ഖുർആൻ

/ഖുർആൻ

എട്ടാം നൂറ്റാണ്ടിനു ശേഷമാണ് ക്വുര്‍ആന്‍ രൂപീകരിക്കപ്പെട്ടു കഴിഞ്ഞതെന്ന് ജറുസലേമിലെ ‘ഖുബ്ബത്തു സ്‌സ്വഖ്‌റ’യില്‍ രേഖപ്പെടുത്തപ്പെട്ട അറബി ലിഖിതങ്ങൾ തെളിവല്ലേ ? അമവീ ഖലീഫയായിരുന്ന അബ്ദുല്‍ മലിക്ക് ബ്‌നു മര്‍വാനിന്റെ ഭരണകാലത്ത് നിര്‍മിക്കപ്പെട്ട ഖുബ്ബത്തു സ്‌സ്വഖ്റയിലെ പുറത്തും അകത്തും മൊസൈക്കില്‍ രേഖപ്പെടുത്തപ്പെട്ട അറബി ലിഖിതങ്ങളിൽ ക്വുർആൻ വചനങ്ങൾ രേഖപ്പെടുത്തിയിരിക്കുന്നത് ഇന്ന് മുസ്ഹഫുകളിൽ കാണുന്ന ക്രമത്തിലല്ല. അത് എഴുതപ്പെടുന്ന കാലത്ത് ക്വുര്‍ആന്‍ പൂര്‍ണമായി രൂപീകരിക്കപ്പെട്ടിട്ടുണ്ടായിരുന്നില്ലെന്നല്ലേ ഇത് വ്യക്തമാക്കുന്നത്?

എട്ടാം നൂറ്റാണ്ടിന്റെ തുടക്കം വരെ ക്വുര്‍ആന്‍ പൂര്‍ണമായും രൂപീകരിക്കപ്പെട്ടു കഴിഞ്ഞിരുന്നില്ലെന്നും അത് തലമുറകളെടുത്ത് രൂപീകരിക്കപ്പെട്ട വചനങ്ങളുടെ സമാഹാരമാണെന്നുമാണ് ചില ഓറിയന്റലിസ്റ്റുകളുടെ വാദം. ഇങ്ങനെ വാദിക്കുന്നവര്‍ ജറുസലേമിലെ ‘ഖുബ്ബത്തു സ്‌സ്വഖ്‌റ’യില്‍ (Dome of the Rock) ല്‍ രേഖപ്പെടുത്തപ്പെട്ട അറബി ലിഖിതങ്ങളാണ് തങ്ങള്‍ക്കുള്ള തെളിവായി അവതരിപ്പി ക്കുന്നത് (Patricia Crone & Michael Cook: Hagarism: The Making of the Islamic World, Cambridge, 1980, Page 139-149.)

അമവീ ഖലീഫയായിരുന്ന അബ്ദുല്‍ മലിക്ക് ബ്‌നു മര്‍വാനിന്റെ ഭരണകാലത്ത് നിര്‍മിക്കപ്പെട്ട സുന്ദരമായൊരു അഷ്ടഭുജ (octagon) കെട്ടിടമാണ് ഖുബ്ബത്തു സ്‌സ്വഖ്റ. ഇതിന്റെ പുറത്തും അകത്തും മൊസൈക്കില്‍ രേഖപ്പെടുത്തപ്പെട്ട അറബി ലിഖിതങ്ങളെയാണ് അത് എഴുതപ്പെടുന്ന കാലത്ത് ക്വുര്‍ആന്‍ പൂര്‍ണമായി രൂപീകരിക്കപ്പെട്ടിട്ടുണ്ടായിരുന്നില്ലെന്നതിന് തെളിവായി ചൂണ്ടിക്കാണിക്കപ്പെടുന്നത്. ആദ്യകാല മുസ്‌ലിം സമ്പ്രദായങ്ങളെയോ അവര്‍ ക്വുര്‍ആനിനെ ഉപയോഗിച്ച രീതിയെയോ കുറിച്ച് യാതൊന്നും അറിയാത്തതുകൊണ്ടാണ് ഇത്തരം ബാലിശമായ വാദങ്ങള്‍ ഉടലെടുക്കുന്നത്.

യഥാര്‍ഥത്തില്‍, ഖുബ്ബത്തു സ്‌സ്വഖ്‌റയിലെ അഷ്ടഭുജത്തിന്മേല്‍ രേഖപ്പെടുത്തപ്പെട്ടിരിക്കുന്നത് ക്രമപ്രകാരമുള്ള ക്വുര്‍ആന്‍ വചനങ്ങളല്ല. ക്വുര്‍ആന്‍ പഠിപ്പിക്കുകയോ രേഖപ്പെടുത്തുകയോ ചെയ്യുകയെന്ന ലക്ഷ്യത്തോടെ എഴുതപ്പെട്ട രേഖയുമല്ല അത്. പ്രത്യുത ഒരു സന്ദേശത്തിന്റെ രേഖീകരണം മാത്രമാണത്. പ്രസ്തുത സന്ദേശത്തിനിടക്ക് ക്വുര്‍ആന്‍ സൂക്തങ്ങളോ അതിന്റെ ഖണ്ഡങ്ങളോ കടന്നു വരുന്നുവെന്ന് മാത്രമേയുള്ളൂ. ഒരു പ്രഭാഷകന്‍ തനിക്കാവശ്യമുള്ള ഉദ്ധരണികള്‍ ഉപയോഗിക്കുന്നതുപോലെ ‘ഖുബ്ബത്തു സ്‌സ്വഖ്‌റാ’യില്‍ സന്ദേശമെഴുതിയവര്‍ അവര്‍ നല്‍കുവാനുദ്ദേശിച്ച ദൂതിന് ഉപോല്‍ബലകമായ ക്വുര്‍ആന്‍ സൂക്തങ്ങളോ ഖണ്ഡങ്ങളോ ഉപയോഗിച്ചുവെന്ന് മാത്രമേയുള്ളൂ. അഷ്ടഭുജത്തിലെ സന്ദേശം വായിക്കുന്ന ആര്‍ക്കും ബോധ്യപ്പെടുന്ന സരളമായ ഒരു വസ്തുതയാണിത്. പ്രസ്തുത സന്ദേശത്തിന്റെ പരിഭാഷ പരിശോധിക്കുക:

അഷ്ടഭുജത്തിനകത്തെ സന്ദേശം (പരിഭാഷ):

പരമകാരുണികനും കരുണാനിധിയുമായ അല്ലാഹുവിന്റെ നാമത്തില്‍; അല്ലാഹുവല്ലാതെ ആരാധ്യനില്ല. അവന്‍ ഏകനാണ്. അവന് പങ്കുകാരൊന്നുമില്ല. അവനാണ് എല്ലാ ആധിപത്യവും; അവന്നുതന്നെയാണ് സ്തുതികളും. അവന്‍ ജീവിപ്പിക്കുന്നു; അവന്‍ മരിപ്പിക്കുകയും ചെയ്യുന്നു. അവനാണ് എല്ലാ കാര്യങ്ങളുടെയും മേലുള്ള അധീശത്വം. മുഹമ്മദ് അല്ലാഹുവിന്റെ ദൂതനും ദാസനുമാകുന്നു.

തീര്‍ച്ചയായും അല്ലാഹുവും മലക്കുകളും പ്രവാചകന്റെ മേല്‍ അനുഗ്രഹങ്ങള്‍ ചൊരിയുന്നു. സത്യവിശ്വാസികളേ നിങ്ങള്‍ അദ്ദേഹത്തിന്റെ മേല്‍ കാരുണ്യവും ശാന്തിയുമുണ്ടാകുവാന്‍ പ്രാര്‍ഥിക്കുക. അദ്ദേഹത്തിന്റെ മേല്‍ അല്ലാഹുവിന്റെ അനുഗ്രഹങ്ങളും ശാന്തിയുമുണ്ടാകട്ടെ.

വേദക്കാരേ, നിങ്ങള്‍ മതകാര്യത്തില്‍ അതിരുകവിയരുത്. അല്ലാഹുവിന്റെ പേരില്‍ വാസ്തവമല്ലാതെ നിങ്ങള്‍ പറയുകയും ചെയ്യരുത്. മര്‍യമിന്റെ മകനായ മസീഹ് ഈസാ അല്ലാഹുവിന്റെ ദൂതനും, മര്‍യമിലേക്ക് അവന്‍ ഇട്ടുകൊടുത്ത അവന്റെ വചനവും, അവങ്കല്‍ നിന്നുള്ള ഒരു ആത്മാവും മാത്രമാകുന്നു. അത് കൊണ്ട് നിങ്ങള്‍ അല്ലാഹുവിലും അവന്റെ ദൂതന്‍മാരിലും വിശ്വസിക്കുക. ത്രിത്വം എന്ന വാക്ക് നിങ്ങള്‍ പറയരുത്. നിങ്ങളുടെ നന്‍മയ്ക്കായി നിങ്ങള്‍ (ഇതില്‍ നിന്ന്) വിരമിക്കുക. അല്ലാഹു ഏക ആരാധ്യന്‍ മാത്രമാകുന്നു. തനിക്ക് ഒരു സന്താനമുണ്ടായിരിക്കുക എന്നതില്‍ നിന്ന് അവനെത്രയോ പരിശുദ്ധനത്രെ. ആകാശങ്ങളിലുള്ളതും ഭൂമിയിലുള്ളതുമെല്ലാം അവന്റെതാകുന്നു. കൈകാര്യകര്‍ത്താവായി അല്ലാഹു തന്നെ മതി.

അല്ലാഹുവിന്റെ അടിമയായിരിക്കുന്നതില്‍ മസീഹ് ഒരിക്കലും വൈമനസ്യം കാണിക്കുന്നതല്ല. (അല്ലാഹുവിന്റെ) സാമീപ്യം സിദ്ധിച്ച മലക്കുകളും (വൈമനസ്യം കാണിക്കുന്നതല്ല.) അവനെ (അല്ലാഹുവെ) ആരാധിക്കുന്നതില്‍ ആര്‍ വൈമനസ്യം കാണിക്കുകയും, അഹംഭാവം നടിക്കുകയും ചെയ്യുന്നുവോ അവരെ മുഴുവനും അവന്‍ തന്റെ അടുക്കലേക്ക് ഒരുമിച്ചുകൂട്ടുന്നതാണ്.

അല്ലാഹുവേ, നിന്റെ ദൂതനും നിന്റെ ദാസനുമായ മര്‍യമിന്റെ പുത്രന്‍ മസീഹിനെ നീ അനുഗ്രഹിക്കേണമേ. അദ്ദേഹം ജനിച്ച ദിവസവും മരിക്കുന്ന ദിവസവും ജീവനോടെ എഴുന്നേല്‍പിക്കപ്പെടുന്ന ദിവസവും അദ്ദേഹത്തിന് സമാധാനം.

അതത്രെ മര്‍യമിന്റെ മകനായ ഈസാ അവര്‍ ഏതൊരു വിഷയത്തില്‍ തര്‍ക്കിച്ച് കൊണ്ടിരിക്കുന്നുവോ അതിനെപ്പറ്റിയുള്ള യഥാർത്ഥമായ വാക്കത്രെ ഇത്. ഒരു സന്താനത്തെ സ്വീകരിക്കുക എന്നത് അല്ലാഹുവിന്നുണ്ടാകാവുന്നതല്ല. അവന്‍ എത്ര പരിശുദ്ധന്‍! അവന്‍ ഒരു കാര്യം തീരുമാനിച്ച് കഴിഞ്ഞാല്‍ അതിനോട് ഉണ്ടാകൂ എന്ന് പറയുക മാത്രംചെയ്യുന്നു. അപ്പോള്‍ അതുണ്ടാകുന്നു. (ഈസാ പറഞ്ഞു) തീര്‍ച്ചയായും അല്ലാഹു എന്റെയും നിങ്ങളുടെയും രക്ഷിതാവാകുന്നു. അതിനാല്‍ അവനെ നിങ്ങള്‍ ആരാധിക്കുക. ഇതത്രെ നേരെയുള്ള മാര്‍ഗം.

താനല്ലാതെ ഒരു ദൈവവുമില്ലെന്നതിന് അല്ലാഹു സാക്ഷ്യം വഹിച്ചിരിക്കുന്നു. മലക്കുകളും അറിവുള്ളവരും (അതിന്ന് സാക്ഷികളാകുന്നു) അവന്‍ നീതി നിര്‍വഹിക്കുന്നവനത്രെ. അവനല്ലാതെ ദൈവമില്ല. പ്രതാപിയും യുക്തിമാനുമത്രെ അവന്‍. തീര്‍ച്ചയായും അല്ലാഹുവിങ്കല്‍ മതം എന്നാല്‍ ഇസ്‌ലാമാകുന്നു. വേദഗ്രന്ഥം നല്‍കപ്പെട്ടവര്‍ തങ്ങള്‍ക്ക് (മതപരമായ) അറിവ് വന്നുകിട്ടിയ ശേഷം തന്നെയാണ് ഭിന്നിച്ചത്. അവര്‍ തമ്മിലുള്ള കക്ഷിമാത്‌സര്യം നിമിത്തമത്രെ അത്. വല്ലവരും അല്ലാഹുവിന്റെ തെളിവുകള്‍ നിഷേധിക്കുന്നുവെങ്കില്‍ അല്ലാഹു അതിവേഗം കണക്ക് ചോദിക്കുന്നവനാകുന്നു.

അഷ്ടഭുജത്തിന് പുറത്തെ സന്ദേശം (പരിഭാഷ):

പരമകാരുണികനും കരുണാനിധിയുമായ അല്ലാഹുവിന്റെ നാമത്തില്‍. അല്ലാഹുവല്ലാതെ ആരാധ്യനില്ല. അവന്‍ ഏകനാണ്. അവന് പങ്കുകാരൊന്നുമില്ല. (നബിയേ,) പറയുക: കാര്യം അല്ലാഹു ഏകനാണ് എന്നതാകുന്നു. അല്ലാഹു ഏവര്‍ക്കും ആശ്രയമായിട്ടുള്ളവനാകുന്നു. അവന്‍ (ആര്‍ക്കും) ജന്‍മം നല്‍കിയിട്ടില്ല. (ആരുടെയും സന്തതിയായി) ജനിച്ചിട്ടുമില്ല. അവന്ന് തുല്യനായി ആരും ഇല്ലതാനും മുഹമ്മദ് അല്ലാഹുവിന്റെ ദൂതനാകുന്നു. അദ്ദേഹത്തില്‍ അല്ലാഹുവിന്റെ അനുഗ്രഹങ്ങളുണ്ടാകട്ടെ.

പരമകാരുണികനും കരുണാനിധിയുമായ അല്ലാഹുവിന്റെ നാമത്തില്‍. അല്ലാഹുവല്ലാതെ ആരാധ്യനില്ല. അവന്‍ ഏകനാണ്. അവന് പങ്കുകാരൊന്നുമില്ല. മുഹമ്മദ് അല്ലാഹുവിന്റെ ദൂതനാകുന്നു. തീര്‍ച്ചയായും അല്ലാഹുവും അവന്റെ മലക്കുകളും നബിയോട് കാരുണ്യം കാണിക്കുന്നു. സത്യവിശ്വാസികളേ, നിങ്ങള്‍ അദ്ദേഹത്തിന്റെ മേല്‍ (അല്ലാഹുവിന്റെ) കാരുണ്യവും ശാന്തിയുമുണ്ടാകാന്‍ പ്രാര്‍ത്ഥിക്കുക.

പരമകാരുണികനും കരുണാനിധിയുമായ അല്ലാഹുവിന്റെ നാമത്തില്‍. അല്ലാഹുവല്ലാതെ ആരാധ്യനില്ല. അവന്‍ ഏകനാണ്. സ്തുതികളെല്ലാം അല്ലാഹുവിനാണ് സന്താനത്തെ സ്വീകരിച്ചിട്ടില്ലാത്തവനും, ആധിപത്യത്തില്‍ പങ്കാളിയില്ലാത്തവനും നിന്ദ്യതയില്‍ നിന്ന് രക്ഷിക്കാന്‍ ഒരു രക്ഷകന്‍ ആവശ്യമില്ലാത്തവനുമായ അല്ലാഹുവിന് സ്തുതി! എന്ന് നീ പറയുകയും അവനെ ശരിയാംവണ്ണം മഹത്വപ്പെടുത്തുകയും ചെയ്യുക. മുഹമ്മദ് അല്ലാഹുവിന്റെ ദൂതനാണ്.

അദ്ദേഹത്തിനു മേലും മലക്കുകളുടെയും പ്രവാചകന്‍മാരുടെയും മേലും അല്ലാഹുവിന്റെ അനുഗ്രഹങ്ങളുണ്ടാവട്ടെ. അദ്ദേഹത്തില്‍മേല്‍ അല്ലാഹുവിന്റെ ശാന്തിയും കാരുണ്യവുമുണ്ടാകട്ടെ. അല്ലാഹുവിന്റെ നാമത്തില്‍, പരമകാരുണികന്‍, കരുണാനിധി. അല്ലാഹുവല്ലാതെ ആരാധ്യനില്ല. അവന്‍ ഏകനാണ്. അവന് പങ്കുകാരില്ല.

അവനാണ് എല്ലാ ആധിപത്യവും. അവനാണ് എല്ലാ കാര്യങ്ങളുടെയും മേലുള്ള അധീശത്വം. മുഹമ്മദ് അല്ലാഹുവിന്റെ ദൂതനാകുന്നു. അദ്ദേഹത്തിനു മേല്‍ അല്ലാഹുവിന്റെ അനുഗ്രഹങ്ങളുണ്ടാവട്ടെ. അദ്ദേഹത്തിന്റെ ജനങ്ങള്‍ക്കുമേല്‍ പുനരുത്ഥാന നാളില്‍ അദ്ദേഹം നടത്തുന്ന ശുപാര്‍ശ അവന്‍ സ്വീകരിക്കട്ടെ.

പരമകാരുണികനും കരുണാനിധിയുമായ അല്ലാഹുവിന്റെ നാമത്തില്‍. അല്ലാഹുവല്ലാതെ ആരാധ്യനില്ല. അവന്‍ ഏകനാണ്. അവന് പങ്കുകാരില്ല. മുഹമ്മദ് അല്ലാഹുവിന്റെ ദൂതനാകുന്നു. അദ്ദേഹത്തിനു മേല്‍ അല്ലാഹുവിന്റെ അനുഗ്രഹങ്ങളുണ്ടാവട്ടെ. ഈ കുംഭഗോപുരം നിര്‍മിച്ചത് ദൈവദാസനായ അബ്ദുല്ലാ അല്‍ ഇമാം അല്‍ മഅ്മൂനാണ്; വിശ്വാസികളുടെ നേതാവ്. എഴുപത്തി രണ്ടാം വര്‍ഷത്തില്‍. അല്ലാഹു അദ്ദേഹത്തില്‍ നിന്ന് ഇത് സ്വീകരിക്കുകയും അദ്ദേഹത്തില്‍ സംപ്രീതനാവുകയും ചെയ്യട്ടെ, ആമീന്‍. സര്‍വലോകരക്ഷിതാവായ, അല്ലാഹുവിന് സ്തുതി”.

ഇത് വായിക്കുന്ന ആര്‍ക്കും മനസ്സിലാക്കാവുന്ന യാഥാര്‍ത്ഥ്യമാണ് ക്വുര്‍ആന്‍ അധ്യായക്രമത്തില്‍ രേഖപ്പെടുത്തിയതല്ല ഇത് എന്നുള്ള വസ്തുത. അല്ലാഹുവിനെ പരിചയപ്പെടുത്തുന്ന ആമുഖത്തില്‍ വ്യത്യസ്ത ക്വുര്‍ആന്‍ സൂക്തങ്ങളില്‍ പ്രയോഗിച്ചിരിക്കുന്ന അല്ലാഹുവിന്റെ നാമ-ഗുണവിശേഷണങ്ങള്‍ പ്രസ്താവിച്ചിരിക്കുന്നുവെന്ന് മാത്രമെയുള്ളൂ. അത് ഉദ്ധരിച്ചുകൊണ്ട് അത് അന്നു നിലനിന്നിരുന്ന ക്വുര്‍ആന്‍ സൂക്തമായിരുന്നുവെന്നും പിന്നീടാണ് അതിലെ ദൈവഗുണ- വിശേഷണങ്ങളെ വേര്‍തിരിച്ചുകൊണ്ടുള്ള സൂക്തങ്ങള്‍ ഇതില്‍ നിന്ന് പരിണമിച്ചുണ്ടായത് എന്നും വാദിക്കുന്നത് മുസ്‌ലിംകള്‍ നടത്തുന്ന പ്രഭാഷണങ്ങളെയും സന്ദേശപ്രചരണത്തെയും കുറിച്ച അജ്ഞത കൊണ്ടാണ്. അല്ലാഹുവിന്റെ നാമത്തില്‍ ആരംഭിക്കുന്ന മുസ്‌ലിം സന്ദേശങ്ങളില്‍ അല്ലാഹുവിനെ സ്തുതിക്കുകയും, അവന്റെ മഹത്വം ഉദ്‌ഘോഷിക്കുകയും മുഹമ്മദ്‌നബി(സ)യുടെ മേല്‍ അനുഗ്രഹങ്ങള്‍ക്കു വേണ്ടി പ്രാര്‍ത്ഥിക്കുകയും ചെയ്ത ശേഷമാണ് മറ്റു കാര്യങ്ങള്‍ പരാമര്‍ശിക്കാറുള്ളത്. ഈ സന്ദേശത്തിലും അങ്ങനെത്തന്നെയാണുള്ളത്. ക്രിസ്ത്യാനികള്‍ താമസിക്കുന്ന സ്ഥലത്താണ് ഖുബ്ബത്തു സ്‌സ്വഖ്‌റാ നിര്‍മിക്കപ്പെട്ടത് എന്നതിനാല്‍ യേശു ക്രിസ്തുവിനെ സംബന്ധിച്ച ഇസ്‌ലാമിക നിലപാട് വ്യക്തമാക്കുകയും ക്രൈസ്തവ നിലപാടിനെ വിമര്‍ശിക്കുകയും ചെയ്യുന്ന വചനങ്ങള്‍ പ്രസ്തുത സന്ദേശത്തിന്റെ ഭാഗമായത് സ്വാഭാവികമാണ്.

ക്വുർആനിലെ പതിനേഴാം അധ്യായത്തിന്റെ 111ാം വചനം രേഖപ്പെടുത്തിയപ്പോള്‍ പ്രസ്തുത വചനത്തിന്റെ തുടക്കത്തിലുള്ള ‘നീ പറയുക’ (വഖുലി)യെന്ന ഭാഗം ഖുവ്വത്തു സ്‌സ്വഖ്‌റായുടെ പടിഞ്ഞാറ് ഭാഗത്ത് രേഖപ്പെടുത്തിയിട്ടില്ലെന്നതാണ് ഇതെഴുതുമ്പോള്‍ ക്വുര്‍ആന്‍ പൂര്‍ണമായിരുന്നില്ലെന്ന് വാദിക്കുന്നവര്‍ക്കുള്ള സുപ്രധാനമായ ഒരു ‘തെളിവ്’. ഒരു സന്ദേശത്തിന്റെ ഭാഗമെന്ന നിലയ്ക്ക് ക്വുര്‍ആന്‍ വചനങ്ങള്‍ ഉദ്ധരിക്കുമ്പോള്‍ ‘നീ പറയുക’ പോലെയുള്ള പരാമര്‍ശങ്ങള്‍ ഒഴിവാക്കി വചനത്തിലെ ആശയപ്രധാനമായ ഭാഗം മാത്രം പരാമര്‍ശിക്കുന്ന സമ്പ്രദായം ഇന്നത്തേതുപോലെ മുസ്‌ലിം സമൂഹത്തില്‍ അന്നും നില നിന്നിരുന്നുവെന്ന് വ്യക്തമാക്കുന്നതാണ് ഈ രേഖ; അതല്ലാതെ, ക്വുര്‍ആന്‍ പരിണമിച്ചുണ്ടായതാണെന്നതിന് അത് തെല്ലും തെളിവു നല്‍കുന്നില്ല.

ഖുബ്ബത്തു സ്‌സ്വഖ്‌റായുടെ പുറത്ത് തെക്ക് ഭാഗത്ത് പൂര്‍ണമായി രേഖപ്പെടുത്തപ്പെട്ട ക്വുര്‍ആനിലെ നൂറ്റി പന്ത്രണ്ടാം അധ്യായത്തിലെ ‘നീ പറയുക’ (ഖുല്‍) എന്ന ഭാഗം ഒഴിവാക്കികൊണ്ടാണ് അബ്ദുല്‍മലിക്കു ബ്‌നു മര്‍വാനിന്റെ കാലത്തും ശേഷവും നിര്‍മിക്കപ്പെട്ട നാണയങ്ങളില്‍ ഈ സൂക്തം മുദ്രീകരിക്കപ്പെട്ടിരിക്കുന്നത് എന്ന വസ്തുത പതിനേഴാം അധ്യായം 111ാം വചനത്തില്‍ ‘നീ പറയുക’യെന്ന ഭാഗം ഒഴിവാക്കിക്കൊണ്ട് രേഖപ്പെടുത്തിയത് ക്വുര്‍ആനിന്റെ രൂപീകരണം പില്‍ക്കാലത്താണ് നടന്നതെന്നതിന് തെളിവാക്കുന്നവരുടെ മുഴുവന്‍ വാദങ്ങളെയും തകര്‍ത്തുകളയുന്നുവെന്ന് പ്രമുഖ ഓറിയന്റലിസ്റ്റായ എസ്‌റ്റെല്ലേ വെലാന്‍ വ്യക്തമാക്കുന്നുണ്ട്. (Estelle Whelan: “Forgotten Witness: Evidence for the Early Codification of the Qur’an”; Journal of American Oriental Society, 1998, Vol. 118, P. 1-14) സമകാലിക രേഖകളെയോ സമ്പ്രദായങ്ങളെയോ കുറിച്ച് പഠിക്കാതെ, ക്വുര്‍ആനിന്റെ ചരിത്രപരതയെ നിഷേധിക്കുവാന്‍ കിട്ടിയ വടികളെല്ലാമെടുത്ത് എറിയാന്‍ ശ്രമിക്കുന്നവരുടെ ‘തെളിവുകള്‍’ അവരുടെ തന്നെ ബൗദ്ധികസത്യസന്ധതക്കു നേരെ തിരിച്ചു വരുന്ന ബൂമറാംഗുകളായിത്തീരുന്നതാണ് നാം ഇവിടെ കാണുന്നത്.

ഹിജ്‌റ 72ലേതാണെന്ന് ഉറപ്പുള്ള ഒരു രേഖയില്‍ ക്വുര്‍ആനിന്റെ വ്യത്യസ്ത ഭാഗങ്ങളില്‍ നിന്നെടുത്ത് ഒരേ വിഷയത്തിലുള്ള വചനങ്ങള്‍ ഉദ്ധരിക്കപ്പെട്ടിരിക്കുന്നുവെന്നത് അക്കാലത്ത് ക്വുര്‍ആനിന്റെ കയ്യെഴുത്ത് പ്രതികള്‍ വ്യാപകമായി ഉപയോഗിക്കപ്പെട്ടിരുന്നുവെന്നതിന് തെളിവാണെന്നാണ് എസ്‌റ്റെല്ലെ വെലാന്‍ സമര്‍ത്ഥിക്കുന്നത്. (Ibid.) ഒരു ഗ്രന്ഥത്തിന്റെ വ്യത്യസ്ത ഭാഗങ്ങളില്‍ പരാമര്‍ശിക്കപ്പെട്ടിരിക്കുന്ന ഒരേ വിഷയസംബന്ധിയായ വചനങ്ങള്‍ ഒരു പ്രത്യേക സന്ദര്‍ഭത്തിലോ രേഖയിലോ ഉദ്ധരിക്കണമെങ്കില്‍ പ്രസ്തുത ഗ്രന്ഥം പൂര്‍ണരൂപത്തില്‍ ഉപലബ്ധമായിരിക്കണമെന്നത് സാമാന്യയുക്തിയാണെന്നാണ് അദ്ദേഹത്തിന്റെ വാദം. അബ്ദുല്ലാ അല്‍ മഅ്മൂനാണ് ഖുബ്ബത്തു സ്‌സ്വഖ്‌റാ നിര്‍മിച്ചതെന്ന് രേഖപ്പെടുത്തപ്പെട്ടിട്ടുണ്ടെങ്കിലും അത് വ്യാജമാണെന്നും അബ്ദുല്‍മലിക്ക് ബ്‌നു മര്‍വാനിന്റെ പേര് മായ്ച്ചു കൊണ്ടാണ് മഅ്മൂനിന്റെ പേര് രേഖപ്പെടുത്തിയിട്ടുള്ളതെന്നും രേഖയില്‍ നിന്നു തന്നെ വ്യക്തമാണ്. സ്വഹാബിമാരുടെ കാലത്ത് ക്വുര്‍ആന്‍ പൂര്‍ണ രൂപത്തില്‍ നിലനിന്നിരുന്നില്ല എന്നതിനുള്ള തെളിവായി കൊട്ടിഘോഷിക്കപ്പെട്ട ഖുബ്ബത്തുസ്‌സ്വഖ്‌റായിലെ ക്വുര്‍ആന്‍ രേഖകള്‍ വിമര്‍ശകര്‍ക്കെതിരായ തെളിവാണ് നല്‍കുന്നതെന്നര്‍ത്ഥം.

അതുകൊണ്ടുതന്നെയായിരിക്കണം പാട്രിഷിയോ ക്രോണിന്റെയും മിഖയേല്‍ കുക്കിന്റെയും വാദങ്ങളെ സമര്‍ത്ഥിക്കുവാന്‍ പാടുപെട്ട് ശ്രമിക്കുന്ന പിന്‍ഗാമികള്‍ പോലും ഖുബ്ബത്തു സ്‌സ്വഖ്‌റായിലെ ക്വുര്‍ആന്‍ ആലേഖനങ്ങളെ ക്വുര്‍ആനിന്റെ ചരിത്രപരതയെ സംശയാസ്പദമാക്കുന്ന തെളിവുകളുടെ കൂടെ പെടുത്താന്‍ മടിക്കുന്നത്. ‘ബുദ്ധിപരമായി സംഭവിക്കാനാവാത്ത വാദങ്ങളാല്‍ നിബിഡവും ബാലിശമായ തെളിവുകള്‍ മാത്രമുള്ളതുമെന്ന് കോണും കുക്കും ചേര്‍ന്നെഴുതിയ ഗ്രന്ഥത്തെ ആധുനിക ഓറിയന്റലിസ്റ്റുകളില്‍ പ്രമുഖനായ മിക്കയേല്‍ ജെ മൊറോണി (Micheal G Morony: Journal of Near Eastern Studies, Volume 41, No:2, April 1982, Page 157-159.) വിശേഷിപ്പിച്ചതും മറ്റൊന്നുകൊണ്ടുമല്ല.

പുരാതനമെന്ന് കരുതപ്പെടുന്ന ഖുര്‍ആന്‍ പ്രതികളില്‍ ഉപയോഗിച്ചിരിക്കുന്ന ലിപിയായ കൂഫീലിപി യഥാർത്ഥത്തിൽ ഉണ്ടായത് എട്ടാം നൂറ്റാണ്ടിൽ മാത്രമാണെന്ന് ഭാഷാശാസ്ത്ര പഠനങ്ങൾ വ്യക്തമാക്കുന്നുണ്ട്. ഇതിനർത്ഥം ആ പ്രതികൾ എഴുതപ്പെട്ടത് എട്ടാം നൂറ്റാണ്ടിലാണെന്നാണ്. മുഹമ്മദ് നബി(സ)ക്ക് ശേഷം പതിറ്റാണ്ടുകള്‍ കഴിഞ്ഞ് രൂപീകരിക്കപ്പെട്ടതാണ് ഇന്ന് നില നിൽക്കുന്ന ഖുർആനിലെ വചനങ്ങള്‍ എന്നല്ലേ ഇത് വ്യക്തമാക്കുന്നത്?

അറബിഭാഷയെയോ അതിന്റെ വികാസത്തെയോ ലിപി പരിണാമത്തെയോ കുറിച്ച് കൃത്യവും വ്യക്തവുമായി പഠിക്കാത്ത ചില ക്രിസ്ത്യന്‍ അപ്പോളജറ്റിക്കുകളാണ് പ്രധാനമായും ഈ വാദമുന്നിയിച്ചിരിക്കുന്നത്. ഉഥ്മാനീ മുസ്ഹഫുകളായി അറിയപ്പെടുന്ന സമര്‍ക്കന്റ്, തോപ്കാപ്പി തുടങ്ങിയ കയ്യെഴുത്ത് പ്രതികള്‍ എഴുതപ്പെട്ടിരിക്കുന്നത് കൂഫിലിപിയിലാണെന്നും പ്രസ്തുത ലിപി തന്നെ അറിയപ്പെടുന്നത് ഇറാക്കിലുള്ള കൂഫാ പട്ടണത്തിന്റെ പേരിലാണെന്നും (Bruce A McDowell & Anees Zaka; Muslims and Christians at the Table: Promoting Biblical Understanding among North American Muslims, Phillipsburg (NJ), 1999, Page 76.) ഖലീഫാ ഉമറിന്റെ (റ)കാലത്ത് നാമകരണം ചെയ്യപ്പെട്ട കൂഫാ പട്ടണത്തിന്റെ പേരിലുള്ള ലിപി ഉഥ്മാനിന്റെ കാലമായപ്പോഴേക്ക് വികസിച്ചു വന്നിരിക്കാന്‍ സാധ്യതയില്ലെന്നും അതുകൊണ്ട്തന്നെ ഈ കയ്യെടുത്തു പ്രതികള്‍ രചിക്കപ്പെട്ടത് ഉഥ്മാനിന്റെ കാലത്താവാന്‍ യാതൊരു സാധ്യതയുമില്ലെന്നും (N.A. Newman: Mohammed, The Qur’an and Islam, Hatfield, 1996, P. 314.) പ്രസ്തുത ക്വുര്‍ആന്‍ ലിപിയില്‍ കുത്തുകളിട്ടിട്ടുണ്ടെന്നും കുത്തുകളിടുന്ന സമ്പ്രദായമാരംഭിച്ചത് ഹിജ്‌റ ഒന്നാം നൂറ്റാണ്ടിന് ശേഷമാണെന്നും (Dr. Robert A Morey: Winning the war against Radical Islam, Las Vegas, 2002, Page 70) അതിനാല്‍ നടേ പറഞ്ഞ കയ്യെഴുത്ത് പ്രതികള്‍ ഉഥ്മാനിന്റെ (റ)ഭരണകാലത്തിന് ഒന്നര നൂറ്റാണ്ടുകളെങ്കിലും കഴിഞ്ഞിട്ടാവണം രചിച്ചിട്ടുള്ളതെന്നുമാണ് (John Gilchrist: Jam’al Qur’an: The Codification of the Qur’an Text, Monder (South Africa), P. 140-146.) അപ്പോളജറ്റിക്കുകളുടെ വാദം. ബൈബിളിനെപ്പോലെത്തന്നെ ആദ്യകാല കയ്യെഴുത്ത് രേഖകളുടെ സാക്ഷ്യം വേണ്ടത്രയില്ലാത്ത ഗ്രന്ഥമാണ് ക്വുര്‍ആനുമെന്ന് വരുത്തിത്തീര്‍ക്കുന്നതിനു വേണ്ടി ഉന്നയിക്കപ്പെടുന്ന ഈ വാദത്തില്‍ ലവലേശം യാഥാര്‍ത്ഥ്യമില്ലെന്ന് പുരാവസ്തു രേഖകളെക്കുറിച്ച സൂക്ഷ്മമായ പഠനം വ്യക്തമാക്കുന്നുണ്ട്.

ക്രിസ്തുവിന് മൂവായിരം കൊല്ലങ്ങള്‍ക്കു മുമ്പ്, ആദ്യകാല വെങ്കല യുഗത്തില്‍ മെസപ്പെട്ടോമിയയില്‍ (ഇന്നത്തെ ഇറാഖ്) വളര്‍ന്നു വികസിച്ച സുമേറിയന്‍ നാഗരികതയിലുള്ളവര്‍ ഉപയോഗിച്ചിരുന്ന ക്യൂനിഫോം (cueniform) ലിപികളില്‍ നിന്നാരംഭിക്കുന്നു ആധുനിക അറബി ലിപിയുടെ ചരിത്രം. ക്യൂനിഫോമുകളില്‍ നിന്നുള്ള പ്രചോദനത്തില്‍ നിന്നാണ് നൈല്‍നദീതടത്തില്‍ ജീവിച്ചിരുന്ന ഈജിപ്തുകാര്‍ അല്‍പ കാലത്തിനുശേഷം അവരുടെ ലിപിയായ ഹൈരോഗ്ലിഫുകള്‍ (heiroglyphs) വികസിപ്പിച്ചെടുത്തത്. (Geoffrey Sampson: A Linguistic Introduction, Stanford University, 1990, P.78.) വെങ്കലയുഗത്തിന്റെ മധ്യകാലഘട്ടത്തില്‍ കാനന്‍ ദേശത്തും (ഇന്നത്തെ ഇസ്രായീലും ഫലസ്തീനും) ഈജിപ്തിലെ സിനായ് പ്രദേശത്തും മധ്യ ഈജിപ്തിലുള്ളവര്‍ ഉപയോഗിച്ച പ്രാഗ് സിനായ് അക്ഷരമാല (proto sinaitic alphabet) രൂപം കൊണ്ടത് ഹൈരോഗ്ലിഫുകളില്‍ നിന്നാണ്. ഇതില്‍നിന്ന് ക്രിസ്തുവിന് 1050 വര്‍ഷങ്ങള്‍ക്ക് മുമ്പ് ചിത്രാധിഷ്ഠിതമല്ലാത്ത (non-pictographic) ആദ്യത്തെ അക്ഷരമാലയായ ഫിനീഷ്യന്‍ അക്ഷരമാല (phoenician alphabet) രൂപപ്പെട്ടു. ഇതില്‍നിന്ന് ക്രിസ്തുവിന് പത്ത് നൂറ്റാണ്ടുകള്‍ക്ക് മുമ്പ് പുരാതന സെമിറ്റിക്കുകാരുടെ ലിപിയായിരുന്ന പാലിയോ ഹിബ്രു അക്ഷരമാലയും (paleo-hebrew alphabet) അതില്‍നിന്ന് എട്ടാം നൂറ്റാണ്ടായപ്പോഴേക്ക് അരാമിക് അക്ഷരമായും (aramic alphabet) രൂപപ്പെട്ടു. (Steven Roger Fischer: History of Writing. London, 2004.) അരാമിക്കില്‍ നിന്ന് ക്രിസ്തുവിന് രണ്ടു നൂറ്റാണ്ടുകള്‍ക്കു മുമ്പ് സുറിയാനിയും (syriac) അതില്‍ നിന്ന് ഏറെ താമസിയാതെ നെബത്തയന്‍ അക്ഷരമാലയും (nabataen alphabet) രൂപപ്പെട്ടു. നെബത്തയന്‍ അക്ഷരങ്ങളില്‍ നിന്നാണ് അറബി അക്ഷരമാലയുണ്ടായതെന്നാണ് ഗവേഷകന്‍മാരുടെ പക്ഷം. ക്രിസ്താബ്ദം നാലാം നൂറ്റാണ്ടിലായിരിക്കണം അറബി അക്ഷരമാലയുണ്ടായതെന്നും 328ല്‍ രചിക്കപ്പെട്ട നെബത്തിയന്‍മാരുടെ രാജകീയ മരണാന്തര ക്രിയകളെക്കുറിച്ചുള്ള രേഖയിലെ ലിപി അറബിയുടെ പ്രാഗ്‌രൂപമാണെന്നും അവര്‍ അഭിപ്രായപ്പെടുന്നു. (“Arabic alphabet”, Brittanica Online www.brittanica.com) എന്നാല്‍ സിറിയയിലെ സെബദില്‍ നിന്ന് കണ്ടെത്തിട്ടുള്ള ഗ്രീക്കിലും സിറിയക്കിലും അറബിയിലുമുള്ള കയ്യെഴുത്ത് രേഖയിലേതാണ് ലഭ്യമായതില്‍വെച്ച് ഏറ്റവും പുരാതനമായ അറബി അക്ഷരങ്ങള്‍ എന്നാണ് ഭൂരിപക്ഷം പണ്ഡിതന്‍മാരുടെയും പക്ഷം (Beatrice Gruendler: The Development of the Arabic Scripts From the Nabatean Era to the first Islamic Century according to the Dated Texts, Atlanta, 1993 Page 13-14)

മുഹമ്മദ് നബി(സ)യുടെ കാലത്ത് നിലനിന്നിരുന്ന മൂന്നുതരം ലിപികളായിരുന്നു ഹിജാസി അഥവാ മാഇല്‍, മശ്ഖ്, കുഫീ എന്നീ ലിപികള്‍. ഇവയിലെല്ലാം രചിക്കപ്പെട്ട ആദ്യകാല ക്വുര്‍ആന്‍ കയ്യെഴുത്ത് പ്രതികളുണ്ട്. ലണ്ടനിലെ ബ്രിട്ടീഷ് ലൈബ്രറിയിലുള്ള MS. Or-2165 കയ്യെഴുത്ത് പ്രതിയും കുവൈത്തിലെ താരിഖ് റജബ് മ്യൂസിയത്തിലുള്ള QUR-1-TSR കയ്യെഴുത്ത് പ്രതിയും പാരീസിലെ ബിബ്‌ളിയോത്തെക്ക് നാഷണേലിലുള്ള Arabe 328 Ca കയ്യെഴുത്തു പ്രതിയും സന്‍ആഇലെ ദാറുല്‍ മഖ്ത്തൂത്താത്തിലുള്ള DA MOI-27.1, DAMOI-25.1, DAMOI=29.1 കയ്യെഴുത്തു പ്രതികളും ഹിജ്‌റ ഒന്നാം നൂറ്റാണ്ടില്‍ ഹിജാസി ലിപിയില്‍ എഴുതപ്പെട്ടവയാണ്. കൈറോയിലെ നാഷണല്‍ ലൈബ്രറിയിലുള്ള ഹിജ്‌റ 107-ല്‍ നിര്‍മ്മിക്കപ്പെട്ടതെന്ന് അതില്‍തന്നെ രേഖയുള്ള കയ്യെഴുത്തു പ്രതി മശ്ഖ് ലിപിയിലുള്ളതാണ്. ഉഥ്മാനീ മുസ്ഹഫുകളായി അറിയപ്പെടുന്നവയും മറ്റ് ഒന്നാം നൂറ്റാണ്ടിലെ കയ്യെഴുത്ത് പ്രതികളുമെല്ലാം കൂഫീ ലിപിയിലാണ് എഴുതപ്പെട്ടിരിക്കുന്നത്. അതുകൊണ്ടുതന്നെയായിരിക്കണം കൂഫി ലിപിയുണ്ടായത് ഹിജ്‌റ ഒന്നാം നൂറ്റാണ്ടിനു ശേഷമാണെന്ന് സ്ഥാപിക്കുവാന്‍ ഓറിയന്റലിസിറ്റുകളും ക്രിസ്ത്യന്‍ അപ്പോളജറ്റിക്കുകളും ആവേശം കാണിക്കുന്നത്. സ്വഹാബിമാരുടെ കാലത്ത് ഉപയോഗിക്കപ്പെട്ടിരുന്ന മുസ്ഹഫുകളിലൊന്നും ഇന്നു ഉപലബ്ധമല്ലെന്നു വന്നാല്‍ അതുപയോഗിച്ച് ക്വുര്‍ആനിന്റെ ചരിത്രപരതയെ ചോദ്യം ചെയ്യാമല്ലോ.

എന്നാല്‍ വസ്തുതകള്‍ അപ്പോളജറ്റിക്കുകള്‍ക്കും അവര്‍ക്കനുസരിച്ച് കാര്യങ്ങള്‍ വളച്ചൊടിച്ചവതരിപ്പിക്കുന്ന ഓറിയന്റലിസ്റ്റുകള്‍ക്കുമെതിരാണ്. മുഹമ്മദ് നബിക്കു(സ) മുമ്പു തന്നെ പ്രചാരത്തിലുണ്ടായിരുന്ന ലിപികളിലൊന്നായിരുന്നു കൂഫീ ലിപിയുമെന്ന വസ്തുത ഓറിയന്റലിസ്റ്റുകളുടെ തന്നെ രചനയായ ‘എന്‍സൈ ക്ലോപീഡിയ ഓഫ് ഇസ്‌ലാം’ തെളിവുകള്‍ നിരത്തി സമർത്ഥിക്കുന്നുണ്ട്. ഹിജ്‌റ 17ല്‍ (ക്രിസ്താബ്ദം 638) കൂഫയുണ്ടാവുന്നതിന് നൂറു വര്‍ഷങ്ങള്‍ക്കു മുമ്പെങ്കിലും മെസപ്പെട്ടോമിയയില്‍ കൂഫി ലിപി പ്രചാരത്തിലുണ്ടായിരുന്നുവെന്നാണ് പ്രമുഖ ഓറിയന്റലിസ്റ്റും ലിപി വിജ്ഞാനീയത്തില്‍ അഗ്രഗണ്യനുമായ ബി. മൊറിട്ട്‌സ് വ്യക്തമാക്കുന്നത് (B. Mortiz: “Arabic Writing”, Encyclopedia of Islam, London, 1913, Page 387.)- അന്‍ബാര്‍, ഹിറ തുടങ്ങിയ മെസെപ്പെട്ടോമിയന്‍ നഗരങ്ങളില്‍ നേരത്തെ പ്രചാരത്തിലുണ്ടായിരുന്ന ലിപിയാണ് കൂഫാ പട്ടണത്തിന്റെ രൂപീകരണത്തിനു ശേഷം ചെറിയ മാറ്റങ്ങളോടെ കൂഫി ലിപിയായി അറിയപ്പെട്ടതെന്ന് നാബിയ അബൊട്ടും വിശദീകരിക്കുന്നുണ്ട്. (Nabia Abbott: The Rise of the North Arabic Script and its Kuranic Development, University of Chicago, 1939 Page 17.) ഗവേഷകരായ അബ്ദുല്‍ കബീര്‍ ഖാത്തിബിയും മുഹമ്മദ് സിജെല്‍മാസ്സിയും കൂടിയെഴുതിയ ‘ഇസ്‌ലാമിക് കാലിഗ്രഫിയുടെ യശസ്സ്’ എന്ന ഗ്രന്ഥത്തില്‍ എങ്ങനെയാണ് ഈ നാമകരണമുണ്ടായതെന്ന് വ്യക്തമാക്കുന്നുണ്ട്. ”ഇസ്‌ലാമിന് മുമ്പ് നിലവിലുണ്ടായിരുന്ന നാല് തരം ലിപികളായിരുന്നു അറബികള്‍ക്ക് പരിചയമുണ്ടായിരുന്നത്. ഹിറയില്‍ പ്രചാരത്തിലുണ്ടായിരുന്ന അല്‍ഹിരിയും, അന്‍ബാറിയിലുണ്ടായിരുന്ന അല്‍അന്‍ബാറിയും മക്കയിലുണ്ടായിരുന്ന അല്‍ മക്കിയും മദീനയിലുണ്ടായിരുന്ന ഇബനു മദനിയും. ക്രിസ്താബ്ദം 999ല്‍ (ഹിജ്‌റ 390) മരണപ്പെട്ട, ഫിഹ്‌രിസ്തിന്റെ കര്‍ത്താവ്, പ്രസിദ്ധനായ അല്‍ നദീമാണ് ഹിരയില്‍നിന്ന് രൂപം പ്രാപിച്ച ലിപിക്ക് കൂഫീ’യെന്ന് നാമകരണം ചെയ്തത്. ഹിജ്‌റ 17ല്‍ (ക്രിസ്താബ്ദം 638) നിര്‍മിക്കപ്പെട്ട നഗരമായ കൂഫയിലുണ്ടായതല്ല ഈ ലിപി. കൂഫയുണ്ടാകുന്നതിന് കാലങ്ങള്‍ക്കു മുമ്പേ ഈ ലിപി നിലനിന്നിരുന്നു. എന്നാല്‍ പ്രസ്തുത ലിപിയില്‍നിന്ന് സുന്ദരമായ കാലിഗ്രഫി വളര്‍ത്തിയെടുത്തതും പരിപോഷിപ്പിച്ചതും മഹത്തായ ഈ ബൗദ്ധിക കേന്ദ്രമാണ്”. (Abdel Kabeer Khattibi & Muhammed Sijelmassi: The Splendour of Islamic Calligraphy, Thames & Hudson, 1994, Page 96-97.)

മാത്രവുമല്ല, ഹിജ്‌റ ഒന്നാം നൂറ്റാണ്ടിന്റെ തുടക്കത്തില്‍ നിര്‍മിച്ചതെന്ന് ഉറപ്പിക്കാവുന്ന ക്വുര്‍ആനല്ലാത്ത മറ്റു കൂഫി ലിഖിതങ്ങള്‍ കണ്ടെടുക്കുകയും ചെയ്തിട്ടുണ്ട്. ഇതിലൊന്നാണ് ഹിജ്‌റ 24ാം വര്‍ഷത്തില്‍ എഴുതിയതാണെന്ന് സ്വയം സാക്ഷ്യം വഹിക്കുന്ന സുഹൈറിന്റെ രേഖ. സഊദി അറേബ്യയിലെ അല്‍ഹിജ്‌റിന് സമീപത്തുള്ള ഖാഅല്‍ മുഅ്തദില്‍ പാറയിലെ ഒരു ചുവന്ന മണല്‍ക്കല്ലില്‍ കൊത്തിവെയ്ക്കപ്പെട്ട രൂപത്തിലുള്ളതാണീ രേഖ. ‘അല്ലാഹുവിന്റെ നാമത്തില്‍, ഞാന്‍ സുഹൈര്‍ ഇതെഴുതുന്നത്. ഇരുപത്തിനാലാം വര്‍ഷത്തില്‍ ഉമര്‍ മര ണപ്പെട്ടപ്പോഴാണ്’ (ബിസ്മില്ലാ, അന സുഹൈര്‍ കതബ്തു സമന്‍ തുവഫ്ഫീ ഉമറ സനത്ത അര്‍ബഅ വ ഇശ്‌രീന്‍) എന്നാണ് രേഖയിലുള്ളത്. (Ali Ibrahim Al-Ghabban (Trans: Robert HolyLand): “The Inscription of Zuhayr, the oldest Islamic Inscription (24Ah/AD644-645), the Rise of the Arabic Scri pt and the nature of the early Islamic State”. “Arabian Archeology and Epigraphy, November 2008, Vol 19, Issue 2, Page 210-237.)

കുത്തുകളുള്ള കൂഫി ലിപിയില്‍ എഴുതപ്പെട്ട ഈ രേഖ ഉമര്‍ (റ) മരണപ്പെട്ട കാലത്ത് കൂഫി ലിപിയിലുള്ള എഴുത്തിന് പ്രചാരമുണ്ടായിരുന്നുവെന്ന് സുതരാം വ്യക്തമാക്കുന്നുണ്ട്. ഉമറിന്‌ ശേഷം ഭരിച്ചയാളാണ് ഉഥ്മാന്‍ (റ) ഉമറിന്റെ (റ) കാലത്തുതന്നെ കൂഫീ ലിപി പ്രചാരത്തിലുണ്ടെങ്കില്‍ ഉഥ്മാനിന്റെ (റ) കാലത്ത് നിര്‍മിക്കപ്പെട്ട മുസ്ഹഫുകള്‍ കൂഫി ലിപിയിലായത് സ്വാഭാവികമാണെന്ന് ആര്‍ക്കും സമ്മതിക്കേണ്ടി വരും. യുനെസ്‌കോയുടെ ലോകസ്മരണകള്‍ രേഖകളില്‍ (http://portal.unesco.org/enev.php_URL_ID= 14264&URL_DO=DO_TOPIC& URL_ SECTION= 201.html) സ്ഥലം പിടിച്ചിട്ടുള്ള ഈ മണല്‍ക്കല്‍ ലിഖിതത്തെ നിഷേധിക്കുവാന്‍ ആര്‍ക്കും കഴിയില്ല. ക്വുര്‍ആന്‍ ചോദ്യത്തെ പരിഹരിക്കാനുതകുന്ന ആദ്യത്തെ ഇസ്‌ലാമികാലേഖനം’ (First Islamic Inscription may solve Qur’an Question) എന്ന തലക്കെട്ടിലാണ് ഡിസ്‌കവറി ചാനലിന്റെ ന്യൂസ് ഇതേക്കുറിച്ച ലേഖനം പ്രസിദ്ധീകരിച്ചിരിക്കുന്നത്. (dsc.discovery.com/news/2008/11/18/islamic-inscription . html) ഉഥ്മാനീ മുസ്ഹഫുകള്‍ എഴുതപ്പെട്ടത് കൂഫി ലിപിയിലാണ് എന്നതിന്റെ പേരില്‍ ക്വുര്‍ആനിന്റെ ചരിത്രപരതയെ ചോദ്യം ചെയ്യാന്‍ ശ്രമിച്ചവരുടെ ഗവേഷണദംഷ്ട്രങ്ങള്‍ പൊഴിക്കുവാന്‍ പര്യാപ്തമാണ് ഈ ചുവന്ന കല്‍രേഖയുടെ കണ്ടുപിടുത്തമെന്നര്‍ഥം.

കൈറോ മ്യൂസിയം ഓഫ് അറബ് ആര്‍ട്ടില്‍ സൂക്ഷിച്ചിരിക്കുന്ന ഹിജ്‌റ 31ല്‍ രചിച്ചതായി സ്വയം സാക്ഷ്യപ്പെടുത്തുന്ന അബ്ദുര്‍റഹ്മാന്‍ ഇബ്‌ നുഖൈര്‍ അല്‍ഹാജിരിയുടെ ഖബര്‍ ഫലകവും (Nabia Abbott: The Rise of the North Arabic Script and its Kur’anic Development, University of Chicago, 1939 Page 18-19.) ഹിജ്‌റ 40ല്‍ എഴുതിയതായി സാക്ഷ്യപ്പെടുത്തുന്ന സഊദി അറേബ്യയിലെ ദര്‍ബ് സുബൈദയിലെ വാദി അല്‍ ശാമിയയിലെ ഒട്ടകപാതയില്‍നിന്ന് 1970ല്‍ ലഭിച്ച ശിലാഫലകവും (A.H.Sharafaddin: “Some Islamic Inscriptions Discovered on the Darb Zubayda”, Atlal (The Journ al of Saudi Arabian Archeology, 1997, Vol 1, Page 69-70.) സഊദി അറേബ്യയിലെ വാദിസബീലില്‍നിന്ന് ലഭിച്ച ഹിജ്‌റ 46ല്‍ രചിച്ചതായി സാക്ഷ്യപ്പെടുത്തുന്ന ചുമര്‍ഫലകവുമെല്ലാം (Beatrice Greundler: Opt it Page 15) കൂഫീലിപിയില്‍ എഴുതപ്പെട്ടവയാണ്. ഇതെല്ലാം സഹാബിമാരുടെ കാലത്ത് കൂഫി ലിപിയിലുള്ള രചനകള്‍ക്ക് പ്രചാരമുണ്ടായിരുന്നുവെന്ന് വ്യക്തമാക്കുന്നുണ്ട്.

ക്രിസ്താബ്ദം 325-ല്‍ ചേർന്ന നിഖിയാ കൗൺസിൽ കാനോനികമായി അംഗീകരിച്ച കൃതികള്‍ മാത്രം നിലനിർത്തി ബാക്കി എല്ലാ ക്രൈസ്തവ ഗ്രന്ഥങ്ങളും ചുട്ടുകരിക്കുവാന്‍ സഭ ആഹ്വാനം നൽകി. ഉഥ്മാൻ (റ) തന്റെ നിർദ്ദേശപ്രകരം തയ്യാർ ചെയ്യപ്പെട്ട ഖുർആൻ പ്രതികള്‍ മാത്രം നിലനിർത്തി ബാക്കിയുള്ളവയെല്ലാം ചുട്ടുകരിക്കാന്‍ കൽപിച്ചു. ഉഥ്മാൻ ചെയ്തതും നിഖിയാ കൗൺസിൽ ചെയ്തതും തമ്മില്‍ എന്തു വ്യത്യാസമാണുള്ളത്?

ഇവിടെ പരാമര്‍ശിക്കപ്പെട്ട സംഭവങ്ങളില്‍ ‘കത്തിക്കുക‘യെന്ന ക്രിയയാണ് ഇരുകൂട്ടരും ചെയ്തതെന്ന കാര്യമൊഴിച്ച് ബാക്കിയെല്ലാം തികച്ചും വ്യത്യസ്തമാണ്. രണ്ടു സംഭവങ്ങളും തമ്മിലുള്ള വ്യത്യാസങ്ങളെ ഇങ്ങനെ സംഗ്രഹിക്കാം.

ഒന്ന്. യേശുവിന് ശേഷം മൂന്നു നൂറ്റാണ്ടുകള്‍ക്കിടക്ക് പലരാലും രചിക്കപ്പെട്ട യേശുവിന്റെ ജീവിതത്തെയും സന്ദേശത്തെയും സംബന്ധിച്ച നാല്‍പതിലധികം ഗ്രന്ഥങ്ങളാണ് നിഖിയാ സൂനഹദോസിനു ശേഷം കത്തിച്ചുകളഞ്ഞത്. മുഹമ്മദി(ﷺ)നു ശേഷം രണ്ടു പതിറ്റാണ്ടിനിടക്ക് പലരും പകര്‍ത്തിയെഴുതിയ ഒരേ ഖുര്‍ആനിന്റെ വിവിധ ഏടുകളില്‍ ഉച്ചാരണ വ്യത്യാസത്തിന് ഇടയാക്കുന്നവയാണ് ഉഥ്മാൻ (റ) കത്തിച്ചുകളയാന്‍ ആവശ്യപ്പെട്ടത്.

രണ്ട്. നിഖിയ കൗണ്‍സില്‍ കാനോനികമായി പ്രഖ്യാപിച്ച നാലു സുവിശേഷങ്ങളിലും അപ്പോസ്തല പ്രവര്‍ത്തനങ്ങളിലും ഇരുപത്തിയൊന്നു ലേഖനങ്ങളിലും വെളിപാടു പുസ്തകത്തിലുമുള്ള പരാമര്‍ശങ്ങള്‍ക്ക് വിരുദ്ധമായ പല പരാമര്‍ശങ്ങളുമുള്ളതുകൊണ്ടും അവ നല്‍കുന്ന യേശു ചിത്രത്തില്‍നിന്ന് തുലോം വ്യത്യസ്തമായ യേശുചിത്രമാണ് അവതരിപ്പിക്കുന്നത് എന്നതുകൊണ്ടുമാണ് അപ്പോക്രിഫാ പുസ്തകങ്ങള്‍ കരിച്ചുകളയുവാന്‍ ആവശ്യപ്പെട്ടത്. വ്യത്യസ്ത ഉച്ചാരണരീതികളുള്ള പ്രാദേശികമൊഴികളില്‍ എഴുതപ്പെട്ട ഏടുകള്‍ തലമുറകളിലേക്ക് കൈമാറ്റം ചെയ്യപ്പെടുമ്പോള്‍ സാരമായ വൈകല്യങ്ങള്‍ക്ക് നിമിത്തമാകാമെന്ന ഭയമാണ് ഉഥ്മാനെ(റ)ഔദ്യോഗിക കൈയ്യെഴുത്തുപ്രതികള്‍ തയാറാക്കാനും സ്വകാര്യ ഏടുകള്‍ നശിപ്പിക്കാനും പ്രേരിപ്പിച്ചത്.

മൂന്ന്. കരിച്ചുകളഞ്ഞ അപ്പോക്രിഫാ ഗ്രന്ഥങ്ങളിലെ ആശയങ്ങള്‍ അവ കരിച്ചുകളഞ്ഞതോടുകൂടി വിസ്മൃതമായി. സ്വകാര്യ ഏടുകളില്‍ എഴുതപ്പെട്ട ഖുര്‍ആന്‍ സൂക്തങ്ങള്‍തന്നെയായിരുന്നു ഔദ്യോഗിക പ്രതികളിലുമുണ്ടായിരുന്നത്. ഉച്ചാരണഭേദങ്ങള്‍ ഒഴിവാക്കാന്‍ വേണ്ടി സ്വകാര്യ ഏടുകള്‍ കത്തിച്ചുകളഞ്ഞുവെങ്കിലും അവയിലുണ്ടായിരുന്ന സൂക്തങ്ങള്‍ അതേ രീതിയില്‍തന്നെ ഇന്നുള്ള ഖുര്‍ആന്‍ കോപ്പികളിലുമുണ്ട്.

നാല്. നിഖിയാ കൗണ്‍സില്‍ തള്ളിക്കളഞ്ഞുവെങ്കിലും അപ്പോക്രിഫാ ഗ്രന്ഥങ്ങളില്‍ പലതും പിന്നീടും ക്രൈസ്തവ മനസ്സുകളില്‍ നിലനിന്നിരുന്നു. അവയിലെ കഥകളില്‍ ചിലത് തലമുറകളില്‍നിന്ന് തലമുറകളിലേക്ക് പ്രേഷണം ചെയ്യപ്പെട്ടു. പതിനാറാം നൂറ്റാണ്ടില്‍ നടന്ന തെന്ത്രോസ് സൂനഹദോസാണ് ഇക്കാര്യത്തില്‍ അന്തിമ തീര്‍പ്പുകല്‍പിച്ചത്. 1540 ഏപ്രില്‍ എട്ടാം തീയതി നടന്ന സൂനഹദോസിന്റെ നാലാം സമ്മേളനം ‘കാനോനിക ഗ്രന്ഥങ്ങളെക്കുറിച്ച്‘എന്ന ഡിക്രിയിലൂടെ പഴയനിയമത്തില്‍ 45-ഉം പുതിയനിയമത്തില്‍ 27-ഉം പുസ്തകങ്ങളാണ് ഉള്ളതെന്ന് പ്രഖ്യാപിച്ചു. ഇതാണ് കാനോനിക ഗ്രന്ഥങ്ങളെക്കുറിച്ച സഭയുടെ അവസാനത്തെ വാക്ക്. എന്നാല്‍, ഉഥ്മാൻ (റ) ഔദ്യോഗികമായി ഖുര്‍ആന്റെ കോപ്പികളെടുത്ത് സ്വകാര്യ ഏടുകള്‍ നശിപ്പിച്ചതിനുശേഷം ഇന്നുവരെ പ്രസ്തുത കോപ്പികളില്‍ നിന്നാണ് മുസ്ഹഫ് പകര്‍ത്തപ്പെടുന്നത്. അതില്‍ ആരും വ്യത്യസ്തത പുലര്‍ത്തുന്നില്ല.

അഞ്ച്. യേശുവിനെക്കുറിച്ച് എഴുതപ്പെട്ട കാനോനികമല്ലാത്ത ഗ്രന്ഥങ്ങള്‍ കരിച്ചുകളയണമെന്ന് കല്‍പിച്ച നിഖിയാ സൂനഹദോസിന്റെ അധ്യക്ഷന്‍ അന്നുവരെ യേശുവില്‍ വിശ്വസിക്കാത്ത കോൺസ്റ്റന്റൈൻ ചക്രവര്‍ത്തിയായിരുന്നു. സ്വകാര്യ കൈയെഴുത്തുപ്രതികള്‍ നശിപ്പിക്കുവാനും ഖുര്‍ആനിന്റെ ഔദ്യോഗിക പ്രതികളെ മാത്രം ആശ്രയിച്ച് പാരായണം ചെയ്യാനും നിര്‍ദേശിച്ച ഉഥ്മാൻ (റ) കറകളഞ്ഞ ഭക്തനും മുഹമ്മദി(ﷺ)ന്റെ ജാമാതാവും അദ്ദേഹത്തോടൊപ്പം ആദര്‍ശ സംരക്ഷണത്തിനുവേണ്ടി നിരവധി യുദ്ധങ്ങളില്‍ പങ്കെടുത്ത വിശ്വാസിയുമായിരുന്നു.

യേശുവിനുശേഷം അനുയായികള്‍ സുവിശേഷങ്ങള്‍ എഴുതി; മുഹമ്മദി(ﷺ)ന് ശേഷം അനുയായികള്‍ ഖുർആൻ എഴുതി; ഇവ തമ്മില്‍ എന്തെങ്കിലും വ്യത്യാസമുണ്ടോ?

വളരെ വലിയ വ്യത്യാസങ്ങളുണ്ട്. അവ ഇങ്ങനെ ക്രോഡീകരിക്കാം.

ഒന്ന്. യേശു‘സുവിശേഷം’ പ്രസംഗിച്ചു(മാര്‍ക്കോസ് 1:14,15, 8:35, 14:9, 10:29,മത്തായി 4:23)വെന്ന് ബൈബിളില്‍ പറയുന്നുണ്ടെങ്കിലും അദ്ദേഹത്തിന്റെ കാലത്ത് ‘സുവിശേഷം’ ഏതെങ്കിലും രൂപത്തില്‍ രേഖപ്പെടുത്തപ്പെട്ടിരുന്നതായി യാതൊരു തെളിവുമില്ല. ഖുര്‍ആനാകട്ടെ മുഹമ്മദി(ﷺ)ന്റെ കാലത്തുതന്നെ രേഖപ്പെടുത്തപ്പെട്ടിരുന്നു. വ്യത്യസ്ത ഏടുകളിലായി.

രണ്ട്. യേശു പ്രസംഗിച്ച‘സുവിശേഷം’ അദ്ദേഹത്തിന്റെ സമകാലികരില്‍ ആരെങ്കിലും പദാനുപദം മനഃപാഠമാക്കിവെച്ചിരുന്നില്ല. ഖുര്‍ആന്‍ മനഃപാഠമാക്കിയ നൂറുകണക്കിന് അനുചരന്മാരുണ്ടായിരുന്നു.

മൂന്ന്. മത്തായിയോ, മാര്‍ക്കോസോ, ലൂക്കോസോ, യോഹന്നാനോ എഴുതിയത് യേശു പ്രസംഗിച്ച സുവിശേഷമല്ല. യേശുവിന്റെ ജീവിതത്തെക്കുറിച്ച് തങ്ങള്‍ അറിഞ്ഞ കാര്യങ്ങളാണ് (ലൂക്കോസ് 1:1-4.) ഖുര്‍ആന്‍ മുഹമ്മദി(ﷺ)ന്റെ ജീവചരിത്രമല്ല, അദ്ദേഹത്തിന് പടച്ചവന്‍ അവതരിപ്പിച്ച വേദഗ്രന്ഥമാണ്.

നാല്. സമൂഹത്തിന്റെ പ്രതിനിധിയായ ഖലീഫ ഉത്തരവാദപ്പെടുത്തിയതിനനുസരിച്ച് സൂക്ഷ്മമായി പരിശോധിച്ചുകൊണ്ടാണ് ഖുര്‍ആന്‍ സമാഹാരണം നടന്നത്. സമാഹര്‍ത്താവായിരുന്ന സൈദുബ്‌നു സാബിത്തിന്റെ വാക്കുകളില്‍ പ്രസ്തുത സമാഹരണത്തിന്റെ സൂക്ഷ്മത വ്യക്തമായി പ്രതിഫലിക്കുന്നുണ്ട്. “ഏതെങ്കിലുമൊരു പര്‍വതത്തെ അതിന്റെ സ്ഥാനത്തുനിന്ന് മാറ്റാനാണ് അബൂബക്കര്‍ (റ) എന്നോട് ആവശ്യപ്പെട്ടിരുന്നതെങ്കില്‍ അതായിരുന്നു എനിക്ക് ഇതിനേക്കാള്‍ നിസ്സാരം”. സുവിശേഷങ്ങളാവട്ടെ, ഓരോരുത്തര്‍ തങ്ങളുടെ ഇച്ഛപ്രകാരം രചിച്ച ഗ്രന്ഥങ്ങളാണ്. അവരുടെ ലക്ഷ്യമാകട്ടെ, തങ്ങളുടെ മുന്നിലുള്ള സമൂഹത്തിന് ക്രിസ്തുവിനെ പരിചയപ്പെടുത്തുക മാത്രമായിരുന്നു. (Raymond E. Brown: Responses to 101 Questions on the Bible, Page 57-58)

അഞ്ച്. യേശുവിന് ശേഷം അഞ്ചു പതിറ്റാണ്ടെങ്കിലും കഴിഞ്ഞാണ് സുവിശേഷങ്ങള്‍ എഴുതപ്പെട്ടത്. ഹിജ്‌റ പന്ത്രണ്ടാം വര്‍ഷത്തിലാണ് – പ്രവാചക നിര്യാണത്തിന് രണ്ടു വര്‍ഷങ്ങള്‍ക്കുശേഷം – ഖുര്‍ആന്‍ സമാഹരണത്തിനു തുടക്കം കുറിക്കപ്പെട്ടത്.

ആറ്. യേശുവിന്റെ ശിഷ്യന്മാരല്ല സുവിശേഷങ്ങള്‍ രചിച്ചിട്ടുള്ളത്. മുഹമ്മദി(ﷺ)ന്റെ ശിഷ്യന്മാരാണ് ഖുര്‍ആന്‍ സമാഹരിച്ചത്.

ഏഴ്. സുവിശേഷങ്ങളുടെ രചനക്ക് ആധാരം യേശുവിനെ സംബന്ധിച്ച കേട്ടുകേള്‍വികള്‍ മാത്രമായിരുന്നു. ഖുര്‍ആന്‍ ക്രോഡീകരണത്തിന് പ്രവാചകന്‍(ﷺ)തന്നെ പറഞ്ഞുകൊടുത്ത് എഴുതിപ്പിടിപ്പിച്ച ഏടുകളും പ്രവാചകനില്‍നിന്ന് നേരിട്ട് ഖുര്‍ആന്‍ കേട്ടു മനഃപാഠമാക്കിയ നൂറുകണക്കിന് അനുചരന്മാരുമായിരുന്നു അവലംബം.

ഹിജ്‌റ 95ൽ മരണപ്പെട്ട ഹജ്ജാജ് ബിൻ യൂസഫ് ഖുർആനിലെ പതിനൊന്ന് സ്ഥലങ്ങളിൽ തന്റേതായ ചെറിയ മാറ്റങ്ങൾ വരുത്തിയെന്ന നിവേദനങ്ങളുണ്ടല്ലോ. പ്രധാനപ്പെട്ട നാല് ഖലീഫമാരുടെ കാലശേഷവും ഖുർആനിൽ തിരുത്തലുകളുണ്ടായി എങ്കിൽ പിന്നെ അത് സംരക്ഷിക്കപ്പെട്ടുവെന്ന് പറയുന്നതിൽ എന്ത് അർത്ഥമാണുള്ളത്?

ഇബ്നു അബീദാവൂദിന്റെ ആൽമസാഹിഫിലുള്ള ഒരു ഉദ്ധരണിയുടെ അടിസ്ഥാനത്തിലുള്ളതാണ് ഈ വിമർശനം. ഉദ്ധരണി ഇങ്ങനെയാണ്: “അബ്ബാദ് ബ്നു ശുഹൈബ്, ഔഫ് ബ്നു അബീജമീലയിൽ നിന്ന് നിവേദനം ചെയ്യുന്നു: അൽ ഹജ്ജാജ് ഖുർആനിലെ പതിനൊന്ന് പദങ്ങൾ മാറ്റിയെഴുതി. സൂറത്തുൽ ബഖറയിൽ (2 :259) ‘ലം യതസന്ന വൻദുർ’ (لَمۡ يَتَسَنَّۚ وَانْظُرۡ) എന്നേ ഉണ്ടായിരുന്നുള്ളൂ. ‘അത് ലം യതസന്നഹു വൻദുർ'(لَمۡ يَتَسَنَّهۡۚ وَانْظُرۡ) എന്നാക്കി മാറ്റി. മാഇദയിൽ(3: 58) ‘ശരീഅത്തൻ വ മിൻഹാജൻ’.’ (شريعة وَّمِنۡهَاجً) എന്നാണുണ്ടായിരുന്നത്. അത് ‘ശിർഅത്തൻ വ മിൻഹാജൻ’ (شِرۡعَةً وَّمِنۡهَاجًا) എന്നാക്കിത്തീർത്തു. യൂനുസിൽ (10:22) ‘ഹുവല്ലദീ യുൻഷിറുക്കും'(هُوَ الَّذِىۡ ينشركم) എന്നായിരുന്നത് ‘ഹുവല്ലദീ യുസയ്യിറുക്കും’ ( هُوَ الَّذِىۡ يُسَيِّرُكُمۡ) എന്നാക്കി മാറ്റി. യൂസുഫിൽ (12:45) ‘അന ആത്തീകും ബി തഅവീലിഹി'(اَنَا آتيكم بِتَاۡوِيۡلِهٖ) എന്നായിരുന്നിടത്ത് ‘അന ഉനബ്ബിഉക്കും ബി തഅവീലിഹി’ (اَنَا اُنَـبِّئُكُمۡ بِتَاۡوِيۡلِهٖ) എന്നാക്കിത്തീർത്തു. സുഖ്‌റൂഫിൽ (43:32)

‘നഹ് നു ഖസംനാ ബൈനഹും മആഇഷഹും’ (نَحۡنُ قَسَمۡنَا بَيۡنَهُمۡ معايشهم) എന്നായിരുന്നിടത്ത് ‘നഹ് നു ഖസംനാ ബൈനഹും മഈഷത്തഹും’ (نَحۡنُ قَسَمۡنَا بَيۡنَهُمۡ مَّعِيۡشَتَهُمۡ) എന്നാക്കി മാറ്റി. തക്‌വീറിൽ (81:24). ‘വമാ ഹുവ അലൽ ഗൈബി ബി ദ്വനീൻ’ (وَمَا هُوَ عَلَى الۡغَيۡبِ بِظنِيۡنٍ) എന്നതിന് പകരമായി ‘വമാ ഹുവ അലൽ ഗൈബി ബി ദനീൻ’ (وَمَا هُوَ عَلَى الۡغَيۡبِ بِضَنِيۡنٍ) എന്നാക്കിത്തീർത്തു” (ഇബ്നു അബീദാവൂദ്: കിതാബുൽ മസാഹിഫ്, പുറം 49)

മുഹമ്മദ് നബിക്ക് അര നൂറ്റാണ്ടുകൾക്ക് ശേഷം പോലും ഖുർആനിൽ കൈകടത്തലുകൾ നടന്നുവെന്ന വിമർശനം ശരിയാണെങ്കിൽ അതേറെ ഗൗരവമുള്ള കാര്യമാണ്. എന്നാൽ വസ്തുതയെന്താണ്?

ഒന്ന്) ഈ നിവേദനം തെളിവിന് ആശ്രയിക്കാൻ കഴിയാത്ത വിധം തീരെ ദുർബലമാണ്. ഇമാം ദഹബി തന്റെ മീസാനുൽ ഇഅ്തിദാലിൽ പറയുന്നു: “ഈ നിവേദനം വളരെ ദുർബലമോ (ദഈഫൻ ജിദ്ദൻ) കെട്ടിയുണ്ടാക്കപ്പെട്ടതോ (മൗദൂഅ്) ആണ്. ഇതിന്റെ നിവേദകപരമ്പരയിൽ അബ്ബാദ് ബിൻ ശുഐബ് ഉള്ളതുകൊണ്ടാണത്. അയാളുടെ ഹദീഥുകളെല്ലാം തള്ളപ്പെടേണ്ടതാണ്. അലിയ്യു ബ്നു മദീനി പറഞ്ഞത് അയാളുടെ ഹദീഥുകൾ കൊള്ളാവുന്നവയല്ലെന്നാണ്. അയാൾ തള്ളപ്പെടേണ്ടവനാണെന്നാണ് ബുഖാരിയുടെയും നസാഇയുടെയും മറ്റുള്ളവരുടെയുമെല്ലാം പക്ഷം. തന്റെ വ്യതിയാനാദർശങ്ങൾ പ്രചരിപ്പിക്കുവാൻ ശ്രമിച്ച (ദൈവികവിധിയെ നിഷേധിച്ചവരായ) ഖദ്‌രിയ്യാക്കളിൽ പെട്ട ഇയാളുടെ നിവേദനങ്ങളിൽ പലതും ഈ രംഗത്തുള്ള ഒരു നവാഗതൻ പോലും കേട്ടാൽ അയാൾക്ക് അവ കെട്ടിയുണ്ടാക്കിയതാണെന്ന് മനസ്സിലാവുന്ന തരത്തിലുള്ളവയാണ്. ദഹബി പറയുന്നു: തള്ളപ്പെടേണ്ടവരിൽ പെട്ടയാളാണ് ഇയാൾ” (4/28)

ഇക്കാര്യം തന്നെയാണ് ഈ നിവേദകനെപ്പറ്റി ഇമാം ഇബ്നു ഹജറുൽ അസ്ഖലാനി തന്റെ ‘കിതാബു ലിസാനുൽ മീസാനി’ലും (പുറങ്ങൾ 230-231) ഇമാം ഇബ്നു ഹിബ്ബാൻ തന്റെ ‘കിത്താബൽ മജ്‌റൂഹീൻ മിനൽ മുഹദ്ദിഥിൻ വൽ ദുഅഫാഅ വൽ മത് റൂഖീൻ’ (പുറങ്ങൾ 164-165) എന്ന കൃതിയിലും പറഞ്ഞിട്ടുള്ളത്.

ആരോ തങ്ങളുടെ ആശയപ്രചാരണത്തിനു വേണ്ടി കെട്ടിയുണ്ടാക്കിയ വാറോല മാത്രമാണ് ഖുർആനിന്റെ അഖണ്ഡതയെ ചോദ്യം ചെയ്യാനായി വിമർശകരുടെ കൈകളിലുള്ള ആയുധങ്ങളിൽ ഏറ്റവും മൂർച്ഛയുള്ളത് എന്ന സത്യം എന്തുമാത്രം പരിഹാസ്യമല്ല!

രണ്ട്) അമവിയ്യാ ഖലീഫയായിരുന്ന അബ്ദുൽ മലിക്ക് ബ്‌നു മർവ്വാനിന്റെ ഇറാഖിലെ ഗവർണറായിരുന്നു ഹജ്ജാജ് ബ്നു യൂസുഫ്. ഉഥ്മാനിനെ ആദരിക്കുകയും അദ്ദേഹത്തെ ആരെങ്കിലും അനാദരിക്കുന്നതിനെ വെറുക്കുകയും വിമർശിക്കുകയും പ്രതിരോധിക്കുകയും ചെയ്യുന്നവരാണ് അമവിയ്യാക്കൾ. ഹജ്ജാജ് അക്കാര്യത്തിൽ ഏറെ മുന്നിലായിരുന്നുവെന്ന് അദ്ദേഹത്തിന്റെ ചരിത്രം വ്യക്തമാക്കുന്നുണ്ട്. ഉഥ്മാനിനെ മറ്റുള്ളവർ വിമർശിക്കുവാനും നിന്ദിക്കുവാനും ഇട വരുത്തുന്ന ഒരു ആരോപണം അദ്ദേഹത്തിൽ നിന്നുണ്ടാകുവാൻ യാതൊരു സാധ്യതയുമില്ല. അൽ മസാഹിഫിലെ നിവേദനം ശരിയാണെങ്കിൽ ഉഥ്മാനിന് ഖുർആൻ ക്രോഡീകരണത്തിൽ തെറ്റു പറ്റിയെന്ന് ഹജ്ജാജ് അംഗീകരിച്ചുവെന്നാണ് അതിനർത്ഥം. അങ്ങനെയൊന്ന് ഉണ്ടാകുവാൻ യാതൊരു സാധ്യതയുമില്ലെന്ന് അന്നത്തെ രാഷ്ട്രീയസാഹചര്യങ്ങൾ അപഗ്രഥിക്കുന്ന ആർക്കും ബോധ്യമാവും.

മൂന്ന്) ഉഥ്മാനിന്റെ കാലത്ത് തന്നെ അദ്ദേഹം നിർമ്മിച്ച ഖുർആൻപതിപ്പുകൾ വ്യത്യസ്ത പ്രവിശ്യകളിലേക്ക് അയച്ചു കൊടുത്തിട്ടുണ്ടായിരുന്നു. ഖലീഫ അബ്ദുൽ മലിക്ക് ബ്‌നു മർവ്വാനിന്റെ കാലമായപ്പോഴേക്കും അവയിൽ നിന്ന് പകർത്തിയെഴുതിയ കോപ്പികൾ എല്ലായിടത്തും വ്യാപകമായിരുന്നു. അവയുടെ അടിസ്ഥാനത്തിൽ പാരായണം അഭ്യസിച്ച പതിനായിരങ്ങൾ ഇസ്‌ലാമികസാമ്രാജ്യത്തിന്റെ വിവിധ സ്ഥലങ്ങളിൽ ജീവിച്ചിരിക്കുകയും ചെയ്തിരുന്നു. അന്ന് നിലവിലുണ്ടായിരുന്ന മുഴുവൻ മുസ്ഹഫുകളിലും ഹജ്ജാജ് തിരുത്തുകയും പ്രസ്തുത തിരുത്തുകൾ പ്രകാരം അന്ന് ജീവിച്ചിരുന്ന ഖുർആൻ മനഃപാഠമാക്കിയിരുന്ന മുഴുവനാളുകളും തങ്ങളുടെ മനഃപാഠം പരിഷ്കരിച്ചുവെന്നും കരുതുന്നത് മൗഢ്യമാണ്; തികച്ചും അസംഭവ്യവും അപ്രായോഗികവുമാണത്.

നാല്) അമവിയ്യാ ഖലീഫയായിരുന്ന അബ്ദുൽ മലിക്ക് ബ്‌നു മർവ്വാനിന്റെ ഇറാഖിലെ ഗവർണർ മാത്രമായിരുന്നു ഹജ്ജാജ്. ബ്രഹത്തായ ഇസ്‌ലാമികസാമ്രാജ്യത്തിലെ നിരവധി പ്രദേശങ്ങളിൽ ഒന്നിന്റെ മാത്രം ഗവർണർ. ഹജ്ജാജ് ഭരിച്ചിരുന്ന ഇറാഖിലെ മുഴുവൻ മുസ്ഹഫുകളിലും തിരുത്താൻ അദ്ദേഹത്തിന് കഴിഞ്ഞുവെന്ന് തന്നെ കരുതുക. എങ്കിൽ പോലും ഇസ്‌ലാമിക സാമ്രാജ്യത്തിൽ ബാക്കിയുള്ള സ്ഥലങ്ങളിലുള്ള ആയിരക്കണക്കിന് ഖുർആൻ പ്രതികളെല്ലാം അദ്ദേഹത്തിന് തിരുത്താൻ കഴിയുന്നതെങ്ങനെ? തികച്ചും അസംഭവ്യമാണ് അൽ മസാഹിഫിൽ ആരോപിക്കപ്പെട്ട സംഭവം എന്നാണിത് വ്യക്തമാക്കുന്നത്.

അഞ്ച്) ഹജ്ജാജ് ഭരിച്ചിരുന്ന ഇറാഖിലെ മുഴുവൻ മുസ്ഹഫുകളിലും അദ്ദേഹം തിരുത്തിയെന്ന് തന്നെ കരുതുക. അക്കാലത്തെ ഇസ്‌ലാമിക സാമ്രാജ്യത്തിൽ മറ്റുള്ള പ്രദേശങ്ങളിൽ പാരായണം ചെയ്യപ്പെട്ടിരുന്ന മുസ്ഹഫുകളുമായി അവയ്ക്ക് അപ്പോൾ വ്യത്യാസമുണ്ടായിരിക്കും. ഇറാഖീമുസ്ഹഫുകളുമായി മുസ്‌ലിംലോകത്തെ മറ്റു മുസ്ഹഫുകൾ ഏതെങ്കിലും കാലത്ത് വ്യത്യസ്തത പുലർത്തിയതായി വ്യക്തമാക്കുന്ന ചരിത്രരേഖകളൊന്നും തന്നെയില്ല. ചരിത്രവിരുദ്ധമാണ് അൽ മസാഹിഫിൽ ആരോപിക്കപ്പെട്ട സംഭവം എന്നർത്ഥം.

ആറ്) അമവിയ്യാക്കൾക്ക് ശേഷം ഇസ്‌ലാമിക സാമ്രാജ്യം ഭരിച്ചത് അബ്ബാസിയാക്കളായിരുന്നു. ഹജ്‌ജാജിനെതിരേയുള്ള നിറം പിടിപ്പിച്ച കഥകളുടെ പലതിന്റെയും സ്രഷ്ടാക്കൾ അബ്ബാസിയാക്കളാണ്. ഖുർആനിൽ ഇങ്ങനെയൊരു കൈകടത്തൽ ഹജ്ജാജ് നടത്തിയിരുന്നുവെങ്കിൽ അക്കാര്യം അബ്ബാസിയാക്കൾ സമൂഹത്തിൽ പാട്ടാക്കുകയും അമവിയ്യാക്കളെക്കുറിച്ച വലിയിരു ആരോപണമായി അത് ഉന്നയിക്കുകയും ചെയ്യുമായിരുന്നു. അതുണ്ടായിട്ടില്ല. അബ്ബാസിയാ രേഖകളിലെവിടെയും അമവിയ്യാക്കൾക്കെതിരായി ഖുർആനിൽ കൈകടത്തലുകൾ നടത്തിയെന്ന ആരോപണം ഉന്നയിച്ചതായി കാണുന്നില്ല. അങ്ങനെയൊന്നുണ്ടായിട്ടില്ലെന്നതിനുള്ള ശക്തമായ തെളിവാണിത്.

ഏഴ്) ഹജ്ജാജ് തിരുത്തിയതായി പറയപ്പെടുന്നതൊന്നും തന്നെ യഥാർത്ഥത്തിൽ തിരുത്തലുകളല്ല പ്രത്യുത, ഖുർആനിന്റെ വ്യത്യസ്തമായ പാരായണങ്ങളാണ്. സൂറത്തുൽ ബഖറയിലെ 259 ആം വചനം ഉദാഹരണം. ‘ലം യതസന്ന വൻദുർ’ (لَمۡ يَتَسَنَّۚ وَانْظُرۡ) എന്ന ഹജ്ജാജ് തിരുത്തിയതിന് മുമ്പുള്ളതായി അൽ മസാഹിഫിലെ നിവേദനത്തിൽ പറയുന്ന ‘യതസന്ന’ക്കു ശേഷം ‘ഹു’ ഇല്ലാത്ത പ്രയോഗം ഹംസയിൽ നിന്നും അൽ കിസാഇയിൽ നിന്നുമുള്ള പാരായണത്തിലുള്ളതാണെന്നും മറ്റ് പ്രധാനപ്പെട്ട അഞ്ച് പാരായണങ്ങളിലും ‘അത് ലം യതസന്നഹു വൻദുർ’ (لَمۡ يَتَسَنَّهۡۚ وَانْظُرۡ) എന്നാണുള്ളതെന്നും ഇബ്നു സഞ്ചലയുടെ ‘ഹുജ്ജത്തുൽ ഖിറാഅത്തി’ൽ നിന്ന് (പുറം 142, 143) ഡോ: മുഹമ്മദ് ബിൻ ഇബ്‌റാഹീം റദ്‌വാൻ ഉദ്ധരിക്കുന്നുണ്ട്. ആരോപിക്കപ്പെട്ട പതിനൊന്ന് വ്യത്യാസങ്ങളും ഖുർആൻ ആയത്തുകളുടെ പാരായണഭേദങ്ങൾ മാത്രമാണെന്ന് ആ വിഷയത്തിൽ പഠനം നടത്തിയ ഡോ: മുഹമ്മദ് റദ്‌വാൻ തന്റെ ‘വിശുദ്ധ ഖുർആനിന്റെയും അതിന്റെ വ്യാഖ്യാനങ്ങളെയും സംബന്ധിച്ച ഓറിയന്റലിസ്റ്റ് വീക്ഷണങ്ങൾ: പഠനവും വിമർശനവും’ എന്ന തന്റെ ഗവേഷണപ്രബന്ധത്തിൽ സമർത്ഥിക്കുന്നുണ്ട്. (ഡോ: മുഹമ്മദ് ബിൻ ഇബ്‌റാഹീം റദ്‌വാൻ:ആറാഅ’ൽ മുസ്തശ്‌രിഖീൻ ഹൌലൽ ഖുർആനൽ കരീം വ തഫ്സീർ: ദിറാഷ് വ നഖ്ദ്, വാല്യം ഒന്ന്, പുറം 430, റിയാദ്, 1992).

പാരായണവ്യത്യാസങ്ങളെക്കുറിച്ച ഹജ്ജാജ് ബ്നു യൂസുഫിന്റെ ഏതോ പരാമർശങ്ങളെ തെറ്റിദ്ധരിച്ചതു കൊണ്ടോ തെറ്റായി വ്യാഖ്യാനിച്ചത് കൊണ്ടോ ഉണ്ടായതാണ് അദ്ദേഹം ഖുർആനിൽ മാറ്റം വരുത്തിയെന്ന ആരോപണമെന്ന് സാരം.

ഖുർആനിൽ സൂറത്തുൽ ബഖറയിലെ 238 ആം വചനത്തിൽ ഇന്നുള്ള ഖുർആനിൽ ഇല്ലാത്ത ഒരു പ്രയോഗം ആയിശ(റ)യുടേതായി നിവേദനം ചെയ്യപ്പെട്ടിട്ടുണ്ടല്ലോ. ഖുർആനിൽ തിരുത്തലുകളുണ്ടായി എന്നല്ലേ അത് വ്യക്തമാക്കുന്നത്?

ആയിശ(റ)യിൽ നിന്ന് നിവേദനം ചെയ്തിരിക്കുന്ന ഒരു ഹദീഥിന്റെ അടിസ്ഥാനത്തിലുള്ളതാണ് ഈ വിമർശനം. ഹദീഥ് ഇങ്ങനെയാണ്: “ആയിശ(റ)യുടെ സ്വാതന്ത്രനാക്കപ്പെട്ട ദാസനായ അബൂയൂനുസ് (റ) പറഞ്ഞു: ആയിശ (റ) അവർക്കുവേണ്ടി ഒരു ഖുർആൻ എഴുതിയുണ്ടാക്കാൻ എന്നോട് ആവശ്യപ്പെട്ടുകൊണ്ട് പറഞ്ഞു. ‘നമസ്കാരങ്ങള്‍, വിശേഷിച്ചും ഉൽകൃഷ്ടനമസ്കാരം നിങ്ങള്‍ സൂക്ഷ്മതയോടെ നിര്‍വഹിക്കുക’ (حَافِظُوا عَلَى الصَّلَوَاتِ وَالصَّلاَةِ الْوُسْطَى) എന്ന വചനമെത്തുമ്പോൾ നീ എന്നെ അറിയിക്കണം. ആ വചനമെത്തിയപ്പോൾ ഞാൻ അവരെ അറിയിച്ചു. അപ്പോൾ അവർ എന്നോട് ഇങ്ങനെ രേഖപ്പെടുത്താൻ പറഞ്ഞു: ‘നമസ്കാരങ്ങള്‍, വിശേഷിച്ചും ഉൽകൃഷ്ടനമസ്കാരവും അസർ നമസ്കാരവും നിങ്ങള്‍ സൂക്ഷ്മതയോടെ നിര്‍വഹിക്കുക; അല്ലാഹുവിനു മുമ്പിൽ ഭയഭക്തിയോടെയാകണം നിങ്ങൾ നിൽക്കേണ്ടത്’ (حَافِظُوا عَلَى الصَّلَوَاتِ وَالصَّلاَةِ الْوُسْطَى وَصَلاَةِ الْعَصْرِ وَقُومُوا لِلَّهِ قَانِتِينَ). എന്നിട്ട് അവർ പറഞ്ഞു. ഇങ്ങനെയാണ് ഞാൻ അല്ലാഹുവിന്റെ ദൂതനിൽ നിന്ന് കേട്ടിട്ടുള്ളത്” (സ്വഹീഹ് മുസ്‌ലിം, കിതാബുൽ മസാജിദി വൽ മവാദിഇ സ്സ്വലാത്തി; ജാമിഉ ത്തിർമിദി, കിതാബു ത്തഫ്സീറിൽ ഖുർആൻ; സുനനു അബീദാവൂദ്, കിതാബുസ്വലാത്ത്; സുനനു ന്നസാഈ, കിതാബുസ്വലാത്ത്; മുവത്വ മാലിക്ക്, കിതാബു സ്വലാത്തിൽ ജമാഅഃ)

എന്താണ് ആയിശ (റ) ഇങ്ങനെ പറയാനുള്ള കാരണം? അത് സ്വഹീഹ് മുസ്‌ലിമിൽ തന്നെയുള്ള അടുത്ത ഹദീഥിൽ നിന്ന് മനസ്സിലാവും.

അൽ ബറാഉ ബിൻ ആസിബിൽ നിന്ന് നിവേദനം: ഈ വചനം ആദ്യം അവതരിക്കപ്പെട്ടത് ഇങ്ങനെയായിരുന്നു: ‘നമസ്കാരങ്ങള്‍, വിശേഷിച്ചും അസർ നിങ്ങള്‍ സൂക്ഷ്മതയോടെ നിര്‍വഹിക്കുക’ (حَافِظُوا عَلَى الصَّلَوَاتِ وَصَلاَةِ الْعَصْرِ). ഈ വചനം അല്ലാഹു ഉദ്ദേശിച്ച കാലത്തോളം ഞങ്ങൾ പാരായണം ചെയ്തിരുന്നത് ഇങ്ങനെയായിരുന്നു. അതിനു ശേഷം അല്ലാഹു ഇത് ദുർബലപ്പെടുത്തുകയും ‘നമസ്കാരങ്ങള്‍, വിശേഷിച്ചും ഉൽകൃഷ്ടനമസ്കാരം നിങ്ങള്‍ സൂക്ഷ്മതയോടെ നിര്‍വഹിക്കുക’ (حَافِظُوا عَلَى الصَّلَوَاتِ وَالصَّلاَةِ الْوُسْطَى) എന്ന വചനം അവതരിപ്പിക്കുകയും ചെയ്തു. (നിവേദക ശൃംഖലയിലെ ഒരാളായ) ഷഖീഖിന് അടുത്തിരുന്ന ഒരാൾ പറഞ്ഞു; ഇപ്പോൾ അത് അർത്ഥമാക്കുന്നത് അസർ നമസ്‌കാരമാണ്. ഇത് സംബന്ധമായി ബറാഅ പറഞ്ഞു: ഈ വചനം എങ്ങനെയാണ് അവതരിച്ചതെന്നും എങ്ങനെയാണ് അല്ലാഹു അത് ദുർബലപ്പെടുത്തിയതെന്നും ഞാൻ നിന്നോട് പറഞ്ഞു കഴിഞ്ഞുവല്ലോ; അല്ലാഹുവിനാണ് കാര്യങ്ങളെല്ലാം കൃത്യമായി അറിയുക.”(സ്വഹീഹ് മുസ്‌ലിം, കിതാബുൽ മസാജിദി വൽ മവാദിഇ സ്സ്വലാത്തി)

ഈ ഹദീഥുകൾ വ്യാഖ്യാനിച്ച പണ്ഡിതന്മാർ ആയിശ (റ) അങ്ങനെ പറഞ്ഞതെന്തുകൊണ്ട് എന്ന് വ്യക്തമാക്കിയിട്ടുണ്ട്. തിർമിദിയുടെ വ്യാഖ്യാതാവായ അബ്ദുറഹ്‌മാൻ അൽമുബാറക് പൂരി എഴുതുന്നു: “അൽ ബാജി പറഞ്ഞു: ‘അവർ(ആയിശ) ഇത് ഖുർആനിലുള്ളതെന്ന രൂപത്തിൽ കേട്ടതിനു ശേഷമായിരിക്കാം മുസ്‌ലിം ബറാഇൽ നിന്ന് നിവേദനം ചെയ്ത ഹദീഥിലുള്ളതുപോലെ അത് ദുർബലപ്പെടുത്തപ്പെട്ടത്; ആയിശ (റ) ഈ വചനം ദുർബലപ്പെടുത്തപ്പെട്ടത് അറിഞ്ഞിട്ടില്ലായിരിക്കാം; അറിഞ്ഞെങ്കിലും അതിലെ നിയമം മാത്രം ദുർബലപ്പെടുത്തപ്പെടുകയും വചനം നിലനിൽക്കുന്നുവെന്ന് അവർ കരുതിയിരിക്കാം; അത് ഖുർആനിന്റെ ഭാഗമല്ലെങ്കിലും അതിന്റെ പ്രാധാന്യം വ്യക്തമാക്കാനായി അല്ലാഹുവിന്റെ ദൂതൻ അവരോട് പറഞ്ഞതിൽ നിന്ന് അത് ഖുർആനിന്റെ ഭാഗം തന്നെയാണെന്ന് അവർ ധരിക്കുകയും കൈയെഴുത്ത്പ്രതിയിൽ അത് ഉൾപ്പെടുത്തണമെന്ന് അവർ ആഗ്രഹിക്കുകയും ചെയ്തതാവാം. (ഇങ്ങനെ പല സാധ്യതകളുമുണ്ട്). മുവത്വയുടെ വിശദീകരണത്തിൽ സർഖാനിയും പറഞ്ഞത് ഇത് തന്നെയാണ്. (തുഹ്ഫത്തുൽ അഹ്‌വാദ്വി ബി ശറഹി ജാമിഉ ത്തിര്മിദി, ഹദീഥ് 2908)

ദുർബലപ്പെടുത്തപ്പെട്ട ഒരു പ്രയോഗം മാത്രമാണ് ഖുർആനിൽ നിന്ന് കളഞ്ഞുപോയതായി വിമർശകർ ആരോപിക്കുന്നത് എന്നാണ് ഇത് വ്യക്തമാക്കുന്നത്. ദുർബലപ്പെടുത്തപ്പെട്ട എല്ലാ പ്രയോഗങ്ങളെയും വചനങ്ങളെയും കുറിച്ച് എല്ലാ പ്രവാചകാനുചരന്മാരും എല്ലായ്‌പ്പോഴും അറിഞ്ഞുകൊള്ളണമെന്നില്ല. അതുകൊണ്ട് തന്നെ അവരുടെ സ്വകാര്യകോപ്പികളിൽ അത്തരം വചനങ്ങളും പ്രയോഗങ്ങളും സ്ഥാനം പിടിച്ചിരിക്കാനിടയുണ്ട്. നസ്ഖ് ചെയ്യപ്പെട്ട പ്രയോഗമാണ് ‘അസർ നമസ്കാരത്തിലും’ (وَصَلاَةِ الْعَصْرِ) എന്നത് എന്ന വസ്തുത അറിയാത്തതുകൊണ്ടുണ്ടായ ആയിശാബീവിയുടെ(റ) ഒരു ആശയക്കുഴപ്പം മാത്രമാണ് ഒന്നാമത്തെ ഹദീഥിലുള്ളത്. ഇത്തരം ആശയക്കുഴപ്പങ്ങൾ വ്യക്തിപരമായി സൂക്ഷിച്ചുവെച്ച ഖുർആൻ കോപ്പികളിൽ ഉണ്ടാവാമെന്നത് കൊണ്ട് തന്നെയാണ് സ്വഹാബിമാരെല്ലാം ഏകകണ്ഠമായി അംഗീകരിച്ച ഖുർആൻ കോപ്പി പുറത്തിറങ്ങിയതോടെ അത്തരം സ്വകാര്യകോപ്പികളെല്ലാം നശിപ്പിക്കാൻ ഉഥ്മാൻ (റ) ഉത്തരവിട്ടത്. പ്രസ്തുത ഉത്തരവിന്റെ സൂക്ഷ്മതയും അനിവാര്യതയുമല്ലാതെ മറ്റൊന്നും തന്നെ ഈ ഹദീഥുകൾ വെളിപ്പെടുത്തുന്നില്ല.

ഉഥ്മാനിന്റെ(റ) കാലത്തെ ഖുർആൻ ക്രോഡീകരണവുമായി ബന്ധപ്പെട്ട് ഇബ്നു മസ്‌ഊദ്‌ (റ) നിരവധി വിമർശനങ്ങളുന്നയിച്ചതായി രേഖകളിലുണ്ടല്ലോ. പ്രവാചകശിഷ്യന്മാരിൽ പ്രമുഖനായ അദ്ദേഹം ഇത്തരം വിമർശനങ്ങളുന്നയിച്ചുവെന്നത് ഉഥ്മാൻ (റ) ക്രോഡീകരിച്ച ഖുർആനിന്റെ വിശ്വാസ്യതയെ ചോദ്യം ചെയ്യുന്നില്ലേ?

ഓറിയന്റലിസ്റ്റുകളും അവരിൽ നിന്ന് ഊർജ്ജമുൾക്കൊള്ളുന്ന ക്രൈസ്തവ മിഷനറിമാരും ഉന്നയിക്കുന്ന പ്രധാനപ്പെട്ട ആരോപണങ്ങളിലൊന്നാണിത്. ഉഥ്മാനിന്റെ കാലത്ത് നടന്ന ഖുർആൻ ക്രോഡീകരണത്തിൽ അബ്ദുല്ലാഹി ബിൻ മസ്ഊദ് സംതൃപ്തനായിരുന്നില്ലെന്നും അന്നുണ്ടാക്കിയ മുസ്ഹഫ് അദ്ദേഹം അംഗീകരിച്ചില്ലെന്നും അതിന് കാരണം മുഹമ്മദ് നബി പഠിപ്പിച്ച ഖുർആനുമായി ഉഥ്മാൻ (റ) ക്രോഡീകരിച്ച ഖുർആനിന് കാര്യമായ അന്തരമുള്ളതിനാലാണെന്നുമെല്ലാമാണ് വിമർശനം. അവർ ഉന്നയിക്കുന്ന പ്രമാണങ്ങൾ പരിധോധിക്കുക:

1) ആസ്സുഹ്‌രി ഉബൈദുല്ലാഹി ബ്നു അബ്ദുല്ലാഹിൽ നിന്ന് നിവേദനം ചെയ്യുന്നു: സൈദ് ബിൻ ഥാബിത്ത് മുസ്ഹഫ് പകർത്തിയെഴുതുന്നത് അബ്ദുല്ലാഹി ബിൻ മസ്ഊദ് വെറുത്തിരുന്നു. സൈദ് ബ്നു ഥാബിത്തിനെപ്പറ്റി അദ്ദേഹം പറഞ്ഞു: ‘മുസ്‌ലിം ജനങ്ങളേ, ഈ മനുഷ്യനിൽ നിന്നുള്ള മുസ്ഹഫും പാരായണവും സ്വീകരിക്കാതിരിക്കുവാൻ ശ്രദ്ധിക്കുക. അല്ലാഹുവാണെ! ഞാൻ ഇസ്‌ലാം സ്വീകരിക്കുന്ന കാലത്ത് അയാൾ അവിശ്വാസിയുടെ മുതുകത്തായിരുന്നു.’ വീണ്ടുമൊരിക്കൽ അദ്ദേഹം പറഞ്ഞു: ‘ഇറാഖിലെ ജനങ്ങളേ, നിങ്ങളുടെ കയ്യിലുള്ള മുസ്ഹഫുകൾ നിങ്ങൾ സൂക്ഷിച്ചു വെക്കുക; അതിന്നായി അവ നിങ്ങൾ ഒളിപ്പിച്ച് വെക്കുക. തീർച്ചയായും അല്ലാഹു പറഞ്ഞിരിക്കുന്നു; ആരെങ്കിലും എന്തെങ്കിലും ഒളിപ്പിച്ചുവെച്ചാൽ അയാൾ അതുമായി വിചാരണനാളിൽ വരും. അതിനാൽ നിങ്ങൾ മുസ്ഹഫുമായി അല്ലാഹുവിനെ കണ്ടുമുട്ടുക” (ജാമിഉ ത്തിർമിദിയിൽ സ്വഹീഹായ സനദോടെ നിവേദനം ചെയ്തത്, അബ്‌വാബു തഫ്സീറുൽ ഖുർആൻ ഹദീഥ് 3104)

2) അബ്ദുല്ലാഹി ബിൻ മസ്ഊദ് പറഞ്ഞു: ‘അതിനാൽ കയ്യെഴുത്ത് രേഖകൾ ഒളിപ്പിച്ച് വെക്കുക. സൈദ് ബ്നു ഥാബിത്തിന്റെതിനേക്കാൾ ഞാൻ ഏറ്റവുമധികം സ്നേഹിക്കുന്നയാളുടെ (പ്രവാചകന്റെ) പാരായണപ്രകാരം പാരായണം ചെയ്യാനാണ് ഞാനിഷ്ടപ്പെടുന്നത്. ആരാധനക്കർഹനായുള്ള ഒരേയൊരുവനാണെ! സൈദ് കുട്ടിയായി മറ്റു കുട്ടികളോടൊപ്പം മുടിയും നീട്ടി കളിച്ചുനടന്നിരുന്ന കാലത്ത് അല്ലാഹുവിന്റെ ദൂതന്റെ ചുണ്ടിൽ നിന്ന് എഴുപതിലധികം സൂറത്തുകൾ ഞാൻ പഠിച്ചിട്ടുണ്ട്.” (ഇബ്നു സഅദ്: കിതാബ് ത്വബഖാത്തുൽ കബീർ, ഭാഗം 2, പുറം 444; ഇബ്‌നു അബീ ദാവൂദ്: അൽമസാഹിഫ് പുറം 60)

3) അല്ലാഹുവാണെ! ഞാൻ ഈ കയ്യെഴുത്തുരേഖ അവർക്ക് കൊടുക്കുകയില്ല. അല്ലാഹുവിന്റെ ദൂതൻ വ്യക്തിപരമായി എഴുപതിലധികം സൂറത്തുകൾ എന്നെ പഠിപ്പിച്ചിട്ടുണ്ട്. ഇപ്പോൾ ഞാനീ കയ്യെഴുത്തുരേഖകൾ അവർക്ക് നൽകണമെന്നോ? അല്ലാഹുവാണെ! ഞാൻ അത് അവർക്ക് നൽകുകയില്ല.” (മുസ്തദ് റക് അൽ ഹാക്കിം: ഹദീഥ് 2896 ഇമാമുമാർ ഹാകിമും ദഹബിയും ഇത് സ്വീകാര്യമായ പരമ്പരയോട് കൂടിയുള്ളതാണെന്ന് സാക്ഷ്യപ്പെടുത്തിയിട്ടുണ്ട്)

ഈ വിഷയത്തിൽ സമാനമായ ആശയങ്ങളുൾക്കൊള്ളുന്ന വേറെയും നിവേദനങ്ങളുണ്ട്. ഉഥ്മാനിന്റെ കാലത്ത് സൈദു ബിൻ ഥാബിത്തിന്റെ നേതൃത്വത്തിൽ ക്രോഡീകരിച്ച മുസ്ഹഫുകൾ മാത്രം നിലനിർത്തി മറ്റുള്ളവയെല്ലാം നശിപ്പിക്കണമെന്ന കല്പനയിൽ നീരസം പ്രകടിപ്പിച്ചുകൊണ്ടുള്ള അബ്ദുല്ലാഹിബ്നു മസ്ഊദിന്റെ പ്രതികരണങ്ങൾ വ്യത്യസ്തമായ വാക്കുകളിൽ രേഖപ്പെടുത്തപ്പെട്ട നിവേദനങ്ങൾ. അവയിൽ ചിലവ ദുർബലമായ പാരമ്പരയോട് കൂടിയുള്ളവയാണെങ്കിലും പൊതുവെയുള്ള ആശയം പ്രബലമായ പരമ്പരയോടു കൂടി സ്ഥാപിക്കപ്പെട്ടതാണ്. എന്താണ് ഈ ഹദീഥുകളിൽ നിന്നും നാം മനസ്സിലാക്കേണ്ടത്? സൈദ് ബിൻ ഥാബിത്തിന്റെ നേതൃത്വത്തിൽ ക്രോഡീകരിക്കപ്പെട്ട ഖുർആൻ കയ്യെഴുത്തുരേഖകളിൽ സാരമായ എന്തെങ്കിലും അബദ്ധങ്ങളുണ്ടെന്ന് ഇബ്നു മസ്ഊദിന് അഭിപ്രായമുണ്ടായിരുന്നുവെന്നാണോ? വിമർശകർ തെറ്റിദ്ധരിപ്പിക്കുന്ന ഇബ്നു മസ്ഊദിന്റെ വചനങ്ങളുടെ യഥാർത്ഥ പൊരുളെന്താണ്? താഴെ പറയുന്ന വസ്തുതകൾ ശ്രദ്ധിക്കുക:

ഒന്ന്) ഇവ്വിഷയകമായ ഇബ്നു മസ്ഊദിന്റെ അഭിപ്രായങ്ങളെയും പ്രതിഷേധപ്രകടനങ്ങളെയുമെല്ലാം മൊത്തമായി പരിശോധനാവിധേയമാക്കിയാൽ രണ്ട് പ്രശ്നങ്ങളിലാണ് അദ്ദേഹത്തിന് വിയോജിപ്പുണ്ടായിരുന്നത് എന്ന കാണാനാവും. ഖുർആൻ വിജ്ഞാനീയങ്ങളിൽ അഗ്രഗണ്യനും പ്രവാചകനോടൊപ്പം ജീവിക്കുവാൻ കൂടുതൽ അവസരമുണ്ടായിരുന്നയാളുമായ തന്നെ പരിഗണിക്കാതെ തന്നെക്കാൾ ഏറെ പ്രായം കുറഞ്ഞ സൈദുബ്നു ഥാബിത്തിനെ അതിമഹത്തായ ഖുർആൻ ക്രോഡീകരണദൗത്യത്തിന്റെ നേതൃത്വം ഏൽപിച്ചതിലുള്ള പ്രതിഷേധമാണ് ഒന്നാമത്തേത്. പ്രവാചകനിൽ നിന്ന് നേർക്കുനേരെ കേട്ട് താൻ എഴുതിത്തയ്യാറാക്കിയ തന്റെ ഖുർആൻ കയ്യെഴുത്തുതരേഖ നശിപ്പിക്കുവാനും പകരം സൈദ് ബിൻ ഥാബിത്തിന്റെ നേതൃത്വത്തിലുള്ള സംഘം തയാറാക്കിയ മുസ്ഹഫ് സ്വീകരിക്കുവാനും ആവശ്യപ്പെട്ടതിലുള്ള അമർഷമാണ് രണ്ടാമത്തേത്. ഇതല്ലാതെ ഉഥ്മാനിന്റെ കാലത്ത് നടന്ന ഖുർആൻ സമാഹരണത്തിലോ ക്രോഡീകരണത്തിലോ എന്തെങ്കിലും സാരമായ അപകടങ്ങളുണ്ടായതായോ പ്രവാചകൻ പഠിപ്പിക്കാത്ത എന്തെങ്കിലും ഖുർആനിൽ കടന്നുകൂടിയതായോ പഠിപ്പിച്ച എന്തെകിലും നഷ്ടപ്പെട്ടതായോ അദ്ദേഹം തന്റെ അമർഷപ്രകടനങ്ങളിലൊന്നും സൂചിപ്പിക്കുന്നേയില്ല. ഖുർആനിന്റെ അഖണ്ഡതയെ ചോദ്യം ചെയ്യുവാനുതകുന്ന തെളിവുകളന്വേഷിച്ച് ഇസ്‌ലാമികഗ്രൻഥശേഖരങ്ങളിൽ മുങ്ങിത്തപ്പുന്നവർക്ക് ഇബ്നു മസ്ഊദിന്റെ പ്രതിഷേധപ്രകടനങ്ങളിൽ നിന്ന് ലഭിക്കുക യാതൊരു ഉപകാരവുമില്ലാത്ത വെറും പായലുകൾ മാത്രമായിരിക്കും.

രണ്ട്) “അല്ലാഹുവിന്റെ ഗ്രന്ഥത്തെ സംബന്ധിച്ച് ഞാനാണ് ഏറ്റവുമധികം അറിവുള്ളവനെന്ന് പ്രവാചകാനുചരന്മാർക്ക് അറിയാവുന്ന കാര്യമാണ്. എന്നേക്കാള്‍ ഈ വിഷയത്തില്‍ അറിവുള്ളവരുണ്ടെങ്കിൽ അവന്റെയടുത്തേക്ക് യാത്രചെയ്തെത്തുവാൻ ഞാൻ സന്നദ്ധനാണ്.” (ഇബ്‌നു അബീ ദാവൂദ് 58) എന്ന ഇബ്നു മസ്ഊദിന്റെ സ്വയംസാക്ഷ്യം നൂറുശതമാനം സത്യസന്ധമാണെന്ന് ഇസ്‌ലാമികപ്രമാണങ്ങളും ചരിത്രവും പഠിക്കുന്ന ആർക്കും മനസ്സിലാവും. “ഞാനും എന്റെ സഹോദരനും യമനില്‍ നിന്നും നബി(സ)യുടെ സമീപം വന്ന സമയത്ത് ഇബ്‌നു മസ്ഊദും(റ) അദ്ദേഹത്തിന്റെ മാതാവും നബി(സ)യുടെ വീട്ടിലെ സ്ഥിരമായ സന്ദർശനങ്ങൾ വഴിയും അവരോടുള്ള നബിയുടെ ബന്ധം വഴിയും അവർ നബിഗൃഹത്തിലെ അംഗങ്ങളാണെന്ന് ഞങ്ങള്‍ ധരിച്ചിരുന്നു”വെന്ന അബൂമൂസൽ അശ്അരിയുടെ വർത്തമാനത്തിൽ (സ്വഹീഹുൽ ബുഖാരി, കിതാബുൽ മഗാസി; സ്വഹീഹു മുസ്‌ലിം, കിതാബ് ഫദാഇലു സ്സ്വഹാബ) നിന്ന് നബിയുമായുള്ള ഇബ്നു മസ്ഊദിന്റെ ബന്ധത്തിന്റെ വലിപ്പം നമുക്ക് മനസ്സിലാക്കാൻ കഴിയും.” അബ്ദുല്ലാഹിബ്നു മസ്ഊദ്, മുആദ് ബിൻ ജബൽ, ഉബയ്യു ബ്നു കഅബ്, അബൂ ഹുദൈഫയുടെ സ്വാതന്ത്രനാക്കിയ അടിമ സാലിം എന്നീ നാലു പേരിൽ നിന്ന് നിങ്ങൾ ഖുർആൻ പഠിക്കുക”യെന്ന അബ്ദുല്ലാഹി ബ്നു അംറ് നിവേദനം ചെയ്ത പ്രവാചകനിർദേശം(സ്വഹീഹുൽ ബുഖാരി, കിതാബു ഫദാഇലിൽ ഖുർആൻ; ജാമിഉ ത്തിർമിദി, കിതാബുൽ മനാഖിബ്) ഇബ്നു മസ്ഊദിന്റെ ഖുർആൻ വിജ്ഞാനത്തിനുള്ള മഹാസാക്ഷ്യമായി നിലകൊള്ളുന്നുമുണ്ട്. അതുകൊണ്ട് തന്നെ ഖുർആനികവിജ്ഞാനീയങ്ങളിലും വ്യാഖ്യാനത്തിലുമെല്ലാം ഇബ്നു അബ്ബാസ് കഴിഞ്ഞാൽ ഏറ്റവുമധികം പരിഗണിക്കപ്പെടുന്നത് ഇബ്നു മസ്ഊദിന്റെ അഭിപ്രായങ്ങളെയാണ്.

പ്രവാചകാനുചാരന്മാരിലെ മഹാപണ്ഡിതനായിരുന്ന ഇബ്നു മസ്ഊദിന് ഖുർആൻ ക്രോഡീകരണത്തിന് നേതൃത്വം വഹിക്കുവാൻ തീർച്ചയായും അവകാശമുണ്ട്. അവിടെ താൻ പരിഗണിക്കപ്പെട്ടിട്ടില്ലെന്ന് തോന്നിയാൽ പ്രതിഷേധിക്കാനുള്ള സ്വാതന്ത്ര്യവുമുണ്ട്. തന്നെ അവഗണിച്ചുകൊണ്ട് തന്നെക്കാളും ഏറെ പ്രായം കുറഞ്ഞ ഒരാളെ ഖുർആൻ പ്രതികളുണ്ടാക്കുകയെന്ന മഹാദൗത്യം ഏൽപിച്ചപ്പോൾ അതിനേക്കാൾ അക്കാര്യത്തിൽ പരിഗണിക്കപ്പെടേണ്ടിയിരുന്നത്‌ താനായിരുന്നുവെന്ന് ആ മഹാപണ്ഡിതൻ വിചാരിച്ചത് സ്വാഭാവികമാണ്. പ്രസ്തുത സ്വാഭാവികതയിൽ കവിഞ്ഞ യാതൊന്നും തന്നെ അദ്ദേഹത്തിന്റെ പ്രതിഷേധപ്രതികരണത്തിൽ കാണാൻ കഴിയില്ല.

മൂന്ന്) എന്തുകൊണ്ടാണ് ഉഥ്മാൻ ഖുർആൻ ക്രോഡീകരണത്തിന്റെ ചുമതല ഇബ്നു മസ്ഊദിനെ ഏൽപ്പിക്കാതെ സൈദു ബ്നു ഥാബിത്തിനെ ഏൽപിച്ചത്? ഉഥ്മാനിന്റെ ന്യായീകരണങ്ങൾ ഇമാം ഇബ്നു ഹജറുൽ അസ്ഖലാനി തന്റെ ഫത്ഹുൽ ബാരിയിൽ വിശദീകരിക്കുന്നുണ്ട്. അവ ഇങ്ങനെയാണ്: ഖുർആൻ സമാഹരണത്തിനും ക്രോഡീകരണത്തിനും മദീനയിൽ വെച്ച് ഉഥ്മാൻ തീരുമാനിച്ച സന്ദർഭത്തിൽ ഇബ്നു മസ്ഊദ് ഇറാഖിലുള്ള കൂഫയിലാണ് ഉണ്ടായിരുന്നത്. അബൂബക്കറിന്റെ കാലത്തെ സുഹ്‌ഫിൽ നിന്ന് എല്ലാവർക്കും പാരായണം ചെയ്യാൻ കഴിയുന്ന രീതിയിലുള്ള പകർപ്പെടുക്കുകയായിരുന്നു ഉഥ്മാനിന്റെ ഉദ്ദേശ്യം. സൈദു ബ്നു ഥാബിത്തായിരുന്നു അബൂബക്കറിന്റെ കാലത്തെ ഖുർആൻ ക്രോഡീകരണത്തിന് നേതൃത്വം വഹിച്ചത്. അതുകൊണ്ട് തന്നെ ഇക്കാര്യത്തിനും ഏറ്റവും അനുയോജ്യം അദ്ദേഹം തന്നെയാണെന്ന് ഉഥ്മാൻ കരുതിയത് സ്വാഭാവികമാണ്. പ്രവാചകന്റെ വഹ്‌യ് എഴുത്തുകാരനായിരുന്നുവെന്ന യോഗ്യത കൂടിയുണ്ട് സൈദിന്. (ഫതഹുല്‍ ബാരി 9/19-20)

അബൂബക്കറിന്റെ കാലത്ത് വ്യത്യസ്തങ്ങളായ വസ്തുക്കളിൽ രേഖീകരിക്കപ്പെട്ടിരുന്ന ഖുർആൻ കയ്യെഴുത്തുകളെയെല്ലാം ഒരുമിച്ച് കൂട്ടി ഒരൊറ്റ ഗ്രൻഥമാക്കുകയെന്ന ദുഷ്കരമായ ജോലി ഏൽപിക്കുമ്പോൾ അബൂബക്കറും ഉമറും കൂടി സൈദിനോട് പറഞ്ഞ കാര്യങ്ങൾ ഇവിടെയും ബാധകമാണ്. “നീ ബുദ്ധിമാനായ യുവാവാണ്. നിന്നെ ഞങ്ങള്‍ തെറ്റിദ്ധരിക്കുകയില്ല. നീ നബി(സ)ക്കായി ദിവ്യസന്ദേശങ്ങള്‍ എഴുതിയിരുന്നു. ആയതിനാല്‍ ക്വുര്‍ആന്‍ രേഖകള്‍ അന്വേഷിച്ചു കണ്ടെത്തി ഒരുമിച്ചു കൂട്ടുക” (സ്വഹീഹുൽ ബുഖാരി, കിതാബുൽ അഹ്‌കാം, ബാബു യൂസ്തഹബ്ബു് ലിൽ കാത്തിബി അൻ യക്കൂന അമീനൻ ആഖിലൻ; ജാമിഉത്തിർമിദി, കിതാബു ത്തഫ്സീർ). ഖുർആൻ സമാഹരണവിഷയത്തിൽ ബുദ്ധിയും കൂർമ്മതയും ഉള്ളയാളെന്ന് തന്റെ മുൻഗാമികൾ സാക്ഷീകരിച്ച വ്യക്തിയെത്തന്നെ അതിന്റെ അടുത്ത ഘട്ടവും ഏൽപിക്കുന്നതാവും ഉത്തമമെന്ന് ഉഥ്മാൻ(റ) കരുതിയിരിക്കണം. ഇബ്നു മസ്ഊദ് (റ) അടക്കമുള്ള സ്വഹാബിമാരൊന്നും തന്നെ അബൂബക്കറി(റ)ന്റെ കാലത്തെ ക്രോഡീകരണത്തിന്റെ നേതൃത്വം സൈദിനെ ഏൽപിച്ചത് ശരിയായില്ല എന്ന് പറഞ്ഞിട്ടുമില്ലെന്ന സത്യം അക്കാര്യത്തിൽ രണ്ടാമത് ഒരു ആലോചന വേണ്ടതില്ലെന്ന തീരുമാനത്തിന് ഉഥ്മാനെ പ്രേരിപ്പിച്ചിരിക്കണം. പ്രവാചകനിൽ നിന്ന് അവസാനമായി ഖുർആൻ കേട്ടവരിൽ ഒരാളായ സൈദിനെ ഈ ഉത്തരവാദിത്തം ഏൽപിച്ചതിൽ ഇബ്നു മസ്ഊദ് ഒഴിച്ച് മറ്റൊരു സ്വഹാബിയും എതിരഭിപ്രായങ്ങളൊന്നും പറഞ്ഞിട്ടില്ലെന്നതു തന്നെ ഉഥ്മാനിന്റെ തീരുമാനം തന്നെയായിരുന്നു ശരിയെന്ന യാഥാർഥ്യം ബോധ്യപ്പെടുത്തുന്നുണ്ട്.

ഖുർആൻ ലിഖിതങ്ങളുടെ ശേഖരണത്തിനും പകർത്തിയെഴുത്തിനും ആവശ്യം ഖുർആനികവിജ്ഞാനീയങ്ങളിലുള്ള വിവരത്തെക്കാൾ ഭാഷാജ്ഞാനവും കയ്യെഴുത്തിനുള്ള കഴിവുമാണ്. ഇക്കാര്യത്തിൽ സൈദ് അഗ്രഗണ്യനായിരുന്നുവെന്ന് പ്രവാചകന്റെ വചനങ്ങളും ചെയ്തികളുമെല്ലാം സാക്ഷ്യം വഹിക്കുന്നുണ്ട്. സൈദ് ബിന്‍ ഥാബിതിൽ നിന്ന് തന്നെയുള്ള രണ്ട് നിവേദനങ്ങൾ ഇക്കാര്യം വ്യക്തമാക്കുന്നുണ്ട്. 1) നബി (സ) എന്നോട് ചോദിച്ചു. നിനക്ക് സുറിയാനി ഭാഷ അറിയുമോ? ഞാന്‍ പറഞ്ഞു, ഇല്ല. നബി (സ) പറഞ്ഞു: നീ അത് പഠിക്കുക, എനിക്ക് ആ ഭാഷയില്‍ ചില കത്തുകള്‍ വരാറുണ്ട്. ഞാന്‍ പതിനേഴ് ദിവസങ്ങൾ കൊണ്ട് അത് പഠിച്ചു. (ഇബ്‌നു ഹിബ്ബാന്‍; സ്വഹീഹാണെന്ന് ശുഐബുല്‍ അര്‍നാഊഥ് 7136). 2) “അല്ലാഹുവിന്റെ ദൂതൻ ജൂതന്മാർ അദ്ദേഹത്തിനെഴുതിയ എഴുത്തുകളിലെ ചില പ്രസ്താവനകൾ പഠിക്കാൻ എന്നോടാവശ്യപ്പെട്ടു. അദ്ദേഹം പറഞ്ഞു: അല്ലാഹുവാണെ! എന്റെ കത്തുകളുടെ കാര്യത്തിൽ ഞാൻ ജൂതന്മാരെ വിശ്വസിക്കുന്നില്ല.’ അര മാസം കഴിയുന്നതിന് മുൻപ് ഞാൻ അവ പഠിച്ചെടുത്തു. പിന്നീട് ജൂതന്മാർക്കായി എന്തെങ്കിലും എഴുതേണ്ടതുണ്ടെങ്കിൽ പ്രവാചകനിർദേശപ്രകാരം ഞാൻ അവർക്ക് അത് എഴുതും; അദ്ദേഹത്തിന് അവർ എന്തെങ്കിലും എഴുതിയാൽ ഞാൻ അത് വായിച്ച് കൊടുക്കും” (ജാമിഉ ത്തിർമിദി; കിതാബുൽ ഇസ്‌തിഅദാൻ; ഹസൻ ആയ സനദോടെയുള്ളത്). ഖുർആൻ അവതരിക്കപ്പെടുന്ന സന്ദർഭങ്ങളിൽ പ്രവാചകൻ പ്രത്യേകമായി ആരെയെങ്കിലും വിളിച്ച് അവ എഴുതാൻ ആവശ്യപ്പെടുമ്പോഴെല്ലാം അദ്ദേഹം വിളിച്ചിരുന്നത് സൈദിനെയായിരുന്നുവെന്നതിൽ നിന്ന് (സ്വഹീഹുൽ ബുഖാരി, കിതാബ് ഫദാഇലിൽ ഖുർആൻ, ബാബു നുസൂലുൽ വഹ്‌യി വ അവ്വ ലു മാ നസല) അദ്ദേഹത്തിന്റെ കയ്യെഴുത്ത് സാമർഥ്യം എത്രത്തോളമായിരുന്നുവെന്ന് നമുക്ക് വായിച്ചെടുക്കാൻ കഴിയും. ഖുർആൻ പകർത്തിയെഴുത്തിന് തികച്ചും അനുയോജ്യനായ വ്യക്തിയെത്തന്നെയാണ് ഉഥ്മാൻ (റ) തെരെഞ്ഞെടുത്തതെന്ന സത്യമാണ് ഇവിടെയും വെളിപ്പെടുന്നത്.

ഭാഷയിലും കയ്യെഴുത്തിലും മാത്രമല്ല മതത്തിലും അഗാധമായ പാണ്ഡിത്യമുള്ളയാളായിരുന്നു സൈദു ബിൻ ഥാബിത്ത്‌. “ഈ സമുദായത്തിലെ അഗാധ പണ്ഡിതന്‍ മരണപ്പെട്ടിരിക്കുന്നു” വെന്ന സൈദുബ്നു ഥാബിത്തിന്റെ മരണസന്ദർഭത്തിലുള്ള അബൂ ഹുറൈറയുടെ(റ) പ്രതികരണം (സിയറു അഅ്‌ലാമി നുബലാഅ് 2/439) മാത്രം മതി അദ്ദേഹത്തിന്റെ പാണ്ഡിത്യത്തിന്റെ ആഴം മനസ്സിലാക്കുവാൻ. ഈ ഉമ്മത്തിലെ പ്രധാനപ്പെട്ട പണ്ഡിതരില്‍ സൈദ് ബിന്‍ സാബിത്തി(റ)നെ എണ്ണിയതായി കാണാം. (മജ്മഉല്‍ ഇമാം ഹൈസമി 9/163)ജനങ്ങള്‍ മതപരമായ പ്രശ്‌നപരിഹാരം തേടി സൈദി(റ)നെ സമീപിച്ചതായി കാണാം. (സ്വഹീഹ് ഇബ്‌നു മാജ 62) ഉമര്‍ (റ) യാത്ര പോകുമ്പോള്‍ ധാരാളമായി സൈദ് ബിന്‍ സാബിത്തി(റ)നെ പ്രതിനിധിയായി നിയമിച്ചിരുന്നു. (സിയറു അഅ്‌ലാമി നുബലാഅ് 2/434)

ഖസ്‌റജ് ഗോത്രക്കാര്‍ അഭിമാനത്തോടെ പറഞ്ഞു: നബി(സ)യുടെ കാലഘട്ടത്തില്‍ ക്വുര്‍ആന്‍ സമ്പൂര്‍ണമായി ഒരുമിച്ച് കൂട്ടിയത് ഞങ്ങളില്‍പ്പെട്ട നാലു പേരാണ്. ഇത് മറ്റാര്‍ക്കുമില്ല. അഥവാ സൈദു ബിന്‍ സാബിത്ത്, അബൂ സൈദ്, ഉബയ്യബിന്‍ കഅ്ബ്, മുആദ് ബിന്‍ ജബല്‍ (ബസാര്‍ 7090, ത്വഹാവി 10/374, ത്വബ്‌റാനി 3488) നബി (സ) പറഞ്ഞു: അനന്തരാവകാശ വിഷയവുമായി ബന്ധപ്പെട്ട വിഷയങ്ങളില്‍ ഏറ്റവും പ്രാവിണ്യമുള്ളയാള്‍ സൈദ് ബിന്‍ സാബിത്ത് (റ) ആണ്. (മുശ്കിലുല്‍ ആഥാർ 810. സ്വഹീഹാണ്)

സാഖിത് ബിന്‍ ഉബൈദ് (റ) പറയുന്നു. ഒരു സദസ്സില്‍ ഏറ്റവും മാന്യതയുള്ളവരും ഒരു ഭവനത്തില്‍ ഏറ്റവും പാണ്ഡിത്യം ഉള്ളവരുമായി സൈദ് ബിന്‍ സാബിത്തി(റ)നെ പോലെ ഞാനൊരാളെയും ദര്‍ശിച്ചിട്ടില്ല. (അല്‍ അദബുല്‍ മുഫ്‌റദ് 219. സ്വഹീഹാണ്).

ഇബ്‌നു മസ്ഊദി(റ)ന്റെ ശിഷ്യനായ മസ്‌റൂക് (റ) മദീനയില്‍ വന്നു. സൈദി(റ)ന്റെ പാണ്ഡിത്യത്തില്‍ അത്ഭുതപ്പെട്ടു. ശിഷ്യന്‍മാര്‍ ചോദിച്ചു. താങ്കള്‍ ഇബ്‌നു മസ്ഊദി(റ)ന്റെ അഭിപ്രായം ഒഴിവാക്കുന്നോ? മസ്‌റൂക് പറഞ്ഞു. സൈദി(റ)ന് അറിവില്‍ അവഗാഹമുള്ളതായി ഞാന്‍ ഗണിക്കുന്നു. (സിയറു അഅ്‌ലാമി നിബലാഅ് 2/437. സ്വഹീഹാണ്). ഭാഷയിലുള്ള കഴിവും എഴുത്തിനുള്ള പാടവവും മതത്തിലുള്ള അഗാധ ജ്ഞാനവുമുള്ള ആളെത്തന്നെയാണ് ഖുർആൻ ക്രോഡീകരണത്തിന്റെ നേതൃത്വത്തിന് ഉഥ്മാൻ തെരഞ്ഞെടുത്തതെന്ന് ഇതിൽ നിന്ന് സുതരാം വ്യക്തമാകുന്നുണ്ട്.

നാല്) യഥാർത്ഥത്തിൽ തന്റെ കൈവശമുള്ള ഖുർആനിന്റെ സ്വകാര്യകോപ്പി നശിപ്പിക്കണമെന്ന് ആവശ്യപ്പെട്ടതാണ് മറ്റെന്തിനേക്കാളുമധികം ഇബ്നു മസ്ഊദിനെ ചൊടിപ്പിച്ചതെന്ന് നടേ സൂചിപ്പിച്ച ഹദീഥുകളുടെ വരികൾക്കിടയിലൂടെ വായിച്ചാൽ ബോധ്യമാവും. അബൂബക്കറിന്റെ കാലത്തെ ക്രോഡീകരണത്തിനും നേതൃത്വം നൽകിയത് സൈദു ബിൻ ഥാബിത്ത് തന്നെയായിരുന്നുവല്ലോ. അപ്പോഴൊന്നും തന്നെ അദ്ദേഹത്തെക്കുറിച്ച് എന്തെങ്കിലും ഒരു ആക്ഷേപം ഇബ്നു മസ്ഊദ് ഉന്നയിച്ചതായി യാതൊരു രേഖയുമില്ല. അതിനർത്ഥം സൈദിന്റെ നേതൃത്വത്തിനല്ല ഇബ്നു മസ്ഊദ് കുഴപ്പം കണ്ടത് എന്നാണ്. സൈദിനെക്കുറിച്ച വിമർശനവാക്കുകൾ അദ്ദേഹം പറയുന്നത് സൈദ് തന്റെ ദൗത്യം പൂർത്തിയാക്കിയതിന് ശേഷമാണ്; അക്കാര്യം ഏല്പിച്ച ഉടനെയല്ല. സൈദിന്റെ ദൗത്യം പൂർത്തിയാക്കിയതിനു ശേഷമാണ് സ്വകാര്യകോപ്പികൾ നശിപ്പിക്കുവാനുള്ള ഖലീഫയുടെ ഉത്തരവുണ്ടാവുന്നത്. അപ്പോൾ തീർച്ചയായും ഇബ്നു മസ്ഊദിന് ദേഷ്യം പിടിച്ചിട്ടുണ്ടാവണം. പ്രവാചകസന്നിധിയിൽ വെച്ച് താൻ കേട്ട് എഴുതിയെടുത്ത, തന്റെ സ്വകാര്യപാരായണത്തിനും പഠനത്തിനും താൻ ഉപയോഗിക്കുന്ന ഖുർആൻ കയ്യെഴുത്തുരേഖയാണ് ഖലീഫ നശിപ്പിക്കുവാൻ ആവശ്യപ്പെട്ടിരിക്കുന്നത്. അദ്ദേഹത്തെ അത് വേദനിപ്പിച്ചിരിക്കണം. പ്രവാചകനോടൊപ്പം ദീർഘകാലം ജീവിച്ച തന്റെ കൈവശമുള്ള കോപ്പി നശിപ്പിച്ച് പകരം തന്റെ മകനാകാൻ മാത്രം പ്രായമുള്ള സൈദിന്റെ നേതൃത്വത്തിൽ നിർമ്മിച്ച മുസ്ഹഫ് താൻ പാരായണം ചെയ്യണമെന്ന ഖലീഫാനിർദേശം ആ വന്ദ്യവയോധികനായ പ്രവാചകാനുചരനെ ചൊടിപ്പിച്ചതിൽ നിന്നാണ് സൈദിനെതിരെയുള്ള അദ്ദേഹത്തിന്റെ പ്രതികരണങ്ങളെല്ലാമുണ്ടായതെന്നാണ് മനസ്സിലാകുന്നത്. ആ പ്രതികരണം പ്രവാചകനോടും അദ്ദേഹത്തിൽ നിന്ന് താൻ കേട്ടെഴുതിയ ഖുർആൻപ്രതിയോടുമുള്ള സ്നേഹത്തിൽ നിന്നുണ്ടായതാണ്. അതുകൊണ്ട് തന്നെ അദ്ദേഹത്തെ വിമർശിക്കുകയല്ല, അനുനയിപ്പിക്കുകയാണ് മറ്റു സ്വഹാബിമാർ ചെയ്തത്.

തന്റെ കയ്യെഴുത്ത് പ്രതി നശിപ്പിക്കാതെ സൂക്ഷിച്ചുവെക്കാൻ തീരുമാനിച്ചതിനോടൊപ്പം, സ്വകാര്യകയ്യെഴുത്തുരേഖകൾ കൈവശമുള്ളവരോടെല്ലാം അവ മറച്ചുവെക്കുകയും സൂക്ഷിച്ചുവെക്കുകയും ചെയ്യണമെന്ന് ആവശ്യപ്പെടുകയുമാണ് ഇബ്നു മസ്ഊദ് ചെയ്തത്. അതല്ലാതെ സൈദ് ബ്നു ഥാബിത്തിന്റെ നേതൃത്വത്തിൽ ക്രോഡീകരിക്കപ്പെടുകയും കോപ്പികളെടുക്കുകയും ചെയ്ത മുസ്ഹഫിൽ ഗുരുതരമായ എന്തെങ്കിലും സ്ഖലിതങ്ങളുള്ളതായി അദ്ദേഹം എവിടെയും പറഞ്ഞിട്ടില്ല; അങ്ങനെയുള്ള സൂചനകൾ പോലും നൽകിയിട്ടില്ല.

അഞ്ച്) തന്റെ കയ്യിലുള്ള സ്വകാര്യകയ്യെഴുത്തുപ്രതി നശിപ്പിക്കുകയില്ലെന്ന് പ്രഖ്യാപിക്കുകയും അങ്ങനെ ചെയ്യേണ്ടതില്ലെന്ന് മറ്റുള്ളവരോട് ആഹ്വാനം നടത്തുകയും ചെയ്ത ഇബ്നു മസ്ഊദിന്റെ നടപടി ശരിയായില്ലെന്നായിരുന്നു സ്വഹാബിമാരിൽ മിക്കവരുടെയും അഭിപ്രായം. ഇക്കാര്യം ഇബ്നു മസ്ഊദിന്റെ പ്രതിസ്വരം രേഖപ്പെടുത്തിയ ഇമാം സുഹ്‌രി തന്നെ വ്യക്തമാക്കുന്നുണ്ട്. “അല്ലാഹുവിന്റെ ദൂതന്റെ അനുചരന്മാരിൽ ഉന്നതമായ നിലവാരത്തിലുള്ള പലരും ഇബ്നു മസ്ഊദിന്റെ ഈ വീക്ഷണത്തെ ഇഷ്ടപ്പെട്ടിരുന്നില്ല” (ജാമിഉ ത്തിർമിദി, അബ് വാബു തഫ്സീറുൽ ഖുർആൻ ഹദീഥ് 3104). ഖുർആനിന്റെ ഉള്ളടക്കത്തിൽ എന്തെങ്കിലും കൈകടത്തലുകളുണ്ടായിയെന്നായിരുന്നു അദ്ദേഹത്തിന്റെ ആരോപണമെങ്കിൽ സ്വഹാബിമാർ ഒന്നുകിൽ അദ്ദേഹത്തോടോപ്പമുണ്ടാകുമായിരുന്നു; അതല്ലെങ്കിൽ അദ്ദേഹത്തിനെതിരെ കടുത്ത വിമർശങ്ങൾ ഉന്നയിക്കുമായിരുന്നു. ഖുർആനിന്റെ ഉള്ളടക്കത്തിൽ സംശയിക്കുകയെന്നത് മുസ്‌ലിംസമൂഹം മതത്തിൽ നിന്ന് പുറത്താകുന്ന പ്രവർത്തനമായാണ് അന്നും ഇന്നും കരുതിപ്പോന്നിട്ടുള്ളത്.

ആറ്) കയ്യെഴുത്തുരേഖകൾ കൈവശമുള്ളവർ അവ നശിപ്പിക്കേണ്ടതില്ലയെന്ന തന്റെ നിലപാടിൽ നിന്ന് ഇബ്നു മസ്ഊദ് തന്നെ പിൽക്കാലത്ത് പിൻവലിയുകയും മുസ്‌ലിംസമൂഹത്തിന്റെ പൊതുനിലപാട് തന്നെയാണ് ശരിയെന്ന് അംഗീകരിക്കുകയും ചെയ്തിട്ടുണ്ട്. ഇക്കാര്യം ഇബ്നു കഥീർ തന്റെ ‘അൽ ബിദായ വ ന്നിഹായ’യിൽ രേഖപ്പെടുത്തിയിട്ടുണ്ട്. “ഉഥ്മാൻ (റ) അദ്ദേഹത്തിന് (ഇബ്നു മസ്ഊദ്) മറ്റു പ്രവാചകാനുചരന്മാരെ താങ്കളും അനുധാവനം ചെയ്യണമെന്ന് ആവശ്യപ്പെട്ടുകൊണ്ട് എഴുതി. സ്വഹാബിമാരെല്ലാം എന്തുകൊണ്ടാണ് പ്രസ്തുത തീരുമാനത്തിലെത്തിയതെന്നും അതുകൊണ്ടുണ്ടാവുന്ന നേട്ടങ്ങളും വിരുദ്ധാഭിപ്രായങ്ങളില്ലാതെയാക്കി ഖുർആനിന്റെ വിഷയത്തിൽ ഒരേ അഭിപ്രായത്തിലേക്ക് എല്ലാവരും എത്തേണ്ടതിന്റെ ആവശ്യകതയുമെല്ലാം എഴുത്തിൽ പരാമർശിച്ചിരുന്നു. അദ്ദേഹത്തിന് കാര്യങ്ങൾ മനസ്സിലാവുകയും എതിർപ്പുകൾ വെടിഞ്ഞ് ഐക്യപ്പെടാൻ അദ്ദേഹം അങ്ങനെ തീരുമാനിക്കുകയും ചെയ്തു.”(7/217)

ശരിയാണ്; പ്രവാചകനിൽ നിന്ന് നേരിട്ട് കേട്ട് താൻ എഴുതിയ ഖുർആനിന്റെ സ്വന്തം കയ്യെഴുത്തുപ്രതി നശിപ്പിക്കാൻ ഇബ്നു മസ്ഊദിന് സമ്മതമുണ്ടായിരുന്നില്ല; തന്റെ മകന്റെ മാത്രം പ്രായമുള്ള സൈദ് ക്രോഡീകരിച്ച പ്രതി സ്വീകരിച്ച് പകരം തന്റെ സ്വന്തം പ്രതി നശിപ്പിക്കില്ലെന്ന് അദ്ദേഹം പരസ്യമായി പ്രഖ്യാപിക്കുകയും കയ്യെഴുത്ത് പ്രതികൾ കൈവശമുള്ളവരൊന്നും അവ നശിപ്പിക്കരുതെന്ന് ആഹ്വാനം നടത്തുകയും ചെയ്തിരുന്നു. എന്നാൽ സൈദ് ക്രോഡീകരിച്ച ഖുർആൻ കൈയ്യെഴുത്ത് രേഖയിൽ ഗുരുതമായ എന്തെങ്കിലും പ്രശ്നങ്ങളുള്ളതായി അദ്ദേഹം ആരോപിച്ചിട്ടില്ല. പ്രവാചകാനുചരന്മാർ മൊത്തത്തിലെടുത്ത തീരുമാനത്തിനെതിരെയുള്ള തന്റെ പ്രതിസ്വരം ശരിയല്ലെന്ന് ബോധ്യപ്പെട്ടതോടെ അദ്ദേഹം അത് തിരുത്തുകയും ചെയ്തിട്ടുണ്ട്. മൂർഖനനെ ലഭിക്കുമെന്ന് കരുതി ഇസ്‌ലാമികഗ്രൻഥങ്ങളുടെ മാളങ്ങളിലെല്ലാം തപ്പി നോക്കിയിട്ട് വിമർശകർക്ക് ലഭിച്ചത് കേവലമൊരു ഞാഞ്ഞൂൽ മാത്രമാണെന്ന സത്യം ഖുർആനിന്റെ അബദ്ധങ്ങളില്ലാതെയുള്ള സംപ്രേഷണത്തെക്കുറിച്ച മതിപ്പ് വർധിപ്പിക്കാൻ മാത്രമേ നിമിത്തമാവുകയുള്ളൂ.

അബൂബക്കറി(റ)ന്റെ കാലത്തെ ഖുർആൻ ക്രോഡീകരണത്തെ അലി (റ) വിമർശിച്ചതായുള്ള നിവേദനവും അബൂബക്കറല്ല ഖുർആൻ ആദ്യമായി ക്രോഡീകരിച്ചത് എന്നുള്ള നിവേദനവുമെല്ലാം ഈ രംഗത്ത് അവ്യക്തതകളുണ്ടെന്നല്ലേ വ്യക്തമാക്കുന്നത്? യഥാർത്ഥത്തിൽ അബൂബക്കറി(റ)ന്റെ കാലത്തെ ഖുർആൻ ക്രോഡീകരണത്തെ അനുകൂലിക്കുകയും അദ്ദേഹം ചെയ്തത് വലിയ പുണ്യമാണെന്ന് പ്രശംസിക്കുകയും അദ്ദേഹത്തിന് വേണ്ടി പ്രാർത്ഥിക്കുകയുമാണ് അലി (റ) ചെയ്തത്.

അലി (റ) പറഞ്ഞു: മുസ്ഹഫിന്റെ വിഷയത്തില്‍ ഏറ്റവും ഉത്തമമായ പ്രതിഫലം അബൂബക്കറി(റ)നാണ്. അല്ലാഹുവിന്റെ കാരുണ്യം അബൂബക്കറി(റ)ന് ഉണ്ടാകട്ടെ. കാരണം അല്ലാഹുവിന്റെ ഗ്രന്ഥം ആദ്യമായി ഒരുമിച്ച് കൂട്ടിയത് അദ്ദേഹമാണ്. (ഇബ്‌നു അബീ ദാവൂദ് പേജ് 153)

അലി (റ) പറഞ്ഞു: അല്ലാഹു അബൂബക്കറി(റ)ന് കരുണ ചെയ്യട്ടെ. കാരണം അദ്ദേഹമാണ് രണ്ടു ചട്ടകള്‍ക്കുള്ളിലായി ഖുര്‍ആനിനെ ഒരുമിച്ചു കൂട്ടിയത്. (ഇബ്‌നു അബീദാവൂദ്, ഇബ്‌നു അബീ ശൈഖ 6/148, ഇബ്‌നു സഅദ് ത്വബമാതില്‍ 3/193. ഈ ഹദീഥ് സ്വഹീഹാണെന്ന് ഇമാം സുയൂത്വി)

ഇതിനു വിരുദ്ധമായ ചില നിവേദനങ്ങളുണ്ടെന്നത് ശരിയാണ്. അവ ബലഹീനവും തെളിവിന് കൊള്ളാത്തതുമാണെന്ന് പണ്ഡിതന്മാർ സമർത്ഥിച്ചിട്ടുണ്ട്. നിവേദനങ്ങൾ ഇവയാണ്: –

ഇഖ്‌രിമ (റ) യിൽ നിന്ന്: അബൂബക്കറി(റ)നുള്ള ബൈഅത്തിനുശേഷം അലി (റ) വീട്ടില്‍ തന്നെയിരുന്നു. കാരണം തിരക്കിയ ഖലീഫയോട് അലി (റ) പറഞ്ഞു. അല്ലാഹുവിന്റെ ഗ്രന്ഥത്തില്‍ വര്‍ധിക്കപ്പെടുമോ എന്ന് എനിക്ക് തോന്നുന്നു. ആയതിനാല്‍ ഖുര്‍ആന്‍ ഒരുമിച്ച് കൂട്ടുന്നതു വരെ നമസ്‌കാരത്തിനല്ലാതെ ഞാന്‍ മേല്‍വസ്ത്രം ധരിക്കുന്നതല്ല. (ഇബ്‌നു ളരീസ് -അല്‍ ഇത്ഖാന്‍ 1/59)

ഈ ഹദീഥിന്റെ നിവേദക പരമ്പര പൂർണമല്ല. നിവേദകശൃംഖലയിൽ കണ്ണികൾ വിട്ടു പോയിട്ടുണ്ട്. അതുകൊണ്ട് തന്നെ ഇത് ദുർബലാണ്; തെളിവിന് കൊള്ളുകയില്ല. ഇമാം ഇബ്‌നു ഹജര്‍ (റ) തന്റെ ഫതഹുൽ ബാരി 9/12 ഗ്രൻഥത്തിൽ ഇക്കാര്യം വ്യക്തമാക്കിയിട്ടുണ്ട്.

ഇതേപോലെത്തന്നെയാണ് സാലിമാണ് ഖുർആൻ ആദ്യമായി ക്രോഡീകരിച്ചതെന്നും അതല്ല ഉമറാണെന്നുമെല്ലാമുള്ള നിവേദനങ്ങൾ. എല്ലാം ദുർബലവും തെളിവിന് കൊള്ളാത്തവയുമാണ്.

ഇബ്‌നു ബുറൈദ (റ) പറയുന്നു: ഒരു ഏടില്‍ ആദ്യമായി ക്വുര്‍ആന്‍ ഒരുമിച്ച് കൂട്ടിയത് സാലിം (റ) ആണ്. (ഇത്ഖാന്‍- 1/166)

ഈ അഥറും പരമ്പര മുറിഞ്ഞതിനാല്‍ ദുര്‍ബലമാണെന്ന് ഇമാം സുയൂത്വി (റ) പറയുന്നു. (അല്‍ ഇത്ഖാന്‍ പേജ് 382)

ഹസന്‍ (റ) പറയുന്നു: ഒരു ആയത്തിനെ സംബന്ധിച്ച് അന്വേഷിച്ചു. അത് യമാമയില്‍ വധിക്കപ്പെട്ട ഒരു സ്വഹാബിയുടെ കൈവശമുണ്ടായിരുന്നു എന്നു പറയപ്പെട്ടു. അപ്പോള്‍ ഉമര്‍ (റ) ഇന്നാലില്ലാഹ്… എന്നുപറഞ്ഞു. ക്വുര്‍ആന്‍ ഒരുമിച്ചു കൂട്ടാല്‍ കല്‍പിച്ചു. അദ്ദേഹമാണ് ആദ്യമായി ക്വുര്‍ആന്‍ മുസ്ഹഫില്‍ ഏകോപിപ്പിച്ചത്. (ഇബ്‌നു അബീദാവൂദ് അല്‍മസാഹിഫ് 1/181)

അഥര്‍ മുന്‍ഖതി ആണെന്ന് ഇബ്‌നു ബസീര്‍ ഫളഇളുല്‍ ക്വുര്‍ആന്‍ പേജ് 27ലും ഇബ്‌നു ഹജര്‍ ഫതഹുല്‍ ബാരി 9/13ലും ഇമാം സുയൂത്വി (റ) അല്‍ ഇത്ഖാന്‍ പേജ് 382ലും വ്യക്തമാക്കുന്നു. ഈ അഥറും അസ്വീകാര്യമാണ് എന്നർത്ഥം.

ഖുർആൻ പകർത്തിയെഴുതിയപ്പോൾ ഗുരുതരമായ ചില വ്യാകരണപ്പിഴവുകൾ വന്നതായി വ്യക്തമാക്കുന്ന നിവേദനങ്ങളുണ്ടല്ലോ. വ്യാകരണപ്പിഴവുകൾ അടക്കം പകർത്തിയെഴുത്തുകാർക്ക് സംഭവിച്ചിട്ടുണ്ടെങ്കിൽ, ഖുർആൻ അവതരിപ്പിക്കപ്പെട്ട രൂപത്തിൽ സംരക്ഷിക്കപെട്ടുവെന്ന് പറയുന്നതിൽ എന്ത് അർത്ഥമാണുള്ളത്? ഇമാം സുയൂഥ്വിയുടെ ‘അൽ ഇത്ഖാൻ ഫീ ഉലൂമിൽ ഖുർആൻ’ എന്ന ഗ്രന്ഥത്തിൽ ഉദ്ധരിച്ചിരിക്കുന്ന ആയിശയിൽ(റ) നിന്നും ഉഥ്മാനിൽ(റ) നിന്നുമുള്ള രണ്ട് നിവേദനങ്ങളാണ് ഈ വിമർശനത്തിന് ആധാരം. അവ ഇങ്ങനെയാണ്:

1) ഹിഷാമു ബ്നു ഉർവ്വ അദ്ദേഹത്തിന്റെ പിതാവിൽ നിന്ന് നിവേദനം ചെയ്യുന്നു: ഖുർആനിലെ വ്യാകരണപ്പിഴവുകളെക്കുറിച്ച് ഞാൻ ആയിശയോട് ചോദിച്ചു. സൂറത്തുൽ മാഇദയിലെ 69 ആം വചനത്തിലെയും (اِنَّ الَّذِيۡنَ اٰمَنُوۡا وَالَّذِيۡنَ هَادُوۡا وَالصَّابِـُٔـوۡنَ وَالنَّصٰرٰى) സൂറത്തു ന്നിസാഇലെ 162ആം വചനത്തിലെയും (وَالۡمُقِيۡمِيۡنَ الصَّلٰوةَ وَالۡمُؤۡتُوۡنَ الزَّكٰوةَ وَ الۡمُؤۡمِنُوۡنَ بِاللّٰهِ ) സൂറത്തു ത്വാഹയിലെ 63ആം വചനത്തിലെയും (اِنۡ هٰذٰٮنِ لَسٰحِرٰنِ) (വ്യാകരണപ്പിശകുകളെക്കുറിച്ചാണ് ഞാൻ ചോദിച്ചത്.) അപ്പോൾ അവർ പറഞ്ഞു: സഹോദരീപുത്രാ… ഇവ എഴുത്തുകാരുടെ അബദ്ധങ്ങളാണ്; എഴുതിയപ്പോൾ അവർക്ക് അബദ്ധങ്ങൾ സംഭവിച്ചുപോയി” (അല്‍ ഇത്ഖാന്‍ 1/174)

2) ഇബ്നു അബ്ബാസിന്റെ മോചിപ്പിക്കപ്പെട്ട ദാസനായ ഇക്‌രിമയിൽ നിന്ന് നിവേദനം: അദ്ദേഹം പറഞ്ഞു: മുസ്ഹഫുകൾ എഴുതപ്പെട്ടശേഷം അവ ഉഥ്മാനെ കാണിച്ചപ്പോൾ അദ്ദേഹം അതിൽ ചില വ്യാകരണപ്പിശകുകൾ കണ്ടു. അദ്ദേഹം പറഞ്ഞു: നിങ്ങല്‍ അത് മാറ്റേണ്ടതില്ല. അറബികളുടെ ഭാഷാശുദ്ധി അതിനെ ശരിയാക്കിക്കൊള്ളും; അല്ലെങ്കിൽ അവർ അത് ശരിയായി വായിച്ചുകൊള്ളും. എഴുത്തുകാരന്‍ സഖീഫ് ഗോത്രക്കാരനും വായിച്ചുകൊടുത്തത് ബുദൈല്‍ ഗോത്രക്കാരനുമായിരുന്നുവെങ്കില്‍ ഈ പിഴവുകള്‍ ഉണ്ടാകുമായിരുന്നില്ല. (അബൂ ഉബൈദ്: ഫദാഇലുല്‍ ക്വുര്‍ആന്‍ 2/103, ഇബ്‌നു അബീ ദാവൂദ്: അല്‍ മസാഹിഫ് 1/235)

താഴെ പറയുന്ന കാര്യങ്ങൾ ശ്രദ്ധിക്കുക:

ഒന്ന്) ആയിശയിൽ(റ) നിന്നുള്ള ഹിശാമുബ്നു ഉർവ്വയുടെ നിവേദനം രണ്ട് കൂഫീ നിവേദകരിലൂടെയാണ് പണ്ഡിതന്മാർ ഉദ്ധരിച്ചിരിക്കുന്നത്. അബൂ മുആവിയ അദ്ദരീർ ആണ് ഒന്നാമത്തെയാൾ. സഈദു ബിൻ മൻസൂർ തന്റെ സുനനിലും (4/1507) അബൂ ഉബൈദ് തന്റെ ഫദാഇലിൽ ഖുർആനിലും (പുറം 229) ഇമാം ത്വബ്‌രി തന്റെ ജാമിഉൽ ബയാനിലും(9/359) ഇബ്നു അബീ ദാവൂദ് തന്റെ അൽ മസാഹിഫിലു(പുറം 43)മെല്ലാം അബൂ മുആവിയയിൽ നിന്നാണ് ഇത് നിവേദനം ചെയ്തിരിക്കുന്നത്. അലിയ്യു ബ്നു മസ്ഹർ അൽകൂഫിയാണ് രണ്ടാമത്തെ നിവേദകൻ. ഉമർ ബ്നു ശുബ്ബാഹിന്റെ താരീഖുൽ മദീനയിൽ (3/1013-1014) അദ്ദേഹത്തിൽ നിന്നുള്ള നിവേദനമാണുള്ളത്. ഇത്തരം ഒറ്റപ്പെട്ടതും അംഗീകൃതമായ തത്ത്വങ്ങൾക്ക് വിരുദ്ധവുമായ ആശയങ്ങൾ ഉൾക്കൊള്ളുന്ന സംഭവങ്ങൾ ഹിശാമു ബ്നു ഉർവ്വയിൽ നിന്ന് അദ്ദേഹത്തിന്റെ അവസാനകാലത്ത് നിവേദനം ചെയ്യപ്പെട്ടാൽ അത് സ്വീകാര്യമാവുകയില്ലെന്നാണ് ഹദീഥ് ശാസ്ത്രജ്ഞന്മാർ പറയുക. ഇക്കാര്യം ഇമാം ദഹബി തന്റെ ‘മീസാനുൽ ഇഅതിദാൽ’ എന്ന ഗ്രൻഥത്തിൽ വ്യക്തമാക്കുന്നുണ്ട്. “തന്റെ ജീവിതാന്ത്യത്തിൽ ഇറാഖിലേക്ക് വന്ന ശേഷം ഹിശാമു ബ്നു ഉർവ്വ മഹത്തായ നിരവധി വിജ്ഞാനങ്ങൾ പഠിപ്പിച്ചിട്ടുണ്ട്; എന്നാൽ ആ സമയത്ത് അദ്ദേഹം നിവേദനം ചെയ്ത ചില ഹദീഥുകൾ പ്രബലമല്ല. മാലിക്കിനും ശുഅബക്കും വക്കീഇനും മറ്റ് അനവധി പ്രഗത്ഭരും സത്യസന്ധരുമായ പണ്ഡിതർക്കും ഇതേപോലെ സംഭവിച്ചിട്ടുണ്ട്.” (4/301)

പ്രായാധിക്യത്താൽ മറവി പറ്റാൻ സാധ്യതയുള്ളതിനാലും വാർധക്യസഹജമായ പ്രയാസങ്ങളുള്ളതിനാലും മറ്റെവിടെയും പറയാത്ത എന്തെങ്കിലും അത്തരം പണ്ഡിതന്മാർ പ്രായാധിക്യമുള്ള കാലത്ത് പറഞ്ഞാൽ അത് സ്വീകാര്യമല്ല എന്നാണ് ഇപ്പറഞ്ഞതിനർത്ഥം. അത്തരക്കാരുടെ നിവേദങ്ങൾക്ക് ഉപോൽബലകമായി മറ്റ് നിവേദനങ്ങളുണ്ടെങ്കിലേ അവ ഹദീഥ് പണ്ഡിതന്മാർ സ്വീകരിക്കുകയുള്ളൂ. ഇറാഖിലെത്തിയ ശേഷമുള്ള ഹിശാമു ബ്നു ഉർവ്വയുടെ ചില നിവേദനങ്ങൾ സ്വീകാര്യമാണെന്ന് ഹദീഥ് നിദാന ശാസ്ത്രജ്ഞർ പറയുന്നത് അവയെ ബലപ്പെടുത്തുന്ന മറ്റു നിവേദനങ്ങൾ ഉള്ളതിനാലാണ്. എന്നാൽ ആയിശയിൽ നിന്നുള്ള ഈ അഥറിനെ ബലപ്പെടുത്തുന്ന മറ്റു നിവേദനങ്ങൾ ഒന്നും തന്നെയില്ല. അത് കൊണ്ട് തന്നെ ഈ നിവേദനം സ്വീകാര്യമായി ഗണിക്കപ്പെടുകയില്ല.

ഹിശാമുബ്നു ഉർവ്വ മാത്രമല്ല, അദ്ദേഹത്തിൽ നിന്ന് ഈ സംഭവം നിവേദനം ചെയ്ത അബൂ മുആവിയ മുഹമ്മദ് ബിൻ ഖാസിം അദ്ദരീർ അൽ കൂഫിയും ഇത്തരം നിവേദങ്ങളുടെ കാര്യത്തിൽ ഹദീഥ് പണ്ഡിതന്മാർക്ക് സ്വീകാര്യനായ വ്യക്തിയല്ല. അബൂ മുആവിയയിലൂടെയുള്ള ചില നിവേദനങ്ങൾ സ്വീകാര്യമല്ലെന്ന് ഇമാം അഹ്‌മദ്‌ ബിൻ ഹമ്പൽ പറഞ്ഞതായി അദ്ദേഹത്തിന്റെ മകൻ അബ്ദുല്ലാഹ് സാക്ഷ്യപ്പെടുത്തുന്നത് ഇമാം ഇബ്നു ഹജർ ഉദ്ധരിക്കുന്നുണ്ട്. അത് ഇങ്ങനെയാണ്: “അഅമഷ് നിവേദനം ചെയ്തതല്ലാതെയുള്ള അബൂ മുആവിയയിൽ നിന്നുള്ള വിവരണങ്ങളൊന്നും സ്വീകാര്യയോഗ്യമല്ല.” (തഹ്ദീബു ത്തഹ്ദീബ് ഭാഗം 9, പുറങ്ങൾ 138, 139) ഇമാം അബൂ ദാവൂദും ഇക്കാര്യം തന്നെ പറയുന്നുണ്ട്. അദ്ദേഹത്തിൽ നിന്ന് ഇമാം ദഹബി ഉദ്ധരിക്കുന്നു: “ഞാൻ അഹ്‌മദ്‌ ബിൻ ഹമ്പലിനോട് ചോദിച്ചു: ഹിശാമുബ്നു ഉർവ്വയിൽ നിന്നുള്ള അബൂ മുആവിയയുടെ നിവേദനങ്ങളെക്കുറിച്ച് താങ്കൾ എന്ത് കരുതുന്നു? അദ്ദേഹം പ്രതിവചിച്ചു: ഇത്തരത്തിലുള്ള അദ്ദേഹത്തിന്റെ നിവേദനങ്ങളൊന്നും തന്നെ സ്വീകാര്യമല്ല. ഇബ്നു ഖർറാഷിൻറെ അഭിപ്രായപ്രകാരം അഅമഷിലൂടെയാണെങ്കിൽ മാത്രമേ അബൂമുആവിയയുടെ നിവേദനങ്ങളെ ആശ്രയിക്കാൻ പറ്റുകയുള്ളൂ” (മീസാനുൽ ഇഅതിദാൽ, ഭാഗം 4 , പുറം 575) ഏത് നിലയ്ക്ക് നോക്കിയാലും തെളിവിന് കൊള്ളാത്ത ദുർബലമായ നിവേദനമാണ് ഇതെന്ന് വ്യക്തമാണെന്നർത്ഥം.

രണ്ട്) ഉഥ്മാൻ (റ) പറഞ്ഞതായി ഇക്‌രിമയിൽ നിന്നുള്ള നിവേദനമാണ് വ്യാകരണപ്പിശക് ആരോപിക്കുന്നവരുടെ രണ്ടാമത്തെ തെളിവ്. അബൂ ഉബൈദ് തന്റെ ഫദാഇലിൽ ഖുർആനിലും (2/103) ഇബ്നു അബീദാവൂദ് തന്റെ അൽ മസാഹിഫിലും (1/235) ഈ നിവേദനം ഉദ്ധരിച്ചിട്ടുണ്ട്. തീരെ ദുർബലമാണീ നിവേദനം. ഉഥ്മാനിന്റെ അടുത്ത് നിന്ന് നേരിട്ട് ഇക്‌രിമ കേട്ടതല്ല ഇത്. ഉഥ്മാൻ പറഞ്ഞതായി മറ്റാരോ പറഞ്ഞത് കേട്ട് നിവേദനം ചെയ്തതാണ്. ഈ നടുവിലെ വ്യക്തി ആരാണെന്നറിയില്ല. ഇത്തരം നിവേദങ്ങളെയാണ് മുർസൽ എന്ന് പറയുക. ഇക്‌രിമയും ഉഥ്മാനും തമ്മിൽ നിവേദകശ്രംഖലയിൽ ഒരു കണ്ണി വിട്ടുപോയിട്ടുണ്ടെന്ന വസ്തുത ഹദീഥ് പണ്ഡിതന്മാർ വ്യക്തമാക്കിയിട്ടുള്ളതാണ്. നിദാനശാസ്ത്രപടുവായ അബൂ സഈദ് അൽ ആലാഈ തന്റെ ജാമിഅ ത്തഹസീൽ ഫീ അഹ്കാമൽ മറാസീൽ (പുറം 239) എന്ന ഗ്രൻഥത്തിലും ഖുർആൻ രേഖീകരണചരിത്രത്തിലെ പണ്ഡിതനായ അബൂ അംദ്ദാനീ തന്റെ കിതാബുൽ മുഖന്നിഅ (പുറം 115) എന്ന ഗ്രന്ഥത്തിലും ഇക്കാര്യം വ്യക്തമാക്കിയിട്ടുണ്ട്. ഇബ്‌നു അബീദാവൂദിന്റെ ഒരു നിവേദനത്തില്‍ ഇഖ്‌രിമതു ഥാഇയ്യ് എന്നാണ് നിവേദകനെ വിളിച്ചിരിക്കുന്നത്. ഹദീഥ് പണ്ഡിതന്മാർക്ക് പരിചയമില്ലാത്ത പേരാണിത്. ഈ നിവേദനത്തിന്റെ മറ്റൊരു ന്യൂനതയാണിത്. ഇത്തരം നിവേദനങ്ങൾ തെളിവിന് കൊള്ളുന്നതല്ലെന്ന് ഹദീഥ് നിദാനശാസ്ത്രത്തിൽ അടിസ്ഥാന വിവരമെങ്കിലുമുള്ള ആർക്കും മനസ്സിലാവും.

യഹ്‌യാ ബിന്‍ യഅ്മര്‍ വഴിയാണ് ഈ അഥറിന്റെ മറ്റൊരു നിവേദകപരമ്പര. ഇദ്ദേഹം ഉഥ്മാനെ(റ) കണ്ടിട്ടില്ലാത്തതിനാല്‍ പരമ്പര മുറിഞ്ഞതാണ്. ഇമാം ബുഖാരി (റ) താരീഖുല്‍ കബീറി(5/170)ലും ഇബ്നു ഹജറുൽ അസ്ഖലാനി തഹ്ദിബു തഹ്ദീബി (11/305)ലും ഇക്കാര്യം വ്യക്തമാക്കുന്നു. ഇതിന്റെ നിവേദകപരമ്പര അവ്യക്തവും തെളിവിന് കൊള്ളാത്തതുമാണെന്ന് ഇമാം ബാഖില്ലാനി തന്റെ ‘അല്‍ ഇന്‍തിദ്വാർ ലില്‍ ക്വുര്‍ആന്‍ (2/136) ല്‍ വ്യക്തമാക്കിയിട്ടുണ്ട്. മറ്റൊരു നിവേദനം അബ്ദുല്‍ അഅ്‌ലാ വഴിയാണ്. ഇദ്ദേഹത്തെ ഒരു നിരൂപകനും വിശ്വസ്തനായി കാണുന്നില്ല. ആയതിനാല്‍ ഇതും ബലഹീനമാണ്. മറ്റൊരു നിവേദനം ഖതാദ (റ) വഴിയാണ്. ഈ പരമ്പരയിലും അവ്യക്തതയുണ്ട്. കാരണം ഒരു നിവേദകന്‍ പറയുന്നത് നമ്മുടെ അനുയായികള്‍ ഉദ്ധരിച്ചുവെന്നാണ്. ആരാണ് ഈ അനുയായികൾ എന്ന് വ്യക്തമല്ല. അതിനാല്‍ ഇതും ബലഹീനമാണ്. ഖുർആൻ എഴുതിയപ്പോൾ അബദ്ധങ്ങളുണ്ടായിയെന്ന പേരിൽ ഉഥ്മാനിന്റേതായി ഉദ്ധരിക്കപ്പെടുന്ന നിവേദനങ്ങളൊന്നും തെളിവിന് കൊള്ളാത്തതാണ് എന്നാണ് ഇതെല്ലാം വ്യക്തമാക്കുന്നത്.

മൂന്ന്) പകർത്തിയെഴുതുമ്പോൾ ഖുർആനിൽ വ്യാകരണത്തെറ്റുണ്ടായിയെന്ന ആയിശയിൽ നിന്നുള്ളതായി ഉദ്ധരിക്കപ്പെടുന്ന വർത്തമാനം പരിഗണന പോലും അർഹിക്കാത്ത ദുർബലവാദമാണെന്ന് അല്പം ചിന്തിക്കുന്ന ആർക്കും മനസ്സിലാവും. ഏതൊരു ഭാഷയിലാണെങ്കിലും അതിലെ അംഗീകരിക്കപ്പെട്ട സാഹിത്യകൃതികളുടെ വെളിച്ചത്തിലാണ് വ്യാകരണനിയമങ്ങൾ രൂപപ്പെടുന്നത്. വ്യാകരണത്തിൽ നിന്ന് ഭാഷയല്ല, ഭാഷയിൽ നിന്ന് വ്യാകരണമാണുണ്ടാവുന്നത്. അതിപുരാതനമായ ഗ്രീക്ക് ഭാഷക്ക് ആദ്യമായി പദരൂപശാസ്ത്ര(word morphology)മുണ്ടാക്കുന്നത് ക്രിസ്തുവിന് രണ്ട് നൂറ്റാണ്ടുകൾക്ക് മുൻപ് ദിയോനീസിയൂസ് ത്രാക്‌സ് ആണ്. ക്രിസ്താബ്ദം രണ്ടാം നൂറ്റാണ്ടിൽ മാത്രമാണ് അപ്പോളോനിയുസ് ഡിസ്കൊലസ് വാക്യങ്ങളുടെ പദവിന്യാസക്രമത്തെക്കുറിച്ച് പറഞ്ഞുകൊണ്ട് വ്യാകരണത്തിലേക്ക് കടക്കുന്നത്. നിലവിലുള്ള ഗ്രീക്ക് സാഹിത്യകൃതികളും സംസാരശൈലിയും പഠനവിധേയമാക്കിയാണ് അദ്ദേഹം അക്കാര്യം നിർവ്വഹിച്ചത്. അതിനുശേഷമാണ് പാശ്ചാത്യഭാഷകളിലെയെല്ലാം വ്യാകരണനിയമങ്ങളുണ്ടാവുന്നത്. ഈ വ്യാകരണനിയമങ്ങളുടെ വെളിച്ചത്തിൽ അവയ്ക്ക് ആധാരമായ സാഹിത്യകൃതികളെ വിമർശിക്കുന്നത് എന്തുമാത്രം വലിയ വങ്കത്തമല്ല! ഏത് കൃതികളിലെ ഭാഷയുടെ വിന്യാസക്രമത്തെ ശാസ്ത്രീയമായി വിശകലനം ചെയ്തുകൊണ്ട് വ്യാകരണമുണ്ടാക്കിയോ അതേ വ്യാകരണനിയമങ്ങൾ ആ കൃതികളിൽ പാലിക്കപ്പെട്ടിട്ടില്ലെന്ന് വാദിക്കുന്നത് എത്രമാത്രം വലിയ വിവരക്കേടാണ്!

വ്യത്യസ്ത ഭാഷകൾ സംസാരിക്കുന്നവർ ഇസ്‌ലാമിലേക്ക് വന്നപ്പോൾ അവരുടെയെല്ലാം മതഭാഷ അറബിയായിത്തീർന്ന സ്വാഭാവികസാഹചര്യത്തിൽ ശുദ്ധഅറബിക്ക് ഭംഗമുണ്ടാവുമോയെന്ന ഭയത്തിൽ നിന്നാണ് അറബി വ്യാകരണനിയമങ്ങൾ ക്രോഡീകരിക്കുകയെന്ന ചിന്തയുണ്ടായതെന്ന് ചരിത്രവിശാരദനായ ഇബ്നു ഖൽദൂൻ തന്റെ മുഖദ്ദിമയിൽ വ്യാകരണത്തെക്കുറിച്ച് ചർച്ച ചെയ്യുന്നിടത്ത് സൂചിപ്പിക്കുന്നുണ്ട്. ക്രിസ്താബ്ദം 688ൽ മരണപ്പെട്ട ‘അബുൽ അസ്‌വദ ദ്ദുവലിയുടെ ‘ഉസൂലു ന്നഹ്‌വ് അൽഅറബി’യാണ് ആദ്യമായി രചിക്കപ്പെട്ട അറബി വ്യാകരണഗ്രൻഥം. അദ്ദേഹത്തിന് ശേഷമാണ് അറബി വ്യാകരണനിയമങ്ങൾ കൃത്യമായും സൂക്ഷ്മമായും ക്രോഡീകരിക്കപ്പെട്ടത്. അതിനുവേണ്ടി വ്യാകരണവിദഗ്ദന്മാർ സ്വീകരിച്ച അടിസ്ഥാനസ്രോതസ്സുകൾ ഖുർആനും ഇസ്‌ലാമിന് മുമ്പുള്ള അറബിക്കവിതകളുമായിരുന്നു. അറബി വ്യാകരണത്തിന്റെ ആധാരമാണ് ഖുർആനും അന്തരാളകകവിതകളുമെന്നർത്ഥം. ഇസ്‌ലാമിന് മുമ്പത്തെ കവികളുടെ കുലപതിയായ ഇമ്രുൽ ഖൈസിന്റെ കവിതയിൽ അറബി വ്യാകരണനിയമങ്ങൾ പാലിക്കപ്പെട്ടിട്ടില്ലെന്ന് ആരെങ്കിലും വിമർശിച്ചാൽ ഭാഷാശാസ്ത്രജ്ഞന്മാർ ആ വിമർശനം ശ്രദ്ധിക്കുക പോലുമില്ല. ഇതേപോലെയാണ് ഖുർആനിൽ വ്യാകരണനിയമങ്ങൾ പാലിക്കപ്പെട്ടിട്ടില്ലെന്ന വിമർശനവും. അങ്ങനെ വിമർശിക്കുന്നവർക്ക് ഒന്നുകിൽ വ്യാകരണമറിയില്ല; അല്ലെങ്കിൽ ഖുർആൻ അറിയില്ല; അതുമല്ലെങ്കിൽ രണ്ടും അറിയില്ല. മൂന്നാമത്തെ വിഭാഗത്തിലാണ് ആ ആരോപണമുന്നയിക്കുന്ന ആധുനികരിലധികവും എന്നതാണ് വസ്തുത.

നാല്) ഖുർആൻ എഴുതിയപ്പോൾ ഗുരുതരമായ വ്യാകരണത്തെറ്റുകളുണ്ടായിട്ട് അത് ആയിഷയും ഉഥ്മാനും മാത്രമേ അറിഞ്ഞുവെന്ന് കരുതുന്നത് തനി വങ്കത്തമാണ്. മറ്റു സ്വഹാബിമാരിൽ നിന്നൊന്നും തന്നെ ഇത്തരമൊരു ആരോപണം ഉദ്ധരിക്കപ്പെട്ടിട്ടില്ല. ഉഥ്മാനെതിരെ അദ്ദേഹത്തിന്റെ ഭരണകാലത്ത് കലാപമുണ്ടാക്കിയവരൊന്നും തന്നെ ഖുർആനിലുണ്ടായിരുന്ന അബദ്ധങ്ങൾ തിരുത്തുവാൻ ഭരണാധികാരിയും ഖുർആൻ ക്രോഡീകരണത്തിന് ഉത്തരവിട്ടയാളും എന്ന നിലയിൽ അദ്ദേഹം നടപടികളൊന്നുമെടുത്തില്ല എന്ന ഒരു ആരോപണം ഉന്നയിച്ചിട്ടില്ല. ഉഥ്മാനെ വധിച്ചതുമായി ബന്ധപ്പെട്ട രാഷ്ട്രീയപ്രശ്നങ്ങളിൽ ഉറച്ച നിലപാടെടുക്കുകയും യുദ്ധത്തിന് വരെ ഒരുങ്ങുകയും ചെയ്ത ധീരവനിതയായ ആയിശ (റ) ഖുർആനിൽ വ്യാകരണപ്പിശകുകളുണ്ടെന്ന് മനസ്സിലാക്കിയിട്ടും അവ തിരുത്തുവാൻ ശ്രമങ്ങളൊന്നും നടത്തിയില്ലെന്ന് കരുതുവാൻ ന്യായമില്ല. മറ്റെന്തിനേക്കാളുമധികം ഖുർആനിൽ കളങ്കങ്ങളൊന്നും വന്നുചേരരുതെന്ന് നിർബന്ധമുണ്ടായിരുന്ന സ്വഹാബിസഞ്ചയത്തിന്റെ പ്രതിനിധിയായ ആയിശ (റ), എഴുത്തുകാരിലൂടെ വന്നുചേർന്ന വ്യാകരണത്തെറ്റുകളെ അതേപോലെ അവശേഷിപ്പിക്കുവാൻ ആരെങ്കിലും അനുമതി നല്കിയെന്നറിഞ്ഞാൽ വെറുതെയിരിക്കുമെന്ന് കരുതിക്കൂടാ. അബൂബക്കറിന്റെ കാലത്ത് ക്രോഡീകരിക്കപ്പെട്ട സുഹ്ഫ് തന്റെ സഹപത്നിയായ ഹഫ്സയുടെ പക്കൽ അവശേഷിച്ചിരുന്ന സമയത്ത് അതുമായി ഒത്തുനോക്കി അത്തരം പിശകുകളുണ്ടായിരുന്നെങ്കിൽ അവ തിരുത്തുവാൻ ആയിശ തയാറാകുമായിരുന്നു. അങ്ങനെയൊന്നും ഉണ്ടായിട്ടില്ലെന്നത് തന്നെ ഉഥ്മാനിന്റെ കാലത്ത് ക്രോഡീകരിക്കപ്പെട്ട ഖുർആനിൽ വ്യാകരണപ്പിശകുള്ളതായുള്ള അവരുടെ പേരിലുള്ള നിവേദനം ശരിയാകാൻ യാതൊരു സാധ്യതയുമില്ലെന്ന് വ്യക്തമാക്കുന്നു.

അഞ്ച്) അറബി വ്യാകരണ നിയമങ്ങൾ പ്രകാരം പിശകുകളാണെന്ന് വിമര്‍ശകര്‍ ചൂണ്ടിക്കാണിക്കുന്ന മുഴുവന്‍ വചനങ്ങളും അറബി വ്യാകരണ നിയമങ്ങളുടെ അടിസ്ഥാനത്തിൽ തന്നെ ആഴമേറിയ ചര്‍ച്ചകള്‍ നടത്തിയിട്ടുണ്ട്, ക്വുര്‍ആന്‍ വ്യാഖ്യാതാക്കള്‍. അവയൊന്നും തന്നെ പിശകുകളല്ലെന്നും ഭാഷയിലെ സവിശേഷമായ സന്ദർഭങ്ങളിലെ പ്രയോഗങ്ങളാണെന്നും ഖുർആനിന് മുൻപ് തന്നെയുള്ള സാഹിത്യകൃതികളുടെ വെളിച്ചത്തിൽ സമർത്ഥിക്കപ്പെട്ടിട്ടുമുണ്ട്. ഭാഷാപരമായ പിഴവുകളിൽ നിന്നും വ്യാകരണസംബന്ധിയായ അബദ്ധങ്ങളിൽ നിന്നും പൂര്‍ണമായും സംരക്ഷിതമാണ് ഖുർആൻ എന്ന വസ്തുത പ്രസ്തുത പഠനങ്ങൾ പരിശോധിച്ചാൽ ആർക്കും ബോധ്യപ്പെടും.

“ഖുർആൻ തനിക്ക് പൂർണമായും ലഭിച്ചുവെന്ന് ആരും പറയരുത്; ഖുർആനിൽ നിന്ന് കുറെയധികം നഷ്ടപ്പെട്ടിട്ടുണ്ട്” എന്ന ഇബ്നു ഉമറിന്റെ പരാമർശം അവതരിക്കപ്പെട്ട രൂപത്തിൽ തന്നെ ഖുർആൻ സംരക്ഷിക്കപ്പെട്ടിട്ടുണ്ടെന്ന വാദം ശരിയല്ലന്നല്ലേ വ്യക്തമാക്കുന്നത് ?

ഇമാം സുയൂഥ്വിയുടെ ‘അൽ ഇത്ഖാൻ ഫീ ഉലൂമിൽ ഖുർആൻ’ എന്ന ഗ്രന്ഥത്തിൽ ഉദ്ധരിച്ചിരിക്കുന്ന ഇബ്നു ഉമറിൽ(റ) നിന്നുള്ള ഒരു നിവേദനമാണ് വിമർശനത്തിന് ആധാരം. നിവേദനം ഇങ്ങനെയാണ്: “നിങ്ങളാരും തന്നെ മുഴുവൻ ഖുർആനും എനിക്ക് ലഭിച്ചുവെന്ന് പറയരുത്. അത് മുഴുവനായും അവൻ അറിയുന്നതെങ്ങനെ? ഖുർആനിൽ നിന്ന് കുറെയേറെ നഷ്ടപ്പെട്ടിട്ടുണ്ട്. അതിൽ അവശേഷിച്ചത് എനിക്ക് ലഭിച്ചിരിക്കുന്നു എന്നാണ് അവൻ പറയേണ്ടത് “(അൽ ഇത്ഖൻ 2/ 25; ഇമാം അബൂ ഉബൈദ്: ഫദാഇലുൽ ക്വുര്‍ആന്‍, പേജ് 320).

താഴെ പറയുന്ന കാര്യങ്ങൾ ശ്രദ്ധിക്കുക:

ഒന്ന്) ഖുർആൻ സമ്പൂർണമായി സംരക്ഷിക്കുമെന്നത് അല്ലാഹുവിന്റെ വാഗ്ദാനമാണ്. “തീര്‍ച്ചയായും നാമാണ്‌ ആ ഉല്‍ബോധനം അവതരിപ്പിച്ചത്‌. തീര്‍ച്ചയായും നാം അതിനെ കാത്തുസൂക്ഷിക്കുന്നതുമാണ്‌.” (ഖുർആൻ 15: 9)

മുഹമ്മദ് നബി(സ)ക്കുപോലും ഖുർആനിൽ എന്തെങ്കിലും കൂട്ടുവാനോ എടുത്തുമാറ്റുവാനോ ഉള്ള അവകാശം അല്ലാഹു നൽകിയിട്ടില്ല. “(നബിയേ) പറയുക: എന്‍റെ സ്വന്തം വകയായി അത്‌ ഭേദഗതി ചെയ്യുവാന്‍ എനിക്ക്‌ പാടുള്ളതല്ല. എനിക്ക്‌ ബോധനം നല്‍കപ്പെടുന്നതിനെ പിന്‍പറ്റുക മാത്രമാണ്‌ ഞാന്‍ ചെയ്യുന്നത്‌. തീര്‍ച്ചയായും എന്‍റെ രക്ഷിതാവിനെ ഞാന്‍ ധിക്കരിക്കുന്ന പക്ഷം ഭയങ്കരമായ ഒരു ദിവസത്തെ ശിക്ഷ ഞാന്‍ പേടിക്കുന്നു.” (10:15)

“ഇത്‌ ലോകരക്ഷിതാവിങ്കല്‍ നിന്ന്‌ അവതരിപ്പിക്കപ്പെട്ടതാകുന്നു. നമ്മുടെ പേരില്‍ അദ്ദേഹം (പ്രവാചകന്‍) വല്ല വാക്കും കെട്ടിച്ചമച്ചു പറഞ്ഞിരുന്നെങ്കില്‍ അദ്ദേഹത്തെ നാം വലതുകൈ കൊണ്ട്‌ പിടികൂടുകയും എന്നിട്ട്‌ അദ്ദേഹത്തിന്‍റെ ജീവനാഡി നാം മുറിച്ചുകളയുകയും ചെയ്യുമായിരുന്നു.” (69: 43- 46)

രണ്ട്) ഖുർആനിൽ ആദ്യം അവതരിപ്പിക്കപ്പെട്ട ചില വചനങ്ങൾ പിന്നീട് ദുർബലപ്പെടുത്തപ്പെട്ടിട്ടുണ്ട്. ഇക്കാര്യം ഖുർആൻ തന്നെ വ്യക്തമാക്കിയിട്ടുള്ളതാണ്.

“വല്ല ആയത്തും നാം ദുര്‍ബലപ്പെടുത്തുകയോ വിസ്മരിപ്പിക്കുകയോ ചെയ്യുകയാണെങ്കില്‍ പകരം അതിനേക്കാള്‍ ഉത്തമമായതോ അതിനു തുല്യമായതോ നാം കൊണ്ടുവരുന്നതാണ്. നിനക്കറിഞ്ഞുകൂടേ, അല്ലാഹു എല്ലാ കാര്യത്തിനും കഴിവുള്ളവനാണെന്ന്” (2:106).

‘ഒരു വേദവാക്യത്തിന്റെ സ്ഥാനത്ത് മറ്റൊരു വേദവാക്യം നാം പകരം വെച്ചാല്‍-അല്ലാഹുവാകട്ടെ താന്‍ അവതരിപ്പിക്കുന്നതിനെപ്പറ്റി നല്ലവണ്ണം അറിയുന്നവനാണു താനും-അവര്‍ പറയും: നീ കെട്ടിച്ചമച്ചു പറയുന്നവന്‍ മാത്രമാകുന്നു എന്ന്. അല്ല, അവരില്‍ അധികപേരും കാര്യം മനസ്സിലാക്കുന്നില്ല’ (16:101).

മൂന്നു രൂപത്തിലാണ് ഖുർആനിൽ ദുർബലപ്പെടുത്തൽ അഥവാ നസ്ഖ് ഉണ്ടായിട്ടുള്ളത്. പാരായണം നിലനിർത്തിക്കൊണ്ട് നിയമങ്ങൾ ദുർബലപ്പെടുത്തുക, പാരായണം ഇല്ലാതെയായി നിയമങ്ങൾ അവശേഷിക്കുക, പാരായണവും അതിലെ നിയമങ്ങളും ദുർബലപ്പെടുത്തുക എന്നിവയാണവ. അല്ലാഹുവിൽ നിന്നുള്ള ബോധനങ്ങളുടെ അടിസ്ഥാനത്തിൽ മുഹമ്മദ് നബി (സ) തന്നെ നിർദേശിച്ചത് അനുസരിച്ച് നടന്നവയാണ് ഈ ദുർബലപ്പെടുത്തലുകളെല്ലാം. പാരായണവും അതിലെ നിയമങ്ങളും ദുർബലപ്പെടുത്തപ്പെട്ട ചില ആയത്തുകളെക്കുറിച്ചുള്ളതാണ് ഇബ്നു ഉമറിന്റെ(റ) മുകളിൽ ഉദ്ധരിച്ച പരാമർശം. അല്ലാഹു അവതരിപ്പിച്ചവയിൽ അവ കൂടി ഉണ്ടായിരുന്നുവെന്നും എന്നാൽ അവ പിൽക്കാലത്ത് ദുർബലപ്പെടുത്തപ്പെട്ടതിനാൽ വിസ്മരിക്കപ്പെടുകയാണുണ്ടായതെന്നുമാണ് ഇബ്നു ഉമർ (റ) വ്യക്തമാക്കുന്നത്. അത് കൊണ്ട് തന്നെ അവതരിക്കപ്പെട്ട മുഴുവൻ ഖുർആനും തനിക്ക് അറിയാം എന്ന് പറയാൻ ആർക്കും അവകാശമില്ലെന്ന വസ്തുത പഠിപ്പിക്കുകയാണ് ഇവിടെ അദ്ദേഹം ചെയ്യുന്നത്.

മൂന്ന്) ഇമാം സുയൂഥ്വി തന്റെ രണ്ട് ഗ്രന്ഥങ്ങളിൽ ഇബ്നു ഉമറിൽ നിന്നുള്ള ഈ അഥർ ഉദ്ധരിക്കുന്നുണ്ട്. ഒന്ന് നേരത്തെ സൂചിപ്പിച്ച അൽ ഇത്ഖാനിലാണ്. രണ്ടാമത്തേത് “മുഅതരിഖ് അൽ അർഖാൻ ഫീ അഅജാസിൽ ഖുർആൻ”(വാല്യം 1, പുറം 95) എന്ന ഗ്രന്ഥത്തിലാണ്. രണ്ടിലും ഖുർആനിലെ ആയത്തുകളെ ദുർബലപ്പെടുത്തുന്നതിനെക്കുറിച്ച് പ്രതിപാദിക്കുന്നിടത്താണ് ഇത് ഉദ്ധരിച്ചിരിക്കുന്നത്. അൽ ഇത്ഖാനിൽ “ദുർബലപ്പെടുത്തലിനെയും ദുർബലപ്പെടുത്തിയവയെയും കുറിച്ച്” എന്ന തലക്കെട്ടിലുള്ള നാല്പത്തിയേഴാമത്തെ ഭാഗത്ത് ഈ അഥർ ഉദ്ധരിച്ചിരിക്കുന്നത് കാണാനാവും. ഹിജ്‌റ 228ൽ മരണപ്പെട്ട അബു ഉബൈദിന്റെ ‘ഫദാ’ഇലിൽ ഖുർആൻ’ എന്ന ഗ്രന്ഥത്തിൽ നിന്നാണ് ഇമാം സുയൂഥ്വി ഈ അഥർ ഉദ്ധരിച്ചിരിക്കുന്നത് (വാല്യം 1, പുറം 320). “വെളിപ്പെട്ടതിനു ശേഷം ഖുർആനിൽ നിന്ന് ദുർബലപ്പെടുത്തിയതും മുസ്ഹഫിൽ ഉൾപ്പെടുത്താത്തതുമായ കാര്യങ്ങളെപ്പറ്റി”എന്ന അധ്യായത്തിലെ ഒന്നാമത്തെ നിവേദനമായാണ് അബു ഉബൈദ് ഇത് ഉദ്ധരിച്ചിരിക്കുന്നത്. ഇത് ഉദ്ധരിച്ച പണ്ഡിതന്മാരെല്ലാം ഖുർആനിലെ ദുർബലപ്പെടുത്തപ്പെട്ട വചനങ്ങളെക്കുറിച്ചാണ് ഇബ്നു ഉമർ ഇങ്ങനെ പറഞ്ഞതെന്നാണ് മനസ്സിലാക്കിയിരുന്നത് എന്ന സത്യമാണ് ഇവ വ്യക്തമാക്കുന്നത്. ഇതേ അഥറിന്റെ തന്നെ ഇബ്നു ഹജറിന്റെ നിവേദനം കാര്യങ്ങൾ കൂടുതൽ വ്യക്തമാക്കുന്നതാണ്. അത് ഇങ്ങനെയാണ്: “ഇബ്നു ഉമറിൽ നിന്നുള്ള ഇബ്നു ദ്ദുറൈസിന്റെ ഒരു നിവേദനത്തിൽ ‘ഞാൻ ഖുർആൻ മുഴുവനായി പാരായണം ചെയ്‍തു’വെന്ന് പറയുന്നതിനെ അദ്ദേഹം ഇഷ്ടപ്പെട്ടിരുന്നില്ല എന്ന് കാണാൻ കഴിയും. അദ്ദേഹം പറയുമായിരുന്നു: നാം പാരായണം ചെയ്തിരുന്ന ചിലവ ഉയർത്തപ്പെട്ടിരിക്കുന്നുവല്ലോ.” (ഫത്ഹുൽ ബാരി വാല്യം 9, പുറം 65)

ഇബ്നു അബ്ബാസ് (റ) നിവേദനം ചെയ്ത ഉബയ്യു ബ്നു കഅ്ബിന്റെ ഇതേ വിഷയത്തിലുള്ള മറ്റൊരു അഥർ കാര്യങ്ങൾ കൂടുതൽ വ്യക്തമാക്കാനുതകുന്നതാണ്: ഉബയ്യിനോട് ഒരാൾ ഇങ്ങനെ പറഞ്ഞു: ‘ഓ അബ്ദുൽ മുൻസർ, ഖുർആൻ മുഴുവനായും ഞാൻ പഠിച്ചെടുത്തിരിക്കുന്നു’. ഉബയ്യ് മറുപടി പറഞ്ഞു: ‘അത് മുഴുവനായും താങ്കൾക്കറിയില്ല. ഖുർആനിൽ നിന്ന് പലതും ദുർബലപ്പെടുത്തിയിട്ടുണ്ട്; അതിന് ശേഷം അവ വിസ്മരിക്കപ്പെട്ടതാണ്.” (അൽ ബാഖിലാനി: അൽ ഇൻതിസാർ ലിൽ ഖുർആൻ, പുറം 406)

ഇബ്നു ഉമറും ഉബയ്യു ബ്നു കഅ്ബുമെല്ലാം പറഞ്ഞത് ഒരേ കാര്യം തന്നെയാണ്: ഖുർആനായി അവതരിക്കപ്പെട്ടത് മുഴുവൻ തനിക്ക് ലഭിച്ചിട്ടുണ്ടെന്നോ താൻ മനഃപാഠമാക്കിയിട്ടുണ്ടെന്നോ പറയാൻ ആർക്കും അവകാശമില്ല; അവതരിക്കപ്പെട്ട ചില വചനങ്ങൾ പ്രവാചകന്റെ(സ) കാലത്ത് തന്നെ ദുർബലപ്പെടുത്തപ്പെട്ടിട്ടുണ്ട്. അത് കൊണ്ട് തന്നെ മനഃപാഠമാക്കിയവരൊന്നും അവ മനഃപാഠമാക്കുകയോ എഴുത്തുകാരൊന്നും അവ എഴുതിവെക്കുകയോ ചെയ്തിട്ടില്ല. പിൽക്കാലത്തുള്ള ആർക്കും തന്നെ പ്രസ്തുത വചനങ്ങളെപ്പറ്റി അറിയുകയില്ല. അത് കൊണ്ടാണ് ഖുർആൻ മുഴുവനായി തനിക്ക് ലഭിച്ചിരിക്കുന്നുവെന്ന് പറയുന്നതിനെ പ്രമുഖരായ സ്വഹാബിമാർ നിരുത്സാഹപ്പെടുത്തിയത്. മുഹമ്മദ് നബി (സ) പൂർത്തിയാക്കിത്തന്ന ഖുർആനിൽ നിന്ന് അദ്ദേഹത്തിന് ശേഷം വല്ലതും നഷ്ടപ്പെട്ടിരിക്കുന്നുവെന്ന് ഒരു സ്വഹാബിയും മനസ്സിലാക്കിയിട്ടില്ല. അങ്ങനെ മനസ്സിലാക്കിയെന്ന് വ്യക്തമാക്കുന്ന യാതൊരു പ്രമാണവുമില്ല.

നാല്) ഉബയ്യു ബ്നു കഅ്ബിൽ നിന്ന് നേരത്തെ പറഞ്ഞത് നിവേദനം ചെയ്ത ഇബ്നു അബ്ബാസിൽ(റ) നിന്ന് ബുഖാരി ഉദ്ധരിച്ചിരിക്കുന്ന അഥർ ഇക്കാര്യത്തിൽ കൂടുതൽ വ്യക്തതയുണ്ടാക്കുന്നതാണ്. അതിങ്ങനെയാണ്: “അബ്ദുൽ അസീസ് ബ്നു രുഫാഇയിൽ നിന്ന് നിവേദനം: ശദ്ദാദ് ബിൻ മഅഖിലും ഞാനും കൂടി ഇബ്നു അബ്ബാസിന്റെ അടുത്ത് പോയി. ശദ്ദാദ് ബിൻ മഅഖിൽ അദ്ദേഹത്തോട് ചോദിച്ചു: (ഖുർആനിനോടൊപ്പം) മറ്റു വല്ലതും പ്രവാചകൻ (സ) അവശേഷിപ്പിച്ചിട്ടുണ്ടോ? അദ്ദേഹം പറഞ്ഞു: (ഖുർആനിന്റെ) രണ്ട് ചട്ടകൾക്കുള്ളിൽ ഉള്ളതല്ലാത്ത മറ്റൊന്നും തന്നെ പ്രവാചകൻ (സ) അവശേഷിപ്പിച്ചിട്ടില്ല. പിന്നീട് ഞങ്ങൾ മുഹമ്മദ് ബിൻ അൽഹനഫിയ്യയുടെ അടുക്കലും പോയി ഇതേ ചോദ്യം ആവർത്തിച്ചു. അദ്ദേഹം പറഞ്ഞു: (ഖുർആനിന്റെ) രണ്ട് ചട്ടകൾക്കുള്ളിൽ ഉള്ളതല്ലാത്ത മറ്റൊന്നും തന്നെ പ്രവാചകൻ (സ) അവശേഷിപ്പിച്ചിട്ടില്ല.” (സ്വഹീഹുൽ ബുഖാരി, കിത്താബു ഫദാഇലിൽ ഖുർആൻ, ബാബു മൻ ഖാല ലം യത്റുഖി നബിയ്യു(സ) ഇല്ലാ മാ ബൈന ദ്ദഫതൈനി)

ഖുർആനിലുണ്ടാകണമെന്ന രൂപത്തിൽ പ്രവാചകൻ (സ) പഠിപ്പിച്ചതൊന്നും തന്നെ പിൽക്കാലത്ത് നഷ്ടപ്പെട്ടിട്ടില്ലെന്ന സത്യം ഇബ്നു അബ്ബാസിന്റെയും മുഹമ്മദ് ബിൻ അൽഹനഫിയ്യയുടെയും ഈ സാക്ഷ്യത്തിൽ നിന്ന് സുതരാം വ്യക്തമാവുന്നുണ്ട്. നഷ്ടപ്പെട്ടതായി ഇബ്നു ഉമറും ഉബയ്യു ബ്നു കഅ്ബുമെല്ലാം പറഞ്ഞത് പ്രവാചകന്റെ(സ) കാലത്ത് അദ്ദേഹം തന്നെ ദൈവികബോധനത്തിന്റെ അടിസ്ഥാനത്തിൽ ദുർബലപ്പെടുത്തിയവയാണ്. ഉബയ്യു ബ്നു കഅ്ബിൽ നിന്ന് ഖുർആനിൽ നിന്ന് കുറെ ഭാഗങ്ങൾ വിസ്മരിക്കപ്പെട്ടിട്ടുണ്ടെന്ന കാര്യം നിവേദനം ചെയ്ത ഇബ്നു അബ്ബാസു(റ) തന്നെയാണ് അബൂബക്കറിന്റെ(റ) കാലത്ത് ക്രോഡീകരിച്ച മുസ്ഹഫിന്റെ രണ്ട് ചട്ടകൾക്കുളിലുള്ള വചനങ്ങൾ മാത്രമാണ് പ്രവാചകൻ (സ) ഖുർആനായി അവശേഷിപ്പിച്ചത് എന്ന് പറയുന്നതെന്നു കാര്യം ശ്രദ്ധേയമാണ്. ഉബയ്യു ബ്നു കഅ്ബ് ഉദ്ദേശിച്ചത് നസ്ഖ് ചെയ്യപ്പെട്ട ഖുർആൻ വചങ്ങളെക്കുറിച്ചാണെന്നാണ് ഇബ്നു അബ്ബാസ് മനസ്സിലാക്കിയതെന്ന് ഇതിൽ നിന്ന് കൃത്യമായി മനസ്സിലാവുന്നുണ്ട്. ഇക്കാര്യം ബുഖാരിയുടെ വ്യാഖ്യാതാവായ ഇബ്നു ഹജർ പറയുന്നുണ്ട്. “ഖുർആൻ അറിയാമായിരുന്നവരുടെ മരണം വഴി കുറെയേറെ ഖുർആൻ വചനങ്ങൾ നഷ്ടപ്പെട്ടുവെന്ന ഊഹത്തെ തിരസ്കരിക്കുന്നതിന് വേണ്ടിയുള്ളതാണ് ഈ അദ്ധ്യായം” (ഫത്ഹുൽ ബാരി, വാല്യം9, പുറം 65) എന്നാണ് ബുഖാരിയുടെ ഇക്കാര്യം നിവേദനം ചെയ്യുന്ന ഉപാധ്യായത്തെപ്പറ്റി അദ്ദേഹം പറയുന്നത്. ഇബ്നു ഉമറിന്റെയും ഉബയ്യു ബ്നു കഅബിന്റെയുമെല്ലാം അഭിപ്രായപ്രകടനങ്ങൾ ഖുർആനിലെ നസ്ഖ് ചെയ്യപ്പെട്ട വചങ്ങളെക്കുറിച്ചാണെന്നാണ് അവരെല്ലാം മനസ്സിലാക്കിയത് എന്ന് സാരം.

മുഹമ്മദ് നബിയുടെ അനുയായികളുടേതായി പല തരം ഖുർആനുകൾ നിലവിലുണ്ടായിരുന്നുവെന്നും അവ തമ്മിൽ വലിയ വ്യത്യാസങ്ങളുണ്ടായിരുന്നുവെന്നും വാദിക്കപ്പെടുന്നുണ്ടല്ലോ. വാസ്തവമെന്താണ്?

പല പ്രവാചകാനുചരന്മാരും അവരവുടെ പാരായണാവശ്യങ്ങൾക്കുവേണ്ടി തങ്ങളുടേതായ ഖുർആൻ രേഖകളുണ്ടാക്കിയിരുന്നുവെന്നത് ശരിയാണ്. അവ അവരുടെ സ്വകാര്യപ്രതികളായിരുന്നു. തങ്ങളുടെ വൈയക്തികമായ പഠന-പാരായണങ്ങൾക്കായി തങ്ങൾക്ക് ലഭിച്ച സൂറത്തുകൾ തങ്ങൾക്ക് ലഭിച്ച ക്രമത്തിൽ എഴുതിവെച്ചവയാരിരുന്നു അവ. ഇത്തരം ഖുർആൻ കയ്യെഴുത്തുരേഖകൾ പല സ്വഹാബിമാർക്കും ഉണ്ടായിരുന്നുവെന്ന് ഹദീഥുകളിൽ നിന്ന് മനസ്സിലാവുന്നുണ്ട്. (ഇമാം സുയൂഥ്വി: അൽ ഇത്ഖാൻ 1/ 62). ഹിജ്‌റ നാലാം നൂറ്റാണ്ടിൽ ജീവിച്ച ബാഗ്ദാദിയൻ ചരിത്രകാരനായ അബൂ ഫറാജ് മുഹമ്മദ് ബിൻ ഇസ്‌ഹാഖ്‌ അന്നദീം തന്റെ പ്രസിദ്ധമായ ‘കിത്താബൽ ഫിഹിരിസ്തി’ൽ ഇബ്നു മസ്ഊദിന്റെയും ഉബയ്യു ബ്നു കഅബിന്റെയും സൈദ് ബ്നു ഥാബിത്തിന്റെയും (റ) മുസ്ഹഫുകളുണ്ടായിരുന്നുവെന്നും അവയിൽ ചിലത് താൻ കണ്ടിട്ടുണ്ടെന്നും പറയുന്നുണ്ട്. (Bayard Dodge: The Fihrist of al-Nadim; A tenth Century Survey of Muslim Culture, New York, 1970, Page 53-63)

ഇബ്നു മസ്ഊദിനും ഉബയ്യു ബ്നു കഅബിനും സൈദ് ബ്നു ഥാബിത്തിനും കൂടാതെ അലി, ഇബ്നു അബ്ബാസ്, അബൂ മൂസൽ അശ്അരി, ഹഫ്സ, അനസ് ബ്നു മാലിക്ക്, ഉമർ ഫാറൂഖ്, ഇബ്നു സുബൈർ, അബ്ദുല്ലാഹി ബ്നു അംറ്, ആയിശ, സാലിം, ഉമ്മു സൽ‍മ, ഉബൈദ് ബ്നു ഉമർ (റ) എന്നിവരുടെ കൈകളിലും സ്വന്തമായ ഖുർആൻ കയ്യെഴുത്ത് രേഖകൾ ഉണ്ടായിരുന്നതായി ഇബ്നു അബീദാവൂദ് തന്റെ മസാഹിഫിൽ വ്യക്തമാക്കുന്നുണ്ട്. (പുറം 14). ഇവയെ കൂടാതെ അബൂബക്കർ, ഉഥ്മാൻ, മുആദ് ബിൻ ജബൽ, അബൂ ദർദാഅ, അബൂ അയൂബ് അൽ അൻസാരി, ഉബാദ ബിൻ അൽ സാമിത്, തമീമുദ്ദാരി (റ) എന്നിവർക്കും പ്രവാചകാലത്ത് തന്നെ സ്വന്തമായി ഖുർആൻ കയ്യെഴുത്ത് രേഖകളുണ്ടായിരുന്നതായി വ്യക്തമാക്കുന്ന രേഖകളുണ്ട്. (Dr. Mohammed Fazalu Rahman Ansari, The Qura’nic Foundations and Structure of Muslim Society, Karachi, 1973, Volume 1,Page 76, Note 2)

ഇബ്നു മസ്ഊദിന്റേതായി അറിയപ്പെടുന്ന ഒരു ഖുർആൻ രേഖയിൽ ഇന്നുള്ള മുസ്ഹഫിലെ ക്രമത്തിലല്ല സൂറത്തുകൾ ക്രമീകരിച്ചിരിക്കുന്നതെന്ന് താൻ കണ്ടതായി സാക്ഷ്യപ്പെടുത്തിക്കൊണ്ട് ഇബ്‌നു ന്നദീം രേഖപ്പെടുത്തുന്നുണ്ട്. അദ്ദേഹം കണ്ട ഇബ്നു മസ്ഊദിന്റെ മുസ്ഹഫിലെ ക്രമം ഇങ്ങനെയാണ്:

2, 4, 3, 7, 6, 5, 10, 9, 16, 11, 12, 17, 21, 23, 26, 37, 33, 28, 24, 8, 19, 29, 30, 36, 25, 22, 13, 34, 35, 14, 38, 47, 31, 35, 40, 43, 41, 46, 45, 44, 48, 57, 59, 32, 50, 65, 49, 67, 64, 63, 62, 61, 72, 71, 58, 60, 66, 55, 53, 51, 52, 54, 69, 56, 68, 79, 70, 73, 74, 83, 80, 76, 75, 77, 78, 81, 82, 88, 87, 92, 89, 85, 84, 96, 90, 93, 94, 86, 100, 107, 101, 98, 91, 95, 104, 105, 106, 102, 97, 110, 108, 109, 111, 112. (The Fihrist, Page 53-57)

ഇതിൽ ആകെയുള്ളത് 106 സൂറത്തുകൾ മാത്രമാണ്. അവയാകട്ടെ ലോകത്തെങ്ങും പ്രചാരത്തിലുള്ള ഉഥ്മാനീ മുസ്ഹഫിലെ ക്രമത്തിലല്ല ക്രോഡീകരിച്ചിരിക്കുന്നത് താനും.

ഉബയ്യുബ്നു കഅബിന്റെ മുസ്ഹഫാണ് താൻ കണ്ടതായി ഇബ്‌നു ന്നദീം രേഖപ്പെടുത്തുന്ന, സ്വഹാബിമാരുടേതായി അറിയപ്പെടുന്ന മറ്റൊരു കയ്യെഴുത്ത് രേഖ. അതിലെ സൂറത്തുകളുടെ ക്രമം ഇങ്ങനെയാണ്: 1, 2, 4, 3, 6, 7, 5, 10, 8, 9, 11, 19, 26, 22, 12, 18, 16, 33, 17, 39, 45, 20, 21, 24, 23, 40, 13, 28, 27, 37, 38, 36, 15, 42, 30, 43, 41, 14, 35, 48, 47, 57, 52, 25, 32, 71, 46, 50, 55, 56, 72, 53, 68, 69, 59, 60, 77, 78, 76, 75, 81, 79, 80, 83, 84, 95, 96, 49, 63, 62, 66, 89, 67, 92, 82, 91, 85, 86, 87, 88, 74, 98, 61, 93, 94, 101, 102, 65, 104, 99, 100, 105, 108, 97, 109, 110, 111, 106, 112, 113, 114. (The Fihrist, Page 58- 60)

ഇതിലുള്ള നൂറ്റിയൊന്ന് സൂറത്തുകളിൽ പലതും ക്രമം തെറ്റിയാണ് ക്രോഡീകരിച്ചിരിക്കുന്നത്.

ഉഥ്മാനീ മുസ്ഹഫിലെ ക്രമത്തിൽ നിന്ന് ഭിന്നമായാണ് ഈ മുസ്ഹഫുകളിൽ സൂറത്തുകൾ ക്രോഡീകരിച്ചിരിക്കുന്നത് എന്ന വസ്തുത ഖുർആനിലെ സൂറത്തുകളുടെ ക്രമത്തിൽ പോലും സ്വഹാബിമാർക്കിടയിൽ ഏകസ്വരമുണ്ടായിരുന്നില്ല എന്നതിനുള്ള തെളിവായാണ് വിമർശകർ എടുത്തുന്നയിക്കാറുള്ളത്. എന്നാൽ എന്താണ് യഥാർത്ഥത്തിലുള്ള വസ്തുത?

സ്വഹാബിമാരുടെ സ്വകാര്യ കയ്യെഴുത്തുപ്രതികൾ അവർ സ്വന്തം പാരായണത്തിനും പഠനത്തിനും വേണ്ടി എഴുതി വെച്ചവയായിരുന്നു. അവർക്ക് ലഭിച്ച സൂറത്തുകൾ അവർക്ക് ലഭിച്ച മുറയിൽ അവർ എഴുതി വെക്കുകയും പിന്നീട് അവർ അവയെ ക്രോഡീകരിക്കുകയും ചെയ്തു. പ്രവാചകൻ (സ) ഇഹലോകവാസം വെടിയുന്നതിന് ഏതാനും ദിവസങ്ങൾക്ക് മുൻപ് വരെ ഖുർആൻ സൂക്തങ്ങൾ അവതരിപ്പിക്കപ്പെടുകയും അവ പല സ്വഹാബിമാരും (റ) എഴുതിവെക്കുകയും മനഃപാഠമാക്കുകയും ചെയ്തു. സ്വകാര്യമായ ഖുർആൻ കയ്യെഴുത്ത് പ്രതികൾ കൈവശമുള്ള സ്വഹാബിമാർ (റ) ജീവിച്ചിരിക്കുമ്പോഴാണ് അബൂബക്കറിന്റെ ഭരണകാലത്ത് ആദ്യമായി കൃത്യമായ ക്രമത്തിലുള്ള ഖുർആൻ ക്രോഡീകരണം നടന്നത്. അതായിരുന്നു ഔദ്യോഗികമായ ആദ്യത്തെ ക്രോഡീകരണം. പ്രസ്തുത മുസ്ഹഫിൽ പ്രവാചകൻ (സ) പറഞ്ഞുകൊടുത്ത ക്രമത്തിൽ തന്നെയാണ് സൂറത്തുകളെ വിന്യസിച്ചിരിക്കുന്നത്. തങ്ങളുടെ സ്വകാര്യലിഖിതങ്ങളിൽ വ്യത്യസ്തമായ ക്രമത്തിലെഴുതിയ സ്വഹാബിമാരിൽ ആരെങ്കിലും അബൂബിക്കറിന്റെ(റ) കാലത്ത് സൂറത്തുകളെ വിന്യസിച്ച് ക്രമം ശരിയല്ലെന്ന് പറഞ്ഞതായി യാതൊരു രേഖയുമില്ല. തങ്ങളുടെ സ്വകാര്യകോപ്പികളിൽ എഴുതിയതിൽ നിന്ന് വ്യത്യസ്തമാണെങ്കിലും അബൂബക്കറിന്റെ(റ) നിർദേശപ്രകാരം തയ്യാറാക്കിയ ഔദ്യോഗികമുസ്ഹഫിലെ സൂറത്തുകളുടെ ക്രമം തന്നെയാണ് അംഗീകരിച്ചുവെന്നാണ് ഇതിൽ നിന്ന് മനസ്സിലാവുന്നത്.

ഉഥ്മാനിന്റെ(റ) ഭരണകാലത്ത് അബൂബക്കറിന്റെ(റ) കാലത്തുണ്ടാക്കിയ ഖുർആൻപ്രതിയെ ആധാരമാക്കി കൂടുതൽ സൂക്ഷ്മവും ക്രത്യവുമായ രീതിയിൽ നിർമിച്ച മുസ്ഹഫുകളിലും അതേ ക്രമം തന്നെയാണ് സ്വീകരിച്ചിരുന്നത്. സ്വകാര്യപ്രതികൾ കൈവശമുണ്ടായിരുന്ന സ്വഹാബിമാരിൽ പലരും മരണപ്പെട്ടിരുന്നില്ല. അവരാരും ഉഥ്മാൻ (റ) സ്വീകരിച്ച സൂറത്തുകളുടെ വിന്യാസക്രമം ശരിയല്ലെന്ന് വാദിച്ചതായി രേഖകളൊന്നുമില്ല. തങ്ങളുടെ സ്വകാര്യപ്രതികളിൽ തങ്ങൾക്ക് ലഭിച്ചതും തങ്ങൾ ഇച്ഛിച്ചതുമായ ക്രമത്തിലാണ് തങ്ങൾ സൂറത്തുകൾ രേഖപ്പെടുത്തിയതെന്നും അങ്ങനെയല്ല പ്രവാചകൻ (സ) പഠിപ്പിച്ച സൂറത്തുകളുടെ ക്രമമെന്നും കൃത്യമായി അറിയാവുന്നവരായിരുന്നു അവർ എന്നതുകൊണ്ടാണ് ഉഥ്മാൻ (റ) സ്വീകരിച്ച സൂറത്തുകളുടെ വിന്യാസക്രമത്തിൽ അഭിപ്രായവ്യത്യാസങ്ങളൊന്നും രേഖപ്പെടുത്താതിരുന്നത്. രണ്ട് ക്രോഡീകരണസന്ദർഭങ്ങളിലുമുള്ള സ്വകാര്യപ്രതികൾ കൈവശമുണ്ടായിരുന്ന സ്വഹാബിമാരുടെ മൗനം സൂറത്തുകളുടെ ക്രമത്തിന്റെ വിഷയത്തിൽ ആർക്കും യാതൊരു വിരുദ്ധാഭിപ്രായങ്ങളും ഉണ്ടായിരുന്നില്ല എന്ന വസ്തുത വ്യക്തമാക്കുന്നതാണ്.

വ്യഭിചാരികളെ എറിഞ്ഞുകൊല്ലണമെന്ന് കൽപിക്കുന്ന ഖുർആൻ വചനം തന്റെ കയ്യിലുണ്ടായിരുന്നുവെന്നും അത് ആട് തിന്ന് നശിച്ചുപോവുകയാണുണ്ടായതെന്നും ആയിഷ പറയുന്നതായി പരാമർശിക്കുന്ന ഹദീഥുണ്ടല്ലോ. മുഹമ്മദ് നബിയുടെ കാലത്തുണ്ടായിരുന്ന ഖുർആനിൽ നിന്ന് പലതും ആടും ഒട്ടകവുമെല്ലാം തിന്ന് നശിച്ചു പോയിട്ടുണ്ടാകാമെന്നല്ലേ ഈ ഹദീഥ് വ്യക്തമാക്കുന്നത്? അവതരിപ്പിക്കപ്പെട്ട രൂപത്തിൽ തന്നെ ഖുർആൻ സംരക്ഷിക്കപ്പെട്ടിട്ടുണ്ടെന്ന വാദം അടിസ്ഥാനരഹിതമാണെന്നല്ലേ ഇത് വ്യക്തമാക്കുന്നത്.

 

ഇമാം ഇബ്നു മാജ തന്റെ സുനനിലും ഇമാം അഹ്‌മദ്‌ തന്റെ മുസ്നദിലും ഉദ്ധരിച്ചിരിക്കുന്ന ഒരു ഹദീഥിന്റെ വെളിച്ചത്തിലുള്ളതാണ് ഈ വിമർശനം. ഹദീഥ് ഇങ്ങനെയാണ്. “ആയിശ (റ) പറഞ്ഞു: കല്ലെറിയലിന്റെ വചനവും പത്ത് പ്രാവശ്യമാണ് മുലകുടിയെന്ന വചനവും അവതരിക്കപ്പെട്ടിരുന്നു. അതെഴുതിയ രേഖ എന്റെ തലയിണക്കടിയിലുണ്ടായിരുന്നു. ദൈവദൂതൻ മരണപ്പെട്ടപ്പോൾ, ഞങ്ങളെല്ലാം അതുമായി ബന്ധപ്പെട്ട തിരിക്കുകൾക്കിടയിലായിരുന്ന സന്ദർഭത്തിൽ ഒരു ആട് അകത്ത് കടന്ന് അത് തിന്നു കളഞ്ഞു.” (സുനനു ഇബ്നു മാജ, കിതാബു ന്നികാഹ്, ഹദീഥ് 1944; മുസ്നദ് അഹ്‌മദ്‌ 43/343)

താഴെ പറയുന്ന കാര്യങ്ങൾ ശ്രദ്ധിക്കുക:

ഒന്ന്) വിവാഹിതരായ വ്യഭിചാരികളെ എറിഞ്ഞു കൊല്ലണമെന്നും മുലകുടി ബന്ധം സ്ഥാപിക്കപ്പെടണമെങ്കിൽ പത്ത് തവണ കുടിക്കേണ്ടതുണ്ടെന്നും പഠിപ്പിക്കുന്ന വചനങ്ങൾ ഖുർആനിന്റെ ഭാഗമായി തന്നെ ആദ്യകാലത്ത് പാരായണം ചെയ്യപ്പെട്ടിരുന്നുവെന്നും പിന്നീട് അവ ദുർബലപ്പെടുത്തുകയാണുണ്ടായത് എന്നും വ്യക്തമാക്കുന്ന ഇതല്ലാതെയുള്ള സ്വഹീഹായ നിവേദനങ്ങളുണ്ട്.

സ്വഹീഹുൽ ബുഖാരി, കിത്താബുൽ ‘ഹുദൂദി’ലെ ബാബുൽ ‘ഇഅതിറാഫി ബി സ്സിനാ’യിൽ ഇബ്നു അബ്ബാസിൽ (റ) നിന്ന് നിവേദനം ചെയ്ത ഹദീഥിൽ ഇങ്ങനെ വായിക്കാം: ഉമർ (റ) പറഞ്ഞു: “കുറെ കാലം കഴിയുമ്പോൾ ആളുകൾ കല്ലെറിഞ്ഞു കൊല്ലലിന്റെ (റജ്മ്) ആയത്തുകൾ വിശുദ്ധഗ്രന്ഥത്തിൽ ഞങ്ങൾ കാണുന്നില്ല എന്ന് പറയുമെന്ന് ഞാൻ ഭയപ്പെടുന്നു. ക്രമേണ അല്ലാഹു ഇറക്കിയ ഈ ഉത്തരവാദിത്വം ഒഴിവാക്കിക്കൊണ്ട് അവർ വഴിപിഴക്കുകയും ചെയ്യും. അറിയുക വ്യഭിചാരി വിവാഹിതനാണെങ്കിൽ, കുറ്റം സാക്ഷികൾ മുഖേനയോ ഗർഭത്തിലൂടെയോ കുറ്റസമ്മതം കൊണ്ടോ തെളിയിക്കപ്പെട്ടാൽ അതിനുള്ള ശിക്ഷയായി കല്ലെറിഞ്ഞു കൊല്ലേണ്ടതാണ്. (നിവേദകന്മാരുടെ പരമ്പരയിൽ പെട്ട ഒരാളായ) സുഫ്‌യാൻ (റ) കൂട്ടിച്ചേർത്തു: ഞാൻ ഈ നിവേദനം ഇങ്ങനെയാണ് മനഃപാഠമാക്കിയത്: ഉമർ (റ) ഇങ്ങനെ കൂടി പറഞ്ഞിട്ടുണ്ട്: “തീർച്ചയായും അല്ലാഹുവിന്റെ ദൂതനും അദ്ദേഹത്തിനുശേഷം ഞങ്ങളുമെല്ലാം കല്ലെറിഞ്ഞു കൊന്നിട്ടുണ്ട്.”

സ്വഹീഹ് മുസ്‌ലിമിലെ കിതാബുൽ ‘ഹുദൂദി’ൽ ബാബു ‘റജ്‌മി ഥയ്യിബി ഫിസ്സിനാ’യിലും സുനനു അബൂദാവൂദിലെ കിതാബുൽ ‘ഹുദൂദി’ൽ ബാബുൻ ‘ഫീ റജ്‌മിലും’ ഈ ഹദീഥ് ചെറിയ വ്യത്യാസങ്ങളോടെ നിവേദനം ചെയ്തിട്ടുണ്ട്.

“ആയിശ(റ)യിൽ നിന്ന് നിവേദനം; അവർ പറഞ്ഞു: വിവാഹബന്ധം നിഷിദ്ധമാകുന്ന രീതിയിൽ മുലകുടി ബന്ധം സ്ഥാപിക്കപ്പെടണമെങ്കിൽ പത്ത് തവണ മുല കുടിക്കണമെന്ന് ഖുർആനിൽ അവതരിക്കപ്പെട്ടിരുന്നു; അത് അഞ്ചു തവണയെന്നാക്കി ദുർബലപ്പെടുത്തപ്പെട്ടു; പ്രവാചകൻ (സ) മരണപ്പെട്ടു; അതിന്നു മുൻപ് അത് ഖുർആനിൽ പാരായണം ചെയ്തിരുന്നു.” (സ്വഹീഹ് മുസ്‌ലിം, കിതാബുർ റിദ്വാഅ, ബാബുത്തുഹ്‌രീമി ബി ഖംസി റദ്വആത്തിൻ)

പാരായണം ദുർബലപ്പെടുത്തി വിധി നിലനിർത്തിയ നസ്‌ഖിന് ഉദാഹരണമായി വ്യഭിചാരിയെ കല്ലെറിഞ്ഞുകൊല്ലണമെന്ന വചനവും പാരായണവും വിധിയും ദുർബലപ്പെടുത്തിയ നസ്‌ഖിന് ഉദാഹരണമായി മുലകുടി ബന്ധത്തെക്കുറിച്ച വചനവും പണ്ഡിതന്മാർ ഉദ്ധരിച്ചിട്ടുണ്ട്.

പ്രവാചകാനുചരന്മാരൊന്നും തന്നെ വ്യഭിചാരിയെ കല്ലെറിഞ്ഞുകൊല്ലണമെന്ന് കല്പിക്കുന്ന വചനം ഖുർആനിന്റെ ഭാഗമായി എഴുതിവെച്ചിട്ടില്ലെന്ന സത്യം വചനം ദുർബലപ്പെട്ടതാണെന്ന വസ്തുതയും പ്രസംഗപീഠത്തിൽ വെച്ച് ഈ വചനത്തിലെ നിയമം നിലനിൽക്കുന്നതാണെന്ന് ഉമർ (റ) പറഞ്ഞപ്പോൾ അവിടെയുണ്ടായിരുന്ന പ്രവാചകാനുചരന്മാരിൽ ഒരാൾ പോലും അതിന്നെതിരെ യാതൊന്നും പറഞ്ഞില്ലെന്ന സത്യം ഇതിലെ നിയമം നിലനിൽക്കുന്നെണ്ടെന്ന വസ്തുതയുമാണ് വ്യക്തമാക്കുന്നതെന്നും ഇമാം നവവി സമർത്ഥിക്കുന്നുണ്ട്. (ഇമാം നവവി: ശറഹ് സ്വഹീഹ് മുസ്‌ലിം, കിതാബുൽ ഹുദൂദ്, ബാബു ‘റജ്‌മി ഥയ്യിബി ഫിസ്സിനാ’ 3201ആം നമ്പർ ഹദീഥിന്റെ വ്യാഖ്യാനം)

പത്ത് മുലകുടിയിലൂടെയാണ് മുലകുടിബന്ധം സ്ഥാപിക്കപ്പെടുകയെന്ന് പഠിപ്പിക്കുന്ന വചനം ഖുർആനിലുണ്ടായിരുന്നുവെന്നും അത് പിന്നീട് ദുർബലപ്പെടുത്തുകയാണുണ്ടായത് എന്നുമുള്ള ആയിശ(റ)യുടെ പരാമർശത്തിൽ നിന്ന് വചനങ്ങളും വിധികളും ദുർബലപ്പെടുത്തുന്ന രീതി പ്രവാചകാനുചരന്മാർക്ക് പരിചയമുണ്ടായിരുന്നുവെന്ന് വ്യക്തമാണ്. മുഹമ്മദ് നബി (സ) ജീവിച്ചിരിക്കുമ്പോൾ തന്നെയാണ് ഇത് സംഭവിച്ചതെന്നും ഈ ഹദീഥ് വ്യക്തമാക്കുന്നുണ്ട്. പ്രവാചകൻ (സ) പഠിപ്പിക്കുകയും അനുചരന്മാരെങ്കിലും എടുത്ത് മാറ്റുകയും ചെയ്ത സൂക്തങ്ങളോ ഖുർആനിൽ നിന്ന് അബദ്ധവശാൽ നഷ്ടപ്പെട്ടുപോയതോ ആട് തിന്നു നശിച്ചതിനാൽ വിസ്മരിക്കപ്പെട്ടതോ ആയ വചനങ്ങളോ അല്ല ഇവയൊന്നും തന്നെ. ഖുർആൻ ജനങ്ങൾക്ക് പറഞ്ഞു കൊടുത്ത പ്രവാചകൻ (സ) തന്നെ, തന്റെ അനുചരന്മാരോട് ഖുർആൻരേഖയിൽ എഴുതേണ്ടതില്ലെന്ന് കൽപിക്കുകയും ദുർബലപ്പെടുത്തുകയും ചെയ്ത വചനങ്ങളാണിവ. അല്ലാഹു അവതരിപ്പിച്ച ചില വചനങ്ങൾ നില നിർത്തേണ്ടതില്ലെന്ന് അവൻ തന്നെ തീരുമാനിച്ചതിനുള്ള ഉദാഹരങ്ങൾ മാത്രമാണ് ഇവ. ഈ വചനങ്ങൾ ഇന്ന് പാരായണം ചെയ്യുന്ന ഖുർആനിൽ ഇല്ലാത്തത് അത് ആട് തിന്നു പോയത് കൊണ്ടല്ല, പ്രത്യുത അവ പഠിപ്പിച്ച പ്രവാചകൻ തന്നെ അവ ഖുർആനിന്റെ ഭാഗമായി സംരക്ഷിക്കേണ്ടതില്ല എന്ന് നിഷ്കർഷിച്ചതിനാലാണ്.

രണ്ട്) ആട് തിന്നു പോയത് വഴി നഷ്ടപ്പെട്ടുവെന്നാരോപിക്കുന്ന വചനങ്ങളെല്ലാം ഇന്നും നിലനിൽക്കുന്നുണ്ട്. വ്യത്യസ്തങ്ങളായ ഹദീഥ് ഗ്രൻഥങ്ങളിൽ അവ നമുക്ക് കാണാൻ കഴിയും. വിവാഹിതരായ വ്യഭിചാരികളെ എറിഞ്ഞു കൊല്ലണമെന്ന ആട് തിന്ന് നശിപ്പിച്ചത് വഴി ഖുർആനിൽ നിന്ന് നഷ്ടപ്പെട്ടുപോയി എന്ന് ആരോപിക്കപ്പെടുന്ന വചനമെടുക്കുക. ഇമാമുമാരായ അബ്ദുല്ലാഹി ബ്നു ഇമാം അഹ് മദിന്റെ സവാഹിദുൽ മുസ്നദിലും (21207) അബ്ദുർറസാഖിന്റെ മുസന്നഫിലും (5990) ഇബ്നു ഹിബ്ബാനിന്റെ സ്വഹീഹയിലും (4428) ഹാക്കിമിന്റെ മുസ്തദ്‌റക്കിലും (8068) ബൈഹഖിയുടെ അസ്സുനനിലും (16911) ഇബ്നു ഹസമിന്റെ അൽ മുഹല്ലയിലും (12/175) സ്വഹീഹായ സനദോടെ ഉദ്ധരിക്കപ്പെട്ട ഹദീഥിൽ സൂറത്തുൽ അഹ്സാബിന്റെ ഭാഗമായി “വൃദ്ധനോ വൃദ്ധയോ വ്യഭിചരിച്ചാൽ അവരെ കല്ലെറിഞ്ഞു കൊല്ലുക; അവർക്ക് അല്ലാഹുവിൽ നിന്നുള്ള ശിക്ഷയാണത്; പ്രതാപവാനും യുക്തിജ്ഞനുമാകുന്നു അല്ലാഹു”(الشَّيْخُ وَالشَّيْخَةُ إِذَا زَنَيَا فَارْجُمُوهُمَا الْبَتَّةَ نَكَالًا مِنَ اللهِ وَاللهُ عَزِيزٌ حَكِيمٌ) വെന്ന വചനം തങ്ങൾ പാരായണം ചെയ്യാറുണ്ടായിരുന്നുവെന്ന് ഉബയ്യു ബ്നു കഅബ് (റ) നിവേദനം ചെയ്തതായി കാണാൻ കഴിയും. ആട് തിന്നതു മൂലമോ സമുദായത്തിന്റെ അശ്രദ്ധയാലോ ഈ വചനം നഷ്ടപ്പെടുകയല്ല ഉണ്ടായതെന്നും ഈ വചനത്തെയും അതിലെ വിധിയെയും വചനം ദുർബലപ്പെട്ടതിനാൽ ഖുർആനിന്റെ ഭാഗമായി അത് പാരായണം ചെയ്യാൻ പാടില്ലെന്ന കാര്യവുമെല്ലാം പ്രവാചകാനുചരന്മാർക്ക് കൃത്യമായി അറിയാമായിരുന്നുവെന്ന് വ്യക്തമാക്കുന്നതാണ് ഇത്തരം ഹദീഥുകൾ. വചനം ദുർബലപ്പെടുത്തുകയും അതിലുള്ള വിധി നിലനിർത്തുകയും ചെയ്ത നസ്ഖിന് ഉദാഹരണമായി സ്വഹാബിമാർ മനസ്സിലാക്കുകയും പണ്ഡിതന്മാർ ഉദ്ധരിക്കുകയും ചെയ്തതാണ് ഈ വചനം. ആട് തിന്നു പോവുക വഴി ഖുർആനിൽ നിന്ന് നഷ്ടപ്പെടുകയും പിന്നീട് വിസ്മരിക്കപ്പെടുകയും ചെയ്ത വചനമായിരുന്നു ഇതെങ്കിൽ പിന്നെയെവിടെനിന്നാണ് ഈ ഹദീഥ് ഉദ്ധരിച്ചവർക്ക് അവ ലഭിച്ചത്?!

മൂന്ന്) ഇമാം ഇബ്നു മാജ തന്റെ സുനനിലും ഇമാം അഹ്‌മദ്‌ തന്റെ മുസ്നദിലും ഉദ്ധരിച്ചിരിക്കുന്ന ഹദീഥിന്റെ നിവേദക പരമ്പര ഇങ്ങനെയാണ്: ആയിശ(റ)യിൽ നിന്ന് അംറാ ബിൻത് അബ്ദുർ റഹ്‌മാനും അവരിൽ നിന്ന് അബ്ദുല്ലാഹി ബ്നു അബീബക്ർ ഇബ്ൻ ഹസമും അവരിൽ നിന്ന് മുഹമ്മദ് ബ്നു ഇസ്ഹാഖും അവരിൽ നിന്ന് അബ്ദുൽ അഅലായും അവരിൽ നിന്ന് അബൂസലമഃ യഹ്‌യ ബിൻ ഖലഫും നിവേദനം ചെയ്യുന്നു. ആയിശ- അംറാ ബിൻത് അബ്ദുർ റഹ്‌മാൻ- അബ്ദുല്ലാഹി ബ്നു അബീബക്ർ ആണ് ഈ നിവേദക പരമ്പരയിലെ അടിസ്ഥാനപരമായ കണ്ണി. അബ്ദുല്ലാഹി ബ്നു അബീബകറിൽ നിന്ന് ഈ ഹദീഥിന്റെ ആദ്യഭാഗം നിരവധി പേർ നിവേദനം ചെയ്തിട്ടുണ്ട്. യഹ്‌യ ബിൻ സഈദ് അൽഅൻസാരി, ഇമാം മാലിക്ക് എന്നിവർ അവരിൽ പ്രധാനികളാണ്. സ്വഹീഹ് മുസ്‌ലിമിലെ കിതാബുർ റിദ്വാഇലും (ഹദീഥ് 1452) മുവത്വയിലെ കിതാബുർ റിദ്വാഇലും (ഹദീഥ് 17) അബ്ദുല്ലാഹി ബ്നു അബീബകർ- യഹ്‌യ ബിൻ സഈദ് അൽഅൻസാരി- മാലിക്ക് ബ്നു അനസ് എന്ന നിവേദകപരമ്പരയോടെയാണ് ഇത് നിവേദനം ചെയ്തിരിക്കുന്നത്. ആ നിവേദകപരമ്പരയിലുള്ള ഹദീഥിൽ ആട് തിന്ന വർത്തമാനമേയില്ല. ആ ഹദീഥ് ഇങ്ങനെയാണ്: “ആയിശ(റ)യിൽ നിന്ന് നിവേദനം; അവർ പറഞ്ഞു: വിവാഹബന്ധം നിഷിദ്ധമാകുന്ന രീതിയിൽ മുലകുടി ബന്ധം സ്ഥാപിക്കപ്പെടണമെങ്കിൽ പത്ത് തവണ മുല കുടിക്കണമെന്ന് ഖുർആനിൽ അവതരിക്കപ്പെട്ടിരുന്നു; അത് അഞ്ചു തവണയെന്നാക്കി ദുർബലപ്പെടുത്തപ്പെട്ടു; പ്രവാചകൻ (സ) മരണപ്പെട്ടു; അതിന്നു മുൻപ് അത് ഖുർആനിൽ പാരായണം ചെയ്തിരുന്നു”.

ഒരേ അബ്ദുല്ലാഹി ബ്നു അബീബക്കറിൽ(റ) നിന്ന് ആയിശ (റ) പറഞ്ഞ ഈ സംഭവം നിവേദനം ചെയ്ത മുഹമ്മദ് ബ്നു ഇസ്ഹാഖൊഴിച്ച് മറ്റാരും തന്നെ അദ്ദേഹത്തിൽ നിന്ന് ആട് തിന്ന സംഭവം, അങ്ങനെയൊരു സംഭവം അദ്ദേഹം പറഞ്ഞു കൊടുത്തിരുന്നുവെങ്കിൽ കേട്ടില്ല എന്ന് കരുതാൻ നിർവാഹമില്ല. അബ്ദുല്ലാഹി ബ്നു അബീബകറിൽ നിന്ന് ആട് തിന്ന സംഭവമില്ലാതെ ഈ ഹദീഥിന്റെ ബാക്കി ഭാഗം നിവേദനം ചെയ്ത യഹ്‌യ ബിൻ സഈദാകട്ടെ ഹദീഥ് നിവേദനത്തിൽ സ്വീകരിക്കുന്ന സൂക്ഷ്മതയുടെയും കൃത്യതയുടെയും കാര്യത്തിൽ ഏറെ പ്രസിദ്ധനായ വ്യക്തിയുമാണ്. ഹദീഥ് വിജ്ഞാനീയത്തിൽ പ്രഗത്ഭനായ സുഫ്‌യാനു സൗരി പറഞ്ഞത് മദീനക്കാരെ സംബന്ധിച്ചിടത്തോളം ഹദീഥുകളുടെ കാര്യത്തിൽ ഇമാം സുഹ്‌രിയേക്കാൾ മഹത്വമുള്ളയാളായാണ് യഹ്‌യ ബിൻ സഈദ് എന്നാണ്. ഏറ്റവുമധികം പ്രാമാണികവും സത്യസന്ധവുമായി ഹദീഥുകൾ നിവേദനം ചെയ്യുന്നയാൾ എന്ന് അലി ബിൻ അൽ മദീനിയും കൃത്യവും കളങ്കങ്ങളൊന്നും കടന്നുവരാത്തതുമായ രീതിയിൽ നിവേദനം ചെയ്യുന്നയാൾ എന്ന് ഇമാം അഹ്‌മദ്‌ ബിൻ ഹമ്പലും സാക്ഷ്യപ്പെടുത്തിയ വ്യക്തികൂടിയാണ് അദ്ദേഹം. (ഇബ്നു ഹജറുൽ അസ്ഖലാനി: തഹ്ദീബ് അത്തഹ്ദീബ് 11/223)

യഹ്‌യ ബിൻ സഈദ് പറഞ്ഞതിനെതിരെ നിവേദനം ചെയ്യുമ്പോൾ ഹദീഥ് നിദാനശാസ്ത്രജ്ഞരെ സംബന്ധിച്ചിടത്തോളം മുഹമ്മദ് ബ്നു ഇസ്‌ഹാഖ്‌ തീരെ സ്വീകാര്യനാവുകയില്ല. ഒരേ അബ്ദുല്ലാഹി ബ്നു അബീബക്കറിൽ നിന്ന് ഒരേ സംഭവം നിവേദനം ചെയ്യുമ്പോൾ യഹ്‌യ ബിൻ സഈദ് കേട്ടതിനേക്കാളധികം മുഹമ്മദ് ബ്നു ഇസ്ഹാഖ്‌ കേട്ടുവെന്ന് പറഞ്ഞാൽ അത് വിശ്വാസയോഗ്യമായി അവർ അംഗീകരിക്കുകയില്ല. മുഹമ്മദ് ബ്നു ഇസ്ഹാഖിനെ തെളിവിന് കൊള്ളുകയില്ലെന്നും പ്രവാചകചര്യയുമായി ബന്ധപ്പെട്ട കാര്യങ്ങളിൽ അദ്ദേഹത്തിൽ നിന്നുള്ള നിവേദനങ്ങൾ സ്വീകരിക്കാനാവില്ലെന്നും അദ്ദേഹം മാത്രമായി നിവേദനം ചെയ്തതാണെങ്കിൽ പ്രസ്തുത ഹദീഥ് താൻ സ്വീകാര്യമായി ഗണിക്കുകയില്ലെന്നും പല നിവേദകരുടെയും വർത്തമാനങ്ങൾ ഒരൊറ്റ ഹദീഥിൽ ഉൾപ്പെടുത്തുകയും അവ തമ്മിൽ വേർതിരിക്കാതിരിക്കുകയും ചെയ്യുന്ന സ്വഭാവം അദ്ദേഹത്തിനുണ്ടെന്നുമെല്ലാം ഇമാം അഹ്‌മദ്‌ ബിൻ ഹമ്പൽ പറഞ്ഞതായി ഇമാമിന്റെ മക്കൾ തന്നെ നിവേദനം ചെയ്തിട്ടുണ്ട്. മുഹമ്മദ് ബ്നു ഇസ്ഹാഖ്‌ ദുർബലനാണെന്ന് യഹ്‌യ ബിൻ മഈനും, ശക്തനല്ലെന്ന് നസാഇയും പ്രാമാണികമല്ലെങ്കിലും മറ്റു നിവേദനങ്ങളോടൊപ്പം സ്വീകരിക്കാവുന്ന നിവേദനങ്ങളാണ് അദ്ദേഹത്തിന്റേതെന്ന് ദാറഖുത്‌നിയും പറഞ്ഞതായി കാണാനാവും. (തഹ്ദീബ് അത്തഹ്ദീബ് 9/45)

അബ്ദുല്ലാഹി ബ്നു അബീബക്കറിൽ നിന്നല്ലാതെയുള്ള മറ്റൊരു പരമ്പരയോട് കൂടി ഇതേ സംഭവം നിവേദനം ചെയ്യപ്പെട്ടപ്പോഴും അതിൽ ആട് തിന്ന കഥയില്ലെന്നതും പ്രസ്തുത കഥയുടെ സത്യസന്ധതയെ ചോദ്യം ചെയ്യുന്നതാണ്. ആയിഷയിൽ നിന്ന് കേട്ടതായി അംറയും അവരിൽ നിന്ന് കേട്ടതായി അൽഖാസിമു ബ്നു മുഹമ്മദും അദ്ദേഹത്തിൽ നിന്ന് അബ്ദു റഹ്‌മാന്‌ ബ്നു ഖാസിമും അദ്ദേഹത്തിൽ നിന്ന് ഹമ്മാദ് ബ്നു സലാമയും അദ്ദേഹത്തിൽ നിന്ന് അൽഹജ്‌ജാജ് ബ്നു മിൻഹാലും അദ്ദേഹത്തിൽ നിന്ന് മുഹമ്മദ് ബ്നു ഖുസൈമയും നിവേദനം ചെയ്യുന്ന ഇതേ സംഭവം പ്രതിപാദിക്കുന്ന ഹദീഥ് അത്തഹാവി തന്റെ ശറഹ് മുഷ്‌കിലൽ അഥറിൽ(11/486) ഉദ്ധരിക്കുന്നുണ്ട്. അതിൽ ആയിശ പറയുന്നതായി “വിവാഹബന്ധം നിഷിദ്ധമാകുന്ന രീതിയിൽ മുലകുടി ബന്ധം സ്ഥാപിക്കപ്പെടണമെങ്കിൽ പത്ത് തവണ മുല കുടിക്കണമെന്ന് ഖുർആനിൽ അവതരിക്കപ്പെട്ടിരുന്നു; അത് അഞ്ചു തവണയെന്നാക്കി ദുർബലപ്പെടുത്തപ്പെട്ടു”എന്ന് മാത്രമേയുള്ളൂ. ആട് തിന്ന സംഭവത്തിലേക്ക് യാതൊരു സൂചനയും അവിടെ നൽകുന്നില്ല. ആട് തിന്ന സംഭവത്തെക്കുറിച്ച വിവരങ്ങൾ തീരെ ദുർബലവും സംഭവിച്ചിരിക്കുവാൻ തീരെ സാധ്യതയില്ലാത്തതുമാണ് എന്ന് വ്യക്തമാക്കുന്ന ബലവത്തായ തെളിവുകളാണ് ഇവയെല്ലാം.

നാല്) ഇമാം ഇബ്നു മാജ തന്റെ സുനനിലും ഇമാം അഹ്‌മദ്‌ തന്റെ മുസ്നദിലും ഉദ്ധരിച്ചിരിക്കുന്ന ഹദീഥിൽ പറഞ്ഞത് പോലെ “കല്ലെറിയലിന്റെ വചനവും പത്ത് പ്രാവശ്യമാണ് മുലകുടിയെന്ന വചനവും എഴുതിയ തന്റെ തലയിണക്കടിയിലുണ്ടായിരുന്ന രേഖ ദൈവദൂതന്റെ മരണവുമായി ബന്ധപ്പെട്ട തിരക്കുകൾക്കിടയിലായിരുന്നപ്പോൾ ഒരു ആട് അകത്ത് കടന്ന് തിന്നു കളഞ്ഞു”വെന്ന ആയിശ(റ)യുടെ വർത്തമാനം സത്യമാണെങ്കിൽ തന്നെ അതിൽ നിന്നെങ്ങനെയാണ് ഖുർആൻ സംരക്ഷിക്കപ്പെട്ടിട്ടില്ലെന്ന് വാദിക്കാൻ കഴിയുക?! അതിൽ നിന്നെങ്ങനെയാണ് ഖുർആനിൽ നിന്ന് ഒരു വചനം നഷ്ടപ്പെട്ടു പോയി എന്നു വരിക? ഈ ഹദീഥിലുള്ളത് അവരുടെ കയ്യിലുണ്ടായിരുന്ന ഈ വചനങ്ങൾ എഴുതിയ രേഖ ആട് തിന്നു എന്ന് മാത്രമാണ്. അവരുടെ കയ്യിൽ മാത്രമായിരുന്നില്ല ആ വചനങ്ങൾ എഴുതി വെച്ചതായി ഉണ്ടായിരുന്നത് എന്ന് നേരത്തെ പറഞ്ഞ ഹദീഥുകളിൽ നിന്ന് വ്യക്തമാണ്. അപ്പോൾ പിന്നെയെങ്ങനെയാണ് ആയിശയുടെ അടുത്തുള്ള രേഖ ആട് തിന്നതു കൊണ്ട് ആ വചനങ്ങൾ നഷ്ടപ്പെട്ടുവെന്ന് വാദിക്കുക?! ഖുർആനിൽ എന്നെന്നും നിലനിർത്തേണ്ട വചനങ്ങൾ ഉൾക്കൊള്ളുന്ന രേഖയായി അതിനെ ആയിശയോ മറ്റ് പ്രവാചകാനുചരന്മാരോ പരിഗണിച്ചതായി വ്യക്തമാക്കുന്ന തെളിവുകളൊന്നുമില്ല. നസ്ഖ് ചെയ്യപ്പെട്ട ആയത്തുകൾ ആയിശ അവരുടെ കയ്യിലുണ്ടായിരുന്ന ഒരു രേഖയിൽ എഴുതിവെച്ചിരുന്നു; തിരക്കുകൾക്കിടയിൽ ആ രേഖ ആട് തിന്നുപോയി. ഖുർആനിൽ നിന്ന് എന്തെങ്കിലും നഷ്ടപ്പെടുകയോ അതിലേക്ക് എന്തെങ്കിലും കൂട്ടിച്ചേർക്കുകയോ ചെയ്തിട്ടുണ്ടെന്ന് ഈ സംഭവം തെളിയിക്കുന്നേയില്ല.

സൂറത്തു തൗബയിലും സൂറത്തുൽ അഹ്സാബിലുമുള്ള ഏതാനും വചനങ്ങൾ അബൂബക്കറി(റ)ന്റെ കാലത്ത് ക്രോഡീകരിക്കപ്പെട്ട ഖുർആനിൽ ആദ്യം ഉണ്ടായിരുന്നില്ലെന്നും മദീനയിലുള്ള ഒരാളിൽ നിന്ന് മാത്രമാണ് അവ ലഭിച്ചതെന്നും അങ്ങനെ ലഭിച്ചതിനു ശേഷം അവ ഖുർആനിൽ കൂട്ടിച്ചേർക്കുകയാണുണ്ടായതെന്നും വ്യക്തമാക്കുന്ന രേഖകളുണ്ടല്ലോ. ഇവ ഖുർആൻ വചനങ്ങളെല്ലാം നിരവധി പേരിലൂടെ സംപ്രേഷണം ചെയ്താണ് (മുതവാത്തിറായാണ്) നമ്മിലേക്കെത്തിയിട്ടുള്ളതെന്ന അവകാശവാദത്തെയും പ്രവാചകവിയോഗം കഴിഞ്ഞ കാലത്ത് ഖുർആൻ പൂർണമായി അറിയാവുന്ന നിരവധി പേർ ഉണ്ടായിരുന്നുവെന്ന വാദത്തെയും ചോദ്യം ചെയ്യുന്നവയല്ലേ?

അബൂബക്കറി(റ)ന്റെ കാലത്ത് നടന്ന ഖുർആൻ ക്രോഡീകരണത്തെക്കുറിച്ച് പ്രതിപാദിക്കുന്ന ഹദീഥുകളുടെ വെളിച്ചത്തിലാണ് ഈ വിമർശനം ഉന്നയിക്കപ്പെടുന്നത്. സ്വഹീഹുൽ ബുഖാരിയിലെ ഇവ്വിഷയകമായ ഹദീഥുകൾ പരിശോധിക്കുക:

സൈദു ബ്നു ഥാബിത് (റ) പറയുന്നു: …….. ഞാന്‍ അന്വേഷണം ആരംഭിച്ചു. പനയോലകള്‍, മിനുസമായ പരന്ന കല്ലുകള്‍, മനഃപ്പാഠമായുള്ളവരുടെ ഹൃദയങ്ങള്‍ എന്നിവയില്‍ നിന്നും അതിനെ ഒരുമിച്ചു കൂട്ടി. സൂറഃ തൗബയിലെ അവസാനത്തെ രണ്ടു ആയത്തുകള്‍ ഖുസൈമ ബിൻ ഥാബിത് അൽ അന്‍സാരി (റ) യിൽ നിന്നു ലഭിക്കുന്നത് വരെ എന്റെ അന്വേഷണം തുടർന്നു. അദ്ദേഹത്തിൽ നിന്നല്ലാതെ മറ്റാരിൽ നിന്നും എനിക്കത് ലഭിച്ചില്ല. സൂറത്തുതൗബയിലെ “തീര്‍ച്ചയായും നിങ്ങള്‍ക്കിതാ നിങ്ങളില്‍ നിന്നുതന്നെയുള്ള ഒരു ദൂതന്‍ വന്നിരിക്കുന്നു. നിങ്ങള്‍ കഷ്ടപ്പെടുന്നത്‌ സഹിക്കാന്‍ കഴിയാത്തവനും, നിങ്ങളുടെ കാര്യത്തില്‍ അതീവതാല്‍പര്യമുള്ളവനും, സത്യവിശ്വാസികളോട്‌ അത്യന്തം ദയാലുവും കാരുണ്യവാനുമാണ്‌ അദ്ദേഹം. എന്നാല്‍ അവര്‍ തിരിഞ്ഞുകളയുന്ന പക്ഷം (നബിയേ,) നീ പറയുക: എനിക്ക്‌ അല്ലാഹു മതി. അവനല്ലാതെ ഒരു ദൈവവുമില്ല. അവന്‍റെ മേലാണ്‌ ഞാന്‍ ഭരമേല്‍പിച്ചിരിക്കുന്നത്‌. അവനാണ്‌ മഹത്തായ സിംഹാസനത്തിന്‍റെ നാഥന്‍”(9: 128-129 ) എന്ന രണ്ട് ആയത്തുകളായിരുന്നു അവ. ആ ഏട് മരണം വരെ അബൂബക്കറിന്റെ (റ) കൈവശമായിരുന്നു. ശേഷം ജീവിതകാലം ഉമറിന്റെ (റ) കരങ്ങളിലും. ശേഷം പുത്രി ഹഫ്‌സ(റ)യുടെ കരങ്ങളിലുമായിരുന്നു. (സ്വഹീഹുൽ ബുഖാരി, കിതാബുൽ അഹ്‌കാം, ബാബു യൂസ്തഹബ്ബു് ലിൽ കാത്തിബി അൻ യക്കൂന അമീനൻ ആഖിലൻ; ജാമിഉത്തിർമിദി, കിതാബു ത്തഫ്സീർ)

സൈദ് ബിൻ ഥാബിത്ത് (റ) പറഞ്ഞു: “ഞങ്ങൾ ഖുർആൻ പകർത്തിയെഴുതിയപ്പോൾ അല്ലാഹുവിന്റെ പ്രവാചകൻ (സ) പാരായണം ചെയ്യുന്നതായി ഞാൻ കേട്ട സൂറത്തുൽ അഹ്സാബിലെ ഒരു വചനം എവിടെ നിന്നും കിട്ടിയില്ല. ഞങ്ങൾ അന്വേഷിച്ചു; അബൂ ഖുസൈമ ബിൻ ഥാബിത് അൽ അന്‍സാരി (റ)യുടെ അടുത്ത് നിന്നാണ് അവസാനം അത് ഞങ്ങൾക്ക് ലഭിച്ചത്.” സത്യവിശ്വാസികളുടെ കൂട്ടത്തില്‍ ചില പുരുഷന്മാരുണ്ട്‌. ഏതൊരു കാര്യത്തില്‍ അല്ലാഹുവോട്‌ അവര്‍ ഉടമ്പടി ചെയ്തുവോ, അതില്‍ അവര്‍ സത്യസന്ധത പുലര്‍ത്തി. അങ്ങനെ അവരില്‍ ചിലര്‍ (രക്ത സാക്ഷിത്വത്തിലൂടെ) തങ്ങളുടെ പ്രതിജ്ഞ നിറവേറ്റി. അവരില്‍ ചിലര്‍ (അത്‌) കാത്തിരിക്കുന്നു. അവര്‍ (ഉടമ്പടിക്ക്‌) യാതൊരു വിധ മാറ്റവും വരുത്തിയിട്ടില്ല”(33: 23) എന്ന വചനമായിരുന്നു അത്.” (സ്വഹീഹുൽ ബുഖാരി, കിതാബ് ഫദാഇലിൽ ഖുർആൻ, ബാബു ജംഇൽ ഖുർആൻ)

ഖുസൈമ ബിൻ ഥാബിത് അൽ അന്‍സാരി(റ)യുടെ പക്കൽ നിന്ന് മാത്രമേ സൂറത്തു തൗബയിലെ അവസാനത്തെ രണ്ട് ആയത്തുകളും സൂറത്തുൽ അഹ്സാബിലെ ഇരുപത്തി മൂന്നാമത്തെ ആയത്തും ലഭിച്ചുള്ളൂവെന്ന് പറയുമ്പോൾ അവ അറിയാവുന്നയാളായി മദീനയിൽ അദ്ദേഹം മാത്രമേയുണ്ടായിരുന്നുള്ളൂ എന്നാണ് അർത്ഥമാക്കുന്നത് എന്ന മുൻധാരണയുടെ അടിസ്ഥാനത്തിലാണ് ഈ വിമർശനം ഉന്നയിക്കപ്പെടുന്നത്. യഥാർത്ഥത്തിൽ ഈ മുൻധാരണയെ ഈ വിഷയം പ്രതിപാദിക്കുന്ന ഹദീഥുകൾ തന്നെ തിരുത്തുന്നുണ്ട്. “ഖുർആൻ പകർത്തിയെഴുതിയപ്പോൾ അല്ലാഹുവിന്റെ പ്രവാചകൻ (സ) പാരായണം ചെയ്യുന്നതായി ഞാൻ കേട്ട സൂറത്തുൽ അഹ്സാബിലെ ഒരു വചനം എവിടെ നിന്നും കിട്ടിയില്ല.” എന്ന സൈദ് ബിൻ ഥാബിത്തിന്റെ(റ) പ്രസ്താവന ശ്രദ്ധിക്കുക. ഇതിന്നർത്ഥമെന്താണ്? സൈദ് ബിൻ ഥാബിത്തടക്കം പല സ്വഹാബിമാരും പ്രവാചകനിൽ നിന്ന് ഈ വചനം കേട്ടിട്ടുണ്ട്; ഖുർആൻ മനഃപാഠമുള്ള സൈദിന് ഈ വചനം അറിയുകയും ചെയ്യാം. പക്ഷെ, പ്രവാചകന്റെ(സ) കാലത്ത് എഴുതപ്പെട്ട രേഖകളിലൊന്നും സൈദി(റ)നും കൂട്ടുകാർക്കും ഈ വചനം കണ്ടെത്താൻ കഴിഞ്ഞില്ല. അത് അവസാനം ഖുസൈമ ബിൻ ഥാബിത് അൽ അന്‍സാരി (റ)യുടെ പക്കൽ നിന്നാണ് കിട്ടിയത്. മനഃപാഠത്തെ മാത്രം ആശ്രയിക്കാതെ രേഖകളിൽ കൂടിയുണ്ടെന്ന് ഉറപ്പു വരുത്തിയ ശേഷമായിരിക്കണം ഖുർആൻ വചനങ്ങനല്ലാം രേഖീകരിക്കേണ്ടത് എന്ന ഭരണാധികാരിയായ അബൂബക്കറിന്റെ നിർദേശം നിഷ്‌കൃഷ്ടമായി അനുസരിക്കുന്ന പ്രവാചകാനുയായിയുടെ ചിത്രം മാത്രമാണ് ഈ ഹദീഥുകളിൽ നാം കാണുന്നത്.

ഇക്കാര്യം ബുഖാരിയുടെ വ്യാഖ്യാതാവായ ഇബ്നു ഹജറുൽ അസ്ഖലാനി (റ) തന്നെ വ്യക്തമാക്കിയിട്ടുണ്ട്.

“ഖുർആനല്ലാത്ത മറ്റൊന്നുംതന്നെ അതോടൊപ്പം എഴുതി വെക്കരുതെന്ന് പ്രവാചക(സ)ന്റെ കല്പനയുണ്ടായിരുന്നു. പ്രവാചകകാലത്ത് എഴുതിയതല്ലാതെ ഖുർആനായി യാതൊന്നും തന്നെ എഴുതേണ്ടതില്ലെന്ന അബൂബക്കറി(റ)ന്റെ നിർദേശവുമുണ്ടായിരുന്നു. സൈദ് ബ്നു ഥാബിത്തും(റ) അദ്ദേഹത്തോടോപ്പമുള്ളവരും സൂറത്തുൽ ബറാഅയിലെ അവസാനത്തെ വചനങ്ങൾ അറിയാവുന്നവരായിരുന്നുവെങ്കിലും പ്രവാചകകാലത്ത് തന്നെ എഴുതപ്പെട്ട രേഖകളിലേതിലെങ്കിലും അവ കണ്ടെത്തുന്നത് വരെ അവർ അവ ഖുർആനിൽ എഴുതിച്ചേർക്കാതിരുന്നത് അതുകൊണ്ടായിരുന്നു…….

ഉമർ (റ) പറഞ്ഞു: പ്രവാചകനിൽ നിന്ന് ഖുർആനുമായി ബന്ധപ്പെട്ട എന്തെങ്കിലും ലഭിച്ചിട്ടുളളവരെല്ലാം അവ കൊണ്ട് വരട്ടെ. ഈത്തപ്പനയോലകളിലും തോൽച്ചുരുളുകളിലും പലകകളിലുമായിരുന്നു അവർ അത് എഴുതിയിരുന്നത്. രണ്ട് പേർ സാക്ഷ്യപ്പെടുത്താതെ അവരിൽ നിന്ന് യാതൊന്നും സ്വീകരിക്കേണ്ടതില്ല. അതുകൊണ്ട് തന്നെ ഏതെങ്കിലുമൊരു ആയത്തുൾക്കൊള്ളുന്ന ഒരു രേഖ ലഭിച്ചാൽ അത് തനിക്ക് മനഃപാഠമുള്ളതാണെങ്കിലും ആരുടെയെങ്കിലും സാക്ഷ്യമില്ലാതെ സ്വീകരിക്കുവാൻ സൈദ് (റ) വൈമനസ്യം പ്രകടിപ്പിച്ചു. ഇക്കാര്യത്തിൽ നന്നായി ശ്രദ്ധിക്കുവാൻ അവർ പ്രത്യേകം ഔൽസുക്യം കാണിച്ചിരുന്നു. ഉമറി(റ)നോടും സൈദിനോടു(റ)മായി “നിങ്ങൾ പള്ളിയുടെ വാതിൽക്കൽ ഇരിക്കുകയും ആരെങ്കിലും ഖുർആനുമായി ബന്ധപ്പെട്ട വല്ലതുമായി വന്നാൽ രണ്ട് സാക്ഷികളുണ്ടെങ്കിൽ നിങ്ങൾ അത് രേഖപ്പെടുത്തുകയും ചെയ്യുക” എന്ന് അബൂബക്കർ (റ) പറഞ്ഞതായി ഹിശാമു ബ്നു അർവ (റ) തന്റെ പിതാവിൽ(റ) നിന്ന് നിവേദനം ചെയ്തതായി അബൂദാവൂദിലുണ്ട്. പരമ്പര മുറിഞ്ഞതാണെങ്കിലും ഇതിന്റെ നിവേദകന്മാരെല്ലാം വിശ്വാസയോഗ്യരാണ്. രണ്ട് സാക്ഷികളെന്നാൽ ഒന്നുകിൽ മനഃപാഠവും രേഖയുമാണ്; അല്ലെങ്കിൽ രേഖയിലുള്ളത് പ്രവാചകസമക്ഷത്തിങ്കൽ വെച്ച് തന്നെ എഴുതിയതാണെന്നതിന് രണ്ട് പേർ സാക്ഷ്യം വഹിക്കലാണ്; അതുമല്ലെങ്കിൽ അത് ഖുർആനിൽ അവതരിപ്പിക്കപ്പെട്ടത് തന്നെയാണെന്നതിനുള്ള രണ്ട് പേരുടെ നിഷ്‌കൃഷ്ടമായ സാക്ഷ്യമാണ്. പ്രവാചകന്റെ(സ) കാലത്ത് എഴുതപെട്ടതാണെന്ന് ഉറപ്പില്ലാത്ത യാതൊന്നും തന്നെ കേവലം മനഃപാഠത്തിന്റെ മാത്രം അടിസ്ഥാനത്തിൽ സ്വീകരിക്കേണ്ടതില്ലെന്നായിരുന്നു അവരുടെ നയം.” (ഇബ്നു ഹജറുൽ അസ്ഖലാനി; ഫത്ഹുൽ ബാരി, കിതാബു ഫദാഇലിൽ ഖുർആൻ, ബാബു ജംഇൽ ഖുർആൻ, 4603 ആം നമ്പർ ഹദീഥിന്റെ വ്യാഖ്യാനം)

സൂറത്തുൽ ബറാഅയിലെ അവസാനത്തെ വചനങ്ങളും അഹ്സാബിലെ ഇരുപത്തിമൂന്നാം വചനവും സൈദ്‌ ബ്നു ഥാബിത്തിനും(റ) സ്വഹാബിമാർക്കും മനഃപാഠമുണ്ടായിരുന്നെങ്കിലും അതിന്റെ മാത്രം അടിസ്ഥാനത്തിൽ അവ തങ്ങളുണ്ടാക്കുന്ന ഖുർആൻരേഖയിൽ എഴുതിച്ചേർക്കുവാൻ അവർ ഒരുക്കമായിരുന്നില്ല. അതുകൊണ്ടാണ് തങ്ങൾക്കറിയാവുന്ന ആയത്തുകൾ ഉൾക്കൊള്ളുന്ന രേഖയന്വേഷിച്ച് അവർ പ്രായാസപ്പെട്ടത്. ഖുസൈമ ബിൻ ഥാബിത് അൽ അന്‍സാരി(റ)യുടെ പക്കൽ നിന്ന് പ്രസ്തുത രേഖകൾ ലഭിച്ചതിന്റെ സന്തോഷമാണ് സൈദ് ബ്നു ഥാബിത്തിന്റെ ബുഖാരിയിലുള്ള നിവേദനത്തിൽ തെളിഞ്ഞു കാണുന്നത്. തങ്ങൾക്കറിയാവുന്ന പ്രസ്തുത ആയത്തുകളെ സാക്ഷ്യപ്പെടുത്തുന്ന രേഖ ലഭിച്ചതോടെ അവർ അവ തങ്ങൾ നിർമ്മിക്കുന്ന സുഹ്‌ഫിൽ എഴുതിച്ചേർക്കുകയും ചെയ്തു.

സൂറത്തുൽ ബറാഅയിലെ അവസാനത്തെ വചനങ്ങളുടെ കാര്യത്തിൽ പ്രവാചകനിൽ നിന്ന് കേൾക്കുകയും മനഃപാഠമാക്കുകയും ചെയ്തുവെന്നതിന് ഉമറും(റ) അബ്ദുല്ലാഹിബ്നു ഉമറും(റ) തന്നെ സാക്ഷി നിന്നതായി ഇമാം ഇബ്നു കഥീർ തന്റെ തഫ്സീറിൽ 9: 129ന്റെ വ്യാഖ്യാനസന്ദർഭത്തിൽ സ്വീകാര്യമായ പരമ്പരയോടെ നിവേദനം ചെയ്യുന്നുണ്ട്.

സൂറത്തുൽ അഹ്സാബിലെ ഇരുപത്തിമൂന്നാം വചനത്തിന്റെ കാര്യത്തിലാണെങ്കിൽ സൈദു ബ്നു ഥാബിത്തും ഖുസൈമ ബിൻ ഥാബിത്തും തന്നെയായിരുന്നു പ്രവാചകനിൽ(സ) നിന്ന് നേരിട്ട് കേട്ടതായി സാക്ഷ്യം വഹിച്ച രണ്ട് സ്വഹാബിമാരെന്ന് ഇമാം ഖുർത്തുബി തന്റെ തഫ്സീർ അൽ ജാമിഅ ലി അഹ്‌കാമൽ ഖുർആനിൽ 9: 129ന്റെ വ്യാഖ്യാനസന്ദർഭത്തിലും നിവേദനം ചെയ്യുന്നുണ്ട്.

ഇനി ഖുസൈമ ബിൻ ഥാബിത്ത്(റ) മാത്രമാണ് പ്രവാചകനിൽ(സ) നിന്ന് കേട്ടതായി സാക്ഷ്യം വഹിക്കുന്നതെങ്കിൽ പോലും അദ്ദേഹത്തിന്റേത് രണ്ട് പേരുടെ സാക്ഷ്യത്തിന് തുല്യമാണെന്ന് പ്രവാചകൻ (സ) തന്നെ പറഞ്ഞിട്ടുള്ളതിനാൽ അത് മതിയാകുന്നതാണെന്ന് പണ്ഡിതന്മാർ വ്യക്തമാക്കിയിട്ടുമുണ്ട്. ഒരു ഗ്രാമീണ അറബിയുമായുള്ള ഇടപാടിനിടയിൽ തന്റെ സത്യസന്ധതയെ പരിഗണിച്ചുകൊണ്ട് തനിക്ക് സാക്ഷ്യം നിന്ന ഖുസൈമ ബിൻ ഥാബിത്തി(റ)നോട് താങ്കളുടെ സാക്ഷ്യത്തിന് രണ്ട് പേരുടെ സാക്ഷ്യത്തിന്റെ മൂല്യമുണ്ടെന്ന് പ്രവാചകൻ (സ) പറഞ്ഞതായി ഉമാറാ ബ്നു ഖുസൈമ(റ)യുടെ അമ്മാവനിൽ(റ) നിന്ന് സ്വഹീഹായ പരമ്പരയോടെ ഇമാം അബൂദാവൂദ് (റ) നിവേദനം ചെയ്യുന്നുണ്ട്. (ശൈഖ് അൽബാനിയുടെ സുനനു അബൂദാവൂദ്, ഹദീഥ് 3607). ഖുസൈമയുടെ പക്കൽ നിന്ന് മാത്രമായി സൂറത്തുൽ അഹ്സാബിലെ വചനങ്ങൾ കിട്ടിയതിനെക്കുറിച്ച് പ്രതിപാദിക്കുന്ന ഹദീഥിലും സൈദ് ബ്നു ഥാബിത്ത്‌ (റ) അദ്ദേഹത്തെക്കുറിച്ച് പറയുന്നത് ‘അല്ലാഹുവിന്റെ ദൂതൻ രണ്ടുപേരുടെ സാക്ഷ്യത്തിന് തുല്യമെന്ന് പറഞ്ഞിട്ടുള്ള ഖുസാമ ബിൻ ഥാബിത്ത്‌” എന്നാണ്. (സ്വഹീഹുൽ ബുഖാരി, കിത്താബുൽ ജിഹാദ്). ഖുസാമായെക്കൂടാതെ മറ്റാരും തന്നെ സാക്ഷികളായി ഉണ്ടായിരുന്നില്ലെങ്കിൽ പോലും ഈ വചനങ്ങൾ ഖുർആനിലുള്ളതാണെന്ന് തീരുമാനിക്കപ്പെടുമായിരുന്നുവെന്നാണ് ഇതിനർത്ഥം. എന്നാൽ സൂറത്തുൽ ബറാഅയിലും അഹ്സാബിലുമുള്ള ഖുസാമയുടെ പക്കൽ നിന്ന് ലഭിച്ച ആയത്തുകളുടെ കാര്യത്തിൽ അദ്ദേഹത്തെക്കൂടാതെ മറ്റുള്ളവരും സാക്ഷികളായി ഉണ്ടായിരുന്നുവെന്ന സത്യം നേരത്തെ ഉദ്ധരിച്ച ഹദീഥുകളിൽ നിന്ന് വ്യക്തമാണ്.

ഖുർആൻ ക്രോഡീകരണവുമായി ബന്ധപ്പെട്ട ഈ ഹദീഥുകൾ ഖുർആനിലെ ഏതെങ്കിലും ആയത്തുകളുടെ വിശ്വാസ്യതയെ ചോദ്യം ചെയ്യുകയോ നിരവധി പേരിലൂടെ സംപ്രേക്ഷണം ചെയ്യപ്പെട്ടതാണ് ഖുർആനിലെ ഓരോ ആയത്തുകളുമെന്ന അവകാശവാദത്തെ നിഷേധിക്കുകയോ ചെയ്യുന്നില്ല. മറിച്ച്, മനഃപാഠത്തോടൊപ്പം പ്രവാചകസന്നിധിയിൽ വെച്ച് എഴുതപ്പെട്ട രേഖകളെക്കൂടി പരിഗണിച്ചുകൊണ്ട് വളരെ സൂക്ഷമമായും തെറ്റുകളൊന്നും കടന്നുവരാൻ യാതൊരു സാധ്യതയുമില്ലാത്ത രൂപത്തിലുമാണ് അബൂബക്കറിന്റെ കാലത്തെ ഖുർആൻ ക്രോഡീകരണം നടന്നത് എന്ന സത്യമാണ് ഇവ വെളിപ്പെടുത്തുന്നത്. ഖുർആൻ വചനങ്ങളുടെ സമാഹരണത്തിലും ക്രോഡീകരണത്തിലും പ്രവാചകശിഷ്യന്മാർ കാണിച്ച സൂക്ഷ്മതയും കൃത്യതയും ആരുടെയും ആദരവ് പിടിച്ച് പറ്റുന്നതാണെന്ന യാഥാർഥ്യത്തിന് ഈ ഹദീഥുകൾ അടിവരയിടുകയും ചെയ്യുന്നു.

ഉബയ്യു ബ്നു കഅ്ബി(റ)ന്റെ മുസ്ഹഫിൽ ഇന്നത്തെ മുസ്ഹഫിൽ ഇല്ലാത്ത രണ്ട് സൂറത്തുകൾ കൂടിയുണ്ടായിരിന്നുവെന്നും അത് പിൽക്കാലത്ത് വിസ്മരിക്കപ്പെടുകയാണ് ചെയ്തതെന്നും ആരോപണമുണ്ടല്ലോ. എന്താണിതിന്റെ വസ്തുത?

ഉബയ്യു ബ്നു കഅ്ബി(റ)ന്റെ മുസ്ഹഫിൽ ഇന്നത്തെ മുസ്ഹഫിൽ ഇല്ലാത്ത സൂറത്തുൽ ഹഫദ്, സൂറത്തുൽ ഖലാഅ എന്നീ സൂറത്തുകൾ കൂടി ഉണ്ടായിരുന്നതായി ഹമ്മാദ് ബ്നു സലാമയിൽ നിന്നുള്ള ഒരു നിവേദനം ഇമാം സുയൂഥ്വി തന്റെ ഇത്ഖാനിലും (2/ 66) ഇബ്നു ദുറൈസ് തന്റെ ഫദാഇലൽ ഖുർആനിലും (പുറം 157) ഉദ്ധരിച്ചിട്ടുണ്ട്. നിവേദനം ചെയ്ത ഹമ്മാദ് ബ്നു സലാമയും ഉബയ്യു ബ്നു കഅ്ബും തമ്മിൽ മൂന്ന് തലമുറകളുടെ വ്യത്യാസമെങ്കിലുമുള്ളതു കൊണ്ട് തന്നെ നിദാനശാസ്ത്രപ്രകാരം ഇത് തീരെ ദുർബലമാണ്.

എന്നാൽ ഇങ്ങനെ രണ്ട് സൂറത്തുകൾ ഉണ്ടായിരിക്കുന്നതായി സൂചിപ്പിക്കുന്ന വിശ്വാസയോഗ്യമായ മറ്റു ചില നിവേദനങ്ങളുണ്ട്. ഉമർ (റ)

اللهم إنا نستعينك ونستغفرك ونثني عليك الخير كله ونشكرك ولا نكفرك ونخلع ونترك من يفجرك എന്ന പ്രാർത്ഥനയും

اللهم إياك نعبد ولك نصلي ونسجد وإليك نسعى ونحفد نرجو رحمتك ونخشى عذابك إن عذابك بالكفار ملحق എന്ന പ്രാർത്ഥനയും ഖുനൂത്തായി പാരായണം ചെയ്യാറുണ്ടായിരുന്നുവെന്ന ഇബ്നു അബീശൈബ അദ്ദേഹത്തിന്റെ മുസന്നഫിലും(2/315) അബ്ദുർറസാഖ് അദ്ദേഹത്തിന്റെ മുസന്നഫിലും ഉദ്ധരിച്ചിട്ടുള്ള (4969) ഉബൈദ് ബ്നു ഉമൈറിൽ നിന്നുള്ള നിവേദനം സ്വഹീഹായ പരമ്പരയോട് കൂടിയുള്ളതാണ്. ഇതിൽ അദുർറസാഖിന്റെ മുസന്നഫിലുള്ള നിവേദനത്തിൽ നിവേദകനായ ഉബൈദ് ബ്നു ഉമൈർ ഇത് ഇബ്നു മസ്ഊദിന്റെ മുസ്ഹഫിലുള്ള രണ്ട് സൂറത്തുകളാണെന്ന് താൻ കേട്ടുവെന്നു കൂടി പറയുന്നുണ്ട്. ഇതാണ് ഇങ്ങനെ രണ്ട് സൂറത്തുകൾ ഉണ്ടായിരുന്നുവെന്നതിനുള്ള തെളിവ്.

മറ്റു ചില നിവേദനങ്ങളും ഇവ്വിഷയകമായി ഉദ്ധരിക്കപ്പെട്ടിട്ടുണ്ട്.

താഊസ് (റ) നിവേദനം ചെയ്യുന്നു: ”ഞാന്‍ ഉമറിന്റെ (റ) പിന്നില്‍ സുബ്ഹ് നമസ്‌കരിച്ചു. റുകൂഇനുശേഷം ഈ രണ്ടു സൂറത്തുകള്‍ കൊണ്ട് ഖുനൂത്ത് ഓതി.” (ത്വബ്‌രി 1/353). സമാനമായ ഉദ്ധരണികള്‍ അബ്ദു റസാഖ് മുസന്നഫ് 3/114, ത്വബ്‌രി -തഹ്ദീബുല്‍ ആസാര്‍ 1/319 എന്നിവയിലും കാണാം.

ഇക്കാര്യത്തിൽ ഉണ്ടാകാനിടയുള്ള സംശയങ്ങൾക്കെല്ലാം ഹദീഥ് പണ്ഡിതന്മാർ മറുപടി പറഞ്ഞിട്ടുണ്ട്. അബുൽ ഹുസ്സൈൻ അഹ്‌മദ്‌ ബിൻ ജഅഫർ അൽ മനാദി തന്റെ ‘അന്നാസിഖ് വൽ മൻസൂഖ്’ എന്ന ഗ്രൻഥത്തിൽ എഴുതുന്നു: “ഖുർആനിൽ നിന്ന് ദുർബലപ്പെടുത്തപ്പെട്ടതിനു ശേഷവും ജനമനസ്സുകളിൽ നിന്ന് മാഞ്ഞുപോകാതിരുന്ന വാക്യങ്ങൾക്ക് ഉദാഹരണമാണ് വിത്ർ നമസ്കാരത്തിലെ ഖുനൂത്തിൽ പാരായണം ചെയ്തുകൊണ്ടിരുന്ന ഈ രണ്ട് സൂറത്തുകൾ. ഉബയ്യു ബ്നു കഅ്ബിന്റെ പേരിൽ അറിയപ്പെട്ടിരുന്ന മുസ്ഹഫിൽ ഈരണ്ട് അധ്യായങ്ങളുമുണ്ടായിരുന്നുവെന്ന കാര്യത്തിലും അത് പ്രവാചകനിൽ നിന്ന് നിവേദനം ചെയ്യപ്പെട്ടതും അദ്ദേഹം പാരായണം ചെയ്തതുമാണെന്നും അവയെ സൂറത്തുൽ ഹഫദ്, സൂറത്തുൽ ഖലാഅ എന്നിങ്ങനെയാണ് വിളിക്കപ്പെട്ടിരുന്നത് എന്ന കാര്യത്തിലും ആദ്യകാല പണ്ഡിതന്മാർക്കിടയിൽ യാതൊരുവിധ അഭിപ്രായവ്യത്യാസങ്ങളുമുണ്ടായിരുന്നില്ല” (ബദറുദ്ദീനു സ്സർക്കശി ഉദ്ധരിച്ചത്: അൽ ബുർഹാൻ ഫീ ഉലൂമിൽ ഖുർആൻ, വാല്യം രണ്ട്, പുറം 37)

ഇമാം സുയൂഥ്വിയും തന്റെ ഇത്ഖാനിൽ ഹുസ്സൈൻ അഹ്‌മദ്‌ അൽ മനാദിയെ ഉദ്ധരിച്ച് ഇതേ കാര്യം തന്നെ സമർത്ഥിക്കുന്നുണ്ട്. (ഭാഗം രണ്ട്, പുറം 68)

അവതരിക്കപ്പെട്ടതിനു ശേഷം ദുർബലപ്പെടുത്തപ്പെട്ട സൂറത്തുകളാണ് സൂറത്തുൽ ഹഫദ്, സൂറത്തുൽ ഖലാഅ എന്നീ സൂറത്തുകൾ എന്നും മൻസൂഖായെങ്കിലും പ്രാർത്ഥനയായതിനാൽ അവ സ്വാഹാബിമാർ നമസ്കാരത്തിലും മറ്റും ഉപയോഗിക്കാറുണ്ടായിരുന്നുവെന്നും അതുകൊണ്ടാണ് ഉബയ്യിന്റെ മുസ്ഹഫിൽ അവ നില നിന്നത് എന്നുമാണ് ഇതിൽ നിന്ന് മനസ്സിലാവുന്നത്. ഉബയ്യു ബ്നു കഅ്ബി(റ)ന്റെ മുസ്ഹഫില്‍ നേരത്തെ ഉണ്ടായിരുന്ന അവയെ, ദുര്‍ബലപ്പെട്ടശേഷവും ദുആ ആയതിനാല്‍ അദ്ദേഹം അത് നിലനിർത്തി. എന്നാല്‍ ഉബയ്യി(റ)ന്റെ ഖിറാഅത്ത് ഉദ്ധരിച്ച ഇമാം നാഫിഅ്, ഇബ്‌നു കഥീർ, അബൂ അംറ് (റ) മുതലായവരൊന്നും ഖുര്‍ആനായി അത് ഉദ്ധരിച്ചിട്ടില്ല. എന്നാൽ അവ പ്രാർത്ഥനയായി നില നിർത്തിയതിനാൽ ഉമർ (റ) നമസ്കാരത്തിൽ ഖുനൂത്തായി അവയുടെ പാരായണം തുടരുകയും ചെയ്തു.

പ്രവാചകൻ (സ) പഠിപ്പിച്ചതല്ലാത്ത യാതൊന്നും അവരൊന്നും ഖുർആനിൽ ഉൾപ്പെടുത്തിയിട്ടില്ല; പഠിപ്പിച്ചതൊന്നും സ്വന്തം താൽപര്യപ്രകാരം എടുത്ത് മാറ്റിയിട്ടുമില്ല. ദൈവികബോധനത്തിന്റെ വെളിച്ചത്തിൽ പ്രവാചകൻ (സ) ദുർബലപ്പെടുത്തിയ വചനങ്ങളിൽ ചിലവ അവരുടെ മുസ്ഹഫിൽ അവർ എഴുതിവെക്കുകയും അവ പ്രാർത്ഥനകളായി ഉപയോഗിക്കുകയും ചെയ്തുവെന്ന് മാത്രമേയുള്ളൂ. അവ പ്രാർത്ഥനകളായി ഇന്നും മുസ്‌ലിംകൾ ഉപയോഗിക്കുന്നുണ്ട്.

പ്രമുഖ സ്വഹാബിയായിരുന്ന ഇബ്നു മസ്ഊദിന്റെ മുസ്ഹഫിൽ സൂറത്തുകളുടെ എണ്ണം പോലും വ്യത്യസ്തമായിരുന്നുവെന്നത് സ്വഹാബിമാർക്ക് പോലും ഖുർആനിന്റെ വിഷയത്തിൽ ഏകാഭിപ്രായമുണ്ടായിരുന്നില്ലെന്നല്ലേ മനസ്സിലാക്കിത്തരുന്നത്. അദ്ദേഹം സൂറത്തുൽ ഫാതിഹയും സൂറത്തുൽ ഫലഖും സൂറത്തുന്നാസും തന്റെ മുസ്ഹഫിൽ നിന്ന് നീക്കം ചെയ്തതായി ചില നിവേദനങ്ങളുണ്ടല്ലോ. ഇതിന്റെ യാഥാർഥ്യമെന്താണ്?

ഇബ്നു മസ് ഊദിന്റെ(റ) മുസ്ഹഫിൽ ഇന്നുള്ള മുസ്ഹഫിലുള്ള ചില സൂറത്തുകൾ ഉണ്ടായിരുന്നില്ലെന്നുള്ള നിവേദനങ്ങളുണ്ട്. സൂറഃ ഫാത്വിഹ, ഫലഖ്, നാസ് എന്നിവ അദ്ദേഹത്തിന്റെ മുസ്ഹഫിൽ ഉണ്ടായിരുന്നില്ലെന്നാണ് ചില അഥ്റുകളിലുള്ളത്. ഇമാം സുയൂഥ്വി തന്റെ ഇത്ഖാൻ ഫീ ഉലൂമിൽ ഖുർആനിൽ ഈ നിവേദനങ്ങൾ ഉദ്ധരിച്ചിട്ടുണ്ട്. ഇബ്നു മസ്ഊദിന്റെ മുസ്ഹഫിൽ ഫാത്തിഹയും ഫലഖ്, നാസ് സൂറത്തുകളും ഇല്ലായിരുന്നുവെന്നതാണ് ഒന്നാമത്തെ നിവേദനം. ഇവയിൽ ഫാത്തിഹ ഇല്ലായിരുന്നുവെന്നതും ഫലഖ്, നാസ് എന്നീ സൂറത്തുകൾ ഇല്ലായിരുന്നുവെന്നതും രണ്ട് വിഷയങ്ങളാണ്. ഓരോന്നിനെയും വേറെ വേറെ പരിശോധിക്കാം:

ഒന്ന്) ഫാത്തിഹ ഇല്ലായിരുന്നുവെന്ന വിമർശനം:

ഫാതിഹ ഖുർആനിന്റെ ഭാഗമാണെന്ന വസ്തുത ഖുർആൻ തന്നെ സാക്ഷ്യപ്പെടുത്തുന്നതാണ്. “ആവര്‍ത്തിച്ചു പാരായണം ചെയ്യപ്പെടുന്ന ഏഴ്‌ വചനങ്ങളും മഹത്തായ ഖുര്‍ആനും തീര്‍ച്ചയായും നിനക്ക്‌ നാം നല്‍കിയിട്ടുണ്ട്‌” (15: 87) എന്ന ഖുർആൻ വചനത്തിലെ “ആവർത്തിച്ച് പാരയണം ചെയ്യപ്പെടുന്ന ഏഴു വചനങ്ങൾ കൊണ്ടുള്ള വിവക്ഷ ഫാത്തിഹത്തുൽ കിതാബാണെന്ന്”(عن ابن مسعود في قوله: {ولقد آتيناك سبعا من المثاني} قال: فاتحة الكتاب) ഇബ്നു മസ്ഊദ് (റ) പറഞ്ഞതായി ഇമാം ത്വബ്‌രി തന്റെ തഫ്സീറിൽ ഉദ്ധരിക്കുന്നുണ്ട്. ഫാത്തിഹത്തുൽ കിതാബെന്നാൽ ‘ഗ്രന്ഥത്തിന്റെ ആമുഖം’ എന്നാണർത്ഥം. ഏതൊരു അധ്യായത്തെയാണോ ഗ്രന്ഥത്തിന്റെ ആമുഖം എന്ന് ഇബ് മസ്ഊദ് (റ) വിശേഷിപ്പിച്ചത് ആ അദ്ധ്യായം ഖുർആനിലുള്ളതല്ലെന്ന് അദ്ദേഹം കരുതിയെന്നു പറയുന്നത് അടിസ്ഥാനരഹിതമാണ്.

ഫാത്തിഹ ഖുർആനിലെ പ്രാരംഭാദ്ധ്യായമായി അംഗീകരിച്ചിരുന്ന ഇബ്നു മസ്ഊദ് (റ) പിന്നെയെന്തുകൊണ്ടാണ് തന്റെ കൈവശമുള്ള ഖുർആൻ കയ്യെഴുത്തുരേഖയിൽ അത് എഴുതാതിരുന്നത് എന്നതിന് അദ്ദേഹം തന്നെ മറുപടി പറഞ്ഞിട്ടുണ്ട്. അത് ഇങ്ങനെയാണ്: “അബൂബക്കർ അൽ അൻബരിയിൽ (റ) നിന്ന് ഇമാം ഖുർതുബി (റ) ഉദ്ധരിക്കുന്നു: എന്തുകൊണ്ടാണ് താങ്കളുടെ ഖുർആനിൽ ഫാത്തിഹ എഴുതാത്തത് എന്ന് ചോദിച്ചപ്പോൾ അബ്ദുല്ലാഹിബ്നു മസ്ഊദിന്റെ (റ) മറുപടി ഇങ്ങനെയായിരുന്നു. “ഞാൻ അഥവാ അത് എഴുതുകയായിരുന്നുവെങ്കിൽ എല്ലാ സൂറത്തുകളുടെയും തുടക്കത്തിൽ അത് എഴുതുമായിരുന്നു.” നമസ്കാരത്തിലെ ഓരോ റക്അത്തിലും സൂറത്തുകൾ പാരായണം ചെയ്യുന്നതിന് മുൻപ് ഫാത്തിഹ ഓതുന്നതുകൊണ്ടാണ് അദ്ദേഹം അങ്ങനെ പറഞ്ഞത് എന്ന് അബൂബക്കർ അൽ അൻബരി (റ) വിശദീകരിക്കുന്നുണ്ട്. ഇബ്ൻ മസ്ഊദ് (റ) തന്നെ ഇങ്ങനെ പറഞ്ഞതായി നിവേദനങ്ങളുണ്ട്. “ഹൃസ്വമായി എഴുതുന്നതിനു വേണ്ടിയാണ് ഞാൻ അത് ഉപേക്ഷിച്ചത്. മുസ്‌ലിംകൾ അത് സംരക്ഷിക്കുമെന്ന് ഞാൻ വിശ്വസിക്കുകയും ചെയ്തു” (ഇമാം ഖുർത്തുബി: അൽ ജാമിഉൽ അഹകാമിൽ ഖുർആൻ, വാല്യം 1, പുറം 115, കൈറോ, 1964)

രണ്ട്) ഫലഖ്, നാസ് സൂറത്തുകളെക്കുറിച്ച വിമർശനം

ഇവ്വിഷയകമായ നിവേദനങ്ങൾ ഇങ്ങനെയാണ്:

ആസിം (റ) സിർറിൽ (റ) നിന്ന് നിവേദനം ചെയ്യുന്നു: അദ്ദേഹം ഉബയ്യിനോട് (റ) പറഞ്ഞു: ഇബ്നു മസ്ഊദ് (റ) അദ്ദേഹത്തിന്റെ മുസ്ഹഫിൽ മുഅവ്വദതൈൻ (ഫലഖ്, നാസ് സൂറത്തുകൾ) രേഖപ്പെടുത്തിയിട്ടില്ല” (മുസ്നദ് അഹ് മദ്, ഹദീഥ് 21186)

അൽ അഅ്മഷ് അബൂ ഇസ്ഹാഖിൽ നിന്നും അദ്ദേഹം അബ്ദുർ റഹ്‌മാനു ബ്നു യസീദിൽ(റ) നിന്നും നിവേദനം ചെയ്യുന്നു: ഇബ്നു മസ്ഊദ് (റ) അദ്ദേഹത്തിന്റെ മസാഹിഫിൽ നിന്ന് മുഅവ്വദതൈൻ മായ്ച്ചു കളയുകയും അവ ഖുർആനിന്റെ ഭാഗമല്ലെന്ന് പറയുകയും ചെയ്തു.” (മുസ്നദ് അഹ് മദ്, ഹദീഥ് 21226)

ഇബ്നു ഉയയ്ന അബ്ദയിൽ(റ) നിന്നും ആസിമിൽ നിന്നും(റ) അവർ സിർറിൽ(റ) നിന്നും നിവേദനം ചെയ്യുന്നു: ഞാൻ ഉബയ്യിനോട് ചോദിച്ചു: “താങ്കളുടെ സഹോദരൻ അവയെ (ഫലഖ്, നാസ് സൂറത്തുകളെ) അദ്ദേഹത്തിന്റെ മുസ്ഹഫിൽ നിന്ന് മായ്ച്ച് കളഞ്ഞിട്ടുണ്ടല്ലോ” അപ്പോൾ അദ്ദേഹം അത് എതിർത്തില്ല. ഇത് ഇബ്നു മസ്ഊദിനെക്കുറിച്ചാണോയെന്ന ചോദ്യത്തിന് ഇബ്നു ഉയയ്ന അതേയെന്ന മറുപടിയാണ് നൽകിയത്. (മുസ്നദ് അഹ്‌മദ്‌, ഹദീഥ് 21189)

എന്തുകൊണ്ടാണ് ഇബ്നു മസ്ഊദ് (റ) തന്റെ മുസ്ഹഫിൽ ഖുർആനിലെ അവസാനത്തെ രണ്ട് അധ്യായങ്ങൾ ചേർക്കാതിരുന്നത്? അവ അല്ലാഹു അവതരിപ്പിച്ചതല്ല എന്ന് അദ്ദേഹത്തിന് അഭിപ്രായം ഉണ്ടായിരുന്നുവോ? അവ ഖുർആനിൽ പെട്ടതല്ല എന്നായിരുന്നുവോ അദ്ദേഹത്തിന്റെ അഭിപ്രായം? താഴെ പറയുന്ന വസ്തുതകൾ ശ്രദ്ധിക്കുക:

ഒന്ന്) ഇബ്നു മസ്ഊദിൽ നിന്ന് ഈ നിവേദനങ്ങൾ ഉദ്ധരിച്ച ആസ്വിമിൽ നിന്ന് സിർറിലൂടെ തന്നെ നിവേദനം ചെയ്യപ്പെട്ട മുതവാത്തിറായ ഖിറാഅത്തുകളിലെല്ലാം ഫലഖ്, നാസ് സൂറത്തുകളുണ്ട്. അസ്സിർറിനെ കൂടാതെ ഇബ്നു മസ്ഊദിൽ നിന്ന് ഖുർആൻ പാരായണം നിവേദനം ചെയ്ത അൽഖമ, അൽ അസ്‌വദ്, മസ്‌റൂഖ്‌, അസ്സുലമി, അബൂവാഇൽ, അസ്ശൈബാനി, അൽഹമദാനി എന്നിവരും നൂറ്റിപ്പതിനാല് സൂറത്തുകളും പൂർണമായി നിവേദനം ചെയ്തിട്ടുണ്ട്. ആസിം, ഹംസ, അൽ കിസായ്, അൽ ഖലഫ് എന്നീ നാല് പേരുടെ പേരിലും അറിയപ്പെടുന്ന പാരായണങ്ങൾ ഇബ്നു മസ്ഊദിൽ നിന്ന് നിവേദനം ചെയ്യപ്പെട്ടവയാണ്. ഇവയിലെല്ലാം ഇന്നുള്ള ഖുർആനിലെ മുഴുവൻ അധ്യായങ്ങളുമുണ്ട്. ഇതിനർത്ഥം ഇബ്നു മസ്‌ഊദ്‌ (റ) തന്റെ ശിഷ്യന്മാർക്ക് ഖുർആനിലെ നൂറ്റിപതിനാല് സൂറത്തുകളും ഖുർആനിന്റെ ഭാഗമായിത്തന്നെ പഠിപ്പിച്ചുവെന്നാണ്. അവസാനത്തെ രണ്ട് അധ്യായങ്ങളെക്കുറിച്ച് അദ്ദേഹത്തിന് വല്ല സംശയങ്ങളുമുണ്ടായിരുന്നുവെങ്കിൽ അദ്ദേഹത്തിന്റെ ശിഷ്യന്മാർക്ക് അവ ഖുർആനിന്റെ ഭാഗമായി അദ്ദേഹം പഠിപ്പിക്കുകയില്ലായിരുന്നു.

രണ്ട്) സൂറത്തുൽ ഫലഖിനെയും സൂറത്തുന്നാസിനെയും ഇബ്നു മസ്ഊദ് (റ) ഖുർആനിന്റെ ഭാഗമായിത്തന്നെയായിരുന്നു മനസ്സിലാക്കിയിരുന്നത് എന്ന തെളിയിക്കുന്ന വേറെയും നിവേദനങ്ങളുണ്ട്. ദൈലമിയിൽ നിന്ന് അലി അൽമുത്തഖി ഉദ്ധരിക്കുന്ന നിവേദനം ഉദാഹരണം: അത് ഇങ്ങനെയാണ്: ” ഇബ്നു മസ്ഊദ് (റ) പറഞ്ഞു: രണ്ട് സൂറത്തുകൾ നിങ്ങൾ ധാരാളമായി പാരായണം ചെയ്യുക. ഇഹലോകത്തെയും പരലോകത്തെയും ഉയർന്ന സ്ഥാനങ്ങളിൽ അത് വഴി അല്ലാഹു നിങ്ങളെ എത്തിക്കും. മുഅവ്വദത്തൈൻ ആണവ” (അലി അൽ മുത്തഖി: കൻസുൽ ഉമ്മാൽ, ബെയ്റുത്ത്, 1981, ഹദീഥ് 2743)

മൂന്ന്) ഇബ്നു മസ്ഊദ് (റ) ഖുർആനിലെ അവസാനത്തെ രണ്ട് സൂറത്തുകൾ തന്റെ മുസ്ഹഫിൽ നിന്ന് മായ്ച്ചു കളഞ്ഞുവെന്നും അത് കണ്ടിട്ടും ഉബയ്യ് (റ) അതിനെ എതിർത്തില്ലെന്നും വ്യക്തമാക്കുന്ന ഹദീഥ് നൽകുന്ന വിവരം വളരെ പ്രസക്തമാണ്. അവ രണ്ടും ഖുർആനിന്റെ ഭാഗമാണെന്ന് തന്നെയായിരുന്നു ഉബയ്യ് (റ) അടക്കമുള്ള സ്വഹാബിമാരുടെ മുഴുവൻ അഭിപ്രായമെന്ന് ഈ നിവേദനത്തിൽ നിന്ന് തന്നെ വ്യക്തമാണ്. ഖുർആനിൽ നിന്ന് രണ്ട് സൂറത്തുകൾ നിഷേധിക്കുകയെന്നാൽ ഇസ്‌ലാമിൽ നിന്ന് പുറത്തുപോകുന്ന കൊടിയ പാപമാണെന്ന കാര്യത്തിൽ സംശയമില്ല. “ആരെങ്കിലും ഖുർആനിലെ ഒരു അക്ഷരമെങ്കിലും നിഷേധിച്ചാൽ അയാൾ ഖുർആൻ മുഴുവൻ നിഷേധിച്ചവനെപ്പോലെയാണ്” എന്ന് ഇബ്നു മസ്ഊദ് (റ) പറഞ്ഞതായി ഇമാം അബ്ദുർറസാഖ് തന്റെ ‘മുസന്നഫി’ൽ (ഹദീഥ് നമ്പർ 15946) നിവേദനം ചെയ്തത് ശ്രദ്ധേയമാണ്. തന്റെ മുസ്ഹഫിൽ നിന്ന് ഇബ്നു മസ്ഊദ് (റ) ഈ രണ്ട് സൂറത്തുകൾ മായ്ച്ചു കളയുക വഴി ഉദ്ദേശിച്ചത് അവയെ നിഷേധിക്കുകയായിരുന്നുവെങ്കിൽ അത് വലിയ പ്രത്യാഘാതങ്ങളുണ്ടാക്കുമായിരുന്നു. അങ്ങനെയൊന്നും ഉണ്ടായിട്ടില്ലെന്നത് തന്നെ അദ്ദേഹം അവയെ നിഷേധിക്കുകയല്ല, എഴുതിവെക്കുന്നത് ശരിയല്ലെന്ന് കരുതുകയാണ് ചെയ്തതെന്ന് വ്യക്തമാണ്.

ഷെയ്ഖ് അബൂ ബക്കർ ബാക്വിലാനി പറഞ്ഞത് ശ്രദ്ധേയമാണ്: “ഈ രണ്ട് സൂറത്തുകൾ ഖുർആനിലുള്ളതല്ലെന്ന വാദം അദ്ദേഹത്തിനുള്ളതായി സ്ഥാപിക്കപ്പെട്ടിട്ടില്ല. അവ അദ്ദേഹം മായ്ച്ചു കളയുകയും തന്റെ മുസ്ഹഫിൽ ഉൾപ്പെടുത്താതിരിക്കുകയും ചെയ്തത് അവ ഖുർആനിന്റെ ഭാഗമാണെന്ന വസ്തുത അദ്ദേഹം നിഷേധിക്കുന്നത് കൊണ്ടായിരുന്നില്ല. പ്രവാചകൻ (സ) എഴുതാനായി പറഞ്ഞതല്ലാതെ യാതൊന്നും തന്നെ മുസ്ഹഫിൽ എഴുതാൻ പാടില്ലെന്നായിരുന്നു അദ്ദേഹത്തിന്റെ വീക്ഷണം. പ്രവാചകൻ (സ) അവ എഴുതിപ്പിച്ചതായോ എഴുതിവെക്കാൻ ആവശ്യപ്പെട്ടതായോ അദ്ദേഹം കണ്ടിട്ടുണ്ടായിരുന്നില്ല. (ഇമാം സുയൂഥ്വി: അൽ ഇത്ഖാൻ 1/ 271)

നാല്) മുഅവ്വദതൈൻ ഖുർആനിലുള്ളതാണെന്ന് വ്യക്തമാക്കുന്ന നബി(സ)യിൽ നിന്ന് സ്ഥിരപ്പെട്ട നിവേദനങ്ങളുണ്ട്.

അബുല്‍ അളാഅ് (റ) പറയുന്നു. നബി (സ) ഒരു സ്വഹാബിക്ക് സൂറഃ ഫലഖും നാസും പഠിപ്പിച്ചു കൊടുത്തു. തങ്ങള്‍ പറഞ്ഞു: നീ ഈ സൂറത്തുകള്‍ ഓതി നമസ്‌കരിക്കുക. (മുസ്നദ് അഹ്‌മദ്- ഹദീഥ് സ്വഹീഹാണെന്ന് ഇമാം ഇബ്‌നു ഹജര്‍ ഫതഹുല്‍ ബാരി 8/615ലും ഇമാം ഖാരി ഉംദ 2/16ലും പറയുന്നു)

ഉഖ്ബ(റ)യിൽ നിന്ന് നിവേദനം: നബി (സ) പറഞ്ഞു: ഈ രാത്രിയില്‍ എനിക്ക് ചില ആയത്തുകള്‍ അവതീര്‍ണമായി. സമാനമായവ തീരെ കാണപ്പെട്ടിട്ടില്ല. അത് സൂറത്തുൽ ഫലഖും നാസുമാണ്. (സ്വഹീഹ് മുസ്‌ലിം കിതാബ് സ്വലാത്ത്, ബാബുൽ ഫദാഇലി മുഅവ്വദതൈൻ). ഇതേപോലെയുള്ള നിരവധി ഹദീഥുകള്‍ ഇമാം ഇബ്‌നു കഥീർ (റ) തന്റെ തഫ്‌സീറില്‍ ഉദ്ധരിക്കുന്നുണ്ട്. അതുകൊണ്ട് തന്നെ അവ ഖുർആനിലുള്ളതല്ലെന്ന് പ്രവാചകശിഷ്യന്മാരിൽ പ്രമുഖനായ ഇബ്നു മസ്ഊദ് (റ) കരുതുമെന്ന് വിചാരിക്കാൻ യാതൊരു ന്യായവുമില്ല.

അഞ്ച്) ഇബ്നു മസ്ഊദിന്റെ(റ) ശിഷ്യന്മാരെല്ലാം ഈ രണ്ട് സൂറത്തുകളും ഖുർആനിൽ പെട്ടത് തന്നെയാണെന്ന് സാക്ഷ്യം വഹിച്ചതിനുള്ള തെളിവാണ് അവരിലൂടെ നിവേദനം ചെയ്യപ്പെട്ട ഖിറാഅത്തുകളിലൊന്നും അവ വിട്ടുകളഞ്ഞിട്ടില്ല എന്നത്. ഈ സൂറത്തുകൾ ഖുർആനിൽ പെട്ടതല്ലെന്ന് ഗുരു പറഞ്ഞിരുന്നെങ്കിൽ ശിഷ്യന്മാർ ആരെങ്കിലും ആ രൂപത്തിൽ ഖുർആൻ പാരായണം പഠിക്കുമായിരുന്നു. അതുണ്ടായിട്ടില്ലെന്ന് മാത്രമല്ല, ഇബ്‌നു മസ്ഊദിന്റെ(റ) ശിഷ്യന്‍മാരില്‍ പ്രമുഖനായ അസ്‌വദ് ബിന്‍ യസീദി(റ)നോട് ആ രണ്ടു സൂറത്തുകള്‍ ഖുര്‍ആനില്‍ പെട്ടതാണോ എന്നു ചോദിച്ചപ്പോള്‍ “അതെ, അവ രണ്ടും ഖുര്‍ആനില്‍പെട്ടതു തന്നെയാണ്” എന്ന് മറുപടി പറഞ്ഞതായി ഇബ്നു അബീശൈബ ഉദ്ധരിക്കുന്നുമുണ്ട്. (ഇബ്‌നു അബീ ശൈബ 30206)

ആറ്) അബ്ദുർ റഹ്‌മാനു ബ്നു യസീദിൽ (റ) നിന്ന് ഇമാം അഹ്‌മദ്‌ തന്റെ മുസ്നദിൽ നിവേദനം ചെയ്ത അഥറിൽ (ഹദീഥ് 21226) ഈ സൂറത്തുകൾ ഖുർആനിലുള്ളതല്ലെന്ന് ഇബ്നു മസ്ഊദ് (റ) പറഞ്ഞതായി ഉണ്ടെന്നത് ശരിയാണ്. ഇത് ഇമാം ത്വബ്റാനിയും തന്റെ മുജമ്മഉൽ കബീറിൽ നിവേദനം ചെയ്യുന്നുണ്ട്. (മുജമ്മഉൽ കബീർ 9150). എന്നാൽ ഈ നിവേദനം സ്വീകാര്യമല്ലെന്ന് ഇമാം നവവിയടക്കമുള്ള നിരവധി പണ്ഡിതന്മാർ വ്യക്തമാക്കിയതായി ഇമാം സുയൂഥ്വി വിശദീകരിക്കുന്നുണ്ട്. (അൽ ഇത്ഖാൻ 1/ 271) നിരവധി എതിർ തെളിവുകളുള്ളതിനാൽ ഈ നിവേദനം മുഅല്ലലും മുതവാത്തിറായ നിവേദനങ്ങൾക്ക് വിരുദ്ധമായതിനാൽ ശാദ്ദുമാണെന്നാണ് പണ്ഡിതാഭിപ്രായം.

ഇബ്നു മസ്ഊദ് (റ) തന്റെ മുസ്ഹഫിൽ ഫാതിഹയും സൂറത്തുൽ ഫലഖും സൂറത്തുന്നാസും എഴുതാതിരുന്നത് അവ ഖുർആനിൽ ഉള്ളതല്ലെന്ന് അദ്ദേഹം കരുതിയതുകൊണ്ടല്ലെന്നും പ്രത്യുത അദ്ദേഹത്തിന്റെതായ വ്യക്തിപരമായ ചില കാരണങ്ങളാലാണെന്നും ഇവയിൽ നിന്ന് വ്യക്തമാണ്. ഖുർആനിൽ ഇന്നുള്ള ഏതെങ്കിലും സൂറത്തുകൾ ഖുർആനിന്റെ ഭാഗമല്ലെന്ന് കരുതിയിരുന്ന സ്വഹാബിമാരാരും ഉണ്ടായിരുന്നിട്ടില്ല. ഖുർആനിൽ കളങ്കമാരോപിക്കുന്നവർക്ക്, അതുകൊണ്ട് തന്നെ, ഇബ്നു മസ്‌ഊദിന്റെ(റ) നടപടിയെ എങ്ങനെ അപഗ്രഥിച്ചാലും തെളിവുകൾ ഒന്നും ലഭിക്കുകയില്ല; അവർ നിരാശപ്പെടുകയേയുള്ളൂ.

സ്വഹാബിമാരിൽ ചിലർ സ്വന്തം താൽപര്യപ്രകാരം ആയത്തുകളും സൂറത്തുകളും ക്രമീകരിച്ചുവെന്ന് സൂചിപ്പിക്കുന്ന ചില നിവേദനങ്ങളുണ്ടല്ലോ. അവയുടെ യാഥാർഥ്യമെന്താണ്?

സ്വഹാബിമാർ സ്വതാല്‍പര്യപ്രകാരം ആയത്തുകളും സൂറത്തുകളും ക്രമീകരിച്ചോ എന്ന ചോദ്യത്തിന് ഇല്ലെന്നാണ് മറുപടി. ഖുർആനിന്റെ സമാഹരണവും സംരക്ഷണവും സ്വന്തം ബാധ്യതയാണെന്ന അല്ലാഹുവിന്റെ വചനത്തിന്റെ അടിസ്ഥാനത്തിൽ ആയത്തുകളുടെയും സൂറത്തുകളുടെയും ക്രമം പോലും വഹ്‌യിന്റെ അടിസ്ഥാനത്തിൽ സ്ഥിരപ്പെട്ടതാണെന്നായിരുന്നു സ്വഹാബിമാർ മനസ്സിലാക്കിയിരുന്നത് എന്ന് വ്യക്തമാക്കുന്ന നിരവധി നിവേദനങ്ങളുണ്ട്. പ്രവാചകാനുചരന്മാരിൽ ചിലർ സ്വന്തം അഭിപ്രായപ്രകാരം ആയത്തുകളുടെ ക്രമം തീരുമാനിച്ചുവെന്ന് തോന്നിപ്പിക്കുന്ന ചില നിവേദനങ്ങളുണ്ടെന്നത് ശരിയാണ്. പക്ഷെ അവയെല്ലാം ദുർബലവും തെളിവിന് കൊള്ളാത്തതുമാണെന്ന് അവയെ അപഗ്രഥിച്ച് പഠിച്ച പണ്ഡിതന്മാർ വ്യക്തമാക്കിയിട്ടുണ്ട്.

ഒന്ന്: അബാദ് ബിന്‍ അബ്ദില്ല (റ) പറയുന്നു:- ഹാരിസ് ബിന്‍ ഖുസൈമ (റ) തൗബയിലെ അവസാന രണ്ടായത്തുകളുമായി ഉമറി(റ)ന് സമീപത്തെത്തി. അദ്ദേഹം ചോദിച്ചു, ആരാണ് സാക്ഷിയുള്ളത്? അദ്ദേഹം പറഞ്ഞു, അറിയില്ല. എന്നാല്‍ ഞാനിത് നബി(സ)യില്‍ നിന്നും കേട്ടതും മനഃപാഠമാക്കിയതുമാണ്. ഉമര്‍ (റ) പറഞ്ഞു: ഇത് മുൻ ആയതായിരുന്നുവെങ്കില്‍ ഇതിനെ സ്വതന്ത്രമായ അധ്യായമാക്കുമായിരുന്നു. നിങ്ങള്‍ ക്വുര്‍ആനിലെ ഒരു സൂറത്തില്‍ അത് ചേര്‍ക്കുക. ഞാനതിനെ ബറാഅത് -തൗബ സൂറയുടെ അവസാനം ചേര്‍ത്തു. (അഹ്‌മദ് 1715, ത്വബ്‌രി 60, ഇബ്‌നു അബീദാവൂദ് -അല്‍ മസാഹിഫ് )

തികച്ചും ബലഹീനമായ ഒരു നിവേദനമാണിത്. ഇമാം അഹ്‌മദിന്റെ മുസ്നദിനുള്ള നിരൂപണത്തിൽ ശൈഖ് അര്‍നാഊഥ് (റ) ഈ അഥര്‍ ബലഹീനമാണെന്ന് രേഖപ്പെടുത്തിയിട്ടുണ്ട്. നിവേദകനായ അബ്ബാദ് ഖുര്‍ആന്‍ ക്രോഡീകരണത്തിനു ദൃക്‌സാക്ഷിയല്ല. അദ്ദേഹത്തിന് ഈ വിവരം നല്‍കിയത് ആരാണെന്ന് അറിയുകയുമില്ല. ആയതിനാല്‍ അഥര്‍ ബലഹീനമായി ഗണിക്കപ്പെടുന്നു. ഇമാം ബുഖാരി (റ) ഇതുമായി ബന്ധപ്പെട്ട് ഉദ്ധരിച്ചതാണ് ശരിയെന്നും ഈ അഥറിലെ ആശയം പ്രബല നിവേദനങ്ങളോട് എതിരായതാണെന്നും ശൈഖ് അഹ്‌മദ് ശാഖിറും(റ) പറഞ്ഞിട്ടുണ്ട്. വിഷയബന്ധിതമായ സ്വീകാര്യമായ നിവേദനങ്ങള്‍ ദുർബലമായ ഈ അഥറിന് എതിരാണ് താനും.

ഇത് ഉദ്ധരിച്ച ഇബ്നു അബൂദാവൂദ് തന്നെ സ്വീകാര്യമായ ഇവ്വിഷയകമായ നിവേദനങ്ങളും ഉദ്ധരിച്ചിട്ടുണ്ട്. അവ ഇവയാണ്. “സൈദ് (റ) പറയുന്നു: നബി(സ)യില്‍ നിന്നും ഞാന്‍ കേട്ട ആയത്ത് (സൂറത്തുല്‍ തൗബയിലെ പ്രസ്തുത ആയത്തുകള്‍) എനിക്ക് ലഭിച്ചില്ല. ഞാന്‍ അത് അന്വേഷിച്ചു. അങ്ങനെ അതിനെ ഖുസൈമ ബിന്‍ സാബിതി(റ)ല്‍ നിന്നും എനിക്കത് ലഭിച്ചു. ഞാന്‍ അതിനെ അതിന്റെ സൂറത്തില്‍ തന്നെ ചേര്‍ത്തു” (ഇബ്‌നു അബീദാവൂദ് -മസാഹിഫില്‍ നമ്പര്‍ 24, ത്വയാലിസി, തിര്‍മിദി 4/346. നിവേദനം സ്വഹീഹാണെന്ന് ഇമാം തിര്‍മിദി (റ) വ്യക്തമാക്കി. ഇമാം ദാരിമി – മുഖന്നഇല്‍ 15-16)

അബുല്‍ ആലിയ (റ) പറയുന്നു: :- അവര്‍ അബൂബക്കറിന്റെ (റ) കാലഘട്ടത്തില്‍ ക്വുര്‍ആന്‍ ശേഖരിച്ചു. ഉബയ്യ് (റ) വായിച്ചു കൊടുക്കുകയും എഴുതുകയും ചെയ്തു. സൂറഃ തൗബയിലെ 127-ാം ആയത്ത് എത്തിയപ്പോള്‍ ഇത് അവസാനം ഇറങ്ങിയ ആയത്താണെന്നു വിചാരിച്ചു. ഉബയ്യ്‌ (റ) പറഞ്ഞു:- ഇതിനുശേഷം നബി (സ) രണ്ടു ആയത്തുകളെ ഓതുന്നതായി ഞാന്‍ കേട്ടിട്ടുണ്ട്. (ശേഷം ആ രണ്ട് ആയത്തുകള്‍ ഓതി). (അല്‍ മസാഹിഫ് -ഹദീഥ് നമ്പര്‍ 29 – ഹദീഥ് മുന്‍തിഖ ആണ്).

വിമര്‍ശനവിധേയമായ ഹദീഥ് ബലഹീനമാണെന്നും അതിനെതിരില്‍ അന്യൂനമായ നിവേദനങ്ങൾ ഉണ്ടെന്നും വ്യക്തമാക്കുന്നതാണ് ഈ തെളിവുകൾ. സ്വഹാബിമാർ സ്വന്തം ഇഷ്ടപ്രകാരം അവർക്കിഷ്ടമുള്ളിടത്ത് ലഭിച്ച ആയത്തുകൾ ചേർക്കുകയായിരുന്നില്ല, നബി(സ) നിർദേശം ഇവ്വിഷയകമായി പൂർണമായും അനുസരിക്കുകയായിരുന്നുവെന്ന് ഈ തെളിവുകൾ സുതരാം വ്യക്തമാക്കുന്നുണ്ട്.

രണ്ട്: ഇബ്‌നു അബ്ബാസ് (റ) പറയുന്നു. ഞാന്‍ ഉഥ്മാനോട്(റ) ചോദിച്ചു, നിങ്ങള്‍ സൂറത്ത് അന്‍ഫാല്‍, തൗബ എന്നിവക്കിടയില്‍ ബിസ്മി രേഖപ്പെടുത്താതെ ചേര്‍ത്ത് എഴുതിയതിന്റെ കാരണമെന്താണ്? ഉഥ്മാൻ (റ) പറഞ്ഞു: നബി(സ)ക്ക് നീണ്ട കാലയളവില്‍ ധാരാളം സൂറത്തുകള്‍ അവതരിച്ചിരുന്നു. ഖുര്‍ആന്‍ അവതരിക്കുമ്പോള്‍ എഴുത്തുകാരില്‍ ഒരാളെ വിളിച്ച് ഇതിനെ ഇന്ന വിഷയം പരാമര്‍ശിക്കുന്ന സൂറത്തില്‍ ചേര്‍ക്കുക എന്ന് കല്‍പിക്കുമായിരുന്നു. സൂറഃ അന്‍ഫാല്‍ മദീനയില്‍ ആദ്യം അവതരിച്ചതാണ്. സൂറഃ തൗബ അവസാനം അവതരിച്ചതും. ഇരു സൂറത്തുകളുടെയും പ്രമേയവിഷയം പരസ്പരം യോജിച്ചതാണ്. അതിന്റെ തുടര്‍ച്ചയാണ് ഇതെന്നു ഞാന്‍ വിചാരിക്കുന്നു. വ്യക്തത നല്‍കാതെയാണ് നബി (സ) ഇഹലോകം വെടിഞ്ഞത്. ആയതിനാല്‍ ഇടയില്‍ ബിസ്മി ചേര്‍ക്കാതെ ചേര്‍ത്ത് രണ്ടു സൂറത്തുകളും രേഖപ്പെടുത്തി. (അഹ്‌മദ്‌ 1/244, തിര്‍മിദി 3086, മിശ്കാത് 2163)

ഹദീഥ് പണ്ഡിതന്‍മാരായ ശൈഖ് ശുഐബ് അര്‍നഊത്, ശൈഖ് അല്‍ബാനി, ശൈഖ് അഹ് മദ് ശാഖിര്‍ (റ) എന്നിവര്‍ ഈ അഥർ ബലഹീനമാണെന്ന് പറഞ്ഞിട്ടുണ്ട്. അഥര്‍ ഉദ്ധരിച്ച യസീദുര്‍റുഖാശി (റ) എന്ന വ്യക്തിയില്‍ അവ്യക്തതയുണ്ടെന്ന് ഇമാം തിര്‍മിദി (റ) രേഖപ്പെടുത്തി. യസീദു ബിന്‍ അബാ(റ)നെ കണ്ടിട്ടില്ലെന്നും ഇമാം തന്നെ പറയുന്നുണ്ട്. (തിര്‍മിദി ഹദീഥ് നമ്പര്‍ 3086)

അന്യൂനമായ ഹദീഥല്ല ഇതെന്നു സാരം, ഇതിന്റെ ആശയം സ്ഥിരപ്പെട്ട കാര്യങ്ങള്‍ക്ക് വിരുദ്ധമാണ്.

1) നബി(സ)യുടെയും അബൂബക്കറി(റ)ന്റെയും കാലഘട്ടത്തില്‍ നടന്ന രണ്ടു രീതിയിലുള്ള ക്രോഡീകരണത്തിലും ഈ രണ്ടു സൂറത്തുകളും രണ്ടായിട്ടാണ് രേഖപ്പെടുത്തിയത്. സൂറഃ തൗബയില്‍ ബിസ്മി ഇല്ല എന്ന വിഷയത്തില്‍ സ്വഹാബാക്കള്‍ ഭിന്നിച്ചിട്ടില്ല. ഇത് കേവലം ഉഥ്മാന്റെ(റ) ഗവേഷണ ഫലമായിരുന്നുവെങ്കില്‍ അവര്‍ ഭിന്നിക്കുമായിരുന്നു.

2) സൂറത്തുകളെല്ലാം തുടങ്ങേണ്ടത് ബിസ്മി കൊണ്ടാണെന്നത് നബി(സ)യുടെ കല്പനയാണ്. അതെ നബി (സ) തന്നെയാണ് അത് സൂറഃ തൗബയില്‍ രേഖപ്പെടുത്തേണ്ടതില്ലെന്ന് പഠിപ്പിച്ചത്. അതിനാലാണ് എഴുത്തുകാർ അത് രേഖപ്പെടുത്താതിരുന്നത്.

3) ബിസ്മി രേഖപ്പെടുത്താതിന്റെ കാരണം എന്താണെന്നതിലാണ് സ്വഹാബാക്കള്‍ വ്യത്യസ്ത അഭിപ്രായം പറഞ്ഞത്. ഈ വിഷയത്തില്‍ നബി(സ)യില്‍ നിന്നും വ്യക്തത ലഭിച്ചിട്ടില്ല എന്ന് ഉഥ്മാൻ (റ) അദ്ദേഹത്തിന് ലഭിച്ച അറിവിന്റെ അടിസ്ഥാനത്തിൽ അഭിപ്രായം പറഞ്ഞതാണ്. മറ്റു സ്വഹാബിമാരും ഇവ്വിഷയകമായ അവരുടെ അഭിപ്രായങ്ങൾ പറഞ്ഞിട്ടുണ്ട്.

ഇബ്‌നു അബ്ബാസ് (റ) പറയുന്നു:- അലി (റ) പറഞ്ഞു: ബിസ്മില്ലാഹി നിര്‍ഭയത്വമാണ്. സൂറഃ തൗബ യുദ്ധ കല്‍പന പ്രഖ്യാപിച്ച് ഇറങ്ങിയതാണ്. അതില്‍ നിര്‍ഭയത്വമില്ല. അതുകൊണ്ടാണ് അതിൽ ബിസ്മി ഒഴിവായത്. (തഫ്‌സീര്‍ ത്വബ്‌രി)

സൂറത്തു തൗബയുടെ ആദ്യ ഭാഗങ്ങള്‍ ദുര്‍ബലപ്പെടുത്തപ്പെട്ടതോടുകൂടി ഇതിലെ ബിസ്മിയും ഉയര്‍ത്തപ്പെട്ടതായി ഇമാം മാലിക്കും(റ) അഭിപ്രായപ്പെട്ടുണ്ട്.

ഖുർആനിൽ പതിനേഴായിരം ആയത്തുകളുള്ളതായി ചിലർ എഴുതിയിട്ടുണ്ടല്ലോ. ആറായിരത്തിലധികം ആയത്തുകളെ ഇന്നുള്ള ഖുർആനിലുള്ളൂ. ചില ആയത്തുകൾ നഷ്ടപ്പെട്ടിട്ടുണ്ടെന്നല്ലേ അതിനർത്ഥം?

ശിയാക്കളുടെ ചില കൃതികളിലാണ് ഇത്തരം പരാമർശങ്ങളുള്ളത്. ശിയാക്കളുടെ ഹദീഥ് ഗ്രന്ഥമായ അൽ കുലൈനിയുടെ ഉസൂലിൽ കാഫിയിൽ, അവരുടെ ഹദീഥ് നിദാനശാസ്ത്രപ്രകാരം അവർ സ്വീകാര്യമാണെന്ന് കരുതുന്ന ഒരു നിവേദനത്തിൽ പറയുന്നത് ഇങ്ങനെയാണ്: “അബൂ അബ്ദില്ല പറയുന്നു: മുഹമ്മദ് നബി(സ)ക്ക് ജിബ്‌രീല്‍ (അ) നല്‍കിയ ഖുര്‍ആനില്‍ പതിനേഴായിരം ആയത്തുകള്‍ ഉണ്ടായിരുന്നു.” (ഉസൂലില്‍ കാഫി വാല്യം രണ്ട്, പുറം 634, ഹദീഥ് 28)

അലി(റ)യോടുള്ള കൃത്രിമ സ്‌നേഹം പ്രകടമാക്കി ഇസ്‌ലാമില്‍ നിന്നും ജനങ്ങളെ തിരിച്ചുവിടാനായി ജൂതനായ ഇബ്‌നു സബഅ് ജന്മം നല്‍കിയതാണ് ശീഇസം. പ്രമുഖരായ സ്വഹാബാക്കളെല്ലാം അവിശ്വാസികളാണെന്നും അവര്‍ ഖുര്‍ആന്‍ മാറ്റിമറിച്ചു എന്നും അവര്‍ വാദിച്ചു. സനദുകള്‍ ഇല്ലാതെയോ അല്ലെങ്കില്‍ സനദുകൾ കൃത്രിമമായി അവര്‍ നിര്‍മിച്ചോ പല അന്ധവിശ്വാസങ്ങളും അവർ പ്രചരിപ്പിച്ചു. ഗ്രന്ഥകര്‍ത്താവ് തന്നെ മഹാനുണയനായതിനാൽ അയാളുടെ കൃതികളുടെ സ്ഥാനം ചവറ്റുകൂനയാണ്. ജൂതനായ പൗലോസ് ക്രിസ്തുമതത്തെ നശിപ്പിച്ചതില്‍ പാഠമുള്‍ക്കൊണ്ട മുസ്‌ലിം പണ്ഡിതന്‍മാര്‍ ജൂതനായ ഇബ്‌നു സബഇനെ തിരിച്ചറിഞ്ഞുവെങ്കിലും വൈകിയിരുന്നു. അങ്ങനെയാണ് ശീഇസം എന്ന മതം ലോകത്ത് വലിയ സ്വാധീനമുണ്ടാക്കിയത്.

ഖുര്‍ആന്‍ തിരുത്തലുമായി ബന്ധപ്പെട്ട് അവര്‍ ആരോപിക്കുന്ന ദുരാരോപണങ്ങള്‍ ഇസ്‌ലാമിന്റെ ശത്രുക്കളുടെ ആരോപണങ്ങളായി മാത്രമാണ് കാണേണ്ടത്. ശരിയായ സനദുകളിലൂടെ അവ തെളിയിക്കാന്‍ സാധ്യമല്ല. ശിയാക്കളുടെ ഇത്തരം ആരോപണങ്ങൾക്ക് അഹ്ലുസുന്നയുടെ മഹാപണ്ഡിതന്മാർ മറുപടി പറഞ്ഞിട്ടുണ്ട്. അഞ്ചാം നൂറ്റാണ്ടിൽ ജീവിച്ച അന്തലൂസിയൻ പണ്ഡിതനായ ഇബ്ൻ ഹസം എഴുതി: “പരിശുദ്ധ ഖുർആനിൽ കൈകടത്തലുകൾ നടന്നിട്ടുണ്ടെന്ന റാഫിദികളുടെ വാദത്തെക്കുറിച്ച് പറയാനുള്ളത്, അവർ മുസ്‌ലിംകളല്ലെന്നാണ്. നിരവധി വിഭാഗങ്ങളുള്ള അവരിലെ ഒന്നാമത്തെ വിഭാഗം ഉടലെടുത്തത് പ്രവാചകവിയോഗത്തിന് ഇരുപത്തിയഞ്ച് വർഷങ്ങൾക്ക് ശേഷമാണ്. ജൂതന്മാരെയും ക്രിസ്ത്യാനികളെയും കള്ളപ്രചാരണങ്ങളുടെയും പാഷാണ്ഡതയുടെയും കാര്യത്തിൽ പിന്തുടർന്നുകൊണ്ട് ഇസ്‌ലാമിനെ തകർക്കാൻ ശ്രമിച്ചതിന്റെ ഫലമായാണ് അവർ രൂപമെടുത്തത്. അലി ബിൻ അബീതാലിബിനും അദ്ദേഹത്തോടൊപ്പമുണ്ടായിരുന്ന ചിലർക്കും ദിവ്യത്വം കൽപിക്കുന്നത് വരെ ചെന്നെത്തിയവരാണ് അവരിലെ അതിതീവ്രവിഭാഗങ്ങൾ. അവരിലെ വ്യതിയാനം കുറഞ്ഞ വിഭാഗത്തിന്റെ വിശ്വാസം അലിക്കുവേണ്ടി സൂര്യൻ രണ്ട് തവണ പിന്നിലേക്ക് നടന്നിട്ടുണ്ടെന്നാണ്. പച്ചക്കള്ളങ്ങളിൽ അഭിരമിക്കുന്ന ഇത്തരം വിഭാഗങ്ങൾ ഖുർആനിനെക്കുറിച്ച് കള്ളം പറയുന്നതിൽ അത്ഭുതപ്പെടാനെന്തുണ്ട്?” (ഇബ്നു ഹസമിൽ നിന്ന് ഇസ്‌റാഈൽ ഫ്രീഡ്ലാൻഡർ ഉദ്ധരിച്ചത്: Israel Friedlaender (1908). “The Heterodoxies of the Shiites in the Presentation of Ibn Hazm”, Journal of the American Oriental Society. American Oriental Society. 29: 61–2. Retrieved 11 April 2015.)

ശിയാക്കളുടെ അടിസ്ഥാന ഗ്രന്ഥങ്ങളിൽ ഖുർആനിൽ മാറ്റത്തിരുത്തലുകളുണ്ടായിട്ടുണ്ട് എന്ന ആരോപണം ഉന്നയിച്ചിട്ടുണ്ടെങ്കിലും, പല പ്രഗത്ഭരായ ഷിയാ പണ്ഡിതന്മാരും ഈ ആരോപണം ശരിയല്ലെന്ന് പറഞ്ഞിട്ടുണ്ട്. 1992 ൽ മരണപ്പെട്ട ഇറാഖിലെ ഷിയാപണ്ഡിതനായ ആയത്തുല്ലാ അബുൽ ഖാസിം അൽഖൊയി പറയുന്നത് ഇങ്ങനെയാണ്: “ഖുർആനിൽ യാതൊരു മാറ്റത്തിരുത്തലും വന്നിട്ടില്ലെന്നും മഹാപ്രവാചകന് (സ) അവതരിക്കപ്പെട്ട ഖുർആൻ അതേപോലെ പൂർണമായും യാതൊരു വ്യത്യാസവുമില്ലാതെയുമാണ് നമ്മുടെ കൈവശമുള്ളത് എന്നുമാണ് മുസ്‌ലിംകളെല്ലാം സ്വീകരിച്ചിരിക്കുന്ന അടിസ്ഥാന വീക്ഷണം. നിരവധി പ്രാമാണികരായ പണ്ഡിതന്മാർ ഇക്കാര്യം പ്രഖ്യാപിച്ചിട്ടുണ്ട്. ഇഥനാ അശ്അരികളിൽക്കിടയിലെ പ്രഗത്ഭ ഹദീഥ് പണ്ഡിതനായ മുഹമ്മദ് ബിൻ ബാബവയ്ഹ് ഖുർആനിൽ മാറ്റമൊന്നുമുണ്ടായിട്ടില്ലെന്നത് ഇഥനാ അശ്അരികളുടെ ഒരു അടിസ്ഥാനവിശ്വാസമാണെന്ന് എണ്ണിപ്പറഞ്ഞിട്ടുണ്ട്. ഇമാമീ ശിയാക്കളുടെ നിയമവിശാരദനായ അബൂ ജഅ്ഫർ മുഹമ്മദ് ബിൻ അൽഹസൻ അത്തൂസി, തന്റെ ഖുർആൻ വ്യാഖ്യാനഗ്രന്ഥമായ അൽതിബയാനിന്റെ തുടക്കത്തിൽ തന്നെ ഈ വീക്ഷണം വ്യക്തമാക്കുകയും തെളിവുകളുടെ വെളിച്ചത്തിൽ അത് സമർത്ഥിക്കുകയും തന്റെ ഗുരുവായ അശ്ശരീഫുൽ മുർതദക്ക് ഇതേ കാഴ്ചപ്പാടാണുണ്ടായിരുന്നതെന്ന് വ്യക്തമാക്കുകയും ചെയ്തിട്ടുണ്ട്. പ്രസിദ്ധ ഖുർആൻ വ്യാഖ്യാതാവായ അത്തബ്റാസിയും തന്റെ വ്യാഖ്യാനമായ മജ്മഉൽ ബൈദാനിന്റെ മുഖവുരയിൽ ഇക്കാര്യം തന്നെയാണ് സമർത്ഥിച്ചിരിക്കുന്നത്.” (Al-Sayyid Abu al-Qasim al-Musawi al-Khu’i, Prolegomena to the Qur’an, Oxford, 1998, Page 137-138)

ആരോപണമുന്നയിച്ചവർ പോലും സ്വയം തന്നെ പിതൃത്വം നിഷേധിക്കാൻ മാത്രം ദുർബലമായ തെളിവുകളാണ് ഖുർആനിൽ മാറ്റങ്ങളുണ്ടായിയെന്ന് വാദിക്കുന്നവരുടെ പക്കലുള്ളത് എന്നാണ് ഈ ശിയാപണ്ഡിതന്റെ ഉദ്ധരണി വെളിപ്പെടുത്തുന്നത്. ഉസൂലുൽ കാഫിയിലെ ഹദീഥ് ദുർബലമാണെന്ന് സ്ഥാപിക്കുവാൻ പാടുപെടുകയാണ് ഇന്ന് ജീവിക്കുന്ന ശിയാബുദ്ധിജീവികളെന്ന് അവരുടെ രചനകളും ഇന്റർനെറ്റിലെ ഇടപെടലുകളും ശ്രദ്ധിച്ചാൽ മനസ്സിലാവും. ഖുർആനിന്റെ അഖണ്ഡതയെ വെല്ലുവിളിക്കാൻ ശ്രമിക്കുന്നവരുടെ കൈകളിലുള്ള തെളിവുകൾ എത്രത്തോളം അടിസ്ഥാനരഹിതമാണെന്ന് ഇവ വ്യക്തമാക്കുന്നുണ്ട്.

ആകാശഭൂമികൾ സൃഷ്ടിക്കപ്പെട്ടത് ആറു ദിവസങ്ങളിലായിട്ടാണെന്ന് പല സ്ഥലങ്ങളിലും പറയുന്ന ഖുർആനിൽ തന്നെ അല്ലാഹു ഒരു കാര്യം സൃഷ്ടിക്കാൻ തീരുമാനിച്ചാൽ ഉണ്ടാകൂ എന്ന് പറയുമ്പോഴേക്ക് അതുണ്ടാകുമെന്നും പ്രസ്താവിക്കുന്നു. ഇത് വൈരുദ്ധ്യമല്ലേ? വൈരുദ്ധ്യം ആരോപിക്കപ്പെട്ട ഖുര്‍ആന്‍ സൂക്തങ്ങള്‍ കാണുക: തീര്‍ച്ചയായും നിങ്ങളുടെ രക്ഷിതാവ് ആകാശങ്ങളേയും ഭൂമിയേയും ആറ് ദശകളിലായി സൃഷ്ടിക്കുകയും, പിന്നീട് കാര്യങ്ങള്‍ നിയന്ത്രിച്ചുകൊണ്ട് സിംഹാസനസ്ഥനാവുകയും ചെയ്ത അല്ലാഹുവാകുന്നു. (10:3) ആകാശങ്ങളുടേയും ഭൂമിയുടേയും നിര്‍മ്മാതാവത്രെ അവന്‍. അവന്‍ ഒരു കാര്യം തീരുമാനിച്ചാല്‍ ഉണ്ടാകൂ എന്ന് പറയുക മാത്രമേ വേണ്ടതുള്ളു. ഉടനെ അതുണ്ടാകുന്നു. (2:117) ആകാശ ഭൂമികള്‍ ആറു ഘട്ടങ്ങളിലായിട്ടാണ് സൃഷ്ടിക്കപ്പെട്ടതെന്ന് പ്രസ്താവിക്കുന്ന ഖുര്‍ആന്‍ സൂക്തങ്ങള്‍ അവന്റെ സൃഷ്ടി വൈഭവം വ്യക്തമാക്കുന്ന സൂറത്തുല്‍ ബഖറയിലെ വചനവുമായി (2:117) യാതൊരു വിധത്തിലും വൈരുധ്യം പുലര്‍ത്തുന്നില്ല. ദൈവപുത്രവാദമുന്നയിക്കുന്ന ക്രൈസ്തവരെ വിമര്‍ശിച്ചുകൊണ്ടാണ് സൂറത്തുല്‍ബഖറയില്‍ ഇക്കാര്യം പ്രസ്താവിക്കുന്നത്. അവര്‍ പറയുന്നു, അല്ലാഹു സന്താനത്തെ സ്വീകരിച്ചിരിക്കുന്നുവെന്ന്. അവനത്രെ പരിശുദ്ധന്‍! അങ്ങനെയല്ല, ആകാശഭൂമികളിലുള്ളതെല്ലാം തന്നെ അവന്റെതാകുന്നു. എല്ലാവരും അവന് കീഴ്‌പ്പെട്ടിരിക്കുന്നവരാകുന്നു (2:116) എന്ന് പറഞ്ഞ ശേഷമാണ് സൂറത്തുല്‍ ബഖറയിലെ നടേ പറഞ്ഞ വചനമുള്ളത്. ആകാശ ഭൂമികളിലുള്ള ചെറുതും വലുതുമായ വസ്തുക്കളെല്ലാം പടച്ചവന്റെ കല്‍പ്പന പ്രകാരംഉണ്ടാവുകയെന്ന അവന്റെ വചനപ്രകാരം ഉണ്ടായതാണെന്നിരിക്കെ, ദൈവത്തിന്റെ വചനപ്രകാരം അത്ഭുതകരമായി ജനിച്ച യേശുക്രിസ്തുമാത്രം ദൈവപുത്രനാണെന്നു പറയുന്നതില്‍ യാതൊരു ന്യായവുമില്ലെന്ന വസ്തുത വ്യക്തമാക്കുകയാണ് ഈ സൂക്തങ്ങള്‍ ചെയ്യുന്നത്. സൂറത്തുല്‍ ബഖറയിലെ സൂക്തം (2:117), അല്ലാഹുവിന്റെ സൃഷ്ടിവൈഭവം വ്യക്തമാക്കുക മാത്രമാണ് ചെയ്യുന്നത്. യാതൊരു മുന്‍മാതൃകയുമില്ലാതെ സൃഷ്ടി നിര്‍വഹിക്കുന്നവനാണ് അല്ലാഹു. അവന്റെ സൃഷ്ടി ഒന്നുമില്ലായ്മയില്‍ നിന്നാണ്. ശൂന്യതയില്‍ നിന്നു ള്ള സൃഷ്ടിപ്പ് അവന്നു മാത്രം കഴിയുന്ന കാര്യമാണ്. ഒരു വസ്തു ഉണ്ടാക്കണമെന്ന് അവന്‍ തീരുമാനിച്ചു കഴിഞ്ഞാല്‍ അവന്ന് ഉണ്ടാവുക എന്ന് പറയേണ്ടതേയുള്ളൂ, ആ വസ്തു ഉണ്ടാവും. ഉണ്ടാവുക എന്ന ദൈവവചനത്തില്‍ തന്നെ പ്രസ്തുത വസ്തു എത്രകാലം കൊണ്ടാണ് ഉണ്ടാവേണ്ടത് എന്നും എത്ര ഘട്ടങ്ങളായാണ് ഉണ്ടാവേണ്ടത് എന്നുമുള്ള ദൈവിക തീരുമാനങ്ങളും ഉള്‍ക്കൊണ്ടിരിക്കും. പ്രസ്തുത തീരുമാനങ്ങളുടെ അടിസ്ഥാനത്തില്‍ വസ്തു ഉണ്ടാകുന്നു. ആകാശഭൂമികള്‍ ഉണ്ടായിരിക്കുന്നതും അല്ലാഹുവിന്റെ ഉണ്ടാവുകയെന്ന വചനപ്രകാരം തന്നെയാണ്. പ്രസ്തുത വചനത്തില്‍ തന്നെ അത് ആറുഘട്ടങ്ങളായാണ് ഉണ്ടാവേണ്ടതെന്നും പ്രസ്തുത സൃഷ്ടി പ്രക്രിയക്ക്ആവശ്യമായ കാലയളവ് ഇത്രയാണെന്നും എങ്ങനെയാണ് അത് ഉണ്ടാകേണ്ടതെന്നുമുള്ള ദൈവിക തീരുമാനങ്ങള്‍ കൂടി ഉള്‍ക്കൊണ്ടിരിക്കും. അതനുസരിച്ചാണ് അവന്‍ നിശ്ചയിച്ച കാലയളവിനുള്ളില്‍ ആറു ഘട്ടങ്ങളായിഅവ സൃഷ്ടിക്കപ്പെട്ടത്. ചുരുക്കത്തില്‍, ആറു ഘട്ടങ്ങളിലായാണ് ആല്ലാഹു ആകാശഭൂമികളെ സൃഷ്ടിച്ചത് എന്ന പരാമര്‍ശവും അവന്‍ ഒരു കാര്യം തീരുമാനിച്ചാല്‍ ഉണ്ടാവുക എന്ന വചനപ്രകാരം അതുണ്ടാവുന്നതാണെന്ന പ്രസ്താവനയും തമ്മില്‍ യാതൊരുവിധ വൈരുധ്യങ്ങളുമില്ല. ഒന്ന് ആകാശ ഭൂമികളുടെ സൃഷ്ടിക്രമം വ്യക്തമാക്കുന്നു, മറ്റേതാകട്ടെ അല്ലാഹുവിന്റെ സൃഷ്ടിവൈഭവമാണ് വിശദീകരിക്കുന്നത്.
ആകാശഭൂമികൾ ഒന്നായിരുന്നുവെന്നും പിന്നീട് അവ വേര്‍പെടുത്തപ്പെട്ടതാണെന്നുമുള്ള 21:30 ലെ പരാമര്‍ശത്തിന് വിരുദ്ധമായി അവ രണ്ടും വേര്‍പ്പെട്ടവയായിരുന്നുവെന്നും പിന്നീട് ഒന്നിച്ച് വരികയാണ് ചെയ്തതെന്നുമാണ് 41:11 ല്‍ പറയുന്നത്. ഇതെങ്ങനെ വിശദീകരിക്കാനാവും? ആകാശങ്ങളും ഭൂമിയും ഒട്ടിച്ചേര്‍ന്നതായിരുന്നുവെന്നും, എന്നിട്ട് നാം അവയെ വേര്‍പെടുത്തുകയാണുണ്ടായതെന്നും സത്യനിഷേധികള്‍ കണ്ടില്ലേ? വെള്ളത്തില്‍ നിന്ന് എല്ലാ ജൈവ വസ്തുക്കളെയും നാം ഉണ്ടാക്കുകയുംചെയ്തു. എന്നിട്ടും അവര്‍ വിശ്വസിക്കുന്നില്ലോ? (21:30) അതിനു പുറമെ അവന്‍ ആകാശത്തിന്റെ നേര്‍ക്ക് തിരിഞ്ഞു. അത് ഒരുപുകയായിരുന്നു. എന്നിട്ട് അതിനോടും ഭൂമിയോടും അവന്‍ പറഞ്ഞു. നിങ്ങള്‍ രണ്ടും അനുസരണ പൂര്‍വമോ നിര്‍ബന്ധിതമായോ വരിക. അവരണ്ടും പറഞ്ഞു: ഞങ്ങളിതാ അനുസരണമുള്ള വരായി വന്നിരിക്കുന്നു. (41:11) വൈരുധ്യമാരോപിക്കപ്പെട്ട സൂക്തങ്ങളാണിവ. സത്യത്തില്‍ പ്രപഞ്ചോല്‍പത്തിയുടെ രണ്ടു ഘട്ടങ്ങളാണ് ഈ സൂക്തങ്ങളില്‍ വിശദീകരിക്കപ്പെട്ടിരിക്കുന്നത്. ആധുനിക ശാസ്ത്രത്തിന്റെ നിഗമനങ്ങള്‍ ഖുര്‍ആനില്‍ സൂചിപ്പിക്കപ്പെട്ട രണ്ടു ഘട്ടങ്ങളും കൃത്യമായി സംഭവിച്ചതു തന്നെയാണെന്നാണ് മനസ്സിലാക്കി തന്നിരിക്കുന്നത്. ഏകദേശം രണ്ടായിരം കോടി കൊല്ലങ്ങള്‍ക്കു മുമ്പ് പ്രപഞ്ചം ആദിമ ഭ്രൂണാവസ്ഥയിലായിരുന്നുവെന്നും ഒരു ഉഗ്രസ്‌ഫോടനത്തോടെയാണ് പ്രപഞ്ചം നിലവില്‍ വന്നതെന്നുമാണ് ഇന്ന് ഏറ്റവുമധികം അംഗീകരിക്കപ്പെട്ടിട്ടുള്ള മഹാവിസ്‌ഫോടന സിദ്ധാന്തം പറയുന്നത്. ഈസിദ്ധാന്തം അംഗീകരിച്ചാലും ഇല്ലെങ്കിലും പ്രാപഞ്ചിക വസ്തുക്കളെല്ലാം ഒന്നിച്ച് ഒരൊറ്റ ആദിപദാര്‍ഥത്തിന്റെ ഭാഗമായിരുന്നുവെന്ന വസ്തുത ഇന്ന്ശാസ്ത്ര ലോകത്ത് ഏതാണ്ട് തര്‍ക്കമറ്റ സംഗതിയാണ്. ഈ ആദിപദാര്‍ഥത്തില്‍ നിന്ന് വേര്‍പ്പെട്ടാണ് ആകാശ ഗോളങ്ങളും ഭൂമിയുമെല്ലാം ഉണ്ടായത്. പ്രപഞ്ചോല്‍പത്തിയുടെ ആദ്യ നിമിഷത്തില്‍ സംഭവിച്ചതാണ് ഈവേര്‍പെടല്‍. ശക്തമായ ഒരു പൊട്ടിത്തെറിയിലൂടെ നടന്ന ഈ വേര്‍പെടലിനാണ് സാങ്കേതികമായി മഹാവിസ്‌ഫോടനം എന്ന് പറയുന്നത്. സൂറത്തുല്‍ അമ്പിയാഇലെ സൂചിത സൂക്തത്തില്‍(21:30) പരാമര്‍ശിക്കപ്പെട്ടിരിക്കുന്നത് ഈ വേര്‍പ്പെടുത്തലാണ്. ആകാശവും ഭൂമിയും ഒട്ടിച്ചേര്‍ന്ന് ഒരു ആദിമ പിണ്ഡാവസ്ഥയിലായിരുന്നുവെന്നും പിന്നീട് അല്ലാഹു അവയെ വേര്‍പെടുത്തിയെന്നുമുള്ള ഖുര്‍ആനിക പരാമര്‍ശങ്ങള്‍ പ്രപഞ്ചോല്‍പത്തിയുടെ ആദ്യ ഘട്ടത്തെയാണ്കുറിക്കുന്നതെന്നര്‍ഥം. എന്നാല്‍, മഹാ വിസ്‌ഫോടനത്തോടെ പ്രാപഞ്ചിക വസ്തുക്കളെല്ലാം ഇന്നു നിലനില്‍ക്കുന്ന രീതിയില്‍ ആയിക്കഴിയുകയല്ല ചെയ്തത്. അതിന് വീണ്ടും പരിണാമദശകള്‍ കഴിയേണ്ടതായി ഉണ്ടായിരുന്നു. ആകാശവസ്തുക്കളുടെ രൂപീകരണത്തില്‍ പ്രധാനം നക്ഷത്രരൂപീകരണത്തിനാണല്ലോ. ഒരു നക്ഷത്രം ഉണ്ടാകുന്നത് എങ്ങനെയെന്ന് ഏകദേശം കൃത്യമായിത്തന്നെ ഇന്ന് നമുക്ക് പറയാന്‍ കഴിയും. ഉപരിലോകത്തുള്ള ധൂളി വാതകപടലങ്ങളായ നെബുലകളില്‍ നിന്നാണ് നക്ഷത്രങ്ങള്‍ പിറവിയെടുക്കുന്നത്. നെബുലകളിലെ പൊടിപടലങ്ങള്‍ക്കകത്ത് പെട്ടെന്നുണ്ടാകുന്ന ആഘാതത്തിന്റെ ഫലമായി അവ ഘനീഭവിച്ച് സാന്ദ്രമാവുകയും നക്ഷത്രഭ്രൂണമായിത്തീരുകയും ചെയ്യുന്നു. ഈഭ്രൂണം ചുറ്റുമുള്ള വാതകപടലത്തില്‍നിന്ന് കണികകളെ ആകര്‍ഷിച്ചുകൊണ്ട് വലുപ്പം വര്‍ധിപ്പിച്ച് പ്രാഗ് നക്ഷത്രമായിത്തീരുന്നു. ഈ പ്രാഗ് നക്ഷത്രം സ്വന്തം ഗുരുത്വാകര്‍ഷണംമൂലം ചുരുങ്ങികൊണ്ടാണ് നക്ഷത്രങ്ങളുണ്ടാകുന്നത്. 1100 പ്രകാശവര്‍ഷങ്ങള്‍അകലെയുള്ള എം ജി സി 1333 എന്ന നെബുലയിലെ നക്ഷത്രാന്തരീയ പുകപടലങ്ങളില്‍ നിന്ന് ഒരു പ്രാഗ് നക്ഷത്രം പിറവിയെടുക്കുന്നത് ഹാവായിലെ മാക്‌സ്വെല്‍ ദൂരദര്‍ശിനിയിലൂടെ കാണാന്‍ ശാസ്ത്രജ്ഞന്‍മാര്‍ക്ക് കഴിഞ്ഞതോടെ നക്ഷത്ര രൂപീകരണത്തെ കുറിച്ച ഈസിദ്ധാന്തം ഏതാണ്ട് സര്‍വ്വാംഗീകൃതമായിട്ടുണ്ട്. പ്രപഞ്ചോല്‍പത്തിയുടെ രണ്ടാം ഘട്ടത്തിലുള്ള നക്ഷത്രങ്ങളുടെ സൃഷ്ടിയെ കുറിച്ചായിരിക്കാം സൂറത്തുഫുസ്സിലത്തിലുള്ള 11-ാം വചനത്തില്‍ പ്രതിപാദിക്കപ്പെട്ടിരിക്കുന്നത്. പുകപടലങ്ങളില്‍ നിന്നുള്ള ആകാശഗോളങ്ങളുടെ സൃഷ്ടിയാണല്ലോ ഈവചനത്തിലെ പ്രതിപാദ്യവിഷയം. ഇത് മഹാവിസ്‌ഫോടനത്തിനു ശേഷം സംഭവിച്ചുകൊണ്ടിരിക്കുന്നതെന്ന് ശാസ്ത്രം പറയുന്ന നെബുലകളില്‍ നിന്നുള്ള നക്ഷത്ര രൂപികരണത്തെ കുറിച്ചു തന്നെയാകാനാണ് സാധ്യത. ഖുര്‍ആനില്‍ രണ്ടു സ്ഥലങ്ങളില്‍ പറയുന്ന പ്രപഞ്ചസൃഷ്ടിയുടെ ക്രമം രണ്ടും ശരിയാണെന്ന വസ്തുതയാണ് നമുക്ക് ഇവിടെ ബോധ്യമാകുന്നത്. പ്രത്യക്ഷത്തില്‍ വൈരുധ്യമുള്ളതെന്ന് തോന്നുന്ന ഖുര്‍ആനിക പരാമര്‍ശങ്ങള്‍ പോലും പരസ്പരപൂരകങ്ങളും വസ്തുതകള്‍ മാത്രം വിവരിക്കുന്നവയുമാണെന്ന വസ്തുതയാണ് ആധുനിക ശാസ്ത്രം നമ്മെ പഠിപ്പിക്കുന്നതെന്ന കാര്യം എന്തു മാത്രം വലിയ അത്ഭുതമാണ്!. ഖുര്‍ആനിന്റെ അമാനുഷികതയും ദൈവികതയും വ്യക്തമാക്കുന്നവയാണ് ഈ വചനങ്ങളെന്നുള്ളതാണ് വാസ്തവം.
അല്ലാഹു സിംഹാസനസ്ഥനാണെന്ന് 57:4ലും പ്രസ്തുത സിംഹാസനം ജലത്തിനു മുകളിലാണെന്ന് 11:7ലും പറയുന്നതിന് വിരുദ്ധമായി 50:16ല് അവന് നിങ്ങളുടെ ജീവനാഡിയേക്കാള്‍ അടുത്താണെന്ന് പറയുന്നുണ്ടല്ലോ. ഇത് വൈരുധ്യമല്ലേ ? പ്രപഞ്ചത്തിലെ ചെറുതും വലുതുമായ സകല പ്രതിഭാസങ്ങളുടെയും സ്രഷ്ടാവാണ് അല്ലാഹു. പദാര്‍ത്ഥ പ്രപഞ്ചത്തിന്റെ സ്രഷ്ടാവായ അവന്‍ പ്രപഞ്ചാതീതനാണ് പദാര്‍ഥാതീതനാണ്. പദാര്‍ത്ഥ ലോകത്തെ കുറിച്ച് മാത്രമെ മനുഷ്യന് പരീക്ഷണ നിരീക്ഷണങ്ങളിലൂടെ മനസ്സിലാകൂ. സ്ഥലകാല സാതത്യത്തിന് അതീതമായ യാതൊന്നിനെക്കുറിച്ചും മനസ്സിലാക്കുവാനുള്ള കഴിവ് മനുഷ്യ മസ്തിഷ്‌കത്തിന് നല്‍കപ്പെട്ടിട്ടില്ല. ഈപരിമിതി മനസ്സിലാക്കിക്കൊണ്ടു വേണം അല്ലാഹുവിനെയും അവന്റെ ഉണ്മയെയുമെല്ലാം കുറിച്ച് ഖുര്‍ആന്‍ പറഞ്ഞകാര്യങ്ങള്‍ നാം വിലയിരുത്താന്‍. ദൈവികോണ്മയെക്കുറിച്ച് അറിയുവാന്‍ മനുഷ്യന്റെ പക്കല്‍ മാര്‍ഗ്ഗങ്ങളൊന്നുമില്ല, ദൈവിക വെളിപാടുകളല്ലാതെ. അല്ലാഹു തന്നെ സ്വയം വെളിപ്പെടുത്തിയ കാര്യങ്ങള്‍ മാത്രമേ അവന്റെ അസ്തിത്വത്തെകുറിച്ച് നമുക്കറിയൂ. ദൈവിക വെളിപാടുകളുടെ മാത്രം സമാഹാരമായി ഇന്ന് നിലനില്‍ക്കുന്ന ഒരു ഗ്രന്ഥം മാത്രമേയുള്ളൂ, ഖുര്‍ആന്‍. മനുഷ്യരുടെ കൈകടത്തലുകളില്‍ നിന്ന് അല്ലാഹുവിനാല്‍ തന്നെ സംരക്ഷിക്കപ്പെടുന്ന ഗ്രന്ഥം. ദൈവികാസ്തിത്വത്തെകുറിച്ച് ഖുര്‍ആന്‍ നല്‍കുന്ന അറിവ് അപ്പടി സ്വീകരിക്കുകയല്ലാതെ അവ വിശദീകരിക്കുകയോ വ്യഖ്യാനിക്കുകയോ ചെയ്യാന്‍ മനുഷ്യര്‍ അശക്തരാണ്. മനുഷ്യ വിജ്ഞാനത്തിന്റെ വരുതിയില്‍ വരാത്ത കാര്യത്തെ അവര്‍ എങ്ങിനെ വ്യാഖ്യാനിക്കാനാണ് ? ദൈവിക സിംഹാസനത്തെ കുറിച്ച് ഖുര്‍ആന്‍ പറയുന്ന ഏതാനും സൂക്തങ്ങള്‍ കാണുക: ആകാശങ്ങളും ഭൂമിയും ആറ് ദിവസങ്ങളിലായി സൃഷ്ടിച്ചവനാണ് അവന്‍. പിന്നീട് അവന്‍ സിംഹാസനസ്ഥനായി. (57:4) പരമകാരുണികന്‍ സിംഹാസനസ്ഥനായിരിക്കുന്നു. (20:5) അവന്റെ അധികാരപീഠം ആകാശഭൂമികളെ മുഴുവന്‍ ഉള്‍കൊള്ളുന്നതാണ്. (2:255) ആറു ദിവസങ്ങളിലായി ആകാശഭൂമികളെ സൃഷ്ടിച്ചവന്‍ അവനത്രെ. അവന്റെ സിംഹാസനം ജലത്തിന്‍ മേലായിരുന്നു.(11:6) ഈ സൂക്തങ്ങളില്‍ പ്രതിപാദിക്കപ്പെട്ട സിംഹാസനം കൊണ്ടുള്ള വിവക്ഷയെന്താണെന്നോ അത് ജലത്തിലായിരുന്നു എന്ന് പറഞ്ഞതിന്റെ അര്‍ഥമെന്താണെന്നോ നമുക്കറിയില്ല. സ്ഥലകാല നൈരന്തര്യത്തിന്നതീതനായ സ്രഷ്ടാവിനെപ്പറ്റി അറിയാന്‍ വെളിപാടല്ലാത്ത മറ്റുമാര്‍ഗ്ഗങ്ങളൊന്നുമില്ലെങ്കില്‍ അവന്റെ സിംഹാസനത്തെകുറിച്ച് അറിയുവാനും പ്രസ്തുത മാര്‍ഗം മാത്രമെ സ്വീകരിക്കാന്‍ നിര്‍വ്വാഹമുള്ളൂ. ഖുര്‍ആനും പ്രവാചക വചനങ്ങളും പറഞ്ഞ കാര്യങ്ങള്‍ അപ്പടി സ്വീകരിക്കുകയും സ്വന്തമായ വ്യാഖ്യാനങ്ങള്‍ നല്‍കാതിരിക്കുകയും ചെയ്യുകയാണ് ഇത്തരം സൂക്തങ്ങളുടെ കാര്യത്തില്‍ സച്ചരിതരായ പ്രവാചക ശിഷ്യന്‍മാരുടെ നിലപാട്. ആ നിലപാട് സ്വീകരിക്കുക മാത്രമാണ് നമുക്കു കരണീയം. അല്ലാഹു മനുഷ്യരുടെ സമീപത്താണുള്ളത് എന്നുവ്യക്തമാക്കുന്ന സൂക്തങ്ങള്‍ ഖുര്‍ആനിലുണ്ട്. പ്രസ്തുത സൂക്തങ്ങള്‍ എന്താണ് അര്‍ഥമാക്കുന്നത് എന്നറിയാന്‍ അവയുടെ സാരം ഒന്നുപരിശോധിച്ചാല്‍ മതിയെന്നതാണ് യാഥാര്‍ത്ഥ്യം. നിന്നോട് എന്റെ ദാസന്‍മാര്‍ എന്നെപ്പറ്റി ചോദിച്ചാല്‍ ഞാന്‍ (അവര്‍ക്ക് ഏറ്റവും) അടുത്തുള്ളവനാകുന്നു (എന്നു പറയുക). പ്രാര്‍ഥിക്കുന്നവന്‍ എന്നെവിളിച്ച് പ്രാര്‍ഥിച്ചാല്‍ ഞാന്‍ ആ പ്രാര്‍ഥനക്ക് ഉത്തരം നല്‍കുന്നതാണ്. (2:186) നീ പറയുക: ഞാന്‍ പിഴച്ചു പോയിട്ടുണ്ടെങ്കില്‍ ഞാന്‍ പിഴക്കുന്നതിന്റെ ദോഷം എനിക്കുതന്നെയാണ്. ഞാന്‍ നേര്‍മാര്‍ഗ്ഗം പ്രാപിച്ചുവെങ്കിലോ, അത് എനിക്ക് എന്റെ രക്ഷിതാവ് ബോധനം നല്‍കുന്നതിന്റെ ഫലമായിട്ടാണ്. തീര്‍ച്ചയായും അവന്‍ കേള്‍ക്കുന്നവനും സമീപസ്ഥനുമാകുന്നു. 34:50 തീര്‍ച്ചയായും മനുഷ്യനെ നാം സൃഷ്ടിച്ചിരിക്കുന്നു. അവന്റെ മനസ്സ് മന്ത്രിച്ചുകൊണ്ടിരിക്കുന്നത് നാം അറിയുകയും ചെയ്യുന്നു. നാം (അവന്റെ)കണ്ഠനാഡിയേക്കാള്‍ അവനോട് അടുത്തവനുമാകുന്നു. (50:16) ഈ സൂക്തങ്ങളെല്ലാം അല്ലാഹുവിന്റെ ഗുണ വിശേഷങ്ങളെ കുറിച്ചാണ് പ്രതിപാദിക്കുന്നത്, അവന്റെ സത്തയെ സംബന്ധിച്ചല്ല. അതുകൊണ്ട് തന്നെ അവന്റെ സത്ത മനുഷ്യരുടെ സമീപത്താണുള്ളത് എന്നോ അവന്‍ കണ്ഠനാഡിയേക്കാള്‍ അടുത്താണ് സ്ഥിതി ചെയ്യുന്നതെന്നോ അല്ല ഈസൂക്തങ്ങള്‍ അര്‍ഥമാക്കുന്നത്; പ്രത്യുത അവന്റെ ശക്തിയും കഴിവും മനുഷ്യരുടെ സമീപസ്ഥമാണെന്നാണ്. പ്രപഞ്ചം അല്ലാഹുവിന്റെ സൃഷ്ടിയാണ്. അതുകൊണ്ട് തന്നെ അല്ലാഹുവിന്റെ സത്ത പ്രപഞ്ചത്തിന് അതീതമായിരിക്കും. പ്രപഞ്ചസ്രഷ്ടാവായ അല്ലാഹു പദാര്‍ത്ഥലോകത്തെവിടെയോ കുടിയിരിക്കുന്നവനാണെന്ന് കരുതുന്നത് ശരിയല്ല. എന്നാല്‍ അവന്റെ കഴിവുകളും ശക്തിയും പ്രപഞ്ചമാസകലം വ്യാപിച്ചുകിടക്കുകയാണ്, പ്രപഞ്ചത്തിലെ ഒരോ വസ്തുവിനെയും ചൂഴ്ന്നുകിടക്കുകയാണ്. അതിനാല്‍ അവനോട് പ്രാര്‍ഥിക്കുവാന്‍ ഒരു ഇടയാളന്റെ ആവശ്യമില്ല. അവനെ സമീപിക്കുവാന്‍ ഒരു ശുപാര്‍ശകനും വേണ്ടതില്ല. അവനില്‍ നിന്ന് എന്തെങ്കിലും മറച്ചുവെക്കാന്‍ കഴിയുമെന്ന് മനുഷ്യര്‍ വിചാരിക്കേണ്ടതുമില്ല. അവരുടെ മനസ്സിനകത്തുള്ളതു പോലും അറിയുന്നവനാണവന്‍. ഇതാണ് ഉപര്യുക്ത സൂക്തങ്ങള്‍ അര്‍ഥമാക്കുന്നത്. അല്ലാഹു സിംഹാസനസ്ഥനായതിനെ കുറിച്ച് പ്രതിപാദിക്കുന്ന സൂക്തങ്ങള്‍ അവന്റെ സത്തയെ കുറിച്ച് മനുഷ്യര്‍ അറിയേണ്ടതായ കാര്യങ്ങള്‍ വെളിപ്പെടുത്തുന്നു. മനുഷ്യരുമയി ബന്ധപ്പെട്ട് അല്ലാഹുവിനെക്കുറിച്ച് പ്രതിപാദിക്കുന്ന സൂക്തങ്ങളാകട്ടെ, അവന്റെ ഗുണവിശേഷങ്ങളെയാണ് ദ്യോതിപ്പിക്കുന്നത്. അതുകൊണ്ട് തന്നെ ഇവ തമ്മില്‍ യാതൊരു വൈരുധ്യവുമില്ല. അല്ലാഹു സിംഹാസനസ്ഥന്‍ തന്നെയാണ്. എന്നാല്‍ അവന്റെ കഴിവുകളും ശക്തിയും മനുഷ്യരുടെ കണ്ഠനാഡിയേക്കാള്‍ അടുത്താണുള്ളത്. അവരുടെ ശരീരത്തെയും മനസ്സിനെയും ആ കഴിവുകള്‍ ചൂഴ്ന്നുനില്‍ക്കുന്നു.
ഈ പ്രപഞ്ചത്തെ സൃഷ്ടിച്ചത് ആറ് ദിവസം കൊണ്ടാണെന്ന് ഖുർആനിൽ പല തവണ പറയുന്നുണ്ട്. (ഉദാ: 7:54, 10:3, 11:7, 25:59). എന്നാൽ 41:9-12 സൂക്തങ്ങളിലെ സൃഷ്ടി വിവരണ പ്രകാരം എട്ട് ദിവസം കൊണ്ടാണ് പ്രപഞ്ചസൃഷ്ടി നടന്നതെന്നാണ് മനസ്സിലാവുന്നത്. ഇത് വ്യക്തമായ വൈരുധ്യമല്ലേ? ആറു ദിവസം കൊണ്ടാണ് പ്രപഞ്ചം സൃഷ്ടിക്കപ്പെട്ടതെന്ന് വ്യക്തമാക്കുന്ന ഒട്ടനവധി സൂക്തങ്ങള്‍ ഖുര്‍ആനിലുണ്ട്. ഒരു ഉദാഹരണം 7:54: തീര്‍ച്ചയായും നിങ്ങളുടെ രക്ഷിതാവ് ആറു ദിവസങ്ങളിലായി ആകാശങ്ങളും ഭൂമിയും സൃഷ്ടിച്ചവനായ ആല്ലാഹുവാകുന്നു. ദിവസം എന്ന് പരിഭാഷപ്പെടുത്തപ്പെട്ടിരിക്കുന്ന യൗം എന്ന അറബിപദത്തിന് ഘട്ടം എന്നും അര്‍ത്ഥമുണ്ട്. ഇവിടെ ആറു ദിവസങ്ങള്‍ എന്നതുകൊണ്ട് സൂര്യോദയം മുതല്‍ സൂര്യാസ്തമയം വരെയുള്ള ഒരുദിവസമല്ല വിവക്ഷിച്ചിട്ടുള്ളതെന്ന് വ്യക്തമാണ്. അല്ലാഹുവിന്റെയടുക്കല്‍ ദിവസമെന്നാല്‍ മനുഷ്യ പരിഗണനയിലുള്ള ദിവസമല്ലെന്ന വസ്തുത ഖുര്‍ആന്‍ വ്യക്തമാക്കുകയും ചെയ്തിട്ടുണ്ട്. ഉപരിലോകങ്ങളും ഭൂമിയുമെല്ലാം സൃഷ്ടിക്കപ്പെട്ടത് ആറ് വ്യത്യസ്ത ഘട്ടങ്ങളായി കൊണ്ടാണെന്ന് വ്യക്തമാക്കുകയാണ് അക്കാര്യം വിവരിച്ച സൂക്തങ്ങളെല്ലാം ചെയ്യുന്നത്. ഈ ഘട്ടങ്ങളുടെ കാലദൈര്‍ഘ്യം എത്രയാണെന്ന് നമുക്കറിയില്ല. അത് ഖുര്‍ആന്‍ വ്യക്തമാക്കുന്നുമ്മില്ല. ആറു ഘട്ടങ്ങളിലായാണ് ആകാശ ഭൂമികളുടെ സൃഷ്ടി സംഭവിച്ചത് എന്നകാര്യം ഖുര്‍ആനില്‍ ഖണ്ഡിതമായി പറഞ്ഞിട്ടുണ്ട്. അതിലധികമോ കുറച്ചോ ഘട്ടങ്ങളായിട്ടാണ് സൃഷ്ടി സംഭവിച്ചതെന്ന് പറയുന്ന ഒരൊറ്റ സൂക്തവും ഖുര്‍ആനിലില്ല. എട്ടു ദിവസം കൊണ്ടാണ് സൃഷ്ടി നടത്തിയതെന്ന് ഖുര്‍ആനില്‍ ഒരിടത്തുമില്ല. എന്നാല്‍ ഖുര്‍ആനില്‍ ഇങ്ങനെ പറയുന്നുവെന്ന് ദുര്‍വ്യാഖ്യാനം ചെയ്യപ്പെടുന്നത് സൂറത്തു ഫുസ്സിലത്തിലെ (അധ്യായം 41) 9 മുതല്‍ 12 വരെയുള്ള സൂക്തങ്ങളുടെ അടിസ്ഥാനത്തിലാണ്. പ്രസ്തുതസൂക്തങ്ങള്‍ പരിശോധിക്കുക: നീ പറയുക; രണ്ടു ഘട്ടങ്ങളിലായി ഭൂമിയെ സൃഷ്ടിച്ചവനില്‍ നിങ്ങള്‍ അവിശ്വസിക്കുകയും അവന് നിങ്ങള്‍ സമന്‍മാരെ സ്ഥാപിക്കുകയും തന്നെയാണോ ചെയ്യുന്നത്? അവനാകുന്നു ലോകങ്ങളുടെ രക്ഷിതാവ്. അതില്‍ അതിന്റെ ഉപരിഭാഗത്ത് ഉറച്ചുനില്‍ക്കുന്ന പര്‍വ്വതങ്ങള്‍ അവന്‍ സ്ഥാപിക്കുകയും അതില്‍ അഭിവൃദ്ധി ഉണ്ടാക്കുകയും, അതിലെ ആഹാരങ്ങള്‍ അവിടെ വ്യവസ്ഥപ്പെടുത്തി വെക്കുകയും ചെയ്തിരിക്കുന്നു, നാലു ഘട്ടങ്ങളിലായി. ആവശ്യപ്പെടുന്നവര്‍ക്കുവേണ്ടി ശരിയായ അനുപാതത്തില്‍. അതിനു പുറമെ അവന്‍ ഉപരിലോകത്തിന്റെ നേര്‍ക്കു തിരിഞ്ഞു. അത് ഒരു പുകയായിരുന്നു. എന്നിട്ട് അതിനോടും ഭൂമിയോടും അവന്‍ പറഞ്ഞു: നിങ്ങള്‍ രണ്ടുപേരും അനുസരണ പൂര്‍ണ്ണമോ നിര്‍ബന്ധിതമായോ വരിക. അവ രണ്ടും പറഞ്ഞു: ഞങ്ങളിതാ അനുസരണമുള്ളവരായി വന്നിരിക്കുന്നു. അങ്ങനെ രണ്ട് ഘട്ടങ്ങളിലായി അവയെ അവന്‍ ഏഴ് ആകാശങ്ങളാക്കിത്തീര്‍ത്തു.’ ഈ സൂക്തങ്ങളിലെവിടെയും എട്ട് ഘട്ടങ്ങളിലായിട്ടാണ് അല്ലാഹു ആകാശ ഭൂമികളെ സൃഷ്ടിച്ചതെന്ന് പറഞ്ഞിട്ടില്ല. ഒന്‍പതാം സൂക്തത്തിലെ രണ്ടു ഘട്ടങ്ങളും പത്താം സൂക്തത്തിലെ നാലു ഘട്ടങ്ങളും പന്ത്രണ്ടാം സൂക്തത്തിലെ രണ്ടു ഘട്ടങ്ങളും കൂട്ടിയാല്‍ എട്ടു ഘട്ടങ്ങളാകുമെന്നതിനാല്‍ ഈ സൂക്തങ്ങള്‍ ആറു ഘട്ടങ്ങളായിട്ടാണ് പ്രപഞ്ച സൃഷ്ടി നടന്നതെന്ന സൂക്തങ്ങളുമായി വൈരുധ്യം പുലര്‍ത്തുന്നുണ്ടെന്നാണ് വിമര്‍ശകന്‍മാരുടെവാദം. ഈ വാദത്തില്‍ യാതൊരു കഴമ്പുമില്ല. ഒമ്പത്, പത്ത് സൂക്തങ്ങളില്‍ ഭൂമിയുടെയും അതിലുള്ളതിന്റെയും സൃഷ്ടിപ്പിനെ സംബന്ധിച്ചാണ് പരാമര്‍ശിക്കുന്നത്. ഭൂമിയുടെ സൃഷ്ടിപ്പ് രണ്ടു ഘട്ടങ്ങളിലായി നിര്‍വ്വഹിച്ചതിനെ കുറിച്ച് ഒന്‍പതാം സൂക്തത്തില്‍ പറയുന്നു. ഭൂമിയെയും അതിലെ പര്‍വ്വതങ്ങളെയും ആഹാര സംവിധാനങ്ങളെയുമെല്ലാം സൃഷ്ടിച്ചത് നാലു ഘട്ടങ്ങളായിട്ടാണെന്ന് പത്താം സൂക്തത്തിലും പറയുന്നു. ഒന്‍പതാം സൂക്തത്തില്‍ പ്രതിപാദിക്കപ്പട്ട രണ്ടു ഘട്ടങ്ങള്‍ കൂടി പത്താം സൂക്തത്തിലെ നാലു ഘട്ടത്തില്‍ ഉള്‍പ്പെടുന്നുവെന്നര്‍ഥം. ഒന്‍പതും പത്തും, സൂക്തങ്ങളില്‍ പ്രതിപാദിക്കപ്പെട്ട സൃഷ്ടി മൊത്തമായി നാലു ഘട്ടങ്ങളായിട്ടാണ് നിര്‍വഹിക്കപ്പെട്ടതെന്നാണ് പത്താമത്തെ സൂക്തത്തിന്റെ അവസാനത്തില്‍ നാലു ഘട്ടങ്ങളിലായി എന്ന് പറഞ്ഞതിനര്‍ഥം. അപ്പോള്‍ ആകാശത്തെ സൃഷ്ടിച്ച രണ്ടു ഘട്ടങ്ങളും കൂടി കൂട്ടുമ്പോള്‍ ആകെ പ്രപഞ്ച സൃഷ്ടി നടന്നത് ആറു ഘട്ടങ്ങളായിട്ടാണെന്ന വസ്തുത സുതരാം വ്യക്തമാവുന്നു. യഥാര്‍ത്ഥത്തില്‍ ഈ വചനങ്ങള്‍, സൃഷ്ടി നടന്നത് ആറുഘട്ടങ്ങളിലായിട്ടാണെന്ന് പറയുന്ന മറ്റു വചനങ്ങളുമായി വൈരുദ്ധ്യങ്ങളൊന്നും വെച്ചുപുലര്‍ത്തുന്നില്ല.
വൈരുധ്യം ആരോപിക്കപ്പെട്ടിരിക്കുന്ന ഖുര്‍ആന്‍ സൂക്തങ്ങളുടെ സാരംപരിശോധക്കുക: (നബിയെ) നിന്നോട് അവര്‍ ശിക്ഷയുടെ കാര്യത്തില്‍ ധൃതികൂട്ടികൊണ്ടിരിക്കുന്നു. അല്ലാഹു തന്റെ വാഗ്ദാനം ലംഘിക്കുകയേയില്ല. തീര്‍ച്ചയായും നിന്റെ നാഥന്റെയടുക്കല്‍ ഒരുദിവസമെന്നാല്‍ നിങ്ങള്‍ എണ്ണിവരുന്ന തരത്തിലുള്ള ആയിരം കൊല്ലം പോലെയാകുന്നു. (22:47) അവന്‍ ആകാശത്തുനിന്ന് ഭൂമിയിലേക്ക് കാര്യങ്ങള്‍ നിയന്ത്രിച്ചയക്കുന്നു. പിന്നീട് ഒരു ദിവസം കാര്യം അവങ്കലേക്ക് ഉയര്‍ന്നുപോകുന്നു. നിങ്ങള്‍ കണക്കാക്കുന്ന തരത്തിലുള്ള ആയിരം വര്‍ഷമാകുന്നു ആ ദിവസത്തിന്റെ അളവ്. (32:5) അമ്പതിനായിരം കൊല്ലത്തിന്റെ അളവുള്ളതായ ഒരു ദിവസത്തി ല്‍മലക്കുകളും ആത്മാവും അവങ്കലേക്ക് കയറിപ്പോകുന്നു. (70:4) ഈ മൂന്ന് സൂക്തങ്ങളിലും ദിവസം എന്നു പരിഭാഷപ്പെടുത്തിയിരിക്കുന്നത് ‘യൗം‘ എന്ന അറബി പദത്തെയാണ്. സാധാരണയായി ഇരുപത്തിനാല് മണിക്കൂറുള്ള ഒരു ദിവസത്തിനാണ് അറബിയില്‍ യൗം എന്നു പറയാറുള്ളത്. എന്നാല്‍ ഘട്ടം, കാലയളവ് എന്നീ അര്‍ത്ഥങ്ങളിലും ഈ പദം ഉപയോഗിക്കാറുണ്ട്. ഖുര്‍ആനിലും ഈ അര്‍ത്ഥകല്‍പ്പനകളിലെല്ലാം വ്യത്യസ്ത സാഹചര്യങ്ങളില്‍ യൗം എന്ന പദംപ്രയോഗിച്ചിട്ടുണ്ട്. മരണാനന്തര ജീവിതത്തെകുറിച്ച് പ്രതിപാദിക്കുമ്പോള്‍ വിവിധസംഭവങ്ങളുടെ കാലയളവിനെ കുറിക്കുന്നതിനു വേണ്ടിയാണ് ഖുര്‍ആന്‍ ‘യൗം‘ എന്നു പ്രയോഗിച്ചിരിക്കുന്നത്. അനന്തമായ മരണാനന്തര ജീവിതത്തെക്കുറിച്ച് മൊത്തമായി തന്നെ യൗമുദ്ദീന്‍ (പ്രതിഫലത്തിന്റെ ദിവസം) എന്നാണ് ഖുര്‍ആനില്‍ പരാമര്‍ശിക്കപ്പെട്ടിരിക്കുന്നത്. അനന്തതയുടെ ദിവസത്തെ കുറിച്ചും ഖുര്‍ആനില്‍ പരാമര്‍ശമുണ്ട്. (അവരോട് പറയപ്പെടും) സമാധാനപൂര്‍വ്വം നിങ്ങളതില്‍ പ്രവേശിച്ചുകൊള്ളുക. അനശ്വര ജീവിതത്തിന്റെ ദിവസ (യൗമുല്‍ഖുലൂദ്)മാണത്. (50:34). അനശ്വരതയുടെ ദിവസമെന്നത് ഏതായിരുന്നാലും സൂര്യനുദിച്ച് അസ്തമിക്കുന്നതിനിടയിലെ കാലയളവാകുകയില്ലല്ലോ. അത് അനന്തമായ ദിവസമാണ്. ഒരിക്കലും അവസാനിക്കാത്ത ദിവസം. ശാശ്വത ജീവിതത്തിന്റെ ദിവസം. ഇവിടെ ദിവസം എന്ന് പ്രയോഗിക്കപ്പെട്ടിരിക്കുന്നത് ഏത് അര്‍ത്ഥത്തിലാണെന്ന് പ്രസ്തുത പ്രയോഗത്തില്‍ നിന്ന് തന്നെ വ്യക്തമാണല്ലോ. അന്ത്യനാളില്‍ സംഭവിക്കുന്ന പല കാര്യങ്ങളെ കുറിച്ചും ഖുര്‍ആനില്‍ ദിവസം (യൗം) എന്നാണ് പ്രയോഗിക്കപ്പെട്ടിരിക്കുന്നത.് മനുഷ്യന്‍ ചിന്നിച്ചിതറിയ പാറ്റകളെപ്പോലെയും പര്‍വ്വതങ്ങള്‍ കടഞ്ഞ രോമം പോലെയുമാകുന്ന ദിവസം (10:4,5.), ഭൂമി ഈ ഭൂമിയല്ലാതെ മറ്റൊന്നായും അതുപോലെ ഉപരിലോകങ്ങളും മാറ്റപ്പെടുകയും ഏകനും സര്‍വ്വാധകാരിയുമായ അല്ലാഹുവിങ്കലേക്ക് അവരെല്ലാം പുറപ്പെട്ടുവരികയും ചെയ്യുന്ന ദിവസം. (14:48), അന്നേ ദിവസം മനുഷ്യര്‍ പലസംഘങ്ങളായി പുറപ്പെടുന്നതാണ് (99:6), അന്നേ ദിവസം ചില മുഖങ്ങള്‍ താഴ്മ കാണിക്കുന്നതും പണിയെടുത്ത് ക്ഷീണിച്ചതുമായിരിക്കും. (88: 2, 3), രഹസ്യങ്ങള്‍ പരിശോധിക്കപ്പെ ടുന്ന ദിവസം. (86: 9), ലോകരക്ഷിതാവിങ്കലേക്ക്് ജനങ്ങള്‍ എഴുന്നേറ്റ് വരുന്ന ദിവസം. (83:6), ഒരാളും മറ്റൊരാള്‍ക്ക് വേണ്ടി യാതൊന്നും അധീനപ്പെടുത്താത്ത ഒരു ദിവസം, അന്നേദിവസം കൈകാര്യ കര്‍തൃത്വം അല്ലാഹുവിനായിരിക്കും. മനുഷ്യന്‍ തന്റെ സഹോദരനെയും മാതാവിനെയും പിതാവിനെയും തന്റെ ഭാര്യയെയും തന്റെ മക്കളെയും വിട്ടോടിപ്പോകുന്ന ദിവസം. (80:34-36.), മനുഷ്യന്‍ താന്‍ അധ്വാനിച്ചു വെച്ചതിനെ കുറിച്ച് ഓര്‍മ്മിക്കുന്ന ദിവസം (79:35) കാണുന്നവര്‍ക്കു വേണ്ടി നരകം വെളിവാക്കപ്പെടുന്ന ദിവസം. (79: 35), ആ നടുക്കുന്ന സംഭവം നടുക്കമുണ്ടാക്കുന്ന ദിവസം. (79:6), ആത്മാവും മലക്കുകളും അണിയണിയായി നില്‍ക്കുന്ന ദിവസം. (78:38), മനുഷ്യന്‍ തന്റെ കൈകള്‍ മുന്‍കൂട്ടി ചെയ്തു വെച്ചത് നോക്കികാണുകയും, അയ്യോ ഞാന്‍ മണ്ണായിരുന്നെങ്കില്‍ എത്ര നന്നായിരുന്നേനെയെന്ന് സത്യനിഷേധി പറയുകയും ചെയ്യുന്ന ദിവസം. (78:40). അതത്രെ യഥാര്‍ത്ഥ ദിവസം. (78:39). ഈ സൂക്തങ്ങളില്‍ പ്രയോഗിക്കപ്പെട്ട ദിവസത്തിന് എന്താണ് അര്‍ത്ഥം? ഓരോ സംഭവങ്ങളോടൊപ്പവും പ്രതിപാദിക്കപ്പെട്ട ദിവസത്തിന് ആ സംഭവം നടക്കുന്നതിനുള്ള കാലയളവ് എന്നാണ് അര്‍ത്ഥമാക്കുന്നതെന്ന് സുതരാം വ്യക്തമാണ്. അതുകൊണ്ട് തന്നെ പ്രസ്തുത ദിവസങ്ങളുടെ കാലദൈര്‍ഘ്യം വ്യത്യസ്തമായിരിക്കും. മനുഷ്യര്‍ ചിന്നിച്ചിതറിയ പാറ്റകളെപ്പോലെയാകുന്ന ദിവസത്തിന്റെ കാലദൈര്‍ഘ്യമാവുകയില്ല അവര്‍ പല സംഘങ്ങളായിപുറപ്പെടുന്ന ദിവസത്തിന്. രഹസ്യങ്ങള്‍ പരിശോധിക്കപ്പെടുന്ന ദിവസത്തിന്റെ ദൈര്‍ഘ്യമാവുകയില്ല നരകം വെളിവാക്കപ്പെടുന്ന ദിവസത്തിനുണ്ടാവുക. ഓരോ ദിവസത്തിന്റെയും ദൈര്‍ഘ്യംവ്യത്യസ്തമായിരിക്കും. അവയുടെ ദൈര്‍ഘ്യം എത്രയാണെന്ന് അല്ലാഹുവിന് മാത്രമേ അറിയൂ. പ്രസ്തുത ദിവസങ്ങളുടെ ദൈര്‍ഘ്യത്തെകുറിച്ച് അറിയുവാന്‍ നമ്മുടെ കൈയില്‍ മാര്‍ഗ്ഗങ്ങളൊന്നുമില്ല. അന്ത്യനാളിനോടനുബന്ധിച്ച് നടക്കുന്ന രണ്ട് കാര്യങ്ങളുടെ സമയദൈര്‍ഘ്യം മാത്രമേ ഖുര്‍ആനിലൂടെ അല്ലാഹു നമുക്ക് അറിയിച്ചുതന്നിട്ടുള്ളൂ. കാര്യങ്ങള്‍ അല്ലാഹുവിങ്കലേക്ക് ഉയര്‍ന്നു പോകുന്ന ഒരു ദിവസത്തിന്റെ അളവ് മനുഷ്യ ഗണനയിലുള്ള ആയിരം വര്‍ഷത്തിന് സമമാണെന്ന വസ്തുതയാണ് അല്ലാഹു സൂറത്തു സജദയിലൂടെ (32:5)വെളിപ്പെടുത്തുന്നത്. മലക്കുകളും ആത്മാവും അല്ലാഹുവിങ്കലേക്ക് കയറിപോകുന്ന ദിവസത്തിന്റെ ദൈര്‍ഘ്യം നമ്മുടെ അമ്പതിനായിരം കൊല്ലങ്ങള്‍ക്ക് തുല്യമാണെന്ന് സൂറത്തുല്‍ മആരിജിലും(70:4) വ്യക്തമാക്കുന്നു. രണ്ട് സൂക്തങ്ങളിലും പരാമര്‍ശിക്കപ്പെട്ടിട്ടുള്ളത് അന്ത്യദിനവുമായി ബന്ധപ്പെട്ട രണ്ട് സംഭവങ്ങളാണ്. പ്രസ്തുതസംഭവങ്ങള്‍ക്ക് എടുക്കുന്ന കാലദൈര്‍ഘ്യം വ്യത്യസ്തമാണെന്ന വസ്തുത ഈ ഖുര്‍ആന്‍ സൂക്തങ്ങളില്‍ നിന്ന് നമുക്ക് മനസ്സിലാകുന്നുവെന്നല്ലാതെ ഇവതമ്മില്‍ യാതൊരു വിധ വൈരുധ്യങ്ങളുമില്ലെന്നതാണ് വാസ്തവം. രണ്ടും രണ്ട് സംഭവങ്ങള്‍, അവയുടെ സമയ ദൈര്‍ഘ്യം വ്യത്യസ്തമാണെന്നു മാത്രം. ഇവയെങ്ങനെ വൈരുധ്യമാകും? എന്നാല്‍ സൂറത്തുല്‍ ഹജ്ജില്‍ നിന്ന് ഉദ്ധരിക്കപ്പെട്ട സൂക്ത(22:47)ത്തിന്റെ പശ്ചാത്തലം വ്യത്യസ്തമാണ്. സത്യനിഷേധികളുടെ പരിഹാസത്തിനുള്ള മറുപടിയായാണ് പ്രസ്തുത സൂക്തം അവതരിപ്പിക്കപ്പെട്ടത്. നിഷേധികള്‍ക്ക് ദൈവിക ശിക്ഷ ലഭിക്കുമെന്ന മുഹമ്മദ് നബി(ﷺ)യുടെ മുന്നറിയിപ്പിനെ പരിഹസിച്ചുകൊണ്ട്, ഞങ്ങള്‍ നിഷേധസ്വഭാവം സ്വീകരിച്ചിട്ട് വര്‍ഷങ്ങളായെങ്കിലും ശിക്ഷയുണ്ടാകാത്തത് എന്തേ എന്ന ചോദ്യത്തിനുള്ള മറുപടി. ദൈവിക ശിക്ഷ ഏതാനും ദിവസങ്ങള്‍ക്കുള്ളില്‍ തന്നെ വന്നുഭവിച്ചു കൊള്ളണമെന്നില്ലെന്നും ചരിത്രത്തിലെ ദൈവിക ഇടപെടലുകള്‍ നടക്കുന്നത് മനുഷ്യരുടെ ഗണനാക്രമത്തിനനുസരിച്ചല്ല, പ്രത്യുത അല്ലാഹുവിന്റെ തീരുമാനപ്രകാരമാണെന്നും വ്യക്തമാക്കുകയാണ് ഈ സൂക്തത്തില്‍ചെയ്യുന്നത്. അല്ലാഹുവിന്റെയടുക്കല്‍ ഒരു ദിവസമെന്നാല്‍ മനുഷ്യഗണനയിലെ ഒരു സഹസ്രാബ്ദത്തിന് തുല്യമാണെന്നും അതുകൊണ്ട് തന്നെ ശിക്ഷ വന്നുഭവിച്ചില്ലെന്ന് കളിയാക്കേണ്ടതില്ലെന്നുമാണ് ഈ സൂക്തം വ്യക്തമാക്കുന്നത്. മനുഷ്യ ചരിത്രത്തിലെ ദൈവിക ഇടപെടലുകളെ മനുഷ്യഗണനയുടെ അടിസ്ഥാനത്തില്‍ മനസ്സിലാക്കേണ്ടതില്ലെന്നാണ് ഈസൂക്തം നല്‍കുന്ന പാഠം. ഈ സൂക്തത്തിലെ പരാമര്‍ശവുമായി ഉദ്ധരിക്കപ്പെട്ട മറ്റു രണ്ട് സൂക്തങ്ങള്‍ക്കും ബന്ധമൊന്നുമില്ല. മൂന്നുസൂക്തങ്ങളിലും വിശദീകരിക്കപ്പെട്ടിരിക്കുന്നത് മൂന്നു തരം ദിവസങ്ങള്‍, അവയുടെ കാലദൈര്‍ഘ്യം വ്യത്യസ്തമായിരിക്കാം. അവയിലെ സംഭവങ്ങളും വ്യത്യസ്തമാണ്. അതുകൊണ്ട് തന്നെ അവ തമ്മില്‍ വൈരുധ്യമുണ്ടെന്ന് പറയാനാവില്ല. അവ തമ്മില്‍ യാതൊരു വൈരുധ്യവും ഇല്ല തന്നെ!
ലൂത്ത് നബിയുടെ സമുദായത്തെ അല്ലാഹു നശിപ്പിച്ചപ്പോൾ ലൂത്ത് നബിയെയും അദ്ദേഹത്തിന്റെ കുടുംബത്തെയും അല്ലാഹു രക്ഷിച്ചുവെന്ന് ഖുർആനിൽ പലയിടത്തും പറയുന്നുണ്ട്. ഇതിൽ 26:171 ഒരു കിഴവി ഒഴികെയുള്ള കുടുംബക്കാരെയെന്നും 7:83 ൽ ഭാര്യ ഒഴികെയുള്ള കുടുംബക്കാരെയെന്നുമാണ് പറഞ്ഞിരിക്കുന്നത്. ഇത് വൈരുധ്യമല്ലേ? സൂക്തങ്ങളില്‍ ഒരേ വ്യക്തിയെ കുറിച്ചു തന്നെയാണ് സൂചിപ്പിച്ചിരിക്കുന്നത്. ലൂത്ത് നബിയുടെ വൃദ്ധയായ ഭാര്യ ഒഴികെയുള്ള കുടുംബക്കാരെയെല്ലാം ദൈവിക ശിക്ഷയില്‍ നിന്ന് അല്ലാഹു രക്ഷിച്ചുവെന്ന വസ്തുത തന്നെയാണ് ഈ രണ്ട് സൂക്തങ്ങളിലുമുള്ളത്. വൃദ്ധയായിരുന്നിട്ടും ലൂത്ത് നബിയുടെ സമുദായം സ്വീകരിച്ചിരുന്ന സ്വവര്‍ഗരതിയെന്ന ദുര്‍വൃത്തിക്ക് കൂട്ടുനിന്ന ഭാര്യയെ സൂചിപ്പിക്കുവാന്‍ വേണ്ടിയാണ് ഖുർആന്‍ കിഴവി എന്ന് വിളിച്ചിരിക്കുന്നത്. പ്രവാചകപത്‌നിയായിരുന്നിട്ടും ധര്‍മ്മത്തിന്റെ പാതയിലേക്ക് കടന്നുവരാന്‍ കഴിയാതിരുന്ന അവരെ സത്യനിഷേധികള്‍ക്കുള്ളഉദാഹരണമായി 66:10ല്‍ എടുത്തു കാണിച്ചിട്ടുമുണ്ട്.

ക്വുർആനിന് മുമ്പുള്ള വേദഗ്രൻഥങ്ങളിലെല്ലാം മനുഷ്യരുടെ കൈകടത്തലുകൾ നടന്നുവെന്ന് മുസ്‌ലിംകൾ വിശ്വസിക്കുന്നു. സ്വാർത്ഥികളായ മനുഷ്യരുടെ പ്രവർത്തനങ്ങളെപ്പറ്റി ശരിക്കും അറിയാവുന്ന ദൈവത്തിന് എന്തേ അത്തരം കൈകടത്തലുകൾ നിയന്ത്രിച്ച്കൂടായിരുന്നോ? എന്ത് കൊണ്ട് അല്ലാഹു ആ വേദഗ്രൻഥങ്ങളെ ക്വുർആനിനെ പോലെ സംരക്ഷിച്ചില്ല?

അൻവർ ഹുസൈൻ കെ.വി

ക്വുർആനിന് മുൻപ് അവതരിപ്പിക്കപ്പെട്ട വേദഗ്രൻഥങ്ങളിൽ കൈകടത്തലുകൾ നടന്നുവെന്നത് മുസ്‌ലിംകളുടെ കേവല വിശ്വാസമല്ല. തെളിയിക്കപ്പെട്ട ഒരുയാഥാർഥ്യമാണ്. മോശെ പ്രവാചകന് അവതരിപ്പിക്കപ്പെട്ട തോറയോ ദാവീദിന്റെ സങ്കീർത്തനങ്ങളോ യേശുപഠിപ്പിച്ച ദൈവത്തിന്റെ സുവിശേഷമോ ഇന്ന് മാറ്റങ്ങളൊന്നുമില്ലാതെ ലഭ്യമാണെന്ന് അവയെ ദൈവികമെന്ന് കരുതി ആദരിക്കുന്നവർതന്നെ കരുതുന്നില്ല. അവയിൽ ദൈവവചനങ്ങളും മാനുഷികവചനങ്ങളുമുണ്ട്., പ്രവാചകന്മാര്‍ക്ക് അവതരിപ്പിക്കപ്പെട്ടിട്ടുള്ള ദൈവവചനങ്ങളിൽ ചിലവ അവയിൽ ഉദ്ധരിക്കപ്പെട്ടിട്ടുണ്ട്. അതോടൊപ്പം മനുഷ്യരുടെ വചനങ്ങളും അവയിലുണ്ട്. ഏതാണ് ദൈവികവചനം, ഏതാണ് മാനുഷികവചനമെന്ന് മനസ്സിലാക്കാന്‍ സാധ്യമല്ലാത്ത അവസ്ഥയിലാണ് ഇന്ന് അവ സ്ഥിതി ചെയ്യുന്നത്.

വേദഗ്രന്ഥങ്ങളുടെ അവതരണത്തിനുശേഷം നൂറ്റാണ്ടുകള്‍ കഴിഞ്ഞപ്പോള്‍ വേദഗ്രന്ഥങ്ങളുടെ പുറംചട്ടകളണിഞ്ഞു കൊണ്ട് മനുഷ്യരാല്‍ രചിക്കപ്പെട്ട ചില ഗ്രന്ഥങ്ങള്‍ വന്നു എന്നും ആ ഗ്രന്ഥങ്ങളാണ് മനുഷ്യരെ തിന്മയിലേക്കും തെറ്റുകളിലേക്കും അധര്‍മത്തിലേക്കും പൈശാചിക പ്രലോഭനങ്ങളിലേക്കുമെല്ലാം കൊണ്ടുപോയത് എന്നുമാണ് പരിശുദ്ധ ക്വുര്‍ആന്‍ പഠിപ്പിക്കുന്നത്. ''സ്വന്തം കൈകള്‍ കൊണ്ട് ഗ്രന്ഥം എഴുതിയുണ്ടാക്കുകയും സ്വാര്‍ഥമായ താല്‍പര്യങ്ങള്‍ക്കുവേണ്ടിയും തുച്ഛമായ താല്‍പര്യങ്ങള്‍ക്കുവേണ്ടിയും അത് ദൈവികമാണെന്ന് പറയുകയും ചെയ്യുന്നവര്‍ക്കാകുന്നു നാശം' (ക്വുര്‍ആന്‍ 2:79)

ക്വുർആനിന് മുമ്പ് അവതരിച്ച വേദങ്ങളെ എന്തു കൊണ്ട് പടച്ചവൻ സംരക്ഷിച്ചില്ല എന്ന ചോദ്യത്തിനുള്ള ഉത്തരം അവയുടെ ദൗത്യം അത്ര മാത്രമേ ഉണ്ടായിരുന്നുള്ളൂവെന്നാണ്. ക്വുർആനിന് മുമ്പ് അവതരിപിപ്പിക്കപ്പെട്ട വേദഗ്രന്ഥങ്ങളുടെയെല്ലാം ദൗത്യം ആ സമൂഹങ്ങളെ സത്യമാർഗത്തിലൂടെ വഴി നടത്തുകമാത്രമായിരുന്നു. അത് കൊണ്ട് തന്നെ അവയുടെസംരക്ഷണത്തിന്റെ ഉത്തരവാദിത്തം ആ സമൂഹങ്ങളെത്തന്നെ ഏൽപ്പിക്കുകയാണ് പ്രവാചകൻമാർ ചെയ്തത്. ആ ഉത്തരവാദിത്തം നിർവഹിക്കുന്നതിൽ അവർ പരാജയപ്പെട്ടു. വേദഗ്രൻഥങ്ങളിൽ അവർ കൈകടത്തലുകൾ നടത്തി. ഇക്കാര്യം ക്വുർആൻ വ്യക്തമാക്കുന്നുണ്ട്. "വേദഗ്രന്ഥത്തിലെ വാചകശൈലികള്‍ വളച്ചൊടിക്കുന്നചിലരും അവരുടെ കൂട്ടത്തിലുണ്ട്‌. അത്‌ വേദഗ്രന്ഥത്തില്‍ പെട്ടതാണെന്ന്‌ നിങ്ങള്‍ ധരിക്കുവാന്‍ വേണ്ടിയാണത്‌. അത്‌ വേദഗ്രന്ഥത്തിലുള്ളതല്ല. അവര്‍ പറയും; അത്‌ അല്ലാഹുവിന്‍റെ പക്കല്‍ നിന്നുള്ളതാണെന്ന്‌. എന്നാല്‍അത്‌ അല്ലാഹുവിങ്കല്‍ നിന്നുള്ളതല്ല. അവര്‍ അറിഞ്ഞുകൊണ്ട്‌ അല്ലാഹുവിന്‍റെ പേരില്‍ കള്ളംപറയുകയാണ്‌." (3 :78)

മുമ്പുള്ള വേദഗ്രൻഥങ്ങളെ സംരക്ഷിക്കുവാൻ പടച്ചവന് കഴിയുമായിരുന്നില്ലേ എന്ന ചോദ്യത്തിനുത്തരം 'തീർച്ചയായും കഴിയുമായിരുന്നു' എന്ന് തന്നെയാണ്. എന്നാൽ പരിമിതമായ ദൗത്യം മാത്രമുള്ള അവയെ സംരക്ഷിക്കേണ്ട ഉത്തരവാദിത്തം അവ അവതരിക്കപ്പെട്ട സമൂഹത്തെ ഏൽപ്പിക്കുകയാണ് ചെയ്തത്. അതിൽ അവർ വീഴ്ച വരുത്തി. സ്വാതന്ത്ര്യം നൽകപ്പെട്ട മറ്റു കാര്യങ്ങളിലൊന്നും പടച്ചവൻ നേരിട്ട് ഇടപെടാത്തതു പോലെത്തന്നെ ഇക്കാര്യത്തിലും അവൻ ഇടപെട്ടില്ല. അവയുടെ ദൗത്യം പരിമിതമായതുകൊണ്ടായിരിക്കാം ഇത്.

ക്വുർആനിന്റെ സ്ഥിതി ഇതിൽ നിന്ന് ഭിന്നമാണ്. അന്തിമപ്രവാചകന്റെ ദൗത്യം അവസാന നാളു  വരെയുള്ള മുഴുവൻ മനുഷ്യരെയും നന്മയിലൂടെ നയിക്കുകയാണ്. "നിന്നെ നാം മനുഷ്യര്‍ക്കാകമാനം സന്തോഷവാര്‍ത്ത അറിയിക്കുവാനും താക്കീത്‌ നല്‍കുവാനും ആയികൊണ്ട്‌ തന്നെയാണ്‌ അയച്ചിട്ടുള്ളത്‌." (ക്വുർആൻ 34 :28) അദ്ദേഹത്തിലൂടെ അവതരിപ്പിക്കപ്പെട്ട ക്വുർആനും മുഴുവൻ ലോകർക്കുമുള്ളതാണ്: "ഇത്‌ ലോകര്‍ക്കുള്ള ഒരു ഉല്‍ബോധനം മാത്രമാകുന്നു." (ക്വുർആൻ 38:87) അവസാന നാളുവരെ മാറ്റമൊന്നുമില്ലാതെ നിലനിൽക്കേണ്ട ഗ്രൻഥമാണ് ക്വുർആൻ എന്നതു കൊണ്ട്തന്നെ അതിന്റെ സമ്പൂർണമായ സംരക്ഷണം അല്ലാഹുതന്നെ ഏറ്റെടുത്തതായി ക്വുർആൻ വ്യക്തമാക്കുന്നു. "തീര്‍ച്ചയായും നാമാണ്‌ ആ ഉല്‍ബോധനം അവതരിപ്പിച്ചത്‌. തീര്‍ച്ചയായും നാം അതിനെ കാത്തുസൂക്ഷിക്കുന്നതുമാണ്‌" (15:9) എന്ന അല്ലാഹുവിന്റെ വാഗ്ദാനം പൂർണമായും പാലിക്കപ്പെട്ടിട്ടുണ്ട്. യാതൊരു മാറ്റത്തിരുത്തലുകളുമില്ലാതെ നില നിൽക്കുന്ന ഏകവേദഗ്രൻഥമാണ് ക്വുർആൻ. അത് അങ്ങനെത്തന്നെ ലോകാവസാനം വരെ നില നിൽക്കുകയും ചെയ്യും.

'ഹൃദയം കൊണ്ട് ചിന്തിക്കുന്നില്ലേ' എന്ന് പല തവണ ചോദിക്കുന്ന ക്വുർആൻ വ്യക്തമായ അശാസ്ത്രീയതയല്ലേ പറയുന്നത്? യഥാർത്ഥത്തിൽ ചിന്തയും ഹൃദയവും തമ്മിൽ യാതൊരു ബന്ധവുമില്ലെന്നും മസ്തിഷ്‌കം മാത്രമാണ് ചിന്തയുടെ കേന്ദ്രമെന്നും പഠിപ്പിക്കുന്ന ആധുനിക വൈദ്യശാസ്ത്രത്തിന്റെ മുന്നിൽ ഈ ക്വുർആനിക പരാമർശങ്ങളെ എങ്ങനെ ന്യായീകരിക്കുവാൻ കഴിയും?

അബ്ദുൽ ലത്തീഫ് അഹ്മദ്

 

ഹൃദയത്തെക്കുറിച്ച് ക്വുർആൻ 110 തവണയെങ്കിലും പറയുന്നുണ്ട്. ചിന്തിക്കുവാനുള്ള കഴിവിനെ ഹൃദയവുമായി ബന്ധപ്പെടുത്തിയാണ് ക്വുർആൻ എല്ലായ്പ്പോഴും പരാമർശിക്കുന്നത്. ഏതാനും വചനങ്ങൾ നോക്കുക:

"ഇവര്‍ ഭൂമിയിലൂടെ സഞ്ചരിക്കുന്നില്ലേ? എങ്കില്‍ ചിന്തിച്ച്‌ മനസ്സിലാക്കാനുതകുന്ന ഹൃദയങ്ങളോ, കേട്ടറിയാനുതകുന്ന കാതുകളോ അവര്‍ക്കുണ്ടാകുമായിരുന്നു. തീര്‍ച്ചയായും കണ്ണുകളെയല്ല അന്ധത ബാധിക്കുന്നത്‌. പക്ഷെ, നെഞ്ചുകളിലുള്ള ഹൃദയങ്ങളെയാണ്‌ അന്ധത ബാധിക്കുന്നത്‌" (22: 46)

"നീ പറയുന്നത്‌ ശ്രദ്ധിച്ച്‌ കേള്‍ക്കുന്ന ചിലരും അവരുടെ കൂട്ടത്തിലുണ്ട്‌. എന്നാല്‍ അത്‌ അവര്‍ ഗ്രഹിക്കാത്ത വിധം അവരുടെ ഹൃദയങ്ങളിന്‍മേല്‍ നാം മൂടികള്‍ ഇടുകയും, അവരുടെ കാതുകളില്‍ അടപ്പ്‌ വെക്കുകയും ചെയ്തിരിക്കുന്നു. എന്തെല്ലാം ദൃഷ്ടാന്തങ്ങള്‍ കണ്ടാലും അവരതില്‍ വിശ്വസിക്കുകയില്ല. അങ്ങനെ അവര്‍ നിന്‍റെ അടുക്കല്‍ നിന്നോട്‌ തര്‍ക്കിക്കുവാനായി വന്നാല്‍ ആ സത്യനിഷേധികള്‍ പറയും; ഇത്‌ പൂര്‍വ്വികന്‍മാരുടെ കെട്ടുകഥകളല്ലാതെ മറ്റൊന്നുമല്ല എന്ന്‌." (6: 25)

"അല്ലാഹുവെപ്പറ്റി പറയപ്പെട്ടാല്‍ ഹൃദയങ്ങള്‍ പേടിച്ച്‌ നടുങ്ങുകയും, അവന്‍റെ ദൃഷ്ടാന്തങ്ങള്‍ വായിച്ചുകേള്‍പിക്കപ്പെട്ടാല്‍ വിശ്വാസം വര്‍ദ്ധിക്കുകയും, തങ്ങളുടെ രക്ഷിതാവിന്‍റെ മേല്‍ ഭരമേല്‍പിക്കുകയും ചെയ്യുന്നവര്‍ മാത്രമാണ്‌ സത്യവിശ്വാസികള്‍." (8: 2)

സ്വഹീഹായ നിരവധി ഹദീഥുകളിലും ചിന്തയെയും സന്മാർഗ - ദുർമാർഗങ്ങളുടെ സ്വീകരണത്തെയുമെല്ലാം ഹൃദയവുമായി ബന്ധപ്പെടുത്തി പരാമർശിച്ചത് കാണാൻ കഴിയും.

ശരീരത്തിന്റെ മൊത്തത്തിലുള്ള നിയന്ത്രണം നിർവഹിക്കുന്നത് മസ്തിഷ്കമാണെന്ന് ശരീരശാസ്ത്രം പഠിച്ചിട്ടുള്ളവർക്കറിയാം. കേന്ദ്രനാഡി വ്യവസ്ഥയാണ് ശരീരത്തെ മൊത്തത്തിൽ നിയന്ത്രിക്കുന്നതെങ്കിലും ഹൃദയത്തിന് അതിന്റേതായ ഒരു നിയന്ത്രണവ്യവസ്ഥയുണ്ട്. ഹൃദയനാഡീവ്യവസ്ഥയെന്നാണ് (cardiac nervous system) അതിനെ ശാസ്ത്രജ്ഞർ വിളിക്കാറുള്ളത്. സ്വന്തമായ നാഡികളും(neurons) നാഡീപ്രസാരകരും (neurotransmitters) പ്രോട്ടീനുകളും മറ്റു അനുബന്ധകോശങ്ങളുമുള്ള സ്വതന്ത്രമായ നാഡീവ്യവസ്ഥയാണിത്. ഈ നാഡീവ്യവസ്ഥക്ക് ഹൃദയമസ്തിഷ്‌കം (heart brain) എന്ന പേര് നൽകിയത് 1991ൽ പ്രസിദ്ധ കനേഡിയൻ നാഡീശാസ്ത്രജ്ഞനായ ഡോ: ആൻഡ്രു ആർമറാണ്. ഹൃദയവും തലച്ചോറും തമ്മിലുള്ള ബന്ധത്തെയും ഹൃദയത്തിന്റെ സ്വയംഭരണത്തെയും കുറിച്ച ഒരു പഠനശാഖ തന്നെ ഇന്നുണ്ട്. ഹൃദയനാഡീശാസ്ത്രം (neurocardiology) എന്നാണ് അതിന് പേര്. ഇവിഷയകമായ പഠനങ്ങളെ സമാഹരിച്ച് കൊണ്ട് ഡോ: ആൻഡ്രു ആർമറും ജെഫ്‌റി എൽ ആർഡറും കൂടി 1994 ൽ രചിച്ച പുസ്തകത്തിന്റെയും തലക്കെട്ട് 'ഹൃദയനാഡീശാസ്ത്രം' എന്ന് തന്നെയാണ്.

ഹൃദയം ചുരുങ്ങിയത് നാല് രൂപത്തിലെങ്കിലും മസ്തിഷ്കവുമായി ആശയവിനിമയം നടത്തുന്നുണ്ടെന്നാണ് ഹൃദയനാഡീശാസ്ത്രം പറയുന്നത്. നാഡീആവേഗങ്ങളിലൂടെയുള്ള (nerve impulses) നാഡീയവും ഹോർമോണുകളിലൂടെയും നാഡീപ്രസാരകരിലൂടെയുമുള്ള ജൈവരസതന്ത്രപരവും സമ്മർദ്ദതരംഗങ്ങളിലൂടെയുള്ള (pressure waves) ജൈവഭൗതികവും വിദ്യുത്കാന്തികക്ഷേത്രത്തിന്റെ (electromagnetic field) വ്യവഹാരങ്ങളിലൂടെയുള്ള ഉർജ്ജപരവുമായ ആശയക്കൈമാറ്റങ്ങൾ. ഈ ആശയക്കൈമാറ്റങ്ങളിലൂടെയാണ് ഹൃദയത്തിന് സ്വയംഭരണം സാധിക്കുന്നത്. ഈ സ്വയംഭരണത്തിൽ രക്തം പമ്പു ചെയ്യുകയെന്ന ഒരേയൊരു ധർമ്മം മാത്രമാണോ നിർവഹിക്കപ്പെടുന്നത്? ഈ ചോദ്യത്തിന് ഖണ്ഡിതമായ ഒരു ഉത്തരം നൽകാൻ ഇന്ന് നാഡീശാസ്ത്രം വളർന്നിട്ടില്ലെങ്കിലും 'അല്ല' എന്ന് പറയുന്നവരാണ് ആ രംഗത്തെ ഗവേഷകരിൽ പലരുമെന്നുള്ളതാണ് വസ്തുത.

പ്രസിദ്ധ ഡോക്യുമെന്ററി ഫിലിം നിർമ്മാതാവായ ഡേവിഡ് മാലോണിന്റെ 'ഹൃദയങ്ങളുടെയും മനസ്സുകളുടെയും' (Of Hearts and Minds) എന്ന ശാസ്ത്രഡോക്യൂമെന്ററിയിൽ ഇന്റർവ്യൂ ചെയ്ത പ്രഗത്ഭരായ പല നാഡീശാസ്ത്രവിദഗ്ധരും ഹൃദയശാസ്ത്രജ്ഞന്മാരും കരുതുന്നത് കേവലം രക്തം പമ്പു ചെയ്യുകയെന്ന ദൗത്യം മാത്രമല്ല ഹൃദയം നിർവഹിക്കുന്നത് എന്ന് തന്നെയാണ്. ഹൃദയത്തിൽ നിന്നുള്ള സിഗ്നലുകൾക്കനുസരിച്ചാണ് മസ്‌തിഷ്‌കത്തിനകത്തെ അമിഗ്ദാല ഭയം ഉത്പാദിപ്പിക്കുന്നതെന്ന കണ്ടെത്തൽ വികാരങ്ങളുടെ രൂപീകരണത്തിൽ ഹൃദയത്തിന് പങ്കുണ്ടെന്നു തന്നെയാണ് വ്യക്തമാക്കുന്നതെന്ന് അവർ അഭിപ്രായപ്പെടുന്നു. ഓക്സ്ഫോർഡ് സർവകലാശാലയിലെ പ്രൊഫസറായ ഡേവിഡ് പാറ്റേഴ്സണിന്റെ പഠനങ്ങൾ കാണിക്കുന്നത് വികാരങ്ങൾ സൃഷ്ടിക്കുന്നത് മസ്തിഷ്‌കം മാത്രമായിട്ടല്ല, ഹൃദയത്തിനു കൂടി അതിൽ പങ്കുണ്ടെന്നാണ്. ചുരുക്കത്തിൽ രക്തം പാമ്പു ചെയ്യുകയെന്ന ധർമ്മം മാത്രം നിർവഹിക്കുന്ന ഒരു ശാരീരികാവയവം മാത്രമാണ് ഹൃദയമെന്ന സങ്കൽപ്പമല്ല ഇന്ന് ശാസ്ത്രലോകത്തുള്ളത്. ചിന്തയുടെയും വികാരങ്ങളുടെയും നിർമാണത്തിൽ ഹൃദയം എങ്ങനെയൊക്കെയോ പങ്കു വഹിക്കുന്നുണ്ട്. എങ്ങനെയൊക്കെയാണെന്ന് പറയാൻ മാത്രം ഇന്ന് ശാസ്ത്രം വളർന്നിട്ടില്ലെന്ന് മാത്രമേയുള്ളൂ.

ഹൃദയനാഡീശാസ്ത്രം കൂടുതൽ വളരുമ്പോൾ, ക്വുർആനും ഹദീഥുകളും അർത്ഥശങ്കക്കിടയില്ലാത്തവണ്ണം വ്യക്തമാക്കുന്നത് പോലെ ഹൃദയം തന്നെയാണ് ചിന്തയുടെയും സത്യാസത്യവിവേചനത്തിന്റെയും കേന്ദ്രമെന്ന വസ്തുത ശാസ്ത്രലോകവും അംഗീകരിക്കുമെന്ന് നമുക്ക് പ്രത്യാശിക്കാം.

'ലോകത്തിന്റെ വ്യത്യസ്ത ഭാഗങ്ങളിൽ പാരായണം ചെയ്യപ്പെടുന്ന ഖുർആനുകൾ തമ്മിൽ വലിയ വ്യത്യാസങ്ങളുണ്ട്. മൊറോക്കോയിലെ ഖുർആൻ അല്ല സൗദി അറേബിയയിലേത്. മുഹമ്മദ് നബി പഠിപ്പിച്ച ഖുർആൻ പുള്ളിയും കുത്തുകളുമില്ലാതെയാണ് ഏഴുതിയത്. ആ ഖുർആൻ വ്യത്യസ്ത ദേശത്തുള്ളവർ ഇസ്‌ലാമിലേക്ക് വന്നപ്പോൾ അവർ അവർക്ക് തോന്നിയതനുസരിച്ച് ഓതിയതിനാലാണ് ഈ വ്യത്യാസമുണ്ടായത്. ഖുർആനിൽ മാറ്റങ്ങളൊന്നുമുണ്ടായിട്ടില്ലെന്ന മുസ്‌ലിംകളുടെ വാദം കള്ളമാണെന്നാണ് ഇത് കാണിക്കുന്നത്.'ഒരു യുക്തിവാദിയുടെ പ്രസംഗത്തിൽ നിന്ന്.എന്താണ് മറുപടി ?

ജബ്ബാർ, കുന്നുമ്മൽ, മലപ്പുറം.

ഖുർആൻ പാരായങ്ങൾ തമ്മിലുള്ള വ്യത്യാസം യുക്തിവാദികളോ ഇസ്‌ലാം വിരോധികളോ ഗവേഷണം ചെയ്തു കണ്ട് പിടിച്ചതല്ല. വ്യത്യസ്ത ഖിറാഅത്തുകളിലെ വ്യത്യസ്ത പാരായണ രീതികളെക്കുറിച്ച് വിവരിക്കുന്ന നിരവധി ഗ്രന്ഥങ്ങൾ മുസ്‌ലിം ലോകത്തുണ്ട്. പാരായണ വ്യത്യാസങ്ങളെക്കുറിച്ച അറിവ് സാധാരണക്കാരന് ആവശ്യമില്ലാത്തതു കൊണ്ടാണ് പണ്ഡിതന്മാർ അത് എല്ലാവരെയും പഠിപ്പിക്കാത്തത്. സാധാരണക്കാരുടെ അറിവില്ലായ്മ മുതലെടുത്തതുകൊണ്ട് ഖുർആനിന്റെ അഖണ്ഡതയിൽ സംശയം ജനിപ്പിക്കാനാണ് പാരായണ വ്യത്യാസത്തെ ഇസ്‌ലാമിന്റെ ശത്രുക്കൾ ചർച്ചക്കെടുക്കുന്നത്. ഖുർആനിന്റെ പാരായണ വ്യത്യാസങ്ങൾ അതിന്റെ ദൈവികതയെ ഉജ്ജ്വലമായി വെളുപ്പെടുത്തുന്നവയാണെന്ന് അൽപം ചിന്തിച്ചാൽ ബോധ്യപ്പെടും.

മനുഷ്യസമൂഹത്തിന് മുന്നിൽ പാരായണം ചെയ്തു കേൾപ്പിച്ച മുഹമ്മദ് നബി (സ) തന്നെ ഖുർആൻ ഏഴ് ശൈലികളിൽ അവതരിക്കപ്പെട്ടതാണെന്ന് പഠിപ്പിച്ചിട്ടുണ്ട്. അല്ലാഹു ജിബ്‌രീലിലൂടെ (അ) നബി (സ)ക്ക് ഏഴു ശൈലികളിൽ (ഹർഫുകൾ) ഖുർആൻ അവതരിപ്പിച്ചതായി സ്വഹീഹായ നിരവധി ഹദീഥുകളാൽ സ്ഥിരീകരിക്കപ്പെട്ടിട്ടുണ്ട്. ഏഴ് വ്യത്യസ്ത ശൈലികളിൽ ഖുർആൻ അവതരിക്കപ്പെട്ടതിനാൽ തന്നെ പലരും പാരായണം ചെയ്തിരുന്നത് പലശൈലികളിലായിരുന്നുവെന്ന് കാണാൻ കഴിയും. ഏഴു ഹർഫുകളിലായാണ് അവസാനത്തെ ദൈവിക ഗ്രന്ഥം അവതരിക്കപ്പെട്ടത് എന്നറിയാതെ ചില സ്വഹാബിമാർ തമ്മിൽ ഇവ്വിഷയകമായി നടന്ന തർക്കങ്ങളെക്കുറിച്ച വിവരണങ്ങളിൽ നിന്ന് ഇവയെല്ലാം അല്ലാഹുവിൽ നിന്ന് അവതരിക്കപ്പെട്ടതാണെന്നും അവന്റെ നിർദ്ദേശമാണ് ഇവയിലെല്ലാം ഖുർആൻ പാരായണം ചെയ്യാമെന്നും ഇവയിലേതിലെങ്കിലും ഒന്നിൽ പാരായണം ചെയ്താൽ മതിയെന്നും ഒന്ന് മറ്റേതിൽ നിന്ന് ഉത്തമമോ അധമമോ അല്ലെന്നുമുള്ള വസ്തുതകൾ മനസ്സിലാവും.

ഉമറുബ്നുൽ ഖത്ത്വാബിൽ (റ) നിന്ന്: 'റസൂലിന്റെ കാലത്ത് ഹിശാമുബ്നു ഹകീം ഒരിക്കൽ 'സൂറത്തുൽ ഫുർഖാൻ' ഓതുന്നത് ഞാൻ കേട്ടു. ഞാൻ അദ്ദേഹത്തിന്റെ പാരായണം ശ്രദ്ധിച്ചു. എനിക്ക് റസൂൽ ഓതിത്തന്നിട്ടില്ലാത്ത പലവിധ ശൈലികളിലും അദ്ദേഹം ഓതുന്നു. നമസ്കാരത്തിലായിരിക്കെത്തന്നെ, അദ്ദേഹവുമായി വഴക്കിടാൻ എനിക്ക് തോന്നി. നമസ്കാരം കഴിയുംവരെ ഞാൻ ക്ഷമിച്ചു. നമസ്കാരത്തിൽ നിന്ന് വിരമിച്ചയുടനെ, അദ്ദേഹത്തിന്റെ തട്ടം കഴുത്തിന് ചുറ്റിപ്പിടിച്ച് ഞാൻ ചോദിച്ചു: 'നിങ്ങളിപ്പോൾ ഓതുന്നതായി ഞാൻ കേട്ട സൂറത്ത് നിങ്ങൾക്കാരാണ് ഓതിത്തന്നത്?' അദ്ദേഹം പറഞ്ഞു: 'അല്ലാഹുവിന്റെ റസൂലാണ് എന്നെയത് ഓതിപ്പഠിപ്പിച്ചത്.' ഞാൻ പറഞ്ഞു: ''കള്ളം. റസൂൽ എനിക്ക് പഠിപ്പിച്ചുതന്നത് നിങ്ങൾ ഓതിയ രൂപത്തിലല്ല.' അദ്ദേഹത്തെയും പിടിച്ച് ഞാൻ റസൂലിന്റെ അടുത്തേക്ക്പുറപ്പെട്ടു. ഞാൻ റസൂലിനോട് പറഞ്ഞു: 'നിങ്ങൾ എനിക്ക് ഓതിത്തരാത്ത വിധം സൂറത്തുൽ ഫുർഖാൻ ഇദ്ദേഹം ഓതുന്നത് ഞാൻ കേട്ടു.' റസൂൽ പറഞ്ഞു: 'അദ്ദേഹത്തെ വിട്ടേക്കൂ. ഹിശാം, നിങ്ങൾ ഓതൂ.' ഹിശാം ഞാൻ കേട്ട അതേപ്രകാരം തന്നെ ഓതി. അപ്പോൾ റസൂൽ പറഞ്ഞു: 'ഇപ്രകാരം തന്നെയാണ് ഇത് അവതരിപ്പിക്കപ്പെട്ടത്.' തുടർന്ന് അവിടുന്ന് പറഞ്ഞു: 'ഉമറേ, നിങ്ങളൊന്ന് ഓതൂ.' റസൂൽ എന്നെ പഠിപ്പിച്ച പോലെ ഞാൻ ഓതി. അപ്പോൾ റസൂൽ പറഞ്ഞു: 'ഇങ്ങനെയും ഇത്  അതരിപ്പിക്കപ്പെട്ടിട്ടുണ്ട്. ഈ ഖുർആൻ ഏഴ് വ്യത്യസ്ത ശൈലികളിൽ (ഹർഫുകൾ) അവതരിപ്പിക്കപ്പെട്ടിട്ടുണ്ട്. നിങ്ങൾക്ക് എളുപ്പമായ വിധം അത് ഓതിക്കൊള്ളുക.' (സ്വഹീഹുൽ ബുഖാരി, കിതാബു ഫദാഇലിൽ ഖുർആൻ; സ്വഹീഹു മുസ്‌ലിം, കിതാബു ഫദാഇലിൽ ഖുർആൻ വമായത അല്ലഖു ബിഹി)

ഇതേ പോലെ നിരവധി ഹദീഥുകൾ വ്യത്യസ്ത ഹദീഥ് ഗ്രന്ഥങ്ങളിൽ നിവേദനം ചെയ്യപ്പെട്ടിട്ടുണ്ട്. മുഹമ്മദ് നബി (സ) യുടെ ആവശ്യപ്രകാരം അല്ലാഹു അവതരിപ്പിച്ചതാണ് ഖുർആനിന്റെ ഈ ഏഴ് ശൈലികളുമെന്ന് നബി (സ) തന്നെ വ്യക്തമാക്കിയതായും സ്വഹീഹായ ഹദീഥുകളിലുണ്ട്.

എഴുത്ത് വ്യാപകമായി നിലനിന്നിരുന്നിട്ടില്ലാത്ത കാലത്ത്, വ്യത്യസ്ത നിലവാരത്തിലുള്ളവർക്ക് ഒരേ ശൈലിയിൽ പാരായണം പ്രയാസകരമാണെന്നതിനാൽ അല്ലാഹുതന്നെ അവതരിപ്പിച്ച ഏഴ് ഹർഫുകളിലായുള്ള ഖുർആൻ പാരായണം നബി (സ) യുടെ കാലത്ത് തന്നെ നിലനിന്നിരുന്നുവെന്ന വസ്തുത മനസ്സിലാക്കാത്തതുകൊണ്ടാണ് വ്യത്യസ്ത ശൈലികളിലുള്ള ഖുർആനുകൾ തമ്മിൽ വ്യത്യാസമുണ്ടെന്നും അത് പിൽക്കാലത്ത് പകർത്തിയെഴുതിയപ്പോൾ സംഭവിച്ച പിഴവാണെന്നും വിമർശകന്മാർ ആരോപിക്കുന്നത്. അല്ലാഹു അവതരിപ്പിച്ച ഏഴ് ശൈലികളിലുമുള്ള ഖുർആൻ പാരായണം നബി (സ) തന്റെ അനുയായികളെ പഠിപ്പിച്ചിരുന്നുവെന്ന വസ്തുത നടേ ഉദ്ധരിച്ച നിവേദനങ്ങൾ വ്യക്തമാക്കുന്നു. നബി (സ) പഠിപ്പിച്ച ഏഴ് ഹർഫുകളിലുള്ള ഖുർആൻ വചനങ്ങൾ തമ്മിൽ ചില പാരായണ വ്യത്യാസങ്ങളുണ്ട്. ഈപാരായണ വ്യത്യാസങ്ങളിൽ ചിലവ അർത്ഥ വ്യത്യാസങ്ങളുമുള്ളവയാണ്. നിസ്സാരവും വൈരുധ്യങ്ങളൊന്നുമില്ലാത്തതുമായ ഈ അർത്ഥ വ്യത്യാസങ്ങൾ പോലും ഖുർആനിന്റെ അമാനുഷികതയെ സ്ഥിരീകരിക്കുന്നതാണ് എന്നതാണ് അത്ഭുതം.

സൂറത്തുൽ ഫാത്തിഹയിലെ ഒരു പാരായണ വ്യത്യാസം ഉദാഹരണമായെടുക്കുക. മൂന്നാമത്തെ ആയത്തിന് 'മാലിക്കി യൗമിദ്ദീൻ' എന്നും 'മലിക്കി യൗമിദ്ധീൻ' എന്നും രണ്ട് പാരായണകളുണ്ട്. പ്രതിഫലനാളിന്റെ ഉടമസ്ഥൻ എന്നാണ് ഒന്നാമത്തെ പാരായണത്തിന്റെ അർഥം. 'പ്രതിഫലനാളിന്റെ രാജാവ്' എന്ന് രണ്ടാമത്തെ പാരായണത്തിന്റെയും. അല്ലാഹുവാണ് പ്രതിഫലനാളിന്റെ രാജാവും ഉടമസ്ഥനും. അത് കൊണ്ട് തന്നെ രണ്ട് പാരായങ്ങളും തമ്മിൽ യാതൊരു വൈരുധ്യവുമില്ല. രണ്ടും നബി (സ) പഠിപ്പിച്ചതും അക്കാലം മുതൽ ഇന്ന് വരെ നിരവധി പരമ്പരകളിലൂടെ നിവേദനം ചെയ്യപ്പെട്ടവയുമാണ്. ഒരു അർഥം മറ്റേ അർത്ഥത്തിന് ഉപോൽബലകമാണെന്നർത്ഥം. ഇതേപോലെയുള്ളതാണ് വ്യത്യസ്ത ഖിറാഅത്തുകളിലുള്ള പാരായണവ്യത്യാസങ്ങൾ. അവയെല്ലാം നബി പഠിപ്പിച്ചതാണ്. ആരും യാതൊന്നും ഖുർആനിൽ കൂട്ടിച്ചേർക്കുകയോ എടുത്ത് മാറ്റുകയോ ചെയ്തിട്ടില്ല.

ഏഴ് ശൈലികളിൽ അവതരിക്കപ്പെട്ടിട്ടും ഖുർആനിൽ യാതൊരു വൈരുധ്യവുമില്ലെന്നത് അത്ഭുതകരമാണ്. ''അവർ ഖുർആനിനെപ്പറ്റി ചിന്തിക്കുന്നില്ലേ? അത് അല്ലാഹു അല്ലാത്തവരുടെ പക്കൽ നിന്നുള്ളതായിരുന്നെങ്കിൽ അവരതിൽ ധാരാളം വൈരുധ്യം കണ്ടെത്തുമായിരുന്നു.''(4:82) വെന്ന ഖുർആൻ” വചനത്തിലെ പരാമർശം ഏഴ് ഹർഫുകൾക്കും ഒരേപോലെ ബാധകമാണ്. ഒരേ ഹർഫിലുള്ള ഖുർആനിലെ വചനങ്ങൾ തമ്മിലോ വ്യത്യസ്ത ഹർഫുകൾ തമ്മിലോ വൈരുധ്യങ്ങളൊന്നുമില്ല. വ്യത്യസ്ത നിലവാരത്തിലുള്ളവരെ പരിഗണിച്ചുകൊണ്ട് വ്യത്യസ്ത ശൈലികളിൽ അവതരിക്കപ്പെട്ടിട്ടുപോലും ഖുർആനിൽ വൈരുധ്യങ്ങളൊന്നുമില്ലെന്ന അത്ഭുതകരമായ വസ്തുത അതിന്റെ ദൈവികത വ്യക്തമാക്കുന്ന പല തെളിവുകളിലൊന്നാണ്. ഖുർആനിനെതിരെ ഉന്നയിക്കപ്പെട്ട ഒരു വിമർശനം ഖുർആനിന്റെ സത്യതയെ വെളിപ്പെടുത്തുന്നതാണ് നാം ഇവിടെ കാണുന്നത്.

വിഷയവുമായി ബന്ധപ്പെട്ട വീഡിയോ

സൂര്യനെക്കുറിച്ചും ചന്ദ്രനെക്കുറിച്ചുമൊന്നും അറബികൾക്കുണ്ടായിരുന്നതിൽ കൂടുതലായ വിവരമൊന്നും ഖുർആൻ രചയിതാവിനുണ്ടായിരുന്നില്ലെന്ന് വ്യക്തമാക്കുന്ന നിരവധി വചനങ്ങൾ ഖുർആനിലുണ്ട്. അവയിലൊന്നാണ് 36 ആം അധ്യായത്തിലെ 39 ആം വചനം. ചന്ദ്രന്‍ ഈന്തപ്പനക്കുലയുടെ വളഞ്ഞ തണ്ടു പോലെയായിത്തീരുന്നുവെന്നാണ് ഈ വചനത്തിൽ പറഞ്ഞിരിക്കുന്നത്. യഥാർത്ഥത്തിൽ ചന്ദ്രൻ അങ്ങനെ ആയിത്തീരുന്നില്ലെന്നും സൂര്യപ്രകാശം ചന്ദ്രോപരിതലത്തിൽ പതിക്കുന്നത് ഒരു പ്രത്യേക രീതിയിലായതിനാൽ ഭൂമിയിലുള്ളവർക്ക് അങ്ങനെ തോന്നുക മാത്രമാണ് ചെയ്യുന്നതെന്നുമുള്ള വസ്തുത ഇന്ന് നമുക്കറിയാം. ദൈവീകമായിരുന്നു ഖുർആനെങ്കിൽ ഇത്തരം അബദ്ധ പരാമർശങ്ങൾ ഉണ്ടാകുമായിരുന്നുവോ?

- സുമേഷ് ചന്ദ്രൻ, കരുനാഗപ്പള്ളി

ന്ദ്രന്‍ ഈന്തപ്പനക്കുലയുടെ വളഞ്ഞ തണ്ടു പോലെയായിത്തീരുന്നുവെന്ന ഖുര്‍ആന്‍ സൂറത്ത് യാസീനിലെ മുപ്പത്തിയൊമ്പതാം വചനത്തിലെ പരാമര്‍ശം അശാസ്ത്രീയമാണെന്നാണ് വിമര്‍ശനം.

വിമര്‍ശിക്കപ്പെട്ട ഖുര്‍ആന്‍ വചനം ശ്രദ്ധിക്കുക. ''ചന്ദ്രന് നാം ചില ഘട്ടങ്ങള്‍ നിശ്ചയിച്ചിരിക്കുന്നു. അങ്ങനെ അത് പഴയ ഈന്തപ്പനക്കുലയുടെ വളഞ്ഞ തണ്ടു പോലെ ആയിത്തീരുന്നു.'' (36:39)

യഥാര്‍ഥത്തില്‍ ചന്ദ്രന്‍ ഈന്തപ്പനക്കുലയുടെ വളഞ്ഞ തണ്ടു പോലെയായിത്തീരുന്നി ല്ലെന്നും അങ്ങനെ ഭൂമിയിലുള്ള മനുഷ്യര്‍ക്ക് തോന്നുകയാണ് ചെയ്യുന്നതെന്നും അതുകൊണ്ടുതന്നെ ഈ ഖുര്‍ആന്‍ വചനം അശാസ്ത്രീയമാണ് എന്നുമാണ് വാദം. ഈ ഖുര്‍ആന്‍ വചനത്തെ സൂക്ഷ്മമായി അപഗ്രഥിച്ചാല്‍ ഈ വിമര്‍ശനം അടിസ്ഥാനരഹിതമാണെന്ന് ബോധ്യപ്പെടുമെന്നുള്ളതാണ് വസ്തുത.

മനുഷ്യര്‍ക്ക് അല്ലാഹു നല്‍കിയ അനുഗ്രഹങ്ങളെയും പ്രകൃതിയിലുള്ള ദൃഷ്ടാന്തങ്ങളെയും കുറിച്ച് വിവരിക്കുന്നതിനിടയിലാണ് സൂറത്തു യാസീനില്‍ ചന്ദ്രന് അല്ലാഹു കണക്കാക്കിയ ഘട്ടങ്ങളെ ക്കുറിച്ച് പരാമര്‍ശിക്കുന്നത്. 'ചന്ദ്രന്‍' എന്ന് പരിഭാഷപ്പെടുത്തിയിരിക്കുന്നത് 'ഖമര്‍' എന്ന അറബി പദത്തെയാണ്. ചന്ദ്രൻ പ്രകാശമാണെന്നാണ് ഖുര്‍ആനിലുടനീളം പറഞ്ഞിട്ടുള്ളത്. നൂര്‍, മുനീര്‍ എന്നിങ്ങനെ ചന്ദ്രനെ വിശേഷിപ്പിക്കുവാന്‍ ഖുര്‍ആന്‍ ഉപയോഗിച്ച പദങ്ങളെല്ലാം പ്രതിഫലിപ്പിക്കപ്പെടുന്ന പ്രകാശത്തെ ദ്യോതിപ്പിക്കുന്നവയാണ്. ഖമര്‍ (ചന്ദ്രന്‍) എന്നതുകൊണ്ടുള്ള വിവക്ഷ ആകാശത്തു നിലനില്‍ക്കുന്ന ഖരഗോളമെന്നതിലുപരിയായി ഭൂമിയിൽ നിന്ന് കാണുന്നതെന്താണോ അതാണെന്ന് ഇത് വ്യക്തമാക്കുന്നു.

ഖമറിന്റെ രൂപ വ്യത്യാസങ്ങള്‍ക്കനുസരിച്ച് ഹിലാല്‍ (ചന്ദ്രക്കല), ബദ്‌ർ (പൂര്‍ണ ചന്ദ്രന്‍) എന്നിങ്ങനെയുള്ള പ്രയോഗങ്ങളും അറബിയിലുണ്ട്. ചന്ദ്രന് ഘട്ടങ്ങളെ നിര്‍ണയിച്ചതായും അത് ഈന്തപ്പന ക്കുലയുടെ വളഞ്ഞ തണ്ടുപോലെ ആയിത്തീരുന്നതുമായുള്ള ഖുര്‍ആന്‍ പരാമര്‍ശങ്ങള്‍ ഭൂമിക്ക് ആപേക്ഷികമായി നടക്കുന്ന ചന്ദ്രപ്രതിഭാസങ്ങളെ കുറിക്കുന്നവയാണ്. ഖമര്‍ തന്നെയാണ് ബദ്‌ർ; ഖമര്‍ തന്നെയാണ് ഹിലാല്‍. ''ഖമര്‍ ഹിലാലായിത്തീരുന്നു''വെന്ന പരാമര്‍ശം ഒരു സാധാരണ അറബി പ്രയോഗമാണ്. ഹിലാലിന്റെ ഉപമാലങ്കാരമാണ് ''ചന്ദ്രന്‍ പഴയ ഈന്തപ്പനയുടെ വളഞ്ഞ തണ്ടുപോലെ'' ആയിത്തീരുകയെന്നത്. ''ഖമറിന്റെ ആദ്യഘട്ടമാണ് ഹിലാല്‍'' എന്ന പരാമര്‍ശം അശാസ്ത്രീയമല്ലാത്തതു പോലെത്തന്നെ ഈ ഉപമാലങ്കാരത്തിലും യാതൊരുവിധ അശാസ്ത്രീയതകളുമില്ല. ചന്ദ്രനിലെ ഖരപദാർത്ഥം വളഞ്ഞ ഈത്തപ്പനത്തണ്ട് പോലെയാകുന്നുവെന്നോ ഓരോ ചാന്ദ്രഘട്ടത്തിലും ചന്ദ്രൻ എന്ന ഖരവസ്തുവിനാണ് മാറ്റമുണ്ടാവുന്നതെന്നോ ഈ വചനത്തിൽ നിന്ന് സ്വഹാബിമാർ മുതൽ ഇന്ന് വരെയുള്ള വിശ്വാസികളൊന്നും മനസ്സിലാക്കിയിട്ടില്ല.

സൂര്യന്‍ ചെളിവെള്ളമുള്ള ഒരു ജലാശയത്തില്‍ മറഞ്ഞുപോവുന്നതായി ദുൽഖർനൈനിന്റെ കഥ പറയുമ്പോൾ ഖുര്‍ആൻ 18:86 ല്‍ പരാർശിക്കുന്നുണ്ട്. ഭൂമിയേക്കാള്‍ ലക്ഷക്കണക്കിന് ഇരട്ടി വലിപ്പമുള്ള സൂര്യന്‍ ഒരുജലാശയത്തില്‍ ആഴ്ന്നു പോവുകയെന്നു പറയുന്നത് വ്യക്തമായും അശാസ്ത്രീയമല്ലേ? ദുൽഖർനൈൻ സൂര്യോദയസ്ഥാനത്തും അസ്തമയസ്ഥാനത്തുമെല്ലാം എത്തിയതായും 18:86ലും 18:90ലും പറയുന്നുണ്ട്. ഇതെല്ലാം സൂര്യനെയും ഭൂമിയെയും ഉദയത്തിന്റെയും അസ്തമയത്തിന്റെയും പിന്നിലുള്ള ശാസ്ത്രത്തെയുമൊന്നും അറിയാത്ത ആരോ എഴുതിയതാണ് ക്വുർആൻ എന്നല്ലേ കാണിക്കുന്നത്?

- പ്രകാശൻ കെ.പി, അത്താണിക്കൽ, വള്ളിക്കുന്ന്

ദുൽഖർനൈനിന്റെ കഥ പറയുമ്പോഴുള്ള സൂര്യന്‍ ചെളിവെള്ളമുള്ള ഒരു ജലാശയത്തില്‍ മറഞ്ഞുപോവുന്നതായും സൂര്യോദയ സ്ഥാന ത്തും അസ്തമയസ്ഥാനത്തുമെല്ലാം അദ്ദേഹം എത്തിയതായുമുള്ള പരാമർശങ്ങൾ സൂര്യനെയും ഭൂമിയെയും ഉദയത്തിന്റെയും അസ്തമ യത്തിന്റെയും പിന്നിലുള്ള ശാസ്ത്രത്തെയുമൊന്നും അറിയാത്ത ആരോഎഴുതിയതാണ്ക്വുർആൻ എന്ന് വ്യക്തമാക്കുന്നതായാണ് വിമർശനം.

വിമര്‍ശിക്കപ്പെട്ട ഖുര്‍ആൻ വാക്യങ്ങള്‍ പരിശോധിക്കുക. ”അവര്‍ നിന്നോട് ദുല്‍ഖര്‍നൈനിയെപ്പറ്റി ചോദിക്കുന്നു. നീ പറയുക: അദ്ദേഹത്തെപ്പറ്റിയുള്ള വിവരം ഞാന്‍ നിങ്ങള്‍ക്ക് ഓതികേള്‍പിച്ച് തരാം. തീര്‍ച്ചയായും നാംഅദ്ദേഹത്തിന് ഭൂമിയില്‍ സ്വാധീനം നല്‍കുകയും, എല്ലാ കാര്യത്തിനുമുള്ള മാര്‍ഗം നാം അദ്ദേഹത്തിന് സൗകര്യ പ്പെടുത്തികൊടുക്കുകയും ചെയ്തു. അങ്ങനെ അദ്ദേഹം ഒരു മാര്‍ഗം പിന്തുടര്‍ന്നു. അങ്ങനെ അദ്ദേഹംസൂര്യാസ്തമനസ്ഥാനത്തെത്തിയപ്പോള്‍ അത് ചെളിവെള്ളമുള്ള ഒരു ജലാശയത്തില്‍ മറഞ്ഞ് പോകുന്നതായി അദ്ദേഹം കണ്ടു. അതിന്റെ അടുത്ത് ഒരു ജനവിഭാഗത്തെയും അദ്ദേഹം കണ്ടെത്തി. അദ്ദേഹത്തോട്) നാം പറഞ്ഞു: ഹേ, ദുല്‍ഖര്‍നൈൻ, ഒന്നുകില്‍ നിനക്ക് ഇവരെ ശിക്ഷിക്കാം. അല്ലെങ്കില്‍ നിനക്ക്അവരില്‍ നന്‍മയുണ്ടാക്കാം.” (18:83-86)

ഈ വചനത്തില്‍ സൂര്യന്‍ ചെളിവെള്ളത്തില്‍ ആഴ്ന്നു പോകുന്നുവെന്ന് പറഞ്ഞിട്ടില്ലെന്ന വസ്തുത ശ്രദ്ധിക്കുക. ദുല്‍ഖര്‍നൈനിയെ കുറി ച്ചും അദ്ദേഹത്തിന്റെ യാത്രകളെക്കുറിച്ചുമാണ് ഈ വചനങ്ങളിലെ പ്രതിപാദ്യം. അദ്ദേഹത്തിന്റെ യാത്രകള്‍ക്കിടയിൽ സൂര്യന്‍ അസ്ത മിക്കുന്ന സ്ഥലത്ത് എത്തിയപ്പോള്‍ ”സൂര്യന്‍ ചെളിവെള്ളമുള്ള ഒരു ജലാശയത്തില്‍ മറഞ്ഞു പോകുന്നതായി അദ്ദേഹം കണ്ടു” വെന്നാണ് ഖുര്‍ആൻ ഈ സൂക്തങ്ങളില്‍വ്യക്തമാക്കിയിട്ടുള്ളത്.

സൂര്യന്‍ ഉദിക്കുകയോ അസ്തമിക്കുകയോ ചെയ്യുന്നില്ല എന്ന് ഇന്ന് നമുക്കറിയാവുന്നതാണ്. ഭൂമിക്ക് ആപേക്ഷികമായി സൂര്യന്‍ നിശ്ചലാ വസ്ഥയിലാണെന്നും ഭൂമിയുടെ സ്വയംഭ്രമണം മൂലമാണ് സൂര്യന്‍ ഉദിക്കുന്നതുംഅസ്തമിക്കുന്നതുമായി നമുക്കനുഭവപ്പെടുന്നതെന്നുമുള്ള താണല്ലോ വസ്തുത. എന്നാല്‍ഭൂമിയില്‍ ജീവിക്കുന്ന ഓരോ മനുഷ്യരും സൂര്യോദയവും അസ്തമയവും അനുഭവിക്കുന്നുണ്ട്. ഭൂമിയിലു ള്ളവര്‍ക്ക് ആപേക്ഷികമായിസൂര്യന്‍ ഉദിക്കുകയും അസ്തമിക്കുകയും ചെയ്യുന്നുണ്ടെന്ന് സാരം. ഭൂമിയില്‍ ജീവിച്ചിരുന്ന ഒരാളായിരുന്ന ദുല്‍ഖര്‍നൈനിയും സൂര്യോദയവും സൂര്യാസ്തമയവും കണ്ടിട്ടുണ്ടാവണം. അദ്ദേഹത്തിന്റെ യാത്രയ്ക്കിടയിൽ സൂര്യാസ്തമയം നടക്കു ന്ന സ്ഥലത്തെത്തിയപ്പോള്‍ ചെളിവെള്ളമുള്ള ഒരു ജലാശയത്തിൽ സൂര്യൻ അസ്തമിക്കുന്നതായി അദ്ദേഹം കണ്ട കാര്യമാണ് ഖുര്‍ആനിൽ ഇവിടെ പരാമര്‍ശിച്ചിരിക്കുന്നത്.

‘ചെളിവെള്ളമുള്ള ജലാശയത്തില്‍ സൂര്യന്‍ മറഞ്ഞുപോയി’യെന്നത് ഖുര്‍ആനിന്റെ കേവല പരാമര്‍ശമല്ല, പ്രത്യുത ദൂര്‍ഖര്‍നൈനി കണ്ട കാര്യത്തിന്റെ പ്രതിപാദനം മാത്രമാണ്. ‘ഞാന്‍ ഇന്നലെ സൂര്യാസ്തമയ സമയത്ത്കോഴിക്കോട് കടപ്പുറത്ത് പോയപ്പോള്‍ സമുദ്രത്തില്‍ സൂര്യന്‍ മറഞ്ഞു പോകുന്നതായി കണ്ടു’വെന്ന പരാമര്‍ശത്തിൽ എന്തെങ്കിലും അശാസ്ത്രീയതയുണ്ടോ? ഇല്ലെങ്കില്‍ സൂചിപ്പിക്കപ്പെട്ട ഖുർആൻ വചനങ്ങളിലും യാതൊരുഅശാസ്ത്രീയതയുമില്ല.

പരലോകത്തെ മനുഷ്യരെ രണ്ടു വിഭാഗങ്ങളാക്കി തിരിക്കുമെന്ന് (90:17-20, 99:6-8) ലും അങ്ങനെ മറ്റ് പല സൂക്തങ്ങളിലും പറയുന്നതിന് വിരുദ്ധമായി മൂന്ന് വിഭാഗങ്ങളാക്കുമെന്ന് (56:7)ൽ പ്രസ്താവിക്കുന്നുണ്ടല്ലോ. എന്താണ് ഇതിനുള്ള വിശദീകരണം? ഇവിടെ പരാമര്‍ശിക്കപ്പെട്ട സൂക്തങ്ങള്‍ പരിശോധിക്കുക: പുറമെ വിശ്വസിക്കുകയും, ക്ഷമ കൊണ്ടും കാരുണ്യം കൊണ്ടുംപരസ്പരം ഉപദേശിക്കുകയും ചെയ്യുന്നവരുടെ കൂട്ടത്തില്‍ അവന്‍ ആയിതീരുകയും ചെയ്യുക. അങ്ങനെ ചെയ്യുന്നവരാണ് വല തുപക്ഷക്കാര്‍. നമ്മുടെദൃഷ്ടാന്തങ്ങള്‍ നിഷേധിച്ചവരാരോ അവരത്രെ ഇടതുപക്ഷത്തിന്റെആള്‍ക്കാര്‍. അവരുടെ മേല്‍ അടച്ചുമൂടിയ നരകാഗ്നിയുണ്ട്. (90: 17-20). അന്നേ ദിവസം മനുഷ്യര്‍ പല സംഘങ്ങളായി പുറപ്പെടുന്നതാണ്; അവര്‍ക്ക് അവരുടെ കര്‍മ്മങ്ങള്‍ കാണിക്കപ്പെടേണ്ടതിനായിട്ട്. അപ്പോള്‍ ആര് ഒരണുത്തൂക്കം നന്‍മ ചെയ്തിരുന്നുവോ അത് അവന്‍കാണും. ആര് ഒരണുത്തൂക്കം തിന്‍മ ചെയ്തിരുന്നുവോ അതും അവന്‍കാണും. (99:6-8) നിങ്ങള്‍ മൂന്ന് തരക്കാരായി തീരുന്ന സന്ദര്‍ഭമത്രെ അത്.’ (56:7) മുകളില്‍ പറഞ്ഞ സൂക്തങ്ങളിലൊന്നും തന്നെ പരലോകത്ത് രണ്ടുവിഭാഗക്കാരേ ഉണ്ടാവൂ എന്ന് പറഞ്ഞിട്ടില്ല. ആദ്യം ഉദ്ധരിക്കപ്പെട്ട സൂറത്തുല്‍ ബലദിലെ (90:17-20) സൂക്തങ്ങളില്‍ സ്വര്‍ഗ്ഗത്തില്‍ പ്രവേശിപ്പിക്കപ്പെടുന്ന വലതു പക്ഷക്കാരെ കുറിച്ചും നരകത്തിലേക്ക് എറിയപ്പെടുന്ന ഇടതുപക്ഷക്കാരെ കുറിച്ചുമാണ് പരാമര്‍ശിക്കപ്പെട്ടിട്ടുള്ളത്. പരലോകത്ത് ഇങ്ങനെ രണ്ടു വിഭാഗം മാത്രമേ ഉണ്ടാവൂയെന്ന് അവിടെയൊന്നും യാതൊരു പരാമര്‍ശവുമില്ല. രണ്ടാമതായി ഉദ്ധരിക്കപ്പെട്ട സൂറത്തു സല്‍സലഃയിലെ സൂക്തങ്ങളിലാകട്ടെ (99:68) അണുത്തൂക്കം നന്‍മചെയ്തവര്‍ അതും തിന്‍മ ചെയ്തവര്‍ അതും കാണുമെന്ന് മാത്രമാണ് പറയുന്നത്. തങ്ങളുടെ കര്‍മ്മഫലങ്ങള്‍ കാണുന്നതിനായി മനുഷ്യര്‍ പലസംഘങ്ങളായി പുറപ്പെടുന്നതിനെ കുറിച്ച് സൂചിപ്പിക്കുന്ന ഈ സൂക്തങ്ങളില്‍ എത്ര സംഘങ്ങളാണെന്ന കൃത്യമായ പ്രതിപാദനം ഉള്‍കൊള്ളുന്നില്ല. സൂറത്തുല്‍ വാഖിഅ:യിലാകട്ടെ (56:7) കൃത്യമായി തന്നെ പരലോകത്തിലെ മൂന്ന് വിഭാഗങ്ങളെ പറ്റി പറഞ്ഞിരിക്കുന്നു. ആ മൂന്ന് വിഭാഗക്കാര്‍ ആരൊക്കെയാണെന്നും പ്രസ്തുത സൂറത്തിലെ മറ്റു സൂക്തങ്ങള്‍ വ്യക്തമാക്കുന്നുണ്ട്. സ്വര്‍ഗത്തിലേക്ക് അയക്കപ്പെടുന്ന വലതു പക്ഷക്കാരില്‍തന്നെ സല്‍ഗുണങ്ങളിലെ മികവ് കൊണ്ട് അല്ലാഹുവിങ്കല്‍ കൂടുതല്‍ സാമീപ്യം നല്‍കപ്പെട്ടവരും ഉന്നതസ്ഥാനീയരുമായ ഒരു പ്രത്യേകവിഭാഗം വേറെ ഉണ്ടാകുമെന്ന് ഈ വചനങ്ങളില്‍ വ്യക്തമാക്കിയിരിക്കുന്നു. ഇവരെക്കൂടി പരിഗണിച്ചു കൊണ്ടാണ് മൂന്നു വിഭാഗക്കാരെന്ന് പരാമര്‍ശിച്ചിരിക്കുന്നത്. പരലോകത്ത് മനുഷ്യരില്‍ ഒരു വിഭാഗം നരകാവകാശികളും മറ്റൊരു വിഭാഗം സ്വര്‍ഗ്ഗാവകാശികളും ആയി തിരിക്കപ്പെടുന്നതോടൊപ്പം തന്നെ സ്വര്‍ഗ്ഗവാസികളില്‍ ദൈവസാമീപ്യം കൂടുതലായി ലഭിക്കുന്ന ഒരു ശ്രേഷ്ഠ വിഭാഗം കൂടി ഉണ്ടാവുന്നതില്‍ വൈരുധ്യമൊന്നുമില്ല. പരലോകത്ത് മൂന്ന് വിഭാഗങ്ങള്‍ ഉണ്ടാവുകയില്ലെന്നോ രണ്ട് വിഭാഗങ്ങള്‍ മാത്രമേ ഉണ്ടാവുകയുള്ളൂവെന്നോ ഖുര്‍ആനിലെവിടെയും പരാമര്‍ശിക്കുന്നില്ല എന്നതുകൊണ്ട് തന്നെ ഈ സൂക്തവുമായി വൈരുധ്യം പുലര്‍ത്തുന്ന ഒരു ഭാഗവും ഖുര്‍ആനിലില്ലെന്ന് അര്‍ഥശങ്കയ്ക്കിടയില്ലാതെ പറയാന്‍ സാധിക്കും.
പരലോകത്ത് എത്ര സ്വർഗമാണുള്ളത്? ഖുര്‍ആനിലെ ചില സൂക്തങ്ങളിൽ ഒരു സ്വർഗമെന്നും (ഉദാ: 39:73, 41:30, 57:21, 79:41) മറ്റു ചിലവയിൽ ധാരാളം സ്വർഗങ്ങളെന്നും (ഉദാ: 18:31, 22:23, 35:33, 78:32) പരാമര്ശിച്ചിട്ടുണ്ടല്ലോ. ഇത് വൈരുധ്യമല്ലേ? മരണാനന്തര ജീവിതത്തെക്കുറിച്ച് പ്രതിപാദിക്കുമ്പോള്‍ ഖുര്‍ആനില്‍ സ്വര്‍ഗ്ഗം (ജന്നത്ത്) എന്നും സ്വർഗങ്ങള്‍ (ജന്നാത്ത്) എന്നും പ്രയോഗിക്കപ്പെട്ടിട്ടുണ്ട്. രണ്ടു പ്രയോഗങ്ങള്‍ക്കും ഓരോ ഉദാഹരണങ്ങള്‍കാണുക. തങ്ങളുടെ രക്ഷിതാവിനെ സൂക്ഷിച്ചു ജീവിച്ചവര്‍ സ്വർഗത്തിലേക്ക് കൂട്ടം കൂട്ടമായി നയിക്കപ്പെടും. അങ്ങനെ അതിന്റെ കവാടങ്ങള്‍ തുറന്നുവെക്കപ്പെട്ടനിലയില്‍ അവര്‍ അതിനടുത്ത് വരുമ്പോള്‍ അവരോട് അതിന്റെ കാവല്‍ക്കാര്‍ പറയും: നിങ്ങള്‍ക്ക് സമാധാനം നിങ്ങള്‍ സംശുദ്ധരായിരിക്കുന്നു. അതിനാല്‍ നിത്യവാസികളെന്ന നിലയില്‍ നിങ്ങള്‍ അതില്‍ പ്രവേശിച്ചുകൊള്ളുക. (39:73) അക്കൂട്ടര്‍ക്കാകുന്നു സ്ഥിരവാസത്തിനുള്ള സ്വർഗത്തോപ്പുകള്‍. അവരുടെ താഴ്ഭാഗത്തുകൂടി അരുവികള്‍ ഒഴുകികൊണ്ടിരിക്കുന്നതാണ്.(18:31) ഈ പ്രയോഗങ്ങള്‍ തമ്മില്‍ യാതൊരു വൈരുധ്യവുമില്ല. സത്യവിശ്വാസികള്‍ കൂട്ടം കൂട്ടമായി ആനയിക്കപ്പെടുന്നത് സ്വർഗലോകത്തിലേക്കാണ്. ആ സ്വർഗലോകത്ത് ഒരുപാട് സ്വർഗങ്ങളുണ്ട്. ഓരോരുത്തരുടെയും സല്‍കര്‍മ്മങ്ങളുടെ തോതനുസരിച്ച് വ്യത്യസ്ത സ്വർഗങ്ങളിലായിരിക്കും പ്രവേശിപ്പിക്കപ്പെടുകയെന്ന് ഹദീസുകളില്‍ വന്നിട്ടുണ്ട്. ഒരു തോട്ടത്തിനകത്ത് തന്നെ വിവിധ തരം തോട്ടങ്ങളുള്ളത് നമുക്ക് പരിചയമുള്ളതാണ്. റോസാചെടിയുടെ തോട്ടവും ഡാലിയയുടെ തോട്ടവും മല്ലികാ തോട്ടവും ഓര്‍ക്കിഡുകളുടെ തോട്ടവുമെല്ലാം ഉള്‍ക്കൊള്ളുന്ന സ്ഥലത്തെ നാം തോട്ടം (ഗാര്‍ഡന്‍) എന്നു തന്നെയാണ് പറയുക. ഒരു തോട്ടത്തില്‍ തന്നെ അനേകം തോട്ടങ്ങളുണ്ടാകുമെന്നര്‍ത്ഥം. ഇതേപോലെത്തന്നെ സ്വർഗലോകത്ത് അനേകം സ്വർഗത്തോപ്പുകളുണ്ട്. ഒരേയൊരു സ്വർഗത്തിനകത്തു തന്നെയുള്ള സ്വർഗത്തോപ്പുകളെക്കുറിച്ച് പരാമര്‍ശിക്കുമ്പോള്‍ ഖുര്‍ആന്‍ ബഹുവചനവും മൊത്തം സ്വര്‍ഗലോകത്തെപ്പറ്റി പറയുമ്പോള്‍ ഏകവചനവും ഉപയോഗിക്കുന്നു എന്നുമാത്രമേയുള്ളൂ. ഇതില്‍ യാതൊരുവിധ വൈരുധ്യവുമില്ല.
ആകാശങ്ങളോ ഭൂമിയോ ഏതാണ് ആദ്യം സൃഷ്ടിക്കപ്പെട്ടത് ? ആദ്യം ഭൂമിയാണെന്ന് 2:29ലും ആദ്യം ആകാശമാണെന്ന് 79: 27-30ലും പറയുന്നു. ഇത് വൈരുധ്യമല്ലേ ? അവനാണ് നിങ്ങള്‍ക്കുവേണ്ടി ഭൂമിയിലുള്ളതെല്ലാം സൃഷ്ടിച്ചുതന്നത്. പുറമെ ഏഴു ആകാശങ്ങളായി ക്രമീകരിച്ചുകൊണ്ട് ഉപരിലോകത്തെ സംവിധാനിച്ചവനും അവന്‍ തന്നെയാണ്. അവന്‍ എല്ലാ കാര്യത്തെപ്പറ്റിയും അറിവുള്ളവനാകുന്നു.’’ (2:29) നിങ്ങളാണോ സൃഷ്ടിക്കപ്പെടാന്‍ കൂടുതല്‍ പ്രയാസമുള്ളവര്‍ അതല്ല ആകാശമാണോ ? അതിനെ (ആകാശത്തെ) അവന്‍ നിര്‍മ്മിച്ചിരിക്കുന്നു. അതിന്റെ വിതാനം അവന്‍ ഉയര്‍ത്തുകയും, അതിനെ അവന്‍ വ്യവസ്ഥപ്പെടുത്തുകയും ചെയ്തിരിക്കുന്നു. അതിലെ രാത്രിയെ അവന്‍ ഇരുട്ടാക്കുകയും, അതിലെ പകലിനെ അവന്‍ പ്രത്യക്ഷപ്പെടുത്തുകയും ചെയ്തിരിക്കുന്നു: അതിനു ശേഷം അവന്‍ ഭൂമിയെ വികസിപ്പിച്ചിരിക്കുന്നു’. (79:27-30) ഈ സൂക്തങ്ങളിലാണ് വൈരുധ്യം ആരോപിക്കപ്പെട്ടിരിക്കുന്നത്. സൂറത്തുല്‍ ബഖറയിലെ ഇരുപത്തിയൊമ്പതാം സൂക്തത്തില്‍ പുറമെയെന്ന് പരിഭാഷപ്പെടുത്തപ്പെട്ടിരിക്കുന്നത്. ‘ഥുമ്മ‘യെന്ന അറബി അവ്യയമാണ്. പിന്നെയെന്നാണ് ‘ഥുമ്മ‘ക്ക് സാധാരണയായി നല്‍കപ്പെടാറുള്ള അര്‍ത്ഥം. പ്രസ്തുത അര്‍ത്ഥം ഈ സൂക്തത്തില്‍ നല്‍കുമ്പോള്‍ ഭൂമിയിലുള്ളതെല്ലാം സൃഷ്ടിച്ചതിനു ശേഷമാണ് ആകാശം സൃഷ്ടിക്കപ്പെട്ടതെന്നാണ് ഖുര്‍ആനില്‍ പറയുന്നതെന്ന് വരുത്താനാവും. ഈ അടിസ്ഥാനത്തിലാണ് വിമര്‍ശകന്‍മാര്‍ ഈ സൂക്തം സൂറത്തുന്നാസിആത്തിലെ ഉദ്ധരിക്കപ്പെട്ട വചനങ്ങളുമായി(79:27-30) വൈരുധ്യം പുലര്‍ത്തുന്നുവെന്ന് ആരോപിക്കുന്നത്. ഥുമ്മയെന്ന അവ്യയം സംഭവങ്ങളുടെ കാലക്രമത്തെ സൂചിപ്പിക്കുവാന്‍ മാത്രമല്ല അറബിയില്‍ പ്രയോഗിക്കപ്പെടുന്നത്. വിവരണത്തിലെ ക്രമാനുഗതികത്വത്തെ സൂചിപ്പിക്കുവാനും ഥുമ്മയെന്ന് പ്രയോഗിക്കാറുണ്ട്. ഇത് പൊതുവെ എല്ലാ ഭാഷകളിലുമുള്ള പ്രയോഗ രീതിയാണ്. മലയാളത്തില്‍ നാം ഇങ്ങനെ പറയാറുണ്ട്. നീ ഇന്ന് എങ്ങോട്ട് പോയതായിരുന്നുവെന്ന് എനിക്കറിയാം, പിന്നെ, നീ ഇന്നലെ എങ്ങോട്ടായിരുന്നു പോയതെന്നും എനിക്കറിയാം. ഈ പ്രയോഗത്തില്‍ നിന്ന് ഇന്നലത്തെ യാത്ര ഇന്നത്തേതിന് ശേഷമാണുണ്ടായതെന്ന് ആരും അര്‍ഥമാക്കാറില്ല. സംഭവക്രമത്തിനു പകരം വിവരണക്രമം സൂചിപ്പിക്കുന്നതിന് വേണ്ടിയാണ് ഇവിടെ പിന്നെയെന്ന് പറഞ്ഞിട്ടുള്ളതെന്ന് ആര്‍ക്കാണ് അറിഞ്ഞുകൂടാത്തത്? ഇതേ പോലെതന്നെ വിവരണക്രമം സൂചിപ്പിക്കുന്നതിന് വേണ്ടിയാണ് 2:29 ല്‍ ഥുമ്മയെന്ന് പ്രയോഗിച്ചിരിക്കുന്നത്. അല്ലാതെ ഭൂമിയുണ്ടായതിന് ശേഷമാണ് ആകാശങ്ങള്‍ ഉണ്ടായതെന്ന് പ്രസ്തുത സൂക്തം അര്‍ഥമാക്കുന്നേയില്ല. മാത്രവുമല്ല, സൂറത്തുല്‍ ബഖറയിലെ സൂചിത വാക്യം അല്ലാഹുവിന്റെ സൃഷ്ടിക്രമം വിവരിക്കുകയല്ല, പ്രത്യുത അവന്റെ അനുഗ്രഹങ്ങളെ സംബന്ധിച്ച് മനുഷ്യരെ തെര്യപ്പെടുത്തുകയാണ് ചെയ്യുന്നത്. അതിനു നേരെമുമ്പുള്ള വാക്യവുമായി അത് ചേര്‍ത്തു വായിച്ചാല്‍ ഇക്കാര്യം സുതരാം വ്യക്തമാവും. നിങ്ങള്‍ക്കെങ്ങനെയാണ് അല്ലാഹുവിനെ നിഷേധിക്കുവാന്‍ കഴിയുക? നിങ്ങള്‍ നിര്‍ജീവ വസ്തുക്കളായിരുന്ന അവസ്ഥയ്ക്കു ശേഷം അവന്‍ നിങ്ങള്‍ക്ക് ജീവന്‍ നല്‍കി. പിന്നെ അവന്‍ നിങ്ങളെ മരിപ്പിക്കുകയും വീണ്ടും ജീവിപ്പിക്കുകയും ചെയ്യുന്നു. പിന്നീട് അവങ്കലേക്കു തന്നെ നിങ്ങള്‍ തിരിച്ചുവിളിക്കപ്പെടുകയും ചെയ്യും (2:78) എന്നു പറഞ്ഞ ശേഷമാണ്, അവനാണ് നിങ്ങള്‍ക്ക് വേണ്ടി ഭൂമിയിലുള്ളതെല്ലാം സൃഷ്ടിച്ചുതന്നത് എന്നു പറഞ്ഞുകൊണ്ട് അടുത്ത സൂക്തം ആരംഭിക്കുന്നത്. മനുഷ്യനെ സംബന്ധിച്ചിടത്തോളം പ്രഥമ പ്രാധാന്യമുള്ളത് ഭൂമിയും അതിലെ അനുഗ്രഹങ്ങളുമാണ്. ദൈവനിഷേധികളോട് ഒന്നാമതായി സ്വന്തത്തെകുറിച്ചും പിന്നീട് അവര്‍വസിക്കുന്ന ഭൂമിയെ കുറിച്ചും അതിനുശേഷം ആകാശ ക്രമീകരണത്തെകുറിച്ചുമെല്ലാം ചിന്തിക്കുവാന്‍ വേണ്ടി ആവശ്യപ്പെടുകയാണ് ഈ സൂക്തങ്ങളില്‍ ചെയ്യുന്നത്. അതുകൊണ്ട് തന്നെ ഇത് ആകാശ ക്രമീകരണങ്ങള്‍ക്കു ശേഷമാണ് ഭൂമിയെ വികസിപ്പിച്ചെടുത്തതെന്ന സൂറത്തു നാസിആത്തിലെ വചനവുമായി (79:30) യാതൊരുവിധ വൈരുധ്യവും പുലര്‍ത്തുന്നില്ലെന്നതാണ് വാസ്തവം.
ശിർക്ക് (ബഹുദൈവത്വം) മഹാപാപമാണെന്ന് ഖുർആനിൽ പല തവണ പറയുന്നുണ്ടല്ലോ. എന്നാൽ, വിശ്വാസികളുടെ നേതാവായി പരിചയപ്പെടുത്തപ്പെടുന്ന ഇബ്റാഹീം നബി (അ) ചന്ദ്രനെയും സൂര്യനെയും നക്ഷത്രങ്ങളെയുമെല്ലാം ദൈവമാക്കിയെന്ന് 6:76-78 സൂക്തങ്ങളിൽ പറയുന്നുണ്ട്. ഇബ്റാഹീം ബഹുദൈവാരാധകനായിരുന്നുവെന്നല്ലേ ഇതിന്നർത്ഥം? ഇബ്‌റാഹീം നബി(അ) ശിര്‍ക്ക് ചെയ്തുവെന്ന് ഖുര്‍ആനില്‍ ഒരിടത്തും പറയുന്നില്ല. ഏകദൈവാദര്‍ശത്തിനു വേണ്ടി ഏറെ ത്യാഗങ്ങള്‍ സഹിച്ച വ്യക്തിയാണ് ഇബ്‌റാഹീം (അ). ബഹുദൈവാരാധനയുടെ വ്യര്‍ഥതയും, സ്രഷ്ടാവും സംരക്ഷകനുമായ അല്ലാഹുവിനെ മാത്രം ആരാധിക്കേണ്ടതിന്റെ ആവശ്യകതയും തന്റെ ജനതയ്ക്ക് വിശദീകരിച്ചു കൊടുക്കുന്നതിന്നായി വിവിധ മാര്‍ഗങ്ങള്‍ അദ്ദേഹം ഉപയോഗിച്ചതായി ഖുര്‍ആന്‍ വ്യക്തമാക്കുന്നുണ്ട്. ഉപദേശ നിര്‍ദ്ദേശങ്ങള്‍ വഴി വിഗ്രഹാരാധനയുടെ വ്യര്‍ഥത വ്യക്തമാക്കാന്‍ അദ്ദേഹം ശ്രമിച്ചു (21:51-56); അവരോട് വാദപ്രതിവാദം നടത്തി (6:80-83); അവരെ ശക്തമായി വിമര്‍ശിച്ചു (6:74,75).അവരുടെ ചിന്തയെ തൊട്ടുണര്‍ത്തുന്നതിന് വേണ്ടി ക്ഷേത്രത്തിലെ വിഗ്രഹങ്ങളെയെല്ലാം ഉടക്കുകയും അവയിലെ വലിയതിനെ ബാക്കിയാക്കി വിഗ്രഹഭഞ്ജനമെന്ന കുറ്റം അതിന്റെ മേല്‍ ആരോപിക്കുകയും ചെയ്തു(21:57-67). ഇതേപോലെ ഒരു സംഭവമാണ് സൂര്യചന്ദ്ര നക്ഷത്രാദികളെയൊന്നും പൂജിക്കുവാന്‍ കൊള്ളുകയില്ലെന്ന് വ്യക്തമാക്കുന്നതിന് വേണ്ടി നക്ഷത്രപൂജകരായിരുന്ന ജനങ്ങളുടെ മുന്നില്‍ ഇബ്‌റാഹീം (അ) ചെയ്തതായി സൂറത്തു അന്‍ആമില്‍ (76-79) വിവരിച്ചിരിക്കുന്നത്. സൂര്യദേവനെയും ചന്ദ്രദേവനെയും ശുക്രദേവനെയുമെല്ലാം ആരാധിച്ചിരുന്ന ജനങ്ങള്‍ക്ക് അവരുടെ പ്രവര്‍ത്തനങ്ങളിലെ വ്യര്‍ഥത വ്യക്തമാക്കികൊടുക്കുയാണ് ഇബ്‌റാഹീം(അ) ചെയ്തത്. ഉദിച്ചുയര്‍ന്ന നക്ഷത്രത്തെ നോക്കി ഇതാണ് എന്റെ രക്ഷിതാവ് എന്ന് പ്രഖ്യാപിക്കുകയും, അത് മറഞ്ഞപ്പോള്‍ മറഞ്ഞുപോകുന്നുവയെ ഞാന്‍ ഇഷ്ടപ്പെടുന്നില്ലെന്ന് പറഞ്ഞ് നക്ഷത്രപൂജകര്‍ക്ക് അവരുടെ പ്രവര്‍ത്തനത്തിന്റെ വ്യര്‍ഥത വ്യക്തമാക്കികൊടുക്കുകയുമാണ് ഇബ്‌റാഹീം (അ) ചെയ്തത്. ചന്ദ്രന്‍ ഉദിച്ചപ്പോള്‍ ഇതാണെന്റെ രക്ഷിതാവ് എന്നു പറയുകയും അതും അസതമിച്ചപ്പോള്‍, ഇതിനെയും ആരാധിക്കാന്‍ കൊള്ളുകയില്ലെന്ന് പ്രഖ്യാപിക്കുകയും ചെയ്ത അദ്ദേഹം ചന്ദ്ര പൂജകരെ തങ്ങളുടെ വിഡ്ഢിത്തം തെര്യപ്പെടുത്തുകയാണ് ചെയ്യുന്നത്. ഇതേ പോലെ തന്നെ സൂര്യപൂജകരെ ചിന്തിപ്പിക്കുന്നതിനായി, വലിയവനായ സൂര്യനാണ് രക്ഷിതാവെന്ന് പറഞ്ഞ് അത് അസ്തമിച്ചപ്പോള്‍ ഇതും ആരാധനക്ക് കൊള്ളുകയില്ലെന്ന് പഠിപ്പിക്കുകയാണ് അദ്ദേഹം ചെയ്തത്. യഥാര്‍ത്ഥത്തില്‍ സൂര്യനോ ചന്ദ്രനോ നക്ഷത്രങ്ങളോ ആരാധനകള്‍ അര്‍ഹിക്കുന്നുവെന്ന വിശ്വാസം ഇബ്‌റഹീ(അ)മിന്ന് ഉണ്ടായിരുന്നില്ല എന്നത് കൊണ്ട് തന്നെ അദ്ദേഹം ബഹുദൈവാരാധന നടത്തിയെന്ന് പറയുന്നത് അടിസ്ഥാനരഹിതമാണ്. ഇങ്ങനെയെല്ലാം ചെയ്തത് എന്റെ ജനങ്ങളേ, നിശ്ചയമായും ഞാന്‍, നിങ്ങള്‍ പങ്ക് ചേര്‍ക്കുന്നതില്‍ നിന്നെല്ലാം ഒഴിവായവനാകുന്നു. ഞാന്‍ ആകാശങ്ങളെയും ഭൂമിയെയും സൃഷ്ടിച്ചുണ്ടാക്കിയവനിലേക്ക് ഋജുമനസ്‌കനായി ക്കൊണ്ട് എന്റെ മുഖം തിരിച്ചിരിക്കുന്നു. ഞാന്‍ ശിര്‍ക്ക് ചെയ്യുന്നവരില്‍ പെട്ടവനല്ലതാനും (6:78,79)എന്നു പ്രഖ്യാപിക്കുന്നതിന്നു വേണ്ടിയായിരുന്നുവെന്നാണ് ഖുര്‍ആന്‍ വ്യക്തമാക്കുന്നത്. മനസ്സില്‍ വിശ്വാസമില്ലാതെ ഞാന്‍ അതിനെ രക്ഷിതാവായി സ്വീകരിച്ചിരിക്കുന്നുവെന്ന് പറയുന്നതുകൊണ്ട് മാത്രം ഒരാള്‍ ശിര്‍ക്കു ചെയ്യുന്നവനായി തീരുകയില്ലെന്ന് ഖുര്‍ആനില്‍ തന്നെ(16:106) വ്യക്തമാക്കുകയും ചെയ്തിട്ടുണ്ട്.
ആകാശഭൂമികളിലുള്ളതെല്ലാം അല്ലാഹുവിനെ അനുസരിക്കുന്നു (30:26, 3:83)വെന്ന് ഖുർആനിന്റെ അവകാശവാദത്തിനു വിരുദ്ധമായി ചെകുത്താന് അവനോട് അനുസരണക്കേട് കാണിച്ചുവെന്ന് ഖുർആനിൽ തന്നെ (7: 11, 15: 28-31, 17: 61, 20: 116, 38: 71-74, 18: 50) പലതവണ പറയുന്നു. മനുഷ്യരുടെ അനുസരണക്കേടിനെ കുറിച്ച പരാമർശങ്ങളും എമ്പാടുമുണ്ട്. ഇത് വൈരുധ്യമല്ലേ? ഈ പ്രപഞ്ചത്തിലെ ചെറുതും വലുതുമായ വസ്തുക്കള്‍ക്കെല്ലാം അവയുടേതായ ചില സ്വഭാവസവിശേഷതകളും വ്യവസ്ഥളും അല്ലാഹു നിശ്ചയിച്ചിട്ടുണ്ട്. ഈ ദൈവിക നിശ്ചയത്തില്‍ നിന്ന് തെന്നിമാറികൊണ്ട് സചേതനമോ അചേതനമോ ആയ യാതൊരു വസ്തുവിനും നിലനില്‍ക്കാനാവില്ല. മനുഷ്യരും മറ്റു ജീവജാലങ്ങളുമടക്കം സൂക്ഷ്മ സ്ഥൂല പ്രപഞ്ചങ്ങളിലെ മുഴുവന്‍ പ്രതിഭാസങ്ങളും ദൈവവിധിപ്രകാരം വ്യവസ്ഥാപിതമായാണ് നിലനില്‍ക്കുന്നത്. നക്ഷത്രങ്ങളും സൂര്യനും ചന്ദ്രനും ഭൂമിയുമെല്ലാം ദൈവികവ്യവസ്ഥ പ്രകാരമാണ് ചലിച്ചുകൊണ്ടിരിക്കുന്നത്. പ്രസ്തുത വ്യവസ്ഥ അന്യൂനവും പ്രമാദമുക്തവുമാണ്. ഈവ്യവസ്ഥയില്‍ നിന്ന് തെന്നിമാറുവാന്‍ സൃഷ്ടികള്‍ക്കൊന്നും കഴിയില്ല. അല്ലാഹുവിന്റെ അത്യുല്‍കൃഷ്ട സൃഷ്ടിയായ മനുഷ്യനും ഈ ദൈവിക വ്യവസ്ഥ പ്രകാരം തന്നെയാണ് ജീവിച്ചുകൊണ്ടിരിക്കുന്നത്. ബീജസങ്കലനം മുതല്‍ വളര്‍ച്ചയെത്തി മനുഷ്യശിശു പുറത്തു വരുന്നത് വരെയുള്ള ഘട്ടങ്ങളിലെല്ലാം കാര്യങ്ങള്‍ മുന്നോട്ട് പോകുന്നത് ദൈവവിധി പ്രകാരമാണ്. സ്വയം നിയന്ത്രിക്കാനാവുന്ന ഏതാനും അവയവങ്ങള്‍ മാത്രമേ മനുഷ്യനുള്ളൂ. അവ തന്നെ അവന്റെ പൂര്‍ണ നിയന്ത്രണത്തിലാണെന്ന് പറയാന്‍ വയ്യ. കൈവിരലുകള്‍ ഇളക്കുവാനുള്ള നമ്മുടെ കഴിവ് അവയ്ക്കുള്ളിലെ അസ്ഥികളുടെ നിലനില്‍പുമായി ബന്ധപ്പെട്ടാണിരിക്കുന്നത്. അസ്ഥികൂടമടക്കം മനുഷ്യശരീരത്തിലെ വ്യവസ്ഥകളൊന്നും തന്നെ നമ്മുടെ പൂര്‍ണ നിയന്ത്രണത്തിനു കീഴിലല്ല. നമ്മുടെ ആകാരം മുതല്‍ വികാരങ്ങള്‍വരെ ജീനുകളില്‍ രേഖപ്പെടുത്തപ്പെട്ടിട്ടുണ്ടത്രെ. മനുഷ്യന്റെ ഇന്ദ്രിയങ്ങളും അവയവങ്ങളും വ്യവസ്ഥകളുമെല്ലാം പ്രവര്‍ത്തിക്കുന്നത് ഈ ജനിതക നിര്‍ദ്ദേശങ്ങള്‍ക്കനുസരിച്ചാണ്. മനുഷ്യ ശരീരമടക്കം പ്രപഞ്ചത്തിലെ സചേതനവും അചേതനവുമായ വസ്തുക്കളെല്ലാം ദൈവിക വിധിപ്രകാരമാണ് നിലനില്‍ക്കുന്നതെന്ന വസ്തുത ഖുര്‍ആന്‍ വ്യക്തമാക്കിയിട്ടുണ്ട്. എല്ലാ വസ്തുക്കളും അല്ലാഹുവിനെ അനുസരിച്ചുകൊണ്ടിരിക്കുകയാണെന്നാണ് ഖുര്‍ആന്‍ പറയുന്നത്: ആകാശങ്ങളിലും ഭൂമിയിലുമുള്ളവരെല്ലാം അവന്റെ അധീനത്തിലത്രെ. എല്ലാവരും അവന് കീഴടങ്ങുന്നവരാകുന്നു. (30:26). അപ്പോള്‍ അല്ലാഹുവിന്റെ മതമല്ലാത്ത മറ്റു വല്ല മതവുമാണോ അവര്‍ ആഗ്രഹിക്കുന്നത്? (യഥാര്‍ത്ഥത്തില്‍) ആകാശങ്ങളിലും ഭൂമിയിലും ഉള്ളവരെല്ലാം അനുസരണയോടെയോ നിര്‍ബന്ധിതമായോ അവന് കീഴ്‌പ്പെട്ടിരിക്കുകയാണ്. അവനിലേക്ക് തന്നെയാണ് അവര്‍ മടക്കപ്പെടുന്നതും. (3:83) എന്നാല്‍ കര്‍മ്മങ്ങളുടെ കാര്യത്തില്‍ മൃഗങ്ങളുടേതില്‍ നിന്ന് വ്യത്യസ്തമായ ചില സ്വാതന്ത്ര്യങ്ങള്‍ അല്ലാഹു മനുഷ്യന് നല്‍കിയിട്ടുണ്ട്. ഈ സ്വാതന്ത്ര്യം പോലും അലംഘനീയമായ ദൈവിക വിധിയുടെ പരിധിക്കുള്ളിലാണ്. പേനയുപയോഗിക്കുവാന്‍ മനുഷ്യര്‍ക്ക് സാധിക്കുന്നത് അല്ലാഹുവിന്റെ വ്യവസ്ഥ പ്രകാരമാണ്. എന്നാല്‍ പേനകൊണ്ട് എന്തെഴുതണമെന്ന് തീരുമാനിക്കുന്നത് മനുഷ്യരുടെ ഇഛാശക്തിയാണ്. ഈ ഇഛാശക്തിയാകട്ടെ ഒരു പരിധിവരെ ദൈവിക വ്യവസ്ഥയുടെ വരുതിക്കുള്ളില്‍ സ്വതന്ത്രമാണ്. ഈസ്വാതന്ത്ര്യം എങ്ങനെ ഉപയോഗിക്കുന്നുവെന്നതാണ് മനുഷ്യനെ ഉദാത്തനും അധമനുമാക്കുന്നത്. തൂലികയെ മനുഷ്യരെ സംസ്‌കരിക്കാനും അവരെ വഴിതെറ്റിക്കുവാനും ഉപയോഗിക്കാം. ഒന്നാമത്തേത് ഔന്നത്യത്തിന്റെ മാര്‍ഗം, രണ്ടാമത്തേത് അധമത്വത്തിന്റെ പാത. ഇതില്‍ ഏതു സരണി തിരഞ്ഞെടുക്കണമെന്ന് മനുഷ്യന് തീരുമാനിക്കാം. അത് അവന്റെ സ്വാതന്ത്ര്യത്തില്‍ പെട്ടതാണ്. തീര്‍ച്ചയായും നാം അവന് വഴി കാണിച്ചുകൊടുത്തിരിക്കുന്നു. എന്നിട്ട് ഒന്നുകില്‍ അവന്‍ നന്ദിയുള്ളവനാകുന്നു. അല്ലെങ്കില്‍ അവന്‍ നന്ദികെട്ടവനാകുന്നു. (76;3) മനുഷ്യരെ പോലെ തന്നെ ഇഛാസ്വാതന്ത്ര്യം നല്‍കപ്പെട്ട സൃഷ്ടികളാണ് ജിന്നുകള്‍ എന്നാണ് ഖുർആനില്‍ നിന്നും ഹദീസില്‍ നിന്നും നമുക്ക് മനസ്സിലാക്കാന്‍ കഴിയുന്നത്. ജിന്നുകളില്‍പ്പെട്ടവനാണ് ഇബ്‌ലീസ് (18:50). ജിന്നുകളും പൂര്‍ണമായി ദൈവിക വ്യവസ്ഥക്ക് കീഴിലുള്ളവര്‍ തന്നെയാണ്. എന്നാല്‍ മനുഷ്യന് സാഷ്ടാംഗം നമിക്കുവാനുള്ള ദൈവിക കല്‍പന അവന്‍ അതിലംഘിച്ചു. കല്‍പന ലംഘിക്കുവാനും സ്വീകരിക്കുവാനുമുള്ള സ്വാതന്ത്ര്യം മനുഷ്യനെപോലെ അവനുമുണ്ടായിരുന്നു. ഈ സ്വാതന്ത്ര്യം പോലും അല്ലാഹുവിന്റെ ദാനമാണ്. അതുകൊണ്ട് തന്നെ ഈ സ്വാതന്ത്ര്യമുപയോഗിച്ച് ദൈവിക കല്‍പന ലംഘിച്ചതിനാല്‍ അല്ലാഹുവിന്റെ പൊതുവ്യവസ്ഥയെ അവന്‍ ലംഘിച്ചുവെന്ന് പറയാവതല്ല. ലംഘിക്കുവാനും സ്വീകരിക്കുവാനും അല്ലാഹു അവന് നല്‍കിയ സ്വാതന്ത്ര്യത്തില്‍ തന്നിഷ്ടത്തിന്റെ-അഹങ്കാരത്തിന്റെയും-മാര്‍ഗം അവന്‍ തെരെഞ്ഞെടുത്തുവെന്നേയുള്ളൂ. ഈ സ്വാതന്ത്ര്യം നല്‍കപ്പെടാത്ത മലക്കുകളാകട്ടെ, അല്ലാഹുവിന്റെ ആജ്ഞ അതിലംഘിക്കാതെ നിറവേറ്റുകയും ചെയ്തു. ജിന്നുകള്‍ക്കും മനുഷ്യര്‍ക്കുമെല്ലാം നല്‍കപ്പെട്ടിരിക്കുന്ന ഇഛാ സ്വാതന്ത്ര്യം അവയുടെ സൃഷ്ടിവ്യവസ്ഥയുമായി താരതമ്യം ചെയ്യുമ്പോള്‍ വളരെ ചെറുതാണ്. ഈ സ്വാതന്ത്ര്യം പോലും അല്ലാഹുവിന്റെ നിയന്ത്രണത്തിനു വിധേയമാണുതാനും. അതുകൊണ്ട് തന്നെ ആകാശ ഭൂമികളുടെ എല്ലാവസ്തുക്കളും അല്ലാഹുവെ അനുസരിക്കുന്നുവെന്ന് പറഞ്ഞ സൂക്തങ്ങളുമായി പിശാചോ മനുഷ്യനോ അനുസരണക്കേട് കാണിക്കുന്നുവെന്ന് പ്രസ്താവിക്കുന്ന സൂക്തങ്ങള്‍ വൈരുധ്യം പുലര്‍ത്തുന്നുവെന്ന് പറയാനാവില്ല. ഇവര്‍ചെയ്യുന്ന അനുസരണക്കേട് പോലും ഇവരുടെ സൃഷ്ടി വ്യവസ്ഥയിലെ അനുസരണത്തിന്റെ ഭാഗമാണെന്നര്‍ഥം.
അല്ലാഹുവല്ലാതെ ആരും തന്നെ രക്ഷാധികാരികളായി ഇല്ലെന്ന് ഖണ്ഡിതമായി പ്രഖ്യാപിക്കുന്ന ഖുർആൻ വാക്യങ്ങൾ(2:107, 29:22)ക്കെതിരല്ലേ ഐഹിക ജീവിതത്തിലും പരലോകത്തിലും ഞങ്ങൾ നിങ്ങളുടെ രക്ഷാധികാരികളാകുന്നു(41:31)വെന്ന് മലക്കുകൾ പറയുമെന്ന് പ്രഖ്യാപിക്കുന്ന ഖുർആൻ വാക്യം? നിനക്കറിഞ്ഞുകൂടെ അല്ലാഹുവിന് തന്നെയാണ് ആകാശ ഭൂമികളുടെ ആധിപത്യമെന്നും, നിങ്ങള്‍ക്ക് അല്ലാഹുവിനെ കൂടാതെ ഒരു രക്ഷകനും സഹായിയും ഇല്ലെന്നും.(2:107) ഭൂമിയിലാകട്ടെ ആകാശത്താകട്ടെ നിങ്ങള്‍ക്ക് (അവനെ) തോല്‍പ്പിക്കാനാവില്ല. നിങ്ങള്‍ക്ക് അല്ലാഹുവിന് പുറമേ ഒരു രക്ഷാധികാരിയോ സഹായിയോ ഇല്ല.(29:22) അല്ലാഹുവിന്റെ പരമാധികാരത്തെ കുറിക്കുന്ന സൂക്തങ്ങളാണിവ. രക്ഷാധികാരിയെന്ന് പരിഭാഷ നല്‍കിയിരിക്കുന്നത് വലിയ്യ് എന്നപദത്തിനാണ്. രക്ഷിതാവ്, ബന്ധു, മിത്രം, എന്നെല്ലാം ഈ പദത്തിനര്‍ത്ഥമുണ്ട്. ‘ഔലിയാഅ്‘ എന്നാണ് ഇതിന്റെ ബഹുവചനം. പരമമായ അര്‍ഥത്തില്‍ അല്ലാഹു മാത്രമാണ് മനുഷ്യരുടെ രക്ഷാധികാരി. അവന്‍ നല്‍കുന്ന രക്ഷയെതടയുവാനോ ശിക്ഷയെ നിയന്ത്രിക്കുവാനോ ആര്‍ക്കും കഴിയില്ല. മനുഷ്യര്‍ പരസ്പരം സഹായിക്കുന്നതും രക്ഷിക്കുന്നതുമെല്ലാം അല്ലാഹുവിന്റെ പരമാധികാരത്തിന് വിധേയമായിട്ടാണ്. അവന്‍ അഭയം നല്‍കുന്നു അവനെതിരായി (എവിടെ നിന്നും)അഭയം ലഭിക്കുകയില്ല (23:88) എന്ന ഖുര്‍ആന്‍ വചനം വ്യക്തമാക്കുന്നത് ഈ ആശയമാണ്. എന്നാല്‍ മനുഷ്യര്‍ക്കു തമ്മില്‍ സഹായിക്കുവാന്‍ കഴിയില്ലെന്നോ രക്ഷിക്കാനാകില്ലെന്നോ ഖുര്‍ആനിലൊരിടത്തും പറയുന്നില്ല. യഹൂദരും ക്രൈസ്തവരും പരസ്പരം സഹായികളാണെന്നും (5:51)സത്യനിഷേധികള്‍ അന്യോന്യം മിത്രങ്ങളാണെന്നും (8:73) അക്രമകാരികളില്‍ ചിലര്‍ ചിലര്‍ക്ക് രക്ഷാകര്‍ത്താക്കളാണെന്നും (45:19) മുഹാജിറുകളും അന്‍സാറുകളും പരസ്പരം ഉറ്റ മിത്രങ്ങളാണെന്നും(8:72) സത്യവിശ്വാസികളായ പുരുഷന്‍മാരും സ്ത്രീകളും അന്യോന്യം മിത്രങ്ങളാണെന്നു (9:71)മെല്ലാമുള്ള പരാമര്‍ശങ്ങള്‍ ഖുര്‍ആനില്‍ കാണാവുന്നതാണ്. ഈ വചനങ്ങളിലെല്ലാം വലിയ്യ് എന്ന പദത്തിനുതന്നെയാണ് സഹായി, മിത്രം, രക്ഷാ കര്‍ത്താവ് തുടങ്ങിയ പരിഭാഷകള്‍ നല്‍കപ്പെട്ടിരിക്കുന്നത്. മനുഷ്യര്‍ക്ക് പരസ്പരം മിത്രങ്ങളും രക്ഷകരുമാകാന്‍ കഴിയുമെന്ന വസ്തുത ഖുര്‍ആന്‍ അംഗീകരിക്കുന്നുണ്ട്. എന്നാല്‍ പരമമായ വലിയ്യ് അല്ലാഹുവാണ്. അഭൗതിക രീതിയില്‍ സഹായിക്കുകയും രക്ഷിക്കുകയും ചെയ്യുന്നവനാണ് യഥാര്‍ഥത്തിലുള്ള വലിയ്യ്. മറ്റുള്ളവരെല്ലാം അവനെ ആശ്രയിച്ചുകൊണ്ട് മുന്നോട്ട് പോകുന്ന ഔലിയാക്കളാണ്. ഇതേ പോലെത്തന്നെയാണ് മാലാഖമാരും. അവര്‍ സത്യവിശ്വാസികളുടെ സഹായികളും ഉറ്റ മിത്രങ്ങളുമാണ്. ഇഹലോകത്ത് സത്യവിശ്വാസികളെ സന്‍മാര്‍ഗത്തിലൂടെ മുന്നോട്ട് പോകാനും പരലോകത്ത് അല്ലാഹുവിന്റെ അനുഗ്രഹത്തിന് പാത്രമാകുവാനും മലക്കുകള്‍ സഹായിച്ചുകൊണ്ടിരിക്കുന്നു. ഇതാണ് ഐഹിക ജീവിതത്തിലും പരലോകത്തിലും ഞങ്ങള്‍ നിങ്ങളുടെ മിത്രങ്ങളാകുന്നു (41:31) എന്ന് സ്വര്‍ഗപ്രവേശത്തിന് അര്‍ഹരായ സദ്‌വൃത്തരോടായി മലക്കുകള്‍ പറയുന്നതിന്റെ താല്‍പര്യം. മലക്കുകളുടെ സഹായം അല്ലാഹുവിന്റെ കല്‍പന പ്രകാരമാണെന്ന വസ്തുതയും ഖുര്‍ആന്‍ വ്യക്തമാക്കുന്നുണ്ട്. മനുഷ്യന് അവന്റെ മുമ്പിലൂടെയും പിന്നിലൂടെയും തുടരെത്തുടരെ വന്നുകൊണ്ട് അല്ലാഹുവിന്റെ കല്‍പന പ്രകാരം അവനെ കാത്തു സൂക്ഷിച്ചുകൊണ്ടിരിക്കുന്നവര്‍ (മലക്കുകള്‍) ഉണ്ട് (13:11). അല്ലാഹു വിധിച്ച രക്ഷയും ശിക്ഷയും നടപ്പാക്കുകയാണ് അവന്റെ ഹിതപ്രകാരം മാത്രം പ്രവര്‍ത്തിക്കുവാന്‍ കഴിയുന്ന മലക്കുകള്‍ ചെയ്യുന്നത്. അതിനാല്‍ത്തന്നെ അല്ലാഹു മാത്രമാണ് രക്ഷാധികാരി എന്ന വചനങ്ങളും മലക്കുകള്‍ നല്‍കുന്ന സംരക്ഷണത്തെപ്പറ്റി സൂചിപ്പിക്കുന്ന സൂക്തങ്ങളും തമ്മില്‍ യാതൊരു വിധ വൈരുധ്യങ്ങളുമില്ല.

ലിംഗനിര്‍ണയവുമായി ബന്ധപ്പെട്ട ക്വുര്‍ആന്‍ പരാമര്‍ശങ്ങള്‍ ശ്രദ്ധിക്കുക.

''ആണ്‍, പെണ്‍ എന്നീ രണ്ട് ഇണകളെ അവനാണ് സൃഷ്ടിച്ചതെന്നും.  ഒരു ബീജം സ്രവിക്കപ്പെടുമ്പോള്‍ അതില്‍ നിന്ന്.'' (53: 45-46) (1)

''പിന്നെ അവന്‍ ഒരു ഭ്രൂണമായി. എന്നിട്ട് അല്ലാഹു (അവനെ) സൃഷ്ടിച്ചു സംവിധാനിച്ചു. അങ്ങനെ അതില്‍ നിന്ന് ആണും പെണ്ണുമാകുന്ന രണ്ടു ഇണകളെ അവന്‍ ഉണ്ടാക്കി. അങ്ങനെയുള്ളവന്‍ മരിച്ചവരെ ജീവിപ്പിക്കാന്‍ കഴിവുള്ളവനല്ലെ?'' (75: 38-40)(2)

ഹദീഥുകളിലാണ് ലിംഗനിര്‍ണയത്തെപ്പറ്റി കുറേക്കൂടി വ്യക്തമായ പരാമര്‍ശമുള്ളത്.

  1. അനസില്‍ നിന്ന്: പ്രവാചകന്‍ മദീനയില്‍ വന്ന വിവരം അബ്ദുല്ലാഹിബ്‌നു സലാമിനു കിട്ടി. അദ്ദേഹം നബിയുടെ അടുത്തുവന്ന് പറഞ്ഞു: 'ഒരു പ്രവാചകനു മാത്രം അറിയാവുന്ന മൂന്നു കാര്യങ്ങള്‍ ഞാന്‍ താങ്കളോട് ചോദിക്കുകയാണ്..... ഇനി കുട്ടിക്ക് സാദൃശ്യം ലഭിക്കുന്ന കാര്യം; പുരുഷന്‍ സ്ത്രീയുമായി വേഴ്ച നടത്തുന്ന വേളയില്‍ അവന്റെ സ്രവം അവളുടെ സ്രവത്തെ അതിജയിച്ചാല്‍ കുട്ടിക്ക് സാദൃശ്യം അയാളോടായി. അവളുടെ സ്രവം അവന്റെ സ്രവത്തെയാണ് അതിജയിക്കുന്നതെങ്കില്‍ അവളോടും.' അബ്ദുല്ല പറഞ്ഞു: 'താങ്കള്‍ അല്ലാഹുവിന്റെ ദൂതനാണെന്നു ഞാന്‍ സാക്ഷ്യപ്പെടുത്തുന്നു.'(3)
  2. അനസ് ബ്‌നുമാലികി(റ)ല്‍ നിന്ന്: പുരുഷന് സ്വപ്‌നസ്ഖലനമുണ്ടാവുന്നതുപോലെ സ്ത്രീക്കും സ്വപ്‌നസ്ഖലനമുണ്ടായാല്‍ അവള്‍ എന്താണ് ചെേയ്യണ്ടത് എന്നതിനെ സംബന്ധിച്ച് ഉമ്മുസുലൈം പ്രവാചകനോട് ചോദിച്ചു. ...........നിശ്ചയമായും പുരുഷന്റെ‚ഇന്ദ്രിയം വെളുത്തതും കട്ടിയുള്ളതുമാണ്. സ്ത്രീയുടെ ഇന്ദ്രിയം മഞ്ഞനിറമുള്ളതും നേര്‍മയുള്ളതുമാണ്. ഏത് മുകളില്‍ വരുന്നുവോ അല്ലെങ്കില്‍ മുന്‍കടക്കുന്നുവോ അതിനോടാണ് കുട്ടിക്ക് സാദൃശ്യമുണ്ടാവുക.'(4)
  3. നബി (സ) സ്വാതന്ത്ര്യം നല്‍കിയ ഥൗബാനി(റ)ല്‍ നിന്ന്: ഞാന്‍ നബി(സ)യുടെ അടുക്കല്‍ നില്‍ക്കുമ്പോള്‍ƒജൂത പണ്ഡിതന്‍മാരില്‍ നിന്നുള്ള ഒരു പണ്ഡിതന്‍ വരികയും 'അസ്സലാമു അലൈക്ക യാ മുഹമ്മദ് (മുഹമ്മദ്, നിനക്ക് സമാധാനമുണ്ടാകട്ടെ)' എന്ന് പറയുകയും ചെയ്തു. ............ അയാള്‍ തുടര്‍ന്നു പറഞ്ഞു: 'ഭൂനിവാസികളില്‍നിന്നും ഒരു പ്രവാചകനോ അല്ലെങ്കില്‍ ഒന്നോ രണ്ടോ ആളുകള്‍ക്കോ അല്ലാതെ മറ്റൊരാക്കും അറിയാത്ത ഒരു കാര്യത്തെ സംബന്ധിച്ച് ചോദിക്കുവാനാണ് ഞാന്‍ വന്നിട്ടുള്ളത്.' നബി (സ) ചോദിച്ചു: 'ഞാനത് പറഞ്ഞാ ല്‍ നിനക്കത് ഉപകരിക്കുമോ?'. 'ഞാന്‍ എന്റ ചെവികള്‍ കൊണ്ട് കേള്‍ക്കും'. അയാള്‍ പറഞ്ഞു: '(പ്രസവിക്കപ്പെടുന്ന) ശിശുവിനെക്കു റിച്ച് ചോദിക്കുവാനാണ് ഞാന്‍ വന്നത്' നബി (സ) പറഞ്ഞു: 'പുരുഷന്റെ‚ ഇന്ദ്രിയം വെളുത്ത നിറത്തിലുളളതും സ്ത്രീയുടെ ഇന്ദ്രിയം മഞ്ഞനിറത്തിലുള്ളതുമാണ്. അത് രണ്ടും ഒരുമിച്ച് ചേരുകയും പുരുഷ ഇന്ദ്രിയം സ്ത്രീ ഇന്ദ്രിയത്തെ അതിജയിക്കുകയും ചെയ്താല്‍ അല്ലാഹുവിന്റെ അനുമതിയോടെ അത് ആണ്‍ കുട്ടിയായിതീരുന്നു. സ്ത്രീയുടെ ഇന്ദ്രിയം പുരുഷ ഇന്ദ്രിയത്തെ അതിജയിച്ചാല്‍ അല്ലാഹു വിന്റെ‚അനുമതിയോടെ അത് പെണ്‍കുട്ടിയായി തീരുന്നു.' ജൂതന്‍ പറഞ്ഞു: 'തീര്‍ച്ചയായും താങ്കള്‍ പറഞ്ഞത് സത്യമാണ്. തീര്‍ച്ചയായും താങ്കള്‍ ഒരു പ്രവാചകന്‍ തന്നെയാണ്'. പിന്നെ അയാള്‍ തിരിച്ചുപോയി. അപ്പോള്‍ നബി (സ) പറഞ്ഞു: 'അയാള്‍ എന്നോടു ചോദിച്ച കാര്യങ്ങളെക്കുറിച്ചൊന്നും അല്ലാഹു അറിയിച്ചുതരുന്നതുവരെ എനിക്ക് യാതൊരു വിവരവും ഉണ്ടായിരുന്നില്ല.'(5)
  4. ഹുദൈഫത്ത് ബ്‌നുഅസീദി(റ)ണ്‍ നിന്ന്: നബി (സ) പറഞ്ഞു: 'ഗര്‍ഭാശയത്തിണ്‍ ബീജം നാല്‍പത് ദിവസം അല്ലെങ്കില്‍ നാല്‍പത്തഞ്ച് ദിവസം ആയിത്തീരുമ്പോള്‍ അതിന്‍മേല്‍ ഒരു മലക്ക് പ്രവേശിക്കും. എന്നിട്ടവന്‍ ചോദിക്കും: രക്ഷിതാവേ, ദൗര്‍ഭാഗ്യവാനോ അതോ സൗഭാഗ്യവാനോ? എന്നിട്ട് അത് രേഖപ്പെടുത്തും. പിന്നെ ചോദിക്കും: രക്ഷിതാവേ, ആണോ അതോ പെണ്ണോ? എന്നിട്ട് അതും രേഖപ്പെ ടുത്തും. അവന്റെ കര്‍മവും അവന്റെ‚ ഫലവും, അവന്റെ‚അവധിയും, അവന്റെ‚ ഉപജീവനവും എഴുതപ്പെടും. പിന്നീട് ഏടുകള്‍ ചുരുട്ടപ്പെടും. അതില്‍ ഒന്നും വര്‍ദ്ധിപ്പിക്കപ്പെടുകയില്ല; ഒന്നും ചുരുട്ടപ്പെടുകയുമില്ല.'(6)
  5. അബ്ദാഹി ബ്‌നുമസ്ഊദി(റ)ല്‍ നിന്ന്: നബി (സ) പറയുന്നത് ഞാന്‍ കേട്ടു: 'ബീജത്തിന്‍മേല്‍ നാല്‍പത്തിരണ്ട് ദിവസം കഴിഞ്ഞാല്‍ അല്ലാഹു ഒരു മലക്കിനെ നിയോഗിക്കും. എന്നിട്ട് അവന്‍ അതിനെ രൂപപ്പെടുത്തുകയും, അതിന് കേള്‍വിയും കാഴ്ചയും ചര്‍മവും മാംസവും അസ്ഥിയും രൂപപ്പൈടുത്തുകയും ചെയ്യും. പിന്നീട് ആ മലക്ക് ചോദിക്കും: രക്ഷിതാവേ, ആണോ അതോ പെണ്ണോ? അപ്പോള്‍ നിന്റെ രക്ഷിതാവ് അവന്‍ ഉദ്ദേശിക്കുന്നത് വിധിക്കും. മലക്ക് അത് രേഖപ്പെടുത്തും. പിന്നീട് മലക്ക് ചോദിക്കും: രക്ഷിതാവേ ഇവന്റെ അവധി? അപ്പോള്‍ നിന്റെ രക്ഷിതാവ് അവന്‍ ഉദ്ദേശിച്ചത് പറയുകയും മലക്ക് അത് രേഖപ്പെടുത്തുകയും ചെയ്യും. പിന്നെ മലക്ക് ചോദി ക്കും: രക്ഷിതാവേ, ഇവന്റെ ഉപജീവനം? അപ്പോള്‍ നിന്റെ രക്ഷിതാവ് അവന്‍ ഉദ്ദേശിച്ചത് വിധിക്കുകയും മലക്ക് അത് രേഖപ്പെടുത്തു കയും ചെയ്യും. പിന്നീട് മലക്ക് തന്റെ‚കയ്യില്‍ ആ ഏടുമായി പോകും. കല്‍പിക്കപ്പെട്ടതിനേക്കാള്‍ വര്‍ദ്ധിപ്പിക്കുകയോ ചുരുക്കുകയോ ഇല്ല.'(7)
  6. അനസ് ബ്‌നുമാലികില്‍ (റ) നിന്ന്: നബി (സ) പറഞ്ഞു: 'പ്രതാപവാനും മഹാനുമായ അല്ലാഹു ഗര്‍ഭാശയത്തിന്റെ കാര്യം ഒരു മലക്കിനെ ഏല്‍പിച്ചിട്ടുണ്ട്. ആ മലക്ക് പറയും: രക്ഷിതാവേ, ബീജമാണ്. രക്ഷിതാവേ സിക്താണ്ഡമാണ്. രക്ഷിതാവേ മാംസപിണ്ഡമാണ്. അല്ലാഹു ഒരു സൃഷ്ടിയില്‍ വിധിക്കാന്‍ ഉദ്ദേശിച്ചാല്‍ മലക്ക് പറയും: രക്ഷിതാവേ, ആണോ പെണ്ണോ? ദൗര്‍ഭാഗ്യവാനോ അതോ സൗഭാഗ്യവാനോ? ഉപജീവനം എങ്ങനെയാണ്? അവധി എത്രയാണ്? അങ്ങനെ അവയെല്ലാം തന്റെ മാതാവിന്റെ വയറ്റിലായിരിക്കെ തന്നെ രേഖപ്പെടുത്ത പ്പെടും.(8)
  7. (നബി(സ)യോട് ചോദിക്കപ്പെട്ടു:) സ്വപ്‌നസ്ഖലനമുണ്ടായാല്‍ സ്ത്രീ കുളിക്കേണ്ടതുണ്ടോ? അപ്പോള്‍ നബി(സ) പറഞ്ഞു: 'അതെ; അവള്‍ ഇന്ദ്രിയം കണ്ടാല്‍'. അപ്പോള്‍ ഉമ്മുസുലൈം (റ) ചോദിച്ചു: 'സ്ത്രീക്ക് സ്ഖലനമുണ്ടാകുമോ?' അപ്പോള്‍ അദ്ദേഹം പറഞ്ഞു: 'എന്തൊരു കഷ്ടം! പിന്നെ? എങ്ങനെയാണ് കുട്ടിക്ക് അവളോട് സാദൃശ്യം ഉണ്ടാകുന്നത്?' മറ്റൊരു നിവേദനത്തില്‍ ആഇശ (റ) ഉമ്മുസുലൈം(റ)യോട് 'ഛെ! സ്ത്രീക്ക് അതുണ്ടാകുമോ?' എന്ന് ചോദിച്ചുവെന്നാണുള്ളത്. മറ്റൊരു റിപ്പോര്‍ട്ടില്‍, ഈ ഹദീഥിന്റെ‚അവസാന ഭാഗത്ത് ഇങ്ങനെ കൂടിയുണ്ട്. 'ഇന്ദ്രിയം കാരണമായിട്ടാണ് കുട്ടിക്ക് സാദൃശ്യമുണ്ടാകുന്നത്. സ്ത്രീയുടെ ഇന്ദ്രിയം പുരുഷന്റെ ഇന്ദ്രിയത്തിന് മുകളില്‍ വന്നാല്‍ കുട്ടിക്ക് മാതൃ സഹോദരന്‍മാരോട് സാദൃശ്യമുണ്ടാകും. പുരുഷന്റെ‚ ഇന്ദ്രിയം സ്ത്രീയുടെ ഇന്ദ്രിയത്തിന് മുകളില്‍ വന്നാല്‍ കുട്ടിക്ക് അവന്റെ പിതൃവ്യന്‍മാരോട് സാദൃശ്യമുണ്ടാകും.'(9)

മുകളില്‍ പറഞ്ഞ അല്ലാഹുവിന്റെയും അവന്റെ ദൂതന്റെയും വചനങ്ങളിലൊന്നും തന്നെ ആശാസ്ത്രീയമായ പരാമര്‍ശങ്ങളൊന്നുമില്ല. ലിംഗനിര്‍ണയത്തെപ്പറ്റിയുള്ള ഏറ്റവും പുതിയ വിവരങ്ങളുമായിപ്പോലും അവ പൂര്‍ണമായും യോജിച്ചു വരുന്നുവെന്നത് അത്ഭുതകരം തന്നെയാണ്.

  1. സ്രവിക്കപ്പെടുന്ന ശുക്ലത്തില്‍ നിന്നാണ് ആണും പെണ്ണുമുണ്ടാകുന്നതെന്ന് സൂറത്തുല്‍ ഖിയാമയിലെ 38 മുതല്‍ 40 വരെയുള്ള വചനങ്ങ ളില്‍ പറയുന്നു. ശുക്ലദ്രാവകത്തിലെ പുരുഷബീജം X ക്രോമസോം വഹിക്കുന്നതാണെങ്കില്‍ അത് അണ്ഡവുമായി ചേര്‍ന്നാല്‍ പെണ്‍കു ഞ്ഞും Y ക്രോമസോം വഹിക്കുന്നതാണെങ്കില്‍ അത് അണ്ഡവുമായി ചേര്‍ന്നാല്‍ ആണ്‍കുഞ്ഞുമുണ്ടാകുന്നു. ശുക്ലദ്രാവകമാണ് കുഞ്ഞ് ആണോ പെണ്ണോ എന്നു തീരുമാനിക്കുന്നത് എന്നര്‍ത്ഥം.
  2. സ്രവിക്കപ്പെടുന്ന ബീജത്തില്‍ തന്നെ ആണ്‍, പെണ്‍ എന്നിവയുണ്ടെന്നും അതാണ് ആണ്‍-പെണ്‍ ഇണകളുടെ ഉല്‍പത്തിക്ക് കാരണമാകുന്ന തെന്നും സൂറത്തുന്നജ്മിലെ 45,46 വചനങ്ങള്‍ വ്യക്തമാക്കുന്നു. സ്രവിക്കപ്പെടുന്ന ബീജത്തില്‍ തന്നെ ആണ്‍ ക്രോമസോമായ Yയെ വഹി ക്കുന്ന ബീജാണുക്കളും പെണ്‍ക്രോമസോമായ Xനെ വഹിക്കുന്ന ബീജാണുക്കുളുമുണ്ട്. ബീജദ്രാവകത്തിലെ Y ആണ്‍ബീജം അണ്ഡവു മായി ചേര്‍ന്നാല്‍ ആണ്‍കുട്ടിയും X പെണ്‍ബീജമാണ് അണ്ഡവുമായി ചേരുന്നതെങ്കില്‍ പെണ്‍കുട്ടിയുമാണുണ്ടാവുക.
  3. അനസില്‍ നിന്ന് ബുഖാരി നിവേദനം ചെയ്ത അബ്ദുല്ലാഹിബ്‌നു സലാമുമായി പ്രവാചകന്‍ (സ) നടത്തിയ സംഭാഷണത്തെക്കുറിച്ച് വിവരിക്കുന്ന ഹദീഥിലും അദ്ദേഹത്തില്‍ നിന്നുതന്നെ മുസ്‌ലിം നിവേദനം ചെയ്ത സ്വപ്നസ്ഖലനത്തെക്കുറിച്ച ഹദീഥിലും ഥൗബാ നി(റ)ല്‍ നിന്ന് മുസ്‌ലിം നിവേദനം ചെയ്ത ജൂതപുരോഹിതനു നല്‍കിയ മറുപടികയെക്കുറിച്ച് വിശദീകരിക്കുന്ന ഹദീഥിലും പുരുഷ ബീജം സ്ത്രീബീജത്തെ അതിജയിച്ചാല്‍ ആണ്‍കുഞ്ഞും, സ്ത്രീബീജം പുരുഷബീജത്തെയാണ് അതിജയിക്കുന്നതെങ്കില്‍ പെണ്‍കുട്ടിയുമാ ണുണ്ടാവുകയെന്ന് പ്രവാചകന്‍ (സ) പറഞ്ഞതായി ഉദ്ധരിച്ചിരിക്കുന്നു. ഈ പരാമര്‍ശത്തെ സുരതക്രിയയില്‍ പുരുഷനാണ് ആദ്യം സ്ഖലിക്കുന്നതെങ്കില്‍ ആണ്‍കുട്ടിയും സ്ത്രീക്കാണ് ആദ്യം സ്ഖലിക്കുകയെങ്കില്‍ പെണ്‍കുട്ടിയുമാണുണ്ടാവുകയെന്നാണ് പല പണ്ഡിത ന്‍മാരും വ്യാഖ്യാനിച്ചിരിക്കുന്നത്. രതിമൂര്‍ച്ചയോടനുബന്ധിച്ച് ചില സ്ത്രീകള്‍ക്ക് പാരായൂറിത്രല്‍ നാളിയില്‍ നിന്ന് പുറത്തേക്കുവ രുന്ന ദ്രാവകത്തിന് കുഞ്ഞിന്റെ ജനനത്തില്‍ യാതൊരു പങ്കുമില്ല എന്ന് ഇന്ന് നമുക്കറിയാം. പെണ്ണിന്റെ സ്ഖലനത്തിന് കുഞ്ഞിന്റെ ഉല്‍പത്തി പ്രക്രിയയില്‍ യാതൊരു പങ്കും വഹിക്കുവാനില്ലെങ്കില്‍ അതോടനുബന്ധിച്ചുണ്ടാകുന്ന ദ്രാവകം ആദ്യമോ പിന്നെയോ ഉണ്ടാകുന്നതെന്നത് ലിംഗനിര്‍ണയത്തെ ബാധിക്കുവാന്‍ സാധ്യതയൊന്നുമില്ല. ഈ ഹദീഥുകളില്‍ ബീജത്തിന്റെ അധീശത്വത്തെക്കുറിക്കു വാന്‍ പ്രയോഗിച്ചിരിക്കുന്നത് 'സബഖ'യെന്നും 'അലാ' എന്നുമുള്ള ക്രിയകളാണ്. ഒന്നിനുമേല്‍ മറ്റൊന്ന് മുന്‍കടക്കുന്നതിനോ ആദ്യമാകു ന്നതിനോ വിജയിക്കുന്നതിനോ അധികാരം സ്ഥാപിക്കുന്നതിനോ ആണ് 'സബഖ'യെന്നു പറയുകയെന്ന് അംഗീകൃത ഭാഷാ നിഘണ്ടുക്കള്‍ പരിശോധിച്ചാല്‍ വ്യക്തമാകും(10).

ഒന്നിനുമുകളില്‍ മറ്റൊന്ന് ആധിപത്യം സ്ഥാപിക്കുന്നതിനാണ് 'അലാ'യെന്ന് പ്രയോഗിക്കുകയെന്ന് ക്വുര്‍ആനില്‍നിന്നു തന്നെ വ്യക്തമാകു ന്നുണ്ട്. സൂറത്തുല്‍ മുഅ്മിനൂനിലെ 91-ാം വചനം നോക്കുക.

''അല്ലാഹു യാതൊരു സന്താനത്തെയും സ്വീകരിച്ചിട്ടില്ല. അവനോടൊപ്പം യാതൊരു ദൈവവുമുണ്ടായിട്ടില്ല. അങ്ങനെയായിരുന്നുവെങ്കില്‍ ഓരോ ദൈവവും താന്‍ സൃഷ്ടിച്ചതുമായി പോയിക്കളയുകയും, അവരില്‍ ചിലര്‍ ചിലരെ അടിച്ചമര്‍ത്തുകയും ചെയ്യുമായിരുന്നു. അവര്‍ പറഞ്ഞുണ്ടാക്കുന്നതില്‍ നിന്നെല്ലാം അല്ലാഹു എത്ര പരിശുദ്ധന്‍!'' (23: 91)(11)

ഈ വചനത്തില്‍ 'ചിലര്‍ ചിലരെ അടിച്ചമര്‍ത്തുകയും ചെയ്യുമായിരുന്നു'വെന്നു പരിഭാഷപ്പെടുത്തിയിരിക്കുന്നത് 'വ ലഅലാ ബഅദുഹും അലാ ബഅദിന്‍' എന്ന പ്രയോഗത്തെയാണ്. 'അലാ'യെന്നാല്‍ ആധിപത്യം സ്ഥാപിക്കുക, അടിച്ചമര്‍ത്തുക എന്നിങ്ങനെയാണ് യഥാര്‍ത്ഥത്തിലുള്ള സാരമെന്നര്‍ത്ഥം.

പുരുഷബീജത്തിലെ Y പെണ്‍ബീജത്തിലെ Xനുമേല്‍ ആധിപത്യം സ്ഥാപിക്കുമ്പോഴാണ് ആണ്‍കുഞ്ഞുണ്ടാകുന്നത് എന്നും പെണ്‍ബീജത്തിലെ X പുരുഷബീജത്തിലെ Yക്കുമേല്‍ ആധിപത്യം സ്ഥാപിക്കുമ്പോഴാണ് പെണ്‍കുഞ്ഞുണ്ടാകുന്നത് എന്നുമുള്ള ജനിതകശാസ്ത്ര വസ്തുതകളു മായി ഈ ഹദീഥുകള്‍ പൂര്‍ണമായും പൊരുത്തപ്പെടുന്നു. മധ്യകാലഘട്ടത്തിലുണ്ടായിരുന്ന അറിവിന്റെ അടിസ്ഥാനത്തില്‍ ഹദീഥ് മനസ്സിലാക്കിയവര്‍ ആണ്‍സ്ഖലനം ആദ്യം നടന്നാല്‍ ആണ്‍കുഞ്ഞും പെണ്‍സ്ഖലനം നടന്നാല്‍ പെണ്‍കുഞ്ഞുമുണ്ടാകുമെന്ന് ഇതില്‍നിന്ന് മനസ്സിലാക്കിയെന്നത് നബിവചനത്തിന്റെ ആശാസ്ത്രീയതയല്ല, അറിവിന്റെ കാലനിബന്ധതയെയാണ് വെളിപ്പെടുത്തുന്നത്. 'സബഖ' യെന്ന ക്രിയയെ വ്യാഖ്യാനിച്ചാല്‍ ആദ്യമുണ്ടാകുന്നത് ഏത് ദ്രവമാണോ അതിന്റെ അടിസ്ഥാനത്തിലാണ് ലിംഗനിര്‍ണയമെന്ന് വേണമെ ങ്കില്‍ പറയാനാകുമെങ്കിലും 'അലാ'യെന്ന പ്രയോഗം അത്തരമൊരു വ്യാഖ്യാനത്തിന് പഴുതുകളൊന്നും നല്‍കുന്നില്ല. ഈ ഹദീഥുകളെ ഒന്നിച്ചു പരിഗണിച്ചുകൊണ്ട്, നിലനില്‍ക്കുന്ന അറിവിന്റെ അടിസ്ഥാനത്തില്‍ വ്യാഖ്യാനിച്ചപ്പോഴാണ് പുരുഷ-പെണ്‍ സ്ഖലനങ്ങളുടെ ക്രമമാണ് ലിംഗനിര്‍ണയത്തിന് നിദാനമെന്നാണ് ഈ ഹദീഥുകള്‍ പഠിപ്പിക്കുന്നതെന്ന നിഗമനത്തില്‍ വ്യാഖ്യാതാക്കള്‍ എത്തിച്ചേര്‍ന്നത്. ഹദീഥുകളെ മൊത്തത്തിലെടുത്ത് പരിശോധിച്ചാല്‍ ഒരു ദ്രവത്തിനു മേലുള്ള മറ്റേ ദ്രവത്തിന്റെ ആധിപത്യം തന്നെയാണ് അവയില്‍ വ്യക്തമാക്കപ്പെട്ടിരിക്കുന്നതെന്ന് മനസ്സിലാകും. കഴിഞ്ഞ നൂറ്റാണ്ടിന്റെ പകുതിയില്‍ മാത്രം ശാസ്ത്രലോകത്തിന് മനസ്സിലായ ബീജത്തി ന്റെ ആധിപത്യമാണ് ലിംഗനിര്‍ണയത്തിന് കാരണമാകുന്നതെന്ന വസ്തുത എത്ര കൃത്യമായാണ് ഈ ഹദീഥുകള്‍ വരച്ച് കാണിക്കുന്നത്!

  1. മുസ്‌ലിം ഹുദൈഫത്തു ബ്‌നു അസീദില്‍ (റ) നിന്നും അബ്ദുല്ലാഹിബ്‌നു മസ്ഊദില്‍ (റ) നിന്നും നിവേദനം ചെയ്ത രണ്ട് വ്യത്യസ്ത ഹദീഥുകളില്‍ നിന്ന് ഗര്‍ഭസ്ഥശിശുവിലുളള ലിംഗമാറ്റത്തിനുവേണ്ടിയുള്ള മലക്ക് പ്രത്യക്ഷപ്പെടുന്നതും കുട്ടി ആണോ പെണ്ണോയെന്ന് ആത്യന്തികമായി തീരുമാനിക്കപ്പെടുന്നതും ബീജസങ്കലനത്തിന് ശേഷം നാല്‍പത് ദിവസങ്ങള്‍ക്കും നാല്‍പത്തിയഞ്ച് ദിവസങ്ങള്‍ക്കുമിടയിലാണെന്ന് വ്യക്തമാവുന്നു.

SRY ജീന്‍ പ്രവര്‍ത്തനക്ഷമമാകുന്നത് ആറാമത്തെ ആഴ്ചയാണെന്ന വിവരം നമുക്ക് ലഭിച്ചത് മൂന്നു പതിറ്റാണ്ടുകള്‍ക്ക് മുമ്പു മാത്രമാണ്. XX സിക്താണ്ഡമാണെങ്കിലും XY സിക്താണ്ഡമാണെങ്കിലും അപൂര്‍വമായുണ്ടാകുന്ന സിക്താണ്ഡങ്ങളാണെങ്കിലുമെല്ലാം അവയുടെ ലിംഗമെന്താ ണെന്ന് ആത്യന്തികമായി തീരുമാനിക്കപ്പെടുക SRY ജീന്‍ പ്രവര്‍ത്തനക്ഷമമാകുന്നതിന്റെ അടിസ്ഥാനത്തിലാണ്. ആറാമത്തെ ആഴ്ചയാണ് SRY ജീന്‍ പ്രവര്‍ത്തനക്ഷമമാവുന്നതെന്ന ഭ്രൂണശാസ്ത്രം 1985ല്‍ മാത്രം നമുക്കു പറഞ്ഞുതന്ന വിവരവും നാല്‍പതു ദിവസങ്ങള്‍ക്കും നല്‍പത്തിയഞ്ച് ദിവസങ്ങള്‍ക്കുമിടയിലാണ് ലിംഗതീരുമാനവുമായി മലക്ക് നിയോഗിക്കപ്പെടുന്നതെന്ന പതിനാലു നൂറ്റാണ്ടുകള്‍ക്ക് മുമ്പ് നബി (സ) പറഞ്ഞ വിവരവും എത്ര ക്രൃത്യമായാണ് ഇവിടെ യോജിച്ചുവരുന്നത്! എന്തുകൊണ്ടാണ് ഹദീഥുകളിലെ പരാമര്‍ശങ്ങള്‍ ഇത്ര യും കൃത്യമാകുന്നതെന്ന ചോദ്യത്തിന് ക്വുര്‍ആന്‍ തന്നെ ഉത്തരം നല്‍കിയിട്ടുണ്ട്.

''നിങ്ങളുടെ കൂട്ടുകാരന്‍ വഴിതെറ്റിയിട്ടില്ല. ദുര്‍മാര്‍ഗിയായിട്ടുമില്ല. അദ്ദേഹം തന്നിഷ്ടപ്രകാരം സംസാരിക്കുന്നുമില്ല. അത് അദ്ദേഹത്തിന് ദിവ്യസമ്പേശമായി നല്‍കപ്പെടുന്ന ഒരു ഉല്‍ബോധനം മാത്രമാകുന്നു.'' (53: 2-4)

കുറിപ്പുകൾ

  1. ക്വുര്‍ആന്‍ 53: 45-46
  2. ക്വുര്‍ആന്‍ 75: 38-40
  3. സ്വഹീഹുല്‍ ബുഖാരി, കിതാബു അഹാദീഥുല്‍ അംബിയാഅ്, ബാബു ഖല്‍ഖി ആദം വ ദുര്‍റിയ്യത്തിഹി, ഹദീഥ്
  4. സ്വഹീഹു മുസ്‌ലിം, കിതാബുല്‍ ഹൈദ്വ്, ബാബു വുജുബില്‍ ഗസ്‌ലി അലല്‍ മര്‍അത്തി ബി ഖുറൂജില്‍ മനിയ്യി മിന്‍ഹ, ഹദീഥ്
  5. സ്വഹീഹു മുസ്‌ലിം, കിതാബുല്‍ ഹൈദ്വ്, ബാബു ബയാനി സ്വിഫത്തില്‍ മനിയിര്‍റജുലി വല്‍ മര്‍അത്തി വ അന്നല്‍ വലദ മഖ്‌ലൂഖുന്‍ മിന്‍ മാഇ.
  6. സ്വഹീഹു മുസ്‌ലിം, കിതാബുല്‍ ക്വദ്ര്‍, ബാബു കൈഫിയ്യത്തില്‍ ഖല്‍ബില്‍ ആദമിയ്യി ഫീ ബദനി ഉമ്മിഹി വ കിതാബത്തി രിസ്‌കിഹി വ അജലിഹി, വ അമലിഹി വ ശകാവത്തിഹി വ സഅദത്തിഹി, ഹദീഥ്
  7. സ്വഹീഹു മുസ്‌ലിം, കിതാബുല്‍ ക്വദ്ര്‍, ബാബു കൈഫിയ്യത്തില്‍ ഖല്‍ബില്‍ ആദമിയ്യി ഫീ ബദനി ഉമ്മിഹി വ കിതാബത്തി രിസ്‌കിഹി വ അജലിഹി, വ അമലിഹി വ ശകാവത്തിഹി വ സഅദത്തിഹി, ഹദീഥ്
  8. സ്വഹീഹു മുസ്‌ലിം, കിതാബുല്‍ ക്വദ്ര്‍, ബാബു കൈഫിയ്യത്തില്‍ ഖല്‍ബില്‍ ആദമിയ്യി ഫീ ബദനി ഉമ്മിഹി വ കിതാബത്തി രിസ്‌കിഹി വ അജലിഹി, വ അമലിഹി വ ശകാവത്തിഹി വ സഅദത്തിഹി, ഹദീഥ്
  9. സ്വഹീഹു മുസ്‌ലിം, കിതാബുല്‍ ഹൈദ്വ്, ബാബു വുജുബില്‍ ഗസ്‌ലി അലല്‍ മര്‍അത്തി ബി ഖുറൂജില്‍ മനിയ്യി മിന്‍ഹ, ഹദീഥ്
  10. Edward William Lane : Arabic-English Lexicon, London, 1863, Book 1, Page 1300.
  11. ക്വുര്‍ആന്‍ 23: 91

ക്വുർആനിലെ 2: 2:233 വചനത്തില്‍ മുലകുടി പ്രായം രണ്ടു വര്‍ഷമാണെന്നും 46:15 വചനത്തില്‍ ഗര്‍ഭകാലവും മുലകുടി പ്രായവും കൂടി മുപ്പതു മാസമാണെന്നും പറഞ്ഞതിനെ താരതമ്യം ചെയ്താൽ ഗർഭകാലം ആറ് മാസമാണ് എന്നാണ് വന്നു ചേരുക. ഇത് വ്യക്തമായ അബദ്ധമല്ലേ ?

സൂറത്തുല്‍ ബക്വറയിലെ 2:233-ാം വചനത്തിലും സൂറത്തു ലുഖ്മാനിലെ പതിനാലാം വചനത്തിലും മുലകുടി പ്രായം രണ്ടു വര്‍ഷമാണെന്ന് വ്യക്തമാക്കുന്നുണ്ട്. മുലകുടി പൂർത്തിയായ്ക്കാനുദ്ദേശിക്കുന്നവർ കുഞ്ഞുങ്ങൾക്ക് രണ്ട് വര്‍ഷമാണ് മുല കൊടുക്കേണ്ടതെന്ന് ഈ വചനങ്ങൾ വ്യക്തമാക്കുന്നു. സൂറത്തുല്‍ അഹ്ക്വാഫിലെ പതിനഞ്ചാം വചനത്തിൽ "അവന്റെ ഗര്‍ഭകാലവും മുലകുടിനിര്‍ത്തലും കൂടി മുപ്പത് മാസക്കാലമാകുന്നു" എന്ന് വ്യക്തമായി പരാമർശിച്ചിട്ടുണ്ട്. ഇതിൽ നിന്ന് ഗർഭകാലം ആറ് മാസമാണ് എന്നല്ലേ മനസ്സിലാവുകയെന്നാണ് വിമർശനം. അങ്ങനെത്തന്നെയാണ് മനസ്സിലാക്കേണ്ടത് എന്നാണ് അതിനുള്ള വിശദീകരണം. ഒൻപത് മാസം കഴിഞ്ഞാണ് സാധാരണഗതിയിൽ പ്രസവം നടക്കാറുള്ളതെന്ന് ആരും പറഞ്ഞു കൊടുക്കാതെതന്നെ എല്ലാവർക്കും അറിയാവുന്ന കാര്യമാണ്. ഇക്കാര്യം മുഹമ്മദ് നബിക്ക് (സ) അറിയുമായിരുന്നില്ല എന്ന കരുതുന്നത് ശുദ്ധ അസംബന്ധമാണ്.

ഈ വചനങ്ങളിൽ നിന്ന് നാം മനസിലാക്കേണ്ടത് കുറഞ്ഞ ഗർഭകാലം ആറു മാനസമാണ് എന്നാണ്. ഇങ്ങനെ മനസ്സിലാക്കിയവരായിരുന്നു ആദ്യകാല മുസ്ലിംകൾ. ഒരു സംഭവം നോക്കുക:

വിവാഹത്തിനുശേഷം ആറുമാസങ്ങള്‍ കഴിയുന്നയുടനെ പ്രസവിച്ച ഒരു സ്ത്രീയെക്കുറിച്ച ഒരു പരാതി ഖലീഫ ഉമറിന്റെ (റ) അടുത്തെത്തി. പ്രസവിക്കപ്പെട്ട കുഞ്ഞിന് ജീവനും ആരോഗ്യവുമുള്ളതിനാല്‍ വിവാഹപൂര്‍വരതിയിലൂടെയുണ്ടായതാവണം അവരുടെ ഗര്‍ഭധാരണമെന്നും അതിനാല്‍ അവര്‍ക്ക് വ്യഭിചാരത്തിനുള്ള ശിക്ഷ നല്‍കണമെന്നുമായിരുന്നു പരാതിക്കാരുടെ പക്ഷം. പ്രശ്‌നത്തിനു പരിഹാരം തേടി പ്രവാചകാനുചരന്‍മാരുമായി ഉമര്‍ (റ) കൂടിയാലോചന നടത്തി. അബ്ദുല്ലാഹിബ്‌നു അബ്ബാസാണ് (റ) പ്രസ്തുത പ്രസവത്തെ ക്വുര്‍ആനിന്റെ അടിസ്ഥാനത്തില്‍ ന്യായീകരിച്ചത്. സൂറത്തുല്‍ ബക്വറയിലെ 2:233-ാം വചനവും സൂറത്തുല്‍ അഹ്ക്വാഫിലെ പതിനഞ്ചാം വചനവും ഉദ്ധരിച്ചുകൊണ്ട് ഈ വചനങ്ങള്‍പ്രകാരം കുറഞ്ഞ ഗര്‍ഭകാലം ആറുമാസമാണെന്ന് സ്ഥാപിക്കുകയാണ് അദ്ദേഹം ചെയ്തത്. സൂറത്തുല്‍ ബക്വറയിലെ വചനത്തില്‍ മുലകുടി പ്രായം രണ്ടു വര്‍ഷമാണെന്നും സൂറത്തുല്‍ അഹ്ക്വാഫില്‍ ഗര്‍ഭകാലവും മുലകുടി പ്രായവും കൂടി മുപ്പതു ദിവസമാണെന്നും പറഞ്ഞതിനെ താരതമ്യം ചെയ്തുകൊണ്ടാണ് പ്രവാചകാനുചരന്‍മാരിലെ ക്വുര്‍ആന്‍ വ്യാഖ്യാതാവെന്ന് അറിയപ്പെട്ടിരുന്ന ഇബ്‌നു അബ്ബാസ് (റ) കുറഞ്ഞ ഗര്‍ഭകാലം ആറു മാസമാണെന്ന് സമര്‍ത്ഥിച്ചത്. ഭരണാധികാരിയായ ഉമര്‍ (റ) അടക്കമുള്ള സ്വഹാബിമാരെല്ലാം അത് അംഗീകരിക്കുകയും കുറ്റാരോപിതയെ വെറുതെ വിടാന്‍ ഖലീഫ കല്‍പിക്കുകയും ചെയ്തു.(ഇമാം അബ്ദുര്‍റസാഖ് തന്റെ മുസന്നഫിലും (7: 352) ഇമാം സുയൂത്തി തന്റെ ദുര്‍റുല്‍ മന്‍സൂറിലും (7: 442) നാഫിഉ ബിന്‍ ജുബൈറില്‍ നിന്ന് നിവേദനം ചെയ്തത്.)

ഉഥ്മാന്റെ (റ) ഭരണകാലത്തും സമാനമായ സംഭവമുണ്ടായതായി ഇമാം മാലിക് (റ) നിവേദനം ചെയ്യുന്നുണ്ട്. ആറാം മാസം കഴിഞ്ഞയുടനെ ആരോഗ്യമുള്ള കുഞ്ഞിനെ പ്രസവിച്ച സ്ത്രീക്ക് വ്യഭിചാരക്കുറ്റത്തിന് ശിക്ഷ വിധിച്ച ഖലീഫയെ തിരുത്തിയത് അലി(റ)യാണ്. നടേ പറഞ്ഞ ആയത്തുകള്‍ ഉദ്ധരിച്ചുകൊണ്ട് ഖലീഫയുടെ വിധിയെ വിമര്‍ശിച്ചത് ഉഥ്മാന്‍ (റ) അംഗീകരിക്കുകയും സ്ത്രീയെ വെറുതെ വിടുകയും ചെയ്തു.(ഇമാം മാലികിന്റെ മുവത്വ 41: 11)

കുറഞ്ഞ ഗര്‍ഭകാലമെത്രയാണെന്ന കാര്യത്തില്‍ പ്രവാചകാനുചരന്‍മാരുടെ കാലം മുതല്‍ മുസ്‌ലിം ലോകത്ത് കാര്യമായ തര്‍ക്കങ്ങളുണ്ടായിട്ടില്ല. നാലു കര്‍മശാസ്ത്ര സരണികളും കുറഞ്ഞ ഗര്‍ഭകാലം ആറുമാസമാണെന്ന് അംഗീകരിക്കുന്നു. പിതൃത്വവും ശിക്ഷാവിധികളുമായി ബന്ധപ്പെട്ട മദ്ഹബീ നിയമങ്ങളിലെല്ലാം ഈ അംഗീകാരത്തിന്റെ സ്വാധീനം കാണാനാവും. മുസ്‌ലിം ലോകത്ത് പതിനാലു നൂറ്റാണ്ടുകളായി അംഗീകരിക്കപ്പെട്ടുവരുന്ന കുറഞ്ഞ ഗര്‍ഭകാലം തന്നെയാണ് ശരിയെന്ന വസ്തുത അംഗീകരിക്കുകയാണ് ആധുനിക സാങ്കേതികവിദ്യകളുടെ സഹായത്തോടെ ശാസ്ത്രം ഇന്നു ചെയ്യുന്നത്. കുറഞ്ഞ ഗര്‍ഭകാലത്തെക്കുറിച്ച സംവാദങ്ങളും തര്‍ക്കങ്ങളും ഭ്രൂണശാസ്ത്രലോകത്ത് സജീവമാണെങ്കിലും നിയമപരമായി അംഗീകരിക്കാവുന്ന കുറഞ്ഞ ഗര്‍ഭകാലം ആറുമാസമാണെന്ന വസ്തുത ഇന്ന് എല്ലാവരും സമ്മതിക്കുന്നുണ്ട്.

ഗര്‍ഭാശയത്തിനുപുറത്ത് ഗര്‍ഭസ്ഥശിശുവിന് ജീവിക്കാനുള്ള കഴിവിനെയാണ് ശിശുജീവനസാമര്‍ത്ഥ്യം (Fetal Viability) എന്നുവിളിക്കുന്നത്. ഗര്‍ഭകാലത്തെ മൂന്നു ത്രൈമാസിക യൂണിറ്റുകളായാണ് (trimester) ഭ്രൂണശാസ്ത്രജ്ഞന്‍മാര്‍ പഠിക്കുന്നത്. ആദ്യത്തെ ത്രൈമാസികത്തിലാണ് ഭ്രൂണത്തില്‍ അടിസ്ഥാനപരമായ മാറ്റങ്ങളെല്ലാം ഉണ്ടാകുന്നത്. ഒന്നാം ത്രൈമാസത്തിനകത്ത് പ്രസവിക്കപ്പെട്ടാല്‍ ശിശുജീവനസാമര്‍ത്ഥ്യം പൂജ്യമായിരിക്കും. അഥവാ അങ്ങനെ പ്രസവിക്കപ്പെടുന്ന കുഞ്ഞ് ഒരു കാരണവശാലും ജീവിച്ചിരിക്കുകയില്ല. രണ്ടാം ത്രൈമാസികത്തില്‍ നടക്കുന്നത് പ്രധാനമായും അവയവങ്ങളുടെ വികാസമാണ്. രണ്ടാമത്തെ തൈമാസികം അവസാനിക്കുമ്പോള്‍ പ്രസവിക്കപ്പെടുന്ന കുഞ്ഞിന് നല്ല പരിചരണം നല്‍കിയാല്‍ അത് ജീവിക്കും. ഈ സമയത്തെ ശിശുജീവനസാമര്‍ത്ഥ്യം (Fetal Viability) 90 ശതമാനമാണ്. നല്ല പരിചരണം നല്‍കിയാല്‍ കുഞ്ഞിനെ രക്ഷിക്കുവാനും കാര്യമാത്രപ്രസക്തമായ വൈകല്യങ്ങളൊന്നുമില്ലാതെ നിലനിര്‍ത്തുവാനും കഴിയുന്ന പ്രായമാണിത് എന്നര്‍ത്ഥം.

ഗര്‍ഭസ്ഥ ശിശുവിന് ഇരുപത്തിരണ്ടാമത്തെ ആഴ്ച പ്രായമാകുന്നതുമുതല്‍ തന്നെ ശിശുജീവനസാമര്‍ത്ഥ്യത്തിന് നേരിയ സാധ്യതകളുണ്ടെന്നാണ് പുതിയ പഠനങ്ങള്‍ വ്യക്തമാക്കുന്നത്. ഇരുപത്തിമൂന്നാമത്തെ ആഴ്ച ഇത് പത്തുമുതല്‍ മുപ്പത്തിയഞ്ച് വരെ ശതമാനവും ഇരുപത്തിനാലാമത്തെ ആഴ്ച ഇത് നാല്‍പത് മുതല്‍ എഴുപത് വരെ ശതമാനവും ഇരുപത്തിയഞ്ചാമത്തെ ആഴ്ച ഇത് അമ്പത് മുതല്‍ എണ്‍പതു വരെ ശതമാനവും ഇരുപത്തിയാറാമത്തെ ആഴ്ച ഇത് എണ്‍പത് മുതല്‍ തൊണ്ണൂറുവരെ ശതമാനവും ഇരുപത്തിയേഴാമത്തെ ആഴ്ച മുതല്‍ ഇത് തൊണ്ണൂറ് ശതമാനത്തിനു മുകളിലുമാണ്. ആറു മാസങ്ങള്‍ക്ക് മുമ്പുള്ള ശിശുജീവനസാമര്‍ത്ഥ്യത്തിന്റെ ശതമാനക്കകണക്ക് ഉയരാനുള്ള കാരണം ചികിത്സാരംഗത്തും സാങ്കേതിക വിദ്യയാലുമുണ്ടായ പുരോഗതിയാണ്. ഈ പുരോഗതിയുണ്ടായിട്ട് ഏതാണ്ട് പതിറ്റാണ്ടുകളേ ആയിട്ടുള്ളൂ. 1973ലെ പ്രസിദ്ധമായ ഒരു ഗര്‍ഭഛിദ്ര കേസില്‍ പോലും അമേരിക്കന്‍ സുപ്രീം കോടതി വിധിച്ചത് ശിശുജീവനസാമര്‍ത്ഥ്യം ഇരുപത്തിയെട്ട് ആഴ്ചകളെങ്കിലും പൂര്‍ത്തിയായാലേ ഉണ്ടാവുകയുള്ളുവെന്നാണ് പൊതുവെ കരുതി വരാറുള്ളതെന്നാണ്. ഇരുപത്തിനാല് ആഴ്ചകളെങ്കിലും പൂര്‍ത്തിയായാലേ ശിശുവിന് ജീവനസാമര്‍ത്ഥ്യമുണ്ടാകൂവെന്നാണ് ഇന്ന് പൊതുവെ ചികിത്സാരംഗത്തുള്ളവര്‍ പറയാറുള്ളതെങ്കിലും അതിനേക്കാള്‍ മുമ്പ് പ്രസവിക്കപ്പെട്ടിട്ടും ജീവിച്ച റിക്കാര്‍ഡുകളുണ്ട്.

2006 ഒക്‌ടോബര്‍ 24ന് ഫ്‌ളോഡിറിയില്‍ ഇരുപത്തിരണ്ട് ആഴ്ചകള്‍ മാത്രം കഴിഞ്ഞ് ജനിച്ച അമില്ലിയ ടൈലറെന്ന പെണ്‍കുട്ടിയാണ് ഏറ്റവും കുറഞ്ഞ ഗര്‍ഭകാലം കഴിഞ്ഞ് ജീവനസാമര്‍ത്ഥ്യത്തോടെയിരിക്കുകയും പിന്നീട് വളര്‍ന്നു വലുതാവുകയും ചെയ്തയാളായി രേഖപ്പെടുത്തപ്പെട്ടിരിക്കുന്നത്. ശ്വാസകോശങ്ങള്‍ക്കും ദഹനവ്യവസ്ഥക്കും തലച്ചോറിനുമെല്ലാം നിരവധി തകരാറുകളുണ്ടായിരുന്നുവെങ്കിലും മികച്ച സാങ്കേതികവിദ്യകളുടെ സഹായത്താല്‍ ഒരു കൂട്ടം ഭിഷഗ്വരന്‍മാര്‍ ഭഗീരഥപ്രയത്‌നം നടത്തി കുട്ടിയെ ജീവനോടെ നിലനിര്‍ത്തുകയാണുണ്ടായത്. നീണ്ട നാലുമാസങ്ങളില്‍ ആശുപത്രിയിലെ ശിശു തീവ്രപരിചരണ വിഭാഗത്തില്‍ കിടത്തിയുള്ള നിരന്തരമായ പരിശ്രമങ്ങളുടെ ഫലമായാണ് അവരുടെ മാതാപിതാക്കള്‍ക്ക് ജീവനുള്ള കുഞ്ഞിനെ ലഭിച്ചത് എന്നര്‍ത്ഥം.

രണ്ടാമത്തെ ത്രൈമാസം കഴിയുമ്പോഴേക്ക് ഗര്‍ഭസ്ഥശിശുവില്‍ ഒരുവിധം എല്ലാ ബാഹ്യാവയവങ്ങളും ആന്തരാവയവങ്ങളും വളര്‍ന്നുവന്നിരിക്കുമെന്നതിനാല്‍ തന്നെ അതിനുശേഷം പ്രസവിക്കപ്പെടുന്ന കുഞ്ഞുങ്ങള്‍ ജീവിച്ചിരിക്കുവാനുള്ള സാധ്യത അഥവാ ശിശുജീവനസാമര്‍ത്ഥ്യം തൊണ്ണൂറു ശതമാനത്തിനു മുകളിലാണ്. ഗര്‍ഭാശയത്തില്‍വെച്ചു തന്നെ പൂര്‍ണ വളര്‍ച്ചയെത്തി പുറത്തുവരുന്ന കുഞ്ഞ് മാതൃശരീരത്തിനകത്ത് തന്റെ ആദ്യകോശമുണ്ടാകുന്നതു മുതല്‍ മുപ്പത്തിയൊന്‍പത് ആഴ്ചക്കാലമാണ് കഴിച്ചുകൂട്ടുന്നത്. പൂര്‍ണമായ ഗര്‍ഭകാലമാണിത്. ഇതിനുമുമ്പ് ഏതു സമയത്തും കുഞ്ഞ് പ്രസവിക്കപ്പെടാം. ഗര്‍ഭാശയത്തിനകത്തും പുറത്തും കുഞ്ഞിന് വളരാനാവശ്യമായ സംവിധാനങ്ങളെല്ലാം ചെയ്തുവെച്ചിരിക്കുന്നവനാണ് സ്രഷ്ടാവ്. മാതൃശരീരത്തില്‍ നിന്ന് പുറത്തുവരുന്ന ശിശുവിന് പിന്നെ മാതാവുമായുള്ള ജൈവികബന്ധം അതിന്റെ മുലകുടിയാണ്. മനുഷ്യശിശുവിന്റെ മുലകുടി പ്രായം രണ്ടു വര്‍ഷമാണെന്ന കാര്യത്തില്‍ ശാസ്ത്രവും ക്വുര്‍ആനും ഒരേ അഭിപ്രായമാണ് പുലര്‍ത്തുന്നത്.

പൂര്‍ണമായ മുലകുടി പ്രായം രണ്ടു വര്‍ഷമാണെന്നു പറയുമ്പോള്‍ അതിനുമുമ്പ് ഏതുസമയത്തും മാതാവിന്റെയോ കുഞ്ഞിന്റെയോ ആരോഗ്യപരമായ കാരണങ്ങളാല്‍ മുലകുടി നിന്നുപോകുവാനുള്ള സാധ്യത ക്വുര്‍ആന്‍ അംഗീകരിക്കുന്നു. മുലകുടിയോടു കൂടി ബന്ധപ്പെടുത്തിയാണ് കുറഞ്ഞ ഗര്‍ഭകാലത്തെക്കുറിച്ച് ക്വുര്‍ആന്‍ പറഞ്ഞിരിക്കുന്നതെന്ന വസ്തുത ശ്രദ്ധേയമാണ്. മുലകുടിയും ഗര്‍ഭകാലവും കൂടി മുപ്പത് മാസമാണെന്ന ക്വുര്‍ആനിക പരാമര്‍ശമാണ് ചുരുങ്ങിയ ഗര്‍ഭകാലം ആറുമാസമാണെന്ന നിഗമനത്തിലെത്താന്‍ പ്രവാചകാനുചരന്മാരെ സഹായിച്ചത്. കാര്യമായ സാങ്കേതിക സഹായങ്ങളൊന്നുമില്ലെങ്കില്‍ പോലും, ആറു മാസങ്ങള്‍ പൂര്‍ത്തിയാക്കി ഗര്‍ഭാശയത്തിനകത്തുനിന്ന് പുറത്തുവരുന്ന കുഞ്ഞിന് ജീവിക്കുവാന്‍ കഴിയുമെന്നാണ് പുതിയ പഠനങ്ങളും വ്യക്തമാക്കുന്നത്. ആറുമാസം പൂര്‍ത്തിയാക്കുന്നതോടെ ശിശുവിന്റെ ജീവനസാമര്‍ത്ഥ്യം തൊണ്ണൂറ് ശതമാനമാണെന്നാണല്ലോ പഠനങ്ങള്‍ കാണിക്കുന്നത്.

ആറു മാസങ്ങള്‍ പൂര്‍ത്തീകരിക്കുന്നതിന് ഏതാനും ദിവസങ്ങള്‍ക്ക് മുമ്പ് പ്രസവിക്കുന്ന കുഞ്ഞുങ്ങളുടെ ജീവന്‍ രക്ഷിക്കുവാന്‍ സാങ്കേതിക സഹായങ്ങളോടെ സാധിക്കുമെന്നതിനാലാണ് കഴിഞ്ഞ അരനൂറ്റാണ്ടിനിടെ ജീവനസാമര്‍ത്ഥ്യത്തില്‍ കാര്യമായ പുരോഗതിയുണ്ടായിട്ടുണ്ടെന്നു പറയുന്നത്. പ്രസ്തുത പുരോഗതിയുടെ ഫലമായി ഇരുപത്തിനാല് ആഴ്ചകളെങ്കിലും പൂര്‍ത്തിയാക്കിയ കുഞ്ഞുങ്ങളെ രക്ഷിക്കാന്‍ കഴിഞ്ഞേക്കുമെന്ന ഒരു ധാരണ ചികിത്സാരംഗത്തുണ്ടായിട്ടുണ്ട്. വിദഗ്ധരായ ചികിത്സകരുടെ മേല്‍നോട്ടത്തില്‍ ശക്തമായ സാങ്കേതിക സഹായത്തോടെയാണ് പ്രസ്തുത രക്ഷിക്കല്‍ ശ്രമം നടക്കുന്നത്. അങ്ങനെ രക്ഷപെടുന്ന കുഞ്ഞുങ്ങള്‍ വ്യത്യസ്തതരം വൈകല്യങ്ങള്‍ക്ക് വിധേയമായിരിക്കും. തലച്ചോറ് വേണ്ടത്ര വികസിച്ചിട്ടില്ലാത്തതിനാല്‍ ഓട്ടിസമടക്കമുള്ള വൈകല്യങ്ങളുണ്ടാവാനുള്ള സാധ്യതയേറെയാണ്. ശ്വാസകോശങ്ങളുടെയും കണ്ഠനാളികളുടെയും വളര്‍ച്ച പൂര്‍ത്തീകരിച്ചു കഴിഞ്ഞിട്ടില്ലാത്തതിനാല്‍ അത്തരം ശിശുക്കള്‍ക്ക് മുല കുടിക്കുവാന്‍ പലപ്പോഴും കഴിയാറില്ല. മാതൃമുലപ്പാല്‍ പിഴിഞ്ഞ് വായിലേക്ക് ഉറ്റിച്ചുകൊടുക്കുകയോ സമാന്തര പോഷകങ്ങള്‍ നല്‍കിയോ ആണ് ചികിത്സകന്‍മാര്‍ ഈ പ്രശ്‌നം പരിഹരിക്കാറുള്ളത്. ആറു മാസങ്ങള്‍ പൂര്‍ത്തിയാക്കുന്നതിന് ഏതാനും ദിവസങ്ങള്‍ക്ക് മുമ്പ് പ്രസവിക്കുന്ന കുഞ്ഞുങ്ങളുടെ ജീവന്‍ രക്ഷിക്കാന്‍ കഴിയുമെങ്കിലും മുലകുടിയടക്കമുള്ള പല ശൈശവക്രിയകളും ചെയ്യാന്‍ അവയ്ക്ക് കഴിയുകയില്ലെന്നര്‍ത്ഥം.

മാതാപിതാക്കളോടുള്ള ബാധ്യതകളെക്കുറിച്ചു പറയുമ്പോള്‍ മാതാവ് തനിക്കുവേണ്ടി സഹിച്ച ത്യാഗങ്ങളെപ്പറ്റി ഓര്‍മിപ്പിച്ചുകൊണ്ടാണ് ക്വുര്‍ആന്‍ മുലകുടി പ്രായവും ഗര്‍ഭകാലവും കൂടി മുപ്പത് മാസങ്ങളാണെന്ന് പരാമര്‍ശിക്കുന്നതെന്ന വസ്തുത ശ്രദ്ധേയമാണ്. ആറുമാസമെങ്കിലുമുള്ള പൊക്കിള്‍കൊടി ബന്ധവും രണ്ടു വര്‍ഷത്തെ മുലകുടി ബന്ധവുമാണ് മാതൃശരീരവുമായി കുഞ്ഞിനുള്ള ജൈവികബന്ധമെന്ന ക്വുര്‍ആന്‍ പരാമര്‍ശം വളര്‍ന്നു വലുതായ ശേഷമുള്ള മാതാപിതാക്കളോടുള്ള ബാധ്യതയെക്കുറിച്ച് ഓര്‍മപ്പെടുത്തുന്നതിനിടയിലാണ് കടന്നുവരുന്നത്. മുലകുടി പ്രായവും കുറഞ്ഞ ഗര്‍ഭകാലവും കൂടി മുപ്പത് മാസങ്ങളാണെന്ന ക്വുര്‍ആന്‍ പരാമര്‍ശം ശാസ്ത്രീയമായ കൃത്യത മാത്രമല്ല വൈകാരിക ബന്ധത്തിനുണ്ടാവേണ്ട ആഴവും വ്യക്തമാക്കുന്നതാണ്.

അല്ല. ക്വുർആൻ പറഞ്ഞതാണ് ശരി!

ഭ്രൂണ ഘട്ടങ്ങളെക്കുറിച്ച് പറയുന്ന സൂറത്തുൽ മുഅമിനൂനിലെ വചനത്തിൽ പതിനാലാം വചനത്തിൽ നാം വായിക്കുന്നത് 'നാം ആ മാംസപിണ്ഡത്തെ (മുദ്‌അ) അസ്ഥികൂടമായി (ഇദ്വാമ്) രൂപപ്പെടുത്തി.' എന്നാണ്.

മുദ്അയില്‍ നിന്നാണ് എല്ലുകളുണ്ടാവുന്നതെന്നാണ് ഭ്രൂണവളര്‍ച്ചയെക്കുറിച്ചു പറയുമ്പോള്‍ ക്വുര്‍ആന്‍ വ്യക്തമാക്കുന്നത്.

ഗര്‍ഭസ്ഥശിശുവിന് അസ്ഥികളുണ്ടാകുവാനാരംഭിക്കുന്നത് നാല്‍പത്തിരണ്ടു ദിവസങ്ങള്‍ക്കുശേഷമാണെന്ന് നബി (സ) പഠിപ്പിച്ചതായി അബ്ദുല്ലാഹിബ്‌നു മസ്ഊദില്‍ (റ) നിന്ന് ഇമാം മുസ്‌ലിം നിവേദനം ചെയ്തിട്ടുണ്ട്.

അസ്ഥിരൂപീകരണവുമായി ബന്ധപ്പെട്ട് ആധുനിക ശാസ്ത്രം സാങ്കേതികസഹായത്തോടെ നമുക്ക് നല്‍കുന്ന അറിവുകള്‍ ക്വുര്‍ആനും നബിവചനങ്ങളും നല്‍കുന്ന വിവരങ്ങളുമായി പൂര്‍ണമായും യോജിച്ചു വരുന്നവെന്നതാണ് വസ്തുത. അസ്ഥിരൂപീകരണ പ്രക്രിയ അഥവാ ഓസിഫിക്കേഷന്‍ ആരംഭിക്കുന്നത് ആറ് ആഴ്ചകള്‍ക്കു ശേഷമാണെന്ന് പഠനങ്ങള്‍ വ്യക്തമാക്കുന്നു. നാല്‍പത്തിരണ്ടു ദിവസങ്ങള്‍ക്കുശേഷമാണ് അസ്ഥികള്‍ ഉണ്ടാകുന്നതെന്നാണ് നബി (സ) പഠിപ്പിച്ചത്. കടിച്ച മാംസപിണ്ഡത്തെപ്പോലെ തോന്നിപ്പിക്കുന്ന സോമൈറ്റുകള്‍ നിറഞ്ഞ ഭ്രൂണഘട്ടത്തിനുശേഷമാണ് അസ്ഥിരൂപീകരണം നടക്കുന്നതെന്നും സോമൈറ്റുകളില്‍ നിന്നാണ് നട്ടെല്ലുണ്ടാകുന്നതെന്നും ശാസ്ത്രം നമുക്ക് പറഞ്ഞുതരുന്നു. കടിച്ച മൃദുലമാംസപിണ്ഡം എന്നു അര്‍ത്ഥം വരുന്ന മുദ്വ്അയില്‍ നിന്നാണ് ഇദ്വാമ് (അസ്ഥികള്‍) ഉണ്ടാകുന്നതെന്ന് ക്വുര്‍ആന്‍ നമുക്ക് നല്‍കുന്ന വിവരം തന്നെയാണിത്. ക്വുര്‍ആനിക വിജ്ഞാനീയങ്ങളെല്ലാം ആധുനികശാസ്ത്രത്തിനുമുമ്പില്‍ അടിപതറാതെ നിലനില്‍ക്കുമെന്ന വസ്തുത ഇത് ഒരിക്കല്‍കൂടി വ്യക്തമാക്കുന്നു.

'മുദ്വ്അ'യെന്ന ഒരു ഘട്ടം ഭ്രൂണത്തിനുണ്ടെന്ന് ക്വുർആൻ പറയുന്നുണ്ടെന്നത് ശരിയാണ്. നുത്വഫ, അലഖ എന്നീ ഘട്ടങ്ങക്കു ശേഷമുള്ള മൂന്നാം ഘട്ടത്തെയാണ് ക്വുര്‍ആന്‍ 'മുദ്വ്അ'യെന്ന് വിളിക്കുന്നത്. അലഖയില്‍ നിന്നാണ് മുദ്വ്അയുണ്ടായതെന്ന് ക്വുര്‍ആന്‍ രണ്ടുതവണ പ്രസ്താവിക്കുന്നുണ്ട്. അവ ഇങ്ങനെയാണ്.

''പിന്നെ ആ ബീജത്തെ (നുത്വഫ) നാം ഒരു ഭ്രൂണമായി (അലഖ) രൂപപ്പെടുത്തി. ശേഷം ആ ഭ്രൂണത്തെ നാം ഒരു മാംസപിണ്ഡമായി (മുദ്വ്അ) രൂപപ്പെടുത്തി. (ക്വുര്‍ആന്‍ 23:14)

മനുഷ്യരേ, ഉയിര്‍ത്തെഴുന്നേല്‍പിനെ പറ്റി നിങ്ങള്‍ സംശയത്തിലാണെങ്കില്‍ (ആലോചിച്ച് നോക്കുക:) തീര്‍ച്ചയായും നാമാണ് നിങ്ങളെ മണ്ണില്‍ നിന്നും, പിന്നീട് ബീജത്തില്‍ നിന്നും, പിന്നീട് ഭ്രൂണത്തില്‍ നിന്നും, അനന്തരം രൂപം നല്‍കപ്പെട്ടതും രൂപം നല്‍കപ്പെടാത്തതുമായ മാംസപിണ്ഡത്തില്‍(മുദ്വ്അ) നിന്നും സൃഷ്ടിച്ചത്. (ക്വുര്‍ആന്‍ 22:5)

മീം, ദ്വ, ഗൊയ്‌ന് എന്നീ അക്ഷരത്രയങ്ങളില്‍ നിന്നാണ് 'മുദ്വ്അ'യെന്ന പദമുണ്ടായിരിക്കുന്നത്. ഈ അക്ഷരത്രയങ്ങളില്‍ നിന്നുണ്ടാകുന്ന പദങ്ങള്‍ക്കെല്ലാം വായിലിട്ട് ചവയ്ക്കുകയെന്ന ക്രിയയുമായി എന്തെങ്കിലും ബന്ധമുണ്ടായിരിക്കും. 'മുദ്വ്അ'യെന്നാല്‍ ചവയ്ക്കുകയെന്നര്‍ത്ഥം. ചവയ്ക്കപ്പെട്ടത് എന്ന അര്‍ത്ഥത്തില്‍ 'യുമ്ദുഅ്' എന്നും 'മംദൂഅ്വ്' എന്നും പ്രയോഗിക്കും. ചവയ്ക്കുന്നവന് മാദ്വിഅ് എന്നാണ് പറയുക. മുകളില്‍ പറഞ്ഞ അക്ഷരത്രയത്തില്‍ നിന്നുണ്ടായ ഒരു സവിശേഷനാമമാണ് 'മുദ്വ്അ'. 'ചവയ്ക്കാനുള്ളത്' എന്ന അര്‍ത്ഥത്തിലും 'ചവയ്ക്കപ്പെട്ടത്' എന്ന അര്‍ത്ഥത്തിലുമുപയോഗിക്കുന്ന നാമം. ചെറിയൊരു മാംസപിണ്ഡത്തിനും ഹൃദയത്തെയും നാവിനെയും പോലെയുള്ള ചവച്ചുതിന്നാന്‍ പറ്റുന്ന അവയവങ്ങളെയും മൃദുലമാംസത്തെയും ചവയ്ക്കാനുള്ള ച്യൂയിംഗം പോലുള്ള വസ്തുക്കളെയുമെല്ലാം 'മുദ്വ്അ'യെന്നു പറയും.

ഭ്രൂണത്തിന്റെ ഇരുപത് മുതല്‍ നാല്‍പതു വരെ ദിവസങ്ങളിലെ രൂപത്തെക്കുറിക്കുവാന്‍ 'മുദ്വ്അ'യെന്ന പദം വളരെ കൃത്യമാണെന്ന വസ്തുത അത്ഭുതകരമാണ്. ഏതാനും മില്ലീമീറ്റര്‍ മാത്രം നീളമുള്ള മൃദുലമായ ഒരു മാംസപിണ്ഡം. എല്ലില്ലാത്ത കടിച്ചു ചവയ്ക്കാന്‍ പാകത്തിലുള്ള ഒരു മുദ്വ്അ. ഇരുപതാം ദിവസം അതിന്മേല്‍ കടിച്ചതു പോലെയുള്ള അടയാളങ്ങളെപ്പോലെ തോന്നിക്കുന്ന ഒന്നാമത്തെ ജോഡി സോമൈറ്റുകള്‍ പ്രത്യക്ഷപ്പെടുന്നു. മുദ്വ്അയില്‍ ആദ്യത്തെ കടി വീണുവെന്ന് പറയാം. പിന്നെ ദിവസേന രണ്ടും മൂന്നും സോമൈറ്റുകള്‍ ഉണ്ടായിക്കൊണ്ടിരിക്കും. മുദ്വ്അയില്‍ രണ്ടും മൂന്നും നാലും കടികള്‍ വീണുകൊണ്ടിരിക്കും എന്നര്‍ത്ഥം. ചവയ്ക്കാനുള്ളത് എന്നും ചവയ്ക്കപ്പെട്ടത് എന്നും അര്‍ത്ഥം പറയാന്‍ കഴിയുന്ന 'മുദ്വ്അ'യെക്കാള്‍ കൃത്യമായ പദമേതാണുളളത്, ഈ ഘട്ടത്തെക്കുറിയ്ക്കുവാന്‍?

അലഖയില്‍ നിന്ന് മുദ്വ്അയുണ്ടാകുന്നതിനെക്കുറിച്ച് പ്രതിപാദിക്കുന്ന സൂറത്തുല്‍ ഹജ്ജിലെ അഞ്ചാമത്തെ വചനത്തില്‍ മുദ്വ്അയെ രണ്ടാക്കിത്തിരിച്ചത് പ്രത്യേകം ശ്രദ്ധേയമാണ്. മുദ്വ്അത്തിന് മുഖല്ലഖ വ ഗൊയ്‌റി മുഖല്ലഖയെന്നാണ് ഇവിടെ പ്രയോഗിച്ചിരിക്കുന്നത്. രൂപം നല്‍കപ്പെട്ടതും രൂപം നല്‍കപ്പെടാത്തതുമായ മുദ്വ്അയെന്നാണ് ഇതിനെ പരിഭാഷപ്പെടുത്തിയിരിക്കുന്നത്. മുഅല്ലഖ വഗൊയ്‌റി മുഅല്ലഖയെന്ന മുദ്വ്അക്കു നല്‍കിയ വിശേഷണങ്ങള്‍ ഭ്രൂണപരിണാമത്തിന്റെ വിവിധ ഘട്ടങ്ങളെക്കുറിക്കുന്നുവെന്ന് ഇബ്‌നു അബ്ബാസും ഖതാദ(റ)യും വിശദീകരിച്ചതായി ഇമാം തബ്‌രിയും ബഗാവിയും തങ്ങളുടെ ക്വുര്‍ആന്‍ വ്യാഖ്യാനഗ്രന്ഥങ്ങളില്‍ വ്യക്തമാക്കുന്നുണ്ട്. മുദ്വ്അയുടെ വിശേഷണങ്ങളായി മുഖല്ലഖയെന്നും ഗ്വൊയ്‌റു മുഖല്ലഖയെന്നും ഉപയോഗിച്ചതില്‍നിന്ന് ഇവരണ്ടും തന്നെ ആ ഘട്ടത്തിലെ രണ്ട് അവസ്ഥകളെയാണ് കുറിക്കുന്നതെന്ന് മനസ്സിലാകുന്നുണ്ട്.

മൂന്നാമത്തെ ആഴ്ച മുതല്‍ ആറാമത്തെ ആഴ്ച വരെയുള്ള കാലയളവില്‍ സോമൈറ്റുകളുടെ ഉല്‍പത്തിയോടൊപ്പം ഭ്രൂണത്തില്‍ നടക്കുന്ന മറ്റു മാറ്റങ്ങളെന്തൊക്കെയാണെന്ന് പരിശോധിക്കുമ്പോഴാണ് രൂപം നല്‍കപ്പെടാത്തതും രൂപം നല്‍കപ്പെട്ടതുമായ മുദ്വ്അയെന്ന പ്രയോഗത്തിന്റെ സൗന്ദര്യം മനസ്സിലാവുക. സോമൈറ്റുകളുണ്ടാക്കുവാന്‍ തുടങ്ങുന്ന ഇരുപതാം ദിവസം ഭ്രൂണം ഒരു കോശക്കൂട്ടം മാത്രമാണ്. സോമൈറ്റുകളുടെ ഉല്‍പത്തിയോടൊപ്പം തന്നെ ഭ്രൂണത്തിന്റെ രൂപം മാറാന്‍ തുടങ്ങുന്നു. ഇരുപത്തിമൂന്നാമത്തെ ദിവസം പത്ത് സോമൈറ്റ് ജോഡികള്‍ ഉണ്ടായതിനുശേഷവും ഗര്‍ഭാശയഭിത്തിയില്‍ പറ്റിക്കിടക്കുന്ന ഒരു ചെറിയ അട്ടയില്‍നിന്ന് കാര്യമായ രൂപവ്യത്യാസങ്ങളൊന്നും തന്നെ ഭ്രൂണത്തിനുണ്ടാവുകയില്ല. ഇരുപത്തിനാലാമത്തെ ദിവസം മുതല്‍ക്കാണ് പ്രകടമായ രൂപവ്യത്യാസം തുടങ്ങുന്നത്. അട്ടയുടെ തലയ്ക്കു കീഴിലായി മാംസകമാനങ്ങള്‍ പ്രത്യക്ഷപ്പെടുന്നതോടെയാണ് ഈ മാറ്റം പ്രകടമാവുക. ഫാരിന്‍ജിയല്‍ കമാനങ്ങള്‍ (pharyngeal arches) എന്നാണ് ഈ കമാനങ്ങളുടെ പേര്. ഒന്നാമത്തെയും രണ്ടാമത്തെയും കമാനങ്ങള്‍ പ്രകടമാവുന്നത് ഇരുപത്തിനാലാം ദിവസമാണ്. ഇരുപത്തിയഞ്ചാം ദിവസമാകുമ്പോള്‍ തലയും വാലും അല്‍പം മുന്നിലേക്കുവളഞ്ഞ് ഏകദേശം അര്‍ദ്ധവൃത്താകൃതിയിലേക്ക് പതിനാറ് സോമൈറ്റുകളുള്ള ഭ്രൂണം പരിണമിക്കുന്നു. ഇരുപത്തിയേഴാം ദിവസമാകുമ്പോഴേക്ക് മൂന്നാമത്തെ ഫാരന്‍ജിയല്‍ കമാനമുണ്ടാവുകയും അതിനുമുകളിലായി ആന്തരിക കര്‍ണമായിത്തീരാനുള്ള കര്‍ണദ്വാരം (oticpit) വ്യക്തമായി കാണാന്‍ തുടങ്ങുകയും ചെയ്യുന്നു. C ആകൃതിയിലുള്ള ഭ്രൂണത്തിനു മധ്യത്തില്‍ കൈമുകുളങ്ങളും (arm buds) കാണാന്‍ തുടങ്ങുന്നത് അന്നുതന്നെയാണ് ഇരുപത്തിയെട്ടാം ദിവസത്തില്‍ നാലാമത്തെ ആഴ്ചയുടെ അവസാനത്തിലാണ് ഫാരിന്‍ജിയല്‍ കമാനങ്ങളുടെ നാലാമത്തെ ജോഡിയും കാല്‍മുകുളങ്ങളും (leg buds)പ്രത്യക്ഷപ്പെടുന്നത്. തലച്ചോറിന്റെ വര്‍ദ്ധനവുകാരണമുള്ള തലയുടെ വളര്‍ച്ചയാണ് അഞ്ചാം ആഴ്ചയില്‍ പ്രധാനമായും നടക്കുന്നത്. തല വളര്‍ന്ന് അതിന്റെ മുഖം ഹൃദയഭാഗത്തെ സ്പര്‍ശിക്കുന്ന അവസ്ഥയിലെത്തുന്നു. ആറാമത്തെ ആഴ്ചയിലാകുമ്പോഴേക്ക് കൈപ്പത്തിയുടെയും കൈമുട്ടുകളുടെയും കൈവിരലുകളുടെയുമെല്ലാം പ്രാഗ്‌രൂപങ്ങള്‍ കാണാന്‍ കഴിയും. നാല്‍പതാമത്തെ ദിവസമാകുമ്പോള്‍ തല കുറേക്കൂടി വലുതാവുകയും ചെവിയായിത്തീരുവാനുള്ള കര്‍ണഅറയും (otic vesicle) കണ്ണിന്റെ ഭാഗത്ത് റെറ്റിനല്‍ വര്‍ണവും (retinal pigment) പ്രത്യക്ഷപ്പെടുകയും ചെയ്യും. സോമൈറ്റുകളെല്ലാം ഉത്ഭവിച്ചു കഴിയുമ്പോഴേക്ക് രൂപം നല്‍കെപ്പട്ട മുദ്വ്അയുടെ പൂര്‍ണരൂപം നമുക്ക് കാണാനാകുമെന്നര്‍ത്ഥം. അപ്പോഴുള്ള മുദ്വ്അക്ക് കണ്ണും കാതും തലയും കൈകാലുകളുമെല്ലാം ഉണ്ടായിരിക്കും.

ഇരുപതാം ദിവസം മുതല്‍ നാല്‍പതാം ദിവസം വരെയുള്ള ചെറിയ കാലയളവില്‍ ഭ്രൂണത്തിനകത്തും പുറത്തുമുണ്ടാകുന്നതുപോലെയുള്ള മാറ്റങ്ങള്‍ മറ്റൊരു ഭ്രൂണഘട്ടത്തിലുമുണ്ടാകുന്നില്ല. കേവലമൊരു കോശക്കൂട്ടമായി ഗര്‍ഭാശയഭിത്തിയില്‍ അള്ളിപ്പിടിച്ച് കിടക്കുകയായിരുന്ന കാര്യമാത്ര പ്രസക്തമായ രൂപങ്ങളൊന്നുമില്ലാതിരുന്ന ഭ്രൂണത്തിന് കൈകാലുകളും തലയും കണ്ണും കാതുമെല്ലാം കാണാന്‍ കഴിയുന്ന രീതിയിലായിത്തീരുന്നത് ഈ ഘട്ടത്തിലാണ്. രൂപം നല്‍കപ്പെട്ടതും (മുഖല്ലഖ) രൂപം നല്‍കപ്പെടാത്തതുമായ (ഗയ്‌റു മുഖല്ലഖ) ചവയ്ച്ച മാംസപിണ്ഡം (മുദ്വ്അ) എന്ന ക്വുര്‍ആനിക പ്രയോഗം എത്രമാത്രം കൃത്യമാണെന്ന് ഇത് മനസ്സിലാക്കിത്തരുന്നു.

മുദ്വ്അയെന്ന പദത്തിന് ചവയ്ക്കപ്പെട്ടത് എന്ന് അര്‍ത്ഥമില്ലെന്നും സോമൈറ്റുകളെക്കുറിച്ച് മനസ്സിലാക്കിയശേഷം ഇസ്‌ലാമിക പ്രബോധകര്‍ പടച്ചുണ്ടാക്കിയതാണ് പ്രസ്തുത അര്‍ത്ഥമെന്നും വാദിക്കുന്നവര്‍ക്ക് ആദ്യകാല ക്രൈസ്തവരേഖകള്‍ തന്നെ മറുപടി പറയുന്നുണ്ട്. ബൈസന്റൈന്‍കാരുടെ ഭരണപ്രദേശത്ത് ഇസ്‌ലാമില്‍നിന്ന് ക്രിസ്തുമതം സ്വീകരിപ്പിക്കുവാന്‍ ഉപയോഗിച്ചിരുന്ന ക്രിസ്താബ്ദം ഒന്‍പതാം നൂറ്റാണ്ടിലെ ഒരു ശാഖാ വര്‍ത്തമാനം ഇങ്ങനെയാണ് രേഖപ്പെടുത്തപ്പെട്ടിട്ടുള്ളത്. ''മനുഷ്യസൃഷ്ടിയെക്കുറിച്ച മുഹമ്മദിന്റെ പരികല്‍പനകളെ ഞാന്‍ തള്ളിക്കളയുന്നു. മനുഷ്യന്‍ പൊടിയില്‍ നിന്നും ദ്രാവകത്തുള്ളിയില്‍നിന്നും അട്ടകളില്‍നിന്നും ചവയ്ക്കപ്പെട്ടതുപോലുള്ള വസ്തുവില്‍ നിന്നുമാണ് സൃഷ്ടിക്കപ്പെട്ടത് എന്നാണ് അയാള്‍ പറയുന്നത്.''(A Byzantine anathema recorded during Muslim conversions to Christianity reads: I anathematize Muhammad’s teaching about the creation of man, where he says that man was created from dust and a drop of fluid , and leeches and chewed-like substance )

പരിശുദ്ധ ക്വുര്‍ആനിന് ഇസ്‌ലാം വിമര്‍ശകര്‍ നല്‍കിയ പരിഭാഷകളില്‍ പോലും അലഖിന് അട്ടയെന്നും മുദ്വ്അക്ക് ചവച്ചരയ്ക്കപ്പെട്ടത് എന്നുമാണ് അര്‍ത്ഥം നല്‍കിയിരുന്നത് എന്ന് ഇതില്‍നിന്ന് വ്യക്തമാണ്. മുസ്‌ലിംകള്‍ ക്വുര്‍ആന്‍ പരിഭാഷകള്‍ ഉപയോഗിക്കാന്‍ തുടങ്ങുന്നതിനു നൂറ്റാണ്ടുകള്‍ക്ക് മുമ്പുതന്നെ ഗ്രീക്കുഭാഷയിലേക്ക് ക്വുര്‍ആന്‍ ഭാഷാന്തരം ചെയ്ത ഇസ്‌ലാമിന്റെ ശത്രുക്കള്‍ക്ക് അലഖയുടെ അര്‍ത്ഥം അട്ടയെന്നാണെന്നും മുദ്വ്അയുടെ അര്‍ത്ഥം ചവയ്ക്കപ്പെട്ടത് എന്നാണെന്നും മനസ്സിലായിയെന്നാണ ല്ലോ ഇതു മനസ്സിലാക്കിത്തരുന്നത്. ശാസ്ത്രവസ്തുതകള്‍ക്കനുസരിച്ച് ക്വുര്‍ആന്‍ വ്യാഖ്യാനിക്കുകയല്ല, ക്വുര്‍ആന്‍ പ്രയോഗങ്ങളുടെ സത്യതയിലേക്ക് ശാസ്ത്രം തെളിക്കുന്ന വെളിച്ചത്തെക്കുറിച്ച് ബോധ്യപ്പെടുത്തുക മാത്രമാണ് ഇസ്‌ലാമിക പ്രബോധകര്‍ ചെയ്യുന്നതെന്ന യാഥാര്‍ത്ഥ്യമാണ് ഇവിടെ അനാവൃതമാകുന്നത്.

ല്ല. വളരെ കൃത്യമായ പരാമർശമാണ് ഇവിടെ ഖുർആൻ നടത്തുന്നത്. വിമർശക്കപ്പെട്ട വചനം നോക്കുക:

"നിങ്ങളുടെ മാതാക്കളുടെ വയറുകളില്‍ നിങ്ങളെ അവന്‍ സൃഷ്ടിക്കുന്നു. മൂന്ന്‌ തരം അന്ധകാരങ്ങള്‍ക്കുള്ളില്‍ സൃഷ്ടിയുടെ ഒരു ഘട്ടത്തിന്‌ ശേഷം മറ്റൊരു ഘട്ടമായിക്കൊണ്ട്‌." (39: 6)

മാതാക്കളുടെ വയറുകള്‍ക്കകത്തെ ഘട്ടംഘട്ടമായ മനുഷ്യസൃഷ്ടി നടക്കുന്നത് മൂന്നുതരം അന്ധകാരങ്ങള്‍ക്കകത്താണ് എന്നാണ് ക്വുര്‍ആന്‍ ഇവിടെ പ്രസ്താവിച്ചിരിക്കുന്നത്. അടിവയറും ഗര്‍ഭാശയവും ആംനിയോണ്‍-കോറിയോണ്‍ സ്തരവുമാണ് കുഞ്ഞിനെ സംരക്ഷിച്ചുനിര്‍ത്തുന്ന പാളികള്‍ എന്ന് നമുക്കറിയാം. മൂന്നുതരം ഇരുട്ടുകള്‍ എന്ന ക്വുര്‍ആനിക പ്രയോഗം എത്രമാത്രം കൃത്യമാണെന്ന് നോക്കുക. ഈ മൂന്ന് പാളികളാണ് ഭ്രൂണവളര്‍ച്ചയ്ക്കാവശ്യമായ ഇരുട്ട് പ്രദാനം ചെയ്യുന്നത് എന്നുകൂടി അറിയുമ്പോഴാണ് ഈ പ്രയോഗം എത്രത്തോളം സൂക്ഷ്മവും കൃത്യവുമാണെന്ന് മനസ്സിലാവുക. കുഞ്ഞിനെ സംരക്ഷിക്കുകയും അതിന്ന് വളരാനാവശ്യമായ അന്ധകാരം സൃഷ്ടിക്കുകയും ചെയ്തുകൊണ്ടുള്ള മൂന്നുതരം പാളികള്‍ക്കുള്ളിലാണ് കുഞ്ഞിന്റെ ഘട്ടങ്ങളായുള്ള വളര്‍ച്ച നടക്കുന്നതെന്ന് ക്വുര്‍ആനിന്റെ സമകാലികരോ പൂര്‍വികരോ ആയ ഭ്രൂണപഠിതാക്കളൊന്നും തന്നെ പറഞ്ഞതായി കാണാന്‍ കഴിയുന്നില്ല. പുരാതന ഗ്രീക്കുകാരോ ജൂതന്‍മാരോ മാത്രമല്ല, ആധുനിക കാലം വരെയുള്ള ഇവ്വിഷയകമായി പഠിച്ചവരൊന്നും തന്നെ കുഞ്ഞിന് അന്ധകാരവും സുരക്ഷയും നല്‍കുന്ന മൂന്ന് പാളികളെക്കുറിച്ച് പരാമര്‍ശിക്കുന്നില്ല. എന്നിട്ടും എങ്ങനെയാണ് നിരക്ഷനായ ഒരാളുടെ നാവില്‍ നിന്ന് ലോകം കേട്ട വചനങ്ങളില്‍ ഇത്ര കൃത്യമായ പരാമര്‍ശങ്ങളുണ്ടാവുന്നതെന്ന ചോദ്യത്തിന് ഒരൊറ്റ ഉത്തരം മാത്രമേയുള്ളു: ''ഈ ഗ്രന്ഥത്തിന്റെ അവതരണം സര്‍വലോകരക്ഷിതാവിങ്കല്‍ നിന്നാകുന്നു. ഇതില്‍ യാതൊരു സംശയവുമില്ല.''(32 : 2)

ക്വുർആൻ 86: 5 -7 വചനങ്ങൾ പറയുന്നത് നട്ടെലിന്റെയും വാരിയെല്ലിന്റെയും ഇടയിൽ നിന്നാണ് ബീജമുണ്ടാവുന്നത് എന്നാണല്ലോ. അരക്കെട്ടിലുള്ള വൃഷണത്തിലാണ് ബീജങ്ങൾ ഉത്പാദിപ്പിക്കപ്പെടുന്നത് എന്നിരിക്കെ ഈ പരാമർശം ശുദ്ധ അബദ്ധമല്ലേ? ഗ്രീക്ക് വൈദ്യനായിരുന്ന ഹിപ്പോക്രാറ്റസിന്റെ ഭ്രൂണ ശാസ്ത്ര ചിന്തകളെ മുഹമ്മദ് നബി (സ) കോപ്പിയടിച്ചത് കൊണ്ടാണ് ഈ അബദ്ധം സംഭവിച്ചത് എന്ന വിമര്ശനനത്തിന് എന്ത് മറുപടി പറയും ?

അല്പം വിശദമായി മറുപടി പറയേണ്ട വിഷയമാണിത്. നിലവിലുള്ള മലയാളം ഖുർആൻ പരിഭാഷകളിലും പ്രധാന ഇംഗ്ലീഷ് പരിഭാഷകളിലുമെല്ലാം ഈ വചനത്തിന് നൽകിയിരിക്കുന്ന ഭാഷാന്തരം വിമർശകൻ സൂചിപ്പിക്കുന്നത് പോലെത്തന്നെയാണ്. അത് ഇങ്ങനെയാണ്: ''എന്നാല്‍ മനുഷ്യന്‍ ചിന്തിച്ചു നോക്കട്ടെ താന്‍ എന്തില്‍ നിന്നാണ് സൃഷ്ടിക്കപ്പെട്ടിരിക്കുന്നത് എന്ന്. തെറിച്ചു വീഴുന്ന ഒരു ദ്രാവകത്തില്‍ നിന്നത്രെ അവന്‍ സൃഷ്ടിക്കപ്പെട്ടിരിക്കുന്നത്. മുതുകെല്ലിനും, വാരിയെല്ലുകള്‍ക്കുമിടയില്‍ നിന്ന് അത് പുറത്തു വരുന്നു.''(1)

വിഷയം കൃത്യമായി മനസ്സിലാക്കാൻ മുകളിലെ വിമർശനത്തെ മൂന്നായി തരം തിരിച്ച് പരിശോധിക്കാം എന്ന് തോന്നുന്നു:

1) മുഹമ്മദ് നബി(സ)ക്ക് ബീജത്തിന്റെ ഉല്‍ഭവസ്ഥാനമാണ് വൃഷണങ്ങള്‍ എന്ന് അറിയുമായിരുന്നില്ല.

2) വാരിയെല്ലുകളുടെയും മുതുകെല്ലിന്റെയും മധ്യെ നിന്നാണ് ശുക്ലമുണ്ടാവുന്നതെന്ന ഹിപ്പോക്രാറ്റിസിന്റെ ഭ്രൂണപരിണാമ ചിന്തകളെ പകര്‍ത്തുകയാണ് ഖുര്‍ആനിൽ ചെയ്തിരിക്കുന്നത്.

3 ) ക്വുർആനിലെ ഈ പരാമർശം പരമാബദ്ധമാണ്.

ഈ ധാരണകള്‍ എത്രത്തോളം ശരിയാണ്? നാം പരിശോധിക്കുക:

1) ഖുര്‍ആനിന്റെ അവതരണകാലത്ത് ജീവിച്ചിരുന്ന അറബികള്‍ക്കെല്ലാം അറിയാമായിരുന്ന ഒരു യാഥാര്‍ഥ്യമായിരുന്നു പുനരുല്‍പാദനത്തിന് കാരണമാകുന്ന ബീജങ്ങളുടെ ഉല്‍പാദനം നടക്കുന്നത് വൃഷണങ്ങളിലാണെന്ന വസ്തുത. വരിയുടയ്ക്കലിനെ (castration) സംബന്ധിച്ച് അക്കാലത്ത് നിലനിന്നിരുന്ന ധാരണകളില്‍ നിന്ന് ഇക്കാര്യം ബോധ്യപ്പെടും. പച്ചിലകള്‍ ചേര്‍ത്ത ചൂടുവെള്ളത്തില്‍ ഇരുത്തിയശേഷം മെല്ലെമെല്ലെ വൃഷണത്തില്‍ തടവുകയും പിന്നെ ശക്തി ഉപയോഗിച്ച് വൃഷണം ഉടച്ചുകളയും ചെയ്യുന്ന രീതിയായിരുന്നു വ്യാപകമായി നിലവിലുണ്ടായിരുന്ന ഷണ്ഡീകരണ രീതി. ഇങ്ങനെ വന്ധ്യംകരണം നടത്തുന്നവര്‍ക്ക് വൃഷണങ്ങള്‍ക്കകത്താണ് പുനരുല്‍പാദനത്തിന് കാരണമാകുന്ന ബീജം ഉല്‍പാദിപ്പിക്കപ്പെടുന്നതെന്ന് കൃത്യമായി അറിയാമായിരുന്നുറപ്പാണല്ലോ.

മുഹമ്മദ് നബി(സ)ക്കും സമകാലികര്‍ക്കുമെല്ലാം വരിയുടച്ച് ഷണ്ഡീകരിക്കുന്ന രീതിയെക്കുറിച്ച് അറിയാമായിരുന്നുവെന്ന് വ്യക്തമാക്കുന്ന നിരവധി ഹദീഥുകളുണ്ട്. സ്വഹീഹുല്‍ ബുഖാരി, കിത്താബുത്തഫ്‌സീറിലും സ്വഹീഹു മുസ്‌ലിം കിത്താബുന്നികാഹിലും ഇബ്‌നു മസ്ഊദിൽ (റ) നിന്ന് നിവേദനം ചെയ്യുന്ന ഹദീഥും സ്വഹീബുല്‍ ബുഖാരി, കിത്താബുന്നികാഹിലും സ്വഹീഹു മുസ്‌ലിം, കിത്താബുന്നികാഹിലും അബൂഹുറൈറയിൽ (റ) നിന്ന് നിവേദനം ചെയ്യുന്ന ഹദീഥും ഇതേ കിതാബുകളിൽ സഅദിൽ (റ) നിന്ന് നിവേദനം ചെയ്യുന്ന ഹദീഥും ഇവയിൽ ചിലതാണ്. പ്രത്യുല്‍പാദനത്തിന് കാരണമാകുന്ന ബീജത്തിന്റെ ആവിര്‍ഭാവം വൃഷണത്തില്‍ നിന്നാണെന്ന് അറിയാമായിരുന്നു എന്നതുകൊണ്ടാണല്ലോ വൃഷണമുടച്ച് ലൈംഗികശേഷിയെയും പ്രത്യുല്‍പാദനത്തെയും ഇല്ലാതാക്കുന്നത് വിരോധിക്കപ്പെട്ടത്. വൃഷണങ്ങളാണ് ബീജോല്‍പാദനത്തിന്റെ കേന്ദ്രമെന്ന വസ്തുത മുഹമ്മദ് നബി(സ) ക്കും അനുചരന്‍മാര്‍ക്കും അക്കാലത്ത് ജീവിച്ചിരുന്ന അറബികള്‍ക്കുമെല്ലാം അറിയാമായിരുന്നുവെന്നാണ് ഇതില്‍നിന്ന് വ്യക്തമായി മനസ്സിലാകുന്നത്.

2) ഗ്രീക്ക് വൈദ്യനായിരുന്ന ഹിപ്പോക്രാറ്റസിന്റെ ബീജോല്‍പാദനത്തെക്കുറിച്ച അഭിപ്രായങ്ങളുള്ളത് The seed and the Nature of the child, On Generation എന്നീ പ്രബന്ധങ്ങളിലാണ്. ഇവയിലൊന്നും തന്നെ നെഞ്ചെല്ലിനും മുതുകെല്ലിനും മധ്യെ നിന്നാണ് ബീജോല്‍പാദനം നടക്കുന്നതെന്ന് അദ്ദേഹം അഭിപ്രായപ്പെടുന്നില്ല. ബീജോല്‍പാദനത്തെക്കുറിച്ച് ഹിപ്പോക്രാറ്റസ് പറയുന്ന കാര്യങ്ങളെ ഇങ്ങനെ സംക്ഷേപിക്കാം.

a) ശരീരത്തിലാകമാനം വ്യാപിച്ചുകിടക്കുന്ന രസത്തിന്റെ ഏറ്റവും ശക്തമായ ഭാഗമാണ് ബീജമായി പുറത്തേക്കു വരുന്നത്.

b) ശരീരത്തിന്റെ എല്ലാഭാഗത്തുനിന്നും ലിംഗത്തിലേക്ക് നീളുന്ന ധമനികളുണ്ട്. അതിലൂടെയാണ് രസം പ്രവഹിക്കുന്നത്.

c) തലച്ചോറില്‍നിന്ന് അരക്കെട്ടുവരെയുള്ള ശരീരഭാഗങ്ങളിലെല്ലാം വിശേഷിച്ചും നട്ടെല്ലിന്റെ മജ്ജയില്‍ രസം വ്യാപിക്കുന്നു.

d) സുഷുമ്‌നാ മജ്ജയിലെത്തുന്ന ശുക്ലം വൃക്കയിലൂടെയുള്ള ധമനികളിലൂടെ കടന്നുപോകുന്നു.

e) വൃക്കയില്‍നിന്നും ഇത് വൃഷണങ്ങളിലൂടെ ലിംഗത്തിലേക്ക് ഒഴുകുന്നു.

ഇതിലെവിടെയും തന്നെ വാരിയെല്ലിന്റെയും മുതുകെല്ലിന്റെയും മധ്യത്തുനിന്നാണ് ബീജം ഉല്‍പാദിപ്പിക്കപ്പെടുന്നതെന്നു പറയുന്നില്ല. ഹിപ്പോക്രാറ്റസിന് ഇല്ലാത്ത ഒരു വാദം അദ്ദേഹത്തില്‍നിന്ന് കോപ്പിയടിക്കുന്നതെങ്ങനെയാണ്? ഹിപ്പോക്രാറ്റസിന്റെ ഭ്രൂണശാസ്ത്രചിന്തകള്‍ അറബികള്‍ക്കിടയില്‍ പ്രചാരത്തിലുണ്ടായിരുന്നുവെന്നത് തെളിവുകളൊന്നും ഇല്ലാത്ത ഒരു ഊഹംമാത്രമാണെന്ന വസ്തുത കൂടി ഇതോട് ചേർത്ത് വായിക്കുക.

മുഹമ്മദ് നബി(സ) ജീവിച്ചിരുന്ന കാലത്ത് വാരിയെല്ലുകള്‍ക്കും മുതുകെല്ലിനുമിടയില്‍ നിന്നാണ് ബീജം ഉല്‍പാദിപ്പിക്കപ്പെടുന്നതെന്ന വിശ്വാസം അറബികള്‍ക്കിടയില്‍ നിലനിന്നിരുന്നുവെങ്കില്‍ ഇസ്‌ലാം പൂര്‍വകാലത്തെ രചനകളിലോ പ്രവാചകശിഷ്യന്‍മാരുടെ വര്‍ത്തമാനങ്ങളിലോ അതിന്റെ സൂചനകളെന്തെങ്കിലുമുണ്ടാകുമായിരുന്നു. അങ്ങനെ ഇല്ലെന്നുമാത്രമല്ല, ഇത്തരമൊരു പരാമര്‍ശം നടത്തുന്ന ഒരേയൊരു കൃതി ഖുര്‍ആനാണുതാനും. അറബികള്‍ക്കിടയില്‍ പ്രചാരത്തിലുണ്ടായിരുന്ന ഹിപ്പോക്രാറ്റിയന്‍ ചിന്തകളെ പകര്‍ത്തുകയാണ് ഖുര്‍ആന്‍ ചെയ്യുന്നതെന്ന വാദം തീരെ അടിസ്ഥാന രഹിതമാണെന്നർത്ഥം.

3) വിമര്‍ശിക്കപ്പെടുന്ന വചനങ്ങള്‍ പരിശോധിക്കുക:

''ഖുലിഖ മിന്‍ മാഇന്‍ ദാഫിഖ്. യഖ്‌റുജു മിന്‍ ബൈനിസ്സ്വുല്‍ബി വ ത്തറാഇബ്''

''തെറിച്ചു വീഴുന്ന ഒരു ദ്രാവകത്തില്‍ നിന്നത്രെ അവന്‍ സൃഷ്ടിക്കപ്പെട്ടിരിക്കുന്നത്. സ്വുല്‍ബിനും, തറാഇബിനുമിടയില്‍ നിന്ന് അത് പുറത്തുവരുന്നു.''

ഈ വചനങ്ങൾക്ക് പ്രവാചകനോ(സ) അനുചരന്മാരോ വല്ല വ്യാഖ്യാനവും നൽകിയിട്ടുണ്ടോ? ഉണ്ടെങ്കിൽ എന്താണ് ? അതാണ് ആദ്യം പരിശോധിക്കേണ്ടത്. സൂറത്തുത്വാരിഖിലുള്ള 'ബൈന സ്വുല്‍ബി വത്തറാഇബി' എന്ന പ്രയോഗത്തിന് 'പുരുഷന്റെ സ്വുല്‍ബില്‍നിന്നും സ്ത്രീയുടെ തറാഇബുകളില്‍നിന്നും എന്ന അര്‍ഥമാണ് ആദ്യകാല ഖുര്‍ആന്‍ വ്യാഖ്യാതാക്കളില്‍ മിക്കവരും നല്‍കിയിട്ടുള്ളത്. സ്വഹാബിമാരില്‍ നിന്നുള്ള ഖുര്‍ആന്‍ വ്യാഖ്യാതാക്കളില്‍ പ്രമുഖനായ ഇബ്‌നു അബ്ബാസ്(റ) ഈ വചനത്തിന് നല്‍കിയ വ്യാഖ്യാനം ''സ്വുല്‍ബുര്‍റജുലി വതറാഇബുല്‍ മര്‍അത്തി വല്‍ വലദു ലാ യകൂനു ഇല്ലാ മിനല്‍ മാഅയ്‌നി'' എന്നാണെന്ന് ഇമാം ത്വബ്‌റാനി ഉദ്ധരിച്ചതായി കാണാം. 'പുരുഷന്റെ സ്വുല്‍ബില്‍ നിന്നും സ്ത്രീയുടെ തറാഇബില്‍ നിന്നുമുള്ള രണ്ട് ദ്രാവകങ്ങളില്‍ നിന്നുമായാണ് കുഞ്ഞുണ്ടാവുന്നത്' എന്നര്‍ഥം. മറ്റൊരു സ്വഹാബിയായ ഇക്‌രിമ (റ)വും ഇതേ അര്‍ഥം പറഞ്ഞതായി ഇമാം ത്വബ്‌രി തന്റെ തഫ്‌സീറില്‍ ഉദ്ധരിക്കുന്നുണ്ട്. സ്വഹാബിമാരുടെ അഭിപ്രായങ്ങളുടെ വെളിച്ചത്തില്‍, പ്രസിദ്ധ ഖുര്‍ആന്‍ വ്യാഖ്യാതാക്കളായ ത്വബ്‌രി, സമഖ്ശരി, ത്വബ്‌റാനി, റാസി, ഖുര്‍തുബി, ഇബ്‌നു കഥീര്‍, ജലാലൈനി, ശൗക്കാനി തുടങ്ങിയവരെല്ലാം 'പുരുഷന്റെ സ്വുല്‍ബില്‍ നിന്നും സ്ത്രീയുടെ തറാഇബില്‍ നിന്നുമുള്ള ദ്രാവകങ്ങളുടെ സംയോജനത്തിൽ നിന്നാണ് കുഞ്ഞുണ്ടാവുന്നത് എന്നാണ് ഈ ആയത്തുകള്‍ക്ക് നല്‍കിയ വ്യാഖ്യാനം. പ്രവാചകനിൽ നിന്ന് നേർക്ക് നേരെ ഖുർആൻ വ്യാഖ്യാനം മനസ്സിലാക്കിയ ഇബ്നു അബ്ബാസും ഇക്‌രിമയും (റ) മനസ്സിലാക്കിയത് സ്ത്രീയുടെ സ്രവവും പുരുഷന്റെ സ്രവവും കൂട്ടിച്ചെർന്നാണ് കുഞ്ഞുണ്ടാവുന്നതെന്ന വസ്തുതയാണ് ഈ വചങ്ങൾ പഠിപ്പിക്കുന്ന ആശയം എന്നാണ് ഇത് വ്യക്തമാക്കുന്നത്.

എന്താണ് സ്വുല്‍ബ് ? സ്വാദ്, ലാമ്, ബാഅ് തുടങ്ങിയ മൂലാക്ഷരങ്ങളില്‍നിന്ന് നിഷ്പന്നമായ സ്വുല്‍ബ് എന്ന പദം ക്രിയാരൂപത്തില്‍ വരുമ്പോള്‍ ഉറച്ചതായിത്തീരുക, ശക്തമാവുക, മാറ്റമില്ലാതാവുക എന്നെല്ലാമാണ് അർഥം. സ്വുല്‍ബ് എന്ന് നട്ടെല്ലിനും (back bone) അരക്കെട്ടിനും (loins) പറയാറുണ്ട്. Loins എന്ന ഇംഗ്ലീഷ് പദത്തിന് The Concise Oxford Arabic-English Dictionary യും Hippocrene Standard Dictionary (Arabic-English) യും നല്‍കുന്ന അറബി അര്‍ഥം സ്വുല്‍ബ് എന്നാണ്. അടിയിലെ വാരിയെല്ലുകള്‍ക്കും (lower ribs) വസ്തിപ്രദേശത്തിനു (pelvis) മിടയ്ക്കുള്ള ശരീരഭാഗമാണ് അരക്കെട്ട് അഥവാ കടിപ്രദേശം (loins). 'സ്വുല്‍ബ്' എന്ന ഖുര്‍ആനിക പ്രയോഗത്തിന് (loin) എന്ന് അര്‍ഥം പറഞ്ഞവരാണ് മിക്ക ഇംഗ്ലീഷ് പരിഭാഷകരുമെന്നതാണ് വസ്തുത. ഇതില്‍ ആദ്യകാല ഇംഗ്ലീഷ് പരിഭാഷകനും ഓറിയന്റലിസ്റ്റുമായ ജോര്‍ജ് സെയിലും ഉള്‍പ്പെടുന്നു. ക്രിസ്തുമതത്തില്‍നിന്ന് ഇസ്‌ലാം സ്വീകരിച്ച ഖുര്‍ആന്‍ പരിഭാഷകനായ മുഹമ്മദ് മാര്‍മഡ്യൂക് പിക്താളും ജൂതമതത്തില്‍ നിന്ന് ഇസ്‌ലാം സ്വീകരിച്ചശേഷം ഖുര്‍ആന്‍ പരിഭാഷപ്പെടുത്തിയ മുഹമ്മദ് അസദും ബ്രിട്ടീഷ് ഓറിയന്റലിസ്റ്റായ ഖുര്‍ആന്‍ പരിഭാഷകന്‍ ആര്‍തര്‍ ജോണ്‍ ആര്‍ബെറിയുമെല്ലാം സ്വുല്‍ബിന് നല്‍കിയിട്ടുള്ള അര്‍ഥം loins എന്നു തന്നെയാണ്.

മനുഷ്യരെക്കുറിച്ച് 'പുരുഷന്റെ അരക്കെട്ടില്‍ നിന്ന് ഉണ്ടാകുന്നവര്‍' എന്ന അര്‍ഥത്തില്‍ 'സ്വുല്‍ബില്‍ നിന്നുള്ളവര്‍' എന്ന പ്രയോഗം മധ്യപൂര്‍വദേശത്ത് വ്യാപകമായിരുന്നു. ഇക്കാര്യം ബൈബിളില്‍ നിന്ന് വ്യക്തമായി മനസ്സിലാകുന്നുണ്ട്. ഒരു ബൈബിള്‍ വചനം കാണുക: ''ദൈവം പിന്നെയും അവനോട് ഞാന്‍ സര്‍വശക്തിയുള്ള ദൈവമാകുന്നു. നീ സന്താനപുഷ്ടിയുള്ളവനായി പെരുകുക. ഒരു ജാതിയും ജാതികളുടെ കൂട്ടവും നിന്നില്‍ നിന്ന് ഉല്‍ഭവിക്കും. രാജാക്കന്മാരും നിന്റെ അരക്കെട്ടിൽ നിന്ന് പുറപ്പെടും.''(ഉല്‍ 35:11)

മക്കളെക്കുറി ച്ച് 'അരക്കെട്ടിൽ നിന്നുണ്ടാവുന്നവർ' എന്ന പ്രയോഗം രാജാക്കന്മാർ 8:19, 2 ദിനവൃത്താന്തം 6:9 പുറപ്പാട് 1:5 എന്നീ പഴയനിയമ ഉദ്ധരണികളിൽ കാണാം. പുതിയനിയമത്തിലെ അപ്പോസ്തലപ്രവൃത്തികള്‍2:30, എബ്രായര്‍7:5, എബ്രായര്‍7:10 എന്നീ വചനങ്ങളിലും സമാനമായ പ്രയോഗങ്ങൾ കാണാം. ഉല്‍പത്തി 35:11ലെ "നിന്റെ അരക്കെട്ടിൽ നിന്ന് പുറപ്പെടും" എന്ന പ്രയോഗത്തിന് അറബി ബൈബിളില്‍ നല്‍കപ്പെട്ടിരിക്കുന്ന പരിഭാഷ 'മിന്‍ സ്വുല്‍ബിക്ക' എന്നാണ്. 'നിന്റെ കടിപ്രദേശത്തുനിന്നുണ്ടാകുന്നവര്‍' എന്ന അര്‍ഥത്തില്‍ 'മിന്‍ സ്വുല്‍ബിക്ക' എന്ന പ്രയോഗം അറബികള്‍ക്കിടയിലും തത്തുല്യമായ പ്രയോഗങ്ങള്‍ മധ്യപൂര്‍വപ്രദേശത്തെ മറ്റു ഭാഷക്കാര്‍ക്കിടയിലും വ്യാപകമായിരുന്നുവെന്നാണ് ഇത് വ്യക്തമാക്കുന്നത്.

ഇത്തരത്തിലുള്ള ചില പ്രയോഗങ്ങള്‍ ഖുര്‍ആനിലും ഹദീഥുകളിലുമെല്ലാം കാണാന്‍ കഴിയും. ഖുര്‍ആന്‍ സൂറത്തുന്നിസാഇലെ ഇരുപത്തിമൂന്നാമത്തെ വചനത്തിലും സ്വഹീഹു മുസ്‌ലിം, കിത്താബുല്‍ ഖദ് റിൽ ആയിഷയിൽ (റ) നിന്ന് നിവേദനം ചെയ്ത ഹദീഥിലും സമാനമായ പ്രയോഗമാണ് കാണാം. സ്വുല്‍ബിന്റെ ബഹുവചനമായ അസ്വ്്‌ലാബ് എന്നാണ് ഇവിടെ പ്രയോഗിച്ചിരിക്കുന്നത്. ഇവയിലെ 'മിന്‍ അസ്വ്്‌ലാബിക്കും' എന്ന പ്രയോഗത്തിന് 'നിങ്ങളുടെ അണക്കെട്ടുകളിൽ നിന്നുള്ള ' എന്ന അര്‍ഥമാണുള്ളത്. ഖുര്‍ആനിന്റെ അവതരണകാലത്ത് മക്കളെ ഉദ്ദേശിച്ചുകൊണ്ട് 'സ്വുല്‍ബില്‍ നിന്നുള്ളവര്‍' എന്ന പ്രയോഗം വ്യാപകമായിരുന്നുവെന്ന വസ്തുതയാണിത് കാണിക്കുന്നത്. സ്വുല്‍ബ് എന്ന പദത്തിന് ഉറച്ചത്, നട്ടെല്ല്, കടിപ്രദേശം തുടങ്ങിയ അര്‍ഥങ്ങളുണ്ടെന്നും, മനുഷ്യജനനവുമായി ബന്ധപ്പെട്ട് പ്രയോഗിക്കുമ്പോള്‍ 'സ്വുല്‍ബില്‍ നിന്ന്' എന്ന പ്രയോഗം ഖുര്‍ആനിന്റെ അവതരണത്തിന് മുമ്പുതന്നെ മധ്യപൂര്‍വദേശത്ത് വ്യാപകമായിരുന്നുവെന്നും ഈ പ്രയോഗത്തിന് അവര്‍ അര്‍ഥമാക്കിയത് അരക്കെട്ടിൽ നിന്ന് എന്നാണെന്നുമുള്ള വസ്തുതകളാണ് ഇതിൽ നിന്നെല്ലാം മനസ്സിലാവുന്നത് സൂറത്തു ത്വാരിഖിലെ ഏഴാമത്തെ വചനത്തിലെ 'സ്വുല്‍ബി'നും അരക്കെട്ട് (loins) എന്ന അര്‍ഥം തന്നെയാണ് പ്രമുഖരായ ഇംഗ്ലീഷ് പരിഭാഷകരെല്ലാം നല്‍കിയിരിക്കുന്നത്. ആദ്യകാല ഖുർആൻ വ്യാഖ്യാതാക്കളും നൽകിയ അർഥം അത് തന്നെ!

എന്താണ് തറാഇബ്? ത,റ,ബ എന്നീ മൂലാക്ഷരങ്ങളില്‍ നിന്ന് നിഷ്പന്നമായ 'തരീബത്തി'ന്റെ ബഹുവചനമായ 'തറാഇബി'നെയാണ് ഈ വചനത്തില്‍ വാരിയെല്ലുകള്‍ എന്ന് പരിഭാഷപ്പെടുത്താറുള്ളത്. ഇംഗ്ലീഷ് പരിഭാഷകളിൽ തറാഇബി'ന് breast bone എന്നും ribs എന്നും അര്‍ഥം നല്‍കിയിട്ടുണ്ട്.. ഇതില്‍നിന്ന് വ്യത്യസ്തമായി മുഹമ്മദ് അസദ് നല്‍കിയിട്ടുള്ള പരിഭാഷ pelvic arch എന്നാണ്; ഇത് സ്ത്രീ ശരീരത്തെ കുറിക്കുന്ന പദപ്രയോഗമാണെന്നാണ് ഖുര്‍ആനില്‍ ഉപയോഗിക്കപ്പെട്ടിരിക്കുന്ന അസാധാരണ പ്രയോഗങ്ങളെക്കുറിച്ച് ഗവേഷണം നടത്തിയിട്ടുള്ളവരുടെ അഭിപ്രായമെന്നും മുഹമ്മദ് അസദ് രേഖപ്പെടുത്തുന്നുണ്ട്.

തറാഇബ് എന്ന പ്രയോഗം അറബിസാഹിത്യങ്ങളില്‍ സാധാരണയായി കാണപ്പെടാത്തതാണ്. ഖുര്‍ആനില്‍ ഒരു തവണ മാത്രമെ ഇങ്ങനെ പ്രയോഗിച്ചിട്ടുള്ളൂ. പ്രവാചകവചനങ്ങളില്‍ വ്യാപകമായി 'തറാഇബ്' എന്നോ അതിന്റെ ഏകവചനരൂപമായ 'തരീബത്ത്' എന്നോ പ്രയോഗിക്കപ്പെട്ടിട്ടില്ല. ഖുര്‍ആനിനു മുമ്പുള്ള കവിതകളില്‍നിന്ന് ഇംറുല്‍ ഖൈസിന്റെ ഒരു കവിതയെ ഉദ്ധരിച്ചുകൊണ്ടാണ് ഇത് സ്ത്രീയുടെ ശരീരഭാഗങ്ങളെ കുറിക്കുവാന്‍ മാത്രമുപയോഗിക്കുന്നതാണെന്ന് ലിസാനുന്‍ അറബ്, താജുല്‍ അറൂസ് തുടങ്ങിയ ആധികാരിക അറബിഭാഷാ നിഘണ്ടുകള്‍ അഭിപ്രായപ്പെടുന്നത്. ആധുനിക അറബിഭാഷാ നിഘണ്ടുകളില്‍ പലതിലും ഈ പദം തന്നെയില്ല. വാരിയെല്ലിനെ കുറിക്കുന്നതിന് 'ള്വില്‍അ്' എന്നും അതിന്റെ ബഹുവചനമായി 'ള്വുലൂഅ്' എന്നുമാണ് അവയിലുള്ളത്. 'സ്വുല്‍ബി'നെപ്പോലെ വ്യാപകമായ പ്രയോഗത്തിലുള്ളതോ സാഹിത്യങ്ങളില്‍ നിറഞ്ഞുനില്‍ക്കുന്നതോ ആയ പദമല്ല 'തറാഇബ്' എന്നര്‍ഥം.

വാരിയെല്ലുകള്‍ക്ക് മാത്രമാണോ ഇങ്ങനെ പ്രയോഗിക്കാറുള്ളത്? അല്ലെന്ന വസ്തുത പുരാതന അറബി നിഘണ്ടുകളെല്ലാം സാക്ഷ്യപ്പെടുത്തുന്നുണ്ട്. സ്തനങ്ങള്‍, സ്തനങ്ങള്‍ക്കിടയിലുള്ള സ്ഥലം, നെഞ്ചെല്ല്, മാറിടം, വാരിയെല്ലുകളില്‍ താഴത്തെ നാലെണ്ണം, സ്തനങ്ങള്‍ക്കും, പൂണെല്ലിനു (collar bone) മിടയിലുള്ള സ്ഥലം, പൂണെല്ലിനടുത്തുള്ള വാരിയെല്ലുകള്‍, രണ്ടു കൈകളും രണ്ടുകാലുകളും രണ്ട് കണ്ണുകളും ഇങ്ങനെ വിവിധ അര്‍ഥങ്ങളില്‍ തറാഇബ് എന്ന് പ്രയോഗിക്കാമെന്നാണ് നിഘണ്ടുകള്‍ വ്യക്തമാക്കുന്നത്. വില്യം ലെയിനിന്റെ പ്രസിദ്ധമായ ലെക്‌സിക്കണില്‍ നല്‍കുന്ന അര്‍ഥങ്ങളും ഇങ്ങനെത്തന്നെയാണ്

എന്താണ് 'തറാഇബ്' എന്ന വിഷയത്തില്‍ ആദ്യകാലം മുതല്‍ക്കേ പണ്ഡിതന്‍മാര്‍ക്ക് അഭിപ്രായവ്യത്യാസമുണ്ടായിരുന്നുവെന്ന വസ്തുത ഇമാം റാസി തന്റെ തഫ്‌സീറില്‍ വ്യക്തമാക്കിയിട്ടുണ്ട്. എന്നാല്‍ സ്വഹാബിമാരും ആദ്യകാല വ്യാഖ്യാതാക്കളുമെല്ലാം ഇത് സ്ത്രീയുടെ ശരീരഭാഗമാണെന്ന നിലയ്ക്കു തന്നെയാണ് കാര്യങ്ങള്‍ വിശദീകരിച്ചിരിക്കുന്നത്. സ്ത്രീയുടെ കഴുത്തിനു താഴെയുള്ള ശാരീരികാവയവങ്ങളിലേതോ ഒന്നാണ് തറാഇബു കൊണ്ട് വിവക്ഷിച്ചത് എന്ന് ഇവയില്‍നിന്ന് മനസ്സിലാക്കാം.

സ്വുല്‍ബ്, തറാഇബ് എന്നീ പദങ്ങളെക്കുറിച്ച പഠനത്തെ ഇങ്ങനെ സംഗ്രഹിക്കാം:

1) ബലിഷ്ഠമായത്, നട്ടെല്ല്, അരക്കെട്ട്, എന്നിങ്ങനെ അര്‍ഥങ്ങളുള്ള അറബിപദമാണ് സ്വുല്‍ബ്. പുരുഷന്റെ സ്വുല്‍ബില്‍നിന്നാണ് കുഞ്ഞുങ്ങളുണ്ടാകുന്നെതന്ന മധ്യപൂര്‍വദേശത്ത് നൂറ്റാണ്ടുകളായി നിലനിന്ന പ്രയോഗം അവന്റെ അരക്കെട്ടിനെ ഉദ്ദേശിച്ചുകൊണ്ടുള്ളതാണ്. അരക്കെട്ടിലുള്ള വൃഷണമായിരിക്കണം സൂറത്തുത്വാരിഖിലെ സ്വുല്‍ബു കൊണ്ടുള്ള വിവക്ഷ.

2) വാരിയെല്ലുകള്‍, സ്തനങ്ങള്‍, കൈകാലുകള്‍, വസ്തികമാനം എന്നിങ്ങനെയുള്ള അര്‍ഥങ്ങളില്‍ പ്രയോഗിക്കപ്പെട്ടിരുന്ന അപൂര്‍വപദങ്ങളിലൊന്നാണ് 'തറാഇബ്'. അറബി ഭാഷാകാരന്‍മാരും ആദ്യകാല ഖുര്‍ആന്‍ വ്യാഖ്യാതാക്കളുടെയും അഭിപ്രായപ്രകാരം ഈ പദപ്രയോഗം മാലയിടുന്ന ഭാഗത്തുള്ള ഏതോ പെണ്ണവയവത്തെക്കുറിക്കുന്നതാണ്. അന്ധരാശയം സ്ഥിതിചെയ്യുന്ന ഭാഗത്തെക്കുറിച്ച കൃത്യമായ പ്രയോഗമാണിത്.

ഇനി പ്രവാചകനിൽ (സ) നിന്ന് ക്വുർആൻ പഠിച്ച ഇബ്നു അബ്ബാസും ഇക്രിമയുമെല്ലാം മനസ്സിലാക്കിയ രൂപത്തിൽ സൂറത്തുത്വാരികിലെ വചനങ്ങളുടെ മലയാള അർഥം ഇങ്ങനെ എഴുതാം:

''തെറിച്ചു വീഴുന്ന ദ്രാവകത്തില്‍ നിന്നാണ് അവന്‍ സൃഷ്ടിക്കപ്പെട്ടിരിക്കുന്നത്. പുരുഷന്റെ അരക്കെട്ടിൽ നിന്നും സ്ത്രീയുടെ മാലയിടുന്ന ഭാഗത്ത് നിന്നുമായി അത് പുറത്ത് വരുന്നു."

പ്രവാചകനിൽ നിന്ന് സ്വഹാബിമാരും അവരിൽ നിന്ന് താബിഉകളുമെല്ലാം ഈ അർത്ഥമാണ് മനസ്സിലാക്കിയതെങ്കിൽ പിന്നെയെങ്ങനെയാണ് നട്ടെല്ലിനും വാലിയെല്ലിനുമിടയിൽ നിന്ൻ പുറത്തുവരുന്ന ശുക്ലത്തിൽ നിന്നാണ് കുഞ്ഞുണ്ടാവുന്നതെന്നാണ് ഈ വചനം അര്ഥമാക്കുന്നതെന്ന വ്യാഖ്യാനമുണ്ടായത് എന്ന ചോദ്യം പ്രസക്തമാണ്. സ്വഹാബിമാരുടെ വ്യാഖ്യാനത്തിൽ നിന്ന് വ്യത്യസ്തമായി പുരുഷാവയവത്തിനും 'തറാഇബ്' എന്നു പറയാമെന്നും ശുക്ളത്തെക്കുറിച്ച് മാത്രമായിരിക്കണം ഈ വചനങ്ങളിൽ പറഞ്ഞിരിക്കുന്നതെന്നും വ്യാഖ്യാനിച്ച പണ്ഡിതന്‍മാരുമുണ്ട്. ഇബ്‌നുല്‍ഖയ്യിം (റഹ്) അവരില്‍ പ്രധാനിയാണ്. പുരുഷന്റെ തന്നെ സ്വുല്‍ബിനും തറാഇബിനുമിടയില്‍നിന്ന് പ്രവഹിക്കുന്ന ശുക്ലദ്രാവകത്തെ കുറിച്ചാണ് സൂറത്തുത്വാരിഖിലെ പരാമര്‍ശമെന്നാണ് അദ്ദേഹത്തിന്റെ പക്ഷം. താബിഉകളില്‍പെട്ട ഖത്താദ (റഹ്) യില്‍നിന്നും ഇത്തരമൊരു അഭിപ്രായം ജലാലുദ്ദീന്‍ സുയൂത്വി (റഹ്) തന്റെ ദുര്‍റുല്‍ മന്‍ഥൂറില്‍ ഉദ്ധരിച്ചിട്ടുമുണ്ട്. സ്ത്രീയുടെ തറാഇബില്‍നിന്നും പുരുഷന്റെ സ്വുല്‍ബില്‍നിന്നും തെറിച്ചുവീഴുന്ന ദ്രാവകങ്ങളുടെ സംഗമത്തെക്കുറിച്ചാണ് സൂറത്തു ത്വാരിഖിലെ വചനങ്ങള്‍ വ്യക്തമാക്കുന്നതെന്ന വ്യാഖ്യാനത്തില്‍ സംശയം പ്രകടിപ്പിച്ചവര്‍ പ്രധാനമായും ഉന്നയിച്ചത് 'സ്ത്രീയുടെ ദ്രാവകം തെറിച്ചുവീഴുന്നതല്ലല്ലോ'യെന്ന ന്യായമായിരുന്നു.

സ്ത്രീയുടെ ദ്രാവകം കൊണ്ട് വിവക്ഷിക്കുന്നത് രതിബാഹ്യലീലകള്‍ നടക്കുമ്പോള്‍ പുറത്തുവരുന്ന ബര്‍ത്തോലിന്‍ സ്രവമോ (bartholin fluid) രതിമൂര്‍ച്ഛാ സമയത്ത് പാരായുറിത്രല്‍ നാളികളില്‍നിന്ന് (paraurethral ducts) പുറത്തുവരുന്ന ദ്രാവകമോ ആണെന്ന് ധരിച്ചവരായിരുന്നു ഈ വാദമുന്നയിച്ച വ്യാഖ്യാതാക്കള്‍. ഈ രണ്ട് ദ്രാവകങ്ങളും തെറിച്ചു വീഴുന്നതല്ലെന്ന് അവർ മനസ്സിലാക്കിയിരുന്നു. അണ്ഡോല്‍സര്‍ജനത്തെയോ ആ സമയത്തുണ്ടാവുന്ന ശാരീരിക മാറ്റങ്ങളെയോ കുറിച്ച അറിവ് അവര്‍ ഈ വാദമുന്നയിക്കുന്ന കാലത്ത് ഉണ്ടായിരുന്നില്ല. അതുകൊണ്ടുതന്നെ സ്ത്രീപുരുഷ ദ്രാവകങ്ങളുടെ സമന്വയത്തെപ്പറ്റിത്തന്നെയാണോ സ്വുല്‍ബില്‍നിന്നും തറാഇബില്‍നിന്നും പുറത്തുവരുന്ന ദ്രാവകത്തെകുറിച്ച് ഖുര്‍ആന്‍ പരാമര്‍ശിക്കുമ്പോള്‍ അര്‍ഥമാക്കുന്നതെന്ന് അവര്‍ സംശയിച്ചു. പുരുഷദ്രാവകം തെറിച്ചു വീഴുന്നതാണെന്ന് അവര്‍ക്ക് അറിയാമായിരുന്നു. എന്നാല്‍ സ്ത്രീയുടെ ദ്രാവകം തെറിച്ചുവീഴുന്നതല്ലെന്ന അന്നത്തെ അറിവിന്റെ വെളിച്ചത്തില്‍ ഈ വചനം പുരുഷബീജത്തെക്കുറിച്ചു മാത്രമാണെന്ന് വാദിക്കുകയാണ് അവര്‍ ചെയ്തത്. എന്നാല്‍ ഇന്നു നമുക്കറിയാം, ബര്‍ത്തോലിന്‍ സ്രവമോ പാരായുറിത്രല്‍ സ്രവമോ കുഞ്ഞിന്റെ പിറവിയില്‍ പങ്കാളിയാവുന്നില്ലെന്നും അതില്‍ പങ്കാളിയാവുന്നത് അണ്ഡാശയത്തിനകത്ത് ഉത്പാദിപ്പിക്കപ്പെടുന്ന അണ്ഡദ്രാവകം മാത്രമാണെന്നും.

ശുക്ലദ്രാവകത്തെപ്പോലെ അണ്ഡദ്രാവകവും തെറിച്ചു വീഴുന്നതാണോ? പെൺശരീരത്തിന്റെ ആന്തരിക ഭാഗത്ത് നടക്കുന്ന അണ്ഡോൽസർജനത്തെപ്പറ്റി ഈയടുത്ത കാലം വരെ നമുക്ക് കാര്യമായൊന്നും അറിയില്ലായിരുന്നു. ആന്തരികാവയവങ്ങളിൽ നടക്കുന്നതെന്താണെന്ന് ക്രത്യമായി മനസ്സിലാക്കാൻ ഇന്ന് മാർഗങ്ങളുണ്ട്. ആര്‍ത്തവചക്രത്തിന്റെ പതിനാലാം ദിവസം ഹൈപോതലാമസിന്റെ ഉദ്ദീപനത്തിന് വിധേയമാകുന്ന അണ്ഡാശയത്തിനകത്തെ പൂര്‍ണവളര്‍ച്ചയെത്തിയ ഫോളിക്കിളില്‍ പ്രത്യക്ഷപ്പെടുന്ന സ്റ്റിഗ്മയെന്ന ദ്വാരത്തിലൂടെ പ്രായപൂര്‍ത്തിയായ അണ്ഡത്തെ വഹിച്ചുകൊണ്ട് ഫോളിക്കുളാര്‍ ദ്രവവും ക്യുമുലസ് കോശങ്ങളും പുറത്തേക്ക് തെറിക്കുകയും അത് ഫലോപ്പിയന്‍ നാളിയുടെ അറ്റത്തുള്ള ഫിംബ്രയകളില്‍ പതിക്കുകയും ചെയ്യുന്ന പ്രക്രിയയാണ് അണ്ഡോല്‍സര്‍ജനം (ovulation). ഇങ്ങനെ ഉല്‍സര്‍ജിക്കപ്പെട്ടഅണ്ഡമാണ് പുരുഷബീജവുമായി സംയോജിക്കുന്നത്. ഇതെല്ലാം നടക്കുന്നത് വാരിയെല്ലിന്റെ കൂടിനുതാഴെ വസ്തികമാനത്തിന് മുകളിലായി സ്ഥിതിചെയ്യുന്ന ഗര്‍ഭാശയത്തിന്റെ രണ്ടറ്റത്തായി കാണപ്പെടുന്ന അണ്ഡാശയങ്ങളിലും അനുബന്ധ അവയവങ്ങളിലുമായാണ്. പുരുഷലിംഗത്തിൽ നിന്ന് ശുക്ലം തെറിച്ചു വീഴുന്നതുപോലെ ഫോളിക്യൂൾ പൊട്ടി അണ്ഡദ്രാവകം തെറിച്ചു വീഴുന്നത് ഇന്ന് നമുക്ക് കാണാൻ കഴിയും. കാണണമെന്നാഗ്രഹിക്കുന്നവർ ഈ ലിങ്ക് സന്ദർശിക്കുക: https://www.youtube.com/watch?v=dq3MdeSDDC4

വിമര്ശിക്കപ്പെട്ടുകൊണ്ടിരിക്കുന്ന വചനത്തിന് സ്വഹാബിമാർ നൽകിയ അർഥം നമുക്കൊന്ന് കൂടി വായിക്കാം:

'തെറിച്ചു വീഴുന്ന ദ്രാവകത്തില്‍ നിന്നാണ് അവന്‍ സൃഷ്ടിക്കപ്പെട്ടിരിക്കുന്നത്. പുരുഷന്റെ അരക്കെട്ടിൽ നിന്നും സ്ത്രീയുടെ മാലയിടുന്ന ഭാഗത്ത് നിന്നുമായി അത് പുറത്ത് വരുന്നു."

താഴെപറയുന്ന വസ്തുതകൾ ശ്രദ്ധിക്കുക:

1) രതിമൂര്‍ച്ഛയിലെത്തുമ്പോള്‍ അനുതപ്ത നാഡീവ്യവസ്ഥയുടെ ഉദ്ദീപനപ്രകാരം വൃഷണത്തില്‍നിന്ന് ബീജാണുക്കള്‍ ബീജവാഹിനിക്കുഴലിലൂടെ മുകളിലേക്ക് കയറി സ്ഖലനനാളിയിലെത്തുകയും അതോടടുത്തുള്ള പ്രോസ്റ്റേറ്റ്, സെമിനല്‍ വെസിക്കിളുകള്‍, കൗപേഴ്‌സ് ഗ്രന്ഥി എന്നിവയില്‍ നിന്ന് ഉല്‍പാദിപ്പിക്കപ്പെടുന്ന സ്രവങ്ങളുമായിച്ചേര്‍ന്ന് സ്ഖലനനാളിയില്‍ നിന്ന് ലിംഗനാളിയിലൂടെ പുറത്തേക്ക് തെറിക്കുകയും ചെയ്യുന്നു. നട്ടെല്ലിന്റെ വാലിന് (coccyx) മുന്‍പിലായാണ് സെമിനല്‍ വെസിക്കിളുകളും പ്രോസ്റ്റേറ്റും കൗപേഴ്‌സ് ഗ്രന്ഥിയും സ്ഖലനനാളിയുമെല്ലാം സ്ഥിതിചെയ്യുന്നത്. പുരുഷശുക്ലം പുറത്തേക്ക് തെറിക്കുന്നത് അരക്കെട്ടിലെ വ്യത്യസ്ത അവയവങ്ങളുടെ ഒന്നിച്ചുള്ള പ്രവര്‍ത്തനം വഴിയാണ്. കൃത്യമായും അരക്കെട്ടിൽ നിന്നുതന്നെയാണ് പുരുഷദ്രാവകം പുറത്തേക്ക് തെറിക്കുന്നത് സ്വുല്‍ബില്‍നിന്നുതന്നെ!

2) ആര്‍ത്തവചക്രത്തിന്റെ പതിനാലാം ദിവസം ഹൈപോതലാമസിന്റെ ഉദ്ദീപനത്തിന് വിധേയമാകുന്ന അണ്ഡാശയത്തിനകത്തെ പൂര്‍ണവളര്‍ച്ചയെത്തിയ ഫോളിക്കിളില്‍ പ്രത്യക്ഷപ്പെടുന്ന സ്റ്റിഗ്മയെന്ന ദ്വാരത്തിലൂടെ പ്രായപൂര്‍ത്തിയായ അണ്ഡത്തെ വഹിച്ചുകൊണ്ട് ഫോളിക്കുളാര്‍ ദ്രവവും ക്യുമുലസ് കോശങ്ങളും പുറത്തേക്ക് തെറിക്കുകയും അത് ഫലോപ്പിയന്‍ നാളിയുടെ അറ്റത്തുള്ള ഫിംബ്രയകളില്‍ പതിക്കുകയും ചെയ്യുന്നു. ഇത് നടക്കുന്നത് വാരിയെല്ലിന്റെ കൂടിനുതാഴെ വസ്തികമാനത്തിന് മുകളിലായി സ്ഥിതിചെയ്യുന്ന ഗര്‍ഭാശയത്തിന്റെ രണ്ടറ്റത്തായി കാണപ്പെടുന്ന അണ്ഡാശയങ്ങളിലും അനുബന്ധ അവയവങ്ങളിലുമായാണ്. ഖുര്‍ആന്‍ 'തറാഇബ്' എന്നു വിളിച്ച സ്ഥലത്തുവെച്ചുതന്നെയാണ് അണ്ഡദ്രാവകം (ovular fluid) അണ്ഡാശയത്തില്‍നിന്ന് പുറത്തേക്ക് തെറിക്കുന്നത്. കൃത്യമായും 'തറാഇബിൽ നിന്ന് തന്നെ !

ഇനി നാം ചിന്തിക്കുക. ക്വുർആനിനാണോ വിമര്ശകര്ക്കാനോ തെറ്റുപറ്റിയതെന്ന്...!!!
വിഷയവുമായി ബന്ധപ്പെട്ട വീഡിയോ

ഇണകളെ കുറിച്ച ഖുര്‍ആന്‍ വചനങ്ങളിൽ എല്ലാ വസ്തുക്കളും ഇണകളായാണ് സൃഷ്ടിക്കപ്പെട്ടത് എന്ന് പറയുന്നുണ്ടല്ലോ. ജീവനുള്ളവയും ഇല്ലാത്തവയുമെല്ലാം എലാ വസ്തുക്കളെയും എന്ന് പറഞ്ഞതിൽ ഉൾപ്പെടെണ്ടതാണ്. അജൈവവസ്തുക്കളിൽ എങ്ങനെയാണ് ഇണകളുണ്ടാവുക? സസ്യങ്ങളെല്ലാം ഇണകളായാണ് സൃഷ്ടിക്കപ്പെട്ടത് എന്ന് പറയുന്ന ക്വുർആനിന് അലൈംഗിക പ്രത്യുത്പാദനം നടക്കുന്ന സസ്യങ്ങളെക്കുറിച്ചറിയില്ലെന്ന് വ്യക്തമാണ്. പിന്നെയെങ്ങനെ ഖുർആൻ സസ്യങ്ങളെയെല്ലാം സൃഷ്ടിച്ച ദൈവത്തിൽ നിന്നുള്ളതാവും ?

ണകളായാണ് എല്ലാം സൃഷ്ടിക്കപ്പെട്ടതെന്ന് ക്വുർആൻ പറയുന്നുണ്ട്; എന്നാൽ എല്ലാം ഉണ്ടാകുന്നത് ഇണകൾ തമ്മിലുള്ള ബന്ധം വഴിയാണെന്ന് ഖുർആനിലെവിടെയും പറയുന്നില്ല. ഇണകൾ എന്ന് പറയുമ്പോഴേക്ക് അത് പ്രത്യുത്പാദനവുമായി ബന്ധപ്പെട്ട പരാമര്ശമാണെന്ന് തെറ്റിദ്ധരിക്കുന്നതു കൊണ്ടാണ് ഈ വിമര്ശനമുണ്ടാവുന്നത്.

ഇണകളെക്കുറിച്ച് പറയുന്ന ഒരു ക്വുർആൻ സൂക്തം നോക്കുക:

''എല്ലാ വസ്തുക്കളില്‍ നിന്നും ഈരണ്ട് ഇണകളെ നാം സൃഷ്ടിച്ചിരിക്കുന്നു. നിങ്ങള്‍ ആലോചിച്ച് മനസ്സിലാക്കുവാന്‍ വേണ്ടി.'' (51:49)

എല്ലാവസ്തുക്കളിലും ഇണകളുണ്ട് എന്ന വചനത്തെ ബാഹ്യമായി അപഗ്രഥിച്ചാല്‍ ജീവിവര്‍ഗങ്ങളിലും സസ്യജാലങ്ങളിലും പെട്ട ഇണകളെകുറിച്ചാകാം ഇതെന്ന് ആര്‍ക്കും മനസ്സിലാവും. മനുഷ്യരിലും സസ്യവര്‍ഗങ്ങളിലും എല്ലാം പെട്ട ഇണകളെകുറിച്ച് ഖുര്‍ആന്‍ പ്രത്യേകം എടുത്ത് പറയുന്നുമുണ്ട്.

''നിങ്ങള്‍ക്ക് വേണ്ടി ഭൂമിയെ തൊട്ടിലാക്കുകയും, നിങ്ങള്‍ക്ക് അതില്‍ വഴികള്‍ ഏര്‍പെടുത്തിത്തരികയും, ആകാശത്ത് നിന്ന് വെള്ളം ഇറക്കിത്തരികയും ചെയ്തവനത്രെ അവന്‍. അങ്ങനെ അത് വഴി വ്യത്യസ്ത തരത്തിലുള്ള സസ്യങ്ങളുടെ ഇണകൾ നാം (അല്ലാഹു) ഉല്‍പാദിപ്പിക്കുകയും ചെയ്തിരിക്കുന്നു.'' (20:53)

''നിങ്ങള്‍ക്ക് സമാധാനപൂര്‍വ്വം ഒത്തുചേരേണ്ടതിനായി നിങ്ങളില്‍ നിന്ന് തന്നെ നിങ്ങള്‍ക്ക് ഇണകളെ സൃഷ്ടിക്കുകയും, നിങ്ങള്‍ക്കിടയില്‍ സ്‌നേഹവും കാരുണ്യവും ഉണ്ടാക്കുകയും ചെയ്തതും അവന്റെ ദൃഷ്ടാന്തങ്ങളില്‍ പെട്ടതത്രെ. തീര്‍ച്ചയായും അതില്‍ ചിന്തിക്കുന്ന ജനങ്ങള്‍ക്ക് ദൃഷ്ടാന്തങ്ങളുണ്ട്.'' (30:21)

മനുഷ്യരില്‍ നിന്നുള്ള ഇണകളെ കുറിച്ച് പരാമര്‍ശിക്കുമ്പോള്‍ ഖുര്‍ആന്‍ വളരെ കൃത്യമായ ചില പ്രയോഗങ്ങള്‍ നടത്തുന്നുണ്ട്. ആണിനെയും പെണ്ണിനെയും വ്യവഛേദിക്കുന്നത് സ്രവിക്കപ്പെടുന്ന ബീജമാണെന്ന വസ്തുത ഖുര്‍ആന്‍ വ്യക്തമാക്കുന്നു.

''ആണ്‍, പെണ്‍ എന്നീ രണ്ട് ഇണകളെ അവനാണ് സൃഷ്ടിച്ചതെന്നും ഒരു ബീജം സ്രവിക്കപ്പെടുമ്പോള്‍ അതില്‍ നിന്ന്...'' (53:45,46)

പുരുഷ ബീജത്തിലെ ക്രോമോസോമുകളാണ് കുഞ്ഞിന്റെ ലിംഗനിര്‍ണയം നടത്തുന്നതിന്റെ അടിത്തറയായി വര്‍ത്തിക്കുന്നതെന്ന വസ്തുത ഇന്ന് നമുക്കറിയാം. പെണ്‍കോശങ്ങളില്‍ ലിംഗക്രോമോസോമായ x മാത്രമെ കാണൂ; ഒരേ തരത്തിലുള്ള രണ്ട് ക്രോമസോമുകള്‍. അതിന് ഊനഭംഗം നടന്നുണ്ടാവുന്ന ലിംഗ കോശത്തില്‍-അണ്ഡം-ഒരേയൊരു x ക്രോമസോം മാത്രമെയുണ്ടാവൂ. എന്നാല്‍ ആണ്‍ കോശങ്ങളില്‍ XY എന്നീ രണ്ട് ലിംഗ ക്രോമസോമുകളുമുണ്ടാവൂം. ഊനഭംഗത്തിലൂടെ പുംബീജങ്ങളുണ്ടാവുമ്പോള്‍ അതില്‍ പകുതി x ക്രോമസോം ഉള്‍ക്കൊള്ളുന്നതും പകുതി Y ക്രോമസോം ഉള്‍ക്കൊള്ളുന്നതുമായിരിക്കും. x ഉള്‍ക്കൊള്ളുന്ന ബീജമാണ് അണ്ഡവുമായി യോജിക്കുന്നതെങ്കില്‍ അതുമൂലമുണ്ടാകുന്ന സിക്താണ്ഡം വളര്‍ന്ന് പെണ്‍കുട്ടിയും Y ക്രോമസോം ഉള്‍ക്കൊള്ളുന്ന ബീജവുമായാണ് അണ്ഡവുമായി സങ്കലിക്കുന്നതെങ്കില്‍ അത് ആണ്‍കുട്ടിയുമായിത്തീരുമെന്നതാണ് പൊതുവായ അവസ്ഥ. സ്രവിക്കപ്പെടുന്ന ബീജത്തില്‍ നിന്നാണ് ആണ്‍, പെണ്‍ തുടങ്ങിയ ഇണകളുണ്ടായിത്തീരുന്നതെന്ന ഖുര്‍ആനിക പരാമര്‍ശം എത്ര കൃത്യം! സൂക്ഷ്മം! ''ആണ്‍, പെണ്‍ എന്നീ രണ്ട് ഇണകളെ അവനാണ് സൃഷ്ടിച്ചതെന്നും ഒരു ബീജം സ്രവിക്കപ്പെടുമ്പോള്‍ അതില്‍ നിന്ന്...'' (53:45,46)

കൂറേക്കൂടി സൂക്ഷ്മമായ പരിശോധനയില്‍ ഓരോ തവണ സ്രവിക്കപ്പെടുന്ന ബീജങ്ങളെയും നമുക്ക് ആണ്‍ ബീജങ്ങളായും പെണ്‍ബീജങ്ങളായും വിഭജിക്കുവാനാകുമെന്ന് ബോധ്യപ്പെടുന്നു. x ക്രോമസോം ഉള്‍ക്കൊള്ളുന്നവ പെണ്‍ബീജങ്ങള്‍; Y ക്രോമസോം ഉള്‍കൊള്ളുന്നവ ആണ്‍ബീജങ്ങള്‍. സ്രവിക്കപ്പെടുന്ന ബീജത്തില്‍ തന്നെ ആണ്‍, പെണ്‍ എന്നീ രണ്ടു തരം ഇണകളുമുണ്ടെന്ന ഖുര്‍ആനിക പരാമര്‍ശം വളരെ കൃത്യമാണെന്ന് സൂക്ഷ്മ പരിശോധനയില്‍ തെളിയുന്നു.

നടേ ഉദ്ധരിച്ച ഇണകളെ കുറിച്ച് പരാമര്‍ശിക്കുന്ന ഖുര്‍ആന്‍ സൂക്തം പരിശോധിക്കുക. ''എല്ലാ വസ്തുക്കളില്‍ നിന്നും ഈരണ്ട് ഇണകളെ നാം സൃഷ്ടിച്ചിരിക്കുന്നു. നിങ്ങള്‍ ആലോചിച്ച് മനസ്സിലാക്കുവാന്‍ വേണ്ടി.'' (51:49).

എല്ലാ വസ്തുക്കളില്‍ നിന്നും ഈരണ്ട് ഇണകളെ സൃഷ്ടിച്ചിരിക്കുന്നുവെന്നാണ് ഖുര്‍ആന്‍ ഇവിടെ പറയുന്നത്. വസ്തുകളെല്ലാം നിര്‍മിക്കപ്പെട്ടിരിക്കുന്നത് ആറ്റങ്ങളെ കൊണ്ടാണെന്ന് ഇന്ന് നമുക്കറിയാം. എന്തുകൊണ്ടാണ് ആറ്റങ്ങള്‍ നിര്‍മിക്കപ്പെട്ടിരിക്കുന്നത്? പോസിറ്റീവ് ചാര്‍ജുള്ള ന്യൂക്ലിയസിന് പുറത്ത് പിടികൊടുക്കാതെ തെന്നിമാറിക്കൊണ്ടിരിക്കുന്ന ഇലക്‌ട്രോണ്‍ മേഘപടലമാണ് ആറ്റമെന്ന ചിത്രമാണ് ക്വാണ്ടം ബലതന്ത്രത്തിന്റേത്. പോസിറ്റീവ് ചാര്‍ജുള്ള പ്രോട്ടോണുകളും അതിനു തുല്യമായ എണ്ണം നെഗറ്റീവ് ചാര്‍ജുള്ള ഇലക്‌ട്രോണുകളും ചേര്‍ന്നാണ് ആറ്റത്തിന്റെ ഘടനയും സ്വഭാവങ്ങളുമെല്ലാം നിര്‍ണയിക്കുന്നത്. ഇലക്‌ട്രോണുകളും പ്രോട്ടോണുകളുമാകുന്ന ഇണകളുടെ പാരസ്പര്യമാണ് ആറ്റോമികലോകത്ത് നടക്കുന്നത്. ഖുര്‍ആന്‍ പറഞ്ഞതാണ് ശരി. എല്ലാ വസ്തുക്കളിലും പെട്ട ഇണകളെ സൃഷ്ടിച്ചവന്‍ എത്ര പരിശുദ്ധന്‍!

നമുക്ക് അറിയുന്നതും അറിയാത്തതുമായ വസ്തുകളെല്ലാം നിലനില്‍ക്കുന്നത് ഇണകളുടെ പാരസ്പര്യത്താലാണെന്നാണ് ഖുര്‍ആന്‍ നല്‍കുന്ന സൂചന. സൂറത്തു യാസീനിലെ ശ്രദ്ധേയമായ ഒരു വചനം ശ്രദ്ധിക്കുക. ''ഭൂമി മുളപ്പിക്കുന്ന സസ്യങ്ങളിലും, അവരുടെ സ്വന്തം വര്‍ഗങ്ങളിലും, അവര്‍ക്കറിയാത്ത വസ്തുക്കളിലും പെട്ട എല്ലാ ഇണകളെയും സൃഷ്ടിച്ചവന്‍ എത്ര പരിശുദ്ധന്‍!'' (36:36) ഈ വചനത്തിലെ 'അവര്‍ക്കറിയാന്‍ പറ്റാത്ത വസ്തുക്കളിലും പെട്ട എല്ലാ ഇണകളെയും സൃഷ്ടിച്ചവന്‍' എന്ന പരാമര്‍ശം ഏറെ ശ്രദ്ധേയമാണ്.

നമുക്ക് അറിയാവുന്നതും അല്ലാത്തതുമായ വസ്തുക്കളെല്ലാം സൃഷ്ടിക്കപ്പട്ടിട്ടുള്ളത് ഇണകളായിട്ടാണ് എന്ന വസ്തുതയാണ് ആറ്റോമിക് ഭൗതികം നമുക്ക് നല്‍കിക്കൊണ്ടിരിക്കുന്ന അറിവുകളിലൊന്ന്. ഇലക്‌ട്രോണ്‍, പ്രോട്ടോണ്‍ എന്നീ ഇണകളുടെ പാരസ്പര്യത്താലാണ് ആറ്റത്തിന്റെ നിലനില്‍പെന്ന് പറഞ്ഞുവല്ലോ. ന്യൂട്രോണുകളും പ്രോട്ടോണുകളും നിര്‍മിക്കപ്പെട്ടിരിക്കുന്നത് ആറു തരം ക്വാര്‍ക്കുകളെ കൊണ്ടാണ്. ഈ ക്വാര്‍ക്കുകളെ വേര്‍പിരിക്കുവാന്‍ സാധ്യമല്ല. ന്യൂട്രോണുകള്‍ക്കും പ്രോട്ടോണുകള്‍ക്കുമകത്തുള്ള ഓരോ ക്വാര്‍ക്കും അതിന്റെ ആന്റിക്വാര്‍ക്കുമായി പരസ്പരം ഇണചേര്‍ന്നു കിടക്കുകയാണ്. അവയെ വേര്‍പിരിക്കുവാനേ സാധ്യമല്ല. ഒരിക്കലും വേര്‍പിരിക്കാനാവാത്ത ഈ ഇണചേരലിനെയാണ് 'ഇന്‍ഫ്രാറെഡ് അടിമത്തം' (infrared slavery) അല്ലെങ്കില്‍ 'വര്‍ണപരിമിതപ്പെടുത്തല്‍' (colour confinement) എന്നു വിളിക്കുന്നത്. ക്വാര്‍ക്കുകള്‍ തമ്മിലുള്ള അതിശക്തമായ ഇണചേരലിനെ കുറിച്ച പഠനശാഖയാണ് ക്വാണ്ടം ക്രോമോഡൈനാമിക്‌സ് (quantum chromodynamics). അത് പഠിക്കുമ്പോൾ ഖുര്‍ആനിനോടൊപ്പം നമ്മളും പറഞ്ഞു പോകുന്നു, നമുക്കറിയാത്ത വസ്തുക്കളില്‍ പോലും ഇണകളെ സൃഷ്ടിച്ചവന്‍ എത്ര പരിശുദ്ധന്‍!

ഇങ്ങനെ, അറിയും തോറും എല്ലാ വസ്തുകളിലുമുള്ള ഇണകളെ പറ്റി നമുക്ക് കൂടുതല്‍ കൂടുതല്‍ മനസ്സിലാവുന്നു! ഇപ്പോള്‍ നടന്നുകൊണ്ടിരിക്കുന്ന ലാര്‍ജ് ഹൈഡ്രോണ്‍ കൊളൈഡര്‍ പരീക്ഷണം ഇത്തരമൊരു ഇണയെക്കൂടി തിരഞ്ഞുകൊണ്ടുള്ളതാണല്ലോ. പ്രപഞ്ചത്തെ വിശദീകരിക്കുവാന്‍ ഇന്ന് ഉപയോഗിക്കുന്ന സ്റ്റാന്റേര്‍ഡ് മോഡല്‍ പ്രകാരം, ശ്യാമഊര്‍ജത്തെയും ശ്യാമദ്രവ്യത്തെയും കുറിച്ച് കൃത്യമായി അറിയുവാന്‍ ഉപയോഗിക്കുന്ന സൂപ്പര്‍ സിമ്മട്രിയിലെ ഓരോ കണത്തിനുമുള്ള സൂപ്പര്‍ പങ്കാളികളെ (super partners) കണ്ടെത്തുകയാണല്ലോ ആയിരം കോടി ഡോളര്‍ ചെലവു ചെയ്തു നിര്‍മിച്ച എല്‍.എച്ച്.സി യുടെ ലക്ഷ്യങ്ങളിലൊന്ന്. വസ്തുക്കള്‍ നിര്‍മിക്കാനുപയോഗിക്കപ്പെട്ട കൂടുതല്‍ സൂക്ഷ്മമായ ഇണകളെ കുറിച്ച് ലോകം അറിഞ്ഞുകൊണ്ടിരിക്കുകയാണ്. അഗാധതകളിലേക്ക് പോകുമ്പോള്‍ ഇണകളുടെ പാരസ്പര്യമാണ് സൃഷ്ടിപ്രപഞ്ചത്തിലെ എല്ലാത്തിനും നിദാനമെന്ന് മാനവരാശി മനസ്സിലാക്കിക്കൊണ്ടിരിക്കുന്നു; ഒപ്പം തന്നെ ഒരിക്കലും തെറ്റുപറ്റാത്ത വചനങ്ങളാണ് ഖുര്‍ആനിലുള്ളതെന്നും.

എല്ലാ സസ്യങ്ങളെയും ഇണകളായി സൃഷ്ടിച്ചുവെന്ന ഖുര്‍ആനിക പരാമര്‍ശം കാണ്ഡം മുറിച്ച് നടുന്ന സസ്യങ്ങളുണ്ടെന്ന വസ്തുതയുടെ വെളിച്ചത്തില്‍ അബദ്ധമാണെന്നാണ് മറ്റൊരു വിമർശനം.

സസ്യങ്ങള്‍ക്കിടയില്‍ ഇണകളുണ്ടെന്ന് വ്യക്തമാക്കുന്ന ഖുര്‍ആന്‍ സൂക്തങ്ങളുടെ സാരം ഇങ്ങനെയാണ്. ''നിങ്ങള്‍ക്ക് വേണ്ട ഭൂമിയെ തൊട്ടിലാക്കുകയും, നിങ്ങള്‍ക്ക് അതില്‍ വഴികള്‍ ഏര്‍പെടുത്തിത്തരികയും, ആകാശത്ത് നിന്ന് വെള്ളം ഇറക്കിത്തരികയും ചെയ്തവനത്രെ അവന്‍. അങ്ങനെ അത് (വെള്ളം) മൂലം വ്യത്യസ്ത തരത്തിലുള്ള സസ്യങ്ങളുടെ ജോടികള്‍ നാം (അല്ലാഹു) ഉല്‍പാദിപ്പിക്കുകയും ചെയ്തിരിക്കുന്നു.'' (20:53)

''ഭൂമി മുളപ്പിക്കുന്ന സസ്യങ്ങളിലും, അവരുടെ സ്വന്തം വര്‍ഗങ്ങളിലും, അവര്‍ക്കറിയാത്ത വസ്തുക്കളിലും പെട്ട എല്ലാ ഇണകളെയും സൃഷ്ടിച്ചവന്‍ എത്ര പരിശുദ്ധന്‍!'' (36:36)

ഈ സൂക്തങ്ങളിലൊന്നും തന്നെ സസ്യങ്ങളിലെല്ലാം പ്രത്യുല്‍പാദനം നടക്കുന്നത് ഇണകള്‍ തമ്മിലുള്ള ലൈംഗികബന്ധം വഴിയാണെന്ന സൂചനകളൊന്നും തന്നെയില്ല. സസ്യങ്ങള്‍ക്കിടയില്‍ ഇണകളുണ്ടെന്ന് മാത്രമാണ് ഈ സൂക്തങ്ങള്‍ വ്യക്തമാക്കുന്നത്. ലൈംഗിക പ്രത്യുല്‍പാദനവും അലൈംഗിക പ്രത്യുല്‍പാദനവും സസ്യങ്ങള്‍ക്കിടയില്‍ നടക്കുന്നുണ്ടെന്ന വസ്തുതയെ ഈ വചനങ്ങള്‍ നിഷേധിക്കുന്നില്ല.

പൂക്കളാണ് സസ്യങ്ങളിലെ പ്രത്യുല്‍പാദന കേന്ദ്രം. രണ്ടുതരം പൂക്കളുണ്ട്. ഏകലിംഗികളും (unisexual) ദ്വിലിംഗികളും (bisexual). ആണ്‍ ലൈംഗികാവയവമായ കേസരങ്ങളോ (androecium) പെണ്‍ലൈംഗികാവയവമായ ജനിയോ (gynoecium) മാത്രമുള്ള പുഷ്പങ്ങളാണ് ഏകലിംഗികള്‍. ഒരേ പുഷ്പത്തില്‍ തന്നെ ഇവ രണ്ടുമുണ്ടെങ്കില്‍ അവയെ ദ്വിലിംഗികള്‍ എന്നും വിളിക്കുന്നു. കേസരങ്ങളിലെ പരാഗികളില്‍ (anther) നിന്ന് പരാഗം ജനിയില്‍ പതിക്കുമ്പോഴാണ് ബീജസങ്കലനം നടക്കുന്നത്. പരാഗം ജനിയില്‍ പതിക്കുന്ന പ്രക്രിയക്കാണ് പരാഗണം (pollination) എന്നു പറയുന്നത്. ഒരു പുഷ്പത്തിലെ പരാഗം അതേ പുഷ്പത്തിലെ ജനിയില്‍ പതിക്കുന്നതിന് സ്വയംപരാഗണം എന്നും മറ്റൊരു പുഷ്പത്തിലെ ജനിയില്‍ പതിക്കുന്നതിന് പരപരാഗണം എന്നും പറയുന്നു. ചില ചെടികള്‍ സ്വയം പരാഗണം നടത്തുന്നു; മറ്റു ചിലവ പരപരാഗണവും. ഇങ്ങനെ പരാഗണം നടത്തുന്ന ചെടികളില്‍ ചിലതിനെ കാണ്ഡത്തില്‍ നിന്ന് മാത്രമായി വളര്‍ത്തിയെടുക്കാന്‍ കഴിയും. മരച്ചീനിയും ചെമ്പരത്തിയും റോസാചെടിയുമെല്ലാം ഇതിന് ഉദാഹരണങ്ങളാണ്. ഇവയില്‍ പുഷ്പങ്ങളും അതില്‍ ലൈംഗികാവയവങ്ങളുമുണ്ട്. അവ തമ്മില്‍ പരാഗണം നടക്കുന്നുണ്ടെങ്കിലും കായുണ്ടാകുന്നതിന് അത് നിമിത്തമാകുന്നില്ല; അതിന് മറ്റുചില ധര്‍മങ്ങളാണുള്ളത്. മുറിച്ച് നട്ടുകൊണ്ട്, കാണ്ഡത്തില്‍ നിന്നാണ് പുതിയ ചെടിയുണ്ടാവുന്നത്. ചെടിയുണ്ടാവുന്നത് ലൈംഗിക പ്രത്യുല്‍പാദനം വഴിയല്ലെങ്കിലും ഇവയിലും പൂക്കളുണ്ട്, അവയില്‍ ആണവയവങ്ങളും പെണ്ണവയവങ്ങളുമുണ്ട്. അവയും ഇണകളായാണ് സ്ഥിതി ചെയ്യുന്നത് എന്ന് സാരം.

അലൈംഗിക പ്രത്യുല്‍പാദനം മാത്രം നടത്തിവരുന്ന ജീവികളായി വ്യവഹരിക്കപ്പെട്ടു പോന്നിരുന്ന അമീബയെപ്പോലുള്ള ജീവികളില്‍ പോലും ചില ലൈംഗിക പെരുമാറ്റങ്ങളുണ്ടെന്ന് ഈയിടെയായി ശാസ്ത്രജ്ഞന്മാര്‍ നിരീക്ഷിച്ചിട്ടുണ്ട്. ചില അമീബകള്‍ മറ്റു ചിലവയുടെ ഇണകളായി വര്‍ത്തിക്കുന്നുണ്ടത്രെ! എഡിന്‍ ബര്‍ഗ് സര്‍വകലാശാലയുടെ ഔദ്യോഗിക വെബ്‌സൈറ്റ് (www.ed.ac.uk) ഇക്കാര്യം സ്ഥിരീകരിച്ചുകൊണ്ടുള്ള പ്രബന്ധങ്ങള്‍ പ്രസിദ്ധീകരിച്ചത് കാണാം. എല്ലാം ഇണകളായാണ് സൃഷ്ടിക്കപ്പെട്ടത് എന്ന ഖുര്‍ആനിക പരാമര്‍ശത്തിന്റെ കൃത്യതയിലേക്കാണ് ഈ ഗവേഷണങ്ങളെല്ലാം വിരല്‍ചൂണ്ടുന്നത്.

സൂര്യനെ വിളക്കായും ചന്ദ്രനെ പ്രകാശമായും വിശേഷിപ്പിച്ച ക്വുർആൻ സൂര്യൻ പ്രക്സശസ്രോതസ്സാണെന്നും ചന്ദ്രൻ അതിന്റെ പ്രകാശം പ്രതിഫലിപ്പിക്കുകയാണ് ചെയ്യുന്നതെന്നുമാണ് വ്യക്തമാക്കുന്നതെന്നും കൃത്യമായ പരാമർശങ്ങളാണ് ക്വുർആൻ നടത്തുന്നതെന്ന് ഇതിന്റെ അടിസ്ഥാനത്തിൽ വാദിക്കുന്നത് ശുദ്ധ തട്ടിപ്പാനിന്നും യുക്തിവാദികൾ പറയുന്നു. ഖുര്‍ആന്‍ 33:45,46ല്‍ മുഹമ്മദ് നബിയെ വിളക്കായും (സിറാജ്) 24:35ല്‍ അല്ലാഹുവിനെ പ്രകാശമായും (നൂര്‍) ഉപമിച്ചിട്ടുണ്ട്. മുഹമ്മദ് നബിയാണ് പ്രകാശ സ്രോതസ്സെന്നും അല്ലാഹു അദ്ദേഹത്തിന്റെ പ്രകാശം പ്രതിഫലിപ്പിക്കുകയാണെന്നുമല്ലേ നടേ പറഞ്ഞ വ്യാഖ്യാനം അംഗീകരിച്ചാല്‍ വന്നു ചേരുക. യുക്തിവാദികളുടെ വിമര്ശനത്തെപ്പറ്റി എന്ത് പറയുന്നു?

സൂര്യനെ വിളക്കായും ചന്ദ്രനെ പ്രകാശമായും ക്വുർആൻ വിശേഷിപ്പിച്ചിട്ടുണ്ട്. സൂര്യൻ പ്രക്സശസ്രോതസ്സാണെന്ന് അതിനെ വിശേഷിപ്പിച്ച സിറാജ് എന്ന പദം തന്നെ വ്യക്തമാക്കുന്നുണ്ട്. വിളക്ക് എന്നാണ് ആ പദത്തിന്റെ നേർക്ക് നേരെയുള്ള അർഥം. എന്നാൽ ചന്ദ്രനെ വിശേഷിപ്പിച്ച നൂർ എന്ന പദത്തിന് നേർക്ക് നേരെ പ്രതിഫലിക്കപ്പെട്ട പ്രകാശം എന്ന അർത്ഥമില്ല; പ്രകാശം എന്ന് മാത്രമാണ് അതിന്റെ അർത്ഥം . ചന്ദ്രൻ സൂര്യന്റെ പ്രകാശം പ്രതിഫലിപ്പിക്കുകയാണ് ചെയ്യുന്നതെന്ന് നമുക്കറിയാം. ഏത് തരം പ്രകാശമായാലും അതിന് നൂർ എന്ന് പറയും; പ്രതിഫലിക്കപ്പെട്ടതാവട്ടെ അല്ലാത്തതാകട്ടെ. സൂര്യനെക്കുറിച്ച് മാത്രമേ സിറാജ് എന്ന് ക്വുർആൻ പ്രയോഗിച്ചിട്ടുള്ളൂവെന്നും ചന്ദ്രനെക്കുറിച്ച് നൂർ എന്നും മുനീർ എന്നുമാണ് പ്രയോഗിച്ചതെന്നുമുള്ള വസ്തുതകൾ വ്യക്തമാക്കുന്നത് പ്രകാശസ്രോതസ്സാണ് സൂര്യനെന്ന വസ്തുത അറിയാവുന്നവനിൽ നിന്നാണ് അത് അവതരിപ്പിക്കപ്പെട്ടിരിക്കുന്നത് എന്നാണ്. ഇതാണ് ഇവ്വിഷയകമായി ഇസ്‌ലാമിക പ്രബോധകർ പറയാറുള്ളത്. രാത്രിയിൽ കാണുന്ന ചന്ദ്രനെയാണ് വിളക്ക് എന്ന അർത്ഥത്തിൽ സിറാജ് എന്ന് വിളിക്കാൻ ഒരു മരുഭൂനിവാസിക്ക് അനുയോജ്യമെങ്കിലും ഖുർആൻ ഒരിക്കലും ചന്ദ്രനെ അങ്ങനെ വിളിക്കുന്നില്ലെന്നതാണ് അതിലെ പദപ്രയോഗങ്ങളിലെ കൃത്യതയും സൂക്ഷ്മതയും അങ്ങനെ ദൈവികതയും വ്യക്തമാക്കുന്നത്.

ഖുര്‍ആന്‍ 33:45,46ല്‍ മുഹമ്മദ് നബിയെ വിളക്കായും (സിറാജ്) 24:35ല്‍ അല്ലാഹുവിനെ പ്രകാശമായും (നൂര്‍) ഉപമിച്ചിട്ടുണ്ടെന്നും മുഹമ്മദ് നബിയാണ് പ്രകാശ സ്രോതസ്സെന്നും അല്ലാഹു അദ്ദേഹത്തിന്റെ പ്രകാശം പ്രതിഫലിപ്പിക്കുകയാണെന്നുമല്ലേ നടേ പറഞ്ഞ വിശദീകരണം അംഗീകരിച്ചാല്‍ വന്നു ചേരുകയെന്നുമാണ് വിമര്‍ശകർ ചോദിക്കുന്നത്.

ഉദ്ധരിക്കപ്പെട്ട ഖുര്‍ആന്‍ വചനങ്ങള്‍ പരിശോധിക്കുക. ''നബിയേ, തീര്‍ച്ചയായും നിന്നെ നാം ഒരു സാക്ഷിയും സന്തോഷവാര്‍ത്ത അറിയിക്കുന്നവനും, താക്കീതുകാരനും ആയിക്കൊണ്ട് നിയോഗിച്ചിരിക്കുന്നു. അല്ലാഹുവിന്റെ ഉത്തരവനുസരിച്ച് അവങ്കലേക്ക് ക്ഷണിക്കുന്നവനും, പ്രകാശം നല്‍കുന്ന ഒരു വിളക്കും ആയിക്കൊണ്ട്.'' (33:45,46)

''അല്ലാഹു ആകാശങ്ങളുടെയും ഭൂമിയുടെയും പ്രകാശമാകുന്നു. അവന്റെ പ്രകാശത്തിന്റെ ഉപമയിതാ: (ചുമരില്‍ വിളക്ക് വെക്കാനുള്ള) ഒരു മാടം അതില്‍ ഒരു വിളക്ക്. വിളക്ക് ഒരു സ്ഫടികത്തിനകത്ത്. സ്ഫടികം ഒരു ജ്വലിക്കുന്ന നക്ഷത്രം പോലെയിരിക്കുന്നു. അനുഗൃഹീതമായ ഒരു വൃക്ഷത്തില്‍ നിന്നാണ് അതിന് (വിളക്കിന്) ഇന്ധനം നല്‍കപ്പെടുന്നത്. അതായത് കിഴക്ക് ഭാഗത്തുള്ളതോ പടിഞ്ഞാറ് ഭാഗത്തുള്ളതോ അല്ലാത്ത ഒലീവ് വൃക്ഷത്തില്‍ നിന്ന്. അതിന്റെ എണ്ണ തീ തട്ടിയില്ലെങ്കില്‍ പോലും പ്രകാശിക്കുമാറാകുന്നു. (അങ്ങനെ) പ്രകാശത്തിന്‍മേല്‍ പ്രകാശം. അല്ലാഹു തന്റെ പ്രകാശത്തിലേക്ക് താന്‍ ഉദ്ദേശിക്കുന്നവരെ നയിക്കുന്നു. അല്ലാഹു ജനങ്ങള്‍ക്ക് വേണ്ടി ഉപമകള്‍ വിവരിച്ചുകൊടുക്കുന്നു. അല്ലാഹു ഏത് കാര്യത്തെപ്പറ്റിയും അറിവുള്ളവനത്രെ.'' (24:35)

ഈ രണ്ട് വചനങ്ങളും രണ്ട് സ്വതന്ത്ര വചനങ്ങളാണ്; ഒന്ന് മറ്റേതിന്റെ ബാക്കിയോ വിശദീകരണമോ അല്ല. സൂറത്തു അഹ്‌സാബിലെ 45,46 വചനങ്ങള്‍ മുഹമ്മദ് നബി(സ)യുടെ സവിശേഷതകള്‍ വിവരിക്കുകയാണ് ചെയ്യുന്നത്. അദ്ദേഹം സാക്ഷിയും സന്തോഷവാര്‍ത്ത അറിയിക്കുന്നവനും താക്കീതുകാരനും അല്ലാഹുവിന്റെ ഉത്തരവനുസരിച്ച് അവങ്കലേക്ക് ക്ഷണിക്കുന്നവനുമാണ്; അതോടൊപ്പംതന്നെ അദ്ദേഹം ജനങ്ങള്‍ക്ക് പ്രകാശം നല്‍കുന്നവനും സ്വയം തന്നെ പ്രകാശിക്കുന്നവനമാണ്. അത് ക്പന്റാണ് അദ്ദേഹത്തെ സിറാജന്‍ മുനീറാ എന്ന് വിളിച്ചിരിക്കുന്നത്. ഇതൊരു ഉപമാലങ്കാരമാണ്. മുഹമ്മദ് നബി (സ) സ്വയം പ്രകാശിക്കുന്ന വിളക്കാണ് എന്ന കാര്യത്തില്‍ സംശയമൊന്നുമില്ല. അദ്ദേഹത്തിന്റെ കര്‍മ്മങ്ങളും നിര്‍ദ്ദേശങ്ങളും അനുവാദങ്ങളുമെല്ലാം അവസാനനാളുവരെയുള്ള മനുഷ്യര്‍ക്കെല്ലാം വെളിച്ചമായിത്തീരുന്നവയാണ്. മുഹമ്മദ് നബി (സ)യെന്ന വിളക്കില്‍ നിന്ന് പുറപ്പെടുന്ന വെളിച്ചമാണ് സുന്നത്ത്. ഇസ്‌ലാമിന്റെ രണ്ടാമത്തെ പ്രമാണമാണത്. മുഹമ്മദ് നബി (സ) സ്വയം വിളക്കായിത്തീര്‍ന്നതല്ല, പ്രത്യുത അല്ലാഹു അദ്ദേഹത്തെ വിളക്കാക്കിത്തീര്‍ത്തതാണ്. സ്വന്തം ജീവിതത്തിന്റെ പ്രകാശത്തിലൂടെ അവസാനനാളുവരെയുള്ള മുഴുവന്‍ മനുഷ്യര്‍ക്കും വഴികാട്ടിയായിത്തീരുവാനുള്ള വിളക്ക്. കെട്ടുപോയ വിളക്കല്ല അദ്ദേഹം; പ്രകാശം നല്‍കികൊണ്ടിരിക്കുന്ന സജീവമായ വിളക്കാണ്-സിറാജന്‍ മുനീറാ. എത്ര സുന്ദരമായ ഉപമാലങ്കാരം!

അല്ലാഹുവിനെ പരിചയപ്പെടുത്തുന്ന അതിസുന്ദരമായ ഖുര്‍ആന്‍ വചനങ്ങളിലൊന്നാണ് സൂറത്തുന്നൂറിലെ 35ാമത്തെ വചനം. ഇതും ഒരു ഉപമാലങ്കാരമാണ്. ആകാശഭൂമികളുടെ പ്രകാശമാണ് അല്ലാഹു. പ്രപഞ്ചത്തിന് മുഴുവന്‍ വെളിച്ചം നല്‍കുന്ന അവന്റെ പ്രകാശം മറ്റേതെങ്കിലും സ്രോതസ്സില്‍ നിന്ന് വരുന്നതല്ല. അവന്‍തന്നെയാണ് വിളക്കും വിളക്കുമാടവും അത് വെച്ചിരിക്കുന്ന സ്ഫടികക്കൂടുമെല്ലാം. പ്രകാശത്തിനു മേല്‍ പ്രകാശമാണവന്‍. അവന്റെ പ്രകാശത്തിലേക്ക് ആളുകളെ നയിക്കുന്നതും അവന്‍തന്നെ. ഇവിടെ അല്ലാഹുവിനെ കേവല പ്രകാശത്തോടല്ല ഉപമിച്ചിട്ടുള്ളതെന്ന കാര്യം പ്രത്യേകം ശ്രദ്ധേയമാണ്. അവന്‍തന്നെയാണ് വിളക്കും വിളക്കുമാടവും സ്ഫടികക്കൂടുമെല്ലാം എന്ന് വ്യക്തമാക്കുകയും അവന്റെ പ്രകാശത്തിലേക്ക് അവന്‍ തന്നെയാണ് ജനങ്ങളെ നയിക്കുന്നതെന്ന് പഠിപ്പിക്കുകയും ചെയ്യുന്നതാണ് ഈ വചനം. അല്ലാഹുവിനെ എത്ര സുന്ദരമായാണ് ഈ ഉപമയിലൂടെ ഖുര്‍ആന്‍ പരിചയപ്പെടുത്തിയിരിക്കുന്നത്!

സൂറത്തുല്‍ അഹ്‌സാബിലെ വചനം മുഹമ്മദ് നബി(സ)യെയും സൂറത്തുന്നൂറിലെ വചനം അല്ലാഹുവിനെയും സ്വതന്ത്രമായി ഉപമാലങ്കാരത്തിലൂടെ പരിചയപ്പെടുത്തുകയാണ് ചെയ്യുന്നത്. മുഹമ്മദ്‌നബി (സ) വിളക്കും അദ്ദേഹത്തില്‍നിന്നു പുറപ്പെടുന്ന പ്രകാശം അല്ലാഹുവുമാണെന്ന് ഈ വചനങ്ങള്‍ സൂചിപ്പിക്കുന്നുപോലുമില്ല. സ്വയം പ്രകാശിച്ചുകൊണ്ട് മനുഷ്യര്‍ക്ക് വെളിച്ചമാകുവാന്‍ അല്ലാഹു നിയോഗിച്ചതാണ് മുഹമ്മദ് നബി (സ)യെയെന്ന് ഒന്നാമത്തെ വചനവും പ്രപഞ്ചത്തിന്റെ വിളക്കും വെളിച്ചവുമാണ് അല്ലാഹുവെന്ന് രണ്ടാമത്തെ വചനവും വ്യക്തമാക്കുന്നു. സൂര്യനെ വിളക്കും ചന്ദ്രനെ പ്രകാശവുമായി പരിചയപ്പെടുത്തിയ വചനങ്ങളിലാകട്ടെ രണ്ടും ഒരേ വചനത്തില്‍തന്നെ പ്രതിപാദിക്കുകയും ഒന്ന് മറ്റേതിന് ഉപോല്‍ബലകമാണെന്ന് വ്യക്തമാക്കുകയും ചെയ്യുന്നുണ്ട്. സൂര്യനെ സിറാജും ചന്ദ്രനെ നൂറുമായി പരിചയപ്പെടുത്തിയതും അല്ലാഹുവിനെ നൂറും മിസ്വ്ബാഹുമായും മുഹമ്മദ് നബിയെ സിറാജന്‍ മുനീറയായും പരിചയപ്പെടുത്തിയതും തമ്മില്‍ താരതമ്യത്തിനുതന്നെ പറ്റാത്തത്ര വ്യത്യാസമുണ്ടെന്ന് സാരം.

സൂര്യനെയും ചന്ദ്രനെയും കുറിച്ച് നിരവധി പരാമർശങ്ങൾ ഖുര്ആനിലുണ്ട്. അവയിലെല്ലാം ഖുർആൻ സൂക്ഷ്മത പുലർത്തുന്നുണ്ട്. ബൈബിളിലെ ആകാശഗോളങ്ങളെക്കുറിച്ച പരാമർശങ്ങളിൽ അബദ്ധങ്ങളുണ്ടെന്ന് ചില ഗവേഷകന്മാർ അഭിപ്രായപ്പെട്ടിട്ടുണ്ട്. എങ്ങനെയായിരിക്കും ഖുർആനിലും എന്ന കരുതുന്ന ചില യുക്തിവാദികളാണ് ഈ വിമർശനം ഉന്നയിക്കാറുള്ളത്.

''ദൈവം മഹാദീപങ്ങള്‍ സൃഷ്ടിച്ചു; പകലിനെ നയിക്കാന്‍ വലുത്, രാത്രിയെ നയിക്കാന്‍ ചെറുത്'' (ഉല്‍പത്തി 1:6) എന്ന ബൈബിള്‍ വചനം സൂക്ഷ്മമായി അപഗ്രഥിച്ചാല്‍ ബൈബിള്‍ രചയിതാക്കള്‍ക്കിടയിലുണ്ടായിരുന്ന അബദ്ധധാരണകളുടെ സ്വാധീനമുള്‍ക്കൊള്ളുന്നതായി മനസ്സിലാക്കാനാവുമെന്ന് ബൈബിള്‍ ഗവേഷകരില്‍ ചിലര്‍ വ്യക്തമാക്കിയിട്ടുണ്ട്. ഇവിടെ 'മഹാദീപ'മെന്ന് പരിഭാഷപ്പെടുത്തിയിരിക്കുന്നത് 'ഗഡോള്‍ മ'ഓര്‍'(gadowl ma'owr) എന്ന ഹിബ്രു ശബ്ദത്തെയാണ്. വിളക്കിനാണ് മ'ഓര്‍ എന്ന് പറയുകയെന്ന് ബൈബിളിന്റെ ആധികാരിക ശബ്ദകോശമായ സ്‌ട്രോങ്ങ് ലക്‌സിക്കണ്‍ വ്യക്തമാക്കുന്നു. (Strongs Lexicon H -3974) പ്രകാശവുമായി ബന്ധപ്പെടുത്തി പറയുമ്പോള്‍ സൂര്യനെയും ചന്ദ്രനെയും കുറിക്കുവാന്‍ ഒരേപദം ഉപയോഗിച്ചിരിക്കുന്നത് സൂക്ഷ്മമായ അര്‍ഥത്തിലുള്ള ഒരു അബദ്ധമാണെന്നാണ് വാദം. സൂര്യചന്ദ്രന്മാര്‍ ആകാശത്ത് നിര്‍വഹിക്കുന്ന ദൗത്യം രണ്ടാണെന്നിരിക്കെ, രണ്ടിനെയും ദീപമായി ഉപമിച്ചിരിക്കുന്നത് ശരിയല്ലെന്നും പ്രകാശം പുറപ്പെടുവിക്കുന്ന സൂര്യന്‍ ദീപമാണെങ്കില്‍ അത് പ്രതിഫലിപ്പിക്കുന്ന ചന്ദ്രന്‍ പ്രകാശപ്രതിബിംബം മാത്രമാണെന്നും രണ്ടും പ്രകാശം പുറപ്പെടുവിക്കുന്ന ആകാശഗോളങ്ങളാണെന്ന അബദ്ധധാരണയില്‍ നിന്നാണ് ഈ ഉപമാപ്രയോഗമുണ്ടായിരിക്കുന്നത് എന്നതുകൊണ്ടു തന്നെ ഈ പ്രയോഗം സ്ഖലിതമാണെന്നുമുള്ള വിമര്‍ശനങ്ങളില്‍ കഴമ്പുണ്ടെന്ന് തന്നെയാണ് ബൈബിള്‍ പരാമര്‍ശങ്ങളെയും അതിന്റെ രചനാചരിത്രത്തെയും കുറിച്ച് പഠിച്ചാല്‍ നമുക്ക് മനസ്സിലാവുക.

ഇതിൽ നിന്ന് തികച്ചും വ്യത്യസ്തമാണ് ഖുർആനിലെ പരാമർശങ്ങൾ. സൂര്യചന്ദ്രന്‍മാരെക്കുറിച്ച ഖുര്‍ആന്‍ പരാമര്‍ശങ്ങള്‍ ശ്രദ്ധിക്കുക.

''ചന്ദ്രനെ അവിടെ ഒരു പ്രകാശമാക്കിയിരിക്കുന്നു. സൂര്യനെ ഒരു വിളക്കുമാക്കിയിരിക്കുന്നു.'' (71:16)

''സൂര്യനെ ഒരു പ്രകാശമാക്കിയത് അവനാകുന്നു. ചന്ദ്രനെ അവനൊരു ശോഭയാക്കുകയും, അതിന് ഘട്ടങ്ങള്‍ നിര്‍ണയിക്കുകയും ചെയ്തിരിക്കുന്നു. നിങ്ങള്‍ കൊല്ലങ്ങളുടെ എണ്ണവും കണക്കും അറിയുന്നതിന് വേണ്ടി. യഥാര്‍ഥ മുറപ്രകാരമല്ലാതെ അല്ലാഹു അതൊന്നും സൃഷ്ടിച്ചിട്ടില്ല. മനസ്സിലാക്കുന്ന ആളുകള്‍ക്കു വേണ്ടി അല്ലാഹു തെളിവുകള്‍ വിശദീകരിക്കുന്നു.'' (10:5)

''ആകാശത്ത് നക്ഷത്രമണ്ഡലങ്ങള്‍ ഉണ്ടാക്കിയവന്‍ അനുഗ്രഹപൂര്‍ണനാകുന്നു. അവിടെ അവന്‍ ഒരു വിളക്കും (സൂര്യന്‍) വെളിച്ചം നല്‍കുന്ന ചന്ദ്രനും ഉണ്ടാക്കിയിരിക്കുന്നു.'' (25:61)

ഈ വചനങ്ങളില്‍ സൂര്യനെ വിളിച്ചിരിക്കുന്നത് സിറാജ്, ദ്വിയാഅ് എന്നിങ്ങനെയാണ്. സിറാജ് എന്നാല്‍ 'വിളക്ക്' എന്നാണര്‍ഥം; ദ്വിയാഅ് എന്നാല്‍ ' തിളങ്ങുന്ന ശോഭ'യെന്നും. ചന്ദ്രനെ വിളിച്ചരിക്കുന്നതാകട്ടെ നൂര്‍ എന്നോ മുനീര്‍ എന്നോ ആണ്. നൂര്‍ എന്നാല്‍ 'പ്രകാശം' എന്നാണര്‍ഥം; മുനീര്‍ എന്നാല്‍ 'വെളിച്ചം നല്‍കുന്നത്' എന്നും. സിറാജ് പ്രകാശത്തിന്റെ സ്രോതസ്സാണ്. നൂര്‍ അത് നിര്‍മിക്കുന്ന പ്രകാശവും. സൂര്യനാണ് പ്രകാശത്തിന്റെ സ്രോതസ്സ് എന്നും ചന്ദ്രനില്‍ നിന്ന് ലഭിക്കുന്നത് സൂര്യനില്‍ നിര്‍മിക്കപ്പെടുന്ന പ്രകാശമാണെന്നും സ്വയം പ്രകാശിക്കാത്ത ചന്ദ്രനില്‍ സൂര്യപ്രകാശം പ്രതിചലിക്കുന്നതുകൊണ്ടാണ് അതില്‍നിന്ന് നമുക്ക് വെളിച്ചം ലഭിക്കുന്നത് എന്നും ഇന്നു നമുക്കറിയാം. ഖുര്‍ആന്‍ അവതരിപ്പിക്കപ്പെടുന്ന കാലത്ത് മനുഷ്യര്‍ക്ക് ഇല്ലാതിരുന്ന അറിവാണിത്. എത്ര കൃത്യമാണ് ഖുര്‍ആനിക പരാമര്‍ശങ്ങള്‍!

'സിറാജ്' എന്ന അറബി പദത്തിന്റെ നേര്‍ക്കുനേരെയുള്ള അര്‍ഥം 'വിളക്ക്' എന്നാണ്. രാത്രിയിലാണ് മനുഷ്യര്‍ക്ക് വിളക്ക് ആവശ്യമായി വരാറുള്ളത്. നല്ല നിലാവുള്ള രാത്രിയില്‍ ചന്ദ്രന്‍ നമുക്ക് വിളക്കിന് പകരമാവാറുണ്ട്. അതുകൊണ്ടുതന്നെ സാധാരണഗതിയില്‍ ചന്ദ്രനെയാണ് വിളക്കിനോട് ഉപമിക്കുവാന്‍ ഏറ്റവും അനുയോജ്യം. മനുഷ്യരുടെ ഉപമാലങ്കാരങ്ങളില്‍ അങ്ങനെയാണ് കാണപ്പെടുക. ഖുര്‍ആന്‍ ഇവിടെ കൃത്യത പുലര്‍ത്തുന്നു. സൂര്യനാണ് യഥാര്‍ഥത്തില്‍ വിളക്ക്; പ്രകാശത്തിന്റെ സ്രോതസ്സ്. ചന്ദ്രനില്‍ നാം കാണുന്നത് പ്രതിഫലിക്കപ്പെട്ട പ്രകാശം മാത്രമാണ്. ഖുര്‍ആന്‍ സൂര്യനെ സിറാജായും ചന്ദ്രനെ നൂറായും പരിചയപ്പെടുത്തുന്നു. പതിനാലു നൂറ്റാണ്ടു മുമ്പത്തെ അറിവിന്റെ അടിസ്ഥാനത്തില്‍ എഴുതപ്പെട്ടതായിരുന്നുവെങ്കില്‍ ഇത്ര കൃത്യമായ പരാമര്‍ശങ്ങള്‍ കാണുവാന്‍ നമുക്ക് കഴിയുകയില്ലായിരുന്നു. സര്‍വ്വേശ്വരനായ തമ്പുരാന്റെ വചനങ്ങളാണ് ഖുര്‍ആന്‍ എന്ന വസ്തുത വ്യക്തമാക്കുന്നതാണ് ഈ കൃത്യത.

രിക്കലും തെറ്റുപറ്റാത്തതെന്ന് വിശ്വസിക്കപ്പെടുന്ന ഖുര്‍ആന്‍ വചനങ്ങളെ തെറ്റാന്‍ സാധ്യതയുള്ള ശാസ്ത്ര സിദ്ധാന്തങ്ങളുടെ വെളിച്ചത്തില്‍ വ്യാഖ്യാനിക്കുന്നത് അപടകരവും ബാലിശവുമാണെന്ന് കരുതുന്നവരുണ്ട്. ഖുര്‍ആന്‍ ശാസ്ത്രപഠനങ്ങള്‍ പരിധിവിടുമ്പോള്‍ അവ അപകടകരമായിത്തീരാറുണ്ടെന്നത് ശരിയാണ്. എന്നാല്‍ ഖുര്‍ആനിന്റെ ദൈവികത വ്യക്തമാക്കുന്ന പഠനങ്ങളെ മുഴുവന്‍ നിഷേധിക്കുവാന്‍ അത് കാരണമായിക്കൂടാ. ഖുര്‍ആന്‍-ശാസ്ത്ര പഠനങ്ങളുടെ യഥാര്‍ഥധര്‍മം മനസ്സിലാവാത്തതുകൊണ്ടാണ് ഇത്തരം വിമര്‍ശനങ്ങളുണ്ടാവുന്നത്.

ശാശ്വത മൂല്യങ്ങളാണ് ഖുര്‍ആന്‍ പഠിപ്പിക്കുന്നത്. അതിലെ പ്രധാനപ്പെട്ട പ്രതിപാദ്യങ്ങളെല്ലാം ശാസ്ത്രീയമായ അപഗ്രഥനത്തിന് പുറത്തുള്ളവയാണ്. സ്രഷ്ടാവും സംരക്ഷകനുമായ തമ്പുരാനിലുള്ള വിശ്വാസവും അവനെ മാത്രം ആരാധിക്കേണ്ടതിന്റെ ആവശ്യകതയുമാണ് ഖുര്‍ആനിക പരാമര്‍ശങ്ങളുടെ കേന്ദ്രബിന്ദു. പ്രപഞ്ചത്തെ സൃഷ്ടിച്ച് പരിപാലിക്കുന്ന, പദാര്‍ഥം ലോകത്തിന് അതീതനായ അല്ലാഹുവിന്റെ അസ്തിത്വമോ അവന്റെ ആരാധ്യതയോ ശാസ്ത്രീയമായ അപഗ്രഥനത്തിന് കഴിയുന്നതല്ല. മരണാനന്തരജീവിതവും അതിലെ രക്ഷാ ശിക്ഷകളുമാണ് ഖുര്‍ആനില്‍ പ്രതിപാദിക്കപ്പെടുന്ന രണ്ടാമത്തെ പ്രധാനപ്പെട്ട കാര്യം. ഇവയും ശാസ്ത്രീയമായ നിരീക്ഷണങ്ങള്‍ക്ക് അതീതമായ വസ്തുതകളാണ്.

നന്‍മതിന്‍മകളെക്കുറിച്ച ഉല്‍ബോധനമാണ് പിന്നീട് ഖുര്‍ആനിലുള്ളത്. ധര്‍മാധര്‍മങ്ങളെ വ്യവഛേദിക്കുവാന്‍ ശാസ്ത്രത്തിന്റെ പക്കല്‍ മാനദണ്ഡങ്ങളൊന്നുമില്ല. ഖുര്‍ആനിന്റെ പ്രധാനപ്പെട്ട പ്രമേയങ്ങളൊന്നും തന്നെ ശാസ്ത്രീയമായ അപഗ്രഥനത്തിന് പറ്റുന്നതല്ല. അതുകൊണ്ടുതന്നെ 'ഖുര്‍ആനിനെ ശാസ്ത്രീയമായി വ്യാഖ്യാനിക്കുക'യെന്ന് പറയുന്നത് തന്നെ ശുദ്ധഭോഷ്‌ക്കാണ്. ശാസ്ത്രത്തിന്റെ അപഗ്രഥന വിശദീകരണ പരിധിയില്‍ വരാത്ത കാര്യങ്ങളെ എങ്ങനെയാണ് ശാസ്ത്രീയമായി വ്യാഖ്യാനിക്കുക?

പ്രകൃതി പ്രതിഭാസങ്ങളെക്കുറിച്ച ഖുര്‍ആനിക പരാമര്‍ശങ്ങള്‍ തെറ്റു പറ്റാത്തവയാണെന്നതിന് ശാസ്ത്രീയ നിരീക്ഷണങ്ങള്‍ നല്‍കുന്ന തെളിവുകളെക്കുറിച്ച പഠനം ഖുര്‍ആനിനെ ശാസ്ത്രീയമായി വ്യാഖ്യാനിക്കലല്ല; അങ്ങനെ ആയിക്കൂടാ. ഖുര്‍ആനിന്റെ വെളിച്ചത്തില്‍ ശാസ്ത്രഗവേഷണങ്ങളെ പരിശോധനാവിധേയമാക്കലാണ് അത്. ഖുര്‍ആനിക പരാമര്‍ശങ്ങള്‍ എത്രത്തോളം കൃത്യവും തെറ്റുപറ്റാത്തവയുമാണെന്ന് മനസ്സിലാക്കുവാന്‍ ശാസ്ത്രീയ ഗവേഷണങ്ങളെ ഉപയോഗപ്പെടുത്തുക മാത്രമാണ് ഇവിടെ ചെയ്യുന്നത്. അതല്ലാതെ, നിലവിലുള്ള ശാസ്ത്രജ്ഞാനത്തിന് അനുസൃതമായി ഖുര്‍ആന്‍ വചനങ്ങളെയോ പരാമര്‍ശങ്ങളെയോ വ്യാഖ്യാനിച്ച് വികലമാക്കലല്ല ഖുര്‍ആന്‍-ശാസ്ത്ര പഠനങ്ങള്‍ ചെയ്യേണ്ടത്. അങ്ങനെ വ്യാഖ്യാനിക്കുന്ന പഠനങ്ങള്‍ യാതൊരു ന്യായീകരണവുമര്‍ഹിക്കുന്നില്ല. തെറ്റു പറ്റാത്ത അല്ലാഹുവിന്റെ വചനങ്ങളെ വ്യാഖ്യാനിക്കുവാന്‍ അബദ്ധങ്ങള്‍ സംഭവിച്ചുകൊണ്ടിരിക്കുന്ന ശാസ്ത്രത്തെ ഉപയോഗപ്പെടുത്തുന്നത് നീതീകരിക്കുവാന്‍ കഴിയാത്ത കാര്യമാണ്.

ശാസ്ത്രത്തിന്റെ തെറ്റുപറ്റാനുള്ള സാധ്യതയെക്കുറിച്ചു പറയുമ്പോള്‍ ശാസ്ത്രീയ നിഗമനങ്ങള്‍, സിദ്ധാന്തങ്ങള്‍, വസ്തുതകള്‍ എന്നിവ തമ്മിലുള്ള വ്യത്യാസം നാം കൃത്യമായി മനസ്സിലാക്കേണ്ടതാണ്. തനിക്ക് ലഭിച്ച വിവരങ്ങളുടെ വെളിച്ചത്തില്‍ ശാസ്ത്രജ്ഞന്‍ ആദ്യമായി ഒരു 'നിഗമന'ത്തില്‍ (hypothesis) എത്തിച്ചേരുന്നു. പ്രസ്തുത നിഗമനത്തിന് ഉപോല്‍ബലകമായ തെളിവുകള്‍ ശേഖരിക്കുകയും പ്രസ്തുത തെളിവുകളുടെ വെളിച്ചത്തില്‍ ഒരു സിദ്ധാന്തത്തിന് (theory) അയാള്‍ രൂപം നല്‍കുകയും ചെയ്യുന്നു. പ്രസ്തുത സിദ്ധാന്തം ശരിയാണെങ്കില്‍ കണ്ടുപിടിക്കപ്പെടേണ്ട കാര്യങ്ങളെക്കുറിച്ച പ്രവചനങ്ങള്‍ ശരിയാണെന്ന് സ്ഥിരീകരിക്കപ്പെടുന്നതോടെ അത് ശാസ്ത്രലോകം അംഗീകരിക്കുന്ന സിദ്ധാന്തങ്ങളുടെ ഗണത്തിലെത്തിച്ചേരുന്നു. സ്വീകരിക്കപ്പെട്ട സിദ്ധാന്തവുമായി ബന്ധപ്പെട്ട സംശയങ്ങള്‍ ദൂരീകരിക്കപ്പെടുകയും ഉത്തരം കണ്ടെത്തേണ്ട പ്രശ്‌നങ്ങള്‍ക്ക് പരിഹാരമാവുകയും ചെയ്യുന്നതോടെ അത് ഒരു യാഥാര്‍ഥ്യമായി (fact) അംഗീകരിക്കപ്പെടുന്നു.

ശാസ്ത്രലോകത്ത് അംഗീകരിക്കപ്പെട്ട യാഥാര്‍ഥ്യങ്ങള്‍ തെറ്റുപറ്റാത്തവയാകാമെങ്കിലും അവ വീണ്ടും വികസിക്കുവാന്‍ സാധ്യതയുള്ളതാണ്. സൂര്യപ്രകാശത്തില്‍ അടങ്ങിയിരിക്കുന്നത് സപ്തവര്‍ണങ്ങളാണ് (vibgyor) എന്നത് ശാസ്ത്രീയമായി അംഗീകരിക്കപ്പെട്ട ഒരു യാഥാര്‍ഥ്യമാണ്. എന്നാല്‍ സപ്തവര്‍ണങ്ങളെ കൂടാതെ അള്‍ട്രാവയലറ്റ്, ഇന്‍ഫ്രാറെഡ് തുടങ്ങിയ കിരണങ്ങള്‍ കൂടി സൂര്യപ്രകാശത്തിലുണ്ട് എന്നത് പ്രസ്തുത യാഥാര്‍ഥ്യത്തിന്റെ വികാസമാണ്. ശാസ്ത്രീയമായി സ്ഥിരീകരിക്കപ്പെട്ട വസ്തുതകള്‍പോലും പൂര്‍ണമായിക്കൊള്ളണമെന്നില്ല എന്നര്‍ഥം.

വിശുദ്ധ ഖുര്‍ആനില്‍ അബദ്ധങ്ങളൊന്നുമില്ലെന്ന് ശാസ്ത്രീയമായ പഠനങ്ങള്‍ വ്യക്തമാക്കുന്നുവെന്ന് പറയുമ്പോള്‍ സ്ഥിരീകരിക്കപ്പെട്ട വസ്തുതകള്‍ ഖുര്‍ആനിന്റെ അപ്രമാദിത്വത്തെ അംഗീകരിക്കുന്നുവെന്ന് മാത്രമെ അര്‍ഥമാക്കുന്നുള്ളൂ. ശാസ്ത്രജ്ഞരുടെ നിഗമനങ്ങളും തെളിയിക്കപ്പെടാത്ത സിദ്ധാന്തങ്ങളും ചിലപ്പോള്‍ ഖുര്‍ആനിക പരാമര്‍ശങ്ങളോട് വൈരുധ്യം പുലര്‍ത്തുന്നുണ്ടാവാം. അവ ഖുര്‍ആനിന്റെ സാധുതയെ ഒരുവിധത്തിലും ബാധിക്കുന്നില്ല. പ്രസ്തുത സിദ്ധാന്തങ്ങള്‍ക്ക് അനുസൃതമായി ഖുര്‍ആനിനെ വ്യാഖ്യാനിക്കുവാന്‍ ശ്രമിക്കുന്നത് ദൈവികഗ്രന്ഥത്തോട് ചെയ്യുന്ന വലിയ പാതകമാണ്. ശാസ്ത്രലോകംതന്നെ അംഗീകരിച്ചു കഴിഞ്ഞിട്ടില്ലാത്ത സങ്കല്‍പ്പങ്ങള്‍ക്കു അനുസരിച്ച് സ്രഷ്ടാവിന്റെ വചനങ്ങളെ വ്യാഖ്യാനിച്ച് വികലമാക്കുന്നത് യാതൊരുവിധ ന്യായീകരണവുമര്‍ഹിക്കുന്നില്ല. തെളിയിക്കപ്പെട്ട വസ്തുകള്‍ക്ക് അനുസൃതമായി ഖുര്‍ആന്‍ വ്യാഖ്യാനിക്കേണ്ടതില്ല; വസ്തുതകള്‍ വ്യാഖ്യാനങ്ങളില്ലാതെത്തന്നെ ഖുര്‍ആന്‍ പരാമര്‍ശങ്ങളെ സത്യപ്പെടുത്തുന്നവയായിരിക്കും എന്നതാണ് യാഥാര്‍ഥ്യം. ഖുര്‍ആന്‍ ദൈവികമാണെന്നതിന് അംഗീകരിക്കപ്പെട്ട ശാസ്ത്രീയ വസ്തുതകള്‍ നല്‍കുന്ന തെളിവുകള്‍ വെളിപ്പെടുത്തുകയാണ്, ശാസ്ത്രത്തിനനുസരിച്ച് ഖുര്‍ആന്‍ വ്യാഖ്യാനിക്കുകയല്ല ഖുര്‍ആന്‍-ശാസ്ത്ര പഠനങ്ങളുടെ ലക്ഷ്യമെന്ന് ചുരുക്കം.

ഖുര്‍ആനില്‍ എല്ലാ ശാസ്ത്രീയ വസ്തുതകളും പരാമര്‍ശിക്കപ്പെട്ടിട്ടുണ്ടെന്നോ അതില്‍ പറഞ്ഞിട്ടുള്ള കാര്യങ്ങള്‍ മാത്രമാണ് ശാസ്ത്രീയ ഗവേഷണങ്ങള്‍ പുറത്തുകൊണ്ടുവരുന്നത് എന്നോ ഉള്ള അവകാശ വാദങ്ങളൊന്നും മുസ്‌ലിംകള്‍ക്കില്ല. ഖുര്‍ആന്‍ ശാസ്ത്രം പഠിപ്പിക്കുവാന്‍ വേണ്ടി അവതരിപ്പിക്കപ്പെട്ടതല്ല എന്നതുകൊണ്ടുതന്നെ അതില്‍ സകല ശാസ്ത്രവും ഉണ്ടെന്ന് ആര്‍ക്കും അവകാശപ്പെടാനാവില്ല; അങ്ങനെ ആരും അവകാശപ്പെടുന്നുമില്ല. പ്രകൃതി പ്രതിഭാസങ്ങളെക്കുറിച്ച് പരാമര്‍ശങ്ങളുള്‍ക്കൊള്ളുന്ന ഖുര്‍ആന്‍ വചനങ്ങളുടെ കൃത്യതയും അപ്രമാദിത്വവും ശാസ്ത്രീയ ഗവേഷണങ്ങള്‍ വ്യക്തമാക്കുകയാണ് ചെയ്യുന്നത് എന്നാണ് മുസ്‌ലിംകള്‍ അവകാശപ്പെടുന്നത്. ഇതൊരു കേവലമായ അവകാശവാദമല്ല. ആര്‍ക്കും പരിശോധിച്ച് സ്വയം തന്നെ ബോധ്യപ്പെടാന്‍ കഴിയുന്ന വസ്തുതയാണത്. തങ്ങളുടെ കൈവശമുള്ള പൗരാണികമോ ആധുനികമോ ആയ ഏത് മാനദണ്ഡമുപയോഗിച്ച് പരിശോധനാവിധേയമാക്കിയാലും ഖുര്‍ആന്‍ അബദ്ധങ്ങളില്‍ നിന്നു മുക്തമാണെന്ന് ആര്‍ക്കും മനസ്സിലാവും.

ശാസ്ത്രീയ വസ്തുതകള്‍ മുഴുവനുമോ ഗവേഷണങ്ങളിലൂടെ നിര്‍മ്മിച്ചെടുക്കുന്ന സാമഗ്രികളെക്കുറിച്ച വിവരങ്ങളോ ഖുര്‍ആനില്‍ മുമ്പേ പരാമര്‍ശിക്കപ്പെട്ടിട്ടുണ്ട് എന്നതല്ല നമ്മുടെ അവകാശവാദമെന്ന വസ്തുത ഖുര്‍ആനിന്റെ അനുകൂലികളും പ്രതികൂലികളും ഒരേപോലെ മനസ്സിലാക്കേണ്ടതുണ്ട്. ഖുര്‍ആനില്‍ അബദ്ധങ്ങളില്ലെന്ന യാഥാര്‍ഥ്യത്തിന് ശാസ്ത്രീയമായ ഗവേഷണങ്ങള്‍ തെളിവു നല്‍കുന്നുവെന്നാണ് മുസ്‌ലിംകളുടെ വാദം.

ഇത് വേണ്ട രൂപത്തില്‍ മനസ്സിലാക്കാത്തതിനാല്‍ ചിലപ്പോഴെല്ലാം ഖുര്‍ആന്‍ ശാസ്ത്ര പഠനങ്ങള്‍പരിധിവിട്ട അവകാശവാദങ്ങള്‍ ഉന്നയിക്കുന്ന അവസ്ഥയിലെത്തിച്ചേറാറുണ്ട്. ഖുര്‍ആനിലുള്ളതെല്ലാം ശാസ്ത്രമാണെന്നും ഖുര്‍ആനിന്റെ അംഗീകാരമില്ലാത്തതൊന്നും ശാസ്ത്രമല്ലെന്നുമുള്ള രീതിയിലുള്ള പരാമര്‍ശങ്ങളും ശാസ്ത്രത്തിന് വിശദീകരിക്കുവാന്‍ കഴിഞ്ഞിട്ടില്ലാത്ത മേഖലകളെക്കുറിച്ച ഖുര്‍ആന്‍ പരാമര്‍ശങ്ങളെ ശാസ്ത്രീയമായി വ്യാഖ്യാനിക്കുവാനുള്ള ത്വരയുമെല്ലാം പരിധിവിട്ടതും അംഗീകരിക്കുവാന്‍ കഴിയാത്തതുമാണ്. ഒരു കളങ്കവുമില്ലാത്ത വിശുദ്ധ നെയ്യാണ് ശാസ്ത്രമെന്ന ധാരണയുടെ വെളിച്ചത്തിലാണ് ഇത്തരം കസര്‍ത്തുകളെല്ലാം അരങ്ങേറാറുള്ളത്. ശാസ്ത്ര നിഗമനങ്ങളും സിദ്ധാന്തങ്ങളും വസ്തുതകളും തമ്മിലുള്ള ബന്ധത്തെക്കുറിച്ച ശാസ്ത്രദര്‍ശനത്തിന്റെ കാഴ്ചപ്പാട് എന്താണെന്നുപോലും ഇത്തരം വ്യാഖ്യാനവിശാരദന്‍മാര്‍ പരിഗണിക്കാറില്ല. ശാസ്ത്രത്തിന്റെ ലേബലില്‍ കാണപ്പെടുന്നതെല്ലാം സത്യമാണെന്ന ധാരണയുടെ വെളിച്ചത്തില്‍ നടക്കുന്ന ഇത്തരം ഖുര്‍ആന്‍-ശാസ്ത്ര പഠനങ്ങള്‍ക്ക് ഖുര്‍ആനിന്റെ അംഗീകാരമില്ല; അവയ്ക്ക് ശാസ്ത്രീയമായ അടിത്തറയുമുണ്ടാകാറില്ല. അങ്ങനെയുള്ള പഠനക്കസര്‍ത്തുകള്‍ മുന്നില്‍വെച്ച്, ഖുര്‍ആനില്‍ അബദ്ധങ്ങളൊന്നുമില്ലെന്ന വസ്തുതയ്ക്ക് ശാസ്ത്രം സാക്ഷ്യം വഹിക്കുന്നുവെന്ന വസ്തുത വ്യക്തമാക്കുന്ന ഗവേഷണങ്ങളെ പിന്തിരിപ്പിക്കാനായി അവതരിപ്പിക്കുന്നത് ന്യായീകരണമര്‍ഹിക്കുന്നില്ല.

വിഷയവുമായി ബന്ധപ്പെട്ട വീഡിയോ

ര്‍വ്വശക്തനായ സ്രഷ്ടാവില്‍നിന്ന് അവതരിപ്പിക്കപ്പെട്ടതാണെന്ന് സ്വയം പ്രഖ്യാപിക്കുന്ന ഒരേയൊരു വേദഗ്രന്ഥമാണ് ഖുര്‍ആന്‍.

''തീര്‍ച്ചയായും ഇത് (ഖുര്‍ആന്‍) ലോകരക്ഷിതാവ് അവതരിപ്പിച്ചത് തന്നെയാകുന്നു.'' (ഖുര്‍ആന്‍ :26:192)

''. ഈ ഗ്രന്ഥത്തിന്റെ അവതരണം സര്‍വ്വലോകരക്ഷിതാവിങ്കല്‍ നിന്നാകുന്നു. ഇതില്‍ യാതൊരു സംശയവുമില്ല.'' (ഖുര്‍ആന്‍ : 32: 2)

''പരമകാരുണികന്‍; ഈ ഖുര്‍ആന്‍ പഠിപ്പിച്ചു.'' (ഖുര്‍ആന്‍ : 55:1,2)

ദൈവികഗ്രന്ഥത്തില്‍ അബദ്ധങ്ങളൊന്നുമുണ്ടാകുവാന്‍ പാടില്ല; തെറ്റുപറ്റാത്തവനായ പടച്ചവനില്‍നിന്ന് അവതീര്‍ണമായതെന്ന് അവകാശപ്പെടുന്ന ഗ്രന്ഥത്തില്‍ തെറ്റുകളുണ്ടാകുവാന്‍ പാടില്ലെന്നത് സാമാന്യമായ ഒരു വസ്തുതയാണ്. എന്നാല്‍ മനുഷ്യരുടെ രചനകള്‍ അങ്ങനെയല്ല; എത്ര വലിയ ബുദ്ധിജീവിയുടെ രചനയാണെങ്കിലും അതില്‍ അബദ്ധങ്ങളുണ്ടാകാവുന്നതാണ്. അവ ചിലപ്പോള്‍ അയാളുടെ ജീവിതകാലത്ത്തന്നെ വെളിപ്പെടും. അതല്ലെങ്കില്‍ തലമുറകള്‍ കഴിഞ്ഞായിരിക്കും അത് ബോധ്യപ്പെടുക. മനുഷ്യരുടെ അറിവ് പരിമിതമായതിനാലും അത് വളര്‍ന്നുകൊണ്ടിരിക്കുന്നതിലുമാണ് ഇത്. ഇന്നലെയുള്ള അറിവിന്റെ അടിസ്ഥാനത്തില്‍ എഴുതിയതിലെ അബദ്ധങ്ങള്‍ ഇന്ന് കൂടുതല്‍ കൃത്യമായ അറിവ് ലഭിക്കുമ്പോള്‍ നാം തിരുത്തുന്നു. വിജ്ഞാനവര്‍ധനവിനനുസരിച്ച് സംഭവിക്കുന്ന സ്വാഭാവികമായ മാനവിക പ്രക്രിയയാണിത്. എല്ലാം അറിഞ്ഞു കഴിഞ്ഞുവെന്ന ഒരു അവസ്ഥ മാനവസമൂഹത്തിന് ഒരിക്കലും ഉണ്ടാകുവാന്‍ പോകുന്നില്ല എന്നിരിക്കെ ഈ തിരുത്തല്‍ പ്രക്രിയ മനുഷ്യാവസാനംവരെ തുടര്‍ന്നുകൊണ്ടിരിക്കും. ഒരു വിജ്ഞാനീയവും ഒരിക്കലും സ്വയം സമ്പൂര്‍ണമാകുന്നില്ല എന്നതുകൊണ്ടുതന്നെ അതിലുള്ള അറിവ് എപ്പോഴും വര്‍ധമാനമായിരിക്കുകയും പ്രസ്തുത വര്‍ധനവിനനുസരിച്ച് ഇന്നലെത്തെ വിവരങ്ങള്‍ തിരുത്തപ്പെട്ടുകൊണ്ടിരിക്കുകയും ചെയ്യും.

സ്രഷ്ടാവ് സര്‍വ്വജ്ഞനാണ്. അവന്റെ അറിവ് എന്നും സ്വയം സമ്പൂര്‍ണമാണ്. പ്രസ്തുത അറിവിലേക്ക് യാതൊന്നും കൂട്ടിച്ചേര്‍ക്കപ്പെടുകയോ അതില്‍നിന്ന് എന്തെങ്കിലും കുറയുകയോ ചെയ്യുന്നില്ല. അതുകൊണ്ടുതന്നെ അവന്റെ വചനങ്ങളില്‍ അബദ്ധങ്ങളുണ്ടായിക്കൂടാ. ദൈവികമെന്നവകാശപ്പെടുന്ന ഏതെങ്കിലുമൊരു ഗ്രന്ഥത്തില്‍ അബദ്ധങ്ങളുണ്ടെങ്കില്‍ പ്രസ്തുത അവകാശവാദം തെറ്റാണെന്നതിന് അതുതന്നെ മതിയായ തെളിവാണ്.

പ്രപഞ്ചത്തെയും പ്രകൃതിയെയും കുറിച്ച വസ്തുനിഷ്ഠമായ പഠനമാണ് ശാസ്ത്രം. ശാസ്ത്രീയമായ വിജ്ഞാനീയങ്ങളെല്ലാം വര്‍ധമാനമായ അറിവിന്റെ അടിസ്ഥാനത്തില്‍ ചിട്ടപ്പെടുത്തപ്പെട്ടവയാണ്. നമ്മുടെ അന്വേഷണം പുരോഗമിക്കുന്നതിനനുസരിച്ച് ഓരോ വിജ്ഞാനത്തിലുമുള്ള പൂര്‍വ്വകാല ധാരണകളില്‍ പലതും ചോദ്യം ചെയ്യപ്പെടുകയും തിരുത്തപ്പെടുകയും ചെയ്തിട്ടുണ്ട്. ഇന്നലെ ശരിയായിരുന്നുവെന്ന് കരുതിയവ തെറ്റാണെന്ന് മനസ്സിലാവുകയും പുതിയതും കൃത്യവുമായ ശരികളിലെത്തിച്ചേരുകയും ചെയ്യുകയെന്നത് ശാസ്ത്രലോകത്തെ സ്വാഭാവികമായ പ്രതിഭാസമാണ്. വിജ്ഞാനീയങ്ങളെക്കുറിച്ച പ്രതിപാദനങ്ങളുള്‍ക്കൊള്ളുന്ന ഒരു ഗ്രന്ഥത്തില്‍ തലമുറകള്‍ കഴിഞ്ഞിട്ടും തിരുത്തലുകളൊന്നും വേണ്ടിവരുന്നില്ലയെന്ന വര്‍ത്തമാനം ശാസ്ത്രലോകത്തിന് തീരെ ഉള്‍ക്കൊള്ളാനാവാത്തതാണ്. പ്രകൃതിയെയും പ്രാപഞ്ചിക പ്രതിഭാസങ്ങളെയും കുറിച്ച പരാമര്‍ശങ്ങളുണ്ടെങ്കില്‍ അതില്‍ തിരുത്തലുകള്‍ ആവശ്യമായിവരും എന്നതാണ് ശാസ്ത്രലോകത്തിന്റെ പൊതുവായ കാഴ്ചപ്പാട്.

ഖുര്‍ആന്‍ ദൈവികമാണെന്ന വസ്തുത ബോധ്യപ്പെടുന്ന പ്രധാനപ്പെട്ട മേഖലകളിലൊന്നാണിത്. പ്രപഞ്ചത്തെയും പ്രകൃതിയെയും കുറിച്ച നിരവധി പരാമര്‍ശങ്ങള്‍ പരിശുദ്ധ ഖുര്‍ആനിലുണ്ട്. മനുഷ്യരുടെ ദൈനംദിന ജീവിതവുമായി ബന്ധപ്പെടുന്ന വിജ്ഞാനീയങ്ങളിലുള്ള പരാമര്‍ശങ്ങളാല്‍ നിബിഢമാണ് ഖുര്‍ആന്‍ വചനങ്ങള്‍. പതിനാല് നൂറ്റാണ്ടുകള്‍ക്ക് മുമ്പ് നിരക്ഷരനായ ഒരു മനുഷ്യന്റെ (സ) നാവിലൂടെയാണ് ലോകം ഈ വചനങ്ങള്‍ ശ്രവിച്ചത്. ഇന്നു നാം ഉപയോഗിക്കുന്ന ശാസ്ത്രീയമായ മാനദണ്ഡങ്ങള്‍ വെച്ചു നോക്കുമ്പോള്‍ വിജ്ഞാനീയങ്ങള്‍ അവയുടെ ഭ്രൂണദശപോലും പ്രാപിച്ചിട്ടില്ലാത്ത സമയത്താണ് ഖുര്‍ആന്‍ വചനങ്ങള്‍ അവതരിപ്പിക്കപ്പെട്ടത്. മറ്റു ഗ്രന്ഥങ്ങളെ പരിശോധിക്കുവാന്‍ ഉപയോഗിക്കുന്ന മാനദണ്ഡങ്ങള്‍ വെച്ചു നോക്കുമ്പോള്‍ ഖുര്‍ആന്‍ അബദ്ധങ്ങളാല്‍ നിബിഢമാകേണ്ടതാണ്. എന്നാല്‍, അത്ഭുതം! ഖുര്‍ആനില്‍ അബദ്ധങ്ങളൊന്നും തന്നെ കണ്ടെത്തുവാന്‍ കഴിയുന്നില്ല. ഖുര്‍ആന്‍ പരാമര്‍ശിച്ച കാര്യങ്ങളുമായി ബന്ധപ്പെട്ട വിജ്ഞാനീയങ്ങളുടെ വളര്‍ച്ച അതില്‍ അബദ്ധങ്ങളുണ്ടെന്ന് സ്ഥാപിക്കുകയല്ല, പ്രത്യുത അതില്‍ സുബദ്ധങ്ങളേയുള്ളുവെന്ന് സ്ഥിരീകരിക്കുകയാണ് ചെയ്തിട്ടുള്ളത്.

പതിനാലു നൂറ്റാണ്ടുകള്‍ക്കു മുമ്പ് ലോകം ശ്രവിച്ച ഒരു ഗ്രന്ഥത്തില്‍, പ്രപഞ്ചത്തെയും അതിന്റെ നിലനില്‍പിനെയും, ഭൂമിയെയും അതിലെ ജീവജാലങ്ങളെയും, മനുഷ്യനെയും അവനെ നിലനിര്‍ത്തുന്ന പ്രകൃതി പ്രതിഭാസങ്ങളെയും, സൂര്യനെയും ചന്ദ്രനെയും സമുദ്രത്തെയും കാറ്റിനെയും മഴയെയും മനുഷ്യന്റെ ഭ്രൂണ പരിണാമത്തെയുമെല്ലാമുള്ള പരാമര്‍ശങ്ങളുണ്ടായിട്ട് അതിലൊന്നും യാതൊരു അബദ്ധങ്ങളുമില്ലെന്നത് അത്ഭുതകരം തന്നെയാണ്. വര്‍ധമാനമായ അറിവിന്റെ ഉടമയായ മനുഷ്യനില്‍ നിന്നുള്ളതല്ല ഈ ഗ്രന്ഥമെന്ന വസ്തുത വ്യക്തമാക്കുന്നതാണ് ഇത്. പൂര്‍ണമായ അറിവിന്റെ നാഥന് മാത്രമെ ഒരിക്കലും തെറ്റു പറ്റാത്ത ഒരു ഗ്രന്ഥം അവതരിപ്പിക്കാനാവൂ. പുതിയ പുതിയ ശാസ്ത്രീയ ഗവേഷണങ്ങള്‍ ഖുര്‍ആന്‍ വചനങ്ങളുടെ കൃത്യതയും അപ്രമാദിത്വവും വ്യക്തമാക്കുമ്പോള്‍ അത് ദൈവികമാണെന്ന വസ്തുത കൂടുതല്‍ തെളിഞ്ഞു വരികയാണ് ചെയ്യുന്നത്. ശാസ്ത്രീയമായ ഗവേഷണങ്ങള്‍ ഖുര്‍ആനിന്റെ ദൈവികതയ്ക്ക് തെളിവുകള്‍ നല്‍കിക്കൊണ്ടിരിക്കുകയാണ് ചെയ്യുന്നതെന്ന് പറയുന്നത് ഇതുകൊണ്ടാണ്.

ശാസ്ത്രം എത്ര തന്നെ വളർന്നാലും ഖുർആനിൽ തെറ്റുകളൊന്നും കണ്ടെത്താനാവില്ലെന്ന് ബോധ്യപ്പെടുത്തുന്നതിനു വേണ്ടിയുള്ളതാണ് ക്വുർആൻ-ശാസ്ത്രപഠനങ്ങൾ. പ്രകൃതിപ്രതിഭാസങ്ങളെക്കുറിച്ച ക്വുർആനിലെ പരാമർശങ്ങൾ കൃത്യവും അബദ്ധമുക്തവുമാണെന്ന വസ്തുത ശാസ്ത്രീയമായ പഠനങ്ങളുടെ വെളിച്ചത്തിൽ വ്യക്തമാക്കുന്നതിനു വേണ്ടിയുള്ളതാണത്. ക്വർആനിനെ ശാസ്ത്രീയമായി വ്യാഖ്യാനിക്കുകയോ ശാസ്തത്തെ ക്വുർആനിനനുസരിച്ച് വളച്ചോടിക്കുകയോ ചെയ്യുന്നതിന് വേണ്ടിയുള്ളതല്ല അത്.

വിഷയവുമായി ബന്ധപ്പെട്ട വീഡിയോ

ല്ല. കഞ്ഞിന്റെ സൃഷ്ടിയിൽ പുരുഷസ്രവത്തിനും സ്ത്രീസ്രവത്തിനും പങ്കുണ്ടെന്നും അവ കൂട്ടിച്ചെർന്നാണ് കുഞ്ഞുണ്ടാവുന്നത് എന്നും തന്നെയാണ് ഖുർആനും ഹദീഥുകളും വ്യക്തമാക്കുന്നത്. മനുഷ്യനെ ജലത്തില്‍നിന്നാണ് സൃഷ്ടിച്ചിരിക്കുന്നതെന്ന് പ്രസ്താവിക്കുന്ന ക്വുര്‍ആന്‍ വചനങ്ങള്‍ സൂചിപ്പിക്കുന്നത് പുരുഷസ്രവത്തില്‍നിന്നുള്ള മനുഷ്യ സൃഷ്ടിയാണെന്നാണ് പ്രമുഖരായ ക്വുര്‍ആന്‍ വ്യാഖ്യാതക്കളെല്ലാം അഭിപ്രായപ്പെട്ടിരിക്കുന്നതെന്നത് ശരിയാണ് . ജലത്തില്‍ നിന്ന് മനുഷ്യനെ സൃഷ്ടിച്ചതായി പരാമര്‍ശിക്കുന്ന സുറത്തുല്‍ ഫുര്‍ക്വാനിലെ 25ാം വചനത്തിന് വ്യാഖ്യാനമായി നിസ്സാരമായ ജലത്തില്‍നിന്നാണ് മനുഷ്യ സൃഷ്ടി നടന്നതെന്ന സൂറത്തുല്‍ മുര്‍സലാത്തിലെ 20ാം വചനവും 'നിസാരമായ ഒരു ജലത്തിന്റെ സത്തില്‍' നിന്നാണ് അത് നടന്നതെന്ന സൂറത്തുസ്സജദയിലെ എട്ടാം വചനവും നിലകൊള്ളുന്നുണ്ട്. ഈ വചനങ്ങള്‍ താരതമ്യം ചെയ്ത് പരിശോധിച്ചാല്‍ മനുഷ്യനെ സൃഷ്ടിച്ച ജലമായി ക്വുര്‍ആന്‍ പരിചയപ്പെടുത്തുന്നത് പുരുഷസ്രവമാണെന്നു തന്നെയാണ് മനസ്സിലാവുക.

സ്ത്രീയുടെ സ്രവത്തെക്കുറിച്ച് ക്വുര്‍ആനില്‍ നേര്‍ക്കുനേരെയുള്ള പരാമര്‍ശങ്ങളൊന്നുമില്ലെങ്കിലും സ്വുല്‍ബിന്റെയും തറാഇബിന്റെയും ഇടയില്‍നിന്ന് പുറപ്പെടുന്ന തെറിച്ചുവീഴുന്ന ദ്രാവകത്തില്‍നിന്നാണ് മനുഷ്യനെ സൃഷ്ടിച്ചതെന്ന് പറയുന്ന സൂറത്തുത്ത്വാരിഖിലെ ആറും ഏഴും വചനങ്ങളെ വ്യാഖ്യാനിച്ച പ്രവാചകാനുചരന്‍മാരില്‍ ക്വുര്‍ആന്‍ വ്യാഖ്യാനത്തിന് പ്രസിദ്ധനായ ഇബ്‌നു അബ്ബാസും(റ) മറ്റൊരു സ്വഹാബിയായ ഇക്‌രിമ(റ)യും പുരുഷന്റെ സ്വുല്‍ബില്‍നിന്ന് പുറപ്പെടുന്ന ദ്രാവകവും സ്ത്രീയുടെ തറാഇബില്‍നിന്ന് പുറപ്പെടുന്ന ദ്രാവകവും ഒരുമിച്ചു ചേര്‍ന്നാണ് കുഞ്ഞുണ്ടാകുന്നതെന്ന് വ്യാഖ്യാനിച്ചതായി ഇമാം ത്വബരി രേഖപ്പെടുത്തുന്നുണ്ട്.(തഫ്‌സീര്‍ അത്ത്വബ്‌രി) പ്രസിദ്ധ ക്വുര്‍ആന്‍ വ്യാഖ്യാതക്കളായ ത്വബ്‌രി, സമഖ്ശരി, ത്വബ്‌റാനി, റാസി, ക്വുര്‍തുബി, ഇബ്‌നുകഥീര്‍, ജലാലൈനി, ശൗക്വാനി തുടങ്ങിയവരെല്ലാം പുരുഷന്റെ സ്വുല്‍ബില്‍നിന്നും സ്ത്രീയുടെ തറാഇബില്‍നിന്നും പുറപ്പെടുന്ന ദ്രാവകങ്ങളുടെ മിശ്രണത്തില്‍നിന്നാണ്് കുഞ്ഞുണ്ടാവുന്നതെന്നാണ് ഈ ആയത്ത് അര്‍ത്ഥമാക്കുന്നതെന്നാണ് അഭിപ്രായപ്പെട്ടിരിക്കുന്നത്. സ്ത്രീസ്രവവും പുരുഷസ്രവവും കൂടിച്ചേര്‍ന്നാണ് കുഞ്ഞുണ്ടാകുന്നതെന്ന് പ്രവാചകാനുചരന്‍മാര്‍ പരിശുദ്ധ ക്വുര്‍ആനില്‍ നിന്നു മനസ്സിലാക്കിയിരുന്നുവെന്ന് ഇത് വ്യക്തമാക്കുന്നു.

ഹദീഥുകള്‍ ഇവ്വിഷയകമായ കൂടുതല്‍ വിശദീകരണങ്ങള്‍ നല്‍കുന്നുണ്ട്. 'സ്ത്രീകള്‍ക്ക് സ്രവമുണ്ടാകുമോ?'യെന്ന ഉമ്മുസുലൈമി (റ)ന്റെ ചോദ്യത്തിന് പ്രവാചകന്‍(സ) നല്‍കിയ മറുപടിയില്‍നിന്ന് അക്കാലത്തെ പൊതുവിശ്വാസവും അതിലെ കൃത്യമായ പ്രവാചകതിരുത്തലും നമുക്ക് ലഭിക്കുന്നു. സ്വഹീഹുല്‍ ബുഖാരിയില്‍ ഉമ്മുസലമ(റ)യില്‍നിന്ന് നിവേദനം ചെയ്യപ്പെട്ട ഈ ഹദീഥില്‍നിന്ന് സ്ത്രീയുടെ സ്രവത്തെക്കുറിച്ച് അക്കാലത്തെ സ്ത്രീകള്‍ക്കുതന്നെ അറിയില്ലായിരുന്നുവെന്ന് മനസ്സിലാക്കാം.

അത്ഭുതത്തോടുകൂടിയാണ് ഉമ്മുസുലൈം 'സ്ത്രീകള്‍ക്ക് സ്രവമുണ്ടാകുമോ?'യെന്ന് ചോദിക്കുന്നത്. സംശയം ചോദിക്കുകയെന്നതിലുപരി അങ്ങനെ ഉണ്ടാവില്ലല്ലോയെന്ന് ദ്യോതിപ്പിച്ചുകൊണ്ടുള്ള പ്രസ്തുത ചോദ്യത്തിന് 'അതെ! ഇതെന്തൊരു ചോദ്യം? പിന്നെയെങ്ങനെയാണ് കുട്ടിക്ക് അവളോട് സാദൃശ്യമുണ്ടാവുക?' എന്ന മറുചോദ്യമാണ് പ്രവാചകന്‍ (സ) മറുപടിയായി നല്‍കുന്നത്. സ്ത്രീകള്‍ക്ക് സ്രവമുണ്ടെന്ന് വ്യക്തമാക്കുക മാത്രമല്ല, അത് കുട്ടിയുടെ പാരമ്പര്യദാതാവുകൂടിയാണെന്ന് പഠിപ്പിക്കുകകൂടി ചെയ്യുന്നുണ്ട് ഈ പ്രവാചകവചനം. ഉമ്മുസുലൈമും(റ) പ്രവാചകനും(സ) തമ്മില്‍ നടന്ന ഈ സംഭാഷണം കൂറേക്കൂടി വിശദമായി ഇമാം മുസ്്‌ലിം(റ) അനസുബ്‌നു മാലിക്കില്‍ (റ) നിന്ന് നിവേദനം ചെയ്തിട്ടുണ്ട്. 'പുരുഷന്റെ സ്രവം വെളുത്തതും കട്ടിയുള്ളതുമാണ്; സ്ത്രീയുടെ സ്രവം മഞ്ഞ നിറത്തിലുള്ളതും നേര്‍മയുള്ളതുമാണ്. ഏത് സ്രവമാണോ മുന്‍കടക്കുന്നത് അതിനോടാണ് കുഞ്ഞിന് സാദൃശ്യമുണ്ടാവുക' എന്നുകൂടി ഉമ്മുസുലൈമിനോട്(റ) പ്രവാചകന്‍(സ) പറഞ്ഞതായി ഈ നിവേദനത്തിലുണ്ട്. വെളുത്ത, കട്ടിയായ പുരുഷസ്രവത്തോട് മഞ്ഞ, നേര്‍മയായ സ്ത്രീസ്രവം കൂടിച്ചേര്‍ന്നാണ് കുഞ്ഞുണ്ടാകുന്നതെന്നാണ് ഇവിടെ പ്രവാചകന്‍(സ) പഠിപ്പിക്കുന്നത്.

ഒരു ജൂത പണ്ഡിതന്റെ ചോദ്യങ്ങള്‍ക്കുള്ള പ്രവാചകന്റെ(സ) ഉത്തരത്തെപ്പറ്റി വിശദീകരിക്കുന്ന ഥൗബാന്‍(റ) നിവേദനം ചെയ്ത സ്വഹീഹ് മുസ്ലിമിലുള്ള ദീര്‍ഘമായ ഹദീഥിലും ശിശുവിന്റെ സൃഷ്ടിയെക്കുറിച്ച ചോദ്യത്തിനുള്ള വിശദമായ ഉത്തരം ആരംഭിക്കുന്നത് 'പുരുഷസ്രവം വെളുത്തനിറത്തിലുള്ളതും സ്ത്രീസ്രവം മഞ്ഞനിറത്തിലുള്ളതുമാണ്; അവ രണ്ടും കൂട്ടിച്ചെരുമ്പോൾ....' എന്നു പറഞ്ഞുകൊണ്ടാണ്. ജൂത ചോദ്യങ്ങള്‍ക്കെല്ലാം മറുപടി പറഞ്ഞശേഷം 'അയാള്‍ എന്നോട് ചോദിച്ച കാര്യങ്ങളെക്കുറിച്ചൊന്നും അല്ലാഹു അറിയിച്ചുതരുന്നതുവരെ എനിക്ക് യാതൊരു വിവരവുമുണ്ടായിരുന്നില്ല' എന്ന് പറഞ്ഞതായുള്ള ഥൗബാനി (റ)ന്റെ പരാമര്‍ശം ശ്രദ്ധേയമാണ്. സ്വന്തം സ്രവത്തെക്കുറിച്ച് അറിയാത്ത സ്ത്രീകള്‍ക്കടക്കം നിങ്ങളുടെ സ്രവം മഞ്ഞനിറത്തിലുള്ളതാണ് എന്ന് പ്രവാചകന്‍(സ) പറഞ്ഞുകൊടുത്തത് വ്യക്തമായ ദൈവബോധനത്തിന്റെ അടിസ്ഥാനത്തിലാണെന്ന് വ്യക്തമാക്കുന്നതാണീ പ്രവാചകപരാമര്‍ശം.

ഏതാണീ മഞ്ഞ ദ്രാവകം? കുഞ്ഞിന്റെ സൃഷ്ടിയില്‍ പങ്കെടുക്കുന്ന പുരുഷസ്രവത്തിന്റെ നിറം 'അബ്‌യദ്വ്' ആണെന്നു പറഞ്ഞതിനുശേഷമാണ് സ്ത്രീ സ്രവത്തിന്റെ നിറം 'അസ്വ്ഫര്‍' (മഞ്ഞ) ആണെന്ന് പ്രവാചകന്‍ (സ) പറഞ്ഞത്. രണ്ടും കൂടിച്ചേര്‍ന്നാണ് കുഞ്ഞുണ്ടാകുന്നതെന്നും അതിനുശേഷം അദ്ദേഹം വ്യക്തമാക്കി. വെള്ള നിറത്തിലുള്ള പുരുഷസ്രവത്തെപോലെതന്നെ ബീജ സങ്കലനത്തില്‍ പങ്കെടുക്കുന്ന സ്ത്രീസ്രവത്തിന്റെ നിറം മഞ്ഞയാണെന്നാണ് പ്രവാചകന്‍ (സ) ഇവിടെ പഠിപ്പിക്കുന്നതെന്നുറപ്പാണ്. സ്ത്രീശരീരത്തില്‍നിന്ന് നിര്‍ഗളിക്കുന്ന ഏതു സ്രവത്തിനാണ് മഞ്ഞനിറമുള്ളതെന്ന കാര്യത്തില്‍ കര്‍മശാസ്ത്ര പണ്ഡിതന്‍മാര്‍ ഏറെ ചര്‍ച്ച ചെയ്തതായി കാണാന്‍ കഴിയും. സ്ത്രീജനനേന്ദ്രിയത്തില്‍നിന്ന് നിര്‍ഗളിക്കുന്ന കാണാനാവുന്ന സ്രവങ്ങള്‍ക്കൊന്നും തന്നെ മഞ്ഞനിറമില്ലെന്ന വസ്തുതയാണ് വിശാലമായ ഇത്തരം ചര്‍ച്ചകളുടെ ഉല്‍ഭവത്തിന് നിമിത്തമായത്.

സ്ത്രീകളുടെ ജനനേന്ദ്രിയത്തില്‍നിന്ന് പുറത്തുവരുന്ന സ്രവങ്ങള്‍ മൂന്നെണ്ണമാണ്. തന്റെ ശരീരം ലൈംഗികബന്ധത്തിന് സജ്ജമായിയെന്ന് അറിയിച്ചുകൊണ്ട് സ്ത്രീജനനേന്ദ്രിയത്തില്‍നിന്ന് കിനിഞ്ഞിറങ്ങുന്ന ബര്‍ത്തോലിന്‍ സ്രവം(Bartholin fluid) ആണ് ഒന്നാമത്തേത്. യോനീമുഖത്തിനകത്തായി സ്ഥിതി ചെയ്യുന്ന പയര്‍വിത്തിന്റെ വലിപ്പത്തിലുള്ള രണ്ട് ബര്‍ത്തോലിന്‍ഗ്രന്ഥികള്‍ സ്ത്രീശരീരം ലൈംഗികമായി ഉത്തേജിപ്പിക്കപ്പെടുമ്പോള്‍ പുറപ്പെടുവിക്കുന്ന ഈ സ്രവത്തിന് നിറമില്ല. രതിമൂര്‍ച്ചയുടെ അവസരത്തില്‍ ചില സ്ത്രീകളുടെ ജനനേന്ദ്രിയത്തില്‍നിന്ന് പുറത്തുവരുന്ന പാരായുറിത്രല്‍ സ്രവമാണ്(Para urethral fluid) രണ്ടാമത്തെ യോനീ സ്രവം. യോനിയുടെ ആന്തരികഭിത്തിയില്‍ സ്ഥിതി ചെയ്യുന്ന പാരായുറിത്രല്‍ ഗ്രന്ഥികളില്‍നിന്നു വളരെ ചെറിയ അളവില്‍മാത്രം പുറത്തുവരുന്ന ഈ സ്രവം താരതമ്യേന കട്ടിയുള്ളതും വെള്ള നിറത്തിലുള്ളതുമായിരിക്കും. സ്ത്രീ ജനനേന്ദ്രിയത്തെ എല്ലായ്‌പ്പോഴും വരളാതെ സൂക്ഷിക്കുന്ന സെര്‍വിക്കല്‍ ശ്ലേഷ്മ (Cervical mucus) ആണ് മൂന്നാമത്തെ യോനീ സ്രവം. അണ്ഡോല്‍സര്‍ജനസമയമല്ലെങ്കില്‍ ഈ സ്രവം വഴുവഴുപ്പുള്ളതും നല്ല വെളുത്ത ക്രീം നിറത്തിലുള്ളതുമായിരിക്കും. അണ്ഡോല്‍സര്‍ജനത്തോടടുക്കുമ്പോള്‍ വെള്ളനിറം മങ്ങുകയും വഴുവഴുപ്പ് കുറയുകയും ചെയ്യുന്ന ഈ സ്രവം ഉല്‍സര്‍ജനസമയമാകുമ്പോഴേക്ക് ജലത്തെപ്പോലെ വര്‍ണരഹിതമാവുകയും മുട്ടയുടെ വെള്ളക്കരുവിനെപ്പോലെയായിത്തീരുകയും ചെയ്യും. അണുബാധയുണ്ടാകുമ്പോള്‍ മാത്രമാണ് സെല്‍വിക്കല്‍ ശ്ലേഷ്മത്തിന് മങ്ങിയ മഞ്ഞനിറമുണ്ടാകുന്നത്. സ്ത്രീജനനേന്ദ്രിയത്തില്‍നിന്ന് സാധാരണഗതിയില്‍ നിര്‍ഗളിക്കപ്പെടുന്ന മൂന്ന് സ്രവങ്ങളും വെളുത്തതോ നിറില്ലാത്തതോ ആണെന്നും ഹദീഥുകളില്‍ പറഞ്ഞ മഞ്ഞസ്രവമല്ല ഇവയെന്നും വ്യക്തമാണ്. ഇവയ്‌ക്കൊന്നുംതന്നെ കുഞ്ഞിന്റെ രൂപീകരണത്തില്‍ നേരിട്ട് പങ്കൊന്നുമില്ലതാനും.

കുഞ്ഞിന്റെ രൂപീകരണത്തിന് നിമിത്തമാകുന്ന സ്രവമെന്താണ് എന്ന ചോദ്യത്തിന് ഉത്തരം കാണാന്‍ ശ്രമിക്കുമ്പോഴാണ് ഹദീഥുകളില്‍ പറഞ്ഞ മഞ്ഞ സ്രവമേതാണെന്ന് നമുക്ക് മനസ്സിലാവുക. ആര്‍ത്തവചക്രത്തിന്റെ പതിനാലാം ദിവസം അണ്ഡാശയത്തിനകത്തെ പൂര്‍ണ വളര്‍ച്ചയെത്തിയ ഫോളിക്കിളില്‍ പ്രത്യക്ഷപ്പെടുന്ന ദ്വാരത്തിലൂടെ പ്രായപൂര്‍ത്തിയെത്തിയ അണ്ഡത്തെവഹിച്ചുകൊണ്ട് ഫോളിക്കുളാര്‍ ദ്രവവും ക്യൂമുലസ് കോശങ്ങളും പുറത്തേക്ക് തെറിച്ച് ഫലോപ്പിയന്‍ നാളിയുടെ അറ്റത്തുള്ള ഫിംബ്രയകളില്‍ പതിക്കുന്നതിനാണ് അണ്ഡോല്‍സര്‍ജനം (Ovulation) എന്നു പറയുന്നത്. രതിമൂര്‍ച്ചയോടനുബന്ധിച്ച് പുരുഷശരീരത്തില്‍ നടക്കുന്ന ശുക്ലസ്ഖലന(Ejaculation) ത്തിന് തുല്യമായി സ്ത്രീശരീരത്തില്‍ നടക്കുന്ന പ്രക്രിയയാണ് ഇതെങ്കിലും ഒരു ആര്‍ത്തവചക്രത്തില്‍ ഒരു തവണ മാത്രമാണ് ഇത് സംഭവിക്കുന്നത്. ശുക്ല സ്ഖലനവും അണ്ഡോല്‍സര്‍ജനവുമാണ് കുഞ്ഞിന്റെ സൃഷ്ടിക്ക് നിദാനമായി പുരുഷശരീരത്തിലും സ്ത്രീശരീരത്തിലും യഥാക്രമം സംഭവിക്കുന്ന രണ്ട് പ്രക്രിയകള്‍. പുരുഷബീജങ്ങളെ വഹിക്കുന്ന ശുക്ലദ്രാവകത്തെപ്പോലെ സ്ത്രീയുടെ അണ്ഡത്തെ വഹിക്കുന്ന ഫോളിക്കുളാര്‍ ദ്രവവും കുഞ്ഞിന്റെ നിര്‍മാണത്തിന് നിമിത്തമാകുന്ന ദ്രാവകമാണ്. ഹദീഥുകളില്‍ പറഞ്ഞ കുഞ്ഞിന്റെ സൃഷ്ടിക്ക് കാരണമായ സ്ത്രീസ്രവം അണ്ഡത്തെ വഹിക്കുന്ന ഫോളിക്കുളാര്‍ ദ്രാവകമാണെന്നാണ് ഇത് വ്യക്തമാക്കുന്നത്. അങ്ങനെയാണെങ്കില്‍ പുരുഷദ്രാവകം വെളുത്തതും സ്ത്രീദ്രാവകം മഞ്ഞയുമെന്ന് പരാമര്‍ശത്തിന്റെ വെളിച്ചത്തില്‍ പരിശോധിക്കുമ്പോള്‍ ഫോളിക്കുളാര്‍ ദ്രാവകത്തിന്റെ നിറം മഞ്ഞയായിരിക്കണം. എന്നാല്‍ എന്താണ് വസ്തുത?

പ്രായപൂര്‍ത്തിയെത്തുന്നതിനുമുമ്പുള്ള അണ്ഡാവസ്ഥയായ അണ്ഡത്തെ(Oocyte) സംരക്ഷിക്കുകയും വളര്‍ത്തിക്കൊണ്ടുവന്ന് ബീജസങ്കലനത്തിന് പറ്റിയ അണ്ഡമാക്കിത്തീര്‍ക്കുകയും ചെയ്യുകയാണ് ഫോളിക്കിളിന്റെ ധര്‍മം. പെണ്‍കുഞ്ഞ് ജനിക്കുമ്പോള്‍ തന്നെ അവളുടെ അണ്ഡാശയത്തിലുള്ള പ്രായപൂര്‍ത്തിയെത്താത്ത അണ്ഡകങ്ങളെ പൊതിഞ്ഞ് ആദിമ ഫോളിക്കിളുകളുണ്ടാവും (Primordial follicles).  അവള്‍ പ്രായപൂര്‍ത്തിയാകുന്നതോടെ ഇതില്‍ ചില ഫോളിക്കിളുകള്‍ വളര്‍ന്നുവരികയും ഓരോ ആര്‍ത്തവചക്രത്തിന്റെയും ശരാശരി 14-16 ദിവസങ്ങള്‍ കഴിഞ്ഞ് പൊട്ടി പൂര്‍ണവളര്‍ച്ചയെത്തിയ അണ്ഡത്തെ (Ovum) പുറത്തുവിടുന്നതോടെ അവയുടെ ധര്‍മം അവസാനിക്കുകയും ചെയ്യുന്നു. ജനനസമയത്തുള്ള ഏകദേശം 1,80,000 ഫോളിക്കിളുകളില്‍ നാനൂറെണ്ണത്തോളം മാത്രമാണ് അണ്ഡോല്‍സര്‍ജനത്തിനുമുമ്പത്തെ വളര്‍ച്ചയെത്തുവാനുള്ള ഭാഗ്യമുണ്ടാകുന്നത്. പ്രസ്തുത വളര്‍ച്ചയ്ക്ക് വ്യത്യസ്തങ്ങളായ ഘട്ടങ്ങളുണ്ട്. ഇതിലെ ഓരോ ഘട്ടങ്ങളിലും അതു കടന്നുപോകാന്‍ കഴിയാത്ത ഫോളിക്കിളുകള്‍ മരിച്ചുപോകുന്നുണ്ട്. ഓരോ ആര്‍ത്തവചക്രത്തിലും ഇരുപതോളം ഫോളിക്കിളുകള്‍ വളര്‍ച്ചയെത്തുന്നുവെങ്കിലും ഒരെണ്ണത്തിന് മാത്രമാണ് ഫോളിക്കിള്‍ മരണമായ അട്രീഷ്യ(atresia)യില്‍നിന്ന് രക്ഷപ്പെട്ട് അണ്ഡോല്‍സര്‍ജനത്തിന് കഴിയുന്നത്. അട്രീഷ്യയില്‍ നിന്ന് രക്ഷപ്പെട്ട് അണ്ഡോല്‍സര്‍ജനത്തിന് കഴിയുന്ന ഫോളിക്കിളുകള്‍ രണ്ട് ദശകളിലൂടെയാണ് കടന്നുപോകുന്നത്. അണ്ഡോല്‍സര്‍ജനത്തിലൂടെ അവസാനിക്കുന്ന ഒന്നാമത്തെ ദശയെ ഫോളിക്കുളാര്‍ ദശfollicular phase) എന്നും അതിനുശേഷമുള്ള ദശയെ ലൂടിയല്‍ ദശ (luteal phase) എന്നുമാണ് വിളിക്കുക. ആര്‍ത്തവം മുതല്‍ അണ്ഡോല്‍സര്‍ജനം വരെയുള്ള ഫോളിക്കുളാര്‍ ദശയില്‍ അണ്ഡകം പൂര്‍ണവളര്‍ച്ചയെത്തിയ അണ്ഡമായിത്തീരുന്നതിനും യഥാരൂപത്തിലുള്ള അണ്ഡോല്‍സര്‍ജനം നടക്കുന്നതിനും വേണ്ടി വ്യത്യസ്തങ്ങളായ പ്രക്രിയകള്‍ നടക്കേണ്ടതുണ്ട്. ഈ പ്രക്രിയകളുടെ അവസാനമായി ശരീരത്തിലെ ഈസ്ട്രജന്‍ നില പരമാവധി ഉയരുകയും ലൂറ്റിനൈസിംഗ് ഹോര്‍മോണ്‍ (LH), ഫോളിക്കിള്‍ സ്റ്റിമുലേറ്റിംഗ് ഹോര്‍മോണ്‍ (FSH) എന്നീ ഹോര്‍മോണുകളെ ഇതിന്റെ ഫലമായി ഉത്പാദിപ്പിക്കുകയും ചെയ്യുന്നു. 24 മുതല്‍ 36 വരെ മണിക്കൂറുകള്‍ നീണ്ടുനില്‍ക്കുന്ന ഈ പ്രക്രിയയുടെ അന്ത്യം കുറിച്ചുകൊണ്ടാണ് അണ്ഡം വഹിക്കുന്ന പൂര്‍ണവളര്‍ച്ചയെത്തിയ ഫോളിക്കിളില്‍(Ovarian follicle) സ്റ്റിഗ്മയെന്ന് പേരുള്ള ദ്വാരമുണ്ടാവുകയും അത് പൊട്ടി അണ്ഡത്തെ വഹിച്ചുകൊണ്ട് ഫോളിക്കുളാര്‍ ദ്രവം പുറത്തേക്ക് തെറിക്കുകയും ചെയ്യുന്നത്. ഈ പുറത്തേക്കു തെറിക്കല്‍ പ്രക്രിയക്കാണ് അണ്ഡോല്‍സര്‍ജനം (Ovulation)എന്നു പറയുക.

ഫോളിക്കുളാര്‍ ദശയിലുടനീളം നടക്കുന്ന അണ്ഡവളര്‍ച്ചയ്ക്കും അതിന് ഉല്‍സര്‍ജിക്കാനാവശ്യമായസംവിധാനങ്ങളൊരുക്കുന്നതിനും നിമിത്തമാകുന്നത് FSHപ്രവര്‍ത്തനങ്ങളാണ്. പ്രസ്തുത ഉത്പാദനത്തോടനുബന്ധിച്ചാണ് ഹൈപ്പോതലാമസില്‍നിന്നുള്ള ഗൊണാടോട്രോപിന്‍ റിലീസിംഗ് ഹോര്‍മോണിന്റെ(GnRH) പ്രേരണയാല്‍ പിറ്റിയൂട്ടറിയില്‍നിന്ന് LHന്റെ ഉത്പാദനം നടക്കുന്നത്. ഈ ഹോര്‍മോണ്‍ ഉത്പാദിപ്പിക്കുന്ന പ്രോട്ടീന്‍ വിഘാടക രസങ്ങളായ പ്രോട്ടിയോലിറ്റിക് എന്‍സൈമുകളാണ്ഫോ(Proteolytic enzymes) ളിക്കിളിലുണ്ടാവുന്ന ദ്വാരമായ സ്റ്റിഗ്മക്ക് കാരണമാകുന്നത്. അണ്ഡോല്‍സര്‍ജനത്തിനുശേഷമുള്ള ഫോളിക്കിള്‍ അവശിഷ്ടങ്ങളെ നിയന്ത്രിക്കുന്നതും പ്രധാനമായി ഈ ഹോര്‍മോണാണ്. ലൂട്ടിയല്‍ ദശയില്‍ അണ്ഡം നഷ്ടപ്പെട്ട ഫോളിക്കിള്‍ അവശിഷ്ടങ്ങള്‍ കോര്‍പസ് ലൂടിയം(Lorpus Luteum) ആയിത്തീരുകയും മാതൃസ്വഭാവങ്ങളെ ഉദ്ദീപിക്കുന്ന പ്രോജസ്റ്ററോണ്‍ (Progesterone) ഹോര്‍മോണിന്റെ വര്‍ധിതമായ ഉത്പാദനത്തിന് നിമിത്തമാവുകയും ചെയ്യുന്നു.

എന്താണീ ലൂറ്റിനൈസിംഗ് ഹോര്‍മോണ്‍? മഞ്ഞയെന്ന് അര്‍ത്ഥം വരുന്ന ലൂറ്റിയസ് (Luteus) എന്ന ലാറ്റിന്‍ പദത്തിന്റെ നപുംസകരൂപമായ ലൂറ്റിയത്തില്‍നിന്നാണ് (Luteum) ലൂറ്റിനൈസ് (Luteinize)എന്ന ക്രിയയുണ്ടായിരിക്കുന്നത്. കോര്‍പ്പസ് ലൂടിയത്തിന്റെ നിര്‍മിതിക്ക് നിമിത്തമായ പ്രവര്‍ത്തനങ്ങള്‍ക്കാണ് സാങ്കേതികമായി ലൂറ്റിനൈസ് എന്ന് പറയുന്നതെങ്കിലും പദപരമായി അതിനര്‍ത്ഥം 'മഞ്ഞയാക്കുന്നത്' എന്നാണ്. ലൂറ്റിനൈസിംഗ് ഹോര്‍മോണിന്റെ പ്രവര്‍ത്തനഫലമായാണ് ഫോളിക്കുളാര്‍ ദശ പിന്നിട്ട ഫോളിക്കിള്‍ അവശിഷ്ടങ്ങള്‍ കോര്‍പസ് ലൂടിയം ആയിത്തീരുന്നത്. കോര്‍പസ് ലൂടിയം എന്ന പദദ്വയത്തിനര്‍ത്ഥം മഞ്ഞ വസ്തുവെന്നാണ് (Yellow body). ലൂടിയല്‍ ദശയിലേക്ക് കടന്ന അണ്ഡം നഷ്ടപ്പെട്ട ഫോളിക്കിള്‍ അവശിഷ്ടങ്ങളെല്ലാം കൂടി രണ്ടു മുതല്‍ അഞ്ചു സെന്റീമീറ്റര്‍ വരെ വ്യാസത്തില്‍ ശരീരത്തില്‍ ഏതാനും ദിവസങ്ങള്‍ കൂടി അവശേഷിക്കും. മനുഷ്യരില്‍ ഇത് ഓറഞ്ചു നിറത്തിലാണ് കാണപ്പെടുന്നത്. അണ്ഡോല്‍സര്‍ജനത്തിന്റെ അവസാനഘട്ടത്തില്‍ ഉത്പാദിപ്പിക്കപ്പെടുന്ന LH അതിന്റെ പ്രവര്‍ത്തനമാരംഭിക്കുകയും ഫോളിക്കുളാര്‍ ദ്രവത്തെ മഞ്ഞവല്‍ക്കരിക്കുകയും ചെയ്യും. ഫോളിക്കിളിലെ സ്റ്റിഗ്മ പൊട്ടി അണ്ഡത്തോടെ പുറത്തേക്ക് തെറിക്കുന്ന ഫോളിക്കുളാര്‍ ദ്രാവകത്തിന്റെ നിറം മഞ്ഞയായിരിക്കും. പുരുഷ ശുക്ലവുമായി താരതമ്യം ചെയ്യുമ്പോള്‍ കട്ടിയില്ലാത്തതും മഞ്ഞ നിറത്തിലുള്ളതുമായ ദ്രാവകമാണ് ഫോളിക്കിള്‍ പൊട്ടി പുറത്തേക്കൊഴുകുന്ന കുഞ്ഞിന്റെ നിര്‍മാണത്തിന് നിമിത്തമാകുന്ന സ്ത്രീസ്രവം എന്നര്‍ത്ഥം.

കുഞ്ഞിന്റെ സൃഷ്ടിക്ക് നിമിത്തമാകുന്ന സ്ത്രീസ്രവം മഞ്ഞനിറത്തിലുള്ളതും കട്ടി കുറഞ്ഞതുമാണെന്ന പ്രവാചകവചനം എത്രമാത്രം കൃത്യമാണെന്ന് നമുക്ക് ബോധ്യപ്പെടുന്നത് ഫോളിക്കിള്‍ രൂപാന്തീകരണത്തെക്കുറിച്ച (folliculogenesis) പുതിയ പഠനങ്ങളുടെ വെളിച്ചത്തിലാണ്. കോര്‍പ്പസ് ലൂടിയത്തെയും ലൂറ്റിനൈസിംഗ് ഹോര്‍മോണിന്റെ ധര്‍മത്തെയുമെല്ലാം കുറിച്ച് കൃത്യമായി മനസ്സിലാക്കാന്‍ കഴിഞ്ഞത് കഴിഞ്ഞ നൂറ്റാണ്ടിന്റെ അന്ത്യത്തിലും ഈ നൂറ്റാണ്ടിന്റെ തുടക്കത്തിലുമായി നടന്ന സാങ്കേതിക വിപ്ലവങ്ങളുടെ ഫലമായി ഉണ്ടായിവന്ന സൂക്ഷ്മദര്‍ശിനികളുപയോഗിച്ചുള്ള പഠനങ്ങള്‍ വഴിയാണ്. ഇപ്പോള്‍ മാത്രം നമുക്ക് മനസ്സിലായ ഇക്കാര്യം എങ്ങനെ പ്രവാചകന്‍(സ) അറിഞ്ഞുവെന്നതിന് അദ്ദേഹം തന്നെ മറുപടി പറഞ്ഞിട്ടുണ്ട്. 'അയാള്‍ എന്നോട് ചോദിച്ച കാര്യങ്ങളെക്കുറിച്ചൊന്നും അല്ലാഹു അറിയിച്ചുതരുന്നതുവരെ എനിക്ക് യാതൊരു വിവരവുമുണ്ടായിരുന്നില്ല' എന്ന പ്രവാചക പ്രസ്താവനയില്‍നിന്ന് നബിവചനങ്ങളുടെ സ്രോതസ് എന്താണെന്ന് മനസ്സിലാക്കാനാവും. തന്റെ ചോദ്യങ്ങള്‍ക്ക് കൃത്യമായി മറുപടി നല്‍കിയ നബി (സ)യോട് 'താങ്കള്‍ പറഞ്ഞത് സത്യമാണ്; താങ്കളൊരു ദൈവദൂതന്‍ തന്നെയാണ്'(സ്വഹീഹ്മുസ്‌ലിം) എന്ന് സാക്ഷ്യപ്പെടുത്തിക്കൊണ്ടാണ് ജൂതപണ്ഡിതന്‍ തിരിച്ചുപോയതെന്ന വസ്തുത ശ്രദ്ധേയമാണ്. പൂര്‍വവേദങ്ങളെക്കുറിച്ച് അറിയാവുന്നവര്‍ക്ക് മുഹമ്മദ് നബി(സ)യെപ്പറ്റി സ്വന്തം മക്കളെ അറിയുന്നതുപോലെ അറിയാന്‍ കഴിഞ്ഞിരുന്നുവെന്ന ക്വുര്‍ആന്‍ പ്രസ്താവനയുടെ സത്യത കൂടി ഇവിടെ വെളിപ്പെടുന്നുണ്ട്: ''നാം വേദം നല്‍കിയിട്ടുള്ളവര്‍ക്ക് സ്വന്തം മക്കളെ അറിയാവുന്നത് പോലെ അദ്ദേഹത്തെ (റസൂലിനെ) അറിയാവുന്നതാണ്. തീര്‍ച്ചയായും അവരില്‍ ഒരു വിഭാഗം അറിഞ്ഞുകൊണ്ടുതന്നെ സത്യം മറച്ചുവെക്കുകയാകുന്നു.'' (ക്വുര്‍ആന്‍ 2:146)

വിഷയവുമായി ബന്ധപ്പെട്ട വീഡിയോ

ല്ല. പാരായണ വ്യത്യാസത്തിനനുസരിച്ച് ചില ആശയവ്യത്യാസങ്ങളുണ്ടാകാമെങ്കിലും അവ ഖുർആനിന്റെ അഖണ്ഡതയെ ചോദ്യം ചെയ്യുന്നവയല്ല. ഏറെ പ്രചാരത്തിലുള്ള രണ്ട് ക്വിറാഅത്തുകളിലുള്ള പാരായണ വ്യത്യാസങ്ങള്‍ പരിശോധിച്ചാല്‍ അവ എത്രമാത്രം ക്വുര്‍ആനിന്റെ അഖണ്ഡതയെ ബാധിക്കാത്തതാണെന്ന് മനസ്സിലാവും.

സൂറത്തുല്‍ ബക്വറയിലെ 85-ാമത്തെ വചനത്തിന്റെ ഹഫ്‌സ് ഖിറാഅത്ത് (അല്‍ ക്വുര്‍ആനില്‍ കരീം ബി രിവായത്തി ഹഫ്‌സ്വ് അന്‍ ആസ്വിം, മുജമ്മ ഉല്‍ മലിക് ഫഹദ്, അല്‍ മദീനതുല്‍ മുനവ്വറ, 2002) പ്രകാരം 'തഅ്മലൂന്‍'’(നിങ്ങള്‍ പ്രവര്‍ത്തിക്കുന്നത്) എന്നതിനു പകരം വര്‍ശ് ഖിറാഅത്തിലുള്ളത് (അല്‍ ക്വുര്‍ആനില്‍ കരീം ബി രിവായത്തി വര്‍ഷ് അന്‍ നാഫിഅ്, ദാറുല്‍ മഅ്‌രിഫത്, ദിമശ്ഖ്, 2003) യഅ്മലൂന്‍’ (അവര്‍ പ്രവര്‍ത്തിക്കുന്നത്) എന്നാണ്. സൂറത്തുല്‍ ഹിജ്‌റിലെ 8ാം വചനത്തിന്റെ ഹഫ്‌സ് ഖിറാഅത്തില്‍ ‘മാ നുനസ്സിലു (നാം ഇറക്കുന്നതല്ല) എന്നാണെങ്കില്‍ വര്‍ശ് ഖിറാഅത്തില്‍ 'മാ തുനസ്സിലു' (നീ ഇറക്കുന്നതല്ല) എന്നാണുള്ളത്. സൂറത്തുല്‍ അമ്പിയാഇലെ നാലാമത്തെ വചനത്തിന്റെ തുടക്കം ഹഫ്‌സ് പ്രകാരം 'ഖാല'’(അദ്ദേഹം പറഞ്ഞു) എന്നാണെങ്കില്‍ വര്‍ശ് പ്രകാരം 'ഖുല്‍'’(നീ പറയുക) എന്നാണ്. സൂറത്തുല്‍ അഹ്‌സാബിന്റെ 68-ാം വചനം ഹഫ്‌സ് ക്വിറാഅത്തു പ്രകാരം അവസാനിക്കുന്നത് 'ലഅ്‌നന്‍ കബീറാ' (വമ്പിച്ച ശാപം) എന്ന പാരായണത്തോടെയാണെങ്കില്‍ വര്‍ശ് പ്രകാരം അത് 'ലഅ്‌നന്‍ കഥീറാ'’(വര്‍ധിച്ച ശാപം) എന്നാണ്. സൂറത്തുല്‍ ഫത്ഹിലെ 17ാമത്തെ വചനത്തില്‍‘'യുദ്ഖില്‍ഹു'’(അവന്‍ അവനെ പ്രവേശിപ്പിക്കും) എന്നാണ് ഹഫ്‌സ് ക്വിറാഅത്തിലുള്ളതെങ്കില്‍ അതിന്റെ വര്‍ശ് ഖിറാഅത്ത് 'നുദ്ഖില്‍ഹു' (നാം അവനെ പ്രവേശിപ്പിക്കും) എന്നാണ്.

ക്വുര്‍ആനിന്റെ സാരത്തെയോ പദവിന്യാസത്തെയോ യാതൊരു തരത്തിലും ബാധിക്കാത്ത ഇത്തരം പാരായണ വ്യത്യാസങ്ങള്‍ പോലും വളരെ പരിമിതമാണെന്ന വസ്തുത അതില്‍ കൂട്ടിച്ചേര്‍ക്കലുകളോ എടുത്തുമാറ്റലുകളോ നടന്നിട്ടില്ലെന്നാണ് വ്യക്തമാക്കുന്നതെന്ന് ഇവ്വിഷയകമായ വസ്തുനിഷ്ഠപഠനം നടത്തിയവരെല്ലാം ഉറക്കെ പറഞ്ഞിട്ടുണ്ട്. ഹഫ്‌സ്-വര്‍ശ് പാരായണഭേദങ്ങളെക്കുറിച്ച ഗവേഷണത്തിന് ഡോക്ടറേറ്റ് ലഭിച്ച ഡോക്ടര്‍ ആഡ്രയന്‍ ബ്രോക്കറ്റ് പറയുന്നത് 'ഇത്തരം പാരായണ വ്യത്യാസങ്ങളുടെ എണ്ണം പരിമിതമാണെന്ന വസ്തുത വ്യക്തമാക്കുന്നത് അതിന് ഒരേയൊരു പാഠമേയുള്ളുവെന്ന സത്യമാണ്' (Adrian Brockett: "The Value of Hafs and Warsh Transmissions for the Textual History of The Qur'an'' in Andrew RÆpin (Ed.), Opt. Cit. Page 33.) എന്നാണ്.

അദ്ദേഹം എഴുതുന്നത് കാണുക:‘'ക്വുര്‍ആന്‍ വാചികമായി മാത്രമായിരുന്നു ആദ്യനൂറ്റാണ്ടുകളില്‍ സംപ്രേഷണം ചെയ്തിരുന്നതെങ്കില്‍ ഹദീഥ് സാഹിത്യങ്ങളിലും ഇസ്‌ലാംപൂര്‍വകവിതകളിലും കാണപ്പെടുന്നതുപോലെ പാഠങ്ങള്‍ (text) തമ്മില്‍ കാര്യമാത്രപ്രസക്തമായ വ്യത്യാസങ്ങള്‍ അതില്‍ കാണപ്പെടുമായിരുന്നു. എഴുത്തുരൂപത്തില്‍ മാത്രമാണ് അത് സംപ്രേഷണം ചെയ്യപ്പെട്ടിരുന്നതെങ്കില്‍ മദീനാഭരണഘടനയുടെ ഒറിജിനല്‍ രേഖകളിലുള്ളതുപോലെ പരിഗണനക്കര്‍ഹമായ വ്യത്യാസങ്ങള്‍ രേഖകളിലും ഉണ്ടാകുമായിരുന്നു. എന്നാല്‍ ക്വുര്‍ആനിന്റെ കാര്യം ഇതു രണ്ടുമല്ല. ഒരേ സമയം തന്നെ വാചികമായ സംപ്രേഷണവും സമാന്തരമായി രേഖകളിലൂടെയുള്ള സംപ്രേഷണവും നിലനിന്നതിനാല്‍ അവ പരസ്പരം സംരക്ഷിക്കുകയും എല്ലാ തരത്തിലുമുള്ള കൈകടത്തലുകളില്‍ നിന്നും ക്വുര്‍ആനിനെ മുക്തമാക്കുകയും ചെയ്തു'.  (Ibid, Page 44.)

‘''മുഹമ്മദിനു ശേഷമുള്ള ക്വുര്‍ആനിന്റെ സംപ്രേഷണം മാറ്റങ്ങളൊന്നുമില്ലാത്ത രീതിയില്‍ തികച്ചും അദ്ദേഹം പറഞ്ഞുകൊടുത്ത പോലെത്തന്നെയായിരുന്നു. ഒരേയൊരു പാഠം മാത്രമേ അതിനുണ്ടായിരുന്നുള്ളൂ. ദുര്‍ബലപ്പെടുത്തപ്പെട്ടതായി ആരോപിക്കപ്പെടുന്ന വചനങ്ങളടക്കം യാതൊന്നും തന്നെ അതില്‍ നിന്ന് എടുത്തു മാറ്റപ്പെട്ടിട്ടില്ല; ഒന്നും കൂട്ടിച്ചേര്‍ക്കപ്പെട്ടിട്ടുമില്ല''.  (Ibid, Page 44.)

അതെ! അവതരിപ്പിക്കപ്പെട്ട രൂപത്തില്‍ മാറ്റങ്ങളൊന്നുമില്ലാതെ നിലനില്‍ക്കുന്ന ഗ്രന്ഥമാണ് ക്വുര്‍ആന്‍. വ്യത്യസ്ത ഖിറാഅത്തുകളിലുള്ള സൂക്ഷമമായ വ്യത്യാസങ്ങള്‍ക്കുപോലും ദൈവികബോധനത്തിന്റെ പിന്‍ബലമുണ്ട്. കഴിഞ്ഞ പതിനാലുനൂറ്റാണ്ടുകളായി മാറ്റമൊന്നുമില്ലാതെ നിലനില്‍ക്കുന്ന ഒരേയൊരു ഗ്രന്ഥമാണ് ക്വുര്‍ആന്‍. മാറ്റമൊന്നുമില്ലാതെ അതിനെ സംരക്ഷിക്കുമെന്ന ദൈവിക വാഗ്ദാനം പാലിക്കപ്പെട്ടുകൊണ്ടിരിക്കുന്നുവെന്നതിനുള്ള തെളിവാണ് നടേ സമര്‍ഥിച്ച വസ്തുതകള്‍. അല്ലാഹുവിന്റെ വാഗ്ദാനം എത്ര സത്യമാണ്!

''തീര്‍ച്ചയായും നാമാണ് ആ ഉല്‍ബോധനം അവതരിപ്പിച്ചത്. തീര്‍ച്ചയായും നാം അതിനെ കാത്തുസൂക്ഷിക്കുന്നതുമാണ്.''

 

ഖുര്‍ആനില്‍ ഉപയോഗിക്കപ്പെട്ടിരിക്കുന്ന ഭാഷാശൈലിയും അതിലെ വിവരണരീതിയുമെല്ലാം മാനുഷിക രചനകളില്‍നിന്നു തികച്ചും വ്യത്യസ്തമാണ്. ഏതാനും സവിശേഷതകള്‍ താഴെ:

  1. ഖുര്‍ആനിലെ വചനങ്ങളെല്ലാം വിവരിക്കപ്പെട്ട വിഷയങ്ങളുമായി താരതമ്യം ചെയ്യുമ്പോള്‍ ഹ്രസ്വവും അമിതവികാര പ്രകടനം ഉള്‍ ക്കൊള്ളാത്തവയുമാണ്.

മാനുഷിക വചനങ്ങള്‍ എപ്പോഴും വ്യക്തിയുടെ മാനസികാവസ്ഥയെ ആശ്രയിച്ചാണ് പ്രകടമാക്കപ്പെടുന്നത്. കോപത്തിലിരിക്കുന്ന ഒരാളു ടെ വാക്കുകളില്‍ കോപം പ്രകടമായിരിക്കും. അന്നേരം ദയയും പ്രശംസയും ആ വാക്കുകളിലുണ്ടാവുകയില്ല. സന്തോഷാവസ്ഥയിലും സ്ഥിതി തഥൈവ!

കോപത്തിന്റെയും സന്തോഷത്തിന്റെയും തീവ്രമായ അവസ്ഥകളില്‍ പ്രകടിപ്പിക്കപ്പെടുന്ന പദങ്ങളെ പ്രസ്തുത വികാരം നിലനില്‍ക്കുന്ന അവസ്ഥകളുടെ അടിസ്ഥാനത്തില്‍ മാത്രമേ വ്യാഖ്യാനിക്കുവാനാകൂ. പ്രസ്തുത പദങ്ങളില്‍ വികാരങ്ങളുടെ അമിതപ്രകടനം കാണാനാ വും. ഏതുസാഹിത്യകാരന്മാരുടെയും കൃതികളില്‍ ഈ അമിതവികാരപ്രകടനം കാണാം. കാരണം അവര്‍ വികാരങ്ങളുള്ള മനുഷ്യരാണെ ന്നതുതന്നെ!

ഖുര്‍ആനിലെ വചനങ്ങള്‍ സന്തോഷവാര്‍ത്ത അറിയിക്കുന്നതാകട്ടെ, മുന്നറിയിപ്പ് നല്‍കുന്നതാകട്ടെ, നിയമങ്ങള്‍ വിശദീകരിക്കുന്നതാകട്ടെ, ദൈവാനുഗ്രഹങ്ങളെക്കുറിച്ച് സംസാരിക്കുന്നതാകട്ടെ, എവിടെയും അമിതമായ വികാര പ്രകടനങ്ങള്‍ കാണുക സാധ്യമല്ല. പരമപരിശുദ്ധനായ പടച്ചതമ്പുരാനില്‍നിന്ന് അവതരിപ്പിക്കപ്പെട്ടതുകൊണ്ടാണിത്.

  1. ഖുര്‍ആന്‍ ഏതു വിഷയത്തെക്കുറിച്ച് വിശദീകരിക്കുമ്പോഴും അതിന്റെ വാഗ്മിതയും രചനാസൗഷ്ടവവും നിലനിര്‍ത്തുന്നു.

വ്യക്തികളുടെ രചനാ സൗഷ്ടവം ചില പ്രത്യേക വിഷയങ്ങളോട് ബന്ധപ്പെട്ടായിരിക്കും പ്രകടമാക്കപ്പെടുക. പ്രസ്തുത വിഷയങ്ങളില്‍ അവരുടെ രചനകള്‍ ഉന്നത നിലവാരം ഉള്‍ക്കൊള്ളുന്നതാകാം. എന്നാല്‍, അവര്‍തന്നെ മറ്റു വിഷയങ്ങളില്‍ രചന നടത്തിയാല്‍ അവ പല പ്പോഴും ശരാശരി നിലവാരം പോലും പുലര്‍ത്തുകയില്ല. രചയിതാവിന്റെ മാനസിക ഘടന, കുടുംബാന്തരീക്ഷം, വികാരവിചാര ങ്ങള്‍, സമൂഹത്തിന്റെ അവസ്ഥ എന്നിവയെല്ലാം അയാളുടെ താല്‍പര്യത്തെ സ്വാധീനിക്കും.

ഖുര്‍ആനിലെ വചനങ്ങള്‍ പ്രകൃതിയെക്കുറിച്ച് വിവരിക്കുമ്പോഴും പരലോകത്തെക്കുറിച്ച് സംസാരിക്കുമ്പോഴും ഒരേ വാഗ്മിത പ്രകടിപ്പി ക്കുന്നു. ദൈവ മഹത്വത്തെ പ്രകീര്‍ത്തിക്കുമ്പോഴും നിയമനിര്‍ദേശങ്ങള്‍ അവതരിപ്പിക്കുമ്പോഴും ഒരേ രചനാസൗഷ്ടവമാണ് അവയ്ക്കു ള്ളത്. സ്ഥലകാലങ്ങള്‍ക്ക് അതീതനായ സ്രഷ്ടാവില്‍നിന്ന് അവതരിപ്പിക്കപ്പെട്ടതുകൊണ്ടാണിത്.

  1. ഖുര്‍ആന്‍ വചനങ്ങള്‍ ഉയര്‍ന്ന സാഹിത്യനിലവാരം പുലര്‍ത്തുന്നതോടൊപ്പം സൂക്ഷ്മതയും സത്യസന്ധതയും പുലര്‍ത്തുന്നവയുമാണ്.

സാഹിത്യം സുന്ദരമാകുന്നത്, ഇല്ലാത്തത് വിവരിക്കുമ്പോഴാണല്ലോ. അര്‍ധസത്യങ്ങളുടെയും അസത്യങ്ങളുടെയും മേമ്പൊടിയില്ലാതെ സാഹിത്യത്തെ സൗന്ദര്യവത്കരിക്കാന്‍ കഴിയില്ലെന്ന് പറയാറുണ്ട്. കവിത നന്നാകണമെങ്കില്‍ കളവ് പറയണമെന്നാണല്ലോ ആപ്തവാ ക്യം. സത്യസന്ധമായ വിവരങ്ങള്‍ മാത്രം നല്‍കുന്ന സാഹിത്യകൃതികള്‍ വിരസവും വരണ്ടതുമായിരിക്കും. അതുകൊണ്ടുതന്നെ സത്യം പറയണമെന്നാഗ്രഹിക്കുന്ന സാഹിത്യകാരന്മാര്‍ക്കുപോലും അസത്യത്തിന്റെ മേമ്പൊടിയോടുകൂടി മാത്രമേ പ്രസ്തുത സത്യം അവതരിപ്പി ക്കുവാനാകൂ. പൊടിപ്പും തൊങ്ങലുമില്ലാതെ മനുഷ്യമനസ്സിന്റെ വൈകാരികതലങ്ങളെ സംതൃപ്തമാക്കാന്‍ കഴിയുകയില്ലെന്ന ധാരണയാ ണ് ഇതിനു കാരണം.

ഖുര്‍ആന്‍ വചനങ്ങള്‍ ഈ പൊതുധാരയില്‍നിന്ന് തികച്ചും വ്യത്യസ്തമാണ്. വസ്തുതകള്‍ മാത്രമാണ് അതിലെ പ്രതിപാദ്യം. പക്ഷേ, ഉന്നത മായ സാഹിത്യനിലവാരം നിലനിര്‍ത്തുവാനും മനുഷ്യമനസ്സുകളെ സംതൃപ്തമാക്കുവാനും അവയ്ക്ക് സാധിക്കുന്നു. മനസ്സിനെക്കുറിച്ച് ശരിയ്ക്കറിയാവുന്ന സര്‍വജ്ഞനില്‍നിന്ന് അവതരിപ്പിക്കപ്പെട്ടതുകൊണ്ടാണിത്.

  1. ഖുര്‍ആന്‍ തുടക്കം മുതല്‍ ഒടുക്കം വരെ ഉന്നതമായ സാഹിത്യ നിലവാരം പുലര്‍ത്തുന്നു.

ഒരു കവിത മനോഹരമാണെന്ന് നാം വിധിയെഴുതുന്നത് അതിലെ ഏതാനും വരികളുടെ സൗന്ദര്യത്തിന്റെ അടിസ്ഥാനത്തിലായിരിക്കും. പ്രസ്തുത കവിതയിലെതന്നെ എല്ലാ വരികളും അതേനിലവാരം പുലര്‍ത്തിക്കൊള്ളണമെന്നില്ല. ഒരു സാഹിത്യകാരനെ ഉന്നത നിലവാരമു ള്ളവനെന്ന് വിളിക്കുന്നത് അയാളുടെ ഏതാനും ചില കൃതികളുടെ അടിസ്ഥാനത്തിലുമായിരിക്കും. അയാളുടെ തന്നെ മറ്റു രചനകള്‍ പ്രസ്തുത നിലവാരം പുലര്‍ത്തിക്കൊള്ളണമെന്നില്ല. ഓരോരുത്തര്‍ക്കും ഉന്നതമായരചനകള്‍ നിര്‍വഹിക്കപ്പെടുന്ന ചില പ്രത്യേക പ്രായ വും സന്ദര്‍ഭവുമെല്ലാം ഉണ്ടായിരിക്കും. പ്രായം, ചുറ്റുപാട്, അന്തരീക്ഷം തുടങ്ങിയവ രചയിതാവിനെ സ്വാധീനിക്കുന്നതുകൊണ്ടാണിത്.

ഖുര്‍ആന്‍ വചനങ്ങള്‍ മുഴുവനും ഉന്നതമായ സാഹിത്യനിലവാരം പുലര്‍ത്തുന്നവയാണ്. ആറായിരത്തിലധികം സൂക്തങ്ങളില്‍ ഒന്നുപോ ലും നിലവാരം കുറഞ്ഞതാണെന്ന് പറയാന്‍ ആര്‍ക്കും സാധ്യമല്ല. നീണ്ട ഇരുപത്തിമൂന്ന് വര്‍ഷക്കാലത്തെ പ്രവാചകദൗത്യത്തിനിടയില്‍ വ്യത്യസ്ത സന്ദര്‍ഭങ്ങളിലായിരുന്നു ഖുര്‍ആന്‍ അവതരിപ്പിക്കപ്പെട്ടുകൊണ്ടിരുന്നത്. അത് പ്രവാചക രചനയായിരുന്നുവെങ്കില്‍ അവതര ണസന്ദര്‍ഭങ്ങളിലെ പ്രവാചകന്റെ മാനസികാവസ്ഥകള്‍ക്ക് അനുസൃതമായി അവയുടെ നിലവാരത്തില്‍ മാറ്റമുണ്ടാകേണ്ടതായിരുന്നു. എന്നാല്‍, ഖുര്‍ആനിലെ ഓരോ സൂക്തവും മറ്റുള്ളവയോട് കിടപിടിക്കുന്നവയാണ്. സര്‍വശക്തനായ തമ്പുരാനില്‍നിന്നായതുകൊണ്ടാ ണിത്.

  1. ഒരേ സംഗതിതന്നെ ഒന്നിലധികം തവണ വിവരിക്കുമ്പോഴും ഖുര്‍ ആന്‍ ഉന്നതമായ സാഹിത്യനിലവാരം പുലര്‍ത്തുന്നു.

ഒരേ കാര്യംതന്നെ ഒന്നിലധികം തവണ വിവരിക്കുമ്പോള്‍ സാധാരണ സാഹിത്യകൃതികളില്‍ ആദ്യത്തെ വിവരണം പോലെ മനോഹരമാ വുകയില്ല രണ്ടാമത്തെ വിവരണം. ആവര്‍ത്തന വിരസത രചയിതാവിന്റെ വചനങ്ങളിലും ആസ്വാദകന്റെ മനസ്സിലും രൂപപ്പെടുന്നതു കാണാം. മനുഷ്യന്‍, അവന്‍ എത്ര ഉന്നതനായ സാഹിത്യകാരനാണെങ്കിലും അടിസ്ഥാനപരമായ പരിമിതികള്‍ ഉള്‍ക്കൊള്ളുന്നവനായതു കൊണ്ടാണിത്.

ഖുര്‍ആനാകട്ടെ പല വിഷയങ്ങളും പല തവണ ആവര്‍ത്തിക്കുന്നുണ്ട്. സൃഷ്ടി, മരണം, മരണാനന്തര ജീവിതം, ദൈവ മഹത്വത്തെക്കുറിച്ച വിവരണങ്ങള്‍, അവനെ മാത്രം ആരാധിക്കേണ്ടതിന്റെ ആവശ്യകത തുടങ്ങിയ വിഷയങ്ങള്‍ ഖുര്‍ആനില്‍ ആവര്‍ത്തിച്ചു പ്രതിപാദിക്കു ന്നുണ്ട്. എന്നാല്‍, ഓരോ തവണ വിവരിക്കുമ്പോഴും ശ്രോതാവിന് അത് പുതുമയുള്ളതായി അനുഭവപ്പെടുകയും അവന്റെ മനസ്സില്‍ മാറ്റത്തിന്റെ ആന്ദോളനങ്ങള്‍ സൃഷ്ടിക്കുകയും ചെയ്യുന്നു. പരിമിതികള്‍ക്ക് അതീതനായ പരമോന്നത നില്‍നിന്ന് അവതരിപ്പിക്കപ്പെ ട്ടതുകൊണ്ടാണിത്.

  1. സാഹിത്യകൃതികള്‍ക്ക് വഴങ്ങാത്ത വിഷയങ്ങളാണ് ഖുര്‍ആനില്‍ പ്രതിപാദിക്കപ്പെടുന്നതെങ്കിലും പ്രസ്തുത വിവരണങ്ങളിലെല്ലാം അത് ഉന്നതമായ നിലവാരം പുലര്‍ത്തുകയും മനോഹാരിത കാത്തുസൂക്ഷിക്കുകയും ചെയ്യുന്നു.

മരണാനന്തര ജീവിതം, ദൈവാസ്തിത്വം, അനുഷ്ഠാനമുറകള്‍, നിയമനിര്‍ദേശങ്ങള്‍, വിധിവിലക്കുകള്‍, നന്മചെയ്യുവാനുള്ള പ്രേരണ, സത്യസന്ധമായ ചരിത്രം തുടങ്ങിയവയെല്ലാം സാഹിത്യകാരന്റെ ദൃഷ്ടിയില്‍ വരണ്ട വിഷയങ്ങളാണ്. അതുകൊണ്ടുതന്നെ പ്രസ്തുത വിഷയങ്ങളില്‍ രചന നിര്‍വഹിച്ചാല്‍ സാഹിത്യം സുന്ദരമാവുകയില്ലെന്നാണ് പൊതുവേയുള്ള ധാരണ. അവ ഭാവനയ്ക്ക് വഴങ്ങുന്ന വിഷയങ്ങളല്ല. അതിനാല്‍ ഇത്തരം വിഷയങ്ങളെടുത്തുകൊണ്ട് നിര്‍വഹിക്കപ്പെട്ട രചനകളില്‍ ഒന്നുംതന്നെ ലോകോത്തര കൃതികളായി അറിയപ്പെടുന്നില്ല. മനുഷ്യന്റെ പരിമിതിയാണ് ഇവിടെയും പ്രകടമാവുന്നത്.

ഖുര്‍ആനിലെ പ്രതിപാദ്യങ്ങളാകട്ടെ, മിക്കവാറും ഇത്തരം വിഷയങ്ങളാണ് ഉള്‍ക്കൊള്ളുന്നത്. എന്നാല്‍, അവയെല്ലാം ഉന്നതമായ സാഹി ത്യനിലവാരം പുലര്‍ത്തുകയും ആസ്വാദകന്റെ മനസ്സിനെ സംതൃപ്തമാക്കുകയും ചെയ്യുന്നു. പദാര്‍ഥാതീതനായ പടച്ചതമ്പുരാനില്‍നിന്ന് അവതരിപ്പിക്കപ്പെട്ടതുകൊണ്ടാണിത്!

  1. ഒരു വിഷയത്തില്‍നിന്ന് മറ്റൊന്നിലേക്ക് മാറുമ്പോഴും സാഹിത്യഭംഗി ചോര്‍ന്നുപോകാതെ സൂക്ഷിക്കുവാന്‍ ഖുര്‍ആനിന് കഴിയുന്നു.

ഒരൊറ്റ സാഹിത്യകൃതിയില്‍തന്നെ ഒരു വിഷയത്തില്‍നിന്ന് മറ്റൊന്നിലേക്ക് മാറുമ്പോള്‍ അതുവരെ പുലര്‍ത്തിപ്പോന്ന നിലവാരം പുലര്‍ ത്താന്‍ പലപ്പോഴും കഴിയാറില്ല. ഒരു വിഷയത്തെക്കുറിച്ച് വിവരിക്കുന്ന സാഹിത്യകാരന്റെ മനസ്സില്‍ രൂപപ്പെടുന്ന ബിംബങ്ങളുടെ ചാരുത അടുത്ത വിഷയ ത്തെക്കുറിച്ച് സംസാരിക്കാനാരംഭിക്കുമ്പോള്‍ മങ്ങുകയും പുതിയ ബിംബങ്ങള്‍ പ്രശോഭിക്കുവാന്‍ സമയമെടു ക്കുകയും ചെയ്യുന്നതുകൊണ്ടാണിത്. വിദഗ്ധമായി ജോലി നിര്‍വഹിച്ചുകൊണ്ടിരിക്കുന്ന ഒരാളെ പെട്ടെന്ന് മറ്റൊരു ജോലിയില്‍ ഏല്‍പി ക്കുന്നതുപോലെയാണിത്. ഇതും മനുഷ്യന്റെ പൊതുവായ പരിമിതിയാണ്.

ഖുര്‍ആനിലുടനീളം വിഷയങ്ങളില്‍നിന്ന് വിഷയങ്ങളിലേക്കുള്ള ചാട്ടം കാണാം. എന്നാല്‍ ഈ ചാട്ടങ്ങളിലൊന്നുംതന്നെ അതിന്റെ ചാരുത ക്ക് ഭംഗം വരുകയോ മനോഹാരിതക്ക് ഹാനി സംഭവിക്കുകയോ ചെയ്യുന്നില്ല. സര്‍വശക്തനില്‍നിന്നായതുകൊണ്ടാണിത്.

  1. ഏതാനും പദങ്ങള്‍ മാത്രമുപയോഗിച്ച്, മനോഹാരിതയും സ്ഫുടതയും നഷ്ടപ്പെടാത്ത രൂപത്തില്‍, അര്‍ഥഗംഭീരമായ ആശയം പ്രകടിപ്പി ക്കുന്ന ഗ്രന്ഥമാണ് ഖുര്‍ആന്‍.

സാധാരണ സാഹിത്യകൃതികളില്‍ പദങ്ങളുടെ സമുദ്രമാണുണ്ടാവുക; പ്രസ്തുത സമുദ്രത്തില്‍ ആശയങ്ങളുടെ മുത്തുകള്‍ തുലോം പരിമി തവും. പ്രൗഢമായ ആശയങ്ങള്‍ പ്രകടിപ്പിക്കുന്നതിനുവേണ്ടി രചിക്കപ്പെട്ട കൃതികളിലാകട്ടെ പദങ്ങളുടെ വേലിയേറ്റം തന്നെ കാണാനാ വും. താന്‍ ഉദ്ദേശിക്കുന്ന ആശയങ്ങള്‍ ആസ്വാദകനിലെത്തുവാന്‍ എന്തൊക്കെ രീതിയിലാണ് പദപ്രയോഗം നടത്തേണ്ടതെന്നതിനെക്കുറിച്ച് ഓരോ രചയിതാവിനും അയാളുടേതായ വീക്ഷണമുണ്ടായിരിക്കും. പ്രസ്തുത വീക്ഷണം അയാളുടേതായതുകൊണ്ടുതന്നെ ആസ്വാദകന് അയാളുടെ പദപ്രയോഗങ്ങളില്‍ പലതും അനാവശ്യമായാണ് അനുഭവപ്പെടുക. ഒരു ആസ്വാദകന് അനാവശ്യമെന്നു തോന്നുന്ന പദങ്ങള്‍ മറ്റൊരാളുടെ വീക്ഷണത്തില്‍ അനിവാര്യമാകാം. അതുകൊണ്ടുതന്നെ എല്ലാവരെയും സംതൃപ്തരാക്കുന്നതിനുവേണ്ടി പദങ്ങള്‍ ഒരുപാട് പ്രയോഗിക്കുവാന്‍ അയാള്‍ നിര്‍ബന്ധിതനായിരിക്കും. അന്യരുടെ മനസ്സുകള്‍ വായിക്കുവാനുള്ള മനുഷ്യരുടെ കഴിവില്ലായ്മയാണ് ഇതിന് കാരണം.

ഖുര്‍ആനിലാകട്ടെ, അനിവാര്യമായ പദങ്ങള്‍ മാത്രമേ ഉപയോഗിച്ചിട്ടു ള്ളൂ. പാരായണം ചെയ്യുന്നവന് അത് ഉദ്ദേശിക്കുന്ന ആശയം പകര്‍ ന്നുനല്‍കുവാന്‍ ഈ പദങ്ങള്‍ കൊണ്ടുതന്നെ സാധിക്കുന്നു. പ്രൗഢമായആശയങ്ങള്‍ അനിവാര്യമായ പദങ്ങള്‍ മാത്രമുപയോഗിച്ച് പ്രകടി പ്പിക്കുകയും അത് മനോഹരമായി അവതരിപ്പിച്ച് എല്ലാത്തരം വായനക്കാരെയും സംതൃപ്തരാക്കുകയും ചെയ്യുന്ന ഗ്രന്ഥമാണ് ഖുര്‍ആന്‍. മനുഷ്യമനസ്സിന്റെ സൂക്ഷ്മ തലങ്ങളെക്കുറിച്ച് വ്യക്തമായി അറിയാവുന്നവനില്‍നിന്ന് അവതരിപ്പിക്കപ്പെട്ടതുകൊണ്ടാണിത്.

  1. സാഹിത്യത്തിന്റെ ഏതു മാനത്തിലൂടെ നോക്കിയാലും ഖുര്‍ആന്‍ ഒരു ഉന്നതമായ സാഹിത്യ കൃതിയാണ്.

സാഹിത്യ കൃതികളെല്ലാം  മനുഷ്യരുടെ ഏതെങ്കിലുമൊരു വികാരത്തെ ഉത്തേജിപ്പിക്കുവാന്‍ വേണ്ടിയുള്ളതായിരിക്കും. ദുഃഖം, സന്തോ ഷം, ദയ, കാരുണ്യം, വെറുപ്പ്, പ്രതിഷേധം എന്നിങ്ങനെ. അതുപോലെതന്നെ പ്രഭാവം, മാധുര്യം, സൗന്ദര്യം, ചാരുത തുടങ്ങിയവയെല്ലാം ഒരേ സാഹിത്യകൃതിയില്‍തന്നെ കണ്ടെത്തുക പ്രയാസമാണ്. സാഹിത്യത്തിന്റെ ഏതെങ്കിലും പ്രത്യേകമായ മാനങ്ങളിലൂടെ നോക്കിയാല്‍ മാത്രമേ സാഹിത്യകൃതികളെ ആസ്വദിക്കുവാനും വിലയിരുത്തുവാനും കഴിയൂ. എല്ലാ അംശങ്ങളെയും ഒരേപോലെ ഉള്‍ക്കൊണ്ടു കൊണ്ട് ഒരു രചന നടത്തുക സാധ്യമല്ല. ഇതും മനുഷ്യന്റെ പരിമിതിയാണ്.

ഖുര്‍ആനാകട്ടെ മനുഷ്യവികാരത്തിന്റെ എല്ലാ തലങ്ങളെയും സ്പര്‍ശിക്കുന്നു. മനുഷ്യനെ സന്തോഷിപ്പിക്കുവാനും ദുഃഖിപ്പിക്കുവാനും ദയയും കാരുണ്യവും പ്രകടിപ്പിക്കുന്നവനാക്കിത്തീര്‍ക്കുവാനും വെറുപ്പും പ്രതിഷേധവും ഉത്തേജിപ്പിക്കുവാനുമെല്ലാം കഴിയുന്ന വരികളാണ് അതിലുള്ളത്. അതോടൊപ്പംതന്നെ അത് മനുഷ്യബുദ്ധിയെ പ്രവര്‍ത്തനക്ഷമമാക്കുകയും ചെയ്യുന്നു. പ്രഭാവം, മാധുര്യം, സൗന്ദര്യം, ചാരുത തുടങ്ങിയ ആസ്വാദക പ്രധാനമായ സാഹിത്യത്തിന്റെ സവിശേഷതകള്‍ ഖുര്‍ആനിക വചനങ്ങളില്‍ സമഞ്ജസമായി സമ്മേളിക്കുകയും ചെയ്തിരിക്കുന്നു. സാഹിത്യത്തിന്റെ ഏതു മാനത്തിലൂടെ നോക്കിയാലും അത് ഉന്നതമായ നിലവാരം പുലര്‍ത്തുന്നതാണെന്ന് കാണാം.

  1. ഖുര്‍ആനില്‍ മറ്റാരുടെയെങ്കിലും ശൈലിയോ പ്രയോഗങ്ങളോ രീതിയോ ആശയങ്ങളോ കടമെടുക്കപ്പെട്ടിട്ടില്ല.

സാഹിത്യകൃതികള്‍ എത്രതന്നെ മൗലികങ്ങളാണെങ്കിലും മറ്റു സാഹി ത്യകാരന്മാരുടെ ശൈലികളും പ്രയോഗങ്ങളുമെല്ലാം അതില്‍ സ്വാധീ നം ചെലുത്തിയിട്ടുണ്ടാകും. അത് സ്വാഭാവികമാണ്. മുന്‍ഗാമികളുടെ രചനകളുടെ സ്വാധീനമുള്‍ക്കൊള്ളാതെ ഒരാള്‍ക്കും സാഹിത്യകൃ തികള്‍ രചിക്കുക സാധ്യമല്ല. നേരിട്ടുള്ള കോപ്പിയടിയല്ല ഇവിടെ വിവക്ഷിക്കപ്പെട്ടിരിക്കുന്നത്. ശൈലികളുടെയും ബിംബങ്ങളുടെയും സ്വാധീനമാണ്. അതില്ലാതെ രചന നടത്തുവാന്‍ കഴിയില്ല. ഇത് മനുഷ്യമനസ്സിന്റെ പരിമിതിയാണ്. മുന്‍ഗാമിയില്‍നിന്ന് പഠിക്കുകയും അത് വികസിപ്പിച്ചെടുക്കുകയും ചെയ്യുന്നവനാണല്ലോ മനുഷ്യന്‍.

ഖുര്‍ആന്‍ ഇത്തരം കടമെടുക്കലുകളില്‍നിന്ന് തികച്ചും മുക്തമാണ്. അറബ് സാഹിത്യ രംഗത്തുണ്ടായിരുന്ന ആരുടെയും ശൈലിയോ രൂപ മോ രീതിയോ ആശയങ്ങളോ ഖുര്‍ആന്‍ കടമെടുത്തിട്ടില്ല. ആരുടെ കൃതിയുടെയും യാതൊരു സ്വാധീനവും ഖുര്‍ആനില്‍ ഇല്ല താനും. എല്ലാ നിലയ്ക്കും ഒരു മൗലിക കൃതിയാണ് ഖുര്‍ആന്‍. പരിധികളോ പരിമിതിയോ ഇല്ലാത്ത അറിവിന്റെ ഉടമസ്ഥനില്‍നിന്ന് അവതരിപ്പിക്കപ്പെട്ടതിനാലാണ് ഇത്.

ര്‍വശക്തനായ സ്രഷ്ടാവിനാല്‍ നിയുക്തരാവുന്ന പ്രവാചകന്മാര്‍ക്ക് തങ്ങളുടെ പ്രവാചകത്വത്തിന്റെ സത്യത ജനങ്ങളെ ബോധ്യപ്പെടുത്തു ന്ന തിനായി ചില ദൃഷ്ടാന്തങ്ങള്‍ ദൈവം നല്‍കിയിരുന്നതായി വേദഗ്രന്ഥങ്ങ ളില്‍നിന്ന് മനസ്സിലാക്കാന്‍ കഴിയും. അവര്‍ ജീവിച്ചിരുന്ന സമൂ ഹത്തിലെ ജനങ്ങള്‍ക്ക് അവരുടെ പ്രവാചകത്വത്തെക്കുറിച്ച അവകാശവാദം ശരിതന്നെയാണെന്ന് ബോധ്യപ്പെടുത്തുകയായിരുന്നു പ്രസ്തു ത ദൃഷ്ടാന്തങ്ങ ളുടെ ഉദ്ദേശ്യം. മൂസാ നബി(അ)ക്ക് നല്‍കപ്പെട്ട സര്‍പ്പമായി മാറുന്ന വടി ഒരുദാഹരണം. ഇതുപോലുള്ള അത്ഭുതങ്ങള്‍ മുഹ മ്മദ് നബി(സ)യിലൂടെയും വെളിപ്പെട്ടിട്ടുണ്ട്. ചന്ദ്രനെ പിളര്‍ത്തിയത് ഒരു ഉദാഹരണം മാത്രം.

ഇത്തരം അത്ഭുതങ്ങള്‍ പ്രവാചകന്മാരുടെ ജീവിതകാലത്ത് മാത്രം നില നിന്നിരുന്നവയാണ്. അവര്‍ക്കുശേഷം ആ അത്ഭുതങ്ങളൊന്നും നില നിന്നിട്ടില്ല; നിലനില്‍ക്കുകയുമില്ല. അന്തിമ പ്രവാചകനിലൂടെ വെളിപ്പെട്ട ഏറ്റവും പ്രധാനപ്പെട്ട അത്ഭുതത്തിന്റെ സ്ഥിതിയിതല്ല. അത് അദ്ദേ ഹത്തിന്റെ ദൗത്യം പോലെതന്നെ അവസാനനാള്‍ വരെ നിലനില്‍ക്കുന്നതാണ്. ഖുര്‍ആനാണ് പ്രസ്തുത അമാനുഷിക ദൃഷ്ടാന്തം. അവസാ നനാള്‍ വരെ ആര്‍ക്കും ഖുര്‍ആന്‍ പരിശോധിക്കാം. അതിലെ അത്ഭുതങ്ങള്‍ ആസ്വദിക്കാം. അങ്ങനെ മുഹമ്മദ് നബി(സ)യുടെ പ്രവാചക ത്വം സത്യമാണോയെന്ന് തീര്‍ ച്ചപ്പെടുത്താം. ഒരേസമയം, വേദഗ്രന്ഥവും ദൈവിക ദൃഷ്ടാന്തവുമായ ഖുര്‍ആന്‍ അവസാനനാള്‍ വരെ നില നില്‍ക്കുന്ന അത്ഭുതങ്ങളുടെ അത്ഭുതമാണ്.

ഖുര്‍ആനിനെ അമാനുഷിക ദൃഷ്ടാന്തമാക്കുന്നത് എന്താണ്?

ഖുര്‍ആനിലെ ആശയങ്ങളും ശൈലിയും ഭാഷയുമെല്ലാം അത്ഭുതംതന്നെയാണ്. അറബി സാഹിത്യത്തിലെ അതികായന്മാര്‍ക്കിടയിലേ ക്കാണ് ഖുര്‍ആനിന്റെ അവതരണം. പതിനാലു നൂറ്റാണ്ടു മുമ്പത്തെ കവിതകള്‍ അറബ് സാഹിത്യത്തിലെ മാസ്റ്റര്‍പീസുകളാണിന്നും. അവര്‍ക്കിടയില്‍ ജനിച്ചുവളര്‍ന്ന ഒരു നിരക്ഷരനിലൂടെയാണ് ഖുര്‍ആന്‍ ലോകം ശ്രവിക്കുന്നത്. അദ്ദേഹമാകട്ടെ നാല്‍പതു വയസ്സുവരെ യാതൊരുവിധ സാഹിത്യാഭിരുചിയും കാണിക്കാത്ത വ്യക്തിയും. ഖുര്‍ആനിന്റെ സാഹിത്യമേന്മയെ സംബന്ധിച്ച് അത് അവതരിപ്പി ക്കപ്പെട്ട സമൂഹത്തില്‍ അഭിപ്രായവ്യത്യാസമൊന്നുമുണ്ടായിരുന്നില്ല. വിശ്വാസികളും അവിശ്വാസികളുമായ അറബികളെല്ലാം ഖുര്‍ആ നിന്റെ ഉന്നതമായ സാഹിത്യമൂല്യം അംഗീകരിക്കുന്നവ രായിരുന്നു. അത് മാരണമാണെന്നും പൈശാചികവചനങ്ങളാണെന്നും പറഞ്ഞ്, അതിന്റെ ദൈവികത അംഗീകരിക്കാതെ മാറിനില്‍ക്കുകയായിരുന്നു അവിശ്വാസികള്‍ ചെയ്തതെന്നു മാത്രം.

ഖുറൈശി നേതാവും അറബി സാഹിത്യത്തിലെ അജയ്യനുമായിരുന്ന വലീദുബ്‌നു മുഗീറയോട് ഖുര്‍ആനിനെതിരെ പരസ്യപ്രസ്താവന നടത്ത ണമെന്നാവശ്യപ്പെട്ട അബൂജഹ്‌ലിന് അദ്ദേഹം നല്‍കിയ മറുപടി ശ്രദ്ധേയ മാണ്. 'ഞാനെന്താണ് പറയേണ്ടത്? ഗദ്യത്തിലും പദ്യത്തി ലും ജിന്നുകളുടെ  കാവ്യങ്ങളിലും അറബി ഭാഷയുടെ മറ്റേതൊരു സാഹിത്യശാഖയിലും നി ങ്ങളേക്കാള്‍ എനിക്ക് അറിവുണ്ട്. അല്ലാഹു വാണ് സത്യം! ഈ മനുഷ്യന്‍ സമര്‍പ്പിക്കുന്ന വചനങ്ങള്‍ക്ക് അവയില്‍ ഒന്നിനോടും സാദൃശ്യമില്ല. അല്ലാഹുവാണെ, അവന്റെ വചനങ്ങ ള്‍ക്ക് വിസ്മയാവഹമായ ഒരു മാധുര്യവും പ്രത്യേകമായൊരു ഭംഗിയുമുണ്ട്. അതിന്റെ കൊമ്പുകളും ചില്ലകളും ഫല ങ്ങള്‍ നിറഞ്ഞതും മുരട് പശിമയാര്‍ന്ന മണ്ണില്‍ ഊന്നിനില്‍ക്കുന്നതുമാണ്. തീര്‍ച്ചയായും അത് സര്‍വവചനങ്ങളേക്കാളും ഉന്നതമാണ്. അതിനെ താഴ്ത്തിക്കാ ണിക്കാന്‍ മറ്റൊരു വചനത്തിനും സാധ്യമല്ല. അതിന്റെ കീഴില്‍ അകപ്പെടുന്ന സകലതിനെയും അത് തകര്‍ത്തുകളയും, തീര്‍ച്ച!

ഇത് ഒരു അമുസ്‌ലിമിന്റെ പ്രസ്താവനയാണെന്ന് നാം ഓര്‍ക്കണം. ഖുര്‍ആനിന്റെ സാഹിത്യമൂല്യത്തെക്കുറിച്ച് ഇതിനേക്കാള്‍ നല്ല ഒരു സര്‍ട്ടിഫിക്കറ്റ് ആവശ്യമില്ല.

അനുകരിക്കാനാകാത്ത ശൈലിയാണ് ഖുര്‍ആനിന്റെത്. ഇക്കാര്യം ആധുനികരായ മുസ്‌ലിംകളല്ലാത്ത അറബി പണ്ഡിതന്മാര്‍ പോലും അംഗീകരിച്ചിട്ടുള്ളതാണ്. ഓറിയന്റലിസ്റ്റായ ജി. സെയ്ല്‍ എഴുതുന്നത് കാണുക:

The style of the Quran is beautiful, it is adorned with bold figures after the Eastern taste, enlivened with florid and sententions expressions and in many places where the Majesty and attributes of God are described, sublime and magnificient (G. Sale, The Koran: Commonly called Al-Quran, with a preliminery discourse, London 1899 vol 1 page 47)

(പൗരസ്ത്യാസ്വാദനത്തിന്റെ വ്യക്തമായ ബിംബങ്ങളാല്‍ അലംകൃതമാക്കപ്പെടുകയും ഉപമാലങ്കാരങ്ങളാലും അര്‍ഥ സമ്പുഷ്ടമായ പദപ്ര യോഗങ്ങളാലും ചൈതന്യവത്താക്കപ്പെടുകയും  ചെയ്തിട്ടുള്ള ഖുര്‍ആനിന്റെ ശൈലി അതിസുന്ദരമാണ്. ദൈവിക ഗുണങ്ങളെയും പ്രതാ പത്തെയും കുറിച്ച് പ്രതിപാദിക്കുമ്പോള്‍ അതിന്റെ ഭാഷ പ്രൗഢവും ഗംഭീരവുമായിത്തീരുന്നു).

മറ്റൊരു ഓറിയന്റലിസ്റ്റായ എ.ജെ. ആര്‍ബറി എഴുതുന്നു:

"The complex prosody, a rich repertory of subtle and complicated rhymes had been completely perfected. A vocabulary of themes, images and figures extensive but nevertheless circumscribed, was firmly established" (A.J. Arberry, The Quran interpreted, London 1955 page 11)

(ഗഹനവും സങ്കീര്‍ണവുമായ കാവ്യശകലങ്ങളുടെ ഒരു സമ്പന്നമായ കലവറ സരളമല്ലാത്ത പദ്യരചനാരീതിയില്‍ പൂര്‍ണമായി കുറ്റമറ്റതാ ക്കപ്പെ ട്ടിരിക്കുന്നു. പ്രമേയങ്ങളുടെയും  ബിംബങ്ങളുടെയും രൂപങ്ങളുടെയും വിപു ലമല്ലെങ്കിലും ക്ലിപ്തമായ പദസഞ്ചയത്തില്‍ അവ ബലിഷ്ഠമായി സ്ഥാപിക്കപ്പെടുകയും ചെയ്തിരിക്കുന്നു)

ഖുര്‍ആനിന്റെ ശൈലിയും ഭാഷയും സാഹിത്യവുമെല്ലാം അതുല്യമാണ്. അനുകരണത്തിന് അതീതമാണ്.  അതിസുന്ദരമാണ്. അറബിയ റിയാവുന്ന എല്ലാവര്‍ക്കും മനസ്സിലാക്കാവുന്നതാണിത്. ഖുര്‍ആനിലെ ഓരോ സൂക്തവും അത്യാകര്‍ഷകവും ശ്രോതാവിന്റെ മനസ്സില്‍ മാറ്റത്തിന്റെ വേലിയേറ്റമുണ്ടാക്കുന്നതുമാണ്.  ഇത് അറബിയറിയാവുന്ന ആധുനികരും പൗരാണികരുമായ വിമര്‍ശകരെല്ലാം സമ്മതി ച്ചിട്ടുള്ളതാണ്.

ഒരു കാര്യം ദൈവിക ദൃഷ്ടാന്തമാകുന്നത് അത് അജയ്യമാകുമ്പോഴാണ്. മോശെ പ്രവാചകന്‍ തന്റെ വടി നിലത്തിട്ടപ്പോള്‍ അത് ഉഗ്രസര്‍പ്പ മായി മാറി. പ്രസ്തുത ദൈവിക ദൃഷ്ടാന്തത്തോട് മല്‍സരിക്കാനായി വന്ന മാന്ത്രികന്മാരുടെ വടികളെയും കയറുകളയുമെല്ലാം ആ സര്‍പ്പം വിഴുങ്ങി. ഇത് ഖുര്‍ആനിലും ബൈബിളിലുമെല്ലാം വിവരിക്കുന്നുണ്ട്.

ഖുര്‍ആന്‍ അവകാശപ്പെടുന്നത് അതിന്റെ ശൈലിയും ഘടനയും ആശയാലേഖനവും സാഹിത്യവുമെല്ലാം അജയ്യമാണെനും അതിന് തുല്യ മായ ഒരു രചന നടത്തുവാന്‍ സൃഷ്ടികള്‍ക്കൊന്നും സാധ്യമല്ലെന്നുമാണ്. 'നമ്മുടെ ദാസന് നാം അവതരിപ്പിച്ചുകൊടുത്തതിനെപ്പറ്റി നിങ്ങള്‍ സംശയാലുക്കളാണെങ്കില്‍ അതിന്‍േറത് പോലെയുള്ള ഒരു അധ്യായമെങ്കിലും നിങ്ങള്‍ കൊണ്ടുവരിക. അല്ലാഹുവിന് പുറമെ നിങ്ങള്‍ ക്കുള്ള സഹായികളെയും വിളിച്ചുകൊള്ളുക. നിങ്ങള്‍ സത്യവാന്മാരാണെങ്കില്‍! നിങ്ങള്‍ക്കത് ചെയ്യാ ന്‍ കഴിഞ്ഞില്ലെങ്കില്‍ -നിങ്ങള്‍ക്ക് ഒരിക്കലും അതു ചെയ്യാന്‍ കഴിയുക യില്ല- മനുഷ്യരും കല്ലുകളും ഇന്ധനമായി കത്തിക്കപ്പെടുന്ന നരകാഗ്‌നിയെ നിങ്ങള്‍ കാത്തുസൂക്ഷിച്ചു കൊള്ളുക. സത്യനിഷേധികള്‍ക്കുവേണ്ടി ഒരുക്കിവെക്കപ്പെട്ടതാകുന്നു അത്' (2:23,24). ഇത് സത്യമാണെന്ന് ഭാഷാ പരിജ്ഞാനമുള്ളവരെല്ലാം സമ്മതിക്കുന്നു.

ഖുര്‍ആനിലേതിന് തുല്യമായ ഒരു അധ്യായമെങ്കിലും കൊണ്ടുവരാനുള്ള അതിന്റെ വെല്ലുവിളിക്ക് ഉത്തരം നല്‍കുവാന്‍ അറബ് സാഹി ത്യരംഗത്തുള്ള ആര്‍ക്കുംതന്നെ ഇതുവരെ കഴിഞ്ഞിട്ടുമില്ല. ഖുര്‍ആന്‍ മുഹമ്മദ് നബി(സ)ക്ക് അവതരിപ്പിക്കപ്പെട്ട ദൈവിക ദൃഷ്ടാന്തമാ ണെന്ന വാദത്തെ പരിഹസിക്കുവാനല്ലാതെ പ്രതിരോധിക്കുവാനോ മറുപടി നല്‍കുവാനോ അതിലെ ഏതെങ്കിലുമൊരു അധ്യായത്തിന് തുല്യമായ അധ്യായം കൊണ്ടുവരാനോ അറബിയറിയാവുന്ന വിമര്‍ശകന്മാര്‍ക്ക് പോലും കഴിയുന്നില്ലെന്നതാണ് വാസ്തവം.

എട്ടാം നൂറ്റാണ്ടിനു ശേഷമാണ് ക്വുര്‍ആന്‍ രൂപീകരിക്കപ്പെട്ടു കഴിഞ്ഞതെന്ന് ജറുസലേമിലെ 'ഖു ബ്ബത്തു സ്‌സ്വഖ്‌റ'യില്‍ രേഖപ്പെടുത്തപ്പെട്ട അറബി ലിഖിതങ്ങൾ തെളിവല്ലേ ? അമവീ ഖലീഫയായിരുന്ന അബ്ദുല്‍ മലി ക്ക് ബ്‌നു മര്‍വാനിന്റെ ഭരണകാലത്ത് നിര്‍മിക്കപ്പെട്ട ഖുബ്ബത്തു സ്‌സ്വഖ്റയിലെ പുറത്തും അകത്തും മൊസൈക്കില്‍ രേഖപ്പെടുത്തപ്പെട്ട അറബി ലിഖിതങ്ങളിൽ ക്വുർആൻ വചനങ്ങൾ രേഖപ്പെടുത്തിയിരിക്കുന്നത് ഇന്ന് മുസ്ഹഫുകളിൽ കാണുന്ന ക്രമത്തിലല്ല. അത് എഴുതപ്പെടുന്ന കാലത്ത് ക്വുര്‍ആന്‍ പൂര്‍ണമായി രൂപീകരിക്കപ്പെട്ടിട്ടുണ്ടായിരുന്നില്ലെന്നല്ലേ ഇത് വ്യക്തമാക്കുന്നത്?

ട്ടാം നൂറ്റാണ്ടിന്റെ തുടക്കം വരെ ക്വുര്‍ആന്‍ പൂര്‍ണമായും രൂപീകരിക്കപ്പെട്ടു കഴിഞ്ഞിരുന്നില്ലെന്നും അത് തലമുറകളെടുത്ത് രൂപീകരിക്കപ്പെട്ട വചനങ്ങളുടെ സമാഹാരമാണെന്നുമാണ് ചില ഓറിയന്റലിസ്റ്റുകളുടെ വാദം. ഇങ്ങനെ വാദിക്കുന്നവര്‍ ജറുസലേമിലെ 'ഖു ബ്ബത്തു സ്‌സ്വഖ്‌റ'യില്‍ (Dome of the Rock) ല്‍ രേഖപ്പെടുത്തപ്പെട്ട അറബി ലിഖിതങ്ങളാണ് തങ്ങള്‍ക്കുള്ള തെളിവായി അവതരിപ്പി ക്കുന്ന ത് (Patricia Crone & Michael Cook: Hagarism: The Making of the Islamic World, Cambridge, 1980, Page 139-149.)

അമവീ ഖലീഫയായിരുന്ന അബ്ദുല്‍ മലി ക്ക് ബ്‌നു മര്‍വാനിന്റെ ഭരണകാലത്ത് നിര്‍മിക്കപ്പെട്ട സുന്ദരമായൊരു അഷ്ടഭുജ (octagon) കെട്ടിടമാണ് ഖുബ്ബത്തു സ്‌സ്വഖ്ഃ. ഇതിന്റെ പുറ ത്തും അകത്തും മൊസൈക്കില്‍ രേഖപ്പെടുത്തപ്പെട്ട അറബി ലിഖിതങ്ങളെയാണ് അത് എഴുതപ്പെടുന്ന കാലത്ത് ക്വുര്‍ആന്‍ പൂര്‍ണമായി രൂപീകരിക്കപ്പെട്ടിട്ടുണ്ടായിരുന്നില്ലെന്നതിന് തെളിവായി ചൂണ്ടിക്കാണിക്കപ്പെടുന്നത്. ആദ്യകാല മുസ്‌ലിം സമ്പ്രദായങ്ങളെ യോ അവര്‍ ക്വുര്‍ആനിനെ ഉപയോഗിച്ച രീതിയെയോ കുറിച്ച് യാതൊന്നും അറിയാത്തതുകൊണ്ടാണ് ഇത്തരം ബാലിശമായ വാദങ്ങള്‍ ഉടലെടുക്കു ന്നത്.

യഥാര്‍ഥത്തില്‍, ഖുബ്ബത്തു സ്‌സ്വഖ്‌റയിലെ അഷ്ടഭുജത്തിന്മേല്‍ രേഖപ്പെടുത്തപ്പെട്ടിരിക്കുന്നത് ക്രമപ്രകാരമുള്ള ക്വുര്‍ആന്‍ വചനങ്ങ ളല്ല. ക്വുര്‍ആന്‍ പഠിപ്പിക്കുകയോ രേഖപ്പെടുത്തുകയോ ചെയ്യുകയെന്ന ലക്ഷ്യത്തോടെ എഴുതപ്പെട്ട രേഖയുമല്ല അത്. പ്രത്യുത ഒരു സന്ദേശ ത്തിന്റെ രേഖീകരണം മാത്രമാണത്. പ്രസ്തുത സന്ദേശത്തിനിടക്ക് ക്വുര്‍ആന്‍ സൂക്തങ്ങളോ അതിന്റെ ഖണ്ഡങ്ങളോ കടന്നു വരുന്നുവെന്ന് മാത്രമേയുള്ളൂ. ഒരു പ്രഭാഷകന്‍ തനിക്കാവശ്യമുള്ള ഉദ്ധരണികള്‍ ഉപയോഗിക്കുന്നതുപോലെ 'ഖുബ്ബത്തു സ്‌സ്വഖ്‌റാ'യില്‍ സന്ദേശമെഴുതി യവര്‍ അവര്‍ നല്‍കുവാനുദ്ദേശിച്ച ദൂതിന് ഉപോല്‍ബലകമായ ക്വുര്‍ആന്‍ സൂക്തങ്ങളോ ഖണ്ഡങ്ങളോ ഉപയോഗിച്ചുവെന്ന് മാത്രമേയു ള്ളൂ. അഷ്ടഭുജത്തിലെ സന്ദേശം വായിക്കുന്ന ആര്‍ക്കും ബോധ്യപ്പെടുന്ന സരളമായ ഒരു വസ്തുതയാണിത്. പ്രസ്തുത സന്ദേശത്തിന്റെ പരിഭാഷ പരിശോധിക്കുക:

അഷ്ടഭുജത്തിനകത്തെ സന്ദേശം (പരിഭാഷ):

പരമകാരുണികനും കരുണാനിധിയുമായ അല്ലാഹുവിന്റെ നാമത്തില്‍; അല്ലാഹുവല്ലാതെ ആരാധ്യനില്ല. അവന്‍ ഏകനാണ്. അവന് പങ്കു കാരൊന്നുമില്ല. അവനാണ് എല്ലാ ആധിപത്യവും; അവന്നുതന്നെയാണ് സ്തുതികളും. അവന്‍ ജീവിപ്പിക്കുന്നു; അവന്‍ മരിപ്പിക്കുകയും ചെയ്യുന്നു. അവനാണ് എല്ലാ കാര്യങ്ങളുടെയും മേലുള്ള അധീശത്വം. മുഹമ്മദ് അല്ലാഹുവിന്റെ ദൂതനും ദാസനുമാകുന്നു.

തീര്‍ച്ചയായും അല്ലാഹുവും മലക്കുകളും പ്രവാചകന്റെ മേല്‍ അനുഗ്രഹങ്ങള്‍ ചൊരിയുന്നു. സത്യവിശ്വാസികളേ നിങ്ങള്‍ അദ്ദേഹത്തി ന്റെ മേല്‍ കാരുണ്യവും ശാന്തിയുമുണ്ടാകുവാന്‍ പ്രാര്‍ഥിക്കുക. അദ്ദേഹത്തിന്റെ മേല്‍ അല്ലാഹുവിന്റെ അനുഗ്രഹങ്ങളും ശാന്തിയുമു ണ്ടാകട്ടെ.

വേദക്കാരേ, നിങ്ങള്‍ മതകാര്യത്തില്‍ അതിരുകവിയരുത്. അല്ലാഹുവിന്റെ പേരില്‍ വാസ്തവമല്ലാതെ നിങ്ങള്‍ പറയുകയും ചെയ്യരുത്. മര്‍യമിന്റെ മകനായ മസീഹ് ഈസാ അല്ലാഹുവിന്റെ ദൂതനും, മര്‍യമിലേക്ക് അവന്‍ ഇട്ടുകൊടുത്ത അവന്റെ വചനവും, അവങ്കല്‍ നിന്നുള്ള ഒരു ആത്മാവും മാത്രമാകുന്നു. അത് കൊണ്ട് നിങ്ങള്‍ അല്ലാഹുവിലും അവന്റെ ദൂതന്‍മാരിലും വിശ്വസിക്കുക. ത്രിത്വം എന്ന വാക്ക് നിങ്ങള്‍ പറയരുത്. നിങ്ങളുടെ നന്‍മയ്ക്കായി നിങ്ങള്‍ (ഇതില്‍ നിന്ന്) വിരമിക്കുക. അല്ലാഹു ഏക ആരാധ്യന്‍ മാത്രമാകുന്നു. തനിക്ക് ഒരു സന്താനമുണ്ടായിരിക്കുക എന്നതില്‍ നിന്ന് അവനെത്രയോ പരിശുദ്ധനത്രെ. ആകാശങ്ങളിലുള്ളതും ഭൂമിയിലുള്ളതുമെല്ലാം അവന്റെതാകുന്നു. കൈകാര്യകര്‍ത്താവായി അല്ലാഹു തന്നെ മതി.

അല്ലാഹുവിന്റെ അടിമയായിരിക്കുന്നതില്‍ മസീഹ് ഒരിക്കലും വൈമനസ്യം കാണിക്കുന്നതല്ല. (അല്ലാഹുവിന്റെ) സാമീപ്യം സിദ്ധിച്ച മലക്കുകളും (വൈമനസ്യം കാണിക്കുന്നതല്ല.) അവനെ (അല്ലാഹുവെ) ആരാധിക്കുന്നതില്‍ ആര്‍ വൈമനസ്യം കാണിക്കുകയും, അഹംഭാ വം നടിക്കുകയും ചെയ്യുന്നുവോ അവരെ മുഴുവനും അവന്‍ തന്റെ അടുക്കലേക്ക് ഒരുമിച്ചുകൂട്ടുന്നതാണ്.

അല്ലാഹുവേ, നിന്റെ ദൂതനും നിന്റെ ദാസനുമായ മര്‍യമിന്റെ പുത്രന്‍ മസീഹിനെ നീ അനുഗ്രഹിക്കേണമേ. അദ്ദേഹം ജനിച്ച ദിവസവും മരിക്കുന്ന ദിവസവും ജീവനോടെ എഴുന്നേല്‍പിക്കപ്പെടുന്ന ദിവസവും അദ്ദേഹത്തിന് സമാധാനം.

അതത്രെ മര്‍യമിന്റെ മകനായ ഈസാ അവര്‍ ഏതൊരു വിഷയത്തില്‍ തര്‍ക്കിച്ച് കൊണ്ടിരിക്കുന്നുവോ അതിനെപ്പറ്റിയുള്ള യഥാര്‍ഥമായ വാക്കത്രെ ഇത്. ഒരു സന്താനത്തെ സ്വീകരിക്കുക എന്നത് അല്ലാഹുവിന്നുണ്ടാകാവുന്നതല്ല. അവന്‍ എത്ര പരിശുദ്ധന്‍! അവന്‍ ഒരു കാര്യം തീരുമാനിച്ച് കഴിഞ്ഞാല്‍ അതിനോട് ഉണ്ടാകൂ എന്ന് പറയുക മാത്രംചെയ്യുന്നു. അപ്പോള്‍ അതുണ്ടാകുന്നു. (ഈസാ പറഞ്ഞു) തീര്‍ച്ചയായും അല്ലാഹു എന്റെയും നിങ്ങളുടെയും രക്ഷിതാവാകുന്നു. അതിനാല്‍ അവനെ നിങ്ങള്‍ ആരാധിക്കുക. ഇതത്രെ നേരെയുള്ള മാര്‍ഗം.

താനല്ലാതെ ഒരു ദൈവവുമില്ലെന്നതിന് അല്ലാഹു സാക്ഷ്യം വഹിച്ചിരിക്കുന്നു. മലക്കുകളും അറിവുള്ളവരും (അതിന്ന് സാക്ഷികളാകുന്നു) അവന്‍ നീതി നിര്‍വഹിക്കുന്നവനത്രെ. അവനല്ലാതെ ദൈവമില്ല. പ്രതാപിയും യുക്തിമാനുമത്രെ അവന്‍. തീര്‍ച്ചയായും അല്ലാഹുവിങ്കല്‍ മതം എന്നാല്‍ ഇസ്‌ലാമാകുന്നു. വേദഗ്രന്ഥം നല്‍കപ്പെട്ടവര്‍ തങ്ങള്‍ക്ക് (മതപരമായ) അറിവ് വന്നുകിട്ടിയ ശേഷം തന്നെയാണ് ഭിന്നിച്ചത്. അവര്‍ തമ്മിലുള്ള കക്ഷിമാത്‌സര്യം നിമിത്തമത്രെ അത്. വല്ലവരും അല്ലാഹുവിന്റെ തെളിവുകള്‍ നിഷേധിക്കുന്നുവെങ്കില്‍ അല്ലാഹു അതിവേഗം കണക്ക് ചോദിക്കുന്നവനാകുന്നു.

അഷ്ടഭുജത്തിന് പുറത്തെ സന്ദേശം (പരിഭാഷ):

പരമകാരുണികനും കരുണാനിധിയുമായ അല്ലാഹുവിന്റെ നാമത്തില്‍. അല്ലാഹുവല്ലാതെ ആരാധ്യനില്ല. അവന്‍ ഏകനാണ്. അവന് പങ്കുകാരൊന്നുമില്ല. (നബിയേ,) പറയുക: കാര്യം അല്ലാഹു ഏകനാണ് എന്നതാകുന്നു. അല്ലാഹു ഏവര്‍ക്കും ആശ്രയമായിട്ടുള്ളവനാകു ന്നു. അവന്‍ (ആര്‍ക്കും) ജന്‍മം നല്‍കിയിട്ടില്ല. (ആരുടെയും സന്തതിയായി) ജനിച്ചിട്ടുമില്ല. അവന്ന് തുല്യനായി ആരും ഇല്ലതാനും മുഹ മ്മദ് അല്ലാഹുവിന്റെ ദൂതനാകുന്നു. അദ്ദേ ഹത്തില്‍ അല്ലാഹുവിന്റെ അനുഗ്രഹങ്ങളുണ്ടാകട്ടെ.

പരമകാരുണികനും കരുണാനിധിയുമായ അല്ലാഹുവിന്റെ നാമത്തില്‍. അല്ലാഹുവല്ലാതെ ആരാധ്യനില്ല. അവന്‍ ഏകനാണ്. അവന് പങ്കുകാരൊന്നുമില്ല. മുഹമ്മദ് അല്ലാഹുവിന്റെ ദൂതനാകുന്നു. തീര്‍ച്ചയായും അല്ലാഹുവും അവന്റെ മലക്കുകളും നബിയോട് കാരുണ്യം കാണിക്കുന്നു. സത്യവിശ്വാസികളേ, നിങ്ങള്‍ അദ്ദേഹത്തിന്റെ മേല്‍ (അല്ലാഹുവിന്റെ) കാരുണ്യവും ശാന്തിയുമുണ്ടാകാന്‍ പ്രാര്‍ഥിക്കുക.

പരമകാരുണികനും കരുണാനിധിയുമായ അല്ലാഹുവിന്റെ നാമത്തില്‍. അല്ലാഹുവല്ലാതെ ആരാധ്യനില്ല. അവന്‍ ഏകനാണ്. സ്തുതികളെ ല്ലാം അല്ലാഹുവിനാണ് സന്താനത്തെ സ്വീകരിച്ചിട്ടില്ലാത്തവനും, ആധിപത്യത്തില്‍ പങ്കാളിയില്ലാത്തവനും നിന്ദ്യതയില്‍ നിന്ന് രക്ഷിക്കാന്‍ ഒരു രക്ഷകന്‍ ആവശ്യമില്ലാത്തവനുമായ അല്ലാഹുവിന് സ്തുതി! എന്ന് നീ പറയുകയും അവനെ ശരിയാംവണ്ണം മഹത്വപ്പെടുത്തുകയും ചെയ്യുക. മുഹമ്മദ് അല്ലാഹുവിന്റെ ദൂതനാണ്.

അദ്ദേഹത്തിനു മേലും മലക്കുകളുടെയും പ്രവാചകന്‍മാരുടെയും മേലും അല്ലാഹുവിന്റെ അനുഗ്രഹങ്ങളുണ്ടാവട്ടെ. അദ്ദേഹത്തില്‍മേല്‍ അല്ലാഹുവിന്റെ ശാന്തിയും കാരുണ്യവുമുണ്ടാകട്ടെ. അല്ലാഹുവിന്റെ നാമത്തില്‍, പരമകാരുണികന്‍, കരുണാനിധി. അല്ലാഹുവല്ലാതെ ആരാധ്യനില്ല. അവന്‍ ഏകനാണ്. അവന് പങ്കുകാരില്ല.

അവനാണ് എല്ലാ ആധിപത്യവും. അവനാണ് എല്ലാ കാര്യങ്ങളുടെയും മേലുള്ള അധീശത്വം. മുഹമ്മദ് അല്ലാഹുവിന്റെ ദൂതനാകുന്നു. അദ്ദേഹത്തിനു മേല്‍ അല്ലാഹുവിന്റെ അനുഗ്രഹങ്ങളുണ്ടാവട്ടെ. അദ്ദേഹത്തിന്റെ ജനങ്ങള്‍ക്കുമേല്‍ പുനരുത്ഥാന നാളില്‍ അദ്ദേഹം നടത്തുന്ന  ശുപാര്‍ശ അവന്‍ സ്വീകരിക്കട്ടെ.

പരമകാരുണികനും കരുണാനിധിയുമായ അല്ലാഹുവിന്റെ നാമത്തില്‍. അല്ലാഹുവല്ലാതെ ആരാധ്യനില്ല. അവന്‍ ഏകനാണ്. അവന് പങ്കുകാരില്ല. മുഹമ്മദ് അല്ലാഹുവിന്റെ ദൂതനാകുന്നു. അദ്ദേഹത്തിനു മേല്‍ അല്ലാഹുവിന്റെ അനുഗ്രഹങ്ങളുണ്ടാവട്ടെ. ഈ കുംഭഗോ പുരം നിര്‍മിച്ചത് ദൈവദാസനായ അബ്ദുല്ലാ അല്‍ ഇമാം അല്‍ മഅ്മൂനാണ്; വിശ്വാസികളുടെ നേതാവ്. എഴുപത്തി രണ്ടാം വര്‍ഷത്തില്‍. അല്ലാഹു അദ്ദേഹത്തില്‍ നിന്ന് ഇത് സ്വീകരിക്കുകയും അദ്ദേഹത്തില്‍ സംപ്രീതനാവുകയും ചെയ്യട്ടെ, ആമീന്‍. സര്‍വലോകരക്ഷിതാവായ, അല്ലാഹുവിന് സ്തുതി''.

ഇത് വായിക്കുന്ന ആര്‍ക്കും മനസ്സിലാക്കാവുന്ന യാഥാര്‍ഥ്യമാണ് ക്വുര്‍ആന്‍ അധ്യായക്രമത്തില്‍ രേഖപ്പെടുത്തിയതല്ല ഇത് എന്നുള്ള വസ്തുത. അല്ലാഹുവിനെ പരിചയപ്പെടുത്തുന്ന ആമുഖത്തില്‍ വ്യത്യസ്ത ക്വുര്‍ആന്‍ സൂക്തങ്ങളില്‍ പ്രയോഗിച്ചിരിക്കുന്ന അല്ലാഹുവി ന്റെ നാമ-ഗുണവിശേഷണങ്ങള്‍  പ്രസ്താവിച്ചിരിക്കുന്നുവെന്ന് മാത്രമെയുള്ളൂ. അത് ഉദ്ധരിച്ചുകൊണ്ട് അത് അന്നു നിലനിന്നിരുന്ന ക്വുര്‍ആന്‍ സൂക്തമായിരുന്നുവെന്നും പിന്നീടാണ് അതിലെ ദൈവഗുണ, വിശേഷണങ്ങളെ വേര്‍തിരിച്ചുകൊണ്ടുള്ള സൂക്തങ്ങള്‍ ഇതില്‍ നിന്ന് പരിണമിച്ചുണ്ടായത് എന്നും വാദിക്കുന്നത് മുസ്‌ലിംകള്‍ നടത്തുന്ന പ്രഭാഷണങ്ങളെയും സന്ദേശപ്രചരണത്തെയും കുറിച്ച അജ്ഞത കൊണ്ടാണ്. അല്ലാഹുവിന്റെ നാമത്തില്‍ ആരംഭിക്കുന്ന മുസ്‌ലിം സന്ദേശങ്ങളില്‍ അല്ലാഹുവിനെ സ്തുതിക്കുകയും, അവന്റെ മഹത്വം ഉദ്‌ഘോഷിക്കുകയും മുഹമ്മദ്‌നബിലയുടെ മേല്‍ അനുഗ്രഹങ്ങള്‍ക്കു വേണ്ടി പ്രാര്‍ഥിക്കുകയും ചെയ്ത ശേഷമാണ് മറ്റു കാര്യങ്ങള്‍ പരാമര്‍ശിക്കാറുള്ളത്. ഈ സന്ദേശത്തിലും അങ്ങനെത്തന്നെയാണുള്ളത്. ക്രിസ്ത്യാനികള്‍ താമസിക്കുന്ന സ്ഥലത്താണ് ഖുബ്ബത്തു സ്‌സ്വഖ്‌ റാ നിര്‍മിക്കപ്പെട്ടത് എന്നതിനാല്‍ യേശു ക്രിസ്തുവിനെ സംബന്ധിച്ച ഇസ്‌ലാമിക നിലപാട് വ്യക്തമാക്കുകയും ക്രൈസ്തവ നിലപാടിനെ വിമര്‍ശിക്കുകയും ചെയ്യുന്ന വചനങ്ങള്‍ പ്രസ്തുത സന്ദേശത്തിന്റെ ഭാഗമായത് സ്വാഭാവികമാണ്.

ക്വുർആനിലെ പതിനേഴാം അധ്യായത്തിന്റെ 111ാം വചനം രേഖപ്പെടുത്തിയപ്പോള്‍ പ്രസ്തുത വചനത്തിന്റെ തുടക്കത്തിലുള്ള 'നീ പറയുക' (വഖുലി) യെന്ന ഭാഗം ഖുവ്വത്തു സ്‌സ്വഖ്‌റായുടെ പടിഞ്ഞാറ് ഭാഗത്ത് രേഖപ്പെടുത്തിയിട്ടില്ലെന്നതാണ് ഇതെഴുതുമ്പോള്‍ ക്വുര്‍ആന്‍ പൂര്‍ണമായിരു ന്നില്ലെന്ന് വാദിക്കുന്നവര്‍ക്കുള്ള സുപ്രധാനമായ ഒരു 'തെളിവ്'. ഒരു സന്ദേശത്തിന്റെ ഭാഗമെന്ന നിലയ്ക്ക് ക്വുര്‍ആന്‍ വചനങ്ങള്‍ ഉദ്ധരിക്കുമ്പോള്‍ 'നീ പറയുക' പോലെയുള്ള പരാമര്‍ശങ്ങള്‍ ഒഴിവാക്കി വചനത്തിലെ ആശയപ്രധാനമായ ഭാഗം മാത്രം പരാമര്‍ശിക്കുന്ന സമ്പ്രദായം ഇന്നത്തേതുപോലെ മുസ്‌ലിം സമൂഹത്തില്‍ അന്നും നില നിന്നിരുന്നുവെന്ന് വ്യക്തമാക്കുന്നതാണ് ഈ രേഖ; അതല്ലാതെ, ക്വുര്‍ആന്‍ പരിണമിച്ചു ണ്ടായതാണെന്നതിന് അത് തെല്ലും തെളിവു നല്‍കുന്നില്ല.

ഖുബ്ബത്തു സ്‌സ്വഖ്‌റായുടെ പുറത്ത് തെക്ക് ഭാഗത്ത് പൂര്‍ണമായി രേഖപ്പെടുത്ത പ്പെട്ട ക്വുര്‍ആനിലെ നൂറ്റി പന്ത്രണ്ടാം അധ്യായത്തിലെ 'നീ പറയുക' (ഖുല്‍) എന്ന ഭാഗം ഒഴിവാക്കികൊണ്ടാണ് അബ്ദുല്‍മലിക്കു ബ്‌നു മര്‍വാനിന്റെ കാലത്തും ശേഷവും നിര്‍മിക്കപ്പെട്ട നാണയങ്ങളില്‍ ഈ സൂക്തം മുദ്രീകരിക്കപ്പെട്ടിരിക്കുന്നത് എന്ന വസ്തുത പതിനേഴാം അധ്യായം 111ാം വചനത്തില്‍ 'നീ പറയുക' യെന്ന ഭാഗം ഒഴിവാക്കിക്കൊണ്ട് രേഖപ്പെടുത്തിയത് ക്വുര്‍ആനിന്റെ രൂപീകരണം പില്‍ക്കാല ത്താണ് നടന്നതെന്നതിന് തെളിവാക്കുന്നവരുടെ മുഴുവന്‍ വാദങ്ങളെയും തകര്‍ത്തുകളയുന്നുവെന്ന് പ്രമുഖ ഓറിയന്റലിസ്റ്റായ എസ്‌റ്റെല്ലേ വെലാന്‍ വ്യക്തമാക്കുന്നുണ്ട്. (Estelle Whelan: "Forgotten Witness: Evidence for the Early Codification of the Qur'an"; Journal of American Oriental Society, 1998, Vol. 118, P. 1-14) സമകാലിക രേഖകളെയോ സമ്പ്രദായങ്ങളെയോ കുറിച്ച് പഠിക്കാതെ, ക്വുര്‍ആനിന്റെ ചരിത്രപരതയെ നിഷേധിക്കുവാന്‍ കിട്ടിയ വടികളെല്ലാമെടുത്ത് എറിയാന്‍ ശ്രമിക്കുന്നവരുടെ 'തെളിവുകള്‍' അവരുടെ തന്നെ ബൗദ്ധികസത്യസന്ധതക്കു നേരെ തിരിച്ചു വരുന്ന ബൂമറാംഗുകളായിത്തീരുന്നതാണ് നാം ഇവിടെ കാണുന്നത്.

ഹിജ്‌റ 72ലേതാണെന്ന് ഉറപ്പുള്ള ഒരു രേഖയില്‍ ക്വുര്‍ആനിന്റെ വ്യത്യസ്ത ഭാഗങ്ങളില്‍ നിന്നെടുത്ത് ഒരേ വിഷയത്തിലുള്ള വചനങ്ങള്‍ ഉദ്ധരിക്കപ്പെട്ടിരിക്കുന്നുവെന്നത് അക്കാലത്ത് ക്വുര്‍ആനിന്റെ കയ്യെഴുത്ത് പ്രതികള്‍ വ്യാപകമായി ഉപയോഗിക്കപ്പെട്ടിരുന്നുവെന്നതിന് തെളിവാണെന്നാണ് എസ്‌റ്റെല്ലെ വെലാന്‍ സമര്‍ഥിക്കുന്നത്. (Ibid.) ഒരു ഗ്രന്ഥത്തിന്റെ വ്യത്യസ്ത ഭാഗങ്ങളില്‍ പരാമര്‍ശിക്കപ്പെട്ടിരിക്കുന്ന ഒരേ വിഷയസംബന്ധിയായ വചനങ്ങള്‍ ഒരു പ്രത്യേക സന്ദര്‍ഭത്തിലോ രേഖയിലോ ഉദ്ധരിക്കണമെങ്കില്‍ പ്രസ്തുത ഗ്രന്ഥം പൂര്‍ണരൂപ ത്തില്‍ ഉപലബ്ധമായിരിക്കണമെന്നത് സാമാന്യയുക്തിയാണെന്നാണ് അദ്ദേഹത്തിന്റെ വാദം. അബ്ദുല്ലാ അല്‍ മഅ്മൂനാണ് ഖുബ്ബത്തു സ്‌സ്വഖ്‌റാ നിര്‍മിച്ചതെന്ന് രേഖപ്പെടുത്തപ്പെട്ടിട്ടുണ്ടെങ്കിലും അത് വ്യാജമാണെന്നും അബ്ദുല്‍മലിക്ക് ബ്‌നു മര്‍വാനിന്റെ പേര് മായ്ച്ചു കൊണ്ടാണ് മഅ്മൂനിന്റെ പേര് രേഖപ്പെടുത്തിയിട്ടുള്ളതെന്നും രേഖയില്‍ നിന്നു തന്നെ വ്യക്തമാണ്.   സ്വഹാബിമാരുടെ കാലത്ത് ക്വുര്‍ആന്‍ പൂര്‍ണ രൂപത്തില്‍ നിലനിന്നി രുന്നില്ല എന്നതിനുള്ള തെളിവായി കൊട്ടിഘോഷിക്കപ്പെട്ട ഖുബ്ബത്തുസ്‌സ്വഖ്‌റായിലെ ക്വുര്‍ആന്‍ രേഖകള്‍ വിമര്‍ശകര്‍ക്കെതിരായ തെളി വാണ് നല്‍കുന്നതെന്നര്‍ഥം.

അതുകൊണ്ടുതന്നെയായിരിക്കണം പാട്രിഷിയോ ക്രോണിന്റെയും മിഖയേല്‍ കുക്കിന്റെയും വാദങ്ങളെ സമര്‍ഥിക്കുവാന്‍ പാടുപെട്ട് ശ്രമിക്കുന്ന പിന്‍ഗാമികള്‍ പോലും ഖുബ്ബത്തു സ്‌സ്വഖ്‌റായിലെ ക്വുര്‍ആന്‍ ആലേഖനങ്ങളെ ക്വുര്‍ആനിന്റെ ചരിത്രപരതയെ സംശയാസ്പദമാക്കുന്ന തെളിവുകളുടെ കൂടെ പെടുത്താന്‍ മടിക്കുന്നത്. 'ബുദ്ധിപരമായി സംഭവിക്കാനാവാത്ത വാദങ്ങ ളാല്‍ നിബിഡവും ബാലിശമായ തെളിവുകള്‍ മാത്രമുള്ളതു (Micheal G Morony: Journal of Near Eastern Studies, Volume 41, No:2, April 1982, Page 157-159.) മെന്ന് കോണും കുക്കും ചേര്‍ന്നെഴുതിയ ഗ്രന്ഥത്തെ ആധുനിക ഓറിയന്റലിസ്റ്റുകളില്‍ പ്രമുഖനായ മിക്കയേല്‍ ജെ മൊറോണി വിശേഷിപ്പിച്ചതും മറ്റൊന്നുകൊണ്ടുമല്ല.

പുരാതനമെന്ന് കരുതപ്പെടുന്ന ഖുര്‍ആന്‍ പ്രതികളില്‍ ഉപയോഗിച്ചിരിക്കുന്ന ലിപിയായ കൂഫീലിപി യഥാർത്ഥത്തിൽ ഉണ്ടായത് എട്ടാം നൂറ്റാണ്ടിൽ മാത്രമാണെന്ന് ഭാഷാശാസ്ത്ര പഠനങ്ങൾ വ്യക്തമാക്കുന്നുണ്ട്. ഇതിനർത്ഥം ആ പ്രതികൾ എഴുതപ്പെട്ടത് എട്ടാം നൂറ്റാണ്ടിലാണെന്നാണ്. മുഹമ്മദ് നബി(സ)ക്ക് ശേഷം പതിറ്റാണ്ടുകള്‍ കഴിഞ്ഞ് രൂപീകരിക്കപ്പെട്ടതാണ് ഇന്ന് നില നിൽക്കുന്ന ഖുർആനിലെ വചനങ്ങള്‍ എന്നല്ലേ ഇത് വ്യക്തമാക്കുന്നത്?

റബിഭാഷയെയോ അതിന്റെ വികാസത്തെയോ ലിപി പരിണാമത്തെയോ കുറിച്ച് കൃത്യവും വ്യക്തവുമായി പഠിക്കാത്ത ചില ക്രി സ്ത്യന്‍ അപ്പോളജറ്റിക്കുകളാണ് പ്രധാനമായും ഈ വാദമുന്നിയിച്ചിരിക്കുന്നത്. ഉഥ്മാനീ മുസ്വ്ഹഫുകളായി അറിയപ്പെടുന്ന സമര്‍ക്ക ന്റ്, തോപ്കാപ്പി തുടങ്ങിയ കയ്യെഴുത്ത് പ്രതികള്‍ എഴുതപ്പെട്ടിരിക്കുന്നത് കൂഫിലിപിയിലാണെന്നും പ്രസ്തുത ലിപി തന്നെ അറിയപ്പെ ടുന്നത് ഇറാക്കിലുള്ള കൂഫാ പട്ടണത്തിന്റെ പേരിലാണെന്നും (Bruce A McDowell & Anees Zaka; Muslims and Christians at the Table: Promoting Biblical Understanding among North American Muslims, Phillipsburg (NJ), 1999, Page 76.)  ഖലീഫാ ഉമറിന്റെ (റ)കാലത്ത് നാമകരണം ചെയ്യപ്പെട്ട കൂഫാ പട്ടണത്തിന്റെ പേരിലുള്ള ലിപി ഉഥ്മാനിന്റെ കാലമായപ്പോഴേക്ക് വികസിച്ചു വന്നിരിക്കാന്‍ സാധ്യതയില്ലെന്നും അതുകൊണ്ട്ത ന്നെ ഈ കയ്യെടുത്തു പ്രതികള്‍ രചിക്കപ്പെട്ടത് ഉഥ്മാനിന്റെ കോലത്താവാന്‍ യാതൊരു സാധ്യതയുമില്ലെന്നും (N.A. Newman: Mohammed, The  Qur'an and Islam, Hatfield, 1996, P. 314.)പ്രസ്തുത ക്വുര്‍ആന്‍ ലിപിയില്‍ കുത്തുകളിട്ടിട്ടുണ്ടെന്നും കുത്തുകളിടുന്ന സമ്പ്രദായമാരംഭിച്ചത് ഹിജ്‌റ ഒന്നാം നൂറ്റാണ്ടിന് ശേഷമാണെന്നും (Dr. Robert A Morey: Winning the war against Radical Islam, Las Vegas, 2002, Page 70)അതിനാല്‍ നടേ പറഞ്ഞ കയ്യെഴുത്ത് പ്രതികള്‍ ഉഥ്മാനിന്റെ  (റ)ഭരണകാലത്തിന് ഒന്നര നൂറ്റാണ്ടുകളെങ്കിലും കഴിഞ്ഞിട്ടാവണം രചിച്ചിട്ടുള്ളതെന്നുമാണ് (John Gilchrist: Jam'al Qur'an: The Codification of the Qur'an Text, Monder (South Africa), P. 140-146.)അപ്പോളജറ്റിക്കു കളുടെ വാദം. ബൈബിളിനെ പ്പോലെത്തന്നെ ആദ്യകാല കയ്യെഴുത്ത് രേഖകളുടെ സാക്ഷ്യം വേണ്ടത്രയില്ലാത്ത ഗ്രന്ഥമാണ് ക്വുര്‍ആനുമെന്ന് വരുത്തിത്തീര്‍ക്കുന്നതിനു വേണ്ടി ഉന്നയിക്കപ്പെടുന്ന ഈ വാദത്തില്‍ ലവലേശം യാഥാര്‍ഥ്യമില്ലെന്ന് പുരാവസ്തു രേഖകളെക്കുറിച്ച സൂക്ഷ്മമായ പഠനം വ്യക്തമാ ക്കുന്നുണ്ട്.

ക്രിസ്തുവിന് മൂവായിരം കൊല്ലങ്ങള്‍ക്കു മുമ്പ്, ആദ്യകാല വെങ്കല യുഗത്തില്‍ മെസപ്പെട്ടോമിയയില്‍ (ഇന്നത്തെ ഇറാഖ്) വളര്‍ന്നു വിക സിച്ച സുമേറിയന്‍ നാഗരികതയിലുള്ളവര്‍ ഉപയോഗിച്ചിരുന്ന ക്യൂനിഫോം (cueniform) ലിപികളില്‍ നിന്നാരംഭിക്കുന്നു ആധുനിക അറബി ലിപിയുടെ ചരിത്രം. ക്യൂനിഫോമുകളില്‍ നിന്നുള്ള പ്രചോദനത്തില്‍ നിന്നാണ് നൈല്‍നദീതടത്തില്‍ ജീവിച്ചിരുന്ന ഈജിപ്തുകാര്‍ അല്‍പ കാലത്തിനുശേഷം അവരുടെ ലിപിയായ ഹൈരോഗ്ലിഫുകള്‍ (heiroglyphs) വികസിപ്പിച്ചെടുത്തത്. (Geoffrey Sampson: A Linguistic Introduction, Stanford University, 1990, P.78.) വെങ്കലയുഗത്തിന്റെ മധ്യകാലഘട്ടത്തില്‍ കാനന്‍ ദേശത്തും (ഇന്നത്തെ ഇസ്രായീലും ഫലസ്തീ നും) ഈജി പ്തിലെ സിനായ് പ്രദേശത്തും മധ്യ ഈജിപ്തിലുള്ളവര്‍ ഉപയോഗിച്ച പ്രാഗ് സിനായ് അക്ഷരമാല ((proto  sinaitic alphabet) രൂപം കൊണ്ട ത് ഹൈരോഗ്ലിഫുകളില്‍ നിന്നാണ്. ഇതില്‍നിന്ന് ക്രിസ്തുവിന് 1050 വര്‍ഷങ്ങള്‍ക്ക് മുമ്പ് ചിത്രാധിഷ്ഠിതമല്ലാത്ത (non-pictographic) ആദ്യത്തെ അക്ഷരമാലയായ ഫിനീഷ്യന്‍ അക്ഷരമാല (phoenician alphabet) രൂപപ്പെട്ടു. ഇതില്‍നിന്ന് ക്രിസ്തുവിന് പത്ത് നൂറ്റാണ്ടുകള്‍ക്ക് മുമ്പ് പുരാതന സെമിറ്റിക്കുകാരുടെ ലിപിയായിരുന്ന പാലിയോ ഹിബ്രു അക്ഷരമാലയും (paleo-hebrew alphabet) അതില്‍നിന്ന് എട്ടാം നൂറ്റാണ്ടായപ്പോ ഴേക്ക് അരാമിക് അക്ഷരമായും (aramic alphabet) രൂപപ്പെട്ടു. (Steven Roger Fischer: History of Writing. London, 2004.) അരാമിക്കില്‍നിന്ന് ക്രി സ്തുവിന് രണ്ടു നൂറ്റാണ്ടുകള്‍ക്കു മുമ്പ് സിറിയക്കും (syriac) അതില്‍ നിന്ന് ഏറെ താമസിയാതെ നെബത്തയന്‍ അക്ഷരമാലയും (nabataen alphabet) രൂപപ്പെട്ടു. നെബത്തയന്‍ അക്ഷരങ്ങളില്‍ നിന്നാണ് അറബി അക്ഷരമാലയുണ്ടായ തെന്നാണ് ഗവേഷകന്‍മാരുടെ പക്ഷം. ക്രിസ്താ ബ്ദം നാലാം നൂറ്റാണ്ടിലായിരിക്കണം അറബി അക്ഷരമാലയുണ്ടായതെന്നും 328ല്‍ രചിക്കപ്പെട്ട നെബത്തിയന്‍മാരുടെ രാജകീയ മരണാന്തര ക്രിയകളെക്കുറിച്ചുള്ള രേഖയിലെ ലിപി അറബിയുടെ പ്രാഗ്‌രൂപമാണെന്നും അവര്‍ അഭിപ്രായപ്പെടുന്നു. ("Arabic alphabet", Brittanica Online www.brittanica.com) എന്നാല്‍ സിറിയയിലെ സെബദില്‍ നിന്ന് കണ്ടെത്തിട്ടുള്ള ഗ്രീക്കിലും സിറിയക്കിലും അറബിയിലുമുള്ള കയ്യെ ഴുത്ത് രേഖയിലേതാണ് ലഭ്യമായതില്‍വെച്ച് ഏറ്റവും പുരാതനമായ അറബി അക്ഷരങ്ങള്‍ എന്നാണ് ഭൂരിപക്ഷം പണ്ഡിതന്‍മാരുടെയും പക്ഷം (Beatrice Gruendler: The Development of the Arabic Scripts From the Nabatean Era to the first Islamic Century according to the Dated Texts, Atlanta, 1993 Page 13-14)

മുഹമ്മദ് നബിലയുടെ കാലത്ത് നിലനിന്നിരുന്ന മൂന്നുതരം ലിപികളായിരുന്നു ഹിജാസി അഥവാ മാഇല്‍, മശ്ഖ്, കുഫീ എന്നീ ലിപികള്‍. ഇവയിലെല്ലാം രചിക്കപ്പെട്ട ആദ്യകാല ക്വുര്‍ആന്‍ കയ്യെഴുത്ത് പ്രതികളുണ്ട്. ലണ്ടനിലെ ബ്രിട്ടീഷ് ലൈബ്രറിയിലുള്ള MS. Or-2165 കയ്യെഴു ത്ത് പ്രതിയും കുവൈത്തിലെ താരിഖ് റജബ് മ്യൂസിയത്തിലുള്ള QUR-1-TSR കയ്യെഴുത്ത് പ്രതിയും പാരീസിലെ ബിബ്‌ളിയോത്തെക്ക് നാഷ ണേലിലുള്ള Arabe 328 Ca കയ്യെഴുത്തു പ്രതിയും സന്‍ആഇലെ ദാറുല്‍ മഖ്ത്തൂത്താത്തിലുള്ള DA MOI-27.1, DAMOI-25.1, DAMOI=29.1 കയ്യെഴു ത്തു പ്രതികളും ഹിജ്‌റ ഒന്നാം നൂറ്റാണ്ടില്‍ ഹിജാസി ലിപിയില്‍ എഴുതപ്പെട്ടവയാണ്. കൈറോയിലെ നാഷണല്‍ ലൈബ്രറിയിലുള്ള ഹിജ്‌റ 107-ല്‍ നിര്‍മ്മിക്കപ്പെട്ടതെന്ന് അതില്‍തന്നെ രേഖയുള്ള കയ്യെഴുത്തു പ്രതി മശ്ഖ് ലിപിയിലുള്ളതാണ്. ഉഥ്മാനീ മുസ്വ്ഹഫുകളായി അറി യപ്പെടുന്നവയും മറ്റ് ഒന്നാം നൂറ്റാണ്ടിലെ കയ്യെഴുത്ത് പ്രതികളുമെല്ലാം കൂഫീ ലിപിയിലാണ് എഴുതപ്പെട്ടിരിക്കുന്നത്. അതുകൊണ്ടുതന്നെ യായിരിക്കണം കൂഫി ലിപിയുണ്ടായത് ഹിജ്‌റ ഒന്നാം നൂറ്റാണ്ടിനു ശേഷമാണെന്ന് സ്ഥാപിക്കുവാന്‍ ഓറിയന്റലിസിറ്റുകളും ക്രിസ്ത്യന്‍ അപ്പോളജറ്റിക്കുകളും ആവേശം കാണിക്കുന്നത്. സ്വഹാബിമാരുടെ കാലത്ത് ഉപയോഗിക്കപ്പെട്ടിരുന്ന മുസ്വ്ഹഫുകളിലൊ ന്നും ഇന്നു ഉപലബ്ധമല്ലെന്നു വന്നാല്‍ അതുപയോഗിച്ച് ക്വുര്‍ആനിന്റെ ചരിത്രപരതയെ ചോദ്യം ചെയ്യാമല്ലോ.

എന്നാല്‍ വസ്തുതകള്‍ അപ്പോളജറ്റിക്കുകള്‍ക്കും അവര്‍ക്കനുസരിച്ച് കാര്യങ്ങള്‍ വളച്ചൊടിച്ചവതരിപ്പിക്കുന്ന ഓറിയന്റലിസ്റ്റുകള്‍ക്കുമെ തിരാണ്. മുഹമ്മദ് നബിലക്കു മുമ്പു തന്നെ പ്രചാരത്തിലുണ്ടായിരുന്ന ലിപികളിലൊന്നായിരുന്നു കൂഫീ ലിപിയുമെന്ന വസ്തുത എന്‍സൈ ക്ലോപീഡിയ ഓഫ് ഇസ്‌ലാം തെളിവുകള്‍ നിരത്തി സമര്‍ഥിക്കുന്നുണ്ട്. ഹിജ്‌റ 17ല്‍ (ക്രിസ്താബ്ദം 638) കൂഫയുണ്ടാവുന്നതിന് നൂറു വര്‍ഷ ങ്ങള്‍ക്കു മുമ്പെങ്കിലും മെസപ്പെട്ടോമിയയില്‍ കൂഫി ലിപി പ്രചാരത്തിലുണ്ടായിരുന്നുവെന്നാണ് പ്രമുഖ ഓറിയന്റലിസ്റ്റും ലിപി വിജ്ഞാ നീയത്തില്‍ അഗ്രഗണ്യനുമായ ബി. മൊറിട്ട്‌സ് വ്യക്തമാക്കുന്നത് (B. Mortiz: "Arabic Writing", Encyclopedia of Islam, London, 1913, Page 387.)-  അന്‍ ബാര്‍, ഹിറ തുടങ്ങിയ മെസെപ്പെട്ടോമിയന്‍ നഗരങ്ങളില്‍ നേരത്തെ പ്രചാരത്തിലുണ്ടായിരുന്ന ലിപിയാണ് കൂഫാ പട്ടണ ത്തിന്റെ രൂപീക രണത്തിനു ശേഷം ചെറിയ മാറ്റങ്ങളോടെ കൂഫി ലിപിയായി അറിയപ്പെട്ടതെന്ന് നാബിയ അബൊട്ടും വിശദീകരിക്കു ന്നുണ്ട്. (Nabia Abbott: The Rise of the North Arabic Script and its Kuranic Development, University of Chicago, 1939 Page 17.)ഗവേഷകരായ അബ്ദുല്‍ കബീര്‍ ഖാത്തിബിയും മുഹമ്മദ് സിജെല്‍മാസ്സിയും കൂടിയെഴുതിയ 'ഇസ്‌ലാമിക് കാലിഗ്രഫിയുടെ യശസ്സ്' എന്ന ഗ്രന്ഥത്തില്‍ എങ്ങനെ യാണ് ഈ നാമകരണമു ണ്ടായതെന്ന് വ്യക്തമാക്കുന്നുണ്ട്. ''ഇസ്‌ലാമിന് മുമ്പ് നിലവിലുണ്ടായിരുന്ന നാല് തരം ലിപികളായിരുന്നു അറബികള്‍ക്ക് പരിചയമുണ്ടാ യിരുന്നത്. ഹിറയില്‍ പ്രചാരത്തിലുണ്ടായിരുന്ന അല്‍ഹിരിയും, അന്‍ബാറിയിലുണ്ടായിരുന്ന അല്‍അന്‍ബാറിയും മക്കയിലുണ്ടായിരുന്ന അല്‍ മക്കിയും മദീനയിലുണ്ടായിരുന്ന ഇബനു മദനിയും. ക്രിസ്താബ്ദം 999ല്‍ (ഹിജ്‌റ 390) മരണപ്പെട്ട, ഫിഹ്‌രിസ്തിന്റെ കര്‍ത്താവ്, പ്രസിദ്ധനായ അല്‍ നദീമാണ് ഹിരയില്‍നിന്ന് രൂപം പ്രാപിച്ച ലിപിക്ക് കൂഫീ'യെന്ന് നാമകരണം ചെയ്തത്. ഹിജ്‌റ 17ല്‍ (ക്രിസ്താബ്ദം 638) നിര്‍മിക്കപ്പെട്ട നഗരമായ കൂഫയിലുണ്ടായതല്ല ഈ ലിപി. കൂഫയുണ്ടാകുന്നതിന് കാലങ്ങള്‍ക്കു മുമ്പേ ഈ ലിപി നിലനിന്നിരുന്നു. എന്നാല്‍ പ്രസ്തുത ലിപിയില്‍നിന്ന് സുന്ദരമായ കാലിഗ്രഫി വളര്‍ത്തിയെടുത്തതും പരിപോഷിപ്പിച്ചതും മഹത്തായ ഈ ബൗദ്ധിക കേന്ദ്രമാണ്''. (Abdel Kabeer Khattibi & Muhammed Sijelmassi: The Splendour of Islamic Calligraphy, Thames & Hudson, 1994, Page 96-97.)

മാത്രവുമല്ല, ഹിജ്‌റ ഒന്നാം നൂറ്റാണ്ടിന്റെ തുടക്കത്തില്‍ നിര്‍മിച്ചതെന്ന് ഉറപ്പിക്കാവുന്ന ക്വുര്‍ആനല്ലാത്ത മറ്റു കൂഫി ലിഖിതങ്ങള്‍ കണ്ടെടു ക്കുകയും ചെയ്തിട്ടുണ്ട്. ഇതിലൊന്നാണ് ഹിജ്‌റ 24ാം വര്‍ഷത്തില്‍ എഴുതിയതാണെന്ന് സ്വയം സാക്ഷ്യം വഹിക്കുന്ന സുഹൈറിന്റെ രേഖ. സഊദി അറേബ്യയിലെ അല്‍ഹിജ്‌റിന് സമീപത്തുള്ള ഖാഅല്‍ മുഅ്തദില്‍ പാറയിലെ ഒരു ചുവന്ന മണല്‍ക്കല്ലില്‍ കൊത്തിവെയ്ക്ക പ്പെട്ട രൂപത്തിലുള്ളതാണീ രേഖ. 'അല്ലാഹുവിന്റെ നാമത്തില്‍, ഞാന്‍ സുഹൈര്‍ ഇതെഴുതുന്നത്. ഇരുപത്തിനാലാം വര്‍ഷത്തില്‍ ഉമര്‍ മര ണപ്പെട്ടപ്പോഴാണ്' (ബിസ്മില്ലാ, അന സുഹൈര്‍ കതബ്തു സമന്‍ തുവഫ്ഫീ ഉമറ സനത്ത അര്‍ബഅ വ ഇശ്‌രീന്‍) എന്നാണ് രേഖയിലു ള്ളത്. (Ali Ibrahim Al-Ghabban (Trans: Robert HolyLand): "The Inscription of Zuhayr, the oldest Islamic Inscription (24Ah/AD644-645), the Rise of the Arabic Scri pt and the nature of the early Islamic State". "Arabian Archeology and Epigraphy, November 2008, Vol 19, Issue 2, Page 210-237.)

കുത്തുകളുള്ള കൂഫി ലിപിയില്‍ എഴുതപ്പെട്ട ഈ രേഖ ഉമര്‍ (റ) മരണപ്പെട്ട കാലത്ത് കൂഫി ലിപിയിലുള്ള എഴുത്തിന് പ്രചാരമുണ്ടായിരുന്നുവെന്ന് സുതരാം വ്യക്തമാക്കുന്നുണ്ട്. ഉമര്‍നേു ശേഷം ഭരിച്ചയാളാണ് ഉഥ്മാന്‍.(റ) ഉമറിെ(റ)ന്റ കാലത്തുതന്നെ കൂഫീ ലിപി പ്രചാരത്തിലുണ്ടെ ങ്കില്‍ ഉഥ്മാ നിെ(റ)ന്റ കാലത്ത് നിര്‍മിക്കപ്പെട്ട മുസ്വ്ഹഫുകള്‍ കൂഫി ലിപിയിലായത് സ്വാഭാവികമാണെന്ന് ആര്‍ക്കും സമ്മതിക്കേണ്ടി വരും. യുനെ സ്‌കോയുടെ ലോകസ്മരണകള്‍ രേഖകളില്‍ (http://portal.unesco.org/enev.php_URL_ID= 14264&URL_DO=DO_TOPIC& URL_ SECTION= 201.html) സ്ഥലം പിടിച്ചിട്ടുള്ള ഈ മണല്‍ക്കല്‍ ലിഖിതത്തെ നിഷേധിക്കുവാന്‍ ആര്‍ക്കും കഴിയില്ല. ക്വുര്‍ആന്‍ ചോദ്യത്തെ പരിഹരിക്കാനുതകുന്ന ആദ്യത്തെ ഇസ്‌ലാമികാലേഖനം' (First Islamic Inscription may solve Qur'an Question) എന്ന തലക്കെട്ടിലാണ് ഡിസ്‌കവറി ചാനലിന്റെ ന്യൂസ് ഇതേക്കുറിച്ച ലേഖനം പ്രസിദ്ധീകരിച്ചിരിക്കുന്നത്. (dsc.discovery.com/news/2008/11/18/islamic-inscription . html) ഉഥ്മാനീ മുസ്ഹഫുകള്‍ എഴു തപ്പെട്ടത് കൂഫി ലിപിയിലാണ് എന്നതിന്റെ പേരില്‍ ക്വുര്‍ആനിന്റെ ചരിത്രപരതയെ ചോദ്യം ചെയ്യാന്‍ ശ്രമിച്ചവരുടെ ഗവേഷണദംഷ്ട്രങ്ങ ള്‍ പൊഴിക്കുവാന്‍ പര്യാപ്തമാണ് ഈ ചുവന്ന കല്‍രേഖയുടെ കണ്ടുപിടുത്തമെന്നര്‍ഥം.

കൈറോ മ്യൂസിയം ഓഫ് അറബ് ആര്‍ട്ടില്‍ സൂക്ഷിച്ചിരിക്കുന്ന ഹിജ്‌റ 31ല്‍ രചിച്ചതായി സ്വയം സാക്ഷ്യപ്പെടുത്തുന്ന അബ്ദുര്‍റഹ്മാന്‍ ഇബ്‌ നുഖൈര്‍ അല്‍ഹാജിരിയുടെ ഖബര്‍ ഫലകവും (Nabia Abbott: The Rise of the North Arabic Script and its Kur'anic Development, University of Chicago, 1939 Page 18-19.) ഹിജ്‌റ 40ല്‍ എഴുതിയതായി സാക്ഷ്യപ്പെടുത്തുന്ന സഊദി അറേബ്യയിലെ ദര്‍ബ് സുബൈദയിലെ വാദി അല്‍ ശാമിയയിലെ ഒട്ടകപാതയില്‍നിന്ന് 1970ല്‍ ലഭിച്ച ശിലാഫലകവും (A.H.Sharafaddin: "Some Islamic Inscriptions Discovered on the Darb Zubayda", Atlal (The Journ al of Saudi Arabian Archeology, 1997, Vol 1, Page 69-70.) സഊദി അറേബ്യയിലെ വാദിസബീലില്‍നിന്ന് ലഭിച്ച ഹിജ്‌റ 46ല്‍ രചിച്ചതായി സാക്ഷ്യ പ്പെടുത്തുന്ന ചുമര്‍ഫലകവുമെല്ലാം (Beatrice Greundler: Opt it Page 15) കൂഫീലിപിയില്‍ എഴുതപ്പെട്ടവയാണ്. ഇതെല്ലാം സഹാബിമാരുടെ കാലത്ത് കൂഫി ലിപിയിലുള്ള രചനകള്‍ക്ക് പ്രചാരമുണ്ടായിരുന്നുവെന്ന് വ്യക്തമാക്കുന്നുണ്ട്.

ണ്ട്. മുഹമ്മദ് നബി(സ)യുടെ അനുയായികള്‍ ഉപയോഗിച്ച ക്വുര്‍ആന്‍ കോപികൾ സംരക്ഷിക്കപെട്ടിട്ടുണ്ട്. ലോകത്തിന്റെ പല ഭാഗങ്ങളിലുമുള്ള മ്യൂസിയങ്ങളിൽ അവ കാണാൻ കഴിയും.

സ്വഹാബിമാരുടെ മുസ്ഹഫുകളോ അവയുടെ ഭാഗങ്ങളോ ഇന്ന് ഉപലബ്ധമല്ലെന്ന ഓറിയന്റലിസ്റ്റുകളില്‍ ചിലരുടെയും അവരുടെ വാദങ്ങള്‍ കോപ്പിയടിച്ച് ഇസ്‌ലാം  വിമര്‍ശനം നടത്തുന്ന ക്രൈസ്തവ മിഷനറിമാരുടെയും വാദം അടിസ്ഥാന രഹിതമാണെന്ന് പ്രമുഖ ഓറിയന്റലിസ്റ്റായ നബിയ എബൊട്ട് തന്റെ ക്വുര്‍ആന്‍ കയ്യെഴുത്ത് രേഖകളെക്കുറിച്ച പഠനത്തില്‍ വ്യക്തമാക്കിയിട്ടുണ്ട്. (Nabia Abbott: The Rise of the North Arabic Script and its Kur'anic Development, with a full description of the Kura'n manuscripts in the Oriental Institute, Chicago, 1939)

ഹിജ്‌റ ഒന്നാം നൂറ്റാണ്ടിന്റെ അവസാന പകുതിയില്‍ രചിക്കപ്പെട്ടതു മുതല്‍ക്കുള്ള ക്വുര്‍ ആന്‍ കയ്യെഴുത്തു പ്രതികളെക്കുറിച്ച വിവരങ്ങള്‍ അവരുടെ പഠനത്തിലുണ്ട്. പ്രസ്തുത നൂറ്റാണ്ടിന്റെ അവസാനംവരെ അറേബ്യന്‍ ഉപദ്വീപിന്റെ വിവിധ ഭാഗങ്ങളിലായി പ്രവാചകാനുചരന്‍മാര്‍ ജീവിച്ചിരുന്നിട്ടുണ്ട്. ഹിജ്‌റ 96-ാം വര്‍ഷത്തില്‍ തന്റെ നൂറാമത്തെ വയസ്സില്‍ മരണപ്പെട്ട സഹലുബ്‌നു സഅദിനെ മദീനയില്‍ ജീവിച്ച അവസാനത്തെ സഹാബിയായും ഹിജ്‌റ 93-ല്‍ തന്റെ 103-ാമത്തെ വയസ്സില്‍ മരണപ്പെട്ട അനസുബ്‌നു മാലിക്കിനെ ബസ്വറയില്‍ ജീവിച്ച അവസാനത്തെ സ്വഹാബിയായും ഹിജ്‌റ 110-ല്‍ മരണപ്പെട്ട അബൂ തുഫൈല്‍ അമിറുബിനു വാഥിലയ്യെ മക്കയില്‍ ജീവിച്ച അവസാനത്തെ സ്വഹാബിയായും കണക്കാക്കപ്പെടുന്നു. ഒന്നാം നൂറ്റാണ്ടിന്റെ അവസാന പകുതിയില്‍ മുസ്‌ലിം ലോകത്തിന്റെ വ്യത്യസ്ത ഭാഗങ്ങളില്‍ ജീവിച്ച സ്വഹാബിമാര്‍ പഠന-പാരായണങ്ങള്‍ക്ക് ഉപയോഗിച്ച താവണം നബിയ എബൊട്ട് രേഖപ്പെടുത്തുന്ന കയ്യെഴുത്ത് പ്രതികളെന്ന് മനസ്സിലാക്കാവുന്നതാണ്.

a) ഉഥ്മാനിന്റെ (റ)കാലത്തെ കയ്യെഴുത്ത് പ്രതികള്‍:

തന്റെ ഭരണകാലത്ത് ഉഥ്മാനിന്റെ (റ) നിര്‍ദ്ദേശപ്രകാരം തയ്യാറാക്കിയതെന്ന് കരുതപ്പെടുന്ന ആറ് കയ്യെഴുത്തു പ്രതികള്‍ പൂര്‍ണമായോ ഭാഗികമായോ ലോകത്തിന്റെ വ്യത്യസ്ത ഭാഗങ്ങളിലായി ഇന്ന് സൂക്ഷിക്കപ്പെട്ടിട്ടുണ്ട്. ഇവ ഉഥ്മാനിന്റെ  (റ)കാലത്ത് പകര്‍ത്തിയെ ഴുതപ്പെട്ട ഏഴു പ്രതികളില്‍പ്പെട്ടവ തന്നെയാണോയെന്ന കാര്യത്തില്‍ വിദഗ്ധര്‍ക്കിടയില്‍ അഭിപ്രായ വ്യത്യാസമുണ്ട്. ഉഥ്മാനീ കയ്യെഴുത്ത് പ്രതികളായി അറിയപ്പെടുന്നവ താഴെ പറയുന്നവയാണ്.

1) ഉസ്‌ബെക്കിസ്താനിലെ താഷ്‌കന്റിലുള്ള കയ്യെഴുത്ത് പ്രതി:

ആകെ 360 പുറങ്ങളുള്ളതില്‍ 69 എണ്ണം പൂര്‍ണമായിത്തന്നെ കീറിപ്പോയ രീതിയിലാണ് ഈ പ്രതി ഇപ്പോഴുള്ളത്. താഷ്‌കന്റിലെ തെല്യാ ശൈഖ് മസ്ജിദിലുള്ള ഹസ്ത് ഇമാം ഗ്രന്ഥാലയത്തില്‍ സൂക്ഷിച്ചിരിക്കുന്ന ഇതില്‍ കാര്യമായ കേടുപാടുകളൊന്നുമില്ലാത്ത പതിനഞ്ച് പുറങ്ങളൊഴിച്ച് ബാക്കിയെല്ലാം കീറിയതോ പേജിന്റെ കുറച്ചുഭാഗംമാത്രം അവശേഷിക്കുന്നതോ ആയ രൂപത്തിലാണുള്ളത്. ഇതില്‍ ഉപയോഗിച്ചിരിക്കുന്ന എഴുത്ത് രീതിയെക്കുറിച്ച് പഠിച്ച ഗവേഷകന്‍മാര്‍ ഹിജ്‌റ രണ്ടാം നൂറ്റാണ്ടിലായിരിക്കണം ഇതിന്റെ രചന നടന്നതെന്നാണ് അഭിപ്രായപ്പെട്ടിരിക്കുന്നത്. (A. Jeffery & I. Mandelsohn: `The Orthography of the Samarqand Qu'ran codex', Journal of the American Oriental Society 1942, Volume 7,  P.65.) എന്നാല്‍ ഓക്‌സ്‌ഫോര്‍ഡില്‍ വെച്ചു നടന്ന കാര്‍ബണ്‍ ഡേറ്റിംഗ് പരീക്ഷണത്തില്‍ ഇത് നേരത്തെ രചിച്ചതാകാന്‍ സാധ്യതയുണ്ടെന്നാണ് തെളിഞ്ഞിരിക്കുന്നത്. ക്രിസ്താബ്ദം 595നും 855നുമിടയില്‍ രചിക്കപ്പെട്ടതാകാന്‍ 95% സാധ്യതയും ക്രിസ്താബ്ദം 640 നും 765നുമിടയില്‍ രചിക്കപ്പെട്ടതാകാന്‍ 68% സാധ്യതയുമുണ്ടെന്നാണ് കാര്‍ബണ്‍ ഡേറ്റിംഗ് പരീക്ഷണം വ്യക്തമാക്കിയത്. (F. Deroehe: 'Mannscripts of the Qu'ran' in J.D. Mc Auliffel (Ed.)Encyclopaedia of the Qu'ran; Leiden & Boston, 2003, volume 3, page 261) ഉഥ്മാനി്യന്റെ കാലത്ത് നിര്‍മിക്കപ്പെട്ട കോപ്പികളിലൊന്നാണ് ഇതെന്ന പാരമ്പര്യം ശരിയാവാന്‍ സാധ്യതയുണ്ടെന്നാണ് കാര്‍ബണ്‍ഡേറ്റിംഗ് പരീക്ഷണം വ്യക്തമാക്കുന്നതെന്നര്‍ഥം.

2) തൂര്‍ക്കിയില്‍ ഇസ്തംബൂളിലെ തോപ്കാപി മ്യൂസിയത്തിലുള്ള കയ്യെഴുത്ത് പ്രതി:

408 പുറങ്ങളിലായി നിലനില്‍ക്കുന്ന ഈ കയ്യെഴുത്ത് പ്രതിയില്‍ ക്വുര്‍ആനിന്റെ 93 ശതമാനവും രേഖപ്പെടുത്തപ്പെട്ടിട്ടുണ്ട്. രണ്ട് പുറങ്ങള്‍ മാത്രമെ നഷ്ടപ്പെട്ടിട്ടുള്ളൂ. ഉഥ്മാന്റെ (റ) കാലത്ത് രചിക്കപ്പെട്ടതാണ് ഇത് എന്ന പാരമ്പര്യത്തിനെതിരാണ് ഇതിലുപയോഗിച്ച രചനാരീതി യെന്നാണ് ഗവേഷകന്‍മാരുടെ പക്ഷം. എന്നാല്‍ ഹിജ്‌റ ഒന്നാം നൂറ്റാണ്ടിന്റെ അവസാനത്തിലോ രണ്ടാം നൂറ്റാണ്ടിന്റെ തുടക്കത്തിലോ ഇത് രചിക്കപ്പെട്ടിട്ടുണ്ടെന്ന കാര്യത്തില്‍ സംശയമില്ല. (T. Altikulac: Al Mushaf Alsharif: Attribured Uthman Bin Affan. (The copy at the Topkapi Palace Meuseum), Isthambul, 2007)

3) തുര്‍ക്കിയിലെ ഇസ്താംബൂളിലുള്ള തുര്‍ക്കിഷ് ആന്റ് ഇസ്‌ലാമിക് ആര്‍ട്ട് മ്യൂസിയത്തിലുള്ള കയ്യെഴുത്ത് പ്രതി:

439 പുറങ്ങളുള്ള ഈ കയ്യെഴുത്തു പ്രതിയുടെ 17 പുറങ്ങള്‍ നഷ്ടപ്പെട്ടിട്ടുണ്ട്. ഇങ്ങനെ നഷ്ടപ്പെട്ടവയില്‍ മൂന്നെണ്ണമൊഴിച്ച് ബാക്കിയെല്ലാം പില്‍ക്കാലത്ത് കണ്ടെടുക്കപ്പെടുകയും സൂക്ഷിക്കുകയും ചെയ്തിട്ടുണ്ട്. ഇതിന്റെ അവസാനത്തെ പുറത്തില്‍ 'കത്തബഹു ഉഥ്മാനുബ്‌നു അഫ്ഫാന്‍ ഫീ  സനത്ത് ഥലാഥീന്‍'' (ഉഥ്മാനുബ്‌നു അഫ്ഫാന്‍ മുപ്പതാം വര്‍ഷത്തില്‍ എഴുതിയത്) എന്ന് രേഖപ്പെടുത്തപ്പെട്ടിട്ടുണ്ടെങ്കിലും അത് വ്യാജമാണെന്നും ഹിജ്‌റ ഒന്നാം നൂറ്റാണ്ടിന്റെ അവസാനത്തിലോ രണ്ടാം നൂറ്റാണ്ടിന്റെ ആദ്യത്തിലോ ആയിരിക്കണം ഇത് എഴുതപ്പെട്ടതെന്നുമാണ് ഗവേഷകന്‍മാരുടെ പക്ഷം. (S. Sahin: The Meuseum of Turkish and Islamic Arts. Thirteen centuries of Glory from the Umayyads to the Ottomans, New York, 2009 page 23-25)

4) റഷ്യയിലെ സെന്റ്പീറ്റേഴ്‌സ് ബര്‍ഗിലും കട്ടാലന്‍ഗാറിലും ബുഖാറയിലും താഷ്‌കന്റിലുമായി സൂക്ഷിക്കപ്പെട്ടിരിക്കുന്ന കയ്യെഴുത്ത് പ്രതി:

ആകെ 97 പുറങ്ങളുള്ളതില്‍ 81 എണ്ണം സെന്റ് പിറ്റേഴ്‌സ് ബര്‍ഗിലെ ഓറിയന്റല്‍ സ്റ്റഡീസിലും ഒരെണ്ണം താഷ്‌കന്റിലെ ബിദുനി ഇന്‍സ്റ്റി റ്റ്യൂട്ട് ഓഫ് ഓറിയന്റല്‍ സ്റ്റഡീസിലും പന്ത്രെണ്ടണ്ണം ഉസ്‌ബെക്കിസ്താനിലുള്ള കട്ടാലന്‍ഗാറിലെ ഇഷ്ഖിയ്യാ സില്‍സിലയിലുള്ള ശൈഖു മാരുടെ മഖ്ബറകള്‍ക്കനുബന്ധമായും രണ്ടെണ്ണം ബുഖാറയിലുള്ള ഇബ്‌നുസീന ബുഖാറ റീജ്യനല്‍ ലൈബ്രറിയിലും ഒരെണ്ണം താഷ്‌കന്റി ലുള്ള ലൈബ്രറി ഓഫ് അഡ്മിനിസ്‌ട്രേഷന്‍ ഓഫ് മുസ്‌ലിം അഫയേഴ്‌സിലുമായാണ് ഈ കയ്യെഴുത്ത് പ്രതി സൂക്ഷിച്ചിട്ടുള്ളത്. ഇത് ഉഥ്മാ നിന്റെ കയ്യെഴുത്തുപ്രതിയാണെന്ന പാരമ്പര്യത്തിന് അടിത്തറയില്ലെന്നും ഹിജ്‌റ രണ്ടാം നൂറ്റാണ്ടിന്റെ തുടക്കത്തിലായിരിക്കണം ഇത് രചിക്കപ്പെട്ടത് എന്നുമാണ് ഗവേഷകന്‍മാരുടെ അഭിപ്രായം. (E.A. Rezvan: The Qur'an of Uthman (Stopete-rsburg, Katta-langar Bukhara, Tashk-ant), St. Petersburg, 2004, Vol-1; page 69-70)  ഇതിന്റെ കാര്‍ബണ്‍ ഡേറ്റിംഗ് പരീക്ഷണവും ഇക്കാര്യം തന്നെയാണ് വ്യക്തമാക്കിയിരിക്കുന്നത്.

5) ഈജിപ്തില്‍ കൈറോയിലെ അല്‍ഹുസൈന്‍ മസ്ജിദിലുള്ള കയ്യെഴുത്ത് പ്രതി:

1087 പുറങ്ങളുള്ള ഈ കയ്യെഴുത്തു പ്രതിയിലെ നാല് പുറങ്ങള്‍ മാത്രമാണ് നഷ്ടപ്പെട്ടിട്ടുള്ളത്. ക്വുര്‍ആനിന്റെ 99 ശതമാനവും അതേപോ ലെത്തന്നെ രേഖപ്പെടുത്തപ്പെട്ടിട്ടുള്ള ഈ 1087 പുറങ്ങളുടെ മൊത്തം ഉയരം 40 സെന്റീമീറ്ററും ഭാരം 80 കിലോഗ്രാമുമാണ്. ഉഥ്മാന്‍(റ) ന്റെ കാലത്ത് മദീനയില്‍ നിന്ന് സിറിയയിലേക്ക് കൊടുത്തയച്ച മുസ്ഹഫ് ആണ് ഇതെന്ന പരമ്പരാഗത വിശ്വാസം ശരിയല്ലെന്ന് അഭിപ്രായപ്പെട്ട ഗവേഷകന്‍മാരുണ്ട്. അമവീ ഖലീഫയായിരുന്ന അബ്ദുല്‍ മലിക്കിബ്‌നു മര്‍വാനിന്റെ സഹോദരനും ഈജിപ്തിലെ ഗവര്‍ണറുമായിരുന്ന അബ്ദുല്‍ അസീസിബ്‌നു മര്‍വാനിന്റെ നിര്‍ദേശപ്രകാരം രചിക്കപ്പെട്ടതാണ് ഇതെന്നാണ് അവരുടെ അഭിപ്രായം. (B. Moritz (Ed): Arabic Paleography: A collection of Arabic texts from the first century of  Hijra till the ye-ar 1000, Khedivial library, Cairo, 1905, plates 13-16.) ഹിജ്‌റ ഒന്നാം നൂറ്റാണ്ടിന്റെ അന്ത്യത്തിലോ രണ്ടാം നൂറ്റാണ്ടിന്റെ തുടക്കത്തിലോ ആയിരിക്കും ഇത് നിര്‍മിക്കപ്പെട്ടതെന്ന് സാരം. (സലാഹുദ് ദീന്‍ അല്‍ മുനജ്ജിദ്: ദിറാസത്ത് ഫീ താരീഖല്‍ ഖത്തല്‍ അറബീ മുന്‍ദു ബിദായത്തി ഇലാ നിഹായത്തല്‍ അസ്ര്‍ അല്‍ ഉമവി, ബെയ്‌റൂത്ത്, 1972, പുറം 53-54)

6) കൈറോയിലെ ഈജിപ്ഷ്യന്‍ നാഷണല്‍ ലൈബ്രറി (ദാറുല്‍ കുത്തുബില്‍ മിസ്‌രിയ്യ) യിലുള്ള കയ്യെഴുത്ത് പ്രതി.

അറുന്നൂറോളം പുറങ്ങളുണ്ടായിരുന്ന ഈ മുസ്ഹഫിന്റെ 306 പുറങ്ങളാണ് ഇന്ന് ലഭ്യമായിട്ടുള്ളത്. ഇതില്‍ 248 എണ്ണം ഈജിപ്ഷ്യന്‍ നാഷണല്‍ ലൈബ്രറിയിലും 46 എണ്ണം പാരീസിലെ ബിബ്‌ളിയോണിക് നാഷണേലിലും പന്ത്രണ്ടെണ്ണം ജര്‍മനിയിലെ ഗോഥയിലുള്ള മ്യൂസിയ ത്തിലുമാണ് സൂക്ഷിച്ചിരിക്കുന്നത്. ഉഥ്മാനി്യന്റെ കാലത്ത് നിര്‍മ്മിക്കപ്പെട്ടതാണെന്ന പരമ്പരാഗത വിശ്വാസത്തിന്  ഉപോല്‍ബലകമായ തെളിവുകളില്ലെങ്കിലും ഹിജ്‌റ ഒന്നാം നൂറ്റാണ്ടില്‍ തന്നെയായിരിക്കണം ഇത് രചിക്കപ്പെട്ടതെന്നാണ് ഗവേഷകന്‍മാരുടെ പൊതുവായ അഭി പ്രായം (M.H. Hussein (Trans: D. Jaeschke & D. Sharp): Origins of the Book: Egypt's contribution to the development of the book from papyrus to codex, Green wich, 1972 page 91-130) ഇതോടൊപ്പമുള്ള ഒരു കയ്യെഴുത്ത് പുറത്തെ കാര്‍ബണ്‍ഡേറ്റിംഗ് പരിശോധനയ്ക്ക് വിധേയമാക്കിയപ്പോള്‍ ക്രി സ്താബ്ദം 609നും 694നുമിടയില്‍ രചിക്കപ്പെട്ടതായിരിക്കുവാന്‍ 95.2% സാധ്യതയുണ്ടെന്നാണ് മനസ്സിലായത്. (Yasin Dutton: "An Umayyad Fragment of the Qu'ran and its Dating", Journal of Qur'anic studies, Vol. 9, No: 2, 2007, Page 76)ഉഥ്മാന്‍്യ പ്രവിശ്യാതലസ്ഥാനങ്ങളിലേക്ക് അയച്ച തോ അതല്ലെങ്കില്‍ അതിനടുത്തകാലത്തെപ്പോഴോ രചിക്കപ്പെട്ടതോ ആയ മുസ്ഹഫിന്റെ ഭാഗങ്ങളാണ് ഇവയെന്ന് ഇത് വ്യക്തമാക്കുന്നു.

b) പുതുതായി സന്‍ആഇല്‍ നിന്ന് കണ്ടെത്തിയ കയ്യെഴുത്തു പ്രതികള്‍:

1972-ല്‍ യമനിന്റെ തലസ്ഥാനമായ സന്‍ആഇലെ പുരാതനമായ പള്ളി പുതുക്കി പണിയുന്നതിനിടയ്ക്ക് പുരാതനമായ ചില കയ്യെഴുത്തു രേഖകള്‍ കണ്ടെത്തുകയുണ്ടായി. ഹിജ്‌റ ആറാം വര്‍ഷത്തില്‍ പ്രവാചകന്റെ നിര്‍ദേശമനുസരിച്ച് അദ്ദേഹത്തിന്റെ അനുചരന്‍മാരില്‍ ഒരാള്‍ നിര്‍മിച്ചതാണ് ഈ പള്ളി. അവിടെ നിന്ന് കണ്ടെടുത്ത കയ്യെഴുത്തു രേഖകളെ ഉള്‍ക്കൊള്ളിച്ചുകൊണ്ട് യുനെസ്‌കോ ഒരു സി.ഡി. പുറത്തിറക്കിയിട്ടുണ്ട്. ഐക്യരാഷ്ട്രസഭയുടെ ശാഖയായ യുനെസ്‌കോയുടെ 'ലോകസ്മരണകള്‍' (Memory of the World) എന്ന പ്രോഗ്രാമി ന്റെ ഭാഗമായാണ് ഈ സി.ഡി. റോം പുറത്തിറക്കിയിരിക്കുന്നത്. (www. unesco.org/webworld/mdn/visite/sanaa/en/present.html)

ഈ കയ്യെഴുത്തു രേഖകളില്‍ ചിലതില്‍ ക്വുര്‍ആനാണ് രേഖപ്പെടുത്തപ്പെട്ടിട്ടുള്ളത്. ഈ ക്വുര്‍ആന്‍ കയ്യെഴുത്ത് രേഖകളില്‍ മിക്കതും ഹിജ്‌റ ഒന്നാം നൂറ്റാണ്ടിലേതാണെന്ന് അതിലെ എഴുത്തുരീതി വ്യക്തമാക്കുന്നുണ്ട്. (Abdul Thaher: "Quering the Qu'ran" The Guardian, 08-08-2000) കാര്‍ബര്‍ ഡേറ്റിംഗ് പരീക്ഷണങ്ങള്‍ കയ്യെഴുത്ത് രേഖകളില്‍ ചിലതെല്ലാം ക്രിസ്താബ്ദം 645 നും 690 നുമിടയില്‍  രചിക്കപ്പെട്ടതാണെന്നാണ് മനസ്സിലാക്കിത്തരുന്നത്. (Carole Hillenbrand: The New Cambridge Medieval History, Cambridge, 2005, vol. 1 Page 330) ഈ രേഖകളിലുള്ള ക്വുര്‍ആന്‍ വചനങ്ങള്‍ സാമ്പ്രദായിക രീതിയിലുള്ളവയല്ലെന്നും ഉഥ്മാന്‍്യകോപ്പിയെടുത്ത് പ്രവിശ്യാതലസ്ഥാനങ്ങളിലേക്ക് അയച്ച മുസ്ഹഫുകളില്‍നിന്ന് ഒരു മാറ്റവും ക്വുര്‍ആനിലുണ്ടായിട്ടില്ലെന്ന മുസ്‌ലിം അവകാശവാദത്തെ ഈ രേഖകളെക്കുറിച്ച പഠനം കടപുഴക്കു മെന്നുമുള്ള അവകാശവാദങ്ങള്‍ ഈ കയ്യെഴുത്ത് രേഖകള്‍ കണ്ടെടുത്ത കാലം മുതല്‍ തന്നെ ആരംഭിച്ചിരുന്നു. (Toby Lester: "What is Qu'ran", The Atlantic Monthly, January 1999)അങ്ങനെ ആഗ്രഹിച്ചു കൊണ്ടുതന്നെയായിരിക്കണം ജര്‍മനിയിലെ അറബി കാലിഗ്രഫി-ക്വുര്‍ആനിക് പാലിയോഗ്രഫി വിദഗ്ധനായ ജെര്‍ഡ് ആര്‍ വുയിനിന്റെ നേതൃത്വത്തിലുള്ള ഒരു സംഘം ഈ കയ്യെഴുത്തു രേഖകള്‍ പരിശോധിക്കുവാന്‍ മുന്നോട്ടു വന്നത്. എന്നാല്‍ പുരാതനമായ പല കയ്യെഴുത്തു രേഖകളിലും ഉപയോഗിക്കപ്പെട്ടിരിക്കുന്ന കൂഫി എഴുത്ത് രീതിയിലല്ല, അതിനേക്കാള്‍ പഴയ ഹിജാസീ എഴുത്ത് രീതിയിലാണ് ഇതിലെ ക്വുര്‍ആന്‍ വചനങ്ങള്‍ എഴുതിയത് എന്നതല്ലാതെ നിലവില്‍ പ്രചാരത്തി ലുള്ള ക്വുര്‍ആനില്‍നിന്ന് യാതൊരു വ്യത്യാസവും കണ്ടെത്താന്‍ ഇതുവരെയുള്ള ഗവേഷണങ്ങള്‍ക്കൊന്നും കഴിഞ്ഞിട്ടില്ല. ("Sana'a Manuscrips: Uncovering a Treasure of words" Unesco courier, 02-12-2008)

ഹിജ്‌റ ഒന്നാം നൂറ്റാണ്ടിന്റെ  ആദ്യ പകുതിയില്‍ നിര്‍മിക്കപ്പെട്ട മുസ്ഹഫിന്റെ ഭാഗങ്ങളാണ് സന്‍ആയില്‍നിന്ന് ലഭിച്ചതെന്ന കാര്‍ബണ്‍  ഡേറ്റിംഗ് പരിശോധനാ ഫലത്തിന്റെ വെളിച്ചത്തില്‍ ഇന്ന് നിലവിലുള്ള ക്വുര്‍ആന്‍ കയ്യെഴുത്തു പ്രതികളില്‍ ഏറ്റവും പഴയതാണിതെന്ന് അഭിപ്രായപ്പെട്ടവരുണ്ട്. (Abdul Taher: "Quareling the Qu'ran", The Guardian, 8-8-2000 )ചില ഗവേഷകന്‍മാരുടെ അഭിപ്രായത്തില്‍ ക്വുര്‍ആനിന്റെ ഈ കയ്യെഴുത്ത് രേഖകളില്‍ ചിലത് മുഹമ്മദ് നബി(സ)യുടെ മരണത്തിനുശേഷം പതിനഞ്ച് വര്‍ഷങ്ങള്‍ക്കകം രചിക്കപ്പെട്ടവയാണ്. ഏതായിരുന്നാലും സ്വഹാബിമാരില്‍ ആരുടെയെ ങ്കിലും കൈവശമുണ്ടായിരുന്ന കയ്യെഴുത്ത് പ്രതിയുടെ അവശിഷ്ടങ്ങളാണ് ഇവയില്‍ ചിലതെന്ന കാര്യത്തില്‍ ഗവേഷകന്‍മാര്‍ക്കിടയില്‍ അഭിപ്രായവ്യത്യാസമില്ല. (B. Sadeghi & U. Bergmann: "The Codex of a Companion of the Prophet and the Qu'ran of the Prophet",  Arabica, 2010, Volume 57 page 344-354)

സ്വഹാബിമാരുടെ കാലം മുതല്‍തന്നെ ക്വുര്‍ആന്‍ രേഖപ്പെടുത്തി സൂക്ഷിക്കുന്ന സമ്പ്രദായം നിലനിന്നിരുന്നുവെന്ന വസ്തുത ഈ രേഖകള്‍ അര്‍തഥശങ്കക്കിടയില്ലാത്ത വിധം വ്യക്തമാക്കുന്നുണ്ട്. സ്വഹാബിമാർ ഉപയോഗിച്ച ഖുർആൻ പ്രതികളൊന്നും ഇന്ന് നിലവിലില്ലെന്ന വാദത്തെ തകർക്കുന്നതാണ് ഈ കയ്യെഴുത്ത് രേഖകൾ.

തെ. മൂന്നാം ഖലീഫയായ ഉഥ്മാനുബ്‌നു അഫ്ഫാനിന്റെ കാലത്ത് ക്രോഡീകരിക്കപ്പെട്ട മുസ്ഹഫുകളുടെ തനിപകർപ്പുകളാണ് ലോകത്തെങ്ങുമുള്ള മുസ്ലിംകൾ ഇന്നുപയോഗിക്കുന്നത്. അഥവാ ഇന്ന് കാണുന്ന രൂപത്തില്‍ ക്വുര്‍ആന്‍ ക്രോഡീകരിക്കുകയും ശേഖരിക്കുകയും ചെയ്തത് ഉഥ്മാനിന്റെ (റ) ഭരണകാലത്താണ്. പന്ത്രണ്ട് വര്‍ഷത്തിലധികം നീണ്ടുനിന്ന തന്റെ ഭരണകാലത്തി നിടയില്‍ ഇസ്‌ലാമിക പ്രബോധനം വ്യാപകമാവുകയും ഇസ്‌ലാമിക രാഷ്ട്രത്തിന്റെ വിസ്തൃതി വര്‍ധിക്കുകയും ചെയ്തപ്പോള്‍ വ്യത്യ സ്ത പ്രദേശങ്ങളിലെ പുതുമുസ്‌ലിംകള്‍ക്ക് ക്വുര്‍ആന്‍ പഠിപ്പിക്കുന്നതിന് വേണ്ടി കൂടുതല്‍ സംവിധാനങ്ങളേര്‍പ്പെടുത്തേണ്ടിവന്നത് സ്വാഭാവികമായിരുന്നു. അങ്ങനെ ഏര്‍പെടുത്തിയ സംവിധാനങ്ങളുടെ ഭാഗമായാണ് വിശുദ്ധ ക്വുര്‍ആനിന്റെ ഔദ്യോഗിക പ്രതികള്‍ പകര്‍ത്തിയെഴുതുവാനും അത് ഇസ്‌ലാമിക രാഷ്ട്രത്തിന്റെ പ്രവിശ്യാ തലസ്ഥാനങ്ങളിലേക്കെല്ലാം കൊടുത്തയച്ച് അവിടെയെല്ലാം തെറ്റുകൂടാതെ, അവതരിക്കപ്പെട്ട രൂപത്തില്‍ തന്നെയാണ് ക്വുര്‍ആന്‍ പാരായണം ചെയ്യുന്നതെന്ന് ഉറപ്പുവരുത്തുവാനും, ആദ്യത്തെ നാല് ഖലീഫമാരില്‍ ഏറ്റവുമധികം കാലം രാഷ്ട്രഭരണം നടത്തിയ വ്യക്തിയായ ഉഥ്മാനിന്(റ) അവസരമുണ്ടായത്.

ക്വുര്‍ആന്‍ കോപ്പികളുടെ ഔദ്യോഗിക പ്രസാധനത്തിലേക്ക് നയിച്ച കാര്യങ്ങളെപ്പറ്റി പ്രവാചകാനുചരന്മാരില്‍ പ്രമുഖനായ അനസ് ബ്‌നു മാലിക്‌ (റ)പറയുന്നത് ഇങ്ങനെയാണ്:‘'ശാമിലെയും ഇറാഖിലെയും ജനങ്ങള്‍ അര്‍മീനിയക്കും അസര്‍ബൈജാനിനും വേണ്ടി യുദ്ധം നടത്തുന്നതിനിടെ ഹുദൈഫ ത്ത്ബ്‌നുല്‍ യമാന്‍ (റ) ഉഥ്മാനിനെ(റ)സന്ദര്‍ശിച്ചു. ക്വുര്‍ആന്‍ പാരായണത്തിലുള്ള ശാമുകാരുടെയും ഇറാഖുകാരുടെയും വ്യതിരിക്ത തകളില്‍ അദ്ദേഹം കുണ്ഠിതനായി. അതിനാല്‍ ഹുദൈഫ (റ)ഉഥ്മാനി(റ)(റ)നാട് പറഞ്ഞു: വിശ്വാസികളുടെ നേതാവേ, അല്ലാഹുവിന്റെ ഗ്രന്ഥത്തിന്റെ കാര്യത്തില്‍ മുമ്പ് ജൂതന്മാരും ക്രൈസ്തവരുമെല്ലാം ഭിന്നിച്ചതുപോലെ ഈ സമുദായം ഭിന്നിക്കാതിരിക്കു വാനായി അവരെ സഹായിച്ചാലും.’അങ്ങനെ ഉഥ്മാന്‍ (റ)ഹഫ്‌സേ(റ)യാട് അവരുടെ കൈവശമുള്ള സുഹുഫ് കൊടുത്തയക്കുവാന്‍ ആവശ്യപ്പെടുകയും അതുപയോഗിച്ച് ഔദ്യോഗിക ക്വുര്‍ആന്‍ പതിപ്പുകള്‍ പകര്‍ത്തിയെഴുതിയ ശേഷം തിരിച്ചു നല്‍കാമെന്ന് ഉറപ്പു നല്‍കുകയും ചെയ്തു കൊണ്ട് ഒരു സന്ദേശം കൊടുത്തയച്ചു. ഹഫ്‌സ (റ) ഉഥ്മാനിന്(റ) അത് കൊടുത്തയച്ചു. അതിന് ശേഷം സൈദ്ബ്‌നു ഥാബിത്, അബ്ദില്ലാഹി ബ്‌നുസ്സുബൈര്‍, സഈദ്ബ്‌നുല്‍ ആസ്വ്, അബ്ദുര്‍റഹ്മാനിബ്‌നുല്‍ ഹാരിഥിബ്‌നു ഹിശാം (റ)എന്നിവരെ വിളിച്ചു വരുത്തി അവരോട് ഉഥ്മാന്‍ (റ)കൃത്യവും അന്യൂനവുമായ ക്വുര്‍ആന്‍ കയ്യെഴുത്ത് പ്രതികള്‍ പകര്‍ത്തിയെഴുതുവാനാവശ്യപ്പെട്ടു. ഉഥ്മാന്‍ (റ)മൂന്ന് ഖുറൈശികളോടായി പറഞ്ഞു: 'ഏതെങ്കിലും വിഷയത്തില്‍ നിങ്ങള്‍ സൈദ്ബ്‌നു ഥാബിതുമായി വിയോജിക്കുന്നുവെങ്കില്‍ നിങ്ങള്‍ അത് ഖുറൈശീ ഉച്ചാരണ രീതിപ്രകാരം എഴുതുക; എന്തുകൊണ്ടെന്നാല്‍ ക്വുര്‍ആന്‍ അവതരിക്കപ്പെട്ടത് ഖുറൈശീ ഉച്ചാരണരീതിയിലാണ്. ’അവര്‍ അങ്ങനെ ചെയ്തു പൂര്‍ത്തിയാക്കിയപ്പോള്‍ ഉഥ്മാന്‍  (റ)ഹഫ്‌സയില്‍ (റ)നിന്ന് വാങ്ങിയ ആദ്യത്തെ കയ്യെഴുത്ത് പ്രതി അവര്‍ക്ക് തിരിച്ചു കൊടുത്തയച്ചു. അവര്‍ പകര്‍ത്തിയെഴുതിയ കോപ്പികള്‍ വ്യത്യസ്ത പ്രവിശ്യകളിലേക്ക് കൊടുത്തയക്കുകയും അതല്ലാത്ത പൂര്‍ണമായോ ഭാഗികമായോ ഉള്ള മറ്റെല്ലാ കയ്യെഴുത്ത് പ്രതികളും കത്തിച്ചു കളയുവാന്‍ ആവശ്യപ്പെടുകയും ചെയ്തു, ഉഥ്മാന്‍.(റ) (സ്വഹീഹുല്‍ ബുഖാരി, കിതാബു ഫദാഇലില്‍ ക്വുര്‍ആന്‍)

ക്വുര്‍ആന്‍ അവതരിക്കപ്പെട്ട ഖുറൈശീ ഉച്ചാരണരീതിയാണ് അറബി ഭാഷയുടെ ആധാര ഉച്ചാരണ രീതിയെന്നതിനാല്‍ അതില്‍ തന്നെയാണ് ക്വുര്‍ആന്‍ പാരായണം ചെയ്യേണ്ടതെന്ന് സ്വഹാബിമാര്‍ ഉഥ്മാനിന്(റ) മുമ്പും നിര്‍ബന്ധം പിടിച്ചിരുന്നു. ഇറാഖിലേക്ക് ക്വുര്‍ആന്‍ പഠിപ്പി ക്കുന്നതിന് വേണ്ടി പറഞ്ഞയച്ച ഇബ്‌നു മസ്ഊദ്  (റ)അന്ന് നിലവിലുണ്ടായിരുന്ന മറ്റൊരു ഉച്ചാരണ രീതിയായ ഹുദൈലില്‍ ജനങ്ങളെ ക്വുര്‍ആന്‍ പഠിപ്പിക്കുന്നുണ്ടെന്നറിഞ്ഞപ്പോള്‍ അത് വിരോധിക്കുകയും ക്വുര്‍ആന്‍ അവതരിച്ചത് ഖുറൈശികളുടെ ഉച്ചാരണ രീതിയിലാ ണെന്നും (ലിസാനു ഖുറൈശ്) അതിനാല്‍ ഖുറൈശീഭാഷ പ്രകാരമാണ്, ഹുദൈല്‍ ഭാഷ പ്രകാരമല്ല ജനങ്ങളെ പഠിപ്പിക്കേണ്ടത് (ഫത്ഹുല്‍ ബാരി, വാല്യം 9 പുറം 17) എന്നും നിഷ്‌കര്‍ഷിച്ചുകൊണ്ട് കത്തെഴുതുകയും ചെയ്ത ഉമറിന്റെ (റ) നടപടിയില്‍ നിന്ന് ഇക്കാര്യം വ്യക്തമാ കുന്നുണ്ട്. അതുകൊണ്ട് തന്നെയാണ് ഉഥ്മാന്‍,(റ) ഖുറൈശീഉച്ചാരണ പ്രകാരം തന്നെ ക്വുര്‍ആന്‍ എഴുതണമെന്ന് നിര്‍ദേശിക്കു കയും അത ല്ലാതെയുള്ള ലിപികളില്‍ എഴുതപ്പെട്ട രേഖകളുള്‍ക്കൊള്ളുന്ന, അനൗദ്യോഗിക ക്വുര്‍ആന്‍ രേഖകളെ കത്തിച്ചുകളയുവാന്‍ നിര്‍ദേശിക്കുക യും ചെയ്തത്.

അര്‍മീനിയക്കും അസര്‍ബൈജാനിനുമെതിരെയുള്ള ഉഥ്മാന്റെ (റ)കാലത്തെ യുദ്ധം നടന്ന ഹിജ്‌റ 25-ാം (ഫത്ഹുല്‍ ബാരി, വാല്യം 9 പുറം 18) വര്‍ഷത്തിനു ശേഷം ഉടനെത്തന്നെ ഉഥ്മാന്റെ (റ)കാലത്തെ ഔദ്യോഗിക ക്വുര്‍ആന്‍ കയ്യെഴുത്ത് രേഖയുടെ പകര്‍ത്തിയെഴുത്ത് നടന്നിട്ടു ണ്ടെന്ന് നടേ സൂചിപ്പിച്ച, അനസ്ബ്‌നു മാലികില്‍(റ) നിന്ന് ബുഖാരി നിവേദനം ചെയ്ത ഹദീഥ് വ്യക്തമാക്കുന്നുണ്ട്. (സ്വഹീഹുല്‍ ബുഖാരി, കിതാബു ഫദാഇലില്‍ ക്വുര്‍ആന്‍) പ്രവാചക വിയോഗം കഴിഞ്ഞ് രണ്ട് പതിറ്റാണ്ടിനകം തന്നെ പ്രസ്തുത പകര്‍ത്തിയെഴുത്ത് നടന്നിട്ടുണ്ടെ ന്നര്‍ഥം. ഖുറൈശീ ആധാരഭാഷ പ്രകാരമുള്ള ക്വുര്‍ആന്‍ കോപ്പികള്‍ തയാറാക്കുകയും വ്യത്യസ്ത പ്രവിശ്യാ തലസ്ഥാനങ്ങളിലേക്ക് അയ ച്ചുകൊടുക്കുകയും വ്യക്തികളുടെ കൈകളിലുണ്ടായിരുന്ന സ്വകാര്യ കയ്യെഴുത്ത് പ്രതികള്‍ നശിപ്പിക്കുവാനാവശ്യപ്പെടുകയും ചെയ്ത പ്പോള്‍ പ്രവാചകനില്‍(സ) നിന്ന് നേര്‍ക്കുനേരെ മതം പഠിച്ച അനുയായികളില്‍ മിക്കവരും ജീവിച്ചിരിക്കുന്നുണ്ടായിരുന്നു. അവരി ലാരും തന്നെ ഉഥ്മാന്റെ (റ)നടപടിയെ വിമര്‍ശിക്കുകയോ എതിര്‍ക്കുകയോ ചെയ്തതാ യി വ്യക്തമാക്കുന്ന രേഖകളൊന്നും തന്നെയില്ല. അല്ലാഹു വിനെയും പ്രവാചകനെയും(സ) സ്വന്തത്തെക്കാളധികം സ്‌നേഹിച്ചിരുന്ന സ്വഹാബിമാര്‍, ക്വുര്‍ആനില്‍ ഉഥ്മാന്‍ (റ)വല്ല മാറ്റവും വരുത്തു കയോ അതില്‍ നിന്ന് വല്ലതും മറച്ചുവെക്കുകയോ ചെയ്തിരുന്നുവെങ്കില്‍ അതിനെ ശക്തമായിത്തന്നെ എതിര്‍ക്കുകയും അദ്ദേഹത്തിനെതി രെ നടപടികള്‍ സ്വീകരിക്കുകയും ചെയ്യുമായിരുന്നു. അങ്ങനെ ചെയ്തില്ലെന്ന് മാത്രമല്ല, അവരെല്ലാവരും ഐകകണ്‌ ഠേന അദ്ദേഹത്തി ന്റെ നടപടി അംഗീകരിക്കുകയും അതോടൊപ്പം സഹകരിക്കുകയുമാണ് ചെയ്തത്.

അബൂബക്‌റിെ(റ)ന്റ കാലത്ത് നിര്‍മിക്കപ്പെട്ട ക്വുര്‍ആന്‍ കയ്യെഴുത്ത് പ്രതി കൈവശമുണ്ടായിരുന്നിട്ടും അതില്‍നിന്ന് നേര്‍ക്കുനേരെ കോപ്പികളെടുക്കാതെ, നിലനില്‍ക്കുന്ന കയ്യെഴുത്ത് രേഖകള്‍ പരിശോധിക്കുകയും മനഃപാഠമുള്ളവരില്‍ നിന്ന് കേള്‍ക്കുകയും ചെയ്തതിന് ശേഷം സ്വതന്ത്രമായ കോപ്പികള്‍ പകര്‍ത്തിയെഴുതുകയും അ ത് ഹഫ്‌സ യേുടെ പക്കലുണ്ടായിരുന്ന, അബൂബക്‌റിെ(റ)ന്റ കാലത്ത് നിര്‍മിക്കപ്പെട്ട 'സുഹുഫു'മായി ഒത്തുനോക്കി. അബദ്ധങ്ങളൊന്നുമുണ്ടായിട്ടില്ലെന്ന് ഉറപ്പ് വരുത്തുകയുമാണ് ഉഥ്മാനിെ(റ)ന്റ ഭരണകാ ലത്ത് സൈദ്ബ്‌നു ഥാബിതിന്റെ(റ) നേതൃത്വത്തിലുള്ള സംഘം ചെയ്തത്. ഇതിനായി അന്‍സ്വാരികളും ഖുറൈശികളുമടങ്ങുന്ന പന്ത്രണ്ട് പേരുടെ ഒരു സംഘത്തെയാണ് ഉഥ്മാന്‍ ഉത്തരവാദപ്പെടുത്തിയതെന്ന് ഹിജ്‌റ110-ല്‍ അന്തരിച്ച ഇബ്‌നു സീരീന്‍ രേഖപ്പെടുത്തുന്നതായി ഇബ്‌നു സഅ്ദ് തന്റെ ത്വബഖാത്തില്‍ ഉദ്ധരിക്കുന്നുണ്ട്.

അബൂബക്‌റിെ(റ)ന്റ കാലത്ത് സൈദുബ്‌നുഥാബിത്തിെ(റ)ന്റ നേതൃത്വത്തില്‍ നടത്തിയ ക്വുര്‍ആന്‍ കയ്യെഴുത്ത് രേഖകളുടെ ക്രോഡീകര ണത്തില്‍ എന്തെങ്കിലും തരത്തിലുള്ള സ്ഖലിതങ്ങളുണ്ടാവുകയോ, അന്ന് ലഭ്യമല്ലാതിരുന്നതിനാല്‍ ഏതെങ്കിലും വചനങ്ങള്‍ വിട്ടുപോ വുകയോ ചെയ്തിട്ടുണ്ടെങ്കില്‍ പ്രസ്തുത പ്രശ്‌നങ്ങള്‍ കൂടി പരിഹരിച്ച ശേഷമുള്ള അന്യൂനമായ ക്വുര്‍ആന്‍ രേഖ തന്നെയാവണം തന്റെ ഭരണകാലത്ത് നിര്‍മിക്കുന്നതെന്ന് വിചാരിച്ചതു കൊണ്ടാവണം ഹഫ്‌സയേുടെ പക്കലുള്ള 'സുഹുഫി'ന്റെ നേര്‍പതിപ്പുകള്‍ നിര്‍മിക്കാതെ സ്വതന്ത്രമായിത്തന്നെ ക്വുര്‍ആന്‍ കയ്യെഴുത്ത് രേഖകള്‍ നിര്‍മിക്കുവാനും അവ ഹഫ്‌സയേുടെ പക്കലുള്ള രേഖയുമായി ഒത്തുനോക്കിയ ശേഷം മാത്രം സ്വീകരിക്കുവാനും ഉഥ്മാന്‍ (റ)തീരുമാനിച്ചത്. ക്വുര്‍ആന്‍ മനഃപാഠമുള്ളവരുടെ മരണം വഴി വചനങ്ങള്‍ നഷ്ടപ്പെട്ടു പോകു മോയെന്ന ഭയമായിരുന്നു അബൂബക്‌റി(റ)നെയും ഉമറി(റ)നെയും 'സുഹുഫ്' നിര്‍മിക്കുന്നതിനെക്കുറിച്ച് ചിന്തിക്കുവാന്‍ പ്രേരിപ്പിച്ച തെങ്കില്‍, ക്വുര്‍ആന്‍ തെറ്റായി പാരായണം ചെയ്യുന്ന പ്രവണതയുണ്ടായി വരുന്നുവെന്ന കണ്ടെത്തെലാണ് ഉഥ്മാനിനെ(റ)ഔദ്യോഗിക ക്വുര്‍ആന്‍ രേഖകള്‍ പകര്‍ത്തിയെഴുതുവാന്‍ പ്രചോദിപ്പിച്ചത്.

തെറ്റായ രീതിയില്‍ രേഖപ്പെടുത്തപ്പെട്ട വ്യക്തിഗത കോപ്പികള്‍ നശിക്കാതെ നിലനിന്നാല്‍ ഭാവിയിലെങ്കിലും അത്തരം കോപ്പികളില്‍ നിന്ന് തെറ്റായ രീതിയില്‍ ക്വുര്‍ആന്‍ പാരായണം ചെയ്യപ്പടുന്ന അവസ്ഥയുണ്ടാകു മെന്ന് മനസ്സിലാക്കിയതിനാലാണ് എല്ലാ അര്‍ഥത്തിലും സമ്പൂര്‍ണമായ ഔദ്യോഗിക കോപ്പികള്‍ പകര്‍ത്തിയെഴുതുവാനും അങ്ങനെ പകര്‍ത്തിയെഴുതിയതിനു ശേഷം അതല്ലാത്ത കോപ്പികളെല്ലാം നശിപ്പിക്കുവാനും ഉഥ്മാന്‍ (റ)തീരുമാനിച്ചത്. ഈ തീരുമാനത്തിന് അന്നു ജീവിച്ചിരുന്ന സ്വഹാബിമാരുടെയെല്ലാം പിന്തുണയുണ്ടായിരുന്നു. ഉഥ്മാന്‍ (റ)ചെയ്തത് തെറ്റായിപ്പോയെന്ന് ഏതെങ്കിലുമൊരു പ്രവാച കാനുചരന്‍ പറഞ്ഞതായി തെളിയിക്കുവാന്‍ ക്വുര്‍ആനില്‍ കൈകടത്തുവാനാണ് അദ്ദേഹം ശ്രമിച്ചതെന്ന് ആരോപിക്കുന്ന ഇസ്‌ലാം വിമര്‍ശകര്‍ക്കൊന്നും കഴിഞ്ഞിട്ടില്ല. സ്വഹാബിമാരില്‍ പലര്‍ക്കും ക്വുര്‍ആന്‍ മനഃപാഠമുണ്ടായിരുന്നതിനാല്‍ അത്തരം വല്ല കൂട്ടിച്ചേര്‍ക്ക ലുകളും നടത്തിയിരുന്നുവെങ്കില്‍ അക്കാര്യം അവര്‍ തുറന്നുപറയുകയും തിരുത്തുകയും ചെയ്യുമായിരുന്നു. അങ്ങനെയൊന്നുമുണ്ടായി ട്ടില്ലെന്ന വസ്തുത അവതരിക്കപ്പെട്ട വിശുദ്ധിയില്‍ തന്നെ നിലനില്‍ക്കുന്നവയായിരുന്നു ഉഥ്മാന്‍ (റ)പകര്‍ത്തിയെഴുതിയ ഔദ്യോഗിക പതിപ്പുകള്‍ എന്ന വസ്തുതയ്ക്ക് അടിവരയിടുന്നുണ്ട്.

ക്വുര്‍ആനിന്റെ ഔദ്യോഗിക രേഖകള്‍ പകര്‍ത്തിയെഴുതി പ്രവിശ്യാതലസ്ഥാനങ്ങളിലേക്ക് കൊടുത്തയക്കുക മാത്രമല്ല ഉഥ്മാന്‍ (റ) ചെ യ്തത്; ഔദ്യോഗിക രേഖകളോടൊപ്പം ഔദ്യോഗിക പാരായണക്കാരെക്കൂടി പറഞ്ഞയച്ചുകൊണ്ട് അംഗീകൃതമല്ലാത്ത പാരായണ ഭേദങ്ങ ളുണ്ടാകുവാനുള്ള സാധ്യതയുടെ വാതില്‍ കൊട്ടിയടക്കുക കൂടി ചെയ്തു അദ്ദേഹമെന്നതാണ് വാസ്തവം. സൈദുബ്‌നു ഥാബിത്തിനെ മദീന യിലും അബ്ദുല്ലാഹിബ്‌നു അസ്സാഇബിനെ മക്കയിലും അല്‍മുഗീറത്തുബ്‌നു ശിഹാബിനെ സിറിയയിലും ആമിറുബ്‌നു അബ്ദില്‍ ഖൈസിനെ ബസ്വറയിലും അബ്ദുറഹ്മാന്‍ അസ്സുലാമിയെ കൂഫയിലും നിശ്ചയിച്ചത് ആ പ്രദേശത്തുകാര്‍ക്ക് ഔദ്യോഗിക രേഖപ്രകാരം എങ്ങനെ പാരാ യണം ചെയ്യണമെന്ന് പഠിപ്പിക്കുവാന്‍ വേണ്ടിയായിരുന്നു. അബ്ദുല്‍ ഫത്താഹ് അല്‍ ഖാദി എഴുതുന്നു: 'ഈ പണ്ഡിതന്മാരെല്ലാം പ്രവാചക നില്‍ നിന്ന് നിരവധി വഴികളില്‍ തങ്ങളിലെത്തുകയും അംഗീകരിക്കപ്പെടുകയും ചെയ്ത പാരായണ രീതകളിലാണ് തങ്ങളുടെ പക്കലുള്ള ഔദ്യോഗിക ക്വുര്‍ആന്‍ രേഖയിലെ വ്യഞ്ജനപ്രധാനമായ രേഖീകരണങ്ങളുപയോഗിച്ച് തങ്ങള്‍ നിയോഗിക്കപ്പെട്ട സമൂഹത്തെ പാരായ ണം ചെയ്തു പഠിപ്പിച്ചത്. ഒരാളിലൂടെ മാത്രമായി സംപ്രേഷണം ചെയ്യപ്പെട്ട പാരായണരീതിക ളൊന്നും അവര്‍ സ്വീകരിച്ചില്ല. നിരവധിയാ ളുകളിലൂടെ നിരവധി വഴികളില്‍ നിവേദനം ചെയ്യപ്പെട്ട പാരായണ രീതിയനുസരിച്ച് മാത്രം ജനങ്ങളെ പഠിപ്പിക്കുന്നതിനായി പാരായണ ക്കാരെക്കൂടി പറഞ്ഞയക്കുക വഴി പ്രവാചകന്‍(സ)പഠിപ്പിച്ച രീതികളില്‍ മാത്രം ഔദ്യോഗിക ക്വുര്‍ആന്‍ രേഖ പാരായണം ചെയ്യപ്പെടുന്ന അവസ്ഥയുണ്ടാക്കുകയാണ് ഉഥ്മാന്‍ (റ)ചെയ്തത്. (അബ്ദുല്‍ ഫത്താഹ് അല്‍ ഖാദിയുടെ അല്‍ ഖിറാആത് ഫീ നദ്വര്‍ അല്‍ മുസ്തശ്‌രിഖീന്‍ വല്‍ മുല്‍ഹിദീന്‍’ എന്ന തലക്കെട്ടില്‍ മജല്ലത്തുല്‍ അസ്ഹറില്‍ (വാല്യം 43/2, 1391 (1971) പുറം 175) വന്ന ലേഖനത്തില്‍ നിന്ന്‌ഡോക്ടര്‍ എം. എം.അഅ്ദമി ഉദ്ധരിച്ചത്, M.M Al Azami, “The History Of The Quranic Text – From Revelation To Compilation, A Comparative Study with the Old and New Testaments, Leicester, 2003 Page 95)

ഉഥ്മാനിെ(റ)ന്റ കാലത്ത് രേഖീകരിക്കപ്പെട്ട രൂപത്തില്‍ തന്നെ യാതൊരു മാറ്റവുമില്ലാതെയാണ് ഇന്നും ക്വുര്‍ആന്‍ നിലനില്‍ക്കുന്നത് എന്ന വസ്തുത ഓറിയന്റലിസ്റ്റ് പണ്ഡിതന്മാര്‍ പോലും അംഗീകരിച്ചിട്ടുള്ളതാണ്. പ്രസിദ്ധ ഓറിയന്റലിസ്റ്റും ക്രിസ്തുമത പ്രചാരകനുമായി രുന്ന സര്‍ വില്യം മ്യൂര്‍ എഴുതുന്നു: ''ഉഥ്മാനിന്റെ പരിശോധിത ഗ്രന്ഥം മാറ്റമൊന്നുമില്ലാതെ നമുക്ക് ലഭിച്ചിട്ടുണ്ട്. എടുത്തുപറയത്തക്ക വ്യത്യാസങ്ങളൊന്നുമില്ലാതെ-യാതൊരുവിധത്തിലുമുള്ള വ്യത്യാസങ്ങളില്ലാതെ എന്നു തന്നെ പറയാം-വളരെ സൂക്ഷ്മവും കൃത്യവുമായി അത് സംരക്ഷിക്കപ്പെട്ടിട്ടുണ്ടെന്ന് വിശാലമായ ഇസ്‌ലാമിക സാമ്രാജ്യത്തിന്റെ അതിര്‍ത്തികള്‍ക്കകത്ത് ചിതറിക്കിടക്കുന്ന അസംഖ്യം ക്വുര്‍ആന്‍ രേഖകള്‍ വ്യക്തമാക്കുന്നു. മുഹമ്മദിന്റെ(സ) മരണത്തിന് കാല്‍ നൂറ്റാണ്ട് കഴിയുന്നതിനു മുമ്പ് നടന്ന ഉഥ്മാനിന്റെ കൊലപാ തകത്തിനു ശേഷം പരസ്പരം വെറുക്കുകയും പോരാടുകയും ചെയ്യുന്ന നിരവധി വിഭാഗങ്ങള്‍ മുസ്‌ലിം ലോകത്തുണ്ടായി ട്ടുണ്ടെങ്കിലും അവര്‍ക്കെല്ലാം ഉണ്ടായിരുന്നത് ഒരേയൊരു ക്വുര്‍ആന്‍ തന്നെയായിരുന്നു. ഒരേയൊരു ഗ്രന്ഥം തന്നെയാണ് അന്നുമുതല്‍ ഇന്നുവരെയുള്ള മുഴുവനാളുകളും പാരായണം ചെയ്തു പോരുന്നത് എന്ന വസ്തുത നിര്‍ഭാഗ്യവാനായ ഖലീഫയുടെ ഉത്തരവ് പ്രകാരം നിര്‍മിക്കപ്പെട്ട ഗ്രന്ഥം തന്നെയാണ് നമുക്ക് ലഭിച്ചിരിക്കുന്നത് എന്നതിനുള്ള അനിഷേധ്യമായ തെളിവാണ്. പന്ത്രണ്ട് നൂറ്റാണ്ടുകാലം ഇത്തരത്തില്‍ യാതൊ രുവിധ മാറ്റങ്ങളൊന്നുമില്ലാതെ സംരക്ഷിക്കപ്പെട്ട ഒരേയൊരു ഗ്രന്ഥം,  ക്വുര്‍ആന്‍ മാത്രമായിരിക്കും. സ്വരചിഹ്നങ്ങളിലും (vowal sign) അക്ഷ രഭേദങ്ങളിലു(diacritical sign)മുള്ള വളരെ ചെറിയ വ്യത്യാസങ്ങള്‍ പോലും ഏറെ പരിമിതമാണ്. ഈ ചിഹ്നങ്ങളിടുന്ന സമ്പ്രദായം പില്‍ക്കാ ലത്ത് ഉണ്ടായതാണ് എന്നതു കൊണ്ടുതന്നെ അത് ആദ്യകാലത്തെ രേഖകളില്‍ നിലനിന്നിരുന്നില്ല. അതിനാല്‍ അവയൊന്നും തന്നെ ഉഥ്മാനി ന്റെ രേഖതന്നെയാണ് ഇന്നും നിലനില്‍ക്കുന്നതെന്ന വസ്തുതയെ ബാധിക്കുന്ന പ്രതിവാദങ്ങളല്ല''. (William Muir: The Life Of Mahomet, Edinburgh, 1912, Pages xxii-xxiii.)

വിതകളും കഥാഖ്യാനങ്ങളും തലമുറകളിലേക്ക് സംപ്രേഷണം ചെയ്യപ്പെടുന്ന മനഃപാഠത്തിലൂടെയും വൈജ്ഞാനിക സാഹിത്യങ്ങള്‍ അടുത്ത തലമുറയ്ക്ക് ലഭിക്കുന്ന രേഖീകരണത്തിലൂടെയും ഒരേപോലെ സംരക്ഷിക്കപ്പെട്ട ഗ്രന്ഥമാണ് ക്വുര്‍ആന്‍. ഒന്നും കടത്തിക്കൂട്ടുകയോ എടുത്തൊഴിവാക്കുകയോ ചെയ്യാനാവാത്തവിധം പതിനാലു നൂറ്റാണ്ടുകളായി തലമുറകളില്‍ നിന്ന് തലമുറകളിലേക്ക് ഈ രണ്ടു രീതികളിലും ക്വുര്‍ആന്‍ പകര്‍ത്തപ്പെട്ടുകൊണ്ടിരിക്കുന്നുണ്ട്. വാക്യങ്ങള്‍ കൂട്ടിച്ചേര്‍ക്കുകയോ എടുത്തുമാറ്റുകയോ ചെയ്തുവെന്ന് വസ്തുനിഷ്ഠ തെളിവുകളുടെ വെളിച്ചത്തില്‍ ഒരാള്‍ക്കും ആരോപിക്കപ്പെടാനാവാത്ത നിലയില്‍, ചരിത്രത്തിന്റെ പൂര്‍ണമായ വെളിച്ചത്തിലാണ് ഒന്നര സഹസ്രാബ്ദക്കാലമായി തലമുറകളില്‍ നിന്ന് തലമുറകളിലേക്കുള്ള ഈ സംപ്രേഷണം നടക്കുന്നത്. അങ്ങനെ സംരക്ഷിക്കുമെന്നത് അല്ലാഹുവിന്റെ വാഗ്ദാനമാണ്. (ക്വുര്‍ആന്‍ 15:9)

അവതരിപ്പിക്കപ്പെടുന്ന മുറയ്ക്ക് മുഹമ്മദ് നബി (സ) ക്വുര്‍ആന്‍ ഹൃദിസ്ഥമാക്കിയിരുന്നുവെന്നും ഓരോവര്‍ഷവും റമദാനില്‍ ജിബ്‌രീല്‍ വന്ന് അവതരിപ്പിക്കപ്പെട്ടിടത്തോളമുള്ള വചനങ്ങള്‍ പാരായണം ചെയ്തു കേട്ട് ഉറപ്പുവരുത്തിയിരുന്നുവെന്നും ഇമാം ബുഖാരി തന്റെ സ്വഹീഹിലെ കിതാബു ഫദാഇലില്‍ ഫാത്വിമ(റ)യില്‍ നിന്ന് നിവേദനം ചെയ്യുന്നുണ്ട്.

ക്വുര്‍ആന്‍ മനഃപാഠമാക്കുവാന്‍ സ്വഹാബിമാരെ പ്രവാചകന്‍ (സ) പ്രോത്സാഹിപ്പിക്കുകയും ഹൃദിസ്ഥമാക്കിയ വചനങ്ങള്‍ മറന്നു പോകാനിടയാകരുതെന്ന് പ്രത്യേകം നിഷ്‌കര്‍ഷിക്കുകയും ചെയ്തിരുന്നതായി വ്യക്തമാക്കുന്ന നിരവധി ഹദീഥുകൾ ബുഖാരിയിലും മുസ്ലിമിലും മറ്റു ഹദീഥ് ഗ്രൻഥങ്ങളിലുമെല്ലാം നിവേദനം ചെയ്യപ്പെട്ടിട്ടുണ്ട്. മനഃപാഠമാക്കുന്നതില്‍ വിദഗ്ധരായിരുന്നു അറബികളെന്നതിനാലും പദ്യ-ഗദ്യ സമ്മിശ്രമായ ക്വുര്‍ആനിന്റെ ശൈലി കാണാതെ പഠിക്കുവാന്‍ എളുപ്പവും ഇമ്പവും പ്രധാനം ചെയ്യുന്നതിനാലും സ്വഹാബിമാര്‍ക്ക് ക്വുര്‍ആന്‍ പഠനം ഒരു പ്രയാസമായി അനുഭവപ്പെട്ടതേയില്ല. പരമാവധി മനഃപാഠമാക്കുവാന്‍ ഓരോ സ്വഹാബിയും പരിശ്രമിച്ചു. കൂടുതല്‍ ക്വുര്‍ആന്‍ പഠിച്ചവന്‍ താനാണെന്ന് അഭിമാനത്തോടെ പറയുന്നവരായിരുന്നു സ്വഹാബിമാരെന്ന് ഇബ്‌നു മസ്ഊദി(റ)ൽ നിന്ന് ബുഖാരി നിവേദനം ചെയ്ത ഹദീഥ് വ്യക്തമാക്കുന്നുണ്ട്.

പ്രവാചകന്റെ (സ) കാലത്ത്, മനഃപാഠമാക്കുന്നതോടൊപ്പം തന്നെ, ക്വുര്‍ആന്‍ രേഖീകരിക്കുന്ന പതിവുമുണ്ടായിരുന്നുവെന്ന് വ്യക്തമാക്കുന്ന നിരവധി ചരിത്രരേഖകളുണ്ട്. ഉമറിന്റെ (റ) ഇസ്‌ലാം സ്വീകരണത്തെക്കുറിച്ച ഇബ്‌നു ഇസ്ഹാഖിന്റെ വിവരണത്തില്‍, തന്റെ സഹോദരിയും ഭര്‍ത്താവും ഇസ്‌ലാം സ്വീകരിച്ചിട്ടുണ്ടെന്ന് മനസ്സിലാക്കി കോപാകുലനായി അവരുടെ വീട്ടിലേക്ക് അദ്ദേഹം കുതിച്ചെത്തിയപ്പോള്‍ അവിടെ അദ്ദേഹത്തിന്റെ സഹോദരിയും ഭര്‍ത്താവും ഖബ്ബാബിനോടൊപ്പമിരുന്ന് ഒരു ചര്‍മപഠത്തിലെഴുതിയ ക്വുര്‍ആനിലെ ത്വാഹാ സൂറത്ത് പാരായണം ചെയ്യുകയായിരുന്നുവെന്നും ഉമര്‍ (റ) പുറത്തു വന്നിട്ടുണ്ടെന്നറിഞ്ഞ സഹോദരി ഫാത്വിമഃ (റ) ആ കയ്യെഴുത്തുരേഖ അവരുടെ തുടയ്ക്ക് താഴെ ഒളിപ്പിച്ചുവെച്ചുവെന്നും പറയുന്നതില്‍ നിന്ന് അന്നുമുതല്‍ തന്നെ ക്വുര്‍ആന്‍ രേഖീകരിച്ചു സൂക്ഷിക്കുന്ന പതിവ് നിലനിന്നിരുന്നുവെന്ന് വ്യക്തമാകുന്നു. ഉമറിന്റെ ഇസ്‌ലാം സ്വീകരണം നടന്ന പ്രവാചകത്വത്തിന്റെ ആറാം വര്‍ഷത്തിനു മുമ്പുതന്നെ ക്വുര്‍ആന്‍ കയ്യെഴുത്ത് രേഖകളിലാക്കി സൂക്ഷിക്കുന്ന പതിവ് മുസ്‌ലിം സമൂഹത്തിലുണ്ടായിരുന്നുവെന്ന് വ്യക്തമാക്കുന്നതാണ് ഈ വിവരണം. മക്കയില്‍വെച്ച് അവതരിക്കപ്പെട്ട വചനങ്ങള്‍ മക്കയില്‍വെച്ചുതന്നെ രേഖപ്പെടുത്തിവെച്ചിരുന്നതായി അബ്ദുല്ലാഹിബ്‌നു അബ്ബാസ് (റ) പറഞ്ഞിട്ടുണ്ടെന്ന് മുഹമ്മദ്ബ്‌നു ശിഹാബ് അസ്സുഹ്‌രി സാക്ഷ്യപ്പെടുത്തിയതായി ഇമാം ഇബ്‌നു കഥീര്‍ രേഖപ്പെടുത്തുന്നുണ്ട്. (അല്‍ ബിദായ വ ന്നിഹായ, വാല്യം 5, പുറം 340)

അബ്ദില്ലാഹിബ്‌നു സഅ്ദ്ബ്‌നു അബീ സര്‍ഹിനെയായിരുന്നു നബി(സ)യില്‍ നിന്ന് ക്വുര്‍ആന്‍ കേള്‍ക്കാനും അത് അന്ന് ഉപലബ്ധമായ എഴുത്തുവസ്തുക്കളില്‍ എഴുതി രേഖപ്പെടുത്തുവാനുമായി മക്കയില്‍വെച്ച് നബി (സ) ഏല്‍പിച്ചത്. ഖാലിദ്ബ്‌നു സഈദ്ബ്‌നുല്‍ ആസ്വ് ആയിരുന്നു നബി(സ)യുടെ നിര്‍ദേശാനുസരണം ക്വുര്‍ആന്‍ രേഖപ്പെടുത്തിയിരുന്നു മറ്റൊരാള്‍. 'ക്വുര്‍ആന്‍ അല്ലാതെ മറ്റൊന്നും നിങ്ങള്‍ എന്നില്‍നിന്ന് എഴുതി സൂക്ഷിക്കരുത്'(സ്വഹീഹു മുസ്‌ലിം, കിതാബു സ്‌സുഹുദു വര്‍ റഖാഇഖ്) എന്ന പ്രവാചകനിര്‍ദേശത്തില്‍ നിന്ന് നിരവധി പേര്‍ ക്വുര്‍ആന്‍ രേഖപ്പെടുത്തിവെക്കാറുണ്ടെന്ന് വ്യക്തമാകുന്നുണ്ട്. മദീനയില്‍ നിന്നെത്തിയവരുമായി പ്രവാചകന്‍ (സ) അഖബയില്‍വെച്ചുണ്ടാക്കിയ ഉടമ്പടിയില്‍ പങ്കെടുത്ത റാഫിഉബ്‌നു മാലിക് അല്‍ അന്‍സ്വാരിക്ക് അതുവരെ അവതരിക്കപ്പെട്ട എല്ലാ ക്വുര്‍ആന്‍ വചനങ്ങളും രേഖപ്പെടുത്തിയ ഒരു കയ്യെഴുത്ത് രേഖ നല്‍കിയതായും തന്റെ നാട്ടിലെത്തിയശേഷം ഗോത്രത്തിലുള്ളവരെയെല്ലാം വിളിച്ചുകൂട്ടി അത് അദ്ദേഹം വായിച്ചു കേള്‍പ്പിച്ചതായും വ്യക്തമാക്കുന്ന രേഖകളുണ്ട്.

മദീനയില്‍ എത്തിയതോടെ പ്രവാചകന്‍ (സ) കൂടുതല്‍ അനുയായികള്‍ ഉണ്ടായിരുന്നതുകൊണ്ടുതന്നെ കൂടുതല്‍ ക്വുര്‍ആന്‍ എഴുത്തുകാരുമുണ്ടായി. അബ്ദുബ്‌നു സഈദ് അബൂ ഉമാമ, അബൂഅയ്യൂബല്‍ അന്‍സ്വാരി, അബൂബക്ര്‍ സിദ്ദീഖ്, അബൂഹുദൈഫ, അബൂസുഫ്‌യാന്‍, അബൂസലമ, അബൂ അബസ്, ഉബയ്യ്ബ്‌നു കഅബ്, അല്‍അര്‍ഖം, ഉസൈദ്ബ്‌നുല്‍ ഹുദൈര്‍, ഔസ്, ബുറൈദ, ബഷീര്‍, ഥാബിത്ബ്‌നു ഖൈസ്, ജഅ്ഫര്‍ബിന്‍ അബീത്വാലിബ്, ജഹ്മ്ബ്‌നു സഅദ്, ജുഹൈം, ഹാതിബ്, ഹുദൈഫ, ഹുസൈന്‍, ഹന്‍ദല, ഹുവൈതിബ്, ഖാലിദ്ബ്‌നു സഈദ്, ഖാലിദ്ബ്‌നു വലീദ്, അസ്സുബൈറ്ബ്‌നു അവ്വാം, സുബൈറ്ബ്‌നു അര്‍ഖം, സൈദ്ബ്‌നു ഥാബിത്, സഅ്ദ്ബ്‌നു റബീഅ്, സഅ്ദ്ബ്‌നു ഉബാദ, സഈദ്ബ്‌നു സഈദ്, കുറഹ്ബില്‍ ബിന്‍ ഹസ്‌ന, ത്വല്‍ഹ, ആമിര്‍ ബിന്‍ ഫുഹൈറ, അബ്ബാസ്, അബ്ദുല്ലാഹിബ്‌നുല്‍ അര്‍ഖം, അബ്ദുല്ലാഹിബ്‌നു അബീബക്ര്‍, അബ്ദുല്ലാഹിബ്‌നു റവാഹ, അബ്ദുല്ലാഹിബ്‌നു സൈദ്, അബ്ദുല്ലാഹിബ്‌നു സഅദ്, അബ്ദുല്ലാഹിബ്‌നു അബ്ദില്ല, അബ്ദുല്ലാഹിബ്‌നു അംറ്, ഉഥ്മാനുബ്‌നു അഫ്ഫാന്‍, ഉഖ്ബ, അല്‍ അലാഅ് അല്‍ ഹദ്‌റമി, അല്‍ അലാഅ്ബ്ന്‍ ഉഖ്ബ, അലിയ്യുബിന്‍ അബീത്വലിബ്, ഉമറുബ്‌നുല്‍ ഖത്വാബ്, അംറുബ്‌നുല്‍ ആസ്വ്, മുഹമ്മദ്ബ്‌നു മസ്‌ലമ, മുആദ്ബ്‌നു ജബല്‍, മുആവിയ, മഅ്‌നുബ്‌നു, അദിയ്യ്, മുഐഖിബ്, മുന്‍ദിര്‍, മുഹാജിര്‍, യസീദിബ്‌നു അബീസുഫ്‌യാന്‍ (റ) എന്നിങ്ങനെ വ്യത്യസ്ത സന്ദര്‍ഭങ്ങളിലായി പ്രവാചകനില്‍ (സ) നിന്ന് ക്വുര്‍ആന്‍ വചനങ്ങള്‍ കേട്ടെഴുതിയ അറുപത്തഞ്ച് അനുചരന്‍മാരുടെ പട്ടിക ഡോ. മുഹമ്മദ് മുസ്തഫ അല്‍ അഅ്ദ്വമി തന്റെ പഠനത്തില്‍ വിവരിക്കുന്നുണ്ട്.(M.M Al Azami, “The History Of The Quranic Text – From Revelation To Compilation, A Comparative Study with the Old and New Testaments, Leicester, 2003, Page 68.)

വഹ്‌യ് അവതരിക്കപ്പെടുന്ന സന്ദര്‍ഭങ്ങളില്‍ ഉടന്‍ തന്നെ പ്രാപ്തനായ ഒരു അനുചരനെ വിളിച്ച് അത് എഴുതിവെക്കാനാവശ്യപ്പെടുക മുഹമ്മദ് നബി(സ)യുടെ പതിവായിരുന്നു. പ്രവാചകന്റെ (സ) പള്ളിക്കടുത്ത് താമസിച്ചിരുന്നതിനാല്‍ സൈദ്ബ്‌നു ഥാബിത്തിന് (റ) പലപ്പോഴും പ്രവാചകനില്‍ (സ) നിന്ന് വഹ്‌യ് എഴുതിവെക്കുവാന്‍ കൂടുതല്‍ അവസരമുണ്ടായിരുന്നതായി അദ്ദേഹം തന്നെ അനുസ്മരിക്കുന്നുണ്ട്. എഴുതാന്‍ കഴിയാത്ത പ്രവാചകാനുചരന്‍മാര്‍ തോല്‍ച്ചുരുളുകളും ചര്‍മപടങ്ങളുമായി വന്ന് എഴുതാന്‍ കഴിയുന്നവരെക്കൊണ്ട് ക്വുര്‍ആന്‍ വചനങ്ങള്‍ എഴുതിപ്പിച്ച് വാങ്ങുന്ന പതിവുണ്ടായിരുന്നുവെന്ന് വ്യക്തമാക്കുന്ന നിവേദനങ്ങളുണ്ട്.

വിശുദ്ധ ക്വുര്‍ആനിന്റെ അവതരണവും സംരക്ഷണവും മാത്രമല്ല ക്രോഡീകരണവും വിശദീകരണവുമെല്ലാം സര്‍വശക്തനായ അല്ലാഹു തന്നെ നിര്‍വഹിക്കുമെന്ന് സ്വയം പ്രഖ്യാപിച്ചിട്ടുള്ളതാണ്. ''തീര്‍ച്ചയായും അതിന്റെ (ക്വുര്‍ആന്റെ) സമാഹരണവും അത് ഓതിത്തരലും നമ്മുടെ ബാധ്യതയാകുന്നു. അങ്ങനെ നാം അത് ഓതിത്തന്നാല്‍ ആ ഓത്ത് നീ പിന്തുടരുക. പിന്നീട് അത് വിവരിച്ചുതരലും നമ്മുടെ ബാധ്യതയാകുന്നു.'' (ക്വുര്‍ആന്‍ 75:17-19) വ്യത്യസ്ത സാഹചര്യങ്ങളില്‍ അവതരിപ്പിക്കപ്പെടുന്ന അല്ലാഹുവിന്റെ വചനങ്ങള്‍ ക്വുര്‍ആനിലെ ഏത് അധ്യായത്തില്‍ എത്രാമത്തെ വചനങ്ങളായാണ് രേഖപ്പെടുത്തേണ്ടതെന്നുകൂടി ദൈവിക ബോധനത്തിന്റെ അടിസ്ഥാനത്തില്‍ നബി (സ) തന്നെ തന്റെ എഴുത്തുകാര്‍ക്ക് പറഞ്ഞുകൊടുക്കുമായിരുന്നു. സൂറത്തുകളെയും അവയിലെ ആയത്തുകളെയും അവയുടെ സ്ഥാനത്തെയുമെല്ലാം കുറിച്ച് പ്രവാചകാനുചരന്‍മാര്‍ക്ക് കൃത്യമായ ബോധ്യമുണ്ടായിരുന്നുവെന്ന് വ്യക്തമാക്കുന്ന നിരവധി നിവേദനങ്ങളുണ്ട്. ഓരോ വചനവും അവതരിക്കപ്പെടുമ്പോള്‍ തന്റെ അനുയായികളായ ക്വുര്‍ആന്‍ എഴുത്തുകാരെ വിളിച്ച് അവ പാരായണം ചെയ്തു കേള്‍പ്പിക്കുകയും ഏത് അധ്യാത്തില്‍ എത്രാമത്തെ വചനമായാണ് അത് ചേര്‍ക്കേണ്ടതെന്ന് നിര്‍ദേശം നല്‍കുകയും ചെയ്തിരുന്നതായി നബി(സ)യുടെ ക്വുര്‍ആന്‍ എഴുത്തുകാരില്‍ പ്രധാനിയായ സൈദ്ബ്‌നു ഥാബിത് (റ) വ്യക്തമാക്കുന്നുണ്ട്.(ജാമിഉത്തിര്‍മിദി, കിതാബു തഫ്‌സീറില്‍ ക്വുര്‍ആന്‍, സുനനു അബീദാവൂദ്, കിതാബുസ്സ്വലാത്)

ക്വുര്‍ആനിലെ പതിനാറാമത്തെ അധ്യായമായ സൂറത്തുന്നഹ്‌ലിലെ തൊണ്ണൂറാമത്തെ വചനം പാരായണം ചെയ്തുകൊണ്ട് ജിബ്‌രീല്‍ ഇപ്പോള്‍ എന്റെയടുക്കല്‍ വന്ന് ഈ വചനം സൂറത്തുന്നഹ്‌ലില്‍ തൊണ്ണൂറാം സൂക്തമായി ചേര്‍ക്കണമെന്നു നിര്‍ദേശിച്ചതായി അപ്പോള്‍ പ്രവാചകനോടൊപ്പമുണ്ടായിരുന്ന ഉഥ്മാനുബ്‌നു അബില്‍ ആസ്വിനോട് പറഞ്ഞതായി മുസ്‌നദ് അഹ്മദ് നിവേദനം ചെയ്യുന്ന ഹദീഥ് ഓരോ സൂക്തങ്ങളും എവിടെ ചേര്‍ക്കണമെന്ന ദൈവിക നിര്‍ദേശമുണ്ടായിരുന്നുവെന്ന വസ്തുത വെളിപ്പെടുത്തുന്നതാണ്. 'സൂറത്തുനിന്നാഇലെ അവസാനത്തെ വചനങ്ങള്‍ താങ്കള്‍ക്ക് മതിയാവുന്നതാണ്' എന്ന് ഉമറി(റ)നോട് പ്രവാചകന്‍ (സ) പറഞ്ഞതില്‍നിന്നും.സ്വഹീഹു മുസ്‌ലിം, കിതാബുല്‍ ഫറാഇദ് "സൂറത്തുല്‍ ബക്വറഃയിലെ അവസാനത്തെ രണ്ടു വചനങ്ങള്‍ രാത്രിയില്‍ പാരായണം ചെയ്യുന്നവര്‍ക്ക് അത് മതിയാകുന്നതാണ്' എന്ന അബൂ മസ്ഊദ് അല്‍ബദ്‌രി (റ) നിവേദനം ചെയ്ത നബിവചനത്തില്‍ നിന്നും(സ്വഹീഹുല്‍ ബുഖാരി, കിതാബു ഫദാഇലില്‍ ക്വുര്‍ആന്‍ എന്റെ അമ്മായിയായ മൈമൂന(റ)യുടെ വീട്ടില്‍ താമസിക്കുമ്പോള്‍ പ്രവാചകന്‍ (സ) രാത്രി ഉറക്കത്തില്‍ നിന്നെണീറ്റ് സൂറത്തു ആലുംറാനിലെ അവസാനത്തെ പത്തു വചനങ്ങള്‍ പാരായണം ചെയ്യുന്നതായി ഞാന്‍ കേട്ടുവെന്ന ഇബ്‌നു അബ്ബാസിന്റെ അനുഭവവിവരണത്തില്‍ നിന്നും(സ്വഹീഹു മുസ്‌ലിം, കിതാബുല്‍ വിദ്വൂഅ്, സ്വഹീഹു മുസ്‌ലിം, കിതാബുല്‍ മുസാഫിരീന്‍) വ്യക്തമാവുന്നത് ഏതെല്ലാം അധ്യായങ്ങളില്‍ എത്രാമത്തെ സൂക്തമാണ് ഓരോ ക്വുര്‍ആന്‍ സൂക്തങ്ങളുമെന്ന് സ്വഹാബിമാര്‍ക്കെല്ലാം കൃത്യമായി അറിയാമായിരുന്നുവെന്നാണ്.

മദീനാ ഇസ്‌ലാമിക രാഷ്ട്രത്തില്‍ ക്വുര്‍ആന്‍ മനഃപാഠമുള്ളവര്‍ ധാരാളമായി ഉണ്ടായിരുന്നതുപോലെ ക്വുര്‍ആന്‍ കയ്യെഴുത്ത് പ്രതികളും ധാരാളമായി ഉപയോഗിക്കപ്പെട്ടിരുന്നു. അതുകൊണ്ടാണല്ലോ ശത്രുക്കളുടെ നാട്ടിലേക്ക് ക്വുര്‍ആനുമായി യാത്ര ചെയ്യുന്നത് നബി (സ) നിരോധിച്ചത്.(സ്വഹീഹുല്‍ ബുഖാരി, കിതാബുല്‍ ജിഹാദ്') മദീനയിലെ മുസ്‌ലിംകളില്‍ പലരുടെയും പക്കല്‍ ക്വുര്‍ആന്‍ രേഖപ്പെടുത്തിയ ചുരുളുകളുണ്ടായിരുന്നുവെന്നും അതുമായി ശത്രുനാട്ടിലേക്കു പോകുന്ന പതിവ് സ്വഹാബിമാര്‍ക്കുണ്ടായിരുന്നുവെന്നുമാണ് അബ്ദുല്ലാഹിബ്‌നു ഉമര്‍ (റ) നിവേദനം ചെയ്ത ഈ ഹദീഥ് വ്യക്തമാക്കുന്നത്. മദീനയില്‍ ക്വുര്‍ആന്‍ കയ്യെഴുത്ത് രേഖകള്‍ വ്യാപകമായിരുന്നുവെന്ന് തന്നെയാണ് ഇത് മനസ്സിലാക്കിത്തരുന്നത്.

അവതരിക്കപ്പെട്ട മുറയിൽ കാണാതെ പഠിച്ചും ലഭ്യമായ ചുരുളുകളിൽ എഴുതി സൂക്ഷിച്ചും എഴുത്ത് രൂപത്തിലും മനഃപാഠമായും സംരക്ഷിക്കപ്പെട്ട ഗ്രൻഥമാണ് ഖുർആൻ എന്നർത്ഥം. അന്ന് മുതൽ ഇന്ന് വരെ ഈ രണ്ട് രുപത്തിലും ക്വുർആൻ സംരക്ഷിക്കപ്പെട്ടുകൊണ്ടിരിക്കുന്നു.

ക്രിസ്താബ്ദം 325-ല്‍ ചേര്‍ന്ന നിഖിയാ കൗണ്‍സില്‍ കാനോനികമായി അംഗീകരിച്ച കൃതികള്‍ മാത്രം നിലനിര്‍ത്തി ബാക്കി എല്ലാ ക്രൈസ്തവഗ്രന്ഥങ്ങളും ചുട്ടുകരിക്കുവാന്‍ സഭ ആഹ്വാനം നല്‍കി. ഉസ്മാന്‍(റ) തന്റെ നിര്‍ദേശപ്രകാരം തയാര്‍ ചെയ്യപ്പെട്ട ഖുര്‍ആന്‍ പ്രതികള്‍ മാത്രം നിലനിര്‍ത്തി ബാക്കിയുള്ളവയെല്ലാം ചുട്ടുകരിക്കാന്‍ കല്‍പിച്ചു. ഉസ്മാന്‍ ചെയ്തതും നിഖിയാ കൗണ്‍സില്‍ ചെയ്തതും തമ്മില്‍ എന്തു വ്യത്യാസമാണുള്ളത്?

വിടെ പരാമര്‍ശിക്കപ്പെട്ട സംഭവങ്ങളില്‍ 'കത്തിക്കുക'യെന്ന ക്രിയയാണ് ഇരുകൂട്ടരും ചെയ്തതെന്ന കാര്യമൊഴിച്ച് ബാക്കിയെല്ലാം തികച്ചും വ്യത്യസ്തമാണ്. രണ്ടു സംഭവങ്ങളും തമ്മിലുള്ള വ്യത്യാസങ്ങള്‍ ഇങ്ങനെ സംഗ്രഹിക്കാം.

  1. യേശുവിന് ശേഷം മൂന്നു നൂറ്റാണ്ടുകള്‍ക്കിടക്ക് പലരാലും രചിക്കപ്പെട്ട യേശുവിന്റെ ജീവിതത്തെയും സന്ദേശത്തെയും സംബന്ധിച്ച നാല്‍പതില ധികം ഗ്രന്ഥങ്ങളാണ് നിഖിയാ സൂനഹദോസ് കത്തിച്ചുകളഞ്ഞത്. മുഹമ്മദി(സ)നു ശേഷം രണ്ടു പതിറ്റാണ്ടിനിടക്ക് പലരും പകര്‍ത്തിയെഴുതിയ ഒരേ ഖുര്‍ആനിന്റെ വിവിധ ഏടുകളില്‍ ഉച്ചാരണ വ്യത്യാസത്തിന് ഇടയാക്കുന്നവയാണ് ഉസ്മാന്‍(റ) കത്തിച്ചുകളയാന്‍ ആവശ്യപ്പെട്ടത്.
  2. നിഖിയ കൗണ്‍സില്‍ കാനോനികമായി പ്രഖ്യാപിച്ച നാലു സുവിശേ ഷങ്ങളിലും അപ്പോസ്തല പ്രവര്‍ത്തനങ്ങളിലും ഇരുപത്തിയൊന്നു ലേഖനങ്ങളിലും വെളിപാടു പുസ്തകത്തിലുമുള്ള പരാമര്‍ശങ്ങള്‍ക്ക് വിരുദ്ധമായ പല പരാമര്‍ശങ്ങളുമുള്ളതുകൊണ്ടും അവ നല്‍കുന്ന യേശു ചിത്രത്തില്‍നിന്ന് തുലോം വ്യത്യസ്തമായ യേശുചിത്രമാണ് അവതരിപ്പിക്കുന്നത് എന്ന തുകൊണ്ടുമാണ് അപ്പോക്രിഫാ പുസ്തകങ്ങള്‍ കരിച്ചുകളയുവാന്‍ ആവശ്യപ്പെട്ടത്. വ്യത്യസ്ത ഉച്ചാരണരീതികളുള്ള പ്രാദേശികമൊഴികളില്‍ എഴുതപ്പെട്ട ഏടുകള്‍ തലമുറകളിലേക്ക് കൈമാറ്റം ചെയ്യപ്പെടുമ്പോള്‍ സാരമായ വൈകല്യങ്ങള്‍ക്ക് നിമിത്തമാകാമെന്ന ഭയമാണ് ഉസ്മാനെ(റ) ഔദ്യോഗിക കൈയെഴുത്തുപ്രതികള്‍ തയാറാക്കാനും സ്വകാര്യ ഏടുകള്‍ നശിപ്പിക്കാനും പ്രേരിപ്പിച്ചത്.
  3. കരിച്ചുകളഞ്ഞ അപ്പോക്രിഫാ ഗ്രന്ഥങ്ങളിലെ ആശയങ്ങള്‍ അവ കരിച്ചുകളഞ്ഞതോടുകൂടി വിസ്മൃതമായി. സ്വകാര്യ ഏടുകളില്‍ എഴുതപ്പെട്ട ഖുര്‍ആന്‍ സൂക്തങ്ങള്‍തന്നെയായിരുന്നു ഔദ്യോഗിക പ്രതികളിലുമുണ്ടായിരുന്നത്. ഉച്ചാരണഭേദങ്ങള്‍ ഒഴിവാക്കാന്‍ വേണ്ടി സ്വകാര്യ ഏടുകള്‍ കത്തിച്ചുകളഞ്ഞുവെങ്കിലും അവയിലുണ്ടായിരുന്ന സൂക്തങ്ങള്‍ അതേ രീതിയില്‍തന്നെ ഇന്നുള്ള ഖുര്‍ആന്‍ കോപ്പികളിലുമുണ്ട്.
  4. നിഖിയാ കൗണ്‍സില്‍ തള്ളിക്കളഞ്ഞുവെങ്കിലും അപ്പോക്രിഫാ ഗ്രന്ഥങ്ങളില്‍ പലതും പിന്നീടും ക്രൈസ്തവ മനസ്സുകളില്‍ നിലനിന്നിരു ന്നു. അവയിലെ കഥകളില്‍ ചിലത് തലമുറകളില്‍നിന്ന് തലമുറകളിലേക്ക് പ്രേഷണം ചെയ്യപ്പെട്ടു. പതിനാറാം നൂറ്റാണ്ടില്‍ നടന്ന തെന്ത്രോസ് സൂനഹദോസാണ് ഇക്കാര്യത്തില്‍ അന്തിമ തീര്‍പ്പുകല്‍പിച്ചത്. 1540 ഏപ്രില്‍ എട്ടാം തീയതി നടന്ന സൂനഹദോസിന്റെ നാലാം സമ്മേളനം 'കാനോനിക ഗ്രന് ഥങ്ങളെക്കുറിച്ച്' എന്ന ഡിക്രിയിലൂടെ പഴയനിയമത്തില്‍ 45-ഉം പുതിയനിയമത്തില്‍ 27-ഉം പുസ്തകങ്ങളാണ് ഉള്ളതെന്ന് പ്രഖ്യാപിച്ചു. ഇതാണ് കാനോനിക ഗ്രന്ഥങ്ങളെക്കുറിച്ച സഭയുടെ അവസാനത്തെ വാക്ക്. എന്നാല്‍, ഉസ്മാന്‍(റ) ഔദ്യോഗികമായി ഖുര്‍ആന്റെ കോപ്പികളെടുത്ത് സ്വകാര്യ ഏടുകള്‍ നശിപ്പിച്ചതിനുശേഷം ഇന്നുവരെ പ്രസ്തുത കോപ്പികളില്‍ നിന്നാണ് മുസ്ഹഫ് പകര്‍ത്തപ്പെടുന്നത്. അതില്‍ ആരും വ്യത്യസ്തത പുലര്‍ത്തുന്നില്ല.
  5. യേശുവിനെക്കുറിച്ച് എഴുതപ്പെട്ട കാനോനികമല്ലാത്ത ഗ്രന്ഥങ്ങള്‍ കരിച്ചുകളയണമെന്ന് കല്‍പിച്ച നിഖിയാ സുനഹദോസിന്റെ അധ്യക്ഷന്‍ അന്നുവരെ യേശുവില്‍ വിശ്വസിക്കാത്ത കോണ്‍സ്റ്റന്‍ൈറന്‍ ചക്രവര്‍ത്തിയായിരുന്നു. സ്വകാര്യ കൈയെഴുത്തുപ്രതികള്‍ നശിപ്പിക്കുവാനും ഖുര്‍ആനിന്റെ ഔദ്യോഗിക പ്രതികളെ മാത്രം ആശ്രയിച്ച് പാരായണം ചെയ്യാനും നിര്‍ദേശിച്ച ഉസ്മാന്‍(റ) കറകളഞ്ഞ ഭക്തനും മുഹമ്മദി(സ)ന്റെ ജാമാതാവും അദ്ദേഹത്തോടൊപ്പം ആദര്‍ശ സംരക്ഷണത്തിനുവേണ്ടി നിരവധി യുദ്ധങ്ങളില്‍ പങ്കെടുത്ത വിശ്വാസിയുമായിരുന്നു.

ഇല്ല. സൈദുബ്‌നു സാബിത്ത്(റ) ക്രോഡീകരിച്ച മുസ്ഹഫ് ഖലീഫയായിരുന്ന അബൂബക്കറി(റ)ന്റെ കൈവശമായിരുന്നു ഉണ്ടായിരുന്നത്. അദ്ദേഹത്തിന്റെ മരണശേഷം രണ്ടാം ഖലീഫ ഉമറി(റ)ന്റെ കൈവശമായി. ഉമറി(റ)ന്റെ കാലശേഷം അദ്ദേഹത്തിന്റെ പുത്രിയും മുഹമ്മദ് നബിയുടെ(സ) പത്‌നിയുമായിരുന്ന ഹഫ്‌സ(റ)യുടെ കൈവശമായി മുസ്ഹഫിന്റെ സൂക്ഷിപ്പ്. ആദ്യം മുതലെ ഹഫ്‌സ(റ)യുടെ കൈവശമായി രുന്നു ഈ കോപ്പിയെന്നും അഭിപ്രായമുണ്ട്. പ്രസ്തുത പതിപ്പിന് ഖുര്‍ആനിന്റെ ഔദ്യോഗിക പതിപ്പിന്റെ സ്ഥാനമുണ്ടായിരുന്നുവെങ്കി ലും മറ്റു പല വ്യക്തികളുടെ കൈവശവും ഖുര്‍ആന്റെ ഏടുകളുണ്ടായിരുന്നു. പ്രവാചകന്റെ കാലത്ത് എഴുതപ്പെട്ടവയും ശേഷം പകര്‍ ത്തിയെഴുതിയതുമായ ഏടുകള്‍. എന്നാല്‍, ഈ രേഖകളെയൊന്നുമായിരുന്നില്ല സാധാരണ ജനങ്ങള്‍ പൊതുവായി തങ്ങളുടെ പഠനത്തിനും പാരായണത്തിനും ആശ്രയിച്ചിരുന്നത്. അവര്‍ അവര്‍ക്കിടയിലുണ്ടായിരുന്ന മനഃപാഠമാക്കിയ വ്യക്തികളെയും അവരില്‍നിന്ന് പകര്‍ത്തി യെഴുതിയ സ്വകാര്യ ഏടുകളെയുമായിരുന്നു ആശ്രയിച്ചിരുന്നത്.

മൂന്നാം ഖലീഫ ഉസ്മാനി(റ)ന്റെ ഭരണകാലം. ഹിജ്‌റ 23-ാം വര്‍ഷമായപ്പോഴേക്ക് ഇസ്‌ലാം കൂടുതല്‍ പ്രചരിക്കുകയും പുതിയ ഭൂപ്രദേശ ങ്ങള്‍ ഇസ്‌ലാമിക സാമ്രാജ്യത്തിന്റെ വരുതിക്കുള്ളില്‍ വരികയും ചെയ്തു. അറബികളും അനറബികളുമായ ആയിരക്കണക്കിനാളുകള്‍ ഇസ്‌ലാമിലേക്ക് കടന്നുവന്നു. അറബിഭാഷ അറിയാത്തവരുടെ ഇസ്‌ലാം ആശ്ലേഷം ഖുര്‍ആന്‍ പാരായണത്തില്‍ ചില പ്രശ്‌നങ്ങളുണ്ടാക്കു ന്നതായി ചിലര്‍ ഉസ്മാന്റെ(റ) ശ്രദ്ധയില്‍ പെടുത്തി. അര്‍മീനിയ, അസര്‍ബൈജാന്‍ എന്നിവിടങ്ങളിലുണ്ടായ യുദ്ധങ്ങളുടെ അവസരത്തില്‍ പ്രസ്തുത പ്രദേശങ്ങളിലെ ജനങ്ങള്‍ ഒരുമിച്ചു കൂടിയപ്പോള്‍ ഖുര്‍ആന്‍ പാരായണത്തിന്റെ രീതിയിലും ഉച്ചാരണക്രമത്തിലും അവര്‍ വമ്പിച്ച വ്യത്യാസം വരുത്തുന്നത് കണ്ട പ്രവാചകാനുചരന്‍ ഇതിനൊരു പരിഹാരമുണ്ടാക്കണമെന്ന് ചിന്തിച്ചു. ഹുദൈഫ(റ)യായിരുന്നു ഈ പ്രശ്‌നം ഖലീഫയുടെ ശ്രദ്ധയില്‍പെടുത്തിയ ആദ്യ വ്യക്തികളില്‍ ഒരാള്‍. ഈ രൂപത്തില്‍ മുന്നോട്ടുപോയാല്‍ ഖുര്‍ആനിനെ സംബന്ധിച്ച് മുസ്‌ലിംകള്‍ക്കിടയില്‍ സാരമായ ഭിന്നിപ്പ് ഉടലെടുക്കാന്‍ കാരണമായേക്കുമെന്ന് ദീര്‍ഘദര്‍ശികളായ പ്രവാചകാനുചരന്മാര്‍ ശ്രദ്ധയില്‍പെടുത്തി. അനിവാര്യമായ നടപടികളുണ്ടാ വണമെന്ന് അവര്‍ ഖലീഫയോട് ആവശ്യപ്പെട്ടു.

ഉസ്മാന്‍ (റ) ഹഫ്‌സ(റ)യുടെ കൈവശമുണ്ടായിരുന്ന ഔദ്യോഗിക ഖുര്‍ആന്‍ പ്രതി കൊണ്ടുവരാന്‍ കല്‍പിച്ചു. ഇതിന്റെ പകര്‍പ്പുകള്‍ ശരിയായ ഖുറൈശി ഉച്ചാരണ രീതി പ്രകാരം തയാറാക്കുന്നതിനായി സൈദുബ്‌നു സാബിത്തി(റ)ന്റെ നേതൃത്വത്തിലുള്ള നാലംഗ സംഘത്തെ അദ്ദേഹം ചുമതലപ്പെടുത്തി. അറബിയുടെ ആധാര ഉച്ചാരണ രീതി ((standard pronunciation) യാണ് ഖുറൈഷി രീതി. അബ്ദുല്ലാഹിബ്‌നു സുബൈര്‍, സൈദുബ്‌നുല്‍ ആസ്വി, അബ്ദുറഹ്മാനുബ്‌നു ഹിശാം തുടങ്ങിയവരായിരുന്നു മറ്റ് അംഗങ്ങള്‍. ഹഫ്‌സ(റ)യുടെ കൈവശമുണ്ടായി രുന്ന ഔദ്യോഗിക മുസ്ഹഫിന്റെ ആധാര ഉച്ചാരണരീതി പ്രകാരമുള്ള പതിപ്പുകള്‍ തയാറാക്കുകയായിരുന്നു ഇവരുടെ ഉത്തരവാദിത്തം. അവര്‍ ഈ ഉത്തരവാദിത്തം ഭംഗിയായി നിര്‍വഹിച്ചു. ഹഫ്‌സയുടെ കൈവശമുണ്ടായിരുന്ന ഔദ്യോഗിക ഖുര്‍ആന്‍ പ്രതി സമാഹരിച്ച സൈദുബ്‌നുസാബിത്തുതന്നെ ഈ ഉത്തരവാദി ത്ത നിര്‍വഹണത്തിന് നേതൃത്വം നല്‍കിയിരുന്നതിനാല്‍ അബദ്ധങ്ങളൊന്നും പിണയാതെ സൂക്ഷ്മതയോടെ കൈകാര്യം ചെയ്യാന്‍പ്രത്യേകം സാധിച്ചുവെന്ന് പറയാവുന്നതാണ്.

ഇങ്ങനെ തയാറാക്കിയ പ്രതികള്‍ ഈജിപ്ത്, ബസറ, കൂഫ, മക്ക, സിറിയ, യമന്‍, ബഹ്‌റൈന്‍ തുടങ്ങിയ നാടുകളിലേക്ക് അയച്ചുകൊടു ത്തു. അതിനു ശേഷം വ്യക്തികള്‍ സൂക്ഷിച്ചിരുന്ന എല്ലാ ഏടുകളും കത്തിച്ചുകളയാന്‍ ഖലീഫ ഉത്തരവ് നല്‍കി. ഈ ആധികാരിക കോപ്പി കള്‍ പ്രകാരം മാത്രമേ ഖുര്‍ആന്‍ പാരായണം പാടുള്ളുവെന്നും കല്‍പന നല്‍കി. ഉസ്മാന്‍(റ) കോപ്പികളെടുത്തു നല്‍കിയ മുസ്ഹഫുക ളുടെ പകര്‍പ്പുകളാണ് ഇന്ന് ലോക വ്യാപകമായി പ്രസിദ്ധീകരിക്കപ്പെട്ടുകൊണ്ടിരിക്കുന്നത്. പ്രസ്തുത ഒറിജിനല്‍ കോപ്പികള്‍ ഇന്നും നിലവിലുണ്ട്.

മുഹമ്മദ് നബി (സ)ക്ക് ശേഷം അബൂബക്കറും പിന്നെ ഉഥ് മാനുമെല്ലാം (റ) ഖുർആൻ എഴുതി സൂക്ഷിച്ചു. ഇന്ന് നമുക്ക് ലഭിക്കുന്നത് അവർ എഴുതിയ കോപ്പികളുടെ പകർപ്പ് മാത്രമാണ്; മുഹമ്മദ് (സ) എഴുതിയതോ അദ്ദേഹത്തിന്റെ കാലത്ത് എഴുതപ്പെട്ടതോ ആയ കോപ്പികളാണ്. കൃസ്തുവിന് ശേഷം അദ്ദേഹത്തിന്റെ അപ്പോസ്തലന്മാർ സുവിശേഷങ്ങൾ എഴുതിയത് പോലെത്തന്നെയല്ലേ ഇതും; എന്ത് വ്യത്യാസമാണുള്ളത് ? വളരെ വലിയ വ്യത്യാസങ്ങളുണ്ട്. അവ ഇങ്ങനെ സംഗ്രഹിക്കാം:

  1. യേശു 'സുവിശേഷം' പ്രസംഗിച്ചു(മാര്‍ക്കോസ് 1:14,15,8:35, 14:9, 10:29, മത്തായി 4:23)വെന്ന് ബൈബിളില്‍ പറയുന്നുണ്ടെങ്കിലും അദ്ദേഹ ത്തിന്റെ കാലത്ത് 'സുവിശേഷം' ഏതെങ്കിലും രൂപത്തില്‍ രേഖപ്പെടുത്തപ്പെട്ടിരുന്നതായി യാതൊരു തെളിവുമില്ല. ഖുര്‍ആനാ കട്ടെ മുഹമ്മദി(സ)ന്റെ കാലത്തുതന്നെ രേഖപ്പെടുത്തപ്പെട്ടിരുന്നു. വ്യത്യസ്ത ഏടുകളിലായി.
  2. യേശു പ്രസംഗിച്ച 'സുവിശേഷം' അദ്ദേഹത്തിന്റെ സമകാലികരില്‍ ആരെങ്കിലും പദാനുപദം മനഃപാഠമാക്കിവെച്ചിരുന്നില്ല. ഖുര്‍ആന്‍ മനഃപാഠമാക്കിയ നൂറുകണക്കിന് അനുചരന്മാരുണ്ടായിരുന്നു.
  3. മത്തായിയോ, മാര്‍ക്കോസോ, ലൂക്കോസോ, യോഹന്നാനോ എഴുതിയത് യേശു പ്രസംഗിച്ച സുവിശേഷമല്ല. യേശുവിന്റെ ജീവിതത്തെ ക്കുറിച്ച് തങ്ങള്‍ അറിഞ്ഞ കാര്യങ്ങളാണ് (ലൂക്കോസ് 1:1-4) ഖുര്‍ആന്‍ മുഹമ്മദി(സ)ന്റെ ജീവചരിത്രമല്ല, അദ്ദേഹത്തിന് പടച്ചതമ്പുരാന്‍ അവതരിപ്പിച്ച വേദഗ്രന്ഥമാണ്.
  4. സമൂഹത്തിന്റെ പ്രതിനിധിയായ ഖലീഫ ഉത്തരവാദപ്പെടുത്തിയതിനനുസരിച്ച് സൂക്ഷ്മമായി പരിശോധിച്ചുകൊണ്ടാണ് ഖുര്‍ആന്‍ സമാഹാരണം നടന്നത്. സമാഹര്‍ത്താവായിരുന്ന സൈദുബ്‌നു സാബിത്തിന്റെ വാക്കുകളില്‍ പ്രസ്തുത സമാഹരണത്തിന്റെ സൂക്ഷ്മത വ്യക്തമായി പ്രതിഫലിക്കുന്നുണ്ട്. ''ഏതെങ്കിലുമൊരു പര്‍വതത്തെ അതിന്റെ സ്ഥാനത്തുനിന്ന് മാറ്റാനാണ് അബൂബക്കര്‍ എന്നോട് ആവശ്യപ്പെട്ടിരുന്നതെങ്കില്‍ അതായിരുന്നു എനിക്ക് ഇതിനേക്കാള്‍ നിസ്സാരം''. സുവിശേഷങ്ങളാവട്ടെ, ഓരോരുത്തര്‍ തങ്ങളുടെ ഇച്ഛപ്രകാരം രചിച്ച ഗ്രന്ഥങ്ങളാണ്. അവരുടെ ലക്ഷ്യമാകട്ടെ, തങ്ങളുടെ മുന്നിലുള്ള സമൂഹത്തിന് ക്രിസ്തുവിനെ പരിച യപ്പെടുത്തുക മാത്രമായിരുന്നു. (Raymond E. Brown: Responses to 101 Questions on the Bible, Page 57-58)
  5. യേശുവിന് ശേഷം അഞ്ചു പതിറ്റാണ്ടെങ്കിലും കഴിഞ്ഞാണ് സുവിശേഷങ്ങള്‍ എഴുതപ്പെട്ടത്. ഹിജ്‌റ പന്ത്രണ്ടാം വര്‍ഷത്തിലാണ് - പ്രവാ ചക നിര്യാണത്തിന് രണ്ടു വര്‍ഷങ്ങള്‍ക്കുശേഷം - ഖുര്‍ആന്‍ സമാഹരണത്തിനു തുടക്കം കുറിക്കപ്പെട്ടത്.
  6. യേശുവിന്റെ ശിഷ്യന്മാരല്ല സുവിശേഷങ്ങള്‍ രചിച്ചിട്ടുള്ളത്. മുഹമ്മദി(സ)ന്റെ ശിഷ്യന്മാരാണ് ഖുര്‍ആന്‍ സമാഹരിച്ചത്.സുവിശേഷ ങ്ങളുടെ രചനക്ക് ആധാരം യേശുവിനെ സംബന്ധിച്ച കേട്ടുകേള്‍വികള്‍ മാത്രമായിരുന്നു. ഖുര്‍ആന്‍ ക്രോഡീകരണത്തിന് പ്രവാചകന്‍(സ) തന്നെ പറഞ്ഞുകൊടുത്ത് എഴുതിപ്പിടിപ്പിച്ച ഏടുകളും പ്രവാചക നില്‍നിന്ന് നേരിട്ട് ഖുര്‍ആന്‍ കേട്ടു മനഃപാഠമാക്കിയ നൂറുകണക്കിന് അനുചരന്മാരുമായിരുന്നു അവലംബം.

മുഹമ്മദ് നബി(സ) യുടെ പിന്‍ഗാമിയായി ഇസ്‌ലാമിക സമൂഹത്തിന്റെ രാഷ്ട്രീയ നേതൃത്വമേറ്റെടുത്ത അബൂബക്‌റിെ(റ)ന്റ ഭരണകാല ത്താണ് വ്യത്യസ്തങ്ങളായ രേഖകളില്‍ എഴുതി സൂക്ഷിച്ചിരുന്ന ക്വുര്‍ആന്‍ വചനങ്ങളെല്ലാം ശേഖരിച്ച് രണ്ട് ചട്ടകള്‍ക്കുള്ളിലാക്കി പുസ്തകരൂപത്തില്‍ ക്രോഡീകരിക്കപ്പെട്ടത്. ക്വുര്‍ആന്‍ മനഃപാഠമുള്ളവര്‍ ജീവിച്ചിരിക്കുകയും എഴുതപ്പെട്ട രേഖകള്‍ അവശേഷിക്കുകയും ചെയ്യുന്ന അവസ്ഥയിലാണ് നബി(സ) ഇഹ ലോകവാസം വെടിയുന്നത്. അദ്ദേഹത്തിന്റെ ജീവിതകാലത്ത് ക്വുര്‍ആന്‍ രണ്ട് ചട്ടകള്‍ക്കുള്ളില്‍ ക്രോഡീകരിച്ച് പുസ്തക രൂപത്തിലാക്കു ക പ്രയാസകരമായിരുന്നു. മരണത്തിന് മുമ്പ് വരെ ദിവ്യബോധനങ്ങള്‍ ലഭിച്ചുകൊണ്ടിരുന്നുവെന്ന് അനസു ബ്‌നു മാലിക്‌ (റ)നിവേ ദനം ചെയ്ത സ്വഹീഹുല്‍ ബുഖാരിയിൽ കിതാബു ഫദാഇലിലുള്ള ഹദീഥിലുണ്ട്. ക്വുര്‍ആനിന്റെ പൂര്‍ത്തീകരണത്തിന് ശേഷം മാത്രം ചെയ്യേണ്ട പുസ്തക രൂപത്തിലുള്ള ക്രോഡീകരണം നിര്‍വഹിക്കുവാന്‍ നബി(സ)യുടെ ജീവിതകാലത്ത് കഴിയുമായിരുന്നില്ല എന്നാണിത് വ്യക്തമാക്കുന്നത്.. എന്നാല്‍ പ്രവാചക നിര്‍ദേശപ്രകാരം മനഃപാഠമാക്കിയവര്‍ക്ക്, രണ്ട് ചട്ടകള്‍ക്കുള്ളിലെന്നത് പോലെത്തന്നെ, ക്വുര്‍ആന്‍ തുടക്കം മുതല്‍ ഒടുക്കം വരെ പൂര്‍ണമായി അറിയാമായിരുന്നു. നിലവിലുള്ള രേഖകള്‍ ശേഖരിച്ച് ക്വുര്‍ആന്‍ ഗ്രന്ഥരൂപത്തിലാക്കിയത് അത്തരക്കാരിലൂടെയായതി നാല്‍ തന്നെ പ്രവാചകന്‍(സ) പഠിപ്പിച്ച അതേ രൂപത്തിലും ക്രമത്തിലുമുള്ള ക്വുര്‍ആനാണ് പിന്‍തലമുറക്ക് ലഭിച്ചത്.

അബൂബക്‌റിെ(റ)ന്റ ഭരണകാലത്തെ ക്രോഡീകരണത്തിന് നേതൃത്വം നൽകിയത് പ്രമുഖ പ്രവാചകാനുചരനായ സൈദ്ബ്‌നു ഥാബിത്ത് ആയിരുന്നു. യമാമ യുദ്ധത്തില്‍ വെച്ച് ക്വുര്‍ആന്‍ മനഃപാഠമാക്കിയിട്ടുള്ള നിരവധിപേര്‍ കൊല്ലപ്പെട്ടപ്പോൾ 'ഇനിയും ഉണ്ടാകാനിരിക്കുന്ന യുദ്ധങ്ങളില്‍ ക്വുര്‍ആന്‍ അറിയാവുന്നവര്‍ മരണപ്പെട്ടാല്‍, അതുവഴി ക്വുര്‍ആന്‍ നഷ്ടപ്പെട്ടു പോകുമോയെന്ന് ഞാന്‍ ഭയപ്പെടുന്നു' വെന്ന ഉമറ്ബ്‌നുല്‍ഖത്താബിന്റെ ‌(റ) ആശങ്കയാണ് അബൂബക്കറിനെ (റ) ക്വുർആനിന്റെ ഗ്രൻഥരൂപത്തിലുള്ള ക്രോഡീകരണത്തെപ്പറ്റി ചിന്തിപ്പിച്ചത്. അവർ രണ്ട് പേരും കൂടി പ്രവാചകന്റെ ക്വുർആൻ എഴുത്തുകാരിലൊരാളും സ്വഹാബീപ്രമുഖനുമായ സൈദ് ബ്നു ഥാബിത് (റ) വിനെ ആ ചുമതലയേൽപ്പിക്കുകയും അദ്ദേഹം ഈത്തപ്പനയോലകളിലും ശുഭ്രശിലകളിലുമുള്ള കയ്യെഴുത്ത് രേഖകളില്‍ നിന്നും മനഃപാഠമുള്ളവരില്‍ നിന്നുമായി ഖുർആൻ ആയത്തുകൾ ശേഖരിക്കുകയും രണ്ട് ചട്ടകൾക്കിടയിലുള്ള ഒരു ഗ്രന്ഥാമായി അവ എഴുതി രേഖപ്പെടുത്തുകയും ചെയ്തു. അങ്ങനെ പൂര്‍ത്തീകരിക്കപ്പെട്ട ക്വുര്‍ആനിന്റെ കയ്യെഴുത്ത് രേഖ തന്റെ മരണം വരെ അബൂബക്‌റിന്റെ (റ)കൈവശമാണുണ്ടായിരുന്നതെന്നും അതിനുശേഷം തന്റെ മരണം വരെ ഉമറിന്റെ (റ)കൈവശവും അതിനു ശേഷം അദ്ദേഹത്തിന്റെ മകളായ (പ്രവാചകപത്‌നി) ഹഫ്‌സയുടെ കൈവശവുമാണ് അത് ഉണ്ടായിരുന്നതെന്നും സ്വഹീഹുല്‍ ബുഖാരിയിലെ കിതാബു ഫദാഇലില്‍ ക്വുര്‍ആന്‍ എന്ന അധ്യായത്തിൽ സൈദ്ബ്‌നു ഥാബിത്തിൽ നിന്ന് തന്നെ നിവേദനം ചെയ്യപ്പെട്ടിട്ടുണ്ട്.

പ്രവാചകവിയോഗത്തിനു ശേഷം ഇരുപത്തിയേഴ് മാസം മാത്രം ഭരിച്ച അബൂബക്‌റിന്റെ (റ)ഭരണ കാലത്ത് തന്നെ രണ്ട് ചട്ടകള്‍ക്കുള്ളി ലായി ക്വുര്‍ആന്‍ ക്രോഡീകരിക്കപ്പെട്ടിട്ടുണ്ടെന്ന് ഇതില്‍ നിന്ന് വ്യക്തമാണ്. അത് നിര്‍വഹിച്ചതാകട്ടെ പ്രവാചകനില്‍(സ) നിന്ന് നേര്‍ക്കു നേരെ ക്വുര്‍ആന്‍ കേള്‍ക്കുവാനും മനഃപാഠമാക്കുവാനും ഏറ്റവും അധികം അവസരമുണ്ടായിരുന്ന വ്യക്തിയും നബി(സ)യുടെ എഴുത്തു കാരില്‍ പ്രമുഖനുമായ സൈദ് ബ്‌നു ഥാബിത്തും!(റ) . ഇതെല്ലാം നടക്കുന്നത് പ്രവാചക വിയോഗത്തിനു ശേഷം കേവലം ആറ് മാസം മാത്രം കഴിഞ്ഞു നടന്ന യമാമ യുദ്ധത്തിനു തൊട്ടുടനെയാണ്. ക്വുര്‍ആന്‍ മനഃപാഠമുള്ളവരില്‍ നിന്ന് കേള്‍ക്കുവാനും പകര്‍ത്തുവാനും, അവ എത്രത്തോളം ലഭ്യമാ യ കയ്യെഴുത്തു രേഖകളുമായി സാമ്യം പുലര്‍ത്തുന്നുണ്ടെന്ന് പരിശോധിക്കുവാനുമെല്ലാം കഴിയുന്ന ഊര്‍ജസ്വലനും സത്യസന്ധനുമായ വ്യക്തിയെത്തന്നെയായിരുന്നു.. സ്വന്തം മനഃപാഠത്തെയോ ജീവിച്ചിരിക്കുന്ന മറ്റു ക്വുര്‍ആന്‍ അറിയുന്നവരുടെ മനഃപാഠത്തെയോ മാത്രം ആശ്രയിക്കുകയല്ല, അവര്‍ മനഃപാഠമാക്കിയ കാര്യങ്ങള്‍ പ്രവാചകന്റെ കാലത്ത് എഴുതപ്പെട്ട രേഖകളിലേതി ലെങ്കിലുമുണ്ടോയെന്ന് പരിശോധിക്കുകകൂടി ചെയ്തതിനു ശേഷമാണ് അദ്ദേഹം ക്വുര്‍ആന്‍ ക്രോഡീകരിച്ചത്.

പലരുടെയും മനഃപാഠത്തി ലുണ്ടായിരുന്നുവെങ്കിലും സൂറത്തുത്തൗബയിലെ അവസാനത്തെ രണ്ട് വചനങ്ങള്‍ എഴുത്ത് രേഖകളിലൊന്നും കണ്ടെത്താനായില്ലെന്നും അത് കണ്ടെത്തുന്നത് വരെ തന്റെ അന്വേഷണം തുടര്‍ന്നുവെന്നും അബൂ ഖുസൈമത്തല്‍ അന്‍സ്വാരിയുടെ പക്കല്‍ നിന്ന് അവകൂടി കണ്ടെത്തിയ ശേഷമാണ് തന്റെ ക്രോഡീകരണദൗത്യം അവസാനിപ്പിച്ചതെന്നും സൈദ് ബ്‌നു ഥാബിത്ത്‌ (റ) തന്നെ നട്ട സൂചിപ്പിച്ച ഹദീഥിൽ പറയുന്നുണ്ട്. പ്രവാചകവിയോഗത്തിന് ശേഷം ആറ് മാസം കഴിഞ്ഞ ഉടനെത്തന്നെ തികച്ചും വിശ്വസ്തനും സത്യസന്ധനും പ്രവാചകനില്‍ നിന്ന് ക്വുര്‍ആന്‍ കേട്ടെഴുതുകയും മനഃപാഠമാക്കുകയും ചെയ്ത വ്യക്തിയുമായ സൈദ് ബ്‌നു ഥാബിത്തിലൂടെ രണ്ട് ചട്ടകള്‍ക്കകത്തായി ക്വുര്‍ആന്‍ ക്രോഡീകരിക്കപ്പെടുകയും അത് പ്രവാചകാനുചരന്മാരുള്‍ക്കൊള്ളുന്ന മുസ്‌ലിം സമൂഹം മൊത്തത്തില്‍ അംഗീകരിക്കുകയും ചെയ്തിട്ടുണ്ട് എന്നര്‍ഥം.

അബൂബക്‌റിെ(റ)ന്റ ഭരണകാലത്ത് രണ്ട് ചട്ടകള്‍ക്കുള്ളിലായി ക്രോഡീകരിക്കപ്പെട്ട ക്വുര്‍ആനിനെ അത് നിര്‍വഹിച്ച സൈദ്ബ്‌നു ഥാബിത്ത് വിളിച്ചിരിക്കുന്നത് 'സുഹുഫ്' (സ്വഹീഹുല്‍ ബുഖാരി, കിതാബു ഫദാഇലില്‍ ക്വുര്‍ആന്‍) എന്നാണ്. ഇബ്‌റാഹീമിനും മൂസായ്ക്കും നല്‍കപ്പെട്ട ഏടുകളെക്കുറിക്കുവാന്‍ ക്വുര്‍ആന്‍ (87:19) ഉപയോഗിച്ച അതേ പ്രയോഗം. താന്‍ മരണപ്പെടുമ്പോള്‍ തന്റെ പിന്‍ഗാമിയായ ഉമറിെ(റ)ന നിര്‍ദേശിച്ചതോടൊപ്പം തന്നെ ഈ 'സുഹുഫ്'അബൂബക്കർ (റ) ഉമറി(റ)നെ ഏല്‍പിക്കുകയും ചെയ്തു. പത്ത് വര്‍ഷക്കാലം ഇസ്‌ലാമിക രാഷ്ട്രത്തിന്റെ ഉന്നതാധികാരിയായി സേവനമനുഷ്ഠിച്ച ഉമറി(റ)ന്റെ ഭരണകാലത്ത് ഈ ക്വുര്‍ആന്‍ കോപ്പി അദ്ദേഹം സൂക്ഷിക്കുകയും ഒപ്പം തന്നെ രാഷ്ട്രത്തിലുടനീളം ക്വുര്‍ആന്‍ പഠിപ്പിക്കുവാനും പ്രചിരിപ്പിക്കുവാനുമാവശ്യമായ സംവിധാനങ്ങളൊരുക്കുകയും ചെയ്തു. ബസ്വറയിലേക്കും കൂഫയിലേക്കും ഹിംസിലേക്കും ദമസ്‌കസിലേക്കും ഫലസ്തീനിലേക്കുമെല്ലാം ക്വുര്‍ആന്‍ പഠിപ്പിക്കുവാനായി വ്യത്യസ്ത പ്രവാചകാനുചരന്മാരെ ഉമർ (റ) പറഞ്ഞയച്ചതായി രേഖപ്പെടുത്തപ്പെട്ടിട്ടുണ്ട്. ശമ്പളം കൊടുത്തതും അല്ലാതെയുമെല്ലാം പലരെയും അദ്ദേഹം കുട്ടികൾക്കും മുതിർന്നവർക്കും ക്വുർആൻ പഠിപ്പിക്കാനായി ചുമതലപ്പെടുത്തത്തിയതായി കാണാൻ കഴിയും.

പേര്‍ഷ്യന്‍ ക്രൈസ്തവനായ അബൂലുഅ്‌ലുഅയുടെ കുത്തേറ്റ് മരണാസന്നനായി കിടന്നപ്പോള്‍ തന്റെ പിന്‍ഗാമിയായി ആരെയും നിശ്ചയിക്കാതിരിക്കുകയും രാഷ്ട്രനേതാവിനെ തെരഞ്ഞെടുക്കേണ്ടതെങ്ങനെയെന്ന തന്റെ വീക്ഷണം മാത്രം സമൂഹത്തിന് മുന്നില്‍ വെക്കുകയും ചെയ്ത ഉമര്‍,(റ) വിശുദ്ധ ക്വുര്‍ആനിന്റെ കയ്യെഴുത്ത് പ്രതിയായ 'സുഹുഫ്' തന്റെ പുത്രിയും പ്രവാചക പത്‌നിയുമായ ഹഫ്‌സയെ (റ) ഏല്‍പിച്ചുകൊണ്ടാണ് ഇഹലോകവാസം വെടിഞ്ഞത്. പ്രസ്തുത കോപ്പിയെ ആധാരമാക്കിയാണ് ഉഥ്മാൻ (റ) കാലത്ത് ഖുർആൻ കോപ്പികളെടുക്കുകയും പ്രവിശ്യാ തലസ്ഥാനങ്ങളിലേക്ക് കൊടുത്തയാക്കുകയും ചെയ്തത്.

ഈ ചോദ്യത്തിന് 'അതെ'യെന്നും 'ഇല്ല'യെന്നും ഉത്തരം പറയാം. ഒരു ഗ്രന്ഥം ക്രോഡീകരിക്കുകയെന്നു പറയുമ്പോള്‍ എന്താണ് അര്‍ഥമാ ക്കുന്നത് എന്നതിനെ ആശ്രയിച്ചിരിക്കുന്നു ഈ ചോദ്യത്തിന്റെ ഉത്തരം. ആദ്യം മുതല്‍ അവസാനം വരെയുള്ള അധ്യായങ്ങള്‍ ഏതെല്ലാമാ ണെന്നും അവയിലെ വാക്യങ്ങള്‍ ഏതെല്ലാമാണെന്നും വ്യക്തമായി പറഞ്ഞുകൊടുക്കുകയും അതുപ്രകാരം തന്റെ അനുയായികളില്‍ നല്ലൊരു ശതമാനത്തെക്കൊണ്ട് മനഃപാഠമാക്കിക്കുകയും ചെയ്യുന്നതുകൊണ്ട് ഒരു ഗ്രന്ഥം ക്രോഡീകരിക്കപ്പെട്ടുവെന്ന് പറയാമെങ്കില്‍ മുഹമ്മദി(സ)ന്റെ കാലത്തുതന്നെ ഖുര്‍ആന്‍ ക്രോഡീകരിക്കപ്പെട്ടുവെന്ന്  പറയാവുന്നതാണ്. എന്നാല്‍, രണ്ടു പുറംചട്ടകള്‍ക്കുള്ളില്‍ ഗ്രന്ഥത്തിലെ എല്ലാ അധ്യായങ്ങളും തുന്നിച്ചേര്‍ത്തുകൊണ്ട് പുറത്തിറക്കുകയാണ് ക്രോഡീകരണം കൊണ്ടുള്ള വിവക്ഷയെങ്കില്‍ ഖുര്‍ആന്‍ മുഹമ്മദി(സ)ന്റെ ജീവിതകാലത്ത് ക്രോഡീകരിക്കപ്പെട്ടിട്ടില്ല എന്നും പറയാവുന്നതാണ്.

പ്രവാചകന്റെ ജീവിതകാലത്തിനിടയില്‍ ഖുര്‍ആന്‍ ക്രോഡീകരിക്കുക അസാധ്യമായിരുന്നുവെന്നതാണ് വാസ്തവം. ഖുര്‍ആന്‍ അവതരണത്തിന്റെ ശൈലി നമുക്കറിയാം.'ജിബ്‌രീല്‍ വരുന്നു. ഖുര്‍ആന്‍ സൂക്തങ്ങള്‍ ഓതികേള്‍പ്പിക്കുന്നു. അത് ഏത് അധ്യായത്തില്‍ എത്രാമത്തെ വാക്യമായി ചേര്‍ക്കണമെന്ന് നിര്‍ദേശിക്കുന്നു'. ഇതായിരുന്നുവല്ലോ രൂപം. വിവിധ സന്ദര്‍ഭങ്ങളില്‍ അവതരിപ്പിക്കപ്പെട്ട സൂക്തങ്ങള്‍ കാലഗണനയനുസരിച്ചല്ല അധ്യായങ്ങളായി സമാഹരിക്കപ്പെട്ടിരിക്കുന്നത് എന്നതിനാല്‍ അവസാനത്തെ സൂക്തം കൂടി അവതരിപ്പിച്ചു കഴിഞ്ഞതിനുശേഷമേ അവസാനമായി ഖുര്‍ആന്‍ ക്രോഡീകരിച്ച് ഗ്രന്ഥമാക്കുവാന്‍ കഴിയുമായിരുന്നുള്ളൂ. പ്രവാചകന്റെ വിയോഗത്തിന് ഒമ്പത് ദിവസങ്ങള്‍ക്കുമുമ്പാണ് അവസാനത്തെ ഖുര്‍ആന്‍ സൂക്തം അവതരിപ്പിക്കപ്പെട്ടത്. ഈ ഒമ്പത് ദിവസങ്ങള്‍ക്കിടക്ക് അത് ഗ്രന്ഥരൂപത്തിലാക്കുക പ്രയാസകരമാണെന്ന് പറയേണ്ടതില്ലല്ലോ. ഖുര്‍ആന്‍ പൂര്‍ണമായും മനഃപാഠമാക്കിയ ഒട്ടനവധി അനുചരന്മാര്‍ ഉണ്ടായിരുന്നതുകൊണ്ടും തുകല്‍ ചുരുളുകളിലും മറ്റു പല വസ്തുക്കളിലുമായി ഖുര്‍ആന്‍ മുഴുവനായി എഴുതിവെച്ചിരുന്നുവെന്നതുകൊണ്ടും ഖുര്‍ആനിനെ സംരക്ഷിക്കുകയെന്നത് പടച്ചതമ്പുരാന്‍തന്നെ ഒരു ബാധ്യതയായി ഏറ്റെടുത്തതു കൊണ്ടും അതൊരു പുസ്തക രൂപത്തിലാക്കാതിരുന്നത് ഒരു വലിയ പ്രശ്‌നമായി പ്രവാചകന്‍(സ) കരുതിയിരുന്നില്ല എന്നുപറയുന്നതാവും ശരി.

ഖുര്‍ആന്‍ അവതരണത്തോടൊപ്പംതന്നെ ക്രോഡീകരണവും നടന്നിരുന്നു. വിശുദ്ധ ഖുര്‍ആന്‍ അവതരിപ്പിച്ച പടച്ചതമ്പുരാന്‍തന്നെ അതിന്റെ ക്രോഡീകരണം തന്റെ ബാധ്യതയായി ഏറ്റെടുത്തിരിക്കുന്നുവെന്നതാണ് വാസ്തവം. അല്ലാഹു പറയുന്നു: ''തീര്‍ച്ചയായും അതിന്റെ (ഖുര്‍ആന്റെ) സമാഹരണവും അത് ഓതിത്തരലും നമ്മുടെ ബാധ്യതയാകുന്നു. അങ്ങനെ നാം അത് ഓതിത്തന്നാല്‍ ആ ഓത്ത് നീ പിന്തുടരുക'' (75:17,18).

മുഹമ്മദി(സ)ന് ഓരോ സൂക്തവും അവതരിപ്പിക്കപ്പെടുമ്പോള്‍ അത് എത്തിച്ചുകൊടുക്കുന്ന ജിബ്‌രീല്‍(റ)തന്നെ അത് ഏത് അധ്യായത്തില്‍ എത്രാമത്തെ വാക്യമായി ചേര്‍ക്കേണ്ടതാണെന്നുകൂടി അദ്ദേഹത്തെ അറിയിച്ചിരുന്നു. ഖുര്‍ആന്‍ എഴുതിവെക്കുന്നതിനുവേണ്ടി സന്നദ്ധ രായ പ്രവാചകാനുചരന്മാര്‍ 'കുത്താബുല്‍ വഹ്‌യ്' (ദിവ്യബോധനത്തിന്റെ എഴുത്തുകാര്‍) എന്നാണ് അറിയപ്പെട്ടിരുന്നത്. അന്‍സാറുക ളില്‍പെട്ട ഉബയ്യ്ബ്‌നു കഅ്ബ്, മുആദുബ്‌നു ജബല്‍, സൈദുബ്‌നുസാബിത്ത്, അബൂസൈദ്(റ) എന്നിവരായിരുന്നു അവരില്‍ പ്രധാനികള്‍. തുകല്‍ കഷ്ണങ്ങളിലായിരുന്നു അവര്‍ പ്രധാനമായും ഖുര്‍ആന്‍ എഴുതിവെച്ചിരുന്നത്. പ്രവാചക(സ)ന്ന് ഏതെങ്കിലും സൂക്തം അവതരിപ്പിക്കപ്പെട്ടാല്‍ അദ്ദേഹം ഈ എഴുത്തുകാരെ വിളിക്കും. ജിബ്‌രീല്‍ അദ്ദേഹത്തോട് നിര്‍ദേശിച്ച ക്രമം അദ്ദേഹം എഴുത്തുകാരോട് പറയും. അഥവാ ഈ സൂക്തങ്ങള്‍ ഏത് അധ്യായത്തില്‍ എത്രാമത്തെ വചനങ്ങളായി ചേര്‍ക്കണമെന്നും നിര്‍ദേശം നല്‍കും. ഇതു പ്രകാരം അവര്‍ എഴുതിവെക്കും. ഇങ്ങനെ, പ്രവാചക(സ)ന്റെ കാലത്തുതന്നെ -ഖുര്‍ആന്‍ അവതരണത്തോടൊപ്പം തന്നെ- അതിന്റെ ക്രോഡീകരണവും നടന്നിരുന്നുവെന്നതാണ് വാസ്തവം.

ഇവ്വിഷയകമായി നിവേദനം ചെയ്യപ്പെട്ട ഏതാനും ഹദീസുകള്‍ കാണുക: ഉസ്മാന്‍(റ) നിവേദനം ചെയ്യുന്നു: ''ദൈവദൂതന് (സ) ഒരേ അവസ രത്തില്‍ വിവിധ അധ്യായങ്ങള്‍ അവതരിക്കാറുണ്ടായിരുന്നു. അങ്ങനെ അവതരിപ്പിക്കപ്പെട്ടാല്‍ അദ്ദേഹം എഴുത്തുകാരെ വിളിച്ച് ഈ ആയത്തുകള്‍ ഇന്ന വിഷയം പ്രതിപാദിക്കുന്ന ഇന്ന സൂറത്തില്‍ രേഖപ്പെടുത്തുകയെന്ന് കല്‍പിക്കുമായിരുന്നു'' (തുര്‍മുദി, അഹ്മദ്).

''ജിബ്‌രീല്‍ എല്ലാ വര്‍ഷവും പ്രവാചക(സ)ന് ഒരു പ്രാവശ്യം ഖുര്‍ആന്‍ കേള്‍പ്പിക്കാറുണ്ടായിരുന്നു. അദ്ദേഹം മരണപ്പെട്ട വര്‍ഷത്തിലാകട്ടെ രണ്ടു പ്രാവശ്യം കേള്‍പ്പിക്കുകയുണ്ടായി'' (ബുഖാരി).

ഓരോ സൂക്തവും അവതരിപ്പിക്കപ്പെടുമ്പോള്‍തന്നെ അത് ഏത് സൂറത്തിലെ എത്രാമത്തെ വാക്യമാണെന്ന ദൈവിക നിര്‍ദേശമുണ്ടാവുന്നു. അത് പ്രകാരം എഴുതിവെക്കാന്‍ പ്രവാചകന്‍(സ) എഴുത്തുകാരോട് നിര്‍ദേശിക്കുന്നു. എല്ലാ വര്‍ഷവും ജിബ്‌രീല്‍(റ) വന്ന് അതുവരെ അവതരിപ്പിക്കപ്പെട്ട സൂക്തങ്ങള്‍ ക്രമത്തില്‍ ഓതിക്കേള്‍പ്പിക്കുന്നു. അത് പ്രവാചകന്‍ (സ) കേള്‍ക്കുന്നു. ശേഷം പ്രവാചകന്‍ ജിബ്‌രീലിനെ ഓതികേള്‍പ്പിക്കുന്നു. അങ്ങനെ ഖുര്‍ആനിന്റെ ക്രമത്തിന്റെ കാര്യത്തിലുള്ള ദൈവിക നിര്‍ദേശം പൂര്‍ണമായി പാലിക്കാന്‍ പ്രവാചകന് (സ) സാധിച്ചിരുന്നു. 'തീര്‍ച്ചയായും അതിന്റെ സമാഹരണവും പാരായണവും നമ്മുടെ ബാധ്യതയാകുന്നു''(75:17)വെന്ന ദൈവിക സൂക്തത്തിന്റെ സത്യസന്ധമായ പുലര്‍ച്ചയാണ് നമുക്കിവിടെ കാണാന്‍ കഴിയുന്നത്.

ല്ല. വിശുദ്ധ ഖുര്‍ആന്‍ പൂര്‍ണമായും ഒറ്റപ്രാവശ്യമായിട്ടല്ല, അല്‍പാല്‍പമായിട്ടാണ് അവതരിപ്പിക്കപ്പെട്ടത്. നീണ്ട ഇരുപത്തിമൂന്ന് വര്‍ഷങ്ങള്‍ക്കിടയ്ക്കായി വ്യത്യസ്ത സാഹചര്യങ്ങളിലായിട്ടാണ് അതിലെ സൂക്തങ്ങളുടെ അവതരണം നടന്നത്. പ്രവാചകന് ലഭിച്ചുകൊ ണ്ടിരുന്ന ദിവ്യബോധനത്തിന് നിര്‍ണിതമായ ഇടവേളകളൊന്നുമുണ്ടായിരുന്നില്ല. ഒരു ദിവസംതന്നെ ഒന്നിലധികം തവണ ദിവ്യബോധനം ലഭിച്ച സന്ദര്‍ഭങ്ങളുണ്ട്. ചിലപ്പോള്‍ ചില വചനങ്ങള്‍ മാത്രമാണ് അവതരിപ്പിക്കപ്പെടുക. ഏതെങ്കിലുമൊരു അധ്യായത്തില്‍ പ്രത്യേക ഭാഗത്ത് ചേര്‍ക്കുവാന്‍ വേണ്ടി നിര്‍ദേശിക്കപ്പെട്ട വചനങ്ങള്‍ മുഴുവനായി ഒറ്റസമയംതന്നെ അവതരിപ്പിക്കപ്പെട്ട അധ്യായങ്ങളുമുണ്ട്. അവസരങ്ങള്‍ക്കും സന്ദര്‍ഭങ്ങള്‍ക്കുമനുസരിച്ച് അവതരിപ്പിക്കപ്പെട്ട ഒരുപാട് സൂക്തങ്ങളുടെ സമുച്ചയമാണ് ഖുര്‍ആന്‍.

മുഹമ്മദി(സ)ന്റെ കാലത്തുതന്നെ അവിശ്വാസികള്‍ ചോദിച്ചിരുന്ന ഒരു ചോദ്യമായിരുന്നു ഇത്.ഖുര്‍ആന്‍ പറയുന്നതു കാണുക: ''സത്യനി ഷേധി കള്‍ പറഞ്ഞു. ഇദ്ദേഹത്തിന് ഖുര്‍ആന്‍ ഒറ്റത്തവണയായി ഇറക്കപ്പെടാത്തതെന്താണെന്ന്. അത് അപ്രകാരം (ഘട്ടങ്ങളിലായി അവത രിപ്പിക്കുക)തന്നെയാണ് വേണ്ടത്. അതുകൊണ്ട് നിന്റെ ഹൃദയത്തെ ഉറപ്പിച്ചുനിര്‍ത്താന്‍വേണ്ടിയാകുന്നു. ശരിയായ സാവകാശത്തോടെ നാമത് പാരായണം ചെയ്ത് കേള്‍പിക്കുകയും ചെയ്തിരിക്കുന്നു''(25:32).

''നീ ജനങ്ങള്‍ക്ക് സാവകാശത്തില്‍ ഓതിക്കൊടുക്കേണ്ടതിനായി ഖുര്‍ആനെ നാം (പല ഭാഗങ്ങളായി) വേര്‍തിരിച്ചിരിക്കുന്നു. നാം അതിനെ ക്രമേണയായി ഇറക്കുകയും ചെയ്തിരിക്കുന്നു''(17:106).

ഈ സൂക്തങ്ങളില്‍നിന്ന് എന്തുകൊണ്ടാണ് ഖുര്‍ആന്‍ ഒന്നിച്ച് ഗ്രന്ഥരൂപത്തില്‍ അവതരിപ്പിക്കാതിരുന്നതെന്ന് നമുക്ക് മനസ്സിലാകുന്നു. അവസാന നാളുവരെയുള്ള മുഴുവന്‍ മനുഷ്യര്‍ക്കും മാര്‍ഗദര്‍ശനം നല്‍കേണ്ട വേദഗ്രന്ഥമാണ് ഖുര്‍ആന്‍. അതു തോറെയെപ്പോലെ കേവലം കുറെ നിയമങ്ങളുടെ സംഹിതയല്ല. വിശ്വാസ പരിവര്‍ത്തനത്തിലൂടെ ഒരു സമൂഹത്തെ എങ്ങനെ വിമലീകരിക്കാമെന്ന് പ്രായോ ഗികമായി കാണിച്ചുതരുന്ന ഗ്രന്ഥമാണത്. ഖുര്‍ആനിന്റെ അവതരണത്തിനനുസരിച്ച് പരിവര്‍ത്തിതമായിക്കൊണ്ടിരുന്ന ഒരു സമൂഹത്തി ന്റെ ചിത്രം നമ്മുടെ മുന്നിലുണ്ട്. ഈ ചിത്രം കൂടി ഉപയോഗിച്ചുകൊണ്ടാണ് ഓേരാ സൂക്തത്തിന്റെയും പൂര്‍ണമായ ഉദ്ദേശ്യം നാം മനസ്സി ലാക്കുന്നത്. ഒറ്റയടിക്കാണ് ഖുര്‍ആന്‍ അവതരിപ്പിക്കപ്പെട്ടതെങ്കില്‍ ഈ രൂപത്തില്‍ നമുക്ക് അത് മനസ്സിലാക്കുവാന്‍ കഴിയുകയില്ലായി രുന്നു. അത് നൂറുശതമാനം പ്രായോഗികമായ ഒരു ഗ്രന്ഥമാണെന്ന് പറയുവാനും സാധിക്കുമായിരുന്നില്ല. വളര്‍ന്നുകൊ ണ്ടിരുന്ന ഒരു സമൂ ഹത്തിന് ഘട്ടങ്ങളായി നല്‍കിയ മാര്‍ഗനിര്‍ദേശങ്ങളെന്ന നിലയ്ക്ക് -പ്രസ്തുത മാര്‍ഗനിര്‍ദേശങ്ങള്‍ക്കൊപ്പം ആ സമൂഹം മാറിക്കൊണ്ടിരു ന്നു -അത് പൂര്‍ണമായും പ്രായോഗികമാണെന്ന് നമുക്ക് ഉറപ്പിച്ചുപറയുവാനാകും.

ഖുര്‍ആന്‍ ഘട്ടങ്ങളായി അവതരിപ്പിക്കപ്പെട്ടതുകൊണ്ടുള്ള ഗുണങ്ങളെ ഇങ്ങനെ സംഗ്രഹിക്കാം.

  1. ദീര്‍ഘകാലമായി സമൂഹത്തില്‍ നിലനിന്നിരുന്ന ദുരാചാരങ്ങളും അധാര്‍മികതകളും ഒറ്റയടിക്ക് നിര്‍ത്തലാക്കുക പ്രയാസകരമാണ്. ഘട്ടങ്ങളായി മാത്രമേ അവ നിര്‍ത്തല്‍ ചെയ്യാനാകൂ. താല്‍ക്കാലിക നിയമങ്ങള്‍ വഴി പ്രസ്തുത പ്രവര്‍ത്തനങ്ങളില്‍നിന്ന് ജനങ്ങളെ ക്രമേണ അകറ്റിക്കൊണ്ട് അവസാനം സ്ഥിരമായ നിയമങ്ങള്‍ നടപ്പില്‍വരുത്തുകയാണ് പ്രായോഗികം. ഇതിന് ഘട്ടങ്ങളായുള്ള അവതരണം സൗകര്യം നല്‍കുന്നു.
  2. ജനങ്ങളില്‍നിന്നും ഉയര്‍ന്നുവരുന്ന സംശയങ്ങള്‍ക്കും അപ്പപ്പോഴുണ്ടാവുന്ന പ്രശ്‌നങ്ങള്‍ക്കും തദവസരത്തില്‍തന്നെ പരിഹാരമുണ്ടാ വുന്നരീതിയില്‍ ദൈവിക സന്ദേശങ്ങള്‍ ലഭിക്കുന്നത് പ്രബോധിത ജനതയില്‍ കൂടുതല്‍ ഫലപ്രദമായ പരിവര്‍ത്തനങ്ങളുണ്ടാവുന്നതിന് നിമിത്തമാകുന്നു.
  3. ഒറ്റപ്രാവശ്യമായി അവതരിപ്പിക്കപ്പെടുന്ന പക്ഷം അതിലെ നിയമനിര്‍ദേശങ്ങള്‍ ഒരൊറ്റ ദിവസംതന്നെ നടപ്പിലാക്കേണ്ടതായി വരും. അത് പ്രയാസകരമാണെന്ന് പറയേണ്ടതില്ലല്ലോ. ഘട്ടങ്ങളായുള്ള അവതരണം വഴി ഈ പ്രയാസം ഇല്ലാതാക്കുവാനും ക്രമേണ പൂര്‍ണ മായി വിമലീകരിക്കപ്പെട്ട ഒരു സമൂഹത്തെ വളര്‍ത്തിയെടുക്കുവാനും സാധിക്കുന്നു.
  4. ഇടയ്ക്കിടക്ക് ദൈവിക ബോധനം ലഭിക്കുന്നത് പ്രവാചകന് മനഃസമാധാനവും ഹൃദയദാര്‍ഢ്യവുമുണ്ടാവുന്നതിന് കാരണമാവുന്നു.
  5. നിരക്ഷരനായ മുഹമ്മദി(സ)ന് ഖുര്‍ആന്‍ പഠിക്കുവാനും മനഃപാഠമാക്കുവാനും ഘട്ടങ്ങളായുള്ള അവതരണം സൗകര്യം നല്‍കുന്നു. മറവിയോ അബദ്ധങ്ങളോ ഇല്ലാതിരിക്കുന്നതിനും ഇത് അവസരമൊരുക്കുന്നു.
  6. പ്രവാചകന്റെ അനുയായികള്‍ക്ക് ഖുര്‍ആന്‍ മനഃപാഠമാക്കുന്നതിനും അതിലെ വിഷയങ്ങള്‍ വ്യക്തമായി പഠിക്കുന്നതിനും അതനുസരി ച്ച് ജീവിതം ക്രമപ്പെടുത്തുന്നതിനും കുറേശ്ശെയുള്ള അവതരണം വഴി സാധിക്കുന്നു.

ല്ലാഹു ജിബ്‌രീലിലൂടെ (അ) നബി (സ)ക്ക് ഏഴു ശൈലികളിൽ (ഹർഫുകൾ) ക്വുർആൻ അവതരിപ്പിച്ചതായി സ്വഹീഹായ നിരവധി ഹദീഥുകളാൽ സ്ഥിരീകരിക്കപ്പെട്ടിട്ടുണ്ട്. ഏഴ് ശൈലികളിൽ അവതരിപ്പിക്കപ്പട്ട ക്വുര്‍ആന്‍ വചനങ്ങള്‍ പാരായണം ചെയ്യേണ്ട വ്യത്യസ്ത രീതികളും (ഖിറാഅത്ത്) പ്രവാചകന്‍(സ) തന്നെ പഠിപ്പിച്ചിരുന്നു. പ്രവാചകനില്‍(സ) നിന്ന് വ്യത്യസ്തരീതികളിലുള്ള ക്വുര്‍ആന്‍ പാരായണം പഠിച്ച സ്വാഹാബിമാര്‍ ആ രീതികളെല്ലാം അടുത്ത തലമുറയ്ക്കും പഠിപ്പിച്ചു കൊടുത്തതായി കാണാനാവും. ഒരേ ഉഥ്മാനീമുസ്ഹഫ് തന്നെ വ്യത്യസ്ത രീതികളില്‍ പാരായണം ചെയ്യുന്ന സമ്പ്രദായങ്ങള്‍ വളര്‍ന്നുവന്നത് അങ്ങനെയാണ്. പ്രസിദ്ധരും പാരായണ രീതികളെപ്പറ്റി കൃത്യമായി അറിയാവുന്നവരുമായ പാരായണ വിദഗ്ധരിലൂടെയും അതല്ലാത്ത അറിയപ്പെടുന്ന പാരായണക്കാരിലൂടെയുമാണ് പ്രവാചകനില്‍ നിന്ന് നേരിട്ട് ക്വുര്‍ആന്‍ പഠിച്ചിട്ടില്ലാത്തവര്‍ അതിന്റെ പാരായണ രീതി അഭ്യസിച്ചത്. വ്യത്യസ്ത തരം പാരായണ രീതികളില്‍ നിന്ന്,  പ്രവാചകനില്‍ നിന്ന് നിരവധി പേരുള്‍ക്കൊള്ളുന്ന ശൃംഖലകളിലൂടെ നിവേദനം ചെയ്യപ്പെട്ട മുതവാത്തിറായ(നിവേദക ശൃംഖലയിലെ ഓരോ കണ്ണിയിലും നിരവധി പേര്‍ ഉള്‍ക്കൊള്ളുന്ന, അബദ്ധങ്ങള്‍ കടന്നു വരാന്‍ യാതൊരു സാധ്യതയുമില്ലാത്ത നിവേദകന്മാരുടെ പരമ്പരകള്‍ക്കാണ് മുതവാതിര്‍ എന്നു പറയുന്നത്.) രീതികള്‍ മാത്രമാണ് അംഗീകരിക്കപ്പെടുന്നത്. ഇങ്ങനെ അംഗീകരിക്കപ്പെട്ട പത്ത് പാരായണ രീതികളാണുള്ളത്.

മുതവാത്തിറായ പത്ത് തരം പാരായണ രീതികളെക്കുറിച്ചും സമഗ്രമായി വിവരിക്കുന്ന ഗ്രന്ഥമാണ് ഹിജ്‌റ 833-ാം വര്‍ഷം അന്തരിച്ച ഇബ്‌നുല്‍ ജസരിയെന്ന് അറിയപ്പെടുന്ന അല്‍ഹാഫിദ് അബുല്‍ഖൈറ് മുഹമ്മദ് ബ്‌നു മുഹമ്മദ് അല്‍ ദിമശ്ഖിയുടെ അന്നശ്‌റ് ഫില്‍ ക്വിറാആത്തില്‍ അശ്ര്‍ എന്ന ബൃഹത്തായ ഗ്രന്ഥം.(അല്‍ ഹാഫിദ് അബുല്‍ ഖൈര്‍ മുഹമ്മദ് ബിന്‍ മുഹമ്മദ് അല്‍ ദിമഷ്ഖി ഇബ്‌നുല്‍ ജസരി: അന്നശ്ര്‍ ഫില്‍ ഖിറാആത്തില്‍ അശര്‍, ബൈറൂത്ത്, ലബനാന്‍.) ഇവ്വിഷയകമായി നിരവധി രചനകള്‍ മുസ്‌ലിം ലോകത്തുണ്ടായിട്ടുണ്ട്; പ്രവാചകന്‍(സ) പഠിപ്പിച്ച എല്ലാ രീതികളിലുമുള്ള പാരായണരീതികളില്‍ ക്വുര്‍ആന്‍ അവസാനനാളുവരെ പാരായണം ചെയ്യപ്പെടുന്ന അവസ്ഥ നിലനില്‍ക്കണമെന്ന് മുസ്‌ലിം സമൂഹം കരുതുന്നതിനാലാണ് ഇത്തരം ഗ്രന്ഥങ്ങളുണ്ടാവുന്നത്. 1994ല്‍ സിറിയയിലെ ദാറുല്‍ മുഹാജിര്‍ പ്രസാധനാലയം പ്രസിദ്ധീകരിച്ച അലവി ബിന്‍ മുഹമ്മദ് ബിന്‍ അഹ്മദ് അല്‍ ഫഖീഹ് യുടെ അല്‍ ക്വിറാഅത്ത് അല്‍ അശറല്‍ മുതവാത്തിറ’എന്ന ഗ്രന്ഥം ഇവ്വിഷയകമായി പുറത്തിറങ്ങിയ താരതമ്യേന പുതിയ ഗ്രന്ഥങ്ങളിലൊന്നാണ്.(അലവി ബിന്‍ മുഹമ്മദ് ബിന്‍ അഹ്മദ് ബില്‍ഫഖീഹ്: അല്‍ ഖിറാആത്തുല്‍ അശ്‌റുല്‍ മുതവാതിറ, ദാറുല്‍ മുഹാജിര്‍, 1994)

നിലനില്‍ക്കുന്ന വ്യത്യസ്ത ഖിറാഅത്തുകള്‍ ക്വുര്‍ആനിന്റെ അഖണ്ഡതയെയും ബാധിക്കുമെന്ന് വിചാരിച്ച് അവയെക്കുറിച്ച അറിവുകള്‍ പൂഴ്ത്തിവെക്കുകയല്ല, അടുത്ത തലുമുറകള്‍ക്ക് പകര്‍ന്നു കൊടുക്കുകയാണ് പ്രസ്തുത വിഷയത്തില്‍ വിവരമുള്ള പണ്ഡിതന്മാര്‍ ചെയ്തുവന്നിട്ടുള്ളതെന്ന് ഈ രചനകള്‍ നമ്മെ ബോധ്യപ്പെടുത്തുന്നുണ്ട്.

മദീനയില്‍ നാഫിഅ്ബ്‌നു അബ്ദിര്‍റഹ്മാന്‍, മക്കയില്‍ നിന്നുള്ള അബ്ദുല്ലാ ഇബ്‌നു കഥീര്‍ അദ്ദാരി, ദമാസ്‌കസില്‍ നിന്നുള്ള അബ്ദുല്ലാഹിബ്‌നു ആമിര്‍, ബസ്വറയില്‍ നിന്നുള്ള അബൂഅംറിബ്‌നു അലാഅ്, യഅ്ഖൂബ് ബ്‌നു ഇസ്ഹാഖ് അല്‍ ഹദ്‌റമി, കൂഫയില്‍ നിന്നുള്ള ആസ്വിം ബിന്‍ അബി അന്നജൂദ് അല്‍ അസദി, ഹംസബിന്‍ ഹബീബ് അത്തൈമി, അലിബിന്‍ ഹംസ അല്‍ അസദി അല്‍ കിസാഇ, ബസ്വറയില്‍ നിന്നുള്ള അബൂ ജാഫര്‍ യസീദുബ്‌നു അല്‍ഖാഖാ അല്‍ മഖ്‌സൂമി, ബാഗ്ദാദില്‍ നിന്നുള്ള അബൂമുഹമ്മദ് അല്‍ അസദി ഖലഫ് എന്നിവ രാണ് പ്രവാചകനില്‍ നിന്ന് മുതവാത്തിറായി പാരായണരീതികള്‍ നിവേദനം ചെയ്ത പണ്ഡിതന്മാര്‍. ഇവരിലൂടെ അറിയപ്പെടുന്ന പാരായണരീതികള്‍ മുഹമ്മദ് നബി(സ)യില്‍ നിന്ന് നിരവധി അനുചരന്മാരിലൂടെ നിവേദനം ചെയ്യപ്പെടുകയും അവരിലൂടെ നിരവധിപേര്‍ പഠിക്കുകയും അങ്ങനെ ഈ പണ്ഡിതന്മാര്‍ക്കടുത്ത് എത്തിപ്പെടുകയും ചെയ്തതാണ് എന്നര്‍ഥം.

ഉദാഹരണത്തിന് ഏറെ പ്രസിദ്ധമായ വര്‍ഷ്, ഖാലൂന്‍ എന്നീ പാരായണരീതികള്‍ പഠിപ്പിച്ച മദീനക്കാരനായ നാഫിഅ്ബ്‌നു അബ്ദുര്‍റഹ്മാനിന്റെ കാര്യമെടുക്കുക. പ്രവാചകനില്‍(സ) നിന്ന് ഉബയ്യുബ്‌നു കഅ്ബും അദ്ദേഹത്തില്‍ നിന്ന് സ്വഹാബിമാരായ അബൂഹുറയ്‌റ,(റ) ഇബ്‌നുഅബ്ബാസ്‌,(റ) എന്നിവരും പഠിച്ചെടുത്തതാണ് ഈ രീതികള്‍. അവരില്‍ നിന്ന് അബ്ദുല്ലാഹിബ്‌നു അയ്യാശ്ബ്‌നു അബീറബീഅത്ത് അല്‍ മഖ്‌സൂമി, യസീദ്ബ്‌നു അല്‍ ഖഅ്ഖാഅ്, അബ്ദുര്‍റഹ്മാനു ബ്‌നു ഹുര്‍മുസ് അല്‍ അഅ്‌റജ്, മുസ്‌ലിമുബ്‌നു ജുന്‍ദുബ് അല്‍ ഹുദലി, യസീദ്ബ്‌നു റുമാന്‍, ശൈബാബ്ന്‍ നിസ്വാഹ് എന്നീ താബിഉകള്‍ ഈ പാരായണരീതികള്‍ പഠിച്ചിട്ടുണ്ട്. അവരില്‍ നിന്നാണ് ഹിജ്‌റ 70ല്‍ ജനിക്കുകയും 117ല്‍ മരണപ്പെടുകയും ചെയ്ത നാഫിഅ് അല്‍ മദനി ഇത് പഠിച്ചെടുത്തത് (Abu Muhammad Ali Ibn Ahmad Ibn Said Ibn Hazm al-Andalusi(384-456 H):  Ar-Rasa'il al-Khamsah (A Booklet In Magazi-ne Al-Azhar), 1993, p. 7. Quoted by M S M Saifullah: “Versions Of  the Qur'an?”  http://www.islamic-awareness.org)

ഇമാം നാഫിഅ് എഴുപതോളം താബിഉകളില്‍ നിന്ന് പാരായണം നേരിട്ട് പഠിച്ച മഹദ് വ്യക്തിയാണ്.(അന്നശ്‌റ് ഫില്‍ ഖിറാആത്തില്‍ അശ്‌റ് (1/112))

അദ്ദേഹത്തിന്റെ പേരില്‍ അറിയപ്പെടുകയും അദ്ദേഹത്തിലൂടെ നിരവധി പഠിതാക്കള്‍ പഠിച്ചെടുക്കുകയും ചെയ്ത വര്‍ഷ്, ഖാലൂന്‍ പാരായണരീതികള്‍ പ്രവാചകന്‍(സ) തന്നെ പഠിപ്പിച്ചുകൊടുത്തതാണെന്ന വസ്തുത സംശയാതീതമായി തെളിയിക്കാനാവുമെന്നാണ് ഇത് വ്യക്തമാക്കുന്നത്. ഇതേ പോലെത്തന്നെയാണ് മുതവാത്തിറായ പത്ത് പാരായണ രീതികളുടെയും സ്ഥിതി.

വ്യത്യസ്ത പാരായണ ശൈലികളെയും (ഹര്‍ഫ്) പാരായണ രീതികളെയും (ക്വിറാഅത്ത്) കുറിച്ച തെറ്റുധാരണകൊണ്ടാണ് ചിലര്‍ ക്വുര്‍ആനിന് പാഠഭേദങ്ങളുണ്ടെന്ന് വാദിക്കുന്നത്. ഹര്‍ഫും ക്വിറാഅത്തും ഒന്നു തന്നെയാണെന്നാണ് അവര്‍ തെറ്റുധരിച്ചിരിക്കുന്നത്. ക്വുര്‍ആനിന്റെ വ്യത്യസ്ത ശൈലികളാണ് ഏഴു ഹര്‍ഫുകള്‍. ക്വുര്‍ആന്‍ അവതരിക്കപ്പെട്ടത് ഖുറൈശികളുടെ ഹര്‍ഫിലായിരുന്നുവെന്നും തന്റെ സമുദായത്തിലെ വ്യത്യസ്ത വിഭാഗക്കാര്‍ക്ക് എളുപ്പത്തില്‍ പാരായണം ചെയ്യുന്നതിനുവേണ്ടി, നബി(സ) ആവശ്യപ്പെട്ട് നേടിയെടുത്തതാണ് ഏഴു ഹര്‍ഫുകളിലുള്ള പാരായണമെന്നും സ്വഹീഹായ ഹദീഥുകള്‍ വ്യക്തമാക്കുന്നുണ്ട്.(സ്വഹീഹുല്‍ ബുഖാരി, കിതാബു ഫദാഇലില്‍ ക്വുര്‍ആന്‍; സ്വഹീഹു മുസ്‌ലിം, കിതാബു ഫദാഇലില്‍ ക്വുര്‍ആന്‍ വ മാ യതഅല്ലഖ ബിഹി; സ്വഹീഹുല്‍ ബുഖാരി, കിതാബുല്‍ ഖുസ്വൂമാത്; സ്വഹീഹുല്‍ ബുഖാരി, കിതാബു ബദ്ഉല്‍ ഖല്‍ഖ്; ജാമിഉ ത്തിര്‍മിദി, കിതാബുല്‍ ക്വിറാആത്; മുസ്‌നദ് ഇമാം അഹ്മദ്, 5/132 ഹദീഥ്: 21523)

വ്യത്യസ്ത ഗോത്രങ്ങളില്‍ നിന്ന് ഇസ്‌ലാം സ്വീകരിച്ച വ്യത്യസ്ത നിലവാരത്തിലുള്ളവര്‍ക്കെല്ലാം ക്വുര്‍ആന്‍ പാരായണത്തിനും മനഃപാഠമാക്കുന്നതിനും എളുപ്പത്തില്‍ സാധിച്ചു. അതോടൊപ്പം തന്നെ ക്വുര്‍ആനിലെ ഒരു അധ്യായത്തിന് തുല്യമായ ഒരു അധ്യായമെങ്കിലും കൊണ്ടുവരാന്‍ സത്യനിഷേധികളെ വെല്ലുവിളിക്കുന്ന ക്വുര്‍ആന്‍ വചനങ്ങളുടെ(ക്വുര്‍ആന്‍ 2:23,24) അര്‍ഥവ്യാപ്തി എല്ലാവര്‍ക്കും മനസ്സിലാക്കുവാനും എല്ലാ ഗോത്രങ്ങളെയും പ്രസ്തുത വെല്ലുവിളിക്ക് വിധേയമാക്കുവാനും അതുവഴി കഴിയുകയും ചെയ്തു. ഒരൊറ്റ ശൈലിയില്‍ മാത്രമായിരുന്നു ക്വുര്‍ആന്‍ അവതരിപ്പിക്കപ്പെട്ടതെങ്കില്‍ ചിലര്‍ക്കെങ്കിലും തങ്ങള്‍ക്ക് ഈ വെല്ലുവിളി ബാധകമല്ലെന്നും ഒരൊറ്റ ശൈലിയില്‍ മാത്രമാണ് ക്വുര്‍ആന്‍ സംസാരിക്കുന്നതെന്നും പറഞ്ഞ് രക്ഷപ്പെടാമായിരുന്നു. അന്ന് നിലവിലുണ്ടായിരുന്ന ഗോത്രങ്ങള്‍ക്കൊന്നിനും ഈ വെല്ലുവിളിയുടെ പരിധിയില്‍ നിന്ന് മാറുവാന്‍ കഴിയാത്തവിധം അവരിലെ സത്യനിഷേധികളെ കുരുക്കുന്നതാണ് ഏഴു ഹര്‍ഫുകളിലായുള്ള ക്വുര്‍ആനിന്റെ അവതരണമെന്നര്‍ഥം.

ഉഥ്മാനി(റ)ന്റ ഭരണകാലത്ത് നടന്നത് ഈ ഏഴുഹര്‍ഫുകളെയും ഉള്‍ക്കൊണ്ടുകൊണ്ട്, ഖുറൈശീഭാഷയിലുള്ള മുസ്വ്ഹഫ് പകര്‍ത്തിയെ ഴുത്തായിരുന്നുവെന്നാണ് ഹദീഥുകള്‍ വ്യക്തമാക്കുന്നത്. ഏതെങ്കിലും കാര്യത്തില്‍ നിങ്ങള്‍ വിയോജിക്കുന്നുവെങ്കില്‍ അത് ഖുറൈശീ രീതിപ്രകാരം എഴുതുക; ക്വുര്‍ആന്‍ അവതരിക്കപ്പെട്ടത് ഖുറൈശീ രീതിയിലാണ് (സ്വഹീഹുല്‍ ബുഖാരി, കിതാബു ഫദാഇലില്‍ ക്വുര്‍ആന്‍) എന്ന ഉഥ്മാനുബ്‌നു അഫ്ഫാനി(റ)ന്റ നിര്‍ദേശത്തില്‍ നിന്ന് അതാണ് മനസ്സിലാവുന്നത്. അറബി ആധാര ഭാഷയായ ഖുറൈശീഭാഷയി ലുള്ള മുസ്വ്ഹഫിന്റെ പകര്‍ത്തിയെഴുത്തിനു ശേഷം വ്യത്യസ്ത ഹര്‍ഫുകള്‍ സ്വാഭാവികമായും നിലനിന്നിരുന്നില്ല. വ്യത്യസ്ത ഹര്‍ഫു കളില്‍ ക്വുര്‍ആന്‍ പാരായണം ചെയ്തുവരികയും മനഃപാഠമാക്കുകയും പഠിപ്പിക്കുകയുമെല്ലാം ചെയ്തുവന്നിരുന്ന സ്വഹാബകളിലാരും തന്നെ ഇത്തരമൊരു ഏകീകരണത്തിന് എതിരെ നിന്നിട്ടില്ലെന്ന വസ്തുത ശ്രദ്ധേയമാണ്.. ഖുറൈശീഭാഷയിലുള്ള ഹര്‍ഫുകളുടെ ഏകീക രണം വഴി ക്വുര്‍ആനിലെ ആശയങ്ങള്‍ക്കോ പദവിന്യാസത്തിനോ മാറ്റങ്ങളെന്തെങ്കിലുമുണ്ടായിട്ടുണ്ടെങ്കില്‍ അവര്‍ അതിനു സമ്മതിക്കില്ലാ യിരുന്നു. സ്വഹാബിമാരുടെ ഐകകണ്ഠമായ അംഗീകാരത്തോടെ പകര്‍ത്തിയെഴുതിയ മുസ്വ്ഹഫുകളാണ് ഉഥ്മാനി(റ)ന്റ കാലം മുതല്‍ ഇന്നുവരെ ലോകത്തെല്ലായിടത്തും ഉപയോഗിച്ചുവന്നിട്ടുള്ളത്.

ഖുറൈശീഭാഷയില്‍ രേഖീകരിക്കപ്പെട്ട വിശുദ്ധ ക്വുര്‍ആനിന്റെ വ്യത്യസ്ത പാരായണങ്ങളാണ് ക്വിറാഅത്തുകള്‍. നടേ സൂചിപ്പിച്ച മുതവാത്തിറായ പത്തു ക്വിറാഅത്തുകളില്‍ ഏഴെണ്ണമാണ് ഏറെ പ്രസിദ്ധമായവ. നാഫിഅ് അല്‍ മദനി, ഇബ്‌നുകഥീര്‍ അല്‍മക്കി, അബൂ അംറുബ്‌നുല്‍ അലാഅ് അല്‍ ബസ്വ്‌രി, ഇബ്‌നു ആമിര്‍ അദ്ദിമശ്ക്കി, ആസിം അല്‍ കൂഫി, ഹംസാ അല്‍ കൂഫി, അല്‍ കിസാഈ അല്‍ കൂഫി എന്നിവരുടേതാണ് അവ. പ്രസിദ്ധമായ ക്വിറാഅത്തുകളുടെ എണ്ണവും ഹര്‍ഫുകളുടെ എണ്ണവും ഏഴ് ആയതുകൊണ്ടാണെന്ന് തോന്നുന്നു, ഹര്‍ഫും ഖിറാഅത്തും ഒന്നു തന്നെയാണെന്ന് ചിലരെല്ലാം തെറ്റുധരിച്ചിട്ടുണ്ട്. വ്യത്യസ്ത ഹര്‍ഫുകളെ ഉള്‍ക്കൊണ്ടുകൊണ്ട് ഖുറൈശീഭാഷയില്‍ ഉഥ്മാന്‍ (റ)ക്രോഡീകരിച്ച ക്വുര്‍ആനിന്റെ വ്യത്യസ്തങ്ങളായ പാരായണ രീതികളാണ് ക്വിറാഅത്തുകള്‍.

നബി(സ)യില്‍ നിന്ന് മുതവാത്തിറായി നിവേദനം ചെയ്യപ്പെട്ട പത്ത് ഖിറാഅത്തുകളില്‍ കാണപ്പെടുന്ന സൂക്ഷ്മമായ വ്യതിരിക്തതകള്‍, വ്യത്യസ്ത ഹര്‍ഫുകളിലുള്ള അടിസ്ഥാന വ്യത്യാസങ്ങളെ ഉള്‍ക്കൊണ്ടുകൊണ്ടു തന്നെയാണ് ഉഥ്മാനി(റ)ന്റ മുസ്ഹഫ് പകര്‍ത്തിയെ ഴുത്ത് നടന്നതെന്ന് വ്യക്തമാക്കുന്നുണ്ട്. ഹര്‍ഫുകളിലെ ഗോത്രഭാഷാപ്രയോഗങ്ങളെ ഏകീകരിച്ച് ഖുറൈശീഭാഷയില്‍ ക്വുര്‍ആന്‍ ക്രോഡീകരിച്ച് ഗ്രന്ഥരൂപത്തിലാക്കിയപ്പോള്‍ തന്നെ വ്യത്യസ്തങ്ങളായ ക്വിറാഅത്തുകളിലൂടെ ഹര്‍ഫുകളിലുള്ള ആശയ വ്യത്യാസമുള്ള പ്രയോഗങ്ങളുടെ ആശയങ്ങള്‍ നിലനിര്‍ത്തുകയും ചെയ്തിട്ടുണ്ടെന്നാണ് ഇതില്‍ നിന്ന് മനസ്സിലാവുന്നത്. ക്വുര്‍ആനിലെ പ്രഥമാധ്യായമായ സൂറത്തുല്‍ ഫാത്തിഹയിലെ നാലാം വചനത്തിലെ വ്യത്യസ്ത പാരായണങ്ങള്‍ ഉദാഹരണമായെടുക്കുക. 'മാലിക്കി യൗമിദ്ദീന്‍' എന്നും 'മലിക്കി യൗമിദ്ദീന്‍' എന്നും രണ്ടു പാരായണഭേദങ്ങളുണ്ട് ഈ വചനത്തിന്. 'പ്രതിഫല നാളിന്റെ ഉടമസ്ഥന്‍' എന്ന് ആദ്യത്തേതിനും, 'പ്രതിഫലനാളിന്റെ രാജാവ്' എന്ന് രണ്ടാമത്തേതിനും യഥാക്രമം അര്‍ഥം പറയാം. ഈ രണ്ടു രൂപത്തിലുമുള്ള പാരായണത്തിന് പ്രവാചകന്റെ (സ) അംഗീകാരമുണ്ട് എന്നതിനാല്‍ ഇവ രണ്ടും ദൈവിക വെളിപാടുകളാണെന്ന് വ്യക്തമാണ്. വ്യത്യസ്ത ഖിറാഅത്തു കളിലുള്ള ഇത്തരം വ്യത്യാസങ്ങള്‍ വ്യതിരിക്തമായ ഹര്‍ഫുകളിലുണ്ടായിരുന്ന വ്യത്യാസത്തെക്കൂടി ഉള്‍ക്കൊള്ളുന്നതാണ്.

വ്യത്യസ്ത ക്വിറാഅത്തുകളിലുള്ള സൂക്ഷ്മമായ വ്യത്യാസങ്ങള്‍ ചൂണ്ടിക്കാണിച്ച് ക്വുര്‍ആനിന് പാഠഭേദങ്ങളുണ്ടെന്ന് സ്ഥാപിക്കാന്‍ ശ്രമിക്കുന്നത് ഹര്‍ഫും ഖിറാഅത്തും അല്ലാഹു തന്നെ അവതരിപ്പിച്ചതാണെന്ന ഇസ്‌ലാമിക പാഠത്തെക്കുറിച്ച് മനസ്സിലാക്കാത്തതു കൊണ്ടാണ്. മനുഷ്യരുടെ കരവിരുതുകളാല്‍ പാഠഭേദങ്ങളുണ്ടായ ബൈബിള്‍ പുസ്തകങ്ങളുടെ അവസ്ഥയല്ല ക്വുര്‍ആനിന്റേത്. അത് അവതരിക്കപ്പെട്ട രൂപത്തില്‍ തന്നെ നിലനില്‍ക്കുന്നുണ്ട്. വ്യത്യസ്ത പാരായണങ്ങളിലൂടെ പടച്ചവന്‍ അവതരിപ്പിച്ച വ്യതിരിക്തതകള്‍ പോലും ഇന്നും നിലനില്‍ക്കുന്നു; ക്വുര്‍ആനിലുള്ള പാരായണ വ്യത്യാസങ്ങളാവട്ടെ വളരെ പരിമിതവും അതിന്റെ അര്‍ഥഘടനയെ ഒരുതരത്തിലും ബാധിക്കാത്തതുമാണെന്ന് തദ്‌വിഷയകമായി പഠനം നടത്തിയിട്ടുള്ള ഓറിയന്റലിസ്റ്റ് പണ്ഡിതന്മാര്‍ പോലും സമ്മതിച്ചിട്ടുള്ളതുമാണ്. അതാണ് ഓറിയന്റലിസത്തിന്റെ പിതാക്കളിലൊരാളായ സര്‍ വില്യം മ്യൂറിന്റെ വാക്കുകളില്‍ നാം കണ്ടത്. (William Muir: Opt. Cit., Pages xxii-xxiii.)

ഏറ്റവുമധികം പ്രചാരത്തിലിരിക്കുന്ന രണ്ട് ക്വുര്‍ആന്‍ ക്വിറാഅത്തുകളായ ഹഫ്‌സിനെയും വര്‍ഷിനെയും കുറിച്ച് ഗവേഷണം നടത്തിയ യോര്‍ക്ക് സെന്റ് ജോണ്‍ യൂനിവേഴ്‌സിറ്റിയിലെ ഫാക്കല്‍ടി ഓഫ് എഡ്യുക്കേഷന്‍ ആന്റ് തിയോളിജിയുടെ പ്രിന്‍സിപള്‍ ലക്ചറും പ്രസിദ്ധ ഓറിയന്റലിസ്റ്റുമായ ഡോക്ടര്‍ ആഡ്രിയന്‍ ബ്രോക്കറ്റ് എഴുതുന്നു. 'ഹഫ്‌സ് നിവേദനത്തിലും വര്‍ഷ് നിവേദനത്തിലുമുള്ള വ്യത്യാസങ്ങള്‍, പാരായണത്തിലുള്ളതാണെങ്കിലും രേഖീകരണത്തിലുള്ളതാണെങ്കിലും വചനങ്ങളുടെ അര്‍ഥത്തെ സാരമായി ബാധിക്കുന്നില്ല എന്നത് ഒരു ലളിതമായ യാഥാര്‍ഥ്യമാണ്. പല വ്യതിരിക്തതകളും അതിന്റെ അര്‍ഥത്തില്‍ യാതൊരുവിധ മാറ്റവുമുണ്ടാക്കുന്നില്ല. രേഖകളുടെ പശ്ചാത്തലത്തില്‍ മാത്രം ചെറിയ അര്‍ഥവ്യത്യാസമുണ്ടാക്കുന്ന മറ്റു ചില വ്യതിരിക്തതകളാവട്ടെ മുസ്‌ലിം ചിന്തയെ ഏതെങ്കിലും തരത്തില്‍ ബാധിക്കാവുന്ന തരത്തില്‍ സ്വാധീനമുണ്ടാക്കുന്നവയല്ല താനും.'(Adrian Brockett: "The Extent To Which The Differences Affect The Sense'', in Andrew RÆpin (Ed.): Approaches Of The History of Interpretation Of The Qur'an, Oxford, 1988, Page 37.)

വ്യത്യസ്ത സ്ഥലങ്ങളില്‍ രൂപാന്തരപ്പെട്ട വ്യത്യസ്ത ക്വുര്‍ആനുകള്‍ തമ്മില്‍ വ്യത്യാസമുണ്ടെന്ന മട്ടിലാണ് ഇസ്‌ലാം വിമര്‍ശകര്‍ വ്യത്യസ്ത ക്വിറാഅത്തുകളെക്കുറിച്ച് പരാമര്‍ശിക്കാറുള്ളത്. മുസ്‌ലിം ലോകത്ത് ആദ്യകാലം മുതല്‍ തന്നെ വ്യത്യസ്ത പാരായണങ്ങളെക്കുറിച്ചറി യാവുന്നവരുണ്ടായിരുന്നു. നടേ പറഞ്ഞ നാടുകളില്‍ പ്രചാരത്തിലിരിക്കുന്ന വര്‍ശ് പാരായണ രീതിയുടെയും ലിബിയ, ടുണീഷ്യ, ഖത്തര്‍ തുടങ്ങിയ സ്ഥലങ്ങളില്‍ നിലനില്‍ക്കുന്ന ഖാലൂന്‍ പാരായണരീതിയുടെയും ഗുരുവായി അറിയപ്പെടുന്നത് നാഫിഅ് അല്‍ മദനി തന്നെയാണ്. മുസ്‌ലിം ലോകത്ത് പൊതുവെ പ്രചാരത്തിലിരിക്കുന്ന ഹഫ്‌സ് പാരായണ രീതിയുടെ ഗുരുനാഥനായ ആസ്വിം അല്‍ കൂഫി തന്നെയാണ് ശുഅ്ബയെന്ന് അറിയപ്പെടുന്ന പാരായണരീതിയുടെയും ഗുരു. പത്തു ഖിറാഅത്തുകളെയും വിശദീകരിക്കുന്ന അല്‍ഖിറാആത്തുല്‍ അശറല്‍ മുതവാത്തിറഃയെന്ന ഗ്രന്ഥത്തില്‍ അതു തയാറാക്കുവാന്‍ സഹായിച്ച, പത്ത് ക്വിറാഅത്തുകളിലും പ്രാവീണ്യവും അവയിലെല്ലാം ക്വുര്‍ആന്‍ മനഃപാഠവുമുള്ള മുഹമ്മദ് ഫഹദ് ഖറൂഫിനെക്കുറിച്ച് പ്രതിപാദിക്കുന്നുണ്ട് (അലവി ബിന്‍ മുഹമ്മദ് ബിന്‍ അഹ്മദ് ബല്‍ഫഖീഹ്: അല്‍ ഖിറാആത്തുല്‍ അശ്‌റുല്‍ മുതവാതിറ, പുറം ചട്ടയുടെ പിന്‍ഭാഗം) എല്ലാ ക്വിറാഅത്തുകളിലും പ്രാവീണ്യമുള്ളവര്‍ എക്കാലത്തും മുസ്‌ലിം ലോകത്ത് ജീവിച്ചിരുന്നുവെന്ന വസ്തുത വ്യക്തമാക്കുന്നതാണ്, പത്തു ക്വിറാഅത്തുകളിലും പ്രാവീണ്യമുള്ളവര്‍ പതിനാലു നൂറ്റാണ്ടുകള്‍ കഴിഞ്ഞും ജീവിച്ചിരിക്കുന്നുവെന്ന യാഥാര്‍ഥ്യമെന്നതില്‍ സംശയമില്ല.

ല്ല. ക്വുർആൻ മനുഷ്യസമൂഹത്തിന് മുന്നിൽ പാരായണം ചെയ്തു കേൾപ്പിച്ച മുഹമ്മദ് നബി (സ) തന്നെ അത് ഏഴ് ശൈലികളിൽ അവതരിക്കപ്പെട്ടതാണെന്ന് പഠിപ്പിച്ചിട്ടുണ്ട്. അല്ലാഹു ജിബ്‌രീലിലൂടെ (അ) നബി (സ)ക്ക് ഏഴു ശൈലികളിൽ (ഹർഫുകൾ) ക്വുർആൻ അവതരിപ്പിച്ചതായി സ്വഹീഹായ നിരവധി ഹദീഥുകളാൽ സ്ഥിരീകരിക്കപ്പെട്ടിട്ടുണ്ട്. ഏഴ് വ്യത്യസ്ത ശൈലികളില്‍ ക്വുര്‍ആന്‍ അവതരിക്കപ്പെട്ടതിനാല്‍ തന്നെ പലരും പാരായണം ചെയ്തിരുന്നത് പല ശൈലികളിലായിരുന്നുവെന്ന് കാണാന്‍ കഴിയും. ഏഴു ഹര്‍ഫുകളിലായാണ് അവസാനത്തെ ദൈവിക ഗ്രന്ഥം അവതരിക്കപ്പെട്ടത് എന്നറിയാതെ ചില സ്വഹാബിമാര്‍ തമ്മില്‍ ഇവ്വിഷ യക മായി നടന്ന തര്‍ക്കങ്ങളെക്കുറിച്ച വിവരണങ്ങളില്‍ നിന്ന് ഇവയെല്ലാം അല്ലാഹുവില്‍ നിന്ന് അവതരിക്കപ്പെട്ടതാണെന്നും അവന്റെ നിര്‍ദേ ശമാണ് ഇവയില്‍ ക്വുര്‍ആന്‍ പാരായണം ചെയ്യാനെന്നും ഇവയിലേതിലെങ്കിലും ഒന്നില്‍ പാരായണം ചെയ്താല്‍ മതിയെന്നും ഒന്ന് മറ്റേതി ല്‍ നിന്ന് ഉത്തമമോ അധമമോ അല്ലെന്നുമുള്ള വസ്തുതകള്‍ മനസ്സിലാവും. രണ്ട് ഹദീഥുകള്‍ കാണുക.

ഉമറുബ്‌നുല്‍ ഖത്ത്വാബില്‍ (റ) നിന്ന്: 'റസൂലിന്റെ കാലത്ത് ഹിശാമുബ്‌നു ഹകീം ഒരിക്കല്‍ 'സൂറത്തുല്‍ ഫുര്‍ഖാന്‍' ഓതുന്നത് ഞാന്‍ കേട്ടു. ഞാന്‍ അദ്ദേഹത്തിന്റെ പാരായണം ശ്രദ്ധിച്ചു. എനിക്ക് റസൂല്‍ ഓതിത്തന്നിട്ടില്ലാത്ത പലവിധ ശൈലികളിലും അദ്ദേഹം ഓതുന്നു. നമസ്‌ കാരത്തിലായിരിക്കെത്തന്നെ, അദ്ദേഹവുമായി വഴക്കിടാന്‍ എനിക്ക് തോന്നി. നമസ്‌കാരം കഴിയുംവരെ ഞാന്‍ ക്ഷമിച്ചു. നമസ്‌കാരത്തി ല്‍നിന്ന് വിരമിച്ചയുടനെ, അദ്ദേഹത്തിന്റെ തട്ടം കഴുത്തിന് ചുറ്റിപ്പിടിച്ച് ഞാന്‍ ചോദിച്ചു: 'നിങ്ങളിപ്പോള്‍ ഓതുന്നതായി ഞാന്‍ കേട്ട സൂറത്ത് നിങ്ങള്‍ക്കാരാണ് ഓതിത്തന്നത്?' അദ്ദേഹം പറഞ്ഞു: 'അല്ലാഹുവിന്റെ റസൂലാണ് എന്നെയത് ഓതിപ്പഠിപ്പിച്ചത്.' ഞാന്‍ പറഞ്ഞു: ''കള്ളം. റസൂല്‍ എനിക്ക് പഠിപ്പിച്ചുതന്നത് നിങ്ങള്‍ ഓതിയ രൂപത്തിലല്ല.' അദ്ദേഹത്തെയും പിടിച്ച് ഞാന്‍ റസൂലിന്റെ അടുത്തേക്ക് പുറപ്പെട്ടു. ഞാന്‍ റസൂലിനോട് പറഞ്ഞു: 'നിങ്ങള്‍ എനിക്ക് ഓതിത്തരാത്തവിധം സൂറത്തുല്‍ ഫുര്‍ഖാന്‍ ഇദ്ദേഹം ഓതുന്നത് ഞാന്‍ കേട്ടു.' റസൂല്‍ പറഞ്ഞു: 'അദ്ദേഹത്തെ വിട്ടേക്കൂ. ഹിശാം, നിങ്ങള്‍ ഓതൂ.' ഹിശാം ഞാന്‍ കേട്ട അതേപ്രകാരം തന്നെ ഓതി. അപ്പോള്‍ റസൂല്‍ പറഞ്ഞു: 'ഇപ്രകാരം തന്നെയാണ് ഇത് അവതരിപ്പിക്കപ്പെട്ടത്.' തുടര്‍ന്ന് അവിടുന്ന് പറഞ്ഞു: 'ഉമറേ, നിങ്ങളൊന്ന് ഓതൂ.' റസൂല്‍ എന്നെ പഠിപ്പിച്ചപോലെ ഞാന്‍ ഓതി. അപ്പോള്‍ റസൂല്‍ പറഞ്ഞു: 'ഇങ്ങനെയും ഇത് അവതരിപ്പിക്കപ്പെട്ടിട്ടുണ്ട്. ഈ ക്വുര്‍ആന്‍ ഏഴ് വ്യത്യസ്ത ശൈലികളില്‍ അവതരിപ്പിക്കപ്പെട്ടിട്ടുണ്ട്. നിങ്ങള്‍ക്ക് എളുപ്പമായ വിധം അത് ഓതിക്കൊള്ളുക.'(സ്വഹീഹുല്‍ ബുഖാരി, കിതാബു ഫദാഇലില്‍ ക്വുര്‍ആന്‍; സ്വഹീഹു മുസ്‌ലിം, കിതാബു ഫദാഇലില്‍ ക്വുര്‍ആന്‍ വ മാ യതഅല്ലഖു ബിഹി)

അബ്ദുല്ലാഹിബ്‌നു മസ്ഊദില്‍ (റ) നിന്ന്: 'നബി പാരായണം ചെയ്തതില്‍നിന്ന് വ്യത്യസ്തമായി ഒരാള്‍ ഒരു സൂക്തം പാരായണം ചെയ്തത് ഞാന്‍ കേട്ടു. ഞാന്‍ അയാളുടെ കൈക്ക് പിടിച്ച് നബിയുടെ അടുത്ത് കൊണ്ടുവന്ന് കാര്യം ബോധിപ്പിച്ചു. അപ്പോള്‍ നബി പറഞ്ഞു: നിങ്ങളി രുവരും (പാരായണം ചെയ്തത്) ശരിയാണ്. നിങ്ങള്‍ ഭിന്നിക്കരുത്. എന്തുകൊണ്ടെന്നാല്‍, നിങ്ങള്‍ക്ക് മുമ്പുള്ളവര്‍ ഭിന്നിച്ചു; അങ്ങനെ അവര്‍ നശിച്ചു.' (സ്വഹീഹുല്‍ ബുഖാരി, കിതാബുല്‍ ഖുസ്വൂമാത്)

മുഹമ്മദ് നബി(സ) യുടെ ആവശ്യപ്രകാരം അല്ലാഹു അവതരിപ്പിച്ചതാണ് ക്വുര്‍ആനിന്റെ ഈ ഏഴ് ശൈലികളുമെന്ന് നബി(സ) തന്നെ വ്യക്തമാക്കിയിട്ടുണ്ട്.

ഇബ്‌നു അബ്ബാസില്‍നിന്ന്: നബി പറഞ്ഞു: ജിബ്‌രീല്‍ ഭാഷയിലെ, ഒരു ശൈലിയിലാണ് ക്വുര്‍ആന്‍ എനിക്ക് ഓതിത്തന്നത്. ഒന്നിലധികം (ശൈലികളില്‍) വേണമെന്ന് ഞാന്‍ ആവശ്യപ്പെട്ടുകൊണ്ടിരുന്നു. അങ്ങനെ അത് ഏഴു ഹര്‍ഫുകളില്‍ എത്തിനിന്നു. (സ്വഹീഹുല്‍ ബുഖാരി, കിതാബു ബദ്ഉല്‍ ഖല്‍ഖ്)

ഉബയ്യ്ബ്‌നു കഅ്ബില്‍ (റ) നിന്ന്: അല്ലാഹുവിന്റെ ദൂതന്‍ ജിബ്‌രീലിനോട്(റ) പറഞ്ഞു: ''ഓ, ജിബ്‌രീല്‍! പ്രായമായ സ്ത്രീകളും പുരുഷന്‍മാരും ആണ്‍കുട്ടികളും പെണ്‍കുട്ടികളുമടങ്ങുന്ന ഒരു നിരക്ഷര സമുദായത്തിലേക്കാണ് ഞാന്‍ നിയോഗിക്കപ്പെട്ടിരിക്കുന്നത്. അവര്‍ക്കെല്ലാവര്‍ക്കും ഒരേ ഗ്രന്ഥം പാരായണം ചെയ്യാന്‍ കഴിയില്ല. അപ്പോള്‍ ജിബ്‌രീല്‍(റ) പറഞ്ഞു: ഓ, മുഹമ്മദ്! ക്വുര്‍ആന്‍ ഏഴു ഹര്‍ഫുകളിലാണ് (ശൈലികള്‍) അവതരിക്കപ്പെട്ടിട്ടുള്ളത്, തീര്‍ച്ച''. (ജാമിഉ ത്തിര്‍മിദി, കിതാബുല്‍ ക്വിറാആത്; മുസ്‌നദ് ഇമാം അഹ്മദ്, 5/132 ഹദീഥ്: 21523: ഇബ്‌നു ഹിബ്ബാന്‍ (ഹദീഥ് 736) ഉദ്ധരിച്ചിട്ടുള്ള ഈ ഹദീഥ് (Abu Khaliyl: Opt. Cit., page 269) ശൈഖ് ശുഐബ് അല്‍ അര്‍നാഊത്ത്വ് ഹസനാണെന്ന് വ്യക്തമാക്കിയിട്ടുണ്ട്. ഇമാം അല്‍ബാനി ഈ ഹദീഥ് ഹസനും സ്വഹീഹുമാണെന്നാണ്പറഞ്ഞിട്ടുള്ളത്:ജാമിഉത്തിര്‍മിദി, ഹദീഥ് 2942

ഉബയ്യുബ്‌നു കഅ്ബില്‍(റ) നിന്ന്: ബനൂ ഗിഫാറുകാരുടെ തടാകത്തിനരികിലിരിക്കുമ്പോള്‍ പ്രവാചകൻ(സ) ജിബ്‌രീല്‍ സന്ദര്‍ശിച്ചുകൊണ്ട് പറഞ്ഞു: താങ്കളുടെ ജനതയ്ക്ക് ക്വുര്‍ആന്‍ ഒരു പാരായണശൈലി (ഹര്‍ഫ്) യില്‍ പഠിപ്പിക്കുവാന്‍ അല്ലാഹു കല്‍പിച്ചിരിക്കുന്നു.’നബി(സ) പറഞ്ഞു: ഞാന്‍ അല്ലാഹുവിനോട് ക്ഷമ യാചിക്കുകയും പാപമോചനം തേടുകയും ചെയ്യുന്നു. എന്റെ ജനതയ്ക്ക് അതിന് കഴിയില്ല . ജിബ്‌രീല്‍ അദ്ദേഹത്തിന്റെയടുക്കല്‍ രണ്ടാമത് വന്നുകൊണ്ട് പറഞ്ഞു: താങ്കളുടെ ജനതയ്ക്ക് ക്വുര്‍ആന്‍ രണ്ടു ഹര്‍ഫുകളില്‍ പഠിപ്പിക്കു വാന്‍ അല്ലാഹു കല്‍പിച്ചിരിക്കുന്നു.’നബി(സ) പറഞ്ഞു: 'ഞാന്‍ അല്ലാഹുവിനോട് ക്ഷമ യാചിക്കുകയും പാപമോചനം തേടുകയും ചെയ്യുന്നു. എന്റെ ജനതയ്ക്ക് അതിന് കഴിയില്ല.' ജിബ്‌രീല്‍ അദ്ദേഹത്തിന്റെയടുക്കല്‍ മൂന്നാമത് വന്നുകൊണ്ട് പറഞ്ഞു: താങ്കളുടെ ജനതയ്ക്ക് ക്വുര്‍ആന്‍ മൂന്നു ഹര്‍ഫുകളില്‍ പഠിപ്പിക്കുവാന്‍ അല്ലാഹു കല്‍പിച്ചിരിക്കുന്നു. നബി(സ) പറഞ്ഞു: 'ഞാന്‍ അല്ലാഹുവിനോട് ക്ഷമ യാചിക്കുകയും പാപമോചനം തേടുകയും ചെയ്യുന്നു. എന്റെ ജനതയ്ക്ക് അതിന് കഴിയില്ല. ജിബ്‌രീല്‍ അദ്ദേഹത്തിന്റെയടുക്കല്‍ നാലാം തവണ വന്നുകൊണ്ട് പറഞ്ഞു: താങ്കളുടെ ജനതയ്ക്ക് ക്വുര്‍ആന്‍ ഏഴ് ഹര്‍ഫുകളില്‍ പഠിപ്പിക്കുവാന്‍ അല്ലാഹു കല്‍പിച്ചിരിക്കു ന്നു. ഇതില്‍ ഏതുതരം പാരായണശൈലിയില്‍ അവര്‍ പാരായണം ചെയ്താലും അത് ശരിയായിരിക്കും.'' (സ്വഹീഹു മുസ്‌ലിം, കിതാബു സ്‌സ്വലാത്ത്)

എഴുത്ത് വ്യാപകമായി നിലനിന്നിരുന്നിട്ടില്ലാത്ത കാലത്ത്, വ്യത്യസ്ത നിലവാരത്തിലുള്ളവര്‍ക്ക് ഒരേ ശൈലിയില്‍ പാരായണം പ്രയാസ കരമാണെന്നതിനാല്‍ അല്ലാഹു തന്നെ അവതരിപ്പിച്ച ഏഴ് ഹര്‍ഫുകളിലായുള്ള ക്വുര്‍ആന്‍ പാരായണം നബി(സ)യുടെ കാലത്ത് തന്നെ നിലനിന്നിരുന്നുവെന്ന വസ്തുത മനസ്സിലാക്കാത്തതുകൊണ്ടാണ് വ്യത്യസ്ത ശൈലികളിലുള്ള ക്വുര്‍ആനുകള്‍ തമ്മില്‍ വൈരുധ്യമുണ്ടെന്ന് ചില വിമര്‍ശകന്മാര്‍ ആരോപിക്കുന്നത്. അല്ലാഹു അവതരിപ്പിച്ച ഏഴ് ശൈലികളിലുമുള്ള ക്വുര്‍ആന്‍ പാരായണം നബി(സ) തന്റെ അനുയായികളെ പഠിപ്പിച്ചിരുന്നുവെന്ന വസ്തുത നടേ ഉദ്ധരിച്ച നിവേദനങ്ങള്‍ വ്യക്തമാക്കുന്നു. ഏഴ് ശൈലികളില്‍ അവതരിക്കപ്പെട്ടിട്ടും ക്വുര്‍ആനില്‍ യാതൊരു വൈരുധ്യവുമില്ലെന്നത് അത്ഭുതകരമാണ്. ''അവര്‍ ക്വുര്‍ആനിനെപ്പറ്റി ചിന്തിക്കുന്നില്ലേ? അത് അല്ലാഹു അല്ലാത്തവരുടെ പക്കല്‍ നിന്നുള്ളതായിരുന്നെങ്കില്‍ അവരതില്‍ ധാരാളം വൈരുധ്യം കണ്ടെത്തുമായിരുന്നു.'' (4:82) വെന്ന ക്വുര്‍ആന്‍” വചനത്തിലെ പരാമര്‍ശം ഏഴ് ഹര്‍ഫുകള്‍ക്കും ഒരേപോലെ ബാധകമാണ്. ഒരേ ഹര്‍ഫിലുള്ള ക്വുര്‍ആനിലെ വചനങ്ങള്‍ തമ്മിലോ വ്യത്യസ്ത ഹര്‍ഫുകള്‍ തമ്മിലോ വൈരുധ്യങ്ങളൊന്നുമില്ല. വ്യത്യസ്ത നിലവാരത്തിലുള്ളവരെ പരിഗണിച്ചുകൊണ്ട് വ്യത്യസ്ത ശൈലികളില്‍ അവതരിക്കപ്പെട്ടിട്ടുപോലും ക്വുര്‍ആനില്‍ വൈരുധ്യങ്ങളൊന്നുമില്ലെന്ന അത്ഭുതകരമായ വസ്തുത അതിന്റെ ദൈവികത വ്യക്തമാക്കുന്ന പല തെളിവുകളിലൊന്നാണ്.

 അബ്രഹാമും ഇശ്മയേലും ഏകദൈവാരാധക്കുവേണ്ടി സ്ഥാപിച്ച പ്രാര്‍ത്ഥനാമന്ദിരത്തോടനുബന്ധിച്ച് പില്‍ക്കാലത്ത് ബഹുദൈവവിശ്വാസപരമായ ചടങ്ങുകള്‍ രൂപം കൊണ്ടതാണെന്നും കഅ്ബയുടെ സാക്ഷാല്‍ ലക്ഷ്യം പ്രപഞ്ചനാഥനെ മാത്രം ആരാധിക്കുകയാണെന്നും പല അറബികള്‍ക്കും പ്രവാചകനിയോഗത്തിന്റെ കാലഘട്ടത്തില്‍പോലും അറിയാമായിരുന്നുവെന്നും അവരാണ് ഹനീഫുകള്‍ എന്നറിയപ്പെട്ടതെന്നും ഉള്ള മുസ്‌ലിം ചരിത്രകാരന്‍മാരുടെ വാദം അടിസ്ഥാനരഹിതമാണ്. ഹനീഫുകള്‍ എന്ന പേരില്‍ ജാഹിലിയ്യ അറബികള്‍ക്കിടയില്‍ ജീവിച്ചിരുന്നവരൊന്നും ഈ വിശ്വാസമുള്ളവരല്ലായിരുന്നുവെന്ന് അവരെക്കുറിച്ചുള്ള നിവേദനങ്ങളില്‍ നിന്ന് സ്പഷ്ടമാണ്. മക്കയുടെ അബ്രഹാമിക പാരമ്പര്യത്തെ നിഷേധിക്കുന്ന ഓറിയന്റലിസ്റ്റ്-മിഷനറി രചനകളില്‍ സര്‍വസാധാരണമായ ഈ വാദങ്ങള്‍ ഹനീഫിയ്യത്തിനെ സംബന്ധിച്ച മുസ്‌ലിം അവകാശവാദത്തെ പുനപരിശോധിക്കാന്‍ പ്രേരിപ്പിക്കുന്നവയല്ലേ?

ല്ല. മക്കയുടെ ഇബ്‌റാഹീമീ പാരമ്പര്യം മുഹമ്മദ് നബി (സ) പറഞ്ഞുണ്ടാക്കിയതാണ് എന്ന വിമര്‍ശനത്തെ എല്ലാ അര്‍ത്ഥത്തിലും കടപുഴക്കുന്നതാണ് പ്രവാചകനിയോഗത്തിനുമുമ്പേ അറേബ്യയിലുണ്ടായിരുന്ന ഹനീഫുകളുടെ സാന്നിധ്യം. അറബികള്‍ ഇബ്‌റാഹീമീ ഏകദൈവാരാധനയില്‍നിന്ന് വ്യതിചലിച്ചുപോയതായി മനസ്സിലാക്കുകയും ബഹുദൈവാരാധനാപരമായ അറബ് അനുഷ്ഠാനങ്ങളോട് വിരക്തി പ്രകടിപ്പിച്ച് ഇബ്‌റാഹീമീ മാര്‍ഗത്തിന്റെ വീണ്ടെടുപ്പ് സ്വന്തം ജീവിതത്തില്‍ ആഗ്രഹിക്കുകയും ചെയ്ത ന്യൂനപക്ഷമാണ് മക്കയിലും പരിസരപ്രദേശങ്ങളിലും ഹനീഫുകള്‍ എന്നു വിളിക്കപ്പെട്ടത് എന്ന് അവരെ സംബന്ധിച്ച നിവേദനങ്ങളെല്ലാം വ്യക്തമാക്കുന്നുണ്ട്. ഇബ്‌റാഹീമീ രക്തത്തോടൊപ്പം ആദര്‍ശവും കുറേയെങ്കിലും അറേബ്യയില്‍ മുഹമ്മദ് നബി(സ)യുടെ കാലം വരെ നിലനിന്നുവെന്ന് ഹനീഫുകളുടെ ചരിത്രം ബോധ്യപ്പെടുത്തുന്നു എന്നതിനാലാണ് ഹനീഫുകള്‍ ഇബ്‌റാഹീമീ നിലപാടുകളുടെ പുനരുജ്ജീവനത്തിന് പരിശ്രമിച്ചവരായിരുന്നില്ലെന്ന് ചില നിവേദനങ്ങളിലെ പരാമര്‍ശങ്ങളുടെ വെളിച്ചത്തില്‍ സ്ഥാപിച്ചെടുക്കാന്‍ ഓറിയന്റലിസ്റ്റുകള്‍ പരിശ്രമിച്ചു നോക്കിയിട്ടുള്ളത്.

വാസ്തവത്തില്‍, അറേബ്യയില്‍ നിലനിന്നിരുന്ന ഹനീഫിയ്യത്തിനെ സംബന്ധിച്ച ചരിത്രനിവേദനങ്ങളുടെ വിശകലനം ഇബ്‌റാഹീം നബി(അ)യുടെ ആദര്‍ശമനുസരിച്ച് ജീവിക്കാനുളള അദമ്യമായ ആഗ്രഹമാണ് ഹനീഫുകളെ വ്യതിരിക്തരാക്കിയത് എന്നുതന്നെയാണ് ബോധ്യപ്പെടുത്തുന്നത്. ഇബ്‌റാഹീമിലേക്ക് മടങ്ങുവാനുള്ള ത്വര മുഹമ്മദ് നബി (സ) പുതിയ ദേശപാരമ്പര്യം മെനഞ്ഞുണ്ടാക്കി മക്കക്കാരില്‍ കൃത്രിമമായി സന്നിവേശിപ്പിച്ചതാണെന്ന വിമര്‍ശക വീക്ഷണം പ്രസ്തുത നിവേദനങ്ങള്‍ക്കുമുന്നില്‍ ഒരിക്കലും നിലനില്‍ക്കുകയില്ല. തങ്ങള്‍ തുടര്‍ന്നുകൊണ്ടിരിക്കുന്ന വിശ്വാസ-കര്‍മ മാര്‍ഗം പൂര്‍ണമായും ഇബ്‌റാഹീമിന്റെയും ഇസ്മാഈലിന്റേതുമാണ് എന്ന് വലിയൊരു വിഭാഗം അറബികള്‍ തെറ്റിദ്ധരിച്ചപ്പോഴും അങ്ങനെയല്ലെന്നും അതില്‍ കലര്‍പ്പുകള്‍ വന്നിട്ടുണ്ടെന്നും ശരിയായി തിരിച്ചറിഞ്ഞ ഒറ്റപ്പെട്ട വ്യക്തികളായിരുന്നു ഹനീഫുകള്‍. അല്ലാഹുവിനു മാത്രം ആരാധനകള്‍ സമര്‍പ്പിക്കണമെന്നു വാദിച്ചിരുന്നതുകൊണ്ടാണ് അവര്‍ ഹനീഫുകള്‍ (ഋജുമാനസ്‌കര്‍-കലര്‍പ്പുകള്‍ അനുവദിക്കാത്തവര്‍) എന്നറിയപ്പെട്ടത്.

മക്കന്‍ മുഖ്യധാരയോട് കലഹിച്ച് ഇബ്‌റാഹീമീ സരണിയോട് വിഗ്രഹാരാധനയും അറബ് അനാചാരങ്ങളും ഒത്തുപോവുകയില്ലെന്ന് ഒറ്റയാനായി പ്രഖ്യാപിച്ച സയ്ദ്ബ്‌നു അംറുബ്‌നു നുഫയ്ല്‍ ആണ് ചരിത്രത്തിലെ ഏറ്റവും പ്രഖ്യാതനായ ഹനീഫ്. മുഹമ്മദ് നബി(സ)യുടെ സമകാലീനനായിരുന്നെങ്കിലും അദ്ദേഹത്തിന്റെ പ്രവാചകത്വത്തിനുമുമ്പ് മരണപ്പെട്ടുപോയ വ്യക്തിയായിരുന്നു സയ്ദ് എന്നാണ് മനസ്സിലാകുന്നത്. മക്കന്‍ വിഗ്രഹാരാധന ശരിയല്ലെന്ന് മനസ്സിലാക്കി ശരിയായ ദൈവികപാത തേടി സിറിയയിലേക്കടക്കം യാത്ര പോയ സയ്ദിന് ജൂത, ക്രൈസ്തവ പണ്ഡിതരടക്കം ഉപദേശിച്ചുകൊടുത്തത് ഇബ്‌റാഹീമീ ഹനീഫിയ്യത്തായിരുന്നുവെന്ന് സ്വഹീഹുല്‍ ബുഖാരിയിലെ തീര്‍ത്തും പ്രബലമായ നിവേദനത്തിലുണ്ട്. സിറിയയില്‍ നിന്നു മടങ്ങിയപ്പോള്‍ അദ്ദേഹം കൈകളുയര്‍ത്തി ”എന്റെ രക്ഷിതാവേ, ഞാന്‍ ഇബ്‌റാഹീമിന്റെ മതത്തിലാണെന്നതിന് നീ സാക്ഷ്യം വഹിച്ചുകൊള്ളുക” എന്ന് പ്രഖ്യാപിച്ചതായി ഇബ്‌നു ഉമര്‍ (റ) റിപ്പോര്‍ട്ട് ചെയ്തിട്ടുണ്ട് (ബുഖാരി). താന്‍ അല്ലാഹുവിനെ മാത്രമേ ആരാധിക്കുകയുള്ളൂവെന്നും അതാണ് ഇബ്‌റാഹീമിന്റെ ശരിയായ മതം എന്നും താനാണ് അതില്‍ നിലനില്‍ക്കുന്നതെന്നും ഇബ്‌റാഹീമിന്റെ പൈതൃകം അവകാശപ്പെടുമ്പോഴും മറ്റു മക്കക്കാര്‍ അദ്ദേഹത്തിന്റെ മതത്തില്‍ നിന്നും വ്യതിചലിച്ചുപോയിരിക്കുന്നുവെന്നും വ്യക്തമാക്കിക്കൊണ്ട് സയ്ദ്ബ്‌നു അംറ് കഅ്ബയുടെ ചാരത്തുനിന്ന് ”ഖുറയ്ശികളേ, സയ്ദിന്റെ ആത്മാവ് ആരുടെ കയ്യിലാണോ, അവനാണ് (അല്ലാഹു) സത്യം; നിങ്ങളിലൊരാളുമല്ല, മറിച്ച് ഞാനാണ് ഇബ്‌റാഹീമിന്റെ മതത്തിലുള്ളത്” (മാ അസ്ബ്ഹ മിന്‍കും അഹദുന്‍ അലാ ദീനി ഇബ്‌റാഹീമ ഗ്വയ്‌രീ) എന്നു പ്രഖ്യാപിച്ചത് ഇബ്‌നു ഇസ്ഹാക്വിന്റെ സീറയിലുണ്ട് (Guillaume, 99-100).

താന്‍ ഇബ്‌റാഹീമിന്റെ മാര്‍ഗം തെരഞ്ഞെടുക്കുന്നു എന്ന് സയ്ദ് പറയുന്നത് ആ മാര്‍ഗം മക്കക്കാര്‍ നേരത്തെ അവകാശപ്പെട്ടുകൊണ്ടിരുന്നതാണ് എന്ന അവബോധത്തോടെയാണ് എന്ന് അദ്ദേഹത്തിന്റെ വാക്കുകള്‍ സുതരാം വ്യക്തമാക്കുന്നുണ്ട്. ഹനീഫിയ്യാ ഏകദൈവാരാധനാനിഷ്ഠ സ്വീകരിച്ചതിന്റെ ഫലമായി, അദ്ദേഹം വിഗ്രഹാരാധനയും വിഗ്രഹങ്ങള്‍ക്ക് നിവേദിക്കപ്പെട്ട ഭക്ഷണം കഴിക്കുന്നതും പൂര്‍ണമായി ഉപേക്ഷിച്ചുവെന്നും ഇബ്‌നു ഇസ്ഹാക്വ് രേഖപ്പെടുത്തുന്നു (Ibid, p. 99). മദീനയില്‍ പ്രവാചകാഗമനത്തിനു മുമ്പുതന്നെ ഹനീഫ് ആയി ജീവിച്ചിരുന്ന ബനൂ അദിയ്യ ഗോത്രക്കാരന്‍ അബൂ ക്വയ്‌സ് ബിന്‍ അബൂ അനസിന്റെയും കഥ ഏതാണ്ട് സമാനം തന്നെയാണ്. വിഗ്രഹങ്ങളുപേക്ഷിക്കുകയും ജൂതനോ ക്രൈസ്തവനോ ആകുന്നതിനുപകരം ശുദ്ധമായ ഇബ്‌റാഹീമീ സരണി പുല്‍കുന്നുവെന്ന് പ്രഖ്യാപിക്കുകയും ചെയ്ത അദ്ദേഹം ”ഞാന്‍ ഇബ്‌റാഹീമിന്റെ നാഥനെയാണ് ആരാധിക്കുന്നത്” എന്ന് വിശദീകരിച്ച് പ്രാര്‍ത്ഥന നിര്‍വഹിക്കുവാന്‍ വേണ്ടി ഒരു ആരാധനാലയം പണിതതായി ഇബ്‌നു ഇസ്ഹാക്വില്‍ തന്നെയുണ്ട്. ഇദ്ദേഹം നബി (സ) മദീനയിലെത്തിയപ്പോള്‍ ഇസ്‌ലാം സ്വീകരിച്ചു (Ibid, pp. 236-9).

സയ്ദിനെയും അബൂക്വയ്‌സിനെയും സംബന്ധിച്ചുള്ള നിവേദനങ്ങള്‍ വ്യക്തമാക്കുന്ന ഒരു കാര്യം, ഇബ്‌റാഹീം നബി(അ)യും ഇസ്മാഈല്‍ നബി(അ)യും പഠിപ്പിച്ചിരുന്നത് ശുദ്ധ ഏകദൈവാരാധനയാണെന്നും എന്നാല്‍ അറബികള്‍ അവരുടെ മാര്‍ഗത്തില്‍നിന്ന് വ്യതിചലിച്ച് വിഗ്രഹാരാധനയിലും അധാര്‍മികതകളിലും എത്തിപ്പെട്ടു എന്നുമുള്ള അടിസ്ഥാന ബോധ്യങ്ങളാണ് അവര്‍ക്കുണ്ടായിരുന്നത് എന്നാണ്. ഇബ്‌റാഹീമീ സരണിക്ക് നിരക്കുന്നതല്ലെന്ന് തങ്ങള്‍ക്ക് ബോധ്യം വന്ന തിന്മകളില്‍ നിന്ന് വിട്ടുനില്‍ക്കുകയും സാധ്യമാകുന്ന തരത്തില്‍ ഏകദൈവാരാധന നിര്‍വഹിക്കുകയും ചെയ്ത് മനസ്സാക്ഷിയെ തൃപ്തിപ്പെടുത്താന്‍ ശ്രമിച്ചവരായിരുന്നു അവര്‍. അതല്ലാതെ, അല്ലാഹുവിനുവേണ്ടി നിര്‍വഹിക്കേണ്ടുന്ന ആരാധനകളുടെ വിശദമായ കര്‍മശാസ്ത്രത്തെക്കുറിച്ചോ അനുഷ്ഠിക്കേണ്ട സല്‍പ്രവര്‍ത്തനങ്ങളെക്കുറിച്ചോ ജീവിതവിശുദ്ധി നിലനിര്‍ത്താന്‍ ഉപേക്ഷിക്കേണ്ട തിന്മകളെക്കുറിച്ചോ കൃത്യവും സമഗ്രവുമായ ധാരണകളൊന്നും അവര്‍ക്കുണ്ടായിരുന്നില്ല. വഹ്‌യ് ലഭിക്കുന്ന ഒരു പ്രവാചകന്റെ അസാന്നിധ്യമായിരുന്നു ഈ പ്രതിസന്ധിക്കു കാരണം.

അബൂക്വയ്‌സും സയ്ദും ആര്‍ത്തവകാരികളുമായുള്ള ലൈംഗിക ബന്ധത്തില്‍നിന്ന് വിട്ടുനിന്നതും സയ്ദ് ശവവും രക്തവും ഭക്ഷിക്കുന്നത് ഒഴിവാക്കിയതും പെണ്‍മക്കളെ ജീവനോടെ കുഴിച്ചുമൂടുന്നതിനെ എതിര്‍ത്തതുമെല്ലാം ഇബ്‌നു ഇസ്ഹാക്വ് വിവരിക്കുന്നുണ്ട്. ഇവയെല്ലാം അവരുടെ അന്വേഷണങ്ങളില്‍ നിന്ന് അവരെത്തിപ്പെട്ട ധാര്‍മിക നിലപാടുകളായിരുന്നു. കുറേക്കൂടി നിഷ്‌കൃഷ്ടമായ മാര്‍ഗദര്‍ശനത്തിനുവേണ്ടി അവര്‍ ദാഹിച്ചിരുന്നുവെന്ന് സയ്ദ്ബ്‌നു അംറിന്റെ വാക്കുകള്‍ വ്യക്തമാക്കുന്നുണ്ട്. അദ്ദേഹം പറഞ്ഞു : ”എന്റെ രക്ഷിതാവേ, നിന്നെ ആരാധിക്കാനുള്ള കൂടുതല്‍ നല്ല മാര്‍ഗങ്ങള്‍ അറിയുമായിരുന്നുവെങ്കില്‍ ഞാനത് സ്വീകരിക്കുമായിരുന്നു; പക്ഷേ എന്തു ചെയ്യാം, എനിക്കതറിയില്ല!” തുടര്‍ന്ന് കഅ്ബക്കുനേരെ തിരിഞ്ഞ് അല്ലാഹുവിനു സുജൂദ് ചെയ്ത് അദ്ദേഹം വാക്കുകള്‍ ഇങ്ങനെ മുഴുമിപ്പിച്ചു: ”എന്റെ നാഥന്‍ ഇബ്‌റാഹീമിന്റെ നാഥനാണ്, എന്റെ മതം ഇബ്‌റാഹീമിന്റെ മതവുമാണ്.” (Ibid, p. 100; Dr. Mahdi  Rizqullah Ahmad, A Biography of the Prophet of Islam in the light of Original Sources  (Riyadh: Darussalam, 2005), p. 58).

മക്കയിലും മദീനയിലുമുണ്ടായിരുന്ന ഹനീഫുകളായി അറിയപ്പെട്ടിരുന്ന  ചില വ്യക്തികള്‍ നബി(സ)യുടെ പ്രവാചകത്വം അംഗീകരിക്കുവാന്‍ വിസമ്മതിക്കുകയും അദ്ദേഹത്തോട് ആശയപരമായ വിയോജിപ്പുകള്‍ രേഖപ്പെടുത്തുകയും ചെയ്തതായി പറയുന്ന ചില നിവേദനങ്ങളുടെ അടിസ്ഥാനത്തിലാണ് വിമര്‍ശകര്‍ ഹനീഫുകള്‍ ഇബ്‌റാഹീമീ മാര്‍ഗത്തിന്റെ പുനരുജ്ജീവനത്തിനു ശ്രമിച്ചവരല്ലായിരുന്നു എന്ന് സ്ഥാപിക്കുവാന്‍ ശ്രമിക്കുന്നത്. അവര്‍ക്കുള്ള മറുപടി സയ്ദിന്റെ വാക്കുകളില്‍ തന്നെയുണ്ട് എന്നുള്ളതാണ് വാസ്തവം. ഇബ്‌റാഹീമീ ഏകദൈവാരാധനയുടെ അടിസ്ഥാനമൂല്യങ്ങള്‍ ഉള്‍ക്കൊണ്ടിരുന്നവരെല്ലാം അറേബ്യയില്‍ ഹനീഫുകളായാണ് അറിയപ്പെട്ടിരുന്നത്. അവരില്‍ നബി(സ)യുടെ പ്രവാചകത്വം അംഗീകരിച്ചവരും നിഷേധിച്ചവരുമുണ്ടാകാം, പ്രവാചകന്‍ സ) പ്രബോധനം ചെയ്ത ധാര്‍മിക പദ്ധതിയുടെ വിശദാംശങ്ങളോട് യോജിച്ചവരും വിയോജിച്ചവരുമുണ്ടാകാം, ഏകദൈവാരാധന മനസ്സിലുള്‍ക്കൊണ്ടാല്‍ മതിയെന്നും നബി (സ) ചെയ്യുന്നതുപോലെ സമൂഹത്തില്‍ അത് വ്യാപകമായി പ്രചരിപ്പിക്കേണ്ടതില്ലെന്നും കരുതിയ ആദര്‍ശ പ്രതിബദ്ധത കുറഞ്ഞ വ്യക്തികളുമുണ്ടാകാം. ഹനീഫുകള്‍ ആരാണെന്ന് മനസ്സിലാക്കിയവര്‍ക്ക് ചരിത്രപരമായി ഇവയിലൊന്നും യാതൊരു അസാംഗത്യവും അനുഭവപ്പെടുകയില്ല.

ഏകശിലാത്മകമായ ഒരു സമൂഹമായിരുന്നില്ല ഹനീഫുകളുടേത്; മറിച്ച് ഇബ്‌റാഹീമീ ഏകദൈവാരാധനയെക്കുറിച്ച് നിശ്ചയവും വ്യക്തതയുമുണ്ടായിരുന്ന, എന്നാല്‍ അനുബന്ധങ്ങളില്‍ ആശയക്കുഴപ്പങ്ങളും അഭിപ്രായാന്തരങ്ങളുമുണ്ടായിരുന്ന ഒറ്റയും തെറ്റയുമായ വ്യക്തിത്വങ്ങളാണ്. അതുകൊണ്ടുതന്നെ വിമര്‍ശകര്‍ എടുത്തുദ്ധരിക്കുന്ന നിവേദനങ്ങള്‍ നിദാനശാസ്ത്രപരമായി ആധികാരികമാണെങ്കിലും അല്ലെങ്കിലും, അവരുടെ വാദം സ്ഥാപിക്കുവാന്‍ പര്യാപ്തമായവയല്ല എന്നതാണ് വാസ്തവം. ചില ഹനീഫുകള്‍ പ്രവാചകന്റെ കൂടെ നിന്നില്ല എന്നുമാത്രമാണ് പരാമൃഷ്ട നിവേദനങ്ങള്‍ വ്യക്തമാക്കുന്നത്. എന്നാല്‍ ഇബ്‌റാഹീം നബി(അ)യുടെ ആശയങ്ങളിലേക്ക് മടങ്ങിപ്പോകണമെന്ന കാര്യത്തില്‍ അവര്‍ക്ക് നബി(സ)യുമായി യാതൊരു അഭിപ്രായവ്യത്യാസവുമുണ്ടായിരുന്നില്ല  എന്ന് അതേ നിവേദനങ്ങള്‍ തന്നെ വ്യക്തമാക്കുന്നുണ്ട്. മക്കയില്‍ പൂര്‍വപിതാക്കളായ ഇബ്‌റാഹീമിെനയും ഇസ്മാഈലിനെയും കുറിച്ചുള്ള ബോധ്യവും അവരുടെ ചര്യകള്‍ മുറുകെപ്പിടിക്കണമെന്ന വികാരവും പ്രവാചകന്‍ (സ) പുതുതായി സൃഷ്ടിച്ചെടുത്തതല്ലെന്ന് എല്ലാ അര്‍ത്ഥത്തിലും സ്ഥാപിക്കുന്നവയാണ് ആ ഉദ്ധരണികള്‍. ഹനീഫുകള്‍ എന്നാല്‍ മുഹമ്മദീയ ഇസ്‌ലാമിന്റെ എല്ലാ ആശയങ്ങളും അംഗീകരിച്ചവരായിരുന്നു എന്ന് മുസ്‌ലിംകള്‍ വാദിക്കുന്നതായി തെറ്റിദ്ധരിപ്പിച്ചുകൊണ്ടാണ് വിമര്‍ശകര്‍ അവയുടെ വിശകലനം നിര്‍വഹിക്കുന്നത്. ഈ തെറ്റിദ്ധാരണയില്‍ നിന്ന് മോചിതരായാല്‍  വിമര്‍ശകര്‍ സ്വന്തം അടിസ്ഥാനങ്ങളെയാണ് തകര്‍ത്തുകളയുന്നത് എന്ന് ആര്‍ക്കും ബോധ്യമാകും. മദീനയിലെ ഔസ് ഗോത്രത്തിന്റെ നേതാക്കളിലൊരാളായിരുന്ന അബൂ ആമിര്‍ അംറുബ്‌നു സയ്ഫും ഔസ് ഗോത്രക്കാരന്‍ തന്നെയായിരുന്ന കവി അബുക്വയ്‌സ് ബിന്‍ അസ്‌ലതും ത്വാഇഫുകാരനായ ഉമയ്യയുമാണ് ഹനീഫുകളായി അറിയപ്പെട്ടിരുന്ന, ഇസ്‌ലാം സ്വീകരിച്ചിട്ടില്ലെന്ന് ചില നിവേദനങ്ങള്‍ പറയുന്ന വ്യക്തികള്‍. അബൂ ആമിറും നബി(സ)യും തമ്മില്‍ മദീനയില്‍വെച്ച് നേരില്‍ കണ്ടുമുട്ടിയപ്പോള്‍ അവര്‍ തമ്മില്‍ നടന്ന സംഭാഷണം ഇബ്‌നു ഇസ്ഹാക്വിന്റെ, വിമര്‍ശകര്‍ ആശ്രയിക്കുന്ന നിവേദനത്തില്‍ തന്നെയുണ്ട്. ഏതു മതവുമായാണ് പ്രവാചകന്‍ (സ) നിയോഗിക്കപ്പെട്ടതെന്ന അബൂ ആമിറിന്റെ ചോദ്യത്തിന് ‘ഹനീഫിയ്യ; ഇബ്‌റാഹീമിന്റെ മതം’ എന്ന് നബി (സ) മറുപടി പറഞ്ഞപ്പോള്‍ താനും ആ മതത്തില്‍ തന്നെയാണ് എന്നായിരുന്നു അബൂ ആമിറിന്റെ പ്രത്യുത്തരം. അബൂ ആമിര്‍ ഇബ്‌റാഹീമിന്റെ മതം ശരിയായി പിന്തുടരുന്നില്ലെന്ന് പറഞ്ഞ പ്രവാചകന്‍(സ)യോട് അദ്ദേഹം പറഞ്ഞത് ‘ഹനീഫിയ്യത്തില്‍ ഇല്ലാത്ത പലതും, മുഹമ്മദ്, താങ്കള്‍ അതില്‍ കൂട്ടിച്ചേര്‍ത്തിരിക്കുന്നു’ (ഇന്നക്ക അദ്ഖല്‍ത, യാ മുഹമ്മദ്, ഫില്‍ ഹനീഫിയ്യ മാ ലയ്‌സ മിന്‍ഹാ) എന്നായിരുന്നുവെന്നും ഇബ്‌റാഹീമിന്റെ മതത്തെ അതിന്റ ശുദ്ധതയില്‍ അപ്പടി പ്രബോധനം ചെയ്യുക മാത്രമാണ് താന്‍ ചെയ്യുന്നതെന്ന് നബി (സ) ഇതിനോട് പ്രതിവചിച്ചുവെന്നും ഇബ്‌നു ഇസ്ഹാക്വ് രേഖപ്പെടുത്തുന്നു. ഇബ്‌റാഹീമീ സരണി പിന്തുടരുന്നുവെന്ന് അവകാശപ്പെട്ടതിനാല്‍ തന്നെയാണ് അബൂ ആമിര്‍ ഹനീഫ് ആയി അറയപ്പെട്ടതെന്നും ഇബ്‌റാഹീമിന്റെ മാര്‍ഗത്തിന്റെ വിശദാംശങ്ങളില്‍ എന്തെല്ലാം വരുമെന്ന കാര്യത്തിലാണ് അദ്ദേഹത്തിന് പ്രവാചകനുമായി അഭിപ്രായവ്യത്യാസമുണ്ടായതെന്നും നിവേദനത്തില്‍ പരാമര്‍ശിക്കപ്പെട്ടിരിക്കുന്ന സംവാദത്തില്‍ നിന്ന് വ്യക്തമാണ്. അറേബ്യയില്‍ പ്രവാചകനുമുമ്പേ ഇബ്‌റാഹീമീ വികാരങ്ങള്‍ നിലനിന്നിരുന്നുവെന്നതിന്റെ സൂചകമാണ് ഫനീഫുകള്‍ എന്ന നിരീക്ഷണത്തെ ഈ നിവേദനം എങ്ങനെ തകര്‍ക്കുമെന്നാണ് വിമര്‍ശകര്‍ പറയുന്നത്? രണ്ടാമത്തെയാളായ അബൂ ക്വയ്‌സ് ബിന്‍ അസ്‌ലത്, ‘ഞാന്‍ ഇബ്‌റാഹീമിന്റെ മതം പിന്തുടരുന്നു; മരണം വരെ ഞാനതില്‍ നിന്ന് പിന്‍മാറുകയില്ല’ എന്ന് പ്രസ്താവിക്കുമായിരുന്നുവെന്ന് ഓറിയന്റലിസ്റ്റുകള്‍ ആശ്രയിക്കുന്ന ഇബ്‌നു സഅദിന്റെ നിവേദനത്തില്‍ തന്നെയുണ്ട്. ഹനീഫിയ്യത്തിന് ഏകദൈവാരാധനാ നിലപാടുകളുമായി ബന്ധമുണ്ടായിരുന്നില്ലെന്ന വിമര്‍ശകവാദത്തെ എല്ലാ അര്‍ത്ഥത്തിലും പൊളിച്ചുകളയുന്നതാണ് ഉമയ്യയെക്കുറിച്ചുള്ള നിവേദനങ്ങളെല്ലാം. അദ്ദേഹം വിഗ്രഹാരാധനയില്‍ നിന്ന് വിട്ടുനില്‍ക്കുകയും അതിന്റെ പേരില്‍ ജൂതനായിപ്പോലും വിചാരിക്കപ്പെടുകയും ചെയ്ത, ഇബ്‌റാഹീമിന്റെയും ഇസ്മാഈലിന്റെയും യഥാര്‍ത്ഥ പൈതൃകം ഹനീഫിയ്യത്താണെന്ന് സ്ഥാപിച്ചുകൊണ്ട് സമൃദ്ധമായി കവിതകളെഴുതിയ ആളായിരുന്നുവെന്നാണ് ചരിത്രഗ്രന്ഥങ്ങള്‍ പറയുന്നത്. അദ്ദേഹത്തിന്റെ ‘ഹനീഫീ’ കവിതള്‍ അറബ് വാമൊഴി പാരമ്പര്യത്തില്‍ സജീവമായി നിലനിന്നതുകൊണ്ടുതന്നെ, ജാഹിലിയ്യാ കാലഘട്ടം മുതല്‍ ഒന്‍പതാം നൂറ്റാണ്ടുവരെയുള്ള അറബിക്കവിതകളുടെ ബൃഹദ്‌ശേഖരം അരനൂറ്റാണ്ടു കാലത്തെ അധ്വാനംകൊണ്ട് അബുല്‍ ഫറജ് ഇസ് ഫഹാനി സി. ഇ പത്താം നൂറ്റാണ്ടില്‍ അഗാനി എന്ന പേരില്‍ പ്രസിദ്ധീകരിച്ചപ്പോള്‍ അതില്‍പോലും ഇടം പിടിച്ചിട്ടുണ്ട്. വിമര്‍ശകര്‍ക്കാവശ്യമുള്ളതൊന്നും ഉമയ്യയെ വിശകലനം ചെയ്തതുകൊണ്ട് ലഭിക്കുകയില്ലെന്ന് ഇതില്‍ നിന്നുതന്നെ വ്യക്തമാണ്. ”ഹനീഫിയ്യത്ത് സത്യമാണെന്നെനിക്കറിയാം; എന്നാല്‍ മുഹമ്മദിന്റെ കാര്യത്തിലാണ് എനിക്ക് തീര്‍ച്ചയില്ലാത്തത്” (വ അന അഅ്‌ലമു അന്നല്‍ ഹനീഫിയ്യ ഹക്വുന്‍ വലാകിന്നശ്ശക്ക്വ യുദാഖിലുനീ ഫീ മുഹമ്മദ്) എന്നാണ് ഉമയ്യ പ്രവാചകനുമായുള്ള തന്റെ അഭിപ്രായവ്യത്യാസത്തെക്കുറിച്ച് പറഞ്ഞത് (ഫത്ഹുല്‍ബാരി). മുഹമ്മദ് നബി(സ)യുടെ പ്രവാചകത്വം അംഗീകരിക്കാന്‍ വിമുഖത കാണിച്ചുവെന്നതുകൊണ്ട് ഉമയ്യ ഇബ്‌റാഹീമീ ആദര്‍ശ പിന്തുടര്‍ച്ച അവകാശപ്പെട്ടിരുന്നുവെന്ന വസ്തുത എങ്ങനെയാണ് ഇല്ലാതാവുക? അറബികള്‍ക്കിയടിലുണ്ടായിരുന്ന ഇബ്‌റാഹീമീ ബോധത്തെ അദ്ദേഹത്തെ സംബന്ധിച്ച നിവേദനങ്ങള്‍ സാധൂകരിക്കുകയല്ലാതെ എങ്ങനെയാണ് നിരാകരിക്കുക? ചുരുക്കത്തില്‍, ഹനീഫുകളെക്കുറിച്ചുള്ള ചരിത്ര/ഹനീഥ് നിവേദനങ്ങള്‍ അവര്‍ ഇബ്‌റാഹീമീ ആദര്‍ശ വ്യതിരിക്തത അവകാശപ്പെട്ടിരുന്നവരല്ല എന്ന് സൂചിപ്പിക്കുന്നുവെന്ന വിമര്‍ശകരുടെ വാദം ഒരു കഴമ്പുമില്ലാത്തതാണ്.

 മക്കന്‍ ബഹുദൈവാരാധക പാരമ്പര്യത്തില്‍ വികസിച്ചുവന്ന ഒരു ശിലാപൂജക ക്ഷേത്രമായിരുന്നു വാസ്തവത്തില്‍ കഅ്ബ. തെക്കനറേബ്യന്‍ വിഗ്രഹാരാധകരുടെ മുന്‍കയ്യില്‍ ശിലാപൂജക്കുവേണ്ടി സ്ഥാപിക്കപ്പെട്ട ക്ഷേത്രമാണ് കഅ്ബയെന്ന് ചരിത്രരേഖകളും കഅ്ബയില്‍ ഇപ്പോഴും സ്ഥിതിചെയ്യുന്ന ഹജറുല്‍ അസ്‌വദും വ്യക്തമാക്കുന്നുണ്ട്. ഒരു പ്രാകൃത ബഹുദൈവാരാധനാ ശേഷിപ്പിനെയാണ് പ്രവാചകന്‍ മുസ്‌ലിംകളുടെ ക്വിബ്‌ലയായി പ്രഖ്യാപിച്ചത് എന്ന് മനസ്സിലാക്കാനേ അവയുടെ വെളിച്ചത്തില്‍ ചരിത്രാന്വേഷകര്‍ക്ക് കഴിയൂ. കഅ്ബ ഇബ്‌റാഹീമും ഇസ്മാഈലും സ്ഥാപിച്ചതല്ലെന്ന് സമര്‍ഥിക്കാന്‍ ചില ഓറിയന്റലിസ്റ്റുകളും മിഷനറിമാരും ഉന്നയിച്ചിട്ടുള്ള ഈ വാദം ശരിയാണോ?

ല്ല. തെക്കേ അറേബ്യയിലെ വിഗ്രഹാരാധകര്‍ ശിലാപൂജക്കുവേണ്ടി സ്ഥാപിച്ച ആരാധനാലയമാണ് കഅ്ബയെന്ന് ‘ചരിത്രരേഖകള്‍’ വ്യക്തമാക്കുന്നുവെന്ന് അവകാശപ്പെടുന്ന ഇസ്‌ലാം വിമര്‍ശകരൊന്നും തന്നെ ഒരു ‘ചരിത്രരേഖ’യും നാളിതുവരെ ഇവ്വിഷയകമായി ഹാജരാക്കിയിട്ടില്ല എന്നതാണ് സത്യം. ദക്ഷിണ അറേബ്യ എന്ന ഭൂമിശാസ്ത്രപരമായ മേല്‍വിലാസം സ്ഥാപിക്കുന്നതുപോയിട്ട് ഏതെങ്കിലും തരത്തിലുള്ള വിഗ്രഹാരാധനക്കുവേണ്ടി ആരെങ്കിലും നിര്‍മിച്ചതാണ് കഅ്ബയെന്ന് സൂചിപ്പിക്കുകയെങ്കിലും ചെയ്യുന്ന ഒരു ചരിത്രരേഖയും ഇല്ല. വിഖ്യാതനായ ഗ്രീക്ക് ചരിത്രകാരന്‍ ഹെറഡോട്ടസ് ‘അലിലത്’ എന്നുപേരുള്ള ഒരു അറേബ്യന്‍ ദേവതയെക്കുറിച്ച് പരാമര്‍ശിക്കുന്നുവെന്നതാണ് കഅ്ബ വിഗ്രഹാരാധകര്‍ സ്ഥാപിച്ചതാണെന്ന് തെളിയിക്കാന്‍ പല ഇസ്‌ലാം വിമര്‍ശകരും എടുത്തുദ്ധരിച്ചിരിക്കുന്നത്. പ്രവാചകന്റെ കാലത്ത് ത്വാഇഫില്‍ പൂജിക്കപ്പെട്ടിരുന്ന ലാത്ത എന്ന വിഗ്രഹമാണ് ഹെറഡോട്ടസിന്റെ പരാമര്‍ശത്തിന് വിധേയമായതെന്ന് അവരില്‍ ചിലര്‍ വ്യാഖ്യാനിക്കുകയും ചെയ്തിട്ടുണ്ട്.

പുരാതനകാലം മുതല്‍ക്കുതന്നെ കഅ്ബ വിഗ്രഹാരാധകരുടെ ദേവാലയമായിരുന്നുവെന്ന് ഹെറഡോട്ടസിന്റെ എഴുത്തുകള്‍ തെളിയിക്കുന്നുവെന്നാണ് വിമര്‍ശകരുടെ വാദം. ഇവിടെ, ഒന്നാമതായി മക്കയെക്കുറിച്ചോ കഅ്ബയെക്കുറിച്ചോ ഹെറഡോട്ടസ് യാതൊരു പ്രസ്താവനയും നടത്തുന്നില്ല എന്നതാണ് സത്യം. ഹെറഡോട്ടസിന്റെ ‘അലിലത്ത്’ ‘ലാത്ത’യാണെങ്കില്‍ ത്വാഇഫിനെക്കുറിച്ചാണ് അദ്ദേഹത്തിന്റെ പരാമര്‍ശം എന്നാണ് വരിക. അങ്ങനെയാണെങ്കിലും അല്ലെങ്കിലും, അറേബ്യയുടെ ഒരു ഭാഗത്ത് ബി.സി. ഇ അഞ്ചാം നൂറ്റാണ്ടില്‍ ഒരു വിഗ്രഹം ദേവതയായി ആരാധിക്കപ്പെടുകയും പൂജിക്കപ്പെടുകയും ചെയ്തിരുന്നുവെന്ന് മാത്രമാണ് അദ്ദേഹത്തിന്റെ വിവരണം തെളിയിക്കുന്നത്. ഇതിന് മക്കയുമായും കഅ്ബയുമായും എന്തു ബന്ധമാണുള്ളത്? കഅ്ബ വിഗ്രഹാരാധനക്കുവേണ്ടി പടുത്തുയര്‍ത്തപ്പെട്ട ഭവനമാണെന്ന  വാദത്തിന് അതെങ്ങനെയാണ് തെളിവാകുക?

രണ്ടാമതായി, ഹെറഡോട്ടസ് ഇനി സാക്ഷാല്‍ കഅ്ബയെക്കുറിച്ചുതന്നെ ഇതേ കാര്യം എഴുതിയാലും അദ്ദേഹത്തിന്റെ കാലമായപ്പോഴേക്കും കഅ്ബക്കു ചുറ്റുമുള്ള ജനത ദേവതാപൂജ ആരംഭിച്ചുവെന്നല്ലാതെ കഅ്ബ സ്ഥാപിക്കപ്പെട്ടത് അന്നായിരുന്നുവെന്നും അതിനുവേണ്ടിയായിരുന്നുവെന്നും എങ്ങനെയാണ് സ്ഥാപിക്കപ്പെടുക? ഇബ്‌റാഹിം നബി(അ)യാണ് കഅ്ബ സ്ഥാപിച്ചതെന്ന പ്രവാചകാധ്യാപനത്തെ അതെങ്ങനെയാണ് തിരുത്തുക? ബൈബിള്‍ കാലഗണന പ്രകാരം ഹെറഡോട്ടസിന് ഒന്നര സഹസ്രാബ്ദത്തോളം മുമ്പ് ജീവിച്ച ഇബ്‌റാഹിം പ്രവാചകന്‍ സ്ഥാപിച്ച ഒരാരാധനാലയത്തിന്റെ പരിസരത്ത് ഹെറഡോട്ടസിന്റെ കാലമായപ്പോഴേക്കും വിശ്വാസവ്യതിചലനങ്ങളുടെ ഫലമായി വിഗ്രഹങ്ങള്‍ വന്നുചേരുന്നതില്‍ എന്ത് അസാംഗത്യമാണുള്ളത്? കഅ്ബയെ ഇബ്‌റാഹിം നബി (അ) പഠിപ്പിച്ച ഏകദൈവാരാധനാപരമായ മൂല്യങ്ങളില്‍ തന്നെ അറബികള്‍ ചരിത്രത്തിലുടനീളം നിലനിര്‍ത്തി എന്ന് മുഹമ്മദ് നബി(സ)യോ മുസ്‌ലിംകളോ അവകാശപ്പെട്ടിട്ടില്ലെന്നു മാത്രമല്ല, ചരിത്രത്തില്‍ സംഭവിച്ച അപഭ്രംശങ്ങള്‍ വഴി പരിശുദ്ധമായ ആ ദൈവഭവനം പില്‍ക്കാലത്ത് വിഗ്രഹങ്ങള്‍കൊണ്ട് മലിനമായിത്തീര്‍ന്നു എന്നാണ് ഇസ്‌ലാം പഠിപ്പിക്കുന്നത്. അത്തരം അപഭ്രംശങ്ങളില്‍ നിന്നു വിമോചിപ്പിച്ച് കഅ്ബയെ അതിന്റെ ആദിമ അബ്രഹാമിക വിശുദ്ധയില്‍ പുനഃസ്ഥാപിക്കുക എന്ന ദൗത്യമാണല്ലോ, മുഹമ്മദ് നബി (സ) ഏഴാം നൂറ്റാണ്ടില്‍ നിര്‍വഹിച്ചത്.

ബി. സി. ഇ ഒന്നാം നൂറ്റാണ്ടില്‍ ജീവിച്ച ഡിയോഡറസ് സിക്കുലസ് (Diodorus Siculus) എന്ന ഗ്രീക്ക് ചരിത്രകാരന്‍ അന്ന് അദ്ദേഹത്തിനറിയാമായിരുന്ന ഭൂപ്രദേശങ്ങളെക്കുറിച്ചെഴുതിയ ബിബഌയോത്തിക്ക ഹിസ്റ്റോറിക്ക (Bibliotheca Historica) എന്ന ഗ്രന്ഥത്തില്‍ മക്കയെ പരമാര്‍ശിച്ചുകൊണ്ട് ”അവിടെ അതിവിശുദ്ധവും എല്ലാ അറബികളും അങ്ങേയറ്റം ആദരിക്കുന്നതുമായ ഒരാരാധനാലയം നിലിവിലുണ്ട്” എന്നെഴുതിയിട്ടുള്ളതാണ് വിമര്‍ശകരുടെ മറ്റൊരു ‘തെളിവ്.’ ഗ്രീക്കുകാര്‍ക്കുപോലും പരിചിതമാകുംവിധമുള്ള പ്രശസ്തി ക്രിസ്തുയേശുവിനു മുമ്പുതന്നെ കഅ്ബ കൈവരിച്ചിരുന്നുവെന്ന് മാത്രമാണ് വാസ്തവത്തില്‍ ഡിയോഡറസിന്റെ ഗ്രന്ഥം തെളിയിക്കുന്നത്. കഅ്ബ ഇബ്‌റാഹിം നബി(അ) സ്ഥാപിച്ചതാണെന്ന യാഥാര്‍ത്ഥ്യത്തെ നിഷേധിക്കുന്ന വിദൂരമായ സൂചനകള്‍പോലും അദ്ദേഹത്തിന്റെ എഴുത്ത് ഉള്‍ക്കൊള്ളുന്നില്ല. കഅ്ബയില്‍ വിഗ്രഹാരാധന നടക്കുന്നതായിത്തന്നെ ഡിയോഡറസിന്റെ എഴുത്തുകള്‍ പറഞ്ഞാല്‍പോലും അദ്ദേഹത്തിനു രണ്ട് സഹസ്രാബ്ദങ്ങള്‍ക്കുമുമ്പ് ഇബ്‌റാഹിം നബി (അ) ഏകദൈവാരാധനക്കുവേണ്ടി സ്ഥാപിച്ചതായിരുന്നു പ്രസ്തുത ഭവനമെന്ന യാഥാര്‍ത്ഥ്യത്തെ അതൊരിക്കലും ബാധിക്കുകയില്ലെന്നതാണ് വാസ്തവം.

കഅ്ബ ഇബ്‌റാഹിം പ്രവാചകനും പുത്രന്‍ ഇസ്മാഈല്‍ പ്രവാചകനും ചേര്‍ന്ന് പ്രപഞ്ചനാഥനെ മാത്രം ആരാധിക്കുവാന്‍വേണ്ടി പടുത്തുയര്‍ത്തിയതാണെന്ന ഇസ്‌ലാമിക നിലപാടിനെ നിരാകരിക്കുകയോ ബഹുദൈവാരാധകര്‍ സ്ഥാപിച്ചതോ ബഹുദൈവാരാധനയ്ക്കുവേണ്ടി സ്ഥാപിക്കപ്പെട്ടതോ ആയ ദേവാലയമാണ് അത് എന്നു സൂചിപ്പിക്കുകയോ ചെയ്യുന്നില്ല വിമര്‍ശകരുടെ കൈവശമുള്ള ‘ചരിത്രരേഖ’കളൊന്നും തന്നെ എന്ന് നമുക്ക് വ്യക്തമായി. കഅ്ബയെ ശിലാപൂജയുമായി ബന്ധിപ്പിക്കുവാന്‍ ഇസ്‌ലാം വിമര്‍ശകര്‍ പിന്നീട് ആശ്രയിക്കുന്നത് ഹജറുല്‍ അസ്‌വദ് എന്ന, കഅ്ബയുടെ ചുമരില്‍ പരിരക്ഷിക്കപ്പെടുന്ന കറുത്ത കല്ലിനെയാണ്. വാസ്തവത്തില്‍, അങ്ങേയറ്റം പരിഹാസ്യമായ ഒരു വാദമാണിത്. ഹജറുല്‍ അസ്‌വദ് എന്ന അറബി വാക്കിനര്‍ത്ഥം കറുത്ത കല്ല് എന്നു മാത്രമാണ്. ആരാധനാലയങ്ങളുടെ ചുമരുകള്‍ നിര്‍മിക്കപ്പെടുക സ്വാഭാവികമായും കല്ലുപയോഗിച്ചു തന്നെയാണ്. കഅ്ബ പടുത്തുയര്‍ത്തുവാനാരംഭിച്ചപ്പോള്‍ മൂലശിലയായി ഇബ്‌റാഹിമും ഇസ്മാഈലും ഉപയോഗിച്ച കറുത്ത കല്ലാണ് ഹജറുല്‍ അസ്‌വദ് എന്ന പേരില്‍ വിശ്രുതമായിത്തീര്‍ന്നത്. ചരിത്രത്തിന്റെ വ്യത്യസ്ത സന്ദര്‍ഭങ്ങളില്‍ നടന്ന പുനര്‍നിര്‍മാണ വേളകളില്‍ ഈ മൂലശില എടുത്തുമാറ്റപ്പെടാതെ പരിരക്ഷിക്കപ്പെട്ടുവെന്നതാണ് അതിന്റെ ചരിത്രപരമായ പ്രാധാന്യം.

ആരാധനാലയങ്ങളുടെ ചുമരുകളില്‍ കല്ലുകളുണ്ടാകുന്നതിന്റെ പേരല്ല ശിലാപൂജ, മറിച്ച് ചുമരിലോ അല്ലാതെയോ ഉള്ള ഏതെങ്കിലും കല്ല് പൂജിക്കപ്പെടുന്നതിന്റെ പേരാണ്. അറബികള്‍ ഇബ്‌റാഹിമീ ഏകദൈവ വിശ്വാസത്തില്‍നിന്ന് പല രീതിയിലും വ്യതിചലിച്ചു പോവുകയും മുഹമ്മദ് നബി (സ)യുടെ കാലമായപ്പോഴേക്കും കടുത്ത വിഗ്രഹാരാധകരായി മാറുകയും ചെയ്തിരുന്നു. എന്നാല്‍ അവരുടെ പൂജാവസ്തുക്കളില്‍ ഒരിക്കല്‍പോലും ഹജറുല്‍ അസ്‌വദ് ഉള്‍പ്പെട്ടിരുന്നില്ലെന്നതാണ് വാസ്തവം. ചരിത്രത്തിന്റെ ഒരു സന്ദര്‍ഭത്തിലും ആരാധിക്കപ്പെട്ടിട്ടില്ലാത്ത, കഅ്ബയുടെ മൂലശിലയെന്ന നിലയില്‍ അപ്പടി നിലനിര്‍ത്തപ്പെടുക മാത്രം ചെയ്തിട്ടുള്ള ഒരു കല്ലാണത്. കഅ്ബയെ ശിലാപൂജയുടെ കേന്ദ്രമായി അവതരിപ്പിക്കാനാഗ്രഹമുള്ള മിഷനറിമാര്‍ക്കും ഓറിയന്റലിസ്റ്റുകള്‍ക്കുമൊന്നും ഹജറുല്‍ അസ്‌വദിനെ അറബികള്‍ പൂജിച്ചിരുന്നുവെന്ന് കാണിക്കുന്ന ഒരു ചരിത്രരേഖയും ഉദ്ധരിക്കാന്‍ കഴിയാത്തത് അതുകൊണ്ടാണ്. കഅ്ബക്കകത്തുപോലും വിഗ്രഹങ്ങള്‍ സ്ഥാപിക്കപ്പെട്ടപ്പോഴും അറബികള്‍ ഹജറുല്‍ അസ്‌വദിനെ ആരാധനാമൂര്‍ത്തിയായി സ്വീകരിച്ചില്ലെന്ന് പറയുമ്പോള്‍ ഹജറുല്‍ അസ്‌വദിനെ ചൂണ്ടിക്കാണിച്ച് ‘ശിലാപൂജ’ സമര്‍ത്ഥിക്കുവാന്‍ ശ്രമിക്കുന്നത് എത്രമേല്‍ ബാലിശമാണെന്ന് ആര്‍ക്കും വ്യക്തമാകും.

ഭക്തി സമര്‍പ്പിക്കപ്പെടുകയോ നിവേദ്യങ്ങള്‍ നേരപ്പെടുകയോ കാര്യകാരണ ബന്ധങ്ങള്‍ക്കതീതമായ ഉപകാരോപദ്രവങ്ങള്‍ പ്രതീക്ഷിക്കപ്പെടുകയോ ഭരമേല്‍പിക്കപ്പെടുകയോ സഹായമഭ്യര്‍ത്ഥിക്കപ്പെടുകയോ പ്രാര്‍ത്ഥിക്കപ്പെടുകയോ ഒന്നും ചെയ്തിട്ടില്ലാത്ത ഹജറുല്‍ അസ്‌വദ്, പ്രപഞ്ചനാഥനുള്ള ആരാധനയായി കഅ്ബയെ വലം വെക്കുന്നവര്‍ക്ക് (ത്വവാഫ്) അത് തുടങ്ങുവാനുള്ള സ്ഥലം അടയാളപ്പെടുത്തുന്ന നാട്ടക്കുറിയായാണ് ഇസ്‌ലാമിക കര്‍മശാസ്ത്രത്തില്‍ നിലനില്‍ക്കുന്നത്. ഇസ്‌ലാം പൂര്‍വ അറബികള്‍ക്കിടയിലും അതങ്ങനെ മാത്രമാണ് നിലനിന്നത്. ഹജറുല്‍ അസ്‌വദിനെ ആരാധിക്കുന്ന യാതൊരു കര്‍മവും ഹജ്ജോ ഉംറയോ ഉള്‍ക്കൊള്ളുന്നില്ല, ഉള്‍ക്കൊണ്ടിട്ടുമില്ല. ചരിത്രത്തില്‍ നിന്നോ കഅ്ബയുടെ നിര്‍മാണരീതിയില്‍ നിന്നോ കഅ്ബയ്ക്ക് ഇബ്‌റാഹിം നബി (അ)യുമായുള്ള പൊക്കിള്‍ക്കൊടി ബന്ധം നിഷേധിക്കാനാവശ്യമായ യാതൊരു തെളിവും ഹാജരാക്കാനാവില്ല എന്നു ചുരുക്കം.

ആരാധിക്കപ്പെടുന്ന കല്ലുകള്‍ ദൈവത്തിന്റെ പ്രവാചകന്‍മാര്‍ സ്ഥാപിക്കുകയില്ലെന്ന കാര്യത്തില്‍ യാതൊരു സംശയവുമില്ല. എന്നാല്‍ സാക്ഷാല്‍ ദൈവത്തെ ആരാധിക്കുവാനുള്ള സ്ഥലം അടയാളപ്പെടുത്താന്‍വേണ്ടി ദൈവനിര്‍ദേശപ്രകാരം കല്ലുകള്‍ വെക്കുക പ്രവാചകന്‍മാരുടെ സമ്പ്രദായം തന്നെയാണ്. കഅ്ബയുടെ കാര്യത്തില്‍ ഇബ്‌റാഹിം നബി(അ) ചെയ്തത് അതുമാത്രമാണ്. ആരാധനാലയത്തിന്റെ നിര്‍മാണം തുടങ്ങാന്‍വേണ്ടി ഉപയോഗിച്ച കല്ലിനെ ചൂണ്ടിക്കാണിച്ച് കഅ്ബ അബ്രഹാമികമല്ലെന്ന് ‘സമര്‍ത്ഥിക്കുന്നവര്‍’, ഈ രീതി പ്രവാചകന്‍മാര്‍ക്കുണ്ടായിരുന്നുവെന്ന് ബൈബിള്‍ തന്നെ വ്യക്തമാക്കുന്നുണ്ടെന്ന വസ്തുതയെ വളരെ സമര്‍ത്ഥമായി മറച്ചുവെക്കുകയാണ് ചെയ്യുന്നത്. അബ്രഹാമിന്റെ പുത്രന്‍ ഇസ്ഹാഖിന്റെ പുത്രനും ഇസ്രാഈല്‍ ഗോത്രങ്ങളുടെ പിതാവുമായ യാക്കോബ് പ്രവാചകന്റെ ജീവിതത്തില്‍ നിന്ന് ബൈബിള്‍ പഴയ നിയമം ഉദ്ധരിക്കുന്ന ഒരു സംഭവം നോക്കുക: ”യാക്കോബ് ബേര്‍ഷെബായില്‍ നിന്ന് ഹാരാനിലേക്ക് പുറപ്പെട്ടു. സൂര്യന്‍ അസ്തമിച്ചപ്പോള്‍ അവന്‍ വഴിക്ക് ഒരിടത്തു തങ്ങുകയും രാത്രി അവിടെ ചെലവഴിക്കുകയും ചെയ്തു. ഒരു കല്ലെടുത്ത് തലയ്ക്കുകീഴെ വെച്ച് അവന്‍ ഉറങ്ങാന്‍ കിടന്നു. അവന് ഒരു ദര്‍ശനം ഉണ്ടായി; ഭൂമിയില്‍ ഉറപ്പിച്ചിരുന്ന ഒരു ഗോവണി, അതിന്റെ അറ്റം ആകാശത്തു മുട്ടിയിരിക്കുന്നു. ദൈവദൂതന്‍മാര്‍ അതിലൂടെ കയറുകയും ഇറങ്ങുകയും ചെയ്തുകൊണ്ടിരുന്നു. ഗോവണിയുടെ മുകളില്‍ നിന്നുകൊണ്ട് കര്‍ത്താവ് അരുളി ചെയ്തു: ഞാന്‍ നിന്റെ പിതാവായ അബ്രഹാമിന്റെയും ഇസ്ഹാഖിന്റെയും ദൈവമായ കര്‍ത്താവാണ്. നീ കിടക്കുന്ന ഈ മണ്ണ് നിനക്കും നിന്റെ സന്തതികള്‍ക്കും ഞാന്‍ നല്‍കും. നിന്റെ സന്തതികള്‍ ഭൂമിയിലെ പൂഴിപോലെ എണ്ണമറ്റവരായിരിക്കും. കിഴക്കോട്ടും പടിഞ്ഞാറോട്ടും തെക്കോട്ടും വടക്കോട്ടും നിങ്ങള്‍ വ്യാപിക്കും. നിന്നിലൂടെയും നിന്റെ സന്തതികളിലൂടെയും ഭൂമിയിലെ ഗോത്രങ്ങളെല്ലാം അനുഗ്രഹിക്കപ്പെടും. ഇതാ, ഞാന്‍ നിന്നോടു കൂടെയുണ്ട്. നീ പോകുന്നിടത്തെല്ലാം ഞാന്‍ നിന്നെ കാത്തുരക്ഷിക്കും. നിന്നെ ഈ നാട്ടിലേക്ക് തിരികെ കൊണ്ടുവരും. നിന്നോട് പറഞ്ഞതൊക്കെ നിറവേറ്റുന്നതുവരെ ഞാന്‍ നിന്നെ കൈവിടുകയില്ല. അപ്പോള്‍ യാക്കോബ് ഉറക്കില്‍ നിന്നുണര്‍ന്നു. അവന്‍ പറഞ്ഞു: തീര്‍ച്ചയായും കര്‍ത്താവ് ഈ സ്ഥലത്തുണ്ട്. എന്നാല്‍, ഞാന്‍ അതറിഞ്ഞില്ല. ഭീതിപൂണ്ട് അവന്‍ പറഞ്ഞു; ഈ സ്ഥലം എത്ര ഭയാനകമാണ്! ഇത് ദൈവത്തിന്റെ ഭവനമല്ലാതെ മറ്റൊന്നുമല്ല. സ്വര്‍ഗത്തിന്റെ കവാടമാണിവിടം. യാക്കോബ് അതിരാവിലെ എഴുന്നേറ്റ് തലയ്ക്കുകീഴെ വെച്ചിരിക്കുന്ന കല്ലെടുത്ത് ഒരു തൂണായി കുത്തിനിര്‍ത്തി അതിന്‍മേല്‍ എണ്ണയൊഴിച്ചു. അവന്‍ ആ സ്ഥലത്തിന് ബഥേല്‍ എന്നു പേരിട്ടു. ലൂസ് എന്നായിരുന്നു ആ പട്ടണത്തിന്റെ ആദ്യത്തെ പേര്. അതുകഴിഞ്ഞ് യാക്കോബ് ഒരു പ്രതിജ്ഞ ചെയ്തു. ദൈവമായ കര്‍ത്താവ് എന്റെ കൂടെ ഉണ്ടായിരിക്കുകയും, ഈ യാത്രയില്‍ എന്നെ സംരക്ഷിക്കുകയും, എനിക്ക് ഉണ്ണാനും ഉടുക്കാനും തരികയും, എന്റെ പിതാവിന്റെ വീട്ടിലേക്ക് സമാധാനത്തോടെ ഞാന്‍ തിരിച്ചെത്തുകയും ചെയ്താല്‍ കര്‍ത്താവായിരിക്കും എന്റെ ദൈവം. തൂണായി കുത്തി നിര്‍ത്തിയിരിക്കുന്ന ഈ കല്ല് ദൈവത്തിന്റെ ഭവനമായിരിക്കും. അവിടുന്ന് എനിക്ക് തരുന്നതിന്റെയെല്ലാം പത്തിലൊന്ന് ഞാന്‍ അവിടുത്തേക്ക് സമര്‍പ്പിക്കുകയും ചെയ്യും.” (ഉല്‍പത്തി 28 : 10-22).

ദൈവം തന്റെ ജനതക്ക് നല്‍കിയ വാഗ്ദത്ത വിശുദ്ധഭൂമിയില്‍ ദൈവനിര്‍ദേശപ്രകാരം ആരാധനാലയമടയാളപ്പെടുത്താന്‍ കല്ലുനാട്ടിയ യാക്കോബ് ചെയ്തുവെന്ന്  ബൈബിള്‍ പറയുന്നതിലധികമൊന്നും അബ്രഹാമും ഇശ്മയേലും മക്ക എന്ന വാഗ്ദത്ത വിശുദ്ധ ഭൂമിയില്‍ ഹജറുല്‍ അസ്‌വദ് എന്ന മൂലശില കൊണ്ട് കഅ്ബയുടെ നിര്‍മാണമാരംഭിച്ചപ്പോള്‍ സംഭവിച്ചിട്ടില്ല എന്നതാണ് സത്യം. ഹജറുല്‍ അസ്‌വദ് ചൂണ്ടിക്കാണിച്ച് കഅ്ബയുടെ അബ്രഹാമിക പ്രവാചകപാരമ്പര്യം നിഷേധിക്കുന്നത് അതിനാല്‍ തന്നെ, എല്ലാ അര്‍ത്ഥത്തിലും അടിസ്ഥാനരഹിതമാണ്.

ഇസ്‌ലാമിക പാരമ്പര്യം പറയുന്നതുപോലെ അബ്രഹാമിനോട് ദൈവം ബലിയായി നല്‍കാന്‍ നിര്‍ദ്ദേശിച്ച് പരീക്ഷിച്ചത് ഇശ്മയേലിനെയല്ല, ഇസ്ഹാഖിനെയാണ്. ഇശ്മയേലിനു പകരമല്ല, ഇസ്ഹാഖിനു പകരമാണ് അബ്രഹാം ആടിനെ ബലിയറുത്തത്. അതിനാല്‍ ഹജ്ജിന്റെ ഭാഗമായുള്ള ഇസ്‌ലാമിക ബലി അബ്രഹാമികമല്ല, മറിച്ച് ബഹുദൈവാരാധനാപരമായ മക്കന്‍ ആചാരങ്ങളുടെ ശേഷിപ്പാണ്. മക്കയുടെ അബ്രഹാമിക പൈതൃകത്തെ നിരാകരിക്കാന്‍ ശ്രമിച്ചുകൊണ്ടുള്ള മിഷനറി വിശകലനങ്ങളില്‍ സര്‍വസാധാരണമായ ഈ വാദങ്ങള്‍ ശരിയാണോ?

ല്ല. ദൈവികനിര്‍ദ്ദേശപ്രകാരം ഇശ്മയേലിനെ ബലിയറുക്കാനൊരുങ്ങിയപ്പോഴല്ല മറിച്ച് ഇസ്ഹാഖിനെ ബലിയറുക്കാനൊരുങ്ങിയപ്പോഴാണ് ആടിനെ ബലിയായി നില്‍കാനുള്ള കല്‍പന അബ്രഹാമിന് ലഭിച്ചതെന്ന് ബൈബിള്‍ പറയുന്നുവെന്നും അതിനാല്‍ ഹജ്ജിന്റെ മൃഗബലി ബഹുദൈവാരാധനാപരമായ പശ്ചാത്തലങ്ങളില്‍ നിന്നുടലെടുത്തതാണെന്നുമാണ് മിഷനറിമാര്‍ വാദിക്കുറുള്ളത്.

ഒന്നാമതായി, ബൈബിള്‍ പറയുന്നുവെന്നതുകൊണ്ടു മാത്രം അബ്രഹാം ബലി നല്‍കാനൊരുങ്ങിയത് ഇസ്ഹാഖിനെയാണെന്ന് സ്ഥാപിക്കപ്പെടുകയില്ല. ആധികാരികമായ ഒരു ചരിത്രസ്രോതസ്സല്ലാത്തതുകൊണ്ടുതന്നെ, ബൈബിള്‍ വിവരണങ്ങളുടെ മാത്രം വെളിച്ചത്തില്‍ ഇശ്മയേലിനെ ബലി നല്‍കാനാണ് ദൈവനിര്‍ദേശമുണ്ടായത് എന്ന നിലപാടിനെ നിരാകരിക്കുന്നത് അര്‍ത്ഥശൂന്യമാണ്.

രണ്ടാമതായി, ഇസ്ഹാഖിനെയായിരുന്നോ ഇശ്മയേലിനെയായിരുന്നോ ദൈവം ബലിക്ക് നിര്‍ദ്ദേശിച്ചത് എന്ന തര്‍ക്കത്തിന്, ഹജ്ജിന്റെ ഭാഗമായ ബലികര്‍മം ബഹുദൈവാരാധനാപരമായ വേരുകളുള്ളതാണെന്ന് സ്ഥാപിക്കാനുള്ള മിഷനറി പരിശ്രമത്തില്‍ ഒരു പങ്കും വഹിക്കാനില്ല. തീര്‍ത്ഥാടകര്‍ സ്വന്തം സമ്പത്ത് ചെലവഴിച്ച് വാങ്ങിയറുത്ത മൃഗങ്ങളുടെ മാംസം പ്രപഞ്ചനാഥന്റെ തൃപ്തി കാംക്ഷിച്ച് പാവങ്ങള്‍ക്ക് വിതരണം ചെയ്യാന്‍ സന്നദ്ധരാകുകയാണ് ഹജ്ജ് ബലിയില്‍ സംഭവിക്കുന്നത്. പ്രപഞ്ചരക്ഷിതാവല്ലാത്ത മറ്റൊരു ശക്തിയോടുമുള്ള ഭക്തി ഇസ്‌ലാമിക ബലികര്‍മത്തില്‍ കടന്നുവരുന്നില്ല. പ്രപഞ്ചനാഥനായ അല്ലാഹുവിലേക്ക്, ബലിയറുക്കപ്പെടുന്ന മൃഗത്തിന്റെ മാംസമോ രക്തമോ അല്ല, മറിച്ച് അതിന് സന്നദ്ധനാകുന്ന വ്യക്തിയുടെ മനസ്സിന്റെ നന്മയാണ് എത്തിച്ചേരുന്നതെന്ന് ക്വുര്‍ആന്‍ വ്യക്തമാക്കുന്നുണ്ട്. ”അവയുടെ മാംസമോ രക്തമോ അല്ലാഹുവിങ്കല്‍ എത്തുന്നതേയില്ല. എന്നാല്‍ നിങ്ങളുടെ ധര്‍മ്മനിഷ്ഠയാണ് അവങ്കല്‍ എത്തുന്നത്.” (22 : 37).

ശുദ്ധ ഏകദൈവാരാധനയുടെ ഭാഗമായി, ദൈവത്തിന് ഭക്ഷണമാവശ്യമാണെന്ന യാതൊരു തെറ്റിദ്ധാരണയുമില്ലാതെ നിര്‍വഹിക്കപ്പെടുന്ന സേവനപ്രവര്‍ത്തനമാണ് ഹജ്ജ് ബലിയും അതോടനുബന്ധിച്ച മാംസവിതരണവുമെന്ന് ചുരുക്കം. ഇതില്‍ ബഹുദൈവാരാധനയുടെ എന്ത് അടരുകളുണ്ടെന്നാണ് മിഷനറിമാര്‍ പറയുന്നത്?

പല ബഹുദൈവാരാധക സമൂഹത്തിന്റെയും അനുഷ്ഠാനമുറകളില്‍ മൃഗബലിയുണ്ടെന്ന കാര്യത്തില്‍ സംശയമൊന്നുമില്ല. പ്രപഞ്ചനാഥനു പുറമെയുള്ള വിഗ്രഹങ്ങളുടെയും സാങ്കല്‍പിക അദൃശ്യശക്തികളുടെയും പൊരുത്തത്തിനുവേണ്ടിയും പലപ്പോഴും അവയെ ‘ഊട്ടാന്‍’ വേണ്ടിയും നിര്‍വഹിക്കപ്പെടുന്ന അന്ധവിശ്വാസ ജഡിലമായ അറവുകളാണവ. പ്രവാചകന്‍മാര്‍ ഏകദൈവവിശ്വാസത്തിന്റെ ഭാഗമായി പഠിപ്പിച്ച ശരിയായ ബലിരീതികളില്‍ നിന്ന് വ്യതിചലിച്ചുപോയാണ് ബഹുദൈവാരാധകര്‍ ഇത്തരം ദുരാചാരങ്ങളില്‍ എത്തിപ്പെട്ടതെന്നാണ് മതഗ്രന്ഥങ്ങളില്‍ നിന്നെല്ലാം മനസ്സിലാക്കാന്‍ കഴിയുന്നത്. മക്കയിലെ ബഹുദൈവാരാധക സംസ്‌കൃതിയിലും ഇത്തരത്തിലുള്ള പല ബലികളുമുണ്ടായിരുന്നു. എന്നാല്‍ ഹജ്ജിന്റെ ഭാഗമായി മിനയില്‍ വെച്ചു നടന്നിരുന്ന ബലി, ഇസ്‌ലാം പൂര്‍വകാലത്തുപോലും ഒരിക്കലും വിഗ്രഹങ്ങളുടെ പ്രസാദത്തിനുവേണ്ടി നിര്‍വഹിക്കപ്പെട്ടിരുന്നില്ല. അത് എപ്പോഴും അല്ലാഹുവിനെ മാത്രമാണ് ലക്ഷ്യമാക്കിയിരുന്നത്. പിന്നെയെങ്ങനെയാണ് ഹജ്ജ് ബലി ബഹുദൈവാരാധനയുടെ ശേഷിപ്പാണെന്ന് മിഷനറിമാര്‍ വാദിക്കുക? എന്ത് തെളിവാണ് ചരിത്രപരമായി അവര്‍ക്കീ വിഷയത്തില്‍ ഹാജരാക്കാനുള്ളത്? ഇബ്‌റാഹീമിനോട് ദൈവം സംസാരിച്ചത് ഏത് പുത്രന്റെ കാര്യമായിരുന്നാലും, ഇബ്‌റാഹീം പഠിപ്പിച്ച ഏകദൈവാരാധനാപരമായ മൃഗബലി ഒരു സമൂഹത്തില്‍ നിലനില്‍ക്കുന്നതില്‍ എന്ത് അസാംഗത്യമാണുള്ളത്?

പ്രപഞ്ചനാഥന്റെ പ്രീതി ഉദ്ദേശിച്ചുള്ള മൃഗബലി പ്രവാചകാധ്യാപനമാണെന്നും ബഹുദൈവാരാധനയുടെ ശേഷിപ്പല്ലെന്നുമാണ് ബൈബിളിന്റെ അസന്നിഗ്ധമായ വീക്ഷണം. ബൈബിളെഴുത്തുകാര്‍ യഹോവക്കുവേണ്ടിയുള്ള മൃഗബലിയുടെ വിശദമായ കര്‍മശാസ്ത്രം വിവരിച്ചിട്ടുള്ളത് പൂര്‍ണമായും പ്രവാചകാധ്യാപനങ്ങളുടെ വെളിച്ചത്തിലാണോ എന്ന കാര്യം സംശയാസ്പദമാണെങ്കിലും, ഏകദൈവാരാധനയുടെ ഭാഗമായി പ്രവാചകനിര്‍ദ്ദേശങ്ങള്‍ പ്രകാരംതന്നെ സെമിറ്റിക് ചരിത്രത്തിലുടനീളം മൃഗബലി നിലനിന്നുവെന്ന് അവ വ്യക്തമാക്കുന്നുണ്ടെന്ന കാര്യം ശ്രദ്ധേയമാണ്.

ഫലസ്ത്വീനിലെ വിശുദ്ധഗേഹം കേന്ദ്രീകരിച്ച് ജൂതന്‍മാര്‍ നടത്തിയിരുന്ന മതാനുഷ്ഠാനങ്ങളുടെ സുപ്രധാനമായ ഭാഗം തന്നെ മൃഗബലിയായിരുന്നു. സി.ഇ എഴുപതില്‍ റോമക്കാര്‍ ആരാധനാലയം തകര്‍ത്തപ്പോഴാണ് മൃഗബലി ജൂതന്‍മാര്‍ക്കിടയില്‍നിന്ന് ഇല്ലാതായിത്തുടങ്ങിയതെന്നാണ് ഗവേഷകരുടെയെല്ലാം അഭിപ്രായം. പഴയനിയമ പ്രവാചകന്‍മാരെല്ലാം പഠിപ്പിച്ച അനുഷ്ഠാന ചട്ടക്കൂടിനെ ദുര്‍ബലപ്പെടുത്തി പൗലോസ് സ്ഥാപിച്ച പുതിയ മതത്തിന്റെ വക്താക്കളായി മാറിയപ്പോഴാണ് ക്രൈസ്തവരില്‍ നിന്ന് ബൈബിളികമായ മൃഗബലി സമ്പ്രദായം അന്യം നിന്നുപോയത്. ബൈബിളില്‍ ഇപ്പോഴും വളരെ വ്യക്തമായി അനുശാസിക്കപ്പെടുന്ന, എന്നാല്‍ അതിന്റെ അനുയായികള്‍ യഥാവിധി പിന്തുടരാന്‍ സന്നദ്ധത കാണിക്കാത്ത പ്രവാചകപാരമ്പര്യമാണ് മൃഗബലിയെന്നര്‍ത്ഥം. ഇത് ഇസ്മാഈലിന്റെ സന്തതിപരമ്പരകളിലെങ്കിലും അവശേഷിക്കുന്നതില്‍ ബൈബിളിന്റെ വക്താക്കള്‍ വേവലാതിപ്പെടുന്നതെന്തിനാണ്?

ആദിമനുഷ്യനായ ആദാമിനു തന്നെ മൃഗബലിയുടെ മഹത്വത്തെക്കുറിച്ച് ദൈവം അറിവു നല്‍കിയിരുന്നുവെന്നാണ് ബൈബിളില്‍ നിന്നു മനസ്സിലാക്കാന്‍ കഴിയുന്നത്. അതുകൊണ്ടാണല്ലോ, ആദാമിന്റെ പുത്രനായ ആബേല്‍ ”തന്റെ ആട്ടിന്‍കുട്ടത്തിലെ കടിഞ്ഞൂല്‍ കുഞ്ഞുങ്ങളെയെടുത്ത് അവയുടെ കൊഴുപ്പുള്ള ഭാഗങ്ങള്‍ അവിടുത്തേക്ക് കാഴ്ചവെച്ചതും, ആബേലിലും അവന്റെ കാഴ്ചവസ്തുക്കളിലും അവിടുന്ന്” പ്രസാദിച്ചതും (ഉല്‍പത്തി 4 : 4). ഇവിടം മുതല്‍ പഴയനിയമത്തില്‍ മുഴുവന്‍ മൃഗബലി വിവരണങ്ങള്‍ നിറഞ്ഞുനില്‍ക്കുന്നതാണ് നമുക്ക് ബൈബിളില്‍ കാണാന്‍ കഴിയുന്നത്. ദൈവത്തില്‍ നിന്ന് ലഭിച്ച അനുഗ്രഹങ്ങള്‍ക്കുള്ള നന്ദിയായും നേര്‍ച്ചയുടെ ഭാഗമായും പെസഹ ആഘോഷത്തിനുവേണ്ടിയും തെറ്റുകള്‍ക്കുള്ള പ്രായശ്ചിത്തമായും അനുശോചനച്ചടങ്ങെന്ന നിലയിലും സ്വതതന്ത്രമായ പുണ്യകര്‍മമെന്ന നിലക്കുമെല്ലാം പുരോഹിതന്‍മാരും രാജാക്കന്‍മാരും സമൂഹവും വ്യക്തികളുമെല്ലാം നടത്തിയ മൃഗബലികളുടെ കഥകള്‍ കൊണ്ട് സമൃദ്ധമാണ് ബൈബിള്‍ പഴയനിയമം. ബലിമൃഗത്തെ അറുത്തശേഷം പുകയും മാംസഗന്ധവും ദൈവത്തിലേക്കയക്കാന്‍ വേണ്ടി മൃഗശരീരം കത്തിക്കുന്നതുമുതല്‍ ബലിമൃഗത്തിന്റെ രക്തം പുരോഹിത നേതൃത്വത്തില്‍ അള്‍ത്താരക്കുചുറ്റും തളിക്കുന്നതുവരെയുള്ള, ഇസ്‌ലാമിക കര്‍മശാസ്ത്രത്തിന് പരിചയമില്ലാത്ത രീതികളും ജൂതസമൂഹത്തില്‍ മൃഗബലിയുടെ ഭാഗമായി നിലനിന്നിരുന്നുവെന്നത് വ്യാപകമായി അറിയപ്പെടുന്ന ചരിത്രവസ്തുതയാണ്. ബൈബിളും ബൈബിളിന്റെ വെളിച്ചത്തില്‍ ജീവിച്ച സമൂഹവും മൃഗബലിയെ മനസ്സിലാക്കിയ രീതിയാണ് ഇതില്‍ നിന്നെല്ലാം വ്യക്തമാകുന്നത്. മൃഗബലി എന്നു കേള്‍ക്കുമ്പോഴേക്കും ബഹുദൈവാരാധനയെക്കുറിച്ച് ചിന്തിക്കുവാനാരംഭിക്കുന്ന ആധുനിക മിഷനറിമാര്‍, തങ്ങളുടെ സ്വന്തം മതഗ്രന്ഥത്തെ തന്നെയാണ് അപഹസിക്കുന്നത് എന്നതത്രെ സത്യം.

മൂന്നാമതായി പരിശോധിക്കുവാനുള്ളത്, ഇസ്ഹാഖിനെ ബലിയറുക്കുവാനാണ് ദൈവനിര്‍ദ്ദേശമുണ്ടായത് എന്നാണ് ബൈബിളെഴുത്തുകാര്‍ മനസ്സിലാക്കിയത് എന്ന അവകാശവാദത്തിന്റെ  വസ്തുതാപരതയാണ്. മക്കക്കടുത്തുള്ള മിനയില്‍വെച്ച് ഇബ്‌റാഹീം ദൈവനിര്‍ദ്ദേശപ്രകാരം ഇസ്മാഈലിനെ ബലിയറുക്കാനൊരുങ്ങിയെന്നും ഇസ്മാഈലിനെ ബലിയാക്കലായിരുന്നില്ല, മറിച്ച് ദൈവനിര്‍ദ്ദേശങ്ങള്‍ അപ്പടി സ്വീകരിക്കുവാനുള്ള ത്യാഗസന്നദ്ധത ഇബ്‌റാഹീമില്‍ നിന്ന് ആവശ്യപ്പെടുകയായിരുന്നു ഇവ്വിഷയകമായ ദൈവിക പദ്ധതി എന്നതിനാല്‍ ബലിയറുക്കാനുള്ള സന്നദ്ധത ഇബ്‌റാഹീം തെളിയിച്ചപ്പോള്‍ മകനെ ബലിയറുക്കേണ്ടതില്ലെന്നു പറഞ്ഞ് പകരം ബലിയറുക്കാനായി ദൈവം ഒരാടിനെ നല്‍കിയെന്നുമുള്ള അറബ് ഇസ്‌ലാമിക പാരമ്പര്യത്തെ നിഷേധിക്കാനാണ് ബൈബിളുപയോഗിച്ച് മിഷനറിമാര്‍ മെനക്കെടുന്നത്.

ഇസ്ഹാഖിനെ ബലി നല്‍കുവാനാണ് ദൈവം ആവശ്യപ്പെട്ടതെന്ന് ബൈബിള്‍ പറയുന്നുവെന്ന കാര്യം ശരിയാണ്. എന്നാല്‍ ഈ പരാമര്‍ശമുള്‍ക്കൊള്ളുന്ന ഖണ്ഡിക പൂര്‍ണമായി വായിച്ചാല്‍ ഇസ്ഹാഖ് എന്ന പദം അതില്‍ പിന്നീടാരോ എഴുതിച്ചേര്‍ത്തതാണെന്ന് പകല്‍പോലെ വ്യക്തമാണ്. ബൈബിള്‍ വചനങ്ങള്‍ നോക്കുക: ”പിന്നീടൊരിക്കല്‍ ദൈവം അബ്രഹാമിനെ പരീക്ഷിച്ചു. അബ്രഹാം, അവിടുന്ന് വിളിച്ചു. ഇതാ ഞാന്‍, അവന്‍ വിളികേട്ടു. നീ സ്‌നേഹിക്കുന്ന നിന്റെ ഏക മകന്‍ ഇസ്ഹാഖിനെയും കൂട്ടിക്കൊണ്ട് കോറിയ ദേശത്തേക്കു പോവുക. അവിടെ ഞാന്‍ കാണിച്ചുതരുന്ന മലമുകളില്‍ നീ അവനെ എനിക്ക് ഒരു ദഹനബലിയായി അര്‍പ്പിക്കണം…. ദൈവം പറഞ്ഞ സ്ഥലത്തെത്തിയപ്പോള്‍ അബ്രഹാം അവിടെ ഒരു ബലിപീഠം പണിതു. വിറക് അടുക്കിവെച്ചിട്ട് ഇസ്ഹാഖിനെ ബന്ധിച്ച് വിറകിനുമീതെ കിടത്തി. മകനെ ബലി കഴിക്കാന്‍ അബ്രഹാം കത്തി കയ്യിലെടുത്തു. തല്‍ക്ഷണം കര്‍ത്താവിന്റെ ദൂതന്‍ ആകാശത്തുനിന്ന് അബ്രഹാം അബ്രഹാം എന്നുവിളിച്ചു. ഇതാ ഞാന്‍, അവന്‍ വിളികേട്ടു. കുട്ടിയുടെ മേല്‍ കൈവെക്കരുത്. അവനെ ഒന്നും ചെയ്യരുത്. നീ ദൈവത്തെ ഭയപ്പെടുന്നുവെന്ന് എനിക്കിപ്പോള്‍ ഉറപ്പായി. കാരണം, നിന്റെ ഏകപുത്രനെ എനിക്കു തരാന്‍ നീ മടി കാണിച്ചില്ല…… കര്‍ത്താവിന്റെ ദൂതന്‍ ആകാശത്തുനിന്ന് വീണ്ടും അബ്രഹാമിനെ വിളിച്ചുപറഞ്ഞു: കര്‍ത്താവ് അരുളി ചെയ്യുന്നു, നീ നിന്റെ ഏകപുത്രനെപ്പോലും എനിക്കുതരാന്‍ മടിക്കായ്ക കൊണ്ട് ഞാന്‍ ശപഥം ചെയ്യുന്നു: ഞാന്‍ നിന്നെ സമൃദ്ധമായി അനുഗ്രഹിക്കും.” (ഉല്‍പത്തി 22 : 1-17).

 ദൈവം ആവശ്യപ്പെട്ടതും അബ്രഹാമിന്റെ ആത്മാര്‍പ്പണത്തെ പ്രശംസിച്ചതുമെല്ലാം ബൈബിള്‍പ്രകാരം അബ്രഹാമിന്റെ ‘ഏക പുത്രനെ’ പരാമര്‍ശിച്ചുകൊണ്ടാണ്. അബ്രഹാമിന് ആദ്യമുണ്ടായ പുത്രന്‍ ഇസ്മാഈല്‍ ആണെന്ന് ഏത് ബൈബിള്‍ വായനക്കാരനാണ് അറിയാത്തത്? ഇസ്ഹാഖ് ജനിക്കുന്നതിനുമുമ്പാണ് ഈ സംഭവങ്ങളെല്ലാമുണ്ടായതെന്ന് ‘ഏകപുത്രന്‍’ എന്ന പ്രയോഗത്തില്‍നിന്ന് സുതരാം വ്യക്തമാണ്. ഇസ്മാഈലിനെയാണ് ഇബ്‌റാഹീം ബലിയറുക്കാനായി കൊണ്ടുപോയതെന്ന് ബൈബിളെഴുത്തുകാര്‍ക്കു പോലും ബോധ്യമുണ്ടായിരുന്നുവെന്നര്‍ത്ഥം.

ഇസ്ഹാഖിനെയും ഇസ്ഹാഖ് ജീവിച്ച പ്രദേശത്തിന്റെ ഭൂമിശാസ്ത്രത്തെയും ഈ സംഭവവിവരണത്തിലേക്ക് പിന്നീടാരോ ചേര്‍ത്തുവെച്ചതാണെന്ന് മനസ്സിലാക്കാനേ നിഷ്പക്ഷരായ ബൈബിള്‍ പഠിതാക്കള്‍ക്ക് കഴിയൂ. ഇതിന് പ്രേരകമായി വര്‍ത്തിച്ചത് യഹൂദവംശീയതയാണെന്ന കാര്യത്തില്‍ സംശയമൊന്നുമില്ല. തങ്ങളുടെ വംശപിതാവായ യഅ്ക്വൂബിന്റെ പിതാവിനില്ലാത്ത മഹത്വം അറബികളുടെ പിതാവായ ഇസ്മാഈലിനുണ്ടായിക്കൂടെന്ന യഹൂദശാഠ്യത്തില്‍ നിന്നുണ്ടായ ഒരു  കൈക്രിയയെയാണ് മിഷനറിമാര്‍ ‘ചരിത്ര’മായി അവതരിപ്പിക്കുന്നത് എന്ന കാര്യം എന്തുമാത്രം സഹതാപാര്‍ഹമല്ല!

ഹാജറ അടിമസ്ത്രീയായിരുന്നുവെന്നും അവരിലുണ്ടായ പുത്രന്‍ അബ്രഹാമിന്റെ യഥാര്‍ത്ഥ പുത്രനല്ലെന്നും ചില മിഷനറിമാര്‍ വാദിച്ചുനോക്കാറുണ്ട്. മാനവവിരുദ്ധമെന്നതിനു പുറമെ, ബൈബിള്‍ വിരുദ്ധം കൂടിയാണ് ഈ നിലപാട്. അടിമസ്ത്രീയായിരുന്ന ഹാജറയെ ഭാര്യയായാണ് അബ്രഹാം സ്വീകരിച്ചതെന്ന് ബൈബിള്‍ വ്യക്തമാക്കുന്നുണ്ട്. ”കാനാന്‍ ദേശത്ത് പത്തു വര്‍ഷം താമസിച്ചു കഴഞ്ഞപ്പോള്‍ അവന്റെ ഭാര്യ സാറ ദാസിയായ ഈജിപ്തുകാരി ഹാഗറിനെ തന്റെ ഭര്‍ത്താവിന് ഭാര്യയായി നല്‍കി.” (ഉല്‍പത്തി 16 : 3).

ഇസ്മാഈലിനെ, വളര്‍ച്ചയുടെ ഓരോ ഘട്ടത്തിലും പുത്രനായിത്തന്നെയാണ് അബ്രഹാം പരിഗണിച്ചതെന്നാണ് ബൈബിളിന്റെയും പക്ഷം. സാറയുടെ അടിമസ്ത്രീയായിരുന്ന ഹാഗാറിനെ അബ്രഹാമിനും സാറക്കുമൊന്നും ഇഷ്ടമുണ്ടായിരുന്നില്ലെന്നും സാറയെയാണ് തന്റെ പ്രിയതമയായി അബ്രഹാം പരിഗണിച്ചിരുന്നതെന്നും വംശീയത തലക്കുപിടിച്ച ചില മിഷനറിമാര്‍ സമര്‍ത്ഥിക്കുവാന്‍ ശ്രമിക്കാറുണ്ട്. അവരുടെ സങ്കുചിതത്വത്തെ വാദത്തിനുവേണ്ടി അംഗീകരിച്ചുകൊടുത്താല്‍ തന്നെ ബൈബിള്‍ പ്രകാരം ഇസ്മാഈല്‍ മാത്രമേ അബ്രഹാമിന്റെ ആദ്യ ജാതനാകൂ എന്നുള്ളതാണ് വാസ്തവം. ആവര്‍ത്തന പുസ്തകം പറയുന്നത് കാണുക: ”ഒരാള്‍ക്ക് രണ്ട് ഭാര്യമാരുണ്ടായിരിക്കുകയും അവന്‍ ഒരുവളെ സ്‌നേഹിക്കുകയും മറ്റവളെ ദ്വേഷിക്കുകയും ഇരുവരിലും അവന് സന്താനങ്ങളുണ്ടാവുകയും ആദ്യജാതന്‍ ദ്വേഷിക്കുന്നവളില്‍ നിന്നുള്ളവനായിരിക്കുകയും ചെയ്താല്‍ അവന്‍ തന്റെ വസ്തുവകകള്‍ പുത്രന്‍മാര്‍ക്ക് ഭാഗിച്ചുകൊടുക്കുമ്പോള്‍ താന്‍ വെറുക്കുന്നവളുടെ മകനും ആദ്യജാതനുമായവനെ മാറ്റി നിര്‍ത്തിയിട്ട് പകരം താന്‍ സ്‌നേഹിക്കുന്നവളുടെ മകനെ ആദ്യജാതനായി കണക്കാക്കരുത്. അവന്‍ തന്റെ സകല സമ്പത്തുക്കളുടെയും രണ്ടോഹരി വെറുക്കുന്നവളുടെ മകനുകൊടുത്ത് അവനെ ആദ്യജാതനായി അംഗീകരിക്കണം. അവനാണ് തന്റെ പുരുഷത്വത്തിന്റെ ആദ്യഫലം. ആദ്യജാതന്റെ അവകാശം അവനുള്ളതാണ്.” (ആവര്‍ത്തനം 21 : 15-17).

ചുരുക്കത്തില്‍, ഇസ്മാഈലിനെയല്ല മറിച്ച് ഇസ്ഹാഖിനെയാണ് ബലി നല്‍കാന്‍ ദൈവം നിര്‍ദ്ദേശിച്ചത് എന്ന് ബൈബിള്‍ സ്ഥാപിക്കുന്നുവെന്നും  അതിനാല്‍ ഹജ്ജ് ബലി അബ്രഹാമികമല്ലെന്നുമുള്ള മിഷനറിവാദം എല്ലാ അര്‍ത്ഥത്തിലും അടിസ്ഥാനരഹിതമാണ്.

മുമ്പ് വേദക്കാരികളായിരുന്ന രണ്ട് സ്ത്രീകള്‍ പ്രവാചകന്റെ (സ)ജീവിതപങ്കാളികളായിരുന്നുവെ ന്നത് ശരിയാണ്. യഹൂദനായ ഹുയയ്യുബ്‌നു അക്തബിന്റെ മകള്‍ സഫിയ്യയാണ് ഒന്ന്. ഈജിപ്തിലെ കിബ്ത്തി നേതാവ് സമ്മാനിച്ച മാരിയത്തുല്‍ കിബ്ത്തിയ്യയെ ന്ന ക്രൈസ്തവ വനിതയാണ് മറ്റൊന്ന്. ഇവര്‍ രണ്ടുപേരും പ്രവാചക ജീവിതത്തിലേക്ക് കടന്നുവന്നത് മദീനാ കാലഘട്ടത്തിലാണ്. ബനൂനളീ ര്‍ ഗോത്ര ഉപരോധവുമായി ബന്ധപ്പെട്ട് ഗോത്രത്തലവനായ ഹുയയ്യും സഫിയ്യയുടെ ഭര്‍ത്താവും കൊല്ലപ്പെട്ടതിനെ തുടര്‍ന്നാണ് ഹിജ്‌റ ഏഴാം വര്‍ഷത്തില്‍ അവരെ മുഹമ്മദ് നബി (സ)വിവാഹം ചെയ്യുന്നത്. മാരിയത്തുല്‍ കിബ്ത്തിയ്യയും പ്രവാചക ജീവിതത്തിലേക്ക് കടന്നുവരുന്നത് ഹിജ്‌റ ഏ ഴാം വര്‍ഷത്തില്‍ തന്നെയാണ്. പൂര്‍വ്വ പ്രവാചകന്മാരുടെ ചരിത്രം വിവരിക്കുന്ന ഖുര്‍ആന്‍ സൂ ക്തങ്ങളില്‍ ബഹുഭൂരിഭാഗവും അവതരിപ്പിക്കപ്പെട്ടത് മക്കയിലാണെന്നിരിക്കെ മദീനാ ജീവിതത്തി ന്റെ ഏഴു വര്‍ഷങ്ങള്‍ക്കുശേഷം പ്രവാചക ജീവിതത്തിലേക്ക് കടന്നുവന്ന രണ്ട് വനിതകളെങ്ങനെ യാണ് പ്രസ്തുത ചരിത്രങ്ങളുടെ സ്രോതസ്സായിത്തീരുക?

കോപ്റ്റിക് ക്രൈസ്തവര്‍ക്കിടയില്‍ പ്രചാരത്തിലുണ്ടായിരുന്ന ശൈശവ സുവിശേഷ (Gospel of the Infancy) ത്തിലുള്ള കഥകളാണ് തൊട്ടിലില്‍വെച്ച് ഉണ്ണിയേശു സംസാരിച്ചതായും പ്രസവവേദനയുടെ സമയത്ത് ഈത്തപ്പന കുലുക്കി പഴം ലഭിച്ചതായുമുള്ള കഥകളെല്ലാമെന്നും ഇവ കോപ്റ്റിക് ക്രി സ്ത്യാനിയായിരുന്ന മാരിയത്തുല്‍ കിബ്ത്തിയ്യ പറഞ്ഞുകൊടുത്തതാണെന്നുമാണ് മറ്റൊരു വാദം. ഈ വാദവും അടിസ്ഥാന രഹിതമാണ്. തൊട്ടിലില്‍വെച്ച് യേശു സംസാരിച്ചതായി സൂചിപ്പിക്കുന്ന വാക്യമുള്‍ക്കൊള്ളുന്ന ഖുര്‍ആനിലെ പത്തൊന്‍പതാം അധ്യായം സൂറത്തുമര്‍യം മക്കയില്‍വെച്ച് അവതരിപ്പിക്കപ്പെട്ടതാണ്. ഹിജ്‌റ ഏഴാം നൂറ്റാണ്ടില്‍ മാത്രം പ്രവാചക ജീവിതത്തിലേക്ക് കടന്നു വന്ന മാരിയത്തുല്‍ കിബ്ത്തിയ്യ പറഞ്ഞുകൊടുത്ത കഥയുടെ അടിസ്ഥാനത്തില്‍ മക്കയില്‍ വെച്ച് എങ്ങനെയാണ് മുഹമ്മദ് നബി (സ)ഈ സംഭവങ്ങളെഴൂതുക? മുഹമ്മദ് നബി (സ)യുടെ കാലത്ത് കോപ്റ്റിക് ക്രൈസ്തവര്‍ക്കിടയില്‍ ശൈശവ സുവിശേഷം പ്രചാരത്തിലിരുന്നുവെന്ന് ഖണ്ഡിതമാ യി തെളിയിക്കാന്‍ ഈ വിമര്‍ശനമുന്നയിച്ചവര്‍ക്കൊന്നും കഴിഞ്ഞിട്ടില്ല. ഇനി കഴിഞ്ഞാല്‍തന്നെ യേശുവിന്റെ ശൈശവകാല സംഭവങ്ങളെക്കുറിച്ച ഖുര്‍ആനിക പരാമര്‍ശങ്ങള്‍ ശൈശവ സുവിശേ ഷത്തിന്റെ അടിസ്ഥാനത്തില്‍ എഴുതിയതാണെന്ന് വസ്തുനിഷ്ഠമായി സ്ഥാപിക്കുവാന്‍ ആര്‍ക്കുംതന്നെ സാധിക്കുകയില്ല.

പ്രഗല്‍ഭനായ ഒരു പ്രവാചക ശിഷ്യനായിരുന്നു സല്‍മാനുല്‍ ഫാരിസി (റ). മദീനയ്ക്ക് ചുറ്റും കിടങ്ങ് കുഴിച്ചുകൊണ്ട് മക്കക്കാരുടെ ആക്രമണത്തെ പ്രതിരോധിക്കാമെന്ന അദ്ദേഹത്തിന്റെ നിര്‍ദ്ദേശമാണ് ഖന്‍ദഖ് യുദ്ധത്തില്‍ മുസ്‌ലിംകളുടെ വിജയത്തിന് നിമിത്തമായ പല കാരണങ്ങളിലൊന്ന്. സല്‍മാനു ല്‍ ഫാരിസിയെക്കുറിച്ച് പറയുമ്പോള്‍ ഖന്‍ദഖ് യുദ്ധമാണ് ഇസ്‌ലാമിക ചരിത്രം പഠിച്ചവരുടെ മന സ്സില്‍ ആദ്യമായി ഓടിയെത്തുക.

അഗ്‌നി ആരാധനയിലധിഷ്ഠിതമായ സരതുഷ്ട്രമതത്തിലായിരുന്ന സല്‍മാന്‍ പിന്നീട് ക്രിസ്തുമതം സ്വീകരിച്ചു. സത്യാന്വേഷിയായിരുന്ന അദ്ദേഹത്തിന് ക്രിസ്തുമതത്തിന്റെ ആശയങ്ങള്‍ പൂര്‍ണ     സം തൃപ്തി നല്‍കാത്തതുകൊണ്ട് തന്റെ അന്വേഷണം തുടരുകയും അവസാനം ഇസ്‌ലാമിലെത്തി ച്ചേരുകയും ചെയ്തു. സല്‍മാനുല്‍ ഫാരിസി (റ) ഇസ്‌ലാം സ്വീകരിച്ചത് മദീനയില്‍വെച്ചാണ്. അതി നുശേഷമാണ് അദ്ദേഹം പ്രവാചകന്റെ (സ) സഹചാരിയായിത്തീര്‍ന്നത്.

ഖുര്‍ആനിന്റെ ഏകദേശം മൂന്നില്‍ രണ്ടുഭാഗവും അവതരിപ്പിക്കപ്പെട്ടത് മക്കയില്‍വെച്ചാണ്. പൂര്‍വ്വ പ്രവാചകന്മാരെക്കുറിച്ച പരാമര്‍ങ്ങളധികവും മക്കയില്‍ അവതരിപ്പിക്കപ്പെട്ട സൂക്തങ്ങളിലാണുള്ളത്. മദീനയില്‍ വെച്ച് പ്രവാചകന്റെ അനുചരനായിത്തീര്‍ന്ന സല്‍മാനുല്‍ ഫാരിസി പറഞ്ഞുകൊടുത്ത വിവരങ്ങളുടെ അടിസ്ഥാനത്തില്‍ എങ്ങനെയാണ് മക്കയില്‍വെച്ച് മുഹമ്മദ് നബി (സ) പൂര്‍വ്വ പ്രവാചകന്മാരുടെ ചരിത്രമെഴുതുക?

ഖുര്‍ആനിന് സമാന്തരമായ ഒരു രചനയുണ്ടാക്കുവാനുള്ള അതിന്റെ വെല്ലുവി ളിയും പ്രസ്തുത വെല്ലുവിളിക്ക് ഉത്തരം നല്‍കുന്നതില്‍ അറബി സാഹിത്യകാരന്മാര്‍ കാലാകാലങ്ങ ളായി പരാജയപ്പെടുകയാണ് ചെയ്യുന്നതെന്ന യാഥാര്‍ത്ഥ്യവും  ഖുര്‍ആനിന്റെ സാഹിത്യശൈലി അതുല്യവും അനുകരണാതീതവുമാണെന്ന വസ്തുത വ്യക്തമാക്കുന്നുണ്ട്. അറബിയല്ലാത്ത-പേര്‍ഷ്യ ക്കാരനായ ഒരാളെങ്ങനെയാണ് അതുല്യമായ ഒരു അറബി സാഹിത്യസൃഷ്ടിയുടെ സ്രോതസ്സായിത്തീ രുക?

ഇങ്ങനെ ഏത് കോണിലൂടെ നോക്കിയാലും സല്‍മാനുല്‍ ഫാരിസി (റ)യാണ് ഖുര്‍ആനിലെ ചരി ത്ര കഥനങ്ങളുടെ സ്രോതസ്സെന്ന വാദം പരിഗണന പോലുമര്‍ഹിക്കാത്ത ഒരു കേവല വാദം മാത്രമാണെന്ന വസ്തുത വ്യക്തമാവും.

ഹൂദ ക്രൈസ്തവരോടൊപ്പം ജീവിക്കുവാന്‍ അവസരം ലഭിച്ച മുഹമ്മദ് നബി (സ) അവര്‍ പറ ഞ്ഞിരുന്ന പ്രവാചകകഥകള്‍ കേട്ടിരിക്കാനിടയുണ്ടെന്നും പ്രസ്തുത കഥകളില്‍ സ്വന്തമായ ഭാവന കൂട്ടിക്കലര്‍ത്തി അദ്ദേഹം രൂപപ്പെടുത്തിയെടുത്തതാണ് ഖുര്‍ആനിലെ ചരിത്രകഥകളെന്നും വാദിക്കു ന്നവരുണ്ട്. ഈ വാദം തീരെ ദുര്‍ബ്ബലവും വ്യക്തമായ ചരിത്ര വസ്തുതകള്‍ക്ക് വിരുദ്ധവുമാണ്. താഴെ പറയുന്ന വസ്തുതകള്‍ ശ്രദ്ധിക്കുക:

(1) ജൂതന്മാരൊ ക്രൈസ്തവരോ ഒരു മതസമൂഹമെന്ന നിലയ്ക്ക് മക്കയില്‍ ഉണ്ടായിരുന്നതായി യാ തൊരു രേഖയുമില്ല; ഒരു തെളിവുമില്ല. മുഹമ്മദ് നബി (സ) യുടെ കാലത്തോ മുമ്പോ യഹൂദ മതക്കാ രോ ക്രൈസ്തവരോ മക്കയില്‍ മതസമൂഹങ്ങളായി നിലനിന്നിരുന്നില്ലെന്നാണ് ചരിത്രം വ്യക്തമാ ക്കുന്നത്.

-(1)-  മുഹമ്മദ് നബി (സ) ക്കുമുമ്പുതന്നെ അറേബ്യന്‍ ബഹുദൈവാരാധന വെറുത്ത ഏതാനും മക്ക ക്കാര്‍ സ്വന്തമായി അബ്രാഹാമീ മതത്തിന്റെ വേരുകള്‍ തേടുകയും ഏകദൈവാരാധകരായി നില നില്‍ക്കുകയും ചെയ്തിരുന്നതായി ചരിത്രഗ്രന്ഥങ്ങളില്‍ കാണാന്‍ കഴിയും. 'ഹനീഫുകള്‍' എന്ന് വിളിക്കപ്പെടുന്ന ഇവര്‍ നാല് പേരാണ്. വറഖത്തുബ്‌നു നൗഫല്‍, അബ്ദുല്ലാഹിബ്‌നു ജഹ്ശ്, ഉഥ്മാ നുബ്‌നു ഹുവാരിഥ്, സൈദുബ്‌നു അംറ് എന്നിവരാണവര്‍. തങ്ങളുടെ സമൂഹത്തില്‍ നിലനിന്ന വിഗ്രഹാരാധനയെ വെറുക്കുകയും അബ്രഹാമീ മാര്‍ഗത്തില്‍നിന്ന് സ്വസമൂഹം വഴിതെറ്റിയതില്‍ ദുഃഖിക്കുകയും യഥാര്‍ത്ഥ ദൈവിക മതത്തിന്റെ വേരുകള്‍ തേടിപ്പോവുകയും ചെയ്ത വരായി രുന്നു അവര്‍. അവരിലൊരാളായ വറഖത്തുബ്‌നു നൗഫല്‍ ഈ അന്വേഷണത്തിന്റെ ഫലമായാണ് ക്രിസ്തുമതം സ്വീകരിച്ചത്. ഇവരെല്ലാവരും ഇബ്രാഹീമിന്റെ മതമായ യഥാര്‍ത്ഥ ദൈവിക മതത്തി ന്റെ വേരുകള്‍ തേടി മക്കവിട്ട് വ്യത്യസ്ത നാടുകളില്‍ അലഞ്ഞുതിരിഞ്ഞതായി ചരിത്രഗ്രന്ഥങ്ങള്‍ വ്യക്തമാക്കുന്നു. യഹൂദരോ ക്രൈസ്തവരോ മക്കയില്‍ ഉണ്ടായിരുന്നെങ്കില്‍ ഏകദൈവ വിശ്വാസ ത്തിലധിഷ്ഠിതമായ ഇബ്രാഹീമീ മാര്‍ഗത്തിന്റെ വേരുകള്‍ തേടി അവര്‍ ഒരിക്കലും മക്ക വിടേണ്ടി വരികയില്ലായിരുന്നു.

(ii) യമനില്‍ അതിശക്തമായ ക്രൈസ്തവ ഭരണമായിരുന്നു മുഹമ്മദ് നബി (സ) യുടെ ജനനകാലത്ത് നിലനിന്നിരുന്നത്. ക്രൈസ്തവ ഭരണാധികാരിയായിരുന്ന അബ്‌റഹ മക്കക്കെതിരെ നയിച്ച വിപ്ലവം പ്രസിദ്ധമാണ്. 'ആനക്കലഹം' എന്നറിയപ്പെട്ട പ്രസ്തുത വിപ്ലവം നടന്ന വര്‍ഷമാണ് മുഹമ്മദ് നബി (സ) യുടെ ജനനം. മക്കയിലെ കഅ്ബാലയം പൊളിച്ചു കളയുകയും താന്‍ സന്‍ആയില്‍ നിര്‍മ്മിച്ച ഖുലൈസ് എന്ന ദേവാലയത്തിലേക്ക് ജനശ്രദ്ധയാകര്‍ഷിക്കുകയും ചെയ്യുകയെന്ന ലക്ഷ്യത്തോടെ നടത്തപ്പെട്ട ആനക്കലഹത്തെ അല്ലാഹു അമ്പേ പരാജയപ്പെടുത്തിയ കഥ ഖുര്‍ആനിലെ 105-ാം അധ്യാ യത്തില്‍ സംക്ഷിപ്തമായി വിവരിക്കുന്നുണ്ട്. അതിങ്ങനെയാണ്: ''ആനക്കാരെക്കൊണ്ട് നിന്റെ രക്ഷിതാവ് പ്രവര്‍ത്തിച്ചത് എങ്ങനെയെന്ന് നീ കണ്ടില്ലേ? അവരുടെ തന്ത്രം അവന്‍ പിഴവിലാക്കിയി ല്ലേ? ചുട്ടുപഴുപ്പിച്ച കളിമണ്‍ കല്ലുകള്‍ കൊണ്ട് അവരെ എറിയുന്ന കൂട്ടംകൂട്ടമായിക്കൊണ്ടുള്ള പക്ഷികളെ അവരുടെ നേര്‍ക്ക് അവന്‍ അയക്കുകയും ചെയ്തു. അങ്ങനെ അവന്‍ അവരെ തിന്നൊ ടുക്കപ്പെട്ട വൈക്കോല്‍ തുരുമ്പുപോലെയാക്കി'' (വി.ഖു. 10).

കഅ്ബാലയം തകര്‍ക്കുകയും മക്കക്കാരെ ക്രൈസ്തവവല്‍ക്കരിക്കുകയും ചെയ്യുകയെന്ന ലക്ഷ്യ ത്തോടെയാണ് അബ്‌റഹത്തിന്റെ ആനപ്പടയുടെ പുറപ്പാടുണ്ടായത്. മക്കയില്‍ ക്രൈസ്തവ സമൂഹ മുണ്ടായിരുന്നുവെങ്കില്‍ ഇത്തരമൊരു പടനീക്കമുണ്ടാകുമായിരുന്നില്ലെന്ന് വ്യക്തമാണ്.

(iii) മക്കയില്‍ ഇസ്‌ലാമിനുമുമ്പ് നിലനിന്നിരുന്ന രേഖകളിലോ കവിതകളിലോ ഒന്നുംതന്നെ ജൂതരെ യോ ക്രൈസ്തവരെയോ സംബന്ധിച്ച യാതൊരു പരാമര്‍ശവുമില്ല.

(2) ഖുര്‍ആനില്‍ ആകെ 114 അധ്യായങ്ങളാണുള്ളത്. ഇതില്‍ 27 എണ്ണം മദീനയില്‍ വെച്ചും 87 എണ്ണം മക്കയില്‍വെച്ചുമാണ് അവതരിപ്പിക്കപ്പെട്ടത്.

പ്രവാചക ചരിത്രങ്ങള്‍ വിശദീകരിക്കുന്ന സൂക്തങ്ങളിലധികവും അവതരിപ്പിക്കപ്പെട്ടത് മക്കയി ലാണ്. അവിടെയാകട്ടെ മുഹമ്മദ് നബി (സ) ക്ക് കണ്ടുമുട്ടാനോ സംസാരിക്കാനോ ആയി യഹൂദരോ ക്രൈസ്തവരോ തീരെയുണ്ടായിരുന്നുമില്ല. പിന്നെയെങ്ങനെയാണ് പൂര്‍വ്വ പ്രവാചകന്മാരുടെ ചരി ത്രങ്ങള്‍ ഖുര്‍ആനിലുണ്ടായത്? ഖുര്‍ആന്‍ പറയുന്നതാണ് ശരിയായ ഉത്തരം! ''നിങ്ങളുടെ കൂട്ടുകാ രന്‍ വഴിതെറ്റിയിട്ടില്ല, ദുര്‍മാര്‍ഗിയായിട്ടുമില്ല. അദ്ദേഹം തന്നിഷ്ടപ്രകാരം സംസാരിക്കുന്നുമില്ല. അത് അദ്ദേഹത്തിന് ദിവ്യസന്ദേശമായി നല്‍കപ്പെടുന്ന ഒരു ഉത്‌ബോധനം മാത്രമാകുന്നു'' (വി.ഖു.53:2-4).

(3) മുഹമ്മദ് നബി (സ) യുടെ ജീവിതത്തില്‍, മക്കയിലോ മദീന യിലോവെച്ച് ക്രൈസ്തവ സമൂഹ വുമായി സമ്പര്‍ക്കത്തിലാകേണ്ട സാഹചര്യങ്ങളൊന്നുമുണ്ടായതായി ചരിത്രത്തില്‍നിന്ന് മനസ്സിലാ ക്കാനാവുന്നില്ല. മദീനയില്‍വെച്ച് ജൂത സമൂഹങ്ങളുമായി സമ്പര്‍ക്കത്തിലായിരുന്നു പ്രവാചകനും അനുയായികളുമെന്നത് നേരാണ്. എന്നാല്‍ അവിടെയും ക്രൈസ്തവര്‍ ഒരു സമൂഹമായി നിലനില്‍ ക്കുന്നുണ്ടായിരുന്നില്ലെന്നാണ് മനസ്സിലാക്കാന്‍ കഴിയുന്നത്. ഇക്കാ ര്യം കാത്തോലിക്ക വിജ്ഞാനകോ ശം പോലും സമ്മതിക്കുന്നുണ്ട്. ''ഹിജാസിനെ (അറേബ്യന്‍ ഉപദ്വീപ്) ഒരിക്കലും ക്രിസ്തുമത സന്ദേശ പ്രചരണം സ്പര്‍ശിച്ചിട്ടില്ല. അതുകൊണ്ടുതന്നെ അവിടെ ക്രൈസ്തവസഭകളുടെ ശുചീകരണം പ്രതീക്ഷിക്കാവതല്ല; അതൊട്ടുകാണാനും കഴിയുന്നില്ല'' (The New Catholic Encyclopaedia Vol. I, Page 721-722)

പ്രഗത്ഭ ഗവേഷകനായ റിച്ചാര്‍ഡ് ബെല്ലിന്റെ നിരീക്ഷണവും ഇതുതന്നെ! ''ഹിജാസിലോ മക്കയുടെ യോ മദീനയുടെയെങ്കിലുമോ പ്രാന്തപ്രദേശങ്ങളിലോ ക്രൈസ്തവത നിലനിന്നിരുന്നുവെന്നതിന് യാ തൊരു തെളിവുമില്ല'' (Richard Bell: The Origin of Islam in Its Christian Environment Page 42).

ഖുര്‍ആനില്‍ യേശുവിനെയും മാതാവിനെയും അവരുടെ കുടുംബത്തെയുംകുറിച്ച് പലസ്ഥലങ്ങ ളിലും പരാമര്‍ശിച്ചിട്ടുണ്ട്. ബൈബിളില്‍പോലും പരാമര്‍ശിച്ചിട്ടില്ലാത്ത പല സംഭവങ്ങളും യേശു വിന്റെ ജീവിതവുമായി ബന്ധപ്പെടുത്തി ഖുര്‍ആന്‍ ഉദ്ധരിക്കുന്നുണ്ട്. ഈ കാര്യങ്ങള്‍ മുഹമ്മദ് നബി (സ) ക്ക് എവിടെനിന്നു കിട്ടി?  മറ്റ് പ്രവാചകന്മാരുമായി ബന്ധപ്പെട്ട പരാമര്‍ശങ്ങള്‍ യഹൂദന്മാരുമാ യുള്ള സഹവര്‍ത്തിത്വത്തിന്റെകാലത്ത് അവര്‍ പറഞ്ഞുകൊടുത്ത കഥകളുടെ വെളിച്ചത്തില്‍ എഴു തിയതാണെന്ന് വാദിക്കുന്നവര്‍ യേശുവിനെയും മാതാവിനെയുംക്കുറിച്ച ഖുര്‍ആനിക വിവരണങ്ങ ളുടെ സ്രോതസ്സെന്തായിരുന്നുവെന്ന്, മറ്റ് പ്രവാചകന്മാരുടെ ചരിത്രകഥകളുടെ കാര്യത്തില്‍ അവര്‍ സ്വീകരിച്ച അതേ മാനദണ്ഡമുപയോഗിച്ച്, വ്യക്തമാക്കുവാന്‍ ബാധ്യസ്ഥരാണ്. പക്ഷെ, അവര്‍ക്ക് അതിന് സാധ്യമല്ല. യഥാര്‍ത്ഥത്തില്‍ യേശുവിനെയും മാതാവിനെയും കുറിച്ച ഖുര്‍ആനിലുള്ള അറിവിന്റെ സ്രോതസ്സെന്തായിരുന്നുവെന്ന് ഖുര്‍ആന്‍തന്നെ സൂചിപ്പിക്കുന്നുണ്ട്. അതല്ലാതെ മറ്റൊ രു വിശദീകരണവും ഇക്കാര്യത്തില്‍ മനുഷ്യബുദ്ധിയെ സംതൃപ്തമാക്കുന്നതായി നിലവിലില്ല. ''(നബിയേ) നാം നിനക്ക് ബോധനം നല്‍കുന്ന അദൃശ്യവാര്‍ത്തകളില്‍ പെട്ടതാകുന്നു അവയൊക്കെ. അവരില്‍ ആരാണ് മര്‍യത്തിന്റെ സംരക്ഷണം ഏറ്റെടുക്കേണ്ടതെന്ന് തീരുമാനിക്കുവാനായി അവര്‍ തങ്ങളുടെ അമ്പുകള്‍ ഇട്ടുകൊണ്ട് നറുക്കെടുപ്പ് നടത്തിയിരുന്ന സമയത്ത് നീ അവരുടെ അടുത്തുണ്ടായിരുന്നില്ലല്ലോ. അവര്‍ തര്‍ക്കത്തിലേര്‍പ്പെട്ടുകൊണ്ടിരുന്നപ്പോഴും നീ അവരുടെ അടുത്തുണ്ടായിരുന്നില്ല'' (വി.ഖു. 3:44)

മുഹമ്മദ് (സ) നബിക്ക് വഹ്‌യ് കിട്ടിയശേഷം അദ്ദേഹത്തെ പത്‌നി ഖദീജ (റ) തന്റെ ബന്ധുവായ വറഖത്തുബ്‌നു നൗഫലിന്റെ അടുക്കലേക്ക് കൊണ്ടുപോയതായി പറയുന്ന സഹീഹുല്‍ ബുഖാരിയിലെ രണ്ട് ഹദീസുകളുടെ വെളിച്ചത്തിലാണ് ഇസ്‌ലാം വിമർശകന്മാര്‍ വറഖയാവാം മുഹമ്മദി(സ)ന് ബൈബിളിലെ വിവരങ്ങള്‍ പറഞ്ഞുകൊടുത്തതെന്ന് സമര്ത്ഥിക്കുന്നത്. പ്രസ്തുത ഹദീസുകള്‍ കാണുക:

ആയിശ പറയുന്നു: ''നബി തിരുമേനി(സ)ക്ക് തുടക്കത്തിൽ ലഭിച്ച ദൈവിക സന്ദേശങ്ങളുടെ ആരംഭം ഉറക്കത്തില്‍ ദൃശ്യമാകുന്ന നല്ല സ്വപ്‌നങ്ങളായിരുന്നു. അവിടുന്ന് കാണുന്ന എല്ലാ സ്വപ്‌നങ്ങളും പ്രഭാതോദയം പോലെ സ്പഷ്ടമായി പുലർന്നു കൊണ്ടേയിരുന്നു. പിന്നീട് തിരുമേനിക്ക് ഏകാന്ത വാസം പ്രിയങ്കരമായിത്തോന്നി. അങ്ങനെ ഏതാനും രാത്രികള്‍ ഹിറാഗുഹയില്‍ ഏകാന്തവാസം അനുഷ്ഠിച്ചു. ആ രാത്രികള്ക്കുള്ള ആഹാരപദാര്ത്ഥ ങ്ങളുമായി ഗുഹയിലേക്ക് പോകും. കുറെ രാത്രി ആരാധനയില്‍ മുഴുകി അവിടെ കഴിച്ചുകൂട്ടും. പിന്നെ ഖദീജാ (റ)യുടെ അടുക്കലേക്ക് തിരിച്ചുവരും. വീണ്ടും ആഹാരപദാര്ത്ഥ്ങ്ങള്‍ തയ്യാറാക്കി പുറപ്പെടും. ഹിറാ ഗുഹയില്വെിച്ച് തിരുമേനിക്ക് സത്യം വന്നുകിട്ടുന്നതുവരെ ഈ നില തുടർന്നു പോന്നു. അങ്ങനെ മലക്ക് തിരുമേനി (സ)യുടെ മുമ്പില്‍ പ്രത്യക്ഷപ്പെട്ടു. ''വായിക്കുക'' എന്ന് പറഞ്ഞു. നബി (സ) പ്രതിവചിച്ചു. എനിക്ക് വളരെ വിഷമം അനുഭവപ്പെട്ടു. അനന്തരം എന്നെ വിട്ട് വീണ്ടും ''വായിക്കുക'' എന്ന് കല്പിപച്ചു. വായിക്കാന്‍ അറിയില്ലെന്ന് ഞാന്‍ അപ്പോഴും മറുപടി നല്കില. മലക്ക് എന്നെ പിടിച്ച് ശക്തിയായി ആശ്ലേഷിച്ചു. എനിക്ക് വളരെ വിഷമം തോന്നി. പിന്നീട് എന്നെ വിട്ടശേഷം ''വായിക്കുക'' എന്ന് പറഞ്ഞു. എനിക്ക് വായന അറിയില്ലായെന്ന് പിന്നെയും ഞാന്‍ പറഞ്ഞ പ്പോള്‍ മൂന്നാമതും മലക്ക് എന്നെ പിടിച്ച് ശക്തിയോടെ ആശ്ലേഷിച്ചു. അനന്തരം എന്നെ വിട്ടിട്ട് പറഞ്ഞു: ''സ്രഷ്ടാവായ നിന്റെ രക്ഷിതാവിന്റെ നാമത്തില്‍ വായിക്കുക. മനുഷ്യനെ അവന്‍ ഭ്രൂണത്തില്നികന്ന് സൃഷ്ടിച്ചിരിക്കുന്നു. നീ വായിക്കുക, നിന്റെ രക്ഷിതാവ് അത്യുദാരനത്രെ'' ഉടനെ പിടക്കുന്ന ഹൃദയത്തോടെ ഈ സന്ദേശവുമായി തിരുമേനി (സ) മടങ്ങി. ഖുവൈലിദിന്റെ മകള്‍ ഖദീജയുടെ അടുക്കല്‍ കയറിച്ചെന്ന് പുതച്ചുതരിക, പുതച്ചുതരിക എന്ന് അവിടുന്ന് അഭ്യര്ത്ഥിച്ചു. അവര്‍ പുതച്ചുകൊടുത്തു. ആ ഭയം നിശ്ശേഷം നീങ്ങിയപ്പോള്‍ നടന്ന സംഭവങ്ങളെല്ലാം ഖദീജാബീവിയെ ധരിപ്പിച്ചു. തന്റെ ജീവന് എന്തെങ്കിലും ആപത്ത് സംഭവിക്കുമോ എന്ന് ഭയപ്പെടുന്നതായി അദ്ദേഹം അവരോട് പറഞ്ഞു. അപ്പോള്‍ ഖദീജ പറഞ്ഞു: ഇല്ല, അല്ലാഹുവാണെ സത്യം. അവന്‍ അങ്ങയെ ഒരിക്കലും അപമാനിക്കുകയില്ല. താങ്കള്‍ കുടുംബബന്ധം പുലര്ത്തുന്നു. പരാശ്രയരുടെ ഭാരം ചുമക്കുന്നു. അഗതികള്ക്ക് സ്വയം അധ്വാനിച്ച് സഹായം ചെയ്തുകൊടുക്കുന്നു. അതിഥികളെ സല്ക്കടരിക്കുന്നു. വിപല്ഘ നട്ടങ്ങളില്‍ ശരിയായ സഹായം നല്കുകന്നു. പിന്നീട് തിരുമേനി(സ)യെയും കൂട്ടി ഖദീജ (റ) തന്റെ പിതൃവ്യപുത്രനായ വറഖത്ത്ബ്‌നു നൗഫലിബ്‌നി അസദിബ്‌നി അബ്ദില്‍ ഉസ്സയുടെ അടുക്കലേക്ക് ചെന്നു. വറഖത്ത് അജ്ഞാനകാലത്ത് ക്രിസ്ത്യാനിയായവനും ഹിബ്രു ഭാഷയില്‍ എഴുതാന്‍ പഠിച്ചവനുമായിരുന്നു. തന്നിമിത്തം അദ്ദേഹം സുവിശേഷത്തില്നിതന്ന് ചില ഭാഗങ്ങള്‍ ഹിബ്രുവില്‍ എഴുതിയെടുക്കാറുണ്ടായിരുന്നു. അദ്ദേഹം വയോവൃദ്ധനായി കണ്കാഴ്ച തന്നെ നഷ്ടപ്പെട്ടു കഴിഞ്ഞിരുന്നു. ഖദീജ (റ) പറഞ്ഞു: ''പിതൃവ്യപുത്രാ താങ്കളുടെ സഹോദര പുത്രന്റെ വിശേഷങ്ങള്‍ ഒന്ന് ശ്രദ്ധിക്കുക''. വറഖത്ത് ചോദിച്ചു: ''എന്റെ സഹോദര പുത്രാ നീ എന്താണ് ദര്ശിച്ചത്?'' കണ്ട കാഴ്ചകളെല്ലാം തിരുമേനി (സ) വറഖത്തിനെ അറിയിച്ചു. വറഖത്ത് പറഞ്ഞു: ഇത് അല്ലാഹു മൂസാ(അ)യുടെ അടുക്കലേക്ക് അയച്ചിരുന്ന അതേ നന്മ യുടെ രഹസ്യ സന്ദേശവാഹകനാണ്. താങ്കള്‍ മതപ്രബോധനം ചെയ്യുന്ന സന്ദര്ഭദത്തി ല്‍ ഞാനൊരു യുവാവായിരുന്നെങ്കില്‍! താങ്കളെ സ്വദേശത്ത് നിന്ന് സ്വജനത ബഹിഷ്‌കരിക്കുന്ന ഘട്ടത്തില്‍ ഞാനൊരു യുവാവായിരുന്നുവെങ്കില്‍!!'' തിരുമേനി (സ) ചോദിച്ചു. അവർ എന്നെ ബഹിഷ്‌കരിക്കുകയോ? വറഖത്ത് പറഞ്ഞു. താങ്കള്‍ കൊണ്ടുവന്നതുപോലെയുള്ള സന്ദേശങ്ങളുമായി വന്ന ഒരു മനുഷ്യനും തന്റെ ജനതയുടെ ശത്രുതയ്ക്ക് പാത്രമാകാതിരുന്നിട്ടില്ല. താങ്കളുടെ പ്രവര്ത്തളനങ്ങള്‍ നടക്കുന്നദിവസം ഞാന്‍ ജീവിച്ചിരിപ്പുണ്ടെങ്കില്‍ സുശ ക്തമായ ഒരു സഹായം താങ്കള്ക്ക്് നല്കുുമായിരുന്നു. പക്ഷെ, പിന്നീട് അധികം കഴിഞ്ഞില്ല. വറഖത്ത് മരണമടഞ്ഞു. ദൈ വിക സന്ദേശങ്ങളുടെ അവതരണം നിലയ്ക്കുകയും ചെയ്തു'' (സഹീഹുല്‍ ബുഖാരി).

ഈ ഹദീസുകള്‍ സത്യസന്ധവും മുന്ധാണരണയില്ലാത്തതുമായ വായനയ്ക്ക് വിധേയമാക്കിയാല്ത നന്നെ വറഖത്തു ബ്‌നു നൗഫലില്നിതന്നാണ് പ്രവാചകന്‍ (സ) ചരിത്രകഥകള്‍ മനസ്സിലാക്കിയത് എന്ന വാദം അടിസ്ഥാനരഹിതമാ ണെന്ന് മനസ്സിലാവും. താഴെ പറയുന്ന വസ്തുതകള്‍ ശ്രദ്ധിക്കുക:

(1) മുഹമ്മദ് നബി (സ)ക്ക് പ്രവാചകത്വം ലഭിക്കുമ്പോള്‍ വറ ഖത്തുബ്‌നു നൗഫല്‍ വാര്ധരക്യംമൂലം കാഴ്ച നഷ്ടപ്പെട്ട വ്യക്്തിയായിരുന്നു. ഇതുകഴിഞ്ഞ് അല്പ‍കാലത്തിനകം അദ്ദേഹം മരണപ്പെട്ടിരിക്കണം. പൂർവ്വ പ്രവാചക ന്മാരെക്കുറിച്ച ഖുർആനിക പരാമര്ശങ്ങള്‍ അവതരിക്കുന്ന കാലത്ത് അദ്ദേഹം ജീവിച്ചിരുന്നുവെന്ന് കരുതാന്‍ വയ്യ. അദ്ദേഹം പറഞ്ഞുകൊടുത്ത് എഴുതിയതാകാം ഖുർആനിലെ പ്രവാചക കഥനങ്ങളെന്ന് കരുതുന്നത് അതുകൊണ്ടുതന്നെ യുക്തിസഹമല്ല.

(2) പൂർവ്വ പ്രവാചകന്മാരില്‍ ചിലരുടെ കഥകള്‍ അടങ്ങിയ ഖുർആൻ സൂക്തങ്ങള്‍ അവതരി പ്പിക്കപ്പെട്ടത് പ്രവാചകനും (സ) അനുചരന്മാരും തമ്മില്‍ സംഭാഷണം നടത്തുമ്പോഴുള്ള പ്രശ്‌ന ങ്ങള്ക്ക്ക പരിഹാരം നിര്ദ്ദേശിക്കുന്നതിനും വേദക്കാരായ യഹൂദ ക്രൈസ്തവരുമായി സംവദി ക്കുമ്പോള്‍ അവരുടെ ചോദ്യങ്ങള്ക്ക്ക ഉത്തരം നല്‍കുന്നതിനു മായിരുന്നു. ഈ സമയത്തൊന്നും വറഖത്തുബ്‌നു നൗഫല്‍ പ്രവാചകനോടൊപ്പമുണ്ടായിരുന്നില്ലല്ലോ. പിന്നെയെങ്ങനെയാണ് പ്രവാചകന്‍ (സ) പൂർവ്വ പ്രവാചകന്മാരുടെ ചരിത്രത്തില്നികന്ന് സൂക്ഷ്മവും കൃത്യവുമായി കാര്യങ്ങളുദ്ധരിക്കുക?

(3) മുഹമ്മദ് നബി (സ)ക്ക് വറഖത്തുബ്‌നു നൗഫല്‍ അദ്ദേഹത്തിന്റെ പ്രവാചകത്വ ലബ്ധിക്കുമുമ്പ് എന്തെങ്കിലും കാര്യങ്ങള്‍ പഠിപ്പിച്ചുകൊടുത്തിരുന്നെങ്കില്‍ ആ സമൂഹത്തിലെ ചിലര്ക്കെങ്കിലും അക്കാര്യം അറിയാമായിരുന്നി രിക്കണം. പ്രവാചകന്റെ അനുചരന്മാരിലോ ശത്രുക്കളിലോ പെട്ട സമകാലികരായ ആരുംതന്നെ വറഖത്ത് ബ്‌നു നൗഫല്‍ പഠിപ്പിച്ചുകൊടുത്ത കാര്യങ്ങളാണ് മുഹ മ്മദ് നബി (സ) ഖുർആനില്‍ ഉള്ക്കൊചള്ളിക്കുന്നത് എന്ന ആരോ പണമുന്നയിച്ചിരുന്നില്ല.

4) ജൂത-ക്രൈസ്തവ വേദങ്ങളില്‍ പാണ്ഡിത്യമുണ്ടായിരുന്ന വറഖത്തുബ്‌നു നൗഫല്‍ മുഹമ്മദ് നബിക്ക് പൂർവ്വ പ്രവാചകന്മാരുടെ കഥകള്‍ പഠിപ്പിച്ചുകൊടുത്തിരുന്നുവെങ്കില്‍ യഹൂദരും ക്രൈസ്ത വരും വികലമാക്കിയ പ്രവാചക കഥനങ്ങളായിരിക്കണം അദ്ദേഹം പറഞ്ഞുകൊടുത്തിരിക്കുക. പ്രസ്തുത വിശദീകരണങ്ങളില്‍ ബൈബിളില്‍ ഇന്ന് കാണപ്പെടുന്ന രീതിയിലുള്ള അശാസ്ത്രീയവും ചരിത്രവിരുദ്ധവും വൈരുദ്ധ്യങ്ങളാല്‍ നിബിഡവുമായ കഥാകഥനങ്ങളു മുണ്ടാവും. അദ്ദേഹം പറഞ്ഞുകൊടുത്തതിന്റെ അടിസ്ഥാനത്തില്‍ എഴുതിയതായിരുന്നു ഖുർആനെങ്കില്‍ അതിലും ഇത്തരം അബദ്ധങ്ങളുണ്ടാവേണ്ടതായിരുന്നു. എന്നാല്‍ ഖുർആനില്‍ ഇത്തരം അബദ്ധങ്ങളൊന്നുമില്ല.

5) പൂർവ്വ വേദങ്ങളില്‍ പണ്ഡിതനായിരുന്ന വറഖത്തുബ്‌നു നൗഫല്‍ മുഹമ്മദ് നബി (സ) ക്കുണ്ടായ ആദ്യ ദിവ്യ ബോധനത്തിന്റെ അനുഭവങ്ങള്‍ കേട്ടപ്പോള്‍ ഇത് ദൈവിക ബോധനത്തിന്റെ ആരംഭമാണെന്നും ''താങ്കളെ ജനം കയ്യൊഴിയുമ്പോള്‍ ഞാന്‍ ജീവിച്ചിരിക്കുന്നുവെങ്കില്‍ ഞാന്‍ താങ്കളെ ശക്തമായി പിന്തുണക്കു''മെന്നും പറഞ്ഞതായി നടേ ഉദ്ധരിച്ച ഹദീസുകള്‍ വ്യക്തമാക്കുന്നത്. തന്നില്‍ നിന്ന് കേട്ടുപഠിച്ച പ്രവാചകകഥകളുടെ അടിസ്ഥാനത്തില്‍ താനും പ്രവാചകനാണെന്ന് വരുത്തിത്തീർക്കാന്‍ മുഹമ്മദ് (സ) ശ്രമിക്കുകയാണെന്ന ഒരു ചെറിയ ശങ്കപോലും വറഖത്തുബ്‌നു നൗഫലിനു ണ്ടായില്ല. മുമ്പൊരിക്കലും മുഹമ്മദി(സ)ന് പൂര്വ്വലപ്രവാചകന്മാരുടെ കഥകള്‍ വറഖത്തുബ്‌നു നൗഫല്‍ പഠിപ്പിച്ചുകൊടുത്തിട്ടില്ലെന്ന സത്യം ഇതിലൂടെ സുതരാം വ്യക്തമാവുന്നുണ്ട്.

പ്രതിഫലനാളിൽ അവിശ്വാസികൾക്ക് ഗ്രന്ഥം ലഭിക്കുക പിന്നിലൂടെയാണെന്ന് 84:10 ലും ഇടത് കൈയ്യിലാണെന്ന് 69:25ലും പറയുന്നു. ഇത് വൈരുധ്യമല്ലേ? ഇവിടെ സൂചിപ്പിക്കപ്പെട്ടിരിക്കുന്ന ഖുര്‍ആന്‍ സൂക്തങ്ങളുടെ സാരം കാണുക: എന്നാല്‍ ഏതൊരുവന്ന്, തന്റെ രേഖ അവന്റെ മുതുകിന്റെ പിന്നിലൂടെ കൊടുക്കപ്പെട്ടുവോ അവന്‍ നാശമേ എന്ന് നിലവിളിക്കുകയും, ആളി കത്തുന്ന നരകാഗ്നിയില്‍ കിടന്ന് എരിയുകയും ചെയ്യും.’’ (84:10) എന്നാല്‍ ഇടത് കൈയ്യില്‍ ഗ്രന്ഥം നല്‍കപ്പെട്ടവനാകട്ടെ ഇപ്രകാരം പറയുന്നതാണ്. ഹാ! എനിക്ക് എന്റെ ഗ്രന്ഥം നല്‍കപ്പെടാതിരുന്നെങ്കില്‍. (69:25) പരലോകത്ത് വെച്ച് അവിശ്വാസികള്‍ക്ക് അവരുടെ കര്‍മങ്ങള്‍ രേഖപ്പെടുത്തിയ ഗ്രന്ഥം നല്‍കപ്പെടുക അവരുടെ പിന്‍ഭാഗത്തു കൂടി ഇടത് കൈയ്യിലായിരിക്കുമെന്ന് മുഹമ്മദ് നബി(ﷺ) വ്യക്തമാക്കിയതായി ഹദീസുകളില്‍ വന്നിട്ടുണ്ട്. വൈരുധ്യം ആരോപിക്കപ്പെട്ടിരിക്കുന്ന സൂക്തങ്ങളില്‍ ഇക്കാര്യമാണ് വ്യക്തമാക്കിയിരിക്കുന്നത്. സൂറത്തുല്‍ –ഇന്‍ശിഖാഖി–ലെ പത്താം വചനത്തില്‍ (84:10) അവിശ്വാസിക്ക് അവന്റെ മുതുകിന് പിന്നിലൂടെയാണ് രേഖ ലഭിക്കുന്നത് എന്നും സൂറത്തുല്‍ ഹാഖയിലെ 25ാം വചനത്തില്‍(69:25) അവന്റെ ഇടത് കൈയിലാണ് അത് കിട്ടുകയെന്നും പറയുമ്പോള്‍ ഒരു സൂക്തം മറ്റൊരു സൂക്തത്തെ വിശദീകരിക്കുകയാണ് ചെയ്യുന്നത്.

കനായ സ്രഷ്ടാവ് നിയോഗിച്ചയച്ച പ്രവാചകന്മാരെക്കുറിച്ച് ബൈബിളിലും ഖുര്‍ആനിലും വന്ന സമാനമായ ചരിത്രപരാമര്‍ശങ്ങളുടെ വെളിച്ചത്തില്‍ ബൈബിളില്‍നിന്ന് പകര്‍ത്തിയെഴുതിയതാണ് ഖുര്‍ആന്‍ എന്ന വാദം മിഷനറിമാരും ഓറിയന്റലിസ്റ്റുകളും ഭൗതികവാദികളുമെല്ലാമായ വിമര്‍ ശകര്‍ ഒരേസ്വരത്തില്‍ ഉന്നയിക്കാറുണ്ട്. ഈ വാദത്തില്‍ എത്രത്തോളം കഴമ്പുണ്ട്? താഴെ പറയുന്ന വസ്തുതകളുടെ വെളിച്ചത്തില്‍ ചിന്തിക്കുമ്പോള്‍ ഈ വാദം ശുദ്ധ അസംബ ന്ധമാണെന്ന് ബോധ്യ മാകും.

ഒന്ന്) മുഹമ്മദ് നബി (സ) നിരക്ഷരനായിരുന്നു. ബൈബിള്‍ പഴയനിയമവും പുതിയനിയമവും വായിച്ചു മനസ്സിലാക്കി അതില്‍നിന്ന് പകര്‍ത്തിയെഴുതുക അദ്ദേഹത്തിന് സ്വന്തമായി അസാധ്യ മായിരുന്നു. ശിഷ്യന്മാരില്‍ ആരുടെയെങ്കിലും സഹായത്തോടെ അദ്ദേഹം അത് നിര്‍വഹിച്ചുവെന്ന് കരുതാനും വയ്യ. അങ്ങനെ ചെയ്തിരുന്നുവെങ്കില്‍ ശിഷ്യന്മാരില്‍ ചിലര്‍ക്കെങ്കിലും അത് അറിയാന്‍ കഴിയേണ്ടതായിരുന്നു. അത് മുഖേന മുഹമ്മദ് നബി (സ)യുടെ വിശ്വാസ്യതയില്‍ അവര്‍ സംശയി ക്കുകയും അവര്‍ തമ്മിലുള്ള ബന്ധത്തിന് ഉലച്ചില്‍തട്ടുകയും ചെയ്യുമായിരുന്നു. മുഹമ്മദ് നബി (സ)യുടെ ശരീരത്തില്‍ ഒരു പോറലെങ്കിലുമേല്‍ക്കുന്നതിന് പകരം സ്വന്തം ജീവന്‍ ബലിയര്‍പ്പിക്കു വാന്‍ സന്നദ്ധരായവരായിരുന്നു പ്രവാചക ശിഷ്യന്മാര്‍ എന്നോര്‍ക്കുക. പ്രവാചകനില്‍ (സ) ഏതെ ങ്കിലുംതരത്തിലുള്ള അവിശ്വാസ്യതയുണ്ടായിരുന്നുവെങ്കില്‍ ഇങ്ങനെ ത്യാഗം ചെയ്യാന്‍ സന്ന ദ്ധരായ ഒരു അനുയായിവൃന്ദത്തെ വളര്‍ത്തിയെടുക്കുവാന്‍ അദ്ദേഹത്തിന് കഴിയുമായിരുന്നില്ലെന്ന് തീര്‍ച്ച യാണ്.

''ഇതിന് മുമ്പ് നീ വല്ല ഗ്രന്ഥവും പാരായണം ചെയ്യുകയോ, നിന്റെ വലതുകൈകൊണ്ട് അത് എഴുതു കയോ ചെയ്തിരുന്നില്ല. അങ്ങനെയാണെങ്കില്‍ ഈ സത്യനിഷേധികള്‍ക്ക് സംശയിക്കാമായിരുന്നു'' (വി.ഖു. 29:48).

രണ്ട്) മുഹമ്മദ് നബി (സ) യുടെ ജീവിതകാലത്ത് ബൈബിള്‍ പഴയനിയമമോ പുതിയനിയമമോ അറ ബിയിലേക്ക് പരിഭാഷപ്പെടുത്തപ്പെട്ടിട്ടുണ്ടായിരുന്നില്ല. അറബിയിലുള്ള പഴയനിയമവും പുതിയ നിയമവുമെല്ലാം ഉണ്ടായതുതന്നെ ഇസ്‌ലാമിന്റെ ദിഗ്‌വിജയങ്ങള്‍ക്ക് ശേഷമാണ്. പഴയ നിയമ രേഖകളെക്കുറിച്ച് സൂക്ഷ്മ പഠനം നടത്തിയ ഏണസ്റ്റ് വൂര്‍ഥ്‌വിന്‍ എഴുതുന്നത് കാണുക: ''ഇസ്‌ലാ മിന്റെ വ്യാപനത്തോടുകൂടി അറബിയുടെ ഉപയോഗം വ്യാപകമാവുകയും ഇസ്‌ലാമിക രാജ്യങ്ങ ളിലെ ജൂതന്മാരുടെയും ക്രിസ്ത്യാനികളുടെയും ദൈനംദിനജീവിതത്തിലെ ഭാഷയായി അറബി മാറു കയും ചെയ്തു. ബൈബിളിന്റെ അറബി പതിപ്പുകള്‍ അനിവാര്യമാക്കി ത്തീര്‍ത്ത ഈ സാഹചര്യ ത്തില്‍ സ്വതന്ത്രവും പ്രാഥമികമായ വ്യാഖ്യാന സംബന്ധിയുമായ നിരവധി പതിപ്പുകള്‍ പുറത്തു വന്നു''.(Ernst Wurthewein: The Text of The Old Testament Page 104).

ഒമ്പതാം നൂറ്റാണ്ടിന്റെ ആദ്യപകുതിയിലാണ് പഴയ നിയമബൈ ബിള്‍ അറബിയിലേക്ക് പരിഭാഷപ്പെടുത്തപ്പെട്ടതെന്നാണ് ലഭ്യമായ കയ്യെഴുത്ത് രേഖകള്‍ വ്യക്തമാക്കുന്നത് (Ibid Page 224-225).

ഏക ദേശം ഇക്കാലത്തുതന്നെയാവണം പുതിയ നിയ മവും അറബിയിലേക്ക് ഭാഷാന്തരം ചെയ്യപ്പെട്ടത്. പ്രഗത്ഭനായ സിഡ്‌നി എച്ച്. ഗ്രിഫിത്തിന്റെ വരികള്‍ കാണുക:''അറബിയിലുള്ള സുവിശേഷങ്ങളടങ്ങിയ ഏറ്റവും പുരാതനമായ കയ്യെഴുത്ത് രേഖ 'സിനായ് അറ ബി കയ്യെഴുത്ത് പ്രതി 72' (Sinai Arabic MS72) ആണ്. ജറുസലേം സഭയുടെ ഗ്രീക്ക് പ്രാര്‍ത്ഥനാ കലണ്ടറി ന്റെ കാലക്രമാടിസ്ഥാനത്തില്‍ അധ്യായങ്ങള്‍ രേഖപ്പെടുത്തി യ നാല് കാനോനിക സുവിശേഷങ്ങ ളും ഇതിലുണ്ട്. രേഖയുടെ അന്ത്യത്തിലെ കുറിപ്പ് വ്യക്തമാക്കുന്നത് ഈ കയ്യെഴുത്ത് രേഖ അറബി കലണ്ടര്‍ 284ല്‍ അഥവാ ക്രിസ്താബ്ദം 897ല്‍ റംലയിലെ സ്റ്റീഫന്‍ (Stephen of Ramlah) എഴുതിയതാണെന്ന് (Sidney H Griffith: The Gospel in Arabic: An Enquiry Into its Appearance In the First Abbasi Century Page 132) എന്നാല്‍ അപ്പോസ്തല പ്രവൃത്തികളും പൗലോസിന്റെ ലേഖനങ്ങളും കാതോലിക ലേഖനങ്ങ ളുമുള്‍ക്കൊള്ളുന്ന Sinai Arabic MS151 എന്ന കയ്യെഴുത്ത് രേഖ ഹിജ്‌റ253 ല്‍ അഥവാ ക്രിസ്താബ്ദം 867ല്‍ സുറിയാനിയില്‍നിന്ന് അറബിയിലേക്ക് ബിസ്ര്‍ബ്‌നുസിര്‍റി എന്നയാള്‍ വിവര്‍ത്തനം ചെയ്തതായി കാണുന്നുണ്ട്. ഇതില്‍ സുവിശേഷങ്ങളില്ലെന്ന കാര്യം പ്രത്യേകം ശ്രദ്ധേയമാണ്. (Ibid Page 131).

മുഹമ്മദ് നബി(ല)ക്ക് ശേഷം രണ്ട് നൂറ്റാണ്ടുകളെങ്കിലും കഴിഞ്ഞാണ് പുതിയനിയമവും പഴയനി യമവുമെല്ലാം അറബിയിലേക്ക് വിവര്‍ത്തനം ചെയ്യപ്പെട്ടത്. നിരക്ഷരനായിരുന്ന മുഹമ്മദ് നബി (സ) മറ്റാരില്‍നിന്നെങ്കിലും അറബിയിലുള്ള ബൈബിള്‍ വായിച്ചുകേട്ടശേഷം അതിലെ കഥകള്‍ ഉള്‍ക്കൊ ള്ളിച്ചുകൊണ്ട് എഴുതിയതാണ് ഖുര്‍ആന്‍ എന്ന വാദവും ഇവിടെ അപ്രസക്തമാവുകയാണ്. അറ ബിയില്‍ നിലവിലില്ലാത്ത ഒരു ഗ്രന്ഥം വായിച്ചുകേട്ടുവെന്ന് കരുതുന്നത് നിരര്‍ത്ഥകമാണെന്ന് പറ യേണ്ടതില്ലല്ലോ.

മൂന്ന്) പ്രവാചകന്മാരുടെ ചരിത്രം വിവരിക്കുന്നിടത്ത്  അധാര്‍മ്മികരും അസാന്മാര്‍ഗികരുമായി രുന്നു അവരെന്ന് വരുത്തിത്തീര്‍ക്കുന്ന തരത്തിലാണ് ബൈബിള്‍ അത് നിര്‍വ്വഹിച്ചിരിക്കുന്നത്. മദ്യ പിച്ച് നഗ്‌നനായ നോഹും ലഹരിമൂത്ത് സ്വപുത്രിമാരുമായി ശയിച്ച ലോത്തും ചതിയനായ യാ ക്കോബും വിഷയലമ്പടനായ ദാവീദും മദ്യം വിളമ്പിയ യേശുവുമെല്ലാം, ധര്‍മ്മത്തിലേക്ക് ജനങ്ങളെ നയിക്കാനായി നിയോഗിക്കപ്പെട്ടവരായിരുന്നു പ്രവാചകന്മാര്‍ എന്ന സങ്കല്‍പത്തിന് കടക വിരുദ്ധ മായ കഥകളാണെന്ന് പറയേണ്ടതില്ലല്ലോ. ഖുര്‍ആനിലെ ചരിത്രവിവരണത്തില്‍ ഇത്ത രം യാതൊരു കഥകളും കാണുന്നില്ല. ബൈബിളില്‍നിന്ന് മുഹമ്മദ് നബി (സ) പകര്‍ത്തിയെഴു തിയതായിരുന്നു ഈ കഥകളെങ്കില്‍  പ്രവാചകന്മാരില്‍ ബൈബിള്‍ ആരോപിച്ച അധാര്‍മ്മികതകളി ലേതെങ്കിലും ഖുര്‍ആ നിലും സ്ഥാനം പിടിക്കേണ്ടതായിരുന്നു. അങ്ങനെയില്ലെന്ന് മാത്രമല്ല, പ്രവാ ചകന്മാരെല്ലാം ഉന്നതരും വിശുദ്ധരുമായിരുന്നുവെന്ന വസ്തുത വ്യക്തമാക്കുന്നതാണ് ഖുര്‍ആനിലെ പ്രവാചക കഥനങ്ങളെ ല്ലാമെന്ന കാര്യം അത് ബൈബിളില്‍നിന്ന് പകര്‍ത്തിയെഴുതിയതാണെന്ന വാദത്തിന്റെ നട്ടെല്ലൊടി ക്കുന്നുണ്ട്.

നാല്) ചരിത്രത്തിന്റെ അളവുകോലുകള്‍ വെച്ചുനോക്കുമ്പോള്‍ വസ്തുനിഷ്ഠചരിത്രത്തിന് നിരക്കാ ത്ത നിരവധി പ്രസ്താവനകള്‍ ബൈബിള്‍ നടത്തുന്നുണ്ട്. ഇത് ബൈബിള്‍ പണ്ഡിതന്മാര്‍ തന്നെ അംഗീ കരിക്കുന്നതാണ്. ''ചരിത്രപരമായി കൃത്യമല്ലാത്ത ചില പ്രസ്താവനകളും ബൈബിളില്‍ കണ്ടെന്നു വരാം'' (ബൈബിള്‍ വിജ്ഞാനകോശം പുറം 12). ബൈബിളില്‍നിന്ന് പകര്‍ത്തിയെഴുതിക്കൊണ്ട് മുഹമ്മദ് നബി (സ)രചിച്ചതായിരുന്നു ഖുര്‍ആനെങ്കില്‍ അതില്‍ ബൈബിളിലേതുപോലെ ചരിത്രപ രമായി കൃത്യമല്ലാത്ത പ്രസ്താവനകള്‍ കാണപ്പെടേണ്ടതായിരുന്നു. എന്നാല്‍, അത്തരം യാതൊരു പ്രസ്താവനയും ഖുര്‍ആനിലില്ല.

അഞ്ച്) ആധുനിക ശാസ്ത്രത്തിന്റെ കാഴ്ചപ്പാടിലൂടെ നോക്കുമ്പോള്‍ ബൈബിളില്‍ നിരവധി അശാ സ്ത്രീയമായ പരാമര്‍ശങ്ങള്‍ കാണാനാവും. സൂര്യന്റെ സൃഷ്ടിക്ക് മുമ്പുതന്നെ രാപ്പകലുകളുണ്ടായ തായി വിവരിക്കുന്ന ഉല്‍പത്തി പുസ്തകം മുതലാരംഭിക്കുന്നു ബൈബിളിലെ ശാസ്ത്രവിരുദ്ധമായ പരാമര്‍ശങ്ങള്‍. രാപ്പകലുകളുണ്ടാവുന്നത് സൂര്യചന്ദ്രന്മാരുടെ ചലനം മൂലമാണെന്നും (യേശു 10:12,13), ഭൂമി ഇളകാതെ നിശ്ചലമായി നില്‍ക്കുകയാണെന്നും (സങ്കീ 104:5) മുയല്‍ അയവിറക്കുന്ന ജീവിയാണെന്നു (ആവ 14:7) മെല്ലാമുള്ള ബൈബിള്‍ പരാമര്‍ശങ്ങള്‍ അതിന്റെ അശാസ്ത്രീയതക്ക് ഉദാഹരണങ്ങളാണ്. ഈ പരാമര്‍ശങ്ങളെല്ലാം വരുന്നത് പ്രവാചകകഥനങ്ങള്‍ക്കിടയിലാണെന്ന കാര്യം പ്രത്യേകം ശ്രദ്ധേയമാണ്. ബൈബിളായിരുന്നു ഖുര്‍ആനിന്റെ രചനയ്ക്കുപയോഗിച്ചിരുന്ന സ്രോതസ്സെങ്കില്‍ ഈ അശാസ്ത്രീയമായ പരാമര്‍ശങ്ങളെല്ലാം ഖുര്‍ആനിലും സ്ഥാനം പിടിക്കുമായി രുന്നു. ഈ പരാമര്‍ശങ്ങള്‍ വസ്തുതകള്‍ക്ക് നിരക്കാത്തതാണെന്ന അറിവ് മുഹമ്മദ് നബി (സ)യുടെ കാലത്തുണ്ടായിരുന്നില്ലെന്നോര്‍ക്കുക. എന്നാല്‍ ഖുര്‍ആനില്‍ ഇത്തരം യാതൊരുവിധ പരാമര്‍ശങ്ങ ളുമില്ല. ഖുര്‍ആനിലെ ഒരൊറ്റ വചനമെങ്കിലും ഏതെങ്കിലും ശാസ്ത്രവസ്തുതകളുമായി വൈരുദ്ധ്യം പുലര്‍ത്തുന്നതായി തെളിയിക്കപ്പെട്ടിട്ടില്ല. ബൈബിളില്‍നിന്ന് പകര്‍ത്തിക്കൊണ്ട് മുഹമ്മദ് നബി (സ) രചിച്ച ഗ്രന്ഥമാണ് ഖുര്‍ആനെന്ന് വാദിക്കുകയാണെങ്കില്‍ തനിക്ക് ശേഷം നൂറ്റാണ്ടുകള്‍ കഴിഞ്ഞ് വരാനിരിക്കുന്ന ശാസ്ത്രമുന്നേറ്റങ്ങള്‍ കൂടി മുന്‍കൂട്ടി കാണാന്‍ കഴിയുകയും അതിന്റെ അടിസ്ഥാന ത്തില്‍ ബൈബിളിലുള്ള അശാസ്ത്രീയതകള്‍ അറിഞ്ഞ് അവയെല്ലാം അരിച്ചൊഴിവാക്കി സംശുദ്ധ മായ ചരിത്രം മാത്രം എടുത്തുദ്ധരിക്കുകയും ചെയ്ത അതിമാനുഷനാണ് അദ്ദേഹമെന്ന് പറയേണ്ടി വരും. സര്‍വ്വശക്തനായ സ്രഷ്ടാവിന്റെ വചനങ്ങളാണ് ഖുര്‍ആനിലുള്ളതെന്ന വസ്തുത നിഷേധിക്കു വാന്‍ തെളിവ് പരതുന്നവര്‍ മുഹമ്മദ് നബി (സ)യെ ദൈവമാക്കുന്ന പരിണാമഗുപ്തിയിലാണ് എത്തിച്ചേരുകയെന്നര്‍ത്ഥം.

ആറ്) ബൈബിളില്‍ പറയാത്ത ചില പ്രവാചകന്മാരുടെയും സമുദായങ്ങളുടെയും ചരിത്രം ഖുര്‍ ആന്‍ വിവരിക്കുന്നുണ്ട്. ആദ്, സമൂദ് ഗോത്രങ്ങളിലേക്ക് നിയോഗിക്കപ്പെട്ട ഹൂദ് നബിയുടെയും സാലിഹ് നബിയുടെയും ചരിത്രം ഉദാഹരണം. ബൈബിളിലെവിടെയും കാണാനാവാത്ത പ്രവാച കന്മാരാണിവര്‍. ബൈബിളില്‍നിന്ന് കോപ്പിയടിക്കുകയാണ് മുഹമ്മദ് നബി (സ)ചെയ്തതെങ്കില്‍ ഈ ചരിത്ര ങ്ങള്‍ അദ്ദേഹത്തിന് എവിടെനിന്നാണ് കിട്ടിയത്?

ഏഴ്) ബൈബിളില്‍ പരാമര്‍ശിക്കപ്പെട്ട പ്രവാചകന്മാരുടെ ചരിത്രം പറയുമ്പോള്‍തന്നെ ബൈബിളി ലൊരിടത്തും പരാമര്‍ശിക്കാത്ത നിരവധി സംഭവങ്ങള്‍ ഖുര്‍ആനില്‍ വിശദീകരിക്കുന്നുണ്ട്. നൂഹ് നബി (അ) യും അവിശ്വാസിയായ മകനും തമ്മില്‍ നടന്ന സംഭാഷണവും മകന്‍ പ്രളയത്തില്‍പെട്ട സംഭവവിവരണവും സൂറത്തു ഹൂദില്‍ (11:42-46) കാണാം. ഇങ്ങനെ യാതൊന്നും ബൈബിളിലെ വിടെയുമില്ല. ഇബ്രാഹീം നബിയും നംറൂദും തമ്മില്‍ നടന്ന സംവാദവും (ഖുര്‍ആന്‍ 2:258) പിതാവു മായി നടന്ന സംഭാഷണവും (ഖുര്‍ആന്‍ 6:74, 19:41-49, 43:26,27) മരണാനന്തര ജീവിതത്തിന്റെ സത്യത ബോധ്യപ്പെടുന്നതിനായി, പക്ഷികളെ കഷ്ണിച്ച് നാല് മലകളില്‍വെച്ചശേഷം അവയെ വിളിച്ചാല്‍ അവ ഓടിവരുന്നതാണെന്ന് അല്ലാഹു അദ്ദേഹത്തോട് പറഞ്ഞ സംഭവവും (2:260) തീയിലേക്ക് വലി ച്ചെറിയപ്പെടുകയും അതില്‍ നിന്ന് അദ്ദേഹം അത്ഭുതകരമായി രക്ഷപ്പെടുകയും ചെയ്ത ചരിത്ര വു(21:56-70)മൊന്നും ബൈബിളിലൊരിടത്തും കാണാന്‍ കഴിയില്ല. ദൈവിക കല്‍പന പ്രകാരം ഒരു പശുവിനെ അറുക്കാന്‍ മൂസാ (അ) ഇസ്രായീല്യരോട് നിര്‍ദേശിക്കുകയും, പശുവിന്റെ പ്രത്യേക തകള്‍ ചോദിച്ച് അതിന്റെ നിര്‍വ്വഹണം അവര്‍ പ്രയാസകരമാക്കുകയും ചെയ്ത സംഭവവും (ഖുര്‍ആന്‍ 2:67-71) കൊലപാതകക്കുറ്റം തെളിയിക്കാനായി പശുവിനെ അറുത്ത് അതിന്റെ ഒരുഭാ ഗംകൊണ്ട് അടിക്കുവാന്‍ കല്‍പിച്ച കഥനങ്ങളും (2:72, 73) മൂസാ നബി ((അ))യുടെ ജീവിതവുമായി ബന്ധപ്പെടുത്തി ബൈബിളിലൊരിടത്തും പ്രസ്താവിക്കുന്നില്ല. ഈസാ നബി (അ) യുടെ ജനനം മുതല്‍ തന്നെയുള്ള ബൈബിളില്‍ പറയാത്ത പല സംഭവങ്ങളും ഖുര്‍ആനില്‍ പരാമര്‍ശിക്കുന്നുണ്ട്. സകരി യ്യായുടെ സംരക്ഷണത്തില്‍ പ്രാര്‍ത്ഥനാസ്ഥലത്ത് താമസിച്ചുകൊണ്ടിരുന്ന മര്‍യമിന്റെ കുട്ടിക്കാ ലത്ത് അവര്‍ക്ക് അത്ഭുതകരമായി ഭക്ഷണസാധനങ്ങള്‍ ലഭിച്ച സംഭവം (ഖുര്‍ആന്‍ 3:37), മര്‍യമിന്റെ പ്രസവസമയത്ത് അവര്‍ക്ക് നല്‍കപ്പെട്ട പ്രത്യേക അനുഗ്രഹങ്ങളെക്കുറിച്ച വിവരണം (19:23-26), ഈസാ (അ) തൊട്ടിലില്‍വെച്ച് സംസാരിച്ച് തന്റെ നിയോഗം പ്രഖ്യാപിച്ചുകൊണ്ട് തന്റെ ആദ്യത്തെ അത്ഭുതം പ്രവര്‍ത്തിച്ച ചരിത്രം (19:29,30), കളിമണ്ണുകൊണ്ട് പക്ഷിയുടെ രൂപമുണ്ടാക്കി ഈസാ (അ) അതില്‍ ഊതിയപ്പോള്‍ അല്ലാഹുവിന്റെ അനുമതിപ്രകാരം അതൊരു പക്ഷിയായി രൂപാന്തരപ്പെട്ട സംഭവം (3:49) ഇതൊന്നുംതന്നെ ബൈബിളില്‍ ഒരിടത്തും പരാമര്‍ശിക്കുന്നുപോലുമില്ല.

ബൈബിളി ല്‍നിന്ന് മുഹമ്മദ് നബി (സ) പകര്‍ത്തിയെഴുതിക്കൊണ്ടാണ് ഖുര്‍ആന്‍ രചിച്ചതെങ്കില്‍ ബൈബിളി ലൊരിടത്തും പരാമര്‍ശിക്കാത്ത പ്രവാചകന്മാരുടെ ജീവിതവുമായി ബന്ധപ്പെട്ട കഥകള്‍ അദ്ദേഹ ത്തിന് എവിടെനിന്നുകിട്ടി? സത്യത്തില്‍ ഖുര്‍ആന്‍ ദൈവ വചനമായതുകൊണ്ടാണ് ബൈബിളിലെ വിടെയും സൂചിപ്പിക്കാത്ത സംഭവങ്ങള്‍പോലും അതില്‍ നമുക്ക് കാണാന്‍ കഴിയുന്നത്. മര്‍യത്തി ന്റെ ബാല്യകാല സംഭവങ്ങള്‍ വിവരിക്കവെ ഖുര്‍ആന്‍ പറഞ്ഞത് എത്ര ശരി! ''(നബിയേ) നാം നിനക്ക് ബോധനം നല്‍കുന്ന അദൃശ്യവാര്‍ത്തകളില്‍ പെട്ടതാകുന്നു അവയൊക്കെ. അവരില്‍ ആരാണ് മര്‍യത്തിന്റെ സംരക്ഷണം ഏറ്റെടുക്കേണ്ടതെന്ന് തീരുമാനിക്കുവാനായി അവര്‍ തങ്ങളുടെ അമ്പുകള്‍ ഇട്ടുകൊണ്ട് നറു ക്കെടുപ്പ് നടത്തിയിരുന്ന സമയത്ത് നീ അവരുടെ അടുത്തുണ്ടായിരുന്നി ല്ലല്ലോ. അവര്‍ തര്‍ക്കത്തിലേര്‍പ്പെട്ടുകൊണ്ടിരുന്നപ്പോഴും നീ അവരുടെ അടുത്തുണ്ടായിരുന്നില്ല'' (വി.ഖു. 3:44).

എട്ട്) ബൈബിളില്‍ പരാമര്‍ശിക്കപ്പെട്ട കഥകള്‍ പറയുമ്പോഴും ബൈബിളില്‍നിന്ന് വ്യത്യസ്തമായി കൃത്യതയും സൂക്ഷ്മതയും ഖുര്‍ആന്‍ കാത്തുസൂക്ഷിക്കുന്നത് കാണാം. ഉദാഹരണത്തിന് മോശ-സീനായ് പര്‍വതത്തിലേക്ക് പോയ അവസരത്തില്‍ ഇസ്രായീല്യര്‍ക്ക് അവരുടെ ആവശ്യപ്രകാരം സ്വര്‍ണംകൊണ്ട് കാളക്കുട്ടിയെ നിര്‍മിച്ച് ആരാധനക്കായി നല്‍കിയത് മോശയുടെ കൂട്ടാളിയും പ്രവാ ചകനുമായ അഹരോണായിരുന്നുവെന്നാണ് പുറപ്പാട് പുസ്തകം (32:1-6) പറയുന്നത്. ഖുര്‍ആനും ബൈബിളുമെല്ലാം പരിശുദ്ധ പ്രവാചകനായി പരിചയപ്പെടുത്തുന്ന ഹാറൂനി((അ))ല്‍ നിന്ന് വിഗ്ര ഹാരാധനക്ക് കൂട്ടുനില്‍ക്കുകയെന്ന മഹാപാപം സംഭവിക്കാനിടയില്ലെന്ന് ഏത് സാമാന്യ ബുദ്ധി ക്കും മനസ്സിലാവും. ഖുര്‍ആനും പ്രസ്തുത സംഭവം വിവരിക്കുന്നുണ്ട്. പക്ഷെ, സ്വര്‍ണപശുവിനെ യുണ്ടാക്കുകയും അതിനെ ആരാധിക്കുവാന്‍ ഇസ്രായീല്യരെ പ്രേരിപ്പിക്കുകയും ചെയ്തത് ഹാറൂ ന(അ)ല്ല; പ്രത്യുത ഇസ്രായീല്യരില്‍പെട്ട ഒരു കപടനായ സാമിരിയാണ് ഇത് ചെയ്തതെന്നും അതു മൂലം അയാള്‍ ദൈവകോപത്തിനും ശപിക്കപ്പെട്ട രോഗത്തിനും വിധേയനായെന്നുമാണ് ഖുര്‍ആന്‍ പഠിപ്പിക്കുന്നത് (20:85-95). ബൈബിളില്‍ പരാമര്‍ശിക്കപ്പെട്ട കഥകള്‍ വിവരിക്കുമ്പോഴും അതിലെ നെല്ലും പതിരും വേര്‍ തിരിച്ച് സത്യസന്ധവും സൂക്ഷ്മവുമായ രീതിയില്‍ അവ ജനസമക്ഷം വെക്കു ന്ന ഖുര്‍ആന്‍ ദൈവികമാണെന്ന് അതിന്റെ ഈ പ്രത്യേകത തന്നെ സുതരാം വ്യക്തമാക്കുന്നു.

ബ്രഹാമിന്റെ ചരിത്രം പറയുന്നിടത്ത് ബൈബിളിലും ഖുർആനിലും ഏകദേശം സമാനമായ കഥകളാണുള്ളതെങ്കിലും ഈ രണ്ട് ഗ്രന്തങ്ങളിലെയും പ്രതിപാദനരീതികൾ തമ്മിൽ സാരമായ ചില അന്തരങ്ങളുണ്ട്. ബൈബിളിലെ ചരിത്ര വിശദീകരണത്തിലുടനീളം ഇസ്രായേലീ വംശീയത നിഴലിക്കുന്നതായി കാണാന്‍ കഴിയും. .അബ്രഹാമിന്റെ ചരിത്രകഥനത്തിൽ ഇത് വ്യക്തമായി കാണാനാവും. അദ്ദേഹത്തിന്റെ പുത്രന്മാരായ യിശ്മായേലിനെയും ഇസഹാഖി നെയും കുറിച്ചു വിവരിക്കുന്ന ഉല്‍പത്തി പുസ്തകഭാഗങ്ങളില്‍ യഹൂദ വംശീയതയുടെയും അടിമ കളോടുള്ള അവരുടെ ക്രൂരമായ പെരുമാറ്റത്തിന്റെയും പ്രതിബിംബമാണ് കാണാന്‍ കഴിയുന്നത്.

അബ്രഹാമിന് തന്റെ ആദ്യഭാര്യയായ സാറായില്‍ സന്താന സൗഭാഗ്യമില്ലായിരുന്നതിനാല്‍ സാറാ തന്നെ തന്റെ അടിമയായിരുന്ന ഹാഗാറിനെ അദ്ദേഹത്തിന് ഭാര്യയായി നല്കുികയാണുണ്ടായത്. 'ഭാര്യയായ സാറായ് തന്റെ ഈജിപ്തുകാ രിയായ ദാസി ഹാഗാറിനെ ഭര്ത്താാവായ അബ്രാമിന്ന് ഒരു ഭാര്യയായി കൊടുത്തു. അബ്രാം ഹാഗാറിനെ പ്രാപിച്ചു. അവള്‍ ഗര്ഭിരണിയായി' (ഉല്പതത്തി16:3). അബ്രഹാമില്‍ നിന്ന് ഗര്ഭിണിയായ ഹാഗാര്‍ സാറായോട് അപമര്യാദയായി പെരുമാറിയപ്പോള്‍ ഹാഗാറിനെ ക്രൂരമായി മര്ദിാക്കുവാനും ഗര്ഭി്ണിയായ അവളെ വീട്ടില്‍ നിന്നും ഓടിക്കുവാനും സാറായെ അബ്രഹാം അനുവദിച്ചുവെന്നാണ് ഉല്പ ത്തി പുസ്തകത്തില്‍ നാം വായിക്കുന്നത്. 'അബ്രാം സാറായിയോട് പറഞ്ഞു:' നോക്കൂ, നിന്റെ ദാസി നിന്റെ കീഴില്‍ തന്നെ യാണ്. നിനക്ക് ഇഷ്ടമുള്ള വിധം നീ അവളോട് പെരുമാറിക്കൊള്ളുക! തുടര്ന്നു സാറായ് ഹാഗാറി നോട് ക്രൂരമായി പെരുമാറി. ഹാഗാര്‍ അവിടെ നിന്ന് ഓടിപ്പോയി' (ഉല്പരത്തി 16:6). തന്നില്‍ നിന്ന് ഗര്ഭിാണി യായ സ്ത്രീയെ അവര്‍ അടിമയായി എന്ന കാരണത്താല്‍ അടിച്ചോടി ക്കുവാന്‍ കൂട്ടുനി ല്ക്കുന്ന അബ്രഹാമിന്റെ ചിത്രം നിര്മിറക്കുക വഴി അടിമസ്ത്രീകളോട് ഏതു തരം ക്രൂരതയും പ്രവര്ത്തിാക്കുവാനുള്ള നിയമം നിര്മിനക്കുവാനാണ് യഹൂദ റബ്ബിമാര്‍ ശ്രമിച്ചിരിക്കുന്നത്.

തങ്ങളുടെ ക്രൂരമായ ജീവിതരീതിക്കനുസൃതമായി മഹാന്മാരുടെ ജീവിതത്തെ മാറ്റിയെഴുതിയ പ്പോഴുണ്ടായ വൈരുദ്ധ്യങ്ങള്‍ ചരിത്ര മെഴുത്തിലുടനീളം കാണന്‍ കഴിയും. വാര്ദ്ധ്ക്യകാലത്തു ണ്ടായ തന്റെ പുത്രന്‍ യിശ്മായേലിനെയും ഭാര്യ ഹാഗാറിനെയും മരുഭൂമിയില്‍ തനിച്ചാക്കുവാ നുള്ള ദൈവിക കല്പഭന നിറവേറ്റിയ മഹാനാണ് വിശുദ്ധ ഖുര്ആചന്‍ പരിചയപ്പെടുത്തുന്ന ഇബ്രാഹീം (അ). ഉല്പാത്തി പുസ്തകമാകട്ടെ ഈ സംഭവത്തെയും ഹാഗാര്‍ എന്ന അടിമസ്ത്രീ യെയും പുത്രനെയും പീഡിപ്പിക്കാനുള്ള സാറായുടെ ശ്രമത്തിന് അബ്രഹാം കൂട്ടുനിന്നതിന് ഉദാ ഹരണമായിട്ടാണ് ഉദ്ധരിക്കുന്നത്. ഇരുപത്തൊന്നാം അധ്യായം നോക്കുക: 'ശിശു വളര്ന്നു . ഇസഹാ ഖിന്റെ മുലകുടി മാറ്റിയ ദിവസം അബ്രഹാം വലിയൊരു വിരുന്നു നല്കി്. ഈജിപ്റ്റുകാരിയായ ഹാഗാറില്‍ അബ്രഹാമിന് ജനിച്ച പുത്രന്‍ തന്റെ പുത്രനായ ഇസഹാഖിനോടൊന്നിച്ചു കളിക്കുന്നത് സാറാ കണ്ടു. സാറാ അബ്രഹാമിനോടുപറഞ്ഞു: 'ഈ അടിമപ്പെണ്ണിനെയും പുത്രനെയും അടിച്ചു പുറത്താക്കുക. ഈ അടിമപ്പെണ്ണിന്റെ പുത്രന്‍ എന്റെ പുത്രനായ ഇസഹാഖിന്നൊപ്പം അവകാശി യായി ത്തീര്ന്നുണ കൂടാ'. യിശ്മായേലും തന്റെ പുത്രനാകയാല്‍ ഇക്കാര്യം അബ്രഹാമിന് ഏറെ അനിഷ്ടമായി. എന്നാല്‍ ദൈവം അബ്രഹാമി നോട് അരുള്‍ ചെയ്തു. 'അടിമപ്പെണ്ണിനേയും കുട്ടി യേയും പ്രതി നീ അനിഷ്ടം വിചാരിക്കേണ്ട. സാറാ പറയുന്നതു പോലെ പ്രവര്‍ത്തിക്കുക. ഇസ ഹാഖിലൂടെയായിരിക്കും നിന്റെ സന്തതിപരമ്പര അറിയപ്പെടുക. അടിമപ്പെണ്ണിന്റെ പുത്രനെയും ഞാന്‍ ഒരു ജനത യാക്കും. അയാളും നിന്റെ സന്തതിയാണല്ലോ'. അബ്രഹാം അതി രാവിലെ എഴു ന്നേറ്റ് അപ്പവും ഒരു തുരുത്തിവെള്ളവും ഹാഗാറിനെ ഏല്പിെച്ചു. കുട്ടിയെ തോളില്വെനച്ച് അവളെ പറഞ്ഞയച്ചു.അവര്‍ അവിടം വിട്ടു, ബേര്ശേ്ബ മരുഭൂമിയില്‍ അലഞ്ഞു നടന്നു' (ഉല്പ.ത്തി 21:8-14).

യഥാര്ത്ഥ ത്തില്‍ യിശ്മായേലിനെ മരുഭൂമിയില്‍ ഉപേക്ഷിച്ച സംഭവം നടക്കുന്നത് ഇസ്ഹാഖിന്റെ ജനനത്തിനു മുമ്പാണ്. ഇസ്ഹാഖിനുള്ള ഔന്നത്യത്തിന്റെയും അടിമസ്ത്രീയോടും മകനോടും ചെയ്യാവുന്ന ക്രൂരതകളുടെയും തെളിവായുദ്ധരിക്കുന്ന തിന്നു വേണ്ടിയാണ് പ്രസ്തുത സംഭവത്തെ ഇസ്ഹാഖിന്റെ ജനനത്തിന് ശേഷത്തേക്ക് വലിച്ചിഴച്ചത്. യിശ്മയേല്‍ വളരെ ചെറിയ ഒരു കുഞ്ഞായിരുന്ന കാലത്താണ് ഈ സംഭവം നടന്നതെന്ന് മുകളി ലുദ്ധരിച്ച ഉല്പനത്തി വചനത്തില്‍ നിന്നു വ്യക്തമാണ്. 'കുട്ടിയെ തോളില്‍ വെച്ച് അവളെ പറഞ്ഞയച്ചു' (21:14) വെന്ന് പറഞ്ഞതില്‍ നിന്ന് തോളത്ത് വെക്കാന്‍ മാത്രമേ യിശ്മായിലിന് അന്ന് പ്രായമു ണ്ടായിരുന്നുള്ളൂവെന്ന് മനസ്സിലാ കുന്നു.'തുരുത്തിയിലെ വെള്ളം തീര്ന്നലപ്പോള്‍ അവള്‍ കുട്ടിയെ ഒരു കുറ്റിക്കാട്ടില്‍ ഉപേക്ഷിച്ചു. അവള്‍ അവിടെ നിന്ന് കുറച്ചകലെ, അതായത് ഏകദേശം ഒരു വില്പാളടു ദൂരെ മാറി പുറം തിരിഞ്ഞിരുന്ന്. 'എനിക്ക് കുഞ്ഞിന്റെ മരണം കാണേണ്ട' എന്നു പറഞ്ഞു. അവള്‍ ദൂരെ മാറി പിന്തി രിഞ്ഞിരുന്നപ്പോള്‍ കുട്ടി ഉറക്കെ കരഞ്ഞു' (ഉല്പ‍ത്തി 21:15,16). ഈ വചനങ്ങളെല്ലാം വളരെ ചെറിയ ഒരു ശിശുവായിരിക്കുമ്പോഴാണ് യിശ്മായേല്‍ തന്റെ മാതാവിനോടൊപ്പം മരുഭൂമിയില്‍ ഉപേക്ഷി ക്കപ്പെട്ടതെന്ന് വ്യക്തമാക്കുന്നു.

'ഹാഗാര്‍ യിശ്മായിലിനെ പ്രസവിച്ചപ്പോള്‍ അബ്രഹാമിന് എണ്പെ ത്താറ് വയസ്സായിരുന്നു' (ഉല്പപത്തി 16:16). 'പുത്രനായ ഇസ്ഹാഖ് പിറന്നപ്പോള്‍ അബ്രഹാമിന്ന് നൂറ് വയസ്സായിരുന്നു' (ഉല്പപത്തി 21:5). ഇതില്‍ നിന്ന് ഇസ്ഹാഖ് ജനിക്കുമ്പോള്‍ യിശ്മായേലിന് പതിനാലു വയസ്സ് പ്രായമായിരു ന്നുവെന്ന് മനസ്സിലാക്കാം. ഇസ്ഹാഖിന്റെ മുലകുടി മാറിയ ദിവസമാണ് ഉല്പനത്തി പുസ്തകം പറയുന്നതുപോലെ യിശ്മായിലിനെയും മാതാവിനെയും മരുഭൂമിയില്‍ ഉപേക്ഷിച്ചതെങ്കില്‍ അന്ന് യിശ്മായീല്‍ പതിനാറു വയസ്സു പ്രായമുള്ളയാളായിരി ക്കണം. ഒരു പതിനാറു വയസ്സുകാരനെ മാതാവ് തോളില്‍ വെക്കുമോ? മരുഭൂമിയില്‍ ഉപേക്ഷിക്ക പ്പെട്ട യിശ്മായേലിന്റെ ചിത്രം ഉല്പ?ത്തിപുസ്തകം ഇരുപത്തി യൊന്നാം അധ്യായപ്രകാരം ഒരു പതിനാറുകാരന്‍േറതല്ലെന്നുറപ്പാണ്. ഈ വൈരുദ്ധ്യത്തിനുള്ള കാരണം യഹൂദ റബ്ബിമാരുടെ കൈക്രിയകളാണ്. സ്വന്തം ഇച്ഛയ്ക്കനുസരിച്ച് പ്രവാചകന്മാരുടെ ചരിത്രം മാറ്റിയെഴുതിയപ്പോള്‍ സ്വാഭാവികമായുണ്ടായ വൈരുദ്ധ്യങ്ങളെക്കുറിച്ച് റബ്ബിമാര്‍ അജ്ഞരായിരുന്നുവെന്നു വേണം മനസ്സിലാക്കുവാന്‍.

സ്വപുത്രനെ ബലിയറുക്കുവാനുള്ള ദൈവകല്പഞന നിറവേറ്റുവാന്‍ സന്നദ്ധനായ അബ്രഹാമിനെ ക്കുറിച്ച് വിവരിക്കുന്നിടത്തും ഈ വൈരുദ്ധ്യം പ്രകടമാവുന്നുണ്ട്. അവ ഇസ്രായേല്യരില്‍ മാത്രമേ ദൈവാനുഗ്രഹമുണ്ടായിട്ടുള്ളുവെന്ന് സ്ഥാപിക്കുന്നതിന് വേണ്ടി ബലികര്മ്മഹത്തെ മാറ്റിയെഴുതി യതുകൊണ്ടുണ്ടായതാണ്. വാര്ധ്ികക്യത്തില്‍ ഇബ്രാഹീമിന് ആദ്യമുണ്ടായ പുത്രനെ ബലിയറു ക്കുവാന്‍ കല്പി്ച്ചു കൊണ്ട് അദ്ദേഹത്തെ അല്ലാഹു പരീക്ഷിച്ചുവെന്നും ത്യാഗങ്ങളുടെ തീച്ചൂളയി ലൂടെ ജീവിച്ചു വളര്ന്ന് മഹാനായ ഇബ്രാഹീം പ്രസ്തുത പരീക്ഷണത്തില്‍ വിജയിച്ചുവെന്നുമാണ് ഖുര്ആഹന്‍ പഠിപ്പിക്കുന്നത്. ബലിയറുക്കുവാന്‍ ദൈവം കല്പിാച്ചത് അബ്രഹാമി ന്റെ ആദ്യത്തെ പുത്രനെത്തന്നെയായിരുന്നുവെന്നാണ് ഉല്പിത്തി പുസ്തകത്തില്‍ നിന്ന് മനസ്സിലാവുന്നത്. സ്വന്തം പുത്രനെ അറുക്കുവാന്‍ വേണ്ടി കൈകാലുകള്‍ കെട്ടി ബലിപീഠത്തിന് മുകളില്‍ കിട ത്തിക്കൊണ്ട് കത്തി എടുത്ത സമയത്ത് ആകാശത്തു നിന്ന് കര്ത്താ്വിന്റെ മാലാഖ പറഞ്ഞതായി ഉല്പ്ത്തി പുസ്തകം ഉദ്ധരിക്കുന്നത് നോക്കുക: 'കുട്ടിയുടെ മേല്‍ കൈവെക്കരുത്. അവനെ ഒന്നും ചെയ്യരുത്. നിന്റെ പുത്രനെ, നിന്റെ ഏകജാതനെ തന്നെ തരുവാന്‍ നീ വൈമനസ്യം കാണിക്കായ്കയാല്‍, നീ ദൈവത്തെ ഭയപ്പെടുന്നു എന്ന് ഞാന്‍ ഇപ്പോള്‍ അറിയുന്നു (ഉല്പനത്തി 22:12). 'നിന്റെ ഏകജാതനെ' (Your only son) എന്നാണ് മാലാഖ പറയുന്നത് എന്ന കാര്യം ശ്രദ്ധിക്കുക. ബലിയറുക്കുവാനായി കല്പിളക്കപ്പെട്ട കാലത്ത് അബ്രഹാമിന് ഒരൊറ്റപുത്രന്‍ മാത്രമേയുണ്ടായിരുന്നുള്ളുവെന്നാണല്ലോ ഇതില്‍ നിന്ന് മനസ്സിലാക്കേണ്ടത്. ആദ്യപുത്രനായ യിശ്മായേലിനെയല്ലാതെ ദ്വിതീയനായ ഇസ്ഹാ ഖിനെ ബലിയറുക്കാനാണ് കല്പുനയുണ്ടായതെങ്കില്‍ 'നിന്റെ ഏകജാതനെ' എന്നു മാലാഖ പറയു മായിരുന്നുവോ?

യഥാര്ത്ഥ ത്തില്‍ ബലിയറുക്കുവാനുള്ള ദൈവകല്പപനയില്‍ നിന്നു തന്നെ ഇക്കാര്യം വ്യക്തമായും മനസ്സിലാകുന്നുണ്ട്. ' ദൈവം കല്പിിച്ചു: 'നിന്റെ പുത്രനെ, നീ അത്യധികം സ്‌നേഹിക്കുന്ന ഏകജാ തനായ ഇസ്ഹാഖിനെ, കൂട്ടിക്കൊണ്ട് മോറിയാ ദേശത്തേക്കു പോകുക. അവിടെ ഞാന്‍ കല്പിക്കു ന്ന മലയില്‍ അവനെ എനിക്കു ഹോമിക്കുക' (ഉല്പ ത്തി 22:2).ഇവിടെ, ഏകജാതനായ ഇസ്ഹാഖി' നെ എന്നാണ് ദൈവകല്പ്നയിലുള്ളത്. ഇസഹാഖ് എങ്ങനെയാണ് ഏകജാതനാകുന്നത്? അദ്ദേഹം അബ്രഹാമിന്റെ ദ്വിതീയ പുത്രനാണല്ലോ. ഇവിടെ, ഈ കല്പേനയില്‍ 'ഇസ്ഹാഖിനെ'യെന്ന് യഹൂദ റബ്ബിമാര്‍ കൂട്ടിച്ചേര്ത്തിതാണെന്നാണ് മനസ്സിലാക്കാന്‍ കഴിയുന്നത്. അടിമസ്ത്രീയില്‍ ജനിച്ച മക്കള്‍ സ്വന്തം പുത്രന്മാരായി അറിയപ്പെടുന്നത് അപമാനമായി കരുതപ്പെട്ടിരുന്ന യഹൂദ പാരമ്പര്യ ത്തിന് അനു സൃതമായ രീതിയില്‍ അബ്രാഹാമിന്റെ ചരിത്രം വളച്ചൊടിക്കപ്പെട്ട പ്പോഴാണ് 'ഇസ്ഹാഖ്' ഏകജാതനായി മാറിയത്.

അടിമ സ്ത്രീയിലുണ്ടായ പുത്രനെ അബ്രഹാം മകനായി ത്തന്നെ പരിഗണിച്ചിരുന്നില്ല എന്നാണല്ലോ ഇതില്‍ നിന്ന് മനസ്സിലാക്കേണ്ടത്.എന്നാല്‍ ഉല്പത്തി പുസ്തകം തന്നെ നല്കുുന്ന അബ്രഹാമിന്റെ ചരിത്രവുമായി ഇത് വ്യക്തമായ വൈരുദ്ധ്യം പ്രകടി പ്പിക്കുന്നു. അബ്രഹാം യിശ്മായിലിനെ പുത്രനായിത്തന്നെയാണ് പരിഗണിച്ചിരു ന്നത് എന്നാണ് ഉല്പമത്തി പുസ്തകത്തില്‍ നിന്ന് മനസ്സിലാകുന്നത്. ജനനം മുതല്‍ (16:15,16) പരിഛേ ദനയിലും മറ്റു കര്മ്മണങ്ങളിലും (17:23) പ്രാര്ത്ഥ്നയിലും (17:20) അങ്ങനെ സകലവിധ കാര്യങ്ങ ളിലും യിശ്മാ യിലിനെ സ്വപുത്രനായിത്തന്നെയാണ് അബ്രഹാം പരിഗണിച്ചിരുന്നത് എന്നു തന്നെ യാണ് ഉല്പ ത്തിപുസ്തകത്തിലുള്ളത്. മാത്രവു മല്ല, ഇഷ്ടഭാര്യയില്‍ ജനിച്ച രണ്ടാമത്തെപുത്രന് അനിഷ്ടഭാര്യ യില്‍ ജനിച്ച ആദ്യപുത്രന് നല്കേചണ്ട അവകാശം നല്കുളന്നത് പഴയനി യമപ്രകാരം പാപമാണ്. ആവര്ത്തണന പുസ്തകത്തിലെ ദൈവകല്‍ പന നോക്കുക: 'ഒരാള്ക്ക് രണ്ടു ഭാര്യമാര്‍ ഉണ്ട് എന്നും അവരില്‍ ഒരുവളെ അയാള്‍ സ്‌നേഹിക്കുകയും മറ്റവളെ വെറുക്കുകയും ചെയ്യുന്നു വെന്നും കരു തുക. സ്‌നേഹിക്കുകയും വെറുക്കുകയും ചെയ്യുന്ന ഭാര്യമാരില്‍ അയാള്ക്ക് സന്താന ങ്ങള്‍ ജനിക്കു കയും ആദ്യജാതന്‍ അയാള്‍ വെറുക്കുന്ന ഭാര്യയില്‍ നിന്നു ജനിക്കയും ചെയ്താല്‍, തന്റെ സ്വത്തു ക്കള്‍ സന്താനങ്ങള്ക്ക്് അവകാശമായി ഭാഗിച്ചു കൊടുക്കുമ്പോള്‍, വെറുക്കു ന്നവളില്‍ നിന്ന് ജനിച്ച വനും ആദ്യജാതനുമായവനു പകരം സ്‌നേഹിക്കുന്നവളിലുണ്ടായ പുത്രനെ ആദ്യജാതനായി അയാ ള്‍ പരിഗണിക്കരുത്. അയാള്‍ തനിക്കുള്ള എല്ലാ സ്വത്തില്‍ നിന്നും ഇരട്ടി ഓഹരി നല്കിയ വെറുക്കു ന്ന ഭാര്യയുടെ പുത്രനെ ആദ്യജാതനായി അംഗീകരിക്കണം. അയാളുടെ വീര്യത്തിന്റ ആദ്യഫലം ആ പുത്രനാണല്ലോ. ആദ്യജാതനുള്ള അവകാശം ആ പുത്രനുതന്നെ' (ആവര്ത്തണനം 21:15-17).

ഹാഗാര്‍ അടിമസ്ത്രീയായിരുന്നതിനാല്‍ അബ്രാഹാമിനാല്‍ വെറു ക്കപ്പെട്ടവളായിരുന്നുവെന്ന യഹൂദ റബ്ബി മാരുടെ വാദം അംഗീകരിച്ചാ ല്തോന്നെ, മുകളില്‍ വിവരിച്ച ദൈവകല്പാനപ്രകാരം ആദ്യജാതനു ള്ള അവകാശത്തിന് അര്ഹ,ന്‍ യിശ്മായേല്‍ തന്നെയാണെന്ന് വ്യക്തമാണ്. ഇസ്ഹാഖിനെ അബ്ര ഹാമിന്റെ ഏകജാതനായി പരിചയപ്പെടുത്തുക വഴി ഉല്പഅത്തി 22:2 പ്രകാരം ദൈവംതന്നെ യിശ്മായേല്‍ അബ്രഹാമിന്റെ പുത്രനല്ലെന്ന് അംഗീകരിച്ചുവെന്ന് പറയേണ്ടിവരും.

ഈ വൈരുദ്ധ്യത്തിനുള്ള കാരണം തങ്ങളുടെ വംശീയ ദുരഭിമാ നത്തിന് മാറ്റുകൂട്ടുവാന്‍ വേണ്ടി പ്രവാചകചരിത്രത്തില്‍ യഹൂദ റബ്ബിമാര്‍ നടത്തിയ കൈക്രിയകളാണ്. അബ്രഹാമിന്റെ ദ്വിതീയ പുത്രനായ ഇസ്ഹാഖിന്റെ മകനായ യാക്കോബിന്റെ പുത്രന്മാരാണ് ഇസ്രായേല്യര്‍. ദൈവികമായ സകല അനുഗ്രഹങ്ങളും വര്ഷിുക്കപ്പെട്ടിരിക്കുന്നതും വര്ഷിമക്കപ്പെടാന്പോ്വുന്നതും ഇസ്രായീ ല്യര്ക്കി ടയില്‍ മാത്രമാണെന്നായിരുന്നു യഹൂദ റബ്ബിമാര്‍ പ്രചരിപ്പിച്ചിരുന്നത്. സ്വപുത്രനെ ബലിയ റുക്കുവാനുള്ള കല്പയനയനുസരിച്ചുകൊണ്ട് ദൈവാനുഗ്രഹത്തിന് പാത്രമായ അബ്രഹാമിന് ദൈ വം നല്കു്ന്ന വരദാനങ്ങളെക്കുറിച്ച് ഉല്പരത്തി പുസ്തകത്തിലുണ്ട്. അതിങ്ങനെയാണ് 'നീ ഇതു ചെയ്തിരിക്കയാല്‍, നിന്റെ പുത്രനെ, നിന്റെ ഏകജാതനെ തരാന്‍ നീ മടിക്കായ്കയാല്‍ എന്നാണെ, ഞാന്‍ ഇങ്ങനെ പ്രതിജ്ഞ ചെയ്യുന്നു- ഇതു കര്ത്തായവാണ് അരുള്‍ ചെയ്യുന്നത്: ഞാന്‍ നിന്നെ സമൃ ദ്ധമായി അനുഗ്രഹിക്കും. നിന്റെ സന്തതിയെ ഞാന്‍ ആകാശത്തിലെ നക്ഷത്രങ്ങളെപ്പോലെയും കട ല്ക്ക്രയിലെ മണല്ത്ത്രി കളെപ്പോലെയും അത്യധികം വര്ദ്ധിിപ്പിക്കും. നിന്റെ സന്തതികള്‍ ശത്രുക്ക ളുടെ പട്ടണവാതിലുകള്‍ കൈവശപ്പെടുത്തും. നീ എന്റെ വാക്ക് അനുസരിച്ചതിനാല്‍ നിന്റെ സന്ത തികളിലൂടെ ഭൂമിയിലെ എല്ലാ ജനതകളും അനുഗ്രഹിക്കപ്പെടും'(ഉല്പതത്തി 22:16-19). യിശ്മാ യേലാണ് ബലിയറുക്കപ്പെടാനായി കല്പിെക്കപ്പെട്ടതെങ്കില്‍ ഈ അനുഗ്രഹങ്ങള്‍ മുഴുവനുമുണ്ടാ വുക യിശ്മായേല്‍ സന്തതികളിലാണെന്നു വരും. യഹൂദന്മാരാകട്ടെ യിശ്മായീലിന്റെ അനുജസ ഹോദരനായ ഇസ്ഹാഖിന്റെ സന്തതി പരമ്പരകളിലാണ് ഉള്പ്പെജടുന്നത്. തങ്ങളിലല്ലാതെ ദൈവാ നുഗ്രഹമു ണ്ടാവുകയെന്നത് ഇസ്രാഈല്യര്ക്ക്പ ഒരു കാരണവശാലും അംഗീകരിക്കാന്‍ പറ്റാത്ത കാര്യമായിരുന്നു. അതുകൊണ്ടു കൂടിയാണ് യഹൂദ റബ്ബിമാര്‍ ഇസ്ഹാഖിനെ അബ്രഹാമിന്റെ ഏകജാതനാക്കി മാറ്റിയത്.

തങ്ങളുടെ വംശീയ ദുരഭിമാനത്തിന്റെ സംരക്ഷണത്തിനു വേണ്ടി എന്തും ചെയ്യാന്‍ മടിയില്ലാത്ത വരും അടിമസ്ത്രീയിലുണ്ടായ സ്വപുത്രന് പുത്രപദവി നല്കുാവാന്‍ വിസമ്മതിച്ചിരുന്നവരുമായി രുന്നു ഇസ്രാഈല്യര്‍ എന്നാണ് അബ്രഹാമിന്റെ ചരിത്രത്തില്‍ യഹൂദറബ്ബിമാര്‍ നടത്തിയ കൈക്രിയ കള്‍ കാണിക്കുന്നത്. വംശീയ ദുരഭിമാനത്തിന്റെ കാര്യത്തില്‍ തങ്ങളുടെ പൂര്വ്വി കന്മാരുടെ സ്വഭാ വം പൂര്ണ്ണെമായിത്തന്നെ ഉള്ക്കൊങള്ളുന്ന അഭിനവ ഇസ്രാഈല്യരുടെ സ്ഥിതിയും ഇതില്‍ നിന്ന് വലിയ വ്യത്യാസമുള്ളതല്ലല്ലോ.

ബൈബിളിൽ നിന്ന് തികച്ചും വ്യതിരിക്തമായ ഇബ്രാഹീമീവ്യക്തിത്വത്തെയാണ് ഖുർആനിൽ നാം കാണുന്നത്. ഇബ്‌റാഹീം നബിയെക്കുറിച്ച് ഖുർആൻ പറയുന്നത് അദ്ദേഹം സദ്‌വൃത്തനും ക്ഷമാശീലനും മാതൃകാപുരുഷനുമായിരുന്നുവെന്നാണ്. (11:75) ഏതെങ്കിലും രൂപത്തിലുള്ള സങ്കുചിതത്വങ്ങളില്ലാത്ത ആദര്ശധീരൻ! അല്ലാഹു തന്നെ 'സ്വന്തം ചങ്ങാതി'യെന്ന് വിലക്കാൻ മാത്ത്രം ദൈവസാമീപ്യത്തിന് അർഹനായ വ്യക്തിത്വം ..."സദ്‌വൃത്തനായിക്കൊണ്ട്‌ തന്‍റെ മുഖത്തെ അല്ലാഹുവിന്‌ കീഴ്പെടുത്തുകയും, നേര്‍മാര്‍ഗത്തിലുറച്ച്‌ നിന്ന്‌ കൊണ്ട്‌ ഇബ്രാഹീമിന്‍റെ മാര്‍ഗത്തെ പിന്തുടരുകയും ചെയ്തവനേക്കാള്‍ ഉത്തമ മതക്കാരന്‍ ആരുണ്ട്‌? അല്ലാഹു ഇബ്രാഹീമിനെ സുഹൃത്തായി സ്വീകരിച്ചിരിക്കുന്നു." (ക്വുർആൻ 4:175)

ക്വുർആനിൽ പ്രതിപാദിക്കപ്പെട്ടിരിക്കുന്ന ഇരുപത്തിയഞ്ച് പ്രവാചകന്മാരിൽ പലരെയും കുറിച്ച് ബൈബിളിൽ പരാമർശങ്ങളുണ്ടെന്നത് ശരിയാണ്.ക്വുർആനിലേതിനേക്കാൾ വിശദമായ ചരിത്ര കഥനവും ബൈബിളിലുണ്ട്.എന്നാൽ തങ്ങളുടെ താല്‍പര്യത്തിനനുസരിച്ച് പ്രവാചകന്മാരുടെ കഥനങ്ങ ള്‍ അവതരിപ്പിക്കുകയാണ് ബൈബിള്‍ പുസ്തകങ്ങളുടെ രചയിതാക്കള്‍ ചെയ്തത്. അതുകൊണ്ടു തന്നെ സമൂഹത്തില്‍ നിലനിന്നിരുന്ന സകല വൃത്തികേടുകളും പ്രവാചകന്മാരില്‍ ആരോപിക്കു വാന്‍ ബൈബിള്‍ കര്‍ത്താക്കള്‍ക്ക് യാതൊരു വൈമനസ്യവുമുണ്ടായിരുന്നില്ല. സമൂഹത്തിന്റെ തിന്മകളെ ന്യായീകരിക്കാനായി പ്രസ്തുത തിന്മകളെല്ലാം പ്രവാചകന്മാരില്‍ ആരോപിക്കുവാനുള്ള പ്രവണതയാണ് ബൈബിളില്‍ നാം കാണുന്നത്.

ധര്‍മനിഷ്ഠനും കുറ്റമറ്റ മനുഷ്യനുമായി ബൈബിള്‍ പരിചയപ്പെടുത്തുന്ന നോഹിനെ (ഉല്‍ 6:9, 10) തന്നെയാണ് ആദ്യമായി വീഞ്ഞുണ്ടാക്കിയവനായും കുടിച്ച് തുണിയുരിഞ്ഞുപോയിട്ട് മക്കള്‍ തുണിയുടുത്തുകൊടുക്കേണ്ട അവസ്ഥയോളമെത്തുന്ന തര ത്തില്‍ ലഹരി ബാധിച്ചവനായുമെല്ലാം ഉല്‍പത്തി പുസ്തകം (9:20-23) വരച്ചുകാണിക്കുന്നത്.

വിശു ദ്ധനും നീതിമാനുമെന്ന് ബെബിള്‍ പറയുന്ന (2 പത്രോസ് 2:78) ലോ ത്ത് മദ്യപിച്ച് മത്തനായി തന്റെ പെണ്‍മക്കളുമായി ശയിക്കുകയും അവര്‍ക്ക് സ്വന്തം പിതാവില്‍ മക്കളുണ്ടാവുകയും ചെയ്ത കഥ ഉല്‍പത്തി പുസ്തകം (19:31-36) വിവരിക്കുന്നുണ്ട്.

ചതിയനും വഞ്ചകനുമായിരുന്നു ഇസ്രായേലി ന്റെ പിതാവായ യാക്കോബെന്നാണ് ബൈബിള്‍ പറയുന്നത് (27:1-46). തന്റെ പടയാളിയുടെ ഭാര്യ യുമായി ബന്ധപ്പെടുകയും അവള്‍ ഗര്‍ഭിണിയായപ്പോള്‍ അതിന്റെ ഉത്തരവാദിത്തം അവളുടെ ഭര്‍ത്താവില്‍ കെട്ടിവെക്കാന്‍ ശ്രമിക്കുകയും പ്രസ്തുത ശ്രമം പരാജയപ്പെട്ടപ്പോള്‍ പടയാളിയെ ചതി ച്ചുകൊല്ലുകയും അങ്ങനെ അയാളുടെ ഭാര്യയെ സ്വന്തമാക്കുകയുമെല്ലാം ചെയ്ത വ്യക്തിയാണ് ബൈബിള്‍ പ്രകാരം ദാവീദ് (2 ശാമു 11:1-27)

.ദാ വീദിന്റെ പുത്രനും ലോകത്തിലെ ഏറ്റവും വലിയ ജ്ഞാനിയുമായി ബൈബിള്‍ പരിചയപ്പെടുത്തുന്ന സോളമന്‍ (1 രാജാ 10:23) വിവാഹബന്ധം വിലക്ക പ്പെട്ടവരുമായി ബന്ധപ്പെടുകയും അവരെ അഗാധമായി സ്‌നേഹിക്കുകയും (1 രാജാ 11:2) അവരുടെ സമ്മര്‍ദ്ദത്തിന് വഴങ്ങി അന്യദേവതമാരെ ആരാധിക്കുകയു (1 രാജാ 11:3-7)മെല്ലാം ചെയ്ത വ്യക്തി യാണ്.

ഇസ്രായേലിന്റെ രക്ഷകനായി വന്ന യേശുക്രിസ്തുവിനെയും മാതൃബഹുമാനമില്ലാത്തവ നായും (യോഹ 2:5, 19:26) സഹിഷ്ണുതയില്ലാത്തവനായും (മത്താ 12:34, 12:39, യോഹ 8:44) ക്ഷിപ്രകോപിയായും (യോഹ 2:13-17, മത്താ 21:19) ജനങ്ങള്‍ക്ക് മദ്യമുണ്ടാക്കിക്കൊടുത്ത് അവരെ ലഹരിപിടിപ്പിച്ചവനായും (യോഹ 2:1-11)മെല്ലാമാണ് ബൈബിള്‍ അവതരിപ്പിച്ചിരിക്കുന്നത്.

ഇതില്‍ നിന്ന് തികച്ചും വ്യതിരിക്തമാണ് ഖുര്‍ആനിലെ സംഭവവിവരണങ്ങള്‍. പ്രവാചകന്മാരെല്ലാം സദ്‌വൃ ത്തരും മാതൃകായോഗ്യരും വിശുദ്ധരുമായിരുന്നുവെന്നാണ് ഖുര്‍ആന്‍ പഠിപ്പിക്കുന്നത്. ''നാം വേദ വും വിജ്ഞാനവും പ്രവാചകത്വവും നല്‍കിയിട്ടുള്ളവരത്രെ അവര്‍. ഇനി ഇക്കൂട്ടര്‍ അവയൊക്കെ നിഷേധിക്കുകയാണെങ്കില്‍ അവയില്‍ അവിശ്വസിക്കുന്നവരല്ലാത്ത ഒരു ജനവിഭാഗത്തെ നാം അത് ഭരമേല്‍പിച്ചിട്ടുണ്ട്. അവരെയാണ് അല്ലാഹു നേര്‍വഴിയിലാക്കിയിട്ടുള്ളത്. അതിനാല്‍ അവരുടെ നേര്‍മാര്‍ഗത്തെ നീ പിന്തുടര്‍ന്ന് കൊള്ളുക. (നബിയേ,) പറയുക. ഇതിന്റെ പേരില്‍ യാതൊരു പ്രതി ഫലവും ഞാന്‍ നിങ്ങളോട് ആവശ്യപ്പെടുന്നില്ല. ഇത് ലോകര്‍ക്കുവേണ്ടിയുള്ള ഒരു ഉത്‌ബോധനമ ല്ലാതെ മറ്റൊന്നുമല്ല'' (വി.ഖു. 6:89, 90).

''അവരെ നാം നമ്മുടെ കല്‍പനപ്രകാരം മാര്‍ഗദര്‍ശനം നല്‍കുന്ന നേതാക്കളാക്കുകയും ചെയ്തു. നല്ലകാര്യങ്ങള്‍ ചെയ്യണമെന്നും നമസ്‌കാരം മുറപോലെ നിര്‍വഹിക്കണമെന്നും സക്കാത്ത് നല്‍കണ മെന്നും നാം അവര്‍ക്ക് ബോധനം നല്‍കുകയും ചെയ്തു. നമ്മെ ആയിരുന്നു അവര്‍ ആരാധിച്ചിരു ന്നത്'' (വി.ഖു. 21:73). ദുര്‍മാര്‍ഗമൊന്നുമില്ലാത്ത നോഹയെയും (7:61) സല്‍മാര്‍ഗനിഷ്ഠനായ ലൂത്തി നെയും (70:80-84) സദ്‌വൃത്തനായ യാക്കോബിനെയും (21:72) വിനയാന്വിതരും ദൈവിക മാര്‍ഗ ത്തില്‍ ഉറച്ച് നിന്നവരുമായ ദാവൂദിനെയും സുലൈമാനെയും (27:15, 38:30) പരിശുദ്ധ പ്രവാചകനും മഹാനും (3:45) മാതൃബഹുമാനമുള്ളവനു (19:32)മായ യേശുവിനെയുമാണ് ഖുര്‍ആന്‍ പരിചയപ്പെടുത്തുന്നത്.

പ്രപഞ്ച നാഥൻ ഏകനാണെന്നും അവൻ മാത്രമാണ് ആരാധ്യനെന്നുമുള്ള ആദർശങ്ങളിൽ ക്വുർ ആനും ബൈബിളും യോജിക്കുന്നു.എന്നാൽ ബൈബിൾ വരച്ചു കാണിക്കുന്നതിൽ നിന്ന് തികച്ചും വ്യത്യസ്തമായ ദൈവസങ്കല്പമാണ് ക്വുർആനിലുള്ളത്. വ്യത്യാസങ്ങളെ ഇങ്ങനെ സംക്ഷേപിക്കാം:

  1. സൃഷ്ടികര്‍ത്താവിനെക്കുറിച്ച് പരാമര്‍ശിക്കുമ്പോള്‍ അവന്റെ മഹത്വത്തിന് അനുഗുണവും ഔന്നത്യത്തിന് ഗ്ലാനി സംഭവിക്കാത്തതുമായ പരാമര്‍ശങ്ങള്‍ മാത്രമെ ഖുര്‍ആനിലുള്ളൂ. ബൈബി ളിലാകട്ടെ, യഹോവയുടെ മഹത്വം ഉല്‍ഘോഷിക്കുന്നുണ്ടെങ്കിലും ഇസ്രായീല്‍ വംശീയതയുടെ സ്വാധീനമുള്ള വചനങ്ങളെത്തുമ്പോള്‍ ഈ മഹത്വത്തിന് വിരുദ്ധമായ നിരവധി പരാമര്‍ശങ്ങള്‍ നടത്തുന്നതായി കാണുന്നുണ്ട്. മനുഷ്യനെ ദൈവത്തിന്റെ പ്രതിഛായയില്‍ അവന്ന് സദൃശനാ യാണ് സൃഷ്ടിച്ചിരിക്കുന്നതെന്ന് (ഉല്‍ 1:26) പറയുന്നത് മുതല്‍ക്കാരംഭിക്കുന്നു ദൈവമഹത്വത്തിന് വിരുദ്ധമായ പരാമര്‍ശ ങ്ങള്‍. വിശ്രമമാവശ്യമുള്ളവനായി ദൈവത്തെ ചിത്രീകരിക്കുന്നതോടെ (ഉല്‍ 2:2,3) ദൈവനിന്ദ അതിന്റെ പരമ കാഷ്ഠയിലെത്തുന്നു. ഏദെന്‍ തോട്ടത്തില്‍ ഒളിച്ച ആദാമി നെയും ഹവ്വയെയും അന്വേഷിച്ച് തിരഞ്ഞുനടക്കുന്ന ബൈബിളിലെ ദൈവം (ഉല്‍ 3:8-13) കിരാത വര്‍ഗങ്ങള്‍ക്കിടയിലെ പ്രാകൃത സങ്കല്‍പത്തേക്കാള്‍ താഴ്ന്ന ദൈവസ ങ്കല്‍പമാണ് ദ്യോതിപ്പി ക്കുന്നത്. താന്‍ ചെയ്തുപോയ കാര്യമാലോചിച്ച് ദുഃഖിക്കുകയും (ഉല്‍ 6:6) മുന്‍കൂട്ടി തീരുമാ നിച്ചുറച്ച കാര്യങ്ങളില്‍ നിന്ന് മനസ് മാറുകയും (പുറ 32:14) ചെയ്യുന്ന ഇസ്രായേലിന്റെ ദൈവം (പുറപ്പാട് 20:5) ഗോത്രപിതാവായ യാക്കോബുമായി മല്ലയുദ്ധം നടത്തി പരാജയപ്പെട്ട കഥകൂടി (ഉല്‍ 32:28) ബൈബിള്‍ പറയു മ്പോള്‍ ഇസ്രായേല്‍ വംശീയതയുടെ സ്വാധീനം എത്രത്തോളം അതി ന്റെ രചനയിലുണ്ടായിട്ടുണ്ടെന്ന് സുതരാം ബോധ്യമാകുന്നു. ഖുര്‍ആനാകട്ടെ, പ്രപ ഞ്ചത്തിന്റെ സ്രഷ്ടാവും നിയന്താവുമായി അല്ലാഹുവിനെ പരിചയപ്പെടുത്തുമ്പോഴൊന്നും തന്നെ അവന്റെ മഹത്വത്തിനോ വിശുദ്ധിക്കോ കോട്ടം തട്ടിക്കുന്ന യാതൊരു പരാമര്‍ശവും നടത്തുന്നില്ല. അറബിക ളുടെയോ ഖുറൈശികളുടെയോ മാത്രം ദൈവമല്ല ഖുര്‍ആനിലെ അല്ലാഹു, പ്രത്യുത സര്‍വ്വലോക രക്ഷിതാവാണ്. ഏതാനും ഖുര്‍ആന്‍ സൂക്തങ്ങള്‍ കാണുക: ''അല്ലാഹു-അവനല്ലാ തെ ദൈവമില്ല. എന്നെന്നും ജീവിച്ചിരിക്കുന്നവന്‍, എല്ലാം നിയന്ത്രിക്കുന്നവന്‍, മയക്കമോ ഉറക്കമോ അവനെ ബാധിക്കുകയില്ല. അവന്റേ താണ് ആകാശഭൂമികളില്‍ ഉള്ളതെല്ലാം'' (വി.ഖു. 2:55).

''അവന്‍ തന്നെയാണ് ആകാശങ്ങളിലും ഭൂമികളിലും സാക്ഷാല്‍ ദൈവം. നിങ്ങളുടെ രഹസ്യവും നിങ്ങളുടെ പരസ്യവും അവന്‍ അറി യുന്നു. നിങ്ങള്‍ നേടിയെടുക്കുന്നതും അവന്‍ അറിയുന്നു'' (വി.ഖു.6:3).

''അവന്റെ പക്കലാകുന്നു അദൃശ്യകാര്യത്തിന്റെ ഖജനാവുകള്‍. അവനല്ലാതെ അവ അറിയുക യില്ല. കരയിലും കടലിലുമുള്ളത് അവന്‍ അറിയുന്നു. അവന്‍ അറിയാതെ ഒരു ഇല പോലും അന ങ്ങുന്നില്ല. ഭൂമിയുടെ ഇരുട്ടുകള്‍ക്കുള്ളിലിരിക്കുന്ന ഒരു ധാന്യമണിയാക ട്ടെ പച്ചയോ ഉണങ്ങിയതോ ആയ ഏതൊരു വസ്തുവാകട്ടെ വ്യക് തമായ ഒരു രേഖയില്‍ എഴുതപ്പെട്ടതായിട്ടല്ലാതെ ഉണ്ടാകില്ല'' (വി.ഖു.6:59). ''ആകാശങ്ങളുടെയും ഭൂമിയുടെയും സ്രഷ്ടാവാകുന്നു (അവന്‍). നിങ്ങള്‍ക്കുവേണ്ടി നിങ്ങളുടെ വര്‍ഗത്തില്‍നിന്നുതന്നെ അവന്‍ ഇണകളെ ഉണ്ടാക്കിത്തന്നിരിക്കുന്നു. കന്നുകാലികളില്‍ നിന്നും ഇണകളെ (ഉണ്ടാക്കിയിരിക്കുന്നു) അതിലൂടെ നിങ്ങളെ അവന്‍ സൃഷ്ടിച്ച് വര്‍ദ്ധിപ്പിക്കുന്നു. അവന് തുല്യമായി യാതൊന്നുമില്ല. അവന്‍ എല്ലാം കേള്‍ക്കുന്നവനും കാണുന്നവനുമാകുന്നു'' (വി.ഖു. 42:11).

 ''(നബിയേ) പറയുക; കാര്യം അല്ലാഹു ഏകനാണ് എന്നതാകുന്നു. അല്ലാഹു ഏവര്‍ക്കും ആശ്രയമാ യിട്ടുള്ളവനാകുന്നു. അവന്‍ (ആര്‍ക്കും) ജന്മം നല്‍കിയിട്ടില്ല. (ആരുടെയും സന്തതിയായി) ജനിച്ചിട്ടു മില്ല. അവന് തുല്യനായി ആരുമില്ലതാനും (വി.ഖു.  112:1-4)

  1. ഖുര്‍ആനിലെയും ബൈബിളിലെയും ചരിത്രകഥനങ്ങള്‍ തമ്മിലുള്ള അന്തരത്തിന്റെ ആത്മാവ് സ്ഥിതിചെയ്യുന്നത് അവയിലെ ദൈവസങ്കല്‍പങ്ങള്‍ തമ്മിലുള്ള അടിസ്ഥാനപരമായ വ്യത്യാസ ങ്ങളിലാണ്. ആദമിന്റെ കഥ വിവരിക്കുന്നിടത്തുതന്നെ കാണുന്ന അന്തരങ്ങള്‍ ശ്രദ്ധിക്കുക.

(i)-  ആദമിനോടും ഇണയോടും തിന്നരുതെന്ന് കല്‍പിച്ച പഴം നന്മ തിന്മകളെക്കുറിച്ച അറിവിന്റെ വൃക്ഷത്തിന്‍േറതായിരുന്നുവെന്നാണ് ബൈബിള്‍ പറയുന്നത്. (ഉല്‍ 2:17), ബൈബിള്‍ പ്രകാരം അത് തിന്നുക വഴിയാണ് മനുഷ്യന് നന്മതിന്മകളെക്കുറിച്ച് അറിവുണ്ടായത് (ഉല്‍ 3:6, 7; 3:22) നന്മതിന്മകളെ വ്യവഛേദിച്ച് മനസ്സിലാക്കാന്‍ കഴിവില്ലാത്ത മനുഷ്യനോട് എങ്ങനെയാണ് വിലക്കപ്പെട്ട കനി തിന്ന രുതെന്ന് കല്‍പിക്കുക? വിധിവിലക്കുകള്‍ പ്രസക്തമാകുന്നത് നന്മതിന്മകളെക്കുറിച്ച അറിവുണ്ടാ വുന്നതോടെയാണല്ലോ. (നന്മ തിന്മകളെക്കുറിച്ച ജ്ഞാനത്തിന്റെ അഭാവത്തില്‍ മൃഗങ്ങളുടെ ലോക ത്ത് വിധിവിലക്കുകള്‍ അപ്രസക്തമാണ് എന്നോര്‍ക്കുക). ഖുര്‍ആനിലെവിടെയും വിലക്കപ്പെട്ട കനി യെക്കുറിച്ച് 'നന്മതിന്മകളെക്കുറിച്ച അറിവിന്റെ വൃക്ഷ'മെന്ന് പരിചയപ്പെടുത്തുന്നില്ല. നന്മതിന്മക ളെക്കുറിച്ച അറിവും നന്മ സ്വീകരിച്ച് ഉന്നതനാകുവാനും തിന്മകളിലൂടെ അധമനാകുവാനുമുള്ള സാധ്യതയും അവന്റെ സൃഷ്ടിയില്‍തന്നെ നിലീനമാണെന്നാണ് ഖുര്‍ആനിക പരാമര്‍ശങ്ങളില്‍നിന്ന് മനസ്സിലാകുന്നത്. നന്മ തിന്മകള്‍ വ്യവഛേദിച്ച് മനസ്സിലാക്കുവാനും തദടിസ്ഥാനത്തില്‍ വസ്തുക്ക ള്‍ക്ക് നാമകരണം ചെയുവാനുമുള്ള കഴിവ് നല്‍കപ്പെടുകയും അങ്ങനെ മാലാഖമാരേക്കാള്‍ ഉന്നത നാവുകയും ചെയ്ത മനുഷ്യനെയാണ് ഖുര്‍ആന്‍ വരച്ചുകാണിക്കുന്നത്. വിലക്കപ്പെട്ട കനിയും നന്മതിന്മകളെക്കുറിച്ച അറിവും തമ്മില്‍ യാതൊരു വിധത്തിലും ഖുര്‍ആന്‍ ബന്ധപ്പെടുത്തുന്നില്ല. ''ഞാനിതാ ഭൂമിയില്‍ ഒരു ഖലീഫയെ നിയോഗിക്കുവാന്‍ പോവുകയാണെന്ന് നിന്റെ നാഥന്‍ മല ക്കുളോട് പറഞ്ഞ സന്ദര്‍ഭം. അവര്‍ പറഞ്ഞു: അവിടെ കുഴപ്പം ഉണ്ടാക്കുകയും രക്തം ചിന്തുകയും ചെയ്യുന്നവനെയാണോ നീ നിയോഗിക്കുന്നത്? ഞങ്ങളാകട്ടെ നിന്റെ മഹത്വത്തെ പ്രകീര്‍ത്തിക്കു കയും നിന്റെ പരിശുദ്ധിയെ വാഴ്ത്തുകയും ചെയ്യുന്നവരല്ലോ. അവന്‍ (അല്ലാഹു) പറഞ്ഞു: നിങ്ങ ള്‍ക്കറിഞ്ഞുകൂടാത്തത് എനിക്കറിയാം. അവന്‍ (അല്ലാഹു) ആദമിനെ നാമങ്ങളെല്ലാം പഠിപ്പിച്ചു. പിന്നീട് ആ പേരിട്ടവയെ അവന്‍ മലക്കുകള്‍ക്ക് കാണിച്ചു. എന്നിട്ടവന്‍ ആജ്ഞാപിച്ചു: നിങ്ങള്‍ സത്യവാന്മാരാണെങ്കില്‍ ഇവയുടെ നാമങ്ങള്‍ എനിക്ക് പറഞ്ഞുതരൂ. അവര്‍ പറഞ്ഞു: നിനക്ക് സ്‌തോത്രം! നീ പഠിപ്പിച്ചുതന്നതല്ലാത്ത യാതൊരു അറിവും ഞങ്ങള്‍ക്കില്ല. നീ തന്നെയാണ് സര്‍വ്വജ്ഞ നും അഗാധജ്ഞാനിയും. അനന്തരം അവന്‍ (അല്ലാഹു) പറഞ്ഞു: ആദമേ ഇവര്‍ക്ക് അവയുടെ നാമ ങ്ങള്‍ പറഞ്ഞുകൊടുക്കൂ. അങ്ങനെ അവന്‍ (ആദം) അവര്‍ക്ക് ആ നാമ ങ്ങള്‍ പറഞ്ഞുകൊടുത്ത പ്പോള്‍ അവന്‍ (അല്ലാഹു) പറഞ്ഞു: ആകാശഭൂമികളിലെ അദൃശ്യകാര്യങ്ങളും നിങ്ങള്‍ വെളിപ്പെ ടുത്തുന്നതും ഒളിച്ചുവെക്കുന്നതുമെല്ലാം എനിക്ക് അറിയാമെന്ന് ഞാന്‍ നിങ്ങളോട് പറഞ്ഞിട്ടില്ലേ'' (വി.ഖു.2:30-33).

ii)- വിലക്കപ്പെട്ട കനി തിന്നരുതെന്ന ദൈവിക കല്‍പനയില്‍ പ്രസ്തുത കനിതിന്നാല്‍ നീ മരിക്കുമെന്ന് ദൈവം ആദാമിനോട് പറയുന്നതായാണ് ബൈബിള്‍ ഉദ്ധരിക്കുന്നത് (ഉല്‍ 2:17). ദൈവിക കല്‍പന ലംഘിക്കുവാന്‍ മനുഷ്യരെ പ്രേരിപ്പിച്ച സര്‍പ്പമാകട്ടെ ''നിങ്ങ ള്‍ മരിക്കുകയില്ല. അത് തിന്നാല്‍ നിങ്ങ ളുടെ കണ്ണുകള്‍ തുറക്കുമെ ന്നും നന്മതിന്മകളെ തിരിച്ചറിഞ്ഞ് നിങ്ങള്‍ ദൈവത്തെപോലെ ആയിത്തീ രുമെന്നും ദൈവത്തിനറിയാം'' എന്ന് പറഞ്ഞുകൊണ്ടാണ് അവരെ പ്രലോഭിപ്പിച്ചത് (ഉല്‍ 3:5). വില ക്കപ്പെട്ട കനി തിന്നപ്പോള്‍ ദൈവം ഭീഷണിപ്പെടുത്തിയതുപോലെ ആദിമനുഷ്യര്‍ മരിച്ചില്ല, പ്രത്യുത അ വര്‍ക്ക് നന്മതിന്മകളെക്കുറിച്ച് ജ്ഞാനമുണ്ടാവുകയാണ് ചെയ്തത് (ഉല്‍ 3:6,7, 3:22). ദൈവം കളവ് പറഞ്ഞ് ആദി മനുഷ്യരെ ഭീഷണിപ്പെടുത്തിയെന്നും യാഥാര്‍ത്ഥ്യം ബോധ്യപ്പെടുത്തിയത് സര്‍പ്പമാ ണെന്നുമാണ് ഈ കഥ ശരിയാണെങ്കില്‍ വന്നുചേരുക. ദൈവിക മഹത്വത്തിന് ഇടിവ് വരുത്തുന്ന ഇത്തരം കഥകളൊന്നുംതന്നെ ഖുര്‍ആനിലില്ല.

iii)- നന്മതിന്മകളെക്കുറിച്ച അറിവിന്റെ കനിതിന്ന മനുഷ്യനെ ഭയപ്പെടുകയും ജീവന്റെ കനികൂടി തിന്ന് മനുഷ്യന്‍ ദൈവത്തെപ്പോലെയാകാതിരിക്കുവാന്‍ മുന്‍കരുതലെടുക്കുകയും ചെയ്യുന്ന ദൈവ ത്തെയാണ് ഉല്‍പത്തി പുസ്തകത്തില്‍ നാം കാണുന്നത് (ഉല്‍ 3: 22-24). വിലക്കപ്പെട്ട കനി തിന്നുക വഴി എന്തെങ്കിലും തരത്തിലുള്ള ദൈവികാംശം മനുഷ്യനുണ്ടായതായി ഖുര്‍ആന്‍ പഠിപ്പിക്കുന്നില്ല. ദൈവ ത്തിന്റെ ഔന്നത്യത്തെയും സര്‍വ്വജ്ഞതയെയും ചോദ്യംചെ യ്യുന്ന ഇത്തരം കഥകളൊന്നും ഖുര്‍ആ നിലില്ല.

(iv) - വിലക്കപ്പെട്ട കനി തിന്നുവാന്‍ മനുഷ്യനെ പ്രേരിപ്പിച്ചത് സര്‍പ്പമാണെന്നാണ് ബൈബിള്‍ പറയു ന്നത് (ഉല്‍ 3:1-5, 3:13). ഇങ്ങ നെ ചെയ്യുക വഴി ദൈവശാപത്തിന് സര്‍പം വിധേയമായി എന്നും, പ്രസ്തുത ശാപത്തിന്റെ ഫലമായാണ് സര്‍പ്പം ഉരസ്സുകൊണ്ട് ഇഴ ഞ്ഞുനടക്കുന്നതെന്നും, മനുഷ്യനും സര്‍പ്പവും തമ്മിലുള്ള വിരോധത്തിന്റെ അടിസ്ഥാനകാരണമിതാണെന്നുമാണ് ബൈബിള്‍ പാഠം (ഉല്‍ 3:14-15). മനുഷ്യന്റെ ശത്രുവായ സാത്താനാണ് മനുഷ്യനെ വഴിതെറ്റിക്കുകയും വിലക്കപ്പെട്ട കനി തിന്നുവാന്‍ പ്രേരിപ്പിക്കുകയും ചെയ്തതെന്നാണ് ഖുര്‍ആന്‍ പറയുന്നത് (ഖുര്‍ആന്‍ 2:35, 36). ഖുര്‍ ആനിലെ ഇവ്വിഷയകമായ പരാമര്‍ശങ്ങളിലൊന്നും സര്‍പ്പം കടന്നുവരുന്നേയില്ല. ദൈവികശാ പത്തിന്റെ ഫലമായിട്ടാണ് സര്‍പം ഇഴ ഞ്ഞ് നടക്കുകയും മനുഷ്യരാല്‍ വെറുക്കപ്പെടുന്നവനാവു കയും ചെയ്തതെങ്കില്‍ പ്രസ്തുത ശാപത്തിന് മുമ്പുള്ള സര്‍പം ഏത് തരത്തിലുള്ളതായിരുന്നുവെന്ന ചോദ്യത്തിന് ഖുര്‍ആനിക വിശദീകരണങ്ങളുടെ വെളിച്ചത്തില്‍ യാതൊരു സാധുതയുമില്ല.

(v) വിലക്കപ്പെട്ട കനി തിന്നതുവഴി ദൈവം ശപിച്ചതിനാലാണ് സ്ത്രീക്ക് ഗര്‍ഭപീഢയും പ്രസവ വേദന യുമുണ്ടായത് എന്നാണ് ബൈബിള്‍ പഠിപ്പിക്കുന്നത് (ഉല്‍ 3:16). ആദിമാതാവിന്റെ തെറ്റിനുള്ള ശിക്ഷ യാണ് ഇന്നും മാതാക്കള്‍ അനുഭവിച്ചുകൊണ്ടിരിക്കുന്നത് എന്നര്‍ത്ഥം. ഗര്‍ഭധാരണത്തെയും പ്രസവ ത്തെയുമെല്ലാം ശിക്ഷയായിക്കാണുന്നതിന് പകരം ദൈവികാനുഗ്രഹമായാണ് ഖുര്‍ആന്‍ മനസിലാക്കിത്തരുന്നത് (ഖുര്‍ആന്‍ 29:8, 46:15, 31:14). വിലക്കപ്പെട്ട കനിയുമായി പ്രസവവേദനക്കോ ഗര്‍ഭപീഢ ക്കോ എന്തെങ്കിലും തരത്തില്‍ ബന്ധമുള്ളതായി ഖുര്‍ആന്‍ പഠിപ്പിക്കുന്നേയില്ല.

(vi) മനുഷ്യരുടെ അധ്വാനവും കൃഷിയുമെല്ലാം വിലക്കപ്പെട്ട കനി തിന്നതുമൂലമുണ്ടായ ദൈവിക ശാപത്തിന്റെ ഫലമായുണ്ടായവയായാണ് ബൈബിള്‍ പരിചയപ്പെടുത്തുന്നത് (ഉല്‍ 3:18, 19). ഖുര്‍ ആനിലാകട്ടെ അധ്വാനിക്കുവാനും സമ്പാദിക്കുവാനുമെല്ലാമുള്ള മനുഷ്യ കഴിവിനെ ദൈവികാനു ഗ്രഹമായാണ് (62:10) വിശേഷിപ്പിച്ചിരിക്കുന്നത്. മനുഷ്യാധ്വാനവും വിലക്കപ്പെട്ട കനിയുമായി ബന്ധപ്പെടുത്തുന്ന യാതൊരു ഖുര്‍ആനിക പരാമര്‍ശവുമില്ല.

(vii) വിലക്കപ്പെട്ട കനി തിന്ന ആദാമും ഹവ്വയും പശ്ചാത്തപി ക്കുകയോ ദൈവം അവര്‍ക്ക് പൊറു ത്ത് കൊടുക്കുകയോ ചെയ്ത തായുള്ള യാതൊരു പരാമര്‍ശവും ബൈബിളിലില്ല. ഖുര്‍ആനാകട്ടെ, തങ്ങളുടെ തെറ്റില്‍ പശ്ചാത്താപവിവശരായ ആദിമാതാപിതാക്കളു ടെ ക്ഷമായാചനയും കാരുണ്യ മൂര്‍ത്തിയായ ദൈവം തമ്പുരാന്റെ പൊറുത്തുകൊടുക്കലും പ്രാധാന്യത്തോട് കൂടിത്തന്നെ പരാമര്‍ ശി ക്കുന്നുണ്ട്. ''അവര്‍ രണ്ടുപേരും  പറഞ്ഞു. ഞങ്ങളുടെ രക്ഷിതാവേ! ഞങ്ങള്‍ ഞങ്ങളോടുതന്നെ അക്രമം ചെയ്തിരിക്കുന്നു. നീ ഞങ്ങള്‍ക്ക് പൊറുത്ത് തരികയും കരുണ കാണിക്കുകയും ചെയ്തി ല്ലെങ്കില്‍തീര്‍ ച്ചയായും ഞങ്ങള്‍ നഷ്ടം പറ്റിയവരുടെ കൂട്ടത്തിലായിരിക്കും'' (വി.ഖു.7:23). ''അനന്തരം ആദം തന്റെ രക്ഷിതാവിങ്കല്‍നിന്ന് ചില വചന ങ്ങള്‍ സ്വീകരിച്ചു. (ആ വചനങ്ങള്‍ മുഖേന പശ്ചാ ത്തപിച്ച) ആദമിന് അല്ലാഹു പാപമോചനം നല്‍കി. അവന്‍ പശ്ചാത്താപം ഏറെ സ്വീക രിക്കുന്നവനും കരുണാനിധിയുമത്രെ'' (വി.ഖു. 2:37,38)

ക്രിസ്തു ക്രൂശിക്കപ്പെട്ടിട്ടില്ലന്നാണ് ക്വുർആൻ വ്യക്തമാക്കുന്നത്.

"അല്ലാഹുവിന്‍റെ ദൂതനായ, മര്‍യമിന്‍റെ മകന്‍ മസീഹ്‌ ഈസായെ ഞങ്ങള്‍ കൊന്നിരിക്കുന്നു എന്നവര്‍ പറഞ്ഞതിനാലും ( അവര്‍ ശപിക്കപ്പെട്ടിരിക്കുന്നു. ) വാസ്തവത്തില്‍ അദ്ദേഹത്തെ അവര്‍ കൊലപ്പെടുത്തിയിട്ടുമില്ല, ക്രൂശിച്ചിട്ടുമില്ല. പക്ഷെ ( യാഥാര്‍ത്ഥ്യം ) അവര്‍ക്ക്‌ തിരിച്ചറിയാതാവുകയാണുണ്ടായത്‌. തീര്‍ച്ചയായും അദ്ദേഹത്തിന്‍റെ ( ഈസായുടെ ) കാര്യത്തില്‍ ഭിന്നിച്ചവര്‍ അതിനെപ്പറ്റി സംശയത്തില്‍ തന്നെയാകുന്നു. ഊഹാപോഹത്തെ പിന്തുടരുന്നതല്ലാതെ അവര്‍ക്ക്‌ അക്കാര്യത്തെപ്പറ്റി യാതൊരു അറിവുമില്ല. ഉറപ്പായും അദ്ദേഹത്തെ അവര്‍ കൊലപ്പെടുത്തിയിട്ടില്ല.എന്നാല്‍ അദ്ദേഹത്തെ അല്ലാഹു അവങ്കലേക്ക്‌ ഉയര്‍ത്തുകയത്രെ ചെയ്തത്‌. അല്ലാഹു പ്രതാപിയും യുക്തിമാനുമാകുന്നു." (ക്വുർആൻ 4:157,158)

'ക്രിസ്തു ക്രൂശിക്കപ്പെട്ടിട്ടില്ല' എന്ന് പറയുമ്പോള്‍ സ്വാഭാവികമായും ഉയര്‍ന്നുവരുന്നൊരു ചോദ്യ മുണ്ട്. നാല് സുവിശേഷ കര്‍ത്താക്കളും ക്രിസ്തുവിന്റെ കുരിശുമരണത്തെക്കുറിച്ച് പ്രതിപാ ദിക്കുന്നുണ്ട്. ദൃക്‌സാക്ഷികളായ നാലുപേരുടെ മൊഴി ഏതൊരു പ്രശ്‌നത്തിലും തീര്‍പ്പുകല്‍പിക്കാ നാവശ്യമായ തെളിവാണ്. സുവിശേഷകര്‍ത്താക്കളുടെ മൊഴിയുടെ അടിസ്ഥാനത്തില്‍ ക്രിസ്തു ക്രൂശിക്കപ്പെട്ടുവെന്ന നിഗമനത്തില്‍ത്തന്നെയാണ് ഏതൊരു ന്യായാധിപനും എത്തിച്ചേരുക. എങ്കില്‍ പിന്നെ കുരിശുമരണം നടന്നിട്ടില്ലെന്ന വാദത്തിന്റെ സാധുതയെന്താണ്?

ഈ വാദം തീര്‍ച്ചയായും പരിശോധിക്കപ്പെടേണ്ടതാണ്. ഒന്നാമതായി, സുവിശേഷകര്‍ത്താക്കള്‍ കുരിശുമരണത്തിന് ദൃക്‌സാക്ഷിക ളായിരുന്നുവോയെന്ന് പരിശോധിക്കപ്പെടണം. മത്തായിയുടെ സുവിശേഷം രചിച്ചത് അപ്പോസ്തലനായ മത്തായിയാണെന്ന് ഖണ്ഡിതമായി പറയാന്‍ ബൈബിള്‍ ചരിത്രത്തെക്കുറിച്ച് പഠിച്ചവരൊന്നും മിനക്കെടുന്നില്ല. ഇത് മത്തായിയുടെ പേരില്‍ മറ്റാരോ രചിച്ച താവാനാണ് സാധ്യതയെന്നാണ് വേദപുസ്തക നിഘണ്ടു പറയു ന്നത്. (റവ. എ.സി. ക്ലേയിറ്റന്‍: വേദ പുസ്തക  നിഘണ്ടു. പേജ്: 312. ) പത്രോസിന്റെ ദ്വിഭാഷിയായിരുന്ന മാര്‍ക്കോസ്, ക്രിസ്തുവിനെ കണ്ടിട്ടുപോലുമില്ല.'(Ibid പേജ് 322. ) പൗലോസിന്റെ ശിഷ്യനായ ലൂക്കോസിന്റെ സ്ഥിതിയും തഥൈവ.  യോഹന്നാന്‍ സുവിശേഷത്തിന്റെ കര്‍ത്താവിനെ സംബന്ധിച്ച് തര്‍ക്കങ്ങള്‍ പണ്ഡിത ര്‍ക്കിടയില്‍ ഇന്നും നിലനില്‍ക്കുന്നു.( Ibid പേജ് 430.) ഇതില്‍നിന്ന് കുരിശുമരണം റിപ്പോര്‍ട്ടു ചെയ്തു വെന്ന് പറയുന്ന സുവിശേഷകര്‍ത്താക്കളിലാരുംതന്നെ പ്രസ്തുത സംഭവത്തിനു ദൃക്‌സാക്ഷികളായി രുന്നില്ലെന്ന് സുതരാം വ്യക്തമാവുന്നു.

ഇനി നാം കോടതിയിലേക്ക് കടക്കുക. ഒരു സംഭവത്തിന് നാല് ദൃക്‌സാക്ഷികള്‍ കോടതിയില്‍ പ്രത്യക്ഷപ്പെട്ടുവെന്നിരിക്കട്ടെ, ഒരേ സംഭവത്തെക്കുറിച്ച് നാല് പേര്‍ പരസ്പരവിരുദ്ധമായ വിശദീകരണങ്ങള്‍ നല്‍കിയാല്‍ കോടതി എന്തുവിധിക്കും? ഈ നാലുപേരും തെളിവിന് കൊള്ളില്ലെന്നും മറ്റു തെളിവുകളില്ലെങ്കില്‍ പ്രസ്തുത സംഭവം നടന്നുവെന്നു പറയുക വയ്യെന്നുമായിരിക്കും കോടതിവിധി. ഇതേ അവസ്ഥയാണ് കുരിശുമരണത്തിനുമുള്ളത്. കുരിശുമരണവും അനന്തരസംഭവങ്ങളും വിശദീകരിക്കുന്നിടത്ത് നാലു സുവിശേഷങ്ങളും പരസ്പര വിരുദ്ധങ്ങളായ ഒട്ടനവധി പ്രസ്താവനകള്‍ നടത്തുന്നതായി കാണാന്‍ കഴിയും.

യേശുവിനെ ഒറ്റുകൊടുക്കുന്നത് മുതല്‍ ആരംഭിക്കുന്നു വൈരുധ്യങ്ങള്‍, പ്രസ്തുത സംഭവം മത്തായി വിവരിക്കുന്നത് ഇങ്ങനെയാണ്.

'അവന്‍ സംസാരിച്ചുകൊണ്ടിരിക്കുമ്പോള്‍തന്നെ പന്ത്രണ്ടു പേരില്‍ ഒരുവനായ യൂദാസ് അവിടെ യെത്തി. അവനോടുകൂടെ പ്രധാനപുരോഹിതന്മാരുടെയും ജനപ്രമാണികളുടെയും അടുക്കല്‍നിന്ന് വാളും വടികളുമായി ഒരു ജനക്കൂട്ടവും വന്നിരുന്നു. ഒറ്റുകാരന്‍ അവര്‍ക്ക് ഈ അടയാളം നല്‍കി യിരുന്നു. ഞാന്‍ ആരെ ചുംബിക്കുന്നുവോ, അവന്‍തന്നെ. അവനെ പിടിച്ചുകൊള്ളുക. അവന്‍ പെട്ടെന്ന് യേശുവിന്റെ അടുത്തുചെന്ന് 'ഗുരോ സ്വസ്തി' എന്നു പറഞ്ഞു അവനെ ചുംബിച്ചു. യേശു അവനോട് ചോദിച്ചു. സ്‌നേഹിതാ നീ എന്തിനാണ് വന്നത്? അപ്പോള്‍ അവര്‍ മുന്നോട്ടുവന്നു യേശു വിനെ പിടിച്ചു.'(മത്തായി 26: 47-50.)

ഈ സംഭവം ലൂക്കോസ് വിശദീകരിക്കുന്നത് നോക്കുക. 'അവന്‍ ഇത് പറഞ്ഞുകൊണ്ടിരിക്കുമ്പോള്‍ ഒരു ജനക്കൂട്ടം അവിടെവന്നു. പന്ത്രണ്ടു പേരില്‍ ഒരുവനായ യൂദാസാണ് അവരുടെ മുന്നില്‍ നടന്നി രുന്നത്. യേശുവിനെ ചുംബിക്കാന്‍ അവന്‍ മുന്നോട്ടുവന്നു. യേശു അവനോട് ചോദിച്ചു. യൂദാസെ നീ ചുംബനം കൊണ്ടോ മനുഷ്യ പുത്രനെ ഒറ്റുകൊടുക്കുന്നത്'?( ലൂക്കോസ് 22:47, 48.)

യേശുവിനെ ഒറ്റിക്കൊടുത്തു ബന്ധിച്ച സംഭവം യോഹന്നാന്‍ റിപ്പോര്‍ട്ടു ചെയ്യുന്നത് ഇതില്‍ നിന്നെല്ലാം വളരെ വ്യത്യാസമായി ക്കൊണ്ടാണ്. 'യൂദാസ് ഒരു ഗണം പടയാളികളെയും പുരോഹിത പ്രമുഖന്മാരുടെയും ഫരിസേയരുടെയും അടുക്കല്‍നിന്ന് സേവകരെയും കൂട്ടി പന്തങ്ങളും വിളക്കു കളും ആയുധങ്ങളുമായി അവിടെയെത്തി. തനിക്ക് സംഭവിക്കാനിരിക്കുന്നതെല്ലാം അറിഞ്ഞിരുന്ന യേശു മുന്നോട്ടുവന്ന് അവരോട് ചോദിച്ചു. നിങ്ങള്‍ ആരെയാണ് അന്വേഷിക്കുന്നത്? അവന്‍ പറഞ്ഞു: നസാറായക്കാരനായ യേശുവിനെ. യേശു പറഞ്ഞു: അത് ഞാനാണ്. അവനെ ഒറ്റിക്കൊ ടുത്ത യൂദാസും അവരോട് കൂടെ ഉണ്ടായിരുന്നു. ഞാനാണ് എന്ന് അവന്‍ പറഞ്ഞപ്പോള്‍ അവര്‍ പിന്‍വലിയുകയും നിലംപതിക്കുകയും ചെയ്തു. അവന്‍ വീണ്ടും ചോദിച്ചു. നിങ്ങള്‍ ആരെ അന്വേ ഷിക്കുന്നു. അവര്‍ പറഞ്ഞു. നസാറായക്കാരനായ യേശുവിനെ യേശു പ്രതിവചിച്ചു: ഞാനാണ് എന്നു നിങ്ങളോടു പറഞ്ഞുവല്ലോ. നിങ്ങള്‍ എന്നെ യാണ് അന്വേഷിക്കുന്നതെങ്കില്‍ ഇവര്‍ പൊയ്‌ക്കൊള്ളട്ടെ' (യോഹന്നാന്‍ 18: 3-8)

ഇതുപോലെ കുറേയധികം പരസ്പര വിരുദ്ധമായ പ്രസ്താവനകള്‍ നടത്തുന്നുണ്ട് സുവിശേഷങ്ങള്‍. യേശുവിനെ ബന്ധിച്ച ശേഷം പ്രധാന പുരോഹിതനായ കയ്യാഫാസിന്റെ അടുക്കലേക്കാണ് ആദ്യം കൊണ്ടുപോയതെന്ന് മത്തായിയും,( മത്തായി 26:57.) കയ്യാഫാസിന്റെ അമ്മായിഅപ്പനായ അന്നാ സിന്റെ അടുക്കലേക്കാണെന്ന് യോഹന്നാനും പറയുന്നു.( യോഹന്നാന്‍ 18:13. 87.  ) ഗോല്‍ഗോഥ മല യിലേക്ക് കുരിശ് ചുമന്നത് യേശു സ്വയമാണെന്ന് യോഹന്നാനും(യോഹന്നാന്‍ 19:17.)കാറേനേയക്കാ രനായ ശിമായോനാണെന്ന്(മത്തായി 27:32.) മത്തായിയും പ്രസ്താവിക്കുന്നു. ശേയുവിനോട് കൂടെ ക്രൂശിക്കപ്പെട്ട രണ്ട് കള്ളന്മാരില്‍ ഒരുത്തന്‍ മാത്രം അദ്ദേഹത്തെ തള്ളിപ്പറഞ്ഞുവെന്നും മറ്റവന്‍ അദ്ദേഹത്തില്‍ വിശ്വസിച്ചുവെന്നും ലൂക്കോസ്(ലൂക്കോസ് 23:42. ) പറയുമ്പോള്‍, രണ്ടു പേരും അവനെ പരിഹസിച്ചുവെന്നാണ് മത്തായി (മത്തായി 27:44. ) പറയുന്നത്. ഇങ്ങനെ കുറെയധികം വൈരുധ്യങ്ങളുണ്ട്.

യേശുവിന്റെ മരണത്തെക്കുറിച്ച് വ്യത്യസ്ത സുവിശേഷങ്ങള്‍ എങ്ങനെയാണ് വിവരിക്കുന്നതെ ന്നുകൂടി പരിശോധിക്കാം. 'ആറാം മണിക്കൂര്‍ മുതല്‍ ഒമ്പതാം മണിക്കൂര്‍ വരെ ഭൂമിയിലെങ്ങും അന്ധകാരം വ്യാപിച്ചു. ഏകദേശം ഒമ്പതാം മണിക്കൂറായപ്പോള്‍ യേശു ഉച്ചത്തില്‍ നിലവിളിച്ചു. 'ഏലി, ഏലി, ലമാ സബക്ഥാനീ' അതായത് എന്റെ ദൈവമേ, എന്റെ ദൈവമേ എന്തുകൊണ്ടു നീ എന്നെ ഉപേക്ഷിച്ചു! അടുത്തുനിന്നിരുന്നവരില്‍ ചിലര്‍ ഇതുകേട്ടു പറഞ്ഞു. അവന്‍ ഏലിയായെ വിളിക്കുന്നു. ഉടന്‍ അവരില്‍ ഒരാള്‍ ഓടിച്ചെന്ന് നീര്‍പ്പഞ്ഞിയെടുത്തു വിനാഗരിയില്‍ മുക്കി, ഒരു ഞാണിന്മേല്‍ ചുറ്റി അവനു കുടിക്കാന്‍ കൊടുത്തു. അപ്പോള്‍ മറ്റുള്ളവര്‍ പറഞ്ഞു:നില്‍ക്കൂഏലിയാ വന്ന് അവനെ രക്ഷിക്കുമോയെന്ന് കാണട്ടെ. യേശു ഉച്ചത്തില്‍ നിലവിളിച്ചുകൊണ്ട് ജീവന്‍ വെടിഞ്ഞു.'(മത്തായി 27:45-50. )

ഈ സംഭവം യോഹന്നാന്‍ വിശദീകരിക്കുന്നതിങ്ങനെയാണ്. 'അനന്തരം, എല്ലാം നിറവേറ്റിക്കഴിഞ്ഞു വെന്നറിഞ്ഞ് തിരുവെഴുത്ത് പൂര്‍ത്തിയാക്കാന്‍ വേണ്ടി യേശു പറഞ്ഞു: എനിക്കു ദാഹിക്കുന്നു. ഒരു പാത്രം നിറയെ വിനാഗരി അവിടെയുണ്ടായിരുന്നു. അവര്‍ വിനാഗരിയില്‍ കുതിര്‍ത്ത ഒരു നീര്‍ പ്പഞ്ഞി ഹിസോപ്പ് ചെടിയുടെ തണ്ടില്‍വെച്ച് അവന്റെ ചുണ്ടോടടുപ്പിച്ചു യേശു വിനാഗരി സ്വീക രിച്ചിട്ട് പറഞ്ഞു: എല്ലാം പൂര്‍ത്തിയായിരിക്കുന്നു. അവന്‍ തലചായ്ച്ച് ആത്മാവിനെ സമര്‍പ്പിച്ചു. '(യോഹന്നാന്‍ 19:28-30.)

ക്രൂശിതന്റെ അവസാനത്തെ വാക്കുകളെന്തായിരുന്നുവെന്നു പോലും ഖണ്ഡിതമായി, ഏകസ്വര ത്തില്‍ പറയാന്‍ പറ്റാത്ത സുവിശേഷകര്‍ത്താക്കളെ പ്രസ്തുത സംഭവത്തിന്റെ ദൃക്‌സാക്ഷി കളാ ക്കാന്‍ പറ്റുമോ? ഒരു പ്രാവശ്യമെങ്കിലും കോടതി വരാന്തയില്‍ പോയിട്ടുള്ളവരെല്ലാം ഉത്തരം പറയും. 'ഇല്ല' അവരുടെ പ്രസ്താവനകള്‍ പരസ്പരവിരുദ്ധമാണ്. അവരെയൊരിക്കലും സാക്ഷ്യ ത്തിന് കൊള്ളുകയില്ല.

ക്രിസ്തുവിന്റെ മരണത്തെക്കുറിച്ച് വ്യക്തമായ വിവരം നമുക്കെവിടെനിന്നുകിട്ടുമെന്ന് പരിശോ ധിക്കുമ്പോള്‍ നമുക്ക് മുന്നില്‍ പിന്നീട് വരുന്നത് ബര്‍ണബാസിന്റെ സുവിശേഷമാണ്. പുതിയ നിയ മത്തിലെവിടെയും കാണാത്ത ബര്‍ണബാസിന്റെ സുവിശേഷ ത്തെക്കുറിച്ച് കേള്‍ക്കുന്ന ക്രൈസ്ത വര്‍ ചിലപ്പോള്‍ അത്ഭുതപ്പെട്ടേക്കാം.

 ബൈബിള്‍ ഒരുവട്ടം വായിച്ചിട്ടുള്ളവര്‍ക്കൊന്നും ബര്‍ണബാസി നെപ്പറ്റി പറഞ്ഞുകൊടുക്കേണ്ട തായി വരികയില്ല. ബര്‍ണബാസാണ് പൗലോസിനെ അപ്പോസ്തലന്മാര്‍ക്കിടയിലേക്ക് കൊണ്ടു വന്നത്.( അപ്പോ. പ്രവൃ. 9:47.) 'ബര്‍ണാബാസ് സുവിശേഷ വേലകള്‍ ചെയ്തുകൊണ്ട് ചുറ്റി നടന്ന തായി അപ്പോസ്തല പ്രവൃത്തികളില്‍ പലപ്രാവശ്യം പറഞ്ഞി രിക്കുന്നു.

ക്രിസ്തുവിനുശേഷം ഒന്നാം നൂറ്റാണ്ടിലും, രണ്ടാം നൂറ്റാണ്ടിലും പ്രചാരത്തിലിരുന്ന സുവിശേഷങ്ങ ളിലൊന്നാണ് ബര്‍ണബാസിന്റെ സുവിശേഷം. ഈ സുവിശേഷം ക്രിസ്തുവിനെറ ദിവ്യത്വത്തെ നിരാകരിക്കുകയും ക്രിസ്തുവിന്റെ സുവിശേഷത്തിലെ സുപ്രധാന ഉപദേശമായ ഏകദൈവ വിശ്വാസത്തിന് ഊന്നല്‍ നല്‍കുകയും ചെയ്യുന്നു. പ്രാകൃത റോമന്‍ പുരാണങ്ങളില്‍നിന്ന് കടമെടുത്ത വിശ്വാസങ്ങളും ആചാരങ്ങളും ക്രിസ്ത്യാനികള്‍ക്കിടയില്‍ പ്രചരിപ്പിച്ചതിനെതിരെ ശക്തമായി നിലകൊണ്ട ഇറാനിയൂസ് (130þ200 CE)  അദ്ദേഹത്തിറെ കാഴ്ചപ്പാടുകളെ ന്യായീകരിക്കുന്നതിനു വേണ്ടി ബര്‍ണബാസിന്റെ സുവിശേഷത്തില്‍നിന്നും ഉദ്ധരിക്കുന്നുണ്ട്. ഇതില്‍നിന്ന് ഒന്ന്, രണ്ട് നൂറ്റാ ണ്ടുകളില്‍ പ്രസ്തുത സുവിശേഷത്തിനുണ്ടായിരുന്ന പ്രാധാന്യം വ്യക്തമാവുന്നുണ്ട്. അലക്‌സാ ണ്ട്രിയ ചര്‍ച്ചുകള്‍ (CHURCHES OF ALEXANDRIA) ക്രിസ്താബ്ദം 325 വരെ ബര്‍ണബാസിന്റെ സുവി ശേഷത്തെ കാനോനിക സുവിശേഷങ്ങളിലൊന്നായി അംഗീകരിച്ചിരുന്നുവെന്നതിന് തെളിവുകളുണ്ട്.

ക്രിസ്താബ്ദം 325-ല്‍ നടന്ന നിഖിയാ കൗണ്‍സിലിന്റെ തീരുമാനപ്രകാരം ഹിബ്രുവിലുള്ള സുവിശേ ഷങ്ങളുടെ എല്ലാ കൈയെഴുത്തു പ്രതികളും നശിപ്പിക്കപ്പെടുകയുണ്ടായി. ഹിബ്രു സുവിശേഷങ്ങള്‍ കൈവശം വെക്കുന്നവര്‍ക്ക് മരണശിക്ഷ വിധിച്ചുകൊണ്ടുള്ള കല്‍പന പുറപ്പെടുവിച്ചു. ഈ നിരോധ നാജ്ഞകളെയെല്ലാം അതിജീവിച്ചുകൊണ്ട് നിലനിന്ന സുവിശേഷമത്രേ ബര്‍ണബാസിന്റെ സുവി ശേഷം. വ്യക്തികളില്‍നിന്നും വ്യക്തികളിലേക്ക് കൈമാറ്റം ചെയ്യപ്പെട്ടുകൊണ്ട് 1738-ല്‍ വിയന്നയിലെ ഇംപീരിയല്‍ ലൈബ്രറിയില്‍ പ്രസ്തുത ഗ്രന്ഥം എത്തി. ഇന്ന് ഗ്രന്ഥത്തിന്റെ കൈയെഴുത്ത് കോപ്പി പ്രസ്തുത ലൈബ്രറിയിലാണുള്ളത്.

ക്രിസ്തുവല്ല, പ്രത്യുത ഒറ്റുകാരനായ യൂദാസാണ് മരത്തില്‍ തറച്ചുകൊല്ലപ്പെട്ടതെന്നാണ് ബര്‍ണ ബാസിന്റെ സുവിശേഷം പറയുന്നത്. യൂദാസാണത്രേ കുരിശില്‍ തൂങ്ങിക്കിടന്നുകൊണ്ട് ഇങ്ങനെ വിലപിച്ചത്. 'ദൈവമേ, നീ എന്തിനാണെന്നെ ഉപേക്ഷിച്ചത്, കുറ്റവാളി രക്ഷപ്പെടുന്നതും ഞാന്‍ അന്യാ യമായി മരിക്കുന്നതും കണ്ടുകൊണ്ട്? (THE GOSPEL OF BARNABAS: TRANSLATED BY LONSDALE AND LAURA RAGG, Chapter 217.)

വഞ്ചകനായ യൂദാസ് പിടിയിലായ സംഭവത്തെപ്പറ്റി ബര്‍ണബാസ് പറയുന്നതിങ്ങനെയാണ്. ''യേശു നിന്നിരുന്ന സ്ഥലത്തിനടുത്ത് യൂദാസും പട്ടാളക്കാരുമെത്തിയപ്പോള്‍ യേശു ജനങ്ങളുടെ ആരവം  കേട്ടുകൊണ്ട് വീട്ടില്‍നിന്നും പിന്‍വലിഞ്ഞു. പതിനൊന്ന് അപ്പോസ്തലന്മാരും അപ്പോള്‍ ഉറങ്ങുക യായിരുന്നു.

അപ്പോള്‍ ദൈവം, അവന്റെ സേവകന്‍ അപകടത്തിലാണെന്ന് മനസ്സിലാക്കിക്കൊണ്ട് അവന്റെ മന്ത്രിമാരായ ഗബ്രിയേല്‍, മിഖായേല്‍, റാഫേല്‍, യുറിയേല്‍ എന്നിവരോട് യേശുവിനെ ലോക ത്തില്‍നിന്ന് പുറത്തേക്കെടുക്കാന്‍ കല്‍പിച്ചു. തെക്കുഭാഗത്തേക്ക് തുറന്നിരിക്കുന്ന ജനവാതിലിലൂടെ യേശുവിനെ വിശുദ്ധ മാലാഖമാര്‍ പുറത്തേക്കെടുത്തു. അവര്‍ അദ്ദേഹത്തെ വഹിച്ചുകൊണ്ട് ദൈവാനുഗ്രഹം എന്നെന്നും നിലനില്‍ക്കുന്ന മാലാഖമാരുടെ സഹവാസത്തില്‍ മൂന്നാം ആകാശ ത്തില്‍ കൊണ്ടു ചെന്നുവെച്ചു.

യേശു എടുക്കപ്പെട്ട ഉടന്‍തന്നെ യൂദാസ് മറ്റുള്ളവര്‍ക്കു മുന്നില്‍ മുറിയിലേക്ക് എടുത്തുചാടി. എല്ലാ അപ്പോസ്തലന്മാരും ഉറങ്ങുകയായിരുന്നു. അപ്പോള്‍ അല്‍ഭുതകാരനായ ദൈവം അത്ഭുതം പ്രവര്‍ ത്തിച്ചു. യൂദാസിന്റെ സംസാരവും മുഖവും യേശുവിന്‍േറത് പോലെയായിത്തീര്‍ന്നു. ഞങ്ങ ളെല്ലാം അദ്ദേഹം യേശുവാണെന്ന് വിചാരിക്കുന്ന പരുവത്തിലായി മാറി. ഞങ്ങളെ ഉണര്‍ത്തി ക്കൊണ്ട് അവന്‍ ഗുരു എവിടെയാണെന്ന് തിരക്കി. അപ്പോള്‍ ഞങ്ങള്‍ അമ്പരന്നുകൊണ്ടു മറുപടി പറഞ്ഞു. 'കര്‍ത്താവേ, അങ്ങ് ഞങ്ങളുടെ ഗുരുവല്ലയോ, ഇപ്പോള്‍ ഞങ്ങളെയെല്ലാം അങ്ങു മറന്നുപോയോ? അവന്‍ പുഞ്ചിരിച്ചുകൊണ്ടു പറഞ്ഞു: ഞാന്‍ യൂദാസ് ഇസ്‌ക്കാരിയോസാണെന്ന് മനസ്സിലാക്കാത്ത നിങ്ങള്‍ ഇപ്പോള്‍ വിഡ്ഢികള്‍തന്നെ!'

ഇതുപറഞ്ഞുകൊണ്ടു നില്‍ക്കുമ്പോള്‍ പട്ടാളക്കാര്‍ പ്രവേശിച്ചു. എല്ലാ നിലക്കും യേശുവിനെപ്പോ ലെയായി മാറിയിരുന്ന യൂദാസിന്റെ മുകളില്‍ കൈവെച്ചു. ഞങ്ങള്‍ക്കു ചുറ്റും നിരന്നിരുന്ന പട്ടാള ക്കാര്‍ക്കിടയില്‍നിന്നും ഓടുമ്പോള്‍ യൂദാസ് പറയുന്നത് ഞങ്ങള്‍ക്ക് കേള്‍ക്കാമായിരുന്നു. ലിനെന്‍ തുണിയില്‍ പൊതി ഞ്ഞിരുന്ന യോഹന്നാന്‍ ഉണര്‍ന്നുകൊണ്ട് ഓടിയപ്പോള്‍ ഒരു പട്ടാളക്കാരന്‍ ലിനെന്‍തുണിയില്‍ കയറിപ്പിടിച്ചതിനാല്‍ അവന്റെ തുണിയഴിയുകയും നഗ്‌നനായി അവന്‍ ഓടി രക്ഷപ്പെടുകയും ചെയ്തു. യേശുവിന്റെ പ്രാര്‍ഥന ദൈവം ശ്രവിക്കുകയും പതിനൊന്ന് പേരും രക്ഷപ്പെടുകയും ചെയ്തു.

പട്ടാളക്കാര്‍ യൂദാസിനെ പിടിച്ചുബന്ധിച്ചത് അയാളെ അവഹേ ളിച്ചുകൊണ്ടായിരുന്നു. അയാള്‍ താന്‍ യേശുവല്ലെന്ന് നിഷേധിച്ചുകൊണ്ടിരുന്നു. പട്ടാളക്കാര്‍ അയാളെ കളിയാക്കിക്കൊണ്ട് പറഞ്ഞു 'സര്‍' പേടിക്കേണ്ട, താങ്കളെ ഇസ്രായീലിന്റെ രാജാവാക്കാനാണ് ഞങ്ങള്‍ വന്നിരിക്കുന്നത്. നിങ്ങള്‍ രാജാധികാരം നിഷേധിക്കുകയില്ലെന്ന് ഞങ്ങള്‍ക്കറിയാവുന്നതിനാലാണ് ഞങ്ങള്‍ നിങ്ങളെ ബന്ധിക്കു ന്നത്''. (Ibid Chapter 215, 217.)

യേശുവിനെ ചതിക്കാന്‍ ശ്രമിച്ച യൂദാസ് കുഴിച്ച കുഴിയില്‍ അദ്ദേഹം വീണുവെന്നും അയാളാണ് ക്രൂശിക്കപ്പെട്ടതെന്നും ബര്‍ണ ബാസിന്റെ സുവിശേഷം വ്യക്തമാക്കുന്നു. ഇവിടെ ക്രിസ്തുവിന്റെ പ്രവചനം പുലരുന്നു. 'മനുഷ്യപുത്രന്‍ എഴുതപ്പെട്ടതുപോലെ പോകുന്നു. പക്ഷേ, മനുഷ്യപുത്രനെ ഒറ്റിക്കൊടുക്കുന്നവനാരോ അവന് ദുരിതം. ജനിക്കാതിരുന്നെങ്കില്‍ അവന്നു നന്നായിരുന്നു. (മത്തായി 26:23, 20.) മനുഷ്യ പുത്രന്‍ ദൈവിക വിധിപ്രകാരം പോയി. ഒറ്റിക്കൊടുത്ത യൂദാസ് കുരിശില്‍ കിടന്നുപിടച്ചപ്പോള്‍ വിചാരിച്ചിരിക്കണം. 'ഞാന്‍ ജനിക്കാതിരുന്നെങ്കില്‍ എത്ര നന്നായിരുന്നു'വെന്ന്.

ഏതായിരുന്നാലും, ക്രിസ്തു ക്രൂശീകരിക്കപ്പെടുകയോ കൊല്ലപ്പെടുകയോ ചെയ്തിട്ടില്ലെന്ന ക്വുർആനികപ്രസ്താവനയെ വെല്ലാനാവശ്യമായ തെളിവുകളൊന്നും ബൈബിളോ മറ്റു ക്രൈസ്തവരചനകളോ നൽകുന്നില്ല.

 
വിഷയവുമായി ബന്ധപ്പെട്ട വീഡിയോ
ക്രൈസ്തവർക്ക് യാതൊന്നും ഭയപ്പെടുകയോ ദുഖിക്കുകയോ വേണ്ടി വരികയില്ലെന്ന് 5:69 ൽ പറയുന്നു. ഇതിന് വിരുദ്ധമായി അവർക്ക് സ്വർഗം നിഷിദ്ധമാണെന്ന് 5:72 ലും പറയുന്നു. ഇതിലേതാണ് ശരി? ജന്‍മമല്ല, വിശ്വാസവും കര്‍മവുമാണ് മനുഷ്യരുടെ മോചനത്തിനുള്ള മാര്‍ഗമെന്ന് വ്യക്തമാക്കുന്നതാണ് സൂറത്തുല്‍ മാഇദയിലെ 69-ാം വചനം. ഈ വചനം തന്നെ 2:62 ലും ആവര്‍ത്തിക്കപ്പെട്ടിട്ടുണ്ട്. പ്രസ്തുത വചനത്തിന്റെ സാരം ഇങ്ങനെയാണ്: സത്യവിശ്വാസികളോ യഹൂദരോ സാബികളോ ക്രൈസ്തവരോ ആരാകട്ടെ അവരില്‍ നിന്ന് അല്ലാഹുവിലും അന്ത്യദിനത്തിലും വിശ്വസിക്കുകയും സല്‍കര്‍മ്മങ്ങള്‍ പ്രവര്‍ത്തിക്കുകയും ചെയ്തവര്‍ക്ക് യാതൊന്നും ഭയപ്പെടേണ്ടതില്ല. അവര്‍ ദു:ഖിക്കേണ്ടി വരികയുമില്ല.” ഈ സൂക്തത്തില്‍ മോക്ഷത്തിനുള്ള മാര്‍ഗമായി നിര്‍ദേശിക്കപ്പെട്ടിട്ടുള്ളത് മൂന്ന് കാര്യങ്ങളാണ്. (ഒന്ന്) അല്ലാഹുവിലുള്ള വിശ്വാസം. (രണ്ട്) അന്ത്യദിനത്തിലുള്ള വിശ്വാസം. (മൂന്ന്) സല്‍കര്‍മങ്ങള്‍ പ്രവര്‍ത്തിക്കുക. ഏത് വിഭാഗത്തില്‍ പെട്ടവരായിരുന്നാലും ഈ മൂന്നു കാര്യങ്ങളുമുണ്ടെങ്കില്‍ സ്വര്‍ഗപ്രവേശം ലഭിക്കുമെന്നാണ് ഖുര്‍ആന്‍ പഠിപ്പിക്കുന്നത്. സ്രഷ്ടാവും സംരക്ഷകനുമായ അല്ലാഹു ഏകനും അദ്വിതീയനും അതുല്യനുമാണെന്ന് വിശ്വസിക്കുകയും അവന്നു മാത്രം ആരാധനകള്‍ അര്‍പ്പിക്കുകയും ചെയ്യുന്നവനാണ് അല്ലാഹുവില്‍ വിശ്വസിച്ചവന്‍ എന്നാലര്‍ത്ഥം. മരണാനന്തര ജീവിതത്തില്‍ അടിയുറച്ച് വിശ്വസിക്കുകയും കര്‍മ്മങ്ങള്‍ക്കുള്ള പ്രതിഫലം പരലോകത്ത് വെച്ച് ലഭിക്കണമെന്ന് കാംക്ഷിക്കുകയുമാണ് അന്ത്യദിനത്തിലുള്ള വിശ്വാസം കൊണ്ട് വിവക്ഷിക്കുന്നത്. ദൈവികമെന്ന് ഉറപ്പുള്ള വിജ്ഞാന സ്രോതസ്സുകളില്‍ പരാമര്‍ശിക്കപ്പെട്ട നന്‍മകള്‍ ചെയ്യുകയും തിന്‍മകളില്‍നിന്ന് അകന്ന് നില്‍ക്കുകയും ചെയ്യുന്നവനാണ് സല്‍കര്‍മങ്ങള്‍ ചെയ്യുന്നവന്‍. ഇക്കാര്യങ്ങള്‍ ജീവിതത്തിലുള്ള ഒരാള്‍ക്ക് ദു:ഖിക്കേണ്ടി വരികയില്ലെന്ന സുവിശേഷമറിയിക്കുകയാണ് മുകളില്‍ വിവരിക്കപ്പെട്ട സൂക്തം ചെയ്യുന്നത്. ക്രിസ്തുവിനെ അനുധാവനം ചെയ്തുകൊണ്ട് മോക്ഷത്തിന്റെ മാര്‍ഗത്തിലെത്തിച്ചേരേണ്ട ക്രൈസ്തവര്‍ അദ്ദേഹത്തിന്റെ ഉപദേശങ്ങളെ അവഗണിച്ചുകൊണ്ട് നരകത്തിന്റെ പാതയിലെത്തിച്ചേര്‍ന്നതിനെ വിമര്‍ശിക്കുകയാണ് 5:72 മുതല്‍ 75 വരെ സൂക്തങ്ങളില്‍ ഖുര്‍ആന്‍ ചെയ്യുന്നത്. യേശു തന്നെയാണ് ദൈവമെന്നും ത്രിയേകത്വത്തിലെ ഒരു വ്യക്തിമാത്രമാണ് അല്ലാഹുവെന്നും വിശ്വസിക്കുന്ന ക്രൈസ്തവര്‍ മോക്ഷത്തിലെത്തിച്ചേരുവാന്‍ 5:69ല്‍ നിര്‍ദേശിക്കപ്പെട്ട ഒന്നാമത്തെ കാര്യത്തില്‍ തന്നെ പിഴച്ചുപോവുകയാണ് ചെയ്തിട്ടുള്ളതെന്നും അതിനാല്‍ അവര്‍ക്ക് സ്വര്‍ഗം നിഷിദ്ധമാണെന്നും വ്യക്തമാക്കുകയുമാണ് ഈസൂക്തങ്ങളില്‍ ചെയ്യുന്നത്. ഇവ 5:69 ന്റെ അനുപൂരകമായ സൂക്തങ്ങളാണ് എന്നര്‍ഥം. ത്രിയേക വിശ്വാസത്തില്‍ നിന്ന് രക്ഷപ്പെട്ട് കൊണ്ട് ഏകദൈവാരാധനയിലേക്ക് കടന്നുവരികയും ദൈവികമെന്ന് ഉറപ്പ് പറയാവുന്ന ഖുര്‍ആനിലും അതിന്റെ പ്രായോഗിക ജീവിത മാതൃകയായ നബിചര്യയിലും വിശദീകരിക്കപ്പെട്ട സല്‍കര്‍മ്മങ്ങള്‍ ചെയ്യുകയും പരലോകമോക്ഷത്തിനു വേണ്ടി പ്രാര്‍ഥിക്കുകയും ചെയ്താല്‍ ക്രൈസ്തവരടക്കമുള്ള സകല മനുഷ്യര്‍ക്കും സമാധാനത്തിന്റെ ഭവനമായ സ്വര്‍ഗത്തില്‍ പ്രവേശനം കിട്ടുമെന്നാണ് 5:69 ല്‍ പറയുന്നത്. ക്രൈസ്തവരുടെ വിശ്വാസാചാരങ്ങളെ വിമര്‍ശിക്കുകയും വേദഗ്രന്ഥത്തിലെ തിരുത്തലുകളെ വ്യക്തമാക്കുകയും ചെയ്യുന്ന ഒട്ടനവധി സൂക്തങ്ങള്‍ ഖുര്‍ആനിലുണ്ട്. ഇവയുമായി യാതൊരു രീതിയിലും 5:69 സൂക്തം വൈരുധ്യം പുലര്‍ത്തുന്നില്ല.
അല്ലാഹു കാരുണ്യത്തെ സ്വന്തത്തിൽ ബാധ്യതയായി രേഖപ്പെടുത്തിയെന്ന് പറയുന്ന ഖുര്‍ആന്‍ സൂക്തത്തിന് (6:12) എതിരല്ലേ അവന് താന് ഉദ്ദേശിക്കുന്നവരെ വഴിതെറ്റിക്കുകയും നരകത്തിലിടുകയും ചെയ്യുമെന്ന് പറയുന്ന (ഉദാ 6:35,14:4) സൂക്തങ്ങൾ ? കരുണാവാരിധിയാണ് അല്ലാഹു. അളവറ്റ കാരുണ്യത്തിന്റെ സ്രോതസ്സാണവന്‍. പരമാണുവിനകത്തെ ചലനങ്ങള്‍ മുതല്‍ താരാസമൂഹങ്ങളിലെ സ്‌ഫോടനങ്ങള്‍ വരെ അല്ലാഹുവിന്റെ അപാരമായ കാരുണ്യത്താലാണ് സംഭവിച്ചുകൊണ്ടിരിക്കുന്നത്. മൃഗങ്ങളും സസ്യങ്ങളും തമ്മില്‍ നിലനില്‍ക്കുന്ന പാരസ്പര്യവും വന്യമൃഗങ്ങളില്‍ പോലും കണ്ടുവരുന്ന പരസ്പര സഹകരണവുമെല്ലാം ദിവ്യ കാരുണ്യത്തിന്റെ ഫലമാണ്. എന്റെ കാരുണ്യം സകല വസ്തുക്കള്‍ക്കും വ്യാപകമായിരിക്കുന്നു (7:156) വെന്നാണ് അല്ലാഹു പറയുന്നത്. അവന്‍ കാരുണ്യത്തെ സ്വന്തം പേരില്‍ (ബാധ്യതയായി) രേഖപ്പെടുത്തിയിരിക്കുന്നു (6:12) വെന്നും അവന്‍ പറയുന്നു. സൂറത്തുല്‍ അന്‍ആമിലെ 54-ാമത്തെ വചനത്തിലും ഇക്കാര്യം തന്നെ സൂചിപ്പിക്കുന്നുണ്ട്. സ്വന്തത്തെയും തന്റെ ചുറ്റുപാടുകളെയും കുറിച്ച് പഠിക്കുന്ന മനുഷ്യന് ദിവ്യ കാരുണ്യത്തെകുറിച്ച് വ്യക്തമായി മനസ്സിലാക്കാന്‍കഴിയും. മാതാവിന്റെയും പിതാവിന്റെയും ശരീരത്തിലെ രണ്ട് അര്‍ധ കോശങ്ങളെ കൂട്ടിയോജിപ്പിച്ച് സിക്താണ്ഡമാകുന്നത് മുതല്‍ ഭ്രൂണത്തിന്റെ വളര്‍ച്ചയുടെ ഓരോ ഘട്ടത്തിലും അതിനു നല്‍കുന്ന പരിരക്ഷവരെയുള്ള കാര്യങ്ങളിലും ശേഷമുള്ള കാര്യങ്ങളിലും ശേഷമുള്ള ജനനത്തിലും വളര്‍ച്ചയിലുമെല്ലാം അല്ലാഹുവിന്റെ അപാരമായ കാരുണ്യമാണ് നമുക്ക് കാണാന്‍ കഴിയുന്നത്. പടച്ചതമ്പുരാന്റെ ഈ കാരുണ്യത്തിന്റെ ഭാഗമായിട്ടാണ് മനുഷ്യര്‍ക്ക് വിശേഷബുദ്ധിയും ചിന്താസ്വാതന്ത്ര്യവും നല്‍കിയത്. സ്വതന്ത്രമായ കൈകാര്യകര്‍ത്തൃത്വത്തിന് കഴിയുന്ന ഏക ജീവിയാണല്ലോ മനുഷ്യന്‍. ചിന്തിച്ച് കാര്യങ്ങളുടെ നിജസ്ഥിതി മനസ്സിലാക്കി പ്രവര്‍ത്തിക്കുവാന്‍ കഴിയുന്ന അവന്ന് നന്മ ചെയ്ത് ഉല്‍കൃഷ്ടനാകുവാനും തിന്മ ചെയ്ത് നികൃഷ്ടനാകാനും സാധിക്കും. നന്മയുടെ മാര്‍ഗം തിരഞ്ഞെടുത്ത് ഉത്തമ മനുഷ്യരായി ജീവിക്കണമെന്നതാണ് ദൈവിക ശാസന. ദൈവികമായ വിധിവിലക്കുകള്‍ അനുസരിച്ച് ജീവിക്കുമ്പോള്‍ മാത്രമേ മനുഷ്യസമൂഹം ഭദ്രമായി നിലനില്‍ക്കുകയുള്ളൂ. ധാര്‍മിക നിയമങ്ങള്‍ അനുസരിക്കപ്പെടുന്ന സമൂഹത്തില്‍ മാത്രമേ സന്തോഷവും സമാധാനവും നിലനില്‍ക്കൂ. അപ്പോള്‍, ധാര്‍മ്മിക നിയമങ്ങള്‍ അനുസരിക്കേണ്ടത് മനുഷ്യരുടെയെല്ലാം ബാധ്യതയാണ്. പക്ഷേ, ഈ നിയമങ്ങള്‍ അനുസരിക്കാതിരിക്കുവാനുള്ള പൈശാചികമായ ഒരു തൃഷ്ണ അകത്തളത്തിലുണ്ട്. ഈ തൃഷ്ണക്ക് കടിഞ്ഞാണിട്ടാല്‍ മാത്രമെ ആഹ്ലാദഭരിതവും സമാധാന സമ്പുഷ്ഠവുമായ മനുഷ്യ സമൂഹത്തിന്റെ സൃഷ്ടി സാധിക്കൂ. ചിന്താശേഷിയുള്ള മനുഷ്യരെ സൃഷ്ടിച്ച അല്ലാഹുവിന്റെ കാരുണ്യത്തിന്റെ ഭാഗം തന്നെയാണ് ഉല്‍കൃഷ്ടമായ ഒരു സമൂഹത്തിന്റെ സൃഷ്ടിക്കുവേണ്ട കാര്യങ്ങള്‍ ചെയ്യുകയെന്നത്. അതുകൊണ്ടാണ് പരമകാരുണികനായ അല്ലാഹു ഈ ഭൂമിയില്‍ നന്മ ചെയ്തവര്‍ക്ക് മരണാനന്തരം നന്മയും തിന്മ ചെയ്തവര്‍ക്ക് പരലോകത്ത് തിന്മയും പ്രതിഫലമായി ലഭിക്കുമെന്ന നിയമമുണ്ടാക്കിയത്. നന്മയെയും, തിന്മയെയും, സത്യത്തെയും, അസത്യത്തെയും, നീതിയെയും അനീതിയെയുമെല്ലാം അല്ലാഹുവിന്റെ കാരുണ്യം തുല്യമായാണ് ഗണിക്കുന്നതെങ്കില്‍ പിന്നെ സത്യത്തിനും നന്മക്കും നീതിക്കുമെന്തു വില? അതു ശരിയല്ല നന്മക്കും തിന്മക്കും അവക്കനുസൃതമായ പ്രതിഫലം നല്‍കണം. ഇതാണ് അല്ലാഹുവിന്റെ നീതി. പരലോകത്ത് വെച്ച് ഇഹലോകത്തിലെ ചെയ്തികള്‍ക്ക് പ്രതിഫലം നല്‍കപ്പെടും. തെറ്റു ചെയ്ത വ്യക്തികള്‍ക്ക്മരണാനന്തരം ലഭിക്കുന്ന കഠിനമായ ശിക്ഷപോലും മാനവരാശിയോടുള്ള അല്ലാഹുവിന്റെ കാരുണ്യത്തിന്റെയും സ്‌നേഹത്തിന്റെയും ഭാഗമാണെന്ന് അല്‍പം ചിന്തിച്ചാല്‍ ബോധ്യമാകും. ഓരോരുത്തരും ചെയ്ത നന്മകള്‍ക്ക് പരലോകത്ത് വെച്ച് തക്കതായ പ്രതിഫലവും തിന്മകള്‍ക്ക് ശിക്ഷയും ലഭിക്കുമെന്നാണ് ഇസ്‌ലാം പഠിപ്പിക്കുന്നത്. അവനവന്‍ ചെയ്ത നന്മതിന്മകള്‍ക്ക് അവന്‍ തന്നെയാണ് ഉത്തരവാദി. നന്മ തെരഞ്ഞെടുക്കുവാനും തിന്മ തെരെഞ്ഞടുക്കുവാനും ഓരോരുത്തര്‍ക്കും സ്വാതന്ത്ര്യമുണ്ട്. ഈ സ്വാതന്ത്ര്യം ദൈവികശാസനകള്‍ക്കനുസൃതമായി വിനിയോഗിക്കപ്പെടുമ്പോഴാണ് ഒരാള്‍ സ്വര്‍ഗ്ഗാവകാശിയായി തീരുന്നത്. മറിച്ചാകുമ്പോള്‍ നരകാവകാശിയും. (3:57,2:39,2:286,73:19 എന്നിവ നോക്കുക). സന്മാര്‍ഗപ്രാപ്തി ഒരു ദൈവിക ദാനമാണ്. അത് ലഭിക്കുന്നത് മനുഷ്യപ്രയത്‌നം കൊണ്ടാണെന്ന് മാത്രം. തുറന്ന മനസ്സും സത്യം കണ്ടെത്തണമെന്ന ആഗ്രഹവും വെച്ചുകൊണ്ട് സന്മാര്‍ഗത്തിലെത്താന്‍ കഠിനാധ്വാനം ചെയ്യുന്നവര്‍ക്ക് അല്ലാഹു നല്‍കുന്ന ദാനമാണ് സന്മാര്‍ഗ പ്രാപ്തി. അതുനല്‍കുവാന്‍ ഒരാള്‍ക്കും കഴിയില്ല. ഒരാളെയും സന്മാര്‍ഗത്തിലെത്തിക്കുവാന്‍ മറ്റൊരാള്‍ക്കാവില്ല. സല്‍പന്‍ഥാവിനെ കുറിച്ച് ഉപദേശിച്ചുകൊടുക്കുക മാത്രമാണ് ഇവ്വിഷയകമായി മനുഷ്യര്‍ക്ക് പരസ്പരം ചെയ്യാന്‍ കഴിയുന്നകാര്യം. സത്യമാര്‍ഗത്തിലെത്തിക്കുന്നത് അല്ലാഹു മാത്രമാണ്. പാശ്ചാതപിച്ചു മടങ്ങിയവരെ തന്റെ മാര്‍ഗത്തിലേക്കവന്‍ നയിക്കുന്നു. (13:27). തീര്‍ച്ചയായും വിശ്വസിക്കുകയും സല്‍കര്‍മങ്ങള്‍ പ്രവര്‍ത്തിക്കുകയും ചെയ്തവരാരോ അവരുടെ വിശ്വാസത്തിന്റെ ഫലമായി അവരുടെ രക്ഷിതാവ് അവരെ നേര്‍വഴിയിലാക്കുന്നതാണ്. (10:9). സത്യനിഷേധത്തിന്റെ കാര്യവും ഇതേപോലെ തന്നെയാണ്. അഹങ്കാരംമൂലം ദൈവശാസനകള്‍ ലംഘിച്ചു ജീവിക്കുകയും ദൈവം നല്‍കിയ ഇന്ദ്രിയങ്ങള്‍ സത്യാന്വേഷണത്തിന് ഉപയോഗിക്കാതിരിക്കുകയും ചെയ്യുന്നവരെ ദുര്‍മാര്‍ഗത്തിലാക്കുന്നതും അല്ലാഹുതന്നെയാണ്. മനുഷ്യരുടെ കര്‍മങ്ങള്‍ ദുഷിക്കുകയും വിശ്വാസം പിഴക്കുകയും ചെയ്യുമ്പോള്‍ ദൈവവിധിപ്രകാരം അവര്‍ ദുര്‍മാര്‍ഗത്തില്‍ ചെന്ന് ചാടുന്നു. ദുര്‍മാര്‍ഗത്തില്‍നിന്ന് ഒരാളെ രക്ഷിക്കാന്‍ ആരു വിചാരിച്ചാലും കഴിയില്ല; അല്ലാഹുവാണ് ആളുകളെ ദുര്‍മാര്‍ഗത്തിലാക്കുന്നത്. അങ്ങനെ ആക്കുന്നതിനു കാരണം അവരുടെ കര്‍മങ്ങളാണെന്നു മാത്രം. അവിശ്വസിക്കുകയും അന്യായം പ്രവര്‍ത്തിക്കുകയും ചെയ്തവരാരോ അവര്‍ക്ക് അല്ലാഹു ഒരിക്കലും പൊറുത്തു കൊടുക്കുന്നതല്ല. നരകത്തിന്റെ മാര്‍ഗത്തിലേക്കല്ലാതെ മറ്റൊരു മാര്‍ഗത്തിലേക്കും അവന്‍ അവരെനയിക്കുന്നതുമല്ല. (4:168). താന്‍ ഉദ്ദേശിക്കുന്നവരെ അല്ലാഹു ദുര്‍മാര്‍ഗത്തിലാക്കുകയും താനുദ്ദേശിക്കുന്നവരെ നേര്‍വഴിയിലാക്കുകയും ചെയ്യുന്നു (14:4) വെന്ന ഖുര്‍ആന്‍ വചനത്തിനര്‍ഥം അല്ലാഹു യാതൊരു മാനദണ്ഡവുമില്ലാതെ കുറേയാളുകളെ സന്മാര്‍ഗത്തിലും വേറെ കുറേപേരെദുര്‍മാര്‍ഗത്തിലുമാക്കും എന്നല്ല. സന്മാര്‍ഗത്തിന്റെയും ദുര്‍മാര്‍ഗത്തിന്റെയും വിഷയത്തിലെ അല്ലാഹുവിന്റെ ഉദ്ദേശ്യം മനുഷ്യരുടെ കര്‍മങ്ങളുടെ അടിസ്ഥാനത്തിലായിരിക്കുമെന്ന് മറ്റുസൂക്തങ്ങളില്‍ ഖുര്‍ആന്‍ വ്യക്തമാക്കുന്നുണ്ട്. സന്‍മാര്‍ഗത്തിലെത്തുവാനുള്ളആത്മാര്‍ത്ഥമായ അഭിവാഞ്ഛയും പശ്ചാത്താപ വിവശമായ ഹൃദയവും തുറന്ന മനസ്സുമുള്ളവരെയാണ് അല്ലാഹു സന്‍മാര്‍ഗത്തിലാക്കുന്നത്. സത്യാന്വേഷണ തൃഷ്ണയില്ലാതിരിക്കുകയും അഹങ്കാരം മൂലം ദൈവശാസനകള്‍ ലംഘിച്ചു ജീവിക്കുകയും ചെയ്യുന്നവരെയാണ് അല്ലാഹു ദുര്‍മാര്‍ഗത്തിലെത്തിക്കുന്നത്. ഓരോരുത്തരും ആഭിമുഖ്യം കാണിക്കുന്ന മാര്‍ഗത്തിലേക്കാണ് അല്ലാഹു അവരെ നയിക്കുകയെ ന്ന വസ്തുതയും ഖുര്‍ആന്‍ വ്യക്തമാക്കുന്നുണ്ട്. എന്നാല്‍ ഏതൊരാള്‍ ദാനം നല്‍കുകയും സൂക്ഷ്മത പാലിക്കുകയും ഏറ്റവും ഉത്തമമായതിനെ സത്യപ്പെടുത്തുകയും ചെയ്തുവോ അവന് നാം ഏറ്റവും എളുപ്പമുള്ളതിലേക്ക് സൗകര്യപ്പെടുത്തിക്കൊടുക്കുന്നതാണ്. എന്നാല്‍ ആര്‍ പിശുക്ക് കാണിക്കുകയും സ്വയം പര്യാപ്തത നടിക്കുകയും ഏറ്റവും ഉത്തമമായതിനെ നിഷേധിച്ചു തള്ളുകയും ചെയ്യുന്നുവോ അവന്ന് നാം ഏറ്റവും ഞെരുക്കമുള്ളതിലേക്ക് സൗകര്യമൊരുക്കി കൊടുക്കുന്നതാണ് (92:5-10). നന്മയും തിന്മയും പ്രവര്‍ത്തിക്കുവാന്‍ മനുഷ്യര്‍ക്ക് സ്വാതന്ത്ര്യം നല്‍കപ്പെട്ടതിനാലാണ്, സന്‍മാര്‍ഗത്തിനും ദുര്‍മാര്‍ഗത്തിനും നിമിത്തമായിതീരുന്ന പ്രവര്‍ത്തനങ്ങള്‍ ചെയ്യാന്‍ അവര്‍ക്ക് സാധിക്കുന്നത്. മറ്റു ജീവികളെപോലെ നന്മ തിന്മകള്‍ വിവേചിച്ചറിയുവാനുള്ള കഴിവ് മനുഷ്യര്‍ക്ക് നല്‍കപ്പെട്ടിട്ടില്ലായിരുന്നുവെങ്കില്‍ സന്‍മാര്‍ഗത്തിന്റെയും ദുര്‍മാര്‍ഗത്തിന്റെയും പ്രശ്‌നം തന്നെ ഉല്‍ഭവിക്കുകയില്ലായിരുന്നു. അല്ലാഹു ഉദ്ദേശിച്ചിരുന്നെങ്കില്‍ മനുഷ്യരെയും മറ്റു ജീവികളെപ്പോലെ ജന്മവാസനകള്‍ക്കനുസരിച്ചു മാത്രം ജീവിക്കുന്നവരാക്കിത്തീര്‍ക്കാമായിരുന്നു. അവരെയെല്ലാം വിശ്വാസികളാക്കി തീര്‍ക്കാമായിരുന്നു. എന്നാല്‍ മറ്റു സൃഷ്ടികളില്‍ നിന്ന് തികച്ചും വ്യതിരിക്തമായ ഒരു അസ്തിത്വമാണ് മനുഷ്യന് നല്‍കപ്പെട്ടിരിക്കുന്നത്. നന്മയും തിന്മയും സ്വപ്രയത്‌നം കൊണ്ട് വ്യവഛേദിച്ചു മനസ്സിലാക്കുവാന്‍ കഴിയുന്ന ഏക ജീവിയാണവന്‍. പ്രസ്തുത സവിശേഷത തന്നെ എടുത്തുകളഞ്ഞു കൊണ്ട് ജന്മനാ തന്നെ മനുഷ്യരെ വിശ്വാസികളായി സൃഷ്ടിക്കാന്‍ അല്ലാഹുവിന് സാധിക്കുമായിരുന്നു. ഇക്കാര്യമാണ്, അല്ലാഹു ഉദ്ദേശിച്ചിരുന്നുവെങ്കില്‍ അവരെയൊക്കെ അവന്‍ സന്മാര്‍ഗത്തില്‍ ഒരുമിച്ചുകൂട്ടുക തന്നെ ചെയ്യുമായിരുന്നു (6:35) വെന്ന് പറയുമ്പോള്‍ അര്‍ഥമാക്കുന്നത്. അല്ലാഹുവിന്റെ കാരുണ്യത്തെ ക്കുറിച്ച് പ്രതിപാദിക്കുന്ന സൂക്തങ്ങള്‍ മാര്‍ഗദര്‍ശനം നല്‍കുവാനുള്ള അവന്റെ പരമാധികാരം പ്രഖ്യാപിക്കുന്ന സൂക്തങ്ങളുമായോ മനുഷ്യ സമൂഹത്തിലെ വിശ്വാസ വൈവിധ്യത്തെ അംഗീകരിക്കുന്ന സൂക്തങ്ങളുമായോ യാതൊരു വിധ വൈരുധ്യങ്ങളും പുലര്‍ത്തുന്നില്ലെന്നതാണ് വാസ്തവം.

മനുഷ്യന് എന്തിൽ നിന്നാണ് സൃഷ്ടിക്കപ്പെട്ടിട്ടുള്ളത്? രക്തക്കട്ടയിൽ നിന്നാണെന്നും (96:2) വെള്ളത്തിൽ നിന്നാണെന്നും (21: 30, 24: 45, 25: 54) ശബ്ദമുണ്ടാക്കുന്ന കളിമണ്ണിൽ നിന്നാണെന്നും(15:26) മണ്ണിൽ നിന്നാണെന്നും (3:59,30:20,35:11) ഭൂമിയിൽ നിന്നാണെന്നും (11:61) ശുക്ലത്തിൽ നിന്നാണെന്നു (16:4,75:37)മെല്ലാം ഖുര്‍ആനിൽ പറയുന്നുണ്ടല്ലോ. ഇത് വ്യക്തമായ വൈരുധ്യമല്ലേ?

മനുഷ്യന്റെ സൃഷ്ടിയെകുറിച്ച് പ്രതിപാദിക്കുമ്പോള്‍ ഖുര്‍ആന്‍ നടത്തുന്ന പ്രസ്താവനകളില്‍ വൈവിധ്യം കാണപ്പെടുന്നുവെന്നത് ഒരു നേരാണ്. ഈ വൈവിധ്യങ്ങള്‍ പക്ഷേ വൈരുധ്യങ്ങളല്ല. മനുഷ്യ സൃഷ്ടിയെപ്പറ്റി ഖുര്‍ആനില്‍ പരാമര്‍ശിക്കപ്പെട്ട എല്ലാം ശരിയാണെന്നാണ് പഠനങ്ങള്‍ വ്യക്തമാക്കുന്നത്.

ഖുര്‍ആനില്‍ മനുഷ്യ സൃഷ്ടിയെ കുറിച്ച് പറയുന്ന സൂക്തങ്ങള്‍ രണ്ടുതരത്തിലുള്ളവയാണ്. മനുഷ്യന്റെ ആദിമ സൃഷ്ടിയെ കുറിച്ച് പ്രതിപാദിക്കുന്നവയാണ് ഒന്ന്. രണ്ടാമത്തെതാകട്ടെ, സ്ത്രീപുരുഷ സംഗമത്തിലൂടെയുള്ള പ്രത്യുല്‍പാദനത്തെ സൂചിപ്പിക്കുന്നവയാണ്. രണ്ടും കൂട്ടിക്കലര്‍ത്തി കൊണ്ടാണ് പലപ്പോഴും വൈരുധ്യങ്ങള്‍ ആരോപിക്കപ്പെടുന്നത്. ഇവയെ രണ്ടായി കണ്ടുകൊണ്ട് തന്നെ പഠനത്തിന്വിധേയമാക്കിയാല്‍ ഇവയില്‍ യാതൊരു വിധ വൈരുധ്യങ്ങളുമില്ലെന്ന് മാത്രമല്ല, ഇവയെല്ലാം കൃത്യവും ശാസ്ത്രീയവുമാണെന്ന് വ്യക്തമാവും.

ആദിമ മനുഷ്യന്റെ സൃഷ്ടിയെ കുറിച്ച് പ്രതിപാദിക്കുന്ന സൂക്തങ്ങള്‍താഴെ:

കറുത്ത ചെളി പാകപ്പെടുത്തിയുണ്ടാക്കിയ (മുട്ടിയാല്‍)മുഴക്കമുണ്ടാകുന്ന കളിമണ്‍ രൂപത്തില്‍ നിന്ന് നാം മനുഷ്യരെസൃഷ്ടിച്ചിരിക്കുന്നു-(15:26)

അവനെ(ആദമിനെ) മണ്ണില്‍ നിന്നും അവന്‍ സൃഷ്ടിച്ചു. പിന്നീട് അതിനോട് ഉണ്ടാകൂ എന്ന് പറഞ്ഞപ്പോള്‍ അവന്‍ (ആദം) അതാ ഉണ്ടാകുന്നു -(3:59)

നിങ്ങളെ അവന്‍ മണ്ണില്‍ നിന്ന് സൃഷ്ടിച്ചു-(30:20)

അല്ലാഹു നിങ്ങളെ മണ്ണില്‍ നിന്നും പിന്നീട് ബീജകണത്തില്‍ നിന്നുംസൃഷ്ടിച്ചു-(35:11)

അവനത്രേ കളി മണ്ണില്‍ നിന്നും നിങ്ങളെ സൃഷ്ടിച്ചത്-(6:2)

എല്ലാ ജന്തുക്കളെയും അവന്‍ വെള്ളത്തില്‍ നിന്ന് സൃഷ്ടിച്ചിരിക്കുന്നു-(24:45)

അവന്‍ തന്നെയാണ് വെള്ളത്തില്‍നിന്ന് മനുഷ്യനെ സൃഷ്ടിക്കുകയും, അവനെ രക്തബന്ധമുള്ളവനും വിവാഹബന്ധമുള്ളവനും ആക്കുകയും ചെയ്തിരിക്കുന്നത്. (25:54)

ആദി മനുഷ്യന്‍ സൃഷ്ടിക്കപ്പെട്ടത് മണ്ണില്‍ നിന്നാണെന്നാണ് ഈ സൂക്തങ്ങളില്‍ നിന്ന് നാം ഒന്നാമതായി മനസ്സിലാക്കേണ്ടത്. മണ്ണില്‍ നിന്ന് എന്നപ്രയോഗം പൊതുവായ ഒരു പരാമര്‍ശമാണ്. തുറാബ് എന്നാണ് മണ്ണിന് അറബിയില്‍ പറയുക. ഏതു തരം മണ്ണിനും പറയാവുന്ന ഒരു പൊതുനാമമാണിത്. ഏതുതരം മണ്ണില്‍ നിന്നാണ് മനുഷ്യ സൃഷ്ടി നടന്നത് എന്ന ചോദ്യത്തിന് കളിമണ്ണില്‍ (ഹമഅ്,ത്വീന്‍) നിന്ന് എന്ന ഉത്തരമാണ് ഖുര്‍ആന്‍ നല്‍കുന്നത്. ഏതുതരം കളിമണ്ണ് എന്ന ചോദ്യത്തിനാണ് ഖുര്‍ആന്‍ സ്വല്‍സ്വാല്‍ എന്നും മസ്‌നൂന്‍ എന്നും ഉത്തരം പറയുന്നത്. കളിമണ്ണിന്റെ രണ്ട് സ്വഭാവങ്ങളാണ് ഈ നാമങ്ങളില്‍ പ്രകടമാവുന്നത്. മുട്ടിയാല്‍ ശബ്ദിക്കുന്ന മണ്ണ് എന്നും പശിമയുള്ള കുഴഞ്ഞ മണ്ണ് എന്നുമാണ് യഥാക്രമം ഈ പദങ്ങളുടെ അര്‍ത്ഥം. അഥവാ, ആദിമനുഷ്യന്‍ സൃഷ്ടിക്കപ്പെട്ടത് മണ്ണില്‍ നിന്നാണ് എന്ന പൊതു പ്രസ്താവനയുടെ വിശദീകരണങ്ങളാണ് നാം മറ്റു സൂക്തങ്ങളില്‍ കാണുന്നത്.

മനുഷ്യന്‍ ജലത്തില്‍ നിന്നാണ് സൃഷ്ടിക്കപ്പെട്ടത് എന്ന് പറയുന്ന ഖുര്‍ആന്‍ സൂക്തം(25:54) മണ്ണില്‍ നിന്നാണ് സൃഷ്ടിക്കപ്പെട്ടത് എന്നു വ്യക്തമാക്കുന്ന സൂക്തങ്ങളുമായി വൈരുധ്യം പുലര്‍ത്തുന്നില്ലേയെന്ന് ചോദിക്കാവുന്നതാണ്. ഇല്ലയെന്നു തന്നെയാണ് ഉത്തരം. മനുഷ്യനെ മണ്ണില്‍നിന്നു മാത്രമായി സൃഷ്ടിച്ചിരിക്കുന്നുവെന്നോ ജലത്തില്‍ നിന്നു മാത്രമായി പടക്കപ്പെട്ടിരിക്കുന്നുവെന്നോ ഖുര്‍ആനിലൊരിടത്തും പരാമര്‍ശമില്ല. അതുകൊണ്ട് തന്നെ ഈ വചനങ്ങള്‍ തമ്മില്‍ വൈരുധ്യമുണ്ടെന്ന് പറയാനാവില്ല. മനുഷ്യനെ വെള്ളത്തില്‍ നിന്നും മണ്ണില്‍ നിന്നുമായി വെള്ളത്തിന്റെയും മണ്ണിന്റെയും മിശ്രിതത്തില്‍ നിന്ന് സൃഷ്ടിക്കപ്പെട്ടുവെന്ന് മാത്രമെ ഈ വചനങ്ങളുടെ അടിസ്ഥാനത്തില്‍ പറയാനാകൂ. വെള്ളം ചേര്‍ത്ത് മണ്ണ് കുഴച്ച് കളിമണ്‍ രൂപമുണ്ടാക്കുക എന്നത് സ്വാഭാവികമായകാര്യമാണല്ലോ. ഇപ്രകാരമായിരിക്കും അല്ലാഹു ആദിമനുഷ്യന്റെ രൂപം നിര്‍മിച്ചത്. ആദി മനുഷ്യന്റെ രൂപം കളിമണ്ണില്‍ നിന്ന് രൂപപ്പെടുത്തിയശേഷം അല്ലാഹുവിന്റെ ആത്മാവില്‍ നിന്ന് ഊതിയപ്പോഴാണ് മനുഷ്യനുണ്ടായതെന്നത്രെ ഖുര്‍ആനില്‍ പറയുന്നത്.(15:28,29). ആദ്യത്തെ മനുഷ്യന്റെ മാത്രമല്ല എല്ലാ മനുഷ്യരുടെ ഘടനയിലും വെള്ളം ഒരുനിര്‍ണ്ണായക ഘടകം തന്നെയാണ്. അതുപോലെ തന്നെ മണ്ണിലെ മൂലകങ്ങളും.

എല്ലാ ജൈവ വസ്തുക്കളും ജലത്തില്‍ നിന്നാണ് സൃഷ്ടിക്കപ്പെട്ടത് എന്ന ഖുര്‍ആനിക പരാമര്‍ശ(24:45,21:30)ത്തിന്റെ വരുതിയില്‍ വരുന്നവനാണ് മനുഷ്യനുമെന്ന വസ്തുത വ്യക്തമാക്കുക കൂടിയാണ് സൂറത്തുല്‍ ഫുര്‍ഖാനിലെ വചനം(25:54) ചെയ്യുന്നത്. സത്യത്തില്‍ ജൈവ വസ്തുവിന്റെ അടിസ്ഥാന ഘടകം ജലമാണ്. കോശത്തിന്റെ ചൈതന്യം നിലനില്‍ക്കുന്നതു തന്നെ ജലത്തിന്റെ സാന്നിധ്യത്തെ ആശ്രയിച്ചാണ്. ഏതൊരു ജൈവ ശരീരത്തെയും വിഘടനത്തിന് വിധേയമാക്കിയാല്‍ പ്രധാനമായും ലഭിക്കുക ജലമായിരിക്കും. മനുഷ്യശരീരമെടുക്കുക അതിന്റെ മൂന്നില്‍ രണ്ടു ഭാഗവും ജലമാണ്. അസ്ഥികളില്‍ പോലും 22 ശതമാനത്തോളം ജലം അടങ്ങിയിട്ടുണ്ട്. മനുഷ്യന്‍ ജലത്തില്‍ നിന്നാണ് സൃഷ്ടിക്കപ്പെട്ടിട്ടുള്ളത് എന്ന ഖുര്‍ആനിക പരാമര്‍ശം തികച്ചും വാസ്തവമാകുന്നു. മനുഷ്യശരീരത്തില്‍ മണ്ണിലുള്ള ധാതുലവണങ്ങളും ജലവുമാണ് അടങ്ങിയിരിക്കുന്നത്. മനുഷ്യന്‍ കളിമണ്ണില്‍ നിന്നും ജലത്തില്‍ നിന്നുമാണ് സൃഷ്ടിക്കപ്പെട്ടതെന്ന ഖുര്‍ആന്‍ സൂക്തങ്ങള്‍ മനുഷ്യ സൃഷ്ടി വിശദീകരിക്കുമ്പോള്‍ വന്ന വൈരുധ്യങ്ങളല്ല, പ്രത്യുത, സൃഷ്ടിക്കുവേണ്ടി അല്ലാഹു ഉപയോഗിച്ച വസ്തുക്കളുടെ വൈവിധ്യമാണ് വ്യക്തമാക്കുന്നത്.

സ്ത്രീപുരുഷ സംഗമത്തിലൂടെയുള്ള മനുഷ്യസൃഷ്ടിയെ കുറിച്ച് പ്രതിപാദിക്കുന്ന ഖുര്‍ആന്‍ സൂക്തങ്ങള്‍ തമ്മിലും വൈരുധ്യങ്ങളൊന്നും വെച്ചു പുലര്‍ത്തുന്നതായി കാണുവാന്‍ സാധിക്കുന്നില്ല. ഏതാനും സൂക്തങ്ങള്‍ കാണുക:

അവന്‍ സ്രവിക്കപ്പെടുന്ന ശുക്ലത്തില്‍ നിന്നുള്ള ഒരു കണമായിരുന്നില്ലേ? (75:37)

മനുഷ്യനെ അവന്‍ ഒരു ബീജകണത്തില്‍ നിന്ന് സൃഷ്ടിച്ചു.(16:4)

കൂടിച്ചേര്‍ന്നുണ്ടായ ഒരു ബീജത്തില്‍ നിന്ന് നാം മനുഷ്യനെ സൃഷ്ടിച്ചു. (76:2)

മനുഷ്യനെ അവന്‍ ഭ്രൂണത്തില്‍ നിന്ന് സൃഷ്ടിച്ചിരിക്കുന്നു.(96:2)

ഈ വാക്യങ്ങളെല്ലാം തന്നെ ലൈംഗിക പ്രത്യുല്‍പാദനത്തിന്റെ വ്യത്യസ്ത ഘട്ടങ്ങളെ ക്കുറിക്കുന്നവയാണ്. സൂറത്തുല്‍ ഖിയാമയില്‍ (75:37)പറഞ്ഞിരിക്കുന്ന ശുക്ലത്തില്‍ നിന്നുള്ള കണം (നുത്വ്ഫത്തന്‍ മിന്‍ മനിയ്യ്)ബീജ സങ്കലനം നടക്കാത്ത പുംബീജത്തെ ഉദ്ദേശിച്ചുകൊണ്ടുള്ള പ്രയോഗമാണ്. സൂറത്തു ന്നഹ്‌ലില്‍ (16:4) പ്രതിപാദിക്കപ്പെട്ട ബീജകണം (നുത്വ്ഫ) എന്ന പദപ്രയോഗത്തിന്റെ വിവക്ഷയും അതുതന്നെ. ബീജ സങ്കലനം കഴിഞ്ഞശേഷമുള്ള അവസ്ഥയെകുറിക്കുന്നതാണ് സൂറത്തുല്‍ ഇന്‍സാനിലെ(76:2) കൂടിച്ചേര്‍ന്നുണ്ടായ ബീജം (നുത്വ്ഫത്തിന്‍ അംശാജിന്‍) എന്ന പ്രയോഗം. സൂറത്തുല്‍ അലഖില്‍ ഭ്രൂണമെന്ന് (അലഖ്) പറഞ്ഞിരിക്കുന്നത് ബീജസങ്കലനത്തിന് ശേഷമുള്ള സിക്താണ്ഡത്തെ കുറിക്കുന്നതിനാണ്. അലഖ് എന്നഅറബി പദത്തിനര്‍ത്ഥം പറ്റിപിടിക്കുന്നത് എന്നാണ്. ജന്തുശരീരത്തില്‍ അള്ളിപിടിക്കുന്നതിനാല്‍, അട്ട എന്ന ജീവിക്ക് ‘അലഖ്‘ എന്നുപറയാറുണ്ട്. ബീജസങ്കലനത്തിനു ശേഷമുണ്ടാകുന്ന സിക്താണ്ഡം ഗര്‍ഭാശയഭിത്തിയില്‍ അള്ളിപിടിച്ചാണ് വളരാനാരംഭിക്കുന്നത്. ഈ അവസ്ഥയിലുള്ള ഭ്രൂണത്തിന്റെ രൂപം അട്ടയുടേതിന് സമാനവുമാണ്. (അലഖ് എന്ന പദത്തിനാണ് മുന്‍കാലത്തെ ഖുര്‍ആന്‍ വ്യാഖ്യാതാക്കള്‍ അവരുടെ കാലഘട്ടത്തിലെ ധാരണ പ്രകാരം രക്തക്കട്ടയെന്ന് അര്‍ത്ഥം പറഞ്ഞിരിക്കുന്നത്). ഭ്രൂണവളര്‍ച്ചയുടെ വ്യത്യസ്ത ഘട്ടങ്ങള്‍ പ്രതിപാദിക്കുന്ന ഖുര്‍ആന്‍ സൂക്തങ്ങളെല്ലാം കൃത്യവും സൂക്ഷ്മവുമായ വിവരങ്ങളാണ് നല്‍കുന്നത് എന്ന വസ്തുതയാണ് നമുക്ക് ഇവിടെ കാണാന്‍ കഴിയുന്നത്. മനുഷ്യന്റെ ആദി സൃഷ്ടിയും ബീജസങ്കലനവും ഭ്രൂണവളര്‍ച്ചയുടെ വ്യത്യസ്ത ഘട്ടങ്ങളും പ്രതിപാദിക്കുന്ന സൂറത്തുല്‍ മുഅ്മിനൂനിലെ സൂക്തങ്ങളില്‍ (23:12-14) ഇക്കാര്യം കുറേകൂടി വ്യക്തമായി കാണാനാവും: തീര്‍ച്ചയായും മനുഷ്യനെ കളിമണ്ണിന്റെ സത്തയില്‍ നിന്ന് നാംസൃഷ്ടിച്ചിരിക്കുന്നു. പിന്നീട് ഒരു ബീജ(നുത്വ്ഫ)മായി കൊണ്ട് അവനെ നാംഭദ്രമായ ഒരുസ്ഥാനത്ത് വെച്ചു. പിന്നെ ആ ബീജത്തെ നാം ഒരു ഭ്രൂണമായി(അലഖ) രൂപപ്പെടുത്തി. അനന്തരം ആ ഭ്രൂണത്തെ നാം ഒരു മാംസപിണ്ഡമായി (മുള്ഗഃ) രൂപപ്പെടുത്തി. തുടര്‍ന്ന് നാം ആ മാംസപിണ്ഡത്തെ അസ്ഥികൂട(ഇളാം)മായി രൂപപ്പെടുത്തി. എന്നിട്ട് നാം അസ്ഥികൂടത്തെ മാംസം(ലഹ്മ്) കൊണ്ട് പൊതിഞ്ഞു. പിന്നീട് മറ്റൊരു സൃഷ്ടിയായി നാം അവനെ വളര്‍ത്തിയെടുത്തു. അപ്പോള്‍ ഏറ്റവും നല്ല സൃഷ്ടികര്‍ത്താവായ അല്ലാഹു അനുഗ്രഹപൂര്‍ണനായിരിക്കുന്നു.’’

വിഷയാവതരണത്തില്‍ ഖുര്‍ആന്‍ സ്വീകരിച്ചിരിക്കുന്ന വൈവിധ്യത്തിനുള്ള ഏറ്റവും നല്ല ഉദാഹരണമാണ് മനുഷ്യ സൃഷ്ടിയെ കുറിച്ച് ഖുര്‍ആന്‍ നടത്തുന്ന പരാമര്‍ശങ്ങള്‍. ശുക്ലകണത്തില്‍ നിന്നാണ് മനുഷ്യ സൃഷ്ടിനടന്നത് എന്ന പരാമര്‍ശവും കൂടിച്ചേര്‍ന്നുണ്ടായ ഭ്രൂണമാണ് മനുഷ്യശിശുവായി മാറുന്നത് എന്ന പ്രസ്താവനയും ഒരേ പോലെ ശരിയാണ്. നുത്വ്ഫ, അലഖ, മുള്ഗ, ഇളാം തുടങ്ങി ബീജസങ്കലനത്തെയും ഭ്രൂണവളര്‍ച്ചയുടെ വിവിധ ഘട്ടങ്ങളെയും കുറിക്കാന്‍ വേണ്ടി ഖുര്‍ആന്‍ ഉപയോഗിച്ച പദപ്രയോഗങ്ങളെല്ലാം ശരിയാണെന്ന് ഭ്രൂണശാസ്ത്ര പഠനങ്ങള്‍ വ്യക്തമാക്കുന്നു. ഇതേ പോലെ തന്നെയാണ് മനുഷ്യരുടെ ആദിമ സൃഷ്ടിനടന്നത് മണ്ണില്‍ നിന്നാണെന്നും ഭൂമിയില്‍ നിന്നാണെന്നും കളിമണ്ണില്‍ നിന്നാണെന്നും വെള്ളത്തില്‍ നിന്നാണെന്നുമെല്ലാം ഉള്ള പരാമര്‍ശങ്ങള്‍. ഇവയെല്ലാം ശരിയാണ്. ഇവയില്‍ യാതൊരു വൈരുധ്യവുമില്ല. ഒരേകാര്യത്തിന്റെ വ്യത്യസ്ത മാനങ്ങള്‍ പ്രതിപാദിക്കുകയെന്ന വൈവിധ്യത്തിന്റെ രീതി സ്വീകരിച്ചുകൊണ്ട് മനുഷ്യന് അസ്തിത്വ ബോധംപ്രദാനം ചെയ്യുകയാണ് ഈ സൂക്തങ്ങളെല്ലാം ചെയ്യുന്നത്.

തിന്മകളെല്ലാം ചെകുത്താനിൽ നിന്നാണ് ഉണ്ടാകുന്നതെന്ന് 38:41 ലും നമ്മിൽ നിന്നു തന്നെയാണെന്ന് 4:79ലും, അല്ലാഹുവിൽ നിന്നാണെന്ന് 4:78 ലും പറയുന്നു. ഇതെല്ലാം ഒരേപോലെ ശരിയാവുന്നതെങ്ങിനെ? നന്മയുടെ ഫലം നന്മയും തിന്മയുടെ ഫലം തിന്മയുമായിരിക്കും. ഈവ്യവസ്ഥ നിശ്ചയിച്ചിരിക്കുന്നത് അല്ലാഹുവാണ്. നല്ല ഭക്ഷണങ്ങളുപയോഗിക്കുകയും സദ്‌വൃത്തരായി ജീവിക്കുകയും ചെയ്യുന്നവര്‍ പൊതുവെ അരോഗദൃഢഗാത്രരായിരിക്കും. മദ്യപാനവും അധാര്‍മ്മികവൃത്തികളും ജീവിതചര്യയാക്കിയവര്‍ ദു:ഖങ്ങളിലും ദുരിതങ്ങളിലും പ്രയാസപ്പെടേണ്ടിവരും. ഇത് കര്‍മ്മഫലങ്ങളെ കുറിച്ചദൈവിക വിധിയാണ്. ഈ വിധിക്കനുസൃതമായാണ് കാര്യങ്ങള്‍ നടന്നുകൊണ്ടിരിക്കുന്നത്. ഒരാള്‍ അരോഗദൃഢഗാത്രനാകുന്നത് ദൈവ വിധിപ്രകാരമാണ്. പക്ഷേ, ഈ ദൈവ വിധികള്‍ മനുഷ്യര്‍ക്ക് ബാധകമാകുന്നത് അവരുടെ പ്രവര്‍ത്തനങ്ങളുമായി ബന്ധപ്പെട്ടാണ്. നന്മയുടെയും തിന്മയുടെയും അടിസ്ഥാനപരമായ ഉറവിടം അല്ലാഹുവാണെന്ന വസ്തുതയാണ് സൂറത്തുന്നിസാഇലെ 78ാം സൂക്തം വ്യക്തമാക്കുന്നത്. അവര്‍ക്ക് വല്ലനേട്ടവും വന്നുകിട്ടിയാല്‍ അവര്‍പറയും: ഇത് അല്ലാഹുവിങ്കല്‍ നിന്ന് ലഭിച്ചതാണ് എന്ന്. അവര്‍ക്ക് വല്ല ദോഷവും വന്നുഭവിച്ചാല്‍ അവര്‍ പറയും: അത് നീ കാരണം ഉണ്ടായതാണെന്ന്. പറയുക: എല്ലാം അല്ലാഹുവിന്റെ പക്കല്‍ നിന്നുള്ളതാണ്. അപ്പോള്‍ ഈ ആളുകള്‍ക്ക് എന്തുപറ്റി? അവര്‍ ഒരു വിഷയവും മനസ്സിലാക്കാന്‍ ഭാവമില്ല. (4:78) നന്മതിന്മകളുടെ അടിസ്ഥാനപരമായ കാരണക്കാരന്‍ അല്ലാഹുതന്നെയാണെന്ന് വ്യക്തമാക്കുകയാണ് ഈ ഖുര്‍ആന്‍ സൂക്തം ചെയ്യുന്നത്. എന്നാല്‍ അടിസ്ഥാനപരമായി അല്ലാഹുവിന്റെ സൃഷ്ടികളെല്ലാം നന്മയാകുന്നു. രോഗാണുക്കള്‍പോലും ആത്യന്തികമായി മനുഷ്യരുടെ നന്മക്ക് വേണ്ടി സൃഷ്ടിക്കപ്പെട്ടതാണെന്ന വസ്തുതയാണ് പുതിയ പഠനങ്ങള്‍ വ്യക്തമാക്കുന്നത്. നന്മയായി ഭവിക്കാവുന്ന ദൈവിക സൃഷ്ടികളെ തിന്മയാക്കിത്തീര്‍ക്കുന്നത് മനുഷ്യരുടെ പ്രവര്‍ത്തനങ്ങള്‍ തന്നെയാണ്. ലൈംഗിക ശേഷിയും അവയവങ്ങളും നല്ലൊരു ഉദാഹരണമാണ്. മനുഷ്യരുടെ സമാധാനപൂര്‍ണമായ ജീവിതത്തിനും സംതൃപ്തമായ കുടുംബാന്തരീക്ഷത്തിനും കെട്ടുറപ്പുള്ള സാമൂഹ്യ ജീവിതത്തിനുമെല്ലാം നിമിത്തമാവുന്ന രീതിയിലാണ് ലൈംഗികാനന്ദം സൃഷ്ടിക്കപ്പെട്ടിരിക്കുന്നത്. ലൈംഗികാവയവങ്ങളെ ദൈവിക മാര്‍ഗനിര്‍ദേശത്തിന്റെ അടിസ്ഥാനത്തില്‍ ഉപയോഗപെടുത്തിയാല്‍ അത് വ്യക്തിക്കും കുടുംബത്തിനും സമൂഹത്തിനുമെല്ലാം നന്മ മാത്രമേ വരുത്തൂ. എന്നാല്‍ പ്രസ്തുത അവയവങ്ങള്‍ തന്നെ എയ്ഡ്‌സിനും ഗൊണേറിയക്കും സിഫിലിസിനുമെല്ലാം കാരണമാക്കുന്ന രീതിയിലും ഉപയോഗിക്കുവാന്‍ മനുഷ്യന് സാധിക്കും. ലൈംഗികത കൊണ്ട് ദൈവം നിശ്ചയിച്ചിരിക്കുന്നത് നന്മയാണ്; എന്നാല്‍ അതിനെ തിന്മയാക്കി തീര്‍ക്കുവാന്‍ മനുഷ്യ പ്രവര്‍ത്തനങ്ങള്‍ക്ക് കഴിയുമെന്നര്‍ഥം. ഈ വസ്തുതയിലേക്കാണ് സൂറത്തുന്നിസാഇലെ തൊട്ടടുത്ത സൂക്തം വിരല്‍ ചൂണ്ടുന്നത്. നന്മയായിട്ട് നിനക്ക് എന്തൊന്ന് വന്ന് കിട്ടിയാലും അത് അല്ലാഹുവിങ്കല്‍ നിന്നുള്ളതാണ്. നിന്നെ ബാധിക്കുന്ന ഏതൊരു ദോഷവും നിന്റെ പക്കല്‍നിന്നു തന്നെയുണ്ടാവുന്നതാണ്.(4:79) സത്യത്തില്‍, ദൈവവിധിയും മനുഷ്യ പ്രവര്‍ത്തനങ്ങളും തമ്മില്‍ നിലനില്‍ക്കുന്ന പാരസ്പര്യത്തെ അതിസുന്ദരമായി വരച്ചുകാട്ടുകയാണ് സൂറത്തുന്നിസാഇലെ 78, 79 സൂക്തങ്ങളെന്നുള്ളതാണ് വാസ്തവം. അതു മനസ്സിലാക്കാത്തതാണ് ഈ സൂക്തങ്ങള്‍ തമ്മില്‍ വൈരുധ്യമുണ്ടെന്ന് വാദിക്കുന്നതിന് കാരണം. അല്ലാഹുവില്‍ നിന്നുണ്ടാകുന്ന കാര്യങ്ങളെല്ലാം ആത്യന്തികമായി നന്മയാണെന്നും അതാണ് മനുഷ്യ വര്‍ത്തനങ്ങളിലൂടെ നന്മയും തിന്മയുമായി തീരുന്നതെന്നുമുള്ള വസ്്തുതയാണ് ഈസൂക്തങ്ങളിലൂടെ അല്ലാഹു പഠിപ്പിക്കുന്നത്. ധൂര്‍ത്ത് മനുഷ്യരെ പാപ്പരാക്കുന്നു. ഇതു ദൈവവിധിയാണ്. മനുഷ്യ സമൂഹത്തിന്റെ മൊത്തം നിലനില്‍പ്പുമായി ബന്ധപ്പെടുത്തിനോക്കുമ്പോള്‍ വിധിയാണെന്നു പറയുന്നതില്‍ അപാകതയൊന്നുമില്ല. എന്നാല്‍ അയാള്‍ പാപ്പരാകുവാനുള്ള കാരണക്കാരന്‍ അവന്‍ തന്നെയാണ്. വിവാഹേതര ലൈംഗിക ബന്ധത്തിലേര്‍പ്പെടുന്ന പലര്‍ക്കും ഗുരുതരമായ ലൈംഗിക രോഗമുണ്ടാകുന്നു. ഈ രോഗം അതുണ്ടായ വ്യക്തിക്ക് തിന്മയായാണ് അനുഭവപ്പെടുക. എന്നാല്‍ മനുഷ്യസമൂഹത്തിന്റെ ധാര്‍മ്മികവും സദാചാരപരവുമായ പരിപ്രേക്ഷ്യത്തിലൂടെ നോക്കുമ്പോള്‍ ഈ നിയമം പോലും നന്മയാണ്. തോന്നിവാസിയായ ഒരാള്‍ക്ക് എയ്ഡ്‌സ് വരുന്നത് ദൈവിക വിധി പ്രകാരമാണ്. പ്രസ്തുത വിധി മനുഷ്യസമൂഹത്തിന്റെ കെട്ടുറപ്പിന് അനിവാര്യവുമാണ്. അതുകൊണ്ട് തന്നെ അത് നന്മയാണ്. എന്നാല്‍ അയാള്‍ക്ക് എയ്ഡ്‌സ് വരാനുള്ള കാരണക്കാരന്‍ അയാള്‍ തന്നെയാണ്. പ്രസ്തുത തിന്മ വന്നുഭവിക്കുന്നത് അയാളുടെ തന്നെ പ്രവര്‍ത്തനഫലമായാണ് എന്നര്‍ഥം. ഇക്കാര്യമാണ് ഈ സൂക്തങ്ങളിലൂടെ അല്ലാഹു നമ്മെ തെര്യപ്പെടുത്തുന്നത്. ഈ സൂക്തങ്ങള്‍ തമ്മില്‍ വൈരുധ്യമുണ്ടെന്ന് തോന്നുന്നത് ഇത് മനസ്സിലാക്കാത്തതുകൊണ്ടാണ്. ചെകുത്താനാണ് മനുഷ്യര്‍ക്ക് ദുരിതങ്ങളെല്ലാം ഉണ്ടാക്കുന്നതെന്ന് ഖുര്‍ആനില്‍ ഒരിടത്തും പറയുന്നില്ല. സൂറത്തുസ്വാദിലെ (38:41)വൈരുധ്യമാരോപിക്കപ്പെട്ടിരിക്കുന്ന സൂക്തം അത്തരമൊരു തത്വം പഠിപ്പിക്കുവാന്‍ വേണ്ടി അവതരിപ്പിക്കപ്പെട്ടതുമല്ല. പ്രവാചകനായ അയ്യൂബി(അ)ന്റെ ഒരു പ്രാര്‍ഥനയാണ് ഈ സൂക്തത്തിലുള്ളത്. പിശാച് എനിക്ക് അവശതയും പീഡനവുമേല്‍പ്പിച്ചിരിക്കുന്നു എന്ന് തന്റെര ക്ഷിതാവിനെ വിളിച്ച് അദ്ദേഹം പറഞ്ഞ സന്ദര്‍ഭം… (38:41) സ്മരിക്കുവാനാവശ്യപ്പെടുന്നതാണ് ഈ സൂക്തം. അയ്യൂബ് നബി (അക്കുണ്ടായരോഗങ്ങളെല്ലാം പിശാചിന്റെ സൃഷ്ടിയാണെന്ന് ഈ സൂക്തം പഠിപ്പിക്കുന്നില്ല. ദുരിതങ്ങള്‍ അകറ്റുവാന്‍ അല്ലാഹുവിനോട് അര്‍ഥിച്ചുകൊണ്ടുള്ള അയ്യൂബ് നബി(അ)യുടെ പ്രാര്‍ത്ഥന സൂറത്തുല്‍ അമ്പിയാഇല്‍ ഉദ്ധരിക്കുന്നുണ്ട്. അതിങ്ങനെയാണ്: എനിക്കിതാ കഷ്ടപ്പാട് ബാധിച്ചിരിക്കുന്നു. നീ കാരുണികരില്‍ വെച്ച് ഏറ്റവും കരുണയുള്ളവനാണല്ലോ. (21:83). ഈ പ്രാര്‍ഥനയിലെവിടെയും പിശാചാണ് തന്റെ ദുരിതങ്ങളുടെ കാരണക്കാരനെന്ന സൂചന പോലുമില്ല. പിന്നെ, പിശാച് എനിക്ക് അവശതയും പീഡനവുമേല്‍പിച്ചിരിക്കുന്നു(38:41) എന്ന അയൂബ്(അ)ന്റെസംസാരം എന്താണര്‍ഥമാക്കുന്നത്?. രോഗത്തിന്റെ കാഠിന്യവും സാമ്പത്തിക ഞെരുക്കങ്ങളും കുടുംബക്കാരുടെയും ബന്ധുക്കളുടെയുമെല്ലാം അവഗണനയും സൃഷ്ടിക്കുന്ന പ്രയാസങ്ങളേക്കാള്‍ ദൈവത്തെ കൈവിട്ടുകളയുവാനും അവനോട് കൃതഘ്‌നനാകുവാനും അവന്റെ കാരുണ്യത്തെ കുറിച്ച് നിരാശനാകുവാനും വേണ്ടിയുള്ള പൈശാചിക ദുര്‍ബോധനങ്ങളാണ് തനിക്ക് ദുരിതമായി തീര്‍ന്നിട്ടുള്ളതെന്നാണ് അയ്യൂബ് (അ) അല്ലാഹുവിനോട് ആവലാതിപ്പെടുന്നത്. ശാരീരികവും മാനസികവുമായ ദുരിതങ്ങളോടൊപ്പമുള്ള പിശാചിന്റെ ദുര്‍മന്ത്രണം മൂലം ആത്മാവ് അവശമായി തീരുമോ എന്ന ഭയമാണ് ആ ദൈവദാസന്റെ വചനങ്ങളില്‍ പ്രകടമായി കാണാന്‍ കഴിയുന്നത്. ഈ സൂക്തം (38:41) ഖുര്‍ആനിലെ മറ്റേതെങ്കിലും ഒരു സൂക്തവുമായി വൈരുധ്യം പുലര്‍ത്തുന്നതായി കാണാന്‍ കഴിയുന്നില്ല.
 

കനായ സ്രഷ്ടാവ് നിയോഗിച്ചയച്ച പ്രവാചകന്മാരെക്കുറിച്ച് ബൈബിളിലും ഖുര്‍ആനിലും വന്ന സമാനമായ ചരിത്രപരാമര്‍ശങ്ങളുടെ വെളിച്ചത്തില്‍ ബൈബിളില്‍നിന്ന് പകര്‍ത്തിയെഴുതിയതാണ് ഖുര്‍ആന്‍ എന്ന വാദം മിഷനറിമാരും ഓറിയന്റലിസ്റ്റുകളും ഭൗതികവാദികളുമെല്ലാമായ വിമര്‍ശകര്‍ ഒരേ സ്വരത്തില്‍ ഉന്നയിക്കാറുണ്ട്. ഈ വാദത്തില്‍ എത്രത്തോളം കഴമ്പുണ്ട്? താഴെ പറയുന്ന വസ്തുതകളുടെ വെളിച്ചത്തില്‍ ചിന്തിക്കുമ്പോള്‍ ഈ വാദം ശുദ്ധ അസംബന്ധമാണെന്ന് ബോധ്യമാകും.

ഒന്ന്) മുഹമ്മദ് നബി (സ) നിരക്ഷരനായിരുന്നു. ബൈബിള്‍ പഴയനിയമവും പുതിയനിയമവും വായിച്ചു മനസ്സിലാക്കി അതില്‍നിന്ന് പകര്‍ത്തിയെഴുതുക അദ്ദേഹത്തിന് സ്വന്തമായി അസാധ്യ മായിരുന്നു. ശിഷ്യന്മാരില്‍ ആരുടെയെങ്കിലും സഹായത്തോടെ അദ്ദേഹം അത് നിര്‍വഹിച്ചുവെന്ന് കരുതാനും വയ്യ. അങ്ങനെ ചെയ്തിരുന്നുവെങ്കില്‍ ശിഷ്യന്മാരില്‍ ചിലര്‍ക്കെങ്കിലും അത് അറിയാന്‍ കഴിയേണ്ടതായിരുന്നു. അത് മുഖേന മുഹമ്മദ് നബി (സ)യുടെ വിശ്വാസ്യതയില്‍ അവര്‍ സംശയിക്കു കയും അവര്‍ തമ്മിലുള്ള ബന്ധത്തിന് ഉലച്ചില്‍തട്ടുകയും ചെയ്യുമായിരുന്നു. മുഹമ്മദ് നബി (സ) യുടെ  ശരീരത്തില്‍ ഒരു പോറലെങ്കിലുമേല്‍ക്കുന്നതിന് പകരം സ്വന്തം ജീവന്‍ ബലിയര്‍പ്പിക്കുവാന്‍ സന്നദ്ധരായവരായിരുന്നു പ്രവാചക ശിഷ്യന്മാര്‍ എന്നോ ര്‍ക്കുക. പ്രവാചകനില്‍ (സ)ഏതെങ്കിലും തരത്തിലുള്ള അവിശ്വാസ്യ തയുണ്ടായിരുന്നുവെങ്കില്‍ ഇങ്ങനെ ത്യാഗം ചെയ്യാന്‍ സന്നദ്ധരായ ഒരു അനുയായിവൃന്ദത്തെ വളര്‍ത്തിയെടുക്കുവാന്‍ അദ്ദേഹത്തിന് കഴിയുമായിരുന്നില്ലെന്ന് തീര്‍ച്ച യാണ്.

''ഇതിന് മുമ്പ് നീ വല്ല ഗ്രന്ഥവും പാരായണം ചെയ്യുകയോ, നിന്റെ വലതുകൈകൊണ്ട് അത് എഴുതു കയോ ചെയ്തിരുന്നില്ല. അങ്ങനെയാണെങ്കില്‍ ഈ സത്യനിഷേധികള്‍ക്ക് സംശയിക്കാമായിരുന്നു'' (വി.ഖു. 29:48).

രണ്ട്) മുഹമ്മദ് നബി (സ)യുടെ ജീവിതകാലത്ത് ബൈബിള്‍ പഴയനിയമമോ പുതിയനിയമമോ അറബിയിലേക്ക് പരിഭാഷപ്പെടുത്തപ്പെട്ടിട്ടുണ്ടായിരുന്നില്ല. അറബിയിലുള്ള പഴയനിയമവും പുതിയനിയമവുമെല്ലാം ഉണ്ടായതുതന്നെ ഇസ്‌ലാമിന്റെ ദിഗ്‌വിജയങ്ങള്‍ക്ക് ശേഷമാണ്. പഴയ നിയമ രേഖകളെക്കുറിച്ച് സൂക്ഷ്മ പഠനം നടത്തിയ ഏണസ്റ്റ് വൂര്‍ഥ്‌വിന്‍ എഴുതുന്നത് കാണുക: ''ഇസ്‌ലാമിന്റെ വ്യാപനത്തോടുകൂടി അറബിയുടെ ഉപയോഗം വ്യാപകമാവുകയും ഇസ്‌ലാമിക രാജ്യങ്ങളിലെ ജൂതന്മാരുടെയും ക്രിസ്ത്യാനികളുടെയും ദൈനംദിനജീവിതത്തിലെ ഭാഷയായി അറബി മാറുകയും ചെയ്തു. ബൈബിളിന്റെ അറബി പതിപ്പുകള്‍ അനിവാര്യമാക്കി ത്തീര്‍ത്ത ഈ സാഹചര്യത്തില്‍ സ്വതന്ത്രവും പ്രാഥമികമായ വ്യാഖ്യാന സംബന്ധിയുമായ നിരവധി പതിപ്പുകള്‍ പുറത്തുവന്നു''.(Ernst Wurthewein: The Text of The Old Testament Page 104).

ഒമ്പതാം നൂറ്റാണ്ടിന്റെ ആദ്യപകുതിയിലാണ് പഴയ നിയമബൈബിള്‍ അറബിയിലേക്ക് പരിഭാഷപ്പെടുത്തപ്പെട്ടതെന്നാണ് ലഭ്യമായ കയ്യെഴുത്ത് രേഖകള്‍ വ്യക്തമാക്കുന്നത് (Ibid Page 224 -225). ഏകദേശം ഇക്കാലത്തുതന്നെയാവണം പുതിയ നിയമവും അറബിയിലേക്ക് ഭാഷാന്തരം ചെയ്യപ്പെട്ടത്. പ്രഗത്ഭനായ സിഡ്‌നി എച്ച്. ഗ്രിഫിത്തിന്റെ വരികള്‍ കാണുക:''അറബിയിലുള്ള സുവിശേഷങ്ങളടങ്ങിയ ഏറ്റവും പുരാതനമായ കയ്യെഴുത്ത് രേഖ 'സിനായ് അറബി കയ്യെഴുത്ത് പ്രതി 72' (Sinai Arabic MS72)ആണ്. ജറുസലേം സഭയുടെ ഗ്രീക്ക് പ്രാര്‍ത്ഥനാ കലണ്ടറിന്റെ കാലക്രമാടിസ്ഥാനത്തില്‍ അധ്യായങ്ങള്‍ രേഖപ്പെടുത്തിയ നാല് കാനോനിക സുവി ശേഷങ്ങളും ഇതിലുണ്ട്. രേഖയുടെ അന്ത്യത്തിലെ കുറിപ്പ് വ്യക്തമാക്കുന്നത് ഈ കയ്യെഴുത്ത് രേഖ അറബി കലണ്ടര്‍ 284ല്‍ അഥവാ ക്രിസ്താബ്ദം 897ല്‍ റംലയിലെ സ്റ്റീഫന്‍ (Stephen of Ramlah) എഴുതി യതാണെന്നാണ് (Sidney H Griffith: The Gospel in Arabic: An Enquiry Into its Appearance In the First Abbasi Century Page 132)

എന്നാല്‍ അപ്പോസ്തല പ്രവൃത്തികളും പൗലോസിന്റെ ലേഖനങ്ങളും കാതോലിക ലേഖനങ്ങളുമുള്‍ക്കൊള്ളുന്ന Sinai Arabic MS151 എന്ന കയ്യെഴുത്ത് രേഖ ഹിജ്‌റ253 ല്‍ അഥവാ ക്രിസ്താബ്ദം 867ല്‍ സുറിയാനിയില്‍ നിന്ന് അറബിയിലേക്ക് ബിസ്ര്‍ബ്‌നുസിര്‍റി എന്നയാള്‍ വിവര്‍ത്തനം ചെയ്തതായി കാണുന്നുണ്ട്. ഇതില്‍ സുവിശേഷങ്ങളില്ലെന്ന കാര്യം പ്രത്യേകം ശ്രദ്ധേയമാണ്. (Ibid Page 131).

മുഹമ്മദ് നബി (സ) ക്ക് ശേഷം രണ്ട് നൂറ്റാണ്ടുകളെങ്കിലും കഴി ഞ്ഞാണ് പുതിയനിയമവും പഴയ നിയമവുമെല്ലാം അറബിയിലേക്ക് വിവര്‍ത്തനം ചെയ്യപ്പെട്ടത്. നിരക്ഷരനായിരുന്ന മുഹമ്മദ് നബി (സ) മറ്റാരില്‍നിന്നെങ്കിലും അറബിയിലുള്ള ബൈബിള്‍ വായിച്ചുകേട്ടശേഷം അതിലെ കഥകള്‍ ഉള്‍ ക്കൊള്ളിച്ചുകൊണ്ട് എഴുതിയതാണ് ഖുര്‍ആന്‍ എന്ന വാദവും ഇവിടെ അപ്രസക്തമാവുകയാണ്. അറബിയില്‍ നിലവിലില്ലാത്ത ഒരു ഗ്രന്ഥം വായിച്ചുകേട്ടുവെന്ന് കരുതുന്നത് നിരര്‍ത്ഥകമാണെന്ന് പറയേണ്ടതില്ലല്ലോ.

മൂന്ന്) പ്രവാചകന്മാരുടെ ചരിത്രം വിവരിക്കുന്നിടത്ത് അധാര്‍മ്മികരും അസാന്മാര്‍ഗികരുമായി രുന്നു അവരെന്ന് വരുത്തിത്തീര്‍ക്കുന്ന തരത്തിലാണ് ബൈബിള്‍ അത് നിര്‍വ്വഹിച്ചിരിക്കുന്നത്. മദ്യപിച്ച് നഗ്‌നനായ നോഹും ലഹരി മൂത്ത് സ്വപുത്രിമാരുമായി ശയിച്ച ലോത്തും ചതിയനായ യാക്കോബും വിഷയലമ്പടനായ ദാവീദും മദ്യം വിളമ്പിയ യേശുവുമെല്ലാം, ധര്‍മ്മത്തിലേക്ക് ജനങ്ങളെ നയിക്കാനായി നിയോഗിക്കപ്പെട്ടവരായിരുന്നു പ്രവാചകന്മാര്‍ എന്ന സങ്കല്‍പത്തിന് കടകവിരുദ്ധമായ കഥകളാണെന്ന് പറയേണ്ടതില്ലല്ലോ. ഖുര്‍ആനിലെ ചരിത്രവിവരണത്തില്‍ ഇത്തരം യാതൊരു കഥകളും കാണുന്നില്ല. ബൈബിളില്‍നിന്ന് മുഹമ്മദ് നബി (സ) പകര്‍ത്തിയെഴു തിയതായിരുന്നു ഈ കഥകളെങ്കില്‍  പ്രവാചകന്മാരില്‍ ബൈബിള്‍ ആരോപിച്ച അധാര്‍മ്മികത കളിലേതെങ്കിലും ഖുര്‍ആനിലും സ്ഥാനം പിടിക്കേണ്ടതായിരുന്നു. അങ്ങനെയില്ലെന്ന് മാത്രമല്ല, പ്രവാചകന്മാരെല്ലാം ഉന്നതരും വിശുദ്ധരുമായിരുന്നുവെന്ന വസ്തുത വ്യക്തമാക്കുന്നതാണ് ഖുര്‍ ആനിലെ പ്രവാചക കഥനങ്ങളെല്ലാമെന്ന കാര്യം അത് ബൈബിളില്‍നിന്ന് പകര്‍ത്തിയെഴുതിയതാ ണെന്ന വാദത്തിന്റെ നട്ടെല്ലൊടിക്കുന്നുണ്ട്.

നാല്) ചരിത്രത്തിന്റെ അളവുകോലുകള്‍ വെച്ചുനോക്കുമ്പോള്‍ വസ്തുനിഷ്ഠചരിത്രത്തിന് നിര ക്കാത്ത നിരവധി പ്രസ്താവനകള്‍ ബൈബിള്‍ നടത്തുന്നുണ്ട്. ഇത് ബൈബിള്‍ പണ്ഡിതന്മാര്‍ തന്നെ അംഗീകരിക്കുന്നതാണ്. ''ചരിത്രപരമായി കൃത്യമല്ലാത്ത ചില പ്രസ്താവനകളും ബൈബിളില്‍ കണ്ടെന്നുവരാം'' (ബൈബിള്‍ വിജ്ഞാനകോശം പുറം 12). ബൈബിളില്‍നിന്ന് പകര്‍ത്തി യെഴുതി ക്കൊണ്ട് മുഹമ്മദ് നബി (സ)രചിച്ചതായിരുന്നു ഖുര്‍ആനെങ്കില്‍ അതില്‍ ബൈബിളിലേതു പോലെ ചരിത്രപരമായി കൃത്യമല്ലാത്ത പ്രസ്താവനകള്‍ കാണപ്പെടേണ്ടതായിരുന്നു. എന്നാല്‍, അത്തരം യാതൊരു പ്രസ്താവനയും ഖുര്‍ആനിലില്ല.

അഞ്ച്) ആധുനിക ശാസ്ത്രത്തിന്റെ കാഴ്ചപ്പാടിലൂടെ നോക്കുമ്പോള്‍ ബൈബിളില്‍ നിരവധി അശാസ്ത്രീയമായ പരാമര്‍ശങ്ങള്‍ കാണാനാവും. സൂര്യന്റെ സൃഷ്ടിക്ക് മുമ്പു തന്നെ രാപ്പകലുകളു ണ്ടായതായി വിവരിക്കുന്ന ഉല്‍പത്തി പുസ്തകം മുതലാരംഭിക്കുന്നു ബൈബിളിലെ ശാസ്ത്ര വിരു ദ്ധമായ പരാമര്‍ശങ്ങള്‍. രാപ്പകലുകളുണ്ടാവുന്നത് സൂര്യചന്ദ്രന്മാരുടെ ചലനം മൂലമാണെന്നും (യേശു 10:12,13), ഭൂമി ഇളകാതെ നിശ്ചലമായി നില്‍ക്കുകയാണെന്നും (സങ്കീ 104:5) മുയല്‍ അയവിറ ക്കുന്ന ജീവിയാണെന്നു (ആവ 14:7) മെല്ലാമുള്ള ബൈബിള്‍ പരാമര്‍ശങ്ങള്‍ അതിന്റെ അശാസ്ത്രീയ തക്ക് ഉദാഹരണങ്ങളാണ്. ഈ പരാമര്‍ശങ്ങളെല്ലാം വരുന്നത് പ്രവാചക കഥനങ്ങള്‍ക്കിടയിലാ ണെന്ന കാര്യം പ്രത്യേകം ശ്രദ്ധേയമാണ്. ബൈബിളായിരുന്നു ഖുര്‍ആനിന്റെ രചനയ്ക്കുപയോ ഗിച്ചിരുന്ന സ്രോതസ്സെങ്കില്‍ ഈ അശാസ്ത്രീയമായ പരാമര്‍ശങ്ങളെല്ലാം ഖുര്‍ആനിലും സ്ഥാനം പിടിക്കുമായിരുന്നു. ഈ പരാമര്‍ശങ്ങള്‍ വസ്തുതകള്‍ക്ക് നിരക്കാത്തതാണെന്ന അറിവ് മുഹമ്മദ് നബി (സ)യുടെ കാലത്തുണ്ടായിരുന്നില്ലെന്നോര്‍ക്കുക. എന്നാല്‍ ഖുര്‍ആനില്‍ ഇത്തരം യാതൊരുവിധ പരാമര്‍ശങ്ങളുമില്ല. ഖുര്‍ആനിലെ ഒരൊറ്റ വചനമെങ്കിലും ഏതെങ്കിലും ശാസ്ത്രവസ്തുത കളുമായി വൈരുദ്ധ്യം പുലര്‍ത്തുന്നതായി തെളിയിക്കപ്പെട്ടിട്ടില്ല. ബൈബിളില്‍നിന്ന് പകര്‍ത്തി ക്കൊണ്ട് മുഹമ്മദ് നബി (സ) രചിച്ച ഗ്രന്ഥമാണ് ഖുര്‍ആനെന്ന് വാദിക്കുകയാണെങ്കില്‍ തനിക്ക് ശേഷം നൂറ്റാണ്ടുകള്‍ കഴിഞ്ഞ് വരാനിരിക്കുന്ന ശാസ്ത്രമുന്നേറ്റങ്ങള്‍ കൂടി മുന്‍കൂട്ടി കാണാന്‍ കഴിയുകയും അതിന്റെ അടിസ്ഥാനത്തില്‍ ബൈബിളിലുള്ള അശാസ്ത്രീയതകള്‍ അറിഞ്ഞ് അവ യെല്ലാം അരിച്ചൊഴിവാക്കി സംശുദ്ധമായ ചരിത്രം മാത്രം എടുത്തുദ്ധരിക്കുകയും ചെയ്ത അതിമാനുഷനാണ് അദ്ദേഹമെന്ന് പറയേണ്ടിവരും. സര്‍വ്വശക്തനായ സ്രഷ്ടാവിന്റെ വചനങ്ങളാണ് ഖുര്‍ആനിലുള്ളതെന്ന വസ്തുത നിഷേധിക്കുവാന്‍ തെളിവ് പരതുന്നവര്‍ മുഹമ്മദ് നബി (സ)യെ ദൈവമാക്കുന്ന പരിണാമഗുപ്തിയിലാണ് എത്തിച്ചേരുകയെന്നര്‍ത്ഥം.

ആറ്) ബൈബിളില്‍ പറയാത്ത ചില പ്രവാചകന്മാരുടെയും സമുദായങ്ങളുടെയും ചരിത്രം ഖുര്‍ ആന്‍  വിവരിക്കുന്നുണ്ട്. ആദ്, സമൂ ദ് ഗോത്രങ്ങളിലേക്ക് നിയോഗിക്കപ്പെട്ട ഹൂദ് നബിയുടെയും സാലിഹ് നബിയുടെയും ചരിത്രം ഉദാഹരണം. ബൈബിളിലെവിടെയും കാണാനാവാത്ത പ്രവാ ചകന്മാരാണിവര്‍. ബൈബിളില്‍ നിന്ന് കോപ്പിയടിക്കുകയാണ് മുഹമ്മദ് നബി (സ)ചെയ്തതെങ്കില്‍ ഈ ചരിത്രങ്ങള്‍ അദ്ദേഹത്തിന് എവിടെനിന്നാണ് കിട്ടിയത്?

ഏഴ്) ബൈബിളില്‍ പരാമര്‍ശിക്കപ്പെട്ട പ്രവാചകന്മാരുടെ ചരിത്രം പറയുമ്പോള്‍തന്നെ ബൈബിളി ലൊരിടത്തും പരാമര്‍ശിക്കാത്ത നിരവധി സംഭവങ്ങള്‍ ഖുര്‍ആനില്‍ വിശദീകരിക്കുന്നുണ്ട്. നൂഹ് നബി(അ)യും അവിശ്വാസിയായ മകനും തമ്മില്‍ നടന്ന സംഭാഷണവും മകന്‍ പ്രളയത്തില്‍ പെട്ട സംഭവവിവരണവും സൂറത്തു ഹൂദില്‍(11: 42-46) കാണാം. ഇങ്ങനെ യാതൊന്നും ബൈബിളിലെവി ടെയുമില്ല. ഇബ്രാഹീം നബിയും നംറൂദും തമ്മില്‍ നടന്ന സംവാദവും (ഖുര്‍ആന്‍ 2:258) പിതാവു മായി നടന്ന സംഭാഷണവും (ഖുര്‍ആന്‍ 6:74, 19:41-49, 43:26,27) മരണാനന്തര ജീവിതത്തിന്റെ സത്യത ബോധ്യപ്പെടുന്നതിനായി, പക്ഷികളെ കഷ്ണിച്ച് നാല് മലകളില്‍വെച്ചശേഷം അവയെ വിളിച്ചാല്‍ അവ ഓടിവരുന്നതാണെന്ന് അല്ലാഹു അദ്ദേഹത്തോട് പറഞ്ഞ സംഭവവും (2:260) തീയിലേക്ക് വലിച്ചെറിയപ്പെടുകയും അതില്‍ നിന്ന് അദ്ദേഹം അത്ഭുതകരമായി രക്ഷപ്പെടുകയും ചെയ്ത ചരിത്രവു(21:56-70)മൊന്നും ബൈബിളിലൊരിടത്തും കാണാന്‍ കഴിയില്ല. ദൈവിക കല്‍പന പ്രകാരം ഒരു പശുവിനെ അറുക്കാന്‍ മൂസാ (അ) ഇസ്രായീല്യരോട് നിര്‍ദേശിക്കുകയും, പശുവിന്റെ പ്രത്യേകതകള്‍ ചോദിച്ച് അതിന്റെ നിര്‍വ്വഹണം അവര്‍ പ്രയാസകരമാക്കുകയും ചെയ്ത സംഭവവും (ഖുര്‍ആന്‍ 2:67-71) കൊലപാതകക്കുറ്റം തെളിയിക്കാനായി പശുവിനെ അറുത്ത് അതിന്റെ ഒരു ഭാഗംകൊണ്ട് അടിക്കുവാന്‍ കല്‍പിച്ച കഥനങ്ങളും (2:72, 73) മൂസാ നബി (അ)യുടെ ജീവിതവുമായി ബന്ധപ്പെടുത്തി ബൈബിളിലൊരിടത്തും പ്രസ്താവിക്കുന്നില്ല. ഈസാ നബി (അ)യുടെ ജനനം മുതല്‍ തന്നെയുള്ള ബൈബിളില്‍ പറയാത്ത പല സംഭവങ്ങളും ഖുര്‍ആനില്‍ പരാമര്‍ശിക്കുന്നുണ്ട്. സകരിയ്യായുടെ സംരക്ഷണത്തി ല്‍ പ്രാര്‍ത്ഥനാസ്ഥലത്ത് താമസിച്ചുകൊണ്ടി രുന്ന മര്‍യമിന്റെ കുട്ടിക്കാലത്ത് അവര്‍ക്ക് അത്ഭുതകരമായി ഭക്ഷണസാധനങ്ങള്‍ ലഭിച്ച സംഭവം (ഖുര്‍ആന്‍ 3:37), മര്‍യമിന്റെ പ്രസവസമയത്ത് അവര്‍ക്ക് നല്‍കപ്പെട്ട പ്രത്യേക അനുഗ്രഹങ്ങളെ ക്കുറിച്ച വിവരണം (19:23-26), ഈസാ (അ) തൊട്ടിലില്‍വെച്ച് സംസാരിച്ച് തന്റെ നിയോഗം പ്ര ഖ്യാപിച്ചു കൊണ്ട് തന്റെ ആദ്യത്തെ അത്ഭുതം പ്രവര്‍ത്തിച്ച ചരിത്രം (19:29,30), കളിമണ്ണുകൊണ്ട് പക്ഷിയുടെ രൂപമുണ്ടാക്കി ഈസാ (അ) അതില്‍ ഊതിയപ്പോള്‍ അല്ലാഹുവിന്റെ അനുമതിപ്രകാരം അതൊരു പക്ഷിയായി രൂപാന്തരപ്പെട്ട സംഭവം (3:49) ഇതൊന്നുംതന്നെ ബൈബിളില്‍ ഒരിടത്തും പരാമര്‍ശിക്കുന്നുപോലുമില്ല. ബൈബിളില്‍നിന്ന് മുഹമ്മദ് നബി (സ) പകര്‍ത്തിയെഴുതി ക്കൊണ്ടാണ് ഖുര്‍ആന്‍ രചിച്ചതെങ്കില്‍ ബൈബിളിലൊരിടത്തും പരാമര്‍ശിക്കാത്ത പ്രവാച കന്മാ രുടെ ജീവിതവുമായി ബന്ധപ്പെട്ട കഥകള്‍ അദ്ദേഹത്തിന് എവിടെനിന്നു കിട്ടി? മുഹമ്മദ് നബിയുടെ കാലത്ത് യഹൂദ ക്രൈസ്തവര്‍ക്കിടയില്‍ നിലനിന്നിരുന്ന കഥകളും ഐതിഹ്യങ്ങളുമാണ് ഈ പരാമര്‍ശങ്ങളുടെ സ്രോതസ്സ് എന്നു വാദിക്കപ്പെടാറുണ്ട്. ഈ വാദവും അടിസ്ഥാന രഹിതമാണ്. പ്രസ്തുത കഥകളു ടെയും പുരാണങ്ങളുടെയുമെല്ലാം ചരിത്രപരതയെ ചോദ്യം ചെയ്യുന്ന പഠനങ്ങളും ഗവേഷണങ്ങളും പില്‍ക്കാലത്ത് നടന്നിട്ടുണ്ട്. ഈ ഗവേഷണങ്ങളിലൊന്ന് പോലും ഒരു ഖുര്‍ആന്‍ കഥനത്തെയും വസ്തുനിഷ്ഠമായി വിമര്‍ശിക്കുന്നില്ലെന്ന കാര്യം പ്രത്യേകം ശ്രദ്ധേയമാണ്. സത്യത്തില്‍ ഖുര്‍ആന്‍ ദൈവവചനമായതുകൊണ്ടാണ് ബൈബിളിലെവിടെയും സൂചിപ്പിക്കാത്ത സംഭവങ്ങള്‍പോലും കൃത്യവും പ്രമാദരഹിതവുമായി അതില്‍ നമുക്ക് കാണാന്‍ കഴിയുന്നത്. മര്‍യത്തിന്റെ ബാല്യകാല സംഭവങ്ങള്‍ വിവരിക്കവെ ഖുര്‍ആന്‍ പറ ഞ്ഞത് എത്ര ശരി! ''(നബിയേ) നാം നിനക്ക് ബോധനം നല്‍കുന്ന അദൃശ്യവാര്‍ത്തകളില്‍ പെട്ടതാകുന്നു അവയൊക്കെ. അവരില്‍ ആരാണ് മര്‍യത്തിന്റെ സംരക്ഷണം ഏറ്റെടുക്കേണ്ടതെന്ന് തീരുമാനിക്കുവാനായി അവര്‍ തങ്ങളുടെ അമ്പുകള്‍ ഇട്ടുകൊണ്ട് നറുക്കെടുപ്പ് നടത്തിയിരുന്ന സമയത്ത് നീ അവരുടെ അടുത്തുണ്ടായിരുന്നില്ല ല്ലോ. അവര്‍ തര്‍ക്കത്തിലേര്‍പ്പെട്ടുകൊണ്ടിരുന്നപ്പോഴും നീ അവരുടെ അടുത്തുണ്ടായിരുന്നില്ല'' (വി.ഖു. 3:44).

എട്ട്) ബൈബിളില്‍ പരാമര്‍ശിക്കപ്പെട്ട കഥകള്‍ പറയുമ്പോഴും ബൈബിളില്‍നിന്ന് വ്യത്യസ്തമായി കൃത്യതയും സൂക്ഷ്മതയും ഖുര്‍ആന്‍ കാത്തുസൂക്ഷിക്കുന്നത് കാണാം. ഉദാഹരണത്തിന് മോശ- സീനായ് പര്‍വതത്തിലേക്ക് പോയ അവസരത്തില്‍ ഇസ്രായീല്യര്‍ക്ക് അവരുടെ ആവശ്യപ്രകാരം സ്വര്‍ണംകൊണ്ട് കാളക്കുട്ടിയെ നിര്‍മിച്ച് ആരാധനക്കായി നല്‍കിയത് മോശയുടെ കൂട്ടാളിയും പ്രവാ ചകനുമായ അഹരോണായിരുന്നുവെന്നാണ് പുറപ്പാട് പുസ്തകം (32:1-6) പറയുന്നത്. ഖുര്‍ആനും ബൈബിളുമെല്ലാം പരിശുദ്ധ പ്രവാചകനായി പരിചയപ്പെടുത്തുന്ന ഹാറൂനി(അ)ല്‍ നിന്ന് വിഗ്രഹാ രാധനക്ക് കൂട്ടുനില്‍ക്കുകയെന്ന മഹാപാപം സംഭവിക്കാനിടയില്ലെന്ന് ഏത് സാമാന്യ ബുദ്ധിക്കും മനസ്സിലാവും. ഖുര്‍ആനും പ്രസ്തുത സംഭവം വിവരിക്കുന്നുണ്ട്. പക്ഷെ, സ്വര്‍ണപശുവിനെയുണ്ടാ ക്കുകയും അതിനെ ആരാധിക്കുവാന്‍ ഇസ്രായീല്യരെ പ്രേരിപ്പിക്കുകയും ചെയ്തത് ഹാറൂന(അ)ല്ല; പ്രത്യുത ഇസ്രായീല്യരില്‍പെട്ട ഒരു കപടനായ സാമിരിയാണ് ഇത് ചെയ്തതെന്നും അതുമൂലം അയാള്‍  ദൈവകോപത്തിനും ശപിക്കപ്പെട്ട രോഗത്തിനും വിധേയനായെന്നുമാണ് ഖുര്‍ആന്‍ പഠിപ്പിക്കുന്നത് (20:85-95). ബൈബിളില്‍ പരാമര്‍ശിക്കപ്പെട്ട കഥകള്‍ വിവരിക്കുമ്പോഴും അതിലെ നെല്ലും പതിരും വേര്‍ തിരിച്ച് സത്യസന്ധവും സൂക്ഷ്മവുമായ രീതിയില്‍ അവ ജനസമക്ഷം വെക്കുന്ന ഖുര്‍ആന്‍ ദൈവികമാണെന്ന് അതിന്റെ ഈ പ്രത്യേകത തന്നെ സുതരാം വ്യക്തമാക്കുന്നു.

പതിവ്രതകളുടെ മേൽ വ്യഭിചാരമാരോപിക്കുന്നത് പൊറുക്കപ്പെടാവുന്ന പാപമാണെന്ന് 24: 4, 5 ലും പൊറുക്കപ്പെടുകയില്ലെന്ന് 24: 23 ലും പറയുന്നു ഇത് വൈരുധ്യമല്ലേ? വൈരുധ്യം ആരോപിക്കപ്പെട്ടിരിക്കുന്ന ഖുര്‍ആന്‍ സൂക്തങ്ങളുടെ സാരം പരിശോധിക്കുക: ചാരിത്ര്യവതികളുടെ മേല്‍(വ്യഭിചാരം) ആരോപിക്കുകയും എന്നിട്ട് നാല് സാക്ഷികളെ കൊണ്ട് വരാതിരിക്കുകയും ചെയ്യുന്നവരെ നിങ്ങള്‍ എണ്‍പത് അടി അടിക്കുക. അവരുടെ സാക്ഷ്യം ഒരിക്കലും സ്വീകരിക്കുകയും ചെയ്യരുത്. അവര്‍ തന്നെയാണ് തോന്നിവാസികള്‍. അതിനുശേഷം പശ്ചാത്തപിക്കുകയും, നിലപാട് നന്നാക്കിതീര്‍ക്കുകയും ചെയ്തവരൊഴികെ. എന്നാല്‍ അല്ലാഹു ഏറെ പൊറുക്കുന്നവനും, കരുണാനിധിയും തന്നെയാകുന്നു. (24: 4, 5) പതിവ്രതകളും (ദുര്‍വൃത്തിയെപ്പറ്റി) ഓര്‍ക്കുകപോലും ചെയ്യാത്തവരുമായ സത്യവിശ്വാസിനികളെപറ്റി ദുരാരോപണം നടത്തുന്നവരാരോ അവര്‍ ഇഹത്തിലും പരത്തിലും ശപിക്കപ്പെട്ടിരിക്കുന്നു; തീര്‍ച്ച. അവര്‍ക്ക് ഭയങ്കരമായ ശിക്ഷയുണ്ട്. (24: 23) ഇവിടെ ഉദ്ധരിക്കപ്പെട്ട സൂറത്തു നൂറിലെ നാലാം വചനം സാക്ഷികളില്ലാതെ വ്യഭിചാര ആരോപണം ഉന്നയിക്കുന്നവര്‍ക്ക് ഇസ്‌ലാമിക ഭരണകൂടം നൽകേണ്ട ശിക്ഷയെ കുറിച്ചുള്ളതാണ്. അവര്‍ തന്നെയാണ് തോന്നിവാസികള്‍ എന്ന് പറഞ്ഞുകൊണ്ട് ഈ സൂക്തം അവസാനിക്കുകയും ചെയ്യുന്നു. ദൈവിക വിധിവിലക്കുകള്‍ തൃണവല്‍ഗണിക്കുന്ന അത്തരം ആളുകള്‍ക്കുള്ള അതികഠിനമായ ദൈവിക ശിക്ഷയെ കുറിച്ച് ഖുര്‍ആനിലെ മറ്റു സൂക്തങ്ങളില്‍ വ്യക്തമാക്കുകയും ചെയ്യുന്നുണ്ട്. മ്ലേഛമായ ഈ കുറ്റംചെയ്ത് ഭരണകൂടത്തിന്റെ ശിക്ഷ ഏറ്റു വാങ്ങിയ ആളുകള്‍ പോലും ദൈവിക കാരുണ്യത്തില്‍ നിരാശരാകേണ്ടതില്ലെന്ന സദ്‌വാര്‍ത്തയാണ് 5 ാംമത്തെ സൂക്തത്തിലൂടെ അല്ലാഹു വ്യക്തമാക്കുന്ത്. അതി നികൃഷ്ടമായ ആരോപണങ്ങള്‍ ഉന്നയിച്ചുകൊണ്ട് ദൈവകോപത്തിന്ന് വിധേയരായ ആളുകള്‍ക്ക് പോലും പശ്ചാത്താപത്തിലൂടെ പാപ പരിഹാരം സാധിക്കുമെന്ന് പ്രതീക്ഷ നല്‍കുകയാണ് ഈ സൂക്തം ചെയ്യുന്നത്. പശ്ചാത്തപിക്കാത്തവര്‍ പരലോകത്ത് കഠിനമായ ശിക്ഷ അനുഭവിക്കേണ്ടിവരിക തന്നെ ചെയ്യും. 24: 23 ലാകട്ടെ സദ്‌വൃത്തകളെക്കുറിച്ച വ്യഭിചാരാരോപണത്തിന്റെ കാഠിന്യം കുറേക്കൂടി വ്യക്തമായി പ്രകടിപ്പിക്കുകയാണ് ചെയ്യുന്നത്. സൂറത്തുന്നൂറിലെ പതിനൊന്ന് മുതല്‍ ഇരുപത്തി ആറ് വരെയുള്ള സൂക്തങ്ങള്‍ക്ക് പ്രത്യേകമായ ഒരു അവതരണ പശ്ചാത്തലമുണ്ട്. പ്രവാചകപത്‌നിയായ ആയിശ(റ)യുടെ മേലുള്ള വ്യഭിചാരാരോപണവുമായി കപടവിശ്വാസികളും പ്രവാചകാനുചരന്‍മാരില്‍ ചിലരും രംഗത്ത് വന്നസാഹചര്യത്തിലുണ്ടായ പ്രതിസന്ധിക്ക് പരിഹാരമെന്നോണം അവതരിക്കപ്പെട്ട സൂക്തങ്ങളാണിവ. പതിവ്രതകളും ദുര്‍വൃത്തിയെപ്പറ്റി ഓര്‍ക്കുകപോലും ചെയ്യാത്തവരുമായ സത്യവിശ്വാസിനികളെപ്പറ്റി ദുരാരോപണം നടത്തുന്നവര്‍ക്ക് ഇഹത്തിലും പരത്തിലും ശാപവും മരണാനന്തരം ഭയങ്കരമായ ശിക്ഷയുമാണുള്ളത് എന്ന മുന്നറിയിപ്പാണ് 24:23ല്‍ ഉള്ളത്. ഈ സൂക്തത്തില്‍ പശ്ചാത്താപത്തെക്കുറിച്ച് യാതൊരുപരാമര്‍ശവുമില്ല. അതുകൊണ്ട് തന്നെ പശ്ചാത്തപിച്ചവര്‍ക്ക് ലഭ്യമായേക്കാവുന്ന പാപമോചനത്തെക്കുറിച്ച സൂചനകളൊന്നും ഈസൂക്തം ഉള്‍ക്കൊള്ളുന്നില്ല. പശ്ചാത്തപിക്കുകയും നിലപാട് നന്നാക്കിതീര്‍ക്കുകയും ചെയ്തവര്‍ക്ക് 24: 5 ല്‍ വാഗ്ദാനം ചെയ്യപ്പെട്ട പാപമോചനം ഈ സൂക്തത്തിലെ പരാമര്‍ശങ്ങള്‍ക്കും ബാധകമാണ്. നിഷ്‌കളങ്കമായി പശ്ചാത്തപിക്കുകയും കുറ്റം ആവര്‍ത്തിക്കാതിരിക്കുകയും ചെയ്താല്‍ അല്ലാഹു പൊറുക്കുമെന്ന വാഗ്ദാനം എല്ലാ കുറ്റങ്ങള്‍ക്കും ബാധകമായിട്ടുള്ളതാണ്. 24: 23 ലാകട്ടെ പശ്ചാത്താപം സ്വീകരിക്കപ്പെടുകയില്ലെന്ന് പറഞ്ഞിട്ടുമില്ല. ആയിശ(റ)യുമായി ബന്ധപ്പെട്ട ദുരാരോപണ പ്രചാരണത്തില്‍ പങ്കെടുത്ത പ്രവാചകാനുചരന്‍മാരില്‍ ചിലര്‍, അവര്‍ക്ക് എണ്‍പത് അടി ശിക്ഷ ലഭിച്ചതിന് ശേഷം, തങ്ങളുടെ നിലപാട് നന്നാക്കി തീര്‍ക്കുകയും, പ്രവാചകന്റെ അനുയായി വൃന്ദത്തില്‍ സജീവമായി നിലകൊള്ളുകയും ചെയ്ത ചരിത്രത്തില്‍ നിന്ന് മനസ്സിലാകുന്നതും അതാണ്. അതിനാൽ 24: 4, 5 സൂക്തങ്ങളുമായി ഈ സൂക്തം (24: 23) യാതൊരുവിധ വൈരുധ്യവും പുലര്‍ത്തുന്നില്ലെന്ന് വ്യക്തമാകുന്നു.
മോശയുടെ ജനനകാലത്താണ് ഇസ്റാഈല്യര്‍ക്ക് ജനിക്കുന്ന കുഞ്ഞുങ്ങളെയെല്ലാം കൊന്നുകളയുവാന്‍ ഫറോവ കൽപന പുറപ്പെടുവിച്ചതെന്ന് 20: 38, 39 ല്‍ പറയുന്നതിന് വിരുദ്ധമായി 40: 23-25 ല്‍ മോശ പ്രവാചകനായതിനു ശേഷമാണ് പ്രസ്തുത കൽപന പുറപ്പെടുവിച്ചതെന്ന് കാണുന്നു. ഇതിലേതാണ് ശരി? ഇസ്‌റാഈല്യര്‍ക്ക് ജനിക്കുന്ന ആണ്‍കുഞ്ഞുങ്ങളെയെല്ലാം കൊന്നുകളയാന്‍ ഫറോവ രണ്ടു തവണ കൽപന പുറപ്പെടുവിച്ചതായി ഖുര്‍ആനില്‍ നിന്ന് മനസ്സിലാകുന്നുണ്ട്. ഇസ്‌റാഈല്യരുടെ എണ്ണം വര്‍ധിക്കുന്നത് തങ്ങളുടെ അധികാരസ്ഥാനത്തിന് ഭീഷണിയാകുമെന്ന് കരുതി, മോശയുടെ ജനനകാലത്തെ ഫറോവ ഈ കൽപന പുറപ്പെടുവിച്ചതായി ബൈബിളില്‍ (പുറപ്പാട് 1:8-16) പറയുന്നുണ്ട്. ഇക്കാര്യമാണ് ഖുര്‍ആനില്‍ 28:4ലും 20:38,39 ലും സൂചിപ്പിച്ചിരിക്കുന്നത്. ഇതില്‍ നിന്ന് തികച്ചും വ്യത്യസ്തമായ മറ്റൊരു സാഹചര്യത്തില്‍ ഇസ്‌റാഈല്യരിലെ ആണ്‍കുഞ്ഞുങ്ങളെയെല്ലാം കൊന്നുകളയുവാന്‍ ഫറോവ കൽപനപുറപ്പെടുവിച്ചതായി 7:127 ലും 40:23-25 ലും ഖുര്‍ആന്‍ വ്യക്തമാക്കുന്നുണ്ട്. ഏകദൈവാദര്‍ശത്തിന്റെ പ്രബോധകനായി നിയോഗിക്കപ്പെട്ട മൂസാനബിയുടെ കൂടെ സത്യവിശ്വാസം സ്വീകരിച്ചവരുടെ ആണ്‍മക്കളെ കൊന്നുകളയാനായിരുന്നു പ്രസ്തുത കൽപന. ഇക്കാര്യം ഖുര്‍ആന്‍ വ്യക്തമാക്കുന്നത് കാണുക: ”ഫിര്‍ഔനിന്റെ ജനതയിലെ പ്രമാണിമാര്‍ പറഞ്ഞു: ഭൂമിയില്‍ കുഴപ്പമുണ്ടാക്കുവാനും താങ്കളെയും താങ്കളുടെ ദൈവങ്ങളേയും വിട്ടുകളയാനും താങ്കള്‍ മൂസായേയും അവന്റെ ആള്‍ക്കാരെയും അനുവദിക്കുകയാണോ? അവന്‍ പറഞ്ഞു, നാം അവരുടെ ആണ്‍മക്കളെ കൊന്നൊടുക്കുകയും അവരുടെ സ്ത്രീകളെ ജീവിക്കാന്‍ വിടുകയും ചെയ്യുന്നതാണ്. തീര്‍ച്ചയായും നാം അവരുടെ മേല്‍ സര്‍വാധിപത്യമുള്ളവരായിരിക്കും. (7:127) ഇക്കാര്യം തന്നെയാണ് 40:23 മുതല്‍ 25 വരെയുള്ള സൂക്തങ്ങളിലും ഖുര്‍ആന്‍ വ്യക്തമാക്കുന്നത്. മൂസാ(അ)യുടെ ജനനത്തിനു മുമ്പും അദ്ദേഹത്തിന്റെ പ്രവാചകത്വ ലബ്ധിക്കുശേഷവും ഫിര്‍ഔന്‍ ഒരേ രീതിയിലുള്ള കൽപന പുറപ്പെടുവിച്ചത് ഉദ്ധരിക്കുകയാണ് ഖുര്‍ആന്‍ ചെയ്യുന്നത്. ഇത് ഖുര്‍ആനിലെ ഒരു വൈരുധ്യമല്ലെന്നര്‍ഥം.
നൂഹ് നബിയെ അദ്ദേഹത്തിന്റെ ജനത വിരട്ടിയോടിച്ചുവെന്ന് 54:9 ൽ പറയുന്നു. എന്നാൽ അദ്ദേഹം കപ്പൽ നിർമ്മിച്ചുകൊണ്ടിരുന്നപ്പോള്‍ തന്റെ ജനതയിലെ പ്രമാണിക്കൂട്ടം അദ്ദേഹത്തിന് അടുത്ത് കൂടെ കടന്നുപോയി എന്നും, പരിഹസിച്ചുവെന്നും 11:38 ൽ കാണാം. നാട്ടിൽ നിന്ന് ഓടിക്കപ്പെട്ട നൂഹ് നബിയുടെ കപ്പൽ നിർമാണം നാട്ടുകാർ കണ്ടു എന്നു പറയുന്നത് അടിസ്ഥാനരഹിതമല്ലേ ? എതിരാളികളുടെ പീഡനങ്ങള്‍ സഹിക്കവയ്യാതെ സ്വന്തം നാട്ടില്‍ നിന്ന് നൂഹ് നബി എങ്ങോട്ടെങ്കിലും പലായനം ചെയ്തതായി ഖുര്‍ആനില്‍ എവിടെയും പറയുന്നില്ല. ഇവിടെ പരാമര്‍ശിക്കപ്പെട്ട സൂറത്തുല്‍ ഖമറിലെ വചന (54:9) ത്തിന്റെ സാരം ഇങ്ങനെയാണ്. ”അവര്‍ക്ക് മുമ്പ് നൂഹിന്റെ ജനതയും നിഷേധിച്ചുകളഞ്ഞിട്ടുണ്ട്. അങ്ങനെ നമ്മുടെ ദാസനെ അവര്‍ നിഷേധിച്ചു തള്ളുകയും ഭ്രാന്തന്‍ എന്ന്പറയുകയും ചെയ്തു. അദ്ദേഹം വിരട്ടി ഓടിക്കപ്പെടുകയും ചെയ്തു.’’ ഈ സൂക്തത്തില്‍ അദ്ദേഹം വിരട്ടി ഓടിക്കപ്പെടുകയും ചെയ്തു, എന്നു പരിഭാഷപ്പെടുത്തിയിരിക്കുന്നത് ‘വസ്ദുജിര്‍’ എന്ന പദത്തെയാണ്. ‘സജറ‘ എന്ന മൂലപദത്തില്‍ നിന്ന് വ്യുല്‍പന്നമായതാണ് ഈ പദം. നാട്ടില്‍ നിന്ന് ആട്ടിയോടിക്കുന്നതിനോ, പലായനത്തിന് നിര്‍ബന്ധിക്കുന്നതിനോ അല്ല ഈപദം ഉപയോഗിക്കുന്നത്. മറിച്ച് ഒരു കാര്യത്തില്‍നിന്ന് പിന്തിരിപ്പിക്കുവാന്‍ വേണ്ടിയുള്ള പീഡനത്തിന്റെ ഭാഗമായി പരിഹസിക്കുകയും, ശബ്ദകോലാഹലങ്ങള്‍ ഉണ്ടാക്കുകയും ചെയ്തുകൊണ്ട് വിരട്ടി ഓടിക്കുന്നതിനാണ് ഈ പദം പ്രയോഗിക്കുക. നൂഹ് നബി സത്യമതപ്രചാരണത്തില്‍ ഏര്‍പ്പെട്ടപ്പോള്‍ തന്റെ സമുദായം അത് നിഷേധിക്കുകയും അദ്ദേഹത്തെ പരിഹസിക്കുകയും ചെയ്തതോടൊപ്പം തന്നെ ആദര്‍ശപ്രബോധനത്തിന് സമ്മതിക്കാതെ അദ്ദേഹത്തെ വിരട്ടി ഓടിക്കുകയും ചെയ്തുവെന്ന് മാത്രമാണ് 54:9 ല്‍ പറയുന്നത്. എന്നാല്‍ ഈ പീഡനങ്ങളെല്ലാം സഹിച്ചുകൊണ്ടുതന്നെ നൂഹ് നബി (അ) തന്റെ നാട്ടില്‍ ഒരു സഹസ്രാബ്ദകാലത്തോളം പ്രബോധന പ്രവര്‍ത്തനങ്ങള്‍ തുടര്‍ന്നു കൊണ്ടുപോയി എന്നും എന്നിട്ടും അംഗുലീപരിമിതരായ ആളുകളെ മാത്രമേ സത്യസരണിയിലേക്ക് കൊണ്ടുവരാന്‍ സാധിച്ചുള്ളൂവെന്നും അവസാനമാണ് അദ്ദേഹത്തിന്റെ പ്രാര്‍ഥന പ്രകാരം പ്രളയമുണ്ടായത് എന്നുമുള്ള ചരിത്രങ്ങള്‍ വിവിധ സ്ഥലങ്ങളിലായി ഖുര്‍ആനില്‍ പരാമര്‍ശിക്കപ്പെട്ടിട്ടുണ്ട്. ചുരുക്കത്തില്‍, ഖുര്‍ആനിലെവിടെയും നൂഹ് നബിയുടെ പലായനത്തെപ്പറ്റി യാതൊരു പരാമര്‍ശവും നടത്തിയിട്ടില്ല. അതിനാല്‍ കപ്പല്‍ നിര്‍മ്മിക്കുമ്പോഴും മുമ്പുമെല്ലാം അദ്ദേഹത്തെ പരിഹസിക്കുകയും വിരട്ടുകയും ചെയ്ത നാട്ടുകാരുടെ നടപടിയെ വിമര്‍ശിക്കുന്ന സൂക്തങ്ങളുമായി വൈരുധ്യം പുലര്‍ത്തുന്ന പരാമര്‍ശങ്ങള്‍ ഖുര്‍ആനില്‍ എവിടെയുമില്ല.
നൂഹ് നബിയേയും കുടുംബത്തെയും വെള്ളപ്പൊക്കത്തിൽ നിന്ന് രക്ഷപ്പെടുത്തിയെന്ന 21:76 ലെ പരാമർശത്തിന് വിരുദ്ധമായി അദ്ദേഹത്തിന്റെ മകൻ പ്രളയത്തിലകപ്പെട്ട് നശിച്ചുവെന്ന് 11:42, 43 ൽ പറയുന്നുണ്ടല്ലോ. ഇത് വ്യക്തമായ വൈരുധ്യമല്ലേ? ഈ ചോദ്യത്തിന്റെ ഉത്തരം സൂറത്തു ഹൂദില്‍ നിന്ന് ഉദ്ധരിക്കപ്പെട്ട സൂക്തങ്ങള്‍ക്ക് തൊട്ടു പിറകെയുള്ള സൂക്തങ്ങളില്‍ തന്നെയുണ്ട്, അത്കാണുക: ”നൂഹ് തന്റെ രക്ഷിതാവിനെ വിളിച്ചുകൊണ്ട് പറഞ്ഞു: എന്റെര ക്ഷിതാവേ, എന്റെ മകന്‍ എന്റെ കുടുംബാഗങ്ങളില്‍പ്പെട്ടവന്‍ തന്നെയാണല്ലോ. തീര്‍ച്ചയായും നിന്റെ വാഗ്ദാനം സത്യമാണ്താനും. നീവിധി കര്‍ത്താക്കളില്‍ വെച്ച് ഏറ്റവും നല്ല വിധി കര്‍ത്താവുമാണ്. അവന്‍ (അല്ലാഹു) പറഞ്ഞു: നൂഹേ, തീര്‍ച്ചയായും അവന്‍ നിന്റെ കുടുംബത്തില്‍പ്പെട്ടവനല്ല. തീര്‍ച്ചയായും അവന്‍ ശരിയല്ലാത്തത് ചെയ്തവനാണ്. അതിനാല്‍ നിനക്ക് അറിവില്ലാത്ത കാര്യം എന്നോട് ആവശ്യപ്പെടരുത്. നീ വിവരമില്ലാത്തവരുടെ കൂട്ടത്തിലായിപ്പോകരുതെന്ന് ഞാന്‍ നിന്നോട് ഉപദേശിക്കുകയാണ്”. (11:45, 46) പ്രവാചകന്‍മാരുടെ കുടുംബം സത്യവിശ്വാസികളായ സഖാക്കളാണെന്ന് വ്യക്തമാക്കുകയാണ് ഈ വചനങ്ങളിലൂടെ അല്ലാഹു ചെയ്യുന്നത്. അതുകൊണ്ട് തന്നെ അവിശ്വാസിയായ മകന്‍ നൂഹ് നബിയുടെ കുടുംബത്തില്‍പ്പെട്ടവനല്ല. 21:76 ല്‍ പറഞ്ഞ അദ്ദേഹത്തെയും കുടുംബത്തെയും മഹാദു:ഖത്തില്‍ നിന്ന് രക്ഷപ്പെടുത്തി എന്ന പരാമര്‍ശത്തിന്റെ വരുതിയില്‍ അവിശ്വാസിയായ മകന്‍ വരുന്നില്ല എന്നര്‍ഥം.

ഇബ്റാഹീം നബിയുടെ ചരിത്രം പറയുമ്പോൾ 21:51,59 ൽ തന്റെ ജനതയോട് അദ്ദേഹം അതിശക്തമായി പ്രതികരിക്കുകയും വിഗ്രഹങ്ങളെ തകർക്കുകയുമെല്ലാം ചെയ്തതായി പ്രതിപാദിക്കുന്നതിന് വിരുദ്ധമായി 19:41, 49 ൽ തന്റെ പിതാവിന്റെ ഭീഷണിക്ക് വഴങ്ങിക്കൊണ്ട് അവരെയെല്ലാം വെടിഞ്ഞ് അദ്ദേഹം പോയി എന്നാണ് കാണുന്നത്. ഈ വൈരുധ്യം എങ്ങനെ വിശദീകരിക്കാനാവും ?

ഖുര്‍ആനിന്റെ ചരിത്ര പ്രതിപാദനരീതിയെ പറ്റിയുള്ള തികഞ്ഞ അജ്ഞതയില്‍ നിന്നാണ് ഈ ആരോപണം ഉണ്ടായിരിക്കുന്നത്. ഇബ്‌റാഹീം നബിയുടെ ജീവിതത്തിലുണ്ടായ വ്യത്യസ്ത സംഭവങ്ങള്‍ വിവിധ സൂക്തങ്ങളിലായി ഖുര്‍ആന്‍ പരാമര്‍ശിക്കുന്നുണ്ട്. ഈ പരാമര്‍ശങ്ങള്‍ സംഭവങ്ങള്‍ നടന്ന ക്രമത്തിലല്ല വിശദീകരിക്കപ്പെട്ടിട്ടുള്ളത്. പല സംഭവങ്ങളും പല സൂക്തങ്ങളിലായി പരന്നുകിടക്കുകയാണ്. അതില്‍പ്പെട്ട രണ്ട് സംഭവങ്ങളാണ് സൂറത്തു മര്‍യമിലെ സൂക്തങ്ങളിലും (19:411,49) സൂറത്തു അമ്പിയാഇലെ സൂക്തങ്ങളിലും(21:51-59) വിശദീകരിക്കപ്പെട്ടിരിക്കുന്നത്. ഇവ രണ്ടും ഇബ്‌റാഹീം നബിയുടെ ജീവിതത്തിലെ രണ്ട് സംഭവങ്ങളുടെ വിവരണമാണ് എന്നതിനാല്‍ തന്നെ ഇവ തമ്മില്‍ വൈരുധ്യമുണ്ടെന്ന് പറയുന്നത് അടിസ്ഥാന രഹിതമാണ്.

സൂറത്തുല്‍ അമ്പിയാഇലെ 51 മുതലുള്ള സൂക്തങ്ങളില്‍ തന്റെ സമുദായത്തെ വിഗ്രഹാരാധനക്കെതിരെ ബോധവല്‍ക്കരിക്കുന്നതിനുവേണ്ടി ഇബ്‌റാഹീം നബി സ്വീകരിച്ച മാര്‍ഗവും അതിന്റെ പ്രതികരണമെന്നോണം അദ്ദേഹത്തിന് നേരിടേണ്ടി വന്ന അഗ്നി പരീക്ഷയുമാണ് വിവരിക്കപ്പെട്ടിരിക്കുന്നത്. ഇതില്‍ നിന്ന് തികച്ചും വ്യത്യസ്തമായ മറ്റൊരു സാഹചര്യമാണ് സൂറത്തുമര്‍യമിലെ 41 മുതലുള്ള സൂക്തങ്ങളിലുള്ളത്. വിഗ്രഹാരാധകനും വിഗ്രഹവില്‍പനക്കാരനുമായ തന്റെ പിതാവിനെ ഏകദൈവാരാധനയുടെ സത്യസരണിയിലേക്ക് ക്ഷണിച്ചുകൊണ്ട് അദ്ദേഹം നടത്തിയ പരാമര്‍ശങ്ങളും അതിന്ന് പിതാവ് നല്‍കിയ മറുപടിയുമാണ് ഈ സൂക്തങ്ങളില്‍ വിശദീകരിക്കപ്പെട്ടിരിക്കുന്നത്. വിഗ്രഹാരാധനയില്‍ നിന്ന് വിട്ടുമാറി നിന്നാല്‍ ഞാന്‍ നിന്നെ കല്ലെറിഞ്ഞോടിക്കുമെന്ന പിതാവിന്റെ ഭീഷണിക്ക് മുമ്പില്‍ പതറാതെ ഏകദൈവരാധനയില്‍ നിന്ന് അല്‍പം പോലും വ്യതിചലിക്കാന്‍ താന്‍ സന്നദ്ധനല്ലെന്ന് പ്രഖ്യാപിക്കുന്ന ഇബ്‌റാഹീം (അ)നെയാണ് 19:47-49 സൂക്തങ്ങളില്‍ നമുക്ക്കാണാന്‍ കഴിയുന്നത്.

‘അങ്ങനെ അവരെയും അല്ലാഹുവിന് പുറമേ അവര്‍ ആരാധിക്കുന്നവയെയും വെടിഞ്ഞ് അദ്ദേഹം പോയപ്പോള്‍ അദ്ദേഹത്തിന് നാം ഇസ്ഹാഖിനെയും യഅ്ഖൂബിനെയും നല്‍കി. അവരെയൊക്കെ നാം പ്രവാചകന്‍മാരാക്കുകയും ചെയ്തു‘(19:49) എന്ന് പറഞ്ഞതിനര്‍ഥം പിതാവ് ഭീഷണിപ്പെടുത്തിയ ഉടനെത്തന്നെ ഇബ്‌റാഹീം (അ) നാട്ടില്‍ നിന്ന് വിട്ടുമാറി പോയി എന്നല്ല; പ്രത്യുത, പിതാവില്‍നിന്നും കുടുംബാംഗങ്ങളില്‍നിന്നും വിഗ്രഹാരാധനയില്‍നിന്നും അദ്ദേഹം അകന്ന് മാറി നിന്നുവെന്നും അനന്തരം അദ്ദേഹത്തിന് അല്ലാഹു പുത്രനെയും പൗത്രനെയും നല്‍കി എന്നും പിന്നീട് ഇരുവര്‍ക്കും പ്രവാചകത്വം നല്‍കി അനഗ്രഹിച്ചുവെന്നും വ്യക്തമാക്കുകയാണ്. പിതാവിന്റെയും കുടുംബത്തിന്റെയും ദൈവങ്ങളെ വെടിഞ്ഞ് ഏകദൈവത്വത്തിന്റെ പ്രബോധനത്തില്‍ മുഴുകിയതിനെത്തുടര്‍ന്നാണ് ഇബ്‌റാഹീം നബി(അ)ക്ക് അല്ലാഹു പുത്രപൗത്രന്‍മാരെ നല്‍കുകയും അവര്‍ക്കും പ്രവാചകത്വം നല്‍കി അനുഗ്രഹിക്കുകയും ചെയ്തത് എന്നത്രെ ഈ വചനത്തിന്റെ താല്‍പര്യം. സ്ഥലകാലങ്ങളെ പരാമര്‍ശിക്കാതെ അല്ലാഹു ചെയ്ത അനുഗ്രഹങ്ങളെ ചൂണ്ടിക്കാണിച്ചിരിക്കയാണ് 19: 49, 50 വചനങ്ങളില്‍. അതുകൊണ്ട് തന്നെസൂറത്തു അമ്പിയാഇലും മറ്റു അദ്ധ്യായങ്ങളിലും വിശദീകരിക്കപ്പെട്ട ഇബ്‌റാഹീം നബിയുടെ ജീവിതസംഭവങ്ങളുമായി ഈ വചനങ്ങള്‍ വൈരുധ്യം പുലര്‍ത്തുന്നുവെന്നു പറയുന്നത് അടിസ്ഥാന രഹിതമാണ്.

ലൂത്ത് നബി തന്റെ ജനതയോട് പ്രകൃതിവിരുദ്ധ രതിയെക്കുറിച്ച് ചോദ്യം ചെയ്തപ്പോൾ അവരുടെ പ്രതികരണം എന്തായിരുന്നുവെന്ന് വിവരിക്കുന്നേടത്ത് ഖുർആനിൽ രണ്ട് വചനങ്ങളിൽ വ്യത്യസ്ത പരാമർശം കാണുന്നു: ഇവരെ നിങ്ങളുടെ നാട്ടിൽ നിന്നും പുറത്താക്കുക. ഇവർ പരിശുദ്ധി പ്രാപിക്കുന്ന ആളുകളാകുന്നു എന്നു പറഞ്ഞത് മാത്രമായിരുന്നു അദ്ദേഹത്തിന്റെ ജനതയുടെ മറുപടി. (7:82). നീ സത്യവാന്മാരുടെ കൂട്ടത്തിലാണെങ്കിൽ ഞങ്ങൾക്ക് അല്ലാഹുവിന്റെ ശിക്ഷ നീ കൊണ്ടുവാ എന്നു പറഞ്ഞതു മാത്രമായിരുന്നു അപ്പോൾ അദ്ദേഹത്തിന്റെ ജനതയുടെ മറുപടി (29:29). അദ്ദേഹത്തിന്റെ ജനത ഒരു മറുപടി മാത്രമേ പറഞ്ഞിട്ടുള്ളൂവെങ്കിൽ ഈ രണ്ടു വചനങ്ങളിൽ ഒന്ന് ശരിയാകാൻ സാധ്യതയില്ലല്ലോ? തന്റെ സമുദായത്തില്‍ വ്യാപകമായിരുന്ന സ്വവര്‍ഗരതിയെന്ന മഹാപാപത്തിനെതിരെ ജനങ്ങളെ ബോധവല്‍ക്കരിച്ചിരുന്ന പ്രവാചകനായിരുന്നു ലൂത്ത് നബി(അ). അദ്ദേഹം തന്റെ ജനതയോട് ഒരു പ്രാവശ്യം മാത്രമായിരിക്കുകയില്ല സംസാരിച്ചിരിക്കുക. പലതവണ, പലരോടും അദ്ദേഹം പ്രകൃതിവിരുദ്ധ രതിക്കെതിരെ സംസാരിച്ചിട്ടുണ്ടാകും. അപ്പോള്‍ അവരുടെ മറുപടി വ്യത്യസ്തമായിരിക്കും. ഇവിടെ സൂചിപ്പിക്കപ്പെട്ട സൂക്തങ്ങളില്‍ വ്യത്യസ്തങ്ങളായ രണ്ടു സന്ദര്‍ഭങ്ങളാണ് പ്രതിപാദിപ്പിക്കപ്പെട്ടിരിക്കുന്നത്. അതുകൊണ്ടു തന്നെ അവരുടെ പ്രതികരണം വ്യത്യസ്തമായിരിക്കുക സ്വാഭാവികമാണ്. 7:80 മുതല്‍ 7:82 വരെയുള്ള സൂക്തങ്ങളുടെ സാരം പരിശോധിക്കുക: ”ലൂത്വിനെയും (നാം അയച്ചു) അദ്ദേഹം തന്റെ ജനതയോട്, നിങ്ങള്‍ക്കു മുമ്പ് ഒരാളും ചെയ്തിട്ടില്ലാത്ത നീചവൃത്തിക്ക് നിങ്ങള്‍ ചെല്ലുകയോ? എന്നുപറഞ്ഞ സന്ദര്‍ഭം (ഓര്‍ക്കുക). സ്ത്രീകളെ വിട്ട് പുരുഷന്‍മാരുടെ അടുത്ത് തന്നെ നിങ്ങള്‍ കാമവികാരത്തോടെ ചെല്ലുന്നു. അല്ല, നിങ്ങള്‍ അതിരുവിട്ടു പ്രവര്‍ത്തിക്കുന്ന ഒരു ജനതയാകുന്നു. ഇവരെ നിങ്ങളുടെ നാട്ടില്‍ നിന്ന്പുറത്താക്കുക. ഇവര്‍ പരിശുദ്ധിപ്രാപിക്കുന്ന ആളുകളാകുന്നു എന്നുപറഞ്ഞതു മാത്രമായിരുന്നു അദ്ദേഹത്തിന്റെ ജനതയുടെ മറുപടി”.’ പ്രകൃതി വിരുദ്ധ ലൈംഗിക വൃത്തിയില്‍ ഏര്‍പ്പെട്ടിരുന്ന തന്റെ ജനതയോട് ലൂത്ത് (അ) ഒരു തവണ നടത്തിയ ഉല്‍ബോധനവും അതിന് അവരുടെ പ്രതികരണവുമാണ് മുകളിലെ സൂക്തങ്ങളില്‍ പ്രതിപാദിക്കപ്പെട്ടിട്ടുള്ളത്. ഇതില്‍ നിന്ന് തികച്ചും വ്യത്യസ്തമായ ഒരു സാഹചര്യമാണ് 29:28, 29 സൂക്തങ്ങളിലുള്ളതെന്ന് അവയുടെ സാരം പരിശോധിച്ചാല്‍ ബോധ്യമാവും. ”ലൂത്തിനെയും (ദൂതനായി അയച്ചു) തന്റെ ജനതയോട് അദ്ദേഹം ഇപ്രകാരം പറഞ്ഞ സന്ദര്‍ഭം (ശ്രദ്ധേയമാകുന്നു)തീര്‍ച്ചയായും നിങ്ങള്‍ നീചകൃത്യമാണ് ചെയ്തുകൊണ്ടിരിക്കുന്നത്. നിങ്ങള്‍ക്കുമുമ്പ് ലോകരില്‍ ഒരാളും അത് ചെയ്യുകയുണ്ടായിട്ടില്ല. നിങ്ങള്‍ കാമനിവൃത്തിക്കായി പുരുഷന്‍മാരുടെ അടുത്ത് ചെല്ലുകയും(പ്രകൃതിപരമായ) മാര്‍ഗം ലംഘിക്കുകയും നിങ്ങളുടെ സദസ്സില്‍ വെച്ച് നിഷിദ്ധവൃത്തി ചെയ്യുകയുമാണോ? അപ്പോള്‍ അദ്ദേഹത്തിന്റെ ജനതനല്‍കിയ മറുപടി നീ സത്യവാന്‍മാരുടെ കൂട്ടത്തിലാണെങ്കില്‍ ഞങ്ങള്‍ക്ക് അല്ലാഹുവിന്റെ ശിക്ഷ നീ കൊണ്ടുവാ എന്നു മാത്രമായിരുന്നു.” (29:28,29) ലൂത്ത് നബി (അ) രണ്ട് തവണ തന്റെ ജനതയോട് നടത്തിയ ഉല്‍ബോധനങ്ങളാണ് സൂറത്തുല്‍ അഅ്‌റാഫിലും സൂറത്തു അന്‍കബൂത്തിലും പ്രതിപാദിക്കപ്പെട്ടിട്ടുള്ളത്. പ്രസ്തുത ഉല്‍ബോധനങ്ങളിലെ വാചകങ്ങള്‍ വ്യത്യസ്തമായതുപോലെ ജനതയുടെ മറുപടിയും വ്യത്യസ്തമായിരുന്നു. എന്ന് പറഞ്ഞത് മാത്രമായിരുന്നു അദ്ദേഹത്തിന്റെ ജനതയുടെ മറുപടി എന്ന് പറഞ്ഞതില്‍ നിന്ന് ലൂത്ത് നബി (അ) സംസാരിച്ചപ്പോഴെല്ലാം അദ്ദേഹത്തിന്റെ ജനത ഒരേയൊരു വാചകം മാത്രമാണ് മറുപടിയായി ആവര്‍ത്തിച്ച് പറഞ്ഞത് എന്ന് അര്‍ഥമാക്കുന്നില്ല. പ്രകൃതി വിരുദ്ധ രതിയുടെ അധാര്‍മികതയെക്കുറിച്ച് അവരെ ഉണര്‍ത്തിക്കൊണ്ട് അദ്ദേഹം ഉന്നയിച്ച ചോദ്യത്തിന് ശരിയായ മറുപടി പറയാനോ ചിന്തിച്ച് തങ്ങളുടെ നിലപാട് നന്നാക്കി തീര്‍ക്കുവാനോ അവര്‍ സന്നദ്ധരായില്ല. അദ്ദേഹം അവരോട് സംസാരിച്ചപ്പോഴെല്ലാം ധിക്കാരത്തോട് കൂടിയുള്ള പ്രതികരണം മാത്രമാണ് അവരില്‍ നിന്നുണ്ടായത്. ഇക്കാര്യമാണ് ഈ വിഷയം പ്രതിപാദിക്കപ്പെട്ട ഖുര്‍ആന്‍ സൂക്തങ്ങളിലെല്ലാം വ്യക്തമാക്കപ്പെട്ടിട്ടുള്ളത്. ഈ സൂക്തങ്ങള്‍ തമ്മില്‍ യാതൊരു വിധ വൈരുധ്യവുമില്ല. യഥാര്‍ഥ മറുപടിയുടെ അഭാവം സൂചിപ്പിക്കുന്നതിന് വേണ്ടിയാണ് ”പറഞ്ഞത് മാത്രമായിരുന്നു അവരുടെമറുപടി” എന്ന് പറഞ്ഞിരിക്കുന്നത്.
ഖുര്‍ആനിനെപ്പറ്റി തീർച്ചയായും അത് മുന്‍ഗാമികളുടെ വേദഗ്രന്ഥങ്ങളിലുണ്ട് (26:196) എന്നു പറഞ്ഞിട്ടുണ്ടല്ലോ, ഹിബ്രുവിലും ഗ്രീക്കിലും എഴുതപ്പെട്ട പൂര്‍വ്വവേദഗ്രന്ഥങ്ങളില്‍ ഖുര്‍ആന്‍ അടങ്ങിയിട്ടുണ്ട് എന്നുപറയുന്നത് വിഡ്ഢിത്തമല്ലേ ? ഖുര്‍ആന്‍ മുന്‍ഗാമികളുടെ ഗ്രന്ഥങ്ങളിലുണ്ട് എന്ന് പറഞ്ഞതിന്റെ താല്‍പര്യം അത് അതേപോലെതന്നെ പൗരാണിക വേദഗ്രന്ഥങ്ങളിലുണ്ട് എന്നല്ല. ഖുര്‍ആനിലുള്ളതെല്ലാം പൂര്‍വ്വീക വേദങ്ങളിലുണ്ടായിരുന്നുവെങ്കില്‍ പിന്നെ ഖുര്‍ആന്‍ അവതരിപ്പിക്കേണ്ട കാര്യമുണ്ടായിരുന്നില്ല. മുഹമ്മദ് നബി(ﷺ) യെപ്പറ്റി തങ്ങളുടെ പക്കലുള്ള തൗറാത്തിലും ഇഞ്ചീലിലും രേഖപ്പെടുത്തപ്പെട്ടതായി അവര്‍ക്ക് കണ്ടെത്താന്‍ കഴിയുന്ന അക്ഷരജ്ഞാനമില്ലാത്ത പ്രവാചകനായ ദൈവദൂതന്‍(7:157) എന്നു പറഞ്ഞതില്‍ നിന്ന് മുഹമ്മദ് നബിയെ അദ്ദേഹത്തിന്റെ ഭൗതിക രൂപത്തില്‍ തന്നെ പൂര്‍വ്വവേദങ്ങളില്‍ കണ്ടെത്താന്‍ കഴിയുമെന്ന് മനസ്സിലാക്കുകയല്ലല്ലോ ചെയ്യേണ്ടത്. ഇതേ പോലെ തന്നെയാണ് ഖുര്‍ആന്‍ പൂര്‍വ്വിക ഗ്രന്ഥങ്ങളില്‍ അടങ്ങിയിട്ടുണ്ട് എന്നു പറഞ്ഞതിന്റെ താല്പര്യം. അറബിയിലുള്ള ഖുര്‍ആന്‍ അതേപോലെ തന്നെ ഹിബ്രുവിലും അരാമിക്കിലുമുള്ള പൂര്‍വ്വിക ഗ്രന്ഥങ്ങളില്‍ സ്ഥിതി ചെയ്യുന്നു എന്നല്ല ഇതിന്നര്‍ഥം. പ്രത്യുത, ഖുര്‍ആനിന്റെ അടിസ്ഥാന ആശയങ്ങളെല്ലാം പൂര്‍വ്വിക ഗ്രന്ഥങ്ങളില്‍ പറഞ്ഞതു തന്നെയാണെന്ന് വ്യക്തമാക്കുകയാണ് ഈ സൂക്തത്തിലൂടെ അല്ലാഹു ചെയ്യുന്നത്. ഖുര്‍ആന്‍ ഏതെങ്കിലുമൊരു പുത്തന്‍സിദ്ധാന്തം ലോകര്‍ക്കു മുമ്പില്‍ അവതരിപ്പിക്കുകയല്ല ചെയ്യുന്നതെന്നും പൂര്‍വ്വീക ഗ്രന്ഥങ്ങളെല്ലാം പഠിപ്പിച്ചതും എന്നാല്‍ ആ ഗ്രന്ഥങ്ങളുടെ ആളുകളെന്ന് സ്വയം അവകാശപ്പെടുന്നവര്‍ വിസ്മരിച്ചതുമായ ആദര്‍ശങ്ങള്‍ കളങ്കലേശമില്ലാതെ അവതരിപ്പിക്കുന്ന ഗ്രന്ഥമാണ് അതെന്നും പഠിപ്പിക്കുകയാണ് ഈ പുസ്തകത്തിന്റെ ലക്ഷ്യം. അതോടൊപ്പം തന്നെ അന്തിമ വേദഗ്രന്ഥമായ ഖുര്‍ആനിനെപ്പറ്റി പൂര്‍വ്വികഗ്രന്ഥങ്ങളിലെല്ലാം പ്രവചിച്ചിട്ടുണ്ടെന്ന വസ്തുത കൂടി ഈ വാക്യം വ്യക്തമാക്കുന്നുണ്ട്. ഏതായിരുന്നാലും ഖുര്‍ആന്‍ അതേപടി തന്നെ മുന്‍ വേദങ്ങളില്‍ ഉണ്ടായിരുന്നുവെന്ന് ഈ വചനത്തില്‍ പറയുന്നില്ല. അതിനാല്‍ ചോദ്യത്തില്‍ പരാമര്‍ശിച്ച രീതിയിലുള്ള വിഡ്ഢിത്തം ഖുര്‍ആനില്‍ എവിടെയും നമുക്ക് കാണാന്‍കഴിയുന്നുമില്ല. 09.01.2019
പുതിയ വെളിപാട് പഴയ വെളിപാടുകളെ ശരിവെക്കുന്നുവെന്ന് 2:97 ലും പകരം വെക്കുന്നുവെന്ന് 16:101 ലും പറയുന്നു. ഇവ തമ്മില്‍ വൈരുധ്യമില്ലേ? മുന്‍ വേദങ്ങളെയെല്ലാം ശരിവെച്ചുകൊണ്ടാണ് അവസാനത്തെ വേദഗ്രന്ഥമായ ഖുര്‍ആന്‍ അവതരിക്കപ്പെട്ടിട്ടുള്ളത്. ഇക്കാര്യം വ്യക്തമാക്കുന്ന ഒരു ഖുര്‍ആന്‍ സൂക്തം കാണുക: (നബിയേ), നിനക്കിതാ സത്യപ്രകാരം വേദഗ്രന്ഥം അവതരിപ്പിച്ചുതന്നിരിക്കുന്നു. അതിന്റെ മുമ്പിലുള്ള വേദഗ്രന്ഥങ്ങളെ ശരിവെക്കുന്നതും അവയെ കാത്തുരക്ഷിക്കുന്നതുമത്രെ അത് (5:48). ഈ സൂക്തത്തില്‍ മുന്‍ വേദഗ്രന്ഥങ്ങളെ കാത്തു രക്ഷിക്കുന്ന (മുഹൈമിന്‍) ഗ്രന്ഥമായാണ് ഖുര്‍ആനിനെ പരിചയപ്പെടുത്തിയിരിക്കുന്നത്. മനുഷ്യരുടെ കൈകടത്തലുകള്‍ക്ക് വിധേയമായ പൂര്‍വ്വവേദങ്ങളെ അവയുടെ കളങ്കമില്ലാത്ത രൂപത്തില്‍ സംരക്ഷിക്കുന്ന ഗ്രന്ഥമാണ് ഖുര്‍ആന്‍ എന്നര്‍ഥം. പൂര്‍വ്വവേദങ്ങളിലെ അടിസ്ഥാന ആശയങ്ങളോടെല്ലാം ഖുര്‍ആന്‍ യോജിക്കുന്നു. അവ അവതരിക്കപ്പെട്ട രൂപത്തില്‍ പൂര്‍ണമായും ദൈവികമായിരുന്നുവെന്ന് അംഗീകരിക്കുകയും അതുകൊണ്ട് തന്നെ പൂര്‍വ്വവേദങ്ങളെ സത്യപ്പെടുത്തുകയുമാണ് ഖുര്‍ആന്‍ ചെയ്യുന്നത്. പൂര്‍വ്വവേദങ്ങളെ സത്യപ്പെടുത്തുക എന്ന് പറഞ്ഞാല്‍ അവയുടെ പേരില്‍ ഇന്ന് നിലനില്‍ക്കുന്ന ഗ്രന്ഥങ്ങളെ അംഗീകരിക്കുക എന്ന് അര്‍ഥമില്ല. ഇന്ന് നിലനില്‍ക്കുന്ന ഖുര്‍ആനൊഴിച്ചുള്ള വേദഗ്രന്ഥങ്ങളെല്ലാം മാനുഷിക കൃത്രിമങ്ങള്‍ കൊണ്ട് വികൃതമാക്കപ്പെട്ടവയാണ്. അവയിലെ ദൈവികവചനങ്ങളെയും മനുഷ്യവചനങ്ങളേയും വേര്‍തിരിച്ചെടുക്കാന്‍ വയ്യാത്തവണ്ണം അവ കൂടികുഴഞ്ഞിരിക്കുന്നു. അവയുടെ ഒന്നിന്റെയും ഒറിജിനല്‍ പ്രതി ഇന്ന് ലഭ്യമല്ലതാനും. അതുകൊണ്ട് തന്നെ ഖുര്‍ആനിന്റെ അവതരണത്തോടെ പൂര്‍വീക ഗ്രന്ഥങ്ങളെല്ലാം ദുര്‍ബലപ്പെടുത്തപ്പെട്ടിട്ടുണ്ട്. ഈ ദുര്‍ബലപ്പെടുത്തല്‍ പോലും അവയെ കാത്തുരക്ഷിക്കുന്നതിന്റെ ഭാഗമാണ് എന്നുള്ളതാണ് വസ്തുത. മനുഷ്യവചനങ്ങളും ദൈവികവചനങ്ങളും കൂടിക്കലര്‍ന്ന് സ്ഥിതിചെയ്യുന്ന വേദഗ്രന്ഥങ്ങള്‍ യഥാര്‍ഥത്തില്‍ അവയുടെ അവതരണ ഉദ്ദേശ്യത്തിന് വിരുദ്ധമായ ഫലമാണ് ഉളവാക്കുന്നത്. അതിനാല്‍ അവയെ ദുര്‍ബലപ്പെടുത്തുകയും അവയുടെ മൗലികമായ ആശയങ്ങളെ മനുഷ്യകൈകടത്തലുകളില്ലാതെ വ്യക്തമാക്കുകയും ചെയ്യുന്നതിലൂടെ ഖുര്‍ആന്‍ പൂര്‍വ്വവേദങ്ങളിലെ അടിസ്ഥാന ആശയങ്ങളെ സംരക്ഷിക്കുകയാണ് യഥാര്‍ഥത്തില്‍ ചെയ്യുന്നത്. അതിനാല്‍ ഖുര്‍ആന്‍ പൂര്‍വ്വവേദങ്ങളെ ദുര്‍ബലപ്പെടുത്തിയതും സത്യപ്പെടുത്തിയതും അവയെ സംരക്ഷിക്കുന്നതിന്റെ ഭാഗമായാണ്. പുതിയ വെളിപാട് പഴയ വെളിപാടുകളെ സത്യപ്പെടുത്തുകയാണ് ചെയ്യുന്നതെന്ന് പറയുന്ന ഖുര്‍ആന്‍ വാക്യം അവയെ ദുര്‍ബലപെടുത്തുകയാണ് ചെയ്തിട്ടുള്ളതെന്ന വാക്യവുമായി യാതൊരു വിധ വൈരുധ്യവും പുലര്‍ത്തുന്നില്ല. ഒരു വാക്യം മറ്റേ വാക്യത്തിന്റെ വിശദീകരണമായിട്ടാണ് നിലകൊള്ളുന്നത്.
മലക്കുകൾ ദൈവിക കൽപന ധിക്കരിക്കാത്തവരാണെന്ന് 16:49, 50 സൂക്തങ്ങളിൽ പറയുന്നതിന് വിരുദ്ധമായി, ആദമിനെ സാഷ്ടാംഗം നമിക്കാന്‍ മലക്കുകളോട് പറഞ്ഞപ്പോൾ ഇബ്‌ലീസ് വിസമ്മതിച്ചുവെന്ന് 2:34 ല്‍ പറയുന്നു. ഈ വൈരുധ്യം എങ്ങനെ വിശദീകരിക്കാനാവും? ഇബ്‌ലീസ് മലക്കല്ല, പ്രത്യുത ജിന്നുകളില്‍പ്പെട്ടവനാണെന്ന വസ്തുത ഖുര്‍ആന്‍ 18:50 ല്‍ വ്യക്തമാക്കിയിട്ടുണ്ട്. അതുകൊണ്ട് തന്നെ ദൈവികശാസനകള്‍ ചോദ്യം ചെയ്യാതെ അനുസരിക്കുന്നവരാണ് മലക്കുകള്‍ എന്ന വസ്തുതയുമായി ഇബ്‌ലീസിന്റെ അനുസരണക്കേട് യാതൊരുവിധ വൈരുധ്യവും പുലര്‍ത്തുന്നില്ല. മലക്കുകളോട് ആദമിനെ സാഷ്ടാംഗം നമിക്കുവാന്‍ അല്ലാഹുകല്‍പിച്ചപ്പോള്‍ പിന്നെ എന്തിന് ജിന്നുകളില്‍ പെട്ട ഇബ്‌ലീസ് ആ കല്‍പന അനുസരിക്കണം എന്ന ചോദ്യം ഉയരാറുണ്ട്. ഇതിന് ഖുര്‍ആന്‍ വ്യാഖ്യാതാക്കള്‍ മറുപടി പറഞ്ഞിട്ടുണ്ട്. ജിന്നുകളുടെ തലവനായിരുന്ന ഇബ്‌ലീസ് തന്റെ ജ്ഞാനം കൊണ്ടും വിശുദ്ധികൊണ്ടും മലക്കുകളോടൊപ്പമെത്താന്‍ അര്‍ഹത നേടിയെടുത്തിരുന്നു. മലക്കുകള്‍ക്കിടയില്‍ കഴിഞ്ഞു കൂടിയവനെന്ന നിലക്ക് ആദമിന് സാഷ്ടാംഗം നമിക്കാന്‍ വേണ്ടിയുള്ള കൽപന അവന് കൂടി ബാധകമായിരുന്നു. തന്റെ പുത്രന്‍മാരോടൊപ്പം ജീവിക്കുന്ന ദത്തുപുത്രനുള്ള ഒരു പിതാവ് തന്റെ മക്കളോടായി എന്തെങ്കിലും ഒരു കാര്യം കല്പിച്ചാല്‍ ദത്ത് പുത്രന്‍ കൂടി അത്ചെയ്യണം എന്ന് പ്രത്യേകമായി പറഞ്ഞിട്ടില്ലെങ്കിലും പ്രസ്തുത കല്‍പന പാലിക്കുവാന്‍ അയാള്‍ കൂടി ബാധ്യസ്ഥനാണ് എന്നത് പോലെ മലക്കുകളോടൊപ്പം കഴിയാന്‍ അര്‍ഹത നേടിയെടുത്തിരുന്ന ഇബ്‌ലീസ് കൂടി മലക്കുകളോടുള്ള ദൈവിക കല്‍പന അനുസരിക്കുവാന്‍ ബാധ്യസ്ഥനാണ്. വിദേശത്ത് ജീവിക്കുന്ന ഒരാള്‍ താന്‍ ജീവിക്കുന്ന രാജ്യത്തിലെ നിയമങ്ങള്‍അനുസരിക്കുവാന്‍ ബാധ്യസ്ഥനായതിനോടും ഇത് ഉപമിക്കാവുന്നതാണ്. ഏതായിരുന്നാലും ജിന്നുകളില്‍പ്പെട്ട ഇബ്‌ലീസ് പ്രസ്തുത ദൈവിക കല്‍പന ധിക്കരിക്കുകയും മ്ലേച്ഛനായി ദൈവ കോപത്തിന് പാത്രമാവുകയുമാണുണ്ടായത്. ഇതാണ് 2:34 ല്‍ പറഞ്ഞിരിക്കുന്നത്.
യൂനുസ് നബിയെ മത്സ്യം പാഴ്ഭൂമിയിൽ തള്ളിയെന്ന് ഖുർആനിലെ 37:145 വചനത്തിൽ പറയുന്നു. ഇതിൽ നിന്ന് വ്യത്യസ്തമായി പാഴ്ഭൂമിയിൽ അദ്ദേഹം തള്ളപ്പെട്ടിട്ടില്ലെന്ന രീതിയിലാണ് 68:49 ൽ പരാമര്‍ശിച്ചിരിക്കുന്നത്. ഇത് വ്യക്തമായ വൈരുധ്യമല്ലേ? വൈരുധ്യമാരോപിക്കപ്പെട്ടിരിക്കുന്ന സൂക്തങ്ങളുടെ സാരംപരിശോധിക്കുക: എന്നിട്ട് അദ്ദേഹത്തെ (യൂനുസിനെ) അനാരോഗ്യവാനായ നിലയില്‍ തുറന്ന സ്ഥലത്തേക്ക് നാം തള്ളി. അദ്ദേഹത്തിന്റെ മേല്‍ നാം യഖ്ത്വീന്‍വൃക്ഷം മുളപ്പിക്കുകയും ചെയ്തു. അദ്ദേഹത്തെ നാം ഒരു ലക്ഷമോ അതിലധികമോ വരുന്ന ജനവിഭാഗത്തിലേക്ക് നിയോഗിച്ചു. (37:145-147) അദ്ദേഹത്തിന്റെ രക്ഷിതാവിങ്കല്‍ നിന്നുള്ള അനുഗ്രഹം അദ്ദേഹത്തെ വീണ്ടെടുത്തില്ലായിരുന്നെങ്കില്‍ അദ്ദേഹം ആ പാഴ്ഭൂമിയില്‍ ആക്ഷേപാര്‍ഹനായികൊണ്ട് പുറന്തള്ളപ്പെടുമായിരുന്നു. അപ്പോള്‍ അദ്ദേഹത്തിന്റെ രക്ഷിതാവ് അദ്ദേഹത്തെ തെരഞ്ഞെടുക്കുകയും അദ്ദേഹം സജ്ജനങ്ങളുടെ കൂട്ടത്തിലാവുകയും ചെയ്തു. (68:49, 50) യൂനുസ് നബി(അ)യെ വിഴുങ്ങിയ മത്സ്യം ഏതാനും ദിവസങ്ങള്‍ക്കുശേഷം അദ്ദേഹത്തെ കരയിലേക്ക് ഛര്‍ദ്ദിച്ചതുമായി ബന്ധപ്പെട്ട വിവരണങ്ങളുള്‍കൊള്ളുന്ന സൂക്തങ്ങളാണ് ഇവ. മത്സ്യം അദ്ദേഹത്തെ കരയിലേക്കിടുമ്പോള്‍ വളരെ വിഷമം പിടിച്ച അവസ്ഥയിലായിരുന്നു അദ്ദേഹം. രണ്ടു മൂന്ന് ദിവസം മത്സ്യത്തിന്റെ വയറ്റില്‍ കഴിച്ചുകൂട്ടേണ്ടിവന്ന അദ്ദേഹത്തെ അവശനും രോഗാതുരനുമായിക്കൊണ്ടാണ് പാഴ്ഭൂയിലേക്ക് മത്സ്യം പുറന്തള്ളിയത്. ആ അവസ്ഥയില്‍ അദ്ദേഹത്തിന് നല്‍കപ്പെട്ട പ്രത്യേകമായ ദൈവികാനുഗ്രഹങ്ങളാണ് സൂറത്തൂസ്വാദിലേയും(37:145-147) സൂറത്തു ഖലമിലെയും (68:49, 50) പരാമര്‍ശിക്കപ്പെട്ട വചനങ്ങളില്‍ വ്യക്തമാക്കപ്പെട്ടിരിക്കുന്നത്. അദ്ദേഹത്തിന്റെ അനാരോഗ്യത്തെ അതിജീവിക്കുവാനാവശ്യമായ ഫലങ്ങളുല്‍പാദിപ്പിക്കുന്ന യഖ്ത്വീന്‍ വൃക്ഷത്തെ യൂനുസ് നബി(അ) ഉപേക്ഷിക്കപ്പെട്ട സ്ഥലത്തുതന്നെ മുളപ്പിക്കുകയും അങ്ങനെ ആരോഗ്യം വീണ്ടെടുത്ത അദ്ദേഹത്തെ വീണ്ടും പ്രവാചകനായി നിയോഗിക്കുകയും ചെയ്ത അല്ലാഹുവിന്റെ അനുഗ്രഹത്തെയാണ് സൂറത്തു സ്വാദിലെ വചനങ്ങളില്‍ സൂചിപ്പിച്ചിരിക്കുന്നത്. ഇങ്ങനെ അനുഗ്രഹങ്ങള്‍ നല്‍കി സംരക്ഷിച്ചിട്ടില്ലായിരുന്നുവെങ്കില്‍ ആ പാഴ്ഭൂമിയില്‍ ആരാരുമറിയാതെ ആക്ഷേപാര്‍ഹനായി യൂനുസ് (അ) പുറംതള്ളപ്പെടുമായിരുന്നുവെന്ന യാഥാര്‍ഥ്യം വ്യക്തമാക്കുകയാണ് സൂറത്തുല്‍ ഖലമിലെ വചനങ്ങള്‍ ചെയ്യുന്നത്. ഇവ തമ്മില്‍ യാതൊരു വൈരുധ്യവുമില്ല. ഒരേ കാര്യം തന്നെ രണ്ട് രൂപത്തില്‍ പറഞ്ഞിരിക്കുന്നുവെന്ന് മാത്രമേയുള്ളൂ.
ഇസ്‌റാഈല്യര്‍ കാളക്കുട്ടിയുടെ വിഗ്രഹമുണ്ടാക്കിയ സംഭവവുമായി ബന്ധപ്പെട്ട ഖുര്‍ആന്‍ സൂക്തങ്ങളില് പ്രസ്തുത പ്രവര്ത്തനത്തില് ഹാറൂനിന് പങ്കുണ്ടായിരുന്നില്ലെന്ന് പറയുന്ന സൂക്തങ്ങളും (20;85-90) പങ്കുണ്ടായിരുന്നെന്ന് സൂചിപ്പിക്കുന്ന സൂക്തങ്ങളു(20:92, 7:151)മുണ്ട്. ഇതൊരു വൈരുധ്യമല്ലേ ? ഹാറൂന്‍(അ) അല്ലാഹുവിന്റെ പ്രവാചകനാണ്. പ്രവാചകന്‍മാരെല്ലാം പാപസുരക്ഷിതരാണെന്നാണ് മുസ്‌ലിംകള്‍ വിശ്വസിക്കുന്നത്. വിഗ്രഹനിര്‍മ്മാണവും ആരാധനയുമെല്ലാം പാപങ്ങളില്‍ ഏറ്റവും ഗുരുതരമായ ശിര്‍ക്കിലുള്‍പ്പെടുന്നവയാണ്. അത് ചെയ്തവന്‍ ഇസ്‌ലാമില്‍ നിന്ന് പുറത്താണ്. ഇസ്‌റാഈല്യരില്‍പ്പെട്ട സാമിരിയുടെ വിഗ്രഹനിര്‍മ്മാണത്തിലോ അതിനുള്ള ആരാധനയിലോ ഹാറൂനി(അ)ന് എന്തെങ്കിലും പങ്കുണ്ടായിരുന്നതായി ഖുര്‍ആനില്‍ ഒരിടത്തും യാതൊരു വിധസൂചനയും നല്‍കുന്നില്ല. ദൈവത്താല്‍ നിയോഗിക്കപ്പെട്ട ഒരു പരിശുദ്ധപ്രവാചകനില്‍നിന്ന് അത്തരമൊരു അക്ഷന്തവ്യമായ പാപംവന്നുഭവിച്ചുവെന്ന് ചിന്തിക്കുക പോലും ചെയ്യാന്‍ മുസ്‌ലിംകള്‍ക്കാവില്ല.എന്നാല്‍ പഴയ നിയമത്തില്‍ പറയുന്നത്, വിഗ്രഹം നിര്‍മ്മിച്ചതും അതിനുള്ളആരാധനകള്‍ നിര്‍വഹിക്കാന്‍ നേതൃത്വം നല്‍കിയതും അഹറോന്‍ആണെന്നാണ് (പുറപ്പാട് 32:16). ഇതില്‍ നിന്ന് തികച്ചും വ്യത്യസ്തമായി,വിഗ്രഹം നിര്‍മ്മിച്ചതും അതിനുള്ള ആരാധനകള്‍ക്ക് നേതൃത്വം നല്‍കിയതുംസാമിരിയാണെന്നും ഇസ്‌റാഈല്യര്‍ സാമിരിയുടെ പിന്നില്‍ അണിനിരന്നപ്പോള്‍ എന്റെ ജനങ്ങളെ, ഇതു (കാളക്കുട്ടി)മൂലം നിങ്ങള്‍പരീക്ഷിക്കപ്പെടുക മാത്രമാണുണ്ടായത്. തീര്‍ച്ചയായും നിങ്ങളുടെ രക്ഷിതാവ്പരമകാരുണികനത്രെ. അതുകൊണ്ട് നിങ്ങള്‍ എന്നെ പിന്‍തുടരുകയുംഎന്റെ കല്‍പ്പനകള്‍ അനുസരിക്കുകയും ചെയ്യുക(20:90) എന്ന് ഉപദേശിച്ച്ഹാറൂന്‍ അവരെ നേര്‍മാര്‍ഗത്തിലേക്ക് കൊണ്ടു വരാന്‍ ശ്രമിച്ചുവെന്നുമാണ്ഖുര്‍ആന്‍ വ്യക്തമാക്കുന്നത്. എന്നാല്‍ മൂസാ നബി(അ)യെ പോലെ, അത്രക്ക് നേതൃപാടവമോ ജനത്തെനിയന്ത്രിക്കുവാനുള്ള കഴിവോ ഹാറൂനി(അ) നില്ലായിരുന്നു. അതിനാല്‍ജനങ്ങളെല്ലാം കാളപൂജകരായിത്തീരുകയും അവരെ ഉപദേശിച്ചിട്ട്ഫലമൊന്നുമില്ലെന്ന് അനുഭവപ്പെടുകയും ചെയ്തപ്പോള്‍ അതിശക്തമായനടപടികളൊന്നും എടുക്കാതെ അദ്ദേഹം നിശബ്ദനായി. ഈയൊരുഅവസ്ഥയിലാണ് മൂസാ (അ) ഇസ്‌റാഈല്യര്‍ക്കിടയിലേക്ക്മടങ്ങിയെത്തുന്നത്. അവിടെയുണ്ടായിരുന്ന അവസ്ഥ കണ്ടപ്പോള്‍അദ്ദേഹത്തിന് ദു:ഖവും കോപവും ഉണ്ടായി. ഏകദൈവാദര്‍ശത്തിനുവേണ്ടി നിലകൊള്ളേണ്ട തന്റെ ജനതയാകെഗോപൂജകരായിത്തീര്‍ന്നിരിക്കുന്നു; അവരെ നയിക്കുകയുംനിയന്ത്രിക്കുകയും ചെയ്യേണ്ട ചുമതലയേല്‍പ്പിക്കപ്പെട്ടിരുന്ന ഹാറൂനാകട്ടെ,ശക്തമായ നടപടികളൊന്നുമെടുക്കാതെ നിശബ്ദനായി നില്‍ക്കുകയുമാണ്. ഇത്കണ്ട് കലികയറിയ മൂസാ (അ) ഹാറൂനി (അ) ന്റെ താടിയിലും തലയിലുംപിടിച്ച് ചോദ്യം ചെയ്യുന്നതാണ് സൂറത്തു ത്വാഹായിലെ 92 മുതല്‍ 94വരെയുള്ള വചനങ്ങളില്‍ വിശദമാക്കിയിട്ടുള്ളത്. ഹാറൂനേ, ഇവര്‍ പിഴച്ചുപോയതായി നീ കണ്ടപ്പോള്‍ എന്നെപിന്‍തുടരാതിരിക്കാന്‍ നിനക്ക് എന്തു തടസ്സമാണുണ്ടായിരുന്നത്? നീ എന്റെകല്‍പനയ്ക്ക് എതിരു പ്രവര്‍ത്തിക്കുകയാണോ ചെയ്തത്. (20:92,93)എന്നാണ് മൂസാ (അ) ഹാറൂനി (അ)നോട് ചോദിക്കുന്നത്. ശക്തമായനടപടികളെടുക്കാതെയും അനുയായികളെ ശരിക്ക് നിയന്ത്രിക്കാതെയും നിന്നഹാറൂനി (അ) നെ വിമര്‍ശിക്കുകയാണ് ഇവിടെ മൂസാ(അ) ചെയ്യുന്നത്.ഈവചനത്തില്‍ ഇസ്‌റാഈല്യരോടൊപ്പം ഹാറൂനും (അ) വിഗ്രഹാരാധനയില്‍പങ്കുചേര്‍ന്നുവെന്ന ആശയത്തിന്റെ ലാഞ്ഛന പോലുമില്ല. മൂസാ (അ) യുടെചോദ്യത്തി ന് അദ്ദേഹം നല്‍കുന്ന മറുപടിയില്‍ ഇക്കാര്യം അല്പം കൂടിസ്പഷ്ടമാണ്. ഇസ്‌റാഈല്‍ സന്തതികള്‍ക്കിടയില്‍ നീഭിന്നിപ്പുണ്ടാക്കിക്കളഞ്ഞു, എന്റെ വാക്കിനു നീ കാത്തുനിന്നില്ല എന്ന് നീപറയുമെന്ന് ഞാന്‍ ഭയപ്പെടുകയാണുണ്ടായത്(20:94) എന്ന് സൂറത്തുത്വാഹായി ലും എന്റെ ഉമ്മയുടെ മകനേ, ജനങ്ങള്‍ എന്നെ ദുര്‍ബലനായിഗണിച്ചു. അവരെന്നെ കൊന്നേക്കുമായിരുന്നു. അതിനാല്‍ (എന്നോട്കയര്‍ത്തുകൊണ്ട്) നീ ശത്രുക്കള്‍ക്ക് സന്തോഷത്തിന് ഇടവരുത്തരുത്.അക്രമികളായ ജനങ്ങളുടെ കൂട്ടത്തില്‍ എന്നെ കണക്കാക്കുകയും ചെയ്യരുത്(7:150) എന്നു സൂറത്തുഅഅ്റാഫിലും ഹാറൂനി (അ)ന്റെ മറുപടിഉദ്ധരിച്ചിട്ടുണ്ട്. ഇവയില്‍ നിന്ന് ഹാറൂന്‍(അ) കൂടി കാളകുട്ടി പൂജയില്‍പങ്കാളിയായതുകൊണ്ടല്ല മൂസാ(അ) അദ്ദേഹത്തെ വിമര്‍ശിച്ചതെന്ന് സുതരാംവ്യക്തമാവുന്നുണ്ട്. സൂറത്തുല്‍ അഅറാഫിലെ (7:151) മൂസാനബി (അ)യുടെ പ്രാര്‍ഥനയില്‍എന്റെ രക്ഷിതാവേ, എനിക്കും എന്റെ സഹോദരനും നീ പൊറുത്തുതരികയും, ഞങ്ങളെ നീ നിന്റെ കാരുണ്യത്തില്‍ ഉള്‍പ്പെടുത്തുകയുംചെയ്യേണമേ എന്ന് പറഞ്ഞതില്‍ നിന്ന് ഹാറൂന്‍(അ) വിഗ്രഹാരാധന നടത്തിപാപിയായി തീര്‍ന്നുവെന്ന് വ്യക്തമാകുന്നു വെന്നാണ്, ഖുര്‍ആനില്‍വൈരുധ്യങ്ങള്‍ ആരോപിക്കുന്നവരുടെ വാദം. ഇത് അടിസ്ഥാനരഹിതമായഒരു വാദമെന്നതില്‍ കവിഞ്ഞ യാതൊന്നുമല്ല. ഹാറൂന്‍(അ) കാളകുട്ടിയെ പൂജചെയ്തുവെന്ന് ഈ വചനത്തില്‍ എവിടെയുമില്ല. പാപമോചനത്തിനുവേണ്ടിയുള്ള പ്രാര്‍ഥന വിശ്വാസികളുടെ ജീവിതത്തില്‍എപ്പോഴുമുണ്ടാകുന്നതാണ്. ഇവിടെയാകട്ടെ, ശക്തമായ നടപടിയെടുക്കാത്തഹാറൂന്‍(അ) ന്റെ നിലപാട് തെറ്റാണെന്ന് തന്നെയാണ് മൂസാ (അ)യുടെഅഭിപ്രായം. മാത്രവുമല്ല, എനിക്കും സഹോദരന്നും പൊറുത്തു തരണം എന്നമൂസാ (അ) യുടെ പ്രാര്‍ഥനയില്‍ നിന്ന് ഹാറൂന്‍ വിഗ്രഹപൂജ നടത്തിയെന്ന്അനുമാനിക്കുകയാണെങ്കില്‍ മൂസാ(അ)യും പ്രസ്തുത തെറ്റ് ചെയ്തുവെന്ന്കൂടി സമ്മതിക്കേണ്ടിവരും. അങ്ങനെ ആരും പറയുന്നില്ലല്ലോ. അതിനാല്‍ ഈപ്രാര്‍ഥനയില്‍ നിന്ന് ഹാറൂന്‍(അ) കാളപൂജക്കു കൂട്ടു നിന്നുവെന്നനിഗമനത്തിലെത്തി ഖുര്‍ആനിലെ മറ്റു സൂക്തങ്ങളുമായി വൈരുധ്യംആരോപിക്കുന്നതിന്ന് യുക്തിയുടെ പിന്‍ബലമില്ല; പ്രമാണങ്ങളുടെഅടിത്തറയുമില്ല.
ഇസ്റാഈല്യർ കാളകുട്ടിയുടെ വിഗ്രഹമുണ്ടാക്കി അതിനെ ആരാധിക്കുകയും പിന്നീടതില്‍ പശ്ചാത്തപിച്ചു മടങ്ങുകയും ചെയ്തതായി ഖുര്‍ആനില്‍ പറയുന്നു. അവർ പശ്ചാത്തപിച്ചത് എപ്പോഴാണ്? മോശ സീനായില്‍ നിന്ന് മടങ്ങുന്നതിന് മുമ്പ് തന്നെ പശ്ചാത്തപിച്ചിട്ടുണ്ടെന്ന് 7:149 ല്‍ പറയുന്നതിന് വിരുദ്ധമായി മടങ്ങി വന്നതിന് ശേഷമാണ് പശ്ചാത്തപിച്ചതെന്നാണ് 20:91 ലുള്ളത്. മൂസാ (അ) തൗറാത്ത് സ്വീകരിക്കുന്നതിന് വേണ്ടി സീനായ് പര്‍വ്വതത്തിലേക്ക് പോയപ്പോള്‍ ഇസ്‌റാഈല്യരുടെ നേതൃത്വം സഹോദരനായ ഹാറൂനെ (അ) ഏല്‍പ്പിച്ചതും അദ്ദേഹത്തിന്റെ വിലക്ക് ലംഘിച്ചുകൊണ്ട് സാമിരി കാളക്കുട്ടിയുടെ വിഗ്രഹമുണ്ടാക്കുകയും ഇസ്‌റാഈല്യര്‍ അതിനെ ആരാധിക്കുവാനാരംഭിക്കുകയും ചെയ്തതും മൂസ(അ) വന്ന ശേഷം ഹാറൂനി(അ)നോടും സാമിരിയോടും ഇസ്‌റാഈല്യരോടും കോപിച്ചതും അപ്പോള്‍ അവര്‍ പാശ്ചാത്തപിച്ചതുമായ സംഭവങ്ങള്‍ ഒന്നിനുപിറകെ ഒന്നായി ചരിത്രവിവരണത്തിന്റെ രീതിയില്‍ വിശദീകരിക്കപ്പെട്ടിരിക്കുന്നത് സൂറത്തു ത്വാഹ (20:83-98) യിലാണ്. ഈ വിവരണത്തിന് വിരുദ്ധമായി ഖുര്‍ആനില്‍ ഒരിടത്തും ഈ ചരിത്രം വിശദീകരിച്ചിട്ടില്ല. മൂസാ (അ) സീനായ് പര്‍വ്വതത്തില്‍ നിന്ന് മടങ്ങിവരുന്നതിന് മുമ്പ് തന്നെ ഇസ്‌റാഈല്യര്‍ പശ്ചാത്തപിച്ചു മടങ്ങിയിരുന്നുവെന്ന് ഖുര്‍ആനിലൊരിടത്തും പരാമര്‍ശിക്കുന്നില്ലെന്നതാണ് സത്യം. സൂറത്തുല്‍ അഅ്‌റാഫിലെ വൈരുധ്യമാരോപിക്കപ്പെട്ട സൂക്തങ്ങളിലെവിടെയും (7:149) മൂസാ(അ)യുടെ ആഗമനത്തിനു മുമ്പ് തന്നെ ഇസ്‌റാഈല്യര്‍ പാശ്ചാത്തപിച്ചു മടങ്ങിയെന്ന് പറഞ്ഞിട്ടില്ല. 7:149 ല്‍ഇസ്‌റാഈല്യരുടെ പശ്ചാത്താപത്തെ കുറിച്ച് പരാമര്‍ശിച്ച ശേഷം അടുത്തവചനത്തില്‍ മൂസാ(അ)യുടെ സീനായില്‍ നിന്നുള്ള ആഗമനത്തെ കുറിച്ചു പറഞ്ഞിരിക്കുന്നുവെന്നത് ശരിയാണ്. ഇതിന്നര്‍ഥം ഇസ്‌റാഈല്യര്‍ പശ്ചാത്തപിച്ച ശേഷമാണ് മൂസാ(അ) മടങ്ങി വന്നതെന്നാണെന്ന് പ്രസ്തുത ഖുര്‍ആന്‍ സൂക്തങ്ങള്‍ മനസ്സിരുത്തി വായിച്ച ആരും പറയില്ല. സൂറത്തുല്‍ അഅ്‌റാഫിലെ 148 മുതല്‍ 150 വരെയുള്ള സൂക്തങ്ങളുടെ സാരം നോക്കുക. മൂസായുടെ ജനത അദ്ദേഹം പോയശേഷം അവരുടെ ആഭരണങ്ങള്‍ കൊണ്ടുണ്ടാക്കിയ മുക്രയിടുന്ന ഒരു കാളക്കുട്ടിയുടെ സ്വരൂപത്തെ ദൈവമായി സ്വീകരിച്ചു. അതവരോട് സംസാരിക്കുകയില്ലെന്നും അവര്‍ക്ക് വഴികാണിക്കുകയില്ലെന്നും അവര്‍ കണ്ടില്ലേ? അതിനെ അവര്‍ (ദൈവമായി)സ്വീകരിക്കുകയും അതോടെ അവര്‍ അക്രമികളാവുകയും ചെയ്തിരിക്കുന്നു(148). അവര്‍ക്ക് ഖേദം തോന്നുകയും തങ്ങള്‍ പിഴച്ചുപോയിരിക്കുന്നുവെന്ന് അവര്‍ കാണുകയും ചെയ്തപ്പോള്‍ അവര്‍ പറഞ്ഞു: ഞങ്ങളുടെ രക്ഷിതാവ് ഞങ്ങളോട് കരുണ കാണിക്കുകയും ഞങ്ങള്‍ക്ക് പൊറുത്തുതരികയും ചെയ്തിട്ടില്ലെങ്കില്‍ തീര്‍ച്ചയായും ഞങ്ങള്‍ നഷ്ടക്കാരില്‍ പെട്ടവരായിരിക്കും (149). കുപിതനും ദു:ഖിതനുമായിക്കൊണ്ട് തന്റെ ജനങ്ങളിലേക്ക് മടങ്ങി വന്ന് മൂസ പറഞ്ഞു: ഞാന്‍ പോയശേഷം എന്റെ പിന്നില്‍ നിങ്ങള്‍ പ്രവര്‍ത്തിച്ചകാര്യം വളരെ ചീത്തതന്നെ! നിങ്ങളുടെ രക്ഷി താവിന്റെ കല്‍പ്പന കാത്തിരിക്കാതെ നിങ്ങള്‍ ധൃതികാട്ടിയോ? അദ്ദേഹം പലകകള്‍ താഴെയിടുകയും തന്റെ സഹോദരന്റെ തലപിടിച്ച് തന്റെയടുത്തേക്ക് വലിക്കുകയും ചെയ്തു…. (150) ഇവിടെ 148, 149 സൂക്തങ്ങളില്‍ മൂസാ(അ) യുടെ യാത്രയ്ക്ക് ശേഷം ഇസ്‌റാഈല്യരിലുണ്ടായ മാര്‍ഗഭ്രംശത്തെയും അതില്‍ നിന്ന് അവര്‍ പശ്ചാത്തപിച്ച് മടങ്ങയതിനെയും കുറിച്ചാണ് പ്രതിപാദിക്കുന്നത്. 150-ാംസൂക്തത്തില്‍ മൂസാ(അ) മടങ്ങി വന്നപ്പോള്‍ അദ്ദേഹത്തിനുണ്ടായ ദു:ഖവും ദേഷ്യവും വിവരിക്കുകയും നേതൃത്വമേല്‍പ്പിച്ചിരുന്ന സഹോദരനോട് കയര്‍ക്കുന്നതിന്റെ ചിത്രം വരച്ചുകാണിക്കുകയുമാണ് ചെയ്യുന്നത്. ഇസ്‌റാഈല്യരുടെ വഴികേടിനെക്കുറിച്ച് പറഞ്ഞയുടനെ തന്നെ അവരുടെ പശ്ചാത്താപത്തെ കുറിച്ചു പരാമര്‍ശിച്ചുവെന്ന് മാത്രമേയുള്ളൂ. ശേഷം മൂസാ(അ)യുടെ ആഗമനവും അതോടനുബന്ധിച്ചുണ്ടായ സംഭവങ്ങളും വിശദീകരിക്കുന്നുമുണ്ട്. ഇത് ഖുര്‍ആനില്‍ പലപ്പോഴും കാണാന്‍ കഴിയുന്ന പൊതുവായുള്ള ഒരു ശൈലിയാണ്. ഗുണപാഠത്തിനനുസരിച്ച് വിഷയക്രമത്തില്‍ കാര്യങ്ങള്‍ വിശദീകരിക്കുന്നിടത്ത് സംഭവങ്ങളുടെ കാലക്രമം പാലിക്കണമെന്ന് ഖുര്‍ആന്‍ നിഷ്‌കര്‍ഷിക്കാറില്ല. എന്നാല്‍ ഒരു സംഭവത്തിന്ന് ശേഷമോ മുമ്പോ ആണ് മറ്റൊരു സംഭവമെന്ന് ഖുര്‍ആനില്‍ വ്യക്തമാക്കിയേടത്തൊന്നും ചരിത്ര വസ്തുതകള്‍ക്ക് വിരുദ്ധമായ പരാമര്‍ശങ്ങള്‍ കാണപ്പെടുന്നേയില്ല.
ഉയിർത്തെഴുന്നേല്പ്പിന്റെ നാളില്‍ ആരും പരസ്പരം അന്വേഷിക്കുകയില്ലെന്ന 23:101 ലെ പ്രസ്താവനയക്ക് വിരുദ്ധമായി അവിടെ നടക്കുന്ന അന്വേഷണങ്ങളെപ്പറ്റി 52:25, 37:27 എന്നീ സൂക്തങ്ങളില്‍ പരാമർശിക്കപ്പെട്ടത് കാണാം. ഇവയെ എങ്ങനെ വിശദീകരിക്കും? എന്നിട്ട് കാഹളത്തില്‍ ഊതപ്പെട്ടാല്‍. അന്ന് അവര്‍ക്കിടയില്‍ കുടുംബബന്ധങ്ങളൊന്നും ഉണ്ടായിരിക്കുകയില്ല. അവര്‍ അന്യോന്യം അന്വേഷിക്കുകയുമില്ല. (23:101) പരസ്പരം പലതും ചോദിച്ചുകൊണ്ട് അവരില്‍ ചിലര്‍ ചിലരെ അഭിമുഖീകരിക്കും. (52:25) അവരില്‍ ചിലര്‍ ചിലരുടെ നേരെ തിരിഞ്ഞ് പരസ്പരം ചോദ്യംചെയ്യും. (37:27) വൈരുധ്യം ആരോപിക്കപ്പെട്ടിരിക്കുന്ന മൂന്ന് സൂക്തങ്ങളാണ് ഇവ. അന്ത്യനാളുമായി ബന്ധപ്പെട്ടിരിക്കുന്ന സംഭവങ്ങള്‍ തന്നെയാണ് ഇതുമൂന്നിലും പ്രതിപാദിക്കപ്പെട്ടിരിക്കുന്നത്. എന്നാല്‍ ഇവ മൂന്നും മൂന്നുസമയത്ത് സംഭവിക്കുന്ന കാര്യങ്ങളാകുന്നു. ഉയിര്‍ത്തെഴുന്നേല്‍പ്പിന്റെ ദിവസമാണ് സൂറത്തുല്‍ മുഅ്മിനൂനി (23:101)ല്‍ പ്രതിപാദിക്കപ്പെട്ടിരിക്കുന്നത്. കാഹളത്തില്‍ ഊതപ്പെട്ടശേഷം, ആത്മാവും ജഡവും തമ്മില്‍ കൂടിച്ചേര്‍ന്നു കഴിഞ്ഞ ശേഷമുള്ള വെപ്രാളത്തെയാണ് ഈസൂക്തം ചിത്രീകരിച്ചിരിക്കുന്നത്. ഓരോരുത്തരും അവനവന്റെ കാര്യത്തില്‍മാത്രം ചിന്താകുലനാകുന്ന സമയമാണത്. കുടുംബ ബന്ധത്തെക്കുറിച്ച വിചാരമോ ബന്ധുമിത്രാദികളെക്കുറിച്ച അന്വേഷണമോ ഇല്ലാതെ തന്റെ രക്ഷയുടെ മാര്‍ഗമെന്തെന്ന് വെപ്രാളപ്പെടുന്ന മനുഷ്യന്റെ നിസ്സഹായാവസ്ഥയാണ് ഇവിടത്തെ പ്രതിപാദ്യം. സ്വര്‍ഗ പ്രവേശത്തിന് അര്‍ഹരായിത്തീരുന്ന സച്ചരിതന്‍മാര്‍ അനുഭവിക്കുന്ന അനുഗ്രഹങ്ങളാണ് സൂറത്തു ഖാഫിലെ (52:25) പ്രതിപാദ്യം. സ്വര്‍ഗവാസികള്‍ നിശ്ശബ്ദരായിരിക്കുകയല്ല, പരസ്പരം കുശലാന്വേഷണങ്ങള്‍ നടത്തുകയും സംസാരിക്കുകയും ചെയ്യുമെന്നാണ് ഈ സൂക്തം വ്യക്തമാക്കുന്നത്. സൂറത്തു സ്വാഫ്ഫാത്തിലെ വചനത്തിലാകട്ടെ (37:27), നരകവാസികളുടെ പരസ്പരമുള്ള ചോദ്യം ചെയ്യലാണ് ചിത്രീകരിച്ചിരിക്കുന്നത്. നിങ്ങളുടെ നിര്‍ബന്ധം കൊണ്ടാണ് തങ്ങള്‍ ദുര്‍മാര്‍ഗം സ്വീകരിച്ചതെന്ന് ഒരു വിഭാഗം നരകവാസികള്‍ മറ്റൊരു വിഭാഗത്തോട് പറയുകയും, അപ്പോള്‍ മറ്റേവിഭാഗം അത് നിഷേധിക്കുകയും ചെയ്യുമെന്ന കാര്യം വ്യക്തമാക്കുകയാണ് ഈ സൂക്തത്തില്‍ ചെയ്യുന്നത്. മൂന്ന് സൂക്തങ്ങളിലും പ്രതിപാദിക്കപ്പെട്ടിരിക്കുന്നത് മൂന്ന് സംഭവങ്ങള്‍. ഒന്ന്, ഉയര്‍ത്തെഴുന്നേല്‍പ്പിന്റെ ദിവസത്തിലെ വെപ്രാളം. രണ്ട്, സ്വര്‍ഗത്തിലെ സന്തോഷദായകമായ സംഭാഷണം. മൂന്ന്, നരകത്തിലെ പരസ്പരമുള്ള കുറ്റപ്പെടുത്തല്‍. പിന്നെയെങ്ങനെ ഈ മൂന്ന് സൂക്തങ്ങളും തമ്മില്‍ വൈരുധ്യമുണ്ടാകും?
മനുഷ്യന്‍ മരണപ്പെടുമ്പോള്‍ അവന്റെ ആത്മാവ് പിടിക്കുന്നത് മരണത്തിന്റെ മാലാഖയാണെന് 32:11ലും മാലാഖമാരാണെന്ന് 47:27ലും അല്ലാഹു തന്നെയാണെന്ന് 39:42 ലും പറയുന്നുണ്ടല്ലോ. ഈ സൂക്തങ്ങളിൽ വ്യക്തമായ വൈരുധ്യമില്ലേ? (നബിയേ) പറയുക: നിങ്ങളുടെ കാര്യത്തില്‍ ഏല്‍പ്പിക്കപ്പെട്ട മരണത്തിന്റെ മലക്ക് നിങ്ങളെ മരിപ്പിക്കുന്നതാണ്. പിന്നീട് നിങ്ങളുടെ രക്ഷിതാവിങ്കലേക്ക് നിങ്ങള്‍ മടക്കപ്പെടുന്നതാണ്. (32:11) അപ്പോള്‍ മലക്കുകള്‍ അവരുടെ മുഖത്തും പിന്‍ഭാഗത്തും അടിച്ചുകൊണ്ട് അവരെ മരിപ്പിക്കുന്ന സന്ദര്‍ഭത്തില്‍ എന്തായിരിക്കും അവരുടെ സ്ഥിതി? (47:27) ആത്മാവുകളെ അവരുടെ മരണവേളയില്‍ അല്ലാഹു പൂര്‍ണ്ണമായിഏറ്റെടുക്കുന്നു. മരണപ്പെടാത്തവയെ അവരുടെ ഉറക്കത്തിലും. (39:42) ഈ സൂക്തങ്ങളിലാണ് വൈരുധ്യം ആരോപിക്കപ്പെട്ടിരിക്കുന്നത്. മരണസമയത്തെ സംഭവങ്ങളുമായി ബന്ധപ്പെട്ട സൂക്തങ്ങളാണ് ഇവ. അല്ലാഹുവാണ് ജീവിതവും മരണവുമെല്ലാം നിശ്ചയിക്കുന്നത്. മനുഷ്യന് ജീവന്‍ നല്‍കിയതും മരണത്തിലേക്ക് നയിക്കുന്നതുമെല്ലാം അല്ലാഹുവാണ്. എല്ലാ പ്രാപഞ്ചിക പ്രതിഭാസങ്ങളും നടക്കുന്നത് അല്ലാഹുവിന്റ അലംഘനീയമാ നിയമങ്ങളുടെ അടിസ്ഥാനത്തിലാണ്. എന്നാല്‍ ഇവയെല്ലാം നടന്നു പോകുന്നതിന്നായി അവന്‍ തന്നെ ചിലവ്യവസ്ഥകള്‍ വെച്ചിട്ടുണ്ട്. മഴ പെയ്യിക്കുന്നതിനും ഇടിമിന്നലുണ്ടാകുന്നതിനുമെല്ലാം അല്ലാഹുഭൂമിയില്‍ ചില വ്യവസ്ഥകളേര്‍പ്പെടുത്തിയതുപോലെ മനുഷ്യനെ മരണപ്പെടുത്തുന്നതിനും അവന്‍ ചില സംവിധാനങ്ങള്‍ ചെയ്തിട്ടുണ്ട്. മലക്കുകളിലൂടെയാണ് അല്ലാഹു മനുഷ്യരെ മരണപ്പെടുത്തുന്നത്. മലക്കുകളാണ് മനുഷ്യരെ മരണപ്പെടുത്തുന്നതെങ്കിലും അതിന്റെ ആത്യന്തികമായ കാരണക്കാരന്‍ അല്ലാഹുവാണെന്നര്‍ത്ഥം. അല്ലാഹുവിന്റെ തീരുമാനങ്ങള്‍ നടപ്പാക്കാന്‍ മാത്രമേ മലക്കുകള്‍ക്ക് അധികാരമുള്ളൂ. ഇക്കാര്യമാണ് സൂറത്തു സുമറിലെ (39:42) വാക്യം വ്യക്തമാക്കുന്നത്. മനുഷ്യരേ മലക്ക് മരണപ്പെടുത്തിയ ശേഷം നിങ്ങളുടെ രക്ഷതാവിങ്കലേക്ക് നിങ്ങള്‍ മടക്കപ്പെടുന്നതാണ് എന്ന സൂറത്തു സജദയിലെ (32:11) പരാമര്‍ശവും വ്യക്തമാക്കുന്നത് മരണത്തിലെ അല്ലാഹുവിന്റെ ആത്യന്തികമായ നിയന്ത്രണത്തെയാണ്. മനുഷ്യരെ മരണപ്പെടുത്തുക എന്ന ചുമതലയേല്‍പ്പിക്കപ്പെട്ടിരിക്കുന്നത് ഒരു പ്രത്യേക മാലാഖയെയാണ്. നിങ്ങളുടെ കാര്യത്തില്‍ ഏല്‍പ്പിക്കപ്പെട്ട മരണത്തിന്റെ മലക്ക് നിങ്ങളെ മരിപ്പിക്കുന്നതാണ് എന്ന് പറയുമ്പോള്‍ (32:11)അക്കാര്യത്തിനു വേണ്ടി ചുമതലയേല്‍പ്പിക്കപ്പെട്ട പ്രത്യേക മാലാഖയെയാണ് ഖുര്‍ആന്‍ വിവക്ഷിക്കുന്നത്. മരണത്തിന്റെ ഈ മാലാഖയക്ക് കീഴില്‍ മലക്കുകളില്‍ നിന്ന് തന്നെയുള്ള ഒരു വലിയ പ്രവര്‍ത്തക വ്യൂഹമുണ്ട്. ഈ മാലാഖമാര്‍ മരണത്തോടനുബന്ധിച്ച വിവിധ പ്രവര്‍ത്തനങ്ങളില്‍ ഏര്‍പ്പെട്ടുകൊണ്ടിരിക്കുകയാണ്. ഇതേ കുറിച്ച് വ്യക്തമാക്കുന്ന പലഖുര്‍ആന്‍ സൂക്തങ്ങളില്‍ ഒന്നു മാത്രമാണ് സൂറത്തു മുഹമ്മദില്‍ നിന്ന് ആദ്യം ഉദ്ധരിക്കപ്പെട്ട സൂക്തം (47:27). മരണത്തോടനുബന്ധിച്ച് മലക്കുകള്‍ ചെയ്യുന്ന പ്രവര്‍ത്തനങ്ങളെപ്പറ്റി പ്രതിപാദിക്കുന്ന മറ്റു ചില സൂക്തങ്ങള്‍ കാണുക: സ്വന്തത്തോട് അന്യായം ചെയ്തവരെ മരിപ്പിക്കുമ്പോള്‍ മലക്കുകള്‍ അവരോട് ചോദിക്കും: നിങ്ങള്‍ എന്തൊരു നിലപാടിലായി രുന്നു? (4:97). നിങ്ങള്‍ നിങ്ങളുടെ ആത്മാക്കളെ പുറത്തിറക്കുവിന്‍ എന്ന് പറഞ്ഞുകൊണ്ട് മലക്കുകള്‍ അവരുടെ നേരെ തങ്ങളുടെ കൈകള്‍ നീട്ടിക്കൊണ്ടിരിക്കുകയാണ്. (6:93) നല്ലവരായിരിക്കെ മലക്കുകള്‍ ഏതൊരു കൂട്ടരുടെ ജീവിതം അവസാനിപ്പിക്കുന്നുവോ അവരോട് മാലാഖമാര്‍ പറയും: നിങ്ങള്‍ക്ക് സമാധാനം! നിങ്ങള്‍ പ്രവര്‍ത്തിച്ചുകൊണ്ടിരുന്നതിന്റെ ഫലമായി നിങ്ങള്‍ സ്വര്‍ഗ്ഗത്തില്‍ പ്രവേശിച്ചുകൊള്ളുക. (16:32). ഈ സൂക്തങ്ങളിലെല്ലാം മനുഷ്യരുടെ മരണത്തോടനുബന്ധിച്ച് വിവിധ പ്രവര്‍ത്തനങ്ങളില്‍ ഏര്‍പ്പെടുന്ന മാലാഖമാരുടെ വ്യുഹത്തെപ്പറ്റിയാണ് വിശദീകരിക്കപ്പെട്ടിരിക്കുന്നത്. ഈ വ്യൂഹം മരണത്തിന്റെ മലക്കിന്റെ കീഴിലുള്ളതാണ്. മരണവും മരണത്തോടനുബന്ധിച്ച പ്രവര്‍ത്തനങ്ങളും ചെയ്യാനായി ഒരു മാലാഖവ്യൂഹത്തെ ചുമതലപ്പെടുത്തിയിട്ടുണ്ട്. ഈ വ്യൂഹത്തിന്റെ നേതാവാണ് മരണത്തിന്റെ മലക്ക് എന്ന് അറിയപ്പെടുന്നത്. ഈ വ്യൂഹവും നേതാവുമെല്ലാം അല്ലാഹുവിന്റെ നിയന്ത്രണത്തിന് കീഴിലാണ്. ഇക്കാര്യങ്ങള്‍ മനസ്സിലാക്കിയാല്‍ വൈരുധ്യം ആരോപിക്കപ്പെട്ടിരിക്കുന്ന സൂക്തങ്ങളില്‍ യാതൊരു വൈരുധ്യവുമില്ലെന്ന വസ്തുത സുവ്യക്തമാകും.
യേശുവിന്റെ ജനനത്തെകുറിച്ച്മർയത്തോട് സന്തോഷവാര്ത്തയറിയച്ചത് മലക്കുകള് ആണെന്ന് ഖുര്ആനില് 3:45ലും,എന്നാല് ഒരു മലക്ക് മാത്രമാണെന്ന് 19:17-21ലും പറയുന്നുണ്ടല്ലോ. ഇത് വ്യക്തമായ വൈരുധ്യമല്ലേ?  

വിടെ വൈരുധ്യം ആരോപിക്കപ്പെട്ടിരിക്കുന്ന ഖുര്‍ആന്‍സൂക്തങ്ങളുടെ സാരം പരിശോധിക്കുക:

മലക്കുകള്‍ പറഞ്ഞ സന്ദര്‍ഭം ശ്രദ്ധിക്കുക: മര്‍യമേ, തീര്‍ച്ചയായുംഅല്ലാഹു നിനക്ക് അവന്റെ പക്കല്‍ നിന്നുള്ള ഒരു വചനത്തെപ്പറ്റിസന്തോഷവാര്‍ത്ത അറിയിക്കുന്നു. അവന്റെ പേര് മര്‍യമിന്റെ മകന്‍മസീഹ് ഈസ എന്നാകുന്നു. അവന്‍ ഇഹത്തിലും പരത്തിലുംമഹത്വമുള്ളവനും സാമീപ്യം സിദ്ധിച്ചവരില്‍പെട്ടവനുമായിരിക്കും. (വി.ഖു. 3:45)

എന്നിട്ട് അവര്‍ കാണാതിരിക്കാന്‍ അവള്‍ ഒരു മറയുണ്ടാക്കി. അപ്പോള്‍നമ്മുടെ ആത്മാവിനെ നാം അവളുടെ അടുത്തേക്ക് നിയോഗിച്ചു. അങ്ങനെഅദ്ദേഹം അവളുടെ മുന്‍പില്‍ തികഞ്ഞ മനുഷ്യരൂപത്തില്‍ പ്രത്യക്ഷപ്പെട്ടു.അവള്‍ പറഞ്ഞു: തീര്‍ച്ചയായും നിന്നില്‍ നിന്ന് ഞാന്‍ പരമകാരുണികനില്‍ശരണം പ്രാപിക്കുന്നു. നീ ധര്‍മ്മനിഷ്ഠയുള്ളവനാണെങ്കില്‍ (എന്നെവിട്ടുമാറിപ്പോകൂ).

അദ്ദേഹം പറഞ്ഞു: പരിശുദ്ധനായ ഒരാണ്‍കുട്ടിയെ നിനക്ക് ദാനംചെയ്യുന്നതിന്നു വേണ്ടി നിന്റെ രക്ഷിതാവ് അയച്ച ദൂതന്‍ മാത്രമാ കുന്നുഞാന്‍. അവള്‍ പറഞ്ഞു: എനിക്കെങ്ങനെ ഒരാണ്‍കുട്ടിയുണ്ടാ കും? യാതൊരുമനുഷ്യനും എന്നെ സ്പര്‍ശിച്ചിട്ടില്ല. ഞാന്‍ ഒരു ദുര്‍നടപ്പുകാരിയായിട്ടുമില്ല.അദ്ദേഹം പറഞ്ഞു: കാര്യം അങ്ങനെ തന്നെയാകുന്നു. അത് തന്നെസംബന്ധിച്ചിടത്തോളം നിസ്സാരമായ ഒരു കാര്യമാണെന്ന് നിന്റെ രക്ഷിതാവ്പറഞ്ഞിരിക്കുന്നു. അവനെ മനുഷ്യര്‍ക്ക് ഒരു ദൃഷ്ടാന്തവും നമ്മുടെ പക്കല്‍നിന്നുള്ള ഒരു കാരുണ്യവുമാക്കുവാനും (നാം ഉദ്ദേശിക്കുന്നു). അത്തീരുമാനിക്കപ്പെട്ട ഒരു കാര്യമാകുന്നു. (വി.ഖു. 19:17-21)

മര്‍യമി (റ)ന്റെ ജീവിതത്തിലുണ്ടായ രണ്ടു സംഭവങ്ങളാണ് മുകളില്‍ഉദ്ധരിച്ച ഖുര്‍ആന്‍ സൂക്തങ്ങളില്‍ പരാമര്‍ശിച്ചിട്ടുള്ളതെന്ന് അവഒരാവര്‍ത്തി മനസ്സിരുത്തി വായിച്ചാല്‍ തന്നെ മനസ്സിലാകും. മര്‍യമി (റ)ന്റെജീവിതത്തില്‍ ഒരേയൊരു തവണ മാത്രമേ മലക്കുകളുമായിസംഭാഷണമുണ്ടായിട്ടുള്ളൂവെന്ന് ഖുര്‍ആനിലൊരിടത്തും പറയുന്നതായിനമുക്ക് കാണാന്‍ കഴിയുന്നില്ല. അങ്ങനെ പറയുന്നുണ്ടെങ്കില്‍ ഈ സൂക്തങ്ങള്‍തമ്മില്‍ വൈരുധ്യങ്ങളുണ്ടെന്ന് പറയുന്നതില്‍ അര്‍ത്ഥമുണ്ടാകുമായിരുന്നു.സത്യത്തില്‍ സൂറത്തു ആലുഇംറാനില്‍(3:42,45)പരാമര്‍ശിക്കപ്പെട്ടമാലാഖമാരുടെ സംഭാഷണം ഒരു സന്തോഷവാര്‍ത്ത അറിയിക്കല്‍ മാത്രമാണ്.അതു നിര്‍വ്വഹിച്ചത് മലക്കുകളുടെ ഒരു സമൂഹമായിരുന്നു. ഏതെല്ലാംമാലാഖമാരായിരുന്നു പ്രസ്തുത സമൂഹത്തില്‍ ഉണ്ടായിരുന്നതെന്ന് പ്രസ്തുതസൂക്തങ്ങളില്‍ വ്യക്തമാക്കപ്പെട്ടിട്ടില്ല. ഈ സന്തോഷവാര്‍ത്ത അറിയിക്കപ്പെട്ടകാര്യത്തിന്റെ നിര്‍വ്വഹണത്തിനും പൂര്‍ത്തീകരണത്തിനുമായിട്ടാണ്പരിശുദ്ധാത്മാവ് എന്ന് വിളിക്കപ്പെടുന്ന ജിബ്രീല്‍ (അ) എന്ന മാലാഖയെഅല്ലാഹു മര്‍യമിന്റെ അടുക്കലേക്ക് അയച്ചത്. ജനങ്ങളില്‍ നിന്ന് അകന്ന്ദൈവസ്മരണയില്‍ കഴിയുന്ന മര്‍യമിന്റെ അടുക്കലേക്ക് ദൈവികദൗത്യവുമായെത്തിയ ജിബ്രീല്‍ കടന്നുവന്നപ്പോഴുള്ള സംഭാഷണമാണ്സൂറത്തു മര്‍യമില്‍ (19:17-21) വിശദീകരിക്കപ്പെട്ടിട്ടുള്ളത്. ജിബ്രീലിന്റെആഗമനോദ്ദേശ്യം മാലാഖമാര്‍ ചെയ്തതുപോലെ ദൈവിക വചനത്തെകുറിച്ച സന്തോഷവാര്‍ത്ത അറിയിക്കുകയായിരുന്നില്ല. പ്രത്യുത,പരിശുദ്ധനായ ഒരു ആണ്‍കുട്ടിയെ ദാനം ചെയ്യുകയായിരുന്നു. അഥവാമാലാഖമാര്‍ സന്തോഷവാര്‍ത്തയറിയിച്ച കാര്യത്തിന്റെനിര്‍വ്വഹണമായിരുന്നു ജിബ്രീലിന്റെ ദൗത്യം. ഒരു പുരുഷന്റെസ്പര്‍ശമേല്‍ക്കാതെ ഒരു കുഞ്ഞുണ്ടാവാന്‍ പോകുന്നുവെന്ന മാലാഖമാരുടെഅറിയിപ്പ് നടപ്പിലാക്കുകയായിരുന്നു ജിബ്രീലിന്റെ ആഗമനോദ്ദേശ്യം.ദൈവിക വാഗ്ദാനത്തിന്റെ പൂര്‍ത്തീകരണത്തിന് അനുഗുണമായ രീതിയില്‍മര്‍യമിന്റെ ശരീരത്തിലുണ്ടാവേണ്ട മാറ്റം ഉണ്ടാക്കുകയായിരുന്നുഅദ്ദേഹത്തിന്റെ ദൗത്യമെന്നാണ് മനസ്സിലാക്കാന്‍ കഴിയുന്നത്. ഏതായാലുംഈ സൂക്തങ്ങളിലെ പ്രതിപാദ്യം രണ്ടു സംഭവങ്ങളാണ്. ഒന്ന്, മാലാഖമാരുടെസന്തോഷവാര്‍ത്തയറിയിക്കലും രണ്ടാമത്തേത്, പ്രസ്തുതസന്തോഷവിഷയത്തിന്റെ പൂര്‍ത്തീകരണത്തിനായുള്ള ജിബ്രീലിന്റെആഗമനവും അതോടനുബന്ധിച്ച് നടന്ന സംഭാഷണങ്ങളുമാണ്.രണ്ടും രണ്ടുസംഭവങ്ങള്‍. രണ്ടിലെയും സംഭാഷണങ്ങള്‍ വ്യത്യസ്തം. രണ്ടിലുംസംസാരിക്കുന്നവരും വ്യത്യസ്തം. പിന്നെയെങ്ങനെയാണ് ഈ സൂക്തങ്ങള്‍തമ്മില്‍ വൈരുധ്യമുണ്ടെന്ന് പറയാനാവുക?

 

ഖുര്‍ആനിലെ പത്തൊന്‍പതാം അധ്യായമായ സൂറത്തുമറിയം തുടങ്ങുന്നതുതന്നെ സകരിയ്യാ (അ)  യുടെ വൃത്താന്തവുമായിക്കൊണ്ടാണ്. വാര്‍ധക്യകാലത്ത് വന്ധ്യയായ ഭാര്യയോടൊപ്പം ജീവിക്കുന്ന സകരിയ്യാ (അ) യുടെ ഒരു അനന്തരാവകാശിക്കുവേണ്ടിയുള്ള പ്രാര്‍ത്ഥനയും പ്രസ്തുത പ്രാര്‍ത്ഥനക്കുള്ള ഉത്തരമായി ഒരു ആണ്‍കുഞ്ഞുണ്ടായ കഥയുമെല്ലാം ഖുര്‍ആന്‍ വിവരിക്കുന്നുണ്ട്. സകരിയ്യായുടെ പ്രാര്‍ത്ഥനയ്ക്കുള്ള ഉത്തരമെന്നോണം സര്‍വ്വശക്തന്‍ അദ്ദേഹത്തിന് നല്‍കിയ വാഗ്ദാനം ഏഴാം വചനത്തില്‍ പറയുന്നുണ്ട്. അതിന്റെ സാരം ഇങ്ങനെയാണ്:

''ഹേ സക്കരിയാ, തീര്‍ച്ചയായും നിനക്ക് നാം ഒരു ആണ്‍കുട്ടിയെപറ്റി സന്തോഷവാര്‍ത്ത അറിയിക്കുന്നു. അവന്റെ പേര്‍ യഹ്‌യാ എന്നാകുന്നു. മുമ്പ് നാം ആരെയും അവന്റെ പേരുള്ളവരാക്കിയിട്ടില്ല'' (വി.ഖു.19:7)

ഈ വചനത്തില്‍ ''മുമ്പ് നാം ആരെയും അവന്റെ പേരുള്ളവരാക്കിയിട്ടില്ല''യെന്ന് യഹ്‌യാ (അ) യെക്കുറിച്ച് പറഞ്ഞത് ചരിത്രപരമായി അബദ്ധമാണെന്നാണ് ആരോപണം. 'യോഹന്നാന്‍ സ്‌നാപക'ന് അറബിയില്‍ പറയുന്ന പേരാണ് യഹ്‌യായെന്ന് പല ഖുര്‍ആന്‍ വ്യാഖ്യാതാക്കളും വ്യക്തമാക്കിയിട്ടുണ്ട്. ഇംഗ്ലീഷില്‍ പുറത്തിറങ്ങിയ ചില ഖുര്‍ആന്‍ പരിഭാഷാ ഗ്രന്ഥങ്ങളില്‍ യഹ്‌യായെന്നതിന് പകരമായി ജോണ്‍ (John) എന്നെഴുതുകയും മറ്റുചിലവയില്‍ യഹ്‌യായെന്നെഴുതി ജോണ്‍ എന്ന് ബ്രാക്കറ്റില്‍ ചേര്‍ക്കുകയും ചെയ്തിട്ടുണ്ട്. അപ്പോള്‍ യോഹന്നാന്‍ സ്‌നാപകനുമുമ്പ് യോഹന്നാന്‍ എന്നപേരുള്ളവരായി ആരുംതന്നെ ജീവിച്ചിരുന്നില്ല എന്നാണ് ഈ സൂക്തത്തില്‍ പറഞ്ഞതെന്ന് വരുന്നു. പഴയ നിയമത്തില്‍തന്നെ ഇരുപത്തിയേഴ് പ്രാവശ്യം യോഹന്നാന്‍ എന്ന നാമം പ്രയോഗിക്കപ്പെട്ടിട്ടുണ്ട്. പഴയ നിയമത്തിലെ യോഹന്നന്‍മാരൊന്നും തന്നെ അത്ര പ്രസിദ്ധരല്ലാത്തതിനാല്‍ മുഹമ്മദി(സ)ന് അവരെക്കുറിച്ച് അറിയുമായിരുന്നില്ല. അതുകൊണ്ട് വന്നുഭവിച്ച അബദ്ധമാണിത്': ഖുര്‍ആന്‍ വിമര്‍ശകരുടെ വാദം പോകുന്നത് ഇങ്ങനെയാണ്.

ഈ വിമര്‍ശനം പ്രധാനമായും ഒരു പദത്തെ ചുറ്റിപ്പറ്റിയുള്ളതാണ്. 'യഹ്‌യാ'യെന്നതാണ് ആ പദം. യോഹന്നാന്‍ സ്‌നാപകനെക്കുറിക്കാന്‍ ഖുര്‍ആനില്‍ പ്രയോഗിക്കപ്പെട്ട പദമാണത്.  പദോല്‍പത്തിയെക്കുറിച്ച് സൂക്ഷ്മമായി ഗവേഷണം നടത്താത്ത വ്യാഖ്യാതാക്കള്‍ യഹ്‌യായെന്ന പദത്തിന് പകരമായും തത്തുല്യമായും യോഹന്നാന്‍ എന്ന് പ്രയോഗിച്ചിട്ടുണ്ടെന്നത് നേരാണ്. ഖുര്‍ആനില്‍ പറഞ്ഞിരിക്കുന്നത് ''മുമ്പ് നാം ആരെയും അവന്റെ പേര് (യഹ്‌യാ) ഉള്ളവരാക്കിയിട്ടില്ല''യെന്നാണ്. ഖുര്‍ആനില്‍ മുമ്പ് ആര്‍ക്കുമുണ്ടായിരുന്നില്ലെന്ന് പറയുന്നത് 'യഹ്‌യാ'യെന്ന നാമമാണ്; യോഹന്നാന്‍ എന്ന പേരല്ല. യഹ്‌യ=യോഹന്നാന്‍ എന്ന് കരുതിയ വ്യാഖ്യാതാക്കളാണ് ഖുര്‍ആനില്‍ പറഞ്ഞത് യോഹന്നാന്‍ എന്നാണെന്ന് വരുത്തിത്തീര്‍ത്തത്. ബൈബിള്‍ പഴയ നിയമത്തില്‍ പലതവണ യോഹന്നാന്‍ എന്ന പേര് ഉപയോഗിക്കപ്പെട്ടിട്ടുണ്ടെന്നത് നേരാണ്. ''യഹ്‌യാ''യെന്നല്ല അവിടെയൊന്നും പ്രയോഗിച്ചിരിക്കുന്നത് എന്ന കാര്യം പ്രത്യേകം ശ്രദ്ധേയമാണ്. അറബി ബൈബിളില്‍നിന്നുള്ള ഏതാനും ഉദ്ധരണികള്‍ കാണുക:

                         ثَمانِيَةٌ وَعِشرُونَ رَجُلاً. 12 وَمِن بَنِي عَزْجَدَ يُوحَنانُ بْنُ هِقّاطانَ وَمَعَهُ مِئَةٌ وَعَشْرَةُ رِجالٍ.

                                                                                   (എസ്രാ 8:12)

                                              جاءَ رَجُلٌ مُرسَلاً مِنَ اللهِ اسْمُهُ يُوحَنّا.

                                                                                 (യോഹന്നാന്‍ 1:6)

''യഹ്‌യാ''യെന്ന അറബി പദത്തിന് തത്തുല്യമായ ഹിബ്രു പദമാണോ 'യോഹന്നാന്‍' എന്നാണ് നാം ആദ്യമായി പരിശോധിക്കേണ്ടത്. ഇവ്വിഷയകമായ പ്രാഥമിക പരിശോധനയ്ക്ക് നാം അറബി ബൈബിള്‍ പരിശോധിച്ചാല്‍ മതിയാവും. 1 രാജാക്കന്മാര്‍ 25:23, 1 ദിനവൃത്താന്തരം 3:15, 1 ദിനവൃത്താന്തം 3:24, എസ്രാ 8:12 തുടങ്ങിയ പഴയ നിയമ ഉദ്ധരണികളില്‍ യോഹന്നാനെക്കുറിച്ച് പറയുന്നുണ്ട്. ഇവിടെയെല്ലാം അറബി ബൈബിളില്‍ യൂഹന്നായെന്നാണ് പറഞ്ഞിരിക്കുന്നത് ''യോഹന്നാന്‍ സുവിശേഷം'' എന്ന തലക്കെട്ട് അറബി ബൈബിളില്‍ 'ബിശാറത്തു യൂഹന്നാ'(بشارت يوحنا) യെന്നാണ്. അറബി പുതിയ നിയമത്തില്‍ യോഹന്നാന്‍ സ്‌നാപകനെയും യേശു ശിഷ്യനായ യോഹന്നാനെയുമെല്ലാം 'യൂഹന്നാ'യെന്നുതന്നെയാണ് വിളിച്ചിരിക്കുന്നത്; എവിടെയും 'യഹ്‌യാ'യെന്ന് കാണുന്നില്ല. 'യോഹന്നാന്‍' എന്ന ഹിബ്രു ശബ്ദത്തിന് തത്തുല്യമായ അറബി പദമായിരുന്നു 'യഹ്‌യാ'യെങ്കില്‍ അറബി ബൈബിളില്‍ യോഹന്നാന്‍ എന്ന പദത്തിന് പകരമായി യഹ്‌യായെന്ന് പ്രയോഗിക്കുമായിരുന്നുവെന്ന് തീര്‍ച്ചയാണ്.

സത്യത്തില്‍, യഹ്‌യാ, യോഹന്നാന്‍ എന്നിവ തികച്ചും വ്യത്യസ്തങ്ങളായ രണ്ട് നാമങ്ങളാണ്. യോഹന്നാന്‍ എന്ന ഹിബ്രു പദത്തിനര്‍ത്ഥം 'യഹോവ കാരുണ്യം ചെയ്തിരിക്കുന്നു'' (Jehovah has graced) എന്നാണ്. രണ്ട് പദങ്ങള്‍ ചേര്‍ന്നുണ്ടായ ഒരു നാമമാണ് യോഹന്നാന്‍. യൂ+ഹന്നാന്‍. യഹോവയുടെ ചുരുക്കമായാണ് 'യൂ'യെന്ന് പ്രയോഗിച്ചിരിക്കുന്നത്. 'ഹന്നാന്‍ എന്ന ഹിബ്രു പദം 'ഹനാന്‍' എന്ന അരമായിക് മൂലത്തില്‍ നിന്നുണ്ടായതാണ്. 'അനുകമ്പ'യെന്നാണ് അര്‍ത്ഥം.'യഹോവ അനുകമ്പയുള്ളവനായിരിക്കുന്നു''എന്നോ ''യഹോവയുടെ അനുകമ്പ''എന്നോ ആണ് യോഹന്നാന്‍ എന്ന പദത്തിന്റെ മൂലാര്‍ത്ഥം. എന്നാല്‍ 'യഹ്‌യാ'യെന്ന അറബിപദമുണ്ടായിരിക്കുന്നത് 'ഹയാ' എന്ന മൂലത്തില്‍നിന്നാണ്. ഈ പദത്തിന് രണ്ട് അര്‍ത്ഥമുണ്ട്. ഒന്ന് 'അല്‍ഹയാത്തി'ല്‍നിന്ന് നിര്‍ധരിക്കപ്പെട്ട ത്. 'ജീവന്‍' എന്നര്‍ത്ഥം. മറ്റൊന്ന്  'അല്‍-ഹയാഇ'ല്‍ നിന്നുള്ളത്. 'നാണം' എന്ന് സാരം 'യഹ്‌യാ'യുടെ ഉല്‍പത്തി ഇവ രണ്ടില്‍ ഏതില്‍നിന്നായിരുന്നാലും യോഹന്നാന്‍ എന്ന ഹിബ്രു പദവുമായി യാതൊരു ബന്ധവും ഇതിനില്ലെന്ന് വ്യക്തമാണ്. രണ്ടിന്റെയും മൂലങ്ങള്‍ വ്യത്യസ്തങ്ങളാണ്; അര്‍ത്ഥങ്ങള്‍ തമ്മില്‍ യാതൊരു വിധ സാമ്യവുമില്ലതാനും.

യേശുവിന് തൊട്ടുമുമ്പ് വന്ന സകരിയ്യായുടെ പുത്രനെയാണ് ഖുര്‍ആന്‍ 'യഹ്‌യാ'യെന്ന് വിളിക്കുന്നത്. സകരിയ്യയുടെയും എലിസബത്തിന്റെയും പുത്രനാണ് ബൈബിളിലെ യോഹന്നാന്‍ സ്‌നാപകന്‍. എന്നാല്‍ 'യഹ്‌യാ'യെന്ന പദവും 'യോഹന്നാന്‍' എന്ന പദവും തമ്മില്‍ യാതൊരുവിധ ബന്ധവുമില്ല. രണ്ടും രണ്ട് മൂലങ്ങളില്‍നിന്നുണ്ടായവ; രണ്ട് അര്‍ത്ഥങ്ങളുള്‍ക്കൊള്ളുന്നവ. ഇതെങ്ങനെ സംഭവിച്ചു? ഒരാളുടെ തന്നെ രണ്ട് നാമങ്ങളാണോ യഹ്‌യായും യോഹന്നാനും? അതല്ല വിമര്‍ശകര്‍ ആരോപിക്കുന്നതുപോലെ മുഹമ്മദ് നബി (സ)ക്ക് പറ്റിയ ഒരു കൈപ്പിഴയാണോ ഇത്?

ഈ ചോദ്യങ്ങള്‍ക്ക് വസ്തുനിഷ്ഠമായ ഉത്തരം ലഭിക്കണമെങ്കില്‍ ഖുര്‍ആനും ബൈബിളുമല്ലാത്ത മറ്റുവല്ല രേഖകളും സ്‌നാപക യോഹന്നാനെക്കുറിച്ച് പറയുന്നതായി നിലനില്‍ക്കുന്നുണ്ടോയെന്ന് പരിശോധിക്കണം. അങ്ങനെ വല്ല രേഖകളുമുണ്ടെങ്കില്‍ അവ ഇക്കാര്യത്തില്‍ നല്‍കുന്ന അറിവ് ഏറെ പ്രധാനപ്പെട്ടതാണ്. യോഹന്നാനോ യഹ്‌യയോ എന്താണ് യഥാര്‍ത്ഥ നാമമെന്ന് കണ്ടുപിടിക്കുന്നതിനുവേണ്ടി മാത്രമല്ല പ്രസ്തുത രേഖകള്‍ പ്രയോജനപ്രദമാവുക. അതുവഴി ഏത് ഗ്രന്ഥമാണ് കൃത്യവും സൂക്ഷ്മവുമായ പരാമര്‍ശങ്ങള്‍ നടത്തുന്നതെന്ന് മനസ്സിലാക്കുവാനും ഏതിനാണ് അപ്രമാദിത്വമുള്ളതെന്ന് വ്യക്തമായി അറിയുവാനും കഴിയും.

യോഹന്നാന്‍ സ്‌നാപകനെക്കുറിച്ച്, അദ്ദേഹം ഒരു വിശുദ്ധ പുരുഷനായിരുന്നുവെന്ന് വിശ്വസിക്കുന്ന ക്രൈസ്തവരും മുസ്‌ലിംകളുമല്ലാത്ത മറ്റേതെങ്കിലും വിഭാഗങ്ങളുണ്ടോയെന്ന അന്വേഷണം പ്രസക്തമാണ്. അങ്ങനെയൊന്നുണ്ടെങ്കില്‍ ഇക്കാര്യത്തില്‍ ഖുര്‍ആനും ബൈബിളുമല്ലാത്ത ഒരു സ്രോതസ്സായി അവരുടെ ഗ്രന്ഥങ്ങളോ രേഖകളോ സ്വീകരിക്കുവാന്‍ പറ്റുമെന്ന കാര്യത്തില്‍ സംശയമൊന്നുമില്ല. പക്ഷെ, അങ്ങനെയെന്തെങ്കിലുമുണ്ടോ?

യോഹന്നാന്‍ സ്‌നാപകനെ പിന്തുടരുന്നവരാണ് തങ്ങളെന്നും അദ്ദേഹം അന്തിമ പ്രവാചകനായിരുന്നുവെന്നും അവകാശപ്പെടുന്ന ഒരു വിഭാഗം ഇറാഖിലും ഇറാനിലും ഇന്നുമുണ്ട്. പോര്‍ച്ചുഗീസ് ക്രിസ്ത്യന്‍ മിഷനറിമാര്‍ ഇവരെ വിളിച്ചത് 'യോഹന്നാന്‍ സ്‌നാപകന്റെ ക്രിസ്ത്യാനികള്‍' (Christians of John the Baptist) എന്നായിരുന്നു. ഏകദൈവാരാധകരായ ഇവരുടെ പ്രധാനപ്പെട്ട ഒരു ആചാരമാണ് ജ്ഞാനസ്‌നാനം (Baptism).തങ്ങളുടെ മതത്തെയും വര്‍ഗത്തെയും കുറിക്കുവാന്‍വേണ്ടി മന്‍ഡായി (Mandai) എന്നും മതവിശ്വാസികളെ സൂചിപ്പിക്കുവാന്‍ മാന്‍ഡിയന്മാര്‍ (Mandaens) എന്നുമാണ് അവര്‍ ഉപയോഗിക്കുന്നത്. ഇസ്‌ലാമിനോട് സമാനമായ ഒട്ടനവധി വിശ്വാസാചാരങ്ങള്‍ മാന്‍ഡിയന്‍മാര്‍ക്കുണ്ട്. ജ്ഞാനസ്‌നാനം, പ്രാര്‍ത്ഥനകള്‍, ഉപവാസം, ദാനം തുടങ്ങിയവയാണ് ഇവരുടെ അടിസ്ഥാനാചാരങ്ങള്‍. അരമായ ഭാഷയോട് സാദൃശ്യമുള്ളതും സെമിറ്റിക് മൂലത്തില്‍നിന്ന് നിര്‍ധരിക്കപ്പെട്ടതുമായ മാന്‍ഡിയാക് ഭാഷ (Mandiac language)യിലാണ് ഇവരുടെ മതഗ്രന്ഥങ്ങളെല്ലാം എഴുതപ്പെട്ടിരിക്കുന്നത്. ഗിന്‍സാ റാബ, ദ്രാഷ ഇദ് യഹ്‌യ, ആദാം ബോഗ്‌റ, ദി കിലെസ്ത, നിയാനി എന്നിവയാണ് ഇവരുടെ മതഗ്രന്ഥങ്ങള്‍.

 ബാഗ്ദാദിലെ കൗണ്‍സില്‍ ഓഫ് ജനറല്‍ അഫയേഴ്‌സിന് കീഴില്‍ പ്രവര്‍ത്തിക്കുന്ന മാന്‍ഡിയന്‍ റിസര്‍ച്ച് സെന്ററില്‍നിന്ന് ഈ മതവിഭാഗത്തെക്കുറിച്ച കൂടുതല്‍ അറിവ് ലഭിക്കും www. mandaean.com-au, www.mandaean.org   എന്നീ വെബ്‌സൈറ്റുകളില്‍നിന്ന് ഈ മതവിഭാഗത്തിന്റെ വിശ്വാസാചാരങ്ങളെക്കുറിച്ച് മനസ്സിലാക്കാനാവും.

മാന്‍ഡിയന്‍മാര്‍ തങ്ങളുടെ പ്രവാചകനും ഗുരുവുമായി സ്വീകരിച്ചിരിക്കുന്നത് യോഹന്നാന്‍ സ്‌നാപകനെയാണെന്ന് പറഞ്ഞുവല്ലോ. അവര്‍ അദ്ദേഹത്തെ വിളിക്കുന്നത് യഹ്‌യാ യൂഹന്നായെന്നാണ്. സ്‌നാപക യോഹന്നാന്‍േറതായി അവര്‍ വിശ്വസിക്കുന്ന ഉപദേശങ്ങളുടെയും അധ്യാപനങ്ങളുടെയും സമാഹാരമാണ് 'ദ്രാഷാ ഇദ് യഹ്‌യ'യെന്ന ഗ്രന്ഥം. 'യഹ്‌യായുടെ പുസ്തകം' എന്നര്‍ത്ഥം. അവരുടെ ഏറ്റവും പ്രധാനപ്പെട്ട ഗ്രന്ഥമായ ഗിന്‍സ റാബയിലെ നാനൂറ്റി പത്താം അധ്യായം തന്നെ' യഹ്‌യായുടെ പ്രാര്‍ത്ഥനകള്‍' എന്ന തലക്കെട്ടോടുകൂടിയതാണ്. ഇവയില്‍നിന്നെല്ലാംതന്നെ മാന്‍ഡിയന്‍മാര്‍ യോഹന്നാന്‍ സ്‌നാപകനെ വിളിക്കുന്നത് യഹ്‌യാ യൂഹന്നായെന്നായിരുന്നുവെന്ന് വ്യക്തമാകുന്നു. ഖുര്‍ആനില്‍ പറഞ്ഞതുപോലെ യോഹന്നാന്‍ സ്‌നാപകന് യഹ്‌യായെന്ന പേര് കൂടിയുണ്ടായിരുന്നുവെന്ന് ഇതില്‍നിന്ന് കൃത്യമായി മനസ്സിലാകുന്നുണ്ട്.

മാന്‍ഡായിക്കുകാരുടെ ഗ്രന്ഥങ്ങളില്‍നിന്നും ലേഖനങ്ങളില്‍നിന്നുമെല്ലാം യോഹന്നാന്‍ സ്‌നാപകന്റെ നാമം  യഹ്‌യാ യൂഹന്നായെന്നായിരുന്നുവെന്ന് മനസ്സിലാകുന്നുണ്ടെന്ന് പറഞ്ഞല്ലോ. എന്നാല്‍ ഖുര്‍ആനിലൊരിടത്തും അദ്ദേഹത്തെ യഹ്‌യാ യൂഹന്നായെന്ന് വിളിച്ചിട്ടില്ല. എന്തുണ്ടൊണിത്?

ഈ ചോദ്യത്തിന് ഉത്തരം ലഭിക്കണമെങ്കില്‍ എന്തുകൊണ്ടാണ് ഈ ഇരട്ട നാമം ഉപയോഗിക്കുന്നതെന്ന് കൃത്യമായി മനസ്സിലാക്കണം. മാന്‍ഡിയന്‍മാരെയും അവരുടെ വിശ്വാസസംഹിതകളെയും ആചാര രീതികളെയുംകുറിച്ച് വിശദമായി പഠിച്ചയാളാണ് ഇ.എസ്. ഡ്രോവര്‍. അവരുടെ 'ദി മാന്‍ഡിയന്‍സ് ഓഫ് ഇറാഖ് ആന്റ് ഇറാന്‍', 'ദി കാനോനിക്കല്‍ പ്രെയര്‍ ബുക്ക് ഓഫ് ദി മാന്‍ഡിയന്‍സ്' എന്നീ പുസ്തകങ്ങള്‍ ഇവ്വിഷയകമായ ആധികാരിക രേഖകളായി പരിഗണിക്കപ്പെടുന്നവയാണ്. അവരും ആര്‍. മാക്കൂച്ചും കൂടിച്ചേര്‍ന്ന് എഴുതിയ ഗ്രന്ഥമാണ് എ മാന്‍ഡായിക് ഡിക്ഷ്ണറി. (E.S. Drowoer: & R. Marcuch: A MANDAIC DICTIONARY 1963 OXFORD)  മാന്‍ഡിയന്മാരുടെ സാങ്കേതിക ശബ്ദങ്ങളും അവര്‍ അവയുപയോഗിച്ചിരുന്ന രീതിയുമെല്ലാം ഈ ശബ്ദകോശത്തിലുണ്ട്. പ്രസ്തുത ഡിക്ഷ്ണറിയുടെ 185-ാം പുറത്തില്‍ യഹ്‌യാ (iahia) യുടെയും 190-ാം പുറത്തില്‍ യോഹന്നാ (iuhana) യുടെയും സാരം നല്‍കിയിട്ടുണ്ട്. അവ നോക്കുക:

ഇവയില്‍നിന്നും മറ്റ് മാന്‍ഡിയന്‍ സാഹിത്യങ്ങളില്‍നിന്നുമായി അവരുടെ പേരുകളെക്കുറിച്ച് നമുക്ക് മനസ്സിലാകുന്ന ഒട്ടേറെ കാര്യങ്ങളുണ്ട്. എല്ലാ മാന്‍ഡിയന്മാര്‍ക്കും പൊതുവെ രണ്ട് പേരുകളുണ്ടായിരിക്കും. ഒന്നാമത്തെ പേര് മല്‍വാഷാ നാമമെന്നും (malwasha name) രണ്ടാമത്തെ പേര് ലഖബ് (lagab) എന്നുമാണ് അറിയപ്പെടുക. എന്തിനാണ് ഈ രണ്ട് പേരുകള്‍? ഇവ എന്താണ് അര്‍ത്ഥമാക്കുന്നത്?

ഇ.എസ്. ഡ്രോവര്‍ എഴുതുന്നു: ''രണ്ടാമത്തെ പേര് പൊതുവെ ഒരു മുഹമ്മദന്‍ നാമമായിരിക്കും. ഇതാണ് എല്ലാ സാധാരണ ആവശ്യങ്ങള്‍ക്കും ഉപയോഗിക്കപ്പെടാറുള്ളത്. ആദ്യത്തെ പേര് (malwasha) ആണ് അയാളുടെ യഥാര്‍ത്ഥ ആത്മീയ നാമം. മതപരവും മാന്ത്രികവുമായ സന്ദര്‍ഭങ്ങളിലെല്ലാം ഈ പേരാണ് ഉപയോഗിക്കുക'' (E.S. Drower: The Mandaeans of  Iraq and Iran (1962-Lieden) Page 81)

യോഹന്നാന്‍ സ്‌നാപകന്റെ മാല്‍വാഷാ നാമമാണ് യഹ്‌യ.യോഹന്നാന്‍ എന്നത് അദ്ദേഹത്തിന്റെ ലഖബും. ജനങ്ങള്‍ പൊതുവെ അദ്ദേഹത്തെ വിളിച്ചിരുന്നത് യോഹന്നാന്‍ എന്നായിരിക്കണം. കാരണം മാന്‍ഡിയന്‍മാര്‍ എല്ലാ സാധാരണ ആവശ്യങ്ങള്‍ക്കും പൊതുവായി ലഖബാണ് വിളിച്ചിരുന്നത്. എന്നാല്‍ അദ്ദേഹത്തിന്റെ യഥാര്‍ത്ഥ ആത്മീയ നാമം യഹ്‌യായെന്നായിരുന്നു. മതപരമായ സന്ദര്‍ഭങ്ങളുമായി ബന്ധപ്പെടുത്തി ഈ നാമമാണ് ഏറെ ഉപയോഗിക്കപ്പെട്ടിരിക്കുന്നത്. യഹ്‌യായുടെ പുസ്തകത്തിലെ മിക്ക അധ്യായങ്ങളും ആരംഭിക്കുന്നതുതന്നെ ''യഹ്‌യാ രാത്രികളില്‍ പ്രഖ്യാപിക്കുന്നു; യോഹന്നാ രാത്രിയുടെ സന്ധ്യകളിലും'' എന്ന് പറഞ്ഞുകൊണ്ടാണ്. ചുരുക്കത്തില്‍ യോഹന്നാന്‍ സ്‌നാപകന്റെ യഥാര്‍ത്ഥ ആത്മീയ നാമം 'യഹ്‌യാ'യെന്നായിരുന്നു; ജനങ്ങള്‍ ആ സമൂഹത്തില്‍ നിലനിന്നിരുന്ന സമ്പ്രദായപ്രകാരം അദ്ദേഹത്തിന്റെ ലഖബ് ആയ 'യോഹന്നാ' എന്ന പേരിലാണ് പൊതുവായി അദ്ദേഹത്തെ വിളിച്ചിരുന്നത് എന്ന് മാത്രമേയുള്ളൂ.

ഖുര്‍ആനില്‍ 'യഹ്‌യാ'യെന്ന് മാത്രമെ പ്രവാചക നാമമായി ഉപയോഗിക്കപ്പെട്ടിട്ടുള്ളൂവെന്ന് പറഞ്ഞുവല്ലോ. അതാണ്, അത് മാത്രമാണ് അദ്ദേഹത്തിന്റെ യഥാര്‍ത്ഥ നാമം എന്നുള്ളതുകൊണ്ടാണിത്. ദൈവിക കല്‍പന പ്രകാരം മാതാപിതാക്കള്‍ അദ്ദേഹത്തിന് നല്‍കിയ പേരായാണ് ഖുര്‍ആന്‍ 'യഹ്‌യാ'യെന്ന നാമത്തെ പരിചയപ്പെടുത്തുന്നത്. അതായിരുന്നു അദ്ദേഹത്തിന്റെ യഥാര്‍ത്ഥ ആത്മീയ നാമം എന്ന് മാന്‍ഡിയന്‍ സാഹിത്യങ്ങളില്‍നിന്ന് നമുക്ക് മനസ്സിലാകുന്നതോടെ ഖുര്‍ആനിന്റെ ദൈവികത ഒരിക്കല്‍കൂടി വ്യക്തമാവുകയാണ് ചെയ്യുന്നത്; ഒപ്പം, ബൈബിളില്‍നിന്ന് പകര്‍ത്തിയെഴുതിയതാണ് ഖുര്‍ആന്‍ എന്ന വാദത്തിന്റെ മൂലത്തില്‍തന്നെ ഈ വസ്തുതകള്‍ കഠാരകുത്തിക്കയറ്റുന്നു. ബൈബിളിലെവിടെയും പരാമര്‍ശിക്കപ്പെട്ടിട്ടില്ലാത്ത യോഹന്നാന്‍ സ്‌നാപകന്റെ യഥാര്‍ത്ഥ നാമമായ 'യഹ്‌യാ' ഖുര്‍ആനില്‍ വന്നത് യാദൃച്ഛികമാകാനിടയില്ലെന്ന് ഏതൊരു സാധാരണക്കാരന്നും മനസ്സിലാക്കാവുന്നതേയുള്ളൂ. സകരിയ്യാ (അ) യുടെ വാര്‍ധക്യകാലത്ത് വന്ധ്യയായ ഭാര്യയില്‍ പുത്രനെ പ്രദാനം ചെയ്യുകയും പുത്രന് 'യഹ്‌യാ' യെന്ന് പേരിടാന്‍ നിര്‍ദ്ദേശിക്കുകയും ചെയ്ത തമ്പുരാനില്‍നിന്ന് അവതീര്‍ണമായ ഗ്രന്ഥമായതിനാലാണ് ഖുര്‍ആനില്‍ ഇക്കാര്യത്തിലും കൃത്യവും സൂക്ഷ്മവുമായ പരാമര്‍ശങ്ങളുണ്ടായത് എന്ന് മാത്രമേ ചിന്തിക്കുവാന്‍ കഴിയുകയുള്ളൂ.

ബൈബിളില്‍ സ്‌നാപകനെ കുറിക്കുവാന്‍ യോഹന്നാന്‍ എന്ന് മാത്രമെ പ്രയോഗിക്കപ്പെട്ടിട്ടുള്ളൂവെന്നതും സ്വാഭാവികമാണ്. അദ്ദേഹത്തിന് ശേഷം പതിറ്റാണ്ടുകള്‍ ഏറെക്കഴിഞ്ഞ് രചിക്കപ്പെട്ട സുവിശേഷങ്ങളിലാണ് യോഹന്നാന്‍ സ്‌നാപകനെക്കുറിച്ച പരാമര്‍ശങ്ങളുള്ളത്. നടേ സൂചിപ്പിച്ചതുപോലെ തന്റെ കാലത്ത് അദ്ദേഹം ജനങ്ങളാല്‍ പൊതുവായി വിളിക്കപ്പെട്ടത് യോഹന്നായെന്നായിരിക്കണം. അതുകൊണ്ടുതന്നെ വാമൊഴിയായി പ്രചരിച്ച അദ്ദേഹത്തിന്റെ ജീവിതകഥനങ്ങളിലും ഉപദേശങ്ങളിലും യോഹന്നായെന്ന പേരായിരിക്കണം പ്രധാനമായും ഉപയോഗിക്കപ്പെട്ടത്. പതിറ്റാണ്ടുകള്‍ കഴിഞ്ഞ് രചിക്കപ്പെട്ട സുവിശേഷങ്ങളുടെ കര്‍ത്താക്കള്‍ യോഹന്നായെന്നാണ് അദ്ദേഹത്തിന്റെ നാമമെന്ന് കരുതിയത് സ്വാഭാവികം മാത്രം. എന്നാല്‍ ബൈബിള്‍ പുസ്തകങ്ങളുടെ കര്‍ത്താക്കള്‍ക്ക് തങ്ങളുടെ ഗ്രന്ഥരചനയില്‍ ദൈവനിവേശനമുണ്ടായിരുന്നുവെന്ന ക്രൈസ്തവ വിശ്വാസത്തിന് കോട്ടം തട്ടിക്കുന്ന പല തെളിവുകളിലൊന്നാണ് ഇതുമെന്ന വസ്തുത വിസ്മരിച്ചുകൂടാ. അങ്ങനെയൊരു ദൈവിക ഇടപെടലിന്റെ സ്വാധീനത്താല്‍ രചിക്കപ്പെട്ടതായിരുന്നു സുവിശേഷങ്ങളെങ്കില്‍ തീര്‍ച്ചയായും സ്‌നാപകന്റെ യഥാര്‍ത്ഥമായ ആത്മീയ നാമമായിരുന്നു അവയില്‍ പ്രതിപാദിക്കേണ്ടിയിരുന്നത്. എന്നാല്‍ ജനങ്ങള്‍ക്കിടയില്‍ പ്രചാരത്തിലിരുന്ന യോഹന്നാന്‍ എന്ന പേര് മാത്രമെ സുവിശേഷങ്ങള്‍ ഉപയോഗിക്കുന്നുള്ളൂ. തങ്ങള്‍ക്ക് വാമൊഴിയായി പകര്‍ന്നുകിട്ടിയതിനേക്കാള്‍ അധികമായ യാതൊരു അറിവും സുവിശേഷ കര്‍ത്താക്കള്‍ക്ക് ഉണ്ടായിരുന്നില്ലെന്ന വസ്തുതയാണല്ലോ ഇത് വെളിപ്പെടുത്തുന്നത്.

യഹ്‌യായുടെ ലഖബ് ആയ 'യോഹന്നാ'യെക്കുറിച്ച് എന്തെങ്കിലും അറിവ് ഖുര്‍ആന്‍ നല്‍കുന്നുണ്ടോയെന്ന് പരിശോധിക്കുമ്പോള്‍ അതിന്റെ ദൈവികത ഒന്നുകൂടി നമുക്ക് ബോധ്യപ്പെടുകയും സര്‍വ്വശക്തനായ സ്രഷ്ടാവിന് മുമ്പില്‍ നമ്രശിരസ്‌കരാവുന്നതിലേക്ക് നാം നയിക്കപ്പെടുകയും ചെയ്യുന്നു. 'യൂ', 'ഹന്നാന്‍' എന്നീ രണ്ട് വാക്കുകളുടെ സമ്മേളനത്തില്‍നിന്നാണ് യൂഹന്നായെന്ന പദമുണ്ടായിട്ടുള്ളതെന്നും 'ഹന്നാന്‍' എന്ന ഹിബ്രു പദം 'ഹനാന്‍' എന്ന അരമായ മൂലത്തില്‍നിന്നുണ്ടായതാണെന്നും 'അനുകമ്പ'യെന്നാണ് ഈ പദത്തിന് അര്‍ത്ഥമെന്നും നേരത്തെ സൂചിപ്പിച്ചത് ഓര്‍ക്കുക. 'ഹനാന്‍' എന്ന അറബി പദവും ഇതേ അര്‍ത്ഥമുള്‍ക്കൊള്ളുന്നതാണ്. അറബി-ഹിബ്രു-അരാമിക് തുടങ്ങിയ ഭാഷകളെല്ലാം ഒരേ സെമിറ്റിക് മൂലത്തില്‍നിന്നുണ്ടായവയാണല്ലോ.

ഖുര്‍ആനില്‍ ഒരു തവണമാത്രമെ 'ഹനാന്‍' എന്ന പദം ഉപയോഗിച്ചിട്ടുള്ളൂ; സൂറത്തുമര്‍യമിലെ പതിമൂന്നാം (19:13) സൂക്തത്തില്‍. ആ സൂക്തത്തിന്റെ മലയാളം ലിപ്യന്തരണം ഇങ്ങനെയാണ്:

വ ഹനാനന്‍ മിന്‍ ലദുന്നാ വ സകാത്തന്‍ വ കാന തഖിയ്യാ ''ഈ വചനത്തിന്റെ മലയാള പരിഭാഷ ''നമ്മുടെ പക്കല്‍നിന്നുള്ള അനുകമ്പയും പരിശുദ്ധിയും നല്‍കി; അദ്ദേഹം (യഹ്‌യാ) ധര്‍മ്മനിഷ്ഠയുള്ളവനായിരുന്നു'' (19:13)വെന്നാണ്.

ഈ വചനത്തില്‍ യഹ്‌യായെക്കുറിച്ച് 'നമ്മുടെ പക്കല്‍നിന്നുള്ള അനുകമ്പ' (ഹനാനന്‍ മിന്‍ ലദുന്ന)യെന്ന് പ്രയോഗിച്ചത് പ്രത്യേകം ശ്രദ്ധേയമാണ്. യഹ്‌യാ 'ദൈവത്തില്‍നിന്നുള്ള അനുകമ്പ'യാണെന്നര്‍ത്ഥം. യൂഹന്നയെന്ന പദത്തിനര്‍ത്ഥം 'ദൈവത്തില്‍നിന്നുള്ള അനുകമ്പ'യെന്നാണെന്ന് മുമ്പ് സൂചിപ്പിച്ചത് ഓര്‍ക്കുക. 'യോഹന്ന'യിലെ അതേ ഹനാന്‍ തന്നെയാണ് ഖുര്‍ആന്‍ ഇവിടെ പ്രയോഗിച്ചിരിക്കുന്നത്. യൂഹന്നയിലെ യൂ ഒഴിവാക്കി അതിന്റെ മൂലരൂപത്തിന് തത്തുല്യമായ 'ഹനാന്‍' എന്ന് പ്രയോഗിക്കുകയാണ് ഖുര്‍ആന്‍ ചെയ്തിരിക്കുന്നത്.

ഇവിടെ 'യൂ' ഒഴിവാക്കിയിരിക്കുന്നതും പ്രത്യേകം ശ്രദ്ധിക്കേണ്ടതാണ്. 'യഹോവ'യുടെ ചുരുക്കമായാണ് 'യൂ'യെന്ന് ഉപയോഗിച്ചിരിക്കുന്നതെന്ന് നേരത്തെ സൂചിപ്പിച്ചുവല്ലോ. അറബിയില്‍ ഏകദൈവത്തെക്കുറിക്കുവാന്‍ യഹോവയെന്ന് ഉപയോഗിക്കാറില്ല. അതുകൊണ്ടുതന്നെ യഹോവയുടെ ചുരുക്കപ്പേരായ 'യൂ'യെന്ന് ഖുര്‍ആനില്‍ പ്രയോഗിക്കുന്നത് സംഗതമല്ലല്ലോ. യൂഹന്നായെന്നത് യഹ്‌യായുടെ യഥാര്‍ത്ഥ നാമമല്ലെന്നും അദ്ദേഹത്തിന്റെ സ്വഭാവ സവിശേഷതകളുടെ അടിസ്ഥാനതില്‍ ജനം വിളിച്ചിരുന്ന പേരായിരുന്നുവെന്നും അത് അദ്ദേഹത്തിന്റെ സവിശേഷത മാത്രമാണ് വെളിപ്പെടുത്തുന്നതെന്നും ഓര്‍ക്കുക. അതുകൊണ്ടുതന്നെ 'ദൈവത്തില്‍നിന്നുള്ള അനുകമ്പ'യെന്ന അര്‍ത്ഥത്തിലുള്ള യൂഹന്നായെന്ന് അതേപോലെ അറബിയില്‍ പ്രയോഗിച്ചിരുന്നുവെങ്കില്‍ ആ പദം അര്‍ത്ഥരഹിതമാകുമായിരുന്നു. യൂഹന്നായെന്നത് അദ്ദേഹത്തിന്റെ പേരല്ലല്ലോ. എന്നാല്‍ 'യൂ' ഒഴിവാക്കിക്കൊണ്ട് 'ദൈവത്തില്‍നിന്നുള്ള ഹനാന്‍' എന്ന് കൃത്യമായി ഖുര്‍ആന്‍ പ്രയോഗിച്ചത് കാണുമ്പോള്‍ അതിന്റെ സൂക്ഷ്മതയും കൃത്യതയും നമുക്ക് വ്യക്തമായി മനസ്സിലാവുകയും തെറ്റുപറ്റാത്ത സ്രഷ്ടാവില്‍ നിന്നുള്ളതാണ് ഖുര്‍ആനെന്ന് സുതരാം ബോധ്യപ്പെടുകയും ചെയ്യുന്നു. ഖുര്‍ആനില്‍ ഒരേയൊരു സ്ഥലത്ത് മാത്രമെ ഹനാന്‍ എന്ന പദം ഉപയോഗിച്ചിട്ടുള്ളൂവെന്നും അത് യഹ്‌യായെക്കുറിച്ചാണെന്നതുംകൂടി ഇതോടൊപ്പം ചേര്‍ത്തുവായിക്കുമ്പോള്‍ ആര്‍ക്കാണ് അതിന്റെ ദൈവികത ബോധ്യപ്പെടാതിരിക്കുക?

ഇനി നാം ചോദ്യത്തിലേക്ക് തിരിച്ചുപോവുക. ഖുര്‍ആനിലെ സൂറത്തുമര്‍യം ഏഴാം വചനത്തില്‍ (19:7) പറയുന്നതെന്താണ്?

''ഹേ സക്കരിയാ, തീര്‍ച്ചയായും നിനക്ക് നാം ഒരു ആണ്‍കുട്ടി യെപറ്റി സന്തോഷവാര്‍ത്ത അറിയിക്കുന്നു. അവന്റെ പേര്‍ യഹ്‌യാ എന്നാകുന്നു. മുമ്പ് നാം ആരെയും അവന്റെ പേരുള്ളവരാക്കിയിട്ടില്ല'' (വി.ഖു.19:7)

ഈ വചനം രണ്ടുതരത്തില്‍ വ്യാഖ്യാനിക്കപ്പെട്ടിട്ടുണ്ട്.:

ഒന്ന്) ഇവിടെ ''ലം നജ്അല്‍ ലഹു മിന്‍ ഖബ്‌ലു സമിയ്യാ''യെന്ന വചനഭാഗത്തെയാണ് ''മുമ്പ് ആരെയും അവന്റെ പേരുള്ളവരാക്കിയിട്ടില്ല''യെന്ന് പരിഭാഷപ്പെടുത്തിയിരിക്കുന്നത്. 'സമിയ്യന്‍' എന്ന പദത്തെയാണ് പേരുള്ളവന്‍ എന്ന് ഭാഷാന്തരം ചെയ്തിരിക്കുന്നത്. ഇതിന് ''മിഥ്‌ലന്‍'' എന്നും ''ശബീഹന്‍'' എന്നുമെല്ലാം അര്‍ത്ഥമുണ്ട്. അദ്ദേഹത്തെ പോലെയുള്ളവന്‍ എന്നര്‍ത്ഥം. അപ്പോള്‍ ഈ വചനഭാഗത്തിന് ''മുമ്പ് ആരെയും അദ്ദേഹത്തെപ്പോലെയുള്ളവനാക്കിയിട്ടില്ല''എന്ന അര്‍ത്ഥംവരും. വൃദ്ധനായ പിതാവിന് വന്ധ്യയായ ഭാര്യയിലുണ്ടായ കുഞ്ഞാണ് യഹ്‌യ. ഇങ്ങനെയൊരു സംഭവം അദ്ദേഹത്തിന് മുമ്പുണ്ടായിട്ടില്ല. ഈ അര്‍ത്ഥത്തില്‍ യഹ്‌യായെപ്പോലെ ഒരാള്‍ അദ്ദേഹത്തിന് മുമ്പുണ്ടായിട്ടില്ലെന്നാണ് ഈ വചനത്തിന് ചില പണ്ഡിതന്മാര്‍ നല്‍കിയ വ്യാഖ്യാനം.

രണ്ട്) ഈ വചനഭാഗത്തിന്റെ നേര്‍ക്കുനേരെയുള്ള അര്‍ത്ഥം പരിഗണിച്ചുകൊണ്ട് 'യഹ്‌യാ'യെന്ന പേര് സ്‌നാപകനുമുമ്പ് മറ്റാര്‍ക്കുമുണ്ടായിട്ടില്ലെന്നാണ് മറ്റൊരു വിഭാഗം പണ്ഡിതന്മാരുടെ വ്യാഖ്യാനം.

രണ്ട് വ്യാഖ്യാനങ്ങള്‍ പ്രകാരം പരിശോധിച്ചാലും ഈ ഖുര്‍ആന്‍ സൂക്തത്തില്‍ യാതൊരു വിധ അബദ്ധവുമില്ലെന്നതാണ് വാസ്തവം. യഹ്‌യാ(അ)ക്കുമുമ്പ് വൃദ്ധനായ പിതാവിന് വന്ധ്യയായ മാതാവിലുണ്ടായ ഒരു കുഞ്ഞിന്റെ കഥ ബൈബിളോ ഖുര്‍ആനോ പരാമര്‍ശിക്കുന്നില്ല. രണ്ടാമത്തെ വ്യാഖ്യാനത്തില്‍ കടിച്ചുതൂങ്ങി ഖുര്‍ആനില്‍ അബദ്ധം ആരോപിക്കുവാന്‍ വേണ്ടി ശ്രമിക്കുന്നവരുടെ വിമര്‍ശനങ്ങള്‍ ഖുര്‍ആനിന്റെ പ്രോജ്ജ്വല പ്രകാശത്തിന് മുമ്പില്‍ കരിഞ്ഞുവീഴുന്നതാണ് നാം കണ്ടത്. യഹ്‌യായെന്ന ഒരു നാമം സ്‌നാപകനുമുമ്പ് ആര്‍ക്കെങ്കിലും നല്‍കപ്പെട്ടതായി സൂചിപ്പിക്കുന്ന രേഖകളൊന്നും തന്നെയില്ല. യഹ്‌യാ=യോഹന്നാന്‍ എന്ന സമവാക്യം ഖുര്‍ആനിന്‍േറതല്ല. അതുകൊണ്ടുതന്നെ അത് വിമര്‍ശനങ്ങള്‍ക്കുമുമ്പില്‍ തകരും. എന്നാല്‍ ഖുര്‍ആന്‍ മുന്നോട്ടുവെക്കുന്ന ആശയങ്ങളാകട്ടെ ഓരോ വിമര്‍ശനങ്ങളുന്നയിക്കപ്പെടുമ്പോഴും പൂര്‍വ്വാധികം പ്രോജ്ജ്വലമായി വിളങ്ങുകമാത്രേമയുള്ളൂ.

ബൈബിളിലെ ഉല്‍പത്തി പുസ്തകം മുപ്പത്തിയൊന്‍പതാം അധ്യായത്തില്‍ യാക്കോബിന്റെ പുത്രനായ യോസഫ് ഈജിപ്തിലെത്തിയതും ഫറോവയുടെ ഉദ്യോഗസ്ഥനായ പോത്തിഫറുടെ വീട്ടില്‍ കഴിഞ്ഞതും അവിടെവെച്ച് യജമാനന്റെ ഭാര്യ അദ്ദേഹത്തെ പ്രലോഭിപ്പിക്കാന്‍ ശ്രമിച്ചതും അതില്‍നിന്ന് അദ്ദേഹം രക്ഷപ്പെട്ടതുമായകാര്യങ്ങള്‍ വിശദമായിത്തന്നെ പ്രതിപാദിക്കുന്നുണ്ട്. അത് ഇങ്ങനെയാണ്. ”ഇതിനിടയില്‍ യിസ്മാഈല്യന്‍ യോസഫിനെ ഈജിപ്തിലേക്ക് കൊണ്ടുപോയി. അവിടെ ഫറോവയുടെ ഒരു ഉദ്യോഗസ്ഥനും അംഗരക്ഷകസേനയുടെ നായകനുമായ ഈജിപ്തുകാരന്‍ പോത്തിഫര്‍ അയാളെ വിലക്കുവാങ്ങി. കര്‍ത്താവ് യോസഫിന്റെ കൂടെയുണ്ടായിരുന്നതിനാല്‍ അയാള്‍ വിജയിയായി. യജമാനനായ ഈജിപ്തുകാരന്റെ വീട്ടില്‍ അയാള്‍ താമസിച്ചു. കര്‍ത്താവ് യോസഫിന്റെ കൂടെയുണ്ട് എന്നും അയാള്‍ ചെയ്യുന്നതെല്ലാം വിജയ പ്രദമാക്കുന്നതിന് സഹായിക്കുന്നുണ്ടെന്നും യജമാനന്‍ കണ്ടു. അതുകൊണ്ട് യജമാനന്‍ യോസഫില്‍ പ്രീതനായി. യോസഫ് അയാളെ സ്വീകരിച്ചു. പോത്തിഫര്‍ യോസഫിനെ തന്റെ ഗൃഹത്തിലെ മേല്‍നോട്ടക്കാരനായി നിയമിച്ചു. തനിക്കുള്ളതെല്ലാം അയാളുടെ ചുമതലയില്‍ ഏല്‍പിച്ചുകൊടുത്തു. അയാള്‍ യോസേഫിനെ തന്റെ ഗൃഹത്തിനും തനിക്കുള്ള എല്ലാറ്റിനും ചുമതലക്കാരനാക്കി. അപ്പോള്‍ മുതല്‍ യോസേഫ് നിമിത്തം കര്‍ത്താവ് ഈജിപ്തുകാരന്‍ പോത്തിഫറുടെ കുടുംബത്തെ അനുഗ്രഹിച്ചു. കര്‍ത്താവിന്റെ അനുഗ്രഹം വീട്ടിലും വയലിലും അയാള്‍ക്കുള്ള സകലതിന്മേലുമുണ്ടായി. അതുകൊണ്ട് പോത്തിഫര്‍ തനിക്കുള്ളതെല്ലാം യോസേഫിന്റെ ചുമതലയില്‍വിട്ടു. യോസേഫ് വീട്ടിലുണ്ടായിരുന്നതിനാല്‍ ഭക്ഷണത്തിനപ്പുറം മറ്റൊന്നിനും പോത്തിഫറിന് ശ്രദ്ധിക്കേണ്ടിവന്നില്ല. സുന്ദരനും സുമുഖനുമായിരുന്നു യോസേഫ്. കുറെകാലം കഴിഞ്ഞപ്പോള്‍ യജമാനനന്റെ ഭാര്യ യോസേഫില്‍ കണ്ണുവെച്ചു. ”എന്നോടൊപ്പം ശയിക്കുക” എന്ന് അവള്‍ പറഞ്ഞു. യോസേഫ് വിസമ്മതിച്ചു. യജമാനന്റെ ഭാര്യയോട് അയാള്‍ പറഞ്ഞു; ‘നോക്കൂ, ഞാന്‍ ഇവിടെയുള്ളതുകൊണ്ട് എന്റെ യജമാനന്‍ വീട്ടിലെ ഒരു കാര്യത്തെപ്പറ്റിയും ക്ലേശിക്കുന്നില്ല. തനിക്കുള്ളതെല്ലാം അദ്ദേഹംഎന്നെ ഏല്‍പിച്ചിരിക്കയാണ്. ഈ വീട്ടില്‍ അയാള്‍ എന്നേക്കാള്‍ വലിയവനല്ല. വീട്ടില്‍ എനിക്കൊന്നും അപ്രാപ്യമാക്കിയിട്ടില്ല; നിങ്ങളെയൊഴിച്ച്. കാരണം നിങ്ങള്‍ അദ്ദേഹത്തിന്റെ ഭാര്യയാണല്ലോ.അപ്പോള്‍ പിന്നെ ഞാന്‍ എങ്ങനെ ഈ വലിയ അധര്‍മ്മം പ്രവര്‍ത്തിക്കും; ദൈവത്തിന് എതിരായി പാപം ചെയ്യും? ”. ദിവസംതോറും യോേസഫിനോട് അവള്‍ ഇക്കാര്യം പറഞ്ഞിരുന്നെങ്കിലും അവളുടെ കൂടെ ശയിക്കാനോ അവളുടെയടുക്കല്‍ ഇരിക്കാനോ അയാള്‍ കൂട്ടാക്കിയില്ല. ഒരുദിവസം ജോലിചെയ്യുന്നതിനായി യോേസഫ് വീട്ടിനുള്ളിലേക്ക് കടന്നു. പുരുഷന്മാര്‍ ആരും വീട്ടിലില്ലായിരുന്നു. അയാളുടെ മേലങ്കിയില്‍ കയറിപ്പിടിച്ച് ‘എന്നോടൊപ്പം ശയിക്കുക‘ എന്ന് അവള്‍ ആവശ്യം ഉന്നയിച്ചു. മേലങ്കി അവളുടെ കയ്യില്‍ഉപേക്ഷിച്ച് അയാള്‍ ഓടി വീടിന് വെളിയിലേക്ക് പോയി. അയാള്‍ മേലങ്കിഉപേക്ഷിച്ച് വീടിന് പുറത്തേക്ക് ഓടിപ്പോയി എന്ന് കണ്ടപ്പോള്‍ അവള്‍ വീട്ടിലുള്ളവരെ വിളിച്ചുവരുത്തി ഇപ്രകാരം പറഞ്ഞു: ‘നോക്കൂ, നമ്മെ അപമാനിക്കാന്‍ അദ്ദേഹം ഒരു എബ്രായനെ നമ്മുടെയിടയില്‍ കൊണ്ടുവന്നിരിക്കുന്നു! എന്റെ കൂടെ ശയിക്കുന്നതിന് അയാള്‍ അകത്ത് കയറിവന്നു. ഞാന്‍ ഉച്ചത്തില്‍ നിലവിളിച്ചു. ഞാന്‍ ശബ്ദം ഉയര്‍ത്തി നിലവിളിക്കുന്നതുകേട്ട ഉടന്‍ അയാള്‍ മേലങ്കി ഉപേക്ഷിച്ച് ഓടി വീടിന് വെളിയിലിറങ്ങിപ്പോയി‘. യോസഫിന്റെ യജമാനന്‍ വീട്ടില്‍ വരുന്നതുവരെ അവള്‍ അയാളുടെ മേലങ്കി കൈവശംവെച്ചു. നേരത്തെ പറഞ്ഞ കഥ തന്നെ അവള്‍ അയാളോടും പറഞ്ഞു: ‘അങ്ങ് ഞങ്ങള്‍ക്ക് കൊണ്ടുവന്ന എബ്രായ ദാസന്‍ മാനം കെടുത്താന്‍ എന്റെ അടുക്കല്‍ വന്നു. ഞാന്‍ ശബ്ദംഉയര്‍ത്തി കരഞ്ഞയുടന്‍ അയാള്‍ മേലങ്കി എന്റെയടുത്ത് ഉപേക്ഷിച്ച് വീടിന് വെളിയിലേക്ക് ഓടിപ്പോയി‘. ‘അങ്ങയുടെ ദാസന്‍ ഈ വി ധത്തിലാണ് എന്നോട് പെരുമാറിയതെന്ന് ഭാര്യ പറയുന്നത് കേട്ടപ്പോള്‍ അദ്ദേഹം കോപംകൊണ്ട് ജ്വലിച്ചു. യജമാനന്‍ യോസേഫിനെ പിടിച്ച് രാജാവിന്റെ തടവുകാരെ അടച്ചിരുന്ന ജയിലില്‍ അടച്ചു. യോസേഫ് ജയിലിലായി. എന്നാല്‍, കര്‍ത്താവ് അയാളോട് കൂടെയായിരുന്ന് അചഞ്ചലമായ സ്‌നേഹം പ്രകടിപ്പിച്ചു. തന്മൂലം തടവറ സൂക്ഷിപ്പുകാരന് അയാളോട് പ്രീതി തോന്നി. അവിടെയുള്ള എല്ലാ തടവുകാരെയും തടവറ സൂക്ഷിപ്പുകാരന്‍ യോസേഫിന്റെ മേല്‍നോട്ടത്തില്‍ വിട്ടുകൊടുത്തു; അവിടെ നടന്നതിനെല്ലാം അയാളായിരുന്നു നടത്തിപ്പുകാരന്‍. കര്‍ത്താവ് യോസേഫിനോട് കൂടെയുണ്ടായിരുന്നതിനാല്‍ യോസേഫിനെ ഭരമേല്‍പിച്ച ഒരു കാര്യത്തിനും തടവറ സൂക്ഷിപ്പുകാരന്‍ ശ്രദ്ധിക്കേണ്ടിയിരുന്നില്ല. യോസേഫിന്റെ പ്രവര്‍ത്തികള്‍ എല്ലാം ദൈവം വിജയകരമാക്കി” (ഉല്‍പത്തി 39: 1-23). യൂസുഫ് നബി (عليهالسلام)യുടെ ചരിത്രം അല്‍പം വിശദമായിത്തന്നെ ഒരുകഥാകഥനത്തിന്റെ രീതിയില്‍ ഖുര്‍ആന്‍ പന്ത്രണ്ടാം അധ്യായമായ സൂറത്തുയൂസുഫില്‍ വിവരിക്കുന്നുണ്ട്. നടേ ഉദ്ധരിച്ച ബൈബിളില്‍ വചനങ്ങളില്‍ സൂചിപ്പിക്കപ്പെട്ട സംഭവങ്ങള്‍ വിവരിക്കുന്ന ഖുര്‍ആന്‍ സൂക്തങ്ങളുടെ സാരംകാണുക: ”ഈജിപ്തില്‍നിന്ന് അവനെ (യൂസുഫിനെ) വിലക്കെടുത്തയാള്‍ തന്റെ ഭാര്യയോട് പറഞ്ഞു: ഇവന് മാന്യമായ താമസസൗകര്യം നല്‍കുക. ഇവന്‍ നമുക്ക് പ്രയോജനപ്പെട്ടേക്കാം. അല്ലെങ്കില്‍ നമുക്ക് അവനെ മകനായി സ്വീകരിക്കാം. അപ്രകാരം യൂസുഫിന് നാം ആ ഭൂപ്രദേശത്ത് സൗകര്യമുണ്ടാക്കികൊടുത്തു. സ്വപ്‌നവാര്‍ത്തകളുടെ വ്യാഖ്യാനം അദ്ദേഹത്തിന് നാം അറിയിച്ചുകൊടുക്കുവാന്‍ വേണ്ടിയാണ് അത്. അല്ലാഹു തന്റെ കാര്യം ജയിച്ചടക്കുന്നവനത്രെ. പക്ഷെ, മനുഷ്യരില്‍ അധികപേരും അത് മനസ്സിലാക്കുന്നില്ല. അങ്ങനെ അദ്ദേഹം പൂര്‍ണ്ണ വളര്‍ച്ചയെത്തിയപ്പോള്‍ അദ്ദേഹത്തിന് നാം യുക്തിബോധവും അറിവും നല്‍കി. സുകൃതം ചെയ്യുന്നവര്‍ക്ക് അപ്രകാരം നാം പ്രതിഫലം നല്‍കുന്നു. അവന്‍ (യൂസുഫ്)ഏതൊരുവളുടെ വീട്ടിലാണോ അവള്‍ അവനെ വശീകരിക്കുവാന്‍ ശ്രമംനടത്തി. വാതിലുകള്‍ അടച്ചുപൂട്ടിയിട്ട് അവള്‍ പറഞ്ഞു. ഇങ്ങോട്ട് വാ. അവന്‍പറഞ്ഞു: അല്ലാഹുവില്‍ ശരണം! നിശ്ചയമായും അവനാണ് എന്റെ രക്ഷിതാവ്. അവന്‍ എന്റെ താമസം ക്ഷേമകരമാക്കിയിരിക്കുന്നു. തീര്‍ച്ചയായും അക്രമം പ്രവര്‍ത്തിക്കുന്നവന്‍ വിജയിക്കുകയില്ല. അവള്‍ക്ക് അവനില്‍ ആഗ്രഹം ജനിച്ചു. തന്റെ രക്ഷിതാവിന്റെ പ്രമാണം കണ്ടറിഞ്ഞില്ലായിരുന്നുവെങ്കില്‍ അവന് അവളിലും ആഗ്രഹം ജനിച്ചേനെ. അപ്രകാരം (സംഭവിച്ചത്) തിന്മയും നീചവൃത്തിയും അവനില്‍നിന്ന് നാം തിരിച്ചുവിടുന്നതിന് വേണ്ടിയത്രെ. തീര്‍ച്ചയായും അവന്‍ നമ്മുടെ നിഷ്‌കളങ്കരായ ദാസന്മാരില്‍ പെട്ടവനാകുന്നു. അവര്‍ രണ്ടുപേരും വാതില്‍ക്കലേക്ക് മത്‌സരിച്ചോടി. അവള്‍ പിന്നില്‍നിന്ന് അവന്റെ കുപ്പായം(പിടിച്ചു. അത്) കീറി. അവര്‍ ഇരുവരും വാതില്‍ക്കല്‍വെച്ച് അവളുടെ നാഥനെ (ഭര്‍ത്താവിനെ) കണ്ടുമുട്ടി. അവള്‍ പറഞ്ഞു. താങ്കളുടെ ഭാര്യയുടെ കാര്യത്തില്‍ ദുരുദ്ദേശം പുലര്‍ത്തിയവനുള്ള പ്രതിഫലം, അവന്‍ തടവിലാക്കപ്പെടുക എന്നതോ, വേദനയേറിയ മറ്റേതെങ്കിലും ശിക്ഷയോ തന്നെ ആയിരിക്കണം. യൂസുഫ് പറഞ്ഞു: അവളാണ് എന്നെ വശീകരിക്കുവാന്‍ശ്രമം നടത്തിയത്. അവളുടെ കുടുംബത്തില്‍പെട്ട ഒരു സാക്ഷി ഇപ്രകാരം സാക്ഷ്യപ്പെടുത്തി. അവന്റെ കുപ്പായം മുന്നില്‍ നിന്നാണ് കീറിയിട്ടുള്ളതെങ്കില്‍ അവള്‍ സത്യമാണ് പറഞ്ഞത്. അവനാകട്ടെ കളവ്പറയുന്നവരുടെ കൂട്ടത്തിലാണ്. എന്നാല്‍ അവന്റെ കുപ്പായം പിന്നില്‍നിന്നാണ് കീറിയിട്ടുള്ളതെങ്കില്‍ അവള്‍ കളവാണ് പറഞ്ഞത്. അവനാകട്ടെ സത്യം പറയുന്നവരുടെ കൂട്ടത്തിലാണ്. അങ്ങനെ അവന്റെ(യൂസുഫിന്റെ) കുപ്പായം പിന്നില്‍ നിന്നാണ് കീറിയിട്ടുള്ളത് എന്ന്കണ്ടപ്പോള്‍ അയാള്‍ (ഗൃഹനാഥന്‍ –തന്റെ ഭാര്യയോട്) പറഞ്ഞു. തീര്‍ച്ചയായും ഇത് നിങ്ങളുടെ (സ്ത്രീകളുടെ) തന്ത്രത്തില്‍ പെട്ടതാണ്.നിങ്ങളുടെ തന്ത്രം ഭയങ്കരംതന്നെ. യൂസുഫേ, നീ ഇത് അവഗണിച്ചേക്കുക. (പെണ്ണേ) നീ നിന്റെ പാപത്തിന് മാപ്പ് തേടുക. തീര്‍ച്ചയായും നീ പിഴച്ചവരുടെ കൂട്ടത്തിലാകുന്നു. നഗരത്തിലെ ചില സ്ത്രീകള്‍ പറഞ്ഞു. പ്രഭുവിന്റെ ഭാര്യ തന്റെ വേലക്കാരനെ വശീകരിക്കാന്‍ ശ്രമിക്കുന്നു. അവള്‍ അവനോട് പ്രേമബദ്ധയായി കഴിഞ്ഞിരിക്കുന്നു. തീര്‍ച്ചയായും അവള്‍ വ്യക്തമായ പിഴവില്‍ അകപ്പെട്ടതായി ഞങ്ങള്‍ കാണുന്നു. അങ്ങനെ ആസ്ത്രീകളുടെ കുസൃതിയെപ്പറ്റി അവള്‍ കേട്ടറിഞ്ഞപ്പോള്‍ അവരുടെ അടുത്തേക്ക് അവള്‍ ആളെ അയക്കുകയും, അവര്‍ക്ക് ചാരിയിരിക്കാവുന്ന ഇരിപ്പിടങ്ങള്‍ ഒരുക്കുകയും ചെയ്തു. അവരില്‍ ഓരോരുത്തര്‍ക്കും(പഴങ്ങള്‍ മുറിക്കാന്‍) അവള്‍ ഓരോ കത്തികൊടുത്തു. (യൂസഫിനോട്) അവള്‍പറഞ്ഞു. നീ അവരുടെ മുമ്പിലേക്ക് പുറപ്പെടുക. അങ്ങനെ അവനെ അവര്‍കണ്ടപ്പോള്‍ അവര്‍ക്ക് അവനെപ്പറ്റി വിസ്മയം തോന്നുകയും, അവരുടെ സ്വന്തം കൈകള്‍ തന്നെ അറുത്ത് പോവുകയും ചെയ്തു. അവര്‍ പറഞ്ഞു. അല്ലാഹു എത്ര പരിശുദ്ധന്‍. ഇത് ഒരു മനുഷ്യനല്ല, ആദരണീയനായ ഒരുമലക്ക് തന്നെയാണ്. അവള്‍ പറഞ്ഞു: എന്നാല്‍ ഏതൊരുവന്റെ കാര്യത്തില്‍ നിങ്ങളെന്നെ ആക്ഷേപിച്ചുവോ അവനാണ് ഇത്. തീര്‍ച്ചയായും അവനെ ഞാന്‍ വശീകരിക്കാന്‍ ശ്രമിച്ചിട്ടുണ്ട്. അപ്പോള്‍ അവന്‍ (സ്വയം കളങ്കപ്പെടുത്താതെ) കാത്തു സൂക്ഷിക്കുകയാണ് ചെയ്ത്. ഞാന്‍ അവനോട് കല്‍പിക്കുന്നപ്രകാരം അവന്‍ ചെയ്തില്ലെങ്കില്‍ തീര്‍ച്ചയായും അവന്‍ തടവിലാക്കപ്പെടുകയും നിന്ദ്യരുടെ കൂട്ടത്തിലായിരിക്കുകയും ചെയ്യും. അവന്‍ (യൂസുഫ്) പറഞ്ഞു. എന്റെ രക്ഷിതാവേ, ഇവര്‍ എന്നെ ഏതൊന്നിലേക്ക് ക്ഷണിക്കുന്നുവോ അതിനേക്കാളും എനിക്ക് കൂടുതല്‍ പ്രിയപ്പെട്ടത് ജയിലാകുന്നു. ഇവരുടെ കുതന്ത്രം എന്നെവിട്ട് നീ തിരിച്ചുകളയാത്ത പക്ഷം ഞാന്‍ അവരിലേക്ക് ചാഞ്ഞുപോയേക്കും. അങ്ങനെ ഞാന്‍ അവിവേകികളുടെ കൂട്ടത്തിലായിപ്പോവുകയും ചെയ്യും. അപ്പോള്‍ അവന്റെ പ്രാര്‍ത്ഥന തന്റെ രക്ഷിതാവ് സ്വീകരിക്കുകയും അവരുടെ കുതന്ത്രം അവനില്‍നിന്ന് അവന്‍ തട്ടിത്തിരിച്ച് കളയുകയുംചെയ്തു. തീര്‍ച്ചയായും അവന്‍ എല്ലാം കേള്‍ക്കുന്നവനും കാണുന്നവനുമത്രെ. പിന്നീട് തെളിവുകള്‍ കണ്ടറിഞ്ഞതിനുശേഷവും അവര്‍ക്ക് തോന്നി; അവനെ ഒരവധിവരെ തടവിലാക്കുകതന്നെവേണമെന്ന്” (വി.ഖു. 12:21-35). ഖുര്‍ആനും ബൈബിളും ഒരേ കഥയാണ് പരാമര്‍ശിക്കുന്നതെങ്കിലും അവയുടെ വിശദാംശങ്ങളില്‍ ഏറെ വ്യത്യാസങ്ങളുണ്ടെന്ന് ഇവ രണ്ടും വായിച്ചുനോക്കുന്ന ആര്‍ക്കും മനസ്സിലാക്കാവുന്നതേയുള്ളൂ. യൂസുഫ് കുറ്റക്കാരനാണെന്ന് യജമാനനും മറ്റും കരുതിയെന്ന രൂപത്തിലാണ് ബൈബിളിന്റെ വിവരണം. ഖുര്‍ആനിലാകട്ടെ യൂസുഫിന്റെ നിരപരാധിത്വം ബോധ്യപ്പെട്ടശേഷവും അദ്ദേഹത്തെ തടവിലാക്കുവാന്‍ യജമാനന്‍ തീരുമാനിച്ചതായാണ് പറഞ്ഞിരിക്കുന്നത്. മേലങ്കി ഊരി ഓടിയ യൂസുഫിനെയാണ് ബൈബിള്‍ പരിചയപ്പെടുത്തുന്നത്. യജമാന ഭാര്യ പിന്നില്‍നിന്ന് പിടിച്ചപ്പോള്‍ കുപ്പായം കീറിയതായാണ് ഖുര്‍ആനിലെ വിവരണം. ഇങ്ങനെ വിശദാംശങ്ങളില്‍ നിരവധി വ്യത്യാസങ്ങളുണ്ടെന്നതാണ് വാസ്തവം. യജമാനന്റെ പേരുമായി ബന്ധപ്പെട്ട് ഖുര്‍ആന്‍ നടത്തുന്ന പരാമര്‍ശങ്ങളുടെ വെളിച്ചത്തിലാണ് വിമര്‍ശകന്മാര്‍ ഖുര്‍ആനില്‍ അബദ്ധം ആരോപിക്കുന്നത്. ‘ബൈബിളില്‍ വ്യക്തമായിത്തന്നെ യജമാനന്റെ പേര് പറഞ്ഞിരിക്കുന്നു. പോത്തിഫര്‍. ഖുര്‍ആനിലാകട്ടെ യൂസുഫി(عليه السلام)ന്റെയജമാനനെക്കുറിച്ച് പരാമര്‍ശിക്കുമ്പോള്‍ അസീസ് എന്നാണ്പ്രയോഗിച്ചിരിക്കുന്നത്. പോത്തിഫര്‍ എന്ന നാമവുമായി ഒരു നിലക്കും യോജിക്കുന്ന പേരല്ല അസീസ്. മാത്രവുമല്ല, ഒരു ശുദ്ധ അറബി പദമാണ് ‘അസീസ്‘. യോസഫിന്റെ കാലത്തെ ഈജിപ്തില്‍ ജീവിച്ച ഒരു വ്യക്തിക്ക് ശുദ്ധമായ ഒരു അറബിപ്പേരുണ്ടായിരുന്നുവെന്ന് കരുതുന്നതുപോലും വിഡ്ഢിത്തമാണ്. യഹൂദ ക്രിസ്ത്യാനികളില്‍നിന്ന് യോസഫിന്റെ കഥകേട്ട് നാളുകള്‍ക്കുശേഷം മുഹമ്മദ് ഖുര്‍ആനില്‍ അത് എഴുതിച്ചേര്‍ത്തപ്പോള്‍ ഉണ്ടായ ഒരു അബദ്ധമാണിത്……..’ വിമര്‍ശകരുടെ ന്യായവാദങ്ങള്‍ ഇങ്ങനെപോകുന്നു. തങ്ങളുടെ വാദം സ്ഥാപിക്കുന്നതിനായി വിമര്‍ശകര്‍ ഉദ്ധരിക്കുന്ന ഖുര്‍ആന്‍ സൂക്തങ്ങളുടെ സാരമാണ് താഴെ: ”നഗരത്തിലെ ചില സ്ത്രീകള്‍ പറഞ്ഞു: പ്രഭു(അല്‍-അസീസ്)വിന്റെഭാര്യ തന്റെ വേലക്കാരനെ വശീകരിക്കാന്‍ ശ്രമിക്കുന്നു. അവള്‍ അവനോട് പ്രേമബദ്ധയായിക്കഴിഞ്ഞിരിക്കുന്നു. തീര്‍ച്ചയായും അവള്‍ വ്യക്തമായ പിഴവില്‍ അകപ്പെട്ടതായി ഞങ്ങള്‍ കാണുന്നു” (വി.ഖു: 12:30). ”(ആ സ്ത്രീകളെ) വിളിച്ചുവരുത്തിയിട്ട് അദ്ദേഹം (രാജാവ്) ചോദിച്ചു. യൂസുഫിനെ വശീകരിക്കുവാന്‍ നിങ്ങള്‍ ശ്രമം നടത്തിയപ്പോള്‍ നിങ്ങളുടെ സ്ഥിതി എന്തായിരുന്നു? അവര്‍ പറഞ്ഞു. അല്ലാഹു എത്ര പരിശുദ്ധന്‍. ഞങ്ങള്‍ യൂസുഫിനെപ്പറ്റി ദോഷകരമായ ഒന്നും മനസിലാക്കിയിട്ടില്ല. പ്രഭു(അല്‍-അസീസ്)വിന്റെ ഭാര്യ പറഞ്ഞു: ഇപ്പോള്‍ സത്യം വെളിപ്പെട്ടിരിക്കുന്നു. ഞാന്‍ അദ്ദേഹത്തെ വശീകരിക്കുവാന്‍ ശ്രമിക്കുകയാണുണ്ടായത്. തീര്‍ച്ചയായും അദ്ദേഹം സത്യവാന്മാരുടെ കൂട്ടത്തില്‍ തന്നെയാകുന്നു” വി.ഖു: 12:51). ഈ സൂക്തങ്ങളില്‍ പ്രഭുവെന്ന് പരിഭാഷപ്പെടുത്തിയിരിക്കുന്നത് ‘അല്‍അസീസ്‘ എന്ന അറബി പദത്തെയാണ്. ചില ഇംഗ്ലീഷ് പരിഭാഷകളില്‍ ഈപദം ഭാഷാന്തരം ചെയ്യാതെ അല്‍-അസീസ് എന്നുതന്നെ അതേപോലെ ഒരുനാമമെന്ന രൂപത്തില്‍ കൊടുത്തിട്ടുണ്ട്. അത് എടുത്തുകൊണ്ടാണ് പോത്തിഫറിന് പകരമായി അസീസ് എന്നാണ് ഖുര്‍ആനില്‍ പ്രയോഗിച്ചിരിക്കുന്നതെന്ന് വിമര്‍ശകര്‍ വാദിക്കുന്നത്. അസീസ് എന്ന അറബി നാമത്തിന്റെ അര്‍ത്ഥം ‘പ്രതാപി‘ യെന്നാണ്. അറബികള്‍ ഇത് ഒരു പേരായും പേരിന്റെ ഭാഗമായും ഉപയോഗിക്കാറുണ്ടെന്നത് ശരിയാണ്. അമുസ്‌ലിംകളും ഈ പേര് ഉപയോഗിക്കാറുണ്ട്. ഇറാഖി മന്ത്രിയായ താരീഖ് അസീസിന്റെ പേര് ഉദാഹരണം. അസീസിന്റെ മുമ്പില്‍ ‘അല്‍’ എന്ന അവ്യയം (definite article) ചേര്‍ത്ത്അല്‍-അസീസ് എന്നാണ് നടേ സൂചിപ്പിക്കപ്പെട്ട ഖുര്‍ആന്‍ സൂക്തങ്ങളില്‍ പ്രയോഗിച്ചിരിക്കുന്നത്. ഒരു പേര് എന്ന നിലയ്ക്ക് അറബിയില്‍ അല്‍-അസീസ് എന്നു പയോഗിക്കാറില്ല. ഇംഗ്ലീഷില്‍ the Lord എന്ന് പറയുന്നതിന് സമാനമായ ഒരു പ്രയോഗമാണിത്. അതുകൊണ്ടാണ് മലയാളത്തില്‍ ‘പ്രഭു‘വെന്ന് പരിഭാഷപ്പെടുത്തിയിരിക്കുന്നത്. വ്യക്തിയുടെ സ്ഥാനമാണ് അല്ലാതെ പേരല്ല അല്‍-അസീസ് എന്ന പ്രയോഗം ദ്യോതിപ്പിക്കുന്നത്. ബൈബിളില്‍ പോത്തിഫര്‍ എന്ന് വിളിച്ച വ്യക്തിയുടെ പേര് സൂചിപ്പിച്ചുകൊണ്ടുള്ള ഒരു പ്രയോഗമെന്ന നിലയ്ക്കല്ല, പ്രത്യുത അദ്ദേഹത്തിന്റെ സ്ഥാനം വ്യക്തമാക്കുന്ന ഒരു പദമെന്ന നിലയ്ക്കാണ് ഖുര്‍ആനില്‍ അല്‍-അസീസ് എന്ന് പറഞ്ഞിരിക്കുന്നത് എന്നര്‍ത്ഥം. പോത്തിഫറിനെക്കുറിച്ച് ബൈബിളില്‍ പറഞ്ഞിരിക്കുന്നത്” ഫറോവാന്റെ ഒരു ഉദ്യോഗസ്ഥനും അംഗരക്ഷക സേനയുടെ നായകനു” (ഉല്‍ 39:1)മെന്നാണല്ലോ. ഈ ഉന്നതസ്ഥാനം വ്യക്തമാക്കാനാണ് ഖുര്‍ആന്‍ ”അല്‍-അസീസ്” എന്ന് പ്രയോഗിച്ചതെന്നാണ് മനസ്സിലാവുന്നത്. ബൈബിളില്‍ പ്രയോഗിച്ച പോത്തിഫര്‍ എന്ന നാമം എന്തുകൊണ്ടാണ് ഖുര്‍ആനില്‍ ഉപയോഗിക്കാതിരുന്നത് എന്ന് ഖണ്ഡിതമായിപ്പറയാന്‍ നമുക്കാവില്ല. സര്‍വ്വശക്തന്റെ വചനങ്ങളുടെ സമാഹാരമാണ് ഖുര്‍ആന്‍. അതിലെ ഓരോ പദവും ഉപയോഗിച്ചതിനു പിന്നില്‍ എന്തെന്ത് യുക്തികളാണുള്ളതെന്ന് പറയാന്‍ പരിമിതമായ അറിവ് മാത്രം നല്‍കപ്പെട്ട നമുക്ക് കഴിയില്ല. ഇക്കാര്യത്തില്‍ സ്ഥിരപ്പെട്ട ഹദീസുകളോ പണ്ഡിതാഭിപ്രായങ്ങളോ ഉദ്ധരിക്കപ്പെട്ടതായി കാണാനും കഴിയുന്നില്ല. ‘പോത്തിഫര്‍’ എന്ന് ഖുര്‍ആനില്‍ പറയാതിരുന്നതിന്റെ കാരണം കൃത്യമായി സര്‍വ്വശക്തനേ അറിയൂ. എങ്കിലും, ‘പോത്തിഫര്‍’ എന്ന ഈജിപ്ഷ്യന്‍ നാമത്തിന്റെ അര്‍ത്ഥമെന്താണെന്ന് മനസ്സിലാക്കുമ്പോള്‍ ഖുര്‍ആനില്‍ പദങ്ങള്‍ പ്രയോഗിച്ചതിലുള്ള സൂക്ഷ്മത നമുക്ക് ഒരിക്കല്‍കൂടി ബോധ്യപ്പെടുമെന്നകാര്യം ഇവിടെ ചൂണ്ടിക്കാട്ടാമെന്ന് തോന്നുന്നു. പോത്തിഫര്‍ എന്ന ഈജിപ്ഷ്യന്‍ പദത്തിനര്‍ത്ഥം ‘റേ‘യാല്‍ നല്‍കപ്പെട്ടവന്‍” (he whom Re has given-Harpers Bible Dictionary P. 809) ഈജിപ്തുകാരുടെ സൂര്യദേവന്റെ പേരാണ്‘റേ‘ സൂര്യദേവന്റെ ദാനം‘ എന്ന് വേണമെങ്കില്‍ ”പോത്തിഫര്‍” എന്ന പേര് പരിഭാഷപ്പെടുത്താം. സൂര്യാരാധനയുമായി ബന്ധപ്പെട്ടതാണ് ഈ പേര്. ബഹുദൈവാരാധനയുമായി ബന്ധപ്പെടുന്ന നാമങ്ങളോ പ്രയോഗങ്ങളോ ഉപയോഗിക്കുവാന്‍ പാടില്ലെന്നാണ് ഇസ്‌ലാമിന്റെ അനുശാസന. ഇതുകൊണ്ടു കൂടിയായിരിക്കാം ഖുര്‍ആന്‍ ‘പോത്തിഫര്‍’ എന്നപേരുപയോഗിക്കാതെ അദ്ദേഹത്തിന്റെ സ്ഥാനത്തെ ദ്യോതിപ്പിക്കുന്ന ‘അല്‍-അസീസ് എന്നുമാത്രം പ്രയോഗിച്ചത്. ഖുര്‍ആന്‍ ‘പോത്തിഫര്‍’ എന്ന പേര് ഉപയോഗിക്കുകയാണെങ്കില്‍ ‘സൂര്യദേവന്റെ ദാന‘മാണ് അദ്ദേഹമെന്ന് അംഗീകരിക്കുന്നുവെന്നാണല്ലോ വന്നുചേരുക. ദൈവിക വചനങ്ങളുടെ സമാഹാരമാണ് ഖുര്‍ആന്‍ എന്ന വസ്തുത ഇവിടെയും വ്യക്തമായി വെളിപ്പെടുന്നു. ബൈബിളില്‍ നിന്ന് മുഹമ്മദ് നബി(ﷺ) പകര്‍ത്തിയെഴുതിയതായിരുന്നു യോസേഫിന്റെ കഥയെങ്കില്‍ സ്വാഭാവികമായും പോത്തിഫര്‍ എന്ന യജമാനനാമം ഖുര്‍ആനില്‍ സ്ഥാനം പിടിക്കേണ്ടതായിരുന്നു. പോത്തിഫര്‍ എന്നാല്‍ ”സൂര്യദേവന്റെ ദാനം”എന്നാണ് അര്‍ത്ഥമെന്ന് മുഹമ്മദ് നബിക്ക് അറിയാമായിരുന്നുവെന്നും അത്ബഹുദൈവ വിശ്വാസമുള്‍ക്കൊള്ളുന്നതായതിനാല്‍ ഖുര്‍ആനില്‍ നിന്ന്ഒഴിവാക്കിയെന്നും കരുതുന്നത് ശുദ്ധ ഭോഷ്‌ക്കാണെന്ന് പറയേണ്ടതില്ല. ഖുര്‍ആനിന്റെ അവതരണ കാലത്ത് ഈജിപ്ഷ്യന്‍ ഭാഷ അറിയാവുന്നവരായി ആരും തന്നെ അറേബ്യയിലുണ്ടായിരുന്നില്ലെന്ന വസ്തുത ഓര്‍ക്കുക. അതേപോലെതന്നെ പോത്തിഫര്‍ എന്ന നാമമൊഴിവാക്കിയത് യാദൃച്ഛികമാണെന്ന് കരുതാനും ന്യായമില്ല. ബൈബിളില്‍നിന്ന് കാര്യങ്ങള്‍ പകര്‍ത്തിയെഴുതുമ്പോള്‍ അതില്‍ പല തവണപ്രയോഗിക്കപ്പെട്ട ഒരു പേര് യാദൃച്ഛികമായി ഒഴിവായിപ്പോയെന്ന് കരുതുന്നതെങ്ങനെ? ഖുര്‍ആനിന്റെ ദൈവികതയ്ക്കുള്ള തെളിവാണിതെന്ന് പറയുന്നത് അതുകൊണ്ടാണ്. സര്‍വ്വജ്ഞനായ തമ്പുരാന്റെ വചനങ്ങളിലെവിടെയും യാതൊരു സ്ഖലിതവുമുണ്ടാവുകയില്ലെന്ന വസ്തുത ഇവിടെ നമുക്ക് ബോധ്യപ്പെടുന്നു. ഖുര്‍ആനിനെതിരെ ഉന്നയിക്കപ്പെട്ട എല്ലാ ആരോപണങ്ങളും ഖുര്‍ആനിന്റെ ദൈവികത വ്യക്തമാവുന്നിടത്തേക്കാണ് നമ്മെ നയിക്കുന്നതെന്നുള്ളത് എന്തുമാത്രം വിസ്മയകരമായിരിക്കുന്നു!
 

ഖുര്‍ആനിലെ ഇരുപത്തിയേഴാം അധ്യായമായ സൂറത്തുന്നംലിലെ 20മുതല്‍ 46വരെയുള്ള സൂക്തങ്ങളില്‍ സബഇലെ രാജ്ഞിയായിരുന്ന ബില്‍ഖീസിനെപ്പറ്റി സുലൈമാന്‍ നബി (അ)  മരംകൊത്തിപ്പക്ഷിയില്‍നിന്നും അറിഞ്ഞതും അവരെ ഇസ്‌ലാമിലേക്ക് ക്ഷണിച്ചുകൊണ്ട് എഴുത്തെഴുതിയതും അവര്‍ സുലൈമാന്‍ നബി (അ) യുടെ കൊട്ടാരം സന്ദര്‍ശിച്ചതുമെല്ലാം ഒരു കഥാകഥനത്തിന്റെ രൂപത്തില്‍തന്നെ അവതരിപ്പിക്കുന്നുണ്ട്. പ്രസ്തുത ഖുര്‍ആന്‍ സൂക്തങ്ങളുടെ സാരം കാണുക:

''അദ്ദേഹം പക്ഷികളെ പരിശോധിക്കു കയുണ്ടായി. എന്നിട്ട് അദ്ദേഹം പറഞ്ഞു: എന്തുപറ്റി? മരംകൊത്തിയെ കാണുന്നില്ലല്ലോ, അഥവാ അത് സ്ഥലംവിട്ടുപോയ കൂട്ടത്തിലാണോ? ഞാന്‍ അതിന് കഠിനശിക്ഷ നല്‍കുകയോ അല്ലെങ്കില്‍ അറുക്കുകയോ തന്നെ ചെയ്യും. അല്ലെങ്കില്‍ വ്യക്തമായ വല്ല ന്യായവും അത് എനിക്ക് ബോധിപ്പിച്ചുതരണം. എന്നാല്‍ അത് എത്തിച്ചേരാന്‍ അധികം താമസിച്ചില്ല. എന്നിട്ടത് പറഞ്ഞു: താങ്കള്‍ സൂക്ഷ്മമായി അറിഞ്ഞിട്ടില്ലാത്ത ഒരു കാര്യം ഞാന്‍ സൂക്ഷ്മമായി മനസിലാക്കിയിട്ടുണ്ട്. 'സബഇ'ല്‍ നിന്ന് യഥാര്‍ത്ഥമായ ഒരു വാര്‍ത്തയുംകൊണ്ടാണ് ഞാന്‍ വന്നിരിക്കുന്നത്. ഒരു സ്ത്രീ അവരെ ഭരിക്കുന്നതായി ഞാന്‍ കണ്ടെത്തുകയുണ്ടായി. എല്ലാ കാര്യങ്ങളില്‍നിന്നും അവള്‍ക്ക് നല്‍കപ്പെട്ടിട്ടുണ്ട്. അവള്‍ക്ക് ഗംഭീരമായ ഒരു സിംഹാസനവുമുണ്ട്. അവളും അവളുടെ ജനതയും അല്ലാഹുവിന് പുറമെ സൂര്യനെ പ്രണാമം ചെയ്യുന്നതായിട്ടാണ് ഞാന്‍ കണ്ടെത്തിയത്. പിശാച് അവര്‍ക്ക് തങ്ങളുടെ പ്രവര്‍ത്തനങ്ങള്‍ ഭംഗിയായി തോന്നിക്കുകയും അവരെ നേര്‍മാര്‍ഗത്തില്‍നിന്ന് തടയുകയും ചെയ്തിരിക്കുന്നു. അതിനാല്‍ അവര്‍ നേര്‍വഴിപ്രാപിക്കുന്നില്ല. ആകാശങ്ങളിലും ഭൂമിയിലും ഒളിഞ്ഞുകിടക്കുന്നത് പുറത്ത് കൊണ്ടുവരികയും നിങ്ങള്‍ രഹസ്യമാക്കുന്നതും പരസ്യമാക്കുന്നതും അറിയുകയും ചെയ്യുന്നവനായ അല്ലാഹുവിന് അവര്‍ പ്രണാമം ചെയ്യാതിരിക്കാന്‍വേണ്ടി (പിശാച് അങ്ങനെ ചെയ്യുന്നു). മഹത്തായ സിംഹാസനത്തിന്റെ നാഥനായ അല്ലാഹു അല്ലാതെ യാതൊരു ദൈവവുമില്ല. സുലൈമാന്‍ പറഞ്ഞു: നീ സത്യം പറയുന്നതാണോ അതല്ലാ നീ കള്ളം പറയുന്നവരുടെ കൂട്ടത്തിലായിരിക്കുന്നുവോ എന്ന് നാം നോക്കാം. നീ എന്റെ ഈ എഴുത്ത് കൊണ്ടുപോയി അവര്‍ക്കിട്ട് കൊടുക്കുക. പിന്നീട് നീ അവരില്‍നിന്ന് മാറിനിന്ന് അവ ര്‍ എന്ത് മറുപടി നല്‍കുന്നുവെന്ന് നോക്കുക. അവള്‍ പറഞ്ഞു: ഹേ, പ്രമുഖന്മാരേ, എനിക്ക് ഇതാ മാന്യമായ ഒരു എഴുത്ത് നല്‍കപ്പെട്ടിരിക്കുന്നു. അത് സുലൈമാന്റെ പക്കല്‍നിന്നുള്ളതാണ്. ആ കത്ത് ഇപ്രകാരമത്രെ: 'പരമകാരുണികനും കരുണാനിധിയുമായ അല്ലാഹുവിന്റെ നാമത്തില്‍. എനിക്കെതിരില്‍ നിങ്ങള്‍ അഹങ്കാരം കാണിക്കാതിരിക്കുകയും കീഴൊതുങ്ങിയവരായിക്കൊണ്ട് നിങ്ങള്‍ എന്റെ അടുത്ത് വരികയും ചെയ്യുക'. അവള്‍ പറഞ്ഞു: ഹേ പ്രമുഖന്മാരേ, എന്റെ ഈ കാര്യത്തില്‍ നിങ്ങള്‍ എനിക്ക് നിര്‍ദ്ദേശം നല്‍കുക. നിങ്ങള്‍ എന്റെ അടുത്ത് സന്നിഹിതരായിട്ടല്ലാതെ യാതൊരു കാര്യവും ഖണ്ഡിതമായി തീരുമാനിക്കുന്നവളല്ല ഞാന്‍. അവര്‍ പറഞ്ഞു: നാം ശക്തിയുള്ളവരും ഉഗ്രമായ സമരവീര്യമുള്ളവരുമാണ്. അധികാരം അങ്ങേക്കാണല്ലോ. അതിനാല്‍ എന്താണ് കല്‍പിച്ചരുളേണ്ടതെന്ന് ആലോചിച്ചുനോക്കുക. അവള്‍ പറഞ്ഞു: തീര്‍ച്ചയായും രാജാക്കന്മാര്‍ ഒരു നാട്ടില്‍ കടന്നാല്‍ അവര്‍ അവിടെ നാശമുണ്ടാക്കുകയും അവിടത്തുകാരിലെ പ്രതാപികളെ നിന്ദ്യന്മാരാക്കുകയും ചെയ്യുന്നവതാണ്. അപ്രകാരമാണ് അവര്‍ ചെയ്തു കൊണ്ടിരിക്കുന്നത്. ഞാന്‍ അവര്‍ക്ക് ഒരു പാരിതോഷികം കൊടുത്തയച്ചിട്ട് എന്തൊരു വിവരവും കൊണ്ടാണ് ദൂതന്മാര്‍ മടങ്ങിവരുന്നതെന്ന് നോക്കാന്‍ പോവുകയാണ്. അവന്‍ (ദൂതന്‍) സുലൈമാന്റെ അടുത്ത് ചെന്നപ്പോള്‍ അദ്ദേഹം പറഞ്ഞു: നിങ്ങള്‍ എന്നെ സമ്പത്ത് തന്ന് സഹായിക്കുകയാണോ?എന്നാല്‍ എനിക്ക് അല്ലാഹു നല്‍കിയിട്ടുള്ളതാണ് നിങ്ങള്‍ക്കവന്‍ നല്‍കിയിട്ടുള്ളതിനേക്കാള്‍ ഉത്തമം. പക്ഷെ, നിങ്ങള്‍ നിങ്ങളുടെ പാരിതോഷികം കൊണ്ട് സന്തോഷം കൊള്ളുകയാകുന്നു. നീ അവരുടെ അടുത്തേക്ക് തന്നെ മടങ്ങിച്ചെല്ലുക. തീര്‍ച്ചയായും അവര്‍ക്ക് നേരിടാന്‍ കഴിയാത്ത സൈന്യങ്ങളെയും കൊണ്ട് നാം അവരുടെ അടുത്ത് ചെല്ലുകയും നിന്ദ്യരും അപമാനിതരുമായ നിലയില്‍ അവരെ നാം അവിടെനിന്ന് പുറത്താക്കുകയും ചെയ്യുന്നതാണ്. അദ്ദേഹം (സുലൈമാന്‍) പറഞ്ഞു: ഹേ പ്രമുഖന്മാരേ, അവര്‍ കീഴൊതുങ്ങിക്കൊണ്ട് എന്റെ അടുക്കല്‍ വരുന്നതിന് മുമ്പായി നിങ്ങളില്‍ ആരാണ് അവളുടെ സിംഹാസനം എനിക്ക് കൊണ്ടുവന്ന് തരിക? ജിന്നുകളുടെ കൂട്ടത്തിലെ ഒരു മല്ലന്‍ പറഞ്ഞു: അങ്ങ് അങ്ങയുടെ ഈ സദസ്സില്‍നിന്ന് എഴുന്നേല്‍ക്കുന്നതിന് മുമ്പായി ഞാന്‍ അത് അങ്ങേക്ക് കൊണ്ടുവന്ന് തരാം. തീര്‍ച്ചയായും ഞാന്‍ അതിന് കഴിവുള്ളവനും വിശ്വസ്തനുമാകുന്നു. വേദത്തില്‍നിന്നുള്ള വിജ്ഞാനം കരസ്ഥമാക്കിയിട്ടുള്ള ആള്‍ പറഞ്ഞു: താങ്കളുടെ ദൃഷ്ടി താങ്കളിലേക്ക് തിരിച്ചുവരുന്നതിന് മുമ്പായി ഞാന്‍ അത് താങ്കള്‍ക്ക് കൊണ്ടുവന്ന് തരാം. അങ്ങനെ അത് (സിംഹാസനം) തന്റെ അടുക്കല്‍ സ്ഥിതിചെയ്യുന്നതായി കണ്ടപ്പോള്‍ അദ്ദേഹം പറഞ്ഞു: ഞാന്‍ നന്ദി കാണിക്കുമോ അതല്ല നന്ദികേട് കാണിക്കുമോ എന്ന് എന്നെ പരീക്ഷിക്കുവാനായി എന്റെ രക്ഷിതാവ് എനിക്ക് നല്‍കിയ അനുഗ്രഹത്തില്‍പെട്ടതാകുന്നു ഇത്. വല്ലവനും നന്ദികാണിക്കുന്നപക്ഷം തന്റെ ഗുണത്തിനായിട്ട് തന്നെയാകുന്നു അവന്‍ നന്ദി കാണിക്കുന്നത്. വല്ല വനും നന്ദികേട് കാണിക്കുന്നപക്ഷം തീര്‍ച്ചയായും എന്റെ രക്ഷിതാവ് പരാശ്രയമുക്തനും ഉല്‍കൃഷ്ടനുമാകുന്നു. അദ്ദേഹം (സുലൈമാന്‍) പറഞ്ഞു: നിങ്ങള്‍ അവളുടെ സിംഹാസനം അവള്‍ക്ക് തിരിച്ചറിയാത്ത വിധത്തില്‍ മാറ്റുക. അവള്‍ യാഥാര്‍ത്ഥ്യം മനസിലാക്കുമോ അതല്ല അവള്‍ യാഥാര്‍ത്ഥ്യം കണ്ടെത്താത്തവരുടെ കൂട്ടത്തിലായിരിക്കുമോ എന്ന് നമുക്ക് നോക്കാം. അങ്ങനെ അവള്‍ വന്നപ്പോള്‍ (അവളോട്) ചോദിക്കപ്പെട്ടു: താങ്കളുടെ സിംഹാസനം ഇതുപോലെയാണോ? അവള്‍ പറഞ്ഞു: ഇത് അത് തന്നെയാണെന്ന് തോന്നുന്നു. ഇതിന് മുമ്പുതന്നെ ഞങ്ങള്‍ക്ക് അറിവ് നല്‍കപ്പെട്ടിരുന്നു. ഞങ്ങള്‍ മുസ്‌ലിംകളാവുകയും ചെയ്തിരുന്നു. അല്ലാഹുവിന് പുറമെ അവള്‍ ആരാധിച്ചിരുന്നതില്‍നിന്ന് അദ്ദേഹം അവളെ തടയുകയും ചെയ്തു. തീര്‍ച്ചയായും അവള്‍ സത്യനിഷേധികളായ ജനതയില്‍പെട്ടവളായിരുന്നു. കൊട്ടാരത്തില്‍ പ്രവേശിച്ചുകൊള്ളുക എന്ന് അവളോട് പറയപ്പെട്ടു. എന്നാല്‍ അവള്‍ അത് കണ്ടപ്പോള്‍ അത് ഒരു ജലാശയമാണെന്ന് വിചാരിക്കുകയും തന്റെ കണങ്കാലുകളില്‍നിന്ന് വസ്ത്രം മേലോട്ട് നീക്കുകയും ചെയ്തു. സുലൈമാന്‍ പറഞ്ഞു: ഇത് സ്ഫടിക കഷണങ്ങള്‍ പാകിമിനുക്കിയ ഒരു കൊട്ടാരമാകുന്നു. അവള്‍ പറഞ്ഞു: എന്റെ രക്ഷിതാവേ, ഞാന്‍ എന്നോട് തന്നെ അന്യായം ചെയ്തിരിക്കുന്നു. ഞാനിതാ സുലൈമാനോടൊപ്പം ലോകരക്ഷിതാവായ അല്ലാഹുവിന് കീഴ്‌പ്പെട്ടിരിക്കുന്നു'' (വി.ഖു. 27:20-44)

സോളമന്റെകാലത്ത് ശേബായിലെ രാജ്ഞി അദ്ദേഹത്തിന്റെയടുക്കല്‍ ചെന്നതും അദ്ദേഹത്തിന്റെ ജ്ഞാനവും പ്രതാപവും കണ്ട് വളരെയേറെ പ്രശംസിച്ചതുമായി ബൈബിള്‍ പഴയ നിയമത്തിലുമുണ്ട്. (1 രാജാക്കന്മാര്‍ 10:1-13; 2 ദിനവൃത്താന്തം 9:1-12). എന്നാല്‍ ഖുര്‍ആനിലേതുപോലെയുള്ള വിശദമായ പരാമര്‍ശങ്ങള്‍ ബൈബിളിലെവിടെയുമില്ല. എന്നാല്‍ ചില യഹൂദ തര്‍ഗൂമുകളില്‍ ഈ കഥ ഏകദേശം ഖുര്‍ആനിലേതിന് തുല്യമായ രീതിയില്‍ വിവരിച്ചിട്ടുണ്ട്. തര്‍ഗുമുകളിലെ ഈ വിവരണങ്ങളില്‍നിന്ന് മുഹമ്മദ് നബി (സ) പകര്‍ത്തിയെഴുതിയതാണ് സുലൈമാന്‍-ബില്‍ഖീസ് രാജ്ഞി കഥയെന്നാണ് വിമര്‍ശകരുടെ വാദം.

'തര്‍ഗൂം' എന്ന അരമായ പദത്തിനര്‍ത്ഥം വിവര്‍ത്തനം അല്ലെങ്കില്‍ വ്യാഖ്യാനമെന്നാണ്. പഴയ നിയമഗ്രന്ഥങ്ങള്‍ക്ക് യഹൂദരുടെ ഇടയില്‍ പ്രചാരത്തിലായ അരമായ വിവര്‍ത്തനങ്ങളോ പരാവര്‍ത്തനങ്ങളോ ആണ് തര്‍ഗൂമുകള്‍ എന്ന് അറിയപ്പെടുന്നത്. പല തര്‍ഗൂമുകളും വിവര്‍ത്തനങ്ങള്‍ എന്നതിനേക്കാളുപരി വ്യാഖ്യാനങ്ങളാണ് എന്നാണ് എന്‍സൈക്ലോപീഡിയ ജൂദായിക്ക പറയുന്നത്. പഴയ നിയമത്തെക്കുറിച്ച് സൂക്ഷ്മമായി പഠിക്കുവാനും വിവിധ കാലങ്ങളില്‍ നിലനിന്നിരുന്ന വ്യാഖ്യാന രീതികള്‍ മനസ്സിലാക്കുവാനും തര്‍ഗൂമുകള്‍ പ്രയോജനപ്പെടുന്നു.

'എസ്‌തേറിന്റെ പുസ്തകത്തിനുള്ള രണ്ടാം തര്‍ഗൂം ആയ തര്‍ഗൂം ഷെനി' (Targum Sheni) യില്‍ സോളമനും ശേബാരാജ്ഞിയും തമ്മില്‍ നടന്ന കൂടിക്കാഴ്ചയെക്കുറിച്ച് വിവരിക്കുന്നുണ്ട്. ഈ വിവരണമാകട്ടെ, ഏകദേശം ഖുര്‍ആനിലേതിന് സമാനമാണ് താനും. ഇതില്‍നിന്ന് തര്‍ഗൂം ഷെനിയില്‍നിന്ന് മുഹമ്മദ് നബി (സ)പകര്‍ത്തിയെഴുതിയതാണ് സോളമന്‍-ബില്‍ഖീസ് കഥയെന്ന് പറയാനാകുമോ?

യഹൂദ പുരോഹിതന്മാര്‍ക്ക് മാത്രം പ്രാപ്യമായ താര്‍ഗൂമുകള്‍ പോലും സൂക്ഷ്മമായി പരിശോധിച്ച് അവയില്‍നിന്ന് പകര്‍ത്തിയെഴുതുവാന്‍ നിരക്ഷരനായ മുഹമ്മദ് നബി (സ) ക്ക് കഴിഞ്ഞുവെന്ന വാദംതന്നെ ശുദ്ധ അസംബന്ധമാണ്. അങ്ങനെ ചെയ്തിരുന്നുവെങ്കില്‍ അക്കാലത്തെ യഹൂദ പുരോഹിതന്മാര്‍ ആരെങ്കിലും പ്രസ്തുത ആരോപണം ഉന്നയിക്കേണ്ടതായിരുന്നു. എന്നാല്‍ ഈ ആരോപണ ത്തിന് ഒന്നര നൂറ്റാണ്ടിലേറെ പഴക്കമില്ലെന്നതാണ് വാസ്തവം.

ഉപലബ്ധമായ തര്‍ഗൂമുകളില്‍ ഏറ്റവും പഴക്കമുള്ളതിന്റെ കാലം ക്രിസ്താബ്ദം 700 നടുത്തായിരിക്കുമെന്നാണ് എന്‍സൈക്ലോപീഡിയ ജൂദായിക്ക പറയുന്നത് ("Targums" CD. Rom Edition) തര്‍ഗൂം ഷെനിയാകട്ടെ എട്ടാം നൂറ്റാണ്ടില്‍ രചിക്കപ്പെട്ടതാണെന്നാണ് പണ്ഡിതാഭിപ്രായം (Ibid)

മുഹമ്മദ് നബിക്ക് ശേഷം പതിറ്റാണ്ടുകള്‍ കഴിഞ്ഞാണ് തര്‍ഗൂം ഷെനി രചിക്കപ്പെട്ടതെന്നര്‍ത്ഥം.

മുഹമ്മദ് നബി (സ) ക്ക് ശേഷം രചിക്കപ്പെട്ട തര്‍ഗൂമില്‍നിന്ന് കോപ്പിയടിച്ചാണ് അദ്ദേഹം ഖുര്‍ആന്‍ രചിച്ചതെന്ന വാദം എന്തുമാത്രം ബാലിശമാണ്!

യഥാര്‍ത്ഥത്തില്‍, തര്‍ഗൂം രചയിതാക്കളാണ് ഖുര്‍ആനിലെ സോളമന്‍-ശേബാരാജ്ഞി കഥയില്‍നിന്ന് കടമെടുത്തത് എന്നാണ് മനസ്സിലാക്കാന്‍ കഴിയുന്നത്. ഖുര്‍ആനിനുശേഷം എഴുതപ്പെട്ട ഒരു കൃതിയില്‍ ഇത്തരമൊരു കടമെടുക്കല്‍ നടന്നിരിക്കാനുള്ള സാധ്യത ഒട്ടും നിഷേധിക്കാനാവില്ല. ഇക്കാര്യം എന്‍സൈക്ലോപീഡിയ ജൂതായിക്കതന്നെ സമ്മതിക്കുന്നുമുണ്ട്. ''ഇതിലെ ചില പ്രസക്ത ഭാഗങ്ങള്‍ ഖുര്‍ആനിലും കാണപ്പെടുന്നുണ്ട്. (27:20-40). ഇതില്‍നിന്ന് ഈ തര്‍ഗൂമിന്റെ രചയിതാവ് അറബി സ്രോതസ്സുകളെയും തന്റെ രചനക്ക് ഉപയോഗിച്ചിട്ടുണ്ടെന്നാണ് മനസിലാവുന്നത്'' (``Targum Sheni'', Encyclopaedia Judaica CD-Rom Edition)  യഹൂദ പുരാണങ്ങളെ അവലംബിച്ചുകൊണ്ട് മുഹമ്മദ് നബി (സ) മെനഞ്ഞുണ്ടാക്കിയ കഥയാണ് സുലൈമാന്‍-ബില്‍ഖീസ് കഥയെന്ന വിമര്‍ശകരുടെ വാദം തകരുക മാത്രമല്ല; പ്രത്യുത ഖുര്‍ആന്‍ ദൈവികമാണെന്ന വസ്തുത ഒരിക്കല്‍കൂടി ബോധ്യപ്പെടുക കൂടിയാണ് ഇവിടെ ചെയ്യുന്നത്. യഹൂദ തര്‍ഗൂമുകള്‍ രചിക്കപ്പെട്ടത് ഖുര്‍ആനിന് ശേഷമാണെന്ന വസ്തുത നാം മനസ്സിലാക്കി. ഖുര്‍ആനിലും തര്‍ഗൂമുകളിലുമൊഴിച്ച് മറ്റെവിടെയും ഈ കഥ വിശദാംശങ്ങളോടെ കാണുന്നുമില്ല. മുഹമ്മദ് നബിയാണ് ഖുര്‍ആന്‍ രചിച്ചതെങ്കില്‍ അദ്ദേഹത്തിന് ഈ കഥയെവിടെനിന്നുകിട്ടി? പൂര്‍വകാല ചരിത്രത്തെക്കുറിച്ച് ശരിക്കറിയാവുന്ന സര്‍വ്വശക്തന്റെ വചനങ്ങളാണ് ഖുര്‍ആനെന്ന വസ്തുതയാണ് ഇവിടെയും വ്യക്തമായി വെളിപ്പെടുന്നത്.

 

മൂസാ (അ) യും ഖിള്‌റും (അ)  തമ്മില്‍ നടന്ന സംഭാഷണങ്ങളും പിന്നീടുണ്ടായ സംഭവങ്ങളുമെല്ലാം സാമാന്യം വിശദമായിത്തന്നെ ഖുര്‍ആനിലെ സൂറത്തുല്‍ കഹ്ഫില്‍ (18:65-82) വിവരിക്കുന്നുണ്ട്. ഏലിജായും യോശുവ ബെന്‍ ലെവിയെന്ന റബ്ബിയും കൂടി നടത്തിയതായി യഹൂദ ഐതിഹ്യത്തില്‍ പറയുന്ന യാത്രയ്ക്കും സംഭവങ്ങ ള്‍ക്കും മൂസാ-ഖിള്ര്‍ സംഭവത്തെക്കുറിച്ച ഖുര്‍ആനിക വിശദീകരണങ്ങളുമായി സാമ്യമുണ്ടെന്നും അതുകൊണ്ട് യഹൂദ ഐതിഹ്യ ത്തില്‍നിന്ന് കടമെടുത്തുകൊണ്ട് മുഹമ്മദ് നബി (സ) കെട്ടിച്ചമച്ചുണ്ടാക്കിയ ഒരു കഥയാണിതെന്നുമാണ് വാദം. Jellinek, Betha-Midrasch, V, 1335ല്‍ ഈ ഐതിഹ്യം വിശദീകരിച്ചിട്ടുണ്ടെന്നും, Zunz, Gesmmelt Vortrage, X, 130ലാണ് ഇതും ഖുര്‍ആനിക കഥയും തമ്മില്‍ സാമ്യമുണ്ടെന്ന് ആദ്യമായി സൂചിപ്പിച്ചിട്ടുള്ളതെന്നും ഓറിയന്റലിസ്റ്റുകളുടെ രചനയായ Encyclopedia of Islam (Page 903 Under the title ``Al-Khidr'')ല്‍ പറയുന്നുണ്ട്. ചരിത്ര വസ്തുതകളും പ്രമാണങ്ങളും സൂക്ഷ്മ പരിശോധനയ്ക്ക് വിധേയമാക്കിയാല്‍ ഈ വാദം അടിസ്ഥാനരഹിതമാണെന്ന് ബോധ്യമാകും:

(1) യഹൂദന്മാര്‍ക്കിടയില്‍ മുഹമ്മദ് നബി (സ) യുടെ കാലത്ത് ഇത്തരം ഒരു ഐതിഹ്യം നിലനില്‍ക്കുകയും അതില്‍ ചില മാറ്റങ്ങള്‍ വരുത്തി മുഹമ്മദ് നബി (സ) ഖുര്‍ആനിലൂടെ അവതരിപ്പിക്കുകയും ചെയ്തിരുന്നുവെങ്കില്‍ അന്നുണ്ടായിരുന്ന യഹൂദന്മാര്‍ ഇക്കാര്യം എടുത്തുപറയുകയും നബി (സ) യെ വിമര്‍ശിക്കുകയും ചെയ്യുമായിരുന്നു. അങ്ങനെ യാതൊന്നും സംഭവിച്ചതായി രേഖപ്പെടുത്തപ്പെട്ടിട്ടില്ല.

(2) യഹൂദ മതത്തില്‍നിന്ന് ഇസ്‌ലാം സ്വീകരിച്ച ഒട്ടനവധി പ്രവാചക ശിഷ്യന്മാരുണ്ടായിരുന്നു. തങ്ങള്‍ കേട്ടുവളര്‍ന്ന ഒരു ഐതിഹ്യം ഏതാനും മാറ്റങ്ങളോടെ അവതരിപ്പിച്ച രീതിയാണ് മൂസാ-ഖിള്ര്‍ (അ)  സംഭവ വിവരണത്തിന്റെ കാര്യത്തില്‍ ഖുര്‍ആനിലുള്ളതെ ങ്കില്‍ അവര്‍ ഇക്കാര്യം സൂചിപ്പിക്കുകയും പ്രവാചകനുമായും മറ്റു ഹാബിമാരുമായും ഇത് ചര്‍ച്ച നടത്തുകയും ചെയ്യുമായിരുന്നു. അത്തരം യാതൊരു സംഭവവും രേഖപ്പെടുത്തപ്പെട്ടിട്ടില്ലാത്തതിനാല്‍തന്നെ അക്കാലത്തെ യഹൂദര്‍ക്കിടയില്‍ ഇത്തരമൊരു ഐതിഹ്യം പ്രചാരത്തിലില്ലായിരുന്നുവെന്ന് വ്യക്തമാകുന്നുണ്ട്.

(3) പതിനൊന്നാം നൂറ്റാണ്ടിന് മുമ്പ് നിലനിന്ന യാതൊരു യഹൂദ രേഖയിലും ഏലിജാ-യോശുവാ ഐതിഹ്യം പ്രതിപാദിക്കുന്നില്ല. (Brannon-M. Wheeler: ``The Jewish Origins of Quran IB: 65-82? Re examining Arent Jan Wensinck's Theory'': Journel of the American Oriental Society Vol 118, Page 115). മുഹമ്മദ് നബി (സ) യുടെ കാലത്ത് ഇത് പ്രചാരത്തിലിരുന്നെങ്കില്‍ ഇതിനേക്കാള്‍ പ്രാചീനമായ യഹൂദ രേഖകളില്‍ ഈ കഥ കാണേണ്ടതായിരുന്നു.

(4) ഖുര്‍ആനില്‍ പറഞ്ഞ മൂസാ-ഖിള്ര്‍ സംഭവവും യഹൂദ ഐ തിഹ്യങ്ങളും സൂക്ഷ്മ പഠനത്തിന് വിധേയമാക്കിയശേഷം ഓറിയന്റലിസ്റ്റായ ബ്രന്നോന്‍ എം. വീലര്‍ എത്തിച്ചേരുന്ന നിഗമനമിങ്ങനെയാണ്: ''ഈ പണ്ഡിതരോ (യൂദ ഐതിഹ്യത്തില്‍നിന്ന് കോപ്പിയടിച്ചതാണ് മൂസാ-ഖിള്ര്‍ കഥയെന്ന് വാദിക്കുന്നവര്‍) വെന്‍സില്‍ക്കോ Hibbur Yafeh me-ka-yeshuaയുടെ തലക്കെട്ടിന് കീഴില്‍ നല്‍കിയിട്ടുള്ള ഈ കഥ പതിനൊന്നാം നൂറ്റാണ്ടിലെ ഖൈറവാന്‍കാരനായ നി സ്സിം ബിന്‍ ഷഹിനിന്റെ പേരിലുള്ള ഒരു അറബി രചനയുടെ ഹിബ്രു പരാവര്‍ത്തനം മാത്രമാണെന്ന വസ്തുത ശ്രദ്ധിച്ചിട്ടേയില്ല....... ഇതിന്റെ അറബി ഒറിജിനല്‍ കണ്ടെത്തിയശേഷവും, പതിനൊന്നാം നൂറ്റാണ്ടിന് മുമ്പുള്ള ഒരു ഹിബ്രു സ്രോതസ്സും ഈ കഥയുള്‍ക്കൊള്ളുന്നില്ലെന്ന വസ്തുത പരിഗണിക്കാതെ ഇതിനെയാണ് ഖുര്‍ആന്‍ ആശ്രയിച്ചതെന്ന് പണ്ഡിതന്മാര്‍ ആവര്‍ത്തിച്ച് പറഞ്ഞുകൊണ്ടിരിക്കുകയാണ്....... ഏലിജയുടെയും യോശുവ ബിന്‍ലെവിയുടെയും കഥയെ ആശ്രയിച്ച് എഴുതപ്പെട്ടതല്ല ഖുര്‍ആന്‍ 18:65-82 എന്ന് ലഭ്യമായ തെളിവുകളെല്ലാം വ്യക്തമാക്കുന്നുണ്ട്. ഖുര്‍ആന്‍ 18:65-82ന്റെ വ്യാഖ്യാനങ്ങളെ, വിശേഷിച്ചും ഉബയ്യുബ്‌നു കഅ്ബിന്റെ കഥയെയും അതിന്റെ പില്‍ക്കാല വിശദീകരണങ്ങളെയും ആശ്രയിച്ചുകൊണ്ടാണോ ഇബ്‌നു ഷാഹിനിന്റെ കഥ രചിക്കപ്പെട്ടിട്ടുള്ളതെന്ന കാര്യം ഇപ്പോഴും ഒരു പ്രശ്‌നമായിത്തന്നെ അവശേഷിക്കുന്നു. ഖുര്‍ആനും ഉബയ്യുബ്‌നു കഅ്ബിന്റെ കഥയുടെ മൂലരൂപവുമായി താരതമ്യം ചെയ്യുമ്പോള്‍, ഇബ്‌നു ഷാഹിന്റെ രചന പുതിയതും ആദ്യകാല ഇസ്‌ലാമിക സ്രോതസ്സുകളുമായി യോജിക്കുന്ന നിരവധി കാര്യങ്ങള്‍ ഉള്‍ക്കൊള്ളുന്നതുമാണ്. ഏലീജയുടെയും യോ ശുവ-ബിന്‍ലെവിയുടെയും കഥയില്‍ ഖുര്‍ആന്‍ 18:65-82ലില്ലാത്തതും ഈ വചനങ്ങളുടെ വ്യാഖ്യാന ഗ്രന്ഥങ്ങളിലുള്ളതുമായ പല കാര്യങ്ങളുടെയും പ്രതിഫലനങ്ങളുണ്ട്. ഖിള്‌റിനുപകരം ഏലീജായെ ഉപയോഗിക്കുവാന്‍ ഇബ്‌നുഷാഹിനെ പ്രേരിപ്പിച്ചത് ഇസ്‌ലാമിക സ്രോതസ്സുകളില്‍ ഈ രണ്ട് വ്യക്തിത്വങ്ങളും തമ്മിലുള്ള അടുത്ത ബന്ധമായിരിക്കാമെന്ന് വ്യാഖ്യാനിക്കാവുന്നതാണ്''. (Ibid Page 155-171) ഖുര്‍ആനിക കഥയുടെ അടിസ്ഥാനത്തില്‍ പതിനൊന്നാം നൂറ്റാണ്ടിലെ ഏതോ യഹൂദന്റെ മനസ്സില്‍ രൂപംകൊണ്ട ഐതിഹ്യമെടുത്ത് പൊക്കി ഈ ഐതിഹ്യത്തിന്റെ അടിസ്ഥാനത്തില്‍ മുഹമ്മദ് നബി (സ) പടച്ചുണ്ടാക്കിയതാണ് മൂസാ-ഖിള്‌റ് സംഭവമെന്ന് വാദിക്കുന്നവര്‍ സ്വന്തം കണ്ണുപൊട്ടിച്ച് അന്ധനാകാന്‍ ശ്രമിക്കുന്നവനെപ്പോലെയാണെന്ന് പറയാതിരിക്കാന്‍ നിര്‍വ്വാഹമില്ല.

യേശുവിനെക്കുറിച്ച് സമകാലികരായിരുന്ന ചരിത്രകാരന്മാരൊന്നും രേഖപ്പെടുത്താത്തതിനാല്‍ അങ്ങനെയൊരു വ്യക്തി ജീവി ച്ചിരുന്നിട്ടേയില്ലയെന്ന് വാദിക്കുന്ന യുക്തിവാദികളുണ്ട്. ജോസിഫസ് എന്ന യൂദ ചരിത്രകാരന്‍ ക്രിസ്തുവിനെക്കുറിച്ച് പറഞ്ഞതായി ക്രിസ്തുമത പ്രചാരകന്മാര്‍ ഉദ്ധരിക്കാറുണ്ടെങ്കിലും പ്രസ്തുത പ്രസ്താവനകള്‍ ജോസിഫസിന്റെ ഗ്രന്ഥത്തില്‍ കൂട്ടിച്ചേര്ത്തസതാണെന്നാണ് വിമര്ശികര്‍ വാദിക്കുന്നത്. അത് എന്തായിരുന്നാലും, ക്രൈസ്തവരെ സംബന്ധിച്ചിടത്തോളം യേശുവിന്റെ ജനനം, പ്രബോധനങ്ങള്‍, മരണം, എന്നിവയെക്കുറിച്ച് അറിവു നല്കുനന്ന ഗ്രന്ഥങ്ങള്‍ നാലു സുവിശേഷങ്ങളാണ്. ഖുര്ആിനില്‍ യേശുവിനെക്കുറിച്ച് ഒരുപാട് കാര്യങ്ങള്‍ പറയുന്നുണ്ടെങ്കിലും ക്രൈസ്തവര്ക്ക്് അത് അസ്വീകാര്യമാണല്ലോ. അപ്പോള്‍ യേശുവിനെക്കുറിച്ച് ക്രൈസ്തവരുടെ അറിവ് സുവിശേഷങ്ങളെ ആശ്രയിച്ചാണിരിക്കുന്നത്. സുവിശേഷങ്ങള്‍ ദൈവനിവേശിതങ്ങളായതിനാല്‍ അവ നല്കുുന്ന വിവരങ്ങള്‍ നൂറു ശതമാനം സത്യസന്ധവും സ്വീകരിക്കാന്‍ കൊള്ളുന്നതുമാ ണെന്നായിരുന്നു പൊതുവായ ക്രൈസ്തവ കാഴ്ചപ്പാട്. ബൈബിള്‍ നല്കുാന്ന യേശുചിത്രം ചരിത്രപരവും വസ്തുനിഷ്ഠവുമാണെന്നായിരുന്നു വിശ്വാസം.

പുതിയ നിയമത്തെക്കുറിച്ച ഗവേഷണങ്ങള്‍ ഈ പരമ്പരാഗത ധാരണയ്ക്ക് ഇളക്കം തട്ടിച്ചിട്ടുണ്ട്. യേശുവിന്റെ ചരിത്രവുമായി ബന്ധപ്പെട്ട് സുവിശേഷകര്ത്തായക്കള്‍ ഉദ്ധരിക്കുന്ന സംഭവങ്ങളില്‍ പലതും ഭാവനാസൃഷ്ടിയാണെന്നാണ് ഗവേഷകര്‍ അഭിപ്രായപ്പെടുന്നത്. ക്രിസ്തുവിന്റെ ശിശുകഥയുമായി ബന്ധപ്പെട്ട് സുവിശേഷങ്ങള്‍ വിവരിക്കുന്ന കാര്യങ്ങള്‍ എത്രത്തോളം ചരിത്രപരമാണെന്ന് ലഭ്യമായ തെളിവുകളുടെ അടിസ്ഥാനത്തില്‍ പ്രഗത്ഭ ബൈബിള്‍ പണ്ഡിതനായ റെയ്മണ്ട്-ഇ-ബ്രൗണ്‍ തന്റെ 'മിശിഹായുടെ ജനനം' (The birth of Messiah) എന്ന ഗ്രന്ഥത്തില്‍ വിശദമായി പരിശോധിക്കുന്നുണ്ട്. ഫാദര്‍.ഫ്രാന്സി( കൊടിയന്‍, ഫാ.ജോസ് മാണപ്പറമ്പില്‍, ഫാ:വര്ഗീെസ് പെരേപ്പാടന്‍ തുടങ്ങിയവര്‍ ചേര്ന്ന് വിവര്ത്ത നം ചെയ്തിരിക്കുന്ന 401 പേജുകളുള്ള ഈ പുസ്തകത്തിന്റെ മലയാള പരിഭാഷ അലൂരിലെ ബിബ്‌ളിയ പബ്ലിക്കേഷന്സ്ന ആണ് പ്രസിദ്ധീകരിച്ചിരിക്കുന്നത്. ഇരുപതിലേറെ ഓണററി ഡോക്റ്ററേറ്റുകളുള്ള അമേരിക്കന്‍ ബൈബിള്‍ പണ്ഡിതനായ റെയ്മണ്ട്- ഇ-ബ്രൗണ്‍ വിശദമായ പഠനങ്ങള്ക്ക് ശേഷം എത്തിച്ചേരുന്ന നിഗമ നങ്ങള്‍ ഇങ്ങനെ സംക്ഷേപിക്കാം.

(1) മത്തായിയും ലൂക്കോസും നല്കുിന്ന യേശുവിന്റെ വംശാവലി കൃത്യമല്ല. വരാനിരിക്കുന്ന മിശിഹാ ദാവീദു വംശജനായിരിക്കുമെന്നായിരുന്നു യഹൂദവിശ്വാസം. യേശു മിശിഹയാണെങ്കില്‍ ദാവീദ് വംശജനാകണമല്ലോ. അതിനു വേണ്ടി ദാവീദിന്റെ വംശാവലിയില്‍ യേശുവിനെ തിരുകിക്കയറ്റി അവതരിപ്പിക്കുകയാണ് സുവിശേഷകന്മാര്‍ ചെയ്തിരിക്കുന്നത്. 'മത്തായിയും ലൂക്കായും തരുന്ന വംശാവലി കുടുംബത്തിന്‍േറതാണ് എന്ന നിഗമനത്തെ മുകളില്‍ കണ്ട പ്രശ്നങ്ങളുടെ വെളിച്ചത്തില്‍ മിക്ക ആധുനിക ചിന്തകരും തള്ളിക്കളയുന്നുണ്ട്. ജനകീയമായിരുന്ന ദാവീദ് രാജപരമ്പരയുടെ അന്ത്യത്തില്‍, യൗസേപ്പിന്റെയും യേശുവിന്റെയും പേരുകള്‍ കൂട്ടിച്ചേര്ത്തുതാണ് മത്തായി നല്കുിന്ന വംശാവലി; ലൂക്കോയുടെതാകട്ടെ യൗസേപ്പിന്റെ പൂര്വ്വി കരുടെ കുടുംബപട്ടികയും. ഇതാണ് ആധുനിക നിഗമനം'1

(2) ഇസ്രായീലിനെ ഫറോവയില്‍ നിന്നു രക്ഷിച്ച മോശയുടെ ചരിത്രത്തോട് സാമ്യപ്പെടുത്തിക്കൊണ്ടാണ് മത്തായി തന്റെ യേശുകഥ നിര്മിപക്കുന്നത്. അതിനാല്‍ യേശുവിന്റെ ജീവിതത്തില്‍ സംഭവിക്കാത്ത പലതും മത്തായിയുടെ യേശുകഥയിലുണ്ട്. 2

(3) പഴയനിയമത്തില്‍ പ്രവചിക്കപ്പെട്ട പ്രവാചകനാണ് യേശുവെന്ന് വരുത്തിത്തീര്ക്കു ന്നതിനു വേണ്ടിയാണ് മത്തായി ശ്രമിച്ചിരിക്കുന്നത്. അതിനാല്‍ പഴയനിയമ പ്രവചനങ്ങള്ക്ക്നുസൃതമായി യേശു കഥ മെനഞ്ഞെടുക്കുകയാണ് അദ്ദേഹം ചെയ്തിരിക്കുന്നത്.3

(4) യേശു ജനിച്ച കാലഘട്ടത്തിലെ മുഴുവന്‍ ജ്യേതിശാസ്ത്ര രേഖകളും പരിശോധിച്ചാലും മത്തായി 2:1-12ല്‍ പറയുന്ന രീതിയില്‍ യേശു ജനനത്തോടനുബന്ധിച്ച് ബെത്‌ലഹേമിന് മുകളില്‍ ഒരു നക്ഷത്രം പ്രത്യക്ഷപ്പെട്ടുവെന്ന വിവരണത്തിന് ഉപോല്ബകലകമായ ഒരു തെളിവുപോലും ലഭിക്കുകയില്ല. (ധൂമകേതുക്കള്‍, ഗ്രഹങ്ങളുടെ സംഗമം, നവജാതനക്ഷത്രങ്ങള്‍ തുടങ്ങിയവയെക്കുറിച്ച ജ്യോതിശാ സ്ത്ര രേഖകളെല്ലാം റെയ്മണ്ട് ബ്രൗണ്‍ പരിശോധിക്കുന്നുണ്ട്). ബെത്‌ലഹേമിനു മുകളില്‍ യേശുവിന്റെ ജനനത്തോടനുബന്ധിച്ച് ഒരു നക്ഷത്രം പ്രത്യക്ഷപ്പെടുകയും അതുകണ്ട് പൗരസ്ത്യ ദേശ ത്തു നിന്ന് ജ്ഞാനികള്‍ ഹെറോദോസ് രാജാവിനോട് യഹൂദ രാജാവിനെക്കുറിച്ച് അന്വേഷിക്കുകയും അങ്ങനെ യേശു ജനിച്ച സ്ഥലത്തെത്തിച്ചേരുകയും ചെയ്തുവെന്ന കഥ മത്തായി മെനഞ്ഞെടുത്ത താണ്. അത് ചരിത്രപരമല്ല.

ഈ കഥ ചരിത്രപരമല്ലെന്നതിന് മറ്റു പുതിയനിയമഭാഗങ്ങളും തെളിവു നല്കുുന്നുവെന്നാണ് റെയ്മണ്ട് ബ്രൗണ്‍ പറയുന്നത്. അദ്ദേഹം എഴുതുന്നു: 'മത്തായി രണ്ടാമധ്യായമനുസരിച്ച് ജ്ഞാനികള്‍ ഹേറോദോസിനെ സന്ദര്ശിാച്ചപ്പോഴാണ് അദ്ദേഹവും പ്രധാനപുരോഹിതന്മാരും പശീശന്മാരുമെല്ലാം യൂദരുടെ രാജാവിന്റെ ജനനത്തെപ്പറ്റി അറിഞ്ഞത്. ജറൂസലം മുഴുവന്‍ ആസംഭവത്തെ സംബന്ധിച്ച് അസ്വസ്ഥമാവുകയും ചെയ്തു. എന്നിട്ടുപോലും യേശു തന്റെ പരസ്യശുശ്രൂഷ ആരംഭിച്ചപ്പോള്‍ അവിടുത്തെപ്പറ്റി ആര്ക്കെലങ്കിലും കാര്യമായ അറിവോ പറയത്തക്ക പ്രതീക്ഷകളോ ഉണ്ടായിരുന്നതായി കാണുന്നില്ല. (മത്തായി 13:54-56) ഹെറോദോസിന്റെ മകന്‍ അന്തിപ്പാസിനാകട്ടെ യേശുവിനെപ്പറ്റി യാതൊരു വിവരവുണ്ടായിരുന്നില്ല. (ലൂക്കോസ് 8:7-9). ലൂക്കോസിന്റെ സുവിശേഷത്തില്‍ സ്‌നാപക യോഹന്നാന്റെ അമ്മയായ ഏലിശാ യേശുവിന്റെ അമ്മയായ മറിയത്തിന്റെ ബന്ധുവാണ്. തന്മൂലം അവരുടെ കുട്ടികള്‍ തമ്മിലും ബന്ധമുണ്ടല്ലോ. ഇതെല്ലാമായിട്ടും പരസ്യശുശ്രൂഷാകാലത്ത് യേശുവും സ്‌നാപകയോഹന്നാനും ബന്ധക്കാരാണെന്നതിനെപ്പറ്റി ഒരു സൂചനയും സുവിശേഷത്തിലില്ല. പോരെങ്കില്‍ യോഹന്നാന്‍1:33 ല്‍ സ്‌നാപകന്‍ ''ഞാന്‍ അവനെ അറിഞ്ഞിരുന്നില്ല'' എന്നു പറയുന്നുണ്ട്. ശൈശവ വിവരണങ്ങളുടെ ചരിത്രപരതയെപ്പറ്റി സംശയമുണര്ത്തുലന്ന സുവിശേഷ ഭാഗങ്ങളില്‍ ചിലതുമാത്രമാണിവ'.

(5) യേശു ജനിച്ചത് യഥാര്ത്ഥ ത്തില്‍ ബെത്‌ലഹേമിലല്ല. ദാവീദി ന്റെ പുത്രനായ മിശിഹ ബെത്‌ലഹേമിലാണ് ജനിക്കുകയെന്ന യഹൂദ പാരമ്പര്യത്തിനനുസൃതമായി യേശുകഥ മെനഞ്ഞെടുത്ത മത്തായിയും ലൂക്കോസും ബെത്‌ലഹേമിലാണ് യേശു ജനിച്ചതെന്ന് വരുത്തി ത്തീര്ക്കു്കയാണ് ചെയ്തിരിക്കുന്നത്.

'യഹൂദ ചിന്തയനുസരിച്ച് ദാവീദാത്മജനായ മിശിഹാ ദാവീദി ന്റെ പട്ടണമായ ബെത്‌ലഹേമില്‍ ജനിക്കണം. അതിനാല്‍ ഈശോ ജനിച്ചത് ബെത്‌ലഹേമിലാണ് എന്ന കഥ മെനഞ്ഞെടുത്തു'

'രണ്ടാമതായി, മറ്റു പുതിയ നിയമഭാഗങ്ങള്‍ ഈശോയുടെ ജനന സ്ഥലം ബേത്‌ലഹേമാണെന്നതില്‍ നിശ്ശബ്ദമാണെന്നു മാത്രമല്ല നസ്രത്തും ഗലീലിയും ആണ് ഇശോയുടെ സ്വന്തം പട്ടണമെന്നതിന് തെളിവുകളുമുണ്ട്. അവന്റെ ''സ്വന്തം പട്ടണം'' (his patris) ഈ പദം മത്തായിയും ലൂക്കായും ഉപയോഗിക്കുന്നത് ഈശോ ജനിച്ചത് ബത്‌ലഹേമിലാണെന്ന് അവര്‍ ഉറപ്പിച്ചച്ചു പറഞ്ഞിരിക്കുന്നതിനാല്‍, ഈശോ വളര്ന്നന സ്ഥലത്തെ ഉദ്ദേശിച്ചാണ് (മത്തായി 22-34, ലൂക്കോ 2:51). മാര്ക്കോ സില്‍ നിന്നാണ് (6:1,4) ഇവര്ക്ക് ഈ വിവരം ലഭിച്ചത് എന്നുള്ളതിനാലും മര്ക്കോതസിന് ബേത്‌ലഹേമിനെക്കുറിച്ച് ഒന്നും പറയാനുമില്ലാത്തതിനാലും അവന്റെ സ്വന്തം പട്ടണം നസ്‌റത്ത് അയിരിക്കണം. ഈശോ ജനിച്ചത് ഗലീലിയയിലായിരിക്കും.... നാലാ മത്തെ സുവിശേഷത്തില്‍ (1:46,7:41-42;52) ഈശൊയുടെ ഗാലിലേ യന്‍ ജനനത്തെക്കുറിച്ച് സൂചനകളുമുണ്ട്'.

(6) ലോകമാസകലമുള്ള ജനങ്ങളുടെ പേര് എഴുതിച്ചേര്ക്കനണം എന്ന് അഗസ്റ്റസ് സീസറിന്റെ കല്പഒനയനുസരിച്ച് യോസേഫും മറിയയും ബെത്‌ലഹേമിലേക്ക് പോയപ്പോഴാണ് യേശു ജനിച്ചതെ ന്നാണ് ലൂക്കോസിന്റെ സുവിശേഷം രണ്ടാം അധ്യായത്തില്‍ പറയുന്നത്. ഇത് ചരിത്രപരമായി ശരിയല്ല. അഗസ്റ്റസ് സീസറുടെ കാലത്ത് ലോകവ്യാപകമായ കണക്കെടുപ്പൊന്നും നടന്നിട്ടില്ല. ക്വരീ നിയസിന്റെ കാലത്ത് യൂദായിലുണ്ടായ കണക്കെടുപ്പ് (നസ്‌റോത്ത് അതില്പ്പെ ടുന്നില്ല) ഹെറോദോസ് മരിച്ച് പത്തുകൊല്ലങ്ങള്ക്കു് ശേഷമാണ് നടന്നത്. അപ്പോഴേക്കും യേശു ജനിച്ചിട്ടുണ്ടായിരിക്കണം. യേശുവിന്റെ ജനനത്തെ അതിന് കൊല്ലങ്ങള്ക്കുട മുമ്പു നടന്ന കണക്കെടുപ്പുമായി യോജിപ്പിക്കുകയാണ് ലൂക്കോസ് ചെയ്തിരിക്കുന്നത്. റെയ്മണ്ട് ബ്രൗണ്‍ എഴുതുന്നു: 'മഹാനായ ഹോരോദോസിന്റെ കാലത്ത് പാലസ്തീനായില്‍ ക്വിരീനിയൂസിന്റെ കീഴില്‍ റോമന്‍ സെന്സെസ് നടന്നു എന്നു വിശ്വസിക്കാന്‍ ഗൗരവതരമായ കാര ണങ്ങളില്ല. ലൂക്കോ 1ലെ വിവരം ശരിയായിരിക്കാം. മഹാനായ ഹെറോദോസിന്റെ കാലയളവില്‍ ഈശോ ജനിച്ചു. എന്നാല്‍ A.D 67 ല്‍ ക്വരീനിയൂസിന്റെ കീഴില്‍ യൂദായില്‍ (ഗലീലിയിലല്ല) നടന്ന സെന്സെസുമായി പൊരുത്തപ്പെടുത്തിയ ലൂക്കോയുടെ വിവരം അവിടെ A.D.30കളില്‍ (ഈശോയുടെ മരണത്തിന് ശേഷം) ഗമാലിയേല്‍ നടത്തുന്ന പ്രസംഗത്തില്‍ തെവുദാസിന്റെ (Thevudas) ഉയിര്ത്തെ ഴുന്നേല്പ്പി നെപ്പറ്റി പരാമര്ശിദക്കുന്നു. എന്നാലത് ഗമാലിയലിന്റെ പ്രസംഗത്തിനു 10 വര്ഷ‍ങ്ങള്ക്കുസ ശേഷവും നടന്നില്ല. തെവുദാസിനു ശേഷം ഗാലിലേയനായ യൂദാസിന്റെ (A.D 6-7) സെന്സംസും ഉയിര്ത്തെ ഴുന്നേല്പു്മായി ലൂക്കാ തെറ്റിദ്ധരിച്ചതാവാം.'

യേശുവിന്റെ ജനനവും ശൈശവകാലവുമായി ബന്ധപ്പെട്ടു കൊണ്ട് മത്തായിയും ലൂക്കോസും പറയുന്ന കാര്യങ്ങളൊന്നും ചരിത്രപരമല്ലെന്നും സുവിശേഷകന്മാരുടെ ഭാവനാസൃഷ്ടികളാണെന്നുമാണ് ആധുനിക ബൈബിള്‍ പണ്ഡിതന്മാരില്‍ പ്രമുഖനായ റെയ്മണ്ട്-ഇ-ബ്രൗണ്‍ തെളിവുകളുടെ അടിസ്ഥാനത്തില്‍ സ്ഥാപിക്കുന്നത്. സുവിശേഷകര്‍ തങ്ങളുടെ ഭാവനയില്‍ തോന്നിയ കാര്യങ്ങള്‍ യേശു കഥയോടു ബന്ധപ്പെടുത്തിക്കൊണ്ട് ചരിത്രമെന്ന നിലക്ക് അവതരിപ്പിച്ചെന്നും ജനങ്ങളെ തെറ്റിദ്ധരിപ്പിച്ചെന്നുമാണല്ലോ ഇതില്‍ നിന്നും മനസ്സിലാക്കേണ്ടത്. ശൈശവകാല വിവരണങ്ങളുടെ കാര്യത്തില്‍ മാത്രമാണോ ഇത് സംഭവിച്ചിരിക്കുന്നത്? അങ്ങനെ കരുതുകവയ്യ. യഹൂദന്മാര്‍ പ്രതീക്ഷിച്ചിരിക്കുന്ന മിശിഹാ യേശുവാണെന്ന് വരുത്തിത്തീര്ക്കു ന്നതിന് വേണ്ടി ശൈശവകാല വിവരണങ്ങളില്‍ കൃത്രിമത്വം കാണിച്ച സുവിശേഷകന്മാര്‍, യേശുവിന്റെ ജീവിതത്തേയും പ്രബോധനങ്ങളെയും മരണത്തെയുമെല്ലാം കുറിച്ച വിവരണങ്ങളില്‍ തങ്ങളുടെ ഇച്ഛക്കനുസരിച്ച് മിനുക്കു പണികള്‍ നടത്തിയിട്ടില്ല എന്നതിന് എന്താണുറപ്പ്? ദിവ്യാത്ഭുതങ്ങളായി സുവിശേഷങ്ങള്‍ വിവരിച്ചവ പോലും നൂറുശതമാനം ചരിത്രപരമായിക്കൊള്ളണമെന്നില്ലായെന്നാണ് റെയ്മണ്ട് ബ്രൗണിന്റെ അഭിപ്രായം. 'യേശുവിന്റെ ദിവ്യാത്ഭുതങ്ങളെപ്പറ്റിയുള്ള പാരമ്പര്യം ആധികാരികമാണെ ന്നിരിക്കിലും സുവിശേഷ ഗ്രന്ഥങ്ങളിലുള്ള ഓരോ ദിവ്യാത്ഭുതവും അക്ഷരാര്ത്ഥാത്തില്‍ ചരിത്രപരമായിക്കൊള്ളണമെന്നില്ല'

ഇതില്‍ നിന്ന് ഒരു കാര്യം സുതരാം വ്യക്തമാണ്. യേശുവിനെക്കുറിച്ച് സത്യസന്ധമായ വിവരങ്ങള്‍ ലഭിക്കുവാന്‍ സുവിശേഷങ്ങളെ മാത്രം ആശ്രയിച്ചിട്ട് ഫലമില്ല. അവയിലെ വിവരങ്ങള്‍ സത്യവും അസത്യവും കൂടിക്കലര്ന്ന് നിലയിലാണുള്ളത്. തങ്ങളുടെ ഭാവനക്കനുസൃതമായി ഓരോ സുവിശേഷകനും യേശുകഥ മെനഞ്ഞെടുക്കുകയാണ് ചെയ്തിരിക്കുന്നത്. അവയില്‍ നിന്ന് സത്യം ചികഞ്ഞെടുക്കുക അതീവ ദുഷ്‌കരമാണ്.

യേശുവിനെക്കുറിച്ച കൃത്യവും കളങ്കരഹിതവുമായ അറിവു നല്കുതന്നത് ദൈവീക ഗ്രന്ഥമായ ഖുർആനിൽ മാത്രമാണെന്ന വസ്തുതയാണ് ഇവിടെ വ്യക്തമാവുന്നത്.

വിഷയവുമായി ബന്ധപ്പെട്ട വീഡിയോ

മുഹമ്മദ് നബി(സ)യുടെ പ്രവാചകത്വത്തിന്റെ ദൃഷ്ടാന്തമെന്ന നിലയില്‍ അല്ലാഹു ചന്ദ്രനെ പിളര്‍ത്തിയെന്നും മക്കക്കാര്‍ അതിന് സാക്ഷികളായെന്നും പറയുന്ന ഇസ്‌ലാമിക പാരമ്പര്യങ്ങള്‍ അടിസ്ഥാനരഹിതവും അസ്വീകാര്യവുമാണ്. കാരണം, ചാന്ദ്രപിളര്‍പ്പ് ചരിത്രപരമായി സ്ഥാപിക്കപ്പെട്ടതോ ശാസ്ത്രീയമായി സംഭവ്യമോ അല്ല. ഇനി ചാന്ദ്രപ്പിളര്‍പ്പും പുനസമാഗമവും അത്യത്ഭുതകരമായി സംഭവിച്ചാല്‍ തന്നെ അതിന്റെ ഭൗതികാഘാതങ്ങള്‍ ചന്ദ്രോപരിതലത്തില്‍ ദൃശ്യമാകേണ്ടതാണ്. അങ്ങനെ യാതൊന്നും ഇന്നുവരെ കണ്ടെത്താനായിട്ടില്ല. പ്രവാചകന്റെ മഹത്വം വര്‍ധിപ്പിച്ചു കാണിക്കുന്നതിനുവേണ്ടി നിര്‍മിക്കപ്പെട്ട ഒരു വ്യാജകഥയാണ് ചാന്ദ്രപ്പിളര്‍പ്പിന്റേത് എന്നാണ് ഇതെല്ലാം വ്യക്തമാക്കുന്നത്. മിഷനറി വെബ്‌സൈറ്റുകളില്‍ സര്‍വസാധാരണമായ ഈ വാദങ്ങളില്‍ കഴമ്പില്ലേ?

 ഇല്ല. പ്രവാചകത്വത്തിന് തെളിവുകള്‍ ആവശ്യപ്പെട്ടുകൊണ്ടുള്ള മക്കന്‍ ബഹുദൈവാരാധകരുടെ അന്വേഷണങ്ങള്‍ക്കുള്ള മറുപടിയായും സത്യവിശ്വാസികള്‍ക്കുള്ള അനുഗ്രഹമായും അല്ലാഹു സംഭവിപ്പിച്ച നിരവധി അമാനുഷിക ദൃഷ്ടാന്തങ്ങള്‍കൊണ്ട് നിബിഢമാണ് നബിജീവിതമെന്ന് ചരിത്രം വ്യക്തമാക്കുന്നുണ്ട്. പ്രവാചകത്വത്തിന്റെ ഇരുപത്തിമൂന്ന് വര്‍ഷക്കാലയളവിനെ സമൃദ്ധമാക്കി ആകാശത്തുനിന്ന് ഭൂമിയിലേക്കവതരിപ്പിക്കപ്പെടുകയും ക്വുര്‍ആന്‍ ആയി ക്രോഡീകരിക്കപ്പെടുകയും ചെയ്ത ആറായിരത്തില്‍പരം ദിവ്യവചസ്സുകള്‍ തന്നെയായിരുന്നു അവയില്‍ ഏറ്റവും പ്രധാനപ്പെട്ടത്. ക്വുര്‍ആനിനു പുറമെ, അല്ലാഹുവില്‍നിന്ന് ലഭിച്ച വിസ്മയകരങ്ങളായ സഹായങ്ങളായും സംരക്ഷണങ്ങളായും കിറുകൃത്യമായ പ്രവചനപ്പുലര്‍ച്ചകളായും മനുഷ്യസാധ്യമല്ലാത്ത യാത്രാനുഭവങ്ങളായും പ്രകൃതിയിലും തീന്‍മേശയിലും മനുഷ്യശരീരങ്ങളിലും വരെ പ്രകടമായ അത്ഭുതങ്ങളായും ദിവ്യദൃഷ്ടാന്തങ്ങള്‍ മുഹമ്മദ് നബി(സ)യുടെ പ്രവാചകത്വത്തിന് സാക്ഷി പറഞ്ഞുകൊണ്ടേയിരിന്നിട്ടുണ്ട്.

അത്തരത്തില്‍ ചരിത്രം രേഖപ്പെടുത്തിയ അനേകം അത്ഭുത സംഭവങ്ങളിലൊന്നു മാത്രമാണ് ചന്ദ്രന്റെ പിളര്‍പ്പും പുനസമാഗമവും. പരിശുദ്ധ ക്വുര്‍ആന്‍ തന്നെ നേര്‍ക്കുനേരെ പരാമര്‍ശിച്ചിട്ടുള്ളതും അതുകൊണ്ടുതന്നെ മുസ്‌ലിം ലോകത്ത് സുപ്രസിദ്ധവുമാണ് ഈ സംഭവം. ചാന്ദ്രപിളര്‍പ്പിനെക്കുറിച്ച് ക്വുര്‍ആന്‍ സംസാരിച്ചതിന്റെ ആശയമിങ്ങനെയാണ്:

”ആ (അന്ത്യ) സമയം അടുത്തു. ചന്ദ്രന്‍ പിളരുകയും ചെയ്തു. ഏതൊരു ദൃഷ്ടാന്തം അവര്‍ കാണുകയാണെങ്കിലും അവര്‍ പിന്തിരിഞ്ഞു കളയുകയും, ഇത് നിലനിന്നു വരുന്ന ജാലവിദ്യയാകുന്നു എന്ന് അവര്‍ പറയുകയും ചെയ്യും. അവര്‍ നിഷേധിച്ചു തള്ളുകയും തങ്ങളുടെ തന്നിഷ്ടങ്ങളെ പിന്‍പറ്റുകയും ചെയ്തിരിക്കുന്നു. ഏതൊരു കാര്യവും ഒരു നിശ്ചിത സ്ഥാനം പ്രാപിക്കുന്നതാകുന്നു.” (54: 1-3)

അന്തിമ പ്രവാചകന്റെ ദൈവിക ദൃഷ്ടാന്തമെന്ന നിലയില്‍ ചന്ദ്രന്‍ പിളര്‍ന്നുവെന്നും പക്ഷേ എത്ര വലിയ അത്ഭുതങ്ങള്‍ കണ്ടാലും മാരണമോ ജാലവിദ്യയോ ആയി തള്ളുകയാണ് മക്കയിലെ പല സത്യനിഷേധികളുടെയും രീതിയെന്നും അത്തരക്കാര്‍ ഈ മഹാത്ഭുതത്തെയും ആ ഗണത്തില്‍പ്പെടുത്തി തള്ളിയെന്നുമുള്ള വിവരങ്ങള്‍ ചാന്ദ്രപിളര്‍പ്പിനെക്കുറിച്ച് ഈ ക്വുര്‍ആന്‍ വചനങ്ങള്‍ നല്‍കുന്നുണ്ട്. ക്വുര്‍ആന്‍ വചനങ്ങളെല്ലാം പ്രവാചകകാലഘട്ടത്തിലേതാണെന്ന കാര്യം ചരിത്രപരമായി അവിതര്‍ക്കിതമാംവിധം സ്ഥാപിക്കപ്പെട്ടതാണ്. ആകാശത്ത് ചന്ദ്രന്റെ പിളര്‍പ്പ് ദൃശ്യമാവുകയും അതിനെ ദൈവിക ദൃഷ്ടാന്തമായി പരിഗണിക്കാന്‍ വിസമ്മതിച്ച ചില മക്കക്കാര്‍ പ്രസ്തുത ദൃശ്യത്തിന് മറ്റു വ്യാഖ്യാനങ്ങള്‍ നല്‍കാന്‍ ശ്രമിക്കുകയും ചെയ്തുവെന്ന് പ്രഖ്യാപിക്കുന്ന ക്വുര്‍ആന്‍ വചനങ്ങള്‍ നബിനാവിലൂടെ മക്ക കേട്ടതാണ്; അദ്ദേഹത്തിന്റെ അനുചരന്‍മാര്‍ മറ്റെല്ലാ ക്വുര്‍ആന്‍ വചനങ്ങളെയും പോലെ സമകാലീനരായ അവിശ്വാസികള്‍ക്കിടയില്‍ നിരന്തരമായി അവ പാരായണം ചെയ്ത് പ്രഘോഷിച്ചുകൊണ്ടിരിക്കുകയും ചെയ്തിട്ടുണ്ട്. എന്നിട്ടും മക്കയിലെ ഒരു ബഹുദൈവാരാധകനും ഈ ക്വുര്‍ആന്‍ വചനങ്ങളെ ഒരിക്കല്‍ പോലും ചോദ്യം ചെയ്തു രംഗത്തുവന്നില്ല. അതിനര്‍ത്ഥം ചന്ദ്രന്‍ പിളര്‍ന്നതിന് ബഹുദൈവാരാധകരടക്കമുള്ള പല മക്കക്കാരും സാക്ഷിയായിയെന്നും അതിന്റെ സ്വഭാവത്തെക്കുറിച്ചുള്ള കുതര്‍ക്കങ്ങളുന്നയിക്കുക മാത്രമാണവര്‍ ചെയ്തതെന്നുമുള്ളതിനുള്ള അനിഷേധ്യമായ ചരിത്രരേഖകളായി പരാമൃഷ്ട ക്വുര്‍ആന്‍ വചനങ്ങള്‍ മാറുന്നുവെന്ന് തന്നെയാണ്.

മദീനയിലേക്കുള്ള പലായനത്തിന്റെ ഉദ്ദേശം അഞ്ചു വര്‍ഷം മുമ്പ് ഒരു പൗര്‍ണമി രാവിലാണ് നബി തിരുമേനി (സ) തന്റെ ചുറ്റുമുണ്ടായിരുന്നവര്‍ക്ക് ചന്ദ്രന്‍ പിളര്‍ന്നുമാറുന്നത് കാണിച്ചുകൊടുത്തത് എന്നാണ് ചരിത്രകാരന്‍മാരുടെ പൊതുവായ അഭിപ്രായം. ചാന്ദ്രപിളര്‍പ്പ് ദൃശ്യമായതിനുള്ള ചരിത്രരേഖ ക്വുര്‍ആന്‍ മാത്രമല്ല; പ്രത്യുത മക്കയില്‍ പ്രവാചകന്റെ സമകാലീനരായവരില്‍ നിന്നുള്ള വിശ്വസ്തമായ ധാരാളം നിവേദനങ്ങള്‍ കൂടിയാണ്. മുഹമ്മദ് നബി(സ)യുടെ കാലത്ത് ചന്ദ്രന്‍ രണ്ടുഭാഗങ്ങളായി പിളര്‍ന്നുമാറിയെന്നു പറയുന്ന ഒട്ടനവധി ഹദീഥുകളാണ് ആധികാരികതയില്‍ ആര്‍ക്കും സംശയമില്ലാത്ത അനേകം പരമ്പരകള്‍ വഴി ഏറ്റവും പ്രാമാണികമായ ഹദീഥ് സമാഹാരങ്ങളില്‍ തന്നെ രേഖപ്പെടുത്തപ്പെട്ടിരിക്കുന്നത്. പ്രവാചകകാലം മുതല്‍ക്കുള്ള ഓരോ തലമുറയിലും അനേകമാളുകള്‍ ഒരുമിച്ച് കൈമാറി വന്നുവെന്ന് (മുതവാതിര്‍) പറയാനാകുംവിധം വിഖ്യാതവും അനിഷേധ്യവുമാണ് ഈ ഹദീഥുകളിലെ സംഭവവിവരണങ്ങള്‍. സ്വഹീഹുല്‍ ബുഖാരിയിലെ കിതാബുത്തഫ്‌സീറിലും കിതാബുല്‍ മനാക്വിബില്‍ അന്‍സ്വാറിലും സ്വഹീഹുമുസ്‌ലിമിലെ ക്വിതാബുസ്വിഫാത്തില്‍ ക്വിയാമതി വല്‍ജന്നതി വന്നാറിലും ചന്ദ്രന്‍ രണ്ടായി പിളര്‍ന്നുനീങ്ങിയെന്ന് ഖണ്ഡിതമായി പ്രസ്താവിക്കുന്ന ഹദീഥുകളുണ്ട്. മക്കക്കാര്‍ പ്രവാചകനോട് ദൃഷ്ടാന്തങ്ങള്‍ക്കുവേണ്ടി ചോദിച്ചപ്പോഴാണ് നബി (സ) ചന്ദ്രന്‍ പിളരുന്നത് കാണിച്ചുകൊടുത്തതെന്ന് സൂചിപ്പിക്കുന്ന ഹദീഥുകള്‍ സ്വഹീഹുല്‍ ബുഖാരിയിലെ കിതാബുല്‍ മനാക്വിബില്‍ അന്‍സ്വാറിലും കിതാബു ഫദാഇലി അസ്‌വ്ഹാബിന്നബിയിലും കാണാന്‍ കഴിയും. സംഭവം നടക്കുമ്പോള്‍ നബി (സ) മക്ക നഗരത്തിന് തൊട്ടപ്പുറത്തുള്ള മിനായിലായിരുന്നുവെന്ന് സ്വഹീഹുല്‍ ബുഖാരിയിലെ കിതാബുല്‍ മനാക്വിബില്‍ അന്‍സ്വാറില്‍നിന്ന് മനസ്സിലാക്കാനാകും.

ചന്ദ്രന്‍ പിളര്‍ന്നുണ്ടായ രണ്ട് കഷ്ണങ്ങളില്‍ ഒന്ന് ഒരു പര്‍വതത്തിന്റെ ഒരു വശത്തും മറ്റേത് മറുവശത്തുമായി നിന്നുവെന്നും അപാരമായ വ്യക്തതയുണ്ടായിരുന്ന ആ പിളര്‍ന്നുമാറല്‍ ദൃശ്യത്തെ ചൂണ്ടി പ്രവാചകന്‍ (സ) കൂടെയുണ്ടായിരുന്നവരോട് ‘സാക്ഷിയായിക്കൊള്ളുക’ എന്നു പറഞ്ഞുവെന്നും സ്വഹീഹുല്‍ ബുഖാരിയിലെ കിതാബുത്തഫ്‌സീറിലെയും സ്വഹീഹുമുസ്‌ലിമിലെ ക്വിതാബുസ്വിഫാത്തില്‍ ക്വിയാമതി വല്‍ജന്നതി വന്നാറിലെയും ഹദീഥുകളിലുണ്ട്. വിസ്മയകരമായ ഈ അനുഭവങ്ങള്‍ക്ക് സാക്ഷിയായി പ്രവാചക സന്നിധിയിലുണ്ടായിരുന്നവരില്‍ താനുമുള്‍പ്പെട്ടിരുന്നതായി അബ്ദുല്ലാഹിബ്‌നു മസ്ഊദ് (റ) സാക്ഷ്യപ്പെടുത്തിയത് മുസ്‌ലിമിലെ ക്വിതാബുസ്വിഫാത്തില്‍ ക്വിയാമതി വല്‍ജന്നതി വന്നാറില്‍ ഉദ്ധരിക്കപ്പെട്ടിരിക്കുന്നു.

മുഹമ്മദ് നബി(സ)യുടെ ജീവിതകാലത്ത് തന്റെ പ്രവാചകത്വത്തിന് തെളിവായി അല്ലാഹു ചന്ദ്രനെ രണ്ടു ഭാഗമാക്കി മാറ്റിനിര്‍ത്തുന്നത് അദ്ദേഹം തന്റെ ചുറ്റുമുണ്ടായിരുന്ന ചിലര്‍ക്ക് കാണിച്ചുകൊടുത്തുവെന്ന ഇസ്‌ലാമിക പാരമ്പര്യത്തിന് ചരിത്രപരമായ അടിത്തറയില്ലെന്ന മിഷനറി വാദം ശുദ്ധ അസംബന്ധമാണെന്ന് ക്വുര്‍ആനും ഹദീഥുകളും പരിശോധിക്കുന്ന ആര്‍ക്കും മനസ്സിലാകുമെന്നാണ് പറഞ്ഞുവന്നതിന്റെ ചുരുക്കം. ഒട്ടനേകം അനിഷേധ്യമായ രേഖീകരണങ്ങളുള്ള ചാന്ദ്രപ്പിളര്‍പ്പ് നടന്നത് ചരിത്രത്തിന്റെ വെള്ളിവെളിച്ചത്തിലാണെന്ന് പറയുന്നതായിരിക്കും ശരി.

ഇങ്ങനെയൊരു സംഭവം കണ്ടതായി ഹിജാസിനു പുറത്തുള്ള ആരും രേഖപ്പെടുത്തിയിട്ടില്ലെന്നു വാദിച്ചാണ് ചില മിഷനറിമാര്‍ ചന്ദ്രന്‍ പിളര്‍ന്നതിന്റെ ചരിത്രപരത നിഷേധിക്കുവാന്‍ ശ്രമിക്കാറുള്ളത്. യഥാര്‍ത്ഥത്തില്‍ ചന്ദ്രന്റെ ദൃശ്യത ഒരു സമയത്തും ഭൂഗോളത്തില്‍ എല്ലാ ഭാഗത്തും ഒരു പോലെയല്ലെന്നും മക്കയുടെ ചക്രവാളം പങ്കിടുന്ന ഭൂപ്രദേശങ്ങളില്‍ തന്നെ മുന്‍പ്രഖ്യാപനമോ ആഗോളശ്രദ്ധ നേടിയ വിളംബരങ്ങളോ ഇല്ലാതെ ആകാശത്ത് പൊടുന്നനെ സംഭവിക്കുകയും ഏതാനും സമയത്തിനകം അവസാനിക്കുകയും ചെയ്ത ഒരു സംഭവം ശ്രദ്ധിക്കപ്പെടാതെ പോവുക തികച്ചും സ്വാഭാവികമാണെന്നും വാനനിരീക്ഷണം ആധുനിക കാലത്തേതുപോലെ വ്യവസ്ഥാപിതമായിത്തീരാത്ത ഒരു കാലത്ത് അസാധാരണമായ ഒരു ആകാശക്കാഴ്ചക്ക് നിരീക്ഷകക്കുറിപ്പുകളുടെ ആധിക്യമുണ്ടാകുമെന്ന് പ്രതീക്ഷിക്കുന്നത് ബാലിശമാണെന്നും മേഘപടലങ്ങളോ ഏതാനും സമയം എന്തു സംഭവിച്ചുവെന്ന് മനസ്സിലാകാതിരിക്കലോ മതി ഇത്തരമൊരു സന്ദര്‍ഭം കുറേ പ്രദേശങ്ങളുടെ ‘ചരിത്ര’ത്തില്‍ നിന്ന് ‘പുറത്താകാന്‍’ എന്നും കണ്ടതെല്ലാം എല്ലാ കാലത്തും എല്ലാ മനുഷ്യരും രേഖപ്പെടുത്താറില്ലെന്നും രേഖപ്പെടുത്തിയതെല്ലാം ചരിത്രഗവേഷകര്‍ കണ്ടെത്തിക്കഴിഞ്ഞിട്ടില്ലെന്നുമുള്ള ലളിതവസ്തുതകള്‍ മാത്രം കണക്കിലെടുത്താല്‍ മതി ഈ പരിശ്രമങ്ങള്‍ എന്തുമാത്രം അര്‍ത്ഥശൂന്യമാണെന്നു മനസ്സിലാകാന്‍.

മക്കയില്‍ സാക്ഷികളുണ്ടായതിന് അനിഷേധ്യമായ ചരിത്രരേഖകളുള്ള ഒരു ആകാശദൃശ്യത്തെ തള്ളിക്കളയുവാനുള്ള യാതൊരു ന്യായവും മിഷനറിമാരുന്നയിക്കുന്ന മറുവാദങ്ങളിലൊന്നുമില്ലെന്നു സാരം. പ്രവാചകകാലഘട്ടത്തിലെ ചാന്ദ്രപ്പിളര്‍പ്പിന് സാക്ഷിയാവുകയോ അതിന്റെ വാര്‍ത്തകളോട് പ്രതികരിക്കുകയോ ചെയ്ത ഇന്‍ഡ്യന്‍ രാജാക്കന്‍മാരെ സംബന്ധിച്ച പ്രചുരപ്രചാരമാര്‍ജ്ജിച്ച പാരമ്പര്യങ്ങളിലേക്ക് ഇവിടെ പ്രവേശിക്കാത്തത് അവയ്ക്ക് ചരിത്രപരത ഇല്ലെന്നുവന്നാലും ചാന്ദ്രപ്പിളര്‍പ്പ് ദൃശ്യം ഒരു ചരിത്രസംഭവമാണെന്ന യാഥാര്‍ത്ഥ്യത്തെ അതൊരിക്കലും ബാധിക്കാന്‍ പോകുന്നില്ല എന്നതിനാലാണ്.

ചാന്ദ്രപ്പിളര്‍പ്പ് ആകാശത്ത് മക്കക്കാര്‍ക്ക് ദൃശ്യമായിയെന്നുറപ്പിക്കുകയാണ് ചരിത്രത്തിന്റെ ദൗത്യം; ആ ദൗത്യമാണ് ചരിത്രശാസ്ത്രത്തിന്റെ വീക്ഷണത്തില്‍ നാം ചര്‍ച്ച ചെയ്ത രേഖകള്‍ നിര്‍വഹിക്കുന്നത്. പ്രസ്തുത ദൃശ്യം എങ്ങനെയുണ്ടായതാണെന്ന് പറയാന്‍ ചരിത്രത്തിന് കഴിഞ്ഞുകൊള്ളണമെന്നില്ല. ചന്ദ്രനെ അല്ലാഹു ഒരമാനുഷിക ദൃഷ്ടാന്തമെന്ന നിലയില്‍ പിളര്‍ത്തിയതാണെന്ന മുഹമ്മദ് നബി(സ)യുടെ വിശദീകരണം വിശ്വാസത്തിന്റെ മണ്ഡലത്തിലുള്ളതാണ്. മിഷനറിമാര്‍ക്ക് പരമാവധി ചെയ്യാനാവുക ആ വിശദീകരണത്തോട് വിയോജിക്കുകയാണ്; മക്കയിലെ ബഹുദൈവാരാധകര്‍ ചെയ്തതുപോലെ. ചന്ദ്രന്‍ പിളര്‍ന്നതായുള്ള ഒരു പ്രതീതി തങ്ങളുടെ കണ്ണുകള്‍ക്കുണ്ടാകുന്ന കണ്‍കെട്ടാണ് മുഹമ്മദ് നബി (സ) നടത്തിയതെന്നാണ് അവര്‍ ആരോപിച്ചത്. ഇത് ചാന്ദ്രപ്പിളര്‍പ്പിന്റെ കാര്യത്തില്‍ മാത്രമല്ല, ക്വുര്‍ആന്‍ അടക്കമുള്ള മുഴുവന്‍ നബിദൃഷ്ടാന്തങ്ങളുടെയും കാര്യത്തില്‍ അവരുയര്‍ത്തിയ നിലപാടാണ്. തങ്ങള്‍ നിഷേധിച്ചുതള്ളിയ പ്രവാചകനിലൂടെ അത്യത്ഭുകരമായ ദൃഷ്ടാന്തങ്ങള്‍ പ്രത്യക്ഷപ്പെടുന്നത് കണ്ടുണ്ടായ അമ്പരപ്പ് മറച്ചുവെക്കാന്‍ യാതൊരാത്മാര്‍ത്ഥതയുമില്ലാതെ അവര്‍ പറഞ്ഞുേനാക്കിയ ഒഴിവുകഴിവ് മാത്രമാണതെന്ന് ചരിത്രം സൂക്ഷ്മമായി പരിശോധിക്കുന്നവര്‍ക്കു മനസ്സിലാകും. കണ്‍കെട്ടിന്റെ പരിമിതശേഷികള്‍കൊണ്ട് സാധ്യമാകുന്നവയല്ല തങ്ങള്‍ സാക്ഷ്യം വഹിച്ചുകൊണ്ടിരുന്ന നബിയത്ഭുതങ്ങളെന്നും അവ ദൈവികമായ ഇടപെടലാകാനേ തരമുള്ളൂ എന്നും അവര്‍ക്ക് വ്യക്തമായിരുന്നു. ദുര്‍വാശിയില്‍ നിന്നുണ്ടായ കപട ആരോപണമായിരുന്നു ജാലവിദ്യയുടേത്. ചാന്ദ്രപിളര്‍പ്പ് സംഭവത്തിന്റെ വിശദാംശങ്ങള്‍ തന്നെ ഇക്കാര്യം വ്യക്തമാക്കുന്നുണ്ട്.

സംഭവം മാരണജന്യമായ വ്യാജപ്രതീതിയോ മറ്റോ ആണെന്ന് വരുത്താന്‍, നമ്മള്‍ മാത്രമേ (നബിയുടെ തൊട്ടടുത്തുള്ളവര്‍) ഇങ്ങനെ കാണുന്നുണ്ടായിരിക്കുകയുള്ളൂ എന്നും അങ്ങനെ സംഭവിക്കുന്നത് കണ്‍കെട്ടായതുകൊണ്ടാണെന്നും മറ്റാരെങ്കിലും കണ്ടതായി തെളിഞ്ഞാല്‍ മാത്രമേ ഇത് ജാലവിദ്യയല്ലെന്നു വരൂ എന്നും അവര്‍ പ്രഖ്യാപിച്ചതായി ജാമിഉത്തിര്‍മിദിയിലെ കിതാബു തഫ്‌സീറില്‍ ക്വുര്‍ആനി അന്‍ റസൂലില്ലാഹിയില്‍ രേഖപ്പെടുത്തിയിട്ടുള്ള ഹദീഥില്‍ വായിക്കാന്‍ കഴിയും. സിഹ്ര്‍ എന്നവര്‍ വിളിച്ചിരുന്ന പലതരം കണ്‍കെട്ടു വിദ്യകളുടെ പരിമിതിയെക്കുറിച്ചുള്ള ബോധ്യമാണ് ഈ വര്‍ത്തമാനത്തില്‍ നിഴലിക്കുന്നത്. പിന്നീടെന്ത് സംഭവിച്ചുവെന്ന് പ്രഗല്‍ഭനായ ആദ്യകാല ക്വുര്‍ആന്‍ വ്യാഖ്യാതാവും ചരിത്രകാരനുമായ അബൂ ജഅ്ഫര്‍ ഇബ്‌നു ജരീറുത്ത്വബ്‌രി തന്റെ ബൃഹദ് ക്വുര്‍ആന്‍ വ്യാഖ്യാനഗ്രന്ഥമായ ജാമിഉല്‍ ബയാനി അന്‍ തഅ്‌വീലി അയ്യുല്‍ ക്വുര്‍ആനില്‍ ചാന്ദ്രപ്പിളര്‍പ്പിനെ പരാമര്‍ശിക്കുന്ന ക്വുര്‍ആന്‍ വചനങ്ങളുടെ (54: 1-3) വിശദീകരണക്കുറിപ്പില്‍ ഉള്‍പ്പെടുത്തിയ നിവേദനത്തിലുണ്ട്. ചന്ദ്രന്‍ പിളര്‍ന്നത് കണ്ടുവോ എന്ന് ആ രാത്രി മക്കക്കു പുറത്തായിരുന്ന ഏതെങ്കിലും യാത്രക്കാരോട് ചോദിക്കാന്‍ അവര്‍ തീരുമാനിച്ചു. അങ്ങനെ ഒരു യാത്രാസംഘത്തോട് അവര്‍ അന്വേഷിച്ചു. ‘അതെ, ഞങ്ങള്‍ അത് കണ്ടു’വെന്നായിരുന്നു അവരുടെ മറുപടി. അപ്പോഴാണ് ഈ ക്വുര്‍ആന്‍ വചനങ്ങള്‍ അവതരിപ്പിക്കപ്പെട്ടത്. (കെയ്‌റോ, ദാറുല്‍ ഹിജ്ര്‍, 2001, Vol. 23, pp.106-7).

തങ്ങള്‍ കണ്ട ആകാശദൃശ്യം കണ്‍കെട്ടല്ലെന്ന് ഇതോടുകൂടി സത്യനിഷേധികള്‍ക്ക് ബോധ്യം വന്നുവെന്ന കാര്യമുറപ്പാണ്. കണ്‍കെട്ടിന് തങ്ങള്‍ തന്നെ വെച്ച മാനദണ്ഡങ്ങള്‍ ചാന്ദ്രപിളര്‍പ്പിനു ബാധകമല്ലെന്നു മനസ്സിലായിട്ടും പ്രസ്തുത വാദത്തിലവര്‍ ഉറച്ചുനിന്നത് സത്യത്തോട് വിമുഖരാകുമാറ് അവരില്‍ രൂഢമൂലമായിരുന്ന അഹങ്കാരം കൊണ്ടുമാത്രമാണെന്നര്‍ത്ഥം. ചന്ദ്രന്‍ പിളര്‍ന്നുനിന്ന ആകാശദൃശ്യം എന്ന അനിഷേധ്യമായ ചരിത്രസംഭവത്തെ മക്കന്‍ ബഹുദൈവാരാധകരുടെ കൂടെനിന്ന് മന്ത്രവാദവല്‍കരിക്കുവാനുള്ള ശ്രമങ്ങള്‍ ഇന്നും അഹങ്കാരത്തില്‍ നിന്നുമാത്രമാണ് നിര്‍ഗളിക്കുക; അതിനോട് സംവദിക്കാന്‍ ചരിത്രത്തിന്റെ ആയുധങ്ങള്‍ മാത്രം മതിയാകില്ല.

യഥാര്‍ത്ഥത്തില്‍, പ്രവാചകന്‍മാരിലൂടെ വെളിപ്പെടുന്ന അത്ഭുതസംഭവങ്ങളെ ജാലവിദ്യയോ മാരണമോ വഴിയുള്ള കണ്‍കെട്ടായി തള്ളിക്കളഞ്ഞ് പടച്ചവനു കാണിക്കാന്‍ കഴിയുന്നതൊക്കെ കാണിക്കാന്‍ കഴിയുന്ന സൂപ്പര്‍ പവറുകളായി കണ്‍കെട്ടുകാരെ അവതരിപ്പിക്കാന്‍ മുതിര്‍ന്ന അസംബന്ധ നാടകക്കാര്‍ ചരിത്രത്തിന്റെ എല്ലാ ഘട്ടങ്ങളിലുമുണ്ടായിട്ടുണ്ട്. കണ്‍കെട്ടുകാരുടെ ചെപ്പടിവിദ്യകളുമായി താരതമ്യം പോലുമില്ലാത്ത മഹാത്ഭുതങ്ങളാണ് സംഭവിക്കുന്നതെന്നു മനസ്സിലായിട്ടും അവയെ ഇപ്രകാരം തള്ളിക്കളഞ്ഞ നിര്‍ഭാഗ്യവാന്‍മാരെ സംബന്ധിച്ച് വിവിധ പ്രവാചകന്‍മാരെകുറിച്ച ബൈബിള്‍ കഥനങ്ങളില്‍ നിന്ന് മനസ്സിലാക്കിയിട്ടും നബി(സ)യുടെ കാര്യം വരുമ്പോള്‍ അതേ നിര്‍ഭാഗ്യത്തിലേക്ക് മുഖം കുത്തുന്ന ദുര്യോഗമാണ് മിഷനറിമാര്‍ക്ക് സംഭവിക്കുന്നത്.

വടി നിലത്തിട്ടാല്‍ സര്‍പ്പമാകുന്ന ദൈവികദൃഷ്ടാന്തവുമായി രാജസദസ്സിലേക്കു കടന്നുവന്ന മോശെയോടും അഹറോനോടും അത് കണ്‍കെട്ടാണെന്ന് വാദിക്കുകയും കൊട്ടാരത്തിലെ ആസ്ഥാന കണ്‍കെട്ടുകാരെക്കൊണ്ടുവന്ന് അവരുടെ കയറുകളും വടികളുംവെച്ച് ‘പ്രതീതി സര്‍പ്പങ്ങളെ’ ഉണ്ടാക്കിപ്പിക്കുകയും മോശെയുടെ സര്‍പ്പം തങ്ങളുടെ വ്യാജസര്‍പ്പങ്ങളെ വിഴുങ്ങുന്നതുകണ്ട് മോശെയുടേത് കണ്‍കെട്ടല്ലെന്നു കണ്‍കെട്ടുകാര്‍ക്കടക്കം ബോധ്യം വന്നിട്ടും ‘കണ്‍കെട്ടുവാദ’ത്തില്‍ ധാര്‍ഷ്ഠ്യത്തോടെ നിലനില്‍ക്കുകയും ചെയ്ത ഫറോവയെക്കുറിച്ച് ക്വുര്‍ആന്‍ വിശദമായും (20: 9-71) ബൈബിള്‍ അല്‍പം കൂടി ചുരുക്കിയും (പുറപ്പാട് 7: 1-13) -വിശദാംശങ്ങളില്‍ ചില വ്യത്യാസങ്ങളുണ്ടെങ്കിലും- വിശദീകരിക്കുന്നുണ്ട്. ഫറോവയുടെ അനന്തരഗാമിത്വമാണ് തങ്ങള്‍ക്കുവേണ്ടതെന്ന് തീരുമാനിക്കുവാന്‍ മിഷനറിമാര്‍ക്ക് തീര്‍ച്ചയായും സ്വാതന്ത്ര്യമുണ്ട്; പക്ഷേ അത് മോശെയുടെയും യേശുവിന്റെയും മേല്‍വിലാസത്തിലാകുമ്പോഴാണ് നമുക്ക് സങ്കടം! ഫറോവയുടെ മനഃശാസ്ത്രം കടമെടുക്കുന്നതു കൊണ്ടല്ലാതെ മറ്റെന്തുകൊണ്ടാണ്.

 പ്രപഞ്ചനാഥന്‍ മോശെക്കുവേണ്ടി ചെങ്കടല്‍ പിളര്‍ത്തിയതും (പുറപ്പാട് 14: 21-31) ജലം രക്തമാക്കി മാറ്റിയതും (പുറപ്പാട് 7: 14-25) അഹറോനുവേണ്ടി വെറും വടിയില്‍ തളിരും പൂവും കായുമുണ്ടാക്കിയതും (സംഖ്യ 17: 1-10) യോശുവക്കുവേണ്ടി പ്രളയകാലത്ത് ജോര്‍ദ്ദാന്‍ നദിയുടെ ഒഴുക്കു നിര്‍ത്തിയതും (യോശുവ 3: 1-17) ബാലാമിനോട് അദ്ദേഹത്തിന്റെ കഴുതയെക്കൊണ്ട് സംസാരിപ്പിച്ചതും (സംഖ്യ 22: 22-30) ഏലിശക്കുവേണ്ടി മരിച്ച കുട്ടിക്ക് ജീവന്‍ നല്‍കിയതും (1 രാജാക്കന്‍മാര്‍ 17: 5-24) അടക്കമുള്ള പഴയനിയമപ്രകാരമുള്ള അത്ഭുതങ്ങളിലും യേശുക്രിസ്തുവിന്റെ ജനനവും ജീവിതവും മരണവുമെല്ലാം അത്ഭുതങ്ങളില്‍ കുതിര്‍ത്തുനിര്‍ത്തുന്ന പുതിയ നിയമവിവരണങ്ങളിലും കണ്ണുമടച്ച് വിശ്വസിക്കുന്ന മിഷനറിമാര്‍ മുഹമ്മദ് നബി(സ)യെ വായിക്കുമ്പോള്‍ മാത്രം മുഴുവന്‍ പ്രവാചകന്‍മാരുടെയും ശത്രുപക്ഷത്തുനിന്ന കണ്‍കെട്ടു സിദ്ധാന്തത്തില്‍ അഭയം തേടുന്നത്. ബൈബിള്‍ കഥകളില്‍ വിവരിക്കപ്പെടുന്ന അത്ഭുതങ്ങളെക്കാള്‍ എന്ത് ‘അവിശ്വസനീത’യാണ് നബിജീവിതത്തിലെ ദൃഷ്ടാന്തങ്ങളില്‍ മിഷനറിമാര്‍ ‘കൂടുതലായി’ കാണുന്നത്?

ബൈബിള്‍, വസ്തുനിഷ്ഠമായി പറഞ്ഞാല്‍ ആധികാരികമായ ഒരു ചരിത്രസ്രോതസ്സല്ല. അതുകൊണ്ടുതന്നെ ബൈബിള്‍ വിവരണങ്ങളുടെ മാത്രം അടിസ്ഥാനത്തില്‍ ഒരത്ഭുതത്തെയും സ്ഥിരീകരിക്കാനാവില്ല. മിഷനറിമാര്‍ വിശ്വസിക്കുന്ന അത്ഭുതങ്ങള്‍ മിക്കതും യാതൊരു ചരിത്രപരതയുമില്ലാത്തതാണെന്ന് അതിനാല്‍ തന്നെ പറയാനാകും. എന്നാല്‍ ചന്ദ്രവിഭജനമടക്കമുള്ള നബിദൃഷ്ടാന്തങ്ങളുടെ കാര്യമതല്ലെന്ന് നാം കണ്ടു. ചരിത്രപരമായി സ്ഥാപിക്കാനാവാത്ത അത്ഭുതങ്ങളില്‍ വിശ്വസിക്കുന്നവര്‍ ചരിത്രം പൂര്‍ണമായി രേഖീകരിച്ച മുഹമ്മദ് നബി(സ)യുടെ ജീവിതത്തിലെ അത്ഭുതങ്ങളെ തള്ളിക്കളയുന്നതിലെ യുക്തി എന്താണ്?

അത്ഭുതങ്ങളെ സംബന്ധിച്ച വിശ്വാസിയുടെ സങ്കല്‍പത്തെ പരിഗണിക്കുന്ന ആര്‍ക്കും ചാന്ദ്രപിളര്‍പ്പുമായി ബന്ധപ്പെട്ട മിഷനറി വിമര്‍ശനങ്ങളുടെ അന്തസാര ശുന്യത ബോധ്യപ്പെടാതിരിക്കില്ല. ഒരു പ്രവാചകന്റെ കൈക്ക് സംഭവിച്ചുവെന്ന് ചരിത്രപരമായി സ്ഥിരീകരിക്കപ്പെടുന്നതോടുകൂടിത്തന്നെ പ്രസ്തുത അത്ഭുതങ്ങളെ ദൈവിക ദൃഷ്ടാന്തങ്ങള്‍ എന്ന നിലക്ക് മനസ്സിലാക്കുക എന്നതാണ് വിശ്വാസപരമായ സമീപനം. ‘അത്ഭുത’ങ്ങളുടെ മണ്ഡലം വിശ്വാസമാണെന്നു പറയുവാനുള്ള കാരണമതാണ്. ശാസ്ത്രത്തെ ഈ ചര്‍ച്ചയിലേക്കു വലിച്ചിഴക്കുവാനുള്ള മിഷനറി ശ്രമം ഏറ്റവും വലിയ അശ്ലീലമായിത്തീരുന്നതും അതുകൊണ്ടു തന്നെ. പ്രപഞ്ചവും പദാര്‍ത്ഥവും അവ സാധാരണഗതിയില്‍ പിന്തുടരുന്ന നിയമങ്ങള്‍വെച്ച് മനസ്സിലാക്കുകയാണ് ശാസ്ത്രത്തിന്റെ ധര്‍മം. അത്ഭുതങ്ങള്‍ ആ നിയമങ്ങളെ മറികടന്ന് സംഭവിക്കുന്ന കാര്യങ്ങളാണെന്നും അതുകൊണ്ടാണ് അവ അത്ഭുതങ്ങളാകുന്നതെന്നും മിഷനറിമാര്‍ക്കറിയില്ലേ? പിന്നെ അവയെ ‘ശാസ്ത്രീയമായി’ തെളിയിക്കാന്‍ ആവശ്യപ്പെടുന്നതിന്റെ ന്യായമെന്താണ്?

പ്രാപഞ്ചിക നിയമങ്ങളെ അതിലംഘിച്ചുകൊണ്ട് എപ്പോഴെങ്കിലും എന്തെങ്കിലും നടക്കുമോ എന്ന ചോദ്യത്തിന് ‘ഇല്ല’യെന്ന് തീര്‍ത്തും നിഷേധാത്മകമായി മറുപടി പറയുക ഭൗതികവാദികള്‍ മാത്രമാണ്. പ്രപഞ്ചത്തിനൊരു സ്രഷ്ടാവുണ്ടെന്ന വസ്തുത അവരംഗീകരിക്കുന്നില്ലെന്നതാണ് അതിന്റെ കാരണം. പ്രപഞ്ചസ്രഷ്ടാവില്‍ വിശ്വസിക്കുന്നവരെ സംബന്ധിച്ചടുത്തോളം അവനാണ് സൃഷ്ടിലോകത്തിന്റെ നിയാമകന്‍. ‘സാധാരണ’യായി പിന്തുടരപ്പെടുന്ന പ്രാപഞ്ചിക നിയമങ്ങള്‍ നിര്‍മിച്ചത് അവനാണ്. ആ നിയമങ്ങള്‍ പ്രകാരമല്ലാതെ ചില കാര്യങ്ങള്‍ സംഭവിപ്പിച്ച് ‘അസാധാരണ’മായ സ്ഥിതിവിശേഷങ്ങള്‍ സൃഷ്ടിക്കുക സര്‍വശക്തനായ അവനെ സംബന്ധിച്ചേടുത്തോളം തീര്‍ത്തും സാധ്യമാണ്. അതുകൊണ്ടാണ് പ്രവാചകന്‍മാര്‍ക്ക് സാക്ഷീകരണമായി പ്രപഞ്ചനാഥന്‍ സംവിധാനിക്കുന്ന അത്ഭുതങ്ങളില്‍ വിശ്വാസി സംശയലേശമന്യേ വിശ്വസിക്കുന്നത്. അതുകൊണ്ടുതന്നെയാണ് മിഷനറിമാര്‍ ബൈബിളിലെ അത്ഭുതകഥകളില്‍ വിശ്വസിക്കുന്നതും. മുകളിലുദ്ധരിച്ച ബൈബിള്‍ അത്ഭുതങ്ങളൊന്നും തന്നെ ശാസ്ത്രീയമായി വിശദീകരിക്കാനാവില്ല; ആകുമായിരുന്നെങ്കില്‍ അവ അത്ഭുതങ്ങളാവുകയും ചെയ്യുമായിരുന്നില്ല.

ശരീരത്തില്‍ തീയേറ്റാല്‍ ചൂടും പൊള്ളലുമുണ്ടാകുമെന്നേ ശാസ്ത്രത്തിന് പറയാനാകൂ. നംറൂദ് അഗ്നികുണ്ഠത്തിലേക്കെറിഞ്ഞ അബ്രഹാമിന് തീ തണുപ്പായത് ക്വുര്‍ആന്‍ പറയുന്നുണ്ട് (21: 68-70). വിഗ്രഹപൂജകനായ ബാബിലോണിയന്‍ രാജാവ് പ്രതിമാപൂജക്ക് വിസമ്മതിച്ച മൂന്ന് ജൂതപുരോഹിതന്‍മാരെ തീയിലെറിഞ്ഞപ്പോള്‍ അവര്‍ ഒരു പൊള്ളലുമേല്‍ക്കാതെ അഗ്നിനാളങ്ങളില്‍ സൈ്വരവിഹാരം നടത്തി പുറത്തുവന്ന കഥ ബൈബിളിലുണ്ട് (ദാനിയേല്‍ 3: 10-27). ഇവിടെ ശാസ്ത്രം നിസ്സഹായമാണ്. ദൈവം ഇടപെട്ടുവെന്നു കരുതുന്ന വിശ്വാസിയോടും ദൈവമില്ലെന്നതിനാല്‍ ഇത് അസംഭ്യവമാണെന്നു കരുതുന്ന അവിശ്വാസിയോടും ‘വിഷയം പരിധിക്കു പുറത്താണ്’ എന്നുമാത്രമേ ശാസ്ത്രത്തിന് പറയാന്‍ കഴിയൂ. ചന്ദ്രന്‍ പിളര്‍ന്നത് ‘ശാസ്ത്രീയമായി’ വിശദീകരിക്കാന്‍ ആവശ്യപ്പെടുന്ന മിഷനറിമാര്‍ മറക്കുന്നത് ഈ വസ്തുതയാണ്; അത്ഭുതങ്ങളെ സംബന്ധിച്ച് അവരും മുസ്‌ലിംകളുമെല്ലാം ഒരുപോലെയംഗീകരിക്കുന്ന പ്രാഥമിക വസ്തുതയെ. പ്രകൃതിനിയമങ്ങള്‍വെച്ച് സ്വാഭാവികമായി സംഭവിച്ചിട്ടില്ലാത്തുകൊണ്ടാണ് ചന്ദ്രന്റെ പിളര്‍പ്പ് ഒരു അത്ഭുതവും പ്രവാചകത്വത്തിനുള്ള തെളിവുമായതെന്നും അത്തരമൊരു പ്രതിഭാസത്തിന്റെ പ്രായോഗികതയെക്കുറിച്ചുള്ള സംശയം ദൈവം തമ്പുരാന്റെ ശക്തിവിശേഷങ്ങളെക്കുറിച്ചുള്ള അജ്ഞതയില്‍ നിന്നുണ്ടാകുന്നതാണെന്നും തിരിച്ചറിയാന്‍ മതവും ശാസ്ത്രവും എന്താണെന്നതിനെക്കുറിച്ച പ്രാഥമിക ധാരണകള്‍ മാത്രമേ ആവശ്യമുള്ളൂ.

അത്ഭുതങ്ങളില്‍ വിശ്വസിക്കുകയും അവ ശാസ്ത്രീയമായി വിശദീകരണക്ഷമമാവുകയില്ലെന്നു തിരിച്ചറിയുകയും ഏതെങ്കിലും നിയമങ്ങളുടെ നിര്‍ബന്ധിത വരുതിക്കുള്ളിലുള്ളവനല്ല പടച്ചവനെന്ന് മനസ്സിലാക്കുകയും ചെയ്യുമ്പോള്‍ തന്നെ ചാന്ദ്രപിളര്‍പ്പിനെ ശാസ്ത്രീയമായി നിഷേധിക്കുവാന്‍ ശ്രമിക്കുന്നത് അത് ഒരു വാനലോകാത്ഭുതമായതു കൊണ്ടാണെന്നാണ് ചില മിഷനറിമാര്‍ തങ്ങളുടെ ഊന്നലുകള്‍ കൊണ്ട് ധ്വനിപ്പിക്കുവാന്‍ ശ്രമിക്കാറുള്ളത്. വാനവും ചന്ദ്രനുമെല്ലാം സര്‍വശക്തന്റെ അധികാരപരിധിക്കു പുറത്താണെന്നാണോ ഇവര്‍ ധരിച്ചിരിക്കുന്നത്?

മണ്‍തരിയും പുല്‍ക്കൊടിയും മാത്രമല്ല, ഗ്രഹവും ഉപഗ്രഹവും നക്ഷത്രവുമെല്ലാം അവന്റേതു മാത്രമാണെന്നും അവയ്ക്കു നിയമങ്ങള്‍ നിശ്ചയിക്കുന്നതും ഇച്ഛിക്കുമ്പോള്‍ നിയമങ്ങള്‍ക്കു പുറത്തേക്ക് അവയെ കൊണ്ടുപോകുന്നതും അവന്‍ മാത്രമാണെന്നും എന്നാണ് ഇവരെല്ലാം തിരിച്ചറിയുക? ഭൂമിയും ചന്ദ്രനും സൂര്യനും ഗ്രഹങ്ങളുമെല്ലാമുള്‍ക്കൊള്ളുന്ന ഒരു സംവിധാനത്തില്‍ ചാന്ദ്രപിളര്‍പ്പ് സൃഷ്ടിക്കാവുന്ന ഭൗതിക ‘പ്രത്യാഘാത’ങ്ങളെക്കുറിച്ചാകുലപ്പെടുന്നവര്‍, ഇപ്പറഞ്ഞതൊക്കെയും അവന്റേതാണെന്നും ഭൗതികശാസ്ത്ര സമവാക്യങ്ങള്‍ പ്രകാരമുള്ള പ്രതിഫലനങ്ങള്‍ക്കിടം നല്‍കി മാത്രമല്ല, ‘ചുറ്റപാടുകളെ അറിയിക്കാതെ’യും കാര്യങ്ങള്‍ നടത്താന്‍ കഴിവുള്ളവനാണവന്‍ എന്നുമുള്ള സത്യങ്ങളാണ് മറന്നുപോകുന്നത്. ചന്ദ്രോപരിതലത്തില്‍ പിളര്‍പ്പിന്റെ അടയാളങ്ങള്‍ ശേഷിക്കുന്നുണ്ടോ എന്ന ചോദ്യമുയര്‍ത്തുന്നവര്‍, പിളര്‍ത്താന്‍ മാത്രമല്ല അടയാളങ്ങളൊന്നും ബാക്കിവെക്കാതെ ചേര്‍ക്കാനും പ്രപഞ്ചനാഥനു കഴിയുമെന്നും ചന്ദ്രോപരിതലത്തിലുള്ള എല്ലാ ‘അടയാള’ങ്ങളും നാം വായിച്ചുകഴിഞ്ഞിട്ടില്ലെന്നും ഓര്‍ക്കുന്നതില്‍ ദയനീയമായി പരാജയപ്പെടുന്നതെന്തുകൊണ്ടാണ്?

അന്ധമായ നബിവിരോധം എന്താണ് ഒരു അത്ഭുതം എന്നും ആരാണ് പടച്ചവന്‍ എന്നുമുള്ള അടിസ്ഥാന യാഥാര്‍ത്ഥ്യങ്ങളെപ്പോലും വിസ്മരിച്ചുകൊണ്ടുള്ള പരിമിതബുദ്ധിയുടെ ഉന്മാദത്തിലേക്കും ചോദ്യങ്ങളുടെ പ്രളയത്തിലേക്കും മിഷനറിമാരുടെ കൈപിടിച്ചുകൊണ്ടുപോകുന്നുണ്ടെങ്കില്‍, ഇതേ ഖഗോളശാസ്ത്ര’ജ്ഞാനം’ ബൈബിള്‍ വായിക്കാന്‍ കൂടി അവര്‍ക്കുപയോഗിക്കാവുന്നതാണ്! അഹറോന്റെയും മോശെയുടെയും മരണശേഷം മോവാബ് ദേശത്തുവെച്ച് ഇസ്രയേല്‍ സമൂഹത്തിന്റെ നായകത്വമേറ്റെടുത്തശേഷം അവരെയുംകൊണ്ട് ജോര്‍ദ്ദാന്‍ നദി മുറിച്ചുകടന്ന് വാഗ്ദത്ത കാനാന്‍ ദേശത്തെത്തിയ യോശുവ, അമോര്യ രാജാക്കന്‍മാരുമായി നടത്തിയ യുദ്ധത്തില്‍ സമയം കൂടുതല്‍ ലഭിക്കാന്‍ വേണ്ടി സൂര്യനെ ഗിബയോണ്‍ താഴ്‌വരക്കു മുകളിലും ചന്ദ്രനെ അയ്യലോണ്‍ താഴ്‌വരക്കുമുകളിലും ഒരു ‘ദിവസം’ മുഴുവന്‍ ‘നിശ്ചലമാക്കി’ നിര്‍ത്തിയതിനെക്കുറിച്ച് (യോശുവ 10: 12-14) അവരുടെ അഭിപ്രായമെന്താണ്?

യേശു കുരിശില്‍ തറക്കപ്പെട്ടതിന്റെ ‘ഫലമായി’ ‘സൂര്യപ്രകാശം’ നിലച്ച് രാജ്യം മുഴുക്കെ മണിക്കൂറുകളോളം കനത്ത ‘ഇരുട്ട് പരക്കുകയും’ ദേവാലയത്തിലെ തിരശ്ശീല കീറുകയും ഭൂമി വിറക്കുകയും പാറകള്‍ പിളരുകയും ചെയ്തതിലെ (മത്തായി 27: 45-51, മാര്‍ക്കോസ് 15-33, ലൂക്കോസ് 23-44) ‘കാര്യകാരണ ബന്ധ’ത്തെയും ‘ശാസ്ത്രീയത’യെയും കുറിച്ച് അവര്‍ക്കെന്താണ് പറയാനുള്ളത്? ഒടുവിലൊടുവില്‍ നിങ്ങള്‍ നബി(സ)യെത്തോല്‍പിക്കാന്‍ പറഞ്ഞെത്തുന്നത് ഭൗതികവാദത്തിന്റെ അരമനയിലാണോ?

മുഹമ്മദ് നബി(സ)യുടെ പ്രവാചകത്വത്തിന്റെ ആരംഭത്തെക്കുറിച്ച് മുസ്‌ലിംകള്‍ പറയുന്ന കഥ, വഹ്‌യിനെക്കുറിച്ച അദ്ദേഹത്തിന്റെ അവകാശവാദങ്ങള്‍ അടിസ്ഥാനരഹിതമാണെന്നാണ് വ്യക്തമാക്കുന്നുണ്ട്. ദൈവികമായ പ്രചോദനമുണ്ടാകുമ്പോള്‍ ഉണ്ടാകേണ്ട സമാധാനത്തിനും സന്തോഷത്തിനും പകരം ഭയവിഹ്വലനായി വീട്ടില്‍ തിരിച്ചെത്തുന്ന മുഹമ്മദിനെയാണ് വെളിപാട് തുടങ്ങിയെന്ന് പറയപ്പെടുന്ന ദിവസം ചരിത്രത്തില്‍ നാം കാണുന്നത്. ദൈവദൂതനായ ജിബ്‌രീല്‍ തന്നെയാണ് പ്രവാചകനടുക്കല്‍ വന്നിരുന്നതെങ്കില്‍ ഇതാകുമായിരുന്നില്ല അനുഭവം. അതിനാല്‍ മുഹമ്മദ് നബിയ്ക്ക് നാല്‍പതാം വയസ്സിലുണ്ടായിയെന്ന് പറയപ്പെടുന്ന വെളിപാട് ദൈവത്തില്‍ നിന്നല്ലെന്ന കാര്യം സ്പഷ്ടമാണ്. പ്രവാചകത്വാരംഭവുമായി ബന്ധപ്പെട്ട നിവേദനങ്ങളെ വിശകലനം ചെയ്തുകൊണ്ട് നബി (സ) അനുഭവിച്ച വെളിപാട് ദിവ്യമല്ലെന്ന് വരുത്തിത്തീര്‍ക്കാന്‍ വേണ്ടി നബിവിമര്‍ശകര്‍ ഉന്നയിക്കുന്ന പരാമൃഷ്ടവാദങ്ങള്‍ ന്യായമല്ലേ?

ല്ല. അന്തിമപ്രവാചകന് ദിവ്യവെളിപാടുകള്‍ കിട്ടിത്തുടങ്ങിയ സന്ദര്‍ഭത്തെക്കുറിച്ചുള്ള ചരിത്രരേഖകളൊന്നും പ്രസ്തുതവെളിപാടുകളുടെ ദൈവികതയെ സംശയാസ്പദമാക്കുന്ന യാതൊരു പരാമര്‍ശവും ഉള്‍ക്കൊള്ളുന്നില്ല. പ്രവാചകന്റെ നാല്‍പതാം വയസ്സിലുണ്ടായ പ്രവാചകത്വലബ്ധിയെയും വഹ്‌യിന്റെ ആരംഭത്തെയും കുറിച്ചുള്ള കുറ്റമറ്റ നിവേദകപരമ്പരയിലൂടെയുള്ള വിവരണമുള്ളത് സ്വഹീഹുല്‍ ബുഖാരിയിലാണ്. പ്രവാചകപത്‌നി ആഇശ (റ), തന്റെ സഹോദരീപുത്രന്‍ ഉര്‍വക്ക് ആദ്യവെളിപാടിന്റെ സമയത്തെ പ്രവാചകാനുഭവങ്ങള്‍ പ്രവാചകന്റെ തന്നെ ആത്മകഥനാപരമായ വാചകങ്ങളുദ്ധരിച്ചുകൊണ്ട് സ്വന്തം വാക്കുകളില്‍ വിശദീകരിച്ചുകൊടുത്തതാണ് ബുഖാരി തന്റെ സ്വഹീഹിന്റെ തുടക്കത്തില്‍ ഹദീഥായി രേഖപ്പെടുത്തിയിട്ടുള്ളത് (കിതാബുല്‍ വഹ്‌യ്). പ്രവാചകത്വത്തിന്റെ ആരംഭത്തെക്കുറിച്ച് പരാമൃഷ്ട ഹദീഥ് നല്‍കുന്നത് താഴെക്കൊടുത്തിരിക്കുന്ന വിവരങ്ങളാണ്.

1. മുഹമ്മദ് നബി(സ)ക്ക് പകല്‍വെളിച്ചം പോലെ സത്യമായിപ്പുലരുന്ന സ്വപ്‌നദര്‍ശനങ്ങള്‍ നിരന്തരമായി ഉണ്ടാകാന്‍ തുടങ്ങി. ഇതായിരുന്നു പ്രവാചകനുമായുള്ള ദൈവിക ആശയവിനിമയത്തിന്റെ ആരംഭം.

2. ഉറക്കത്തില്‍ താന്‍ കാണുന്ന സ്വപ്‌നങ്ങളുടെ പുലര്‍ച്ച പ്രവാചകനെ ചിന്താകുലനാക്കുകയും അദ്ദേഹം മക്കയിലെ ഒരു പര്‍വതത്തിനുമുകളിലുള്ള ഹിറാഗുഹയില്‍ ഏകാന്തനായി പ്രാര്‍ത്ഥനകളില്‍ മുഴുകിയിരിക്കുന്ന ശീലം ആരംഭിക്കുകയും ചെയ്തു. രണ്ടോ മൂന്നോ ദിവസങ്ങള്‍ക്കാവശ്യമായ ജീവിതവിഭവങ്ങളുമായി ഗുഹയില്‍ തങ്ങുകയും ശേഷം വീട്ടിലേക്ക് മടങ്ങി വീണ്ടും വിഭവശേഖരണം നടത്തി ഗുഹയിലേക്കുതന്നെ തിരിച്ചുപോവുകയുമായിരുന്നു പതിവ്.

3. ഇങ്ങനെ പ്രവാചകന്‍ ഗുഹയിലിരിക്കുന്ന ഒരു ദിവസമാണ് ക്വുര്‍ആന്‍ അവതരണത്തിന് സമാരംഭം കുറിച്ചുകൊണ്ട് ഒരു മലക്ക് പ്രവാചകനരികില്‍ പ്രത്യക്ഷപ്പെട്ടത്.

4. മലക്ക് പ്രവാചകനോട് ‘ഇക്വ്‌റഅ്’ (വായിക്കുക/ഓതുക) എന്ന് കല്‍പിച്ചു. ‘ഞാന്‍ വായിക്കാനറിയുന്നവനല്ല’ (മാ അന ബി ക്വാരിഅ്) എന്നായിരുന്നു നബി(സ)യുടെ മറുപടി. അപ്പോള്‍ മലക്ക് പ്രവാചകന് താങ്ങാന്‍ കഴിയുന്നതിലുമപ്പുറത്തുള്ള ശക്തിയോടെ അദ്ദേഹത്തെ പിടിച്ചുഞെരുക്കുകയും ശേഷം സ്വതന്ത്രനാക്കുകയും ചെയ്ത് ‘ഇക്വ്‌റഅ്’ എന്ന കല്‍പന ആവര്‍ത്തിച്ചു. വീണ്ടും പ്രവാചകന്‍ പഴയ മറുപടി തന്നെ പറഞ്ഞു. മലക്ക് വീണ്ടും പ്രവാചകനെ ഞെരുക്കുകയും സ്വതന്ത്രനാക്കുകയും കല്‍പന ആവര്‍ത്തിക്കുകയും ചെയ്തു. മൂന്നാം തവണയും ഇതേഘട്ടങ്ങള്‍ കടന്നപ്പോള്‍ മലക്ക് പരിശുദ്ധ ക്വുര്‍ആനില്‍ 96-ാം അധ്യായം സൂറത്തുല്‍ അലക്വിലെ ആദ്യത്തെ അഞ്ച് വചനങ്ങള്‍ പ്രവാചകനെ ഓതിക്കേള്‍പിച്ചു. ഇതായിരുന്നു ക്വുര്‍ആന്‍ അവതരണത്തിന്റെയും പ്രവാചകത്വത്തിന്റെയും തുടക്കം. പ്രസ്തുത വചനങ്ങളുടെ സാരം ഇപ്രകാരമാണ്: ”സൃഷ്ടിച്ചവനായ നിന്റെ രക്ഷിതാവിന്റെ നാമത്തില്‍ വായിക്കുക. മനുഷ്യനെ അവന്‍ ഭ്രൂണത്തില്‍ നിന്ന് സൃഷ്ടിച്ചിരിക്കുന്നു. നീ വായിക്കുക: നിന്റെ രക്ഷിതാവ് പേനകൊണ്ട് പഠിപ്പിച്ചവനായ ഏറ്റവും വലിയ ഔദാര്യവാനാകുന്നു. മനുഷ്യന് അറിയാത്തത് അവന്‍ പഠിപ്പിച്ചിരിക്കുന്നു.”

5. ഗുഹയില്‍ ഏകാന്തനായിരിക്കെ മലക്കുമായുണ്ടായ മുഖാമുഖവും തത്സസമയത്തെ അനുഭവങ്ങളും പ്രവാചകനെ ഭയവിഹ്വലനാക്കി. പരിഭ്രാന്തനായി അതിവേഗത്തില്‍ മിടിച്ചുകൊണ്ടിരിക്കുന്ന ഹൃദയവുമായി വീട്ടിലേക്ക് മടങ്ങിയെത്തിയ അദ്ദേഹം പത്‌നി ഖദീജ ബിന്‍ത് ഖുവയ്‌ലിദിനോട് (റ) തന്നെ പുതപ്പിക്കുവാന്‍ ആവശ്യപ്പെട്ടു. വിറ നിന്ന് സാധാരണ നില കൈവരിക്കുവോളം പ്രവാചകന്‍ (സ) പുതച്ചുമൂടി നിന്നു. 6. അനന്തരം പുതപ്പില്‍നിന്ന് പുറത്തുവന്ന പ്രവാചകന്‍ ഉണ്ടായ സംഭവങ്ങള്‍ വിവരിക്കുകയും തനിക്ക് ഭയം അനുഭവപ്പെടുന്നുവെന്ന് പറയുകയും ചെയ്തു. അപ്പോള്‍ ഖദീജ (റ) ‘ഇല്ല, അല്ലാഹുവാണ് സത്യം, അല്ലാഹു അങ്ങയെ നിന്ദിക്കുകയില്ല. കാരണം അങ്ങ് ബന്ധുക്കളുടെ കാര്യം ശ്രദ്ധിക്കുന്നു, ദുര്‍ബലരുടെ ഭാരങ്ങള്‍ ചുമക്കുന്നു, ദരിദ്രര്‍ക്കും അശരണര്‍ക്കും സംരക്ഷണമേകുന്നു, അതിഥികളെ ആദരിക്കുന്നു, പ്രയാസപ്പെടുന്നവരെ സഹായിക്കുന്നു’ എന്നുപറഞ്ഞുകൊണ്ട് പ്രവാചകനെ ആശ്വസിപ്പിച്ചു. 7. പ്രവാചകനെ ആശ്വസിപ്പിച്ചശേഷം ഖദീജ അദ്ദേഹത്തെയും കൂട്ടി തന്റെ ബന്ധുവും ക്രൈസ്തവ-ജൂതഗ്രന്ഥങ്ങളില്‍ പ്രാവിണ്യമുണ്ടായിരുന്ന പണ്ഡിതനുമായിരുന്ന വറക്വത്ബ്‌നു നൗഫലിനടുത്തേക്കുപോയി. വറക്വ കാഴ്ചശക്തിയടക്കം നഷ്ടപ്പെട്ട് അങ്ങേയറ്റം വൃദ്ധനായിത്തീര്‍ന്നിരുന്നു. പ്രവാചകനുണ്ടായ അനുഭവങ്ങള്‍ കേട്ട വറക്വ, മോശെ (മൂസ) പ്രവാചകന്റെയടുക്കലേക്ക് ദിവ്യവെളിപാടുമായി വന്ന മലക്ക് ജിബ്‌രീല്‍ തന്നെയാണ് ഹിറാ ഗുഹയില്‍ വന്നതെന്ന് പറയുകയും പ്രവാചകന്‍ തനിക്ക് ലഭിച്ചുതുടങ്ങിയ ദിവ്യവെളിപാടുകള്‍ പ്രബോധനം ചെയ്യാന്‍ തുടങ്ങുമ്പോള്‍ പൂര്‍വപ്രവാചകന്‍മാരെപ്പോലെ കടുത്ത എതിര്‍പ്പുകള്‍ നേരിടുമെന്നും മക്കയില്‍നിന്ന് പുറത്താക്കപ്പെടുമെന്നും അന്ന് ഒരു യുവാവായി നാട്ടിലുണ്ടാകണമെന്നും പ്രവാചകനെ പിന്തുണക്കാന്‍ കഴിയണമെന്നും താന്‍ ആഗ്രഹിച്ചുപോകുന്നുവെന്നും പറഞ്ഞു. പക്ഷേ ഈ കൂടിക്കാഴ്ച കഴിഞ്ഞ് അധികമാകും മുമ്പെ അദ്ദേഹം മരണപ്പെട്ടുപോയി.

പരിശുദ്ധ ക്വുര്‍ആനിന്റെ അവതരണസമയത്ത് പ്രവാചകനുണ്ടായ പരാമൃഷ്ടാനുഭവങ്ങളില്‍ അസംഗതമായിട്ടെന്താണുള്ളതെന്ന് വ്യക്തമാക്കേണ്ടത് വിമര്‍ശകര്‍ തന്നെയാണ്. ഭൗതികവാദികളായ നബിവിമര്‍ശകര്‍ക്ക് വഹ്‌യ് എന്ന ആശയത്തെ തന്നെ ഉള്‍ക്കൊള്ളാന്‍ കഴിയാതിരിക്കുക സ്വാഭാവികമാണ്. പ്രപഞ്ചത്തിന് ഒരു സ്രഷ്ടാവ് തന്നെയില്ലെന്ന് വിശ്വസിക്കുന്നവര്‍ക്ക് ആ സ്രഷ്ടാവ് മനുഷ്യര്‍ക്കായുളള ധാര്‍മികപദ്ധതി വെളിപാടുകള്‍ വഴി പ്രവാചകന്‍മാര്‍ക്കെത്തിച്ചു കൊടുക്കുന്നുവെന്ന ആശയത്തെ ഒരിക്കലും അംഗീകരിക്കാനാകില്ല. അവരുടെ നിഷേധത്തിന് വഹ്‌യിന്റെ സമയത്തെ പ്രവാചകാനുഭവങ്ങളെ വിശകലനം ചെയ്തുകൊണ്ട് മറുപടി പറയുന്നത് അര്‍ത്ഥശൂന്യമാണ്. ദൈവാസ്തിത്വത്തെക്കുറിച്ചും ദൈവിക മാര്‍ഗദര്‍ശനത്തിന്റെ അനിവാര്യതയെക്കുറിച്ചുമെല്ലാമുളള സംവാദങ്ങളാണ് അവരുമായി നടക്കേണ്ടത്. പടച്ചവനുണ്ടെന്നും വഹ്‌യ് എന്നൊരു സംവിധാനമുണ്ടെന്നും അംഗീകരിക്കുന്നവര്‍ക്ക് മാത്രമാണ് വഹ്‌യിന്റെ പ്രവാചകാനുഭവങ്ങളെ അപഗ്രഥിക്കേണ്ട ആവശ്യമുണ്ടാകുന്നത്.

മിഷനറിമാര്‍ തീര്‍ച്ചയായും രണ്ടാമതുപറഞ്ഞ ഗണത്തില്‍ വരുന്നവരാണ്. പ്രവാചകനുണ്ടായത് ദൈവിക വെളിപാടല്ലെന്ന് വരുത്തിത്തീര്‍ക്കാന്‍ ഹദീഥുകളിലുള്ള ഹിറാ വിവരണങ്ങളെ ഓറിയന്റലിസ്റ്റ് വ്യാഖാനങ്ങളെ ഉപജീവിച്ചുകൊണ്ട് അവര്‍ വിമര്‍ശനാത്മകമായി വിശകലനം ചെയ്തതിന് കയ്യും കണക്കുമില്ല. ആ വിമര്‍ശനങ്ങളെല്ലാം, സ്വാഭാവികമായും വഹ്‌യിന്റെ സമയത്ത് പ്രവാചകന്‍മാര്‍ക്കുണ്ടാകുന്ന അനുഭവങ്ങളെന്തൊക്കെയാണെന്ന് ബൈബിളുപയോഗിച്ച് വിശദീകരിച്ചശേഷം പ്രവാചകനുണ്ടായത് അത്തരത്തിലുള്ള അനുഭവങ്ങളൊന്നുമല്ലെന്ന് പറയാനാണ് അടിസ്ഥാനപരമായി ശ്രമിക്കുന്നത്. വഹ്‌യ് പ്രവാചകനില്‍ ഭൗതികമായി ഏതുതരം പ്രതിഫലനങ്ങളാണ് സൃഷ്ടിക്കുക എന്നതിനെ സംബന്ധിച്ച തെറ്റുപറ്റാത്ത ധാരണകള്‍ നാം സ്വരൂപിക്കേണ്ടത് ബൈബിളില്‍ നിന്നാണ് എന്ന നിലപാടാണ് ഈ വര്‍ത്തമാനത്തിന്റെ അടിത്തറ.

ബൈബിള്‍ നൂറുശതമാനവും ദൈവികമാണെന്നും അതില്‍ മനുഷ്യ നിഗമനങ്ങളിടം പിടിച്ചിട്ടില്ലെന്നും എഴുതപ്പെട്ട അതേ രീതിയില്‍ തന്നെ ബൈബിള്‍ പുസ്തകങ്ങള്‍ ഇപ്പോഴും നിലനില്‍ക്കുന്നുവെന്നും മിഷനറിമാര്‍ തെളിയിക്കുമ്പോള്‍ മാത്രമേ ബൈബിളുപയോഗിച്ച് മുഹമ്മദ് നബി(സ)യുടെ വഹ്‌യനുഭവങ്ങളെ നിരൂപണം ചെയ്യുന്നതിന് സാധൂകരണമുണ്ടാവുകയുള്ളൂ. ബൈബിളില്‍ വഹ്‌യിനെക്കുറിച്ച് ഏഴുതപ്പെട്ടിരിക്കുന്നതെന്നല്ലാം സത്യമാണെന്ന് വിശ്വസിക്കുന്നതില്‍ -ബൈബിളിന്റെ ദൈവികത തെളിയിക്കപ്പെടാത്തിടത്തോളം കാലം- യാതൊരു യുക്തിയുമില്ല. ഇതുപോലെത്തന്നെ പ്രധാനമാണ് എല്ലാ പ്രവാചകന്‍മാരുടെയും വെളിപാടനുഭവങ്ങള്‍ ഒരുപോലെയായിരിക്കണമെന്ന് ശഠിക്കുന്നത് അര്‍ത്ഥശൂന്യമാണെന്നതും. ദൈവം അവന്റെ പ്രവാചകന്‍മാരോട് സംസാരിക്കാന്‍ വ്യത്യസ്ത മാര്‍ഗങ്ങള്‍ സ്വീകരിക്കുകയും ആശയവിനിമയത്തിന് അവരെ വ്യത്യസ്ത അനുഭവങ്ങളിലൂടെ കൂട്ടിക്കൊണ്ടുപോവുകയും ചെയ്യുന്നതില്‍ എന്ത് അസാംഗത്യമാണുള്ളത്?

ചില പ്രവാചകന്‍മാരുടെ വെളിപാട് സ്വീകരണസമയത്തെ അനുഭവങ്ങള്‍ തന്നെ എല്ലാ പ്രവാചകന്‍മാര്‍ക്കും അതുപോലെ ആവര്‍ത്തിക്കുമെന്ന് ദൈവം പറയാത്തിടത്തോളം കാലം അത്തരം താരതമ്യങ്ങള്‍ തന്നെ അപ്രസക്തമാണെന്നതാണ് വസ്തുത. നബി(സ)യുടെ വ്യക്തിനിഷ്ഠമായ വെളിപാടനുഭവങ്ങളെ ബൈബിളിന്റെ മാത്രം അടിസ്ഥാനത്തില്‍ തള്ളിക്കളയാന്‍ ശ്രമിക്കുന്നത് തീര്‍ത്തും ബാലിശമായ ഒരു മിഷനറി സങ്കുചിതത്വമാണെന്ന് ചുരുക്കം.

ഇനി, ഹിറാ അനുഭവങ്ങളെ ബൈബിളുപയോഗിച്ച് വിശകലനം ചെയ്താല്‍ അവയുടെ ദൈവികതയെ നിഷേധിക്കേണ്ടി വരുമോ എന്ന് നാം പരിശോധിക്കുക. ബൈബിളിനെ അന്ധമായി പിന്‍പറ്റുന്നവര്‍ക്കുമാത്രം ബാധകമായ ഒരു ചര്‍ച്ചയാണ് ഇത്. എങ്കിലും നാം പരിശോധനയ്ക്ക് മുതിരുക. പ്രവാചകന് സത്യമായിപ്പുലരുന്ന സ്വപ്‌നദര്‍ശനങ്ങള്‍ ഉണ്ടായിത്തുടങ്ങിയെന്നും അതായിരുന്നു അല്ലാഹു അദ്ദേഹവുമായി സംവദിക്കാനാരംഭിച്ച രീതി എന്നുമാണ് ഹദീഥ് ഒന്നാമതായി പറയുന്നത്. ബൈബിളികമായ വീക്ഷണത്തില്‍ ഇതിന് എന്ത് കുഴപ്പമാണുള്ളത്? നേരായിപ്പുലരുന്ന സ്വപ്‌നങ്ങള്‍ ഉറക്കില്‍ പ്രവാചകന്‍മാരെ കാണിക്കുന്നത് ദൈവത്തിന്റെ രീതിയാണെന്നു തന്നെയല്ലേ ബൈബിളും പറയുന്നത്? ജോസഫും (ഉല്‍പത്തി 37: 5-8), ജേക്കബും (ഉല്‍പത്തി 28: 12-14), അബ്രഹാമും (ഉല്‍പത്തി 15: 1), സോളമനും (1 രാജാക്കന്‍മാര്‍ 8: 5) എല്ലാം ദൈവത്താല്‍ സത്യസ്വപ്‌നങ്ങള്‍ കാണിക്കപ്പെട്ടതിനെക്കുറിച്ചുള്ള ബൈബിള്‍ വചനങ്ങള്‍ ഉണ്ടെന്നിരിക്കെ മുഹമ്മദ് നബി(സ)ക്കുണ്ടായ സമാനമായ അനുഭവത്തെ ബൈബിളികമായ ഏത് പ്രതലമുപയോഗിച്ചാണ് മിഷനറിമാര്‍ക്ക് തള്ളിക്കളയേണ്ടി വരുന്നത്?

താന്‍ കാണാന്‍ തുടങ്ങിയ, അക്ഷരംപ്രതി പുലര്‍ന്നുകൊണ്ടിരിക്കുന്ന സ്വപ്‌നങ്ങള്‍ പ്രവാചകനെ (സ) അത്യധികം ചിന്താകുലനാക്കുകയും ഒറ്റയ്ക്കിരിക്കുവാന്‍ അദ്ദേഹം ഇഷ്ടപ്പെടുന്ന സ്ഥിതിയുണ്ടാക്കുകയും ചെയ്തതാണ് ഹിറയില്‍ പോയി പ്രാര്‍ത്ഥനകളില്‍ ശാന്തി കണ്ടെത്തുവാന്‍ ശ്രമിക്കുന്ന ഒരു സന്ദര്‍ഭം പ്രവാചകജീവിതത്തിലേക്ക് കൊണ്ടുവന്നത് എന്ന ചരിത്രവസ്തുത, തിരുനബി(സ)ക്ക് കാണിക്കപ്പെട്ട സ്വപ്‌നങ്ങള്‍ക്കുപിന്നില്‍ ജിബ്‌രീല്‍ മലക്കുമായി മുഖാമുഖം കാണാനുള്ള ഇടത്തിലേക്ക് അദ്ദേഹത്തെ നയിച്ചുകൊണ്ടുപോവുക എന്ന ദൈവിക പദ്ധതിയുണ്ടായിരുന്നുവെന്ന് സുതരാം വ്യക്തമാക്കുന്നുണ്ട്. അന്തിമവേദഗ്രന്ഥം അവതരിപ്പിച്ചു തുടങ്ങാനുള്ള സാഹചര്യം പ്രവാചകജീവിതത്തില്‍ സൃഷ്ടിക്കുവാനും പ്രവാചകത്വത്തിന്റെ മൂര്‍ത്തമായ ആരംഭം കുറിക്കുവാനും വേണ്ടി പടച്ചവന്‍ മുഹമ്മദ് നബി(സ)യുമായുള്ള സംവേദനത്തിന്റെ ആമുഖം സ്വപ്‌നങ്ങള്‍ വഴി നിര്‍വഹിച്ചുതുടങ്ങിയതില്‍ ശുദ്ധമായ ബൈബിള്‍ കണ്ണിലൂടെ നോക്കിയാല്‍ പോലും അസാധാരണമായി യാതൊന്നുമില്ലെന്ന് നാം കണ്ടു.

ഇനി ഹിറയിലേക്ക് വരാം. ഹിറാ ഗുഹയില്‍ വന്ന് മലക്ക് പ്രവാചകനെ ക്വുര്‍ആന്‍ വചനങ്ങള്‍ കേള്‍പിച്ച അനുഭവത്തെയാണ് ഓറിയന്റലിസ്റ്റുകളും മിഷനറിമാരും ഏറ്റവും പ്രധാനമായി ആക്രമിക്കാറുള്ളത്. മനഃശാസ്ത്രപരമായി പ്രസ്തുത സന്ദര്‍ഭത്തെ ഉപജീവിച്ചുകൊണ്ടുള്ള വിമര്‍ശനകാഠിന്യം തികച്ചും സ്വാഭാവികമാണ് എന്നുപറയുന്നതായിരിക്കും ശരി. കാരണം വെളിപാട് എന്നു പറയുമ്പോള്‍ അതിനെ കേവലം ഒരു ആന്തരിക പ്രചോദനമായി മാത്രം ചുരുക്കിയെഴുതാനാഗ്രഹിക്കുന്നവരാണ് മിഷനറിമാരെല്ലാം തന്നെ. തങ്ങള്‍ വേദഗ്രന്ഥമായി അവതരിപ്പിക്കുന്ന പഴയ-പുതിയ നിയമ പുസ്തകങ്ങളൊന്നും ദൈവമോ, ദൈവദൂതനോ ഏതെങ്കിലും പ്രവാചകനെ ഓതിക്കേള്‍പ്പിച്ചവയല്ലെന്നും പലരും പലകാലങ്ങളില്‍ സ്വന്തമായി എഴുതിയുണ്ടാക്കിയവയാണെന്നും ബോധ്യമുള്ളവര്‍ക്ക്, ദൈവം മനസ്സില്‍ സൃഷ്ടിക്കുന്ന ‘തോന്നലു’കള്‍ക്കനുസൃതമായി മനുഷ്യര്‍ സ്വന്തം വാചകങ്ങളില്‍ എഴുതിയുണ്ടാക്കുന്നവയാണ് വേദഗ്രന്ഥങ്ങള്‍ എന്ന് പറയേണ്ടി വരുന്നതില്‍ യാതൊരു അത്ഭുതത്തിനും വകയില്ല. ‘ദൈവനിവേശിതവും പരിശുദ്ധാത്മ പ്രചോദിതവുമാണ്, വാചകങ്ങള്‍ ദൈവികമല്ലെങ്കിലും വിശുദ്ധ ബൈബിള്‍’ എന്ന് സമാധാനിക്കുന്നവര്‍ക്കെങ്ങനെയാണ് തങ്ങള്‍ ശത്രുവായി കാണുന്ന മുഹമ്മദ് നബി(സ)ക്ക് ക്വുര്‍ആനിന്റെ പാഠം (text) ദൈവത്തിന്റെ മലക്ക് മൂര്‍ത്തമായി പ്രത്യക്ഷപ്പെട്ട് ഓതിക്കേള്‍പ്പിച്ചുതുടങ്ങി എന്ന വൃത്താന്തം അസ്വസ്ഥതയുണ്ടാക്കാതിരിക്കുക?

ക്വുര്‍ആനിലെ വചനങ്ങള്‍ നൂറുശതമാനം ദൈവികമാണെന്നും അവ അപ്പടി പ്രവാചകന് അവതരിപ്പിക്കപ്പെടുകയാണുണ്ടായതെന്നും വരുമ്പോള്‍ ബൈബിള്‍ ക്വുര്‍ആനുമായി താരതമ്യം പോലുമര്‍ഹിക്കുന്നില്ലെന്ന് തിരിച്ചറിയുന്നവരുടെ ആര്‍ക്കും പ്രവചിക്കാവുന്ന അസഹിഷ്ണുത മാത്രമാണ് ഹിറയില്‍ ചുറ്റിത്തിരിയുന്ന വിമര്‍ശകത്തൂലികകളുടെ മഷിയും ഊര്‍ജ്ജവുമെന്ന കാര്യം സ്പഷ്ടമാണ്. സത്യം സ്വാര്‍ത്ഥതാല്‍പര്യങ്ങള്‍ക്കെതിരായി വരുമ്പോള്‍ അസഹിഷ്ണുത കാണിക്കലല്ല മറിച്ച് അതിനോടുള്ള ശാത്രവം അവസാനിപ്പിക്കലാണ് രക്ഷയുടെ മാര്‍ഗമെന്ന് ഇവര്‍ തിരിച്ചറിഞ്ഞിരുന്നെങ്കില്‍ എന്ന് പ്രാര്‍ത്ഥിക്കുവാന്‍ മാത്രമേ നമുക്ക് കഴിയൂ. വെളിപാടിനെ ഒരു അന്തപ്രചോദനത്തിന്റെ മാത്രം തലമുള്ള മാനസികാനുഭൂതിയില്‍ ഒതുക്കി നിര്‍ത്തുകയല്ല അന്തിമപ്രവാചകന്റെ കാര്യത്തില്‍ പ്രപഞ്ചനാഥന്‍ ചെയ്തത്; മറിച്ച് അതിന്റെ ആദ്യ സന്ദര്‍ഭത്തില്‍ തന്നെ തികച്ചും മൂര്‍ത്തമായി തന്റെ മലക്കിനെ പ്രവാചകനുമുന്നില്‍ കൊണ്ടുചെന്നു നിര്‍ത്തുകയും ശാരീരികമായിത്തന്നെ പ്രവാചകനെ അതിശക്തമായി മലക്കിനെക്കൊണ്ട് സ്പര്‍ശിപ്പിക്കുകയും ശേഷം സ്ഫുടമായ വാചകങ്ങളിലുള്ള സംഭാഷണത്തിന് അവസരമൊരുക്കയും ചെയ്യുകയാണ്. അകത്തുനിന്ന് നിര്‍ഗളിക്കുന്ന യാതൊന്നുമല്ല, മറിച്ച് പുറത്തുനിന്ന് പ്രവാചകനിലേക്ക് പ്രവഹിക്കുന്നതാണ് വഹ്‌യ് എന്ന് ഇതിനേക്കാള്‍ വ്യക്തമായി ഭൗതിക പരിതസ്ഥിതികളുപയോഗിച്ച് പ്രതിഫലിപ്പിക്കാനാവുകയില്ല തന്നെ. വെളിപാടു സ്വീകരണത്തിന്റെ ഏറ്റവും ഉയര്‍ന്നതും സുവ്യക്തവും തീവ്രവുമായ വൈയക്തികാനുഭവങ്ങളാണ് പ്രവാചകശ്രേഷ്ഠനുണ്ടായതെന്ന് സാരം. നബിവിദ്വേഷത്താല്‍ ഉന്മാദം ബാധിച്ച തലച്ചോറുകള്‍ക്ക് പടച്ചവന്‍ പ്രവാചകനു നല്‍കിയ ഈ തുല്യതയില്ലാത്ത ആദരവിനെ തമസ്‌കരിക്കേണ്ടി വരുന്നതിന്റെ രാഷ്ട്രീയം യാതൊരു വിശദീകരണവുമില്ലാതെ തന്നെ വ്യക്തമാണ്.

ഹിറയില്‍വെച്ച് പ്രവാചകനുണ്ടായ അനുഭവങ്ങളെ നാം പരിഗണിക്കുക. പ്രവാചകത്വമോ വെളിപാടുകളോ ദീര്‍ഘകാലത്തേക്ക് ഉണ്ടായിട്ടില്ലാത്ത മക്കയിലെ അറബ് സമൂഹത്തില്‍ ജനിച്ചുവളര്‍ന്ന, വേദപുസ്തകങ്ങളുമായി യാതൊരു പരിചയവുമില്ലാത്ത സാധാരണക്കാരനായ ഒരു മനുഷ്യന്‍ ആള്‍പാര്‍പ്പില്ലാത്ത ഒരു കൂറ്റന്‍ പര്‍വതത്തിന്റെ ചെങ്കുത്തായ ചെരുവിലുള്ള ഹിറ എന്ന ചെറിയ ഗുഹയില്‍ ഒറ്റയ്ക്കിരിക്കുമ്പോള്‍ ഒരു ദിവസം പൊടുന്നനെ അല്ലാഹുവിന്റെ ദിവ്യസന്ദേശവുമായി അതിശക്തനായ ജിബ്‌രീല്‍ എന്ന മലക്ക് പ്രത്യക്ഷപ്പെടുകയും എഴുത്തോ വായനയോ അറിയാത്ത പ്രവാചകനോട് വായിക്കുവാനാജ്ഞാപിക്കുകയും വായിക്കാനറിയില്ലെന്നു പറഞ്ഞപ്പോള്‍ ആരും ഉലഞ്ഞുപോകുംവിധം ചേര്‍ത്തുപിടിച്ചമര്‍ത്തുകയും ഇത് രണ്ടുതവണ കൂടിയാവര്‍ത്തിച്ചശേഷം ഗാംഭീര്യം തുളുമ്പുന്ന അഞ്ച് ദിവ്യവചനങ്ങള്‍ ഓതികേള്‍പിക്കുകയും അപ്രത്യക്ഷനാവുകയും ചെയ്യുന്നു. പ്രവാചകന്‍ ഇത്തരമൊരനുഭവം ജീവിതത്തിലൊരിക്കലും പ്രതീക്ഷിച്ചിട്ടുപോലുമില്ലെന്ന കാര്യം കൂടി നമ്മളോര്‍ക്കണം. പ്രവാചകത്വത്തിന്റെയും വെളിപാടുകളുടെയും രീതിശാസ്ത്രവുമായുള്ള തികഞ്ഞ അപരിചിതത്വം, താന്‍ ഒരു പ്രവാചനായി നിയോഗിക്കപ്പെടുമെന്നോ തന്റെയടുക്കലേക്ക് മലക്ക് വരും എന്നോ ഉള്ള യാതൊരു വിചാരവുമില്ലാത്ത ഒരാളുടെ തയ്യാറെയുപ്പില്ലായ്മ. അതിഭീകരമായ ഏകാന്തത. ആരും ഭയന്നുപോകുംവിധമുള്ള ആശ്ലേഷം. ലോകര്‍ക്കു മുഴുവന്‍ ദിവ്യസന്ദേശമെത്തിക്കാനുള്ള ദൗത്യം ജഗന്നിയന്താവിനാല്‍ ഏല്‍പിക്കപ്പെടുന്നത് സൃഷ്ടിക്കുന്ന മനോവിസ്‌ഫോടനം. ദൈവത്തിന്റെ വചനങ്ങള്‍ അവന്റെ ദൂതനില്‍നിന്ന് നേരിട്ടുകേള്‍ക്കുന്നത് സൃഷ്ടിക്കുന്ന ശാരീരിക വ്യതിയാനങ്ങള്‍. ആരാണ് ഭയന്നുപോകാതിരിക്കുക? ആരാണ് വിറച്ചുപോകാതിരിക്കുക?

വിശുദ്ധ ക്വുര്‍ആനിലെ വചനങ്ങള്‍, അവയുടെ ആശയങ്ങളും പദങ്ങളും ഒരുപോലെ ദൈവത്തില്‍നിന്ന് നിര്‍ഗളിച്ചവയായതിനാല്‍, ശുദ്ധ മനസ്സുള്ള ആരിലും പ്രകമ്പനങ്ങള്‍ സൃഷ്ടിക്കുക തന്നെ ചെയ്യും. ക്വുര്‍ആന്‍ വചനങ്ങളുടെ കേവലമായ ശ്രുതി തന്നെ അതിന്റെ ദൈവികതയെക്കുറിച്ച ബോധമുള്ളവരുടെ മനസ്സില്‍ ആന്ദോളനങ്ങളുളവാക്കാന്‍ പോന്നതാണ്. ആശയങ്ങളറിഞ്ഞു കേള്‍ക്കുന്നവരില്‍ അത് ചെലുത്തുന്ന സ്വാധീനം അതിനേക്കാള്‍ ശക്തമാണ്. മനുഷ്യന്റെ നിസ്സാരതയും ദൈവത്തിന്റെ മഹത്വവും ദൈവതൃപ്തിക്കായുള്ള അധ്വാനങ്ങളില്‍ മനുഷ്യന്‍ വരുത്തുന്ന അലംഭാവത്തിന്റെ ഗൗരവവും ബോധ്യപ്പെടുത്തി ജീവിതത്തെ പൂര്‍ണമായി സംസ്‌കരിക്കുവാന്‍ ശേഷിയുള്ള അതിശക്തമായ ദൈവഭയം കേള്‍വിക്കാരനില്‍ നിറയ്ക്കുകയാണ് ഓരോ ക്വുര്‍ആന്‍ വചനവും ചെയ്യുന്നത്. സത്യവിശ്വാസികളില്‍ ക്വുര്‍ആന്‍ ശ്രവണം ഉണ്ടാക്കുന്ന ഫലത്തെക്കുറിച്ച് ക്വുര്‍ആന്‍ തന്നെ പറയുന്നതിപ്രകരമാണ്: ”അല്ലാഹുവെപ്പറ്റി പറയപ്പെട്ടാല്‍ ഹൃദയങ്ങള്‍ പേടിച്ചുനടുങ്ങുകയും അവന്റെ വചനങ്ങള്‍ വായിച്ചുകേള്‍പിക്കപ്പെട്ടാല്‍ വിശ്വാസം വര്‍ധിക്കുകയും തങ്ങളുടെ രക്ഷിതാവിന്റെ മേല്‍ ഭരമേല്‍പിക്കുകയും ചെയ്യുന്നവര്‍ മാത്രമാണ് സത്യവിശ്വാസികള്‍.” (ക്വുര്‍ആന്‍ 8: 2).

ഈ ഫലം സൃഷ്ടിക്കുന്ന ക്വുര്‍ആന്‍ വചനങ്ങള്‍, ദൈവത്തിന്റെ ഘനഗംഭീരമായ സംസാരം, ആകാശത്തുനിന്ന് ഭൂമിയിലേക്കിറങ്ങിവന്ന അതിശക്തനായ ഒരു മലക്കില്‍നിന്ന് മനുഷ്യരുടെ കൂട്ടത്തില്‍വെച്ച് ആദ്യമായി കേള്‍ക്കുകയാണ് മുഹമ്മദ് നബി (സ). ക്വുര്‍ആന്‍ അവതരണം പ്രവാചകനെ മാനസികമായും ശാരീരികമായും വിറപ്പിച്ച അനുഭവമായിത്തീരാതിരിക്കുക പിന്നെയെങ്ങനെയാണ്? ക്വുര്‍ആന്‍ അവതരണപ്രക്രിയയുടെ ഭാരത്തെക്കുറിച്ച് സുന്ദരമായ ഒരുപമയിലൂടെ ക്വുര്‍ആന്‍ തന്നെ വര്‍ണിക്കുന്നുണ്ട്: ”ഈ ക്വുര്‍ആനിനെ നാം (അല്ലാഹു) ഒരു പര്‍വതത്തിനുമുകളില്‍ അവതരിപ്പിച്ചിരുന്നെങ്കില്‍ അത് (പര്‍വതം) വിനീതമാകുന്നതും അല്ലാഹുവെപ്പറ്റിയുള്ള ഭയത്താല്‍ പൊട്ടിപ്പിളരുന്നതും നിനക്ക് കാണാമായിരുന്നു. ആ ഉദാഹരണങ്ങള്‍ നാം ജനങ്ങള്‍ക്കുേവണ്ടി വിവരിക്കുന്നു. അവര്‍ ചിന്തിക്കുവാന്‍വേണ്ടി.” (59: 21) വഹ്‌യ് പ്രവാചകനില്‍ സൃഷ്ടിച്ച വിഹ്വലത, തികച്ചും സ്വാഭാവികമായിരുന്നുവെന്നര്‍ത്ഥം. വഹ്‌യ് ലഭിച്ച സ്ഥലവും രീതിയും മുതല്‍ അതിന്റെ ഉള്ളടക്കവും ധ്വനികളും വരെ ആ വിഹ്വലതയില്‍ പ്രതിഫലിക്കുന്നുണ്ട്; ഇത്ര വലിയ പ്രകമ്പനങ്ങള്‍ സൃഷ്ടിക്കുന്ന ശാരീരിക-മാനസികാനുഭവങ്ങള്‍ സമ്മാനിക്കുന്ന ദൈവികബോധനപ്രക്രിയയെ താങ്ങാനുള്ള കരുത്ത് തനിക്കുണ്ടോ എന്ന ആവലാതി മുതല്‍ അന്തിമപ്രവാചകന്‍ എന്ന അതിഭയങ്കരമായ ഉത്തരവാദിത്തം ശിരസ്സാവഹിക്കാനുള്ള വലുപ്പം തനിക്കുണ്ടോ എന്ന ഭയപ്പാടുവരെ ആ വിറയലില്‍ പ്രതിധ്വനിക്കുന്നുണ്ട്.

പ്രവാചകന്‍ ഒരു മനുഷ്യനാണ് എന്ന വസ്തുത മാത്രമാണ് ഹിറാ സംഭവം തെളിയിക്കുന്നത്; നിഷ്‌കളങ്കനും ആത്മാര്‍ത്ഥതയുള്ളവനുമായ മനുഷ്യന്‍. പ്രവാചകത്വം ഏല്‍പിക്കുവാന്‍വേണ്ടി അതേ സാഹചര്യത്തിലുള്ള ഏതു മനുഷ്യനെ അതേ അനുഭവങ്ങളിലൂടെ കൊണ്ടുപോയാലും ഇതേ കാര്യങ്ങള്‍ തന്നെയാണ് സംഭവിക്കുക. മുഹമ്മദ് നബി (സ) മലക്കാണെന്നോ ദൈവമാണെന്നോ അല്ല മുസ്‌ലിംകള്‍ വിശ്വസിക്കുന്നത്; മനുഷ്യനാണെന്നു തന്നെയാണ്. ആ മനുഷ്യന് ദിവ്യബോധനങ്ങള്‍ ലഭിച്ചു എന്നാണ് മുസ്‌ലിംകള്‍ പറയുന്നത്. അതിനെ ഖണ്ഡിക്കുവാനുതകുന്ന യാതൊന്നും പ്രവാചകന്റെ ഹിറാ അനുഭവങ്ങളിലില്ല തന്നെ. ക്വുര്‍ആന്‍ പറയട്ടെ: ”( നബിയേ,) പറയുക: ഞാന്‍ നിങ്ങളെപ്പോലെയുള്ള ഒരു മനുഷ്യന്‍ മാത്രമാകുന്നു. നിങ്ങളുടെ ദൈവം ഏകദൈവം മാത്രമാണെന്ന് എനിക്ക് ബോധനം നല്‍കപ്പെടുന്നു. അതിനാല്‍ വല്ലവനും തന്റെ രക്ഷിതാവുമായി കണ്ടുമുട്ടണമെന്ന് ആഗ്രഹിക്കുന്നുവെങ്കില്‍ അവന്‍ സല്‍കര്‍മ്മം പ്രവര്‍ത്തിക്കുകയും, തന്റെ രക്ഷിതാവിനുള്ള ആരാധനയില്‍ യാതൊന്നിനെയും പങ്കുചേര്‍ക്കാതിരിക്കുകയും ചെയ്തുകൊള്ളട്ടെ.” (18: 110)

ജിബ്‌രീലിന്റെ ആഗമനം പ്രവാചകനില്‍ നിറച്ച ഭയവും അത് സൃഷ്ടിച്ച ശാരീരിക പ്രതിഫലനങ്ങളും ബൈബിള്‍ വിവരിക്കുന്ന പ്രവാചകാനുഭവങ്ങളുമായി ചേര്‍ന്നുപോകുന്നതല്ല എന്ന മിഷനറി വാദത്തിന്റെ (അങ്ങനെ ചേര്‍ന്നുപോകുക ഒരാളുടെ പ്രവാചകത്വം തെളിയിക്കുവാന്‍ ഒരു നിലയ്ക്കും ആവശ്യമല്ലെന്ന് നാം സൂചിപ്പിച്ചുകഴിഞ്ഞു) വസ്തുനിഷ്ഠതയാണ് ഇനി പരിശോധിക്കപ്പെടേണ്ടത്. ജിബ്‌രീല്‍ (ഗബ്രിയേല്‍) മലക്കുകളില്‍ ശക്തികൊണ്ടും ഗാംഭീര്യംകൊണ്ടും വേറിട്ടുനില്‍ക്കുന്നയാളാണെന്ന സങ്കല്‍പം ബൈബിള്‍ പ്രദാനം ചെയ്യുന്നുണ്ട്. പ്രശസ്തമായ ബൈബിള്‍ ഓണ്‍ലൈന്‍ വിജ്ഞാനകോശം www.newadvent.org പറയട്ടെ: ”ഗബ്രിയേല്‍, പേര് സൂചിപ്പിക്കുന്നതുപോലെ ദൈവത്തിന്റെ ശക്തിയുടെ ദൂതനാണ്. ഗബ്രിയേലിനെ പരാമര്‍ശിക്കുന്ന ബൈബിള്‍ വചനങ്ങളില്‍ മഹത്വത്തെയും ശക്തിയെയും അധികാരത്തെയും ബലത്തെയുമെല്ലാം സൂചിപ്പിക്കുന്ന പദങ്ങള്‍ നിരന്തരമായി ആവര്‍ത്തിക്കുന്നത് ശ്രദ്ധേയമാണ്… ജൂതന്‍മാര്‍ ഗബ്രിയേലിന്റെ ഈ വിശേഷണങ്ങളെ ആഴത്തില്‍ ഉള്‍ക്കൊണ്ടിട്ടുള്ളതുപോലെയാണ് തോന്നുന്നത്… സൊദോമിന്റെ നാശം ഗബ്രിയേലിന്റെ കൈകളിലൂടെയാണുണ്ടായതെന്ന് അവര്‍ മനസ്സിലാക്കുന്നു.” അതിശക്തനും ഗംഭീരഭാവമുള്ളയാളും ഒരു പ്രദേശത്തെയൊന്നടങ്കം നശിപ്പിക്കുവാന്‍ ദൈവം നിയോഗിച്ച ബലവാനുമെല്ലാമായാണ് ബൈബിള്‍ പഴയനിയമം ജിബ്‌രീലിനെ അവതരിപ്പിക്കുന്നതെന്ന് സാരം. ഹിറാ ഗുഹയില്‍ പ്രവാചകനുമുന്നില്‍ ജിബ്‌രീല്‍ പ്രത്യക്ഷപ്പെട്ടതിനെത്തുടര്‍ന്നുണ്ടായ സംഭവങ്ങളെ ന്യായീകരിക്കുക മാത്രമല്ലേ ഈ ബൈബിള്‍ വര്‍ണനകള്‍ ചെയ്യുന്നത്?

ഇനി ജിബ്‌രീല്‍ പ്രവാചകന്‍മാര്‍ക്ക് പ്രത്യക്ഷപ്പെട്ടതിനെ സംബന്ധിച്ച ബൈബിള്‍ വിവരണങ്ങളിലേക്കു വരാം. ഗബ്രിയേല്‍ ദൂതനുമായി ആശയവിനിമയം നടത്തിയ ഏറ്റവും ശ്രദ്ധേയനായ പഴയനിയമ കഥാപാത്രം ദാനിയേല്‍ ആണ്. എന്തായിരുന്നു ദാനിയേലിന്റെ അനുഭവം? ഭാവിയെക്കുറിച്ചുള്ള ഭീഷണമായ സുചനകളുള്‍ക്കൊള്ളുന്നുവെന്ന് തോന്നിപ്പിക്കുന്ന വിചിത്രദൃശ്യങ്ങളുള്ള ഒരു ദര്‍ശനം ദാനിയേലിനുണ്ടാകുന്നു. ദര്‍ശനത്തിന്റെ ആഘാതത്തില്‍ നില്‍ക്കവെ ഗബ്രിയേല്‍ മനുഷ്യരൂപത്തില്‍ ദാനിയേലിനു പ്രത്യക്ഷപ്പെട്ടതിനെക്കുറിച്ച് ബൈബിള്‍ പറയട്ടെ: ”ദാനിയേലായ ഞാന്‍ ഈ ദര്‍ശനം ഗ്രഹിക്കാന്‍ ശ്രമിച്ചുകൊണ്ടിരിക്കെ, ഇതാ എന്റെ മുമ്പില്‍ മനുഷ്യരൂപമുള്ള ഒരുവന്‍ നില്‍ക്കുന്നു. ഉലായ് തീരങ്ങളില്‍നിന്ന് ഒരുവന്‍ വിളിച്ചുപറയുന്നത് ഞാന്‍ കേട്ടു; ഗബ്രിയേല്‍, ദര്‍ശനം ഇവനെ ഗ്രഹിപ്പിക്കുക. ഞാന്‍ നിന്നിടത്തേക്ക് അവന്‍ (ഗബ്രിയേല്‍) വന്നു. അവന്‍ വന്നപ്പോള്‍ ഞാന്‍ ഭയവിഹ്വലനായി സാഷ്ടാംഗം വീണു… അവന്‍ എന്നോട് സംസാരിച്ചുകൊണ്ടിരിക്കുമ്പോള്‍ ഞാന്‍ മൂര്‍ഛിച്ചുവീണു. എന്നാല്‍ അവന്‍ എന്നെ തൊട്ട് എഴുന്നേല്‍പിച്ചു നിര്‍ത്തി.” (ദാനിയേല്‍ 8: 15-18)

മറ്റൊരിക്കല്‍ ഗബ്രിയേല്‍ ടൈഗ്രീസ് തീരത്തുവെച്ച് ദാനിയേലിനു പ്രത്യക്ഷപ്പെട്ടതിനെക്കുറിച്ച് പത്താം അധ്യായത്തിലുണ്ട്. അതിപ്രാകാരം: ”ഞാന്‍ കണ്ണുയര്‍ത്തി നോക്കിയപ്പോള്‍ ചണവസ്ത്രവും ഊഫാസിലെ സ്വര്‍ണം കൊണ്ടുള്ള അരപ്പട്ടയും ധരിച്ച ഒരുവനെ കണ്ടു. അവന്റെ ശരീരം ഗോമേദകം പോലെയും മുഖം മിന്നല്‍ പോലെയും കണ്ണുകള്‍ ജ്വലിക്കുന്ന ഒരു പന്തം പോലെയും ആയിരുന്നു. അവന്റെ കൈകാലുകള്‍ മിനുക്കിയ ഓടിന്റെ ഭംഗിയുള്ളവയും സ്വരം ജനക്കൂട്ടത്തിന്റെ ഇരമ്പല്‍ പോലെയും ആയിരുന്നു. ദാനിയേലായ ഞാന്‍ മാത്രം ഈ ദര്‍ശനം കണ്ടു; എന്നോടൊപ്പമുണ്ടായിരുന്നവര്‍ അതു കണ്ടില്ല. മഹാഭീതി പിടിപെട്ട് അവര്‍ ഓടിയൊളിച്ചു. അങ്ങനെ തനിച്ചായ ഞാന്‍ ഈ മഹാദര്‍ശനം കണ്ടു; എന്റെ ശക്തി ചോര്‍ന്നുപോയി. എന്റെ മുഖം തിരിച്ചറിയാന്‍ വയ്യാത്തവിധം മാറിപ്പോയി. എന്റെ ശക്തിയറ്റു. അപ്പോള്‍ ഞാന്‍ അവന്റെ സ്വരം കേട്ടു, അവന്റെ സ്വരം ശ്രവിച്ച ഞാന്‍ പ്രജ്ഞയറ്റ് നിലം പതിച്ചു. എന്നാല്‍, ഒരു കരം എന്നെ സ്പര്‍ശിച്ചു. അവന്‍ എന്നെ എഴുന്നേല്‍പിച്ചു. വിറയലോടെയാണെങ്കിലും മുട്ടും കയ്യും ഊന്നി ഞാന്‍ നിന്നു. അവന്‍ എന്നോട് പറഞ്ഞു: ഏറ്റവും പ്രിയങ്കരനായ ദാനിയേലേ, എഴുന്നേല്‍ക്കുക. ഞാന്‍ നിന്നോട് പറയുന്ന വാക്കുകള്‍ ശ്രദ്ധിച്ചുകേള്‍ക്കുക. എന്നെ നിന്റെയടുത്തേക്ക് അയച്ചിരിക്കുകയാണ്. അവന്‍ ഇതുപറഞ്ഞപ്പോള്‍ ഞാന്‍ വിറയലോടെ നിവര്‍ന്നുനിന്നു.” (ദാനിയേല്‍ 10: 5 – 11)

ഹിറയിലുണ്ടായ വെളിപാടിന്റെ ദൈവികതയെ നിഷേധിക്കാന്‍ ബൈബിളുപയോഗിച്ച് കഴിയില്ലെന്നും പ്രവാചകാനുഭവങ്ങളെ സാധൂകരിക്കുക മാത്രമാണ് ബൈബിള്‍ ഈ വിഷയത്തില്‍ ചെയ്യുന്നത് എന്നുമുള്ള കേവല വസ്തുതകളാണ് ഇവിടെ അനാവൃതമാകുന്നത്. ഗബ്രിയേലുമായുള്ള മനുഷ്യമുഖാമുഖത്തെക്കുറിച്ച് ബൈബിള്‍ പുതിയ നിയമവും ഇതേദിശയിലുള്ള സൂചനകള്‍ നല്‍കുന്നുണ്ട്. യോഹന്നാന്‍ സ്‌നാപകന്റെ പിതാവ് സെഖര്യാവിന്റെ അനുഭവങ്ങള്‍ ശ്രദ്ധിക്കുക: ”അപ്പോള്‍, കര്‍ത്താവിന്റെ ദൂതന്‍ ധൂപപീഠത്തിന്റെ വലതുവശത്ത് നില്‍ക്കുന്നതായി അവന് പ്രത്യക്ഷപ്പെട്ടു. അവനെക്കണ്ട് സഖറിയാ അസ്വസ്ഥനാവുകയും ഭയപ്പെടുകയും ചെയ്തു.” (ലൂക്കോസ് 1: 11, 12). പ്രവാചകന്‍ (സ) ഭയവിഹ്വലനായത് ചൂണ്ടിക്കാണിച്ച് അദ്ദേഹത്തിനുണ്ടായത് വഹ്‌യല്ലെന്നും വന്നത് മലക്കല്ലെന്നുമെല്ലാം സമര്‍ത്ഥിക്കുവാന്‍ പാടുപെടുന്ന മിഷനറിമാര്‍ തങ്ങളുടെ സ്വന്തം ബൈബിളിന്റെയടക്കം കഴുത്തിലാണ് ഈ അഭ്യാസപ്രകടനങ്ങള്‍ക്കിടയില്‍ കത്തിവെക്കുന്നതെന്നര്‍ത്ഥം.

ബൈബിള്‍ ശരിയായ രീതിയില്‍ വായിച്ചവര്‍ക്കൊന്നും മുഹമ്മദ് നബി(സ)യ്ക്ക് ഹിറാ ഗുഹയില്‍ വെച്ചുണ്ടായ വെളിപാടിനെ സാക്ഷീകരിക്കുവാനല്ലാതെ നിരാകരിക്കാന്‍ കഴിയില്ലെന്ന വസ്തുതയാണ് ഹദീഥിന്റെ അവസാന ഭാഗത്തുള്ള വറക്വത്ബ്‌നു നൗഫലിന്റെ വാചകങ്ങള്‍ തെളിയിക്കുന്നത്. ജൂത-ക്രിസ്തു ദര്‍ശനങ്ങളോട് ആഭിമുഖ്യവും അവരുടെ ഗ്രന്ഥങ്ങളില്‍ അഗാധപരിജ്ഞാനവുമുണ്ടായിരുന്ന വറക്വ, നബി(സ)യുടെ വെളിപാടനുഭവങ്ങള്‍ കേട്ടപ്പോള്‍ അത് ജിബ്‌രീല്‍ തന്നെയാണെന്ന് സാക്ഷ്യപ്പെടുത്തുകയും നബി(സ)ക്ക് പിന്തുണ പ്രഖ്യാപിക്കുകയുമാണ് ചെയ്തതെന്ന യാഥാര്‍ത്ഥ്യം മിഷനറിമാരുടെ കണ്ണുതുറപ്പിക്കേണ്ടതുണ്ട്. അതെ, ഖദീജ (റ) ആണയിട്ടു പറഞ്ഞതുപോലെ ബന്ധുക്കള്‍ക്കും ദുര്‍ബലര്‍ക്കും ദരിദ്രര്‍ക്കും അശരണര്‍ക്കും അതിഥികള്‍ക്കും പ്രയാസപ്പെടുന്നവര്‍ക്കും തണല്‍മരമായി നിന്നിരുന്ന മുഹമ്മദ് (സ) എന്ന നന്മകളുടെ ഉടല്‍ രൂപത്തെ പ്രവാചകനായി നിയോഗിക്കുവാന്‍ ജിബ്‌രീല്‍ എന്ന മലക്കു തന്നെയാണ് ഒന്നര സഹസ്രാബ്ദത്തോളം മുമ്പ് ഹിറാ ഗുഹയില്‍ പ്രത്യക്ഷപ്പെട്ടത്; മുഹമ്മദ് നബി (സ) ജിബ്‌രീലിനോട് സ്വയം സാക്ഷ്യപ്പെടുത്തിയതുപോലെ എഴുത്തോ വായനയോ അറിയാതിരുന്ന, വേദപുസ്തക പരിജ്ഞാനം അശേഷമില്ലാതിരുന്ന ആ സാധാരണക്കാരന്റെ നാവില്‍ നിന്ന് ലോകം വിസ്മയത്തോടുകൂടി ക്വുര്‍ആന്‍ കേട്ടത് ജിബ്‌രീല്‍ അദ്ദേഹത്തിനത് പഠിപ്പിച്ചുകൊടുത്തതുകൊണ്ടാണ്. ഹിറയില്‍ നിന്ന് പ്രസരിച്ച വെളിച്ചത്തിന് ബൈബിള്‍ മറയാകുമെന്ന് കരുതുന്നവര്‍ വിഡ്ഢികളുടെ സ്വര്‍ഗത്തിലാണെന്നു മാത്രമാണ് നമുക്ക് പറയാനുള്ളത്.

വിശുദ്ധ ക്വുര്‍ആന്‍ പ്രപഞ്ചനാഥന്‍ പ്രവാചകന് അവതരിപ്പിച്ചുകൊടുത്തതാണെന്ന് അതിന്റെ ഉള്ളടക്കം തെളിയിക്കുന്നുവെന്ന മുസ്‌ലിംകളുടെ വാദം അടിസ്ഥാനരഹിതമാണ്. മുഹമ്മദ് നബി (സ) തനിക്ക് പരിചയമുണ്ടായിരുന്ന ജൂതക്രൈസ്തവരില്‍ നിന്ന് നേടിയെടുത്ത മതവിജ്ഞാനീയങ്ങള്‍ ക്വുര്‍ആന്‍ വചനങ്ങളാക്കി പുനരാവിഷ്‌കരിക്കുകയാണ് യഥാര്‍ത്ഥത്തില്‍ ചെയ്തത്. പ്രവാചകപൂര്‍വ കാലഘട്ടത്തില്‍ ബൈസന്റൈന്‍ സിറിയന്‍ പ്രവിശ്യകളിലൂടെ അദ്ദേഹം നടത്തിയ യാത്രകളില്‍ കണ്ടുമുട്ടിയ ക്രൈസ്തവ പണ്ഡിതന്‍മാരുടെ ശിഷ്യത്വമാണ് പ്രവാചകനെ ക്വുര്‍ആനിന്റെ 'രചന'ക്ക് പ്രാപ്തനാക്കിയതെന്ന് മനസ്സിലാക്കുവാന്‍ കഴിയും. ഓറിയന്റലിസ്റ്റുകളും മിഷനറിമാരും മുഹമ്മദ് നബി(സ)യുടെ പ്രവാചകത്വത്തെ നിഷേധിക്കുവാന്‍ വേണ്ടി ഉന്നയിക്കുന്ന ഈ വാദങ്ങള്‍ സത്യസന്ധമാണോ?

ല്ല. ഓറിയന്റലിസ്റ്റുകള്‍ കൊണ്ടുനടക്കുന്ന 'സിറിയന്‍ കഥകള്‍'ക്കൊന്നും -അവയുടെ ചരിത്രപരത വേറെ പരിശോധിക്കപ്പെടേണ്ടതാണ്- മുഹമ്മദ് നബി(സ)യുടെ പ്രവാചകത്വത്തെയോ ക്വുര്‍ആന്റെ ദൈവികതയെയോ ഒരു നിലക്കും ചോദ്യം ചെയ്യാനുളള കെല്‍പില്ലെന്നതാണ് വാസ്തവം. ക്വുര്‍ആനിലെ വചനങ്ങള്‍ പ്രപഞ്ചനാഥന്റേതു മാത്രമാണെന്ന് അവയുടെ ഉള്ളടക്കവും ശൈലിയും സുതരാം വ്യക്തമാക്കുന്നുണ്ട്. ആയിരം സിറിയന്‍ യാത്രകളും പതിനായിരം ക്രൈസ്തവപണ്ഡിതന്‍മാരുടെ ശിഷ്യത്വവുമുണ്ടായാലും ക്വുര്‍ആനിനെപ്പോലൊരു രചന നിര്‍വഹിക്കുവാന്‍ ഒരാള്‍ക്കും സാധ്യമല്ലെന്നതാണ് വാസ്തവം.

ബൈബിള്‍ വിജ്ഞാനീയങ്ങളിലുള്ള അവഗാഹമാണല്ലോ, ക്രൈസ്തവ സമ്പര്‍ക്കങ്ങള്‍ മുഖേന നേടിയെടുക്കുവാന്‍ കഴിയുമെന്ന് വിമര്‍ശകര്‍ കരുതുന്ന ക്വുര്‍ആന്‍ രചനക്കാവശ്യമായ 'ആയുധം'. ബൈബിളില്‍ പ്രവാചകന്‍മാരെയും വേദഗ്രന്ഥങ്ങളെയും മാനവചരിത്രത്തെയും സംബന്ധിച്ച് രേഖപ്പെടുത്തിയിട്ടുള്ള 'വിവരങ്ങള്‍' പ്രമാദമുക്തമാണെന്ന് ഓറിയന്റലിസ്റ്റുകള്‍ക്ക് വാദമുണ്ടോ? അബദ്ധങ്ങള്‍കൊണ്ട് 'സമൃദ്ധ'മായ പ്രസ്തുത 'വിവര'ങ്ങളായിരുന്നു ക്വുര്‍ആനിന്റെ അവവലംബമെങ്കില്‍ ബൈബിളിനെപ്പോലെത്തന്നെ ക്വുര്‍ആനും തെറ്റുകളുടെ ഒരു ഘോഷയാത്രയായിത്തീരുമായിരുന്നുവെന്നതാണ് വാസ്തവം. എന്നാല്‍ ഒരൊറ്റ അബദ്ധംപോലും വരുത്താതെയാണ് സെമിറ്റിക് പ്രവാചകന്‍മാരെക്കുറിച്ചും വേദഗ്രന്ഥങ്ങളെക്കുറിച്ചും ഇസ്രയേല്‍ വംശത്തിന്റെ നാള്‍വഴിയെക്കുറിച്ചുമെല്ലാം പരിശുദ്ധ ക്വുര്‍ആന്‍ സംസാരിക്കുന്നത്. ജൂത-ക്രൈസ്തവ പുരോഹിതന്‍മാരും ബൈബിളെഴുത്തുകാരും ചരിത്രാഖ്യാനത്തില്‍ വരുത്തിയ സ്ഖലിതങ്ങളൊന്നുപോലും പരിശുദ്ധ ക്വുര്‍ആനില്‍ കടന്നുവരുന്നില്ലെന്ന യാഥാര്‍ത്ഥ്യം തന്നെ, ക്വുര്‍ആനിക ഉളളടക്കത്തിന് ക്രൈസ്തവസ്രോതസ്സുകളെ സങ്കല്‍പിക്കുന്നത് എന്തുമാത്രം വലിയ അസംബന്ധമാണെന്ന് വ്യക്തമാക്കുന്നുണ്ട്.

ക്രൈസ്തവമോ ക്രൈസ്തവേതരമോ ആയ ഒരു വൈജ്ഞാനിക പാരമ്പര്യത്തിനും പരിശുദ്ധ ക്വുര്‍ആനിന്റെ ഉള്ളടക്കത്തിനുള്ള വിശദീകരണമായിത്തീരാന്‍ കഴിയില്ല. കാരണം മനുഷ്യകര്‍തൃത്വമുള്ള പരാമൃഷ്ട വിജ്ഞാനശേഖരങ്ങളിലെല്ലാം തന്നെ മനുഷ്യസഹജമായ അബദ്ധങ്ങളുടെ നിറസാന്നിദ്ധ്യമുണ്ട്; കാലഘട്ടത്തിന്റെയും പ്രദേശത്തിന്റെയും ബുദ്ധിശക്തിയുടെയും ഓര്‍മശേഷിയുടെയും സത്യസന്ധതയുടെയും പരിമിതകള്‍ക്കൊണ്ടുവന്ന സ്വാഭാവികമായ അബദ്ധങ്ങള്‍. എന്നാല്‍ വിശുദ്ധ ക്വുര്‍ആനില്‍ സെമിറ്റിക് പാരമ്പര്യത്തെക്കുറിച്ച് എന്നല്ല, ആറായിരത്തില്‍പരം വചനങ്ങളിലായി പരന്നുകിടക്കുന്ന പരശ്ശതം വിഷയങ്ങളെക്കുറിച്ചുള്ള പരാമര്‍ശങ്ങളും ഒരു ചെറിയ സ്ഖലിതം പോലും പേറുന്നില്ലെന്ന സത്യം ക്വുര്‍ആന്‍ മനുഷ്യരചനയല്ലെന്നും മനുഷ്യരുടെയൊന്നും സഹകരണം അത്തരമൊരു ഗ്രന്ഥത്തിന്റെ രചനക്ക് ഉപകാരപ്പെടുകയില്ലെന്നും അസന്നിഗ്ധമായി വ്യക്തമാക്കുന്നുണ്ട്. സര്‍വജ്ഞനും സൂക്ഷ്മജ്ഞനുമായ അല്ലാഹുവിനു മാത്രമേ പരിശുദ്ധ ക്വുര്‍ആന്‍ അവതരിപ്പിക്കുവാന്‍ കഴിയൂ എന്നാണ് അതിലെ വചനങ്ങളുടെയെല്ലാം കണിശമായ കൃത്യത നമ്മെ ബോധ്യപ്പെടുത്തുന്നത്.

ക്വുര്‍ആന്‍ ദൈവികമല്ലെന്ന് വാദിക്കുന്നവരെ ക്വുര്‍ആനില്‍ മനുഷ്യസഹജമായ അബദ്ധങ്ങള്‍ ചൂണ്ടിക്കാണിച്ച് തങ്ങളുടെ വാദം തെളിയിക്കുവാന്‍ ക്വുര്‍ആന്‍ തന്നെ വെല്ലുവിളിച്ചിട്ടുണ്ടെന്ന കാര്യം ശ്രദ്ധേയമാണ്: ''അവര്‍ ക്വുര്‍ആനിനെക്കുറിച്ചാലോചിക്കുന്നില്ലേ; അത് അല്ലാഹു അല്ലാത്തവരുടെ പക്കല്‍ നിന്നായിരുന്നുവെങ്കില്‍ അവര്‍ക്കതില്‍ ധാരാളം അബദ്ധങ്ങള്‍ കണ്ടെത്തുവാന്‍ കഴിയുമായിരുന്നല്ലോ!'' (ക്വുര്‍ആന്‍ 4 : 82).

ഒന്നര സഹസ്രാബ്ദത്തോളമായി ലോകത്ത് അജയ്യമായി നിലനില്‍ക്കുന്ന ഈ വെല്ലുവിളിയെ ഫലപ്രദമായി നേരിടാന്‍ കഴിയാത്തതുകൊണ്ടാണ് ഓറിയന്റലിസ്റ്റുകളും മിഷനറിമാരും 'സിറിയയില്‍ പോയി' മനസ്സമാധാനം കണ്ടെത്താന്‍ ശ്രമിക്കുന്നത്. ഇതുപോലെത്തന്നെയാണ് ക്വുര്‍ആനിന്റെ അനാദൃശമായ പ്രതിപാദന സൗകുമാര്യവും. അറേബ്യയിലോ സിറിയയിലോ റോമിലോ പേര്‍ഷ്യയിലോ യമനിലോ ഭാരതത്തിലോ ഗ്രീസിലോ എല്ലമായി അന്ന് ജീവിച്ചിരുന്ന സാഹിത്യസാമ്രാട്ടുകള്‍ക്കൊന്നും വികൃതാനുകരണങ്ങള്‍ക്കുപോലും ശ്രമിച്ചുനോക്കാന്‍ കഴിയാതിരുന്ന, ഇരുപത്തിമൂന്ന് വര്‍ഷക്കാലം കൊണ്ട് വലിയൊരു സമൂഹത്തെ അപ്പാടെ അനുയായികളായി നേടിയെടുത്ത തികച്ചും മൗലികമായ പരിശുദ്ധ ക്വുര്‍ആനിന്റെ ശൈലി സിറിയയില്‍ നിന്ന് മുഹമ്മദ് നബി(സ)ക്ക് 'വീണുകിട്ടി'യതാണെന്നു പറയാന്‍ അന്ധതയുടെ മൂര്‍ധന്യതയിലുള്ളവര്‍ക്കു മാത്രമേ കഴിയൂ! മഹാപണ്ഡിതന്‍മാരും സാഹിത്യകാരന്‍മാരും മതപുരോഹിതന്‍മാരും പകച്ചുനിന്നുപോയ ക്വുര്‍ആന്‍ വചനങ്ങള്‍ ദിവ്യമെന്ന് സത്യസന്ധര്‍ക്കൊക്കെയും ബോധ്യമാകുംവിധം നിരക്ഷരനായ ഒരു മനുഷ്യന്റെ നാവില്‍നിന്ന് വശ്യമനോഹരമായി നിര്‍ഗളിച്ചതിന് ഏതെങ്കിലും 'യാത്രകള്‍' വിശദീകരണമാകുമെന്ന് കരുതുന്നവര്‍ സഹതാപം പോലുമര്‍ഹിക്കുന്നില്ലെന്നതാണ് വാസ്തവം.

യാത്രകള്‍ പലതവണ ചെയ്തവരും മഹാപണ്ഡിതന്‍മാരുടെ ശിഷ്യത്വം സ്വീകരിച്ചവരും അനേകമായിരമുണ്ടായിട്ടുണ്ടല്ലോ ലോകത്ത്. അവര്‍ക്കാര്‍ക്കും ക്വുര്‍ആനിനെപ്പോലൊരു രചന സാധ്യമാകാത്തതെന്തുകൊണ്ടാണെന്ന് വിമര്‍ശകര്‍ വിശദീകരിക്കുമോ? ക്വുര്‍ആനിലൂടെ പ്രപഞ്ചനാഥന്‍ തന്നെ സംസാരിക്കട്ടെ: ''നമ്മുടെ ദാസന് നാം അവതരിപ്പിച്ചുകൊടുത്തതിനെ (വിശുദ്ധ ക്വുര്‍ആനിനെ)പറ്റി ന്റത് പോലുള്ള ഒരു അദ്ധ്യായമെങ്കിലും നിങ്ങള്‍ കൊണ്ടുവരിക. അല്ലാഹുവിന് പുറമെ നിങ്ങള്‍ക്കുള്ള സഹായികളേയും വിളിച്ചുകൊള്ളുക. നിങ്ങള്‍ സത്യവാന്‍മാരാണെങ്കില്‍ (അതാണല്ലോ വേണ്ടത്). നിങ്ങള്‍ക്കത് ചെയ്യാന്‍ കഴിഞ്ഞില്ലെങ്കില്‍ നിങ്ങള്‍ക്കത് ഒരിക്കലും ചെയ്യാന്‍ കഴിയുകയുമില്ല. മനുഷ്യരും കല്ലുകളും ഇന്ധനമായി കത്തിക്കപ്പെടുന്ന നരകാഗ്‌നിയെ നിങ്ങള്‍ കാത്തുസൂക്ഷിച്ചുകൊള്ളുക. സത്യനിഷേധികള്‍ക്കുവേണ്ടി ഒരുക്കിവെക്കപ്പെട്ടതാകുന്നു അത്.'' (ക്വുര്‍ആന്‍ 2 : 23-24)

പരിശുദ്ധ ക്വുര്‍ആനിന് ക്രൈസ്തവസ്രോതസ്സുകള്‍ ആരോപിക്കുവാനുള്ള അടിസ്ഥാനന്യായമായി ഈ ദിശയില്‍ സംസാരിച്ചിട്ടുള്ള വിമര്‍ശകരെല്ലാം ചൂണ്ടിക്കാണിച്ചിട്ടുള്ളത് ബൈബിള്‍ ചരിത്രത്തില്‍ നിന്നുള്ള ചില ഭാഗങ്ങള്‍ പരിശുദ്ധ ക്വുര്‍ആനിലും കടന്നുവരുന്നുവെന്നതാണ്. ദിവ്യവെളിപാടുകളെയും പ്രവാചകന്‍മാരെയും വേദഗ്രന്ഥങ്ങളെയും കുറിച്ച് സാമാന്യധാരണയെങ്കിലുമുള്ള മുഴുവനാളുകള്‍ക്കും ഈ പ്രതിരോധം അങ്ങേയറ്റം ദുര്‍ബലവും പരിഹാസ്യവുമാണെന്ന് വളരെയെളുപ്പത്തില്‍ മനസ്സിലാകും. ആദം മുതല്‍ യേശു വരെയുള്ള പൂര്‍വപ്രവാചകന്‍മാരുടെ പിന്‍ഗാമിയായി, അവരെയെല്ലാം നിയോഗിച്ച പടച്ചതമ്പുരാനില്‍ നിന്നുള്ള ദിവ്യവെളിപാടുകള്‍ പരിശുദ്ധ ക്വുര്‍ആനിന്റെ രൂപത്തില്‍ ഏറ്റുവാങ്ങിക്കൊണ്ടാണ് മുഹമ്മദ് നബി (സ) എന്ന അന്തിമ പ്രവാചകന്‍ ലോകത്ത് നിയുക്തനായത്. ഒരേ ദൈവത്തില്‍ നിന്ന് വ്യത്യസ്ത പ്രവാചകന്‍മാര്‍ക്ക് ലഭിച്ച ഉപദേശങ്ങളില്‍ സമാനതകള്‍ കാണുന്നതില്‍ എന്താണത്ഭുതം? അന്തിമ പ്രവാചകന് ലഭിച്ച വേദഗ്രന്ഥത്തില്‍ പൂര്‍വപ്രവാചകന്‍മാരെയും പൂര്‍വവേദങ്ങളെയും കുറിച്ച വിവരണങ്ങള്‍ കടന്നുവരുന്നതില്‍ വിചിത്രമായി എന്തുണ്ടെന്നാണ് മിഷനറിമാര്‍ കരുതുന്നത്? ബൈബിള്‍ പ്രവാചകന്‍മാരെക്കുറിച്ച് പൂര്‍ണമായ മൗനം പാലിച്ചുകൊണ്ടോ അവരെ നിഷേധിച്ചുകൊണ്ടോ ആണ് ക്വുര്‍ആന്‍ അവതരിച്ചിരുന്നത് എങ്കിലല്ലേ മിഷനറിമാര്‍ ക്വുര്‍ആനിന്റെ ദൈവികതയെ സംശയിക്കേണ്ടത്?

ബൈബിള്‍ പ്രവാചകന്‍മാരെ സംബന്ധിച്ച സത്യസന്ധമായ വിവരണങ്ങള്‍ നല്‍കുകയും അവരെ സംബന്ധിച്ച് ക്രൈസ്തവസമൂഹത്തില്‍ പ്രചരിച്ചിരുന്ന വ്യാജവാര്‍ത്തകള്‍ നിഷേധിക്കുകയും ചെയ്തുകൊണ്ട് പൂര്‍വപ്രവാചകന്‍മാരെ സംബന്ധിച്ച ആശയക്കുഴപ്പങ്ങള്‍ക്ക് ദൈവികമായ വിരാമം കുറിക്കുകയാണ് പരിശുദ്ധ ക്വുര്‍ആന്‍ അതിന്റെ പ്രവാചകകഥനങ്ങളിലൂടെ ചെയ്യുന്നത്. പ്രസ്തുതകഥനങ്ങളില്‍ അബദ്ധങ്ങളുണ്ടെന്ന് സ്ഥാപിച്ചുകൊണ്ടു മാത്രമേ ക്വുര്‍ആനിന്റെ ദൈവികതയെ നിരാകരിക്കുവാന്‍ കഴിയൂ. അവയുടെ സാന്നിദ്ധ്യം, ക്വുര്‍ആനിന്റെ ദൈവികതയെ അരക്കിട്ടുറപ്പിക്കുക മാത്രമാണ് ചെയ്യുക എന്നതത്രെ യാഥാര്‍ത്ഥ്യം. പൂര്‍വ വേദങ്ങളുമായുള്ള ബന്ധം വിശദീകരിക്കവെ പരിശുദ്ധ ക്വുര്‍ആന്‍ തന്നെ അത് ഈ രംഗത്ത് നിര്‍വഹിക്കുന്ന ദൗത്യത്തെ കൃത്യമായി നിര്‍വചിക്കുന്നുണ്ട്: ''(നബിയേ,) നിനക്കിതാ സത്യപ്രകാരം വേദഗ്രന്ഥം അവതരിപ്പിച്ച് തന്നിരിക്കുന്നു. അതിന്റെ മുമ്പുള്ള വേദഗ്രന്ഥങ്ങളെ ശരിവെക്കുന്നതും അവയെ കാത്തുരക്ഷിക്കുന്നതുമത്രെ അത്. അതിനാല്‍ നീ അവര്‍ക്കിടയില്‍ നാം അവതരിപ്പിച്ച് തന്നതനുസരിച്ച് വിധികല്‍പിക്കുക. നിനക്ക് വന്നുകിട്ടിയ സത്യത്തെ വിട്ട് നീ അവരുടെ തന്നിഷ്ടങ്ങളെ പിന്‍പറ്റിപോകരുത്. നിങ്ങളില്‍ ഓരോ വിഭാഗത്തിനും ഓരോ നിയമക്രമവും കര്‍മ്മമാര്‍ഗവും നാം നിശ്ചയിച്ച് തന്നിരിക്കുന്നു. അല്ലാഹു ഉദ്ദേശിച്ചിരുന്നെങ്കില്‍ നിങ്ങളെ അവന്‍ ഒരൊറ്റ സമുദായമാക്കുമായിരുന്നു. പക്ഷെ നിങ്ങള്‍ക്കവന്‍ നല്‍കിയിട്ടുള്ളതില്‍ നിങ്ങളെ പരീക്ഷിക്കുവാന്‍ (അവന്‍ ഉദ്ദേശിക്കുന്നു). അതിനാല്‍ നല്ല കാര്യങ്ങളിലേക്ക് നിങ്ങള്‍ മത്സരിച്ച് മുന്നേറുക. അല്ലാഹുവിങ്കലേക്കത്രെ നിങ്ങളുടെയെല്ലാം മടക്കം. നിങ്ങള്‍ ഭിന്നിച്ചിരുന്ന വിഷയങ്ങളെപ്പറ്റി അപ്പോഴവന്‍ നിങ്ങള്‍ക്ക് അറിയിച്ച് തരുന്നതാണ്.'' (5 : 48)

യഥാര്‍ത്ഥത്തില്‍ ഒരു പ്രവാചകന്‍ തനിക്കുമുമ്പുവന്ന പ്രവാചകന്‍മാരുടെ കഥ പറയുകയും അവരെ ഉദ്ധരിക്കുകയും അവരെ സംബന്ധിച്ച തെറ്റിദ്ധാരണകളെ തിരുത്തുകയുമെല്ലാം ചെയ്യുന്നത് തികച്ചും സ്വാഭാവികമാണെന്നും അദ്ദേഹത്തിന് ലഭിക്കുന്ന ദിവ്യവെളിപാടുകളുടെ പ്രധാനപ്പെട്ടൊരു ഭാഗം ഇത്തരം കാര്യങ്ങളായിരിക്കുമെന്നും ബൈബിള്‍ പണ്ഡിതന്‍മാര്‍ക്ക് ആരും പറഞ്ഞുകൊടുക്കേണ്ടതില്ല. അത്തരം ഒരു നൈരന്തര്യത്തിന്റെ വിശദമായ വിവരണമാണ് ബൈബിളിന്റെ മുഖ്യപ്രമേയങ്ങളിലൊന്നുതന്നെ. ഇസ്രയേല്‍ തറവാട്ടിലേക്ക് കടന്നുവന്ന ഓരോ പ്രവാചകനും തനിക്കുമുമ്പുള്ള പ്രവാചകന്‍മാരെയും പുസ്തകങ്ങളെയും പരാമര്‍ശിച്ചത് അവരൊന്നും പ്രവാചകന്‍മാരല്ലെന്നും പൂര്‍വിക പ്രവാചകന്‍മാരുടെ പ്രബോധനങ്ങള്‍ പഠിച്ചുമനസ്സിലാക്കി വ്യാജ പ്രവാചകത്വം അവകാശപ്പെട്ടവരാണെന്നുമുള്ളതിന്റെ തെളിവാണെന്ന് വിവേകമുള്ള ആരെങ്കിലും പറയുമോ? യേശുവിന്റെ സംസാരങ്ങള്‍ ദിവ്യപ്രചോദിതമല്ലെന്നും പഴയനിയമ പാഠത്തില്‍ നിന്ന് അദ്ദേഹം സ്വന്തമായി രൂപീകരിച്ചതാണെന്നും വാദിക്കുന്നവരോട് മിഷനറിമാരുടെ നിലപാടെന്തായിരിക്കും? ആ വാദത്തില്‍ നിന്ന് ഒരു നിലക്കും ഭിന്നമല്ല പരിശുദ്ധ ക്വുര്‍ആനിലെ സെമിറ്റിക് പ്രവാചകചരിത്രത്തെ മാത്രം ആധാരമാക്കിക്കൊണ്ടുള്ള മുഹമ്മദ് നബി(സ)യുടെ പ്രവാചകത്വനിഷേധമെന്നു മനസ്സിലാക്കുവാന്‍ സെമിനാരി വിദ്യാഭ്യാസത്തിന്റെ പോലും ആവശ്യമില്ലെന്നതല്ലേ സത്യം?

ബൈബിള്‍ പ്രവാചകന്‍മാരില്‍ മിക്കവരും ജീവിച്ചത് പ്രവാചകന്‍മാര്‍ നിരന്തരമായി കടന്നുവന്ന ഇസ്രാഈല്യര്‍ക്കു മധ്യത്തിലാണ്. മുമ്പുകടന്നുവന്ന പ്രവാചകന്‍മാരുടെ ഉപദേശങ്ങളും പുസ്തകങ്ങളും അവരെ സംബന്ധിച്ച പാരമ്പര്യങ്ങളുമെല്ലാം സമൂഹത്തിലെ ഏതാണ്ടെല്ലാവര്‍ക്കും ചിരപരിചിതമായിരുന്ന സാഹചര്യങ്ങളിലേക്കാണ് ഓരോ പുതിയ ബൈബിള്‍ പ്രവാചകനും കടന്നുവരുന്നത്. 'വിവരങ്ങള്‍ കട്ടതാണെന്ന്' ആരോപിക്കേണ്ടവര്‍ക്ക് ആവശ്യമായ എല്ലാ 'പശ്ചാത്തല സൗകര്യ'ങ്ങളും അവരുടെ ജിവിതങ്ങള്‍ നല്‍കുന്നുണ്ടെന്ന് ചുരുക്കം. എന്നാല്‍ മുഹമ്മദ് നബി(സ)യുടെ ജീവിതപരിസരം ഇതില്‍നിന്നും തീര്‍ത്തുംവിഭിന്നമാണ്. ഇശ്മയേലിനുശേഷം പ്രവാചകന്‍മാരുടെ നിയോഗമൊന്നുമുണ്ടായിട്ടില്ലാത്ത, വേദഗ്രന്ഥങ്ങളൊന്നും കയ്യിലില്ലാതിരുന്ന, തികഞ്ഞ വിഗ്രഹാരാധകരും അധാര്‍മികരുമായി ജീവിച്ച മക്കന്‍ അറബികള്‍ക്കിടയില്‍ ജനിച്ചുജീവിച്ച മുഹമ്മദ് നബി(സ)യാണ് പൂര്‍വപ്രവാചകന്‍മാരെ സംബന്ധിച്ച വര്‍ത്തമാനങ്ങള്‍, അതും ബൈബിള്‍ വരുത്തിയ അബദ്ധങ്ങളില്‍ നിന്നുപോലും മുക്തമായി തന്റെ പ്രബോധിത സമൂഹത്തിനുമുന്നില്‍ വെക്കുന്നത്.

പ്രവാചകന്‍ സമ്പൂര്‍ണ നിരക്ഷരനായിരുന്നുവെന്നും മതപരമോ ഭൗതികമോ ആയിട്ടുള്ള യാതൊരു വിദ്യാഭ്യാസവും അദ്ദേഹം നേടിയിട്ടില്ലെന്നുമുള്ള വസ്തുതകള്‍ നാം ഇതിനോട് ചേര്‍ത്തുവായിക്കണം. പ്രവാചകന്റെ കിറുകൃത്യമായ പൂര്‍വപ്രവാചകാപഥനങ്ങള്‍ ദിവ്യവെളിപാടുകളുടെ വെളിച്ചത്തിലുള്ളതായിരിക്കുവാന്‍ മാത്രമേ തരമുള്ളുവെന്ന് ഈ സാഹചര്യത്തെളിവുകള്‍ മുഴുവന്‍ വ്യക്തമാക്കുന്നുണ്ട്. പ്രപഞ്ചനാഥനില്‍ നിന്നുള്ള വെളിപാടുകള്‍ ഇല്ലായിരുന്നെങ്കില്‍ ഇത്തരം വിഷയങ്ങളില്‍ പ്രവാചകന്‍ (സ) തികഞ്ഞ അജ്ഞനായിത്തന്നെ തുടരുമായിരുന്നുവെന്ന് ക്വുര്‍ആന്‍ തന്നെ എടുത്തുപറയുവാനുള്ള കാരണങ്ങള്‍ ഇതെല്ലമായിരിക്കാം. ക്വുര്‍ആന്‍ പറയുന്നത് കാണുക : ''അലിഫ്-ലാം-റാ. സ്പഷ്ടമായ വേദഗ്രന്ഥത്തിലെ വചനങ്ങളാകുന്നു അവ. നിങ്ങള്‍ ഗ്രഹിക്കുന്നതിന് വേണ്ടി അത് അറബിഭാഷയില്‍ വായിക്കപ്പെടുന്ന ഒരു പ്രമാണമായി അവതരിപ്പിച്ചിരിക്കുന്നു. നിനക്ക് ഈ ഖുര്‍ആന്‍ ബോധനം നല്‍കിയത് വഴി ഏറ്റവും നല്ല ചരിത്രവിവരണമാണ് നാം നിനക്ക് നല്‍കിക്കൊണ്ടിരിക്കുന്നത്. തീര്‍ച്ചയായും ഇതിനുമുമ്പ് നീ അതിനെപ്പറ്റി ബോധമില്ലാത്തവനായിരുന്നു.'' (12 : 1-3)

ക്വുര്‍ആനിലെ പൂര്‍വപ്രവാചക വിവരണങ്ങള്‍ ഏതു ദൈവിക ഗ്രന്ഥത്തിന്റെയും സ്വാഭാവികത മാത്രമാണെന്നും സെമിറ്റിക് സാമൂഹിക പശ്ചാത്തലത്തിലല്ലാതെ വളര്‍ന്നുവന്ന മുഹമ്മദ് നബി(സ)ക്ക് പൂര്‍വപ്രവാചകന്‍മാരെ സംബന്ധിച്ച കൃത്യമായ അറിവുകള്‍ പ്രപഞ്ചനാഥനില്‍ നിന്നാണ് ലഭിച്ചതെന്നും സെമിറ്റിക് സമ്പര്‍ക്കങ്ങള്‍ കൊണ്ടുപോലും സ്വരൂപിക്കാനാവാത്തവിധം അന്യൂനമായ സെമിറ്റിക് പ്രവാചക കഥനമാണ് പരിശുദ്ധ ക്വുര്‍ആന്‍ നടത്തുന്നതെന്നും നാം മനസ്സിലാക്കി. ഇനി ഓറിയന്റലിസ്റ്റുകളുടെയും മിഷനറിമാരുടെയും 'സിറിയന്‍ സിദ്ധാന്ത'ത്തിലേക്കുവരാം. ഒരു സിറിയന്‍ കഥക്കും വഴങ്ങാത്തവിധം സുഭദ്രമാണ് ക്വുര്‍ആനിലെ ചരിത്രാഖ്യാനത്തിന്റെ കെട്ടുറപ്പും കൃത്യതയുമെന്നതുകൊണ്ടുതന്നെ, ഒരു വൈജ്ഞാനികാന്വേഷണം എന്ന നിലക്കുമാത്രമാണ് അവയെക്കുറിച്ചുള്ള അപഗ്രഥനം പ്രസക്തമാകുന്നത്. മുഹമ്മദ് നബി (സ) ജീവിച്ച മക്കയില്‍ ക്രൈസ്തവ സമൂഹത്തിന്റെയോ ജൂത സമൂഹത്തിന്റെയോ സാന്നിദ്ധ്യമുണ്ടായിട്ടില്ലെന്ന കാര്യം ചരിത്രകാരന്‍മാര്‍ക്കിടയില്‍ സുവിദിതമാണ്. ഹിജാസിന്റെ തെക്ക് യമനിലും വടക്ക് ശാമിലും (ഇന്നത്തെ സിറിയന്‍, ഫലസ്ത്വീന്‍ പ്രവിശ്യകള്‍) ചെങ്കടലിനക്കരെ ആഫ്രിക്കന്‍ ഉപഭൂഖണ്ഡത്തില്‍ എത്യോപ്യയിലുമാണ് പ്രവാചകകാലഘട്ടത്തില്‍ പ്രധാനമായും ക്രൈസ്തവരുടെ സാമൂഹികസാന്നിദ്ധ്യമുണ്ടായിരുന്നത്. ഇതില്‍ സിറയയില്‍ നിന്നുള്ള ചില ക്രൈസ്തവ പണ്ഡിതന്‍മാര്‍ പ്രവാചകന് ബൈബിള്‍ കഥകളില്‍ 'ട്യൂഷന്‍' നല്‍കിയതായാണ് ഓറിയന്റലിസ്റ്റുകളും മിഷനറിമാരും വാദിക്കുന്നത്. ചരിത്രപരമായ ഒരു രേഖയുമില്ലാത്ത തികഞ്ഞ ഒരു അപസര്‍പ്പകകഥയാണ് ഇതെന്ന വസ്തുത അവര്‍ സമര്‍ത്ഥമായി മറച്ചുവെക്കുകയും ചെയ്യും!

ഹിജാസില്‍ വന്ന് മതപ്രബോധനത്തിനായി തമ്പടിച്ച ചില ക്രൈസ്തവ പണ്ഡിതന്‍മാരുടെ ശിഷ്യത്വം സ്വീകരിച്ച് പ്രവാചകന്‍ കാലം കഴിച്ചതായുള്ള കള്ളക്കഥകളെഴുതിവെച്ച മധ്യകാല ക്രൈസ്തവപുരോഹിതന്‍മാര്‍ മുതല്‍ പ്രവാചകന്‍ നുബുവ്വത്തിന് മുമ്പ് ബൈസന്റൈന്‍ റോമിന്റെ സിറിയന്‍ പ്രവിശ്യകളില്‍ കാലങ്ങളോളം ആത്മീയാന്വേഷണത്തിനായി അലഞ്ഞുതിരിഞ്ഞതായും മധ്യധാരണാഴിയുടെ തീരം വരെ ചെന്നെത്തിയതായും ചിലപ്പോഴൊക്ക മധ്യധാരണാഴിയില്‍ കപ്പല്‍ യാത്ര വരെ നടത്തിയതായുമുള്ള വന്യമായ ഭാവനകളെ 'ചരിത്ര'മായി പ്രതിഷ്ഠിക്കുവാന്‍ ശ്രമിച്ച കൊളോണിയല്‍കാല ഓറിയന്റലിസ്റ്റുകള്‍ വരെ ഈ അപസര്‍പ്പക കഥാകാരന്‍മാരുടെ നീണ്ടനിരയിലുണ്ട്! മുഹമ്മദ് നബി (സ) കള്ള പ്രവാചകനാണെന്ന തങ്ങളുടെ വാദത്തെ പരിശുദ്ധ ക്വുര്‍ആനിന്റെ ചരിത്രാഖ്യാനത്തിന്റെ കൃത്യത കടപുഴക്കുന്നതായി ബോധ്യപ്പെട്ട ഓറിയന്റലിസ്റ്റ്-മിഷനറി കൂട്ടുകെട്ട്, പ്രസ്തുത കൃത്യതക്ക് വിശദീകരണം നല്‍കുവാന്‍ വേണ്ടി നടത്തിയ മസ്തിഷ്‌ക വ്യായാമങ്ങളുടെ സന്തതികളായിപ്പിറന്ന പെരുംകള്ളങ്ങള്‍ മാത്രമാണ് പ്രവാചകജീവിതത്തിലെ ഈ വ്യാജ സിറിയന്‍ അധ്യായങ്ങള്‍ മുഴുവനുമെന്നതാണ് വസ്തുത.

മുഹമ്മദ് നബി(സ)യുടെ ജീവിതത്തില്‍ തങ്ങളുടെ വകയായി എഴുതിച്ചേര്‍ത്ത ഈ സംഭവങ്ങള്‍ക്ക് ആധാരമായ ചരിത്രരേഖകളേതൊക്കെയാണെന്ന് വ്യക്തമാക്കുവാന്‍ ഓറിയന്റലിസ്റ്റുകളൊന്നും സന്നദ്ധമായിട്ടില്ല. അക്കാദമിക, വൈജ്ഞാനികാന്വേഷണത്തിന്റെ കിരീടം അഭിമാനപൂര്‍വം തലയിലണിയുന്നവര്‍ ചരിത്രപരത ലവലേശവുമില്ലാത്ത കല്‍പിത കഥകളില്‍ അഭിരമിക്കുന്നുണ്ടെങ്കില്‍, ക്വുര്‍ആനിന്റെ ദൈവികത നിഷേധിക്കുവാന്‍ അവരെന്തു കടുംകയ്യും ചെയ്യും എന്ന് മനസ്സിലാക്കാനേ നമുക്ക് കഴിയൂ. സത്യസന്ധമായ മാര്‍ഗങ്ങളിലൂടെ ക്വുര്‍ആനിന്റെ ശോഭ കെടുത്താനാകില്ലെന്ന് തിരിച്ചറിഞ്ഞവര്‍ കല്ലുവെച്ച നുണകള്‍ ബോധപൂര്‍വം പറഞ്ഞ് അതിനെ തമസ്‌കരിക്കുവാന്‍ ശ്രമിക്കുമ്പോള്‍ ക്വുര്‍ആനിന്റെ അജയ്യതക്കും ഔജല്യത്തിനുമാണ് അടിവരയിടപ്പെടുന്നത് എന്നതാണ് സത്യം. ഒരാളെക്കുറിച്ച്, അദ്ദേഹത്തെ തോല്‍പിക്കുവാന്‍ വേണ്ടി, ഭാവനയില്‍ വരുന്ന എന്ത് കള്ളക്കഥയും പടച്ചുണ്ടാക്കുന്നതിന്റെ പേരാണ് ചരിത്രമെഴുത്തെന്ന് കരുതുന്നവരുടെ വ്യവഹാരങ്ങള്‍ അക്കാദമിക പഠനങ്ങളുടെ ഗതി നിശ്ചയിക്കുന്ന വൈചിത്ര്യത്തിനുമുന്നില്‍ പരിശുദ്ധ ക്വുര്‍ആന്‍ പറഞ്ഞതുമാത്രമാണ് നമുക്കും പറയാനുള്ളത്. ''അവരുടെ വായ്‌കൊണ്ട് അല്ലാഹുവിന്റെ പ്രകാശം കെടുത്തിക്കളയാമെന്ന് അവര്‍ ആഗ്രഹിക്കുന്നു. അല്ലാഹുവാകട്ടെ,തന്റെ പ്രകാശം പൂര്‍ണ്ണമാക്കാതെ സമ്മതിക്കുകയില്ല. സത്യനിഷേധികള്‍ക്ക് അത് അനിഷ്ടകരമായാലും. അവനാണ് സന്മാര്‍ഗവും സത്യമതവുമായി തന്റെ ദൂതനെ അയച്ചവന്‍. എല്ലാ മതത്തെയും അത് അതിജയിക്കുന്നതാക്കാന്‍ വേണ്ടി. ബഹുദൈവവിശ്വാസികള്‍ക്ക് അത് അനിഷ്ടകരമായാലും.'' (ക്വുര്‍ആന്‍ 9 : 32-33)

മുഹമ്മദ് നബി(സ)ക്ക് തന്റെ ജീവിതത്തില്‍ രണ്ട് സിറിയന്‍ യാത്രകള്‍ നടത്തിയതായി മാത്രമാണ് നിവേദനങ്ങളുള്ളത്. അതിലൊന്ന്, ഒന്‍പത് വയസ്സിലും പന്ത്രണ്ട് വയസ്സിനുമിടക്കെപ്പോഴോ അബൂത്വാലിബിന്റെ കൂടെ ഒരു കച്ചവടയാത്രയില്‍ മുഹമ്മദ് (സ) സിറിയയിലെ ബുസ്വ്‌റ വരെ പോയി എന്നു പറയുന്ന നിവേദനങ്ങളാണ്. ഇബ്‌നു ഇസ്ഹാഖ് അദ്ദേഹത്തിന്റെ സീറത്തു റസൂലില്ലയില്‍ ഈ കഥ പറയുന്നുണ്ട്. എന്നാല്‍ യാതൊരു നിവേദകപരമ്പരയും ഉദ്ധരിക്കാതെയാണ് അദ്ദേഹം കഥ എടുത്തുചേര്‍ത്തിട്ടുള്ളത് എന്നതിനാല്‍ തന്നെ നിവേദനങ്ങളുടെ നിദാനശാസ്ത്രമനുസരിച്ച് അദ്ദേഹത്തിന്റെ വര്‍ത്തമാനത്തെ ആധികാരികമായി പരിഗണിക്കുവാന്‍ യാതൊരു നിര്‍വാഹവുമില്ല. ഇമാം തുര്‍മുദി തന്റെ ജാമിഇലും ഇമാം ഹാകിം തന്റെ മുസ്തദ്‌റകിലും ഉദ്ധരിച്ചിട്ടുള്ള ഹദീഥുകളാണ് ഈ കഥക്കാധാരമായി പിന്നെയുള്ളത്. രണ്ട് ഹദീഥുകളുടെയും നിവേദകപരമ്പരകള്‍ അനേകം ദൗര്‍ബല്യങ്ങള്‍ ചൂണ്ടിക്കാണിക്കപ്പെട്ടിട്ടുള്ളതും അവ കാരണമായിത്തന്നെ പണ്ഡിതന്‍മാരാല്‍ സംശയാസ്പദമായി പരിഗണിക്കപ്പെട്ടിട്ടുള്ളവയുമാണ്. എന്നാല്‍ തുര്‍മുദി രേഖപ്പെടുത്തിയ ഹദീഥ് ചില പണ്ഡിതന്‍മാരെങ്കിലും ഹസന്‍ ആയോ സ്വഹീഹ് ആയോ എണ്ണിയിട്ടുണ്ടെന്ന കാര്യം ശരിയാണ്. ഹദീഥില്‍ പറയുന്ന സിറിയന്‍ യാത്രക്ക് സാക്ഷിയായിട്ടില്ലാത്ത അബൂമൂസല്‍ അശ്അരി (റ) എവിടെ നിന്നാണ് തനിക്കീ വിവരം കിട്ടിയത് എന്നുപറയാതെ പരാമൃഷ്ട കഥ പറയുന്നതാണ് ഹദീഥിലുള്ളത്. ഹദീഥിന്റെ നിവേദകപരമ്പരയെ വിമര്‍ശിച്ച പണ്ഡിതന്‍മാരുടെ അഭിപ്രായങ്ങളെ തള്ളിക്കളഞ്ഞാല്‍ പോലും, അബൂമൂസല്‍ അശ്അരി ഇവ്വിഷയകമായി കേട്ടകാര്യം എന്നുമാത്രമേ ഹദീഥിലെ കഥയെക്കുറിച്ച് പരമാവധി പറയാന്‍ പറ്റൂവെന്നര്‍ത്ഥം. അത്തരമൊരു 'കേള്‍വി' മാത്രമായതുകൊണ്ടുതന്നെ, ഒരുപാടു പൊരുത്തക്കേടുകളും അസംഭവ്യതകളും ഹദീഥിലെ കഥാകഥനത്തിലുള്ളതായി നിരൂപകര്‍ ചൂണ്ടിക്കാണിച്ചിട്ടുണ്ട്. ഈ സംശയാസ്പദമായ കേള്‍വി മാത്രമാണ് പ്രവാചകനെക്കുറിച്ച് പറയപ്പെടുന്ന ഒന്നാം സിറിയന്‍ യാത്രയെക്കുറിച്ച് ആകെക്കൂടി ചരിത്രത്തിലുള്ളത്.

ഇനി, എന്താണ് അബൂമൂസല്‍ അശ്അരി കേട്ടിട്ടുള്ളതെന്നുകൂടി നാം പരിശോധിക്കുക. അബൂത്വാലിബും സംഘവും സാധാരണയായി മക്കയില്‍ നിന്നുള്ള കച്ചവടസംഘങ്ങള്‍ കടന്നുപോകാറുള്ള ഒരു ക്രൈസ്തവമഠത്തിനുമുന്നിലെത്തിയപ്പോള്‍ അസാധാരണമാംവിധം അവിടുത്തെ മുഖ്യപുരോഹിതന്‍ പുറത്തേക്കിറങ്ങിവന്നുവെന്നും ബാലനായ മുഹമ്മദ് നബി(സ)യില്‍ ഭാവി പ്രവാചകന്റെ അടയാളങ്ങള്‍ ദര്‍ശിച്ചുവെന്നും അബൂത്വാലിബിനോട് ഈ കുട്ടിയെ റോമിലേക്ക് കൂട്ടുന്നത് അപകടമായിരിക്കുമെന്ന് പറയുകയും തിരിച്ചയക്കാന്‍ നിര്‍ബന്ധിക്കുകയും ചെയ്തുവെന്നും മുഹമ്മദ് നബി (സ) കച്ചവടസംഘത്തില്‍ നിന്ന് വേര്‍പിരിഞ്ഞ് അബൂത്വാലിബ് കൂടെ പറഞ്ഞയച്ച രണ്ട് പേരോടൊപ്പം അവിടെനിന്ന് മക്കയിലേക്ക് മടങ്ങിയെന്നും മാത്രമാണ് ഹദീഥിലുള്ളത്. തിരെ ചെറിയ പ്രായത്തില്‍ വളരെ കുറഞ്ഞൊരു സമയം ഒരു പുരോഹിതനെ കണ്ടുവെന്ന് മാത്രമാണ് ഹദീഥ് -അതില്‍ പറഞ്ഞ കാര്യങ്ങള്‍ ശരിയാണെങ്കില്‍- ആകെക്കൂടി നല്‍കുന്ന വിവരം. പ്രവാചകന് ക്രൈസ്തവപുരോഹിതന്‍മാര്‍ മതാധ്യാപനങ്ങള്‍ നല്‍കി എന്ന് ഹദീഥില്‍ എവിടെയാണുള്ളത്? സമൂഹത്തില്‍ ജീവിച്ച ഒരു വ്യക്തി എന്ന നിലക്ക് പ്രവാചകന്‍ പലരെയും കാണാനിടവന്ന നിലയില്‍ ഒരു ക്രൈസ്തവപുരോഹിതനെയും കണ്ടുമുട്ടി എന്ന, പ്രവാചകജീവിതത്തെ സംബന്ധിച്ച് ഒരു നിലക്കും പ്രസക്തമല്ലാത്ത ഒരറിവു മാത്രമാണ് ഹദീഥ് പങ്കുവെക്കുന്നതെന്ന് ചരുക്കും.

അതോടൊപ്പം, ഒരു ക്രൈസ്തവപുരോഹിതന്‍ ബൈബളില്‍ നിന്ന് ലഭിച്ച സൂചനകളുടെ വെളിച്ചത്തില്‍ മുഹമ്മദിനെ (സ) ചെറു്രപായത്തില്‍ തന്നെ ഭാവിപ്രവാചകനായി തിരിച്ചറിഞ്ഞു എന്ന അറിവുകൂടി ഹദീഥ് അവശേഷിപ്പിക്കുന്നു. വാസ്തവത്തില്‍, പ്രവാചകജീവിതവുമായി ബന്ധപ്പെട്ട് പരാമൃഷ്ട ഹദീഥിലുള്ള ഏറ്റവും പ്രസക്തമായ അറിവ് അതാണ്. മുഹമ്മദ് നബി (സ) ക്രൈസ്തവ പുരോഹിതനെ ഗുരുവായി സ്വീകരിച്ചുവെന്നല്ല, പ്രത്യുത മുഹമ്മദ് നബി(സ)യെ ലോകഗുരുവായി പുരോഹിതന്‍ അംഗീകരിച്ചുവെന്നാണ് ഹദീഥിലുള്ളത്. പ്രവാചകബാല്യത്തില്‍ നടന്നുവെന്ന് പറയപ്പെടുന്ന സിറിയന്‍ യാത്രയെ സംബന്ധിച്ച എല്ലാ നിവേദനങ്ങളും ഇക്കാര്യം ഐകകണ്‌ഠേന പറയുന്നുണ്ട്. ആ നിവേദനങ്ങള്‍ സ്വീകാര്യമാണെന്നണ് ഓറിയന്റലിസ്റ്റുകളുടെ പക്ഷമെങ്കില്‍, അവര്‍ പറയാനുദ്ദേശിക്കുന്നതിന് നേര്‍വിപരീതമായ കഥയാണ് അവ ഉല്‍പാദിപ്പിക്കുക എന്നുചുരുക്കം. സിറിയയില്‍വെച്ച് കണ്ടുമുട്ടിയ ക്രൈസ്തവ പാതിരിയുടെ പാത പിന്തുടര്‍ന്ന് പ്രവാചകനെ അംഗീകരിക്കുവാന്‍ മിഷനറിമാര്‍ സന്മനസ്സ് കാണിക്കുമോ? ബൈബിളില്‍ നബിയെക്കുറിച്ച പ്രവചനങ്ങളുണ്ടെന്ന വസ്തുതയെ അവര്‍ ഉള്‍ക്കൊള്ളുമോ ?

ഖദീജ(റ)യുമായുള്ള വിവാഹത്തിന് തൊട്ടുമുമ്പ് അവരുടെ മയ്‌സറ എന്ന ഭൃത്യന്റെ കൂടെ ഖദീജയുടെ കച്ചവടവസ്തുക്കളുമായി മുഹമ്മദ് നബി (സ) ശാമിലേക്ക് ഒരു യാത്രപോയതാണ് ചരിത്രഗ്രന്ഥങ്ങളിലുള്ള രണ്ടാമത്തെ സംഭവം. ഈ സംഭവം കുറേക്കൂടി പ്രശസ്തവും ചരിത്രകാരന്‍മാര്‍ അംഗീകരിക്കുന്നതുമാണ്; വിശദാംശങ്ങളെക്കുറിച്ച് അഭിപ്രായവ്യത്യാസങ്ങളുണ്ടെങ്കിലും. എന്നാല്‍ എല്ലാ അഭിപ്രായവ്യത്യാസങ്ങളെയും തള്ളിക്കളഞ്ഞ് മുഴുവന്‍ ചരിത്രകാരന്‍മാരുടെയും നിവേദനങ്ങളെ ആധികാരികമായി നാം സ്വീകരിച്ചാലും ആ യാത്രയില്‍ ഇവ്വിഷയകമായുണ്ടായത് ഒരു ക്രൈസ്തവപുരോഹിതനും ഒരു ജൂതവ്യാപാരിയും ഭാവി പ്രവാചകന്റെ അടയാളങ്ങള്‍ നബി(സ)യില്‍ ദര്‍ശിച്ചുവെന്നതു മാത്രമാണ്. ഒന്നാം സിറിയന്‍ യാത്രയെക്കുറിച്ചു പറഞ്ഞതുപോലെ, രണ്ടാം സിറിയന്‍ യാത്രയിലും ഓറിയന്റലിസ്റ്റുകള്‍ക്കാവശ്യമുള്ള യാതൊന്നും ഉണ്ടായതായി നിവേദനങ്ങള്‍ വ്യക്തമാക്കുന്നില്ലെന്ന് ചുരുക്കം. ചരിത്രത്തില്‍ നിന്ന് എത്ര വിദൂരമായാണ് ഓറിയന്റലിസ്റ്റുകളുടെയും മിഷനറിമാരുടെയും നബിവിദ്വേഷത്തില്‍ നിന്നുല്‍ഭൂതമാകുന്ന കാല്‍പനിക ഭാവനകള്‍ മേഞ്ഞു നടക്കുന്നതെന്നാലോചിച്ചുനോക്കൂ!

മുഹമ്മദ് നബി(സ)ക്ക് ബൈബിള്‍ പ്രവാചകന്‍മാരെക്കുറിച്ച് സിറിയന്‍ യാത്രകളില്‍ നിന്നുലഭിച്ച അറിവുകളാണ് ക്വുര്‍ആനിലുള്ളതെന്ന വിമര്‍ശക വിശദീകരണത്തിന്റെ പരിഹാസ്യത മനസ്സിലാക്കാന്‍ ക്വുര്‍ആനിലും ഹദീഥുകളിലും ചരിത്രപുസ്തകങ്ങളിലും സവിസ്തരം രേഖപ്പെടുത്തപ്പെട്ടിട്ടുള്ള, പ്രവാചകനും (സ) മക്കയിലെ ബഹുദൈവാരാധാകരും തമ്മില്‍ നടന്നിട്ടുള്ള സംവാദങ്ങളുടെ വിശദാംശങ്ങള്‍ മാത്രം പരിശോധിച്ചാല്‍ മതി എന്നതാണ് വാസ്തവം. രണ്ട് സിറിയന്‍ യാത്രകളില്‍ നിന്ന് സമാഹരിച്ചതാണ് ക്വുര്‍ആനിലെ പല വിവരങ്ങളുമെന്ന ആരോപണം, ക്വുര്‍ആനിനെതിരെ നൂറുകണക്കിന് വിമര്‍ശനങ്ങളുന്നയിച്ചിട്ടും ഒരു മക്കന്‍ ബഹുദൈവാരാധകനും ഒരിക്കല്‍പോലും ഉന്നയിച്ചിട്ടില്ലെന്ന യാഥാര്‍ത്ഥ്യം, 'സിറിയന്‍ സമ്പര്‍ക്കം' പ്രവാചകന്റെ വിയോഗത്തിനുശേഷം ആരുടെെയാക്കെയോ ഭാവനയില്‍ ഉരുത്തിരിഞ്ഞ 'ഫിക്ഷന്‍' മാത്രമാണെന്ന് സുതരാം വ്യക്തമാക്കുന്നില്ലേ? കളവുകള്‍ കൊണ്ട് എത്രകാലം ലോകെത്ത വഞ്ചിക്കാമെന്നാണ് 'ഭാവനാസമ്പന്നരായ' ഓറിയന്റലിസ്റ്റ് 'കാല്‍പനികര്‍' കണക്കുകൂട്ടുന്നത്?

തെ. അവതരിപ്പിക്കപ്പെട്ട വിശുദ്ധിയില്‍ തന്നെ നില നിൽക്കുന്ന ഒരേയൊരു വേദഗ്രൻഥമാണ് ക്വുര്‍ആൻ. അങ്ങനെ സംരക്ഷിക്കുമെന്നത് അല്ലാഹുവിന്റെ വാഗ്ദാനമാണ്. ''തീര്‍ച്ചയായും നാമാണ് ആ ഉല്‍ബോധനം അവതരിപ്പിച്ചത്. തീര്‍ച്ചയായും നാം അതിനെ കാത്തുസൂക്ഷിക്കുന്നതാണ്'' (ക്വുര്‍ആന്‍ 15:9) എന്നാണ് അല്ലാഹു പറഞ്ഞിരിക്കുന്നത്. ഈ ദൈവിക വാഗ്ദാനം പൂര്‍ണമായും പാലിക്കപ്പെട്ടിട്ടുണ്ടെന്ന് ലോകമെങ്ങും പ്രചാരത്തിലിരിക്കുന്ന ക്വുര്‍ആന്‍ കോപ്പികളും ക്വുര്‍ആന്‍ മനഃപാഠമാക്കിയിട്ടുള്ള ആയിരക്കണക്കിനു ഹാഫിളുകളും വ്യക്തമാക്കുന്നു. പ്രവാചകന്റെ കാലം മുതല്‍ ഇന്നുവരെ നൈരന്തര്യത്തോടെ നടന്നുവന്നിട്ടുള്ളതാണ് ക്വുര്‍ആന്‍ രേഖീകരിക്കുകയും ഹൃദിസ്ഥമാക്കുകയും ചെയ്യുന്ന പ്രക്രിയകളെന്ന വസ്തുതയ്ക്ക് ചരിത്രം നല്‍കുന്ന തെളിവുകള്‍ ഇതിനെ ദൃഢീകരിക്കുകയും ചെയ്യുന്നു.

കവിതകളും കഥാഖ്യാനങ്ങളും തലമുറകളിലേക്ക് സംപ്രേഷണം ചെയ്യപ്പെടുന്ന മനഃപാഠത്തിലൂടെയും വൈജ്ഞാനിക സാഹിത്യങ്ങള്‍ അടുത്ത തലമുറയ്ക്ക് ലഭിക്കുന്ന രേഖീകരണത്തിലൂടെയും ഒരേപോലെ സംരക്ഷിക്കപ്പെട്ട ഗ്രന്ഥമാണ് ക്വുര്‍ആന്‍. ഒന്നും കടത്തിക്കൂട്ടുകയോ എടുത്തൊഴിവാക്കുകയോ ചെയ്യാനാവാത്തവിധം പതിനാലു നൂറ്റാണ്ടുകളായി തലമുറകളില്‍ നിന്ന് തലമുറകളിലേക്ക് ഈ രണ്ടു രീതികളിലും ക്വുര്‍ആന്‍ പകര്‍ത്തപ്പെട്ടുകൊണ്ടിരിക്കുന്നുണ്ട്. വാക്യങ്ങള്‍ കൂട്ടിച്ചേര്‍ക്കുകയോ എടുത്തുമാറ്റുകയോ ചെയ്തുവെന്ന് വസ്തുനിഷ്ഠ തെളിവുകളുടെ വെളിച്ചത്തില്‍ ഒരാള്‍ക്കും ആരോപിക്കപ്പെടാനാവാത്ത നിലയില്‍, ചരിത്രത്തിന്റെ പൂര്‍ണമായ വെളിച്ചത്തിലാണ് ഒന്നര സഹസ്രാബ്ദക്കാലമായി തലമുറകളില്‍ നിന്ന് തലമുറകളിലേക്കുള്ള ഈ സംപ്രേഷണം നടക്കുന്നത്.

അവതരിപ്പിക്കപ്പെടുന്ന മുറയ്ക്ക് മുഹമ്മദ് നബി (സ) ക്വുര്‍ആന്‍ ഹൃദിസ്ഥമാക്കിയിരുന്നുവെന്നും ഓരോവര്‍ഷവും റമദാനില്‍ ജിബ്‌രീല്‍ വന്ന് അവതരിപ്പിക്കപ്പെട്ടിടത്തോളമുള്ള വചനങ്ങള്‍ പാരായണം ചെയ്തു കേട്ട് ഉറപ്പുവരുത്തിയിരുന്നുവെന്നും ഇമാം ബുഖാരി തന്റെ സ്വഹീഹിലെ കിതാബു ഫദാഇലില്‍ ഫാത്വിമ(റ)യില്‍ നിന്ന് നിവേദനം ചെയ്യുന്നുണ്ട്.

ക്വുര്‍ആന്‍ മനഃപാഠമാക്കുവാന്‍ സ്വഹാബിമാരെ പ്രവാചകന്‍ (സ) പ്രോത്സാഹിപ്പിക്കുകയും ഹൃദിസ്ഥമാക്കിയ വചനങ്ങള്‍ മറന്നു പോകാനിടയാകരുതെന്ന് പ്രത്യേകം നിഷ്‌കര്‍ഷിക്കുകയും ചെയ്തിരുന്നതായി വ്യക്തമാക്കുന്ന നിരവധി ഹദീഥുകൾ ബുഖാരിയിലും മുസ്ലിമിലും മറ്റു ഹദീഥ് ഗ്രൻഥങ്ങളിലുമെല്ലാം നിവേദനം ചെയ്യപ്പെട്ടിട്ടുണ്ട്. മനഃപാഠമാക്കുന്നതില്‍ വിദഗ്ധരായിരുന്നു അറബികളെന്നതിനാലും പദ്യ-ഗദ്യ സമ്മിശ്രമായ ക്വുര്‍ആനിന്റെ ശൈലി കാണാതെ പഠിക്കുവാന്‍ എളുപ്പവും ഇമ്പവും പ്രധാനം ചെയ്യുന്നതിനാലും സ്വഹാബിമാര്‍ക്ക് ക്വുര്‍ആന്‍ പഠനം ഒരു പ്രയാസമായി അനുഭവപ്പെട്ടതേയില്ല. പരമാവധി മനഃപാഠമാക്കുവാന്‍ ഓരോ സ്വഹാബിയും പരിശ്രമിച്ചു. കൂടുതല്‍ ക്വുര്‍ആന്‍ പഠിച്ചവന്‍ താനാണെന്ന് അഭിമാനത്തോടെ പറയുന്നവരായിരുന്നു സ്വഹാബിമാരെന്ന് ഇബ്‌നു മസ്ഊദി(റ)ൽ നിന്ന് ബുഖാരി നിവേദനം ചെയ്ത ഹദീഥ് വ്യക്തമാക്കുന്നുണ്ട്.

പ്രവാചകന്റെ (സ) കാലത്ത്, മനഃപാഠമാക്കുന്നതോടൊപ്പം തന്നെ, ക്വുര്‍ആന്‍ രേഖീകരിക്കുന്ന പതിവുമുണ്ടായിരുന്നുവെന്ന് വ്യക്തമാക്കുന്ന നിരവധി ചരിത്രരേഖകളുണ്ട്. ഉമറിന്റെ (റ) ഇസ്‌ലാം സ്വീകരണത്തെക്കുറിച്ച ഇബ്‌നു ഇസ്ഹാഖിന്റെ വിവരണത്തില്‍, തന്റെ സഹോദരിയും ഭര്‍ത്താവും ഇസ്‌ലാം സ്വീകരിച്ചിട്ടുണ്ടെന്ന് മനസ്സിലാക്കി കോപാകുലനായി അവരുടെ വീട്ടിലേക്ക് അദ്ദേഹം കുതിച്ചെത്തിയപ്പോള്‍ അവിടെ അദ്ദേഹത്തിന്റെ സഹോദരിയും ഭര്‍ത്താവും ഖബ്ബാബിനോടൊപ്പമിരുന്ന് ഒരു ചര്‍മപഠത്തിലെഴുതിയ ക്വുര്‍ആനിലെ ത്വാഹാ സൂറത്ത് പാരായണം ചെയ്യുകയായിരുന്നുവെന്നും ഉമര്‍ (റ) പുറത്തു വന്നിട്ടുണ്ടെന്നറിഞ്ഞ സഹോദരി ഫാത്വിമഃ (റ) ആ കയ്യെഴുത്തുരേഖ അവരുടെ തുടയ്ക്ക് താഴെ ഒളിപ്പിച്ചുവെച്ചുവെന്നും പറയുന്നതില്‍ നിന്ന് അന്നുമുതല്‍ തന്നെ ക്വുര്‍ആന്‍ രേഖീകരിച്ചു സൂക്ഷിക്കുന്ന പതിവ് നിലനിന്നിരുന്നുവെന്ന് വ്യക്തമാകുന്നു. ഉമറിന്റെ ഇസ്‌ലാം സ്വീകരണം നടന്ന പ്രവാചകത്വത്തിന്റെ ആറാം വര്‍ഷത്തിനു മുമ്പുതന്നെ ക്വുര്‍ആന്‍ കയ്യെഴുത്ത് രേഖകളിലാക്കി സൂക്ഷിക്കുന്ന പതിവ് മുസ്‌ലിം സമൂഹത്തിലുണ്ടായിരുന്നുവെന്ന് വ്യക്തമാക്കുന്നതാണ് ഈ വിവരണം. മക്കയില്‍വെച്ച് അവതരിക്കപ്പെട്ട വചനങ്ങള്‍ മക്കയില്‍വെച്ചുതന്നെ രേഖപ്പെടുത്തിവെച്ചിരുന്നതായി അബ്ദുല്ലാഹിബ്‌നു അബ്ബാസ് (റ) പറഞ്ഞിട്ടുണ്ടെന്ന് മുഹമ്മദ്ബ്‌നു ശിഹാബ് അസ്സുഹ്‌രി സാക്ഷ്യപ്പെടുത്തിയതായി ഇമാം ഇബ്‌നു കഥീര്‍ രേഖപ്പെടുത്തുന്നുണ്ട്. (അല്‍ ബിദായ വ ന്നിഹായ, വാല്യം 5, പുറം 340)

അബ്ദില്ലാഹിബ്‌നു സഅ്ദ്ബ്‌നു അബീ സര്‍ഹിനെയായിരുന്നു നബി(സ)യില്‍ നിന്ന് ക്വുര്‍ആന്‍ കേള്‍ക്കാനും അത് അന്ന് ഉപലബ്ധമായ എഴുത്തുവസ്തുക്കളില്‍ എഴുതി രേഖപ്പെടുത്തുവാനുമായി മക്കയില്‍വെച്ച് നബി (സ) ഏല്‍പിച്ചത്. ഖാലിദ്ബ്‌നു സഈദ്ബ്‌നുല്‍ ആസ്വ് ആയിരുന്നു നബി(സ)യുടെ നിര്‍ദേശാനുസരണം ക്വുര്‍ആന്‍ രേഖപ്പെടുത്തിയിരുന്നു മറ്റൊരാള്‍. 'ക്വുര്‍ആന്‍ അല്ലാതെ മറ്റൊന്നും നിങ്ങള്‍ എന്നില്‍നിന്ന് എഴുതി സൂക്ഷിക്കരുത്'(സ്വഹീഹു മുസ്‌ലിം, കിതാബു സ്‌സുഹുദു വര്‍ റഖാഇഖ്) എന്ന പ്രവാചകനിര്‍ദേശത്തില്‍ നിന്ന് നിരവധി പേര്‍ ക്വുര്‍ആന്‍ രേഖപ്പെടുത്തിവെക്കാറുണ്ടെന്ന് വ്യക്തമാകുന്നുണ്ട്. മദീനയില്‍ നിന്നെത്തിയവരുമായി പ്രവാചകന്‍ (സ) അഖബയില്‍വെച്ചുണ്ടാക്കിയ ഉടമ്പടിയില്‍ പങ്കെടുത്ത റാഫിഉബ്‌നു മാലിക് അല്‍ അന്‍സ്വാരിക്ക് അതുവരെ അവതരിക്കപ്പെട്ട എല്ലാ ക്വുര്‍ആന്‍ വചനങ്ങളും രേഖപ്പെടുത്തിയ ഒരു കയ്യെഴുത്ത് രേഖ നല്‍കിയതായും തന്റെ നാട്ടിലെത്തിയശേഷം ഗോത്രത്തിലുള്ളവരെയെല്ലാം വിളിച്ചുകൂട്ടി അത് അദ്ദേഹം വായിച്ചു കേള്‍പ്പിച്ചതായും വ്യക്തമാക്കുന്ന രേഖകളുണ്ട്.

മദീനയില്‍ എത്തിയതോടെ പ്രവാചകന്‍ (സ) കൂടുതല്‍ അനുയായികള്‍ ഉണ്ടായിരുന്നതുകൊണ്ടുതന്നെ കൂടുതല്‍ ക്വുര്‍ആന്‍ എഴുത്തുകാരുമുണ്ടായി. അബ്ദുബ്‌നു സഈദ് അബൂ ഉമാമ, അബൂഅയ്യൂബല്‍ അന്‍സ്വാരി, അബൂബക്ര്‍ സിദ്ദീഖ്, അബൂഹുദൈഫ, അബൂസുഫ്‌യാന്‍, അബൂസലമ, അബൂ അബസ്, ഉബയ്യ്ബ്‌നു കഅബ്, അല്‍അര്‍ഖം, ഉസൈദ്ബ്‌നുല്‍ ഹുദൈര്‍, ഔസ്, ബുറൈദ, ബഷീര്‍, ഥാബിത്ബ്‌നു ഖൈസ്, ജഅ്ഫര്‍ബിന്‍ അബീത്വാലിബ്, ജഹ്മ്ബ്‌നു സഅദ്, ജുഹൈം, ഹാതിബ്, ഹുദൈഫ, ഹുസൈന്‍, ഹന്‍ദല, ഹുവൈതിബ്, ഖാലിദ്ബ്‌നു സഈദ്, ഖാലിദ്ബ്‌നു വലീദ്, അസ്സുബൈറ്ബ്‌നു അവ്വാം, സുബൈറ്ബ്‌നു അര്‍ഖം, സൈദ്ബ്‌നു ഥാബിത്, സഅ്ദ്ബ്‌നു റബീഅ്, സഅ്ദ്ബ്‌നു ഉബാദ, സഈദ്ബ്‌നു സഈദ്, കുറഹ്ബില്‍ ബിന്‍ ഹസ്‌ന, ത്വല്‍ഹ, ആമിര്‍ ബിന്‍ ഫുഹൈറ, അബ്ബാസ്, അബ്ദുല്ലാഹിബ്‌നുല്‍ അര്‍ഖം, അബ്ദുല്ലാഹിബ്‌നു അബീബക്ര്‍, അബ്ദുല്ലാഹിബ്‌നു റവാഹ, അബ്ദുല്ലാഹിബ്‌നു സൈദ്, അബ്ദുല്ലാഹിബ്‌നു സഅദ്, അബ്ദുല്ലാഹിബ്‌നു അബ്ദില്ല, അബ്ദുല്ലാഹിബ്‌നു അംറ്, ഉഥ്മാനുബ്‌നു അഫ്ഫാന്‍, ഉഖ്ബ, അല്‍ അലാഅ് അല്‍ ഹദ്‌റമി, അല്‍ അലാഅ്ബ്ന്‍ ഉഖ്ബ, അലിയ്യുബിന്‍ അബീത്വലിബ്, ഉമറുബ്‌നുല്‍ ഖത്വാബ്, അംറുബ്‌നുല്‍ ആസ്വ്, മുഹമ്മദ്ബ്‌നു മസ്‌ലമ, മുആദ്ബ്‌നു ജബല്‍, മുആവിയ, മഅ്‌നുബ്‌നു, അദിയ്യ്, മുഐഖിബ്, മുന്‍ദിര്‍, മുഹാജിര്‍, യസീദിബ്‌നു അബീസുഫ്‌യാന്‍ (റ) എന്നിങ്ങനെ വ്യത്യസ്ത സന്ദര്‍ഭങ്ങളിലായി പ്രവാചകനില്‍ (സ) നിന്ന് ക്വുര്‍ആന്‍ വചനങ്ങള്‍ കേട്ടെഴുതിയ അറുപത്തഞ്ച് അനുചരന്‍മാരുടെ പട്ടിക ഡോ. മുഹമ്മദ് മുസ്തഫ അല്‍ അഅ്ദ്വമി തന്റെ പഠനത്തില്‍ വിവരിക്കുന്നുണ്ട്.(M.M Al Azami, “The History Of The Quranic Text – From Revelation To Compilation, A Comparative Study with the Old and New Testaments, Leicester, 2003, Page 68.)

വഹ്‌യ് അവതരിക്കപ്പെടുന്ന സന്ദര്‍ഭങ്ങളില്‍ ഉടന്‍ തന്നെ പ്രാപ്തനായ ഒരു അനുചരനെ വിളിച്ച് അത് എഴുതിവെക്കാനാവശ്യപ്പെടുക മുഹമ്മദ് നബി(സ)യുടെ പതിവായിരുന്നു. പ്രവാചകന്റെ (സ) പള്ളിക്കടുത്ത് താമസിച്ചിരുന്നതിനാല്‍ സൈദ്ബ്‌നു ഥാബിത്തിന് (റ) പലപ്പോഴും പ്രവാചകനില്‍ (സ) നിന്ന് വഹ്‌യ് എഴുതിവെക്കുവാന്‍ കൂടുതല്‍ അവസരമുണ്ടായിരുന്നതായി അദ്ദേഹം തന്നെ അനുസ്മരിക്കുന്നുണ്ട്. എഴുതാന്‍ കഴിയാത്ത പ്രവാചകാനുചരന്‍മാര്‍ തോല്‍ച്ചുരുളുകളും ചര്‍മപടങ്ങളുമായി വന്ന് എഴുതാന്‍ കഴിയുന്നവരെക്കൊണ്ട് ക്വുര്‍ആന്‍ വചനങ്ങള്‍ എഴുതിപ്പിച്ച് വാങ്ങുന്ന പതിവുണ്ടായിരുന്നുവെന്ന് വ്യക്തമാക്കുന്ന നിവേദനങ്ങളുണ്ട്.

വിശുദ്ധ ക്വുര്‍ആനിന്റെ അവതരണവും സംരക്ഷണവും മാത്രമല്ല ക്രോഡീകരണവും വിശദീകരണവുമെല്ലാം സര്‍വശക്തനായ അല്ലാഹു തന്നെ നിര്‍വഹിക്കുമെന്ന് സ്വയം പ്രഖ്യാപിച്ചിട്ടുള്ളതാണ്. ''തീര്‍ച്ചയായും അതിന്റെ (ക്വുര്‍ആന്റെ) സമാഹരണവും അത് ഓതിത്തരലും നമ്മുടെ ബാധ്യതയാകുന്നു. അങ്ങനെ നാം അത് ഓതിത്തന്നാല്‍ ആ ഓത്ത് നീ പിന്തുടരുക. പിന്നീട് അത് വിവരിച്ചുതരലും നമ്മുടെ ബാധ്യതയാകുന്നു.'' (ക്വുര്‍ആന്‍ 75:17-19) വ്യത്യസ്ത സാഹചര്യങ്ങളില്‍ അവതരിപ്പിക്കപ്പെടുന്ന അല്ലാഹുവിന്റെ വചനങ്ങള്‍ ക്വുര്‍ആനിലെ ഏത് അധ്യായത്തില്‍ എത്രാമത്തെ വചനങ്ങളായാണ് രേഖപ്പെടുത്തേണ്ടതെന്നുകൂടി ദൈവിക ബോധനത്തിന്റെ അടിസ്ഥാനത്തില്‍ നബി (സ) തന്നെ തന്റെ എഴുത്തുകാര്‍ക്ക് പറഞ്ഞുകൊടുക്കുമായിരുന്നു. സൂറത്തുകളെയും അവയിലെ ആയത്തുകളെയും അവയുടെ സ്ഥാനത്തെയുമെല്ലാം കുറിച്ച് പ്രവാചകാനുചരന്‍മാര്‍ക്ക് കൃത്യമായ ബോധ്യമുണ്ടായിരുന്നുവെന്ന് വ്യക്തമാക്കുന്ന നിരവധി നിവേദനങ്ങളുണ്ട്. ഓരോ വചനവും അവതരിക്കപ്പെടുമ്പോള്‍ തന്റെ അനുയായികളായ ക്വുര്‍ആന്‍ എഴുത്തുകാരെ വിളിച്ച് അവ പാരായണം ചെയ്തു കേള്‍പ്പിക്കുകയും ഏത് അധ്യാത്തില്‍ എത്രാമത്തെ വചനമായാണ് അത് ചേര്‍ക്കേണ്ടതെന്ന് നിര്‍ദേശം നല്‍കുകയും ചെയ്തിരുന്നതായി നബി(സ)യുടെ ക്വുര്‍ആന്‍ എഴുത്തുകാരില്‍ പ്രധാനിയായ സൈദ്ബ്‌നു ഥാബിത് (റ) വ്യക്തമാക്കുന്നുണ്ട്.(ജാമിഉത്തിര്‍മിദി, കിതാബു തഫ്‌സീറില്‍ ക്വുര്‍ആന്‍, സുനനു അബീദാവൂദ്, കിതാബുസ്സ്വലാത്)

ക്വുര്‍ആനിലെ പതിനാറാമത്തെ അധ്യായമായ സൂറത്തുന്നഹ്‌ലിലെ തൊണ്ണൂറാമത്തെ വചനം പാരായണം ചെയ്തുകൊണ്ട് ജിബ്‌രീല്‍ ഇപ്പോള്‍ എന്റെയടുക്കല്‍ വന്ന് ഈ വചനം സൂറത്തുന്നഹ്‌ലില്‍ തൊണ്ണൂറാം സൂക്തമായി ചേര്‍ക്കണമെന്നു നിര്‍ദേശിച്ചതായി അപ്പോള്‍ പ്രവാചകനോടൊപ്പമുണ്ടായിരുന്ന ഉഥ്മാനുബ്‌നു അബില്‍ ആസ്വിനോട് പറഞ്ഞതായി മുസ്‌നദ് അഹ്മദ് നിവേദനം ചെയ്യുന്ന ഹദീഥ് ഓരോ സൂക്തങ്ങളും എവിടെ ചേര്‍ക്കണമെന്ന ദൈവിക നിര്‍ദേശമുണ്ടായിരുന്നുവെന്ന വസ്തുത വെളിപ്പെടുത്തുന്നതാണ്. 'സൂറത്തുനിന്നാഇലെ അവസാനത്തെ വചനങ്ങള്‍ താങ്കള്‍ക്ക് മതിയാവുന്നതാണ്' എന്ന് ഉമറി(റ)നോട് പ്രവാചകന്‍ (സ) പറഞ്ഞതില്‍നിന്നും.സ്വഹീഹു മുസ്‌ലിം, കിതാബുല്‍ ഫറാഇദ് "സൂറത്തുല്‍ ബക്വറഃയിലെ അവസാനത്തെ രണ്ടു വചനങ്ങള്‍ രാത്രിയില്‍ പാരായണം ചെയ്യുന്നവര്‍ക്ക് അത് മതിയാകുന്നതാണ്' എന്ന അബൂ മസ്ഊദ് അല്‍ബദ്‌രി (റ) നിവേദനം ചെയ്ത നബിവചനത്തില്‍ നിന്നും(സ്വഹീഹുല്‍ ബുഖാരി, കിതാബു ഫദാഇലില്‍ ക്വുര്‍ആന്‍ എന്റെ അമ്മായിയായ മൈമൂന(റ)യുടെ വീട്ടില്‍ താമസിക്കുമ്പോള്‍ പ്രവാചകന്‍ (സ) രാത്രി ഉറക്കത്തില്‍ നിന്നെണീറ്റ് സൂറത്തു ആലുംറാനിലെ അവസാനത്തെ പത്തു വചനങ്ങള്‍ പാരായണം ചെയ്യുന്നതായി ഞാന്‍ കേട്ടുവെന്ന ഇബ്‌നു അബ്ബാസിന്റെ അനുഭവവിവരണത്തില്‍ നിന്നും(സ്വഹീഹു മുസ്‌ലിം, കിതാബുല്‍ വിദ്വൂഅ്, സ്വഹീഹു മുസ്‌ലിം, കിതാബുല്‍ മുസാഫിരീന്‍) വ്യക്തമാവുന്നത് ഏതെല്ലാം അധ്യായങ്ങളില്‍ എത്രാമത്തെ സൂക്തമാണ് ഓരോ ക്വുര്‍ആന്‍ സൂക്തങ്ങളുമെന്ന് സ്വഹാബിമാര്‍ക്കെല്ലാം കൃത്യമായി അറിയാമായിരുന്നുവെന്നാണ്.

മദീനാ ഇസ്‌ലാമിക രാഷ്ട്രത്തില്‍ ക്വുര്‍ആന്‍ മനഃപാഠമുള്ളവര്‍ ധാരാളമായി ഉണ്ടായിരുന്നതുപോലെ ക്വുര്‍ആന്‍ കയ്യെഴുത്ത് പ്രതികളും ധാരാളമായി ഉപയോഗിക്കപ്പെട്ടിരുന്നു. അതുകൊണ്ടാണല്ലോ ശത്രുക്കളുടെ നാട്ടിലേക്ക് ക്വുര്‍ആനുമായി യാത്ര ചെയ്യുന്നത് നബി (സ) നിരോധിച്ചത്.(സ്വഹീഹുല്‍ ബുഖാരി, കിതാബുല്‍ ജിഹാദ്') മദീനയിലെ മുസ്‌ലിംകളില്‍ പലരുടെയും പക്കല്‍ ക്വുര്‍ആന്‍ രേഖപ്പെടുത്തിയ ചുരുളുകളുണ്ടായിരുന്നുവെന്നും അതുമായി ശത്രുനാട്ടിലേക്കു പോകുന്ന പതിവ് സ്വഹാബിമാര്‍ക്കുണ്ടായിരുന്നുവെന്നുമാണ് അബ്ദുല്ലാഹിബ്‌നു ഉമര്‍ (റ) നിവേദനം ചെയ്ത ഈ ഹദീഥ് വ്യക്തമാക്കുന്നത്. മദീനയില്‍ ക്വുര്‍ആന്‍ കയ്യെഴുത്ത് രേഖകള്‍ വ്യാപകമായിരുന്നുവെന്ന് തന്നെയാണ് ഇത് മനസ്സിലാക്കിത്തരുന്നത്.

അവതരിക്കപ്പെട്ട മുറയിൽ കാണാതെ പഠിച്ചും ലഭ്യമായ ചുരുളുകളിൽ എഴുതി സൂക്ഷിച്ചും എഴുത്ത് രൂപത്തിലും മനഃപാഠമായും സംരക്ഷിക്കപ്പെട്ട ഗ്രൻഥമാണ് ഖുർആൻ എന്നർത്ഥം. അന്ന് മുതൽ ഇന്ന് വരെ ഈ രണ്ട് രുപത്തിലും ക്വുർആൻ സംരക്ഷിക്കപ്പെട്ടുകൊണ്ടിരിക്കുന്നു.

തന്നെ അല്ലാഹു ഒറ്റരാത്രി കൊണ്ട് മക്കയിലെ മസ്ജിദുല്‍ ഹറാമില്‍നിന്ന് ഫലസ്ത്വീനിലെ മസ്ജിദുല്‍ അക്വ്‌സയിലേക്ക് അതിവേഗ നിശാപ്രയാണം (ഇസ്രാഅ്) ചെയ്യിച്ചുവെന്ന മുഹമ്മദ് നബിയുടെ അവകാശവാദം വ്യാജമാണെന്ന് അതുസംബന്ധമായി അദ്ദേഹം നല്‍കിയ വിവരണങ്ങളിലെ കാലാനുചിതത്വത്തില്‍ നിന്നും വ്യക്തമാകുന്നുണ്ട്. പ്രവാചകന്റെ കാലഘട്ടത്തില്‍ മസ്ജിദുല്‍ അക്വ്‌സ നിലവിലുണ്ടായിരുന്നില്ല എന്നതാണ് യാഥാര്‍ത്ഥ്യം. ഖലീഫ ഉമറിന്റെ  കാലഘട്ടത്തിലാണ് പ്രസ്തുത ആരാധനാലയം നിര്‍മിക്കപ്പെടുന്നത്.’ ഇസ്രാഅ് കെട്ടുകഥയാണെന്നു സ്ഥാപിക്കുവാന്‍ നബിവിമര്‍ശകര്‍ ഉന്നയിക്കുന്ന ഈ ആരോപണത്തിന് എന്തെങ്കിലും അടിസ്ഥാനമുണ്ടോ

 പ്രബോധന പ്രവര്‍ത്തനങ്ങള്‍ക്ക് താങ്ങും തണലുമായി നിന്നിരുന്ന പിതൃവ്യന്‍ അബൂത്വാലിബും പത്‌നി ഖദീജ(റ)യും പ്രവാചകത്വത്തിന്റെ പത്താം വര്‍ഷം മരണപ്പെട്ടതിനെ തുടര്‍ന്ന് മുഹമ്മദ് നബി (സ) അങ്ങേയറ്റം ഖിന്നനും സാമൂഹികമായി അരക്ഷിതനുമായിത്തീര്‍ന്നതും മക്കക്കാരുടെ കഠിനശാത്രവത്തില്‍ നിന്ന് രക്ഷയാഗ്രഹിച്ച് അദ്ദേഹം ത്വാഇഫിലെ ഥക്വീഫ് ഗോത്രക്കാര്‍ക്കടുത്തേക്ക് അഭയവും പിന്തുണയുമഭ്യര്‍ത്ഥിച്ച് ചെന്നതും അവര്‍ പ്രവാചകനെ അതിനീചമായ രീതിയില്‍ കല്ലെറിഞ്ഞോടിച്ചു മക്കയിലേക്കു തന്നെ തിരിച്ചയച്ചതും ഇസ്‌ലാമിക ചരിത്രത്തിലെ പ്രസിദ്ധമായ അധ്യായങ്ങളാണ്. ഈ സാഹചര്യത്തില്‍ നബി(സ)ക്ക് അളവറ്റ സാന്ത്വനവും ആത്മവിശ്വാസവും പകരുകയും മാനവരില്‍ മഹോന്നതനും പ്രവാചകന്‍മാരുടെ നേതാവുമായി അദ്ദേഹത്തെ ആദരിക്കുകയും ചെയ്തുകൊണ്ട് അല്ലാഹു നടത്തിയ അത്യത്ഭുകരമായ ഇടപെടലായിരുന്നു ഇസ്രാഉം മിഅ്‌റാജും.

ഒരു രാത്രിയില്‍ മക്കയില്‍ നിദ്രയിലേക്കു ചായുകയായിരുന്ന നബി(സ)യെ അതിവേഗം ജറൂസലേമിലേക്ക് സഞ്ചരിപ്പിച്ചു കൊണ്ടുപോവുകയും (ഇസ്‌റാഅ്) അവിടെനിന്ന് ആകാശലോകത്തേക്കുയര്‍ത്തി അവിടുത്തെ കാഴ്ചകള്‍ കാണിക്കുകയും (മിഅ്‌റാജ്) ചെയ്തശേഷം തിരിച്ചു ജറൂസലേമിലേക്കും ആ രാത്രി പുലരുന്നതിനു മുമ്പുതന്നെ മക്കയിലേക്കുമെത്തിക്കുകയും ചെയ്ത ദിവ്യദൃഷ്ടാന്തം നബിജീവിതത്തില്‍ പ്രവാചകത്വത്തെ സാക്ഷീകരിച്ചുകൊണ്ടുണ്ടായ ഏറ്റവും വലിയ അടയാളങ്ങളില്‍ (മുഅ്ജിസത്ത്) ഒന്നായിരുന്നു

. പ്രവാചകന്റെ (സ) ഇസ്രാഅ് അനുഭവത്തെ പരാമര്‍ശിച്ചുകൊണ്ട് അവതരിപ്പിക്കപ്പെട്ട ക്വുര്‍ആന്‍ വചനത്തിലും നബി (സ) നല്‍കിയ വിവരണങ്ങളിലും പ്രമാദമായ ഒരു കാലാനുചിതത്വം മുഴച്ചു നില്‍ക്കുന്നുവെന്നും അത് ഇസ്രാഅ് ഒരു കെട്ടുകഥയാണെന്ന് വ്യക്തമാക്കുന്നുവെന്നുമാണ് പല മിഷനറിമാരും ഓറിയന്റലിസ്റ്റുകളും സ്ഥാപിക്കാന്‍ ശ്രമിച്ചിട്ടുളളത്. ഇസ്രാഇന്റെ സമയത്ത് ഫലസ്ത്വീനില്‍ മസ്ജിദുല്‍ അക്വ്‌സ ഇല്ല എന്നിരിക്കെ എങ്ങനെയാണ് അങ്ങോട്ടു നിശാസഞ്ചാരം നടന്നു എന്ന അവകാശവാദം വിശ്വസനീയമാവുക എന്ന ചോദ്യമാണ് അവര്‍ ഉന്നയിച്ചിരിക്കുന്നത്. എന്താണ് യാഥാര്‍ത്ഥ്യമെന്ന് നമുക്ക് പരിശോധിക്കാം.

ഇസ്രാഅ് സംബന്ധിയായുള്ള ക്വുര്‍ആന്‍ വചനത്തിന്റെ സാരം ഇങ്ങനെയാണ്. ”തന്റെ ദാസനെ (നബിയെ) ഒരു രാത്രിയില്‍ മസ്ജിദുല്‍ ഹറാമില്‍ നിന്ന് മസ്ജിദുല്‍ അക്വ്‌സയിലേക്ക് -അതിന്റെ പരിസരം നാം അനുഗൃഹീതമാക്കിയിരിക്കുന്നു- നിശായാത്ര ചെയ്യിച്ചവന്‍ എത്രയോ പരിശുദ്ധന്‍! നമ്മുടെ ദൃഷ്ടാന്തങ്ങളില്‍ ചിലത് അദ്ദേഹത്തിന് നാം കാണിച്ചുകൊടുക്കാന്‍ വേണ്ടിയത്രെ അത്. തീര്‍ച്ചയായും അവന്‍ (അല്ലാഹു) എല്ലാം കേള്‍ക്കുന്നവനും കാണുന്നവനുമത്രെ.”(1)

സംഭവത്തിന്റെ വിശദാംശങ്ങള്‍ പ്രബലമായ ഹദീഥുകളിലുണ്ട്. നബി (സ) മക്കയില്‍ ഖുറയ്ശികള്‍ പുനര്‍നിര്‍മിച്ച കഅ്ബയുടെ ചാരത്തുള്ള ഹിജ്‌റില്‍ ആയിരിക്കെയാണ് ഇസ്രാഅ് ആരംഭിച്ചതെന്നു അബ്ബാസുബ്‌നു മാലിക്കും(2) (റ), നബി (സ) ഉണര്‍വ്വിനും ഉറക്കിനുമിടയിലുള്ള ഒരവസ്ഥിയിലായിരിക്കെ ജിബ്‌രീല്‍ കടന്നുവന്ന്, കഴുതയെക്കാള്‍ വലതും കോവര്‍ കഴുതയെക്കാള്‍ ചെറുതുമായിരുന്ന, വെള്ള നിറത്തിലുള്ള ബുറാക്വ് എന്ന സവിശേഷ മൃഗത്തിന്റെ പുറത്തിരുത്തി കാഴ്ചയെത്തുന്നേടത്ത് കാലെത്തുന്ന അതിവേഗതയിലാണ് ഇസ്രാഇന് കൊണ്ടുപോയതെന്ന് അനസ്ബ്‌നു മാലിക്കും(3) (റ) പ്രവാചകനെ ഉദ്ധരിച്ചു പറഞ്ഞത് ആധികാരികമായി നിവേദനം ചെയ്യപ്പെട്ടിരിക്കുന്നു.

ഇങ്ങനെ പുറപ്പെട്ട യാത്ര ‘ബയ്തുല്‍ മക്വ്ദിസില്‍’ എത്തിയെന്നും അവിടെ ബുറാക്വിനെ തളച്ചശേഷം താന്‍ രണ്ടു റക്അത്ത് നമസ്‌കരിച്ചുവെന്നും നബി (സ) പറഞ്ഞതായി അനസ്ബ്‌നു മാലിക്കില്‍ നിന്നു തന്നെയുള്ള മറ്റൊരു നിവേദനത്തിലുണ്ട്.(4) ഇസ്രാഈല്‍ സമുദായത്തിന്റെ ജീവിതസിരാകേന്ദ്രമായിരുന്ന ജറൂസലേമിലുള്ള ബയ്തുല്‍ മക്വ്ദിസില്‍ വെച്ച് പൂര്‍വപ്രവാചകന്‍മാരായ അബ്രഹാം, മോശെ, യേശു എന്നിവരുമായുള്ള ആത്മീയ സമാഗമത്തിന് അല്ലാഹു തനിക്ക് അവസരമൊരുക്കിയെന്നും താന്‍ അവരുടെ സംഘപ്രാര്‍ത്ഥനയുടെ നേതാവായി നിശ്ചയിക്കപ്പെട്ടുവെന്നും നബി (സ) വ്യക്തമാക്കിയതായി അബൂ ഹുറയ്‌റ(റ)യില്‍ നിന്ന് ഉദ്ധരിക്കപ്പെടുന്ന മറ്റൊരു ഹദീഥില്‍(5) നിന്നു മനസ്സിലാക്കാം.

ഇസ്രാഅ് കഴിഞ്ഞ് മക്കയില്‍ തിരിച്ചെത്തി നബി (സ) കഅ്ബക്കരികില്‍ നില്‍ക്കുമ്പോള്‍ രാപ്രയാണത്തെക്കുറിച്ചു കേട്ട മക്കക്കാര്‍ ചോദ്യങ്ങളുമായി വളഞ്ഞുവെന്നും കഥനം സത്യമാണെന്നു തെളിയിക്കാന്‍ ബെയ്തുല്‍ മക്വ്ദിസിന്റെ ചില വിശദാംശങ്ങള്‍ -അല്‍പനേരം മാത്രം അവിടെ ചെലവഴിച്ചൊരാള്‍ക്ക് നല്‍കുവാനാകാത്തത്ര സൂക്ഷ്മമായവ- പറയാന്‍ ആവശ്യപ്പെട്ടുവെന്നും അപ്പോള്‍ അല്ലാഹു നബി(സ)ക്ക് മുന്നില്‍ ബയ്ത്തുല്‍ മക്വ്ദിസ് പ്രദര്‍ശിപ്പിച്ചുവെന്നും അതുനോക്കി അദ്ദേഹം അവരുടെ ചോദ്യങ്ങള്‍ക്കെല്ലാം കൃത്യമായ മറുപടി നല്‍കിയെന്നും അതേ ഹദീഥില്‍ തന്നെ സുതരാം വ്യക്തമാക്കപ്പെട്ടിട്ടുണ്ട്.

മക്കയിലെ കഅ്ബക്കരികില്‍ നിന്ന് ഫലസ്ത്വീനിലെ മസ്ജിദുല്‍ അക്വ്‌സ എന്നോ ബെയ്തുല്‍ മക്വ്ദിസ് എന്നോ പറയപ്പെടുന്ന സ്ഥലത്തേക്കാണ് ഇസ്രാഅ് നടന്നതെന്നാണ് നബി (സ) വിശദീകരിച്ചതെന്ന കാര്യം മേല്‍ പരാമര്‍ശിച്ച ക്വുര്‍ആന്‍ വാക്യത്തില്‍ നിന്നും ഹദീഥുകളില്‍ നിന്നും സ്പഷ്ടമാണ്. ആരാധനകള്‍ നടക്കുന്ന ഇടത്തിനാണ് ഇവിടെ യഥാക്രമം ‘മസ്ജിദ്’ എന്നോ ‘ബയ്ത്ത്’ എന്നോ പ്രയോഗിക്കപ്പെട്ടിരിക്കുന്നത് എന്ന കാര്യം വ്യക്തമാണ്. ‘മസ്ജിദ്’ എന്നാല്‍ ഭാഷാപരമായി ‘സാഷ്ടാംഗസ്ഥാനം’ എന്നും ‘ബയ്ത്ത്’ എന്നാല്‍ ഭവനം എന്നുമാണ് അര്‍ത്ഥം. ‘അക്വ്‌സാ’ എന്നാല്‍ വിദൂരമായത്/അങ്ങേയറ്റത്തുള്ളത് എന്നും ‘മക്വ്ദിസ്’ എന്നാല്‍ വിശുദ്ധമായത് എന്നുമാണ് ഭാഷാ വിവക്ഷ. ദൈവാരാധനക്കുവേണ്ടി സ്ഥാപിച്ചിട്ടുള്ളതില്‍വെച്ച് മക്കയില്‍ നിന്നേറ്റവും വിദൂരമായ ഒരു സാഷ്ടാംഗസ്ഥാനം/ആരാധനാ കേന്ദ്രം ജറൂസലേമില്‍ സ്ഥിതി ചെയ്യുന്നുവെന്നും കഅ്ബയില്‍ നിന്ന് അങ്ങോട്ടാണ് നബി(സ)യുടെ രാസഞ്ചാരം നടന്നതെന്നും ചുരുക്കം. ഏതാണ് ഈ ആരാധന കേന്ദ്രം? അത് നിര്‍മിച്ചത് ഖലീഫ ഉമറിന്റെ ഭരണകാലത്താണോ? നമുക്ക് അന്വേഷിക്കുക!

ഇസ്രാഇനെക്കുറിച്ചുള്ള ക്വുര്‍ആന്‍ വചനം നബി (സ) പാരായണം ചെയ്തു കേള്‍പ്പിക്കുന്നത് ഹിജാസിലെ തന്റെ സമകാലീനരെയാണ്. അവരില്‍ വിശ്വാസികള്‍ക്കു പുറമെ അവിശ്വാസികളായ ബഹുദൈവാരാധകരുമുണ്ട്. ഇസ്രാഇനെക്കുറിച്ച് നബി (സ) സ്വന്തം ഭാഷയില്‍ വിവരിക്കുന്നതും അവരോടു തന്നെയാണ്. ഇസ്രാഇനെക്കുറിച്ചുള്ള പ്രവാചകന്റെ (സ) കഥനം സ്വീകരിക്കുവാന്‍ സന്നദ്ധമാകാതെ അതിനെ കെട്ടുകഥയാക്കി മുദ്രകുത്തുകയും നബിനിഷേധം തുടരുകയുമാണ് അവിശ്വാസികളായ അദ്ദേഹത്തിന്റെ സമകാലീനര്‍ സ്വാഭാവികമായും ചെയ്തത്.

ഇസ്രാഇനെയും മിഅ്‌റാജിനെയും കുറിച്ചുള്ള പ്രവാചകാഖ്യാനത്തെ അപ്പടി തല്‍ക്ഷണം സ്വീകരിക്കുവാന്‍ സന്നദ്ധനായതിന്റെ പേരിലാണ് പ്രവാചകന്റെ ആത്മമിത്രം അബൂബക്‌റിന് ‘സ്വിദ്ദീക്വ്’ (വിശ്വസ്തത പുലര്‍ത്തുന്നയാള്‍) എന്ന അപരാഭിധാനം ലഭിച്ചത് എന്നു പറയുമ്പോള്‍, എത്ര വലിയ അവിശ്വസനീയതയോടെയാണ് ഇവ്വിഷയകമായ നബിവചനങ്ങളെ മക്ക എതിരേറ്റതെന്നു വ്യക്തമാകുന്നുണ്ട്. ‘ഇതും വിശ്വസിച്ചോ’ എന്ന മട്ടില്‍ അവിശ്വാസം മുറ്റുന്ന കണ്ണുകളുമായി തന്നെ സമീപിച്ച് ”അദ്ദേഹം രാത്രിയില്‍ ജറൂസലേമില്‍ പോയി പുലരുന്നതിനുമുമ്പ് തിരിച്ചെത്തിയെന്ന് പറയുന്നതിനെ താങ്കള്‍ക്കംഗീകരിക്കാനാകുന്നതെങ്ങനെ” എന്നു ചോദിച്ചവരോട് അബൂബക്ര്‍ പറഞ്ഞ ഈ മറുപടിയാണ് അദ്ദേഹത്തെ ‘സ്വിദ്ദീക്വ്’ ആക്കിയതെന്നാണ് ചരിത്രം പറയുന്നത്: ”അതെ! നിശ്ചയമായും ഞാന്‍ അദ്ദേഹം പറയുന്ന അതിനെക്കാള്‍ അത്ഭുതകരമായ കാര്യങ്ങളില്‍ വിശ്വസിക്കുന്നുണ്ടല്ലോ. രാവിലെയോ ദിനാന്ത്യത്തിലോ അദ്ദേഹത്തിന് ആകാശത്തുനിന്ന് വൃത്താന്തങ്ങളെത്തുന്നുവെന്ന് ഞാന്‍ അംഗീകരിക്കുന്നില്ലേ?”(6)

ദൈവിക വെളിപാടുകള്‍ കൊണ്ടനുഗ്രഹിക്കപ്പെട്ട പ്രവാചകന്‍ പ്രപഞ്ചനാഥന്‍ സംവിധാനിച്ച ഒരു അമാനുഷിക സഞ്ചാരത്തിന് തെരഞ്ഞെടുക്കപ്പെടുന്നതില്‍ യാതൊരു അസാംഗത്യവുമില്ലെന്ന, യുക്തിഭദ്രമായ ബോധ്യമാണ് അബൂബക്‌റിനെ ഇവിടെ നയിക്കുന്നത്. ഇസ്രാഅ് നടന്നുവെന്നംഗീകരിക്കുവാന്‍ വിസമ്മതിച്ച ഒരു വലിയ ആള്‍കൂട്ടം മക്കയിലുണ്ടായിരുന്നുവെന്നും അവരുടെ ‘പ്രശ്‌നവല്‍കരണങ്ങള്‍’ പ്രവാചകത്വം അംഗീകരിക്കുന്നുണ്ടോ ഇല്ലേ എന്നതാണ് അടിസ്ഥാന ചോദ്യമെന്നും അതംഗീകരിക്കുന്നവര്‍ക്ക് ഇസ്രാഅ് വാര്‍ത്ത അംഗീകരിക്കാതിരിക്കാനാവില്ലെന്നുമുള്ള അബൂബക്‌റിനെ പോലുള്ളവരുടെ സമര്‍ത്ഥനങ്ങള്‍ക്കുമുന്നില്‍ ബുദ്ധിപരമായി പരാജയപ്പെട്ടുവെന്നുമാണ് ഈ സംഭാഷണങ്ങള്‍ സൂചിപ്പിക്കുന്നത്.

ഒരു രാത്രികൊണ്ട് ഇത്രയധികം ദൂരം താണ്ടാനാകില്ല എന്നതായിരുന്നു അവിശ്വാസികളുടെ ‘പോയിന്റ്’ എന്നു സാരം. സാധാരണ നിയമങ്ങളെ മറികടന്നുകൊണ്ടുള്ള സവിശേഷമായ ഒരു ദൈവിക ഇടപെടലാണ് ‘ഇസ്രാഅ്’ എന്നതുകൊണ്ടു തന്നെ ആ പോയിന്റിന് സംവാദങ്ങളില്‍ പിടിച്ചുനില്‍ക്കാനാകുമായിരുന്നില്ല. എന്നാല്‍ പ്രവാചകകഥനത്തില്‍ ‘കാലാനുചിതത്വം’ ഉണ്ടായിരുന്നെങ്കില്‍ ഇസ്രാഅ് വൃത്താന്തം കള്ളമാണെന്നു തെളിയിക്കുവാന്‍ അത് തീര്‍ത്തും ഉപയുക്തമാകുമായിരുന്നു. ഫലസ്ത്വീന്‍ ഉള്‍ക്കൊള്ളുന്ന അന്നത്തെ ‘ശാം’ പ്രവിശ്യയിലേക്ക് നിരന്തരമായി കച്ചവടയാത്ര പോയിരുന്നവരായിരുന്നു മക്കയിലെ അറബികള്‍; ഫലസ്ത്വീന്‍ അതുകൊണ്ടു തന്നെ അവര്‍ക്ക് ചിരപരിചിതവുമായിരുന്നു. ഫലസ്ത്വീനില്‍ ഒരു മസ്ജിദുല്‍ അക്വ്‌സ/ബെയ്തുല്‍  മക്വ്ദിസ് ഇല്ലായിരുന്നുവെങ്കില്‍ നബബവിവരണങ്ങളെ ‘പൊളിക്കുവാന്‍’ അവര്‍ക്ക് വളരെ എളുപ്പമായിരുന്നു. എന്നിട്ടെന്തുകൊണ്ട് അവരിലൊരാള്‍ പോലും അക്കാര്യമുന്നയിച്ചില്ല?

ഫലസ്ത്വീനിന് മതപരവും വംശീയവുമായ വിശുദ്ധി കല്‍പിക്കുന്ന ജൂതന്‍മാര്‍ സമൃദ്ധമായിരുന്ന മദീനയിലേക്കാണ് നബി(സ)യും ശിഷ്യന്‍മാരും ഇസ്രാഅ് നടന്ന് ഏതാനും കാലത്തിനകം ഹിജ്‌റ പോകുന്നത്. ഇസ്രാഅ്-മിഅ്‌റാജ് കഥകള്‍ ജൂതന്‍മാരുമായി സ്വാഭാവികമായും മുസ്‌ലിംകളുടെ വിശ്വാസത്തിന്റെ ഭാഗമായി വിനിമയം ചെയ്യപ്പെട്ടു. എന്നിട്ടും ഒരു ജൂതന്‍പോലും ഒരിക്കല്‍ പോലും എന്തുകൊണ്ട് ഏത് ബയ്ത്തുല്‍ മക്വ്ദിസ്/ഏതു മസ്ജിദുല്‍ അക്വ്‌സ എന്നു ചോദിച്ച് പ്രവാചകനരികിലേക്കു വന്നില്ല?

ബെയ്തുല്‍ മക്വ്ദിസ്/മസ്ജിദുല്‍ അക്വ്‌സ എന്ന പേരിലുള്ള ഒരാരാധനാസ്ഥാനം പ്രവാചകകാലത്തുതന്നെ ഫലസ്ത്വീനിലുണ്ടായിരുന്നുവെന്നും ഹിജാസിലെ ജൂതന്‍മാരും ബഹുദൈവാരാധകരുമടക്കം അതിനെക്കുറിച്ചറിവുള്ളവരുമായിരുന്നുവെന്നുമാണ് ഇത് വ്യക്തമാക്കുന്നത്. പ്രവാചകന്റെ ഇസ്രാഅ് വിവരണത്തെ ചോദ്യം ചെയ്യാന്‍ കഅ്ബക്കരികില്‍ അദ്ദേഹത്തിനു ചുറ്റുംകൂടിയ മക്കന്‍ ബഹുദൈവാരാധകര്‍ ബെയ്തുല്‍ മക്വ്ദിസിന്റെ വിശദാംശങ്ങള്‍ പറഞ്ഞ് കഥയുടെ ആധികാരികത തെളിയിക്കുവാന്‍ അദ്ദേഹത്തെ വെല്ലുവിളിച്ചുവെന്നു പറഞ്ഞാല്‍ അതിനര്‍ത്ഥം ഫലസ്ത്വീന്‍ യാത്ര നടത്തിയവരും ബയ്ത്തുല്‍ മക്വ്ദിസ് നേരിട്ടുകണ്ടവരുമായ അനേകര്‍ അക്കൂട്ടത്തിലുണ്ടായിരുന്നുവെന്നാണല്ലോ!

യഥാര്‍ത്ഥത്തില്‍, വിമര്‍ശകരില്‍ ചിലരെങ്കിലും ധരിച്ചുവശായിട്ടുള്ളതുപോലെ ഇസ്രാഅ് വിവരണത്തില്‍ മാത്രം നബി (സ) പരാമര്‍ശിച്ച ഒന്നല്ല ഫലസ്ത്വീനിലെ വിശുദ്ധ മസ്ജിദ്. അദ്ദേഹത്തിന്റെ വര്‍ത്തമാനങ്ങളിലും അനുചരന്‍മാരുടെ ജീവിതവ്യവഹാരങ്ങളിലും നിരന്തരം കടന്നുവന്നിരുന്ന ഒരു റഫറന്‍സ് പോയിന്റ് ആയിരുന്നു അത്. മിഅ്‌റാജിന്റെ അവസരത്തിലാണ് ദിനേന അഞ്ചുനേരം നിര്‍ദ്ദിഷ്ടരീതിയില്‍ നമസ്‌കാരം നിര്‍വഹിക്കേണ്ടത് എല്ലാ മുസ്‌ലിംകള്‍ക്കും നിര്‍ബന്ധമാക്കിെക്കാണ്ടുള്ള ദൈവിക കല്‍പന നബി(സ)ക്ക് ലഭിക്കുന്നത്.(7) ഈ നമസ്‌കാരം ബയ്ത്തുല്‍ മക്വ്ദിസിന്റെ ദിശയിലേക്കു തിരിഞ്ഞുകൊണ്ട് നിര്‍വഹിക്കാനായിരുന്നു ദൈവകല്‍പന. തദടിസ്ഥാനത്തില്‍ മദീനാ ജീവിതത്തിന്റെ ആദ്യമാസങ്ങളില്‍ ബയ്ത്തുല്‍ മക്വ്ദിസിലേക്കു തിരിഞ്ഞാണ് മുസ്‌ലിംകള്‍ പ്രാര്‍ത്ഥിച്ചുകൊണ്ടിരുന്നത്. പിന്നീടാണ് മക്കയിലെ കഅ്ബയുടെ ദിശയിലേക്കു തിരിയുവാന്‍ നിര്‍ദ്ദേശിച്ചുകൊണ്ടുള്ള ക്വുര്‍ആന്‍ വചനം(8) അവതരിപ്പിക്കപ്പെടുന്നതും മുസ്‌ലിംകളുടെ ക്വിബ്‌ല ജറൂസലേമില്‍നിന്ന് മക്കയിലേക്കു മാറുന്നതും.

നബി (സ) മദീനയിലെത്തിയതിനുശേഷം പതിനാറോ പതിനേഴോ മാസങ്ങള്‍ നമസ്‌കാരം ബയ്ത്തുല്‍ മക്വ്ദിസിനു അഭിമുഖമായിട്ടായിരുന്നുവെന്നും ഒരു അസ്വ്ര്‍ നമസ്‌കാരമാണ് കഅ്ബയിലേക്കു തിരിഞ്ഞു ആദ്യമായി നബി(സ)യുടെ നേതൃത്വത്തില്‍ സംഘമായി നിര്‍വഹിക്കപ്പെട്ടതെന്നും പുതിയ ക്വിബ്‌ലക്കഭിമുഖമായി പ്രവാചകന്റെ (സ) കൂടെ ഈ നമസ്‌കാരത്തില്‍ പങ്കെടുത്ത ഒരാള്‍ നമസ്‌കാരം കഴിഞ്ഞു പുറത്തിറങ്ങി മദീനയില്‍ തന്നെയുള്ള മറ്റൊരു പള്ളിക്കരികിലെത്തിയപ്പോള്‍ അവിടെ ക്വിബ്‌ലമാറ്റ വിവരമറിയാതെ ജനങ്ങള്‍ ബയ്ത്തുല്‍ മക്വ്ദിസിന്റെ ദിശയില്‍ തന്നെ നമസ്‌കരിക്കുന്നതു കണ്ടുവെന്നും അപ്പോള്‍ അദ്ദേഹം ഉച്ചത്തില്‍ ക്വിബ്‌ലമാറ്റം വിളംബരം ചെയ്തുവെന്നും അതുകേട്ട ജനങ്ങള്‍ നമസ്‌കരിച്ചുകൊണ്ടിരിക്കെ തന്നെ പുതിയ ക്വിബ്‌ലയിലേക്ക് കൂട്ടമായി തിരിഞ്ഞുവെന്നും പ്രവാചകാനുചരനായ ബറാഉബ്‌നു ആസ്വിബ് (റ) വിശദീകരിച്ചത് ബുഖാരി രേഖപ്പെടുത്തിയിട്ടുണ്ട്.(9) ഇത് നല്‍കുന്ന അറിവെന്താണ്? ജറുസലേമിലെ ബയ്ത്തുല്‍ മക്വ്ദിസിലേക്ക് തിരഞ്ഞുകൊണ്ടാണ് മുസ്‌ലിംകളുടെ നിര്‍ബന്ധ പ്രാര്‍ത്ഥനകള്‍ എന്നകാര്യം പരസ്യമായിരുന്നുവെന്നും മദീനയിലെ പള്ളികള്‍ നിര്‍മിക്കപ്പെട്ടിരുന്നത് ബയ്ത്തുല്‍ മക്വ്ദിസിനുഭിമുഖമായി നമസ്‌കാരം നിര്‍വഹിക്കാനാകുംവിധമായിരുന്നുവെന്നും തന്നെയല്ലേ?

ജറുസലേമിനെക്കുറിച്ച് യഹൂദബൈബിളിന്റെയും തല്‍മൂദിന്റെയുമെല്ലാം വെളിച്ചത്തില്‍ കിറുകൃത്യമായ അറിവുണ്ടായിരുന്ന ജൂതന്‍മാര്‍ക്കു നടുവിലാണിതെല്ലാമെന്നോര്‍ക്കണം. നമസ്‌കാരത്തിനു നേതൃത്വം നല്‍കുന്നയാള്‍ക്ക് (ഇമാം) നില്‍ക്കാന്‍ ആദ്യം ബയ്ത്തുല്‍ മക്വ്ദിസിനഭിമുഖമായി നിര്‍മിക്കപ്പെട്ടിരുന്ന സ്ഥലം (മിഹ്‌റാബ്) കഅ്ബ ക്വിബ്‌ലയായതിനുശേഷവും അതേപടി സംരക്ഷിക്കപ്പെട്ട മദീനയിലെ പള്ളികളിലൊന്നാണ് ‘മസ്ജിദുല്‍ ക്വിബ്‌ലതയ്‌നി’ (രണ്ട് കിബ്‌ലകളുള്ള മസ്ജിദ്) എന്നപേരില്‍ മുസ്‌ലിം ലോകത്ത് കഴിഞ്ഞ പതിനാലു നൂറ്റാണ്ടുകാലവും പ്രശസ്തമായിരുന്നു. ഇന്നും മദീനയിലുള്ള ഈ പള്ളിയില്‍നിന്നും അടുത്തകാലത്തുമാത്രമാണ് ഇത് നീക്കം ചെയ്തത്. ഇത് ബനൂ സലമ ഗോത്രക്കാരുടെ പള്ളിയായിരുന്നുവെന്നും പ്രവാചകന്‍ അവിടെയിരിക്കുമ്പോഴാണ് ക്വിബ്‌ല മാറ്റത്തിനുള്ള സന്ദേശം ലഭിച്ച് കഅ്ബയിലേക്കു തിരിഞ്ഞ് നമസ്‌കരിച്ചതെന്നും അതുകൊണ്ടാണ് ഈ പള്ളി സവിശേഷമായ പേരിലും രീതിയിലും നിലനിന്നതെന്നും പറയുന്ന ഒരു പാരമ്പര്യം ഇബ്‌നു സഅദ് രേഖപ്പെടുത്തിയിട്ടുണ്ട്.(10) പ്രസ്തുത പാരമ്പര്യത്തിന്റെ ആധികാരികത എന്തു തന്നെയായിരുന്നാലും, ബയ്ത്തുല്‍ മക്വ്ദിസ് നമസ്‌കാരദിശയായിരുന്ന ഭൂതകാലം മുസ്‌ലിം സമൂഹത്തില്‍ കാലങ്ങള്‍ക്കുശേഷവും എത്ര സജീവമായി ചര്‍ച്ച ചെയ്യപ്പെട്ടുവെന്ന് അത് ബോധ്യപ്പെടുത്തുന്നുണ്ട്.

ഉമറിന്റെ ഭരണകാലത്തുമാത്രം നിലവില്‍ വന്ന ഒരു കേന്ദ്രത്തിലേക്കു തിരിഞ്ഞ് ഇതെല്ലാം എങ്ങനെ സംഭവിച്ചുവെന്നാണ് വിമര്‍ശകര്‍ മനസ്സിലാക്കുന്നത്? ‘ഇല്ലാത്ത’ ഒരിടത്തേക്കു തിരിഞ്ഞ് നമസ്‌കരിക്കുന്നതിലെ ‘അനൗചത്യം’ ചൂണ്ടിക്കാട്ടി മുസ്‌ലിംകളെ പരിഹസിക്കുവാന്‍ ഒരു യഹൂദന്‍ പോലും അക്കാലത്ത് രംഗത്തുവരാതിരുന്നത് എന്തുകൊണ്ടാണ്? ജറുസലേമിലെ ബയ്ത്തുല്‍ മക്വ്ദിസ് അന്നുമുണ്ടായിരുന്നുവെന്നതും അതിനെക്കുറിച്ച് അവര്‍ക്കെല്ലാം അറിവുണ്ടായിരുന്നുവെന്നതുമാണ് അതിന്റെ കാരണം. അതുകൊണ്ടാണ് പ്രവാചകാനുചരനായ ബറാഉബ്‌നു ആസ്വിബ് പറഞ്ഞത്: ”യഹൂദന്‍മാരും വേദക്കാരും പ്രവാചകന്‍ നമസ്‌കാരത്തില്‍ ബയ്ത്തുല്‍ മക്വ്ദിസിലേക്കു തിരിയുന്നതില്‍ സന്തുഷ്ടരായിരുന്നു. എന്നാല്‍ നമസ്‌കാരദിശ കഅ്ബയിലേക്ക് അദ്ദേഹം മാറ്റിയപ്പോള്‍ അവര്‍ അതില്‍ അദ്ദേഹത്തെ തള്ളിപ്പറഞ്ഞു.”(11)

യഹൂദന്‍മാര്‍ ബയ്ത്തുല്‍ മക്വ്ദിസ് എന്നുകേട്ട് അത്ഭുതപ്പെട്ടില്ലെന്നു മാത്രമല്ല, മുസ്‌ലിംകള്‍ അതിനെ തങ്ങളുടെ ക്വിബ്‌ലയായി സ്വീകരിച്ചതില്‍ ആവേശഭരിതരാവുക കൂടി ചെയ്തുവെന്നാണ് പരാമൃഷ്ട നിവേദനം തെളിയിക്കുന്നത്. വാസ്തവത്തില്‍, ക്വിബ്‌ല മാറ്റം അറിയിച്ചുകൊണ്ടുള്ള ക്വുര്‍ആന്‍ സൂക്തങ്ങളില്‍ തന്നെ അത് യഹുദരില്‍ സൃഷ്ടിച്ച ഭാവഭേദങ്ങളെ സംബന്ധിച്ച സൂചനകളുണ്ട്. ”അപ്രകാരം നാം നിങ്ങളെ ഒരു ഉത്തമ സമുദായമാക്കിയിരിക്കുന്നു. നിങ്ങള്‍ ജനങ്ങള്‍ക്ക് സാക്ഷികളായിരിക്കുവാനും റസൂല്‍ നിങ്ങള്‍ക്ക് സാക്ഷിയായിരിക്കുവാനും വേണ്ടി. റസൂലിനെ പിന്‍പറ്റുന്നതാരൊക്കെയെന്നും, പിന്‍മാറിക്കളയുന്നതാരൊക്കെയെന്നും തിരിച്ചറിയുവാന്‍ വേണ്ടി മാത്രമായിരുന്നു നീ ഇതുവരെ തിരിഞ്ഞു നിന്നിരുന്ന ഭാഗത്തെ നാം ക്വിബ്‌ലയായി നിശ്ചയിച്ചത്. അല്ലാഹു നേര്‍വഴിയിലാക്കിയവരൊഴിച്ച് മറ്റെല്ലാവര്‍ക്കും അത് ( ക്വിബ്‌ല മാറ്റം ) ഒരു വലിയ പ്രശ്‌നമായിത്തീര്‍ന്നിരിക്കുന്നു. അല്ലാഹു നിങ്ങളുടെ വിശ്വാസത്തെ പാഴാക്കിക്കളയുന്നതല്ല. തീര്‍ച്ചയായും അല്ലാഹു മനുഷ്യരോട് അത്യധികം ദയയുള്ളവനും കരുണാനിധിയുമാകുന്നു. (നബിയേ,) നിന്റെ മുഖം ആകാശത്തേക്ക് തിരിഞ്ഞുകൊണ്ടിരിക്കുന്നത് നാം കാണുന്നുണ്ട്. അതിനാല്‍ നിനക്ക് ഇഷ്ടമാകുന്ന ഒരു ക്വിബ്‌ലയിലേക്ക് നിന്നെ നാം തിരിക്കുകയാണ്. ഇനി മേല്‍ നീ നിന്റെ മുഖം മസ്ജിദുല്‍ ഹറാമിന്റെ നേര്‍ക്ക് തിരിക്കുക. നിങ്ങള്‍ എവിടെയായിരുന്നാലും അതിന്റെ നേര്‍ക്കാണ് നിങ്ങള്‍ മുഖം തിരിക്കേണ്ടത്. വേദം നല്‍കപ്പെട്ടവര്‍ക്ക് ഇത് തങ്ങളുടെ രക്ഷിതാവിങ്കല്‍ നിന്നുള്ള സത്യമാണെന്ന് നന്നായി അറിയാം. അവര്‍ പ്രവര്‍ത്തിക്കുന്നതിനെപ്പറ്റിയൊന്നും അല്ലാഹു അശ്രദ്ധനല്ല. വേദം നല്‍കപ്പെട്ടവരുടെ അടുക്കല്‍ നീ എല്ലാവിധ ദൃഷ്ടാന്തവും കൊണ്ട് ചെന്നാലും അവര്‍ നിന്റെ ക്വിബ്‌ലയെ പിന്തുടരുന്നതല്ല. അവരുടെ ക്വിബ്‌ലയെ നീയും പിന്തുടരുന്നതല്ല. അവരില്‍ ഒരു വിഭാഗം മറ്റൊരു വിഭാഗത്തിന്റെ ക്വിബ്‌ലയെ പിന്തുടരുകയുമില്ല. നിനക്ക് ശരിയായ അറിവ് വന്നുകിട്ടിയ ശേഷം നീയെങ്ങാനും അവരുടെ ഇച്ഛകളെ പിന്‍പറ്റിയാല്‍ നീയും അതിക്രമകാരികളുടെ കൂട്ടത്തില്‍ തന്നെയായിരിക്കും.” (2: 143-145)

ഇസ്രഈല്യര്‍ പൂര്‍വിക കാലം മുതല്‍ക്കുതന്നെ വിശുദ്ധമായി ആദരിച്ചിരുന്നതും അറബികളടക്കമുള്ള കച്ചവടയാത്രികര്‍ക്ക് സുപരിചിതമായിരുന്നതും മുഹമ്മദ് നബി(സ)യും അനുയായികളും വിശുദ്ധമായി അംഗീകരിച്ചിരുന്നതുമായ ഫലസ്ത്വീനിലെ ഒരു ആരാധനാ സ്ഥാനം ബയ്ത്തുല്‍ മക്വ്ദിസ് എന്നോ മസ്ജിദുല്‍ അക്വ്‌സ എന്നോ ഉള്ള പേരില്‍ അറേബ്യയിലുടനീളം അറിയപ്പെട്ടിരുന്നുവെന്നും തന്റെ നിശായാത്ര അവിടെയെത്തിയതിനെക്കുറിച്ചാണ് നബി(സ) സംസാരിച്ചതെന്നുമാണ് ഇവയില്‍ നിന്നെല്ലാം അനിഷേധ്യമാംവിധം ബോധ്യപ്പെടുന്നത്. ഏതായിരുന്നു ആ ആരാധാനാസ്ഥാനം? ഇസ്രാഈലി പൂര്‍വപ്രവാചകന്‍മാര്‍ക്ക് നേതൃത്വം നല്‍കി നബി(സ)ക്ക് അവിടെവെച്ച് നമസ്‌കരിക്കുവാനവസരമുണ്ടായതിലെ പ്രതീകാത്മകത സൂചിപ്പിക്കുന്നതുപോലെ ഇസ്രാഈല്‍ സമൂഹത്തില്‍ അവരുടെ പ്രവാചകന്‍മാരാല്‍ പരിപാലിക്കപ്പെട്ടിരുന്ന ആരാധനാകേന്ദ്രമായിരുന്നു ബയ്ത്തുല്‍ മക്വ്ദിസ്. ബൈബിളും ഇസ്‌ലാമിക പ്രമാണങ്ങളും പരിശോധിച്ചാല്‍ ഇക്കാര്യം മനസ്സിലാകും

. മാനവചരിത്രത്തില്‍ പ്രപഞ്ചനാഥനെ ആരാധിക്കുന്നതിനുവേണ്ടി സംവിധാനിക്കപ്പെട്ട രണ്ടാമത്തെ കേന്ദ്രം ബയ്ത്തുല്‍ മക്വ്ദിസ് ആയിരുന്നുവെന്നാണ് മുഹമ്മദ് നബി (സ) പഠിപ്പിച്ചിട്ടുള്ളത്, ഒന്നാമത്തേത് മക്കയിലെ കഅ്ബയും. അബൂ ദര്‍റ് (റ) നബി(സ)യോട് ചോദിച്ചു. ”അല്ലാഹുവിന്റെ ദൂതരെ, ഏതുപള്ളിയാണ് ഭൂമുഖത്ത് ആദ്യമായി നിര്‍മിക്കപ്പെട്ടത്?” അദ്ദേഹം പറഞ്ഞു, ”മസ്ജിദുല്‍ ഹറാം.” അബൂ ദര്‍റ് (റ) വീണ്ടും ചോദിച്ചു. ”പിന്നെയേതാണ് നിര്‍മിക്കപ്പെട്ടത്?” അദ്ദേഹം പ്രതിവചിച്ചു,  (ജറുസലേമിലെ) മസ്ജിദുല്‍ അക്വ്‌സാ.”  അബൂ ദര്‍റ് (റ) വീണ്ടും ചോദിച്ചു. ”അവ രണ്ടിന്റെയും നിര്‍മാണങ്ങള്‍ക്കിടയിലെ കാലവിളംബമെത്രയായിരുന്നു?” അദ്ദേഹം പറഞ്ഞു, ”നാല്‍പ്പത് വര്‍ഷം.”(12)

ഭൂമുഖത്ത് ആദ്യമായി നിര്‍മിക്കപ്പെട്ട ദൈവാരാധനാകേന്ദ്രമായ കഅ്ബയില്‍ നിന്ന് രണ്ടാമതായി നിര്‍മിക്കപ്പെട്ട ബയ്ത്തുല്‍ മക്വ്ദിസിലേക്കാണ് പ്രവാചകന്റെ (സ) നിശാപ്രയാണം നടന്നതെന്ന് സാരം. കഅ്ബയാണ് ചരിത്രത്തില്‍ ഒന്നാമതായി നിര്‍മിക്കപ്പെട്ട ദൈവാരാധനാ ഭവനമെന്ന് ക്വുര്‍ആന്‍ തന്നെ പ്രസ്താവിക്കുന്നുണ്ട്.(13) ആദിമനുഷ്യനായ ആദമിന്റെ കാലത്തുതന്നെ ഇവ രണ്ടും നിര്‍മിക്കപ്പെട്ടുവെന്നാണ് പണ്ഡിതന്‍മാര്‍ പൊതുവില്‍ മനസ്സിലാക്കുന്നത്. കഅ്ബയുടെ സ്ഥാനത്ത് പിന്നീടുള്ള പുനര്‍നിര്‍മാണങ്ങളും അതിന്റെ പരിപാലനവുമെല്ലാം ഇബ്‌റാഹീം, ഇസ്മാഈല്‍ പ്രവാചകന്‍മാരുടെയും അവരുടെ സന്തതിപരമ്പരകളായ അറബികളുടെയും കൈകളിലൂടെ നടന്നപ്പോള്‍ ബയ്ത്തുല്‍ മക്വ്ദിസിന്റേത് അതിനുചുറ്റും അധിവസിക്കുന്ന ഇസ്രാഈല്യരിലൂടെ നടന്നുവെന്നു മാത്രമേയുള്ളൂ. ഇസ്രാഈല്യര്‍ക്കുവേണ്ടി ബയ്ത്തുല്‍ മക്വ്ദിസിന്റെ പുനര്‍നിര്‍മാണം പൂര്‍ത്തിയാക്കിയത് അവരുടെ രാജാവും പ്രവാചകനും ആയിരുന്ന സുലൈമാന്‍ (അ) ആയിരുന്നുവെന്ന് നബി(സ) പറഞ്ഞതായി അബ്ദുല്ലാഹിബ്‌നു അംറില്‍ നിന്നുള്ള ഒരു നിവേദനത്തിലുണ്ട്.(14)

സോളമന്‍ നിര്‍മിച്ച ആരാധനാലയം എന്ന നിലയില്‍ തന്നെയാണ് യഹൂദര്‍ ബയ്ത്തുല്‍ മക്വ്ദിസിനെ മനസ്സിലാക്കിയത്. ജൂത-ക്രൈസ്തവ സാഹിത്യങ്ങളിലെ ‘The Temple’ സോളമന്‍ രാജാവിനാല്‍ നിര്‍മിക്കപ്പെട്ട ബയ്ത്തുല്‍ മക്വ്ദിസ് ആണ്. അത് നിര്‍മിക്കപ്പെട്ട ഉയര്‍ന്ന പ്രദേശമാണ് ‘Temple Mount’ എന്ന പേരില്‍ യഹൂദ ക്രൈസ്തവ പാരമ്പര്യത്തില്‍ അറിയപ്പെടുന്നത്. ജറുസലേമില്‍ നേരത്തെതന്നെ വിശുദ്ധ സ്ഥാനമായി മനസ്സിലാക്കപ്പെട്ടിരുന്ന Temple Mountല്‍ സോളമന്‍ രാജാവ് പണിതീര്‍ത്ത ദേവാലയമാണ് ബയ്ത്തുല്‍ മക്വ്ദിസ് എന്ന് യഹൂദപാരമ്പര്യവും ബൈബിള്‍ പഴയനിയമവും തെളിയിക്കുന്നുണ്ട്. ഉല്‍പത്തി പുസ്തകത്തിലെ ‘മോറിയാ ദേശത്തെ മലരുകള്‍'(15) ആണ് സോളമന്റെ ദേവാലയത്തിന്റെ ആസ്ഥാനമെന്ന നിലയില്‍ Temple Mount ആയി മാറിയത്. മോറിയാ പ്രദേശം ‘സൃഷ്ടിയുടെ ആരംഭം’ മുതല്‍ക്കുതന്നെ വിശുദ്ധമായി പ്രഖ്യാപിക്കപ്പെട്ടതായിരുന്നുവെന്നാണ് യഹൂദവിശ്വാസം.(16) ആദമിനെ സൃഷ്ടിക്കാനുതകുന്ന മണ്ണ് കര്‍ത്താവ് ഒരുക്കൂട്ടിയതിവിടെയാണെന്നു വിശ്വസിക്കുന്ന യഹൂദന്‍മാരുണ്ട്.(17) മോറിയാ പ്രദേശത്തിന്റെ വിശുദ്ധിയെക്കുറിച്ച ഈ ധാരണ ബൈബിളെഴുത്തുകാരെ ആഴത്തില്‍ സ്വാധീനിച്ചുവെന്ന് കാണാനാകും. അബ്രഹാമിനോട് കര്‍ത്താവ് പുത്രന്‍ ഇസ്ഹാക്വിനെ മോറിയായിലേക്കു കൊണ്ടുപോയി അവിടെവെച്ച് ബലിയറുക്കാനാവശ്യപ്പെട്ടുവെന്നാണ് ഉല്‍പത്തി പക്ഷം.(18)

മാനവചരിത്രത്തിന്റെ നന്നേ തുടക്കം മുതല്‍ക്കുതന്നെ വിശുദ്ധമായി നിശ്ചയിക്കപ്പെട്ട സ്ഥാനമായി ബയ്ത്തുല്‍ മക്വ്ദിസിന്റെ ഭൂമിയെ ഇസ്‌ലാമിക പാരമ്പര്യത്തെപ്പോലെത്തന്നെ യഹൂദപാരമ്പര്യവും മനസ്സിലാക്കുന്നുവെന്നു ചുരുക്കം. പ്രസ്തുത ഭൂമിയില്‍ പിതാവായ ദാവീദ് എല്‍പിച്ചതുപ്രകാരം സോളമന്‍ അതിഗംഭീരമായ ദേവാലയം ഏറ്റവും മികച്ച പണിത്തരങ്ങളുടെയും പണിക്കാരെയും വെച്ചു നിര്‍മിക്കുകയും അതിന്റെ കേന്ദ്രസ്ഥാനത്ത് മോശെക്ക് കര്‍ത്താവ് കല്‍പനകള്‍ എഴുതി നല്‍കിയ പലകകള്‍ സൂക്ഷിച്ചിരുന്ന വിശുദ്ധ പ്രമാണപെട്ടകം (Art of the covenant)സ്ഥാപിക്കുകയും മൃഗബലിയുള്‍പ്പെടെയുള്ള ഇസ്രാഈലി അനുഷ്ഠാനകര്‍മങ്ങളുടെയും പ്രാര്‍ത്ഥനകളുടെയും സിരാകേന്ദ്രമായി ദേവാലയം പരിലസിക്കുകയും ചെയ്തതിനെക്കുറിച്ചുള്ള വിശദമായ വര്‍ണനകള്‍ ബൈബിളെഴുത്തുകാര്‍ നടത്തിയിട്ടുണ്ട്.(19) ഈ ദേവാലയത്തെ യഹൂദന്‍മാര്‍ വിളിച്ചത് ഹീബ്രുവില്‍ ‘ബെയ്ത് ഹാമിക്ദാശ്’ (Beit Hamikdash) എന്നായിരുന്നു;(20) അറബികളുടെ ബയ്ത്ത് അല്‍ മക്വ്ദിസ് തന്നെ.

ഏതാണ് പരിശുദ്ധ ക്വുര്‍ആനും ഹദീഥുകളും നിശാപ്രയാണം ചെന്നെത്തിയെന്നു പറയുന്ന ഫലസ്ത്വീനിയന്‍ ആരാധനാസ്ഥാനമെന്ന് ഇത്രയും വിശദീകരിച്ചതില്‍ നിന്ന് സുതരാം വ്യക്തമാണ്. സോളമന്റെ ദേവാലയം മുഹമ്മദ് നബി(സ)യുടെ കാലത്ത് നിലവിലുണ്ടായിരുന്നില്ല എന്നുപറഞ്ഞുകൊണ്ടാണ് വിമര്‍ശകര്‍ ഈ വസ്തുതകളെ മറികടക്കാന്‍ നോക്കാറുള്ളത്. സോളമന്‍ നിര്‍മിച്ച കെട്ടിടം ഫലസ്ത്വീനില്‍ നിലവിലുണ്ടെന്ന് ക്വുര്‍ആനോ മുഹമ്മദ് നബി(സ)യോ എവിടെയും പറഞ്ഞിട്ടില്ല. പിന്നെ ഈ വിമര്‍ശനത്തിന് എന്ത് പ്രസക്തിയാണുള്ളത്?

സോളമന്‍ നിര്‍മിച്ച ദേവാലയം BCE 586ല്‍ ബാബിലോണിയക്കാര്‍ തകര്‍ത്തതും പിന്നീട് ഏഴു പതിറ്റാണ്ടോളം കഴിഞ്ഞ് യഹൂദര്‍ തല്‍സ്ഥാനത്ത് മറ്റൊരു കെട്ടിടം പണിതതും (second temple) ഹെറോദ് രാജാവ് അതിന് മോടി കൂട്ടിയതും CE 70ല്‍ യഹൂദകലാപകാരികളെ അമര്‍ച്ച ചെയ്യാന്‍ വന്ന റോമന്‍ സൈന്യം അതു തകര്‍ത്തുകളഞ്ഞതും(21) ചരിത്രത്തില്‍ പരക്കെ അറിയപ്പെട്ടതാണ്. First Temple എന്നും Second Temple എന്നും ബൈബിള്‍ പണ്ഡിതന്മാര്‍ വിളിക്കുന്ന കെട്ടിടങ്ങള്‍ Temple Mountല്‍ സംരക്ഷിക്കപ്പെട്ടുകിടക്കുന്നില്ലെന്നും അവ തകര്‍ക്കപ്പെട്ടുവെന്നും മുഹമ്മദ് നബി(സ)യുടെ സമകാലീനരായ മദീനാ യഹൂദന്മാര്‍ക്കും ശാമിലേക്ക് നിരന്തരമായി കച്ചവടയാത്ര പോയിരുന്ന മക്കന്‍ ബഹുദൈവാരാധകര്‍ക്കും അറിയാതിരിക്കുവാന്‍ ചരിത്രപരമായി യാതൊരു നിര്‍വാഹവുമില്ല. എന്നിട്ടും അവരൊന്നും നിശായാത്രാ വിവരണങ്ങളോടോ ബയ്ത്തുല്‍ മക്വ്ദിസിനെ ക്വിബ്‌ലയാക്കാനുള്ള തീരുമാനത്തോടോ ഇക്കാര്യം പറഞ്ഞ് ഏറ്റുമുട്ടാന്‍ വരാതിരുന്നത് നബി(സ) സംസാരിച്ചത് സോളമനോ പില്‍ക്കാല ഇസ്രാഈല്യരോ നിര്‍മിച്ച ഏതെങ്കിലും കെട്ടിടങ്ങളെ കുറിച്ചല്ല, മറിച്ച് ബയ്ത്തുല്‍ മക്വ്ദിസിനെക്കുറിച്ചും മസ്ജിദുല്‍ അക്വ്‌സായെക്കുറിച്ചും ആണ് എന്നതുകൊണ്ടാണ്.

മനുഷ്യചരിത്രത്തിന്റെ ആദ്യ നാളുകള്‍ തൊട്ട് പ്രപഞ്ചനാഥന്റെ നിര്‍ദേശപ്രകാരം അവനെ ആരാധിക്കുവാനായി ഉപയോഗിക്കപ്പെട്ട ഒരു ഭൂമിശാസ്ത്ര ഉണ്‍മയാണ് ഇസ്‌ലാമിക പരിപ്രേക്ഷ്യത്തില്‍ അത്. അവിടെ കെട്ടിടങ്ങള്‍ വിവിധ കാലഘട്ടങ്ങളില്‍ വന്നിട്ടും പോയിട്ടുമുണ്ട്, പക്ഷെ അവയുടെ ചുമരുകള്‍ തകര്‍ന്നാലും നിശ്ചിത അതിര്‍ത്തികള്‍ക്കുള്ളില്‍ അത് മസ്ജിദ് തന്നെയാണ്. ആരാധനകള്‍ നിര്‍വഹിക്കാന്‍ നിര്‍ദേശിക്കപ്പെട്ട സ്ഥലം എന്നു മാത്രമേ സാങ്കേതികമായി മസ്ജിദിന് അര്‍ത്ഥമുള്ളൂ, അത് ചുമരുകളുള്ള കെട്ടിടത്തിനകത്താകണമെന്ന് യാതൊരു നിര്‍ബന്ധവുമില്ല. മുന്‍പ്രവാചകന്മാരില്‍ നിന്ന് വ്യത്യസ്തമായി ഭൂമിയിലെവിടെയും വെച്ച് നിര്‍ബന്ധ നമസ്‌കാരം നിര്‍വഹിക്കാന്‍ തന്റെ അനുയായികള്‍ക്ക് അല്ലാഹു അനുമതി നല്‍കിയിരിക്കുന്നു   എന്ന് പഠിപ്പിച്ചുകൊണ്ട് മുഹമ്മദ് നബി (സ) പറഞ്ഞതിപ്രകാരമാണ്. ”ഭൂമി എനിക്ക് വിശുദ്ധവും ശുദ്ധിയുള്ളതും മസ്ജിദും ആക്കിത്തന്നിരിക്കുന്നു; അതിനാല്‍ പ്രാര്‍ത്ഥനാ സമയങ്ങള്‍ ആഗതമാകുമ്പോള്‍ നിങ്ങള്‍ എവിടെയായിരുന്നാലും അവിടെവെച്ച് നമസ്‌കരിക്കുക.”(22)

ആരാധനാസ്ഥാനങ്ങളാണ് മസ്ജിദുകള്‍. ചുമരുകള്‍ അതിനെ പരിപാലിക്കുന്ന മനുഷ്യരുടെ സംഭാവനയാണ്; അതാരെങ്കിലും കെട്ടിയാലും കെട്ടിയില്ലെങ്കിലും കെട്ടിയവ നിലനിന്നാലും ഇല്ലെങ്കിലും മസ്ജിദായി പ്രഖ്യാപിക്കപ്പെടുകയും നീക്കിവെക്കുകയും ചെയ്ത നിലങ്ങള്‍ മസ്ജിദായി തന്നെ നിലനില്‍ക്കും- ആരാധനകള്‍ നിര്‍വഹിക്കാവുന്ന ഒരു തുണ്ട് ഭൂമി എന്ന അര്‍ത്ഥത്തില്‍. ഭൂമി മുഴുവന്‍ ഇനി മുതല്‍ മസ്ജിദ് ആണെന്ന് മുഹമ്മദ് നബി (സ) പറയുമ്പോള്‍ അദ്ദേഹത്തിന്റെ കാലം മുതല്‍ ഭൂമിയിലെവിടെവെച്ചും നമസ്‌കരിക്കാമെന്ന പുതിയ നിയമം പടച്ചവന്‍ അവതരിപ്പിച്ചിരിക്കുന്നുവെന്നാണ് അതിനര്‍ത്ഥം; അല്ലാതെ ഭൂമിക്ക് ചുമരുകളുണ്ടെന്നല്ല.

ഇതുതന്നെയാണ് ‘ബയ്ത്’ എന്ന പ്രയോഗത്തിന്റെ കാര്യവും. മസ്ജിദ് എന്നു പറയുമ്പോഴുള്ള വിവക്ഷ മാത്രമേ അതിനുമുള്ളുവെന്ന് ക്വുര്‍ആന്‍ പരിശോധിച്ചാല്‍ തന്നെ മനസ്സിലാകും. ഇബ്‌റാഹീം നബി(അ)യും പുത്രന്‍ ഇസ്മാഈല്‍ നബി(അ)യും ചേര്‍ന്നാണ് മക്കയിലെ കഅ്ബ പടുത്തുയര്‍ത്തിയതെന്ന് ക്വുര്‍ആന്‍ ഖണ്ഡിതമായി പ്രസ്താവിക്കുന്നുണ്ട്: ”ഇബ്രാഹീമും ഇസ്മാഈലും കൂടി ആ ഭവനത്തിന്റെ (കഅ്ബയുടെ) അടിത്തറ കെട്ടി ഉയര്‍ത്തിക്കൊണ്ടിരുന്ന സന്ദര്‍ഭവും (അനുസ്മരിക്കുക). (അവര്‍ ഇപ്രകാരം പ്രാര്‍ത്ഥിച്ചിരുന്നു:) ഞങ്ങളുടെ രക്ഷിതാവേ, ഞങ്ങളില്‍ നിന്ന് നീയിത് സ്വീകരിക്കേണമേ. തീര്‍ച്ചയായും നീ എല്ലാം കേള്‍ക്കുന്നവനും അറിയുന്നവനുമാകുന്നു.”(23) ‘ബയ്ത്’ എന്നാണ്, മറ്റുപല ക്വുര്‍ആന്‍ വചനങ്ങളിലുമെന്നപോലെ ഇവിടെയും, കഅ്ബയെ സൂചിപ്പിക്കാന്‍ പ്രയോഗിക്കപ്പെട്ടിരിക്കുന്നത്. ഇബ്‌റാഹീം നബി (അ) ഭാര്യ ഹാജറിനെയും കൈക്കുഞ്ഞായ ഇസ്മാഈലിനെയും (അ) ആള്‍താമസമില്ലാത്ത മക്കയില്‍ കൊണ്ടുവന്നു താമസിപ്പിച്ച് സ്വദേശത്തേക്കു മടങ്ങിയും മക്കയില്‍ സംസം ഉറവയുണ്ടായതും അതിനുചുറ്റും ജനവാസമാരംഭിച്ചതും ഇസ്മാഈല്‍ (അ) വളര്‍ന്നുവലുതായതും അപ്പോള്‍ ഇബ്‌റാഹീം (അ) മടങ്ങിവന്നു മകനെയും കൂട്ടി കഅ്ബ നിര്‍മിച്ചതുമെല്ലാം ക്വുര്‍ആനിലും ഹദീഥുകളിലും വിശദമായി വിവരിക്കപ്പെട്ടിട്ടുണ്ട്. ആദം നബി(അ)യുടെ കാലത്ത് ആരാധനലായമുണ്ടായിരുന്ന സ്ഥാനം കണ്ടെത്തി ഇബ്‌റാഹീമും ഇസ്മാഈലും കഅ്ബ നിര്‍മിച്ചുവെന്നര്‍ത്ഥം. ഇബ്‌റാഹീം ഹാജറയെ മക്കയില്‍ കൊണ്ടുവന്നാക്കുമ്പോള്‍ അവിടെ കഅ്ബ എന്ന ഒരു നിര്‍മിതിയില്ല. എന്നാല്‍ ആ സമയത്തെ ഇബ്‌റാഹീമിന്റെ പ്രാര്‍ത്ഥന ക്വുര്‍ആന്‍ വിവരിക്കുന്നതിങ്ങനെയാണ്. ”ഞങ്ങളുടെ രക്ഷിതാവേ, എന്റെ സന്തതികളില്‍ നിന്ന് (ചിലരെ) കൃഷിയൊന്നും ഇല്ലാത്ത ഒരു താഴ്‌വരയില്‍, നിന്റെ പവിത്രമായ ഭവനത്തിന്റെ (കഅ്ബയുടെ) അടുത്ത് ഞാനിതാ താമസിപ്പിച്ചിരിക്കുന്നു. ഞങ്ങളുടെ രക്ഷിതാവേ, അവര്‍ നമസ്‌കാരം മുറപ്രകാരം നിര്‍വഹിക്കുവാന്‍ വേണ്ടിയാണ് (അങ്ങനെ ചെയ്തത്). അതിനാല്‍ മനുഷ്യരില്‍ ചിലരുടെ മനസ്സുകളെ നീ അവരോട് ചായ്‌വുള്ളതാക്കുകയും, അവര്‍ക്ക് കായ്കനികളില്‍ നിന്ന് നീ ഉപജീവനം നല്‍കുകയും ചെയ്യേണമേ. അവര്‍ നന്ദി കാണിച്ചെന്നു വരാം.”(24)

ഇവിടെ നിന്റെ പവിത്രമായ ‘ബയ്ത്തി’നടുത്തുള്ള താഴ്‌വരയില്‍ അവരെ പാര്‍പ്പിച്ചിരിക്കുന്നുവെന്നാണ് ഇബ്‌റാഹീം പടച്ചവനോടു പറയുന്നത്. അദ്ദേഹം കഅ്ബ നിര്‍മിക്കുന്നതിനു മുമ്പാണിതെന്നോര്‍ക്കണം. അപ്പോള്‍, അല്ലാഹുവിനെ ആരാധിക്കാന്‍ വേണ്ടി തെരഞ്ഞെടുക്കപ്പെട്ട ഒരു സ്ഥലം അവിെടയുണ്ടായിരുന്നു, എടുപ്പുകളൊക്കെ ഇല്ലാതായെങ്കിലും ‘ബയ്ത്’ ബയ്തല്ലാതെയായില്ല. ഇതുതന്നെയാണ് ഇസ്‌ലാമിക വീക്ഷണത്തില്‍ ബയ്ത്തുല്‍ മക്വ്ദിസിന്റെയും കാര്യം. സോളമന്റെ എടുപ്പുകള്‍ വന്നതോ പോയതോ അതിന്റെ ബയ്ത്/മസ്ജിദ് എന്ന സ്ഥാനത്തെ ഒരു നിലക്കും ബാധിക്കുന്നില്ല. അതിന്റെ അതിര്‍ത്തികള്‍ക്കുള്ളിലെ സ്ഥലം വിശുദ്ധമായ ഒരു ആരാധനാകേന്ദ്രം അഥവാ ബയ്ത്/മസ്ജിദ് ആകുന്നു; അങ്ങോട്ടു പോകുവാനുള്ള അനര്‍ഘ സൗഭാഗ്യം കൊണ്ടാണ് അന്ത്യപ്രവാചകനായ മുഹമ്മദ് നബി (സ) ഇസ്രാഇന്റെ രാത്രിയില്‍ അനുഗ്രഹിക്കപ്പെട്ടത്.

ഒന്നും രണ്ടും ടെമ്പിളുകള്‍ തകര്‍ക്കപ്പെട്ടതിനുശേഷം ‘Temple Mount’ യഹൂദസമൂഹത്തില്‍ വിഭാവനം ചെയ്യപ്പെട്ടതെങ്ങനെയാണെന്നുകൂടി പരിശോധിച്ചാല്‍ ഇവ്വിഷയകമായുള്ള വിമര്‍ശനങ്ങളുടെ അര്‍ത്ഥശൂന്യത പൂര്‍ണമായി ബോധ്യമാകും. റോമന്‍ ആക്രമണത്തില്‍ രണ്ടാം ദേവാലയം തകര്‍ക്കപ്പെട്ടതിനുശേഷം അവിടെ വേറെ ഏര്‍പ്പാടുകള്‍ക്കൊന്നും ആരും മുതിര്‍ന്നതായി കാണുന്നില്ല. ദേവാലയത്തിന് പവിത്രത കല്‍പിച്ചിരുന്ന യഹൂദന്‍മാര്‍ക്ക് ഫലസ്ത്വീന്‍ നഷ്ടപ്പെട്ടതു തന്നെയായിരുന്നു അതിന്റെ അടിസ്ഥാന കാരണം. ജറുസലേം പിന്നീട് റോമക്കാരുടെയും ബൈസന്റൈന്‍ സാമ്രാജ്യത്തിന്റെയും നിയന്ത്രണത്തിലേക്കാണ് പോയത്. ദേവാലയം നിലനിന്നിരുന്ന സ്ഥലത്തെ ബോധപൂര്‍വം അവഗണിച്ച് അത് പാഴ്ഭൂമിയാക്കാനായിരുന്നു അത് തച്ചുതകര്‍ത്ത റോമക്കാരുടെ ശ്രമം. അവരുടെ അനന്തരാവകാശികളായി വന്ന ബൈസന്റൈന്‍ ഭരണാധികാരികള്‍ ക്രിസ്ത്യാനികളായിരുന്നുവെങ്കിലും പഴയ നിയമപാഠങ്ങളെ പിരിഞ്ഞ് റോമന്‍ പൈതൃകത്തെ പുണര്‍ന്നിരുന്ന അവരും ഈ അവഗണന തുടര്‍ന്നു.

വിശുദ്ധ ദേവാലയ ഭൂമിയില്‍ പ്രാചീനമായ എടുപ്പുകളുടെ അവശിഷ്ടങ്ങള്‍ അനാഥമായിക്കിടന്ന ഈ കാലഘട്ടത്തിലാണ് മുഹമ്മദ് നബി(സ)യുടെ ഇസ്രാഅ് സംഭവിക്കുന്നത്; ക്രൈസ്തവര്‍ ആ സ്ഥലത്തിന്റെ വിശുദ്ധി പോലും മറന്നുതുടങ്ങിയിരുന്ന സന്ദര്‍ഭത്തില്‍! എന്നാല്‍ യഹൂദര്‍ അങ്ങനെയായിരുന്നില്ല. ദേവാലയഭൂമിയെക്കുറിച്ച വികാരസാന്ദ്രമായ ഓര്‍മകള്‍ പേറിയാണ് ഫലസ്ത്വീനില്‍നിന്ന് പുറത്താക്കപ്പെട്ട യഹൂദരുടെ തലമുറകള്‍ ലോകത്തിന്റെ വ്യത്യസ്ത ഭാഗങ്ങളില്‍ ജീവിച്ചത്. എന്തായിരുന്നു രണ്ടാം ദേവാലയം തകര്‍ക്കപ്പെട്ടതിനുശേഷം റോമന്‍-ബൈസന്റൈന്‍ അതിക്രമകാരികള്‍ ചപ്പുചവറുകള്‍ നിറച്ചവഹേളിച്ച മോറിയ കുന്നിനെക്കുറിച്ച് അവരുടെ വിശ്വാസം? ഇസ്‌ലാമിക സങ്കല്‍പം പോലെത്തന്നെ അവര്‍ക്കും അത് വിശുദ്ധസ്ഥാനം തന്നെയായി നിലനിന്നുവെന്നത് മാത്രമാണുത്തരം.

പതിനൊന്നാം നൂറ്റാണ്ടില്‍ ജീവിച്ച ലോകപ്രശസ്തനായ യഹൂദ റബ്ബി മയ്‌മോണിഡസിന്റെ (Maimonides) വാക്കുകളില്‍ പറഞ്ഞാല്‍ ”ബെയ്ത് ഹാമിക്ദാശ് ദൈവത്തിന്റെ അനശ്വര ഭവനമാണ്; ദേവാലയം തകര്‍ക്കപ്പെട്ടെങ്കിലും മോറിയാ കുന്നില്‍നിന്നും ‘ദൈവസാന്നിധ്യം’ പോയിട്ടില്ല; ‘മിശിഹ’ വരുമ്പോള്‍ അദ്ദേഹം അത് പുനര്‍നിര്‍മിക്കും.”(25) മുഹമ്മദ് നബി(സ)യുടെ മരണം കഴിഞ്ഞ് നൂറ്റാണ്ടുകള്‍ പിന്നിട്ടിട്ടും യഹൂദപരികല്‍പന അങ്ങനെതന്നെ നിലനിന്നുവെന്നണിതിനര്‍ത്ഥം. അവിടെച്ചെന്ന് ഇസ്രാഈലി പ്രവാചകന്‍മാരുടെ കൂടെ നമസ്‌കരിക്കുവാന്‍ നബി(സ)ക്ക് അല്ലാഹു അവസരമുണ്ടാക്കിക്കൊടുത്തതില്‍ പിന്നെയെന്ത് അസാംഗത്യമാണുള്ളത്?

ദേവാലയഭൂമി നഷ്ടപ്പെട്ട് പുരോഹിതാസ്ഥാനമില്ലാതായതോടെയാണ് സാധാരണക്കാര്‍ക്ക് മതം പഠിക്കാന്‍ യഹൂദ റബ്ബിമാര്‍ മിശ്‌നയും (Mishnah), ഗമറയുമെല്ലാം (Gemara) രചിച്ചുതുടങ്ങുന്ന സാഹചര്യമുണ്ടായത്. CE ആറാം നൂറ്റാണ്ടുവരെയുള്ള ഇക്കപം പണ്ഡിതരചനകളുടെ സമാഹാരമാണ് തല്‍മൂദ് (Talmud) എന്നറിയപ്പെടുന്നത്. തല്‍മൂദില്‍നിന്നുള്ള ഒരു ഭാഗം ഇപ്രകാരമാണ്: ”വിദേശത്ത് പ്രാര്‍ത്ഥനയില്‍ നില്‍ക്കുന്നവന്‍ ഹൃദയം കേന്ദ്രീകരിക്കേണ്ടത് ഇസ്രയേല്‍ രാജ്യത്തിലേക്കാണ്; കാരണം, ”അവര്‍ അങ്ങ് അവരുടെ പിതാക്കന്‍മാര്‍ക്ക് ദാനം ചെയ്ത ദേശത്തേക്കുനോക്കി അങ്ങയോടു പ്രാര്‍ത്ഥിച്ചാല്‍” (1 രാജാക്കന്‍മാര്‍ 8: 48) എന്നു പറഞ്ഞിരിക്കുന്നു. ഇസ്രായേല്‍ ദേശത്തുള്ളയാള്‍ പ്രാര്‍ത്ഥനയില്‍ ജറുസലേമിലേക്കാണ് ഹൃദയം കേന്ദ്രീകരിക്കേണ്ടത്; ‘അങ്ങ് തെരഞ്ഞെടുത്തിരിക്കുന്ന ഈ നഗരത്തിന് അഭിമുഖമായി നിന്ന് പ്രാര്‍ത്ഥിച്ചാല്‍’ (1 രാജാക്കന്‍മാര്‍ 8: 44) എന്നുമുണ്ട്. ഒരാള്‍ ജറുസലേമിലാണെങ്കില്‍ അയാര്‍ ഹൃദയം കേന്ദ്രീകരിക്കേണ്ടത് ബെയ്ത് ഹാമിക്ദാശിമിലേക്കാണ്. -‘അവര്‍ ഈ ഭവനത്തിലേക്ക് പ്രാര്‍ത്ഥിക്കണം’ (2 ദിനവൃത്താന്തം 6: 32) എന്നാണ് പറഞ്ഞിരിക്കുന്നത്.”(26)

എന്താണിതിനര്‍ത്ഥം? അതെ, ചുമരുകള്‍ താഴെ വീണാല്‍ ഇല്ലാതാകുന്നതായിരുന്നില്ല മതബോധമുള്ള ഇസ്രാഈല്യന് ബയ്തുല്‍ മക്വ്ദിസ്. കെട്ടിയുയര്‍ത്തിയ സൗധം നിലം പൊത്തിയെങ്കിലും അതിനെ ആരാധനാകേന്ദ്രമായി തന്നെ അവന്‍ കണ്ടു. വിശുദ്ധി അതുപോലെ നിലനില്‍ക്കുന്നതായി മനസ്സിലാക്കി, ലോകത്തിന്റെ പല ഭാഗങ്ങളിലേക്ക് റോമന്‍ പീഡനം കാരണമായി ചിതറേണ്ടി വന്നപ്പോഴും അവിടേക്കു തിരിഞ്ഞു തന്നെ പ്രാര്‍ത്ഥിച്ചു. ബയ്തുല്‍ മക്വ്ദിസ് എന്നുതന്നെ പലപ്പോഴും ആ മണ്ണിനെ വിളിച്ചു. ദേവാലയത്തിന്റെ കെട്ട് തകര്‍ത്തെന്നഹങ്കരിച്ചവര്‍ക്ക് ദേവാലയഭൂമിയും തങ്ങള്‍ക്ക് ദേവാലയമാണെന്ന് യഹൂദവൈകാരികത കാണിച്ചുകൊടുത്തു. അതേ അര്‍ത്ഥത്തില്‍ തന്നെയാണ് നബി(സ)യും ബയ്തുല്‍ മക്വ്ദിസില്‍ പോയെന്നു പറഞ്ഞത്; ക്വുര്‍ആന്‍ മസ്ജിദുല്‍ അക്വ്‌സ എന്ന വിശേഷണം നടത്തിയത്.

സുലയ്മാന്‍ നബി (അ) നിര്‍മിച്ച കെട്ടിടം തകര്‍ന്നതുകൊണ്ടോ ക്രൈസ്തവര്‍ പാഴ്ഭൂമിയായി ഉപേക്ഷിച്ചതുകൊണ്ടോ ബയ്തുല്‍ മക്വ്ദിസ് അതല്ലാതാവുകയില്ല എന്നതുകൊണ്ടു തന്നെ, തീര്‍ത്ഥയാത്ര പോകാവുന്ന വിശുദ്ധ കേന്ദ്രങ്ങളിലൊന്നായാണ് മുഹമ്മദ് നബി (സ) അനുചരന്‍മാര്‍ക്ക് മസ്ജിദുല്‍ അക്വ്‌സയെ പരിചയപ്പെടുത്തിക്കൊടുത്തത്. അദ്ദേഹം പറഞ്ഞു: ”മൂന്ന് മസ്ജിദുകള്‍ സന്ദര്‍ശിക്കാനല്ലാതെ നിങ്ങള്‍ തീര്‍ത്ഥയാത്ര പോകരുത്. ഈ എന്റെ മസ്ജിദ് (മദീനയിലെ മസ്ജിദുന്നബവി), (മക്കയിലെ) മസ്ജിദുല്‍ ഹറാം, മസ്ജിദുല്‍ അക്വ്‌സ.”(27)

മുസ്‌ലിംകളുടെ തീര്‍ത്ഥാടനകേന്ദ്രമായി നിശ്ചയിക്കപ്പെട്ടതുകൊണ്ടു തന്നെ രണ്ടാം ഖലീഫ ഉമറിന്റെ (റ) കാലത്ത് ഫലസ്ത്വീന്‍ ബൈസന്റൈന്‍കാരില്‍നിന്ന് മുസ്‌ലിം സൈന്യത്തിന്റെ നിയന്ത്രണത്തിലേക്കു വന്നപ്പോള്‍ ഉമര്‍ (റ) അവിടം സന്ദര്‍ശിക്കുകയും വൃത്തിയാക്കുകയും നമസ്‌കാര സൗകര്യമുളള ഒരു കെട്ടിടം നിര്‍മിക്കുകയും ചെയ്തു. ആ കെട്ടിടമല്ല മസ്ജിദുല്‍ അക്വ്‌സ, പ്രത്യുത മസ്ജിദുല്‍ അക്വ്‌സയായി ക്വുര്‍ആന്‍ വിശേഷിപ്പിച്ച സ്ഥലത്ത് ഒരു ഭാഗത്തായി നിര്‍മിക്കപ്പെട്ട ഒരു സൗകര്യവും സംവിധാനവും മാത്രമായിരുന്നു പ്രസ്തുത കെട്ടിടം.(28) അതിനു പിന്നീടുണ്ടായ രൂപാന്തരങ്ങളോ വികാസങ്ങളോ ചുരുക്കങ്ങളോ ഒന്നുമല്ല മസ്ജിദുല്‍ അക്വ്‌സയുടെ അകവും പുറവും നിര്‍ണയിക്കുന്നത്. കഅ്ബക്ക് ചുറ്റും നമസ്‌കരിക്കുന്നതിനായി ഉമവികളോ അബ്ബാസികളോ തുര്‍ക്കികളോ സുഊദികളോ പണികഴിപ്പിച്ച കെട്ടിടങ്ങളല്ല  മറിച്ച് ആ സ്ഥലമാണ് മസ്ജിദുല്‍ ഹറാം എന്നതുപോലെത്തന്നെ. ഇതുമനസ്സിലാക്കാതെയാണ് വിമര്‍ശകര്‍ മസ്ജിദുല്‍ അക്വ്‌സയും ഉമര്‍(റ) നിര്‍മിച്ച പള്ളിയും ഒന്നാണെന്ന് തെറ്റിദ്ധരിക്കുന്നത്.

ശ്രദ്ധിക്കുക: ഇസ്രാഅ് വിവരണങ്ങളില്‍ വിശ്വസിച്ച, ബയ്ത്തുല്‍ മക്വ്ദിസിലേക്ക് തിരിഞ്ഞ് ഒന്നര വര്‍ഷത്തോളം നബി(സ)ക്കൊപ്പം നമസ്‌കരിച്ച, മസ്ജിദുല്‍ അക്വ്‌സ തീര്‍ത്ഥാടന കേന്ദ്രമായി പ്രവാചകനാല്‍ പഠിപ്പിക്കപ്പെട്ട ഉമര്‍ (റ) ഫലസ്ത്വീനില്‍ എത്തിയപ്പോള്‍ മസ്ജിദുല്‍ അക്വ്‌സ എന്ന പേരില്‍ കറതീര്‍ന്ന ഒരു കെട്ടിടം കാണാതെ അത്ഭുത പരതന്ത്രനായിട്ടില്ല. ആരാധനകള്‍ക്ക് സൗകര്യപ്പെടുംവിധമുള്ള കെട്ടിടങ്ങളൊന്നും മസ്ജിദുല്‍ അക്വ്‌സയുടെ സ്ഥാനത്ത് നിലവിലില്ലെന്നു അറിയാവുന്നതുകൊണ്ട് സന്ദര്‍ശനാവസരത്തില്‍ തികച്ചും സ്വാഭാവികമായി പെരുമാറുകയും അവിടെയൊരു പള്ളി നിര്‍മിക്കാന്‍ മുന്‍കയ്യെടുക്കുകയുമാണ് അദ്ദേഹം ചെയ്തത്.

അതെ, ഒന്നാം ടെമ്പിളോ, രണ്ടാം ടെമ്പിളോ ഫലസ്ത്വീനില്‍ നില്‍ക്കുന്നതായി പ്രവാചകനോ അനുചരന്‍മാരോ ഒന്നും മനസ്സിലാക്കിയിരുന്നില്ല; അവയെ അല്ല അവരാരും മസ്ജിദുല്‍ അക്വ്‌സ എന്നോ ബയ്ത്തുല്‍ മക്വ്ദിസ് എന്നോ വിളിച്ചത്. അതിനാല്‍ ഇസ്രാഅ് വിവരണത്തില്‍ കാലാനുചിതത്വമുെണ്ടന്ന ആരോപണത്തില്‍ യാതൊരു കഴമ്പുമില്ല

കുറിപ്പുകള്‍

  1. ക്വുര്‍ആന്‍ 17: 1.
  2. ബുഖാരി, സ്വഹീഹ്/കിതാബു മനാകിബില്‍ അന്‍സ്വാര്‍.
  3. മുസ്‌ലിം, സ്വഹീഹ്/കിതാബുല്‍ ഈമാന്‍.
  4. മുസ്‌ലിം, സ്വഹീഹ്/കിതാബുല്‍ ഈമാന്‍.
  5. മുസ്‌ലിം, സ്വഹീഹ്/കിതാബുല്‍ ഈമാന്‍.
  6. ഹാകിം, മുസ്തദ്‌റക് അലസ്സ്വഹീഹയ്ന്‍. see Ali Muhammad As-Sallabee, The Noble life of the Prophet (Riyadh. Darussalam, 2005), vol. 1, pp 552-3.
  7. ബുഖാരി, സ്വഹീഹ്/കിതാബു ബദീല്‍ ഹല്‍ക്വ്.
  8. ക്വുര്‍ആന്‍ 2: 143-145.
  9. ബുഖാരി, സ്വഹീഹ്/കിതാബുല്‍ ഈമാന്‍.
  10. S.Moinul Haq, Ibn sa’d’s Al Tabaqat Al Kabir (New Delhi. kitab bhavan, 2009), vol. 1, 284.
  11. ബുഖാരി, സ്വഹീഹ്/കിതാബുല്‍ ഈമാന്‍.
  12. ബുഖാരി, സ്വഹീഹ്/കിതാബുല്‍ അമ്പിയാഅ്.
  13. ക്വുര്‍ആന്‍ 3: 96.
  14. അഹ്മദ് അന്നസാഇ, സുനനുസ്സുഗ്‌റ/കിതാബുല്‍ മസാജിദ്; യസീദ് ഇബ്‌നു മാജ, സുനന്‍/ കിതാബു ഇക്വാമതിസ്സ്വലാതി വസ്സുന്നതു ഫീഹാ.
  15. ബൈബിള്‍; ഉല്‍പത്തി 22: 2
  16. Rabbi Menachan M Sohneerson, ‘G-d’s chosen House-www,chabadozy
  17. see Jochua Hammes, ‘what is beneadh the temple mount?’, www.smithsonianmag.com
  18. ഉല്‍പത്തി 22: 2.ബൈബിള്‍, 1 ദിനവൃത്താന്തം അധ്യായങ്ങള്‍ 28, 29. 2 ദിനവൃത്താന്തം അധ്യായങ്ങള്‍ 1 – 7.
  19. www.chabad.org.op.cit
  20. see Temple Mount, the Temple Jerusalem during first Temple, second Temple of in www.jeuishviofnallibray.org
  21. ബുഖാരി, സ്വഹീഹ്/കിതാബുല്‍ മസാജിദി വ മവാദിഇ സ്സ്വലാത്ത്.
  22. ക്വുര്‍ആന്‍ 2: 127.
  23. ക്വുര്‍ആന്‍ 14: 37.
  24. See www.chabad.org.op.cit
  25. The William Davidson Talmud, berakhot Daf 30 a.

അബ്രഹാം പ്രവാചകന്‍ ഭാര്യ ഹാജറിനെയും പുത്രന്‍ ഇസ്മാഈലിനെയും മക്കയില്‍ കൊണ്ടുചെന്നാക്കിയെന്നും ഇസ്മാഈലിനെയും കൂട്ടി അവിടെ കഅ്ബ സ്ഥാപിച്ചുവെന്നും അതിനു ചുറ്റുമാണ് മക്കന്‍ നാഗരികത വളര്‍ന്നുവന്നതെന്നുമുള്ള മുഹമ്മദ് നബി(സ)യുടെ വിശദീകരണങ്ങള്‍ അടിസ്ഥാനരഹിതമാണെന്ന് ബൈബിള്‍ തെളിയിക്കുന്നുവെന്ന മിഷനറിമാരുടെ വാദത്തില്‍ എന്തെങ്കിലും കഴമ്പുണ്ടോ? ഹാജറും ഇസ്മാഈലും ഇബ്രാഹീമിനാല്‍ ഉപേക്ഷിക്കപ്പെട്ടത് മക്കയിലല്ലെന്ന് ബൈബിള്‍ വചനങ്ങളില്‍ നിന്ന് വ്യക്തമാകുന്നില്ലേ?

ല്ല. അബ്രഹാം ഹാജറിനെയും ഇസ്മാഈലിനെയും കൊണ്ടുപോയി പാര്‍പ്പിച്ചത് മക്കയിലല്ലെന്ന് ബൈബിള്‍ വചനങ്ങളില്‍ നിന്ന് മനസ്സിലാകുന്നുണ്ടെന്നും അതിനാല്‍ ഇബ്‌റാഹീമും ഇസ്മാഈലും ചേര്‍ന്നാണ് കഅ്ബ നിര്‍മിച്ചതെന്ന പ്രവാചകന്‍(സ)യുടെ അധ്യാപനം അടിസ്ഥാനരഹിതമാണെന്നും സമര്‍ത്ഥിക്കുവാനാണ് മിഷനറിമാര്‍ ശ്രമിച്ചുവരാറുള്ളത്. കഅ്ബയുടെ അബ്രഹാമിക പശ്ചാത്തലം മുഹമ്മദ് നബി(സ)ക്ക് നൂറ്റാണ്ടുകള്‍ക്കു മുമ്പു തന്നെ അറബികള്‍ക്കും അവരെ പരിചയമുണ്ടായിരുന്നവര്‍ക്കും ബോധ്യമുണ്ടായിരുന്നതാണെന്നും കഅ്ബ ഏകദൈവാരാധനക്കുവേണ്ടി അബ്രഹാം സ്ഥാപിച്ചതാണെന്ന വസ്തുതയെ നിരാകരിക്കുന്ന യാതൊരു ചരിത്രരേഖയുമില്ലെന്നും നാം നേരത്തെ കണ്ടുകഴിഞ്ഞു. മുഹമ്മദ് നബി (സ) പുതുതായി അവതരിപ്പിച്ച ഒരു വാദത്തെയല്ല, മറിച്ച് അറബികള്‍ക്കും ചരിത്രത്തിനും തര്‍ക്കമില്ലാത്ത ഒരു യാഥാര്‍ത്ഥ്യത്തെയാണ് മിഷനറിമാര്‍ ബൈബിളുപയോഗിച്ച് അട്ടിമറിക്കാന്‍ ശ്രമിക്കുന്നത് എന്നാണിതിനര്‍ത്ഥം. കഅ്ബ ഇബ്‌റാഹീമും ഇസ്മാഈലും ചേര്‍ന്ന് സ്ഥാപിച്ചതല്ലെന്ന് പറയാന്‍ മിഷനറിമാരുടെ കയ്യില്‍ ബൈബിളല്ലാത്ത യാതൊരു പ്രണാണവുമില്ല. അതുകൊണ്ടുതന്നെ, ബൈബിള്‍ പ്രമാദങ്ങളില്‍ നിന്ന് പൂര്‍ണമായും മുക്തമായ ചരിത്രസ്രോതസ്സാണെങ്കില്‍ മാത്രമേ ഈ വാദത്തിന് എന്തെങ്കിലും പ്രസക്തിയുണ്ട് എന്നു പറയാനാകൂ.

ക്വുര്‍ആന്‍ പ്രപഞ്ചനാഥനായ അല്ലാഹു മുഹമ്മദ് നബി(സ)ക്ക് അവതരിപ്പിച്ചുകൊടുത്ത വേദഗ്രന്ഥമാണെന്നാണ് ക്വുര്‍ആനും മുഹമ്മദ് നബി(സ)യും പഠിപ്പിച്ചിട്ടുള്ളത്; മുസ്‌ലിം ലോകം നാളിതുവരെയായി അങ്ങനെയാണ് മനസ്സിലാക്കി വന്നിട്ടുള്ളതും. ബൈബിള്‍ ക്രൈസ്തവരുടെ വേദഗ്രന്ഥമാണെന്നാണ് പറയപ്പെടാറുള്ളതെങ്കിലും അത് പ്രപഞ്ചരക്ഷിതാവിന്റെ വചനങ്ങളുടെ സമാഹാരമാണെന്ന് ബൈബിളെഴുത്തുകാര്‍ക്കോ ക്രൈസ്തവലോകത്തിനുപോലുമോ അവകാശവാദമില്ല.

പ്രവാചകന്‍മാരുടെയും ഇസ്രയേല്‍ സമൂഹത്തിന്റെയും ചരിത്രം പല കാലങ്ങളിലായി മനുഷ്യര്‍ എഴുതിവെച്ചതിന്റെ സമാഹാരമാണ് ബൈബിള്‍ പഴയ നിയമം. ആ ചരിത്രം പറഞ്ഞുപോകുമ്പോള്‍ പ്രവാചകവചനങ്ങളായി അവരുടെ ഓര്‍മയിലുള്ള പലതും ബൈബിളില്‍ കടന്നുവരുന്നുണ്ടെന്ന് മാത്രമേയുള്ളൂ. അതാത് പ്രവാചകന്‍മാരുടെ മരണം കഴിഞ്ഞ് കാലങ്ങള്‍ പിന്നിട്ടതിനുശേഷമാണ് ബൈബിള്‍ പുസ്തകങ്ങള്‍ പലതും രചിക്കപ്പെട്ടത് എന്നതുകൊണ്ടുതന്നെ, ബൈബിളില്‍ നമുക്കിന്ന് വായിക്കാന്‍ കഴിയുന്ന പ്രവാചകചരിത്രത്തിലും അതിന്റെ ഭാഗമായി ഉദ്ധരിക്കപ്പെട്ടിരിക്കുന്ന പ്രവാചക വചനങ്ങളിലും സ്ഖലിതങ്ങളുണ്ടാകാനുള്ള സാധ്യത വളരെയധികമാണ്. ഇങ്ങനെ സ്ഖലിത സാധ്യതകളോടുകൂടി എഴുതപ്പെട്ടു എന്നതിനുപുറമെ, എഴുതപ്പെട്ടതിനുശേഷം പില്‍ക്കാലക്കാരുടെ തിരുത്തലുകള്‍ക്ക് നിരന്തരമായി വിധേയമായി എന്നത് ബൈബിള്‍ വിവരണങ്ങളുടെ ആധികാരികതയെ പിന്നെയും സംശയാസ്പദമാക്കുന്നു. സത്യസന്ധരായ ബൈബിള്‍ പണ്ഡിതന്‍മാര്‍ക്കിടയില്‍ അഭിപ്രായവ്യത്യാസങ്ങളില്ലാത്ത വസ്തുതകളാണിവയെല്ലാം. ബൈബിള്‍ പറയുന്നു എന്നതുകൊണ്ടുമാത്രം ഒരു കാര്യം ശരിയോ തെറ്റോ ആകണമെന്നില്ല എന്നു തന്നെയാണ് ഇതിന്റെയര്‍ത്ഥം.

ബൈബിള്‍ പറയുന്ന ചരിത്രത്തില്‍ അനേകം അബദ്ധങ്ങളുണ്ടെന്ന് ഇതിനകം ചരിത്രഗവേഷകര്‍ തെളിയിച്ചു കഴിഞ്ഞിട്ടുണ്ട്. ഇബ്‌റാഹീം ഹാജറിനെയും ഇസ്മാഈലിനെയും കൊണ്ടുപോയി പാര്‍പ്പിച്ചത് മക്കയിലല്ലെന്ന് ബൈബിള്‍ പറയുന്നതുകൊണ്ടു മാത്രം അത് മക്കയിലല്ല എന്നു വരികയില്ലെന്ന് ചുരുക്കം. ‘ഞങ്ങളുടെ ബൈബിള്‍ പറയുന്നു; അതിനാല്‍ ലോകം അത് അംഗീകരിക്കണം’ എന്നു പറയുന്ന ഭക്തിയുടെയും വിശ്വാസത്തിന്റെയും ഭാഷ മിഷനറിമാരുടെ മനസ്സമാധാനത്തിനു മാത്രമേ ഉപകരിക്കൂ. ബൈബിള്‍ കുറ്റമറ്റ ചരിത്രസ്രോതസ്സാണെന്നും അതില്‍ യാതൊരുവിധ അബദ്ധവുമില്ലെന്നും തെളിയിക്കുവാന്‍ മിഷനറിമാര്‍ സന്നദ്ധമാകാത്തിടത്തോളം കാലം സത്യാന്വേഷികള്‍ അതിന് യാതൊരു വിലയും കല്‍പിക്കുകയില്ല.

ഇബ്‌റാഹീം നബി(അ)യുടെയും അദ്ദേഹത്തിന്റെ കുടുംബത്തിന്റെയും മക്കാ ബന്ധം നിഷേധിക്കുവാന്‍ മിഷനറിമാര്‍ ആശ്രയിക്കുന്നത് പഴയ നിയമത്തിലെ പഞ്ചപുസ്തകങ്ങളെയാണ്; കുറേക്കൂടി കൃത്യമായി പറഞ്ഞാല്‍ അവയില്‍ ആദ്യത്തേതായ ഉല്‍പത്തി പുസ്തകത്തെ. ഉല്‍പത്തി, പുറപ്പാട്, സംഖ്യ, ലേവ്യ, ആവര്‍ത്തനം എന്നിവയടങ്ങുന്ന പഞ്ചഗ്രന്ഥി തോറയാണെന്ന് സാമാന്യമായി പലരും പറഞ്ഞുപോകാറുണ്ടെങ്കിലും മോശെ പ്രവാചകനവതരിപ്പിക്കപ്പെട്ട തോറ അതേപടി സംരക്ഷിക്കപ്പെട്ടതല്ല, മറിച്ച് അതിലേക്ക് പല കാലങ്ങളിലായി പലതും പുരോഹിതന്‍മാര്‍ സ്വധാരണകള്‍ക്കനുസരിച്ച് എഴുതിച്ചേര്‍ത്തുണ്ടാക്കിയതാണ് പഞ്ചപുസ്തകങ്ങളുടെ ഉള്ളടക്കമെന്ന് ബൈബിള്‍ പണ്ഡിതന്‍മാര്‍ തന്നെ വ്യക്തമാക്കിയിട്ടുണ്ട്. ആവര്‍ത്തന പുസ്തകത്തിലെ മുപ്പത്തിനാലാം അധ്യായത്തില്‍ മോശെയുടെ മരണത്തെക്കുറിച്ചുള്ള പ്രതിപാദനങ്ങള്‍ പോലുമുണ്ട്. മോശെക്ക് കര്‍ത്താവ് അവതരിപ്പിച്ചുകൊടുത്ത വചനങ്ങള്‍ അപ്പടി പരിരക്ഷിക്കപ്പെടുന്നതല്ല പഞ്ചഗ്രന്ഥങ്ങളെന്ന് ഇതില്‍ നിന്നുതന്നെ വ്യക്തമാണ്. മോശെ പ്രവാചകനുശേഷവും പഞ്ചപുസ്തകങ്ങളില്‍ പലതും എഴുതിച്ചേര്‍ക്കപ്പെട്ടുവെന്ന് ചുരുക്കം.

അബ്രഹാമിനുശേഷം നൂറ്റാണ്ടുകള്‍ കഴിഞ്ഞ് ജീവിച്ച മോശെയുടെ മരണശേഷവും പുരോഹിതന്‍മാര്‍ വചനങ്ങള്‍ എഴുതിച്ചേര്‍ത്തിട്ടുള്ള ഒരു പുസ്തകത്തിലെ പരാമര്‍ശങ്ങള്‍ മാത്രം വെച്ച് അബ്രഹാം ഹാഗറിനെ കൊണ്ടുചെന്നാക്കിയത് മക്കയിലല്ല എന്നു തീരുമാനിക്കാന്‍ കഴിയുക എങ്ങനെയാണ്? പരിശുദ്ധ ക്വുര്‍ആന്‍ പറഞ്ഞതെത്ര ശരിയാണ്! ”വേദക്കാരേ, ഇബ്‌റാഹീമിന്റെ കാര്യത്തില്‍ നിങ്ങളെന്തിനാണ് തര്‍ക്കിക്കുന്നത്? തൗറാത്തും ഇന്‍ജീലും അവതരിപ്പിക്കപ്പെട്ടത് അദ്ദേഹത്തിനു ശേഷം മാത്രമാണല്ലോ. നിങ്ങളെന്താണ് ചിന്തിക്കാത്തത്?  ഹേ; കൂട്ടരേ, നിങ്ങള്‍ക്ക് അറിവുള്ള കാര്യത്തെപ്പറ്റി നിങ്ങള്‍ തര്‍ക്കിച്ചു. ഇനി നിങ്ങള്‍ക്ക് അറിവില്ലാത്ത വിഷയത്തില്‍ നിങ്ങളെന്തിന്ന് തര്‍ക്കിക്കുന്നു? അല്ലാഹു അറിയുന്നു, നിങ്ങള്‍ അറിയുന്നില്ല. ഇബ്രാഹീം യഹൂദനോ ക്രിസ്ത്യനോ ആയിരുന്നില്ല. എന്നാല്‍ അദ്ദേഹം ഹനീഫും മുസ്‌ലിമും ആയിരുന്നു. അദ്ദേഹം ബഹുദൈവാരാധകരില്‍ പെട്ടവനായിരുന്നിട്ടുമില്ല. തീര്‍ച്ചയായും ജനങ്ങളില്‍ ഇബ്രാഹീമിനോട് കൂടുതല്‍ അടുപ്പമുള്ളവര്‍ അദ്ദേഹത്തെ പിന്തുടര്‍ന്നവരും, ഈ പ്രവാചകനും, (അദ്ദേഹത്തില്‍) വിശ്വസിച്ചവരുമാകുന്നു. അല്ലാഹു സത്യവിശ്വാസികളുടെ രക്ഷാധികാരിയാകുന്നു.” (ക്വുര്‍ആന്‍ 3 : 65-68)

ഹാജറിന്റെയും ഇസ്മാഈലിന്റെയും പാലായനമുണ്ടായത് മക്കയിലേക്കല്ലെന്ന് ബൈബിള്‍ പുസ്തകങ്ങളുടെ മാത്രം അടിസ്ഥാനത്തില്‍ വാദിക്കുന്നത് അര്‍ത്ഥശൂന്യമാണെന്ന് നമുക്ക് വ്യക്തമായി. ഇബ്‌റാഹീമിന്റെ ജീവിതത്തിലെ അറേബ്യന്‍ അധ്യായങ്ങള്‍ ബൈബിള്‍ നിരാകരിക്കുന്നുണ്ടോ എന്നാണ് ഇനി പരിശോധിക്കാനുള്ളത്. ഇശ്മയേലിന്റെ പുത്രനായി ബൈബിള്‍ പരിചയപ്പെടുത്തുന്ന കേദാറിന്റെ (ഉല്‍പത്തി 25 : 13, ദിനവൃത്താന്തം 1 : 29) സന്തതിപരമ്പരകള്‍ അറേബ്യയിലാണ് നിവസിക്കുന്നതെന്ന് ബൈബിളെഴുത്തുകാര്‍ തന്നെ സൂചിപ്പിക്കുന്നുണ്ടെന്നതാണ് വാസ്തവം. യെശയ്യാ പ്രവാചകന്റെ പുസ്തകത്തില്‍ ഇപ്പോള്‍ ‘അറേബ്യയെക്കുറിച്ചുള്ള സന്ദേശം’ എന്ന തലക്കെട്ടോടുകൂടി പ്രസിദ്ധീകരിക്കപ്പെട്ടുവരുന്ന വചനങ്ങള്‍ വായിക്കുക: ”അറേബ്യയെക്കുറിച്ചുളള അരുള്‍പാട്: ദദാന്യരായ സാര്‍ഥവാഹകരേ, നിങ്ങള്‍ അറേബ്യയിലെ കുറ്റിക്കാട്ടില്‍ വസിക്കും. തേമാന്യരേ, നിങ്ങള്‍ ദാഹിക്കുന്നവര്‍ക്ക് ജലം നല്‍കുവിന്‍, പാലായനം ചെയ്യുന്നവര്‍ക്ക് അപ്പം കൊടുക്കുവിന്‍. എന്തെന്നാല്‍, അവര്‍ ഊരിയ വാളില്‍ നിന്നും കുലച്ച വില്ലില്‍ നിന്നും യുദ്ധത്തിന്റെ നടുവില്‍ നിന്നും രക്ഷപെട്ട് ഓടുന്നവരാണ്. കര്‍ത്താവ് എന്നോട് അരുളിചെയ്തു: കൂലിക്കാരന്‍ കണക്കാക്കുന്നതുപോലെ, കണിശം ഒരു വര്‍ഷത്തിനുള്ളില്‍ കേദാറിന്റെ സര്‍വ മഹത്വവും നശിക്കും. കേദാറിന്റെ വില്ലാളിവീരന്‍മാരില്‍ ചുരുക്കം ചിലര്‍മാത്രം അവശേഷിക്കും. ഇസ്രായേലിന്റെ ദൈവമായ കര്‍ത്താവാണ് അരുളിചെയ്തിരിക്കുന്നത്.” (എശയ്യ 21 : 13-17).

ഫലസ്ത്വീനു പുറത്തുള്ള അറേബ്യന്‍ മരുപ്രദേശങ്ങളില്‍ വ്യാപിച്ചുകിടക്കുന്നവരായാണ് ഇസ്മാഈലിന്റെ സന്തതിപരമ്പരകളെ ബൈബിളെഴുത്തുകാര്‍ മനസ്സിലാക്കിയിരുന്നതെന്ന് ഉല്‍പത്തി പുസ്തകം പൂര്‍ണമായി പരിശോധിച്ചാല്‍ തന്നെ മനസ്സിലാകും. ബൈബിള്‍ പരാമര്‍ശിക്കുന്ന ഇസ്മാഈല്‍ സന്തതികളില്‍പ്പെട്ടവരാണ് മക്കയില്‍ നിവസിക്കുന്നതെന്ന് അവരെ പരിചയമുണ്ടായിരുന്ന ജൂതന്‍മാര്‍ക്കും ക്രൈസ്തവര്‍ക്കും ബോധ്യമുണ്ടായിരുന്നതുകൊണ്ടാണ് മദീനയിലെ ജൂതന്‍മാര്‍ മുതല്‍ പ്രാചീന റോമില്‍ ജീവിച്ച ജോസിഫസിനും സോസിമേമസിനും വരെ മക്കയുടെ അബ്രഹാമിക പൈതൃകത്തെ സംബന്ധിച്ച് സംശയങ്ങളുണ്ടാകാതിരുന്നത്. മിഷനറിമാര്‍ നൂറുശതമാനം ആധികാരികമെന്ന് വിശ്വസിക്കുന്ന ബൈബിള്‍ വിവരണങ്ങള്‍പോലും അറേബ്യയുടെ ഇസ്മാഈലി വേരുകളിലേക്ക് വിരല്‍ ചൂണ്ടുന്നുണ്ടെന്ന് സാരം.

‘ഇശ്മയേല്‍ പാറാനിലെ മരുഭൂമിയില്‍ പാര്‍ത്തു’ എന്ന ബൈബിള്‍ വചനമാണ് (ഉല്‍പത്തി 21 : 20) ഇസ്മാഈലും ഹാജറും പാര്‍പ്പുറപ്പിച്ചത് മക്കയിലല്ലെന്ന് ബൈബിള്‍ സൂചിപ്പിക്കുന്നുവെന്ന് പറയാനായി മിഷനറിമാര്‍ ഉപയോഗിക്കാറുള്ളത്. ‘പാറാന്‍’ എന്ന പ്രയോഗം ബൈബിളില്‍ പലയിടങ്ങളിലായി കാണാന്‍ കഴിയും. അവിടെയെല്ലാം ബൈബിള്‍ ഉദ്ദേശിച്ചത് ഇന്ന് നമുക്ക് പരിചയമുള്ള ഏത് പ്രദേശത്തെയാണ് എന്ന് കൃത്യമായി നിര്‍ണയിക്കുവാന്‍ യാതൊരു നിര്‍വാഹവുമില്ലെന്നതാണ് വാസ്തവം. ഇശ്മയേല്‍ പാര്‍ത്ത പാറാന്‍ ഏതാണെന്ന കാര്യത്തിലും ബൈബിള്‍ പണ്ഡിതന്‍മാര്‍ക്കിടയില്‍ അനേകം അഭിപ്രായവ്യത്യാസങ്ങളുണ്ട്. ‘പാറാന്‍’ എന്ന, ഉദ്ദേശ്യം എന്താണെന്ന് വ്യക്തമല്ലാത്ത, അനേകം അര്‍ത്ഥസാധ്യതകള്‍ അവശേഷിപ്പിക്കുന്ന അവ്യക്തമായ പ്രയോഗമാണ് ഇസ്മാഈല്‍ വളര്‍ന്ന പ്രദേശത്തെക്കുറിച്ച് ബൈബിള്‍ നടത്തുന്നതെന്നര്‍ത്ഥം. പാറാന്‍ കൊണ്ട് വിവക്ഷിക്കപ്പെട്ടത് ഇന്ന പ്രദേശമാണെന്ന് തറപ്പിച്ചു പറയാന്‍ നിര്‍വാഹങ്ങളൊന്നുമില്ലെന്നിരിക്കെ, ബൈബിളെഴുത്തുകാരന്‍ ഉദ്ദേശിച്ചത് മക്കയല്ലെന്ന് മിഷനറിമാര്‍ക്ക് തീര്‍ച്ചപ്പെടുത്താന്‍ കഴിയുന്നതെങ്ങനെയാണ്? പാറാന്‍ ഒരു മരുഭൂമിയാണെന്ന അധികവിവരം മാത്രമാണ് പരാമൃഷ്ട ബൈബിള്‍ വചനത്തില്‍ നിന്നു ലഭിക്കുന്നത്. ആ വിവരമാകട്ടെ, പാറാന്‍ മക്കയാകാനുള്ള സാധ്യതയെ ശക്തിപ്പെടുത്തുക മാത്രമാണ് ചെയ്യുന്നത്.

ഫലസ്ത്വീനിലെ ‘ബിഅ്ര്‍ ശബ’യാണ് പാറാന്‍ കൊണ്ട് ഉദ്ദേശിക്കപ്പെട്ടതെന്ന് ബൈബിള്‍ വചനങ്ങള്‍ തന്നെ വ്യക്തമാക്കുന്നതായി ചില മിഷനറിമാര്‍ വാദിച്ചുനോക്കാറുണ്ട്. പാറാന്‍ മരുഭൂമിയെക്കുറിച്ച് പറയുന്നതിനു തൊട്ടുമുമ്പ് ബൈബിള്‍ ഹാജറിന്റെ ബിഅ്ര്‍ ശബ അനുഭവങ്ങളെക്കുറിച്ച് പറയുന്നുവെന്നതാണ് അവരുടെ ന്യായം. വചനങ്ങള്‍ ഇപ്രകാരമാണ്: ”അബ്രഹാം അതിരാവിലെ എഴുന്നേറ്റ് കുറേ അപ്പവും ഒരു തുകല്‍ സഞ്ചിയില്‍ വെള്ളവുമെടുത്ത് ഹാഗറിന്റെ തോളില്‍ വെച്ചുകൊടുത്തു. മകനെയും ഏല്‍പിച്ചിട്ട് അവളെ പറഞ്ഞയച്ചു. അവള്‍ അവിടെ നിന്നുപോയി. ബിഅ്ര്‍ ശബ മരുപ്രദേശത്ത് അലഞ്ഞുനടന്നു. തുകല്‍ സഞ്ചിയിലെ വെള്ളം തീര്‍ന്നപ്പോള്‍ അവള്‍ കുട്ടിയെ ഒരു കുറ്റിക്കാട്ടില്‍ കിടത്തി. കുഞ്ഞ് മരിക്കുന്നത് എനിക്കു കാണാന്‍ വയ്യ എന്നുപറഞ്ഞ് അവള്‍ കുറേ അകലെ, ഒരു അമ്പെയ്ത്ത് ദൂരെച്ചെന്ന് എതിര്‍വശത്തേക്ക് തിരിഞ്ഞിരുന്നു. കുട്ടി ഉച്ചത്തില്‍ കരയാന്‍ തുടങ്ങി. കുട്ടിയുടെ കരച്ചില്‍ ദൈവം കേട്ടു. സ്വര്‍ഗത്തില്‍ നിന്ന് ദൈവത്തിന്റെ ദൂതന്‍ അവളെ വിളിച്ചുപറഞ്ഞു: ഹാഗാര്‍, നീ വിഷമിക്കേണ്ടാ; ഭയപ്പെടുകയും വേണ്ട. കുട്ടിയുടെ കരച്ചില്‍ ദൈവം കേട്ടിരിക്കുന്നു. എഴുന്നേറ്റ് കുട്ടിയെ കയ്യിലെടുക്കുക. അവനില്‍ നിന്ന് ഞാന്‍ വലിയൊരു ജനതയെ പുറപ്പെടുവിക്കും. ദൈവം അവളുടെ കണ്ണുതുറന്നു. അവള്‍ ഒരു കിണര്‍ കണ്ടു. അവള്‍ ചെന്ന് തുകല്‍ സഞ്ചി നിറച്ച്, കുട്ടിക്ക് കുടിക്കാന്‍ കൊടുത്തു. ദൈവം ആ കുട്ടിയോടു കൂടെയുണ്ടായിരുന്നു. അവന്‍ മരുഭൂമിയില്‍ പാര്‍ത്തു. അവന്‍ വളര്‍ന്ന് സമര്‍ത്ഥനായൊരു വില്ലാളിയായിത്തീര്‍ന്നു. അവര്‍ പാറാനിലെ മരുഭൂമിയില്‍ പാര്‍ത്തു.” (ഉല്‍പത്തി 21 : 14-21).

ബിഅ്ര്‍ ശബയാണ് പാറാന്‍ എന്ന വാദം ഈ ബൈബിള്‍ ഖണ്ഡികയില്‍ നിന്ന് നിര്‍ധരിച്ചെടുക്കുന്നത് അബദ്ധമാണ് എന്നാണ് സൂഷ്മ വായനയില്‍ ബോധ്യപ്പെടുന്നത്. ഈ വാചകങ്ങള്‍ക്ക് തൊട്ടുമുകളിലുള്ള ബൈബിള്‍ വചനങ്ങള്‍ കൂടി പരിശോധിക്കുക: ”ഈജിപ്തുകാരിയായ ഹാഗാറില്‍ അബ്രഹാത്തിന് ജനിച്ച മകന്‍, തന്റെ മകനായ ഇസ്ഹാഖിനോടുകൂടെ കളിക്കുന്നത് സാറ കണ്ടു. അവള്‍ അബ്രഹാത്തോടു പറഞ്ഞു: ആ അടിമപ്പെണ്ണിനെയും അവളുടെ മകനെയും ഇറക്കിവിടുക. അവളുടെ മകന്‍ എന്റെ മകന്‍ ഇസ്ഹാഖിനോടൊപ്പം അവകാശിയാകാന്‍ പാടില്ല. തന്മൂലം മകനെയോര്‍ത്ത് അബ്രഹാം വളരെ അസ്വസ്ഥനായി. എന്നാല്‍, ദൈവം അബ്രഹാത്തിനോട് അരുളിചെയ്തു: കുട്ടിയെക്കുറിച്ചും നിന്റെ അടിമപ്പെണ്ണിനെക്കുറിച്ചും നീ ക്ലേശിക്കേണ്ട. സാറാ പറയുന്നതുപോലെ നീ ചെയ്യുക. കാരണം ഇസ്ഹാഖിലൂടെയാണ് നിന്റെ സന്തതികള്‍ അറിയപ്പെടുക. അടിമപ്പെണ്ണില്‍ ജനിച്ച മകനെയും ഞാനൊരു ജനതയാക്കും. അവനും നിന്റെ മകനാണല്ലോ.” (ഉല്‍പത്തി 21 : 9-13).

ഇശ്മയേലിനോടും ഹാജറിനോടും സാറയ്ക്ക് അസഹിഷ്ണുത തോന്നിയെന്നും തന്റെയും ഇസ്ഹാഖിന്റെയും ജീവിതപരിസരങ്ങളില്‍ നിന്ന് ഹാജറിനെയും ഇശ്മയേലിനെയും പുറത്താക്കാന്‍ സാറ അബ്രഹാമിനോട്  ആവശ്യപ്പെട്ടുവെന്നും ഇതില്‍ അസ്വസ്ഥനായ അബ്രഹാമിനെ, സാറയുടെ ആവശ്യം രണ്ട് വ്യത്യസ്ത ഭൂപ്രദേശങ്ങളില്‍ അബ്രഹാമിന്റെ രണ്ട് സന്തതിശാഖകള്‍ മഹാജനസഞ്ചയങ്ങളായി മാറുക എന്ന ദൈവിക പദ്ധതിയുടെ നിവൃത്തിയായി മാറുമെന്ന് പറഞ്ഞ് കര്‍ത്താവ് ആശ്വസിപ്പിച്ചുവെന്നും തദടിസ്ഥാനത്തില്‍ ഇസ്ഹാഖും സാറയും താമസിച്ചിരുന്ന നാട്ടില്‍ നിന്ന് വിദൂരവും സുരക്ഷിതവുമായ മറ്റൊരിടത്തേക്ക് അബ്രഹാം ഹാഗറിനെയും ഇശ്മയേലിനെയും പറഞ്ഞുവിട്ടുവെന്നും ആണ് ഉപര്യുക്ത ബൈബിള്‍ വചനങ്ങള്‍ വിശദീകരിക്കുന്നത്. ബൈബിള്‍പ്രകാരം ഹാഗറിന്റെയും ഇശ്മയേലിന്റെയും പാലായനത്തിന്റെ ലക്ഷ്യം തന്നെ സാറയുടെയും ഇസ്ഹാഖിന്റെയും ചുറ്റുവട്ടങ്ങളില്‍ നിന്ന് രക്ഷപെടലായിരുന്നുവെന്ന് സാരം.

ബിഅ്ര്‍ ശബ ഇസ്ഹാഖിന്റെയും സാറയുടെയും സ്വന്തം നാടാണെന്ന് ബൈബിളില്‍ നിന്ന് വ്യക്തമാണ്.  ബിഅ്ര്‍ ശബ ഉള്‍ക്കൊള്ളുന്ന ഫലസ്ത്വീന്‍ പ്രവിശ്യകളിലാണ് സാറയും ഇസ്ഹാഖും ഇസ്ഹാഖിന്റെ സന്തതിപരമ്പരകളും നിലനിന്നതെന്ന് ബൈബിളെഴുത്തുകാര്‍ സൂചിപ്പിക്കുന്നുണ്ട് (ഉല്‍പത്തി 25). ഇതിനര്‍ത്ഥമെന്താണ്? ബിഅ്ര്‍ ശബയില്‍ തന്നെ ഹാജറും ഇശ്മയേലും പാര്‍പ്പുറപ്പിച്ചാല്‍ അബ്രഹാം എന്തിനുവേണ്ടിയാണോ അവരെ വീട്ടില്‍ നിന്ന് പറഞ്ഞയച്ചത്, ആ ലക്ഷ്യം നിറവേറുകയില്ല എന്നല്ലേ? വാസ്തവത്തില്‍, ഫലസ്ത്വീനില്‍ നിന്ന് പാലായനം ആരംഭിക്കുന്ന ഹാജറും ഇശ്മയേലും  ബിഅ്ര്‍ ശബയില്‍ അലഞ്ഞുതിരിയുക എന്നത് വളരെ സ്വാഭാവികമാണ്; കാരണം അവര്‍ അതുവരെ ജീവിച്ച പ്രദേശമാണത്.

തോല്‍ സഞ്ചിയും കൈക്കുഞ്ഞുമായി  ബിഅ്ര്‍ ശബ മരുഭൂമിയിലൂടെ തന്നെയായിരിക്കണം ഹാജര്‍ പ്രയാണമാരംഭിച്ചിട്ടുണ്ടാവുക. എന്നാല്‍ വെള്ളം തീര്‍ന്നതും കിണറുകള്‍ കാണാതെ ഹാജറ അസ്വസ്ഥമായതും മലക്ക് പ്രത്യക്ഷപ്പെട്ടതുമെല്ലാം അവിടെ വെച്ചാണെന്ന് ബൈബിളെഴുത്തുകാര്‍ ഉദ്ദേശിച്ചിരിക്കണമെന്നില്ല. ഒന്നാമതായി, ബിഅ്ര്‍ ശബ അവര്‍ക്ക് ചിരപരിചിതമാണ്. രണ്ടാമതായി, ബിഅ്ര്‍ ശബ വെള്ളക്കിണറുകള്‍കൊണ്ട് സമൃദ്ധമാണ്. ആ കിണറുകളില്‍ പലതും ബൈബിള്‍പ്രകാരം ഹാഗാറിന്റെ ഭര്‍ത്താവായ അബ്രഹാം പണി കഴിപ്പിച്ചതുമാണ്. ബിഅ്ര്‍ എന്ന വാക്കിനുതന്നെ അര്‍ത്ഥം കിണര്‍ എന്നാണ്. ബിഅ്ര്‍ ശബ എന്നാല്‍ ഏഴ് കിണറുകള്‍ (അറബിയില്‍ ബിഅ്ര്‍ അസ്സബ്അ്) എന്നോ കരാര്‍ പ്രകാരമുള്ള കണറുകള്‍ എന്നോ ആണ്.  ജലസംഭരണികള്‍ സമൃദ്ധമായിരുന്നതുകൊണ്ടാണ് ബിഅ്ര്‍ ശബ  ആ പേരില്‍ അറിയപ്പെടാന്‍ ഇടയായത് (www.bible-archeology.info/bible-city-beershaba.html).

തനിക്ക് ചിരപരിചിതമായിരുന്ന, ജലസ്രോതസ്സുകളുടെ പേരില്‍ അറിയപ്പെട്ടിരുന്ന ഒരു പ്രദേശത്ത് ഹാജറ കുഞ്ഞിനെ മരണവക്കില്‍ കിടത്തി വെള്ളത്തിനുവേണ്ടി കരയുക എന്നത് മനസ്സിലാക്കുവാന്‍ പ്രയാസമുള്ള കാര്യമാണ്. ബൈബിളെഴുത്തുകാരന്‍ ഉദ്ദേശിച്ചിരിക്കാന്‍ സാധ്യത ഇതാണ്: ബിഅ്ര്‍ ശബയില്‍ നിന്നാണ് ഹാജര്‍ പ്രയാണമാരംഭിച്ചത്. ആ പ്രയാണം അവരെ അപരിചിതവും ജലശൂന്യവുമായ മറ്റൊരു മരുപ്രദേശത്തെത്തിച്ചു. ഉല്‍പത്തി പുസ്തകത്തിലെ 21-ാം അധ്യായത്തിലെ പതിനാലാം വചനം പലായനത്തിന്റെ തുടക്കത്തില്‍ ബിഅ്ര്‍ ശബയില്‍ സംഭവിച്ച കാര്യത്തിലേക്കും 15 മുതല്‍ 21 വരെയുള്ള വചനങ്ങള്‍ മറ്റൊരു മരുഭൂമിയില്‍ നടന്ന സംഭവങ്ങളിലേക്കും വിരല്‍ ചൂണ്ടുന്നതാകാനാണ് സാധ്യതയുള്ളതെന്ന് ചുരുക്കം. ആ മരുഭൂമിയെയാണ് വചനങ്ങളുടെ കര്‍ത്താവ് പാറാന്‍ എന്നു വിളിക്കുന്നത്.

കുഞ്ഞിന്റെ കരച്ചില്‍കേട്ട് ദൈവം സവിശേഷദാനമായി സൃഷ്ടിച്ച വെള്ളക്കിണറാണ് പാറാന്റെ അടയാളമായി ഉല്‍പത്തി പുസ്തകം പറയുന്നതെന്ന് നാം കണ്ടു. ഹാജറിന്റെ മനോവ്യഥക്കും ഇശ്മയേലിന്റെ നിലവിളിക്കും പരിഹാരമായി ദിവ്യകാരുണ്യത്തില്‍ നിന്ന് ഉറവയെടുത്ത സംസം കിണര്‍ നിലനില്‍ക്കുന്ന മക്കയല്ലാതെ മറ്റേതാണീ പ്രദേശം? മറ്റൊരു നാട്ടിലും ഇത്തരമൊരു ജലപ്രവാഹമുണ്ടായതായി ബൈബിളില്‍ എവിടെയും പറയുന്നില്ല. ഹാഗാര്‍ പാലായനം ചെയ്തത് മക്കയിലേക്കല്ലെന്ന് ബൈബിളുപയോഗിച്ച് സമര്‍ത്ഥിക്കുവാനാകില്ലെന്ന് ചുരുക്കം. ബൈബിളിനെ കണ്ണുമടച്ച് വിശ്വസിക്കുന്നവര്‍ക്കുമാത്രമേ ബൈബിള്‍ വിവരണങ്ങളുടെ മാത്രം വെളിച്ചത്തിലുള്ള ചരിത്രാഖ്യാനത്തെ അംഗീകരിക്കാന്‍ കഴിയൂ എന്ന് നാം നേരത്തെ പറഞ്ഞു. എന്നാല്‍ ബൈബിളിനുപോലും ഇല്ലാത്ത വാദങ്ങള്‍ ബൈബിളിനുമേല്‍ കെട്ടിവെച്ച് ഇശ്മയേലിനെ മക്കയില്‍ നിന്ന് ‘തള്ളിപ്പുറത്താക്കാന്‍’ ശ്രമിക്കുന്ന മിഷനറിമാര്‍ എന്തു വൈജ്ഞാനിക സത്യസന്ധതയാണ് പുലര്‍ത്തുന്നത്?

തന്റെ പ്രവാചകത്വത്തിന് സെമിറ്റിക് അംഗീകാരം ലഭിക്കാന്‍വേണ്ടി പ്രവാചകന്‍ പുതുതായി ചമച്ചുണ്ടാക്കിയ സിദ്ധാന്തമാണ് മക്കയുടെ അബ്രഹാമിക പൈതൃകം. അബ്രഹാം ഹാഗാറിനെയും ഇശ്മയേലിനെയും മക്കയില്‍ കൊണ്ടുചെന്നാക്കിയെന്നും അവിടെ കഅ്ബ നിര്‍മിച്ചുവെന്നും ഇശ്മയേലിന്റെ സന്തതിപരമ്പരകള്‍ പ്രസ്തുത താഴ്‌വരയില്‍ നിലനിന്നുവെന്നുമെല്ലാം പ്രവാചകന്‍ അറബികളെ പറഞ്ഞുപഠിപ്പിക്കുകയായിരുന്നു. മദീനയിലെ ജൂതന്‍മാരുടെ അനുഭാവം നേടിയെടുക്കാനും കഅ്ബയുടെ നിയന്ത്രണം കൈക്കലാക്കാനും വേണ്ടി ഇസ്‌ലാമിന്റെ അവസാന കാലഘട്ടത്തില്‍ കഅ്ബ അബ്രഹാം സ്ഥാപിച്ചതാണെന്നും തന്റെ ഗോത്രം അബ്രഹാമിന്റെ പരമ്പരയാണെന്നും കഅ്ബയെ അതിന്റെ അബ്രഹാമിക വിശുദ്ധിയിലേക്ക് തിരിച്ചുകൊണ്ടുപോകാനാണ് താന്‍ ശ്രമിക്കുന്നതെന്നും മുഹമ്മദ് നബി (സ) അവകാശപ്പെടുകയാണ് ചെയ്തത് എന്ന് സമര്‍ഥിക്കാന്‍ ശ്രമിക്കുന്ന ചില ഓറിയന്റലിസ്റ്റ്/മിഷനറി പഠനങ്ങളുടെ പരാമൃഷ്ട വാദങ്ങളില്‍ എന്തെങ്കിലും സത്യമുണ്ടോ?

ല്ല. ഓറിയന്റലിസ്റ്റുകളും മിഷനറിമാരും മക്കയുടെയും അതുവഴി മുഹമ്മദ് നബി(സ)യുടെയും അബ്രഹാമിക പൈതൃകത്തെ നിഷേധിക്കുവാന്‍വേണ്ടി പല രീതിയില്‍ ഉന്നയിച്ചിട്ടുള്ള വാദങ്ങളാണ് നടേ പറഞ്ഞവയെല്ലാം. മുഹമ്മദ് നബി(സ)യെയും അറബികളെയും വിശുദ്ധമായ അബ്രഹാമിക ശൃംഖലയില്‍ കണ്ണിചേര്‍ത്തു പറയാന്‍ ഇസ്രാഈലി വംശീയ ദുരഭിമാനമാണ് ഇവരില്‍ പലര്‍ക്കും തടസ്സമാകുന്നതെന്ന് അവരുടെ രചനകള്‍ പരിശോധിച്ചാല്‍ മനസ്സിലാക്കാന്‍ കഴിയും.

നബിചരിത്രത്തിന്റെ മുഴുവന്‍ സ്രോതസ്സുകള്‍ പ്രകാരവും സ്ഥിരപ്പെട്ടിട്ടുള്ള കാര്യമാണ് വാസ്തവത്തില്‍ ഇസ്മാഈല്‍ നബി (അ)യുടെ പുത്രന്‍ അദ്‌നാന്റെ പുത്രപരമ്പരയില്‍ ഖുറയ്ശ് ഗോത്രത്തില്‍ ബനൂഹാശിം കുടുംബത്തിലാണ് പ്രവാചകന്റെ ജനനമുണ്ടായത് എന്നത്. കുടുംബപരമ്പരാ പഠനം വ്യവസ്ഥാപിതമായിത്തന്നെ നിലനിന്നിരുന്ന അറബ് സമൂഹത്തില്‍ ഖുറയ്ശികളുടെ അദ്‌നാനീ പാരമ്പര്യത്തെക്കുറിച്ച് യാതൊരു സംശയങ്ങളുമില്ലാതിരുന്നതുകൊണ്ടാണ് പ്രവാചകചരിത്രത്തിന്റെ സ്രോതസ്സുകള്‍ ഇക്കാര്യത്തില്‍ ഏകോപിക്കുന്നതും അബ്രഹാമും ഇസ്മാഈലും ചേര്‍ന്നുനിര്‍മിച്ച കഅ്ബയുടെ പരിപാലനം മുഴുവന്‍ മക്കക്കാരുടെയും അംഗീകാരത്തോടുകൂടി ഖുറയ്ശികള്‍ നിര്‍വഹിച്ചുപോന്നുവെന്ന വസ്തുത ഐകകണ്‌ഠ്യേന ചൂണ്ടിക്കാണിക്കുന്നതും. പ്രവാചക ജീവചരിത്രകാരന്‍മാര്‍ക്കിടയില്‍ അഭിപ്രായവ്യത്യാസങ്ങളില്ലാതെ സ്ഥിരപ്പെടുവാന്‍ മാത്രം പ്രബലമായ ഒരു ചരിത്രവസ്തുതയെ, പ്രവാചകനെ പ്രവാചകനായി അംഗീകരിക്കുവാന്‍ അഹങ്കാരം സമ്മതിക്കാത്തതുകൊണ്ടു മാത്രമാണ് നബിവിമര്‍ശകര്‍ നിഷേധിക്കാന്‍ ശ്രമിക്കുന്നത്.

ഇബ്രാഹിം നബി (അ) മാതാവ് ഹാജറിനെയും പുത്രന്‍ ഇസ്മാഈലിനെയും മക്കയില്‍ കൊണ്ടുചെന്നാക്കിയെന്നും അവിടെ ദൈവാനുഗ്രഹമായി സംസം ജലം ഉറവയെടുത്തെന്നും ജുര്‍ഹൂം എന്ന അറബിഗോത്രം ജലസാന്നധ്യം കാരണം അവിടെ തമ്പടിച്ചെന്നും അവരുമായുള്ള വൈവാഹികബന്ധത്തിലൂടെ ഇസ്മാഈല്‍ നബി (അ) യുടെ സന്തതികള്‍ മക്കാ താഴ്‌വരയില്‍ നിലവില്‍ വന്നുവെന്നും ഇബ്‌റാഹിം നബി (അ) മക്കയിലേക്കുവന്ന് ഇസ്മാഈലിനെയും കൂട്ടി കഅ്ബ നിര്‍മിച്ചുവെന്നും മുഹമ്മദ് നബി (സ) പറയാതെ തന്നെ അറബികള്‍ നിര്‍വിവാദം അംഗീകരിച്ചു വന്നിരുന്നതാണ്. ചിരപുരാതന കാലം മുതല്‍ തലമുറകളിലൂടെ അവര്‍ കൈമാറി വന്ന ദേശചരിത്രമാണത്.

മുഹമ്മദ് നബി (സ) സമൂഹത്തിനുമുന്നില്‍ അവതരിപ്പിച്ച ദൈവസന്ദേശങ്ങള്‍ ഒട്ടുമിക്കതും ആശയപരമായ കാരണങ്ങളാല്‍ അദ്ദേഹത്തിന്റെ പ്രബോധിത സമൂഹത്തിന്റെ കടുത്ത എതിര്‍പ്പുകള്‍ക്ക് വിധേയമായിട്ടുണ്ട്. ക്വുര്‍ആനിലും ഹദീഥുകളിലും ചരിത്രഗ്രന്ഥങ്ങളിലും അവ വിശദമായി രേഖപ്പെടുത്തിയിട്ടുമുണ്ട്. എന്നാല്‍ കഅ്ബയുടെയും മക്കയുടെയും ഇബ്‌റാഹിമീ-ഇസ്മാഈലീ പാരമ്പര്യത്തെ സംബന്ധിച്ച് നബി(സ) പറഞ്ഞ കാര്യങ്ങളെയൊന്നും ഒരു സമകാലീനനും ചോദ്യം ചെയ്തിട്ടില്ലെന്നതാണ് യാഥാര്‍ത്ഥ്യം. കാരണം, അവര്‍ക്ക് നേരത്തെ തന്നെ ബോധ്യമുണ്ടായിരുന്ന കാര്യങ്ങളായിരുന്നു അവയൊക്കെയും. പ്രവാചകകാലഘട്ടത്തില്‍ മക്കയിലും പരിസരപ്രദേശങ്ങളിലും ജീവിച്ചിരുന്ന ജനങ്ങള്‍ക്ക് വിശ്വാസി-അവിശ്വാസി ഭേദമില്ലാതെ യോജിപ്പുണ്ടായിരുന്ന ചരിത്രമാണിത്. നബി(സ)യുടെ സമകാലീനരായിരുന്ന അറബ് ജൂതരോ ക്രൈസ്തവരോ ഒന്നും ഇത്തരം വിമര്‍ശനങ്ങള്‍ ഉന്നയിച്ചിട്ടില്ലെന്ന കാര്യം ശ്രദ്ധേയമാണ്. തങ്ങളുടെ ആദര്‍ശപിതാവായ അബ്രഹാമിന് മക്കയുമായുള്ള ബന്ധത്തെ അവരാരും നിരാകരിച്ചില്ലെന്നുവേണം മനസ്സിലാക്കാന്‍. പ്രവാചകനില്‍ നിന്ന് ഇക്കാര്യങ്ങള്‍ നേരിട്ടുകേട്ട, ബൈബിളിന്റെ കുറേക്കൂടി പുരാതനമായ പ്രതികളെ ഉപജീവിച്ചിരുന്ന ഏഴാം നൂറ്റാണ്ടിലെ അറബ് ജൂത-ക്രിസ്ത്യാനികള്‍ക്ക് തോന്നിയിട്ടില്ലാത്ത മനപ്രയാസമാണ് അവരുടെ പിന്‍മുറക്കാര്‍ ഇത്തരം വിഷയങ്ങളില്‍ പ്രകടമാക്കുന്നത്.

അറേബ്യയില്‍ ചര്‍ച്ചക്കുപോലും സാധ്യതകളില്ലാത്തവിധം സ്പഷ്ടമായിരുന്ന ഒരു യാഥാര്‍ത്ഥ്യമാണ് കഅ്ബയുടെ ഇബ്‌റാഹിമീ പൈകൃകം എന്ന് ഇതെല്ലാം വ്യക്തമാക്കുന്നുണ്ട്. ഒരു നാടിന്റെ ചരിത്രം ഒരു സുപ്രഭാതത്തില്‍ യാതൊരു ഒച്ചപ്പാടുകളുമില്ലാതെ പ്രവാചകന്‍ മാറ്റി പ്രഖ്യാപിച്ചു എന്ന് സ്ഥാപിക്കാന്‍ ശ്രമിക്കുന്നത് എന്തുമാത്രം വലിയ അസംബന്ധമല്ല!

ഖുറയ്ശികളുടെയും കഅ്ബയുടെയും അബ്രഹാമിക പൈതൃകത്തെ സംബന്ധിച്ച് അറബികള്‍ക്കോ അവരെ പരിചയമുണ്ടായിരുന്നവര്‍ക്കോ സംശയങ്ങളൊന്നുമുണ്ടായിരുന്നില്ല എന്നതുകൊണ്ടുതന്നെ, ഖുറയ്ശികളെയും കഅ്ബയെയും സംബന്ധിച്ച പ്രവാചകപൂര്‍വ അറബ് സംഭാഷണങ്ങളിലെല്ലാം ഇബ്‌റാഹീമി-ഇസ്മാഈലി പാരമ്പര്യത്തെക്കുറിച്ച പരാമര്‍ശങ്ങള്‍ സര്‍വസാധാരണമായിരുന്നുവെന്നതാണ് യാഥാര്‍ത്ഥ്യം. ഖുറയ്ശികളുടെ ഇസ്‌ലാം പൂര്‍വചരിത്രം ക്രോഡീകരിക്കുവാന്‍ ശ്രമിച്ച ചരിത്രകാരന്‍മാരുടെയെല്ലാം രചനകളില്‍, അതുകൊണ്ടുതന്നെ, ഇത്തരത്തിലുള്ള അനേകം നിവേദനങ്ങള്‍ കണ്ടെത്താന്‍ കഴിയും. പ്രവാചകന്റെ പിതാമഹനായിരുന്ന അബ്ദുല്‍ മുത്വലിബ് ”ഞങ്ങള്‍ അല്ലാഹുവിന്റെ നാട്ടിലെ അല്ലാഹുവിന്റെ ജനതയാണ്; അത് എല്ലായ്‌പ്പോഴും (അല്ലാഹുവുമായുള്ള) ഇബ്‌റാഹിമീന്റെ ഉടമ്പടി പ്രകാരമായിരുന്നു” (നഹ്‌നു അഹ്‌ലുല്ലാഹി ഫീ ബലദതിഹി, ലം യസല്‍ ദാക അലാ അഹ്ദി ഇബ്‌റാഹീം) എന്ന് പറഞ്ഞിരുന്നത് യഅ്ക്വൂബി തന്റെ താരിഖില്‍ ഉദ്ധരിക്കുന്നുണ്ട് (1/253).

പ്രവാചകന്റെ പിതൃവ്യനും ഗുണകാംക്ഷിയും എന്നാല്‍ അമുസ്‌ലിമും ആയിരുന്ന അബൂത്വാലിബ്, താന്‍ മുഹമ്മദിന്റെ (സ) പ്രവാചകത്വം അംഗീകരിക്കുന്നില്ലെങ്കിലും അദ്ദേഹത്തെ ശത്രുക്കളുടെ പീഡനത്തിന് വിട്ടുകൊടുക്കുകയില്ലെന്ന് മക്കളുടെ മുഴുവന്‍ പൈതൃകചിഹ്നങ്ങളെയും പിടിച്ചാണയിട്ട് ആലപിച്ച കവിതയില്‍ ”ഇബ്‌റാഹിമീന്റെ പാദമുദ്ര ഇപ്പോഴും ശുദ്ധമായി കിടക്കുന്ന ശില; അതിലെ രണ്ട് പാദമുദ്രകളും ചെരുപ്പില്ലാതെ നഗ്നമാണ്” എന്ന് മക്വാമു ഇബ്‌റാഹീമിനെക്കുറിച്ച് പ്രസ്താവിച്ചതായി ഇബ്‌നു ഇസ്ഹാക്വ്  നിവേദനം ചെയ്യുന്നുണ്ട്. (A. Guillaume, The Life of Muhammed: A Translation of Ibn Ishaq’s Sirat Rasul Allah (Oxford University Press, 2007), p. 123).

മുദാര്‍ ഗോത്രക്കാര്‍ ”ഇസ്മാഈല്‍ നമുക്കായി അവശേഷിപ്പിച്ച മതത്തിന്റെ കടമകള്‍ നമുക്കുവേണ്ടി നിര്‍വഹിച്ചുവന്നത് ഖുറയ്ശികളാണ്” എന്ന് പറയാറുണ്ടായിരുന്നുവെന്ന് മുഹമ്മദ് ഇബ്‌നു ഹബീബ് തന്റെ മുഹബ്ബറില്‍ (264) രേഖപ്പെടുത്തുന്നുണ്ട്.  ഇസ്മാഈലിന്റെ നേര്‍പൈതൃകം കാരണം അറബികള്‍ ഖുറയ്ശികള്‍ക്കു കല്‍പിച്ചുനല്‍കിയിരുന്ന ആദരവിന്റെ രേഖ കൂടിയാണ് മുദാര്‍ ഗോത്രക്കാരില്‍ നിന്നുള്ള പാരമൃഷ്ട ഉദ്ധരണി. ഖുറയ്ശികള്‍ അറബികള്‍ക്കിടയില്‍ അറിയപ്പെട്ടിരുന്നത് ‘സ്വരീഹു വുല്‍ദി ഇസ്മാഈല്‍’ (ഇസ്മാഈലിന്റെ സുവ്യക്തമായ/ഋജുവായ പരമ്പര) എന്നായിരുന്നുവെന്ന് ഇബ്‌നു ഇസ്ഹാക്വ് രേഖപ്പെടുത്തിയത് (4/205) ഇവിടെ ചേര്‍ത്തുവായിക്കുന്നത് പ്രസക്തമാണ്.

ഇസ്മാഈല്‍ നബി(അ)യുടെ പിന്‍മുറക്കാര്‍ എന്ന നിലയിലും കഅ്ബയുടെ കൈകാര്യകര്‍ത്താക്കള്‍ എന്ന നിലയിലുമാണ് ഖുറയ്ശികള്‍ മക്കയില്‍ സവിശേഷമായ സാമൂഹികാംഗീകാരങ്ങള്‍ ആസ്വദിച്ചിരുന്നത്. ഖുറയ്ശികളും ഥക്വീഫ ് ഗോത്രക്കാരും തമ്മില്‍ നടന്ന ഒരു സംഭാഷണത്തില്‍ കഅ്ബാ പരിപാലനത്തില്‍ ഥക്വീഫ് ഗോത്രക്കാരെ പങ്കാളിയാക്കാന്‍ തങ്ങള്‍ സന്നദ്ധമാണെന്നും അതിനുപകരമായി ഥക്വീഫുകാരുടെ നിയന്ത്രണത്തിലുള്ള പ്രവിശ്യകളിലെ സാമ്പത്തിക ഇടപാടുകളില്‍ തങ്ങള്‍ക്കവസരം നല്‍കണമെന്നും ഖുറയ്ശി പ്രതിനിധികള്‍ പറഞ്ഞപ്പോള്‍ ഥക്വീഫ് ഗോത്രനേതാക്കള്‍ നല്‍കിയ മറുപടി ഇബ്‌നു ഹബീബ് നിവേദനം ചെയ്യുന്നതിപ്രകാരമാണ്: ”നിങ്ങളെയെങ്ങനെയാണ് ഞങ്ങള്‍ ഞങ്ങളുടെ പിതാക്കള്‍ ആയുധങ്ങളില്ലാതെ വെറുംകൈകള്‍കൊണ്ട് പാറ വെട്ടിത്തുരന്ന് പാര്‍പ്പുറപ്പിച്ച ഭൂമിയുടെ അവകാശത്തില്‍ പങ്കുകാരാക്കുക? (പകരം നിങ്ങള്‍ ഞങ്ങളുമായി പങ്കുവെക്കാമെന്ന് പറയുന്ന) വിശുദ്ധ ഗേഹം നിങ്ങള്‍ സ്വയം  ഉണ്ടാക്കിയതല്ല. അത് സ്ഥാപിച്ചത് ഇബ്‌റാഹീം ആയിരുന്നു.” (കയ്ഫ നുശ്‌രികുകും ഫീ വാദിന്‍ നസലഹു അബൂനാ വ ഹറഫഹു ബിയദയ്ഹി ഫിസ്സ്വഖ്‌രി ലം യഖ്ഫിര്‍ഹു ബില്‍ ഹദീദ്, വ അന്‍തും ലം തജ്അലുല്‍ ഹറമ, ഇന്നമാ ജഅലഹു ഇബ്‌റാഹീം/ കിതാബുല്‍ മുനമ്മക്വ് ഫീ അഖ്ബാരി ഖുറയ്ശ്, 280. വിര്‍ജീനിയ യൂനിവേഴ്‌സിറ്റി പ്രസ് 1964ല്‍ മുഹമ്മദ് ഇബ്‌നു ഹബീബിന്റെ ഈ ഗ്രന്ഥം പുനഃപ്രസിദ്ധീകരിച്ചിട്ടുണ്ട്).

പ്രവാചകന്റെ മദീനാ കാലഘട്ടത്തില്‍ ഇസ്‌ലാം ആശ്ലേഷിച്ച മദീനയിലെ പ്രഗല്‍ഭനായ ജൂതപണ്ഡിതന്‍ അബ്ദുല്ലാഹിബ്‌നു സലാം ജൂതനായിരിക്കെ മദീനയിലെ ജൂതനേതാക്കളോട് കഅ്ബയെ ഉദ്ദേശിച്ചുകൊണ്ട് ”നമ്മുടെ പിതാവ് ഇബ്‌റാഹീമിന്റെ പള്ളി സന്ദര്‍ശിക്കുവാന്‍ ഞാന്‍ ആഗ്രഹിക്കുന്നു”വെന്ന് പറഞ്ഞതായി വ്യക്തമാക്കുന്ന നിവേദനങ്ങളും (ജലാലുദ്ദീന്‍ സുയൂത്വി, അദ്ദുര്‍റുല്‍ മന്‍ഥൂര്‍ ഫിത്തഫ്‌സീരി ബില്‍ മഅ്ഥൂര്‍, 6/410) അറേബ്യന്‍ സമൂഹത്തില്‍ ഇക്കാര്യങ്ങള്‍ക്കുണ്ടായിരുന്ന സര്‍വസമ്മതിയാണ് സൂചിപ്പിക്കുന്നത്. പ്രവാചകകാലഘട്ടത്തിലോ അതിനുമുമ്പോ, ഖുറയ്ശികളുടെയും കഅ്ബയുടെയും ഇബ്‌റാഹീമി-ഇസ്മാഈലി വേരുകള്‍ ഒരു തര്‍ക്കവിഷയമേ ആയിരുന്നില്ലെന്നും സര്‍വാംഗീകൃതമായ ഒരു ചരിത്രവസ്തുതയില്‍ നിന്ന് പാഠം പഠിച്ച് വിശ്വാസപരമായ വിശുദ്ധി വീണ്ടെടുക്കാന്‍ അറബികളെ ആഹ്വാനം ചെയ്യുക മാത്രമാണ് മുഹമ്മദ് നബി (സ) ചെയ്തത് എന്നുമാണ് ഇതെല്ലാം വ്യക്തമാക്കുന്നത്.

മുഹമ്മദ് നബി(സ)യുടെ കാലത്തെ മക്കന്‍ അറബികള്‍ തങ്ങളില്‍ പലരുടെയും പൂര്‍വപിതാക്കളായും കഅ്ബയുടെ സ്ഥാപകരായും ഇബ്‌റാഹിം(അ)യെയും, ഇസ്മാഈല്‍(അ)യെയും അറിയുകയും ആദരിക്കുകയും ചെയ്തുവന്നവരായിരുന്നുവെന്ന് മുഹമ്മദ് നബി (സ) മക്ക കീഴടക്കുമ്പോഴുള്ള കഅ്ബാലയത്തിന്റെ അവസ്ഥ തന്നെ വ്യക്തമാക്കുന്നുണ്ട്. കഅ്ബാലയത്തിനകത്ത് ഇബ്‌റാഹിമിന്റെയും ഇസ്മാഈലിന്റെയും ചിത്രങ്ങള്‍ കൊത്തിവെച്ചിട്ടുണ്ടായിരുന്നു ബഹുദൈവാരാധക അറബികള്‍. മക്ക, മദീന കേന്ദ്രമായുള്ള ഇസ്‌ലാമിക രാഷ്ട്രത്തിന്റെ ഭാഗമായി ഹിജ്‌റ എട്ടാം വര്‍ഷം മാറിയപ്പോള്‍ രാഷ്ട്രത്തലവന്‍ എന്ന നിലയില്‍ പ്രവാചകന്‍ (സ) പ്രസ്തുത ചിത്രങ്ങള്‍ മായ്ച്ചുകളയാന്‍ കല്‍പിക്കുകയാണ് ചെയ്തത്.

അമ്പുകളുപയോഗിച്ചുള്ള അന്ധവിശ്വാസജഡിലമായ ഭാഗ്യപരീക്ഷണത്തില്‍ ഇബ്‌റാഹീമും ഇസ്മാഈലും ഏര്‍പ്പെട്ടിരിക്കുന്നതായാണ് കഅ്ബക്കകത്ത് ചിത്രീകരിക്കപ്പെട്ടിരുന്നത്.  അറബികള്‍ ഇബ്‌റാഹീമിനെയും ഇസ്മാഈലിനെയും കഅ്ബയുമായുള്ള ബന്ധത്തിന്റെ പേരിലും രക്തബന്ധത്തിന്റെ പേരിലും ആദരിച്ചപ്പോഴും അവര്‍ പഠിപ്പിച്ച ശുദ്ധ ഏകദൈവാരാധനയില്‍ നിന്ന് ബഹുദൂരം അകന്നുപോവുകയും അവരെത്തന്നെ ബഹുദൈവാരാധനാപരമായ പശ്ചാത്തലങ്ങളില്‍ കൊണ്ടുപോയി പ്രതിഷ്ഠിക്കുകയും ചെയ്യുകയാണുണ്ടായതെന്ന പ്രവാചകന്റെ(സ) വിശദീകരണത്തെ കൃത്യമായി സാധൂകരിക്കുന്ന പുരാവസ്തു രേഖ കൂടിയായിരുന്നു വാസ്തവത്തില്‍ പ്രസ്തുത ചിത്രങ്ങള്‍. ഇത്തരം ചിത്രങ്ങള്‍ പ്രതിഷ്ഠിക്കപ്പെട്ടാല്‍ ആരാധനാലയത്തില്‍ പ്രപഞ്ചനാഥന്റെ അനുഗ്രഹവുമായി കടന്നുവരുന്ന മലക്കുകളുടെ സാന്നിധ്യം ഇല്ലാതാവുകയാണ് ചെയ്യുകയെന്നും ഇബ്‌റാഹിം, ഇസ്മാഈല്‍ പ്രവാചകന്‍മാരുമായി യാതൊരു ബന്ധവുമില്ലാത്ത ബഹുദൈവാരാധനാപരമായ ഭാഗ്യപരീക്ഷണാനുഷ്ഠാനത്തെയാണ് അവരുടെ പേരില്‍ ചിത്രകാരന്‍മാര്‍ ആരോപിച്ചിരിക്കുന്നതെന്നും അവ സ്ഥാപിച്ചവര്‍ പ്രപഞ്ചരക്ഷിതാവായ അല്ലാഹുവിന്റെ ശാപത്തിനര്‍ഹരാണെന്നും കഅ്ബക്കകത്തു കയറി പരാമൃഷ്ട ചിത്രങ്ങള്‍ കാണാനിടയായപ്പോള്‍ നബി(സ) പ്രതിവചിച്ചതായി ഇബ്‌നു അബ്ബാസില്‍ നിന്നുള്ള തീര്‍ത്തും ആധികാരികമായ നിവേദനങ്ങളിലുണ്ട് (ബുഖാരി). അമ്പുകളുപയോദിച്ച് ഭാഗ്യം പരീക്ഷിക്കുന്ന ജാഹിലിയ്യാ അറബ് സമ്പ്രദായത്തിനുപോലും ഇസ്മാഈലിനെക്കുറിച്ചുള്ള അവരുടെ ഓര്‍മകളുമായി ബന്ധമുണ്ടായിരുന്നുവെന്നതാണ് വാസ്തവം. ഇസ്മാഈല്‍ നബി (അ) പ്രഗല്‍ഭനായ ഒരു വില്ലാളിയായി മക്കയില്‍ വളര്‍ന്നുവന്നുവെന്ന അറിവില്‍നിന്നാണ് പില്‍ക്കാലത്ത് അവരുടെ അന്ധവിശ്വാസപരമായ ചടങ്ങുകളില്‍ അമ്പും വില്ലും വന്ന് നിറഞ്ഞതും ഇസ്മാഈലിന്റെ(അ) ചിത്രത്തിനും ഹുബുലിന്റെ വിഗ്രഹത്തിനുമെല്ലാം അമ്പുകളുടെ അകമ്പടിയുണ്ടായിത്തീര്‍ന്നതും.

ഇസ്മാഈല്‍ മരുഭൂമിയില്‍ ദൈവസംരക്ഷണത്തില്‍ വളര്‍ന്നുവന്നതിനെക്കുറിച്ച് പരാമര്‍ശിക്കവെ ബൈബിള്‍ തന്നെ ഇക്കാര്യത്തിലേക്ക് സൂചന നല്‍കുന്നുണ്ട്. ”ദൈവം ആ കുട്ടി (ഇശ്മയേല്‍)യോട് കൂടിയുണ്ടായിരുന്നു. അവന്‍ മരുഭൂമിയില്‍ പാര്‍ത്തു. അവന്‍ വളര്‍ന്നു സമര്‍ത്ഥനായൊരു വില്ലാളിയായിത്തീര്‍ന്നു” (ഉല്‍പത്തി 21 : 20). ഇക്കാര്യത്തെ ശരിവെക്കുന്ന പ്രസ്താവന മുഹമ്മദ് നബി(സ)യും നടത്തിയിട്ടുണ്ട്. അമ്പെയ്ത്തു മത്സരം നടത്തിക്കൊണ്ടിരുന്ന ഒരു അറബ് ഗോത്രത്തെക്കണ്ടപ്പോള്‍ പ്രവാചകന്‍ (സ) അവരോട്, ”ഇസ്മാഈല്‍ സന്തതികളേ, നിങ്ങള്‍ അമ്പെയ്ത്ത് പരിശീലിക്കുക; കാരണം നിങ്ങളുടെ പിതാവ് പ്രഗല്‍ഭനായ ഒരു വില്ലാളിയായിരുന്നു” എന്ന് പറയുകയുണ്ടായി (ബുഖാരി).

ഇസ്മാഈല്‍ നബി (അ) പഠിപ്പിച്ച ചില അനുഷ്ഠാനങ്ങള്‍ തികഞ്ഞ ബഹുദൈവാരാധനയിലേക്ക് കൂപ്പുകുത്തിയപ്പോഴും മക്കക്കാര്‍ക്കിടയില്‍ പ്രവാചകകാലഘട്ടം വരെ അതേപടി നിലനിന്നുവെന്നതും ഇതുപോലെത്തന്നെ ശ്രദ്ധേയമാണ്. ഇവയില്‍ ഏറ്റവും പ്രധാനപ്പെട്ട ഒന്നായിരുന്നു ചേലാകര്‍മം. ലിംഗ പരിഛേദനയ്ക്കുള്ള കല്‍പന ദൈവം പുറപ്പെടുവിച്ചത് അബ്രഹാമിന്റെ കാലത്താണ് എന്നാണ് ബൈബിളില്‍ നിന്നു മനസ്സിലാകുന്നത്. അബ്രഹാമിന് തൊണ്ണൂറും ഇശ്മയേലിന് പതിമൂന്നും വയസ്സുള്ളപ്പോള്‍ ലിംഗപരിഛേദനയ്ക്കുള്ള കല്‍പന വന്നുവെന്നും അവര്‍ രണ്ടുപേരും അപ്പോള്‍ പരിഛേദനയറ്റുവെന്നും സകലരെയും എട്ടാം ദിവസം ചേലാകര്‍മം ചെയ്യണമെന്ന നിയമം പ്രഖ്യാപിച്ചുവെന്നും ഉല്‍പത്തി പുസ്തകം വിശദീകരിക്കുന്നു (അധ്യായം 17). ഇസ്മാഈലിന്റെ പൈതൃകമെന്ന നിലയ്ക്കുതന്നെ അറബികള്‍ കുട്ടികളെ ചേലാകര്‍മം ചെയ്യുന്ന പതിവ് നിലനിര്‍ത്തിപ്പോന്നു. മുഹമ്മദ് നബി(സ)യുടെ ആഗമനകാലത്തെ ജാഹിലിയ്യാ അറബികള്‍ക്കിടയില്‍ പോലും ലിംഗാഗ്ര ഛേദനം സാര്‍വത്രികമായിരുന്നു. അവരതിലേക്ക് ബഹുദൈവാരാധനയുടെ ഘടകങ്ങള്‍ ചേര്‍ത്തിരുന്നുവെന്ന് മാത്രമേയുള്ളൂ. കഅ്ബാലയത്തിനകത്ത് തങ്ങള്‍ സ്ഥാപിച്ച ഹുബുല്‍ ദേവന്റെ വിഗ്രഹത്തിന്റെ മുന്നില്‍ പോയി അമ്പുകളുപയോഗിച്ച് ഭാഗ്യപരീക്ഷണം നടത്തിയാണ് ചേലാകര്‍മത്തിന്റെ വിശദാംശങ്ങള്‍ ജാഹിലിയ്യാ അറബികള്‍ നിശ്ചയിച്ചിരുന്നത് എന്ന് ഇബ്‌നു ഇസ്ഹാക്വ് രേഖപ്പെടുത്തിയിട്ടുണ്ട്.

ചേലാകര്‍മം ഇസ്മാഈലി പാരമ്പര്യത്തിന്റെ ഭാഗമായി അറബികള്‍ക്കിടയില്‍ നിലനിന്നുപോന്നതിനെക്കുറിച്ച് ഇസ്‌ലാമിക ചരിത്രസ്രോതസ്സുകള്‍ മാത്രമല്ല സംസാരിക്കുന്നത്. സി. ഇ ഒന്നാം നൂറ്റാണ്ടില്‍ റോമാ സാമ്രാജ്യത്തില്‍ ജീവിച്ച ജൂതപണ്ഡിതനും ചരിത്രകാരനുമായ ജോസിഫസ് ഫ്‌ളേവിയസ് (Josephus Flavius) അദ്ദേഹത്തിന്റെ പ്രശസ്തമായ Antiquities of the Jews എന്ന ഗ്രന്ഥത്തില്‍ അറബികള്‍ ”പതിമൂന്ന് വയസ്സിനുശേഷം ചേലാകര്‍മം നടത്തുന്നു; ഇശ്മയേല്‍, ആ ജനതയുടെ സ്ഥാപകന്‍, അബ്രഹാമിന്റെ വെപ്പാട്ടിയില്‍ പിറന്ന മകന്‍, ആ വയസ്സിലാണ് പരിഛേദനയേറ്റത്” എന്നു വ്യക്തമായി എഴുതുന്നുണ്ട്. മുഹമ്മദ് നബി(സ)ക്ക് ആറു നൂറ്റാണ്ടോളം മുമ്പും അറബികളുടെ ചേലാകര്‍മം പ്രശസ്തമായിരുന്നുവെന്നും ഇസ്മാഈലിന്റെ സന്തതിപരമ്പരകള്‍ പുലര്‍ത്തിപ്പോരുന്ന സ്വാഭാവികത എന്ന നിലയിലാണ് ജൂതപണ്ഡിതന്‍മാര്‍ പോലും ഈ പ്രതിഭാസത്തെ മനസ്സിലാക്കിയിരുന്നത് എന്നും ജോസിഫസിന്റെ പ്രസ്താവന തെളിയിക്കുന്നുണ്ട്. അറബികളുടെ ഇസ്മാഈലി വേരുകള്‍ മുഹമ്മദ് നബി(സ)യുടെയും മുസ്‌ലിംകളുടെയും പുതിയ ‘കണ്ടുപിടുത്ത’മാണെന്ന മിഷനറി വാദം കൂടിയാണ് ജോസിഫസിന്റെ വരികള്‍ക്കുമുന്നില്‍ ദയനീയമായി തകര്‍ന്നുപോകുന്നത്.

ബഹുദൈവാരാധനയുടെ ചിഹ്നങ്ങള്‍കൊണ്ട് നിറഞ്ഞ മക്കയുടെ സാമൂഹ്യപരിസരം ചികഞ്ഞ് ഉള്ളിലേക്കുപോയാല്‍ ഇബ്‌റാഹിം നബി(അ)യുടെയും ഇസ്മാഈല്‍ നബി(അ)യുടെയും പ്രബോധനങ്ങളുടെ ശേഷിപ്പുകള്‍ ഉറങ്ങിക്കിടക്കുന്നത് കണ്ടെത്താന്‍ കഴിയുമെന്ന, ദിവ്യബോധനങ്ങളുടെ വെളിച്ചത്തിലുള്ള മുഹമ്മദ് നബി(സ)യുടെ നിലപാടിനെയാണ് ഈ പശ്ചാതലത്തെളിവുകളെല്ലാം സാധൂകരിക്കുന്നത്. ദിവ്യബോധനങ്ങളുടെ വെളിച്ചത്തില്‍, ഹജ്ജിലെയും ഉംറയിലെയും ഇബ്‌റാഹീമി ഘടകങ്ങളെ അവയുടെ ആദിമവിശുദ്ധിയില്‍ പുനഃസ്ഥാപിക്കുകയും പില്‍ക്കാലത്ത് വന്നുചേര്‍ന്ന ബഹുദൈവാരാധനാപരമായ അനുഷ്ഠാനങ്ങളെയും അന്ധവിശ്വാസങ്ങളെയും തൂത്തുകളയുകയുമാണ് പ്രവാചകന്‍(സ) ചെയ്തത്. പില്‍ക്കാല വ്യതിയാനങ്ങള്‍ വഴി ദുഷിച്ചപോയ അബ്രഹാമിക പാരമ്പര്യമാണ് മക്കയില്‍ നിലനില്‍ക്കുന്നതെന്നും അതിനെ ശുദ്ധീകരിച്ചെടുക്കുകയാണ് ഇബ്‌റാഹീം നബി(അ)യോട് യഥാര്‍ത്ഥത്തില്‍ സ്‌നേഹമുള്ളവര്‍ ചെയ്യേണ്ടതെന്നും വസ്തുനിഷ്ഠമായി കാര്യങ്ങളെ നോക്കിക്കാണുന്നവര്‍ക്കെല്ലാം മനസ്സിലാകുമായിരുന്നു.

അതുകൊണ്ടാണ്, പ്രവാചകനുമുമ്പ് സി. ഇ അഞ്ചാം നൂറ്റാണ്ടില്‍ ഗസ്സയില്‍ ജീവിച്ച സോസമേമസ് (Sozomemes) എന്ന ക്രൈസ്തവ പണ്ഡിതന്‍ അദ്ദേഹത്തിന്റെ രചനയില്‍ ഈസ്മാഈലീ പരമ്പരയിലുള്ള അറബികളെക്കുറിച്ചും ബഹുദൈവാരാധകരായ അയല്‍സമൂഹങ്ങളുമായുള്ള സമ്പര്‍ക്കം അവരുടെ വിശ്വാസത്തെ മലിനമാക്കുന്നതിനെക്കുറിച്ചും തന്റെ രചനകളില്‍ രേഖപ്പെടുത്തുകയും മോശെക്കുമുമ്പ് ഇസ്രാഈല്യര്‍ ജീവിച്ചിരുന്ന നിയമങ്ങളനുസരിച്ചു തന്നെയാണ് വിശ്വാസവ്യതിചലനം സംഭവിക്കുന്നതുവരെ അറബികളും ജീവിച്ചിരുന്നത് എന്നും അവരില്‍ ചിലരെങ്കിലും ഇപ്പോഴും ആ പൈതൃകം മുറുകെപ്പിടിക്കുന്നുണ്ടെന്നും ഇബ്‌റാഹീമീ നിയമങ്ങളെ വീണ്ടെടുക്കാന്‍ ഇസ്രാഈല്യരുമായുള്ള ആശയവിനിമയങ്ങള്‍ അവര്‍ക്ക് സഹായകരമാകുന്നുണ്ടെന്നും നിരീക്ഷിക്കുകയും ചെയ്തത്. ക്രൈസ്തവ സഭാപിതാക്കളുടെ പ്രാചീന ഗ്രീക്ക് എഴുത്തുകള്‍ ക്രോഡീകരിച്ച് 1857-66 കാലഘട്ടത്തില്‍ പാരീസില്‍ നിന്ന് ജെ.പി മിഗ്‌നെയുടെ നേതൃത്വത്തില്‍ 161 വോള്യങ്ങളില്‍ പ്രസിദ്ധീകരിക്കപ്പെട്ട Patrologia Graeciaയില്‍ ഈ പരാമര്‍ശങ്ങളെയുള്‍കൊള്ളുന്ന Sozomen histori ecclesiastia എടുത്തുചേര്‍ത്തിട്ടുണ്ട്. (www.patristica.net എന്ന വെബ്‌സൈറ്റില്‍ ഈ രേഖകള്‍ ലഭ്യമാണ്).

മക്കയുടെ ഇബ്‌റാഹീമീ പൈതൃകം പ്രവാചകന്‍ പറഞ്ഞുണ്ടാക്കിയതാണെന്ന് പറയുന്ന ഓറിയന്റലിസ്റ്റുകളും മിഷനറിമാരും പ്രവാചകനുമുമ്പ് ജീവിച്ച ജൂത-ക്രൈസ്തവ പണ്ഡിതന്മാര്‍ മക്കയുടെ അബ്രഹാമിക പൈതൃകത്തെ സാധൂകരിച്ച് സംസാരിച്ചതിനെക്കുറിച്ച് പാലിക്കുന്ന മൗനം, കാപട്യത്തില്‍ നിന്ന് മാത്രമാണ് ഉയിര്‍കൊള്ളുന്നത്.

ക്രൈസ്തവ വിമര്‍ശകരാണ് മുഹമ്മദ് നബിക്ക് ലഭിച്ച വെളിപാടുകള്‍ പിശാചില്‍നിന്നാണെന്ന ആരോപണം ഉന്നയിക്കുന്നത്. മുഹമ്മദി(സ)ന് ലഭിച്ച വെളിപാടുകള്‍ പിശാചുബാധയുടെ ഫലമായുണ്ടായതാണെന്ന് വരുത്തിത്തീര്‍ക്കാനാണ് സി.ഡി. ഫാണ്ടര്‍, ക്ലേയ്ര്‍ ടിസ്ഡാല്‍, ജോഷ്മാക്ഡവല്‍, ജോണ്‍ജില്‍ ക്രിസ്റ്റ്, ജി. നെഹ്ല്‍സ് തുടങ്ങിയ ക്രൈസ്തവ ഗ്രന്ഥകാരന്മാരെല്ലാം ശ്രമിച്ചിരിക്കുന്നത്. യേശുക്രിസ്തുവിന്റെ ക്രൂശീകര ണത്തെയും അതുമൂലമുള്ള പാപപരിഹാരത്തെയും നിഷേധിച്ചു കൊണ്ട് മനുഷ്യരാശിയെ പാപത്തിന്റെ ഗര്‍ത്തത്തില്‍തന്നെ തളച്ചിടുവാനുള്ള പി ശാചിന്റെ പരിശ്രമമാണ് ഖുര്‍ആനിന്റെ രചനക്കു പിന്നിലുള്ളതെന്ന് അവര്‍ വാദിക്കുന്നു. മനുഷ്യശരീരത്തില്‍ പിശാച് കയറിക്കൂടുമോ? പിശാചുബാധ കൊണ്ട് ഒരാള്‍ക്ക് രോഗങ്ങളുണ്ടാവുമോ? പിശാചുബാധിച്ച ഒരാള്‍ക്ക് വെളിപാടുണ്ടാവുമോ? തുടങ്ങിയ ചര്‍ച്ചകള്‍ ഇവിടെ അപ്രസക്തമാണ്. ബൈ ബിള്‍ പ്രകാരം പിശാചുബാധിച്ച ഒരാളില്‍ കാണപ്പെടുന്ന അസുഖങ്ങള്‍ എന്തെല്ലാമാണെന്ന് പരിശോധിക്കുക.

1. ബുദ്ധിഭ്രമത്താല്‍ അലറി വിളിക്കല്‍ (മാര്‍ക്കോസ് 1:24, ലൂക്കോസ് 9:39, യോഹന്നാന്‍ 10:20)

2. സ്വയം നശീകരണ പ്രവണത (മത്തായി 55:9, 18: 17, 15:32, മര്‍ക്കോസ് 5: 13, ലൂക്കോസ്, 8:33)

3. നഗ്‌നമായി നടക്കുന്നതിനുള്ള പ്രവണത (ലൂക്കോസ് 8:2, 8:35)

4. പിശാചിനാല്‍ തള്ളയിടപ്പെടുക (മത്തായി 17:15, മര്‍ക്കോസ് 1:26,9:18, 9:20,9:26)

5. മൂകത (മര്‍ക്കോസ് 9:25, 9:32, 12:22, ലൂക്കോസ് 11:14)

6. ബധിരത (മര്‍ക്കോസ് 9: 25)

7. അന്ധത (മത്തായി 12:22)

8. മറ്റാരും കാണാത്തത് കാണുകയും അറിയുകയും ചെയ്യുക (മര്‍ക്കോസ് 1:24, ലൂക്കോസ് 4:3, മത്തായി 8:29)

പിശാചുബാധിതനില്‍ കാണപ്പെടുന്നതെന്ന് ബൈബിള്‍ ഉദ്‌ഘോഷിക്കു ന്ന ലക്ഷണങ്ങളൊന്നും മുഹമ്മദി(സ)ല്‍ ഉണ്ടായിരുന്നതായി നമുക്ക് കാ ണാന്‍ കഴിയുന്നില്ല. ദൈവിക വെളിപാടുകള്‍ ലഭിക്കുമ്പോള്‍ അവ ഒരു മണിനാദം പോലെ തനിക്ക് അനുഭവപ്പെടാറുണ്ടെന്നും അതാണ് ഏറ്റവും പ്രയാസകരമായ വെളിപാടു രീതിയെന്നും മുഹമ്മദ്(സ) പറഞ്ഞതാണ് അദ്ദേഹത്തെ പിശാചുബാധിച്ചിരുന്നുവെന്നും പൈശാചിക വെളിപാടുകളാണ് ഖുര്‍ആനെന്നും വാദിക്കുന്നവരുടെ ഒരു തെളിവ്. വെളിപാട് ലഭിച്ചു കൊ ണ്ടിരുന്ന അതിശൈത്യമുള്ള ഒരു ദിവസം പ്രവാചകന്റെ നെറ്റിയില്‍ വിയര്‍പ്പുതുള്ളിയുണ്ടായിരുന്നതായി ഞാന്‍ കണ്ടുവെന്ന പ്രവാചകപത്‌നി ആഇശ(റ) യുടെ നിവേദനമാണ് മറ്റൊരു തെളിവ്. ഇവിടെ പ്രസക്തമായ ഒരു ചോദ്യമുണ്ട്. പിശാചുബാധിതന് ചെവിയില്‍ മണിയടിക്കുന്നതുപോലെ തോന്നുമെന്നോ അവന്റെ നെറ്റിത്തടം അതിശൈത്യമാണെങ്കിലും വിയര്‍പ്പുതുള്ളികളാല്‍ നിറയുമെന്നോ ബൈബിളില്‍ എവിടെയെങ്കിലുമുണ്ടോ? ഇല്ലെങ്കില്‍, പ്രവാചക(സ)നില്‍ പിശാചുബാധ ആരോപിക്കുവാന്‍ ബൈബിളിന്റെ അനുയായികള്‍ക്ക് എന്തടിസ്ഥാനമാണുള്ളത്?

പ്രവാചകന് ലഭിച്ച ദൈവിക സന്ദേശങ്ങള്‍ പിശാചുബാധയുടെ ഉല്‍പ ന്നങ്ങളാണെന്ന് പറയുന്നവര്‍ തങ്ങളുടെതന്നെ വിശുദ്ധന്മാരാണ് പിശാചു ബാധയേറ്റവരെന്ന് പറയാന്‍ നിര്‍ബന്ധിതരാവുമെന്നതാണ് വാസ്തവം.

യേശുവിന്റെ ജീവിതകാലമത്രയും അദ്ദേഹത്തെയും അദ്ദേഹം പഠിപ്പിച്ച ആശയങ്ങളെയും നശിപ്പിക്കുവാന്‍ വേണ്ടി അഹോരാത്രം പരിശ്രമിക്കുക യും (അപ്പോസ്തല പ്രവൃത്തികള്‍ 9:1, 26:10, 8:1) അദ്ദേഹത്തിനുശേഷം ക്രിസ്തു തനിക്ക് വെളിപ്പെട്ടിട്ടുണ്ടെന്ന് അവകാശവാദമുന്നയിക്കുകയും ചെയ്തയാളാണ് 'വിശുദ്ധ പൗലോസ്'. അദ്ദേഹത്തിന് ക്രിസ്തുദര്‍ശനം ലഭിച്ച രീതിയെക്കുറിച്ച് ബൈബിള്‍ വിവരിക്കുന്നത് കാണുക: ''പിന്നെ അയാള്‍ യാത്ര പുറപ്പെട്ട് ഡമാസ്‌കസിനെ സമീപിച്ചപ്പോള്‍, പെട്ടെന്ന് ആകാശത്തുനിന്ന് ഒരു പ്രകാശം അയാളുടെ ചുറ്റും മിന്നലൊളി പരത്തി. സാവൂള്‍ നിലം പതിച്ചു. 'സാവൂള്‍, സാവൂള്‍ നീ എന്നെ പീഡിപ്പിക്കുന്നത് എന്തിന്? എന്ന് തന്നോട് ചോദിക്കുന്ന ഒരു സ്വരം കേള്‍ക്കയായി. അപ്പോള്‍ അയാള്‍ ചോദിച്ചു: 'പ്രഭോ നീ ആരാണ്? അവന്‍ പറഞ്ഞു: നീ പീഡിപ്പിക്കുന്ന യേശുവാണ് ഞാന്‍. എഴുന്നേറ്റ് നഗരത്തില്‍ ചെല്ലുക. നീ ചെയ്യേണ്ടത് എന്തെന്ന് അവിടെ വെച്ച് നിനക്ക് അറിവ് കിട്ടും'. 'അയാളോടൊപ്പം യാത്ര ചെയ്തിരുന്ന ആളുകള്‍ സ്വരം കേട്ടെങ്കിലും ആരെയും കാണായ്കയാല്‍ വിസ്മയ സ്തബ്ധരായി നിന്നുപോയി. വീണുകിടന്നിടത്തുനിന്ന് സാവൂള്‍ എഴുന്നേറ്റു. കണ്ണുതുറന്നിട്ടും അയാള്‍ക്ക് ഒന്നും കാണാന്‍ കഴിഞ്ഞില്ല. അതിനാല്‍ അവര്‍ അയാളെ കൈക്കുപിടിച്ച് ഡമാസ്‌കസിലേക്കു കൊണ്ടുപോയി. മൂന്നു ദിവസത്തേക്ക് അയാള്‍ക്ക് കാഴ്ചയില്ലായിരുന്നു; അയാള്‍ തിന്നുകയോ കുടിക്കുകയോ ചെയ്തതുമില്ല'' (അപ്പോസ്തല പ്രവൃത്തികള്‍ 9:3-9)

നിലംപതിക്കുന്നതും കൂടെയുള്ളവര്‍ കാണാത്തത് കാണുന്നതും കേള്‍ ക്കാത്തത് കേള്‍ക്കുന്നതും കണ്ണു കാണാതാവുന്നതുമെല്ലാം പിശാചുബാധ യുടെ ലക്ഷണങ്ങളായി സുവിശേഷങ്ങളില്‍ സൂചിപ്പിക്കപ്പെട്ടിട്ടുള്ളത് നാം കണ്ടു. ക്രിസ്തുവിനെ താന്‍ കണ്ടുവെന്ന് പൗലോസ് അവകാശപ്പെട്ട സംഭവത്തില്‍ ഇതെല്ലാം അദ്ദേഹം അനുഭവിക്കുന്നുമുണ്ട്. പൗലോസിന് പിശാചുബാധയാണ് ഉണ്ടായതെന്ന് വാദിച്ചാല്‍ അത് അംഗീകരിക്കാന്‍ ക്രൈസ്തവ സമൂഹം സന്നദ്ധമാവുമോ? മുഹമ്മദി(സ)ന് പിശാചുബാധയായിരുന്നുവെന്ന് സമര്‍ഥിക്കുവാന്‍ ബൈബിളില്‍നിന്ന് ഒരു തെളിവെങ്കിലുമുദ്ധരിക്കാ ന്‍ ക്രൈസ്തവ വിമര്‍ശകര്‍ക്ക് കഴിയില്ല. അതേസമയം, നിലവിലുള്ള ക്രിസ്തുമതത്തിന്റെ സ്ഥാപകനായ പൗലോസിന് പിശാചുബാധയാണ് അനുഭ വപ്പെട്ടതെന്ന് ബൈബിള്‍ ഉപയോഗിച്ചു കൊണ്ട് സ്ഥാപിക്കാന്‍ ഒരാള്‍ക്ക് കഴിയും. അപ്പോള്‍ ആര്‍ക്കാണ് പിശാചുബാധ? ഇനി, മുഹമ്മദ് നബി(സ)ക്ക് പിശാച് ബാധിച്ചതുകൊണ്ടാണ് ഖുര്‍ആന്‍ എഴുതിയുണ്ടാക്കിയതെന്ന ക്രൈസ്തവവാദത്തിന്റെ ആണിക്കല്ല് പരിശോധിക്കുക. യേശുക്രിസ്തുവിന്റെ കുരിശുമരണത്തിലൂടെയുള്ള പാപപരിഹാരം എന്ന ആശയത്തെ വിമര്‍ശിക്കുന്നതുമൂലമാണല്ലോ ഖുര്‍ആന്‍ പിശാചിന്റെ സൃഷ്ടിയാണെന്ന് വാദിക്കുന്നത്.

എന്നാല്‍, യാഥാര്‍ഥ്യമെന്താണ്? യേശുക്രിസ്തു പരിശുദ്ധനായിരുന്നുവെന്ന് മുസ്‌ലിംകളും ക്രൈസ്തവരും വിശ്വസിക്കുന്നു. അദ്ദേഹം സര്‍വശക് തനാല്‍ നിയുക്തനായ വ്യക്തിയാണെന്ന് ഇരുകൂട്ടരും സമ്മതിക്കുന്നു. അദ്ദേഹത്തിന് പിശാചുബാധയുണ്ടായിട്ടില്ലെന്ന് ഇരുകക്ഷികളും പറയുന്നു. എങ്കില്‍, മുഹമ്മദ് നബി(സ)ക്കോ പൗലോസിനോ ആര്‍ക്കാണ് പിശാചില്‍നിന്ന് വെളിപാടുണ്ടായതെന്ന് പരിശോധിക്കാന്‍ നമുക്കെന്തുകൊണ്ട് യേശുക്രിസ്തുവിന്റെ ഉപദേശങ്ങളുമായി അവരുടെ ഉപദേശങ്ങളെ താരതമ്യം ചെയ്തുകൂടാ? പിശാചില്‍നിന്ന് വെളിപാടുണ്ടായ വ്യക്തി യേശുവിന്റെ ശത്രുവായിരിക്കുമല്ലോ. ഒരു ദൈവദൂതന്റെ ശത്രു അയാള്‍ പ്രബോധനം ചെയ്യുന്ന ആശയങ്ങളുടെ ശത്രുവായിരിക്കും എന്നോര്‍ക്കുക.

യേശു പറഞ്ഞു: നിയമത്തെ (തോറ)യോ പ്രവാചകന്മാരെയോ റദ്ദാക്കാനല്ല ഞാന്‍ വന്നത്' (മത്തായി 5:17). ഖുര്‍ആന്‍ പറയുന്നു: 'തീര്‍ച്ചയായും നാം തന്നെയാണ് തൗറാത്ത് അവ തരിപ്പിച്ചിരിക്കുന്നത്, അതില്‍ മാര്‍ഗദര്‍ശനവും പ്രകാശവുമുണ്ട്' (5:44).

'മര്‍യമിന്റെ മകന്‍ ഈസ പറഞ്ഞ സന്ദര്‍ഭം: ഇസ്രായേല്‍ സന്തതികളേ, എനിക്കുമുമ്പുള്ള തൗറാത്തിനെ സത്യപ്പെടുത്തുന്നവനായിക്കൊണ്ടും എനിക്ക് ശേഷം വരുന്ന അഹ്മദ് എന്നു പേരുള്ളൊരു ദൂതനെപ്പറ്റി സന്തോ ഷവാര്‍ത്ത അറിയിക്കുന്നവനായിക്കൊണ്ടും നിങ്ങളിലേക്ക് അല്ലാഹുവിന്റെ ദൂതനായി നിയോഗിക്കപ്പെട്ടവനാകുന്നു ഞാന്‍' (61:6).

പൗലോസ് എഴുതി: 'നിയമാനുഷ്ഠാനങ്ങളെ (തോറ) ആശ്രയിക്കുന്നവ രെല്ലാം ശാപഗ്രസ്തരാണ് (ഗലാത്തിയക്കാര്‍ 3:10).

'ക്രിസ്തു നിയമത്തിന്റെ ശാപത്തില്‍നിന്നു നമ്മെ മോചിപ്പിച്ചിരിക്കു ന്നു' (ഗലാത്തിയക്കാര്‍ 3:13). 'അവന്‍ (യേശു) തന്റെ ശരീരത്തില്‍, നിയമത്തെ അതിന്റെ കല്‍പന കളോടും അനുശാസനങ്ങളോടുംകൂടി റദ്ദാക്കി' (എഫേസോസുകാര്‍2:15)

ഞാന്‍ നിയമത്തെ റദ്ദാക്കാനല്ല വന്നതെന്ന് യേശു, ഖുര്‍ആനും അതുതന്നെ പറയുന്നു. പൗലോസാകട്ടെ യേശു നിയമത്തില്‍നിന്ന് ലോകത്തെ രക്ഷിക്കാനാണ് വന്നത് എന്നു സമര്‍ഥിക്കുന്നു. ആര്‍ക്കാണ് പിശാചിന്റെ വെളിപാട്?

യേശുക്രിസ്തു താന്‍ ദൈവമാണെന്ന് പഠിപ്പിച്ചില്ല (മര്‍ക്കോസ് 12:29, മത്തായി 4:10) ഇക്കാര്യം ഖുര്‍ആന്‍ അര്‍ഥശങ്കക്കിടയില്ലാത്തവണ്ണം വ്യക്ത മാക്കുന്നു (3:51), എന്നാല്‍ പൗലോസ് പറഞ്ഞതാകട്ടെ 'പ്രകൃത്യാതന്നെ ദൈവമായിരുന്നിട്ടും ദൈവത്തോടു തനിക്കുള്ള തുല്യതയെ, മുറുകെപ്പിടിച്ചുകൊണ്ടിരിക്കേണ്ട ഒരു കാര്യമായി അവന്‍ പരിഗണിച്ചില്ല? (ഫിലിപ്പിയര്‍ 2:6). 'അവന്‍ അദൃശ്യനായ ദൈവത്തിന്റെ പ്രതിരൂപമാണ്; സര്‍വസൃഷ്ടികളിലും ആദ്യജാതന്‍' (കൊളോസിയക്കാര്‍ 1:15) എന്നിങ്ങനെയാണ്. യേശുക്രിസ്തുവിന് സ്വയം താന്‍ ദൈവമാണെന്ന വെളിപാട് ലഭിച്ചിട്ടില്ല. അങ്ങനെ ലഭിച്ചിരുന്നുവെങ്കില്‍ അദ്ദേഹം അത് പറയുമായിരുന്നു. എന്നാല്‍, പൗലോസിന് യേശു ദൈവമായിരുന്നുവെന്ന് വെളിപാട് കിട്ടി. പ്രസ്തുത വെളിപാട് എവിടെനിന്നായിരിക്കണം?

അബ്രഹാമിനോട് ദൈവം ചെയ്ത ഉടമ്പടിയായിട്ടാണ് പരിച്‌ഛേദനാകര്‍മത്തെ ബൈബിള്‍ പരിചയപ്പെടുത്തുന്നത്. 'നീയും നിനക്കു ശേഷം തലമുറയായി നിന്റെ സന്തതികളും പാലിക്കേണ്ട ഉടമ്പടി'യെന്നു പറഞ്ഞുകൊണ്ടാണ് അബ്രഹാമിനോട് കര്‍ത്താവ് പരിച്‌ഛേദന ചെയ്യുന്നതിനുള്ള കല്‍പന നല്‍കുന്നത് (ഉല്‍പത്തി 17:9-14) കര്‍ത്താവ് മോശയോടു പറഞ്ഞ തായി ബൈബിള്‍ ഉദ്ധരിക്കുന്നു: 'എട്ടാം ദിവസം ശിശുവിന്റെ പരിച്‌ഛേ ദനം നടത്തണം (ലേവിയര്‍ 12:3) ഈ ദൈവിക കല്‍പന യേശുവും അനുസ രിച്ചിരുന്നു. 'എട്ടു ദിവസം പൂര്‍ത്തിയായപ്പോള്‍ ശിശുവിന് പരിച്‌ഛേദനം നടത്തി' (ലൂക്കോസ് 2:21). പരിച്‌ഛേദനം ചെയ്യേണ്ടതില്ലെന്ന് യേശു ആരോടും പറഞ്ഞില്ല. കാരണം അദ്ദേഹത്തിന് അത്തരത്തിലുള്ള ഒരു ബോധനം ലഭിച്ചിരുന്നില്ല. എന്നാല്‍ പൗലോസ് പറയുന്നത് കാണുക: 'പരിച്‌ഛേദനം സ്വീകരിക്കുന്നുവെങ്കില്‍ നിങ്ങള്‍ക്ക് ക്രിസ്തുവിനെക്കൊണ്ട് നേട്ടമില്ല' (ഗലാത്തിയക്കാര്‍ 5:2). ഈ വെളിപാട് പൗലോസിന് എവിടെനിന്ന് കിട്ടി? ദൈവത്തില്‍ നിന്നാകാന്‍ വഴിയില്ല. പിന്നെയോ?

പിശാചില്‍നിന്നാണ് മുഹമ്മദി(ല)ന് വെളിപാടുണ്ടായത് എന്നുപറയാ നുള്ള പ്രധാനപ്പെട്ട കാരണം കുരിശുമരണത്തെയും പാപപരിഹാരബലി യെയും ഖുര്‍ആന്‍ നിഷേധിക്കുന്നുവെന്നതാണല്ലോ. യേശുവിനെയും മാതാവിനെയും പുകഴ്ത്തുകയും ആദരിക്കുകയും ചെയ്യുന്ന ഒട്ടനവധി സൂക്ത ങ്ങള്‍ ഖുര്‍ആനിലുണ്ട്. ഖുര്‍ആനില്‍ പേരു പരാമര്‍ശിക്കപ്പെട്ട ഏക വനിത മര്‍യമാണെന്നോര്‍ക്കുക. യേശു ചെയ്തതായി ബൈബിളില്‍ പറയാത്ത കളിമണ്‍പക്ഷികളില്‍ ഊതി അവയ്ക്ക് ജീവനിടുക തുടങ്ങിയ അത്ഭുതങ്ങ ളെക്കുറിച്ച് ഖുര്‍ആന്‍ പ്രതിപാദിക്കുന്നുമുണ്ട് (3:49). തൊട്ടിലില്‍ വെച്ച് ഉണ്ണിയേശു സംസാരിച്ചതായുള്ള ഖുര്‍ആനിക പരാമര്‍ശം (19:30) ബൈബി ളിലൊരിടത്തും കാണുവാന്‍ സാധ്യമല്ല. യേശുവിന്റെ വിശുദ്ധ വ്യക്തിത്വത്തില്‍ കളങ്കമുണ്ടാക്കുന്ന യാതൊന്നും ഖുര്‍ആനിലില്ല. യോഹന്നാന്റെ സുവിശേഷ പ്രകാരം ക്രിസ്തുവിന്റെ ആദ്യത്തെ അത്ഭുതം കാനായിലെ കല്യാണവിരുന്നില്‍ വെച്ച് മദ്യം നിര്‍മിച്ചു നല്‍കിയതാണെന്ന കാര്യം പ്രത്യേകം പ്രസ്താവ്യമാണ് (യോഹന്നാന്‍ 2:1-11). ഖുര്‍ആനില്‍ ഇത്തരം യാതൊരു പരാമര്‍ശവുമില്ല.

'മരത്തില്‍ തൂക്കിക്കൊല്ലപ്പെടുന്നവന്‍ ദൈവത്താല്‍ ശപിക്കപ്പെട്ടവനാണ്' (ആവര്‍ത്തനം 21:23)എന്നാണ് ബൈബിളിന്റെ സിദ്ധാന്തം. കുരിശില്‍ തറക്കുക വഴി യേശുവിനെ ശപിക്കപ്പെട്ടവനായി മുദ്രയടിക്കുകയാണ് തങ്ങ ള്‍ ചെയ്തതെന്നാണ് യഹൂദര്‍ കരുതിയത്. പൗലോസ് പറയുന്നതും മറ്റൊന്നല്ല. 'മരത്തില്‍ തൂക്കപ്പെടുന്നവരെല്ലാം ശപിക്കപ്പെട്ടവര്‍ എന്ന് എഴുതിയിരിക്കുന്നതുപോലെ ക്രിസ്തു നമുക്കുവേണ്ടി ശാപമായിത്തീരുന്നു' (ഗലാത്യര്‍ 3:13). അപ്പോള്‍ ക്രൂശീകരണം യേശുവിനെ ശപിക്കപ്പെട്ടവനാക്കുകയാണ് ചെയ്യുന്നത്. ലോകത്തിനുവേണ്ടി യേശു ശാപമായിത്തീര്‍ന്നുവെന്ന വാദം ഖുര്‍ആന്‍ അംഗീകരിക്കുന്നില്ല. ശാപത്തിന്റെ മരക്കുരിശില്‍നിന്ന് തന്നെ രക്ഷിക്കേണമേയെന്ന ക്രിസ്തുവിന്റെ പ്രാര്‍ഥന (മത്തായി 26:39) ദൈവം കേട്ടില്ലെന്നു കരുതുന്നത് ദൈവിക കാരുണ്യത്തിന്റെ നിഷേധമല്ലാതെ മറ്റെന്താണ്? ശപിക്കപ്പെട്ട മരക്കുരിശില്‍നിന്ന് പടച്ചതമ്പുരാന്‍ യേശുവിനെ രക്ഷിച്ചുകൊണ്ട് യഹൂദന്മാരുടെ ഗൂഢാലോചനയെ തകര്‍ക്കുകയാണ് ചെയ്തത് എന്നാണ് ഖുര്‍ആന്‍ പഠിപ്പിക്കുന്നത് (4:157,158).

മരക്കുരിശില്‍ ക്രൂശിക്കുക വഴി യേശുവിനെ ശപിക്കപ്പെട്ടവനാക്കിയെന്ന് യഹൂദന്മാര്‍. മരക്കുരിശില്‍ മരിച്ച് യേശു ശപിക്കപ്പെട്ടവനായിത്തീര്‍ന്നുവെന്ന് പൗ ലോസ്. മരക്കുരിശില്‍നിന്ന് പരിശുദ്ധനായ യേശുവിനെ ദൈവം രക്ഷിച്ചുവെന്ന് ഖുര്‍ആന്‍.

ഏതാണ് പിശാചിന്റെ വെളിപാട്? യേശുവിനെ മഹത്വപ്പെടുത്തുന്നതോ അതല്ല ശാപഗ്രസ്തനാക്കുന്നതോ? ചുരുക്കത്തില്‍, ഖുര്‍ആന്‍ പൈശാചിക വെളിപാടാണെന്ന് സമര്‍ഥിക്കു വാന്‍ വേണ്ടി തെളിവുകള്‍ പരതുന്നവര്‍ കുഴിക്കുന്ന കുഴികളില്‍ തങ്ങള്‍തന്നെയാണ് വീഴുന്നത് എന്നുള്ളതാണ് യാഥാര്‍ഥ്യം.

യുക്തിവാദികളായ വിമര്‍ശകന്മാര്‍ പ്രധാനമായും ഉന്നയിക്കുന്ന ആരോ പണമാണ് മുഹമ്മദ്(സ) നബിക്ക് ഉന്മാദരോഗ (Schizophrenia) മായിരുന്നുവെന്നത്. ദൈവത്തിന്റെ അസ്തിത്വം അംഗീകരിക്കാത്തവരെ സംബന്ധിച്ചിടത്തോളം അവര്‍ക്ക് വെളിപാടുകളുടെ സത്യതയെക്കുറിച്ച് എത്രതന്നെ പറഞ്ഞാലും ഉള്‍ക്കൊള്ളാന്‍ കഴിയില്ല. അതുകൊണ്ടുതന്നെ നിരീശ്വരവാദികളോടുള്ള ചര്‍ച്ച തുടങ്ങേണ്ടത് ദൈവാസ്തിത്വത്തെക്കുറിച്ച് സംസാരിച്ചുകൊണ്ടാണ്. പടച്ചതമ്പുരാന്റെ അസ്തിത്വംതന്നെ അംഗീകരിക്കാത്തവരെ അവനില്‍നിന്നുള്ള വെളിപാടുകള്‍ സത്യസന്ധമാണെന്ന് സമ്മതിപ്പിക്കുന്ന തെങ്ങനെ?

ചോദ്യത്തിന്റെ രണ്ടാം ഭാഗമാണ് ആദ്യമായി ചര്‍ച്ച ചെയ്യപ്പെടേണ്ടത്. സമകാലികരാല്‍ മുഹമ്മദ് (സ) ഭ്രാന്തനെന്ന് വിളിക്കപ്പെട്ടിരുന്നുവോ? ഉണ്ടെ ങ്കില്‍ ഭ്രാന്തിന്റെ എന്തെല്ലാം ലക്ഷണങ്ങളുടെ അടിസ്ഥാനത്തിലാണ് അവര്‍ ഈ ആരോപണം ഉന്നയിച്ചത്?

നാല്‍പതു വയസ്സുവരെ സത്യസന്ധനും സര്‍വരാലും അംഗീകരിക്ക പ്പെട്ട വ്യക്തിത്വത്തിന്റെ ഉടമയുമായിരുന്നു മുഹമ്മദ് (സ) . സുദീര്‍ഘമായ ഈ കാലഘട്ടത്തിനിടയ്ക്ക് ആരെങ്കിലും അദ്ദേഹത്തില്‍ ഏതെങ്കിലും തരത്തിലുള്ള മാനസിക വിഭ്രാന്തി ആരോപിച്ചിട്ടില്ല.

പ്രവാചകത്വത്തിനുശേഷം അദ്ദേഹം ഭ്രാന്തനെന്ന് ആരോപിക്കപ്പെട്ടിരുന്നുവെന്നത് ശരിയാണ്. ഭ്രാന്ത നെന്ന് മാത്രമല്ല മുഹമ്മദ്(സ) അധിക്ഷേപിക്കപ്പെട്ടത്; ജ്യോല്‍സ്യന്‍, മാരണ ക്കാരന്‍, മാരണം ബാധിച്ചവന്‍, കവി എന്നിങ്ങനെയുള്ള അധിക്ഷേപങ്ങളെ ല്ലാം അദ്ദേഹത്തിനുനേരെ ഉന്നയിക്കപ്പെട്ടിരുന്നു. അദ്ദേഹത്തിന്റെ വ്യക്തിത്വത്തിലോ മാനസിക സംതുലനത്തിലോ വല്ല വ്യത്യാസവും പ്രകടമായതുകൊണ്ടാണോ അവര്‍ അങ്ങനെ അധിക്ഷേപിച്ചത്? ആണെന്ന് അവരാരുംതന്നെ വാദിച്ചിട്ടില്ല. അവരുടെ പ്രശ്‌നം ഖുര്‍ആനും അതുള്‍ക്കൊള്ളുന്ന ആശയങ്ങളുമായിരുന്നു. തങ്ങളുടെ പാരമ്പര്യ വിശ്വാസങ്ങള്‍ക്കെതിരെയാണ് മുഹമ്മദ്(സ) സംസാരിക്കുന്നത്. അദ്ദേഹം ദൈവികമാണെന്ന് പറഞ്ഞുകൊണ്ട് ഓതിക്കേള്‍പ്പിക്കുന്ന ഖുര്‍ആനിലേക്ക് ജനങ്ങള്‍ ആകൃഷ്ടരാവുക യും ചെയ്യുന്നു. മുഹമ്മദി(സ)നെ സ്വഭാവഹത്യ നടത്താതെ ജനങ്ങളെ അദ്ദേഹത്തില്‍നിന്ന് അകറ്റാന്‍ മറ്റു മാര്‍ഗങ്ങളൊന്നുമില്ലെന്ന് കണ്ട പാരമ്പ ര്യമതത്തിന്റെ കാവല്‍ക്കാര്‍ ബോധപൂര്‍വം കെട്ടിച്ചമച്ച സ്വഭാവഹത്യയായിരുന്നു ഇവയെല്ലാം.

മുഹമ്മദ്(സ) പ്രവാചകത്വം പരസ്യമായി പ്രഖ്യാപിച്ചകാലം. ഹജ്ജ് മാസം ആസന്നമായി. അറേബ്യയുടെ വിവിധ ഭാഗങ്ങളില്‍നിന്നും ഹജ്ജിനു വരുന്നവരോട് മുഹമ്മദ് (സ) മതപ്രബോധനം നടത്തുമെന്നും ഖുര്‍ആനിന്റെ വശ്യതയില്‍ അവര്‍ ആകൃഷ്ടരാവുമെന്നും മക്കയിലെ പ്രമാണിമാര്‍ ഭയന്നു. അവര്‍ യോഗം ചേര്‍ന്നു. ഹജ്ജിന് എത്തിച്ചേരുന്നവരോട് ആദ്യമേതന്നെ മുഹമ്മദി(സ)നെതിരെ പ്രചാരവേലകള്‍ നടത്താന്‍ തീരുമാനിച്ചു. മുഹമ്മദി(സ)നെ എങ്ങനെ വിശേഷിപ്പിക്കണം, എന്നതായി പിന്നീടുള്ള ചര്‍ച്ച. പലരും പല രൂപത്തില്‍ പറയുന്നത് തങ്ങളുടെ വിശ്വാസ്യത തകര്‍ക്കും. എല്ലാവര്‍ക്കും ഒരേ രൂപത്തില്‍ പറയാന്‍ പറ്റുന്ന ആരോപണമെന്ത്? ചിലര്‍ പറഞ്ഞു: ''നമുക്ക് മുഹമ്മദ് ഒരു ജ്യോല്‍സ്യനാണെന്ന് പറയാം''. പൗരപ്രമുഖനായ വലീദുബ്‌നുമുഗീറ പറഞ്ഞു: ''പറ്റില്ല, അല്ലാഹുവാണ് സത്യം അവ ന്‍ ജ്യോല്‍സ്യനല്ല. ജ്യോല്‍സ്യന്മാരെ നാം കണ്ടിട്ടുണ്ട്. മുഹമ്മദിന്റെ വാക്കുകള്‍ ജ്യോല്‍സ്യന്മാരുടെ പ്രവചനങ്ങളല്ല''. മറ്റു ചിലര്‍ പറഞ്ഞു: ''നമുക്ക് അവന്‍ ഭ്രാന്തനാണെന്ന് പറയാം''. വലീദ് പറഞ്ഞു: ''അവന്‍ ഭ്രാന്തനല്ല. ഭ്രാന്തന്മാരെ നാം കണ്ടിട്ടുണ്ട്. അവരുടെ ഭ്രാന്തമായ സംസാരങ്ങളോ ഗോഷ്ഠികളോ പിശാചുബാധയോ ഒന്നും അവനില്ല''. അവര്‍ പറഞ്ഞു: ''എങ്കില്‍ അവന്‍ കവിയാണെന്ന് പറയാം''. വലീദ് പ്രതിവചിച്ചു: ''അവന്‍ കവിയല്ല. കവിതയുടെ എല്ലാ ഇനങ്ങളും നമുക്കറിയാം. അവന്‍ പറയുന്ന ത് കവിതയല്ല''. ജനം പറഞ്ഞു: ''എങ്കില്‍ അവന്‍ മാരണക്കാരനാണെന്ന് പറയാം' വലീദ് പ്രതികരിച്ചു: ''അവന്‍ മാരണക്കാരനുമല്ല. മാരണക്കാരെ നമുക്കറിയാം. അവരുടെ കെട്ടുകളോ, ഊത്തുകളോ ഒന്നും അവന്‍ പ്രയോഗിക്കുന്നില്ല''. അവര്‍ ചോദിച്ചു: ''പിന്നെ എന്താണ് നിങ്ങളുടെ നിര്‍ദേശം?'' അദ്ദേഹം പറഞ്ഞു: ''തീര്‍ച്ചയായും അവന്റെ വചനങ്ങളില്‍ മാധുര്യമുണ്ട്. അതിന്റെ മൂലം വിസ്തൃതവും ശാഖകള്‍ ഫലസമൃദ്ധവുമാണ്. നിങ്ങള്‍ അവനെപ്പറ്റി എന്തു പറഞ്ഞാലും അതു നിരര്‍ഥകമാണെന്നു തെളിയും. പിതാവിനും മക്കള്‍ക്കുമിടയിലും ഭാര്യക്കും ഭര്‍ത്താവിനുമിടയിലും ജ്യേഷ്ഠനും അനുജനുമിടയിലും പിളര്‍പ്പുണ്ടാക്കുവാന്‍ വേണ്ടി വന്ന ജാലവിദ്യക്കാരനാണ് അവനെന്ന് പറയുന്നതാണ് നല്ലത്!'' ജനം ഇതംഗീകരിച്ചു. അവര്‍ പ്രചാര ണം തുടങ്ങി.

ഈ സംഭവം മനസ്സിലാക്കിത്തരുന്ന വസ്തുതയെന്താണ്? പ്രവാചകപ്ര ബോധനങ്ങളില്‍ നിന്ന് ജനങ്ങളെ പിന്തിരിപ്പിക്കുവാന്‍ വേണ്ടി ശത്രുക്കള്‍ മെനഞ്ഞെടുത്ത പലതരം ദുഷ്പ്രചാരണങ്ങളിലൊന്നു മാത്രമാണ് അദ്ദേ ഹം ഭ്രാന്തനാണെന്ന ആരോപണം. ഈ പ്രചാരണം നടത്തിയിരുന്നവര്‍ക്കു തന്നെ അതില്‍ വിശ്വാസമുണ്ടായിരുന്നില്ല. അതുകൊണ്ടുതന്നെ, അവരുടെ പ്രചാരണത്തെ ഒരു തെളിവായി സ്വീകരിക്കുന്നത് അബദ്ധമാണ്.

പ്രവാചകന്‍ ജീവിച്ചത് പതിനാലു നൂറ്റാണ്ടുകള്‍ക്കുമുമ്പാണ്. അദ്ദേഹ ത്തിന് ഉന്മാദരോഗമുണ്ടായിരുന്നുവോയെന്ന് വസ്തുനിഷ്ഠമായി പരിശോധിക്കുവാന്‍ ഇപ്പോള്‍ അദ്ദേഹം നമ്മുടെ മുന്നില്‍ ജീവിച്ചിരിക്കാത്തതിനാല്‍ ഇന്ന് നമുക്ക് കഴിയില്ല. അദ്ദേഹത്തിനുണ്ടായ വെളിപാടുകളും സ്വപ്‌നദര്‍ശ നങ്ങളുമാണ് പ്രവാചകന്‍ ഉന്മാദരോഗിയായിരുന്നുവെന്ന് വാദിക്കുന്നവര്‍ക്കുള്ള തെളിവ്. വെളിപാടുകള്‍ സ്വീകരിക്കുമ്പോള്‍ പ്രവാചകനില്‍ കാണ പ്പെട്ട ഭാവവ്യത്യാസങ്ങളെയും വഹ്‌യ് എങ്ങനെയാണെന്നുള്ള പ്രവാചക ന്റെ വിവരങ്ങളെയും വിശദീകരിക്കുന്ന ഹദീഥുകളുടെ വെളിച്ചത്തിലാണ് വിമര്‍ശകന്മാര്‍ ഈ വാദമുന്നയിക്കുന്നത്. ഉന്മാദരോഗത്തിന്റെ ലക്ഷണ ങ്ങള്‍ പ്രവാചകനില്‍ കാണപ്പെട്ടിരുന്നുവോയെന്ന് വസ്തുനിഷ്ഠമായി പരി ശോധിച്ചാല്‍ ഈ വാദത്തില്‍ യാതൊരു കഴമ്പുമില്ലെന്ന് സുതരാം വ്യക് തമാവും.

ഒന്ന്: ഉന്മാദരോഗികളുടെ സ്വഭാവം നിരന്തരം മാറിക്കൊണ്ടിരിക്കും. മറ്റുള്ളവരോടുള്ള പെരുമാറ്റത്തിലും സംസാരത്തിലുമെല്ലാം ഈ വൈരുധ്യം പ്രകടമായിരിക്കും.

മുഹമ്മദി(സ)ന്റെ ജീവിതവും സംസാരങ്ങളും പരിശോധിക്കുക.

യാതൊരു രീതിയിലുള്ള സ്വഭാവ വൈരുദ്ധ്യങ്ങളും അദ്ദേഹത്തില്‍ നമുക്ക് കാണാന്‍ കഴിയില്ല. മാറിക്കൊണ്ടിരിക്കുന്ന പെരുമാറ്റ രീതികളുടെയും പൂര്‍വാപരബന്ധമില്ലാത്ത സംസാരത്തിന്റെയും ഉടമസ്ഥനായിരുന്നു മുഹമ്മദ് നബി(ല)യെങ്കില്‍ അദ്ദേഹത്തിന് പരശ്ശതം അനുയായികളുണ്ടായതെങ്ങ നെ? സാധാരണയായി നാം മനസ്സിലാക്കുന്ന 'ദിവ്യന്‍'മാരുടെ അനുയായികളെപ്പോലെയായിരുന്നില്ല മുഹമ്മദി(സ)ന്റെ അനുചരന്മാര്‍. അദ്ദേഹത്തിന്റെ ഉപദേശങ്ങള്‍ പ്രാവര്‍ത്തികമാക്കുന്നതിനുവേണ്ടി മല്‍സരിക്കുകയായിരുന്നു അവര്‍. ഒരു ഉന്മാദരോഗിയുടെ വാക്കുകള്‍ അനുസരിക്കുവാന്‍ വേണ്ടി ജനസഹസ്രങ്ങള്‍ മല്‍സരിച്ചുവെന്ന് പറഞ്ഞാല്‍ അത് വിശ്വസിക്കാനാവുമോ?

രണ്ട്: ഉന്മാദരോഗികളുടെ പ്രതികരണങ്ങള്‍ വൈരുധ്യാത്മകമായിരിക്കും. സന്തോഷവേളയില്‍ പൊട്ടിക്കരയുകയും സന്താപവേളയില്‍ പൊട്ടിച്ചിരിക്കുകയും ചെയ്യും. വെറുതെ ചിരിക്കുകയും കരയുകയും ചെയ്യുന്ന സ്വഭാവവും കണ്ടുവരാറുണ്ട്.

മുഹമ്മദ് നബി(സ) യുടെ പ്രതികരണങ്ങള്‍ സമചിത്തതയോടുകൂടിയുള്ളതായിരുന്നു. ഒരു സംഭവം: പ്രവാചകന്‍(സ)ഒരു മരത്തണലില്‍ വിശ്രമിക്കുകയാണ്. പെട്ടെന്ന് ഊരിപ്പിടിച്ച വാളുമായി മുന്നില്‍ ഒരു കാട്ടാളന്‍ പ്രത്യക്ഷപ്പെ ട്ടു. അയാള്‍ ചോദിച്ചു: ''എന്നില്‍നിന്ന് നിന്നെ ഇപ്പോള്‍ ആര് രക്ഷിക്കും?'' പ്രവാചകന്‍ അക്ഷോഭ്യനായി മറുപടി പറഞ്ഞു: 'അല്ലാഹു'. ഈ മറുപടിയു ടെ ദൃഢത കേട്ട് കാട്ടാളന്റെ കൈയില്‍നിന്ന് വാള്‍ വീണുപോയി. (ബുഖാരി, മുസ്‌ലിം)

ഒരു ഉന്മാദരോഗിയില്‍നിന്ന് ദൃഢചിത്തതയോടുകൂടിയുള്ള ഇത്തരം പെരുമാറ്റങ്ങള്‍ പ്രതീക്ഷിക്കുവാന്‍ കഴിയുമോ?

മൂന്ന്: ഉന്മാദരോഗികള്‍ അന്തര്‍മുഖരായിരിക്കും. പുറമെയുള്ള ലോക ത്ത് നടക്കുന്ന സംഭവങ്ങളിലൊന്നും അവര്‍ക്ക് യാതൊരു താല്‍പര്യവും കാണുകയില്ല.

മുഹമ്മദ് നബി(സ)അന്തര്‍മുഖനായിരുന്നില്ല. തന്റെ ചുറ്റുപാടും നടക്കുന്ന സംഭവങ്ങള്‍ അതീവ താല്‍പര്യത്തോടെ നിരീക്ഷിക്കുകയും തന്റെ പങ്ക് ആവശ്യമെങ്കില്‍ നിര്‍വഹിക്കുകയും ചെയ്തിരുന്ന വ്യക്തിയാണദ്ദേ ഹം. ജനങ്ങള്‍ക്ക് ധാര്‍മിക നിര്‍ദേശങ്ങള്‍ നല്‍കുക മാത്രമല്ല, അവര്‍ക്ക് മാതൃകയായി ജീവിച്ച് കാണിച്ചുകൊടുക്കുകകൂടി ചെയ്ത വ്യക്തിയായിരുന്നു അദ്ദേഹം.

ലാമാര്‍ട്ടിന്‍ എഴുതി: 'തത്ത്വജ്ഞാനി, പ്രസംഗകന്‍, ദൈവദൂതന്‍, നിയമ നിര്‍മാതാവ്, പോരാളി, ആശയങ്ങളുടെ ജേതാവ്, അബദ്ധ സങ്കല്‍പങ്ങളില്‍നിന്ന് മുക്തമായ ആചാര വിശേഷങ്ങളുടെയും യുക്തിബന്ധുരമായ വിശ്വാസപ്രമാണങ്ങളുടെയും പുനഃസ്ഥാപകന്‍, ഇരുപത് ഭൗതിക സാമ്രാജ്യങ്ങളുടെ സ്ഥാപകന്‍ -അതായിരുന്നു മുഹമ്മദ്. മനുഷ്യത്വത്തിന്റെ എല്ലാ മാനദണ്ഡങ്ങളും വെച്ച് പരിഗണിക്കുമ്പോള്‍ നാം വ്യക്തമായും ചോദിച്ചേക്കാം. മുഹമ്മദിനേക്കാള്‍ മഹാനായ മറ്റു വല്ല മനുഷ്യനുമുണ്ടോ?''(Historie De La turquie., Vol, 2 Page 277)

അന്തര്‍മുഖനായ ഒരു ഉന്മാദരോഗിയെക്കുറിച്ച വിലയിരുത്തലാണോ ഇത്?

നാല്: ഉന്മാദരോഗികള്‍ക്ക് നിര്‍ണിതമായ എന്തെങ്കിലും ലക്ഷ്യത്തിനുവേണ്ടി വ്യവസ്ഥാപിതമായി പ്രവര്‍ത്തിക്കാന്‍ കഴിയില്ല. കാര്യമായി യാതൊന്നും ചെയ്യാനാവാത്ത ഇവര്‍ ശാരീരികമായും മാനസികമായും തളര്‍ന്നവരായിരിക്കും.

മുഹമ്മദ് നബി(സ) ജനങ്ങളെ സത്യമാര്‍ഗത്തിലേക്ക് നയിക്കുന്നതിനുവേണ്ടി അയക്കപ്പെട്ട ദൈവദൂതന്മാരില്‍ അന്തിമനായിരുന്നു. തന്നിലേല്‍പിക്കപ്പെട്ട ഉത്തരവാദിത്തം രണ്ടു ദശാബ്ദത്തിലധികം ഭംഗിയായി നിര്‍വഹിക്കാന്‍ അദ്ദേഹത്തിന് കഴിഞ്ഞു. ചിട്ടയോടുകൂടിയുള്ള പ്രബോധന പ്രവര്‍ ത്തനങ്ങള്‍ വഴി ജനസഹസ്രങ്ങളെ ദൈവികമതത്തിലേക്ക് ആകര്‍ഷിക്കുവാന്‍ മുഹമ്മദി(സ)ന് സാധിച്ചു. സാംസ്‌കാരിക രംഗത്ത് വട്ടപ്പൂജ്യത്തിലായിരുന്ന ഒരു ജനവിഭാഗത്തെ ലോകത്തിന് മുഴുവന്‍ മാതൃകയാക്കി പരിവര്‍ത്തിപ്പിക്കുവാന്‍ വേണ്ടിവന്നത് കേവലം ഇരുപത്തിമൂന്ന് വര്‍ഷങ്ങള്‍ മാത്രം. ലോകചരിത്രത്തെ ഏറ്റവുമധികം സ്വാധീനിച്ച വ്യക്തിയാണ് മുഹമ്മദ്(സ) എന്ന് ചരിത്രത്തെ നിഷ്പക്ഷമായി നോക്കിക്കണ്ടവരെല്ലാം അഭിപ്രായപ്പെട്ടിട്ടുണ്ട്.

ഇതെല്ലാം ഒരു ഉന്മാദരോഗിക്ക് കഴിയുന്നതാണെന്ന് പ്രസ്തുത രോഗ ത്തെക്കുറിച്ച് അല്‍പമെങ്കിലും അറിയുന്നവരാരെങ്കിലും സമ്മതിക്കുമോ?

അഞ്ച്: ഉന്മാദരോഗി അശരീരികള്‍ കേള്‍ക്കുകയും(Auditory Hallucination) മിഥ്യാഭ്രമത്തിലായിരിക്കുകയും (Delusion)  മായാദൃശ്യങ്ങള്‍ കാണുക യും(Hallucination) ചെയ്യും. ഈ അശരീരികളും മായാദൃശ്യങ്ങളും യാഥാര്‍ഥ്യവുമായി യാതൊരു ബന്ധവുമുള്ളതായിരിക്കില്ല.

മുഹമ്മദ് നബി(സ)ക്കുണ്ടായ വെളിപാടുകളും ദര്‍ശനങ്ങളും ഈ ഗണത്തില്‍ പെടുത്തിക്കൊണ്ടാണ് വിമര്‍ശകര്‍ അദ്ദേഹത്തില്‍ ഉന്മാദരോഗം ആരോപിക്കുന്നത്. ഉന്മാദരോഗത്തിന്റെ മറ്റു ലക്ഷണങ്ങളൊന്നും നബി(സ)യില്‍ ഉണ്ടായിരുന്നില്ലെന്ന് നാം മനസ്സിലാക്കി. അപ്പോള്‍ ഈ വെളിപാടുകളുടെ മാത്രം വെളിച്ചത്തില്‍ അദ്ദേഹം ഉന്മാദരോഗിയാണെന്ന് പറയുന്ന തെങ്ങനെ? ഉന്മാദരോഗിക്കുണ്ടാവുന്ന 'വെളിപാടു'കള്‍ അയാളുടെ രോഗത്തിന്റെ ലക്ഷണമാണ്. ഈ വെളിപാടുകള്‍ അയാളുടെ വൈയക്തിക മേഖലകളുമായി മാത്രം ബന്ധപ്പെട്ടതായിരിക്കും. എന്നാല്‍, മുഹമ്മദി(സ)നുണ്ടായ വെളിപാടുകളോ? ആ വെളിപാടുകള്‍ ഒരു ഉത്തമ സമൂഹത്തെ പടിപടിയായി വാര്‍ത്തെടുക്കുകയായിരുന്നു. ആദ്യം ദൈവബോധവും പര ലോകചിന്തയും ജനങ്ങളില്‍ വളര്‍ത്തി. ഘട്ടം ഘട്ടമായി സമൂഹത്തെ മുച്ചൂടും ബാധിച്ചിരുന്ന എല്ലാ തിന്മകളുടെയും അടിവേരറുത്തു. അങ്ങനെ ഒരു മാതൃകാ സമൂഹത്തിന്റെ സൃഷ്ടിക്ക് നിമിത്തമാകുവാന്‍ മുഹമ്മദി(സ)ന് ലഭിച്ച വെളിപാടുകള്‍ക്ക് കഴിഞ്ഞു. അത് സൃഷ്ടിച്ച വിപ്ലവം മഹത്തരമാണ്. ചരിത്രകാലത്ത് അതിനു തുല്യമായ മറ്റൊരു വിപ്ലവം നടന്നിട്ടില്ല.

ഉന്മാദരോഗി കേള്‍ക്കുന്ന അശരീരികള്‍ക്ക് ഒരു മാതൃകാ സമൂഹത്തിന്റെ സൃഷ്ടിക്കോ നിസ്തുലമായ ഒരു വിപ്ലവത്തിനോ നിമിത്തമാകുവാന്‍ കഴിയുമോ?

മുഹമ്മദി(സ)ന് ഉന്മാദരോഗമായിരുന്നുവെന്നും അദ്ദേഹം ശ്രവിച്ച അശ രീരികളാണ് ഖുര്‍ആനിലുള്ളതെന്നുമുള്ള വാദം പരിഗണനപോലും അര്‍ഹിക്കാത്ത ആരോപണം മാത്രമാണെന്നാണ് ഇതില്‍നിന്നെല്ലാം വ്യക്തമാകുന്നത്.

നങ്ങളെ ധാര്‍മികതയിലേക്ക് നയിക്കുന്ന ഗ്രന്ഥമാണ് ഖുര്‍ആന്‍. മദ്യ ത്തിലും മദിരാക്ഷിയിലും യുദ്ധങ്ങളിലും സായൂജ്യമടഞ്ഞിരുന്ന ഒരു സമൂഹ ത്തെ കേവലം 23 വര്‍ഷക്കാലം കൊണ്ട് ധാര്‍മികതയുടെ പ്രയോക്താക്കളും പ്രചാരകരുമാക്കിയ ഗ്രന്ഥമെന്ന ഖ്യാതി ഖുര്‍ആനിനു മാത്രം അവകാശപ്പെ ട്ടതാണ്. എന്നാല്‍ ധാര്‍മിക നവോത്ഥാനത്തിനുവേണ്ടി മുഹമ്മദ്(സ) രചി ച്ചുകൊണ്ട് ദൈവത്തില്‍ ആരോപിച്ച ഗ്രന്ഥമാണ് ഖുര്‍ആന്‍ എന്ന വാദ ഗതി അടിസ്ഥാന രഹിതമാണെന്ന് അത് ഒരാവര്‍ത്തി വായിക്കുന്ന ഏവര്‍ക്കും ബോധ്യമാവും. താഴെപ്പറയുന്ന വസ്തുതകള്‍ ശ്രദ്ധിക്കുക.

ഒന്ന്: സത്യസന്ധനായിരുന്നു മുഹമ്മദ് (സ) എന്ന കാര്യത്തില്‍ പക്ഷാ ന്തരമില്ല. അത്തരമൊരാള്‍ ധാര്‍മിക നവോത്ഥാനത്തിനുവേണ്ടി ദൈവത്തിന്റെ പേരില്‍ ഒരു പച്ചക്കള്ളം പറഞ്ഞുവെന്നു കരുതുന്നത് യുക്തി സഹമ ല്ല. ധാര്‍മിക നവോത്ഥാനത്തിനുവേണ്ടി ആത്മാര്‍ഥമായി പരിശ്രമിക്കുന്ന ഒരു വ്യക്തി അക്കാര്യത്തിനുവേണ്ടി സ്വന്തമായി ഒരു വലിയ അധര്‍മം ചെയ്യുകയെന്നത് അവിശ്വസനീയമാണ്. ദൈവത്തിന്റെ പേരില്‍ കളവ് പറയുന്നതിനേക്കാള്‍ വലിയ പാപമെന്താണ്?

രണ്ട്: പടച്ചവന്റെ പേരില്‍ കളവു പറയുകയും സ്വയം കൃതരചന കള്‍ ദൈവത്തിന്‍േറതാണെന്ന് വാദിക്കുകയും ചെയ്യുന്നവനാണ് ഏറ്റവും വലിയ അക്രമിയെന്നാണ് ഖുര്‍ആന്‍ പറയുന്നത്. ''അല്ലാഹുവിന്റെ പേരില്‍ കള്ളം കെട്ടിച്ചമയ്ക്കുകയോ, തനിക്ക് യാതൊരു ബോധനവും നല്‍കപ്പെടാതെ 'എനിക്ക് ബോധനം ലഭിച്ചിരിക്കുന്നു' എന്ന് പറയുകയോ ചെയ്തവനേ ക്കാളും അല്ലാഹു അവതരിപ്പിച്ചതുപോലെയുള്ളത് ഞാനും അവതരിപ്പിക്കാമെന്ന് പറഞ്ഞവനേക്കാളും വലിയ അക്രമി ആരുണ്ട്?''(6:93). ഖുര്‍ആന്‍ മുഹമ്മദി(സ)ന്റെ രചനയാണെങ്കില്‍ ഈ സൂക്തത്തില്‍ പറഞ്ഞ 'ഏറ്റവും വലിയ അക്രമി' അദ്ദേഹം തന്നെയായിരിക്കുമല്ലോ. തന്നെത്തന്നെ 'ഏറ്റവും വലിയ അക്രമി'യെന്ന് വിളിക്കുവാനും അതു രേഖപ്പെടുത്തുവാനും അദ്ദേ ഹം തയാറാകുമായിരുന്നുവോ?

മൂന്ന്: സ്വയംകൃത രചനകള്‍ നടത്തി അത് ദൈവത്തില്‍ ആരോപിക്കു ന്നവരെ ഖുര്‍ആന്‍ ശപിക്കുന്നുണ്ട്. ''എന്നാല്‍ സ്വന്തം കൈകള്‍ കൊണ്ട് ഗ്രന്ഥം എഴുതിയുണ്ടാക്കുകയും എന്നിട്ട് അത് അല്ലാഹുവില്‍നിന്ന് ലഭിച്ച താണെന്ന് പറയുകയും ചെയ്യുന്നവര്‍ക്ക് നാശം!''(2:79) ഖുര്‍ആന്‍ മുഹ മ്മദി(സ)ന്റെ സൃഷ്ടിയാണെങ്കില്‍ ഈ ശാപം അദ്ദേഹത്തിനുകൂടി ബാധക മാണല്ലോ. സ്വന്തമായി ഒരു രചന നിര്‍വഹിക്കുക. ആ രചനയില്‍ സ്വന്ത ത്തെത്തന്നെ ശപിക്കുക. ഇത് വിശ്വസനീയമാണോ?

നാല്: ഖുര്‍ആന്‍ ഒന്നിച്ച് അവതരിപ്പിക്കപ്പെട്ട ഗ്രന്ഥമല്ല. നീണ്ട ഇരുപ ത്തിമൂന്ന് വര്‍ഷങ്ങള്‍ക്കിടയില്‍ വ്യത്യസ്ത സാഹചര്യങ്ങളിലാണ് ഖുര്‍ആ ന്‍ സൂക്തങ്ങള്‍ അവതരിപ്പിക്കപ്പെട്ടത്. ഓരോ വിഷയങ്ങളിലും ജനങ്ങളുടെ ചോദ്യങ്ങള്‍ക്കുള്ള മറുപടിയായിട്ടാണ് ചില സന്ദര്‍ഭങ്ങളില്‍ ഖുര്‍ആന്‍ സൂക്തങ്ങള്‍ അവതരിപ്പിച്ചിരിക്കുന്നത്. ഖുര്‍ആനില്‍ പതിനഞ്ചോളം സ്ഥ ലങ്ങളില്‍ 'അവര്‍ നിന്നോട്...നെക്കുറിച്ചു ചോദിക്കുന്നു. പറയുക: ...' എന്ന ശൈലിയിലുള്ള സൂക്തങ്ങളുണ്ട്. ഓരോ വിഷയങ്ങളിലും പ്രവാചകനോട് അവര്‍ ചോദിച്ച സമയത്ത് അദ്ദേഹത്തിന് ഉത്തരം നല്‍കാന്‍ സാധിച്ചില്ലെ ന്നും പിന്നീട് ഖുര്‍ആന്‍ വാക്യം അവതരിപ്പിച്ചതിനുശേഷം മാത്രമാണ് അത് സാധിച്ചതെന്നുമാണല്ലോ ഇതില്‍നിന്ന് മനസ്സിലാവുന്നത്. ധാര്‍മിക നവോത്ഥാനം ലക്ഷ്യമാക്കിക്കൊണ്ടുള്ള പ്രവാചക(സ)ന്റെ രചനയായിരുന്നു ഖുര്‍ആനെങ്കില്‍ ജനം ചോദിച്ചപ്പോള്‍ ഉടന്‍തന്നെ അദ്ദേഹത്തിന് മറുപടി പറയാന്‍ കഴിയുമായിരുന്നു.

ഉദാഹരണത്തിന്, മദ്യത്തില്‍നിന്നും ചൂതാട്ടത്തില്‍നിന്നും ജനങ്ങളെ രക്ഷിക്കണമെന്നായിരുന്നു പ്രവാചകന്റെ ഉദ്ദശ്യമെങ്കില്‍ അവയെക്കുറിച്ച് ചോദിച്ച ഉടന്‍തന്നെ അവ പാപമാണ് എന്ന് അ ദ്ദേഹം മറുപടി പറയുമായിരുന്നു. എന്നാല്‍, അദ്ദേഹം ചെയ്തത് അതല്ല; സ്വയം മറുപടി പറയാതെ ദൈവിക വെളിപാട് പ്രതീക്ഷിക്കുകയായിരുന്നു. ദൈവവചനങ്ങള്‍ വെളിപ്പെട്ടതിനുശേഷമാണ് ഈ തിന്മകള്‍ക്കെതിരെയുള്ള നടപടികള്‍ അദ്ദേഹം സ്വീകരിച്ചത്.

അഞ്ച്: മുഹമ്മദ് നബി(സ)യെ തിരുത്തുന്ന ചില ഖുര്‍ആന്‍ സൂക്തങ്ങളുണ്ട്. ഖുറൈശി പ്രമുഖരുമായി സംസാരിച്ചുകൊണ്ടിരിക്കുമ്പോള്‍ കടന്നുവന്ന അന്ധനായ അബ്ദുല്ലാഹിബ്‌നുഉമ്മിമക്തൂമിനെ പ്രസന്നതയോടെ സ്വീകരിക്കാതിരുന്ന പ്രവാചക(സ)ന്റെ നടപടിയെ തിരുത്തിയ ഖുര്‍ആന്‍ സൂക്തങ്ങള്‍ (80:1-10) സുവിദിതമാണ്.

മറ്റൊരു സംഭവം: മുസ്‌ലിംകള്‍ക്ക് ഏറെ നാശനഷ്ടങ്ങള്‍ വിതച്ച ഉഹ്ദ് യുദ്ധത്തില്‍ പ്രവാചകന്റെ ശരീരത്തിലും ഒരുപാട് മുറിവുകള്‍ ഉണ്ടായി. യുദ്ധശേഷം അദ്ദേഹം അവിശ്വാസികളി ല്‍ ചിലരെ ശപിക്കുകയും 'അവരുടെ പ്രവാചകനെ മുറിപ്പെടുത്തിയ സമൂഹ മെങ്ങനെയാണ് നന്നാവുക?' എന്ന് ആത്മഗതം നടത്തുകയും ചെയ്തു. ഉടന്‍ ഖുര്‍ആന്‍ സൂക്തമവതരിച്ചു; പ്രവാചക(സ)നെ തിരുത്തിക്കൊണ്ട്. ''(നബിയേ), കാര്യത്തിന്റെ തീരുമാനത്തില്‍ നിനക്ക് യാതൊരവകാശവുമില്ല. അവന്‍ (അല്ലാഹു) ഒന്നുകില്‍ അവരുടെ പശ്ചാത്താപം സ്വീകരിച്ചേ ക്കാം. അല്ലെങ്കില്‍ അവരെ അവന്‍ ശിക്ഷിച്ചേക്കാം. തീര്‍ച്ചയായും അവര്‍ അക്രമികളാകുന്നു''(3:128)(തിര്‍മിദി,ഇബ്‌നുമാജ).

ഇതൊന്നും പ്രവാചകനി ല്‍ ബോധപൂര്‍വ്വം വന്ന തെറ്റുകളല്ല. താന്‍ സ്വീകരിച്ച നിലപാടുകളിലുണ്ടാ യ അബദ്ധം മാത്രം. എന്നിട്ടും അവ തിരുത്തുന്ന വചനങ്ങള്‍ ഖുര്‍ആനി ലുണ്ടായി. ജനങ്ങളെ ധര്‍മനിഷ്ഠരാക്കുവാന്‍ വേണ്ടി പ്രവാചകന്‍(സ) പടച്ച ഗ്രന്ഥമായിരുന്നു ഖുര്‍ആനെങ്കില്‍ അദ്ദേഹത്തിന്റെ നടപടികളെ വിമര്‍ശിക്കുന്ന സൂക്തങ്ങള്‍ ഖുര്‍ആനിലുണ്ടാവുമായിരുന്നുവോ?

അറബികളെ ഐക്യപ്പെടുത്തുകയും പുരോഗതിയിലേക്ക് നയിക്കുകയു മായിരുന്നു ഖുര്‍ആനിന്റെ പിന്നിലുള്ള ലക്ഷ്യമെങ്കില്‍ അതിലെ പ്രതിപാദന ങ്ങളില്‍നിന്ന് ഇക്കാര്യം വ്യക്തമാവുമായിരുന്നു. എന്നാല്‍, ഖുര്‍ആന്‍ ഒരാവര്‍ത്തി വായിച്ച ഒരാള്‍ക്ക് അതില്‍ അറബി ദേശീയതയുടെ ഉയിര്‍ത്തെഴുന്നേല്‍പ്പ് ഒരു വിഷയമായി വരുന്നേയില്ലെന്ന വസ്തുത വ്യക്തമാവുന്നതാണ്. അറബികളുടെ നവോത്ഥാനമായിരുന്നു ഖുര്‍ആന്‍ രചനക്കുപിന്നിലുള്ള ഉദ്ദേശ്യമെന്ന വാദം താഴെ പറയുന്ന വസ്തുതകള്‍ക്കുമുന്നില്‍ അടിസ്ഥാന രഹിതമായിത്തീരുന്നു.

ഒന്ന്: അറബികളുടെ നവോത്ഥാനത്തെയോ ഐക്യത്തെയോ പ്രോല്‍സാഹിപ്പിക്കുന്ന രീതിയിലുള്ള ഒരു വചനംപോലും ഖുര്‍ആനിലില്ല.

രണ്ട്: ദേശീയമായ അതിര്‍വരമ്പുകളില്ലാത്ത ആദര്‍ശസമൂഹമെന്ന സങ്ക ല്‍പമാണ് ഖുര്‍ആന്‍ മുന്നോട്ടുവെക്കുന്നത്. 'ഉമ്മത്ത്' എന്ന സാങ്കേതിക സംജ്ഞയാല്‍ വിശേഷിപ്പിക്കപ്പെട്ടിരിക്കുന്ന ആദര്‍ശസമൂഹത്തില്‍ സത്യ വിശ്വാസം സ്വീകരിച്ച ഏവരും ദേശീയതയുടെയോ പ്രാദേശികത്വത്തിന്റെ യോ വര്‍ഗത്തിന്റെയോ ജാതീയതയുടെയോ അതിര്‍വരമ്പുകളില്ലാതെ അംഗങ്ങളാണ്. അറബിദേശീയതയെന്ന സങ്കല്‍പംതന്നെ ഖുര്‍ആനിന് അന്യമാണ്.

മൂന്ന്: അറബികളുടെ നവോത്ഥാനമായിരുന്നു മുഹമ്മദി(സ)ന്റെ ല ക്ഷ്യമെങ്കില്‍ അധികാരം വാഗ്ദാനം ചെയ്യപ്പെട്ടപ്പോള്‍ അത് സ്വീകരിക്കുക യും ശക്തിയും പാടവവുമുപയോഗിച്ച് അവരെ ഐക്യപ്പെടുത്തുകയും പുരോഗതിയിലേക്ക് നയിക്കുകയും ചെയ്യാന്‍ അദ്ദേഹം ശ്രമിക്കുമായിരുന്നു. എന്നാല്‍, അതുണ്ടായില്ല. അധികാരം സ്വീകരിച്ചുകൊണ്ട് നവോത്ഥാനത്തിന് ശ്രമിക്കുന്നതിന് പകരം അത് നിരസിക്കുകയാണ് അദ്ദേഹം ചെയ്തത്.

നാല്: അധികാരം ലഭിച്ചതിനുശേഷവും അദ്ദേഹം അറബികളുടെ ഏതെ ങ്കിലും തരത്തിലുള്ള ഔന്നത്യത്തിനുവേണ്ടി വാദിച്ചിട്ടില്ല. തന്റെ അന്തിമ പ്രസംഗത്തില്‍ അദ്ദേഹം അര്‍ഥശങ്കക്കിടയില്ലാത്ത വണ്ണം പ്രഖ്യാപിച്ചു: ''അറബിക്ക് അനറബിയേക്കാളോ അനറബിക്ക് അറബിയേക്കാളോ യാതൊരു ശ്രേഷ്ഠതയുമില്ല; ധര്‍മനിഷ്ഠയുടെ പേരിലല്ലാതെ''.(അഹ്മദ്)

ഇത് അറ ബ് ദേശീയതയുടെ നവോത്ഥാനത്തിനുവേണ്ടി ശ്രമിച്ച ഒരു വ്യക്തിയുടെ വാക്കുകളാകുമോ?

അഞ്ച്: സത്യവിശ്വാസികള്‍ക്ക് മാതൃകയായി ഖുര്‍ആനില്‍ പരാമര്‍ശിക്ക പ്പെട്ടിട്ടുള്ളത് രണ്ടു വനിതകളാണ്. ഒന്ന്, ഫറോവയുടെ പത്‌നി. രണ്ട്, യേശുവിന്റെ മാതാവ് (66:11,12). രണ്ടു പേരും അറബികളല്ല. അറബ് ദേശീയതക്കുവേണ്ടി ഗ്രന്ഥമെഴുതിയ വ്യക്തി ലോകത്തിന് മാതൃകയായി എടുത്തുകാണിക്കുന്നത് അറബികളുടെ എതിരാളികളെയാകുമോ? മര്‍യമിനെ ക്കുറിച്ച് ഖുര്‍ആന്‍ പറയുന്നതിങ്ങനെയാണ്: ''മലക്കുകള്‍ ഇപ്രകാരം പറ ഞ്ഞ സന്ദര്‍ഭം: മര്‍യമേ, തീര്‍ച്ചയായും അല്ലാഹു നിന്നെ പ്രത്യേകം തെര ഞ്ഞെടുക്കുകയും നിനക്ക് പരിശുദ്ധി നല്‍കുകയും ലോകത്തുള്ള സ്ത്രീകളില്‍ വെച്ച് ഉല്‍കൃഷ്ടയായി നിന്നെ തെരഞ്ഞെടുക്കുകയും ചെയ്തിരിക്കുന്നു''(3:42).

ബൈബിളിലൊരിടത്തും ഇത്ര ബഹുമാനത്തോടുകൂടി മര്‍യമിനെക്കുറിച്ച് പരാമര്‍ശിക്കപ്പെട്ടിട്ടില്ല എന്നുകൂടി ഓര്‍ക്കുക. ലോക വനിതകളില്‍ ഉല്‍കൃഷ്ടയായി ഖുര്‍ആന്‍ എടുത്തുകാണിക്കുന്നത് മുഹമ്മദി(ല)ന്റെ മാതാവിനെയോ ഭാര്യയെയോ മറ്റേതെങ്കിലും അറബ് സ്ത്രീയെയോ അല്ല; ഇസ്രായേല്‍ വനിതയായ മര്‍യമിനെയാണ്. അറബ് ദേശീയതയുടെ വക്താവില്‍നിന്ന് ഇത്തരമൊരു പരാമര്‍ശം പ്രതീക്ഷിക്കുവാന്‍ പറ്റുമോ?

ആറ്: അറബ് ദേശീയതയുടെ നവോത്ഥാനത്തിനുവേണ്ടി പണിയെടുക്കുന്ന ഒരു വ്യക്തി അറബികളുടെ അഹംബോധത്തെ ഉദ്ദീപിക്കുവാനായിരിക്കും തന്റെ രചനയില്‍ ശ്രമിക്കുക. അറബികളുടെ ശ്രേഷ്ഠതയെക്കുറിച്ചുമാത്രമായിരിക്കും അയാള്‍ സംസാരിക്കുക. എന്നാല്‍ ഖുര്‍ആന്‍ ഇസ്രായേല്യര്‍ക്ക് നല്‍കിയ ശ്രേഷ്ഠതയെക്കുറിച്ചും സംസാരിക്കുന്നുണ്ട്. ''ഇസ്രായേല്‍ സന്തതികളേ, നിങ്ങള്‍ക്ക് ഞാന്‍ ചെയ്തുതന്നിട്ടുള്ള അനുഗ്രഹവും മറ്റു ജനവിഭാഗങ്ങളേക്കാള്‍ നിങ്ങള്‍ക്ക് ഞാന്‍ ശ്രേഷ്ഠത നല്‍കിയതും നിങ്ങളോര്‍ക്കുക''(2:47).

ധികാരമോഹമെന്നാല്‍ എന്താണ്? രാജ്യത്തിന്റെ അധികാരം കൈ ക്കലാക്കി സുഖസമൃദ്ധമായ ജീവിതം നയിക്കാനുള്ള ആഗ്രഹം. പതിമൂന്ന് വര്‍ഷത്തെ കഷ്ടപ്പാടുകള്‍ക്കും പീഡനങ്ങള്‍ക്കും ശേഷം പലായനം ചെയ്തു മദീനയിലെത്തിയ പ്രവാചകന് അധികാരം ലഭിച്ചുവെന്നത് നേരാണ്. എന്നാല്‍, അദ്ദേഹത്തിന് അധികാരം സുഖലോലുപതയ്ക്കുള്ള മാര്‍ഗമായി രുന്നില്ല. ഭരണാധികാരിയായിരിക്കുമ്പോഴും ഈത്തപ്പനപ്പായയില്‍ അന്തിയുറങ്ങുകയും വസ്ത്രങ്ങള്‍ സ്വയം അലക്കുകയും പാദരക്ഷകള്‍ തുന്നുകയും ആടിനെ കറക്കുകയും ചെയ്യുന്ന മനുഷ്യനെ അധികാരമോഹിയെന്നു വിളിക്കാന്‍ ആര്‍ക്കാണ് സാധിക്കുക? അധികാരത്തിന്റെ പേരില്‍ ജനങ്ങളാല്‍ ആദരിക്കപ്പെടുകയും അവരില്‍ നിന്ന് ഉയര്‍ന്നുനില്‍ക്കുകയും ചെയ്യാന്‍ ആഗ്രഹിക്കുന്നവരാണ് അധികാരം മോഹിക്കുക. പ്രവാചക(സ)നാവട്ടെ ജനങ്ങളെ സേവിച്ച് ജനങ്ങളോടൊപ്പം ജീവിച്ചയാളായിരുന്നു. തന്നെ ബഹുമാനിച്ചുകൊണ്ട് ആളുകള്‍ എഴുന്നേറ്റു നില്‍ക്കുന്നതുപോലും അവിടുന്ന് ഇഷ്ടപ്പെട്ടിരുന്നില്ല.

അദ്ദേഹം ഉപദേശിച്ചു: ''ക്രിസ്ത്യാനികള്‍ മര്‍യമിന്റെ പുത്രനായ യേശുവിനെ പുകഴ്ത്തിയതുപോലെ എന്നെ നിങ്ങള്‍ പുകഴ്ത്തരുത്'' (ബുഖാരി, മുസ്‌ലിം). ഇതെല്ലാംതന്നെ മുഹമ്മദ് (സ) ഒരു അധികാര മോഹിയായിരുന്നില്ലെന്ന് വ്യക്തമാക്കുന്നു.

മാത്രവുമല്ല, തന്റെ പ്രബോധന പ്രവര്‍ത്തനങ്ങള്‍ നിര്‍ത്തിവെക്കുകയാ ണെങ്കില്‍, മക്കയിലെ പ്രയാസപൂര്‍ണമായ ആദ്യനാളുകളില്‍തന്നെ അധികാരം നല്‍കാമെന്ന് അദ്ദേഹം വാഗ്ദാനം ചെയ്യപ്പെട്ടിരുന്നു. സമൂഹത്തിലെ നേതാക്കന്മാരെല്ലാംകൂടി ഒരു ദിവസം മുഹമ്മദി(സ)ന്റെ അടുത്തുചെന്ന് അദ്ദേഹത്തെ വശീകരിക്കാനായി ആവത് ശ്രമിച്ചു നോക്കി. പക്ഷെ, നിരാശ മാത്രമായിരുന്നു ഫലം. മക്കയിലെ പ്രബോധന പ്രവര്‍ത്തനങ്ങളുടെ ആദ്യനാളുകളിലായിരുന്നു ഈ സംഭവം. ഖുര്‍ആന്‍ രചിച്ചുകൊണ്ട് താന്‍ ദൈവദൂതനാണെന്ന് വരുത്തിത്തീര്‍ത്ത് അധികാരം കൈക്കലാക്കുകയായിരുന്നു പ്രവാചക(ല)ന്റെ ലക്ഷ്യമെങ്കില്‍ പ്രയാസങ്ങള്‍ ഏറെയൊന്നും സഹിക്കാതെ അധികാരം തന്റെ കാല്‍ക്കീഴില്‍ വന്ന സമയത്ത് അദ്ദേഹം അത് സ്വീകരിക്കുവാന്‍ വൈമനസ്യം കാണിച്ചതെന്തിനാണ്? മുഹമ്മദ്അ(സ)ധികാരം കാംക്ഷിച്ചിരുന്നില്ലെന്ന് ഇതില്‍നിന്ന് സുതരാം വ്യക്തമാണ്. ഖുര്‍ആന്‍ കൊണ്ടുവന്നതിനു പിന്നില്‍ അധികാരമോഹമായിരുന്നില്ലെന്ന് സാരം.

നാഥനായി വളര്‍ന്ന മുഹമ്മദ്(സ) ചെറുപ്പത്തില്‍ ഒരുപാട് കഷ്ടപ്പാടുകള്‍ അനുഭവിച്ചിരിക്കാം. എന്നാല്‍, തന്റെ 25-ാം വയസ്സില്‍ നാല്‍പതുകാരിയായ കച്ചവടക്കാരി ഖദീജ( (റ)യെ വിവാഹം ചെയ്തതിനുശേഷം അദ്ദേ ഹത്തിന്റെ ജീവിതനിലവാരം സ്വാഭാവികമായും മെച്ചപ്പെട്ടതായി മാറിയിരിക്കണം. അത്യാവശ്യം നല്ല സാമ്പത്തിക ശേഷിയുണ്ടായിരുന്ന ഖദീജ( (റ)യുടെ ഭര്‍ത്താവായിരുന്ന അദ്ദേഹം സാമ്പത്തിക ക്ലേശങ്ങള്‍ അനുഭവിച്ചിരിക്കാനുള്ള സാധ്യത വിരളമാണ്.

ഖദീജയുമായുള്ള മുഹമ്മദി(സ) ന്റെ വിവാഹം നടന്നത് പ്രവാചകത്വം ലഭിക്കുന്നതിന് 15 വര്‍ഷങ്ങള്‍ക്കുമുമ്പാണ്. പതി നഞ്ച് വര്‍ഷം സാമ്പത്തികക്ലേശം കൂടാതെ ജീവിച്ചതിനുശേഷമാണ് താന്‍ പ്രവാചകനാണെന്നും ഖുര്‍ആന്‍ ദൈവവചനമാണെന്നുമുള്ള അവകാശവാദങ്ങളുമായി മുഹമ്മദ്(സ) രംഗപ്രവേശം ചെയ്യുന്നതെന്നര്‍ഥം. ഖുര്‍ആന്‍ ദൈവികമാണെന്ന് വാദിക്കുക വഴി ഭൗതികലാഭമാണ് അദ്ദേഹം ഇച്ഛിച്ചതെങ്കില്‍ ഈ വാദം ഉന്നയിച്ചതിനുശേഷം അദ്ദേഹത്തിന്റെ സാമ്പത്തിക സ്ഥിതി മെച്ചപ്പെട്ടിരിക്കണമല്ലോ.

എന്നാല്‍, എന്തായിരുന്നു സ്ഥിതി?

പ്രവാചകപത്‌നി ആഇശ(റ) പറയുന്നു: ''ഞങ്ങളുടെ വീട്ടില്‍ ഒന്നും പാചകം ചെയ്യാനില്ലാത്തതിനാല്‍ അടുപ്പു പുകയാതെ ഒന്നോ രണ്ടോ മാസ ങ്ങള്‍ കഴിഞ്ഞുപോകാറുണ്ടായിരുന്നു. ഈത്തപ്പഴവും വെള്ളവുമായിരുന്നു ഞങ്ങളുടെ ഉപജീവനം. ചിലപ്പോള്‍ മദീനത്തുകാര്‍ കൊണ്ടുവന്ന ആട്ടി ന്‍പാലും ഈത്തപ്പഴത്തോടു കൂടെയുണ്ടാവും''. (ബുഖാരി, മുസ്‌ലിം)

ആഇശ(റ) ഒരാളോട് പഴയകാര്യങ്ങള്‍ പറഞ്ഞുകൊണ്ടിരിക്കുകയാണ്. മദീനയിലേക്കുള്ള പലായനത്തിനുശേഷം പ്രവാചകനും കുടുംബവും സ ഹിച്ച പ്രയാസങ്ങളാണ് പ്രതിപാദ്യം. ഒരു രാത്രി തപ്പിത്തടഞ്ഞുകൊണ്ട് വീട്ടുജോലികള്‍ ചെയ്തകാര്യം അവര്‍ പറഞ്ഞു. അയാള്‍ ചോദിച്ചു: ''വിള ക്കില്ലായിരുന്നുവോ? അവര്‍ പ്രതിവചിച്ചു: ''വിളക്കു കത്തിക്കാനുള്ള എണ്ണ ഞങ്ങളുടെ പക്കലുണ്ടായിരുന്നുവെങ്കില്‍ വിശപ്പ് മാറ്റാന്‍ അത് കുടിക്കുമാ യിരുന്നു; കത്തിക്കുന്നതിനു പകരം''. (അഹ്മദ്, ത്വബ്‌റാനി)

ഇത് പ്രവാചകന്റെ ആദ്യകാലത്തെ മാത്രം അവസ്ഥയല്ല. മുഹമ്മദ്(സ) ശക്തമായ ഒരു സാമ്രാജ്യത്തിന്റെ ഭരണാധികാരിയായിരുന്നപ്പോഴും അദ്ദേ ഹത്തിന്റെ അവസ്ഥ ഇതില്‍നിന്ന് ഒട്ടും മെച്ചമായിരുന്നില്ല. ഇസ്‌ലാമിക സാമ്രാജ്യത്തിന്റെ അധിപന്റെ അന്തപുരത്തെക്കുറിച്ച് അദ്ദേഹത്തിന്റെ സഹചാരിയായിരുന്ന ഉമര്‍ (റ)തന്നെ പറയട്ടെ: ''പ്രവാചകന്റെ മുറിയില്‍ ഊറക്കിട്ട മൂന്ന് തോല്‍കഷ്ണങ്ങളും ഒരു മൂലയില്‍ അല്‍പം ബാര്‍ലിയുമല്ലാതെ മറ്റൊന്നുംതന്നെ ഞാന്‍ കണ്ടില്ല. ഞാന്‍ കരഞ്ഞുപോയി. പ്രവാചകന്‍ ചോദിച്ചു: 'എന്തിനാണ് താങ്കള്‍ കരയുന്നത്?' ഞാന്‍ പറഞ്ഞു: 'അല്ലാഹുവിന്റെ ദൂതരേ! ഞാനെങ്ങനെ കരയാതിരിക്കും? താങ്കളുടെ ശരീരത്തില്‍ ഈത്തപ്പനയോലകളുടെ പാട് ഞാന്‍ കാണുന്നു. ഈ മുറിയില്‍ എന്തെല്ലാമുണ്ടെന്നും ഞാനറിയുന്നു. അല്ലാഹുവിന്റെ ദൂതരേ! സമൃദ്ധമായ വിഭവങ്ങള്‍ക്കുവേണ്ടി അല്ലാഹുവിനോട് പ്രാര്‍ ഥിച്ചാലും. അവിശ്വാസികളും അല്ലാഹുവില്‍ പങ്കുചേര്‍ക്കുന്നവരുമായ പേര്‍ഷ്യക്കാരുടെയും റോമാക്കാരുടെയും രാജാക്കന്മാര്‍-സീസറും കൈസറുമെല്ലാം-അരുവികള്‍ ഒഴുകുന്ന തോട്ടങ്ങളില്‍ വസിക്കുമ്പോള്‍ അല്ലാഹുവിന്റെ തെരഞ്ഞെടുക്കപ്പെട്ട പ്രവാചകന്‍ ജീവിക്കുന്നത് ദാരുണമായ പട്ടിണിയില്‍!' എന്റെ ഈ സംസാരം കേട്ടപ്പോള്‍ തലയിണയില്‍ വിശ്രമിക്കുകയായിരുന്ന പ്രവാചകന്‍ എഴുന്നേറ്റിരുന്നു. എന്നിട്ടു പറഞ്ഞു: 'ഉമര്‍! താങ്കള്‍ ഈ വിഷയത്തില്‍ ഇനിയും സംശയാലുവാണോ? ഭൗതിക ജീവിതത്തിലെ സുഖസൗകര്യങ്ങളേക്കാള്‍ നല്ലത് മരണാനന്തര ജീവിതത്തിലെ സുഖസൗ കര്യങ്ങളാണ്. അവിശ്വാസികള്‍ അവരുടെ നന്മയുടെ വിഹിതം ഈ ജീവിതത്തില്‍ ആസ്വദിക്കുന്നു. നമ്മുടേതാകട്ടെ, മരണാനന്തര ജീവിതത്തിലേക്കുവേണ്ടി ബാക്കിവെച്ചിരിക്കുകയാണ്'. ഞാന്‍ അദ്ദേഹത്തോട് അപേക്ഷിച്ചു: 'ദൈവദൂതരെ! എനിക്കുവേണ്ടി മാപ്പിനപേക്ഷിച്ചാലും. എനിക്കു തെറ്റിപ്പോയി''.

ഖുര്‍ആന്‍ ഭൗതിക ലാഭങ്ങള്‍ക്കുവേണ്ടി പടച്ചുണ്ടാക്കിയ മുഹമ്മദി(സ) ന്റെ കൃതിയാണെന്ന വാദമാണിവിടെ തകരുന്നത്. ആകെ സ്വത്തായി ബാക്കിയുണ്ടായിരുന്ന ഏഴു ദീനാര്‍ മരണത്തിനുമുമ്പ് ദാനം ചെയ്യുകയും യഹൂദന് തന്റെ പടച്ചട്ട പണയം വെച്ചുകൊണ്ട് മരണപ്പെടുകയും ചെയ്ത മനുഷ്യന്‍ ധനമോഹിയായിരുന്നുവെന്ന് പറയുന്നത് അടിസ്ഥാനരഹിതമാണ്.

ഖുര്‍ആനിന്റെ രചനക്കുപിന്നില്‍ ധനമോഹമായിരുന്നുവെന്ന വാദം അടിസ്ഥാന രഹിതമാണെന്ന് ദി ന്യു കാത്തോലിക് എന്‍സൈക്ലോപീഡിയ പോലും സമ്മതിച്ചിട്ടുണ്ട്. ''മുഹമ്മദി(സ) ന്റെ മതവിപ്ലവത്തിനു പിന്നില്‍ ധനമോഹമായിരുന്നുവെന്ന ഒരു ധാരണ സൃഷ്ടിക്കപ്പെട്ടിട്ടുണ്ട്. വ്യക്തമാ യി അറിയപ്പെടുന്ന വസ്തുതകള്‍ ഈ ധാരണക്കെതിരാണ്'' (The New Catholic Encyclopedia Vol IX, Page 1001).

മുഹമ്മദ് നബി(സ) ജീവിച്ചത് ചരിത്രത്തിന്റെ വെളിച്ചത്തിലാണ്. അദ്ദേ ഹത്തിലൂടെയാണ് ലോകം ഖുര്‍ആന്‍ ശ്രവിച്ചത്. അതുകൊണ്ടുതന്നെ ഖുര്‍ആനിന്റെ ദൈവികത അംഗീകരിക്കാത്തവരെ സംബന്ധിച്ചിടത്തോളം അവര്‍ക്ക് പറയാനുള്ളത് ഇത് മുഹമ്മദി(സ)ന്റെ രചനയാണെന്നാണ്. ഈ വാദം വിശദമായി ചര്‍ച്ച ചെയ്യേണ്ടതാണ്. ചര്‍ച്ചയുടെ ആമുഖമായി നാം മനസ്സിലാക്കേണ്ട ചില വസ്തുതകളുണ്ട്. അവയുടെ അടിത്തറയില്‍നിന്നുകൊണ്ടായിരിക്കണം മുഹമ്മദ് നബി(സ)യില്‍ ഖുര്‍ആനിന്റെ കര്‍തൃത്വം ആരോപിക്കുന്നത്.

ഒന്ന്: നാല്‍പതു വയസ്സുവരെ അറബികള്‍ക്കിടയില്‍ സുസമ്മതനായ വ്യക്തിയായിരുന്നു മുഹമ്മദ്(സ). ഖുര്‍ആന്‍ ദൈവികമാണെന്നും അതിലെ വിധിവിലക്കുകള്‍ അനുസരിക്കേണ്ടതുണ്ടെന്നും പ്രബോധനം ചെയ്തതുകൊണ്ടാണ് അദ്ദേഹം വെറുക്കപ്പെട്ടവനായത്; ബഹിഷ്‌കരിക്കപ്പെട്ടത്; ജനിച്ച് വളര്‍ന്ന നാട്ടില്‍ നിന്ന് പലായനം ചെയ്യേണ്ടിവന്നത്.

രണ്ട്: സത്യസന്ധനായിരുന്നു മുഹമ്മദ്(സ)എന്ന കാര്യത്തില്‍ അദ്ദേഹ ത്തിന്റെ കഠിന ശത്രുക്കള്‍ക്കുപോലും അഭിപ്രായവ്യത്യാസമുണ്ടായിരുന്നി ല്ല. നാല്‍പതു വയസ്സുവരെ സത്യസന്ധനായി ജീവിച്ച അദ്ദേഹം ഒരു ദിവ സം പടച്ചതമ്പുരാന്റെ പേരില്‍ ഒരു പച്ചക്കള്ളം പറഞ്ഞുവെന്നും അത് പ്രചരിപ്പിക്കുന്നതിനുവേണ്ടി സ്വന്തം ജീവന്‍ തൃണവത്ഗണിച്ചുവെന്നും വിശ്വസിക്കുക പ്രയാസമാണ്.

മൂന്ന്: സാഹിത്യകാരന്മാര്‍ക്ക് അറേബ്യയില്‍ ഉന്നതമായ സ്ഥാനം നല്‍ കപ്പെട്ടിരുന്നു. ഖുര്‍ആന്‍ അത്യുന്നതമായ ഒരു സാഹിത്യ സൃഷ്ടിയാണെന്ന കാര്യത്തില്‍ ആര്‍ക്കും ഭിന്നാഭിപ്രായമുണ്ടായിരുന്നില്ല. അത് തന്‍േറതാണ് എന്ന് അവകാശപ്പെട്ടിരുന്നുവെങ്കില്‍ അദ്ദേഹത്തിന് അറബികള്‍ക്കിടയില്‍ ഉന്നതമായ സ്ഥാനമാനങ്ങള്‍ ലഭിക്കുമായിരുന്നു.

നാല്: മുഹമ്മദി(സ)ന്റെ ചില നടപടികളെ വിമര്‍ശിക്കുന്ന വാക്യങ്ങള്‍ ഖുര്‍ആനിലുണ്ട്. അഞ്ച്: മുഹമ്മദി(സ)നെ ശക്തമായി താക്കീത് ചെയ്യുന്ന വചനങ്ങളും ഖുര്‍ആനിലുണ്ട്. ഈ വസ്തുതകള്‍ മുന്നില്‍ വെച്ചുകൊണ്ടാണ് ഖുര്‍ആന്‍ മുഹമ്മദി(സ)ന്റെ സൃഷ്ടിയാണ് എന്ന വാദത്തിലെ ശരിയും തെറ്റും പരിശോധിക്കേ ണ്ടത്.

സാഹിത്യമൂല്യമുള്ള ഒരു സൃഷ്ടി നടത്തി അത് ദൈവത്തിന്റെ പേരില്‍ ആരോപിച്ചതാണെങ്കില്‍ അതിനു പിന്നില്‍ സ്വാര്‍ഥമായ വല്ല ലക്ഷ്യങ്ങളുമുണ്ടാവണമല്ലോ. അതെന്തായിരുന്നുവെന്നാണ് വിമര്‍ശകര്‍ ആദ്യം വ്യക്തമാക്കേണ്ടത്. അതിന്റെ അടിസ്ഥാനത്തിലാണ് ഈ വാദത്തിന്റെ സത്യത പരിശോധിക്കപ്പെടേണ്ടത്.

''ഖുര്‍ആന്‍ വചനങ്ങള്‍ ക്രോഡീകരിച്ചത് ദൈവിക നിര്‍ദേശം അനുസരിച്ചല്ലെന്ന് വിവിധ ഖുര്‍ആനുകള്‍ പരിശോധിച്ചാല്‍ മനസ്സിലാക്കാന്‍ കഴിയുന്നതാണ്. ഖുര്‍ആന്‍ വചനങ്ങള്‍ ചേര്‍ത്തത് മാത്രമല്ല അതിലെ അധ്യായങ്ങള്‍ ചേര്‍ത്തതിലും ക്രമം തെറ്റിയതായി കാണാം. ഖലീഫ ഉസ്മാന്‍ ക്രോഡീകരിച്ച ഖുര്‍ആന്റെ ഘടനയും ഖലീഫ അലിയുടെ ഖുര്‍ ആന്റെ ഘടനയും തമ്മില്‍ അന്തരമുണ്ട്. അലിയുടെ ഖുര്‍ആനില്‍ അല്‍ അലഖ് ഒന്നാമത്തെ അധ്യായമാണെങ്കില്‍ ഉസ്മാന്റെ ഖുര്‍ആനില്‍ 96-ാം അധ്യായമാണ്. 74ല്‍ കാണുന്ന മുദ്ദസ്സിര്‍ രണ്ടാമത്തേതും 50ല്‍ കാണുന്ന ക്വാഫ് മൂന്നാമത്തേതും 73ല്‍ കാണുന്ന മുസമ്മില്‍ നാലാമത്തേതുമാണ്. അതുപോലെ ഉബയ്യിബ്‌നു കഅ്ബിന്റെ ഖുര്‍ആനില്‍ ഉസ്മാന്റെ ഖുര്‍ആനിലെ നാലാം അധ്യായമായ അന്നിസാഅ് മൂന്നാം അധ്യായമാണ്. ഇബ്‌നു മസ്ഊദിന്റെ ഖുര്‍ആനില്‍ ഉസ്മാന്റെ ഖുര്‍ആനിലെ 2-ാം അധ്യായമായ അല്‍ബഖറയാണ് ഒന്നാം അധ്യായം. അതില്‍ രണ്ടാം അധ്യായമായി വരുന്നത് ഉസ്മാന്റെ 4-ാം അധ്യായമായ അന്നിസാഅ് ആണ്. ഇവയില്‍ ആരുടെ ഖുര്‍ആനാണ് യാഥാര്‍ത്ഥ്യം'' എന്ന ഒരു യുക്തിവാദിയുടെ വിമർശനത്തിന് എന്താണ് മറുപടി ?

വിശുദ്ധ ഖുര്‍ആനിന്റെ ക്രോഡീകരണം അല്ലാഹു ഏറ്റെടുത്ത ബാധ്യതയാണെന്ന് ഖുര്‍ആനില്‍തന്നെ (75:17) പറഞ്ഞിട്ടുണ്ട്. മുസ്‌ലിംകളെ സംബന്ധിച്ചിടത്തോളം ദിവ്യവചനത്തെക്കാള്‍ വലിയ പ്ര മാണമൊന്നുമില്ല. 'വിവിധ ഖുര്‍ആനുകള്‍' എന്ന യുക്തിവാദിയുടെ പ്രയോഗംതന്നെ വലിയ തട്ടിപ്പാണ്. ലോകത്ത് ഒരു ഖുര്‍ആനേയുള്ളൂ. നബി (സ)യുടെ അനുചരന്മാരുടെ കാലം മുതല്‍ ഇന്നുവരെ അനിഷേധ്യമായി അംഗീകരിക്കപ്പെട്ടുവരുന്ന യാഥാര്‍ഥ്യമാണ് മുസ്‌ലിംകള്‍ക്ക് വിഭിന്ന ഖുര്‍ആനുകള്‍ ഇല്ലെന്നത്. ഖലീഫ ഉസ്മാന്‍ ഖുര്‍ആന്‍ ക്രോഡീകരിച്ചു എന്ന പരാമര്‍ശം യുക്തിവാദിയുടെ മറ്റൊ രു കള്ളത്തരമാണ്. നബി (സ)യുടെ വിയോഗത്തെ തുടര്‍ന്ന് ഒന്നാം ഖലീഫ അബൂബക്കറിന്റെ നേതൃത്വത്തില്‍ നബി (സ) പഠിപ്പിച്ച ക്രമത്തില്‍ തയ്യാറാക്കിയ ഖുര്‍ആന്‍പ്രതിയുടെ കുറെ തനിപ്പകര്‍പ്പുകളെടുത്ത് ഇസ്‌ലാമിക രാഷ്ട്രത്തിന്റെ പ്രവിശ്യാതലസ്ഥാനങ്ങളിലേക്ക് കൊടുത്തയക്കുകയാണ് മുന്നാം ഖലീഫ ഉസ്മാന്‍ (റ) ചെയ്തത്. അദ്ദേഹം സ്വന്തമായി ഖുര്‍ആന്‍ ക്രോഡീകരിച്ചിട്ടില്ല.

നാലാം ഖലീഫ അലി (റ), ഉബയ്യിബ്‌നുകഅ്ബ്, ഇബ്‌നു മസ്ഊദ് (റ) എന്നിവരാരും ഒന്നാം ഖലീഫയുടെ നേതൃത്വത്തില്‍ സൂക്ഷ്മ പരിശോധന നടത്തി തയാറാക്കിയതില്‍നിന്ന് വ്യത്യസ്തമായ ഖുര്‍ആന്‍ ക്രോഡീകരിക്കുകയോ പ്രചരിപ്പിക്കുകയോ ചെയ്തിട്ടില്ല. നബി (സ)ക്ക് ദിവ്യസന്ദേശങ്ങള്‍ അവതരിക്കാന്‍ തുടങ്ങിയകാലംമുതല്‍തന്നെ സാക്ഷരരായ അനുചരന്മാര്‍ ഖുര്‍ആന്‍ സൂക്തങ്ങള്‍ എഴുതിസൂക്ഷിച്ചിരുന്നു. ഓരോ സന്ദര്‍ഭത്തിലും അവതരിപ്പിക്കപ്പെടുന്ന വചനങ്ങളും അധ്യായങ്ങളും ഒന്നിന് പുറകെ മറ്റൊന്നായി രേഖപ്പെടുത്തുമ്പോള്‍ അതിന്റെക്രമം അവതരണക്രമംതന്നെയായിരിക്കുക സ്വാഭാവികമാകുന്നു. അധ്യായങ്ങളുടെയും വചനങ്ങളുടെയും ക്രമം അല്ലാഹു നിര്‍ദേശിച്ചതനുസരിച്ച് നബി (സ) അറിയിച്ചുകൊടുക്കുന്നതിന് മുമ്പ് ശിഷ്യന്മാര്‍ എഴുതിവെച്ച പകര്‍പ്പുകളെല്ലാം അവ തരണ ക്രമപ്രകാരമുള്ളതായിരുന്നു. ആ കാര്യമാണ് ചില ഗ്രന്ഥ ങ്ങളില്‍ റിപ്പോര്‍ട്ട് ചെയ്തിട്ടുള്ളത്.

ഖുര്‍ആനിന്റെ ആമുഖമായ ഫാതിഹയെ ആദ്യകാലത്ത് ചിലസഹാബികള്‍ ഒരു അധ്യായമെന്നനിലയില്‍ എണ്ണാത്തതുകൊണ്ടാണ് അവര്‍ അല്‍ബഖറയെ ഒന്നാമ ത്തെ അധ്യായമായി ഗണിച്ചത്. എന്നാല്‍ ഫാതിഹ ഖുര്‍ആനില്‍ പെട്ടതല്ലെന്ന് സത്യവിശ്വാസികളാരും ഒരിക്കലും പറഞ്ഞിട്ടില്ല. 96, 74, 73, 4, 3, 2,1 എന്നീ അധ്യായങ്ങളോ അവയിലെ ഏതെങ്കിലും വചനങ്ങളോ ഖുര്‍ആനില്‍ പെട്ടതല്ലെന്ന് യുക്തിവാദി ചൂണ്ടിക്കാണിച്ച സഹാബികള്‍ ഒരിക്കലും അഭിപ്രായപ്പെട്ടിട്ടേയില്ല. എങ്കിലേ ഇവര്‍ക്കെല്ലാം വ്യത്യസ്തമായ ഖുര്‍ആന്‍ ഉണ്ടായിരുന്നുവെന്ന് പറയുന്ന തിന് ന്യായമുള്ളൂ. അല്ലാഹുവും റസൂലും (സ) നിര്‍ദേശിച്ചപ്രകാരം ക്രോഡീകരിക്കുന്നതിന് മുമ്പ് വിവിധ സഹാബികള്‍ എഴുതിവെച്ചിരുന്നത് സംബന്ധിച്ച റിപ്പോര്‍ട്ടുകള്‍ ഖുര്‍ആനിന്റെ അഖണ്ഡതയെ ചോദ്യം ചെയ്യാന്‍ തെളിവാക്കുന്നത് യുക്തിവാദമല്ല; ദുരാരോപണമാകുന്നു.

പ്രപഞ്ചത്തിന്റെ വികാസത്തെക്കുറിച്ച് ഖുര്‍ആനിലുണ്ടെന്ന് പല മുസ്‌ലിം ഗ്രന്ഥകാരന്മാരും എഴുതിയിട്ടുണ്ട്. ഖുര്‍ആന്‍ 51:47 ആണ് അതിന് തെളിവായി കൊടുക്കാറുള്ളത്. എന്നാല്‍ പ്രസ്തുത സൂക്തത്തിലെ 'വ ഇന്നാ ലമൂസ്ഊന്‍' എന്നതിന്റെ അര്‍ഥം 'ദൈവം വിപുലമായ കഴിവുള്ളവനാണ്' എന്നാണെന്ന് ഒരു മുസ്‌ലിം നാമധാരിയായ യുക്തിവാദി എഴുതിയതായി കണ്ടു. വിശദീകരണം പ്രതീക്ഷിക്കുന്നു.
  • 'ഔസ'അ' എന്ന ക്രിയയില്‍നിന്നുള്ള കര്‍തൃനാമമാണ് 'മൂസിഅ്': ഈ ക്രിയ അകര്‍മകമായും സകര്‍മകമായും പ്രയോഗിക്കാറുണ്ട്. അകര്‍മകമാകുമ്പോള്‍ അതിന്റെ അര്‍ത്ഥം ജീവിത സൗകര്യവും ഐശ്വര്യവും ഉള്ളവനായി തീര്‍ന്നു എന്നാണ്. സകര്‍മകമാകുമ്പോള്‍ അതിന്റെ അര്‍ഥം വിശാലമാക്കി എന്നും വികസിപ്പിച്ചു എന്നുമാണ്. ബൈറൂത്തിലെ കാത്തലിക് പ്രസ് പ്രസിദ്ധീകരിച്ച 'അല്‍മുന്‍ജിദ്' എന്ന നിഘണ്ടുവില്‍ 'ദ്വയ്യക്വ' (ഇടുങ്ങിയതാക്കി) എന്നതിന്റെ വിപരീതമാണ് 'ഔസഅ' എന്ന് രേഖപ്പെടുത്തിയിരിക്കുന്നു. ഇതനുസരിച്ച് 'മൂസിഅ്' എന്നതിന്റെ അര്‍ത്ഥം വിശാലമാക്കുന്നവന്‍ അഥവാ വികസിപ്പിക്കുന്നവന്‍ എന്നാകുന്നു. 'മൂസിഊന്‍' എന്നത് പൂജക ബഹുവചനമാകുന്നു.
വിഷയവുമായി ബന്ധപ്പെട്ട വീഡിയോ

ക്ഷത്രങ്ങളും ഗ്രഹങ്ങളുമടങ്ങുന്ന ആകാശഗോളങ്ങളെല്ലാം പൊതുവെ ഗോളാകൃതിയുള്ളവയാണ്. അത് കൊണ്ട് തന്നെ പ്രപഞ്ചത്തിലെ ദൈവാനുഗ്രഹങ്ങളെക്കുറിച്ച പരാമര്ശങ്ങൾക്കിടയിൽ ഭൂമിയുടെ ഗോളാകൃതി വലിയൊരു ചർച്ചാവിഷയമല്ല. ഈ ആകാശഗോളങ്ങളില്‍ നിന്ന് ഭൂമിക്കുള്ള സവിശേഷത അതില്‍ ജീവന്‍ നിലനില്‍ക്കുന്നുവെന്നതാണ്. ജീവന്‍ നിലനില്‍ക്കുവാന്‍ പാകത്തില്‍ ഭൂമിയെ സംവിധാനിച്ചതിനെക്കുറിച്ചാണ് അല്ലാഹു നല്‍കിയ അനുഗ്രഹങ്ങളെക്കുറിച്ചു പറയുമ്പോള്‍ ക്വുര്‍ആന്‍ ഊന്നുന്നത്.

''നിങ്ങള്‍ക്ക് വേണ്ടി ഭൂമിയെ മെത്തയും ആകാശത്തെ മേല്‍പുരയുമാക്കിത്തരികയും ആകാശത്ത് നിന്ന് വെള്ളം ചൊരിഞ്ഞുതന്നിട്ട് അത് മുഖേന നിങ്ങള്‍ക്ക് ഭക്ഷിക്കുവാനുള്ള കായ്കനികള്‍ ഉല്‍പാദിപ്പിച്ചു തരികയും ചെയ്ത (നാഥനെ നിങ്ങൾ ആരാധിക്കുക ). ഇതെല്ലാം അറിയുന്ന നിങ്ങള്‍ അല്ലാഹുവിന് സമന്‍മാരെ ഉണ്ടാക്കുകയും അരുത്.'' (2:22)

പ്രപഞ്ചത്തിലുള്ള എന്തിനെക്കുറിച്ചും നാം പറയുമ്പോള്‍ ആപേക്ഷികമായാണ്, കേവലമായല്ല പരാമര്‍ശിക്കേണ്ടത് എന്നാണ് ആധുനികഭൗതികം പഠിപ്പിക്കുന്നത്. സൗരയൂഥത്തിലെ ഭൂമിക്കുപുറത്തുള്ള ഏതെങ്കിലും ഒരു കേന്ദ്രത്തിന് അപേക്ഷികമായി ഭൂമി ഉരുണ്ടതാണെന്നു പറയാം. നമ്മുടെ ഗ്യാലക്‌സിക്കു പുറത്തുള്ള ഒരു നിരീക്ഷകന് ആപേക്ഷികമായി ഭൂമിയുടെ ആകൃതിയെക്കുറിച്ച് പരാമര്‍ശിക്കുമ്പോള്‍ അതിന്റെ സ്വയംഭ്രമണവും പരിക്രമണവും ഗാലക്തികകേന്ദ്രത്തെ ചുറ്റിയുള്ള സൂര്യനോടൊപ്പമുള്ള ചലനവുമെല്ലാം പരിഗണിക്കേണ്ടിവരും. ഭൂമിക്കുപുറത്തെ നിരീക്ഷകന് ആപേക്ഷികമായി ഭൂമി ഗോളാകൃതിയിലായിരിക്കുന്നതുപോലെ ഭൂമിയില്‍ ജീവിക്കുന്ന നിരീക്ഷകന് ആപേക്ഷികമായി ഭൂമി പരന്നാണുള്ളത്. പരന്ന ഭൂമിയെ പരിഗണിച്ചുകൊണ്ടാണ് ഭൂമിയിലെ നമ്മുടെ പ്രവര്‍ത്തനങ്ങളെല്ലാം നാം ആസൂത്രണം ചെയ്യുന്നത്. പരന്നഭൂമിയെ പരിഗണിച്ചുകൊണ്ട് ക്രിസ്തുവിന് മൂന്നുനൂറ്റാണ്ടുകള്‍ക്ക് മുമ്പ് ഗ്രീക്ക് ഗണിതജ്ഞനായ യൂക്ലിഡ് നിര്‍ധരിച്ചെടുത്ത തത്വങ്ങളില്‍ തന്നെ രണ്ടായിരത്തിലധികം വര്‍ഷങ്ങള്‍ കഴിഞ്ഞിട്ടും ആധുനിക ജ്യാമിതി നിലനില്‍ക്കുന്നതും അതുപ്രകാരം ഭൂമിയിലെ നമ്മുടെ നിര്‍മാണങ്ങളെല്ലാം നാം ആസൂത്രണം ചെയ്യുന്നതും മനുഷ്യര്‍ക്ക് ആപേക്ഷികമായി ഭൂമി പരന്നതായതുകൊണ്ടാണ്. അല്ലാഹു നല്‍കിയ അനുഗ്രഹങ്ങളെക്കുറിച്ചു പറയുമ്പോള്‍ ഭൂമിയെ അല്ലാഹു പരത്തിയതായി ക്വുര്‍ആന്‍ പരാമര്‍ശിക്കുന്നുണ്ട്.

''ഭൂമിയിലേക്ക് (അവര്‍ നോക്കുന്നില്ലേ?) അത് എങ്ങനെ പരത്തപ്പെട്ടിരിക്കുന്നുവെന്ന്'' (88:20)

''ഭൂമിയാകട്ടെ നാം അതിനെ വികസിപ്പിക്കുകയും, അതില്‍ ഉറച്ചുനില്‍ക്കുന്ന പര്‍വ്വതങ്ങള്‍ നാം സ്ഥാപിക്കുകയും കൗതുകമുള്ള എല്ലാ സസ്യവര്‍ഗങ്ങളും നാം അതില്‍ മുളപ്പിക്കുകയും ചെയ്തിരിക്കുന്നു.'' (50:7)

''അതെ, നിങ്ങള്‍ക്ക് വേണ്ടി ഭൂമിയെ ഒരു തൊട്ടിലാക്കുകയും നിങ്ങള്‍ നേരായ മാര്‍ഗം കണ്ടെത്താന്‍ വേണ്ടി നിങ്ങള്‍ക്കവിടെ പാതകളുണ്ടാക്കിത്തരികയും ചെയ്തവന്‍.'' (43:10)

''നിങ്ങള്‍ക്ക് വേണ്ടി ഭൂമിയെ തൊട്ടിലാക്കുകയും, നിങ്ങള്‍ക്ക് അതില്‍ വഴികള്‍ ഏര്‍പെടുത്തിത്തരികയും, ആകാശത്ത് നിന്ന് വെള്ളം ഇറക്കിത്തരികയും ചെയ്തവനത്രെ അവന്‍. അങ്ങനെ അത് (വെള്ളം) മൂലം വ്യത്യസ്ത തരത്തിലുള്ള സസ്യങ്ങളുടെ ജോടികള്‍ നാം (അല്ലാഹു) ഉല്‍പാദിപ്പിക്കുകയും ചെയ്തിരിക്കുന്നു.'' (20:53)

''ഭൂമിയാകട്ടെ നാം അതിനെ ഒരു വിരിപ്പാക്കിയിരിക്കുന്നു. എന്നാല്‍ അത് വിതാനിച്ചവന്‍ എത്ര നല്ലവന്‍!'' (51:48)

''അല്ലാഹു നിങ്ങള്‍ക്കു വേണ്ടി ഭൂമിയെ ഒരു വിരിപ്പാക്കുകയും ചെയ്തിരിക്കുന്നു.'' (71:19)

''ഭൂമിയെ നാം ഒരു വിരിപ്പാക്കിയില്ലേ? പര്‍വ്വതങ്ങളെ ആണികളാക്കുകയും (ചെയ്തില്ലേ?).'' (78:6,7)

മനുഷ്യന് ആപേക്ഷികമായി, അവനും മറ്റുജീവജാലങ്ങള്‍ക്കും ജീവിക്കുവാന്‍ തക്കരൂപത്തില്‍ ഭൂമിയെ വിശാലമാക്കുകയും വികസിപ്പിക്കുകയും വിതാനിക്കുകയും പരത്തുകയും ചെയ്തതിനെക്കുറിച്ചു പറയുമ്പോള്‍ തന്നെ ഭൂമിക്കു പുറത്തെ നിരീക്ഷകന് ആപേക്ഷികമായുള്ള ഭൂമിയുടെ ഗോളാകൃതിയെപ്പറ്റിയും ക്വുര്‍ആന്‍ സൂചന നല്‍കുന്നുണ്ട്.

''ആകാശങ്ങളും ഭൂമിയും അവന്‍ യാഥാര്‍ത്ഥ്യപൂര്‍വ്വം സൃഷ്ടിച്ചിരിക്കുന്നു. രാത്രിയെക്കൊണ്ട് അവന്‍ പകലിന്‍മേല്‍ ചുറ്റിപ്പൊതിയുന്നു. പകലിനെക്കൊണ്ട് അവന്‍ രാത്രിമേലും ചുറ്റിപ്പൊതിയുന്നു. സൂര്യനെയും ചന്ദ്രനെയും അവന്‍ നിയന്ത്രണവിധേയമാക്കുകയും ചെയ്തിരിക്കുന്നു. എല്ലാം നിശ്ചിതമായ പരിധിവരെ സഞ്ചരിക്കുന്നു. അറിയുക: അവനത്രെ പ്രതാപിയും ഏറെ പൊറുക്കുന്നവനും.'' (39:5)

രാത്രിയും പകലും പരസ്പരം ചുറ്റുന്നതിനെക്കുറിച്ച് പരാമര്‍ശിക്കുമ്പോള്‍ 'യുകവ്വിറു' എന്നാണ് ക്വുര്‍ആന്‍ ഇവിടെ പറഞ്ഞിരിക്കുന്നത്. ഒരു ഗോളത്തിന്‍മേല്‍ ചുറ്റുന്നതിനെപ്പറ്റിയാണ് ഇങ്ങനെ പരാമര്‍ശിക്കുക. ഉദയസ്ഥാനങ്ങളെയും അസ്തമയസ്ഥാനങ്ങളെയും കുറിച്ചു പരാമര്‍ശിക്കുമ്പോഴും ഭൂമിയുടെ ഗോളാകൃതിയെക്കുറിച്ച് ക്വുര്‍ആന്‍ സൂചന നല്‍കുന്നുണ്ട്.

'' ഉദയസ്ഥാനങ്ങളുടെ രക്ഷിതാവും ആയിട്ടുള്ളവന്‍''. (37:5)

''രണ്ട് ഉദയസ്ഥാനങ്ങളുടെ രക്ഷിതാവും രണ്ട് അസ്തമന സ്ഥാനങ്ങളുടെ രക്ഷിതാവുമാകുന്നു അവന്‍.'' (55:17)

ഒന്നിലധികം ഉദയസ്ഥാനങ്ങളും അസ്തമയസ്ഥാനങ്ങളുമുണ്ടാവുക ഭൂമി ഗോളമായിരിക്കുമ്പോള്‍ മാത്രമാണല്ലോ?

ആകാശഗോളങ്ങളും ഭൂമിയുമെല്ലാം ഗോളാകൃതിയിലുള്ളവയാണെന്ന് ആദ്യകാല മുസ്‌ലിംകള്‍ മനസ്സിലാക്കിയിരുന്നതായി ശൈഖുല്‍ ഇസ്‌ലാം ഇബ്‌നുതീമിയ തന്റെ ഫതാവയില്‍ പരാമര്‍ശിക്കുന്നുണ്ട് (6/586-587). ഇമാം അഹ്മദിന്റ (റ) ശിഷ്യനായിരുന്ന അബുല്‍ഹുസൈന്‍ അഹ്മദ്ബ്‌നു ജഅ്ഫര്‍ ബ്‌നു മുനാദീ(റ)യും ഇമാം അബൂ മുഹമ്മദ്ബ്‌നു ഹസമും (റ) അബൂ ഫറാജ്ബ്‌നുല്‍ ജൗസി(റ)യുമെല്ലാം ഇക്കാര്യം വ്യക്തമാക്കുന്നുണ്ടെന്ന് അദ്ദേഹം സമര്‍ത്ഥിക്കുന്നു. പ്രവാചകശിഷ്യനായിരുന്ന ഇബ്‌നു അബ്ബാസില്‍ (റ) നിന്നുള്ള ചില പരാമര്‍ശങ്ങളും ഭൂമിയും മറ്റും ഉരുïതാണെന്നാണ് അദ്ദേഹം മനസ്സിലാക്കിയിരുന്നതെന്ന് വ്യക്തമാക്കുന്നുണ്ടെന്നും അദ്ദേഹം പറയുന്നുണ്ട്.

ഭൂമി ഒരു ഗോളമാണെന്ന് ആദ്യകാല മുസ്‌ലിം പണ്ഡിതന്‍മാര്‍ മനസ്സിലാക്കുകയും അതിന്റെ വെളിച്ചത്തില്‍ അവര്‍ ഗോളീയത്രികോണമിതി(Spherical trigonometry)രൂപീകരിക്കുകയും ചെയ്തതായും അതുപയോഗിച്ചാണ് ലോകത്തിലെ വ്യത്യസ്ത കോണുകളില്‍നിന്ന് മക്കയിലെ ഖിബ്‌ലയിലേക്കുള്ള ദിശ നിര്‍ണയിച്ചതെന്നും ചരിത്രകാരനായ ഡേവിഡ് എ. കിംഗ് തന്റെ അസ്‌ട്രോണമി ഇന്‍ദി സര്‍വീസ് ഓഫ് ഇസ്‌ലാം(Astronomy in the Service of Islam) എന്ന ഗ്രന്ഥത്തില്‍ പറയുന്നത്. ഭൂഗോളത്തിന്റെ ചുറ്റളവ് കണ്ടു പിടിക്കാനായി ഒരുപറ്റം മുസ്‌ലിം ഗോളശാസ്ത്രജ്ഞരെയും ഭൂമിശാസ്ത്രജ്ഞരെയും ഖലീഫ മഅ്മൂന്‍ ഉത്തരവാദിത്തപ്പെടുത്തിയതായും സിറിയയിലെ തദ്മൂറും റാഖ്ബയും തമ്മിലുള്ള ദൂരം അളന്ന് അവതമ്മില്‍ ഒരു ഡിഗ്രി അക്ഷാംശവ്യത്യാസമുണ്ടെന്ന് മനസ്സിലാക്കുകയും ഭൂമിയുടെ ചുറ്റളവ് 24000 മൈലുകളാണെന്ന് കണക്കാക്കുകയും ചെയ്തതായും മറ്റൊരുകൂട്ടം മുസ്‌ലിം ശാസ്ത്രജ്ഞരുടെ കണക്കുകള്‍പ്രകാരം ഭൂമിയുടെ ചുറ്റളവ് 40,284 കിലോമീറ്ററാണെന്നും ആധുനികയന്ത്രങ്ങളുപയോഗിച്ച് നാം ഇന്നുകണക്കാക്കുന്ന 40,068 കിലോമീറ്ററുമായി വളരെ അടുത്തുനില്‍ക്കുന്നതാണ് അവരുടെ കണക്കാക്കലെന്നത് അത്ഭുതകരമാണെന്നും ശാസ്ത്രചരിത്രകാരനായ അഡ്വേര്‍ഡ് എസ്.കെന്നഡി തന്റെ മാത്തമാറ്റിക്കല്‍ ജിയോഗ്രഫി എന്ന ഗ്രന്ഥത്തില്‍ (പുറം 185-201) നിരീക്ഷിക്കുന്നുണ്ട്.

 

ഭൂമിയെ പരത്തിയതായുള്ള ക്വുര്‍ആന്‍ പരാമര്‍ശത്തില്‍നിന്ന് അതൊരു ഗോളമല്ലെന്ന് ആദ്യകാല മുസ്‌ലിംകളൊന്നും മനസ്സിലാക്കിയിരുന്നില്ലെന്നുസാരം.

 

മനുഷ്യന് ജീവിക്കുവാൻ തക്ക രൂപത്തിൽ ഭൂമിയിൽ അല്ലാഹു ഏർപ്പെടുത്തിയ സംവിധാനങ്ങളെക്കുറിച്ച് പറയുമ്പോൾ അതിനെ വിരിപ്പും തൊട്ടിലുമാക്കിയാതായും പരത്തിയതായും വിതാനിച്ചതായുമെല്ലാം ക്വുർആൻ പറയുന്നുണ്ട്. ജീവന്‍ നിലനില്‍ക്കുവാനായി പ്രത്യേകം പടക്കപ്പെട്ട ഭൂമിയിലെ സംവിധാനങ്ങളെക്കുറിച്ചാണ് ഈ സൂക്തങ്ങൾ പരാമര്ശിക്കുന്നതെന്ന് അവ മനസ്സിരുത്തി വായിച്ചാൽ ആർക്കും ബോധ്യപ്പെടും. ചലനാത്മകമായ ഈ പ്രപഞ്ചത്തിൽ എത്രയെത്ര ചലനങ്ങള്‍ക്കാണ് ഭൂമി വിധേയമായിക്കൊണ്ടിരിക്കുന്നത്! ഭൂമിയുടെ സ്വയംഭ്രമണം. സുര്യനു ചുറ്റുമുള്ള പരിക്രമണം. ഗ്യാലക്‌സിക്കുചുറ്റും സൂര്യനോടൊപ്പം നടത്തുന്ന ചലനം. പ്രപഞ്ചകേന്ദ്രത്തെ ആസ്പദമാക്കി ഗ്യാലക്‌സി നടത്തുന്ന ഭ്രമണത്തോടൊപ്പമുള്ള ചലനം. പ്രപഞ്ചവികാസത്തിനനുപൂരകമായി നടത്തുന്ന ചലനം. ഇങ്ങനെ ചുരുങ്ങിയത് അഞ്ചുവിധം ചലനങ്ങള്‍ക്കെങ്കിലും ഭൂമി ഒരേസമയം വിധേയമാണ്.

ഇങ്ങനെ ചലനാത്മകമായ ഭൂമിയില്‍ തങ്ങള്‍ നിശ്ചലരും സുരക്ഷിതരുമാണെന്ന ബോധമുള്‍ക്കൊണ്ടുകൊണ്ടു ജീവിക്കുകയാണ് മനുഷ്യനടക്കമുള്ള ജീവജാലങ്ങള്‍. അവരെ സംബന്ധിച്ചിടത്തോളം ഭൂമി ഒരു മെത്ത തന്നെയാണ്. ഊഷരമായ മരുഭൂമിക്കുനടുവില്‍ ഉയര്‍പ്പെട്ട സകലവിധ സംവിധാനങ്ങളോടും കൂടിയ ഒരു ഗൃഹം പോലെയാണ് അവരെ സംബന്ധിച്ചിടത്തോളം കത്തുന്ന പ്രപഞ്ചത്തിന് നടുവിലുള്ള ഭൂമി. അതിശീഘ്രമായ ചലനങ്ങള്‍ തങ്ങളുടെ ജീവിതത്തിനാവശ്യമായ പ്രതിഭാസങ്ങള്‍ സൃഷ്ടിക്കുന്നുവെന്നതിനാല്‍ ജീവികള്‍ക്ക് ഭൂമിയൊരു തൊട്ടിലാണ്. നമുക്ക് ജീവിക്കാനനുകൂലമായ കാലാവസ്ഥയുള്‍ക്കൊള്ളുന്ന മറ്റൊരു ആകാശഗോളവും ഇതേവരെ കണ്ടെ ത്തിയിട്ടില്ല. ജീവജാലങ്ങള്‍ക്കു വസിക്കാന്‍ പറ്റിയ ഒരേയൊരു വീട് തന്നെയാണ് ഭൂമി. അള്‍ട്രാ വയലറ്റ് രശ്മികളേല്‍ക്കാതെ, ഉല്‍ക്കാ നിപാതങ്ങളേല്‍ക്കാതെ, ഓക്‌സിജന്‍ യഥേഷ്ടം ശ്വസിച്ചും വെള്ളം യഥേഷ്ടം കുടിച്ചും ഭക്ഷണപാനീയങ്ങളുയോഗിച്ചും വിശ്രമിക്കാന്‍ കഴിയുന്ന പ്രപഞ്ചത്തിലെ ഏക മെത്തയാണിത്. ഈ സംവിധാനങ്ങള്‍ക്കുപിന്നില്‍ സര്‍വജ്ഞനും സര്‍വശക്തനുമായ ഒരു സംവിധായകന്റെ കരവിരുതുകളല്ലാതെ മറ്റെന്താണ് ചിന്തിക്കുന്നവര്‍ക്ക് കാണാന്‍ കഴിയുന്നത്? ഇതു സൂചിപ്പിക്കുന്ന ക്വുര്‍ആന്‍ സൂക്തങ്ങള്‍ നോക്കുക:

''നിങ്ങള്‍ക്ക് വേണ്ടി ഭൂമിയെ മെത്തയും ആകാശത്തെ മേല്‍പുരയുമാക്കിത്തന്ന (നാഥന്‍)'' (2:22)

''അവനാണ് ഭൂമിയെ വിശാലമാക്കുകയും, അതില്‍ ഉറച്ചുനില്‍ക്കുന്ന പര്‍വ്വതങ്ങളും നദികളും ഉണ്ടാക്കുകയും ചെയ്തവന്‍.'' (13:3)

''ഭൂമിയെ നാം വിശാലമാക്കുകയും അതില്‍ ഉറച്ചുനില്‍ക്കുന്ന പര്‍വ്വതങ്ങള്‍ സ്ഥാപിക്കുകയും, അളവ് നിര്‍ണയിക്കപ്പെട്ട എല്ലാ വസ്തുക്കളും അതില്‍ നാം മുളപ്പിക്കുകയും ചെയ്തിരിക്കുന്നു.'' (15:19)

''നിങ്ങള്‍ക്ക് വേണ്ടി ഭൂമിയെ തൊട്ടിലാക്കുകയും, നിങ്ങള്‍ക്ക് അതില്‍ വഴികള്‍ ഏര്‍പെടുത്തിത്തരികയും, ആകാശത്ത് നിന്ന് വെള്ളം ഇറക്കിത്തരികയും ചെയ്തവനത്രെ അവന്‍. അങ്ങനെ അത് (വെള്ളം) മൂലം വ്യത്യസ്ത തരത്തിലുള്ള സസ്യങ്ങളുടെ ജോടികള്‍ നാം (അല്ലാഹു) ഉല്‍പാദിപ്പിക്കുകയും ചെയ്തിരിക്കുന്നു.'' (20:53)

''അഥവാ, ഭൂമിയെ നിവാസയോഗ്യമാക്കുകയും, അതിനിടയില്‍ നദികളുണ്ടാക്കുകയും, അതിന് ഉറപ്പ് നല്‍കുന്ന പര്‍വ്വതങ്ങള്‍ ഉണ്ടാക്കുകയും, രണ്ടുതരം ജലാശയങ്ങള്‍ക്കിടയില്‍ ഒരു തടസ്സം ഉണ്ടാക്കുകയും ചെയ്തവനോ? (അതോ അവരുടെ ദൈവങ്ങളോ?) അല്ലാഹുവോടൊപ്പം മറ്റു വല്ല ദൈവവുമുണ്ടോ? അല്ല, അവരില്‍ അധികപേരും അറിയുന്നില്ല.'' (27:61)

''അവനാകുന്നു നിങ്ങള്‍ക്ക് വേണ്ടി ഭൂമിയെ വിധേയമാക്കി തന്നവന്‍. അതിനാല്‍ അതിന്റെ ചുമലുകളിലൂടെ നിങ്ങള്‍ നടക്കുകയും അവന്റെ ഉപജീവനത്തില്‍ നിന്ന് ഭക്ഷിക്കുകയും ചെയ്തു കൊള്ളുക. അവങ്കലേക്ക് തന്നെയാണ് ഉയിര്‍ത്തെഴുന്നേല്‍പ്.'' (67:15)

''അല്ലാഹു നിങ്ങള്‍ക്കു വേണ്ടി ഭൂമിയെ ഒരു വിരിപ്പാക്കുകയും ചെയ്തിരിക്കുന്നു. അതിലെ വിസ്താരമുള്ള പാതകളില്‍ നിങ്ങള്‍ പ്രവേശിക്കുവാന്‍ വേണ്ടി.'' (71:19,20)

''ഭൂമിയാകട്ടെ, നാം അതിനെ ഒരു വിരിപ്പാക്കിയിരിക്കുന്നു. എന്നാല്‍ അത് വിതാനിച്ചവന്‍ എത്ര നല്ലവന്‍!'' (51:48)

ല്ല. ആർത്തവരക്തത്തിന് കുഞ്ഞിന്റെ രൂപീകരണത്തിൽ എന്തെങ്കിലും പങ്കുള്ളതായി ഖുർആൻ പഠിപ്പിക്കുന്നില്ല. ആര്‍ത്തവരക്തത്തെക്കുറിച്ച് ക്വുര്‍ആന്‍ പരാമര്‍ശിക്കുന്നത് രണ്ടു തവണയാണ്.

അവ ഇങ്ങനെയാണ്:

''ആര്‍ത്തവത്തെപ്പറ്റി അവര്‍ നിന്നോട് ചോദിക്കുന്നു. പറയുക; അതൊരു മാലിന്യമാകുന്നു. അതിനാല്‍ ആര്‍ത്തവഘട്ടത്തില്‍ നിങ്ങള്‍ സ്ത്രീകളില്‍ നിന്ന് അകന്നു നില്‍ക്കേണ്ടതാണ്. അവര്‍ ശുദ്ധിയാകുന്നത് വരെ അവരെ സമീപിക്കുവാന്‍ പാടില്ല. എന്നാല്‍ അവര്‍ ശുചീകരിച്ചു കഴിഞ്ഞാല്‍ അല്ലാഹു നിങ്ങളോട് കല്‍പിച്ച വിധത്തില്‍ നിങ്ങള്‍ അവരുടെ അടുത്ത് ചെന്നുകൊള്ളുക. തീര്‍ച്ചയായും അല്ലാഹു പശ്ചാതപിക്കുന്നവരെ ഇഷ്ടപ്പെടുന്നു. ശുചിത്വം പാലിക്കുന്നവരെയും ഇഷ്ടപ്പെടുന്നു.''(ക്വുര്‍ആന്‍ 2:222)

''നിങ്ങളുടെ സ്ത്രീകളില്‍ നിന്നും ആര്‍ത്തവത്തെ സംബന്ധിച്ച് നിരാശപ്പെട്ടിട്ടുള്ളവരെ സംബന്ധിച്ചിടത്തോളം നിങ്ങള്‍ അവരുടെ ഇദ്ദയുടെ കാര്യത്തില്‍ സംശയത്തിലാണെങ്കില്‍ അത് മൂന്ന് മാസമാകുന്നു. ആര്‍ത്തവമുണ്ടായിട്ടില്ലാത്തവരുടേതും അങ്ങനെ തന്നെ. ഗര്‍ഭവതികളായ സ്ത്രീകളാകട്ടെ, അവരുടെ അവധി അവര്‍ തങ്ങളുടെ ഗര്‍ഭം പ്രസവിക്കലാകുന്നു. വല്ലവനും അല്ലാഹുവെ സൂക്ഷിക്കുന്ന പക്ഷം അവന്ന് അവന്റെ കാര്യത്തില്‍ അല്ലാഹു എളുപ്പമുണ്ടാക്കികൊടുക്കുന്നതാണ്.''(ക്വുര്‍ആന്‍ 65:4)

ആര്‍ത്തവത്തെക്കുറിച്ച സംശയത്തിന് മറുപടി പറയുമ്പോള്‍ സൂറത്തുല്‍ ബക്വറയിലെ സൂക്തത്തില്‍ അതൊരു മാലിന്യമാണെന്നും അത് പുറപ്പെടുന്ന സന്ദര്‍ഭത്തില്‍ സ്ത്രീകളുമായി ശാരീരികബന്ധം പാടില്ലെന്നും മാത്രമാണ് പറയുന്നതെന്ന കാര്യം ശ്രദ്ധേയമാണ്. ഇവിടെ കുഞ്ഞിന്റെ രൂപീകരണവുമായി അതിന് ഏതെങ്കിലും തരത്തിലുള്ള ബന്ധമുണ്ടെന്ന് സൂചിപ്പിക്കുന്ന യാതൊരു പരാമര്‍ശവുമില്ല. സൂറത്തുത്ത്വലാക്വിലെ വചനമാകട്ടെ, ആര്‍ത്തവവിരാമക്കാരുടെയും ആര്‍ത്തവമുണ്ടായിട്ടില്ലാത്തവരുടെയും ഇദ്ദ കാലത്തെക്കുറിച്ചുള്ളതാണ്. അവിടെയും ഗര്‍ഭധാരണത്തെയോ കുഞ്ഞിന്റെ രൂപീകരണത്തെയോ കുറിക്കുന്ന യാതൊന്നും തന്നെ പറഞ്ഞിട്ടില്ല. ആര്‍ത്തവകാലത്തെയും ആര്‍ത്തവരക്തത്തെയും കുറിച്ച നിരവധി പരാമര്‍ശങ്ങള്‍ ഹദീഥുകളിലുണ്ട്. സ്വഹീഹുല്‍ ബുഖാരിയിലെ ആറാമത്തെ അധ്യായവും സ്വഹീഹു മുസ്്‌ലിമിലെ മൂന്നാം അധ്യായവും 'കിതാബുല്‍ ഹൈദ്വ്' അഥവാ ആര്‍ത്തവത്തെക്കുറിച്ച അധ്യായങ്ങളാണ്. ബുഖാരി 37 ഹദീഥുകളും മുസ്്‌ലിം 158 ഹദീഥുകളും ഈ അധ്യായത്തില്‍ നല്‍കിയിട്ടുണ്ട്. ഇവയില്‍ മിക്കതും കര്‍മശാസ്ത്ര സംബന്ധിയായ വിഷയങ്ങളാണ് കൈകാര്യം ചെയ്യുന്നത്. സുനനുന്നസാഇയിലെ മൂന്നാം അധ്യായമായ 'കിതാബുല്‍ ഹൈദ്വു വല്‍ ഇസ്തിഹാദ്വ', സുനനു അബൂദാവൂദിലെ ഒന്നാം അധ്യായമായ 'കിതാബുത്ത്വഹാറ', ജാമിഉത്തിര്‍മിദിയിലെ ഒന്നാം അധ്യായമായ 'കിതാബുത്ത്വഹാറത്തു അന്‍ റസൂലുല്ലാഹി സ്വല്ലല്ലാഹു അലൈഹിവസല്ലം', സുനനു ഇബ്‌നുമാജയിലെ ഒന്നാം അധ്യായമായ 'കിതാബുത്ത്വഹാറത്തു വസുനനുഹാ', മുവത്വാ മാലിക്കിലെ രണ്ടാം അധ്യായമായ 'കിതാബുത്ത്വഹാറ' എന്നിവയില്‍ ഉദ്ധരിച്ചിരിക്കുന്ന ആര്‍ത്തവ സംബന്ധിയായ ഹദീഥുകളിലും പ്രധാനമായി പരാമര്‍ശിച്ചിരിക്കുന്നത് കര്‍മപരമായ കാര്യങ്ങളെക്കുറിച്ചാണ്. ആര്‍ത്തവരക്തത്തെക്കുറിച്ചുള്ള നൂറിലധികം വരുന്ന ഹദീഥുകള്‍ക്കിടയിലെവിടെയും അതിന് കുഞ്ഞിന്റെ രൂപീകരണത്തില്‍ എന്തെങ്കിലും വിധത്തിലുള്ള പങ്കുണ്ടെന്ന് സൂചിപ്പിക്കുന്ന ഒരു പരാമര്‍ശം പോലുമില്ല.

വിഷയവുമായി ബന്ധപ്പെട്ട വീഡിയോ
 

കകോശജീവിയിൽ നിന്ന് പ്രകൃതിനിർധാരണത്തിലൂടെ മാത്രമായി പരിണമിച്ചാണ് ജീവികളെല്ലാം ഉണ്ടായത് എന്നും ജൈവപ്രപഞ്ചത്തിന്റെ നിലനിൽപിന് പിന്നിൽ പ്രകൃത്യാതീതമായ യാതൊന്നുമില്ലെന്നും പറയുന്നതാണ് പരിണാമവാദമെങ്കിൽ അതിനെ ഖുർആൻ അംഗീകരിക്കുന്നില്ല. ജീവികൾക്കിടയിൽ മാറ്റങ്ങളൊന്നുമുണ്ടായിട്ടില്ലെന്നാണ് ഖുർആൻ പറയുന്നത് എന്നല്ല ഇതിനർത്ഥം. ജീവജാതികൾക്കകത്ത് നിരവധി മാറ്റങ്ങളുണ്ടായിട്ടുണ്ടായിരിക്കാം; പുതിയ ജീവജാതികളുടെ ഉല്പത്തിക്ക് വരെ പ്രസ്തുത മാറ്റങ്ങൾ കാരണമായിരുന്നിരിക്കാം. ഇതൊന്നും തന്നെ ഖുർആൻ നിഷേധിക്കുന്നില്ല. എന്നാൽ അന്ധമായ പ്രകൃതിനിര്ധാരണം വഴി മാത്രമാണ് പ്രസ്തുത മാറ്റങ്ങളുണ്ടാവുന്നതെന്ന വാദത്തെ ക്വുർആൻ നിരാകരിക്കുന്നു.ജീവവര്ഗങ്ങളുടെ സൃഷ്ടിക്കും നിലനിൽപ്പിനും പിന്നിൽ വ്യക്തമായ ആസൂത്രണമുണ്ടെന്നാണ് ക്വുർആനും ഹദീഥുകളും പഠിപ്പിക്കുന്നത്. "എല്ലാ ജന്തുക്കളെയും അല്ലാഹു വെള്ളത്തില്‍ നിന്ന്‌ സൃഷ്ടിച്ചിരിക്കുന്നു. അവരുടെ കൂട്ടത്തില്‍ ഉദരത്തില്‍മേല്‍ ഇഴഞ്ഞ്‌ നടക്കുന്നവരുണ്ട്‌. രണ്ട്‌ കാലില്‍ നടക്കുന്നവരും അവരിലുണ്ട്‌. നാലുകാലില്‍ നടക്കുന്നവരും അവരിലുണ്ട്‌. അല്ലാഹു താന്‍ ഉദ്ദേശിക്കുന്നത്‌ സൃഷ്ടിക്കുന്നു. തീര്‍ച്ചയായും അല്ലാഹു എല്ലാകാര്യത്തിനും കഴിവുള്ളവനാകുന്നു." (ക്വുർആൻ 24 :45)

ജൈവവൈവിധ്യത്തെ വിശദീകരിക്കുവാനുള്ള സര്‍ഗാത്മകവും സുന്ദരവുമായ ഒരു പരിശ്രമമാണ് പരിണാമസിദ്ധാന്തം. ചാൾസ് ഡാർവിൻ 1859 ൽ എഴുതിയ ദി ഒറിജിൻ ഓഫ് സ്പെസിസ് എന്ന ഗ്രന്ഥത്തിലൂടെയാണ് പ്രകൃതി നിർധാരണത്തിലൂടെയുള്ള ജീവജാതികളുടെ പരിണാമമെന്ന സിദ്ധാന്തം ആദ്യമായി മുന്നോട്ടു വെക്കപ്പെടുന്നത്. പിന്നീട് നിയോഡാർവിനിസ്റ്റുകൾ ജനിതകത്തിന്റെ വെളിച്ചത്തിൽ പരിണാമത്തെ വിശദീകരിക്കാൻ ശ്രമിച്ചു. പ്രസ്തുത ശ്രമം ഇന്നും തുടർന്നുകൊണ്ടിരിക്കുന്നു. ജീവികളുടെ വൈവിധ്യത്തെ വിശദീകരിക്കാൻ ശ്രമിക്കുന്ന ഒരു സങ്കല്പം എന്നതിലുപരിയായി ഒരു ശാസ്ത്രസിദ്ധാന്തത്തിന്റെ രീതിയോ രൂപമോ നല്‍കാനാവുന്ന തത്ത്വമല്ല ജീവപരിണാമമെന്ന വസ്തുത ശക്തരായ പരിണാമവാദികള്‍ക്കുപോലും ബോധ്യമുള്ളതാണ്. പരിണാമവാദം ഒരു കേവല സാങ്കല്‍പിക തത്ത്വം മാത്രമാണെന്നും ഊഹങ്ങളല്ലാതെ ഒരു ശാസ്ത്രസിദ്ധാന്തത്തിനുണ്ടാവേണ്ട വസ്തുനിഷ്ഠ തെളിവുകളുടെ പിന്‍ബലം അതിനില്ലെന്നും പറയുന്നത് ഇവ്വിഷയകമായി ഇതഃപര്യന്തം നടന്ന ഗവേഷണങ്ങളെയൊന്നും അവഗണിച്ചുകൊണ്ടല്ല; പ്രത്യുത, പ്രസ്തുത ഗവേഷണങ്ങളൊന്നും തന്നെ ഉദ്ദേശിച്ച  ഫലം നല്‍കിയിട്ടില്ലെന്ന അതിന്റെ വക്താക്കളുടെ തന്നെ വെളിപ്പെടുത്തലുകളുടെ വെളിച്ചത്തിലാണ്. പരിണാമവാദത്തിനെതിരെ ശാസ്ത്രലോകത്തുനിന്ന് ഉന്നയിക്കപ്പെടുന്ന വിമര്‍ശനങ്ങള്‍ താഴെ പറയുന്നവയാണ്:

1. നാം ജീവിക്കുന്നത് ലക്ഷക്കണക്കിന് ജീവിവര്‍ഗങ്ങള്‍ക്കിടയിലാണ്. ഇവയിലൊന്നും തന്നെ മറ്റൊരു ജീവിവര്‍ഗമായി പരിണമിക്കുന്നത് നാം കാണുന്നില്ല. രേഖപ്പെട്ടിടത്തോളമുള്ള ചരിത്രത്തിലെവിടെയും ആരെങ്കിലും അത്തരമൊരു പരിണാമം നിരീക്ഷിച്ചതായി രേഖപ്പെടുത്തിയിട്ടില്ല.

2. ലഘുജീവികളില്‍ നിന്ന് സങ്കീര്‍ണമായവ പരിണമിച്ചുണ്ടായിയെന്ന തന്റെ വാദത്തെ സത്യപ്പെടുത്തുന്ന തെളിവുകള്‍ പുരാവസ്തുപഠനങ്ങള്‍ നല്‍കുമെന്ന ചാള്‍സ് ഡാര്‍വിന്റെ പ്രതീക്ഷയുടെ പൂര്‍ത്തീകരണത്തിനുവേണ്ടി ഒന്നരനൂറ്റാണ്ടുകള്‍ നീണ്ട പരിശ്രമങ്ങളൊന്നും തന്നെ ഫലം കണ്ടിട്ടില്ല. ഭൂമിയില്‍ നിലനില്‍ക്കുന്നതായി കണക്കാക്കപ്പെടുന്ന 87 ലക്ഷം ജീവിവര്‍ഗങ്ങളില്‍ ഏതെങ്കിലും രണ്ട് ജീവി വര്‍ഗങ്ങളെ കൂട്ടിയോജിപ്പിക്കുന്ന മുറിഞ്ഞ കണ്ണിയെ കണ്ടെത്തുവാന്‍ ഫോസില്‍ പഠനങ്ങള്‍ക്ക് കഴിഞ്ഞിട്ടില്ല.

3. ഫോസില്‍ പഠനങ്ങള്‍ വ്യക്തമാക്കുന്നത് വ്യത്യസ്ത ജീവവര്‍ഗങ്ങള്‍ക്കിടയില്‍ പ്രകടമായ വ്യത്യാസമുണ്ടെന്നും പ്രസ്തുത വ്യത്യാസം എന്നെന്നും നിലനിന്നിരുന്നുവെന്നും തന്നെയാണ്. ഡാര്‍വിന്റെ പ്രകൃതിനിര്‍ധാരണതത്ത്വവും നിയോഡാര്‍വിനിസ്റ്റുകളുടെ ഉല്‍പരിവര്‍ത്തന സിദ്ധാന്തവുമുപയോഗിച്ച് ജീവവര്‍ഗങ്ങള്‍ തമ്മില്‍ നിലനില്‍ക്കുന്ന വിടവ് വിശദീകരിക്കാനാവാത്തതുകൊണ്ടാണ് ‘വിരാമമിടുന്ന സന്തുലിതാവസ്ഥ’ (Punctuated equilibriium) എന്ന, ഓരോ ജീവിവര്‍ഗവും മറ്റേ ജീവിവര്‍ഗത്തില്‍ നിന്ന് പെട്ടെന്ന് പരിണമിക്കുകയായിരുന്നുവെന്ന സിദ്ധാന്തത്തില്‍ ഇക്കാലത്തെ പരിണാമവാദികള്‍ക്ക് അഭയം തേടേണ്ടിവരുന്നത്. ഓരോ ജീവവര്‍ഗവും പ്രത്യേകമായി സൃഷ്ടിക്കപ്പെടുകയാണുണ്ടായത് എന്നു തന്നെയാണ് ഇത് വ്യക്തമാക്കുന്നത്.

4. ജീവന്റെ ഉല്‍പത്തിയെക്കുറിച്ച് വിശദീകരിക്കുകയെന്ന, ജൈവലോകത്ത് നടന്നുവെന്ന് സങ്കല്‍പിക്കപ്പെട്ട പരിണാമത്തെക്കുറിച്ച് പറയുന്ന ഒരു സിദ്ധാന്തത്തിന്റെ ബാധ്യത നിര്‍വഹിക്കുവാന്‍ പരിണാമവാദത്തിന് ഇതുവരെ കഴിഞ്ഞിട്ടില്ല. ഭൂമിയില്‍ നിലനിന്ന ഏതെങ്കിലുമൊരു സാഹചര്യത്തില്‍ ജീവന്‍ യാദൃച്ഛികമായി ഉണ്ടാകുവാന്‍ യാതൊരു സാധ്യതയുമില്ലെന്ന വസ്തുത പരിണാമവാദികള്‍ തന്നെ സമ്മതിക്കുന്നുണ്ട്.(18) നക്ഷത്രാന്തരപടലത്തിലെവിടെയോ നിലനിന്നിരുന്ന ഒരു പ്രത്യേക സാഹചര്യത്തില്‍ അവിടെ രൂപം കൊള്ളുകയും പിന്നീട് ഭൂമിയോട് അടുത്തു വന്ന ഏതോ ധൂമകേതു വഴി ഭൂമിയിലെത്തുകയും ചെയ്ത പ്രതിഭാസമാണ് ജീവനെന്ന വിശദീകരണത്തില്‍ അഭയം തേടുകയാണ് പരിണാമവാദികള്‍ ഇപ്പോള്‍ ചെയ്യുന്നത്. (Sir Fred Hoyle& Chandra Wickramasinghe: Evolution from Space, New York, 1984)

5. പരിണാമത്തിന് അനുകൂലമായി അവതരിപ്പിക്കപ്പെട്ടിരുന്ന തെളിവുകളൊന്നും തന്നെ ജീവപരിണാമത്തെ സാധൂകരിക്കുന്നില്ലെന്ന വസ്തുത വ്യക്തമാക്കുകയാണ് പിന്നീടുള്ള ഗവേഷണങ്ങള്‍ ചെയതത്. സസ്തനികളുടെ ഭ്രൂണഘട്ടങ്ങളിലെ സാദൃശ്യം കെട്ടിച്ചമയ്ക്കപ്പെട്ടതാണെന്ന് ഈ രംഗത്തെ ആദ്യകാല പരിശ്രമങ്ങളിലൊന്നായ ഏണസ്റ്റ് ഹെയ്ക്കലിന്റെ ഭ്രൂണപരിണാമഘട്ടങ്ങളുടെ താരതമ്യചിത്രത്തെ പഠനവിധേയമാക്കിയ പരിണാമവാദികള്‍ തന്നെ വ്യക്തമാക്കിയിട്ടുള്ളതാണ്.(Michael K. Richardson& Gerhard Keuck: Haeckel’s ABC of evolution and development, ýBiological Reviews, 2002, No: 77, Pages 495–528)

വ്യത്യസ്ത ജീവികളുടെ ജനിതകഘടനയുടെ താരതമ്യം വഴിയുള്ള പരിണാമവൃക്ഷത്തിന്റെ നിര്‍മാണമെന്ന ആശയവും പരിഹരിക്കാനാവാത്ത പ്രശ്‌നങ്ങളാല്‍ വഴിമുട്ടി നില്‍ക്കുകയാണ്. മനുഷ്യനും ആള്‍കുരങ്ങും തമ്മിലുള്ള ജനിതകസാദൃശ്യം 96 ശതമാനത്തോളം വരുമെന്നും അതിനാല്‍ ആള്‍ക്കുരങ്ങില്‍ നിന്നാണ് മനുഷ്യനുണ്ടായതെന്നും വാദിക്കുകയാണെങ്കില്‍ പശുവിനോട് കുതിരയെക്കാള്‍ ബന്ധം ഡോള്‍ഫിനാണെന്നുകൂടി വാദിക്കേണ്ടിവരുമെന്നും ഇതേപോലെയുള്ള കാരണങ്ങളാല്‍ ജനിതകവസ്തുവിന്റെ സാദൃശ്യത്തിന്റെ വെളിച്ചത്തില്‍ മാത്രമായി പരിണാമത്തെ സമര്‍ത്ഥിക്കാനാവില്ലെന്നും വാദിക്കുന്ന വലിയൊരു വിഭാഗം ജീവശാത്രജ്ഞന്മാരുണ്ട്.(Michael Denton:  Evolution: Theory in Crisis, London, 1985. Pages 233-249)

ജനിതകരേഖകള്‍ക്കിടയില്‍ കാണപ്പെടുന്ന, ധര്‍മങ്ങളൊന്നും ഇല്ലാത്തതായി  കരുതിയിരുന്ന ചവറ് ഡി.എന്‍.എകള്‍ (Junk DNA) പരിണാമത്തിനുള്ള തെളിവുകളായി കരുതിയത് തെറ്റാണെന്ന് അവ ചവറുകളല്ലെന്നും അവയ്ക്ക് ധര്‍മങ്ങളുണ്ടെന്നും മനസ്സിലായതോടെ ജീവശാസ്ത്രലോകത്തിന് ബോധ്യപ്പെട്ടു. (Alice Park: Junk DNA — Not So Useless After All, Time Magazine, Sept. 06, 2012) പരിണാമത്തിന് അനുകൂലമായി വ്യാഖ്യാനിക്കപ്പെട്ടിരുന്ന തെളിവുകള്‍ യഥാര്‍ത്ഥ തെളിവുകളല്ലെന്ന് ബോധ്യപ്പെട്ടുകൊണ്ടിരിക്കുന്നത് പരിണാമവാദത്തിന്റെ വിശ്വാസ്യത തകര്‍ക്കുന്നതാണെന്നതില്‍ സംശയമില്ല.

6. ജീവപരിണാമമെന്ന ആശയം പ്രകൃതി നിയമങ്ങള്‍ക്ക് വിരുദ്ധമാണെന്നതിനാല്‍ അശാസ്ത്രീയമാണ്. ‘ഒരു വ്യവസ്ഥയുടെ എന്‍ട്രോപ്പി ഒരിക്കലും കുറയുകയില്ല, അത്തരം വ്യവസ്ഥകള്‍ പരമാവധി എന്‍ട്രോപ്പിയിലെത്തി താപഗതിക സന്തുലിതത്വം പാലിക്കുവാനാണ് സദാ ശ്രമിച്ചുകൊണ്ടിരിക്കുകയെന്ന രണ്ടാം താപഗതിക നിയമത്തിന് എതിരാണ് ജീവപരിണാമം എന്ന ആശയം. ഒറ്റപ്പെട്ട ഒരു വ്യവസ്ഥയില്‍ സ്വാഭാവികമായും സങ്കീര്‍ണവസ്തുകള്‍ വിഘടിച്ച് ലഘുവായിത്തീരുകയാണ് ചെയ്യുകയെന്നാണ് പരമാവധി എന്‍ട്രോപ്പിയിലെത്താനാണ് വ്യവസ്ഥ പരിശ്രമിക്കുകയെന്ന് പറഞ്ഞാല്‍ അതിനര്‍ത്ഥം. ജീവപരിണാമം എന്ന ആശയം തന്നെ ലഘു ജീവവസ്തുകളില്‍ നിന്ന് സങ്കീര്‍ണ ജീവജാലങ്ങളിലേക്കുള്ള സ്വാഭാവികപരിവര്‍ത്തനത്തെയാണല്ലോ കുറിക്കുന്നത്. അങ്ങനെ സംഭവിക്കുവാന്‍ ഒരു ബാഹ്യ ഇടപെടലില്ലാതെ   യാതൊരു സാധ്യതയുമില്ലെന്നാണ് രണ്ടാം താപഗതിക നിയമം അര്‍ഥശങ്കയ്ക്കിടയില്ലാത്തവണ്ണം വ്യക്തമാക്കുന്നത്. സൂക്ഷ്മജീവികളില്‍ നിന്ന് സങ്കീര്‍ണജീവികളുണ്ടാവുകയെന്ന പരിണാമം സംഭവിച്ചുവെന്നതിന് സമൃദ്ധമായ മറ്റു തെളിവുകളുണ്ടെങ്കില്‍ പോലും അങ്ങനെ സംഭവിക്കണമെങ്കില്‍ ഒരു ബാഹ്യശക്തിയുടെ ഇടപെടലുണ്ടായിട്ടുണ്ടെന്ന് സമ്മതിക്കേണ്ടിവരും.

പരിണാമവാദം ശാസ്ത്രീയമായി സ്ഥിരീകരിക്കപ്പെടാത്ത ഒരു സിദ്ധാന്തമായി മാത്രം നില നിൽക്കുന്നിടത്തോളം അതേക്കുറിച്ച് ഇസ്ലാമികനിലപാടെന്ത് എന്ന ചോദ്യം അപ്രസക്തമാണ്. മനുഷ്യൻ പടച്ചവന്റെ ഒരു സവിശേഷസൃഷ്ടിയാണെന്ന് ക്വുർആൻ അസന്നിഗ്ധമായി പ്രഖ്യാപിക്കുന്നു. ജീവികൾക്കിടയിൽ നടന്നേക്കാവുന്ന മാറ്റങ്ങളെ ഇസ്‌ലാം നിഷേധിക്കുന്നില്ല. പക്ഷെ, പ്രസ്തുത മാറ്റങ്ങൾക്കു പിന്നിൽ അന്ധമായ യാദൃച്ഛികതയാണെന്ന വാദം ഇസ്‌ലാം നിരാകരിക്കുന്നു. മാറ്റങ്ങളുണ്ടായിട്ടുണ്ടെങ്കിൽ അവയടക്കം അല്ലാഹുവിന്റെ വിശാലമായ ആസൂത്രണത്തിന്റെ ഭാഗമായി നടന്നതായിരിക്കുമെന്നാണ് ഇസ്‌ലാമിന്റെ വീക്ഷണം.

വിഷയവുമായി ബന്ധപ്പെട്ട വീഡിയോ

ബീജസങ്കലനമെന്ന പദം ഖുർആൻ പ്രയോഗിച്ചിട്ടില്ലെന്നത് ശരിയാണ്. എന്നാൽ, സ്ത്രീ-പുരുഷ ബീജങ്ങളുടെ സംയോജനത്തിൽ നിന്നാണ് കുഞ്ഞുണ്ടാവുന്നതെന്ന വസ്തുതയിലേക്ക് ക്വുർആൻ വെളിച്ചം വീശിയിട്ടുണ്ട്. ഇവ്വിഷയകമായ ഖുർആൻ പരാമർശങ്ങളുടെ കൃത്യതയും സൂക്ഷ്മതയുമറിയണമെങ്കിൽ അക്കാലത്ത് നിലവിലുണ്ടായിരുന്ന സങ്കൽപ്പങ്ങൾ എന്തൊക്കെയായിരുന്നുവെന്ന് മനസ്സിലാക്കണം. പുരുഷ ശുക്ലവും ആര്‍ത്തവരക്തവും ചേര്‍ന്നാണ് കുഞ്ഞുണ്ടാവുന്നതെന്ന് കരുതിയ പിപ്പിലാദ ഋഷി മുതല്‍(1) പാലില്‍നിന്ന് തൈരുണ്ടാവുന്നതുപോലെ ശുക്ലദ്രാവകം ഘനീഭവിച്ചാണ് ശിശുനിര്‍മിതി നടക്കുന്നതെന്ന് വിചാരിച്ച ബൈബിളിലെ  ഇയ്യോബ്(2) പുസ്തകത്തിന്റെ കര്‍ത്താവ് വരെയുള്ളവരുടെ വീക്ഷണങ്ങള്‍ വ്യത്യസ്ത അറ്റങ്ങളിലുള്ളവയായിരുന്നു. പുരുഷന്റെയും സ്ത്രീയുടെയും ശുക്ലങ്ങളിലുള്ള ബീജങ്ങള്‍ കൂടിച്ചേര്‍ന്നാണ് കുഞ്ഞുണ്ടാവുന്നതെന്ന് കരുതിയ ഹിപ്പോക്രാറ്റസ്,(3) മാതൃരക്തത്തെ പുരുഷശുക്ലം ഘനീഭവിപ്പിച്ചാണ് ശിശുവുണ്ടാകുന്നതെന്ന് കരുതിയ അരിസ്റ്റോട്ടില്‍,(4) ശുക്ലത്തെ മാതൃരക്തം പരിപോഷിപ്പിക്കുമ്പോഴാണ് അതിന്റെ നിര്‍മിതി നടക്കുന്നതെന്ന് വിചാരിച്ച ഗാലന്‍(5) എന്നിവരുടെ വീക്ഷണങ്ങള്‍ പാശ്ചാത്യന്‍ വൈജ്ഞാനിക മണ്ഡലത്തില്‍ സജീവമായിരുന്ന കാലത്താണ് ക്വുര്‍ആന്‍ അവതരിക്കുന്നത്.

“നുത്വ്ഫ (ബീജം) യില്‍ നിന്നാണ് കുഞ്ഞുണ്ടാവുന്നതെന്ന് പറഞ്ഞതോടൊപ്പം തന്നെ കൂടിച്ചേര്‍ന്നുണ്ടായ നുത്വ്ഫയാണ് ശിശുനിര്‍മിതിക്ക് നിമിത്തമാവുന്നതെന്നു കൂടി ക്വുര്‍ആന്‍ വ്യക്തമാക്കുന്നുണ്ട്. ക്വുര്‍ആന്‍ പറയുന്നത് നോക്കുക: ”നുത്വ്ഫതുന്‍ അംശാജിൽ (കൂട്ടിച്ചെർന്നുണ്ടായ ബീജം) നിന്ന് തീർച്ചയായും നാം മനുഷ്യനെ സൃഷ്ടിച്ചിരിക്കുന്നു; നമുക്ക് അവനെ പരീക്ഷിക്കുവാന്‍. അങ്ങനെ നാം അവനെ കേള്‍ക്കുന്നവനും കാണുന്നവനുമാക്കിയിരിക്കുന്നു.’‘(6)

മനുഷ്യസൃഷ്ടി നടന്നത് ‘നുത്വ്ഫതുന്‍ അംശാജി’ല്‍ നിന്നാണെന്നാണ് ഈ വചനത്തില്‍ വ്യക്തമാക്കിയിരിക്കുന്നത്. മീം, ശീന്‍, ജീം അക്ഷരത്രയത്തില്‍നിന്ന് നിഷ്പന്നമായ മാശിജിന്റെ ബഹുവചനമാണ് അംശാജ്. കൂട്ടിച്ചേര്‍ക്കുക, ആശയക്കുഴപ്പത്തിലാക്കുക, ഒന്നിനെ മറ്റൊന്നുമായി ഒന്നിച്ചുചേര്‍ക്കുക എന്നീ അര്‍ത്ഥങ്ങളിലാണ് ഈ അക്ഷരത്രയം ഉപയോഗിക്കാറുള്ളത്.(7) ‘നുത്വ്ഫതുന്‍ അംശാജുന്‍’ എന്നാല്‍ കൂടിച്ചേര്‍ന്നുണ്ടായ നുത്വ്ഫയെന്നാണ് അര്‍ത്ഥമെന്ന് ഇത് വ്യക്തമാക്കുന്നു. ക്വുര്‍ആനില്‍ ഈ വചനത്തിലല്ലാതെ മറ്റൊരിടത്തും ഈ പദം പ്രയോഗിച്ചിട്ടില്ല. പുരുഷബീജവും അണ്ഡവും ചേര്‍ന്ന സിക്താണ്ഡത്തെ കുറിക്കാനാണ് ക്വുര്‍ആന്‍ ഇങ്ങനെ പ്രയോഗിച്ചതെന്നാണ് മനസ്സിലാവുന്നത്.

പുരുഷ-സ്ത്രീ സ്രവങ്ങളുടെ സംയോജനത്തില്‍നിന്നാണ് കുഞ്ഞുണ്ടാവുന്നതെന്ന വസ്തുത പ്രവാചകന്‍(സ) വ്യക്തമാക്കിയിട്ടുണ്ട്. ശിശുനിര്‍മിതിയെക്കുറിച്ച ജൂത ചോദ്യത്തിനുള്ള പ്രവാചക മറുപടിയില്‍ ”പുരുഷസ്രവം വെളുത്തതും സ്ത്രീസ്രവം മഞ്ഞയുമാണ്; അവ രണ്ടും കൂടിച്ചേര്‍ന്നാല്‍…” എന്നു കാണാം.(8) പുരുഷന്റെ നുത്വ്ഫയും സ്ത്രീയുടെ നുത്വ്ഫയും കൂടിച്ചേര്‍ന്നുണ്ടാവുന്ന “നുത്വ്ഫയെക്കുറിച്ചാണ് ക്വുര്‍ആനില്‍ ‘നുത്വ്ഫതിന്‍ അംശാജിന്‍’’എന്ന് പറഞ്ഞിരിക്കുന്നതെന്ന് ഇതില്‍നിന്ന് വ്യക്തമാണ്. പ്രത്യുല്‍പാദനത്തെയും കുഞ്ഞിന്റെ ലിംഗനിര്‍ണയം, വിധി എന്നിവയെയുമെല്ലാം കുറിച്ച് പ്രതിപാദിക്കുന്ന ഹദീഥുകളിലും സ്ത്രീ-പുരുഷ സ്രവങ്ങളുടെ സംയോജനത്തെക്കുറിച്ച പരാമര്‍ശങ്ങള്‍ കാണാം. (9)

പ്രവാചകനില്‍നിന്ന് മതം പഠിച്ച സ്വഹാബിമാര്‍ സ്ത്രീ-പുരുഷ സ്രവങ്ങളുടെ സംയോജനമാണ് “നുത്വ്ഫതിന്‍ അംശാജിന്‍’ എന്നതുകൊണ്ട് മനസ്സിലാക്കിയതെന്ന് ക്വുര്‍ആന്‍ വ്യാഖ്യാന ഗ്രന്ഥങ്ങള്‍ വ്യക്തമാക്കുന്നുണ്ട്. ”പുരുഷസ്രവവും സ്ത്രീസ്രവവും; അവ യോജിക്കുമ്പോള്‍”’ എന്നാണ് ഇബ്‌നുഅബ്ബാസ്(റ) ഈ വചനത്തെ വ്യാഖ്യാനിച്ചതെന്ന് ഇമാം ത്വബ്‌രി തന്റെ ജാമിഉല്‍ ബയാന്‍ ഫീ തഫ്‌സീറില്‍ ക്വുര്‍ആനില്‍ സമര്‍ത്ഥിക്കുന്നു.(10) ഇക്‌രിമ(റ)യാകട്ടെ, “”പുരുഷസ്രവവും സ്ത്രീസ്രവവും; അതിലൊന്ന് മറ്റേതുമായി കൂടിച്ചേരുമ്പോള്‍” എന്നാണ് ഈ വചനത്തെ വ്യാഖ്യാനിക്കുന്നത്. റബീഉബ്‌നു അനസ് (റ), ഹസന്‍(റ), മുജാഹിദ്(റ) എന്നിവരും ഇതേപോലെ തന്നെയാണ് ഈ വചനത്തെ വ്യാഖ്യാനിച്ചതെന്ന് ഇമാം ത്വബ്‌രി(റ) വിശദീകരിക്കുന്നുണ്ട്. സ്ത്രീയുടെയും പുരുഷന്റെയും നുത്വ്ഫകളുടെ സംയോജനത്തില്‍നിന്നാണ് കുഞ്ഞുണ്ടാകുന്നതെന്നായിരുന്നു സ്വഹാബിമാരും താബിഉകളുമെല്ലാം മനസ്സിലാക്കിയതെന്ന് ഇമാം റാസി(റ) തന്റെ ക്വുര്‍ആന്‍ വ്യാഖ്യാനഗ്രന്ഥമായ മഫാതീഹുല്‍ ഗൈബില്‍, ഈ വചനത്തെ വ്യാഖ്യാനിച്ചുകൊണ്ട് പരാമര്‍ശിക്കുന്നു.(11)

“നുത്വ്ഫതുന്‍ അംശാജുന്‍ എന്നാല്‍ കൂടിച്ചേര്‍ന്നുണ്ടായ ബീജം എന്നു തന്നെയാണ് അര്‍ത്ഥമെന്ന് മുസ്്‌ലിംകളല്ലാത്ത ക്വുര്‍ആന്‍ വ്യാഖ്യാതാക്കള്‍ പോലും സമ്മതിക്കുന്നതാണ്. നടേ പറഞ്ഞ സൂക്തത്തിന് പതിനെട്ടാം നൂറ്റാണ്ടുകാരനായ ഓറിയന്റലിസ്റ്റ് ക്വുര്‍ആന്‍ പരിഭാഷകന്‍ ജോര്‍ജ് സെയില്‍ നല്‍കുന്ന പരിഭാഷ ”Verily, we have created man of mingled seed of btoh sexes” എന്നാണ്.(12) ഇരുപതാം നൂറ്റാണ്ടുകാരനായ ബ്രിട്ടീഷ് ഓറിയന്റലിസ്റ്റ് എ. ജെ. ആര്‍ബെറി ഈ വചനത്തെ പരിഭാഷപ്പെടുത്തിയിരിക്കുന്നത്  ”we created man of a spermdrop, a mingling” എന്നാണ്.(13) സ്ത്രീയുടെയും പുരുഷന്റെയും ബീജങ്ങളുടെ സങ്കലനത്തില്‍നിന്നാണ് അതുണ്ടാവുന്നതെന്ന് ജോര്‍ജ് സെയില്‍ ഈ വചനത്തില്‍നിന്ന് മനസ്സിലാക്കിയത് ഏതെങ്കിലും ഇസ്്‌ലാമിക പ്രബോധകരുടെ സ്വാധീനം കൊണ്ടല്ല, പ്രത്യുത അറബിഭാഷയിലൂടെ ക്വുര്‍ആന്‍ പഠിച്ചപ്പോള്‍ അദ്ദേഹത്തിന് അങ്ങനെ മനസ്സിലായതാണ്. മുന്‍ധാരണയില്ലാതെ ക്വുര്‍ആനെ സമീപിക്കുന്നവര്‍ക്കെല്ലാം ഈ വചനത്തില്‍നിന്ന് സ്ത്രീ-പുരുഷ ബീജങ്ങളുടെ സംഗമമാണ് കുഞ്ഞുണ്ടാവുന്നതിന് നിമിത്തമാകുകയെന്നാണ് മനസ്സിലാവുകയെന്ന് സെയ്‌ലിന്റെ പരിഭാഷ തെര്യപ്പെടുത്തുന്നുണ്ട്.

പുരുഷബീജവും അണ്ഡവും കൂടിച്ചേരുന്ന ബീജസങ്കലന(fertilization)മെന്ന പ്രക്രിയയെക്കുറിച്ച് കൂടുതല്‍ മനസ്സിലാക്കുമ്പോഴാണ് കൂടിച്ചേര്‍ന്നുണ്ടായ നുത്വ്ഫയെന്ന പ്രയോഗം എത്രമാത്രം കൃത്യമാണെന്ന് ബോധ്യപ്പെടുക. ആര്‍ത്തവചക്രത്തിന്റെ മധ്യത്തില്‍ നടക്കുന്ന അണ്ഡോല്‍സര്‍ജ്ജന(Ovulation)മാണ് പെണ്‍ശരീരത്തില്‍ നടക്കുന്ന ബീജസങ്കലനത്തിലേക്കുള്ള ആദ്യപടി. അണ്ഡോല്‍സര്‍ജ്ജനം കഴിഞ്ഞാല്‍ ഒരു ദിവസത്തിലധികം അണ്ഡം ജീവിച്ചിരിക്കില്ല. അതിനകം ബീജസങ്കലനം നടന്നില്ലെങ്കില്‍ അണ്ഡം നശിച്ചുപോകും. അണ്ഡാശയത്തില്‍നിന്ന് പുറത്തുവന്ന് ഫലോപ്പിയന്‍ നാളിയിലെത്തി ബീജത്തെ പ്രതീക്ഷിച്ചുകൊണ്ട് നില്‍ക്കുന്ന അണ്ഡത്തിനടുത്തെത്തുന്ന ഇരുന്നൂറോളം വരുന്ന പുരുഷബീജങ്ങളില്‍ ഒരെണ്ണത്തിന് മാത്രമാണ് അതിന്റെ ‘ഭിത്തി ഭേദിച്ച് അകത്തുകടക്കാനാവുക. ഒരു തവണ സ്ഖലിക്കുന്ന കോടിക്കണക്കിന് ബീജങ്ങളില്‍നിന്ന് നീന്തി അണ്ഡത്തിനടുത്തെത്തുന്നതില്‍ വിജയിക്കുന്ന ഇരുനൂറോളമെണ്ണത്തില്‍നിന്ന് ഒരേ ഒരെണ്ണത്തിനുമാത്രം! അണ്ഡത്തെ പൊതിഞ്ഞുനില്‍ക്കുന്ന മോളിക്കുളാര്‍ കോശങ്ങളുടെ നിരയായ കൊറോണ റേഡിയാറ്റ(Corona radiata)യിലൂടെ വലിഞ്ഞ് അകത്തുകയറി അണ്ഡഭിത്തിയായ സോണ പെല്ലുസിഡ(zona pellucida)യെ  ഭേദിച്ച് അണ്ഡകോശദ്രവ്യത്തിനകത്തെത്തുവാന്‍ കെല്‍പുള്ള ഒരേയൊരു ബീജത്തിനുമാത്രം ലഭിക്കുന്ന അവസരം! ഇങ്ങനെ ഒരു ബീജാണു അകത്തു കയറിക്കഴിഞ്ഞാല്‍ ഉടന്‍ നടക്കുന്ന കോര്‍ട്ടിക്കല്‍ പ്രതിപ്രവര്‍ത്തനങ്ങള്‍ (cortical  reactions) വഴി പിന്നെയൊരു ബീജവും അണ്ഡത്തിനകത്തേക്ക് കടക്കാത്ത സ്ഥിതി സംജാതമാവുന്നു. അതിനുശേഷമാണ് ബീജകോശകേന്ദ്രത്തിലെ ജനിതക വസ്തുക്കള്‍ അണ്ഡകോശത്തിന്റെ കോശദ്രവ്യത്തില്‍ കലരുകയും അവയും അണ്ഡജനതിക വസ്തുക്കളും തമ്മില്‍ യോജിക്കുകയും ചെയ്യുന്നത്. അണ്ഡകോശത്തിലെ 23 ക്രോമോസോമുകളും ബീജകോശത്തിലെ 23 ക്രോമോസോമുകളും കൂടിച്ചേര്‍ന്ന് 46 ക്രോമോസോമുകളുള്ള ഒരു പൂര്‍ണകോശമായിത്തീരുന്ന പ്രക്രിയയാണ് ബീജസങ്കലനം.

ബീജത്തിന്റെയും അണ്ഡത്തിന്റെയും ന്യൂക്ലിയസ്സുകള്‍ ഒരുമിച്ചുചേര്‍ന്ന് 46 ക്രോമോസോമുകളുള്ള ഒരു പൂര്‍ണ്ണ ന്യൂക്ലിയസ് ആകുന്നതിന് മുമ്പ് അണ്ഡത്തിന്റെ കോശദ്രവ്യത്തിനകത്ത് രണ്ട് പ്രോന്യൂക്ലിയസുകള്‍ (pronuclei) ഉള്ള ഒരു ഘട്ടമുണ്ട്. ആണ്‍ പ്രോന്യൂക്ലിയസും പെണ്‍ പ്രോന്യൂക്ലിയസും അണ്ഡകോശദ്രവ്യത്തിനകത്ത് സ്ഥിതി ചെയ്യുന്ന ഘട്ടം. ഈ സമയത്തെ സംയോജിത കോശത്തിന്റെ പുരുഷ പ്രോന്യൂക്ലിയസ് ഒഴികെയുള്ള ഭാഗങ്ങളെല്ലാം പഴയ അണ്ഡത്തിന്റേതിനു സമാനമായിരിക്കും. ശുക്ലകോശത്തിന്റെ കോശസ്തരം അണ്ഡത്തിന്റെ സ്തരവുമായി ചേര്‍ന്ന് അപ്രത്യക്ഷമാവും. ശുക്ലത്തിന്റെ വാലും കോശദ്രവ്യത്തിലുള്ള മൈറ്റോകോണ്‍ട്രിയയുമെല്ലാം പുരുഷ പ്രോന്യൂക്ലിയസ് ഉണ്ടാകുന്നതോടെ നശിച്ചുപോവും. അതുകൊണ്ടാണ് നമ്മുടെയെല്ലാം -പുരുഷനായാലും സ്ത്രീയായാലും- കോശങ്ങള്‍ക്കകത്തെ മൈറ്റോകോണ്‍ട്രിയ നമുക്ക് മാതാവില്‍നിന്ന് ലഭിച്ചതാണെന്ന് പറയുന്നത്. അഥവാ പുരുഷബീജവും സ്ത്രീബീജവും കൂടിച്ചേര്‍ന്ന് ഒരു മൂന്നാം വസ്തുവുണ്ടാവുകയല്ല, സ്ത്രീ ബീജത്തിനകത്ത് പുരുഷന്റെ ജനിതകവസ്തുവിന്റെ കൂടിച്ചേരല്‍ നടക്കുക മാത്രമാണ് ബീജസങ്കലനത്തില്‍ സംഭവിക്കുന്നത്. അണ്ഡത്തിന്റെ കോശദ്രവ്യവും കോശസ്തരവും മൈറ്റോകോണ്‍ട്രിയയുമെല്ലാം തന്നെയാണ് സിക്താണ്ഡത്തിനുമുണ്ടാവുക. അതിന്റെ ന്യൂക്ലിയസിലേക്ക് പുരുഷബീജത്തിന്റെ ജനിതക വസ്തു കൂടിച്ചേരുക മാത്രമാണ് ബീജസങ്കലനത്തില്‍ നടക്കുന്നത്.(14) രണ്ട് അര്‍ധകോശങ്ങള്‍ ചേര്‍ന്ന് പൂര്‍ണകോശമാകുന്ന പ്രക്രിയയെന്ന, ബീജസങ്കലനത്തിന് സാധാരണയായി പറയാറുള്ള നിര്‍വചനത്തിനുപകരം പൂര്‍ണകോശത്തിന്റെ കോശദ്രവ്യവും അര്‍ധന്യൂക്ലിയസുമുള്ള അണ്ഡത്തിലേക്ക് പുരുഷബീജത്തിനകത്തെ അര്‍ധന്യൂക്ലിയസിലെ ജനിതക വസ്തുവിനെ കടത്തിവിട്ട് അതിനെ പൂര്‍ണകോശമാക്കുന്ന പ്രക്രിയയാണ് ബീജസങ്കലനം എന്നു പറയുന്നതാകും കൃത്യമായ നിര്‍വചനം.

ന്യൂക്ലിയസിനെ മാറ്റിനിര്‍ത്തിയാല്‍ അണ്ഡം ഒരു പൂര്‍ണകോശം തന്നെയാണ്. പൂര്‍ണകോശത്തിന്റേതുപോലെയുള്ള ദ്രവ്യവും സ്തരവും മൈറ്റോകോണ്‍ട്രിയയുമെല്ലാമാണ് അണ്ഡകോശത്തിലുമുള്ളത്. അതിനെ പൂര്‍ണകോശമാക്കിതീര്‍ക്കുന്നതിന് ഒരു അര്‍ധന്യൂക്ലിയസ് കൂടി മാത്രം മതി. പ്രസ്തുത അര്‍ധന്യൂക്ലിയസാണ് പുരുഷബീജം നല്‍കുന്നത്. അങ്ങനെ നോക്കുമ്പോള്‍ അണ്ഡത്തിലേക്ക് അര്‍ധന്യൂക്ലിയസ് കൂട്ടിച്ചേര്‍ക്കുന്ന പ്രക്രിയയാണ് ബീജസങ്കലനമെന്ന് പറയാം. സ്ത്രീ നുത്വ്ഫയിലേക്ക് പുരുഷ ജനിതകവസ്തുവിനെ കൂട്ടിച്ചേര്‍ക്കുന്ന പ്രക്രിയ. ഇങ്ങനെ കൂടിച്ചേര്‍ന്നു കഴിഞ്ഞാലും സ്ത്രീ നുത്വ്ഫ, നുത്വ്ഫ തന്നെയായിരിക്കും. അതിന്റെ കോശദ്രവ്യത്തിനോ സ്തരത്തിനോ ആകൃതിക്കോ മാറ്റങ്ങളൊന്നുമുണ്ടാവുകയില്ല. പുരുഷന്റെ ജനിതക വസ്തു കൂടിച്ചേര്‍ന്ന് അംശാജ് ആയിത്തീര്‍ന്നതായിരിക്കും ആ നുത്വ്ഫയെന്നതുമാത്രമാണ് വ്യത്യാസം. മനുഷ്യനെ സൃഷ്ടിച്ചിരിക്കുന്നത് നുത്വ്ഫതുന്‍ അംശാജില്‍’ നിന്നാണെന്ന ക്വുര്‍ആന്‍ പരാമര്‍ശത്തിന്റെ കൃത്യതയാണ് നമുക്കിവിടെ ബോധ്യപ്പെടുന്നത്. പുരുഷജനിതക വസ്തു കൂട്ടിച്ചേര്‍ത്ത സ്ത്രീ നുത്വ്ഫയില്‍ നിന്നാണല്ലോ നമ്മുടെയെല്ലാം തുടക്കം. പ്രസ്തുത നുത്വ്ഫ വിഭജിക്കപ്പെട്ടാണ് നമ്മുടെ ശരീരവും ഇന്ദ്രിയങ്ങളുമെല്ലാം ഉണ്ടായിട്ടുള്ളത്. ”നുത്വ്ഫത്തിന്‍ അംശാജിന്‍” എന്ന പ്രയോഗത്തില്‍നിന്ന് പുരുഷസ്രവത്തിന്റെയും സ്ത്രീസ്രവത്തിന്റെയും സമ്മേളനം വഴിയാണ് കുഞ്ഞുണ്ടാവുന്നതെന്നാണ് സ്വഹാബിമാര്‍ മനസ്സിലാക്കിയതെന്ന് പറയുമ്പോള്‍ അവരാരും തന്നെ ബീജത്തെയും അണ്ഡത്തെയും കുറിച്ച് പറഞ്ഞിട്ടില്ലല്ലോയെന്ന് തര്‍ക്കിക്കുന്നത് ശുദ്ധ അസംബന്ധമാണ്. നുത്വ്ഫയെന്നാല്‍ പുരുഷ സ്രവത്തിന്റെയോ സ്ത്രീ സ്രവത്തിന്റെയോ ഒരു ഒരു തുള്ളിയോ ചെറിയ അളവോയെന്നാണ് അവര്‍ മനസ്സിലാക്കിയിരുന്നതെന്ന് ഹദീഥുകളില്‍ നിന്ന് നമുക്ക് വ്യക്തമായി. അണ്ഡത്തെ വഹിച്ചുകൊണ്ടുള്ള ഫോളിക്കുളാര്‍ ദ്രാവകത്തിന്റെ ചെറിയൊരു അംശമാണ് അണ്ഡമെന്നും ശുക്ലദ്രാവകത്തിന്റെ ചെറിയൊരു അംശമായ ശുക്ലാണുവാണ് അതുമായി യോജിക്കുന്നതെന്നും ഇന്ന് നമുക്കറിയാം. പരീക്ഷണങ്ങളിലൂടെ ആധുനിക മനുഷ്യര്‍ കണ്ടെത്തിയതെല്ലാം പൗരാണികര്‍ക്ക് അറിയാമായിരുന്നുവെന്ന് ആരും വാദിക്കുന്നില്ല. ദിവ്യവെളിപാടുകളുടെ അടിസ്ഥാനത്തില്‍ അന്തിമപ്രവാചകന്‍ പറഞ്ഞതൊന്നുംതന്നെ ആധുനികശാസ്ത്രം കണ്ടെത്തുന്ന വസ്തുതകള്‍ക്ക് എതിരാവുകയില്ലെന്ന് മാത്രമാണ് മുസ്്‌ലിംകളുടെ വാദം. പുരുഷസ്രവത്തിന്റെയോ സ്ത്രീസ്രവത്തിന്റെയോ ചെറിയൊരു അംശമാണ് നുത്വ്ഫയെന്ന് മനസ്സിലാക്കിയവര്‍ നുത്വ്ഫത്തിന്‍ അംശാജിന്‍ എന്ന പ്രയോഗത്തെ വ്യാഖ്യാനിക്കുമ്പോള്‍ സ്ത്രീസ്രവവും പുരുഷസ്രവവും കൂടിച്ചേര്‍ന്നുണ്ടാവുന്ന നുത്വ്ഫയില്‍നിന്നുള്ള മനുഷ്യസൃഷ്ടിയെ കുറിച്ചാണ് ഇവിടെ പറയുന്നതെന്ന് സ്വാഭാവികമായും പരാമര്‍ശിക്കും. അതില്‍നിന്ന് പുരുഷസ്രവവും സ്ത്രീസ്രവവും പൂര്‍ണമായാണ് ശിശുനിര്‍മിതിയില്‍ പങ്കെടുക്കുന്നതെന്നാണ് അവര്‍ മനസ്സിലാക്കിയതെന്ന് കരുതിക്കൂടാത്തതാണ്. സ്ത്രീസ്രവത്തിന്റെ ഭാഗമായ നുത്വ്ഫയും പുരുഷസ്രവത്തിന്റെ ഭാഗമായ നുത്വ്ഫയും കൂടിച്ചേര്‍ന്ന നുത്വ്ഫത്തിന്‍ അംശാജിനില്‍നിന്നാണ് കുഞ്ഞുണ്ടാവുന്നതെന്നാണ് അവര്‍ കരുതിയിരുന്നതെന്നുതന്നെയാണ് അവരുടെ പരാമര്‍ശങ്ങള്‍ വ്യക്തമാക്കുന്നത്.

പുരുഷബീജത്തെയും അണ്ഡത്തെയും കുറിച്ച് നുത്വ്ഫയെന്ന് പ്രയോഗിച്ച ക്വുര്‍ആന്‍ സിക്താണ്ഡത്തെ (zygote) കുറിക്കാന്‍ “നുത്വ്ഫത്തിന്‍ അംശാജിന്‍’ എന്നാണ് പ്രയോഗിച്ചതെന്ന വസ്തുത ശ്രദ്ധേയമാണ്. സ്ത്രീയുടെ നുത്വ്ഫയിലേക്ക് പുരുഷ ജനിതകവസ്തുവിനെ കൂട്ടിച്ചേര്‍ക്കുന്ന പ്രക്രിയയാണ് ബീജസങ്കലനമെന്ന് നാം മനസ്സിലാക്കി. പ്രസ്തുത പ്രക്രിയ കഴിഞ്ഞ ശേഷമുള്ള ബീജത്തെ കുറിക്കാന്‍ ഏറ്റവും കൃത്യമായ പദം തന്നെയാണ് ക്വുര്‍ആന്‍ ഉപയോഗിച്ചിരിക്കുന്നത്. കൂട്ടിച്ചേര്‍ക്കപ്പെട്ട ബീജം-നുത്വ്ഫത്തിന്‍ അംശാജിന്‍! സര്‍വജ്ഞനായ അല്ലാഹുവിനല്ലാതെ ആര്‍ക്കാണ് ഇത്ര കൃത്യമായി പദങ്ങള്‍ പ്രയോഗിക്കാന്‍ കഴിയുക!

  • കുറിപ്പുകള്‍:
  1. ഗര്‍ഭോപനിഷത്ത്, വചനങ്ങള്‍ 2,3; ഉപനിഷദ്‌സര്‍വസ്വം, തൃശൂര്‍, 2001, പുറം 63-68.
  2. ഇയ്യോബ് 10: 9-11.
  3. Hippocrates: ‘The Seed’, Sections 5-7, Hippocratic Writings, Page 319-320.
  4. Aristotle: On the Generation of Animals, Montana, 2004, page 3-229.
  5. Phillip de Lacy: Corpus Medicorum Graecorum: Galeni de Semine (Galen: On Semen) (Greek text with Englisht yrans), Akademie Verlag, 20-Nov-1992, section I: 9:1-10, page 107-109.
  6. വിശുദ്ധ ഖുര്‍ആന്‍ 76: 2.
  7. ലിസാനുല്‍ അറബ്.
  8. സ്വഹീഹു മുസ്‌ലിം, കിതാബുല്‍ ഹൈദ്വ്.
  9. സ്വഹീഹു മുസ്‌ലിം, കിതാബുണ്‍ ക്വദ്യ്യ.
  10. തഫ്‌സീര്‍ അത്ത്വബ്‌രി 76: 2.
  11. ഇമാം റാസി: ജാമിഉല്‍ ബയാന്‍ ഫീ തഫ്‌സീറില്‍ ഖുര്‍ആന്‍ (http://www.altafsir.com/)
  12. George Sale : The Koran (Al-Qur’an) (http://www.gutenberg.org/).
  13. Arthur John Arberry: The Koran Interpreted, Page 315.
  14. Elaine N. Marieb& Katja Hoehn: Anatomy & Physiology,  London, 2012, Pages 1119- 1121.
വിഷയവുമായി ബന്ധപ്പെട്ട വീഡിയോ
ല്ല. സ്രവിക്കപ്പെടുന്ന ശുക്ലത്തിന്റെ ചെറിയ ഒരു ഭാഗം മാത്രമായ ഒരേയൊരു ബീജം മാത്രമാണ് കുഞ്ഞിന്റെ നിർമ്മിതിയിൽ പങ്കെടുക്കുന്നതെന്നാണ് ക്വുർആനും നബിവചനങ്ങളൂം വ്യക്തമാക്കുന്നത്.
 പുരുഷ ശുക്ലം ഘനീഭവിച്ചാണ് കുഞ്ഞുണ്ടാകുന്നതെന്ന് ധരിച്ചവരും സ്ത്രീശുക്ലമോ ആര്‍ത്തവരക്തമോ കട്ടിയായാണ് ഭ്രൂണമുണ്ടാകുന്നതെന്ന് കരുതിയവരുമായ പൗരാണികരെല്ലാം വിചാരിച്ചത് ഭ്രൂണനിര്‍മാണത്തില്‍ പങ്കെടുക്കുന്നത് സ്രവം പൂര്‍ണമായിട്ടാണെന്നായിരുന്നുവെന്ന് ഗര്‍ഭോപനിഷത്ത് മുതല്‍ ഗാലന്റെ ഗ്രന്ഥങ്ങള്‍ വരെയുള്ളവ പരിശോധിച്ചാല്‍ വ്യക്തമാവും. പുരുഷ ശുക്ലത്തില്‍ ഒളിഞ്ഞിരിക്കുന്ന കുഞ്ഞിന്റെ പ്രാഗ് രൂപം സ്ത്രീശരീരത്തില്‍നിന്ന് പുറത്തുവരാതെ തങ്ങിനില്‍ക്കുന്ന ആര്‍ത്തവരക്തത്തില്‍നിന്ന് പോഷണങ്ങള്‍ സ്വീകരിച്ച് ഗര്‍ഭാശയത്തില്‍വെച്ച് വളരുകയാണ് ചെയ്യുന്നതെന്ന് വാദിച്ച നടേരൂപകരണ സിദ്ധാന്തക്കാരും (Preformationists) ആര്‍ത്തവരക്തം പുരുഷ ശുക്ലത്താല്‍ പ്രചോദിതമാകുമ്പോള്‍ അത് ഘനീഭവിക്കുകയും അതിനുശേഷം സ്ത്രീ ശരീരത്തില്‍നിന്ന് പോഷണങ്ങള്‍ സ്വീകരിച്ച് അവയവങ്ങള്‍ രൂപീകരിക്കപ്പെടുകയുമാണ് ചെയ്യുന്നതെന്ന് വാദിച്ച സ്വയം ഉല്‍പാദന സിദ്ധാന്തക്കാരും (epigenesists) തമ്മില്‍ നടന്ന ആശയ സംഘട്ടനങ്ങളുടെ പശ്ചാത്തലത്തിലാണ് ആധുനിക ഭ്രൂണശാസ്ത്രം ജനിക്കുന്നത്.  സ്ത്രീയുടെ ശുക്ലത്തില്‍നിന്നോ ആര്‍ത്തവരക്തത്തില്‍നിന്നോ ഏതിൽനിന്നാണ് കുഞ്ഞുണ്ടാവുന്നതെന്ന് തർക്കിച്ചവരെല്ലാം പക്ഷെ, പ്രസ്തുത സ്രവങ്ങളില്‍നിന്ന് പൂര്‍ണമായാണ് കുഞ്ഞിന്റെ സൃഷ്ടി  നടക്കുന്നതെന്ന് തന്നെയാണ് വിചാരിച്ചിരുന്നത്. സ്രവമേതാണെങ്കിലും അത് പൂര്‍ണമായി തന്നെയാണ് ഭ്രൂണനിര്‍മിതിയില്‍ പങ്കെടുക്കുന്നതെന്നായിരുന്നു എല്ലാവരും കരുതിയിരുന്നത് എന്നര്‍ത്ഥം. (1)
ഇംഗ്ലീഷ് ജീവശാസ്ത്രജ്ഞനായിരുന്ന റോബര്‍ട്ട് ഹുക്കിന്റെ കോശ നിരീക്ഷണമാണ് ഭ്രൂണ ശാസ്ത്രരംഗത്ത് വഴിത്തിരിവായിത്തീര്‍ന്ന പ്രധാനപ്പെട്ട ഒരു സംഭവം ഓസ്‌കാര്‍ ഹെര്‍ട്്‌വിഗും റിച്ചാര്‍ഡ് ഹെര്‍ട്‌വിഗും കൂടി കടല്‍ച്ചൊരുക്കുകളില്‍ നടക്കുന്ന ബീജസങ്കലനത്തെക്കുറിച്ച് നടത്തിയ വിശദമായ പഠനങ്ങളോടെയാണ് പുരുഷശുക്ലത്തിലെ നിരവധി ബീജങ്ങളിലൊന്ന് മാത്രമാണ് അണ്ഡമായി ചേര്‍ന്ന് കുഞ്ഞുണ്ടാകുന്നതില്‍ പങ്കാളിയാവുന്നതെന്ന് ശാസ്ത്രലോകത്തിന് മനസ്സിലായത്. 1677ല്‍ ആന്റണി വാന്‍ ല്യൂവെന്‍ ഹോക്ക് തന്റെ സൂക്ഷ്മ ദര്‍ശിനിയിലൂടെ ശുക്ലദ്രാവകത്തിനകത്തെ ബീജാണുക്കളെ കണ്ടിരുന്നുവെന്നതിനാല്‍ അദ്ദേഹമാണ് പുരുഷബീജം കണ്ടുപിടിച്ചതെന്നാണ് പൊതുവെ വ്യവഹരിക്കുന്നതെങ്കിലും ശുക്ലദ്രാവകത്തിലെ നിരവധി ബീജാണുക്കളില്‍ ഒരെണ്ണം മാത്രമാണ് ബീജസങ്കലനത്തില്‍ പങ്കെടുക്കുന്നതെന്ന് ശാസ്ത്രലോകം പൂര്‍ണാര്‍ത്ഥത്തില്‍ അംഗീകരിച്ചത് ഇരുപതാം നൂറ്റാണ്ടിന്റെ തുടക്കത്തില്‍ മാത്രമാണ് (2)
സ്ത്രീ ശരീരത്തില്‍ വെച്ചുള്ള കുഞ്ഞിന്റെ നിര്‍മിതിയുടെ പ്രഥമഘട്ടത്തെക്കുറിച്ച് ക്വുര്‍ആന്‍ പറയുന്നത് ‘നിസ്സാരമായ ഒരു ദ്രാവകത്തില്‍നിന്ന്’ എന്നാണ്:
  ”അവന്‍ തന്നെയാണ് വെള്ളത്തില്‍ നിന്ന് മനുഷ്യനെ സൃഷ്ടിക്കുകയും, അവനെ രക്തബന്ധമുള്ളവനും വിവാഹബന്ധമുള്ളവനും ആക്കുകയും ചെയ്തിരിക്കുന്നത്. നിന്റെ രക്ഷിതാവ് കഴിവുള്ളവനാകുന്നു.”(3)
  ”നിസ്സാരമായ ഒരു ദ്രാവകത്തില്‍ നിന്ന് നിങ്ങളെ നാം സൃഷ്ടിച്ചില്ലേ?”(4)
മനുഷ്യനെ ജലത്തില്‍ (മാഅ്) നിന്നാണ് സൃഷ്ടിച്ചതെന്ന സൂറത്തുല്‍ ഫുര്‍ക്വാനിലെ വചനത്തിന്റെ വിശദീകരണമാണ് ‘നിസ്സാരമായ ദ്രാവക'(മാഇന്‍ മഹീന്‍)ത്തില്‍ നിന്നാണ് അത് നിര്‍വഹിച്ചതെന്ന സൂറത്തുല്‍ മുര്‍സലാത്തിലെ വചനം. നിസ്സാരമായ ദ്രാവകമെന്നതുകൊണ്ടുള്ള വിവക്ഷ പുരുഷ ശുക്ലമാണെന്ന് വ്യക്തമാണ്. നിസ്സാരവും വിലയൊന്നുമില്ലാത്തതുമായി പരിഗണിക്കപ്പെടുന്ന ശുക്ല ദ്രാവകത്തെക്കുറിച്ച് ‘മാഇന്‍ മഹീന്‍’ എന്ന് പറഞ്ഞതോടൊപ്പം പ്രസ്തുത ദ്രാവകത്തില്‍നിന്ന് പൂര്‍ണമായല്ല മനുഷ്യ സൃഷ്ടി നടക്കുന്നതെന്ന് ക്വുര്‍ആന്‍ തന്നെ വ്യക്തമാക്കുന്നുണ്ട്.
സൂറത്തുസ്സജദയിലെ എട്ടാം വചനം നോക്കുക:”പിന്നെ അവന്റെ സന്തതിയെ നിസ്സാരമായ ഒരു വെള്ളത്തിന്റെ സത്തില്‍ നിന്ന് അവന്‍ ഉണ്ടാക്കി.”(5)
‘സുലാലത്തിന്‍ മിന്‍ മാഇന്‍ മഹീന്‍’ എന്ന പ്രയോഗത്തെയാണ് ഇവിടെ ‘നിസ്സാരമായ ഒരു ദ്രാവകത്തിന്റെ സത്തില്‍നിന്ന്’ എന്ന് പരിഭാഷപ്പെടുത്തിയിരിക്കുന്നത്. ആള്‍ക്കൂട്ടത്തില്‍നിന്ന് വ്യക്തികള്‍ ചോര്‍ന്നുപോവുകയെന്ന അര്‍ത്ഥത്തിലുള്ളതാണ് ഈ പ്രയോഗം. സീന്‍, ലാം, ലാം അക്ഷരത്രയത്തില്‍നിന്ന് നിഷ്പന്നമായ ഇതിന്റെ ക്രിയാധാതു പൊടിയില്‍നിന്ന് മുടിയെടുക്കുന്നതുപോലെയോ ഉറയില്‍നിന്ന് വാള്‍ എടുക്കുന്നതുപോലെയോ വലിയ ഒന്നില്‍നിന്ന് ചെറിയ ഒന്നിനെ പുറത്തെടുക്കുന്നതിനാണ് പൊതുവെ ഉപയോഗിക്കാറുള്ളതെന്നും എന്തെങ്കിലും ഒന്ന് പിഴിഞ്ഞ് അതിന്റെ സത്തെടുക്കുന്നതിനും ഇത് പ്രയോഗിക്കാറുണ്ടെന്നും താജുൽ ഉറൂസിനെപ്പോലെയുള്ള അറബി ശബ്ദതാരാവലികളും (7) ലെയിനിന്റെ അറബി പദവിജ്ഞാനകോശവും വിശദീകരിക്കുന്നുണ്ട്..(8)
ഒരു സാധനത്തിന്റെ സത്ത് എന്നോ അതിന്റെ ഏറ്റവും നല്ല ഭാഗം എന്നോ സംശുദ്ധമായ അതിന്റെ അംശം എന്നോ എല്ലാം സുലാലത്തിന് അര്‍ത്ഥം പറയാം.. പുരുഷന്‍ സ്ഖലിക്കുന്ന രണ്ട് കോടിയോളം വരുന്ന ബീജങ്ങളില്‍ ലക്ഷണമൊത്ത ഒരെണ്ണം, അഥവാ സ്രവത്തിന്റെ ഏറ്റവും അനുയോജ്യമായ അംശം മാത്രമാണ് അണ്ഡവുമായി സംയോജിച്ച് കുഞ്ഞായി തീരുന്നത്. ശുക്ലദ്രാവകത്തില്‍നിന്നുള്ള ഏറ്റവും നല്ല ഭാഗം മാത്രം! ശുക്ല ദ്രാവകത്തിന്റെ ‘സുലാലത്ത്’ അഥവാ സംശുദ്ധമായ സത്ത് തന്നെയാണ് ബീജസങ്കലനത്തില്‍ പങ്കെടുത്ത് കുഞ്ഞിന്റെ ജനനത്തിന് നിമിത്തമായിത്തീരുന്നത്.
പുരുഷന്‍ സ്രവിക്കുന്ന ശുക്ലദ്രാവകത്തിന് അറബിയില്‍ പറയുക ‘മനിയ്യ്’ എന്നാണ്. ക്വുര്‍ആനില്‍ ഒരേയൊരു തവണ മാത്രമാണ് ഈ നാമരൂപം പ്രയോഗിച്ചിരിക്കുന്നത്. പ്രസ്തുത പ്രയോഗമിങ്ങനെയാണ്:
”അവന്‍ സ്രവിക്കപ്പെടുന്ന ശുക്ലത്തില്‍ നിന്നുള്ള ഒരു കണമായിരുന്നില്ലേ?”(9)
ഇവിടെ‘മനിയ്യി’ല്‍ നിന്നാണ് മനുഷ്യന്‍ സൃഷ്ടിക്കപ്പെട്ടത് എന്നല്ല പറഞ്ഞിരിക്കുന്നതെന്ന വസ്തുത ശ്രദ്ധേയമാണ്. ‘നുത്വ്ഫത്തിന്‍ മിന്‍ മനിയ്യിന്‍ യുമ്‌ന’ യെന്നാണ് സ്ത്രീ ശരീരത്തില്‍വെച്ചുള്ള മനുഷ്യസൃഷ്ടിയുടെ പ്രാഥമിക ഘട്ടത്തെക്കുറിച്ച് അവിടെയുള്ള ക്വുര്‍ആനിക പ്രയോഗം. ‘മനിയ്യിന്‍ യുമ്‌ന’ എന്നാല്‍ സ്രവിക്കപ്പെടുന്ന ശുക്ലമെന്നാണ് അര്‍ത്ഥം. ‘മിന്‍’ എന്ന വിവേചക ഭേദകം ഉപയോഗിക്കാറുള്ളത് ‘ഒന്നില്‍നിന്ന്’ എന്ന അര്‍ത്ഥത്തിലാണ്. ‘ഹുദന്‍ മിന്‍ റബ്ബിഹിം’ എന്ന പ്രയോഗം ക്വുര്‍ആനില്‍ നിരന്തരമായി കാണാം. ‘നിങ്ങളുടെ നാഥനില്‍ നിന്നുള്ള മാര്‍ഗദര്‍ശനം’ എന്നാണ് ഇതിനര്‍ത്ഥം. അതുകൊണ്ടാണ് ‘നുത്വ്ഫത്തിന്‍ മിന്‍ മനിയ്യി’ന് ‘ശുക്ലത്തില്‍നിന്നുള്ള ഒരു കണം’ എന്ന് അര്‍ത്ഥം നല്‍കിയിരിക്കുന്നത്. സ്രവിക്കപ്പെടുന്ന ശുക്ലത്തില്‍നിന്നുള്ള ഒരു നുത്വ്ഫയെന്നാണ് ക്വുര്‍ആന്‍ പറഞ്ഞതെന്ന കാര്യം ശ്രദ്ധിക്കുക. ‘നുത്വ്ഫ’ ഏകവചനമാണ്. നിത്വാഫ്, നുത്വ്ഫ് എന്നിവയാണ് അതിന്റെ ബഹുവചനരൂപങ്ങള്‍. ശുക്ലം നിരവധി നുത്വ്ഫകളുള്ള ദ്രാവകമാണെന്നും അതില്‍നിന്നുള്ള ഒരു നുത്വ്ഫയാണ് ബീജ സങ്കലനത്തില്‍ പങ്കെടുക്കുന്നതെന്നും ‘നുത്വ്ഫത്തിന്‍ മിന്‍ മനിയ്യ്’ എന്ന ക്വുര്‍ആന്‍ പ്രയോഗം വ്യക്തമാക്കുന്നുണ്ട്.
പുരുഷസ്രവത്തില്‍നിന്ന് പൂര്‍ണമായല്ല, അതിന്റെ ചെറിയൊരു ഭാഗത്തുനിന്നാണ് കുഞ്ഞുണ്ടാവുന്നതെന്ന വസ്തുത പ്രവാചകന്‍(സ) കൃത്യമായി പഠിപ്പിച്ചിട്ടുണ്ട്. ഒരു ഹദീഥ് കാണുക:
അബൂസഈദുല്‍ ഖുദ്‌രി(റ) നിവേദനം: നബി(സ)യോട് ചിലർ അസ്‌ലിനെപ്പറ്റി ചോദിച്ചു. . നിങ്ങളാരും അങ്ങനെ ചെയ്യരുത് എന്ന് നബി (സ) പറഞ്ഞില്ല. നബി (സ) പറഞ്ഞു: ‘അല്ലാഹുവല്ലാതെ ആരെയും സൃഷ്ടിക്കുന്നില്ല.....മൊത്തം സ്രവത്തില്‍നിന്നല്ല കുഞ്ഞുണ്ടാവുന്നത്. അല്ലാഹു ഒന്നിനെ സൃഷ്ടിക്കണമെന്ന് ഉദ്ദേശിച്ചാല്‍ അതിനെ ഒന്നും തടയുന്നതല്ല.'(10)
ശുക്ലസ്ഖലനത്തിനുമുമ്പ് യോനിയില്‍നിന്ന് ലിംഗം പിന്‍വലിച്ച് പുറത്തുകളയുന്ന മൈഥുനവിരാമ (coitus Interruptus/അസ്ല്‍)ത്തെക്കുറിച്ച ചോദ്യത്തിന് പ്രവാചകന്‍ (സ) നല്‍കിയ മറുപടിയാണ് ഈ ഹദീഥിലുള്ളത്. ലിംഗം യോനിയില്‍നിന്ന് പിന്‍വലിക്കുന്നതിനിടയില്‍ സംഭവിച്ചേക്കാവുന്ന സ്ഖലനത്തിനിടയില്‍ ശുക്ലദ്രാവകത്തിന്റെ അല്‍പമെങ്കിലും ജനനേന്ദ്രിയത്തില്‍ പതിക്കാനിടയായാല്‍ അത് ബീജസങ്കലനത്തിനും അതുവഴി കുഞ്ഞിന്റെ ജനനത്തിനും കാരണമായേക്കാം എന്ന വസ്തുതയാണ് പ്രവാചകന്‍ (സ) ഇവിടെ പഠിപ്പിക്കുന്നത്. ‘മൊത്തം സ്രവത്തില്‍നിന്നല്ല കുഞ്ഞുണ്ടാവുന്നത്’ ( മാ മിന്‍ കുല്ലില്‍ മാഇ യകൂനുല്‍ വലദു) എന്നാണ് ഇവിടുത്തെ പ്രവാചക പ്രയോഗം. പുരുഷ സ്രവത്തിന്റെ ചെറിയ ഒരു അംശം സ്ത്രീ ജനനേന്ദ്രിയത്തില്‍ പതിച്ചാലും കുഞ്ഞുണ്ടാവുമെന്നു പറഞ്ഞാല്‍ അതിനര്‍ത്ഥം ശിശുനിര്‍മിതിക്ക് നിദാനമായതെന്താണെങ്കിലും അത് ശുക്ലദ്രാവകത്തില്‍ പരന്നുകിടക്കുകയാണെന്നതാണ്. അങ്ങനെ പരന്നുകിടക്കുന്ന വസ്തുക്കളെയാണ് ക്വുര്‍ആന്‍ നുത്വ്ഫയെന്ന് വിളിക്കുന്നത്.
‘നുത്വ്ഫത്തിന്‍ മിന്‍ മനിയ്യിന്‍ യുമ്‌ന’യെന്ന ക്വുര്‍ആനിക പ്രയോഗത്തിനുള്ള വിശദീകരണം ഈ ഹദീഥ് നല്‍കുന്നുണ്ട്. സ്രവിക്കപ്പെടുന്ന മനിയ്യിന്റെ ചെറിയൊരു അംശമെങ്കിലും സ്ത്രീ ജനനേന്ദ്രിയത്തില്‍ പതിച്ചാല്‍ അതില്‍നിന്ന് കുഞ്ഞുണ്ടാവുമെങ്കില്‍ അതിന്നര്‍ത്ഥം പ്രസ്തുത അംശത്തില്‍ കുഞ്ഞിന്റെ നിര്‍മിതിക്കാവശ്യമായ നുത്വ്ഫയുണ്ടെന്നാണ്. സ്രവത്തിന്റെ ചെറിയൊരംശത്തിലും നുത്വ്ഫയുണ്ടാകുമെന്ന് പറഞ്ഞാല്‍ ദ്രാവകാംശങ്ങളിലെല്ലാം നുത്വ്ഫകളുടെ സാന്നിധ്യമുണ്ടാകുമെന്നും ഇത്തരം കുറേ നുത്വ്ഫകള്‍ അടങ്ങിയതാണ് ശുക്ല ദ്രാവകമെന്നുമാണ് സാരം. സ്രവിക്കപ്പെടുന്ന ശുക്ലദ്രാവകത്തില്‍ നിരവധി നുത്വ്ഫകളുണ്ടെന്നും അതില്‍ ഒരു നുത്വ്ഫയാണ് ബീജ സങ്കലനത്തില്‍ പങ്കെടുക്കുന്നതെന്നും നടേ പറഞ്ഞ ക്വുര്‍ആന്‍ വചനങ്ങളും ഹദീഥും കൂട്ടി വായിച്ചാല്‍ കൃത്യമായി ബോധ്യപ്പെടുന്നുണ്ട്.
എന്താണീ നുത്വ്ഫ? നൂന്, ത്വ, ഫ എന്നീ അക്ഷരത്രയങ്ങളില്‍ നിന്നുള്ള ക്രിയാധാതു മൃദുവായി ഒഴുക്കുക, പുറംതള്ളുക, സ്രവിക്കുക, വിയര്‍ക്കുക, ഉറ്റിവീഴുക, നിര്‍ഗളിക്കുക, ഒലിച്ചിറങ്ങുക എന്നീ അര്‍ത്ഥങ്ങളിലാണ് പ്രയോഗിക്കാറുള്ളത്. നുത്വ്ഫയെന്ന ഏകവചനനാമത്തിന് ശുദ്ധജലം, ജലകണം, ചെറിയ മുത്ത് എന്നിങ്ങനെയാണ് സാധാരണ നിഘണ്ടുക്കള്‍ അര്‍ത്ഥം പറഞ്ഞുവരുന്നത്.(11) നെറ്റിയില്‍ നിര്‍ഗളിക്കുന്ന വിയര്‍പ്പുകണങ്ങള്‍ക്ക് ‘നിത്വാഫ്’ എന്ന നുത്വ്ഫയുടെ ബഹുവചനമുപയോഗിക്കും. ചെറിയ അളവ് വെള്ളത്തിനോ വെള്ളം നിറക്കുന്ന പാത്രത്തില്‍ അവശേഷിക്കുന്ന അല്‍പം ജലത്തിനോ നുത്വ്ഫയെന്നു പറയുമെന്ന് പ്രസിദ്ധമായ ‘ലിസാനുല്‍ അറബ്’ അറബി ശബ്ദതാരാവലി വ്യക്തമാക്കുന്നുണ്ട്. (12) ജലപാത്രത്തിലുള്ള ചെറിയ അളവ് വെള്ളത്തെ സൂചിപ്പിച്ചുകൊണ്ട് ‘നുത്വ്ഫ’യെന്ന സ്വഹീഹു മുസ്്‌ലിമിലുള്ള ഒരു ഹദീഥില്‍ പ്രയോഗിച്ചിട്ടുണ്ട്.(13) മൊത്തം ദ്രാവകവുമായി താരതമ്യം ചെയ്യുമ്പോള്‍ ചെറിയൊരു അംശം ദ്രാവകത്തെ ഉദ്ദേശിച്ചുകൊണ്ടാണീ പ്രയോഗമെന്നാണ് ഇതെല്ലാം വ്യക്തമാക്കുന്നത്. പ്രസിദ്ധമായ ലെയിനിന്റെ അറബി- ഇംഗ്ലീഷ് ലെക്‌സിക്കണ്‍ നുത്വ്ഫക്ക് നല്‍കുന്ന അര്‍ത്ഥം Sperma of a man or a woman എന്നാണ്.(14) ലാറ്റിനില്‍ Sperma എന്നാല്‍ വിത്ത് അല്ലെങ്കില്‍ ബീജമെന്നാണ് അര്‍ത്ഥം. സ്ത്രീശരീരത്തില്‍ വെച്ചുള്ള കുഞ്ഞിന്റെ നിര്‍മിതിയുടെ പ്രഥമ ഘട്ടത്തെക്കുറിച്ച് നുത്വ്ഫയെന്ന് ക്വുര്‍ആനില്‍ നിരവധി തവണ പ്രയോഗിച്ചിട്ടുണ്ട്:
”മനുഷ്യനെ അവന്‍ ഒരു നുത്വ്ഫയില്‍ നിന്ന് സൃഷ്ടിച്ചു. എന്നിട്ട് അവനതാ വ്യക്തമായ എതിര്‍പ്പുകാരനായിരിക്കുന്നു.”(15)
”അവന്റെ ചങ്ങാതി അവനുമായി സംവാദം നടത്തിക്കൊണ്ടിരിക്കെ പറഞ്ഞു: മണ്ണില്‍നിന്നും അനന്തരം നുത്വ്ഫയില്‍ നിന്നും നിന്നെ സൃഷ്ടിക്കുകയും, പിന്നീട് നിന്നെ ഒരു പുരുഷനായി സംവിധാനിക്കുകയും ചെയ്തവനില്‍ നീ അവിശ്വസിച്ചിരിക്കുകയാണോ?”(16)
”മനുഷ്യരേ, ഉയിര്‍ത്തെഴുന്നേല്‍പിനെ പറ്റി നിങ്ങള്‍ സംശയത്തിലാണെങ്കില്‍ (ആലോചിച്ച് നോക്കുക:) തീര്‍ച്ചയായും നാമാണ് നിങ്ങളെ മണ്ണില്‍നിന്നും, പിന്നീട് നുത്വ്ഫയില്‍ നിന്നും, പിന്നീട് ഭ്രൂണത്തില്‍നിന്നും, അനന്തരം രൂപം നല്‍കപ്പെട്ടതും രൂപം നല്‍കപ്പെടാത്തതുമായ മാംസപിണ്ഡത്തില്‍നിന്നുംസൃഷ്ടിച്ചത്.”(17)
”അല്ലാഹു നിങ്ങളെ മണ്ണില്‍ നിന്നും പിന്നീട് നുത്വ്ഫയില്‍ നിന്നും സൃഷ്ടിച്ചു. പിന്നെ അവന്‍ നിങ്ങളെ ഇണകളാക്കി......”(18)
”മനുഷ്യന്‍ കണ്ടില്ലേ; അവനെ നാം ഒരു നുത്വ്ഫയില്‍ നിന്നാണ് സൃഷ്ടിച്ചിരിക്കുന്നതെന്ന്? എന്നിട്ട് അവനതാ ഒരു പ്രത്യക്ഷമായ എതിര്‍പ്പുകാരനായിരിക്കുന്നു.”(19)
”മണ്ണില്‍നിന്നും, പിന്നെ നുത്വ്ഫയില്‍നിന്നും, പിന്നെ ഭ്രൂണത്തില്‍ നിന്നും നിങ്ങളെ സൃഷ്ടിച്ചത് അവനാകുന്നു. പിന്നീട് ഒരു ശിശുവായി നിങ്ങളെ അവന്‍ പുറത്തുകൊണ്ടുവരുന്നു.”(20)
”ആണ്‍, പെണ്‍ എന്നീ രണ്ട് ഇണകളെ അവനാണ് സൃഷ്ടിച്ചതെന്നും; ഒരു നുത്വ്ഫ സ്രവിക്കപ്പെടുമ്പോള്‍ അതില്‍ നിന്ന്.”(21)
”ഏതൊരു വസ്തുവില്‍ നിന്നാണ് അല്ലാഹു അവനെ സൃഷ്ടിച്ചത്? ഒരു നുത്വ്ഫയില്‍ നിന്ന് അവനെ സൃഷ്ടിക്കുകയും, എന്നിട്ട് അവനെ വ്യവസ്ഥപ്പെടുത്തുകയും ചെയ്തു.”(22)
പുരുഷസ്രവമായ മനിയ്യിന്റെ ഒരു അംശമായ നുത്വ്ഫയെക്കുറിച്ചാണ് ഈ വചനങ്ങളിലെല്ലാം  പറഞ്ഞിരിക്കുന്നതെന്നാണ് ക്വുര്‍ആനിന്റെ പ്രഥമ സംബോധിതര്‍ മനസ്സിലാക്കിയതെന്ന് വ്യക്തമാക്കുന്ന രേഖകളുണ്ട്. ആദ്യകാല ക്വുര്‍ആന്‍ വ്യാഖ്യാതാക്കളില്‍ പ്രമുഖനായ ഇമാം ത്വബ്‌രി തന്റെ ഹിജ്‌റ 270ല്‍ (ക്രിസ്താബ്ദം 883) പൂര്‍ത്തിയാക്കിയ ക്വുര്‍ആന്‍ വ്യാഖ്യാന ഗ്രന്ഥത്തില്‍ സൂറത്തുല്‍ കിയാമയിലെ 37ാം വചനത്തെ വ്യാഖ്യാനിച്ചുകൊണ്ട് പറയുന്നത്് ‘പുരുഷ അരക്കെട്ടുകളില്‍നിന്നുള്ള ശുക്ലത്തിലെ (മനിയ്യ്) വളരെ കുറഞ്ഞ അളവിലുള്ള ദ്രാവകം’ (മാഉന്‍ ഖലീലുന്‍ ഫീ സ്വുല്‍ബിര്‍റജുലി മിന്‍ മനിയ്യി) എന്നാണ്.(23)
ഇരുപതാം നൂറ്റാണ്ടിലെ ആദ്യപാതിയില്‍ ജീവിച്ച പ്രസിദ്ധ ഓറിയന്റലിസ്റ്റ് ക്വുര്‍ആന്‍ വിവര്‍ത്തകനായ ആര്‍തര്‍ ജോണ്‍ ആര്‍ബെറി നുത്വ്ഫക്ക്  നല്‍കുന്ന പരിഭാഷ sperm drop എന്നാണ്.(24) ജൂത വിവര്‍ത്തകനായ എന്‍. ജെ. ദാവൂദ് ‘നുത്വ്ഫത്തുന്‍ മിന്‍ മനിയ്യിന്‍ യുമ്‌ന’ക്ക് നല്‍കുന്ന പരിഭാഷ മ drop of ejaculated semen എന്നാണ്.(25) ഒരുവിധം എല്ലാ ക്വുര്‍ആന്‍ വ്യാഖ്യാതാക്കളും ഈ വചനത്തിലെ നുത്വ്ഫക്ക് നല്‍കുന്ന അര്‍ത്ഥം ഒരു തുള്ളിയെന്നോ ഒരു കണം എന്നോ ആണ്. സ്രവിക്കപ്പെടുന്ന ശുക്ലദ്രാവകത്തിന്റെ ഒരു തുള്ളിയില്‍ നിന്നോ ഒരു കണത്തില്‍നിന്നോ ആണ് ബീജസങ്കലനവും അങ്ങനെ കുഞ്ഞിന്റെ നിര്‍മിതിയും നടക്കുന്നതെന്നായിരുന്നു ക്വുര്‍ആനില്‍നിന്നും ഹദീഥുകളില്‍നിന്നുമെല്ലാം അവര്‍ മനസ്സിലാക്കിയിരുന്നതെന്നാണ് ഇതെല്ലാം വ്യക്തമാക്കുന്നത്.
പുരുഷ ശരീരത്തില്‍നിന്ന് സ്രവിക്കപ്പെടുന്ന നിസ്സാരമായ ഒരു ദ്രാവകത്തിന്റെ സത്തില്‍നിന്നാണ് മനുഷ്യ സൃഷ്ടി നടക്കുന്നതെന്നും ശുക്ലദ്രാവകത്തിലുള്ള നിരവധി നുത്വ്ഫകളില്‍ ഒരു നുത്വ്ഫയാണ് ഭ്രൂണനിര്‍മ്മാണത്തില്‍ പങ്കെടുക്കുന്നതെന്നും ക്വുര്‍ആന്‍ വചനങ്ങളും ഹദീഥുകളും സൂചിപ്പിക്കുന്നത് പതിനാലു നൂറ്റാണ്ടുകള്‍ക്കുമുമ്പാണെന്ന് നാം ഓര്‍ക്കണം. ശുക്ലത്തില്‍നിന്നോ ആര്‍ത്തവരക്തത്തില്‍നിന്നോ ഏതില്‍ നിന്നാണ് കുഞ്ഞുണ്ടാവുന്നതെന്ന് തത്ത്വജ്ഞാനികള്‍ തര്‍ക്കിച്ചിരുന്ന കാലത്താണീ സൂചന നല്‍കുന്നത്. പുരുഷസ്രവത്തിന്റെ പൂര്‍ണതയെ ദ്യോതിപ്പിക്കുന്ന മനിയ്യ് മുഴുവനായിട്ടാണ് ശിശുനിര്‍മിതിയില്‍ പങ്കാളിയാവുന്നതെന്ന ഒരു സൂചന പോലും ക്വുര്‍ആനിലോ ഹദീഥുകളിലോ കാണാനാവുന്നില്ല.
കുറിപ്പുകള്‍:
1. Stephen Ruffenach: Caspar Friedrich Wolff (1734-1794), The Embryo Project Encyclopedia,  (http://embryo.asu.edu/)
2. Professor Scott Gilbert (Ed.): A Conceptual History of Modern Embryology,  Maryland,1994, 8-21
3. വിശുദ്ധ ക്വുര്‍ആന്‍ 25:54
4. വിശുദ്ധ ക്വുര്‍ആന്‍ 77:20
5. വിശുദ്ധ ക്വുര്‍ആന്‍ 32:8
6. വിശുദ്ധ ക്വുര്‍ആന്‍ 24:63
7. താജല്‍ അറൂസ്:  https://archive.org
8. Edward William Lane: Arabic-English Lexicon,London,1863, Volume 4, Page1397
9. വിശുദ്ധ ക്വുര്‍ആന്‍ 75:37
10. സ്വഹീഹുമുസ്‌ലിം
11. താജല്‍ അറൂസ്
12. ലിസാനുല്‍ അറബ്
13. സ്വഹീഹുമുസ്‌ലിം, കിതാബുലുക്തത്
14. Lane’s Arabic-English Lexicon, Page 2810
15. വിശുദ്ധ ക്വുര്‍ആന്‍ 16: 4.
16. വിശുദ്ധ ക്വുര്‍ആന്‍ 18: 37.
17. വിശുദ്ധ ക്വുര്‍ആന്‍ 22: 5.
18. വിശുദ്ധ ക്വുര്‍ആന്‍ 35: 11.
19. വിശുദ്ധ ക്വുര്‍ആന്‍ 36: 77.
20. വിശുദ്ധ ക്വുര്‍ആന്‍ 40: 67.
21. വിശുദ്ധ ക്വുര്‍ആന്‍ 53: 45, 46.
22. വിശുദ്ധ ക്വുര്‍ആന്‍ 80:18, 19.
23. ഇമാംറാസി: ജാമിഉല്‍ബയാന്‍ ഫീ തഫ്‌സീറില്‍ ഖുര്‍ആന്‍ (http://www.altafsir. com/)
24. Arthur John Arberry: The Koran Interpreted, Page 352
25. N.J. Dawood:The Koran, Page 380
26. വിശുദ്ധ ക്വുര്‍ആന്‍ 53:3,4
വിഷയവുമായി ബന്ധപ്പെട്ട വീഡിയോ

തൊരു വൈജ്ഞാനിക മേഖലയിലേക്കും ക്വുര്‍ആനും ഹദീഥുകളും നല്‍കുന്ന വെളിച്ചത്തെക്കുറിച്ച് ചര്‍ച്ച ചെയ്യുമ്പോള്‍ നാം പ്രാഥമികമായി മനസ്സിരുത്തേണ്ട വസ്തുത, ശാസ്ത്രത്തെക്കുറിച്ചോ ഭൗതിക വിജ്ഞാനീയങ്ങളെകുറിച്ചോ അറിവു നല്‍കുന്നതിനുവേണ്ടി അവതരിപ്പിക്കപ്പെട്ട വെളിപാടുകളല്ല ഇവയെന്നുള്ളതാണ്. മനുഷ്യരുടെ ജീവിതവിജയത്തിനാവശ്യമായ മാര്‍ഗനിര്‍ദേശങ്ങള്‍ നല്‍കുകയും മരണാനന്തര ജീവിതത്തിലെ ശാശ്വത ശാന്തിയിലേക്ക് അവരെ നയിക്കുകയുമാണ് വെളിപാടുകള്‍ നിര്‍വഹിക്കുന്ന ധര്‍മം. പ്രസ്തുത ധര്‍മ നിര്‍വഹണത്തിനിടയില്‍, ചുറ്റുപാടുകളെയും തന്നെ തന്നെയും നിരീക്ഷിച്ചുകൊണ്ട് സര്‍വ്വലോക സ്രഷ്ടാവിന്റെ അനുഗ്രഹങ്ങളെയും മാര്‍ഗദര്‍ശനത്തിന്റെ അനിവാര്യതയെയും കുറിച്ച് സ്വയം ബോധ്യപ്പെടുത്തുവാന്‍ മനുഷ്യരോട് ആഹ്വാനം ചെയ്യുന്നതിനിടയിലാണ് ഭൗതിക വിജ്ഞാനീയങ്ങളിലേക്ക് പ്രധാനമായും ക്വുര്‍ആനും ഹദീഥുകളും വെളിച്ചം വീശുന്നത്. തലച്ചോറിന്റെ ഉപയോഗത്തിലൂടെ മനുഷ്യര്‍ നേടിയെടുക്കേണ്ട വിവരങ്ങളോ പ്രസ്തുത വിവരങ്ങളുടെ വെളിച്ചത്തില്‍ വികസിപ്പിച്ചെടുക്കേണ്ട സാങ്കേതികവിദ്യയെയോ കുറിച്ച് പഠിപ്പിക്കുകയല്ല, പ്രത്യുത തലച്ചോറിന് മാത്രമായി മനസ്സിലാക്കിയെടുക്കാനാവാത്ത യഥാര്‍ത്ഥമായ അറിവു നല്‍കുകയാണ് വെളിപാടുകളുടെ ധര്‍മം എന്നതുകൊണ്ടുതന്നെ ഭൗതിക വിജ്ഞാനീയങ്ങളുടെ ഏതെങ്കിലുമൊരു ശാഖയെക്കുറിച്ച പൂര്‍ണമായ വിവരങ്ങളോ വിവരണങ്ങളോ തേടി ക്വുര്‍ആനിലോ ഹദീഥുകളിലോ പരതുന്നത് വിഡ്ഢിത്തമാണ്.

മസ്തിഷ്‌കത്തിന് മനസ്സിലാക്കാനാവുന്ന വസ്തുതകളെ ചൂണ്ടിക്കാണിച്ച് അവയുടെ അപഗ്രഥനത്തിലൂടെ മനസ്സിലാക്കാനാവാത്ത ലോകത്തെക്കുറിച്ച് മനസ്സിലാക്കുവാന്‍ മനുഷ്യരോട് പറയുമ്പോള്‍, പ്രസ്തുത വസ്തുതകളെക്കുറിച്ച പരാമര്‍ശങ്ങളിലൊന്നും അബദ്ധങ്ങള്‍ കടന്നുവരുന്നില്ലെന്നതാണ് ഈ വെളിപാടുകളുടെ സവിശേഷത. എഴുതപ്പെട്ട കാലത്തെ അറിവില്ലായ്മയുടെ സ്വാധീനമില്ലാത്ത മതപരമോ മതേതരമോ ആയ ഗ്രന്ഥങ്ങളൊന്നുമില്ലെന്ന സ്വാഭാവികതയ്ക്ക് അപവാദമാണ് ക്വുര്‍ആനും സ്വഹീഹായ ഹദീഥുകളുമെന്ന വസ്തുത വ്യത്യസ്ത വൈജ്ഞാനിക മേഖലകളെക്കുറിച്ച് ഈ വെൡപാടുകളിലുള്ള പരാമര്‍ശങ്ങളെ ഇന്നു നിലനില്‍ക്കുന്ന തെളിയിക്കപ്പെട്ട യാഥാര്‍ത്ഥ്യങ്ങളുമായി താരതമ്യം ചെയ്താല്‍ സുതരാം ബോധ്യപ്പെടും. തെറ്റുപറ്റാത്തവനില്‍നിന്നുള്ളതാണ് ഈ വെളിപാടുകളെന്ന വസ്തുത വ്യക്തമാക്കുവാന്‍ ഇത്തരം താരതമ്യങ്ങള്‍ നിമിത്തമാകുമെന്നാണ് ഇവ്വിഷയകമായ ഇസ്‌ലാമിക പ്രബോധകരുടെ അവകാശവാദം.

മനുഷ്യരെ സ്വന്തത്തെപ്പറ്റി ചിന്തിക്കുവാന്‍ പ്രേരിപ്പിച്ചുകൊണ്ട് സ്രഷ്ടാവിന്റെ അസ്തിത്വത്തെയും പുനരുത്ഥാനത്തിന്റെ സത്യതയെയും കുറിച്ച് ബോധ്യപ്പെടുത്തുന്ന ക്വുര്‍ആന്‍ സൂക്തങ്ങളിലും താന്‍ പ്രവാചകനാണെന്നുള്ള യാഥാര്‍ത്ഥ്യം വ്യക്തമാക്കിക്കൊണ്ടുള്ള നബിവചനങ്ങളിലുമാണ് മനുഷ്യഭ്രൂണത്തിന്റെ ഉല്‍പത്തിയെയും പരിണാമത്തെയും കുറിച്ച പരാമര്‍ശങ്ങളിലധികവും കടന്നുവരുന്നത്. ക്വുര്‍ആനിലും സ്വഹീഹായ ഹദീഥുകളിലും പ്രതിപാദിക്കപ്പെട്ട ഭ്രൂണശാസ്ത്ര വസ്തുതകളെ ആധുനിക പഠനങ്ങള്‍ വെളിച്ചത്തുകൊണ്ടുവന്ന യാഥാര്‍ത്ഥ്യങ്ങളുമായി താരതമ്യം ചെയ്തു പഠിക്കുന്നവര്‍ക്കൊന്നും തന്നെ ഈ സ്രോതസുകളിലുള്ളത് ദൈവിക വെളിപാടാണെന്ന യാഥാര്‍ത്ഥ്യത്തെ നിഷേധിക്കാനാവുകയില്ല.

അതുകൊണ്ടാണല്ലോ, കാനഡയില്‍ ടൊറന്റോ സര്‍വകലാശാലയിലെ പ്രൊഫസറും അറിയപ്പെടുന്ന ഭ്രൂണശാസ്ത്രജ്ഞനും മെഡിക്കല്‍ കോളേജുകളില്‍ പഠിപ്പിക്കുന്ന ഗ്രന്ഥങ്ങളുടെ കര്‍ത്താവുമായ ഡോക്ടര്‍ കീത്ത് മൂര്‍ ഇങ്ങനെ പറഞ്ഞത്: ”മനുഷ്യ പ്രത്യുല്‍പാദനത്തെയും ഭ്രൂണവളര്‍ച്ചയെയും സംബന്ധിച്ച് വിവരിക്കുന്ന ക്വുര്‍ആനിലെയും സുന്നത്തിലെയും വചനങ്ങളെ വ്യാഖ്യാനിക്കുവാനായി സുഊദി അറേബ്യയിലെ ജിദ്ദ കിംഗ് അബ്ദുല്‍ അസീസ് സര്‍വകലാശാലയിലെ ഭ്രൂണശാസ്ത്ര സമിതിയെ സഹായിക്കുവാനും അവരോടൊപ്പം പ്രവര്‍ത്തിക്കുവാനും കഴിഞ്ഞ മൂന്ന് വര്‍ഷമായി എനിക്കു സാധിച്ചു. ഭ്രൂണശാസ്ത്രം തന്നെ സ്ഥാപിക്കപ്പെട്ടിട്ടില്ലാത്ത ക്രിസ്താബ്ദം ഏഴാം നൂറ്റാണ്ടില്‍ രേഖപ്പെടുത്തപ്പെട്ട കാര്യങ്ങളുടെ കൃത്യത കണ്ട് ആദ്യമേ തന്നെ അത്ഭുതപരതന്ത്രനായിതീര്‍ന്നു. ക്രിസ്താബ്ദം പത്താം നൂറ്റാണ്ടില്‍ ജീവിച്ച മുസ്്‌ലിം ശാസ്ത്രജ്ഞന്‍മാരുടെ മഹത്തായ ചരിത്രത്തെക്കുറിച്ചും രോഗശുശ്രൂഷാരംഗത്തെ അവരുടെ സംഭാവനകളെക്കുറിച്ചും എനിക്ക് അറിയാമായിരുന്നുവെങ്കിലും ക്വുര്‍ആനിലും സുന്നത്തിലുമടങ്ങിയിരിക്കുന്ന മതപരമായ കാര്യങ്ങളെപ്പറ്റി എനിക്ക് യാതൊരുവിധ അറിവുമുണ്ടായിരുന്നില്ല.”(L. Keith Moore and Abdul-Majeed al-Zindani: The Developing Human with Islamic Additions, Third Edition, Philadelphia, 1982.)

”മനുഷ്യവളര്‍ച്ചയെക്കുറിച്ച ക്വുര്‍ആന്‍ പരാമര്‍ശങ്ങളെ വ്യക്തമാക്കുവാനായി സഹായിക്കാനാവുകയെന്നത് എന്നെ സംബന്ധിച്ചിടത്തോളം ഏറെ സന്തോഷകരമാണ്. ക്വുര്‍ആനില്‍ പറഞ്ഞ ഈ വിജ്ഞാനങ്ങളില്‍ ഭൂരിഭാഗവും അതിന്റെ അവതരണത്തിന് ശേഷം നൂറ്റാണ്ടുകള്‍ കഴിഞ്ഞുമാത്രം കണ്ടുപിടിക്കപ്പെട്ടവയാണ് എന്നതുകൊണ്ടുതന്നെ അവ മുഹമ്മദിന് ദൈവത്തില്‍നിന്ന് അഥവാ അല്ലാഹുവില്‍നിന്ന് ലഭിച്ചതായിരിക്കുവാനേ നിര്‍വാഹമുള്ളു. മുഹമ്മദ് ദൈവത്തിന്റെ അഥവാ അല്ലാഹുവിന്റെ ദൂതന്‍ തന്നെയാണെന്ന കാര്യമാണ് ഇത് സമര്‍ത്ഥിക്കുന്നത്.”(Abdul-Majeed al-Zindani: This is the Truth (video tape).)

ഭ്രൂണത്തിന്റെ ഉല്‍പത്തിയെയും പരിണാമത്തെയും കുറിച്ച് ആധുനികശാസ്ത്രം നമുക്ക് നല്‍കുന്ന അറിവുകളുടെ വെളിച്ചത്തില്‍ ഈ പരാമര്‍ശങ്ങള്‍ പഠനവിധേയമാക്കുമ്പോള്‍ ഇതിലെ കൃത്യതയും സൂക്ഷ്മതയും ആരെയും ആശ്ചര്യഭരിതരാക്കുമെന്ന കാര്യത്തില്‍ സംശയമില്ല. ഏതെങ്കിലുമൊരു മനുഷ്യന്റെ തലച്ചോറിനകത്ത് രൂപീകരിക്കപ്പെട്ട ആശയങ്ങളുടെ സമാഹാരമാണ് ക്വുര്‍ആനെങ്കില്‍ മുഹമ്മദ് നബി (സ)യുടെ കാലത്ത് നിലനിന്നിരുന്ന അബദ്ധധാരണകളിലേതെങ്കിലും ക്വുര്‍ആനില്‍ ഉണ്ടാവേണ്ടിയിരുന്നു. അത്തരം അബദ്ധങ്ങളൊന്നുമില്ലെന്നു മാത്രമല്ല, ആധുനിക യന്ത്രങ്ങളുടെ സഹായത്തോടെ മാത്രം നാം മനസ്സിലാക്കിയ കാര്യങ്ങള്‍ പോലും വളരെ കൃത്യമായി ക്വുര്‍ആനിലും ഹദീഥുകളിലും പരാമര്‍ശിക്കപ്പെടുന്നുവെന്ന വസ്തുത എന്തുമാത്രം അത്ഭുതകരമല്ല! ആധുനികഭ്രൂണശാസ്ത്രത്തിന്റെ കണ്ണടയിലൂടെ ക്വുര്‍ആനിലും ഹദീഥുകളിലും പരാമര്‍ശിക്കപ്പെട്ട ഭ്രൂണഘട്ടങ്ങളെ നോക്കുന്ന സത്യസന്ധരായ ആര്‍ക്കും ഈ സ്രോതസുകളുടെ ദൈവികത നിഷേധിക്കാനാവില്ല. അതുകൊണ്ടാണല്ലോ ജീവിച്ചിരിക്കുന്നവരില്‍ ഏറ്റവും പ്രഗത്ഭനായ ഡോ. കീത്ത് മൂറിനെപ്പോലുള്ള ഒരു ഭ്രൂണശാസ്ത്രജ്ഞനുപോലും അത് സമ്മതിക്കേണ്ടിവന്നത്!

നുഷ്യസൃഷ്ടിപ്പിന്റെ വ്യത്യസ്ത ഘട്ടങ്ങളെക്കുറിച്ച് പ്രതിപാദിച്ചുകൊണ്ട് സ്രഷ്ടാവിനെ അറിയുവാനും പുനരുത്ഥാനത്തെ നിഷേധിക്കാതിരിക്കാനും മനുഷ്യരെ ഉദ്്‌ബോധിപ്പിക്കുന്ന ക്വുര്‍ആനിക വചനങ്ങള്‍ അനേകമുണ്ട്. ആധുനിക ഉപകരണങ്ങളുടെ സഹായത്തോടെ മാത്രം മനുഷ്യര്‍ക്ക് മനസ്സിലാക്കാനായ ഭ്രൂണോല്‍പത്തിയെയും പരിണാമത്തെയും കുറിച്ച പരാമര്‍ശങ്ങള്‍ നടത്തുമ്പോഴും ക്വുര്‍ആന്‍ പുലര്‍ത്തുന്ന കൃത്യതയും സൂക്ഷ്മതയും വിസ്മയാവഹമാണ്.

മുഹമ്മദ് നബി(സ)യുടെ മസ്തിഷ്‌കത്തില്‍ വിരിഞ്ഞ ചിന്തകളുടെ സമാഹാരമാണ് ക്വുര്‍ആന്‍ എന്ന് വാദിക്കുന്നവര്‍ക്ക് അതില്‍ പ്രതിപാദിക്കപ്പെട്ട സൂക്ഷ്മവും കൃത്യവുമായ പ്രസ്താവനകള്‍ തലവേദനയുണ്ടാക്കാറുണ്ട്. പതിനാലു നൂറ്റാണ്ടുകള്‍ക്കു മുമ്പു ജീവിച്ച ഒരു അറബിനിരക്ഷരന്റെ വായില്‍നിന്ന് ലോകം ശ്രവിച്ച ഒരു ഗ്രന്ഥത്തില്‍ ഇത്ര കൃത്യമായ പരാമര്‍ശങ്ങളുണ്ടാകുന്നതെങ്ങനെയെന്ന ചോദ്യത്തിന് ഉത്തരം പറയുവാന്‍ അവര്‍ ഏറെ പ്രയാസപ്പെടാറുമുണ്ട്. പ്രസ്തുത തലവേദനയില്‍നിന്നുള്ള മോചനത്തിന് ക്വുര്‍ആന്‍ വിമര്‍ശകര്‍ കണ്ടെത്തിയ പരിഹാരമാണ് മുഹമ്മദ് നബി(സ)ക്കു മുമ്പുതന്നെ ലോകത്തെങ്ങും ഭ്രൂണവിജ്ഞാനീയങ്ങള്‍ നിലനിന്നിരുന്നുവെന്നും പ്രസ്തുത വിജ്ഞാനീയങ്ങളില്‍നിന്ന് കടമെടുത്തതാണ് ക്വുര്‍ആനിലും ഹദീഥുകളിലുമുള്ള ഭ്രൂണശാസ്ത്ര പരാമര്‍ശങ്ങളെന്നുമുള്ള വാദം. ക്വുര്‍ആനിന്റെ അവതരണ കാലത്ത് ലോകത്തിന്റെ വിവിധ വശങ്ങളില്‍ നിലനിന്നിരുന്ന ഭ്രൂണവിജ്ഞാനീയങ്ങളെക്കുറിച്ച പഠനം ഈ വാദത്തിന്റെ നട്ടെല്ലൊടിക്കുമെന്നതാണ് സത്യം

ഭാരതീയ സങ്കൽപം:

മനുഷ്യഭ്രൂണത്തിന്റെ പരിണാമദശകളെക്കുറിച്ച് പ്രതിപാദിക്കുന്ന ലഭ്യമായതില്‍വെച്ച് ഏറ്റവും പുരാതനമായ ഗ്രന്ഥം കൃഷ്ണയജുര്‍വേദത്തിന്റെ സംന്യാസ ഉപനിഷത്തായി ഗണിക്കപ്പെടുന്ന ഗര്‍ഭോപനിഷത്താണ്. ക്രിസ്തുവിന് ആയിരത്തിനാനൂറ് വര്‍ഷങ്ങള്‍ക്കുമുമ്പ് പിപ്പലാദ ഋഷി രചിച്ചതായി  കരുതപ്പെടുന്ന ഗര്‍ഭോപനിഷത്തില്‍ ശുക്ലത്തിന്റെ ആവിര്‍ഭാവത്തെയും ഭ്രൂണനിര്‍മാണത്തില്‍ പെണ്‍ശരീരത്തിന്റെ സംഭാവനയെയും ഗര്‍ഭാവസ്ഥയിലെ ഭ്രൂണമാറ്റങ്ങളെയും കുറിച്ച വിശദമായ പരാമര്‍ശങ്ങളുണ്ട്. ഗര്‍ഭോപനിഷത്തിലെ രണ്ടും മൂന്നും വചനങ്ങളിലുള്ള ആശയങ്ങളെ ഇങ്ങനെ സംഗ്രഹിക്കാം:

1. വിവിധങ്ങളായ പദാര്‍ത്ഥങ്ങളില്‍നിന്ന് ഉല്‍ഭൂതമാകുന്ന രസത്തില്‍നിന്ന് രക്തവും അതില്‍നിന്ന് മാംസവും അതില്‍നിന്ന് മേദസ്സും അതില്‍നിന്ന് സ്‌നായുവും അതില്‍നിന്ന് അസ്ഥിയും അതില്‍നിന്ന് മജ്ജയുമുണ്ടാകുന്നു. മജ്ജയില്‍നിന്നാണ് ശുക്ലമുണ്ടാകുന്നത്.

2. പുരുഷന്റെ ശുക്ലവും സ്്ത്രീയുടെ ആര്‍ത്തവരക്തവും (ശോണിതം) ചേര്‍ന്നാണ് ഗര്‍ഭമുണ്ടാകുന്നത്.

3. ഋതുസമയത്ത് ഉചിതമായ രീതിയില്‍ ഗര്‍ഭധാരണം നടന്നാല്‍ ശുക്ലവും ശോണിതവും ചേര്‍ന്ന് ഒരൊറ്റ രാത്രികൊണ്ട് കലലമായിത്തീരും.

4. ഗര്‍ഭസ്ഥശിശു ഏഴു രാത്രികള്‍കൊണ്ട് ബുദ്ബുദവും പതിനഞ്ച് ദിവസങ്ങള്‍കൊണ്ട് പിണ്ഡാകാരവും ഒരു മാസം കൊണ്ട് കഠിനവുമായിത്തീരുന്നു.

5. രണ്ടുമാസം കൊണ്ട് തലയും മൂന്ന് മാസം കൊണ്ട് കാലും നാലാം മാസം മുട്ടുകളും അരയും വയറും അഞ്ചാം മാസം പൃഷ്ഠം, നട്ടെല്ല് എന്നിവയും രൂപപ്പെടുന്നു.

6. ആറാം മാസത്തിലാണ് വായ്, മൂക്ക്, ചെവി, കണ്ണുകള്‍ എന്നിവ രൂപപ്പെടുന്നത്.

7. ഏഴാം മാസത്തില്‍ കുഞ്ഞ് ജീവസ്പന്ദത്തോടെ പുഷ്ടിപ്പെടുന്നു.

8. പരിപൂര്‍ണ ശരീരം രൂപം പ്രാപിക്കുന്നത് എട്ടാം മാസത്തിലാണ്.

9. ശുക്ലമാണ് അധികമെങ്കില്‍ ആണ്‍കുഞ്ഞും ആര്‍ത്തവരക്തമാണ് അധികമെങ്കില്‍ പെണ്‍കുഞ്ഞും രണ്ടും തുല്യമാണെങ്കില്‍ നപുംസകവുമാണുണ്ടാവുക.

10. സ്ത്രീപുരുഷ സംയോഗം നടക്കുന്ന സന്ദര്‍ഭത്തിലെ ദമ്പതിമാരുടെ മനോവ്യഥ കാരണമാണ് കുഞ്ഞിന് അംഗവൈകല്യങ്ങളുണ്ടാകുന്നത്.

11. വായുവിനാല്‍ ശുക്ല-ശോണിതങ്ങള്‍ ഭേദിക്കപ്പെടുന്നതിനാലാണ് ഇരട്ടകളും മറ്റുമുണ്ടാകുന്നത്.

12. ജ്ഞാനേന്ദ്രിയാദികള്‍ ചേര്‍ന്ന് ശിശു പൂര്‍ണത പ്രാപിക്കുന്നത് ഒമ്പതാം മാസത്തിലാണ്.

ഇസ്രായീല്യരുടെ സങ്കല്‍പം:

ഭാഷാശാസ്ത്ര പ്രകാരം പരിശോധിച്ചാല്‍ എഴുതപ്പെട്ടതില്‍ വെച്ച് ഏറ്റവും പുരാതനമായ ബൈബിള്‍ പുസ്തകമെന്ന് മനസ്സിലാക്കാനാകുന്ന ഇയ്യോബിന്റെ പുസ്തകത്തില്‍ തന്നെ ഭ്രൂണോല്‍പാദനത്തെക്കുറിച്ച പരിമിതമായ ചില പരാമര്‍ശങ്ങളുണ്ട്. അതിങ്ങനെ വായിക്കാം:

”കളിമണ്ണുകൊണ്ടാണ് അങ്ങ് എന്നെ സൃഷ്ടിച്ചതെന്ന് അനുസ്മരിക്കേണമേ! പൊടിയിലേക്കുതന്നെ അങ്ങ് എന്നെ തിരിച്ചയക്കുമോ? അങ്ങ് എന്നെ പാലുപോലെ പകര്‍ന്ന് തൈരുപോലെ ഉറ കൂട്ടിയില്ലേ? അങ്ങ് ചര്‍മവും മാംസവും കൊണ്ട് എന്നെ ആവരണം ചെയ്തു; അസ്ഥിയും സ്‌നായുക്കളുംകൊണ്ട് എന്നെ തുന്നിച്ചേര്‍ത്തു.”( ഇയ്യോബ് 10: 9-11).

മനുഷ്യരെ പാല്‍ പോലെ പകരുകയും തൈരുപോലെ ഉറ കൂടിപ്പിക്കുകയും അതിനുശേഷം ത്വക്കും മാംസവും ധരിപ്പിക്കുകയും അസ്ഥിയും ഞരമ്പും കൊണ്ട് മെടയുകയും ചെയ്തവനായി സ്രഷ്ടാവിനെ അഭിസംബോധന ചെയ്യുന്ന ഇയ്യോബിന്റെ വരികള്‍ ഭ്രൂണോല്‍പത്തിയെക്കുറിച്ച് അന്ന് നിലനിന്നിരുന്ന സങ്കല്‍പമെന്തായിരുന്നുവെന്ന് മനസ്സിലാക്കിത്തരുന്നുണ്ട്. പുരുഷ ശുക്ലം കട്ട പിടിച്ചാണ് കുഞ്ഞുണ്ടാവുന്നതെന്നും അതില്‍നിന്നാണ് അവയവങ്ങള്‍ രൂപീകരിക്കപ്പെടുന്നതെന്നുമായിരുന്നു അന്നു നിലനിന്നിരുന്ന സങ്കല്‍പം. ഇയ്യോബില്‍നിന്ന് വ്യത്യസ്തമായ വീക്ഷണങ്ങളെന്തെങ്കിലും ഔദ്യോഗിക കാനോനില്‍ ഉള്‍പ്പെടുത്തപ്പെട്ട ബൈബിള്‍ ഗ്രന്ഥങ്ങളുടെ കര്‍ത്താക്കള്‍ക്ക് ഉണ്ടെന്ന് വ്യക്തമാക്കുന്ന പരാമര്‍ശങ്ങളൊന്നും തന്നെ ബൈബിള്‍ പുസ്തകങ്ങളിലില്ല.

പുരുഷബീജത്തില്‍ നിന്നാണ് കുഞ്ഞുണ്ടാവുന്നതെങ്കിലും അത് പുഷ്ടി പ്രാപിക്കുന്നത് മാതാവിന്റെ ആര്‍ത്തവരക്തത്താലാണെന്നായിരുന്നു ഇസ്രായീല്യരുടെ വിശ്വാസമെന്ന് വ്യക്തമാക്കുന്ന ഒരു പരാമര്‍ശം കത്തോലിക്കാ ബൈബിളിലുണ്ട്. യഹൂദന്‍മാരുടെ കാനോനിലോ പ്രൊട്ടസ്റ്റന്റ് ബൈബിളിലോ ഉള്‍പ്പെടുത്തിയിട്ടില്ലാത്ത, കത്തോലിക്കര്‍ ഉത്തര കാനോനിക ഗ്രന്ഥങ്ങളെന്ന് വിളിച്ച് അവരുടെ ബൈബിളിന്റെ ഭാഗമായി ആദരിക്കുന്ന സോളമന്റെ വിജ്ഞാനത്തിലാണ് പ്രസ്തുത പരാമര്‍ശമുള്ളത്. അതിങ്ങനെ വായിക്കാം: ”ദാമ്പത്യത്തിന്റെ ആനന്ദത്തില്‍ പുരുഷബീജത്തില്‍ നിന്ന് ജീവന്‍ ലഭിച്ചു, പത്തു മാസം കൊണ്ട് അമ്മയുടെ രക്തത്താല്‍ പുഷ്ടി പ്രാപിച്ചു.”( (വിജ്ഞാനം 7: 2) ഇവയിൽ നിന്നും അന്നത്തെ യഹൂദറബ്ബിമാരുടെ തൽമുദിൽ നിന്നുമായി ഇസ്രാഈല്യർക്കിടയിൽ നിലനിന്നിരുന്ന ഭ്രൂണഘട്ടങ്ങളെക്കുറിച്ച വിവരങ്ങൾ നമുക്ക് ലഭിക്കും. അവയെ ഇങ്ങനെ സംക്ഷേപിക്കാം: 1. പുരുഷ ശുക്ലം ഗര്‍ഭാശയത്തിലെത്തിയ ശേഷം പാലില്‍നിന്ന് തൈരുണ്ടാകുന്നതുപോലെ കട്ടിയായിത്തീര്‍ന്നാണ് ഭ്രൂണമുണ്ടാകുന്നത്.

2. ശുക്ലത്തില്‍നിന്നുണ്ടാകുന്ന ഭ്രൂണം വളരുമ്പോള്‍ ആര്‍ത്തവരക്തം അതിനെ പരിപോഷിപ്പിക്കുന്നു.

3. പിതാവില്‍നിന്നുണ്ടാകുന്ന വെളുത്ത ശുക്ലത്തില്‍നിന്നാണ് അസ്ഥികള്‍, സ്‌നായുക്കള്‍, നഖങ്ങള്‍, മസ്തിഷ്‌കം എന്നിവയുണ്ടാകുന്നത്.

4. മാതാവില്‍നിന്നുള്ള ചുവന്ന ശുക്ലത്തില്‍നിന്നാണ് തൊലി, മാംസം, മുടി, രക്തം, കണ്ണിന്റെ കറുപ്പ് എന്നിവയുണ്ടാകുന്നത്.

5. ദൈവദത്തമായ ആത്മാവിന്റെ പ്രവര്‍ത്തനഫലമായാണ് ജീവനും ജൈവപ്രതിഭാസങ്ങളുമുണ്ടാകുന്നത്.

ഹിപ്പോക്രാറ്റസിന്റെ സങ്കല്‍പം:

പാശ്ചാത്യ ചികിത്സാ ശാസ്ത്രത്തിന്റെ പിതാവായി അറിയപ്പെടുന്ന ഹിപ്പോക്രാറ്റസാണ് (460 BC-377 BC) ഭ്രൂണത്തിന്റെ രൂപീകരണത്തെയും പരിണാമത്തെയും കുറിച്ച വീക്ഷണങ്ങളവതരിപ്പിച്ച ഗ്രീക്കുകാരില്‍ പ്രഥമ ഗണനീയന്‍. ഹിപ്പോക്രാറ്റസ് തന്റെ ഭ്രൂണശാസ്ത്ര വീക്ഷണങ്ങള്‍ അവതരിപ്പിച്ചിരിക്കുന്നത് ചികിത്സാക്രമം (Regimen), വിത്ത് (The Seed), കുഞ്ഞിന്റെ പ്രകൃതി (The Nature of the Child) എന്നീ രചനകളിലാണ്. ഈ കൃതികളിലൂടെ ഹിപ്പോക്രാറ്റസ് അവതരിപ്പിച്ച മനുഷ്യോല്‍പത്തി സങ്കല്‍പങ്ങളെ ഇങ്ങനെ സംഗ്രഹിക്കാം.

1. എല്ലാ പ്രാപഞ്ചിക പ്രതിഭാസങ്ങളും അഗ്നിയും ജലവും ചേര്‍ന്നാണ് ഉണ്ടാകുന്നത്. ഇതില്‍ അഗ്നിയാണ് ഭ്രൂണത്തിന് കാരണമായിത്തീരുന്നത്.

2. മാതാവും പിതാവും ശുക്ലം ഉല്‍പാദിപ്പിക്കുന്നുണ്ട്. ഗര്‍ഭധാരണത്തിന്റെ സമയത്തല്ലാതെ പിതാവും മാതാവും വിസര്‍ജിക്കാത്ത ശുക്ലങ്ങള്‍ മാതാവിന്റെ ജനനേന്ദ്രിയത്തില്‍നിന്ന് പുറംതള്ളപ്പെടുകയാണ് ചെയ്യുന്നത്. ഗര്‍ഭധാരണ കാലത്തെ ഈര്‍പ്പത്താല്‍ ഗര്‍ഭാശയരന്ധ്രം ചുരുങ്ങുന്നതിനാല്‍ രണ്ടുപേരുടെയും ശുക്ലങ്ങള്‍ അവിടെ തങ്ങുകയും കൂടിച്ചേരുകയും ചെയ്യുന്നു.

3. ശരീരത്തിന്റെ വ്യത്യസ്ത ഭാഗങ്ങളില്‍നിന്നാണ് ശുക്ലമുണ്ടാകുന്നത്. ശക്തമായ ശാരീരികാവയവങ്ങളില്‍നിന്ന് ബലമുള്ള ബീജവും അശക്തമായ ശാരീരികാവയവങ്ങളില്‍നിന്ന് അബല ബീജവുമുണ്ടാകുന്നു.

4. രണ്ടുപേരുടെയും ശുക്ലത്തില്‍ സ്ത്രീബീജവും പുരുഷബീജവും ഉണ്ടായിരിക്കും.

5. ബലബീജവും അബല ബീജവും എങ്ങനെ യോജിക്കുന്നുവെന്നതിനനുസരിച്ചായിരിക്കും കുഞ്ഞിന്റെ ലിംഗം നിര്‍ണയിക്കപ്പെടുക. സ്ത്രീയും പുരുഷനും ഉല്‍പാദിപ്പിക്കുന്നത് ശക്തമായ ബീജങ്ങളാണെങ്കില്‍ അവ കൂടിച്ചേര്‍ന്നുണ്ടാകുന്നത് ആണ്‍കുഞ്ഞും അശക്തങ്ങളായ ബീജങ്ങളാണെങ്കില്‍ അവ കൂടിച്ചേര്‍ന്നുണ്ടാകുന്നത് പെണ്‍കുഞ്ഞുമായിരിക്കും.

6. ഒരാള്‍ ശക്തമായ ബീജവും മറ്റേയാള്‍ അശക്തമായ ബീജവുമാണ് ഉല്‍പാദിപ്പിക്കുന്നതെങ്കില്‍ ഏത് ബീജമാണോ കൂടുതലുള്ളത് അതിനനുസരിച്ചാണ് കുഞ്ഞിന്റെ ലിംഗം നിര്‍ണയിക്കപ്പെടുക.

7. ബീജങ്ങള്‍ കൂടിച്ചേര്‍ന്നുണ്ടാകുന്ന ഭ്രൂണത്തിലേക്ക് മാതൃരക്തമെത്തി അത് കട്ടപിടിച്ചുകൊണ്ടാണ് ഗര്‍ഭസ്ഥശിശുവിന്റെ ശരീരത്തില്‍ മാംസം വളരുന്നത്.

8. മാംസം വളരുന്നതനുസരിച്ച് അവയവങ്ങളെല്ലാം ഒന്നിച്ചാണ് രൂപീകരിക്കപ്പെടുന്നത്.

9. ആണ്‍കുഞ്ഞിന്റെ അവയവങ്ങള്‍ വളരാന്‍ മുപ്പതുദിവസങ്ങള്‍ മതി. പെണ്‍കുഞ്ഞിന്റെ അവയവവളര്‍ച്ചയ്ക്ക് പരമാവധി നാല്‍പത്തിരണ്ടു ദിവസങ്ങളാണ് വേണ്ടത്.

അരിസ്റ്റോട്ടിലിന്റെ സങ്കൽപം:

  ഹിപ്പോക്രാറ്റസിന് ശേഷം ശ്രദ്ധേയമായ ഭ്രൂണപരിണാമപരാമര്‍ശങ്ങള്‍ നടത്തിയ ഗ്രീക്കുകാരന്‍ തത്ത്വജ്ഞാനിയും ശാസ്ത്രജ്ഞനുമായിരുന്ന അരിസ്റ്റോട്ടിലാണ് (384-322 BC).താനെഴുതിയ നാനൂറിലധികം പുസ്തകങ്ങളില്‍ ഭ്രൂണത്തിന്റെ ഉല്‍പത്തിയെയും പരിവര്‍ത്തനങ്ങളെയും കുറിച്ച് അരിസ്റ്റോട്ടില്‍ വിവരിക്കുന്നത് ജന്തുക്കളുടെ ഉല്‍പത്തിയെപ്പറ്റി (On the Generation of Animals) എന്ന ഗ്രന്ഥത്തിലാണ്.

അതിൽ പറഞ്ഞിരിക്കുന്ന അരിസ്റ്റോട്ടിലിന്റെ വീക്ഷണങ്ങളെ ഇങ്ങനെ സംഗ്രഹിക്കാം:

1. പാല്‍ ഉറ കൂട്ടാന്‍ വേണ്ടി റെന്നറ്റ് ഉപയോഗിക്കപ്പെടുന്നതുപോലെ മാതൃശരീരത്തില്‍നിന്നുള്ള സ്രവങ്ങളെ കട്ടിയാക്കുകയാണ് പുരുഷശരീരത്തില്‍നിന്ന് സ്രവിക്കപ്പെടുന്ന ശുക്ലത്തിന്റെ ധര്‍മം. അതിനാല്‍ മാതൃസ്രവമാണ് യഥാര്‍ഥത്തില്‍ അടിസ്ഥാന വസ്തു. പാലിനെ കട്ടിയാക്കുകയെന്നതില്‍ കവിഞ്ഞ ധര്‍മമൊന്നും തന്നെ ഉറ കൂട്ടുന്ന പ്രക്രിയയില്‍ റെന്നറ്റിന് ഇല്ലാത്തതുപോലെ മാതൃരക്തത്തെ കട്ടിയാക്കുക മാത്രമാണ് ശുക്ലം ചെയ്യുന്നത്.

2. ഇങ്ങനെ കട്ടിയായിത്തീര്‍ന്ന് ഉറ കൂടിയ മാതൃരക്തത്തില്‍നിന്ന് രൂപപ്പെടുന്ന ഭ്രൂണം മണ്ണില്‍ നട്ടുപിടിപ്പിച്ച വിത്ത് വളരുന്നതുപോലെ മെല്ലെ വളര്‍ന്നുവരികയാണ് ചെയ്യുന്നത്.

3. ഗര്‍ഭസ്ഥ ശിശുവിന്റെ ശരീരത്തില്‍ ആദ്യമായി പ്രവര്‍ത്തനമാരംഭിക്കുന്നത് ഹൃദയമാണ്.

4. ഹൃദയത്തില്‍ നിന്നാരംഭിക്കുന്ന രക്തധമനികള്‍ ശരീരത്തിലാകമാനം വിന്യസിക്കപ്പെടുന്നു.

5. ശരീരാവയവങ്ങളുടെ രൂപീകരണം നടക്കുന്നത് താപത്തിന്റെയും തണുപ്പിന്റെയും പ്രവര്‍ത്തനഫലമായിട്ടാണ്. ചില അവയവങ്ങള്‍ ഘനീഭവിക്കപ്പെടുന്നത് താപത്താലാണെങ്കില്‍ മറ്റ് ചിലവയുണ്ടാകുന്നത് തണുപ്പിനാലാണ്.

6. പോഷകാഹാരം രക്തത്തിലൂടെ ശരീരാവയവങ്ങളിലേക്ക് ഒലിച്ചിറങ്ങുമ്പോഴാണ് അതിന്റെ ഒപ്പം തന്നെയുള്ള മാംസം രൂപീകരിക്കപ്പെടുന്നത്. മാംസം രൂപീകരിക്കപ്പെടുന്നതോടെ തണുപ്പ് അതിനെ ഘനീഭവിപ്പിക്കുന്നു.

7. ആന്തരികതാപത്താല്‍ ദ്രാവകങ്ങള്‍ ദൃഢമായിത്തീരുന്നതിനാലാണ് എല്ലുകളും സ്‌നായുക്കളുമെല്ലാം ഉണ്ടാകുന്നത്. അതുകൊണ്ടാണ് ഇവയെ തീ കൊണ്ട് വിഘടിപ്പിച്ച് നശിപ്പിക്കാന്‍ കഴിയാത്തത്.

8. നാം ഭക്ഷണപദാര്‍ത്ഥങ്ങള്‍ ചൂടാക്കിയും തണുപ്പിച്ചുമെല്ലാം നിര്‍മിച്ചെടുക്കുന്നതുപോലെ തന്നെ ഗര്‍ഭാശയത്തിനകത്തെ ചൂടും തണുപ്പും നമ്മുടെ അവയവങ്ങളെയെല്ലാം ചുട്ടെടുക്കുകയാണ് ചെയ്യുന്നത്.

9.  ചൂടാക്കിയ ദ്രാവകങ്ങള്‍ക്കു മുകളിലുണ്ടാവുന്ന പാടയെപ്പോലെയാണ് മാംസത്തിന് പുറത്തെ പാടയായി തൊലിയുണ്ടാകുന്നത്.

10. എല്ലാവിധ ഇന്ദ്രിയാനുഭവങ്ങളുടെയും കേന്ദ്രമായ ഹൃദയം രൂപീകരിക്കപ്പെട്ടശേഷം അതിന്റെ താപഫലമായിട്ടാണ് മസ്തിഷ്‌കം രൂപപ്പെടുന്നത്. മറ്റു ശാരീരികാവയവങ്ങള്‍ ഉണ്ടാകുന്നതിന് മുമ്പുതന്നെ മസ്തിഷ്‌കത്തെ പൊതിഞ്ഞുകൊണ്ടുള്ള ശാരീരികവ്യവസ്ഥകള്‍ രൂപപ്പെടുന്നത് അതുകൊണ്ടാണ്.

  ഗാലന്റെ സങ്കൽപം

ഹിപ്പോക്രാറ്റസിന്റെയും അരിസ്‌റ്റോട്ടിലിന്റെയുമെല്ലാം ഭ്രൂണപരിണാമത്തെക്കുറിച്ച വീക്ഷണങ്ങളെ അപഗ്രഥിച്ചുകൊണ്ട് തന്റേതായ ഭ്രൂണശാസ്ത്ര കാഴ്ചപ്പാടുകള്‍ അവതരിപ്പിച്ച ശ്രദ്ധേയനായ ഗ്രീക്ക് തത്ത്വജ്ഞാനിയാണ് ഗാലന്‍ എന്നറിയപ്പെടുന്ന ക്ലോഡിയസ് ഗാലെനെസ് (ക്രി.വ 129-200). വിവിധ ശാസ്ത്രവിഷയങ്ങളെപ്പറ്റി പ്രതിപാദിക്കുന്ന അഞ്ഞൂറോളം വരുന്ന പ്രബന്ധങ്ങളില്‍ ‘ശുക്ലത്തെപ്പറ്റി’ (On semen), ‘പ്രകൃതി പ്രഭാവങ്ങളെപ്പറ്റി’ (On the Natural Faculties), ‘ഗര്‍ഭസ്ഥ ശിശുവിന്റെ രൂപീകരണത്തെപ്പറ്റി’ (On the Formation of the Foetus) എന്നീ മൂന്നെണ്ണത്തിലാണ് തന്റെ ഭ്രൂണപരിണാമവീക്ഷണങ്ങള്‍ അദ്ദേഹം അവതരിപ്പിക്കുന്നത്.

ഇവയിലുള്ള ഗാലന്റെ ഭ്രൂണശാസ്ത്ര വീക്ഷണങ്ങളെ ഇങ്ങനെ സംഗ്രഹിക്കാം.

1. ഗര്‍ഭാശയത്തിലെത്തുന്ന ശുക്ലമാണ് ജന്തുവിന്റെ രൂപീകരണത്തിന് നിമിത്തമാകുന്ന സജീവമായ തത്ത്വം.

2. ആറാം ദിവസം ശുക്ലം ഇല്ലാതാവുകയും പകരം ഭ്രൂണം വളരാനാരംഭിക്കുകയും ചെയ്യും. ഭ്രൂണവളര്‍ച്ച നടക്കുന്നത് മാതൃശരീരത്തില്‍ രൂപപ്പെടുന്ന ആര്‍ത്തവരക്തം ശുക്ലത്തെ പോഷിപ്പിക്കുമ്പോഴാണ്.

3. ശുക്ലത്തെ രക്തം പരിപോഷിപ്പിക്കുമ്പോള്‍ അത് ഒരു മാംസപിണ്ഡമായിത്തീരുന്നു. ഹൃദയമോ കരളോ മസ്തിഷ്‌കമോ ഇല്ലാതെതന്നെ ഈ മാംസപിണ്ഡം നിലനില്‍ക്കുകയും വളരുകയും ചെയ്യുന്നു.

4. ശരീരത്തെ ഭരിക്കുന്ന മൂന്ന് അവയവങ്ങള്‍ ഒരു നിഴല്‍ ചിത്രത്തിലെന്നവണ്ണം പ്രത്യക്ഷപ്പെടുന്ന ഘട്ടമാണ് അടുത്തത്. ഈ അവയവങ്ങള്‍ കൂടുതല്‍ വ്യക്തതയോടെ ദൃശ്യമാകുന്നതോടൊപ്പം തന്നെ വയറിന്റെ ഭാഗങ്ങളും മെല്ലെ പ്രത്യക്ഷപ്പെടാനാരംഭിക്കുന്നു.

5. ഇതിനുശേഷം ശാരീരികാവയവങ്ങള്‍ വേര്‍പിരിയുകയും കൃത്യമായി കാണാനാവുന്ന അവസ്ഥ സംജാതമാവുകയും ചെയ്യുന്നു. 6. പിന്നീട് ശരീരത്തിലെ കൊമ്പുകളും ചില്ലകളുമെല്ലാം പ്രത്യക്ഷീഭവിക്കുന്നു.

 

ക്വുര്‍ആനിന്റെ അവതരണ കാലത്ത് ലോകത്തെങ്ങും നിലനിന്നിരുന്ന ഭ്രൂണ വിജ്ഞാനീയങ്ങളുടെ ഒരു ഏകദേശചിത്രമാണിത്. ഇതില്‍ ഹിപ്പോക്രാറ്റസിന്റെയും അരിസ്‌റ്റോട്ടിലിന്റെയും ഗാലന്റെയുമെല്ലാം ഭ്രൂണപരിണാമത്തെക്കുറിച്ച വീക്ഷണങ്ങളാണ് നവോത്ഥാനകാലം വരെ യൂറോപ്പില്‍ നിലനിന്നിരുന്നത്.   ഈ ധാരണകളില്ലെല്ലാം നിരവധി അബദ്ധങ്ങളുണ്ടെന്ന് ഇന്ന് നമുക്കറിയാം. എന്നാല്‍ ഈ അബദ്ധധാരണകള്‍ നിലനിന്നിരുന്ന കാലത്ത് അവതരിപ്പിക്കപ്പെട്ട ക്വുര്‍ആന്‍സൂക്തങ്ങളില്‍ ഈ ധാരണകളുടെ സൂക്ഷ്മമായ സ്വാധീനം പോലുമില്ല. അബദ്ധധാരണകള്‍ മാത്രം നിലനിന്നിരുന്ന ഭ്രൂണവിജ്ഞാനീയത്തെകുറിച്ച് പരാമര്‍ശിക്കുമ്പോള്‍ പ്രസ്തുത ധാരണകളുടെ യാതൊരു സ്വാധീനവുമില്ലാതെ കാര്യങ്ങള്‍ വിശദീകരിക്കുന്നത് തന്നെ അത്ഭുമാണ്. ക്വുര്‍ആനാകട്ടെ അബദ്ധധാരണകളുടെ സ്വാധീനമൊന്നുമില്ലാതെ കാര്യങ്ങള്‍ വ്യക്തമാക്കുക മാത്രമല്ല, പ്രത്യുത ആധുനികശാസ്ത്രവിദ്യകളുടെ സഹായത്താല്‍ മാത്രം നാം കണ്ടെത്തിയ കാര്യങ്ങള്‍ വളരെ കൃത്യമായി പരാമര്‍ശിക്കുക കൂടി ചെയ്യുന്നു. ഇത് ക്വുര്‍ആനിനെ അത്ഭുതങ്ങളുടെ അത്ഭുതമാക്കിത്തീര്‍ക്കുകയാണ് ചെയ്യുന്നത്. സര്‍വജ്ഞനായ അല്ലാഹുവിനല്ലാതെ ഇത്ര കൃത്യമായ പ്രസ്താവനകള്‍ നടത്താന്‍ കഴിയില്ല. അവതരണകാലത്ത് നിലനിന്നിരുന്ന അബദ്ധധാരണകളുടെ സ്വാധീനമില്ലാതെയും ആധുനികശാസ്ത്രം സ്ഥിരീകരിച്ച വസ്തുതകളിലേക്ക് വെളിച്ചംവീശിയും ക്വുര്‍ആന്‍ അതിന്റെ അപ്രമാദിത്വം വെളിപ്പെടുത്തുകയും ദൈവികത പ്രഖ്യാപിക്കുകയുമാണ് ചെയ്യുന്നത്

വിഷയവുമായി ബന്ധപ്പെട്ട വീഡിയോ

ഖുര്‍ആനില്‍ ആറ് ആയത്തുകളില്‍ ഹാമാനെക്കുറിച്ച് പരാമര്‍ശിക്കുന്നുണ്ട്. (28: 6, 28: 38, 29: 39, 40: 24, 40: 36-37) ഇവയെല്ലാം മൂസാനബി(അ)യുടെ ജീവിതവുമായി ബന്ധപ്പെട്ടുകൊണ്ടുള്ള സൂക്തങ്ങള്‍ തന്നെയാണ്. ഫറോവയുടെ കൊട്ടാരവുമായി ബന്ധമുള്ള വ്യക്തിയാണ് ഖുര്‍ആനിലെ ഹാമാന്‍. ഫറോവയുമായി ബന്ധമുള്ള ഒരു ഹാമാനെപ്പറ്റി ബൈബിളിലെവിടെയും നാം വായിക്കുന്നില്ല. എന്നാല്‍ എസ്‌തേറിന്റെ പുസ്തകത്തില്‍ അഹശ്വേറോശ് രാജാവിന്റെ കീഴിലുള്ള ഒരു പ്രഭുവായ ഹാമാനെക്കുറിച്ച് പറയുന്നുണ്ട്. മുഹമ്മദ് നബിക്ക് ഫറോവയും അഹശ്വേറോശ്‌ രാജാവും തമ്മില്‍ പരസ്പരം മാറിപ്പോയതിനാല്‍ സംഭവിച്ചുപോയ ഒരു കൈപ്പിഴയുടെ ഫലമായി വന്നുചേര്‍ന്നതാണ് ഖുര്‍ആനിലെ ഫറോവാ ഹാമാന്‍ കഥയെന്നാണ് വിമര്‍ശകരുടെ ആരോപണം. പതിനേഴാം നൂറ്റാണ്ടിനുശേഷം എഴുതപ്പെട്ട ഓറിയന്റലിസ്റ്റുകളുടെയും ക്രിസ്ത്യന്‍ മിഷനറിമാരുടെയും ഒരുവിധം എല്ലാ ഖുര്‍ആന്‍ വിമര്‍ശന ഗ്രന്ഥങ്ങളിലും ഈ ആരോപണം കാണാം. വാമൊഴിയായി കാര്യങ്ങള്‍ കേട്ടറിഞ്ഞ ശേഷം തന്‍േറതായ ഭാഷാശൈലിയില്‍ ബൈബിള്‍ കഥകള്‍ അവതരിപ്പിക്കാന്‍ ശ്രമിച്ച മുഹമ്മദി(ﷺ)ന് സംഭവിച്ച ഒരു കൈപ്പിഴയായാണ് ഇതിനെ ഓറിയന്റലിസ്റ്റുകള്‍ വിലയിരുത്തുന്നത്. ഈ വിലയിരുത്തലുകള്‍ എത്രത്തോളം സത്യസന്ധമാണ് എന്ന് പരിശോധിക്കുന്നതിനു മുമ്പ് ഫറോവയേയും ഹാമാനെയും ബന്ധപ്പെടുത്തിക്കൊണ്ടുള്ള പരാമര്‍ശങ്ങളുള്‍ക്കൊള്ളുന്ന ഖുര്‍ആന്‍ സൂക്തങ്ങള്‍ പരിശോധിക്കുക.

”ഫിര്‍ഔന്‍ പറഞ്ഞു: പ്രമുഖന്മാരേ ഞാനല്ലാതെ യാതൊരു ദൈവവും നിങ്ങള്‍ക്കുള്ളതായി ഞാന്‍ അറിഞ്ഞിട്ടില്ല. അതുകൊണ്ട് ഹാമാനേ എനിക്കു വേണ്ടി കളിമണ്ണ് കൊണ്ട് (ഇഷ്ടിക) ചുട്ടെടുക്കുക. എന്നിട്ട് എനിക്ക് നീ ഒരു ഉന്നതസൗധം ഉണ്ടാക്കിത്തരിക. മൂസായുടെ ദൈവത്തിങ്കലേക്ക് എനിക്കൊന്ന് എത്തിനോക്കാമല്ലോ. തീര്‍ച്ചയായും അവന്‍ വ്യാജം പറയുന്നവരുടെ കൂട്ടത്തിലാണെന്നാണ് ഞാന്‍ വിചാരിക്കുന്നത്” (വി.ഖു.28:38). ”ഫിര്‍ഔന്‍ പറഞ്ഞു: ഹാമാനേ എനിക്ക് ആ മാര്‍ഗങ്ങളില്‍ അഥവാ ആകാശ മാര്‍ഗങ്ങളില്‍ എത്തിച്ചേരുവാനും എന്നിട്ട് മൂസായുടെ ദൈവത്തിന്റെ അടുത്തേക്ക് എത്തി നോക്കുവാനും തക്കവണ്ണം എനിക്കുവേണ്ടി നീ ഒരു ഉന്നത സൗധം പണിത് തരൂ. തീര്‍ച്ചയായും അവന്‍ (മൂസാ) കളവ് പറയുകയാണെന്നാണ് ഞാന്‍ വിചാരിക്കുന്നത്. അപ്രകാരം ഫിര്‍ഔന് തന്റെ ദുഷ്പ്രവൃത്തി അലംകൃതമായി തോന്നിക്കപ്പെട്ടു. നേരായ മാര്‍ഗത്തില്‍ നിന്ന് അവന്‍ തടയപ്പെടുകയും ചെയ്തു. ഫറോവയുടെ തന്ത്രം നഷ്ടത്തില്‍ തന്നെയാകുന്നു” (വി.ഖു. 40:36, 37). ഈ സൂക്തങ്ങളില്‍ പറയുന്ന കാര്യങ്ങളില്‍ നിന്ന് മനസ്സിലാക്കാന്‍ കഴിയുന്നത് താഴെ പറയുന്ന കാര്യങ്ങളാണ്: (1) ഹാമാന്‍ ഫിര്‍ഔനിന്റെ കീഴിലുള്ള ഒരു ഉദ്യോഗസ്ഥനാണ്. മന്ത്രിയാണെന്നാണ് ഖുര്‍ആന്‍ വ്യാഖ്യാതാക്കളുടെ പക്ഷം. (2) മൂസാ (അ) പറഞ്ഞ ദൈവം ആകാശത്തെവിടെയെങ്കിലും ഉണ്ടോയെന്ന് നോക്കാനായി ഒരു ഗോപുരം നിര്‍മ്മിക്കുവാന്‍ ഫിര്‍ ഔന്‍ ഹാമാനോട് കൽപിച്ചു. (3) കളിമണ്ണുകൊണ്ട് ചുട്ടെടുത്ത ഇഷ്ടിക ഉപയോഗിച്ചുകൊണ്ട് സൗധം നിര്‍മ്മിക്കാനാണ് ഫിര്‍ഔന്‍ ആവശ്യപ്പെട്ടത്. ഖുര്‍ആന്‍ മുന്നോട്ടുവെക്കുന്ന ഈ കാര്യങ്ങളുടെ വെളിച്ചത്തില്‍ വിമര്‍ശനങ്ങള്‍ ഓരോന്നായി പരിശോധിക്കുമ്പോള്‍ അവയെല്ലാം ഖുര്‍ആനിക പ്രകാശത്തിനുമുമ്പില്‍ കരിഞ്ഞുപോകുന്ന കേവലം ധൂളികള്‍ മാത്രമാണെന്ന് ബോധ്യമാകും. വിമര്‍ശനങ്ങളും അവയ്ക്ക് നല്‍കുവാനുള്ള മറുപടിയും ഇങ്ങനെസംഗ്രഹിക്കാം. ഒന്ന്) ഫറോവയുടെ ചരിത്രവുമായി ബന്ധപ്പെടുത്തി ബൈബിളിലെവിടെയും ഹാമാന്‍ എന്ന ഒരാളെക്കുറിച്ച് യാതൊന്നുംപറയുന്നില്ല. അതിനാല്‍ ഖുര്‍ആനില്‍ പറയുന്ന ഫിര്‍ഔന്‍-ഹാമാന്‍ കഥ ഒരു കെട്ടുകഥ മാത്രമാണ്. ബൈബിള്‍ കഥകളെല്ലാം നൂറുശതമാനം സത്യസന്ധവും വസ്തുനിഷ്ഠവുമാണെന്ന സങ്കല്‍പത്തിന്റെ അടിസ്ഥാനത്തിലാണ് ഈ വിമര്‍ശനം ഉന്നയിക്കപ്പെടുന്നത്. ഈ സങ്കല്‍പം തന്നെ അടിസ്ഥാനമില്ലാത്തതാണ്. ചരിത്രകാരന്റെ വീക്ഷണത്തില്‍ നോക്കിയാല്‍ തീരെ വസ്തുതാപരമല്ലാത്ത നിരവധി പരാമര്‍ശങ്ങള്‍ നടത്തുന്ന ഗ്രന്ഥമാണ് ബൈബിളെന്ന വസ്തുത സുതരാം ബോധ്യമാകും. അതുകൊണ്ടുതന്നെ ബൈബിള്‍ വിവരണങ്ങളുടെ മാത്രം വെളിച്ചത്തില്‍ ഫിര്‍ഔന്‍-ഹാമാന്‍കഥയുടെ ചരിത്രപരത സംശയിക്കുന്നതില്‍ യാതൊരര്‍ത്ഥവുമില്ല. അതിന് മറ്റ് സ്രോതസ്സുകളുടെ പിന്‍ബലമുണ്ടോയെന്ന് പരിശോധിച്ചശേഷം മാത്രമെ ഈആരോപണം എത്രമാത്രം വസ്തുതാപരമാണെന്ന് പറയാനൊക്കൂ. ബൈബിളിലെ ഫറോവായുടെ കഥകളില്‍തന്നെ ചരിത്രപരമായി കൃത്യമല്ലാത്ത പരാമര്‍ശങ്ങള്‍ പലതുമുണ്ട്. ഫറോവയെന്ന നാമം ഉപയോഗിക്കുന്നിടത്തുമുതല്‍ ആരംഭിക്കുന്നു പ്രശ്‌നങ്ങളെന്നതാണ് വാസ്തവം. ബൈബിളില്‍ പലരെയും ഫറോവയെന്ന് അഭിസംബോധന ചെയ്തതായി കാണാന്‍ കഴിയും. അബ്രഹാമിന്റെ കാലത്തുണ്ടായിരുന്ന രാജാവിനെ പഴയ നിയമ പുസ്തകം പരിചയപ്പെടുത്തുന്നത് ഫറോവയെന്നാണ്. ഉല്‍പത്തി പുസ്തകം പന്ത്രണ്ടാം അധ്യായത്തില്‍ പത്തുമുതല്‍ ഇരുപതുവരെ വചനങ്ങളില്‍ അബ്രഹാമിന്റെ കാലത്തെ രാജാവിനെക്കുറിച്ച് ആറ് തവണ ഫറോവയെന്ന് പറഞ്ഞിട്ടുണ്ട്. യോസഫിന്റെ കാലത്തെ രാജാവിനെയും ബൈബിള്‍ ഫറോവയെന്നാണ് വിളിക്കുന്നത്. ഉല്‍പത്തി പുസ്തകത്തിന്റെ നാല്‍പത്, നാല്‍പത്തിയൊന്ന് അധ്യായങ്ങളില്‍ യോസഫിന്റെ കാലത്തെ രാജാവിനെ ഫറോവയെന്ന് അഭിസംബോധന ചെയ്യുന്നതായി കാണാം. യോസഫിന്റെ കാലത്തെ രാജാവിനെ ഉല്‍പത്തി പുസ്തകത്തില്‍ തൊണ്ണൂറ് തവണയാണ് ഫറോവയെന്ന് വിളിച്ചിരിക്കുന്നത്. മോശയുടെ കാലത്തെ ഈജിപ്തിലെ രാജാവിനെക്കുറിച്ച് പുറപ്പാട് പുസ്തകത്തില്‍ നൂറ്റി ഇരുപത്തൊന്ന് വശ്യം ഫറോവയെന്ന് പ്രായോഗിച്ചിട്ടുണ്ട്. ചുരുക്കത്തില്‍ അബ്രഹാമിന്റെയും യോസഫിന്റെയും മോശയുടെയുമെല്ലാം കാലത്ത് ഈജിപ്ത് ഭരിച്ച രാജാക്കന്മാരെ ബൈബിള്‍ ഫറോവയെന്നാണ് വിളിച്ചിരിക്കുന്നത്. ചരിത്രപരമായി നോക്കിയാല്‍ ഈ അഭിസംബോധന തന്നെ അബദ്ധമാണെന്ന് കാണാന്‍ കഴിയും. ഈജിപ്തിന്റെ പൗരാണിക ചരിത്രത്തെ മുപ്പത് രാജവംശങ്ങളുടെ(Dynasty) കാലഘട്ടങ്ങളായി വിഭജിച്ചുകൊണ്ടാണ് പഠിക്കുന്നത്. ക്രിസ്തുവിന് മൂന്ന് നൂറ്റാണ്ടുകള്‍ക്ക് മുമ്പ് ജീവിച്ച ഈജിപ്ഷ്യന്‍ ചരിത്രകാരനായ മാനെത്തോ (Manetho) തയ്യാറാക്കിയ രാജാക്കന്മാരുടെ പട്ടികയെയാണ് മുപ്പത്രാജവംശകാലങ്ങളായി തിരിച്ചിരിക്കുന്നത്. യാതൊരുവിധ രേഖകളും ലഭ്യമല്ലാത്ത 3100 ബി.സിക്കുമുമ്പുള്ള കാലത്തെ ‘രാജവംശങ്ങള്‍ക്ക് മുമ്പുള്ളകാലഘട്ട‘ (Predynastic era) മെന്ന് വിളിക്കുന്നു. രാജവംശങ്ങളുടെ കാലംഇങ്ങനെയാണ് രേഖപ്പെടുത്തപ്പെട്ടിരിക്കുന്നത്. Nicholas Grimal: A History of Ancient Egypt: Blackwell Publishers, Oxford. ഈജിപ്തിലെ രാജാക്കന്മാരെ വിളിക്കാന്‍ ബൈബിള്‍ ഉപയോഗിച്ചിരിക്കുന്ന പേരാണ് ഫറോവയെന്ന് സൂചിപ്പിച്ചുവല്ലോ. പെര്‍-ആ(Per-aa) യെന്ന പദത്തില്‍ നിന്നാണ് ഫറോവയുടെ ഉല്‍പത്തി. മഹാഭവനം (Greathouse) എന്നാണ് ഈ പദത്തിനര്‍ത്ഥം. ഫറോവയെന്ന് ഈജിപ്തിലെ രാജാക്കന്മാര്‍ അഭിസംബോധന ചെയ്യപ്പെട്ടു തുടങ്ങിയത് എന്ന് മുതല്‍ക്കാണ്? ഹാര്‍പേര്‍സ് ബൈബിള്‍ ഡിക്ഷ്ണറി എഴുതുന്നത് കാണുക: ‘പെര്‍’ എന്നും‘ആ‘ യെന്നുമുള്ള രണ്ട് ഈജിപ്ഷ്യന്‍ പദങ്ങളില്‍ നിന്നാണ് ഈ പദ(ഫറോവ)ത്തിന്റെ ഉല്‍പത്തി. ”മഹാഭവന”മെന്നാണ് ഈ ഈജിപ്ഷ്യന്‍ പദസമുച്ചയത്തിന്റെ യഥാര്‍ത്ഥസാരം. ക്രിസ്തുവിന് മുമ്പ് മൂന്നാം സഹസ്രാബ്ദം മുതല്‍ രണ്ടാം സഹസ്രാബ്ദത്തിന്റെ ആദ്യ പകുതിവരെ ഈനാമമുപയോഗിച്ചിരുന്നത് രാജകൊട്ടാരത്തിനായിരുന്നു. പതിനെട്ടാം രാജവംശത്തിലെ തുത്‌മോസ് മൂന്നാമന്റെ (Thutmose III 1504-1450 BC)കാലം മുതല്‍ പെര്‍-ആ രാജാവിനെത്തന്നെ വിളിക്കുന്ന നാമമായിത്തീര്‍ന്നു. ഇരുപത്തിരണ്ടാം രാജവംശത്തിലെ ശോെഷന്‍ക് ഒന്നാമന്റെ (Shoshenq I 945-924 BC) കാലംമുതല്‍ രാജനാമത്തോടൊപ്പം, ബൈബിളില്‍ കാണുന്നതുപോലെ ഫറോവയെന്ന് ചേര്‍ത്ത് വിളിക്കുന്ന സമ്പ്രദായമുണ്ടായി (ഉദാ: ഫറോവാശോഷെന്‍ക്) (Harper’s Bible Dictionary Page 781) ഡോ. ബാബുപോള്‍ തന്റെ ബൈബിള്‍ നിഘണ്ടുവില്‍ പറയുന്നതും ഇതുതന്നെയാണ്: ”ഫറവോ. ഈജിപ്തിലെ രാജാവ്: ‘മഹാഭവനം‘ എന്ന്അര്‍ത്ഥമുള്ള ഒരു പദത്തില്‍നിന്ന് ഉരുത്തിരിഞ്ഞ സ്ഥാനനാമമാണ് ഫറവോ.ക്രി. മു. മൂന്നാം സഹസ്രാബ്ദം മുതല്‍ ഉപയോഗമുണ്ടായിരുന്നെങ്കിലും ക്രി. മു. 1500 വരെ ആ പദം രാജാവിനെ സൂചിപ്പിച്ചിരുന്നില്ല. കൊട്ടാരം, ഡര്‍ബാര്‍അഥവാ രാജസദസ്സ് എന്നായിരുന്നു ആദ്യത്തെ അര്‍ത്ഥം. വ്യക്തിനാമത്തോടൊപ്പം ഫറവോ (ഉദാ: ഫറവോ നെക്കോ, ഫറവോഹോഫ്‌റ) എന്ന് ചേര്‍ക്കുന്ന രീതി ക്രി. മു 945 മുതല്‍ തുടങ്ങി” (ഡോ. ഡി.ബാബുപോള്‍: വേദശ ബ്ദരത്‌നാകരം പുറം 445). പുതിയ രാജ്യത്വ (New Kindom) കാലത്ത് പതിനെട്ടാം രാജവംശത്തിന്റെ ഭരണകാലത്താണ് ഈജിപ്തിലെ രാജാക്കന്മാരെ ഫറോവയെന്ന് അഭിസംബോധന ചെയ്യാനാരംഭിച്ചതെന്ന് എന്‍സൈക്ലോപീഡിയ ബ്രിട്ടാണിക്കയും വ്യക്തമാക്കുന്നുണ്ട് (Encyclopaedia-Electronic Edition -Pharoah) ദി ഫങ്ക് ആന്റ് വാഗ്‌നല്‍സ് എന്‍സൈക്ലോപീഡിയ (The Funk and Wagnalls New Encyclopaedia Infopaedia 2.0-Electronic Edition-”Pharoah) നെല്‍സണ്‍സ് ഇല്ലസ്‌ട്രേറ്റഡ് ബൈബിള്‍ ഡിക്ഷ്ണറി (Sr. Herbert Lockyer (General Editor): Nelson’s Illustrated Bible Dictionary (1986) “Pharoah”) തുടങ്ങിയ ആധികാരിക ഗ്രന്ഥങ്ങളെല്ലാം ക്രിസ്തുവിന് മുമ്പ് പതിനഞ്ചാം നൂറ്റാണ്ടിന് മുമ്പ് ഫറോവയെന്ന പദപ്രയോഗം രാജാക്കന്മാരെ അഭിസംബോധന ചെയ്തുകൊണ്ട് നിലനിന്നിരുന്നില്ലെന്ന വസ്തുത അംഗീകരിക്കുന്നു. അബ്രഹാമിന്റെ കാലത്തെ ഇൗജിപ്തിലെ രാജാവിനെ ബൈബിള്‍ ഫറോവയെന്ന് വിളിക്കുന്നുണ്ടെന്ന് പറഞ്ഞല്ലോ. ഉല്‍പത്തി പുസ്തകത്തില്‍ അബ്രഹാമിന്റെ കഥ വിശദമായി വിവരിക്കുന്നുണ്ട് (11:26-25:18). എന്നാല്‍ എന്നാണ് അദ്ദേഹം ജീവിച്ചിരുന്നതെന്ന് വ്യക്തമാക്കുന്ന കൃത്യമായ രേഖകളൊന്നുമില്ല. ദി അക്കാദമിക് അമേരിക്കന്‍ എന്‍സൈക്ലോപീഡിയ ബി.സി. മൂന്നാം സഹസ്രാബ്ദത്തിന്റെ അന്ത്യത്തിലോ രണ്ടാം സഹസ്രാബ്ദത്തിന്റെ തുടക്കത്തിലോ ആയിരിക്കണം അബ്രഹാം ജീവിച്ചതെന്നാണ് അഭിപ്രായപ്പെടുന്നത് (Abraham) ബി.സി. 1850 കളിലായിരിക്കണം അബ്രഹാമിന്റെ കാലഘട്ടമെന്നാണ് ‘ഡിക്ഷ്ണറിഓഫ് പ്രോപര്‍ നെയിംസ് ആന്റ് പ്ലെയി സസ് ഇന്‍ ദ ബൈബിളി‘ന്റെ പക്ഷം(O. odelain & R. Seguineau: Dictionary of Proper Names and Places in the Bible. (London 1991) Page 7 “Abraham”) ദി ലയണ്‍ ഹാന്റ് ബുക്ക് റ്റു ദ ബൈബിളിനും ഏകദേശം ഇതേ അഭിപ്രായം തന്നെയാണുള്ളത്. (David Alexander and Pat Alexander (Ed): The Lion Hand Book to the Bible, (1973-Oxford) Page 152, 153) ആധുനിക ഗവേഷണങ്ങളുടെ വെളിച്ചത്തില്‍ അബ്രഹാം ബി.സി. 2300നടുത്ത് ജീവിച്ചുവെന്നാണ് മനസ്സിലാക്കാന്‍ കഴിയുന്നതെന്ന് ‘ദി ഹച്ചിന്‍സണ്‍സ് ന്യൂസെന്റുറി എന്‍സൈക്ലോപീഡിയ‘ പറയുന്നു. (The Hutchinson New Century Encyclopaedia’ (1995-Electronic Edition) “Abraham”). ബി.സി. 2000ത്തിനും 2300നു മിടയിലായിരിക്കണം അബ്രഹാമിന്റെ കാലമെന്നാണ് പ്രസിദ്ധമായ കോളിന്‍ ജെം ഡിക്ഷ്ണറി ഓഫ് ദി ബൈബിളിന്റെയും പക്ഷം (Rev: James L. Dow, Collins Gem Dictionary of the Bible (1985-Britain) “Abraham” Page 18)ബാബിലോണിലെ ഹമുറബി (ക്രി:മു: 1728-1686)യുടെ സമകാലീനനായിരുന്നു അബ്രഹാമെന്ന അഭിപ്രായവും കി: മു: 1500ന് അടുപ്പിച്ചാണ്അബ്രഹാമിന്റെ കാലമെന്ന മക്കെന്‍സിയുടെ അഭിപ്രായവും ഡോ. ബാബുപോള്‍ തന്റെ വേദശബ്ദ രത്‌നാകരത്തില്‍ ഉദ്ധരിക്കുന്നുണ്ട് (പുറം 50). എന്നാല്‍ ആധുനിക ഉല്‍ഖനന ഗവേഷണങ്ങള്‍ അബ്രഹാമിന്റെ കാലത്തെപുറകോട്ട് കൊണ്ടുപോകുന്നുവെന്നും ബി.സി. 2300നോടടുത്തായിരിക്കണം അദ്ദേഹം ജീവിച്ചിരുന്നതെന്നാണ് ഉല്‍ഖനന രേഖകള്‍ നല്‍കുന്ന തെളിവുകള്‍ വ്യക്തമാക്കുന്നതെന്നും നിരവധി ഗവേഷകന്മാര്‍ വ്യക്തമാക്കിയിട്ടുണ്ട്. (Chain Bermant and Michael Weitzman: Ebla: A Revelation in Archeology (Great Britain 1979) Page 6-184 & Howard La Fay: “Ebla: Splender of an Unknown Empire” National Geographic Magazine, December 1978) ചുരുക്കത്തില്‍, ക്രിസ്തുവിന് 1500വര്‍ഷങ്ങള്‍ക്ക് മുമ്പ് ജീവിച്ചിരുന്ന വ്യക്തിയാണ് അബ്രഹാം എന്ന പുരാതനഗ്രന്ഥകര്‍ത്താക്കളുടെ അഭിപ്രായത്തിന് വിരുദ്ധമായി ബി.സി. 2300നടുത്ത കാലത്തായിരിക്കണം അദ്ദേഹം ജീവിച്ചിരുന്നതെന്ന നിഗമനത്തിലെത്താനാണ് ആധുനിക ഉല്‍ഖനന ഗവേഷണങ്ങള്‍ നല്‍കുന്ന വിവരങ്ങള്‍ നമ്മെ പ്രേരിപ്പിക്കുന്നത്. ഈജിപ്്തിലെ രാജവംശ ചരിത്രവുമായി ബന്ധപ്പെടുത്തി പരിശോധിച്ചാല്‍ ആറുമുതല്‍ പന്ത്രണ്ടുവരെ രാജവംശങ്ങളുടെ കാലങ്ങള്‍ക്കിടയിലായിരിക്കണം അബ്രഹാമിന്റെ ജീവിതെമന്നാണ് മനസ്സിലാകുന്നത്. ആധുനിക ഗവേഷകന്മാരുടെ അഭിപ്രായം പരിഗണിച്ചാല്‍ ആറാം രാജവംശത്തിന്റെ കാലത്തായിരിക്കണം (ബി.സി. 2300) അബ്രഹാം ജീവിച്ചിരുന്നത്. അങ്ങനെയെങ്കില്‍ പുരാതന രാജത്വ (Old Kingdom)കാലത്തായിരിക്കും അദ്ദേഹത്തിന്റെ ജീവിതം. ഇനി പന്ത്രണ്ടാം രാജവംശത്തിന്റെ കാലത്താണ് അബ്രഹാം ജീവി ച്ചിരുന്നതെന്ന പഴയ അഭിപ്രായം പരിഗണിച്ചാലും മധ്യരാജത്വ (Middle Kingdom) കാലത്താണ് അദ്ദേഹം ജീവിച്ചിരുന്നതെന്ന് മാത്രമെ വരികയുള്ളൂ. ഈജിപ്തിലെ രാജാക്കന്മാരെ ഫറോവയെന്ന് വിളിക്കാനാരംഭിച്ചത് പുതിയ രാജത്വകാലത്ത് പതിനെട്ടാം രാജവംശത്തിന്റെ സമയത്താണെന്ന് മുമ്പ് വ്യക്തമാക്കിയിട്ടുണ്ട്. പിന്നെയെങ്ങനെയാണ് അബ്രഹാമിന്റെ കാലത്തെ ഈജിപ്തിലെ രാജാവിനെ ഫറോവയെന്ന് വിളിക്കുക? അബ്രഹാമിന് നൂറ്റാണ്ടുകള്‍ കഴിഞ്ഞ് നിലവില്‍വന്ന ഫറോവയെന്ന അഭിസംബോധന രീതി അദ്ദേഹത്തിന്റെ കാലത്ത് എങ്ങനെയാണ് ഉപയോഗിക്കപ്പെടുക? ഫറോവമാരുടെ കഥ പറയുമ്പോള്‍ ബൈബിള്‍ സൂക്ഷ്മത പുലര്‍ത്തുന്നില്ലെന്ന വസ്തുതയാണ് നമുക്ക് ഇവിടെ ബോധ്യപ്പെടുന്നത്. യോസഫിന്റെ ചരിത്രവിവരണത്തിലും ഇതേ പ്രശ്‌നം നിലനില്‍ക്കുന്നുണ്ട്. യോസഫിന്റെ ഈജിപ്തിലേക്കുള്ള വരവ് എന്നായിരുന്നു? ബൈബിളിലെ സംഭവ വിവരണങ്ങളുടെ വെളിച്ചത്തില്‍ ഹിക് സോസ് വംശത്തിന്റെ ഭരണകാലത്തായിരിക്കണം ഇത് നടന്നതെന്നാണ് പണ്ഡിതന്മാര്‍ അഭിപ്രായപ്പെടുന്നത്. ആരാണ് ഹിക്‌സോസ് വംശം? ഡോ. ഡി.ബാബുപോള്‍ എഴുതുന്നു: ‘ക്രി. മു. 1720-1550 കാലത്ത് ഈജിപ്ത് ഭരിച്ചിരുന്നത് ഹിക്‌സോസ് വംശത്തിലെ ഫറോവമാരായിരുന്നു. ഇവര്‍ ഏഷ്യാ വന്‍കരയില്‍ നിന്നു വന്ന് കാലക്രമത്തില്‍ ഭരണാധികാരികളായിത്തീര്‍ന്നവരാണ്. ആദ്യകാലത്ത് നിലവിലുള്ള ഭരണയന്ത്രം തന്നെ ഉപയോഗപ്പെടുത്തിയെങ്കിലും പിന്നെപ്പിന്നെ കൂടുതലായി സെമിറ്റിക് വംശജരെ ഉന്നത സ്ഥാനങ്ങളില്‍ നിയമിക്കാന്‍ തുടങ്ങി” (വേദശബ്ദരത്‌നാകരം പുറം 272). ഹിക് സോസ് വംശക്കാരുടെ ഭരണകാലത്താണ് യോസഫിന്റെ ഈജിപ്തിലേക്കുള്ള പ്രവേശനമുണ്ടായതെന്ന ഡോ. ഡി.ബാബുപോളിന്റെ അഭിപ്രായം തന്നെയാണ് നെല്‍സണ്‍സ് ഇല്ലസ്‌ട്രേറ്റഡ് ബൈബിള്‍ ഡിക്ഷ്ണറിയും (Nelsons Illustrated Bible Dictionary “Egypt” Page 324) വില്യം നീല്‍സ് വണ്‍ വാള്യം ബൈബിള്‍ കമെന്ററിയും (Willian Neil’s One Volume Bible Commentary “Genesis: The Story of Joseph” Page 63) ദി ന്യൂ ജെംറാം ബിബ്ലിക്കല്‍ കമന്ററിയുമെല്ലാം (The New Jerome Biblical Commentary Page 37) മുന്നോട്ടുവെക്കുന്നത്. ഹിക്‌സോസ് വംശജരുടെ ഭരണകാലത്താണ് യോസഫിന്റെ ഈജിപ്ത് ആഗമനമുണ്ടായതെന്ന കാര്യത്തില്‍ പ്രമുഖരായ ബൈബിള്‍ പണ്ഡിതന്മാര്‍ക്കിടയില്‍ അഭിപ്രായാന്തരമൊന്നുമില്ലെന്നര്‍ത്ഥം. ഹിക്‌സോസ് വംശം ഏതുകാലത്താണ് ഈജിപ്ത് ഭരിച്ചിരുന്നതെന്ന് നടേസൂചിപ്പിച്ചുവല്ലോ. ക്രി.മു. 1720-1550ആണ് ഹിക്‌സോസ് വംശത്തിന്റെ കാലം. ഈജിപ്തിലെ മധ്യരാജത്വകാല (Middle Kingdom) മാണിത്. ഇക്കാലത്താണ് ഈജിപ്തിലേക്കുള്ള യോസഫിന്റെ ആഗമനമുണ്ടായത്. പതിനഞ്ചാം രാജവംശത്തിന്റെ കാലത്തായിരിക്കണം ഇത് സംഭവിച്ചിരിക്കുകയെന്നാണ് പണ്ഡിതാഭിപ്രായം. ഫറോവയെന്ന അഭിസംബോധനാ രീതി പുതിയ രാജവംശകാലത്താണല്ലോ ആരംഭിച്ചത്. യോസഫിന്റെ കാലത്തെ രാജാവിനെ ഫറോവയെന്ന് ബൈബിള്‍ വിളിച്ചിരിക്കുന്നതും അബദ്ധമാണെന്നര്‍ത്ഥം. അബ്രഹാമിന്റെയും യോസഫിന്റെയും കാലത്തെ ഈജിപ്ഷ്യന്‍ ചക്രവര്‍ത്തിമാരെ ഫറോവമാരെന്ന് ബൈബിള്‍ അഭിസംബോധന ചെയ്യുവാനുള്ള കാരണമെന്താണ്? മോശെ പ്രവാചകന്ന് ശേഷം നൂറ്റാണ്ടുകള്‍ കഴിഞ്ഞാണ് ബൈബിള്‍ പഴയ നിയമത്തിലെ പുസ്തകങ്ങളില്‍ മിക്കതും എഴുതപ്പെടുന്നത്. വിവിധ ഗ്രന്ഥകാരന്മാരുടെ രചനകളാണല്ലോ അവയിലുള്ളത്. അബ്രഹാമിന്റെയും യോസഫിന്റെയുമെല്ലാം കഥ ബൈബിളില്‍ എഴുതപ്പെട്ട കാലത്ത് ഈജിപ്തിലെ രാജാക്കന്മാരെ വിളിച്ചിരുന്നത് ഫറോവമാരെന്നായിരുന്നിരിക്കണം. ഈ പ്രയോഗത്തില്‍ നിന്ന് ഈജിപ്തില്‍ എക്കാലത്തും രാജാക്കന്മാരെ ഫറോവമാരെന്നായിരിക്കണം വിളിച്ചതെന്ന നിഗമനത്തിലെത്തിയ ഗ്രന്ഥകാരന്മാരുടെ സൂക്ഷ്മതക്കുറവുകൊണ്ട് സംഭവിച്ച അബദ്ധമാണിത്. ഈജിപ്തിന്റെ പുരാതന ചരിത്രമോ ഫറോവയെന്ന പദത്തിന്റെ വ്യുല്‍പത്തിയോ അറിയാത്ത ഗ്രന്ഥകാരന്മാര്‍ക്ക് സംഭവിച്ച സ്വാഭാവികമായ ഒരു കൈപ്പിഴ മാത്രമാണിത്. ഈജിപ്ഷ്യന്‍ പുരാവിജ്ഞാനീയത്തിന്റെ വളര്‍ച്ചയാണ് ഇതൊരു അബദ്ധമാണെന്ന് നമുക്ക് വ്യക്തമാക്കിത്തരികയും ബൈബിള്‍ രചയിതാക്കള്‍ക്ക് ഇതേപോലെയുള്ള നിരവധി അബദ്ധങ്ങള്‍ സംഭവിച്ചിട്ടുണ്ടെന്ന വസ്തുത വെളിച്ചത്തുകൊണ്ടുവരികയും ചെയ്തത്. ഫറോവയുടെ ചരിത്രവിവരണവുമായി ബന്ധപ്പെട്ട് ബൈബിള്‍ സൂക്ഷ്മത പാലിക്കുന്നില്ലെന്ന വസ്തുത ആ പേര് ഉപയോഗിച്ചതിലുള്ള അബദ്ധം തന്നെസുതരാം വ്യക്തമാക്കുന്നുണ്ട്. ഖുര്‍ആനില്‍ വിവരിച്ചിട്ടുള്ള ഫറോവാ–ഹാമാന്‍ കഥ ബൈബിളിലില്ലെന്ന ഏക കാരണത്താല്‍ നിഷേധിക്കുന്നതിന് യാതൊരുവിധ ന്യായീകരണവുമില്ലെന്ന് ഇതില്‍ നിന്നുതന്നെ ശരിക്കുംവ്യക്തമാണ്. ഫറോവമാരെക്കുറിച്ച് ബൈബിള്‍ കൃത്യവും സൂക്ഷ്മവും കളങ്കരഹിതവുമായ ചരിത്രം പ്രദാനം ചെയ്യുന്നുവെങ്കില്‍ മാത്രമെ അതിന്റെ മാത്രം അടിസ്ഥാനത്തില്‍ ഖുര്‍ആനിലെ ഫറോവാ–ഹാമാന്‍ കഥയെവിലയിരുത്താവൂ. ഫറോവയെന്ന പദത്തിന്റെ ഉപയോഗത്തില്‍പോലും കൃത്യതയില്ലാത്ത ബൈബിളിനെ ഇക്കാര്യത്തില്‍ ഒരു മാനദണ്ഡമാക്കുവാനേ പറ്റില്ല. മോശയ്ക്കുശേഷം നൂറ്റാണ്ടുകള്‍ കഴിഞ്ഞ് എഴുതപ്പെട്ടതിനാല്‍ ഫറോവയെന്ന നാമം പ്രയോഗിക്കുന്നതില്‍ ഇത്തരമൊരു അബദ്ധമുണ്ടായെങ്കില്‍ ഇതേ കാരണത്താല്‍ ഫറോവയുടെ മന്ത്രിയുമായി ബന്ധപ്പെട്ട ഒരു കഥ വിട്ടുപോയിരിക്കുവാനുള്ള സാധ്യതയും നിഷേധിക്കുവാനാവില്ലല്ലോ. ഖുര്‍ആനാകട്ടെ, അതിലെ മറ്റെല്ലാ വിഷയങ്ങളിലുമെന്നപോലെ ഫറോവയുടെ നാമപ്രയോഗത്തിലും സംഭവവിവരണങ്ങളിലുമെല്ലാം തികഞ്ഞ സൂക്ഷ്മത പുലര്‍ത്തിക്കൊണ്ട് അതിന്റെ ദൈവികത വെളിപ്പെടുത്തുന്നത് കാണാന്‍ കഴിയും. അത്ഭുതകരമാണ് ഇക്കാര്യത്തില്‍ ഖുര്‍ആന്‍ പുലര്‍ത്തുന്ന സൂക്ഷ്മതയും കൃത്യതയുമെന്നതാണ് വസ്തുത. ബൈബിളിലെ ഫറോവയെന്ന ശബ്ദത്തിന് തത്തുല്യമാണ് ഖുര്‍ആനിലെ ഫിര്‍ഔന്‍. അബ്രഹാമിന്റെയോ യോസഫിന്റെയോ കാലത്തെ രാജാവിനെക്കുറിച്ച് ഖുര്‍ആനിലൊരിടത്തും ഫിര്‍ഔന്‍ എന്ന് പ്രയോഗിക്കുന്നില്ല. യൂസുഫ് നബി (അ)യുടെ കാലത്ത് ഈജിപ്ത് ഭരിച്ചിരുന്നയാളെ ഖുര്‍ആന്‍ വിളിക്കുന്നത് ‘മലിക്ക്‘ എന്നാണ് (ഉദാ: 12:43-54, 12:72-76). ‘മലിക്ക്‘ എന്നാല്‍ രാജാവ് എന്നര്‍ത്ഥം. യോസഫിന്റെ കാലത്തെ ഭരണാധികാരിയെ കേവലം രാജാവ് എന്ന് വിളിക്കുന്ന ഖുര്‍ആന്‍ മോശയുടെ കാലത്തെ ചക്രവര്‍ത്തിയെ ഫിര്‍ഔന്‍ എന്നുതന്നെ വിളിക്കുന്നുണ്ട്. മോശയുടെ ചരിത്രവുമായി ബന്ധപ്പെടുത്തി ഫിര്‍ഔനിന്റെ നാമം നിരവധിതവണ ഖുര്‍ആനില്‍ കാണാം. (ഉദാ: 7:104-137, 8:52-54, 10:75-90, 20:24-78, 26:10-66, 28:3-42, 24-46, 43:46-85, 51:38-40, 79:17-25). പുതിയ രാജത്വകാലത്താണ് ഫറോവയെന്ന അഭിസംബോധനാ രീതിയുണ്ടായെതന്ന് നടേ സൂചിപ്പിച്ചുവല്ലോ. ബി.സി. 1552- 1069 ആണ്പുതിയ രാജത്വകാലം. ക്രിസ്തുവിന് മുമ്പ് പതിനാലാം നൂറ്റാണ്ടിന്റെ മധ്യത്തില്‍ ബി.സി. 1352നും 1348നും ഇടക്കായിരിക്കണം രാജാക്കന്മാരെ ഫറോവയെന്ന് അഭിസംബോധന ചെയ്യുന്ന രീതിയുടെ തുടക്കമെന്നാണ് പണ്ഡിതാഭിപ്രായം. ബി.സി. പതിമൂന്നാം നൂറ്റാണ്ടിന്റെ ആദ്യപകുതിയിലാണ് മോശ ജനിച്ചതെന്നാണ് കരുതപ്പെടുന്നത്. ഫറോവയെന്ന അഭിസംബോധനാ രീതി പ്രചുരപ്രചാരം സിദ്ധിച്ച കാലത്താണ് മോശയുടെ ജനനവും ദൗത്യവുമെല്ലാം ഉണ്ടായതെന്നര്‍ത്ഥം. മോശയുടെ പ്രബോധനകാലത്തെ രാജാവിനെ ഫിര്‍ഔന്‍ എന്ന് സംബോധന ചെയ്തിരുന്ന ഖുര്‍ആനിക രീതി പൂര്‍ണ മായും ശരിയായ ചരിത്രവും പുരാവസ്തുശാസ്ത്രവുമായി എല്ലാ നിലയ്ക്കും യോജിക്കുന്നതാണെന്ന വസ്തുത അതിന്റെ ദൈവികതയ്ക്കുള്ള തെളിവ് കൂടിയാണ്. ഫറോവയെക്കുറിച്ച പരാമര്‍ശങ്ങളുടെ കാര്യത്തില്‍ ഖുര്‍ആന്‍ കൃത്യതയും സൂക്ഷ്മതയും പുലര്‍ത്തുന്നുവെന്നും ബൈബിള്‍ പല അബദ്ധങ്ങളും വരുത്തുന്നുണ്ടെന്നുമുള്ള വസ്തുതകള്‍ നാം മനസ്സിലാക്കി. ചരിത്രകഥനങ്ങളുടെ വിഷയത്തില്‍ ഖുര്‍ആന്‍ വെച്ചുപുലര്‍ത്തുന്ന കൃത്യതയ്ക്ക് കാരണം അതിന്റെ ദൈവികതയാണെന്ന വസ്തുത അല്‍പം ചിന്തിച്ചാല്‍ തന്നെ ബോധ്യപ്പെടും. ബൈബിളല്ലാത്ത മറ്റൊരു സ്രോതസ്സും പുരാതന ചരിത്രത്തിന്റെ കാര്യത്തില്‍ നിലവിലില്ലാതിരുന്ന കാലത്ത് അവതരിപ്പിക്കപ്പെട്ട ഖുര്‍ആനിക വചനങ്ങള്‍ അബദ്ധങ്ങളില്‍ നിന്ന് പൂര്‍ണമായി മുക്തമാണെന്ന വസ്തുത അതിന്റെ ദൈവികതയല്ലാതെ മറ്റെന്താണ് വ്യക്തമാക്കുന്നത്? ബൈബിളില്‍ അബദ്ധങ്ങള്‍ പരാമര്‍ശിക്കപ്പെട്ട അതേ ചരിത്രം തന്നെ കൈകാര്യം ചെയ്യുമ്പോഴും ഖുര്‍ആനില്‍ അതേ അബദ്ധം ആവര്‍ത്തിക്കപ്പെടുന്നില്ലെന്ന വസ്തുത പൗരാണിക സംഭവങ്ങളെക്കുറിച്ച് സൂക്ഷ്മമായി അറിയുന്നവനില്‍ നിന്ന് അവതരിപ്പിക്കപ്പെട്ടതാണ് അതെന്ന് വ്യക്തമാക്കുന്നു. ഫറോവയുടെ ചരിത്ര കഥനത്തിന്റെ കാര്യത്തില്‍ പല സ്ഥലത്തും അബദ്ധങ്ങള്‍ പറ്റിയിട്ടുള്ള ബൈബിളില്‍ പരാമര്‍ശിക്കുന്നില്ലെന്ന കാരണത്താല്‍ പ്രസ്തുത കഥനത്തില്‍ പൂര്‍ണമായി കൃത്യത പുലര്‍ത്തുന്ന ഗ്രന്ഥമായ ഖുര്‍ആനില്‍ പറഞ്ഞിരിക്കുന്ന ഫറോവാ–ഹാമാന്‍ കഥ മുഹമ്മദ് നബിക്കുണ്ടായ ആശയക്കുഴപ്പത്തില്‍ നിന്നുണ്ടായ കെട്ടുകഥയാണെന്ന് വാദിക്കുന്നത് ഏത് മാനദണ്ഡത്തിന്റെ അടിസ്ഥാനത്തില്‍ നോക്കിയാലും വിവരക്കേടാണ്. രണ്ട്) എസ്‌തേറിന്റെ പുസ്തകത്തില്‍ പറഞ്ഞിരിക്കുന്ന അഹശ്വറോശ് രാജാവിന്റെ കീഴിലുള്ള ഹാമാന്‍ എന്ന പ്രഭുവിന്റെ കഥയും ഫറോവയുടെ കഥയും കൂടിക്കുഴഞ്ഞതു മൂലമാണ് ഖുര്‍ആനില്‍ ഫറോവാ–ഹാമാന്‍ കഥയുണ്ടായത്. ഖുര്‍ആനില്‍ ചരിത്രാബദ്ധമുണ്ടെന്ന് വരുത്തുവാന്‍വേണ്ടി വിമര്‍ശകന്മാര്‍ ഇവിടെ അവലംബമാക്കിയിരിക്കുന്നത് ബൈബിള്‍ പഴയനിയമത്തിലെ എസ്‌തേറിന്റെ പുസ്തകത്തെയാണ്. എസ്‌തേറിന്റെ പുസ്തകം മൂന്ന് മുതല്‍ ഏഴുവരെ അധ്യായങ്ങളില്‍ അഹശ്വറോശ് രാജാവിന്റെ പ്രഭുവായ ഹാമാനെക്കുറിച്ച് പറയുന്നുണ്ടെന്നത് ശരിയാണ്. എസ്‌തേറിന്റെ പുസ്തകത്തിലെ ഹാമാന്‍ ചരിത്രപുരുഷനും ഖുര്‍ആനിലെ ഹാമാന്‍ കെട്ടുകഥയുമാണെന്ന വാദത്തിന് എന്ത് അടിത്തറയാണുള്ളത്? ഖുര്‍ആനിനേക്കാള്‍ ആധികാരികമാണ് എസ്‌തേറിന്റെ പുസ്തകമെന്ന് തെളിയിക്കാന്‍ കഴിഞ്ഞാല്‍ ഈ വാദത്തിന് അല്‍പം പ്രസക്തിയുണ്ടെന്നെങ്കിലും പറയാമായിരുന്നു. എന്നാല്‍ എസ്‌തേറിന്റെ പുസ്തകത്തിന്റെ അവസ്ഥയെന്താണ്? യഹൂദരും ക്രൈസ്തവരുമായ പലപണ്ഡിതന്മാരും ഈ പുസ്തകം ആധികാരികമല്ലെന്ന് പ്രഖ്യാപിക്കുകയും വേദ പുസ്തകത്തിന്റെ ഭാഗമല്ലെന്ന് വാദിക്കുകയും ചെയ്യുന്നുണ്ടെന്നതാണ് വാസ്തവം. റവ. എ.സി. ക്ലെയ്റ്റണ്‍ എഴുതുന്നത് കാണുക: ”യഹൂദന്മാരിലും ക്രിസ്ത്യാനികളിലും ചിലര്‍ ഈ പുസ്തകത്തില്‍ ആദ്ധ്യാത്മിക ഉപദേശങ്ങള്‍ കുറവായിരിക്കുന്നുവെന്നും ഇതില്‍ യഹോവ എന്നോ ദൈവം എന്നോ ഉള്ളനാമം ഒരു പ്രാവശ്യംപോലും കാണുന്നില്ലെന്നുമുള്ള കാരണത്താല്‍ ഇത് ഒരു കഥയാണെന്ന് വിചാരിച്ച് ഇതിനെ വേദപുസ്തകത്തില്‍ ഉള്‍പ്പെടുത്താതിരിക്കുന്നു. എന്നാല്‍ മിക്കവാറും യഹൂദന്മാരും ക്രിസ്ത്യാനികളും അംഗീകരിച്ചിട്ടുണ്ട്” (ബൈബിള്‍ നിഘണ്ടു, പേജ് 83). ഇക്കാര്യം അല്‍പംകൂടി വിശദമായി ഡോ. ഡി. ബാബുപോള്‍ തന്റെ വേദശബ്ദ രത്‌നാകരത്തില്‍ വിവരിക്കുന്നുണ്ട്. ”എസ്‌തേറിന്റെ കഥ ബൈബിളിള്‍ ഉള്‍പ്പെടുത്തുന്നത് ദീര്‍ഘകാലം തര്‍ക്ക വിഷയമായിരുന്നു. ക്രി.പി.90ല്‍ ജാംനിയയില്‍ കൂടിയ സുനഹദോസാണ് യഹൂദന്മാരുടെ വേദഗ്രന്ഥത്തില്‍ –ക്രിസ്ത്യാനികള്‍ക്ക് പഴയ നിയമം– ഈ കഥയ്ക്ക് സ്ഥാനംഅനുവദിച്ചത്. ആദിമ ക്രൈസ്തവരുടെ ഇടയില്‍ എസ്‌തേറിന്റെ സ്വീകാര്യത തര്‍ക്കവിഷയമായി തുടര്‍ന്നു. 396ല്‍ ചേര്‍ന്ന കാര്‍ത്തേജ് സുനഹദോസാണ് ഇത് ബൈബിളില്‍ ഉള്‍പ്പെടുത്താന്‍ നിശ്ചയിച്ചത്. 367ല്‍ അത്താനാഷ്യോസ് തയ്യാറാക്കിയ പട്ടികയില്‍ ഡ്യൂട്രോ കാനോനിക്കല്‍ എന്നായിരുന്നു സ്ഥാനം. മെലിറ്റോ, ടിയോഡോര്‍, അംഫിലോക്കിയൂസ്, നാസിയാന്‍സിലെ ഗ്രിഗോറിയോസ് ഇവരുടെയൊക്കെ അഭിപ്രായവും എസ്‌തേറിന്റെ കാനോനികതയ്ക്ക് അനുകൂലമായിരുന്നില്ല. ആറാം നൂറ്റാണ്ടില്‍ ഇത് തര്‍ക്കവിഷയമായിരുന്നുവെന്ന് ജൂനിനിയോസും രേഖപ്പെടുത്തിയിട്ടുണ്ട്. യഹൂദ പണ്ഡിതന്മാരുടെ എതിര്‍പ്പിന് കാരണം ഗ്രന്ഥത്തിന്റെ മതനിരപേക്ഷതയും വൈദേശിക സ്വഭാവവുമായിരുന്നു. 4:14,16 തുടങ്ങി ചുരുക്കം പരാമര്‍ശങ്ങള്‍ ഒഴിച്ചാല്‍ ദൈവം ഈ കൃതിയില്‍ ദൃശ്യനേയല്ല. പേര്‍ഷ്യന്‍ രാജാവിനെക്കുറിച്ച് 187 പരാമര്‍ശങ്ങളുണ്ട്. പൂരിം തിരുനാള്‍, മോശയുടെ ന്യായപ്രമാണത്തില്‍ ഇല്ലായിരുന്നു എന്നതിനാല്‍ 30 പ്രവാചകന്മാര്‍ ഉള്‍പ്പെടെ 85 മൂപ്പന്മാര്‍ ആ തിരുനാളില്‍ വായിക്കേണ്ടിയിരുന്ന ഈ കൃതിയെ അംഗീകരിക്കാന്‍ വിസമ്മതിച്ചുവെന്ന് മറ്റൊരിടത്തും കാണുന്നു” (വേദശബ്ദരത്‌നാകരം: പേജ് 156). കത്തോലിക്കരുടെ ബൈബിള്‍ വിജ്ഞാനകോശം പറയുന്നതും ഇതുതന്നെ: ”ബി.സി. രണ്ടാം നൂറ്റാണ്ടു മുതല്‍ എ.ഡി. ഒന്നാം നൂറ്റാണ്ടുവരെയുള്ള കാലഘട്ടത്തില്‍ ജീവിച്ചിരുന്ന ഖംമ്രാനിലെ എസീം സമൂഹം എസ്‌തേറിനെ ഹിബ്രുകാനനില്‍ ഉള്‍പ്പെടുത്തിയില്ല. പൂരിംതിരുനാള്‍ ദൈവസ്ഥാപിതമായതല്ലാത്തതിനാല്‍ യഹൂദ തിരുനാളുകളില്‍ ഒന്നായി അതിനെ അവര്‍ കണ്ടില്ല. അതുകൊണ്ടാവാം ഈ ഗ്രന്ഥത്തിന് അവര്‍ വലിയ പ്രാധാന്യം കൊടുക്കാതിരുന്നത്……….. ആദിമകാലങ്ങളില്‍ എസ്‌തേറിനകാനോനികം അല്ലെങ്കില്‍ പ്രാമാണിക ഗ്രന്ഥമായി ക്രിസ്ത്യാനികളും കണ്ടിരുന്നില്ല. ഇതില്‍ വെളിവാകുന്ന കടുത്ത വര്‍ഗസ്‌നേഹവും സ്വജനപക്ഷപാതവും അതേസമയം വിജാതീയരോടുള്ള അസഹിഷ്ണുതയുമാകാം ഇതിന് കാരണം. പൗരസ്ത്യസഭകളില്‍ പ്രത്യേകിച്ച് സിറിയ, അനാതോളിയ തുടങ്ങിയ പ്രദേശങ്ങളിലെ സഭകള്‍ ഇതിന് കാനോനികത്വം കല്‍പിച്ചിരുന്നില്ല. സഭാ പിതാക്കന്മാരായ സാര്‍ദിസിലെ മെലിറ്റോ, കാപ്പതോച്ചിയായിലെ ഗ്രിഗറി നസിയാന്‍സണ്‍ (329-390), മെപ്‌സുവെസ്തയിലെ തിയഡോര്‍ (350?-420) തുടങ്ങിയവരൊക്കെ ഈ ഗ്രന്ഥത്തെ കാനോകിക ഗ്രന്ഥങ്ങളുടെ കൂടെ ഉള്‍പ്പെടുത്തിയിരുന്നില്ല. അത്തനേഷ്യസ് (295-373) ഇതിന്റെ കാനോനികത നിരസിച്ചുവെങ്കിലും യൂദിത്ത്, തോബിത്ത് തുടങ്ങിയ ഉത്തര കാനോനിക ഗ്രന്ഥങ്ങളുടെ കൂടെ ഇതിനെ ഉള്‍പ്പെടുത്തി. ഒറിജന്‍ (185?-254), എപ്പിഫാനോസ് (314-403), ജറുസലേമിലെ സിറില്‍ (386) തുടങ്ങിയ പിതാക്കന്മാര്‍ എസ്‌തേറിനെ കാനോനിക ഗ്രന്ഥമായി കണക്കാക്കി. ട്രുള്ളോ സിനഡില്‍ (എ.ഡി. 692) ഇതിനെ പ്രാമാണിക ഗ്രന്ഥമായി അംഗീകരിച്ചു” (ബൈബിള്‍ വിജ്ഞാനകോശം പേജ്172). എസ്‌തേറിന്റെ പുസ്തകം തീരെ പ്രാമാണികമല്ലെന്നും ചരിത്രരേഖയായി എടുക്കാവുന്നതല്ലെന്നും എന്‍സൈക്ലോപീഡിയ ബ്രിട്ടാണിക്കയും വ്യക്തമാക്കുന്നുണ്ട്. (“Encyclopaedia Brittannica “Esthay”) എസ്‌തേറിന്റെ പുസ്തകത്തിലെ കഥകളെല്ലാം കേവലം കഥകള്‍ മാത്രമാണെന്നും അവയ്ക്ക് യാതൊരുവിധ ചരിത്രപരതയും അവകാശപ്പെടാനാകില്ലെന്നുമാണ് പല ആധുനിക ഗവേഷകന്മാരുടെയും പക്ഷമെന്ന വസ്തുത ദി ജ്യൂയിഷ് എന്‍സൈക്ലോപീഡിയയും സമ്മതിക്കുന്നു(The Jewish Encyclopaedia (1905) Vol. 5 Page 235-236) തീരെ ചരിത്രപരത അവകാശപ്പെടാന്‍ കഴിയാത്ത എസ്‌തേറിന്റെ പുസ്തകത്തിലെ ഒരു കേവല പരാമര്‍ശത്തിന്റെ അടിസ്ഥാനത്തില്‍ ഖുര്‍ആനിലെ ഫിര്‍ഔന്‍-ഹാമാന്‍ കഥയ്ക്ക് യാതൊരുവിധ അടിത്തറയുമില്ലെന്ന് പറയുന്നതെങ്ങനെ? യഥാര്‍ത്ഥത്തില്‍, എസ്‌തേറിന്റെ പുസ്തകത്തില്‍ പറയുന്ന ഹാമാനും ഖുര്‍ആനിലെ ഹാമാനും തമ്മില്‍ യാതൊരു ബന്ധവുമില്ലെന്ന വസ്തുത രണ്ട് ഗ്രന്ഥങ്ങളിലെയും ഹാമാനെക്കുറിച്ച പരാമര്‍ശങ്ങളുള്ള ഭാഗങ്ങള്‍ ഒരു ആവര്‍ത്തി വായിക്കുന്ന ആര്‍ക്കും ബോധ്യമാകും. എന്നാല്‍ ബൈബിളില്‍ നിന്ന് പകര്‍ത്തിയെഴുതിയുണ്ടാക്കിയതാണ് ഖുര്‍ആനെന്ന് വരുത്താന്‍വേണ്ടി ഗവേഷണങ്ങളിലേര്‍പ്പെടുന്നവര്‍ പലപ്പോഴും ഇക്കാര്യം വിസ്മരിക്കുകയോ അവഗണിക്കുകയോ ചെയ്യുന്നുവെന്നതാണ് വാസ്തവം. എസ്‌തേറിന്റെ പുസ്തകമാകുന്ന മാനദണ്ഡത്തിന്റെ വെളിച്ചത്തില്‍ മാത്രമെ ഹാമാനെപ്പറ്റി മനസ്സിലാകൂയെന്ന് വാശി പിടിക്കുന്നവര്‍ ആ പുസ്തകം എത്രത്തോളം വസ്തുനിഷ്ഠവും സത്യസന്ധവും ചരിത്രപരവുമാണെന്ന് പരിശോധിക്കുവാന്‍ മിനക്കെടാറില്ല. പ്രസ്തുത പരിശോധന നടക്കുമ്പോള്‍ മാത്രമെ തങ്ങള്‍ മാനദണ്ഡമായുപയോഗിക്കുന്നത് തീരെ മാനദണ്ഡമാക്കുവാന്‍ കൊള്ളാത്ത ഗ്രന്ഥമാണെന്ന വസ്തുത ബോധ്യപ്പെടുകയുള്ളൂ. ഖുര്‍ആനില്‍ പരാമര്‍ശിച്ചിരിക്കുന്ന ഫിര്‍ഔന്‍-ഹാമാന്‍ കഥയ്ക്ക് ബൈബിളിലെ ഏതെങ്കിലും കഥകളുമായി പ്രത്യക്ഷമോ പരോക്ഷമോ ആയയാതൊരുവിധ ബന്ധവുമില്ലെന്ന വസ്തുത നടേ സൂചിപ്പിച്ചകാര്യങ്ങളില്‍ നിന്ന് നാം മനസ്സിലാക്കി. ഇനി പ്രസ്തുത കഥയ്ക്ക് ഏതെങ്കിലും അര്‍ത്ഥത്തിലുള്ള ചരിത്രപരത അവകാശപ്പെടാനാകുമോയെന്നാണ് പരിശോധിക്കപ്പെടേണ്ടത്. ഫറോവയും ഹാമാനുമുള്‍പ്പെടുന്ന ഖുര്‍ആനിക കഥയ്ക്ക് ഉപോല്‍ബലകമായി ബൈബിളല്ലാത്ത മറ്റുവല്ല സ്രോതസ്സുകളും വല്ല തെളിവുകളും നല്‍കുന്നുണ്ടോ? നാം പരിശോധിക്കുക. ഫറോവ–ഹാമാന്‍ കഥയുമായി ബന്ധപ്പെടുത്തി ഖുര്‍ആന്‍ പറയുന്ന കാര്യങ്ങളെ ഇങ്ങനെ വ്യവഛേദിച്ചെഴുതാം. (1) ഫറോവ ദൈവമാണെന്ന് അവകാശപ്പെട്ടിരുന്നു. (2) ഫറോവ ആകാശത്തേക്ക് കയറിപ്പോകാന്‍ ശ്രമിച്ചിരുന്നു. (3) ഹാമാന്‍ ഫറോവയുടെ ഒരു ഉദ്യോഗസ്ഥനായിരുന്നു. (4) ഫറോവ ഹാമാനോട് കളിമണ്ണുകൊണ്ട് ചുട്ടെടുത്ത ഇഷ്ടികയുപയോഗിച്ച് ഒരു ഗോപുരം നിര്‍മ്മിക്കുവാനാവശ്യപ്പെട്ടിരുന്നു. ബൈബിളിലും ഖുര്‍ആനിലും പരാമര്‍ശിക്കപ്പെട്ടിരിക്കുന്ന ഫറോവയുമായി ബന്ധപ്പെട്ട കഥകള്‍ പരിശോധിക്കാന്‍ ഇവയല്ലാത്ത മറ്റുവല്ല സ്രോതസ്സുകളും നമുക്ക് ലഭ്യമാണോ? ഈ ചോദ്യത്തിന് പത്തൊന്‍പതാം നൂറ്റാണ്ടുവരെ നല്‍കപ്പെട്ടിരുന്ന ഉത്തരം ”ഇല്ല”യെന്നായിരുന്നു. എന്നാല്‍ ഇന്നത്തെ സ്ഥിതി അതല്ല. പുരാതന ഈജിപ്തിനെക്കുറിച്ച പഠനശാഖ(Egyptology) ഇന്ന് ഏറെ പുരോഗമിച്ചിട്ടുണ്ട്. പുരാതന ഈജിപ്തില്‍ നിലനിന്നിരുന്ന ചിത്രലിപി(Hieroglyph)കള്‍ വായിക്കുവാന്‍ കഴിഞ്ഞതുമൂലം ഈജിപ്‌തോളജി കഴിഞ്ഞ രണ്ട് ശതാബ്ദത്തിനുള്ളില്‍ അഭൂതപൂര്‍വ്വമായ വളര്‍ച്ച നേടി. ഫറോവമാരെയും ഹിക്‌സോസുകാരെയുമെല്ലാം കുറിച്ച് ബൈബിളിലോ ഖുര്‍ആനിലോ ഇല്ലാത്ത പല അറിവുകളും ചിത്രലിപിയുടെ വായനമൂലം നമുക്ക് ലഭിച്ചു. ഈജിപ്തുകാരെയും ഫറോവമാരെയും കുറിച്ച് വിജ്ഞാനകോശങ്ങളില്‍ നിന്ന് നടേ ഉദ്ധരിച്ച പല അറിവുകളും ഈവായനമൂലം ലഭിച്ചവയാണ്. ഹിരോഗ്ലിഫ് ലിപികളുടെ വായനയുടെ ചരിത്രം വിവരിക്കുന്നിടത്ത് എന്‍സൈക്ലോപീഡിയ ബ്രിട്ടാണിക്ക പറയുന്നവരികള്‍ ശ്രദ്ധേയമാണ്. ”1799ല്‍ നടന്ന റോസറ്റാ സ്‌റ്റോണിന്റെ (Rosetta Stone) കണ്ടെത്തലാണ് ആ രഹസ്യം തുറക്കുവാനുള്ള അവസാനത്തെ താക്കോല്‍ നമുക്ക് തന്നത്. മൂന്നുതരം വ്യത്യസ്ത എഴുത്തുരേഖകളുണ്ടായിരുന്നു പ്രസ്തുത ശിലയില്‍; ഹീരോഗ്ലിഫിക്, ഡെമോട്രിക്, ഗ്രീക്ക് രേഖകള്‍. ഈ മൂന്ന് രേഖകളും സമാനമാണെന്ന ഗ്രീക്ക് രേഖയിലെ പ്രഖ്യാപനമാണ് ഇത് പരിഭാഷപ്പെടുത്തുന്ന കാര്യത്തിലെ ശ്രദ്ധേയമായ കാല്‍വെപ്പുകള്‍ക്ക് കാരണമായത്…. എ.ഐ.സില്‍വെക്ര ഡി–സാസിയെന്ന ഫ്രഞ്ച് പണ്ഡിതനും ജെ.ഡി. അക്കര്‍ബാദ് എന്ന സ്വീഡിഷ് നയതന്ത്രജ്ഞനും ഡെമോട്രിക് മൂലത്തിലെ നിരവധി നാമങ്ങള്‍ മനസ്സിലാക്കുന്നതില്‍ വിജയിച്ചു. ഏതാനും ചിഹ്‌നങ്ങളുടെ സ്വരമൂല്യം കൃത്യമായി നിശ്ചയിക്കുവാനും അക്കര്‍ബാദിന് കഴിഞ്ഞു. തോമസ് യങ്ങ് എന്ന ഇംഗ്ലീഷുകാരന്‍ അഞ്ച് ഹീരോഗ്ലിഫുകളെ കൃത്യമായി മനസ്സിലാക്കുകയും ചെയ്തു. മറ്റൊരു ഫ്രഞ്ചുകാരനായ ജീന്‍-ഫ്രാന്‍തോയിസ് ചാം പോളിയന്‍ പ്രസ്തുത ശിലാലിഖിതം പൂര്‍ണമായി വായിക്കുന്നതില്‍ വിജയിച്ചു. ഭാഷകളുടെ സ്വാഭാവിക സൗകര്യത്തിലേക്ക് അദ്ദേഹം ഈ ശിലാലിഖിതത്തെ കൊണ്ടുവന്നു. (തന്റെ പതിനാറാമത്തെ വയസില്‍തന്നെ ഗ്രീക്കിനും ലാറ്റിനിനോടുമൊപ്പം ആറ് പൗരാണിക പൗരസ്ത്യ ഭാഷകളില്‍ നൈപുണ്യം നേടിയ വ്യക്തിയാണ് ഇദ്ദേഹം). ഓരോ ചിഹ്‌നവും മറ്റ് ചിഹ്‌നങ്ങളുമായി താരതമ്യം ചെയ്ത് ഹീരോഗ്ലിഫുകളുടെ സ്വരമൂല്യം നിര്‍ണയിക്കാന്‍ അദ്ദേഹത്തിന് കഴിഞ്ഞു. ചാംപോളിയന്റെ കണ്ടെത്തലുകളെ ശക്തിപ്പെടുത്തുകയും നവീകരിക്കുകയുമാണ് പിന്നീട് നടന്ന ഗവേഷണങ്ങള്‍ ചെയ്തത്.”(Encyclopaedia Brittanica Online-“Heiroglyphs”) പത്തൊന്‍പതാം നൂറ്റാണ്ടില്‍ മാത്രമാണ് ആധുനിക മനുഷ്യര്‍ക്ക് പൗരാണിക ഈജിപ്തുകാരെക്കുറിച്ച് വേദഗ്രന്ഥങ്ങളില്‍ നിന്നല്ലാത്ത അറിവുകള്‍ ലഭിക്കുന്നതെന്ന് നാം മനസ്സിലാക്കി. ഈ അറിവുകള്‍ ഇവ്വിഷയകമായ ഖുര്‍ആനിക പരാമര്‍ശങ്ങളെ അനുകൂലിക്കുകയാണോ അതല്ല നിഷേധിക്കുകയാണോ ചെയ്യുന്നതെന്ന അന്വേഷണത്തിന് ഏറെപ്രസക്തിയുണ്ട്. ഹീരോഗ്ലിഫ് ലിപിയുടെ വായനയില്‍ നിന്ന് നമുക്ക് ലഭിക്കുന്ന അറിവുകള്‍ ഖുര്‍ആനിക പരാമര്‍ശങ്ങളുമായി യോജിക്കുന്നുവെന്നുവന്നാല്‍ ഖുര്‍ആന്‍ നല്‍കുന്ന വിജ്ഞാനം പ്രമാദമുക്തമാണെന്ന വസ്തുതയാണ് അതുമൂലം നമുക്ക് മനസ്സിലാക്കാനാവുക. പത്തൊന്‍പതാം നൂറ്റാണ്ടില്‍ മാത്രം ആധുനിക മനുഷ്യന്‍ മനസ്സിലാക്കിയ കാര്യങ്ങളുമായി പൊരുത്തപ്പെടുന്ന പരാമര്‍ശങ്ങള്‍ ഏഴാം നൂറ്റാണ്ടില്‍ അവതരിപ്പിക്കപ്പെട്ട ഗ്രന്ഥത്തിലുണ്ടാവുകയെന്നത് അതിന്റെ ദൈവികതയല്ലാതെ മറ്റെന്താണ് വെളിപ്പെടുത്തുന്നത്? ഫിര്‍ഔന്‍-ഹാമാന്‍ കഥയുമായി ബന്ധപ്പെട്ട് ഖുര്‍ആനില്‍ സൂചിപ്പിക്കപ്പെട്ട കാര്യങ്ങളെ സംഗ്രഹിച്ച് നേരത്തെ പരാമര്‍ശിച്ച നാല് കാര്യങ്ങളിലേക്ക് ഈജിപ്‌റ്റോളജിയില്‍ നടന്ന ഗവേഷണ പഠനങ്ങള്‍ എന്തെങ്കിലും വെളിച്ചം പകരുന്നുണ്ടോയെന്ന് നാം പരിശോധിക്കുക. (1) ദൈവമാണെന്ന ഫറോവയുടെ അവകാശവാദം: ബൈബിളില്‍ അബ്രഹാമിന്റെയും യോസഫിന്റെയും മോശയുടെയുമെല്ലാം കാലത്ത് ജീവിച്ചിരുന്ന ഈജിപ്ഷ്യന്‍ രാജാക്കന്മാരെ ഫറോവയെന്ന് വിളിച്ചിരിക്കുന്നുവെന്നും ഇത് ചരിത്രപരമായി നോക്കിയാല്‍ അബദ്ധമാണെന്നും മൂസാ (അ)യുടെ കാലത്തെ ഈജിപ്ഷ്യന്‍ ചക്രവര്‍ത്തിയെ മാത്രം ഫിര്‍ഔനെന്ന് വിളിച്ച ഖുര്‍ആനിക രീതിയാണ് സൂക്ഷ്മവും കൃത്യവുമായതെന്നും നടേ നാം മനസ്സിലാക്കി. മോശയുടെ കാലത്ത് ജീവിച്ചിരുന്ന ഫറോവ താന്‍ ദൈവമാണെന്ന് അവകാശപ്പെട്ടിരുന്നുവെന്ന് ഖുര്‍ആന്‍ പറയുന്നുണ്ട്. ”ഫിര്‍ഔന്‍ പറഞ്ഞു: പ്രമുഖന്മാരേ, ഞാനല്ലാതെ യാതൊരു ദൈവവും നിങ്ങള്‍ക്കുള്ളതായി ഞാന്‍ അറിഞ്ഞിട്ടില്ല” (വി.ഖു. 28:3). ”അങ്ങനെ അവന്‍ (തന്റെ ആള്‍ക്കാരെ) ശേഖരിച്ചു. എന്നിട്ട് വിളംബരം ചെയ്തു. ഞാന്‍ നിങ്ങളുടെ അത്യുന്നതനായ രക്ഷിതാവാകുന്നു എന്നവന്‍ പറഞ്ഞു” (വി.ഖു. 79:23,24). ഈജിപ്‌റ്റോളജിയുടെ രംഗത്ത് നടന്ന പുതിയ ഗവേഷണങ്ങള്‍ ഈ പരാമര്‍ശങ്ങളുടെ സത്യതയ്ക്ക് സാക്ഷ്യം നില്‍ക്കുന്നു. എന്‍സൈക്ലോപീഡിയ ബ്രിട്ടാണിക്ക എഴുതുന്നത് കാണുക: ”ആകാശദൈവമായ ഹോറസിന്റെയും സൂര്യദൈവങ്ങളായ റേ, ആമോന്‍, ആറ്റോണ്‍ എന്നിവരോടുമൊപ്പം വ്യവഹരിക്കപ്പെട്ടുപോന്ന തങ്ങളുടെ രാജാവായ ഫറോവ ഒരുദൈവമാണെന്നായിരുന്നു ഈജിപ്തുകാര്‍ വിശ്വസിച്ചത്. ഫറോവ മരണപ്പെട്ടാലും അയാളുടെ ദിവ്യമായ കഴിവുകള്‍ നിലനില്‍ക്കുമെന്നും അത് മരണത്തിന്റെ ദൈവവും ഹോറസിന്റെ പിതാവുമായ ഹോസിറസിലൂടെ തന്റെ പുത്രനും പുതിയ ഫറോവയുമായ വ്യക്തിയിലേക്ക് സന്നിവേശിക്കുമെന്നുമായിരുന്നു അവരുടെ വിശ്വാസം. ഫറോവയുടെ ദിവ്യത്വം വഴിയാണ് അയാള്‍ക്ക് മാന്ത്രികമായ കഴിവുകളുണ്ടായതെന്നും അവര്‍ വിശ്വസിച്ചു. അയാളുടെ കിരീടത്തിന്മേലുള്ള സര്‍പ്പമായ യുറായസ് തന്റെ ശത്രുക്കള്‍ക്കുനേരെ തീപ്പൊരി പാറിക്കുമെന്നും യുദ്ധഭൂമിയില്‍ ആയിരക്കണക്കിന് ശത്രുക്കളെ പരാജയപ്പെടുത്താന്‍ അയാള്‍ക്ക് കഴിയുമെന്നും സര്‍വ്വശക്തനും സര്‍വ്വജ്ഞനും പ്രകൃതിയെയും വിളവെടുപ്പിനെയുമെല്ലാം നിയന്ത്രിക്കുന്നവനുമാണ് അയാളെന്നുമായിരുന്നു അവരുടെ വിശ്വാസം” (Brittanica Online-“Pharoah”) ഹാര്‍പേര്‍സ് ബൈബിള്‍ ഡിക്ഷണറിയും ഇക്കാര്യം സൂചിപ്പിക്കുന്നുണ്ട്: ”തങ്ങളുടെ രാജാവ് ഒരു ദൈവമാണെന്നായിരുന്നു ഈജിപ്തുകാരുടെ സങ്കല്‍പം. രാജകീയ ഫാല്‍ക്കന്‍ ദൈവമായ ഹോറസിന്റെ ജഡാവതാരമാണ് ഫറോവ. അഞ്ചാം രാജവംശം മുതല്‍ക്കെങ്കിലും (2494-2345 ബി.സി) മഹാനായ സൂര്യദൈവമായ റേയുടെ പുത്രനാണ് ഫറോവയെന്ന വിശ്വാസം നിലനിന്നിരുന്നു. അയാള്‍ മരിക്കുമ്പോള്‍ ഓസിറസ് ദേവനായിത്തീരുമെന്നും പരലോകത്ത് മറ്റ് ദേവന്മാരോടൊപ്പം കൂടിച്ചേരുമെന്നുമായിരുന്നു വിശ്വാസം. താത്വികമായി, ഈജിപ്തിലെ ഭൂപ്രദേശങ്ങളെല്ലാം ഫറോവയുടേതായിരുന്നു” (Harper’s Bible Dictionary Page 781) ചുരുക്കത്തില്‍, ഖുര്‍ആന്‍ പരാമര്‍ശിക്കുന്നതുപോലെ മോശയുടെ കാലത്ത് ഈജിപ്ത് ഭരിച്ചിരുന്ന ഫറോവ താന്‍ ദൈവമാണെന്ന് അവകാശപ്പെട്ടിരുന്നുവെന്നും പ്രസ്തുത അവകാശവാദം ഈജിപ്തുകാര്‍ അംഗീകരിച്ചിരുന്നുവെന്നും തന്നെയാണ് പൗരാണിക ഈജിപ്തിനെക്കുറിച്ച ആധുനിക പഠനങ്ങള്‍ വ്യക്തമാക്കുന്നത്. ഇക്കാര്യം ബൈബിള്‍ വ്യാഖ്യാനഗ്രന്ഥങ്ങളും അംഗീകരിക്കുന്നു. (ii) ഫറോവയുടെ ആകാശാരോഹണം: മൂസാ നബി(അ)യുടെ കാലത്തെ ഫിര്‍ഔന്‍ ഒരു ആകാശസൗധം നിര്‍മിക്കുവാന്‍ ആവശ്യപ്പെട്ടുവെന്നും അതുവഴി മൂസാ (അ) പറഞ്ഞ രീതിയിലുള്ള ഒരു ദൈവം ആകാശത്തെവിടെയെങ്കിലും ഉണ്ടോയെന്ന് നോക്കാന്‍ ശ്രമിച്ചുവെന്നും ഖുര്‍ആന്‍ വ്യക്തമാക്കുന്നു. ”ഫിര്‍ഔന്‍ പറഞ്ഞു: പ്രമുഖന്മാരേ, ഞാനല്ലാതെ യാതൊരുദൈവവും നിങ്ങള്‍ക്കുള്ളതായി ഞാന്‍ അറിഞ്ഞിട്ടില്ല. അതുകൊണ്ട് ഹാമാനേ, എനിക്കുവേണ്ടി കളിമണ്ണുകൊണ്ട് (ഇഷ്ടിക) ചുട്ടെടുക്കുക. എന്നിട്ട് എനിക്ക് നീ ഒരു ഉന്നത സൗധം ഉണ്ടാക്കിത്തരിക. മൂസായുടെ ദൈവത്തിങ്കലേക്ക് നിക്കൊന്ന് എത്തിനോക്കാമല്ലോ. തീര്‍ച്ചയായും അവന്‍ വ്യാജം പറയുന്നവരുടെ കൂട്ടത്തിലാണെന്നാണ് ഞാന്‍ കരുതുന്നത്” (വി.ഖു. 28:38). ”ഫിര്‍ഔന്‍ പറഞ്ഞു: ഹാമാനേ എനിക്ക് ആ മാര്‍ഗങ്ങളില്‍ അഥവാ ആകാശമാര്‍ഗങ്ങളില്‍ എത്തിച്ചേരുവാനും എന്നിട്ട് മൂസായുടെ ദൈവത്തിന്റെ അടുത്തേക്ക് എത്തിനോക്കുവാനും തക്കവണ്ണം എനിക്കുവേണ്ടി നീയൊരു ഉന്നത സൗധം പണിത് തരൂ. തീര്‍ച്ചയായും അവന്‍ (മൂസാ) കളവ് പറയുകയാണെന്നാണ് ഞാന്‍ വിചാരിക്കുന്നത്. അപ്രകാരം ഫിര്‍ഔന് തന്റെ ദുഷ്പ്രവൃത്തി അലംകൃതമായി തോന്നിക്കപ്പെട്ടു. നേരായമാര്‍ഗത്തില്‍ നിന്ന് അവന്‍ തടയപ്പെടുകയും ചെയ്തു. ഫറോവയുടെ തന്ത്രം നഷ്ടത്തില്‍ തന്നെയായിരുന്നു” (വി.ഖു. 40:36-37). ഹാമാനോട് പ്രത്യേകമായി ഫിര്‍ഔന്‍ ആവശ്യപ്പെട്ട കാര്യമാണ് ഖുര്‍ആനില്‍ പറഞ്ഞിരിക്കുന്നത്. ഇതേക്കുറിച്ച കൃത്യമായ വല്ല അറിവും ഈജിപ്ഷ്യന്‍ ശിലാലിഖിതങ്ങളിലുണ്ടോയെന്ന് ഇനിയും കണ്ടെത്തിക്കഴിഞ്ഞിട്ടില്ല. എന്നാല്‍ ആകാശാരോഹണത്തിന് താല്‍പര്യമുള്ളവരായിരുന്നു ഫറോവമാര്‍ എന്നകാര്യം പൗരാണിക ഈജിപ്തിനെക്കുറിച്ച് പഠിച്ച ഗവേഷകന്മാര്‍ വ്യക്തമാക്കിയിട്ടുണ്ട്. പ്രസിദ്ധ ഈജിപ്‌റ്റോളജിസ്റ്റായ ക്രിസ്ത്യന്‍ ജാക്വ് എഴുതുന്നത് കാണുക: ”ദൈവങ്ങളുടെ മുന്നില്‍ ഫറോവ തന്റെ പ്രാമാണികത്വം പ്രകടിപ്പിക്കുന്നു. അദ്ദേഹം തനിക്ക് ആകാശത്തിലേക്ക് കയറിപ്പോകാനായി ഒരു ഗോവണിയുണ്ടാക്കുവാന്‍ അവരോട് ആവശ്യപ്പെടുന്നു. അവര്‍ അദ്ദേഹത്തെ അനുസരിച്ചില്ലെങ്കില്‍ അവയ്ക്ക് ഭക്ഷണങ്ങളോ നൈവേദ്യങ്ങളോ നല്‍കുകയില്ല”. (Christian Jacgq: Egyptian Magic (Translated by Janet M. Davis U.K. 1985, page II) പൗരാണിക ഈജിപ്തിലെ ശിലാലിഖിതങ്ങളിലൊന്നിന്റെ വായനയില്‍ നിന്ന് മനസ്സിലായ കാര്യമാണ് മുകളില്‍ ഉദ്ധരിച്ചത്. ഇതില്‍ നിന്ന് ഒരു കാര്യം വ്യക്തമാണ്. ഫറോവമാരില്‍ ഒരാളോ അല്ലെങ്കില്‍ ഒന്നിലധികം പേരോ ആകാശത്തിലേക്ക് കയറിപ്പോകാന്‍ ആഗ്രഹിച്ചിരുന്നുവെന്നും അതിനായി ഗോവണി പണിയാന്‍ ശ്രമിച്ചിരുന്നുവെന്നതുമാണവ. മൂസാ (അ) പറഞ്ഞ ദൈവത്തിന്റെ അസ്തിത്വമന്വേഷിച്ച് ആകാശത്തിലേക്ക് കയറിപ്പോകാനായി ഗോവണി നിര്‍മ്മിക്കുവാന്‍ കല്‍പിച്ചവനായി ഫിര്‍ഔനിനെ പരിചയപ്പെടുത്തുന്ന ഖുര്‍ആനിലെ പരാമര്‍ശങ്ങള്‍ക്ക് ഉപോല്‍ബലകമായ തെളിവുകളാണ് പൗരാണിക ഈജിപ്തിനെക്കുറിച്ച പഠനങ്ങള്‍ നല്‍കുന്നത് എന്നര്‍ത്ഥം. (iii) ഹാമാന്‍, ഫറോവയുടെ ഉദ്യോഗസ്ഥന്‍ ഹാമാനിന്റെ നാമം ആറുതവണ ഖുര്‍ആനിലുണ്ട്. 28:6, 28:8, 28:38, 29:39, 40:24, 40:36 എന്നീ സൂക്തങ്ങളിലാണ് പ്രസ്തുത പേരുള്ളത്. എല്ലാം ഫിര്‍ഔനുമായി ബന്ധപ്പെട്ടുകൊണ്ടുള്ള സന്ദര്‍ഭങ്ങളിലുള്ളതാണ്. ചിലസൂക്തങ്ങള്‍ കാണുക: ”ഫിര്‍ഔന്‍ പറഞ്ഞു: പ്രമുഖന്മാരേ ഞാനല്ലാതെ യാതൊരു ദൈവവും നിങ്ങള്‍ക്കുള്ളതായി ഞാന്‍ അറിഞ്ഞിട്ടില്ല. അതുകൊണ്ട് ഹാമാനേ, എനിക്കുവേണ്ടി കളിമണ്ണുകൊണ്ട് (ഇഷ്ടിക) ചുട്ടെടുക്കുക. എന്നിട്ട് എനിക്ക് നീ ഒരു ഉന്നത സൗധം ഉണ്ടാക്കിത്തരിക. മൂസായുടെ ദൈവത്തിങ്കലേക്ക് എനിക്കൊന്ന് എത്തിനോക്കാമല്ലോ. തീര്‍ച്ചയായും അവന്‍ വ്യാജം പറയുന്നവരുടെ കൂട്ടത്തിലാണെന്നാണ് ഞാന്‍ കരുതുന്നത്” (വി.ഖു. 28:38). ”ഫിര്‍ഔന്‍ പറഞ്ഞു: ഹാമാനേ എനിക്ക് ആ മാര്‍ഗങ്ങളില്‍ അഥവാ ആകാശമാര്‍ഗങ്ങളില്‍ എത്തിച്ചേരുവാനും എന്നിട്ട് മൂസായുടെ ദൈവത്തിന്റെ അടുത്തേക്ക് എത്തിനോക്കുവാനും തക്കവണ്ണം എനിക്കുവേണ്ടി നീയൊരു ഉന്നത സൗധം പണിത് തരൂ. തീര്‍ച്ചയായും അവന്‍ (മൂസ) കളവ് പറയുകയാണെന്നാണ് ഞാന്‍ വിചാരിക്കുന്നത്. അപ്രകാരം ഫിര്‍ഔന് തന്റെ ദുഷ്പ്രവൃത്തി അലംകൃതമായി തോന്നിക്കപ്പെട്ടു. നേരായമാര്‍ഗത്തില്‍ നിന്ന് അവന്‍ തടയപ്പെടുകയും ചെയ്തു. ഫറോവയുടെ തന്ത്രം നഷ്ടത്തില്‍ തന്നെയായിരുന്നു” (വി.ഖു. 40:36-37). ഈ സൂക്തങ്ങളില്‍ നിന്ന് ആരാണ് ഹാമാനെന്നോ അദ്ദേഹവും ഫിര്‍ഔനും തമ്മിലുള്ള ബന്ധമെന്തായിരുന്നുവെന്നോ കൃത്യവും സൂക്ഷ്മവുമായി വ്യക്തമാവുന്നില്ലെന്നത് ശരിയാണ്. എന്നാല്‍ ഫിര്‍ഔനിന്റെ രാജസദസ്സിലെ ഒരു പ്രധാനിയായിരുന്നു ഹാമാനെന്നും കെട്ടിടം പണികളുടെ ചുമതല അദ്ദേഹത്തിനായിരുന്നുവെന്നും ഈ ആയത്തുകള്‍ സുതരാം വ്യക്തമാക്കുന്നുണ്ട്. ഈ ആയത്തുകളില്‍ പരാമര്‍ശിക്കപ്പെട്ട ആശയങ്ങളുമായി പൊരുത്തമുള്ള യാതൊരു കഥയും ബൈബിളില്‍ കാണുന്നില്ല. എസ്‌തേറിന്റെ പുസ്തകങ്ങളില്‍ കാണുന്ന അഹശ്വറോശ് രാജാവിന്റെ പ്രഭുവായ ഹാമാനും ഖുര്‍ആനില്‍ പറഞ്ഞിരിക്കുന്ന ഹാമാനും തമ്മില്‍ യാതൊരു ബന്ധവുമില്ലെന്ന വസ്തുത നടേ നാം സമര്‍ത്ഥിക്കുകയുണ്ടായി. ഫറോവയുടെ കൊട്ടാര സദസ്സില്‍ ഹാമാന്‍ എന്നുപേരായ ഒരാളുണ്ടായിരുന്നുവോ? ഇക്കാര്യത്തില്‍ പുരാതന ഈജിപ്തിനെക്കുറിച്ച പഠനങ്ങളിലേതെങ്കിലും വല്ല വെളിച്ചവും നല്‍കുന്നുണ്ടോ? ഇക്കാര്യത്തില്‍ ഏറെ പഠനം നടത്തിയ വ്യക്തിയാണ് ഡോ. മോറിസ് ബുക്കായി. പൗരാണിക ഈജിപ്തിലുണ്ടായിരുന്നതായി ഖുര്‍ആനില്‍പറയുന്ന ഹാമാനെക്കുറിച്ച് എന്തെങ്കിലുമറിയണമെങ്കില്‍ പൗരാണിക ഈജിപ്ഷ്യനില്‍ അവഗാഹമുള്ള ആരോടെങ്കിലും ചോദിക്കുകയാണ് മാര്‍ഗമെന്ന് കരുതിയ മോറിസ് ബുക്കായി ആ വഴിക്ക് തന്റെ അന്വേഷണം തിരിച്ചുവിടുകയും അതുവഴി അത്ഭുതകരമായ ചില വസ്തുതകള്‍ പുറത്തുകൊണ്ടുവരികയും ചെയ്തു. ഡോ. മോറിസ് ബുക്കായി തന്നെഎഴുതട്ടെ! ”എന്റെ ‘ഖുര്‍ആനിനെക്കുറിച്ച പരിചിന്തനങ്ങള്‍, (Reflections on the Qur’an) എന്ന ഗ്രന്ഥത്തില്‍ സൂചിപ്പിച്ചതുപോലെ പന്ത്രണ്ട് വര്‍ഷങ്ങള്‍ക്ക് മുമ്പ് ക്ലാസിക്കല്‍ അറബിയില്‍ നല്ല പരിജ്ഞാനമുള്ള ഒരു വിദഗ്ദ്ധനോട് ഇക്കാര്യം സൂചിപ്പിക്കുന്നതിലേക്ക് പ്രസ്തുത അന്വേഷണം എന്നെ നയിച്ചു. ഇക്കാര്യങ്ങളെല്ലാം പൂര്‍ണമായി പൂര്‍ത്തീകരിക്കുന്ന പ്രഗല്‍ഭനായ ഒരു ഫ്രഞ്ച് ഈജിപ്‌റ്റോളജിസ്റ്റ് എന്റെ ചോദ്യത്തിന് ഉത്തരം നല്‍കുവാന്‍ ദയവുകാണിച്ചു. ഖുര്‍ആനില്‍ എഴുതിയതുപോലെതന്നെ കോപ്പിയെടുത്ത ‘ഹാമാന്‍’ എന്നപദം കാണിച്ചുകൊണ്ട് ഏഴാം നൂറ്റാണ്ടിലുള്ള ഒരു രേഖയില്‍ ഈജിപ്ഷ്യന്‍ ചരിത്രവുമായി ബന്ധപ്പെട്ട ഒരാളെ സൂചിപ്പിക്കുവാനായി ഉപയോഗിച്ച പദമാണ് ഇതെന്ന് ഞാന്‍ അദ്ദേഹത്തെ തെര്യപ്പെടുത്തി. ഈ പദത്തിന്റെ ഹീരോഗ്ലിഫിക് ലിപ്യന്തരണം താന്‍ കണ്ടിട്ടുണ്ടെന്ന് അദ്ദേഹം പറഞ്ഞു. എന്നാല്‍ ഏഴാം നൂറ്റാണ്ടിലുള്ള ഒരു രേഖയില്‍ ഇത്കാണുക തികച്ചും അസംഭവ്യമാണെന്നും അക്കാലത്ത് ഹീരോഗ്ലിഫുകള്‍ പൂര്‍ണമായിത്തന്നെ വിസ്മൃതിയിലായിരുന്നുവെന്നും അദ്ദേഹം കൂട്ടിച്ചേര്‍ത്തു. പ്രസ്തുത നാമത്തെക്കുറിച്ച തന്റെ അനുമാനത്തെ ഉറപ്പിക്കുവാന്‍ ‘രാന്‍കെ‘യുടെ ”പുതിയ രാജത്വകാലത്തെ വ്യക്തിനാമങ്ങളുടെ നിഘണ്ടു‘ (Henmann Ranke: Dictionary of Personal Names of the New Kingdom) എന്ന ഗ്രന്ഥത്തെ അവലംബിക്കുവാന്‍ അദ്ദേഹം എന്നോട് ആവശ്യപ്പെട്ടു. ആ പുസ്തകം വായിച്ചപ്പോള്‍ അദ്ദേഹം എന്റെ മുന്നില്‍ വെച്ച് എഴുതിയതു പോലെ തന്നെയുള്ള ഹീരോഗ്ലിഫ് ലിപികളില്‍ എഴുതപ്പെട്ട പ്രസ്തുത നാമവും അതിന്റെ ജര്‍മ്മന്‍ ഭാഷയിലുള്ള ലിപ്യന്തരണവും കാണാന്‍ എനിക്ക് കഴിഞ്ഞു. ആ വിദഗ്ദ്ധന്‍ സങ്കല്‍പിച്ചു പറഞ്ഞതെല്ലാം ശരിയാണെന്ന് ഈ നിഘണ്ടു എന്നെ ബോധ്യപ്പെടുത്തി. ഹാമാന്റെ തൊഴില്‍ വായിച്ചപ്പോള്‍ ഞാന്‍ അക്ഷരാര്‍ത്ഥത്തില്‍ സ്തബ്ധനായിപ്പോയി. ”കല്ലുവെട്ടുകുഴിയിലെ തൊഴിലാളികളുടെ നേതാവ്” (The Chief of the Workers in the Stone-quarries). ഖുര്‍ആനില്‍ എന്താണോ പറയുന്നത് കൃത്യമായും അതുതന്നെ!ഫറോവയുടേതായി ഖുര്‍ആന്‍ ഉദ്ധരിക്കുന്ന വചനം ഒരു കെട്ടിടനിര്‍മ്മാതാവിനെയാണല്ലോ അഭിസംബോധന ചെയ്യുന്നത്. ഹാമാനെക്കുറിച്ച് നേരത്തെ സൂചിപ്പിച്ച നിഘണ്ടുവില്‍ പരാമര്‍ശിക്കപ്പെട്ട പുറത്തിന്റെ ഫോട്ടോകോപ്പിയും ഖുര്‍ആനിലെ പ്രസ്തുത നാമമുള്ള പേജുകളും നടേ പറഞ്ഞ വിദഗ്ദ്ധന് കാണിച്ചുകൊടുത്തപ്പോള്‍ അയാളും അമ്പരന്നുപോയി; അയാള്‍ക്ക് മിണ്ടാട്ടം നിന്നുപോയി….. നിഘണ്ടുവിന്റെ കര്‍ത്താവായ റാന്‍കെ സൂചിപ്പിക്കുന്ന ഒരു കാര്യംശ്രദ്ധേയമാണ്. ”വാള്‍ട്ടര്‍ റെന്‍സിന്‍സ്‌കിയെന്ന ഈജിപ്‌റ്റോളജിസ്റ്റ് 1906ല്‍ പുറത്തിറക്കിയ ഒരു ഗ്രന്ഥത്തില്‍ താന്‍ ‘ഹാമാന്‍’ എന്ന നാമം ഓസ്ട്രിയയിലുള്ള വിയന്നയിലെ ഹോഫ് മ്യൂസിയത്തില്‍ സൂക്ഷിച്ചിരിക്കുന്ന ശിലാഫലകത്തില്‍ കൊത്തിവെച്ചതായാണ് കണ്ടതെന്ന് പറയുന്നുണ്ട്. കുറെയേറെ വര്‍ഷങ്ങള്‍ക്കുശേഷം എനിക്ക് ആ ശിലാഫലകത്തിലെ ഹീരോഗ്ലിഫുകളില്‍ രേഖപ്പെടുത്തിയിട്ടുള്ള തൊഴില്‍ വായിക്കാന്‍ സാധിച്ചു. അപ്പോഴാണ് പ്രസ്തുത നാമത്തോടൊപ്പം ചേര്‍ത്തിരിക്കുന്ന നിര്‍ണ്ണയപദം(determinative) അദ്ദേഹം ഫറോവയുമായി വളരെ അടുത്ത വ്യക്തിയാണെന്ന് പ്രാധാന്യപൂര്‍വ്വം വ്യക്തമാക്കുന്നുണ്ടെന്ന് മനസ്സിലായത്” (Dr. Maurice Bucaille: Moses and Pharoah, The Hebrews in Egypt (1995-Tokyo) Page 192-193). എന്തുമാത്രം അത്ഭുതകരമാണ് ഈ കണ്ടെത്തല്‍! ഫിര്‍ഔനിന്റെ രാജസദസ്സിലുള്ള കെട്ടിട നിര്‍മ്മാണവുമായി ബന്ധപ്പെട്ട വ്യക്തിയായാണ് ഖുര്‍ആന്‍ ഹാമാനെ പരിചയപ്പെടുത്തുന്നത്. ഈജിപ്ഷ്യന്‍ ഹീരോഗ്ലിഫുകള്‍ നല്‍കുന്ന അറിവും മറ്റൊന്നല്ല. ഫറോവമാരുടെ കാലത്ത് ഹാമാന്‍ എന്ന ഒരാള്‍ ജീവിച്ചിരുന്നിട്ടേയില്ലെന്ന വിമര്‍ശകരുടെ വാദം ഈജിപ്‌റ്റോളജിയില്‍ നടന്ന പുതിയ ഗവേഷണങ്ങള്‍ തിരസ്‌ക്കരിക്കുന്നതാണ് നാം ഇവിടെകാണുന്നത്. ബൈബിളിന്റെ വെളിച്ചത്തില്‍ ഖുര്‍ആനിനെ വിമര്‍ശിക്കുവാന്‍ ധൃഷ്ടരായവര്‍ക്ക് ഹീരോഗ്ലിഫുകളുടെ വായന വല്ലാത്തൊരു തിരിച്ചടിയാണ് നല്‍കുന്നത്. ഖുര്‍ആനാണ് കൃത്യവും സൂക്ഷ്മവുമായ ചരിത്ര പരാമര്‍ശങ്ങള്‍ നടത്തുന്നത് എന്ന് ബോധ്യപ്പെടുത്തുക മാത്രമല്ല, പുതിയ പഠനങ്ങള്‍ചെയ്യുന്നത്; പ്രത്യുത, ബൈബിള്‍ ഇക്കാര്യത്തില്‍ തീരെ സൂക്ഷ്മമല്ലാത്ത പദപ്രയോഗങ്ങളും പരാമര്‍ശങ്ങളുമാണ് നടത്തുന്നത് എന്നുകൂടി അവ വ്യക്തമാക്കുന്നു. ബൈബിള്‍ അപ്രമാദിയും ഖുര്‍ആന്‍ അബദ്ധങ്ങളാല്‍ നിബിഡവുമാണെന്ന് സമര്‍ത്ഥിക്കാനായി ശ്രമിക്കുന്നവര്‍ നിരത്തുന്ന തെളിവുകള്‍ അവര്‍ക്കെതിരെയുള്ള ബൂമറാംഗുകളായി മാറുന്നതാണ് ഇവിടെയും നാം കാണുന്നത്. (iv) ചുട്ടെടുത്ത ഇഷ്ടികകൊണ്ടുള്ള ഗോപുരം: ഫിര്‍ഔന്‍ ഹാമാനോടായി ആവശ്യപ്പെടുന്നതായി ഖുര്‍ആന്‍ പറയുന്നത് ചുട്ടെടുത്ത കളിമണ്ണുകൊണ്ടുള്ള ഇഷ്ടികയുപയോഗിച്ച് ഒരു ഗോപുരം നിര്‍മ്മിക്കുവാനാണ്. അഹങ്കാരത്തോടെയുള്ള ഫിര്‍ഔനിന്റെ കല്‍പന ഖുര്‍ആന്‍ ഉദ്ധരിക്കുന്നു: ”ഫിര്‍ഔന്‍ പറഞ്ഞു: പ്രമുഖന്മാരേ ഞാനല്ലാതെ യാതൊരു ദൈവവും നിങ്ങള്‍ക്കുള്ളതായി ഞാന്‍ അറിഞ്ഞിട്ടില്ല. അതുകൊണ്ട് ഹാമാനേ, എനിക്കുവേണ്ടി കളിമണ്ണുകൊണ്ട് (ഇഷ്ടിക) ചുട്ടെടുക്കുക. എന്നിട്ട് എനിക്ക് നീ ഒരു ഉന്നത സൗധം ഉണ്ടാക്കിത്തരിക. മൂസായുടെ ദൈവത്തിങ്കലേക്ക് എനിക്കൊന്ന് എത്തിനോക്കാമല്ലോ. തീര്‍ച്ചയായും അവന്‍ വ്യാജം പറയുന്നവരുടെ കൂട്ടത്തിലാണെന്നാണ് ഞാന്‍ കരുതുന്നത്.” (വി.ഖു. 28:38) ഖുര്‍ആന്‍ വിമര്‍ശകരുടെ പരിഹാസത്തിന് പാത്രമായ പരാമര്‍ശങ്ങളിലൊന്നാണിത്. ഫറോവമാരുടെ കാലത്ത് ചുട്ടെടുത്ത ഇഷ്ടിക കൊണ്ട് കെട്ടിടം നിര്‍മ്മിക്കുന്ന വിദ്യ ഈജിപ്തില്‍ നിലവിലുണ്ടായിരുന്നില്ലെന്നും റോമാക്കാരുടെ കാലത്താണ് പ്രസ്തുത സമ്പ്രദായം ആരംഭിച്ചത് എന്നും ഈജിപ്ഷ്യന്‍ പുരാവസ്തു ഗവേഷണത്തിന്റെ വെളിച്ചത്തില്‍ പത്തൊന്‍പതാം നൂറ്റാണ്ടില്‍ എഴുതപ്പെട്ട ചില പുസ്തകങ്ങളിലെ ചില പരാമര്‍ശങ്ങളുടെ അടിസ്ഥാനത്തിലാണ് വിമര്‍ശനമുന്നയിക്കപ്പെടാറുള്ളത്. ചുട്ടെടുത്ത കളിമണ്ണുകൊണ്ടുള്ള ഇഷ്ടികയുണ്ടാക്കുന്ന വിദ്യതന്നെ നിലവിലില്ലാതിരുന്ന കാലത്ത് എങ്ങനെയാണ് അങ്ങനെയുള്ള ഇഷ്ടികകൊണ്ട് കെട്ടിടമുണ്ടാക്കാന്‍ ആവശ്യപ്പെടുകയെന്ന് വിമര്‍ശകര്‍ പരിഹസിക്കുകയും ചെയ്യുന്നു. ഈജിപ്ഷ്യന്‍ പുരാവസ്തു പഠനത്തിലെ പുതിയ കണ്ടെത്തലുകള്‍ മനസ്സിലാക്കുമ്പോള്‍ ഈ പരിഹാസം മുഴുവന്‍ വിമര്‍ശകരുടെ വദനത്തിലേക്കുതന്നെ തിരിച്ചുചെല്ലുന്നതാണ് നമുക്ക് കാണാനാകുന്നത്. പൗരാണിക ഈജിപ്തിലെ കെട്ടിട നിര്‍മ്മാണ രീതികളെക്കുറിച്ച് ആധികാരികമായി പഠിച്ചയാളാണ് എ.ജെ. സ്‌പെന്‍സര്‍. 1979ല്‍ പുറത്തിറങ്ങിയ അദ്ദേഹത്തിന്റെ ”പുരാതന ഈജിപ്തിലെ ഇഷ്ടികവാസ്തുവിദ്യ” (Brick Architecture in Ancient Egypt) എന്ന ഗ്രന്ഥമാണ് ഇവ്വിഷയകമായി ഇന്ന് നിലവിലുള്ള ഏറ്റവും ആധികാരികമായ രേഖ. പ്രസ്തുത പുസ്തകത്തില്‍ പറയുന്നത് കാണുക: ”റോമാക്കാരുടെ കാലംവരെ ചുട്ടെടുത്ത ഇഷ്ടികയുപയോഗിക്കുന്ന രീതിക്ക് പൊതുവായ പ്രചാരമുണ്ടായിരുന്നില്ലെങ്കിലും അതിനേക്കാള്‍ കുറെ മുമ്പുതന്നെ ഈ വസ്തുവെക്കുറിച്ച അറിവ് ഈജിപ്തുകാര്‍ക്കുണ്ടായിരുന്നുവെന്നും ഇടയ്‌ക്കൊക്കെ അത് ഉപയോഗിക്കപ്പെട്ടിരുന്നുവെന്നും വ്യക്തമാക്കുന്നമതിയായ തെളിവുകളുണ്ട്. രാജവംശകാലത്തിന് മുമ്പ് (Predynastic) തന്നെ അബിദോസിലും മഹാസ്‌നായിലുമുണ്ടായിരുന്ന പൊടിചൂളകളില്‍ ചുട്ടെടുത്ത കളിമണ്ണുകൊണ്ടുണ്ടാക്കിയ വലിയ കഷണങ്ങളെ ഇഷ്ടികകളെന്ന് വിളിക്കാനാവുകയില്ലെങ്കിലും അത് സാധാരണ കളിമണ്ണ് ചുട്ടെടുക്കുന്ന വിദ്യ അവര്‍ക്ക് അറിവുണ്ടായിരുന്നുവെന്ന് വ്യക്തമാക്കുന്നുണ്ട്. കത്തിക്കുന്നതുവഴി കളിമണ്‍ ഇഷ്ടികകള്‍ ശക്തമായിത്തീരുമെന്ന അറിവ് ആദ്യകാല ഈജിപ്തുകാര്‍ക്ക് ഉണ്ടായിരുന്നില്ലെന്ന് കരുതാന്‍ യാതൊരുന്യായവുമില്ല. ഏതെങ്കിലുമൊരു കെട്ടിടം യാദൃച്ഛികമായോ മറ്റോ അഗ്‌നിക്കിരയായതില്‍ നിന്ന് അവര്‍ ഇത് മനസ്സിലാക്കിയിരിക്കണം. സക്കാറയില്‍ ഒന്നാം രാജവംശകാലത്തുണ്ടായിരുന്ന ചില സ്തൂപങ്ങളില്‍ തന്നെ യാദൃച്ഛികമായി നിര്‍മ്മിക്കപ്പെട്ട ചുട്ടെടുത്ത ഇഷ്ടികകള്‍ കാണപ്പെടുന്നുണ്ട്. കൊള്ളക്കാര്‍ കത്തിച്ചതുമൂലമോ മറ്റോ ആയിരിക്കണം ഇതുണ്ടായതെന്നാണ് അനുമാനം. എങ്കിലും പുരാതന രാജത്വകാലത്ത് ഈജിപ്തുകാര്‍ കരുതിക്കൂട്ടി ചുട്ടെടുത്ത ഇഷ്ടികകള്‍ നിര്‍മ്മിച്ചിരുന്നുവെന്നതിന് ഇതുവരെ തെളിവുകളൊന്നും ലഭിച്ചിട്ടില്ല. മധ്യരാജത്വകാലത്തെ നൂബിയയിലെ വലിയ കോട്ടകളില്‍ ഉപയോഗിച്ചിരിക്കുന്ന വിരിപ്പുപലകക (Paving-slabs) ളാണ് അവര്‍ മനഃപൂര്‍വ്വം നിര്‍മ്മിച്ച ചുട്ടെടുത്ത ഇഷ്ടികകള്‍ക്കുള്ള ഏറ്റവും പഴയതെളിവ്. 30 x 30 x 5 സെ.മീ. അളവിലുള്ള ഈ പലകകള്‍ സാധാരണ കളിമണ്‍ഇഷ്ടികകളെ അപേക്ഷിച്ച് ഈര്‍പ്പത്തിനോടും തേയ്മാനത്തോടും ഉയര്‍ന്ന പ്രതിരോധശേഷി പ്രകടിപ്പിക്കുന്നതാണ് അത് ഈ ആവശ്യാര്‍ത്ഥം ഉപയോഗിക്കുവാന്‍ കാരണം”. (A.J. SPENCER: Brick Architecture In Ancient Egypt (1979-U.K) Page 140). ചുട്ടെടുത്ത ഇഷ്ടികകള്‍ ഉപയോഗിച്ചുകൊണ്ട് കെട്ടിടമുണ്ടാക്കുന്ന രീതിപൗരാണിക ഈജിപ്തുകാര്‍ക്ക് അറിയാമായിരുന്നുവെങ്കിലും അത് സാര്‍വ്വത്രികമായി ഉപയോഗിക്കപ്പെട്ടിരുന്നില്ലെന്ന് സ്‌പെന്‍സര്‍ ചൂണ്ടിക്കാട്ടുന്നു. എന്നാല്‍ ശക്തമായ സൗധങ്ങളുണ്ടാക്കുമ്പോള്‍ അവ പരിഗണിക്കപ്പെട്ടിരുന്നുവെന്ന് അദ്ദേഹം തന്നെ ചൂണ്ടിക്കാട്ടുന്നുണ്ട്. നടേപറഞ്ഞതില്‍ നിന്ന് ചുട്ടെടുത്ത ഇഷ്ടികകള്‍ എല്ലാ കാലങ്ങളിലുമുള്ള ഈജിപ്തുകാര്‍ക്ക് പരിചിതമായിരുന്നുവെന്നും എന്നാല്‍ അതിന്റെ ഈടുകൊണ്ട് സാധാരണ കളിമണ്‍ കട്ടകളില്‍നിന്ന് വ്യത്യസ്തമായ ഉപയോഗമുള്ളപ്പോള്‍ മാത്രമായിരുന്നു അത് ഉപയോഗിച്ചിരുന്നതെന്നുമാണ് നമുക്ക് മനസ്സിലാവുന്നതെന്ന് ഉപസംഹരിക്കാം” (Ibid Page 141) ഖുര്‍ആനില്‍ പരാമര്‍ശിക്കപ്പെട്ട കാര്യങ്ങളുമായി ഈജിപ്‌റ്റോളജിയിലെ പുതിയ പഠനങ്ങള്‍ നല്‍കുന്ന അറിവ് പൂര്‍ണ്ണമായി യോജിച്ചുവരുന്നതാണ് നമുക്ക് ഇവിടെ കാണാന്‍ കഴിയുന്നത്. മൂസാ നബി(അ)യുടെ കാലത്തിന് മുമ്പുതന്നെ കളിമണ്‍ ഇഷ്ടികകള്‍ ചുട്ടെടുത്തു കൊണ്ട് ബലവത്താക്കുന്ന രീതി ഈജിപ്തുകാര്‍ക്ക് അറിയാമായിരുന്നുവെന്ന അറിവ് ഫിര്‍ഔന്‍ ഹാമാനോടായി ”എനിക്കുവേണ്ടി കളിമണ്‍ ഇഷ്ടിക ചുട്ടെടുക്കുക” (28:38)യെന്ന് പറഞ്ഞതായി ഖുര്‍ആന്‍ ഉദ്ധരിക്കുന്നത് പൂര്‍ണമായും ചരിത്രപരമാണെന്ന് വ്യക്തമാക്കുന്നു. ആകാശത്തിലേക്ക് കയറിപ്പോയി മൂസാ (അ) പറയുന്ന ദൈവമെങ്ങാനും അവിടെയുണ്ടോയെന്ന് നോക്കുവാനായാണ് സൗധം പണിയാന്‍ ഫിര്‍ഔന്‍ ആവശ്യപ്പെട്ടത്. സാധാരണയില്‍ നിന്ന് വ്യത്യസ്തമായ ഉന്നത സൗധം നിര്‍മിക്കുകയാണ് ഫിര്‍ഔനിന്റെ ആവശ്യം. ഈജിപ്തുകാര്‍ സാധാരണ കെട്ടിട നിര്‍മാണത്തിന് ഉപയോഗിച്ചിരുന്ന കളിമണ്‍ കട്ടകള്‍ പോര ഈ സൗധത്തിന്. അതുകൊണ്ടാണ് പ്രത്യേകമായിത്തന്നെ ‘കളിമണ്ണുകൊണ്ട് ചുട്ടെടുക്കുക‘യെന്ന് ആജ്ഞാപിച്ചത്. ഉറപ്പുള്ള ഉന്നത സൗധം നിര്‍മ്മിക്കാന്‍ ഈജിപ്തുകാര്‍ പ്രത്യേക ആവശ്യങ്ങള്‍ക്കായുള്ള നിര്‍മ്മാണ പ്രവര്‍ത്തനങ്ങള്‍ക്ക് ഉപയോഗിക്കുന്ന ചുട്ടെടുത്ത ഇഷ്ടിക തന്നെ വേണമെന്നാണ് ഫിര്‍ഔനിന്റെ കല്‍പന. ഫീരോഗ്ലിഫ് ലിപികളെക്കുറിച്ച് യാതൊന്നും അറിയാതിരുന്ന കാലത്ത് അവതരിപ്പിക്കപ്പെട്ട വിശുദ്ധ വേദഗ്രന്ഥത്തിെല പരാമര്‍ശങ്ങളെല്ലാം ഹീരോഗ്ലിഫുകള്‍ നല്‍കുന്ന അറിവുകളുമായി പൂര്‍ണമായും യോജിക്കുന്നുവെന്ന വസ്തുത ഖുര്‍ആനിന്റെ ദൈവികതയല്ലാതെ മറ്റെന്താണ് വ്യക്തമാക്കുന്നത്?
ഖുര്‍ആനിലെ ഇരുപതാം അധ്യായമായ സൂറത്തുത്വാഹയിലെ 85 മുതല്‍ 97വരെയുള്ള വചനങ്ങളില്‍ മൂസാ (عليهالسلام) തൗറാത്ത് സ്വീകരിക്കുന്നതിന്നായി സീനാമലയില്‍ പോയ സമയത്ത് ഇസ്രായീല്യരില്‍പെട്ട ഒരു സാമിരി അവരുടെ സ്വര്‍ണാഭരണങ്ങളെല്ലാം ശേഖരിച്ച് അതുകൊണ്ട് ഒരു സ്വര്‍ണക്കാളയെ നിര്‍മിക്കുകയും അയാളുടെ നിര്‍ദ്ദേശപ്രകാരം മറ്റുള്ളവര്‍ അതിനെ ആരാധിക്കുവാന്‍ ആരംഭിക്കുകയും ചെയ്ത സംഭവങ്ങള്‍ വിശദീകരിക്കുന്നുണ്ട്. ഖുര്‍ആനിലെ പ്രസ്തുത കഥാകഥനം കാണുക: ”അവന്‍ (അല്ലാഹു) പറഞ്ഞു: എന്നാല്‍ നീ പോന്ന ശേഷം നിന്റെ ജനതയെ നാം പരീക്ഷിച്ചിരിക്കുകയാണ്. ‘സാമിരി‘ അവരെ വഴിതെറ്റിച്ച് കളഞ്ഞിരിക്കുന്നു. അപ്പോള്‍ മൂസ തന്റെ ജനങ്ങളുടെ അടുത്തേക്ക് കുപിതനും ദുഃഖിതനുമായിട്ട് തിരിച്ചുചെന്നു. അദ്ദേഹംപറഞ്ഞു: എന്റെ ജനങ്ങളേ, നിങ്ങളുടെ രക്ഷിതാവ് നിങ്ങള്‍ക്ക് ഉത്തമമായ ഒരു വാഗ്ദാനം നല്‍കിയില്ലേ? എന്നിട്ട് നിങ്ങള്‍ക്ക് കാലം ദീര്‍ഘമായിപ്പോയോ? അഥവാ നിങ്ങളുടെ രക്ഷിതാവിങ്കല്‍ നിന്നുള്ള കോപം നിങ്ങളില്‍ ഇറങ്ങണമെന്ന് ആഗ്രഹിച്ചുകൊണ്ടുതന്നെ എന്നോടുള്ള നിശ്ചയം നിങ്ങള്‍ ലംഘിച്ചതാണോ? അവര്‍ പറഞ്ഞു: ഞങ്ങള്‍ ഞങ്ങളുടെ ഹിതം അനുസരിച്ച് താങ്കളോടുള്ള നിശ്ചയം ലംഘിച്ചതല്ല. എന്നാല്‍ ജനങ്ങളുടെ ആഭരണചുമടുകള്‍ ഞങ്ങള്‍ വഹിപ്പിക്കപ്പെട്ടിരുന്നു. അങ്ങനെ ഞങ്ങള്‍ അത്(തീയില്‍) എറിഞ്ഞ് കളഞ്ഞു. അപ്പോള്‍ സാമിരിയും അപ്രകാരം അത്(തീയില്‍) ഇട്ടു. എന്നിട്ട് അവര്‍ക്ക് അവന്‍ (ലോഹംകൊണ്ട്) ഒരു മുക്രയിടുന്ന കാളക്കുട്ടിയുടെ രൂപമുണ്ടാക്കിക്കൊടുത്തു. അപ്പോള്‍ അവര്‍ അന്യോന്യംപറഞ്ഞു. നിങ്ങളുടെ ദൈവവും മൂസായുടെ ദൈവവും ഇതുതന്നെയാണ്. എന്നാല്‍ അദ്ദേഹം മറന്നുപോയിരിക്കുകയാണ്. എന്നാല്‍ അതൊരുവാക്കുപോലും അവരോട് മറുപടി പറയുന്നില്ലെന്നും അവര്‍ക്ക് യാതൊരുഉപദ്രവവും ഉപകാരവും ചെയ്യാന്‍ അതിന് കഴിയില്ലെന്നും അവര്‍കാണുന്നില്ലേ? മുമ്പുതന്നെ ഹാറൂന്‍ അവരോട് പറഞ്ഞിട്ടുണ്ടായിരുന്നു: എന്റെ ജനങ്ങളേ, ഇത് (കാളക്കുട്ടി)മൂലം നിങ്ങള്‍ പരീക്ഷിക്കപ്പെടുക മാത്രമാണുണ്ടായത്. തീര്‍ച്ചയായും നിങ്ങളുടെ രക്ഷിതാവ് പരമകാരുണികനത്രെ. അതുകൊണ്ട് നിങ്ങള്‍ എന്നെ പിന്തുടരുകയും എന്റെ കല്‍പന അനുസരിക്കുകയും ചെയ്യുക. അവര്‍ പറഞ്ഞു: മൂസാ ഞങ്ങളുടെ അടുത്തേക്ക് മടങ്ങിവരുവോളം ഞങ്ങള്‍ ഇതിനുള്ള ആരാധനയില്‍ നിരതരായിതന്നെ ഇരിക്കുന്നതാണ്. അദ്ദേഹം (മൂസ) പറഞ്ഞു: ഹാറൂനേ, ഇവര്‍ പിഴച്ചുപോയതായി നീ കണ്ടപ്പോള്‍ എന്നെ പിന്തുടരാതിരിക്കാന്‍ നിനക്ക് എന്ത് തടസ്സമാണ് ഉണ്ടായത്. നീ എന്റെ കല്‍പനക്ക് എതിര് പ്രവര്‍ത്തിക്കുകയാണോ ചെയ്തത്? അദ്ദേഹം (ഹാറൂന്‍) പറഞ്ഞു: എന്റെ ഉമ്മയുടെ മകനേ നീ എന്റെ താടിയിലും തലയിലും പിടിക്കാതിരിക്കൂ. ‘ഇസ്രാഈല്‍ സന്തതികള്‍ക്കിടയില്‍ നീ ഭിന്നിപ്പുണ്ടാക്കിക്കളഞ്ഞു. എന്റെ വാക്കിന് നീ കാത്തുനിന്നില്ല.’ എന്ന് നീ പറയുമെന്ന് ഞാന്‍ ഭയപ്പെടുകയാണുണ്ടായത് (തുടര്‍ന്ന് സാമിരിയോട്) അദ്ദേഹം പറഞ്ഞു: ഹേ സാമിരി, നിന്റെ കാര്യം എന്താണ്? അവന്‍ പറഞ്ഞു: അവര്‍ (ജനങ്ങള്‍)കണ്ടുമനസ്സിലാക്കാത്ത ഒരു കാര്യം ഞാന്‍ കണ്ടുമനസ്സിലാക്കി. അങ്ങനെ ദൈവദൂതന്റെ കാല്‍പ്പാടില്‍നിന്നും ഞാന്‍ ഒരു പിടിപിടിക്കുകയും എന്നിട്ട് അത് ഇട്ടുകളയുകയും ചെയ്തു. അപ്രകാരം ചെയ്യാനാണ് എന്റെ മനസ്സ് അന്നെ പ്രേരിപ്പിച്ചത്. അദ്ദേഹം (മൂസ) പറഞ്ഞു: എന്നാല്‍ നീ പോ. തീര്‍ച്ചയായും നിനക്ക് ഈ ജീവിതത്തിലുള്ളത് ‘തൊട്ടുകൂടാ‘ എന്ന് പറയലായിരിക്കും. തീര്‍ച്ചയായും നിനക്ക് നിശ്ചിതമായ ഒരു അവധിയുണ്ട്. അത് അതിലംഘിക്കപ്പെടുകയേ ഇല്ല. നീ പൂജിച്ചുകൊണ്ടിരിക്കുന്ന നിന്റെ ആ ദൈവത്തിന്റെ നേരെ നോക്കൂ. തീര്‍ച്ചയായും നാം അതിനെ ചുട്ടെരിക്കുകയും എന്നിട്ട് നാം അത് പൊടിച്ച് കടലില്‍ വിതറിക്കളയുകയും ചെയ്യുന്നതാണ്” (വി.ഖു. 20:85-97) ഈ വചനങ്ങളില്‍ ഒരു ‘സാമിരി‘യാണ് സ്വര്‍ണ്ണക്കാളയെ നിര്‍മ്മിച്ചതെന്നാണല്ലോ പറയുന്നത്. ‘സാമിരി‘യെന്നത് ഒരു വ്യക്തിയുടെ പേരല്ലയെന്നാണ് ഖുര്‍ആനിലെ ‘അസ്‌സാമിരി‘യെന്ന പദപ്രയോഗത്തില്‍നിന്ന് മനസ്സിലാകുന്നത്. ശമരിയക്കാരന്‍ (Samiritan) എന്നാണ് ചില ഖുര്‍ആന്‍വ്യാഖ്യാതാക്കള്‍ ‘അസ്‌സാമിരി‘ക്ക് അര്‍ത്ഥം നല്‍കിയിരിക്കുന്നത്. എന്നാല്‍ ശമര്യ പട്ടണമുണ്ടായതു തന്നെ ഏകദേശം ബി.സി. 870ലെ ഇസ്രായേല്‍ ഭരണാധികാരിയായിരുന്ന ഒമ്രിയുടെ കാലത്തായിരുന്നുവെന്നാണ് ബൈബിള്‍ പഴയ നിയമം വ്യക്തമാക്കുന്നത്: ”യഹൂദ രാജാവായ ആസായുടെ വാഴ്ചയുടെ 31-ാം വല്‍സരം ഒമ്‌റി ഇസ്രായീലില്‍ ഭരണം ആരംഭിച്ചു. അയാള്‍ പന്ത്രണ്ട് വല്‍സരം ഭരണം നടത്തി. അതില്‍ ആറ് വല്‍സരം തിറുസായില്‍ ഭരണം നടത്തി. അയാള്‍ രണ്ട് താലന്ത് വെള്ളികൊടുത്ത് ശമര്യാമലശമറിനോട് വാങ്ങി. അയാള്‍ ആ മല കോട്ടകെട്ടി സുരക്ഷിതമാക്കി. മലയുടെ ഉടമയായിരുന്ന ശമറിന്റെ പേരിന് അനുസൃതമായി ആ നഗരത്തിന് ശമര്യായെന്ന് പേരിട്ടു” (1 രാജാ 16:24) മോശെയ്ക്കുശേഷം നൂറ്റാണ്ടുകള്‍ കഴിഞ്ഞാണ് ‘ശമരിയ‘യെന്ന നഗരമുണ്ടായത്. പിന്നെയെങ്ങനെയാണ് ഒരു ശമരിയക്കാരന്‍ മോശയുടെ കാലത്ത് സ്വര്‍ണം കൊണ്ട് കാളക്കുട്ടിയെയുണ്ടാക്കുക? ബൈബിളില്‍ പറയുന്നതിന് വിരുദ്ധമായി അഹറോണല്ല പ്രത്യുത ‘സാമിരി‘യാണ്സ്വര്‍ണവിഗ്രഹമുണ്ടാക്കിയതെന്ന് മുഹമ്മദ് (ﷺ) പറഞ്ഞത് യഹൂദഗ്രന്ഥമായ പിര്‍ഗ്വി റബ്ബി എലിയെസറിലെ (Pirgey Rabbi Eliezer) ഒരു പ്രയോഗം തെറ്റിദ്ധരിച്ചുകൊണ്ടാണെന്നാണ് മനസിലാകുന്നത്. ഇസ്രായീല്യരിലെ ഒരുവിഭാഗമാണ് ശമരിയക്കാര്‍ എന്ന് മനസിലാക്കിയ മുഹമ്മദ് (ﷺ) യഹൂദഗ്രന്ഥത്തിലെ പരാമര്‍ശങ്ങള്‍ തെറ്റായി മനസിലാക്കിയതിനാലാണ് സാമിരിയാണ് സ്വര്‍ണ വിഗ്രഹമുണ്ടാക്കിയതെന്ന കഥയുണ്ടായത്–ഇവ്വിഷയകമായ വിമര്‍ശനങ്ങളുടെ സംക്ഷിപ്തമാണിത്. ഈ വിമര്‍ശനത്തെ മൂന്നായി വിഭജിക്കാം. ഒന്ന്) ബൈബിളില്‍ പറയുന്നതുപോലെ മോശയുടെ സഹോദരനായ അഹറോണാണ് സ്വര്‍ണവിഗ്രഹമുണ്ടാക്കിയത്. സാമിരിയാണെന്ന് മുഹമ്മദ് നബി തെറ്റിദ്ധരിച്ചതാണ്. രണ്ട്) ‘സാമിരി‘യെന്ന പേര് ലഭിച്ചത് യഹൂദഗ്രന്ഥമായ പിര്‍ഗ്വി റബ്ബി എലിയെസറിലെ ഒരു പരാമര്‍ശം തെറ്റായി മനസ്സിലാക്കിയതുമൂലമാണ്. ഈഗ്രന്ഥമാണ് ഇവ്വിഷയകമായി മുഹമ്മദി(ﷺ)ന്റെ പ്രധാന സ്രോതസ്സ്. മൂന്ന്). ശമരിയ പട്ടണമുണ്ടായത് മോശയ്ക്കുശേഷം നൂറ്റാണ്ടുകള്‍കഴിഞ്ഞാണ് എന്നിരിക്കെ ശമര്യക്കാരനാണ് സ്വര്‍ണവിഗ്രഹം നിര്‍മിച്ചതെന്ന പരാമര്‍ശം. ചരിത്രപരമായി നോക്കിയാല്‍ ശുദ്ധ വങ്കത്തമാണ്. ഈ വിമര്‍ശനങ്ങളില്‍ എത്രത്തോളം കഴമ്പുണ്ടെന്ന് പരിശോധിക്കുക. ഒന്ന്). അഹറോണാണ് സ്വര്‍ണംകൊണ്ട് കാളക്കുട്ടിയെ നിര്‍മിച്ചതെന്ന് ബൈബിള്‍ പറയുന്നുണ്ടെന്നത് ശരിയാണ്. പുറപ്പാട് പുസ്തകം പറയുന്നത് നോക്കുക: ”മോശെയെ പര്‍വ്വതത്തില്‍നിന്ന് വരാന്‍ വൈകുന്നത് കണ്ട് ജനം അഹറോന്റെ ചുറ്റുംകൂടി പറഞ്ഞു: ‘എഴു ന്നേല്‍ക്കൂ, ഞങ്ങളെ നയിക്കാന്‍ ഞങ്ങള്‍ക്ക് ദേവന്മാരെ ഉണ്ടാക്കിത്തരൂ. ഞങ്ങളെ ഈജിപ്തില്‍നിന്ന്കൊണ്ടുവന്ന ഈ മോശെക്ക് എന്ത് സംഭവിച്ചെന്ന് ഞങ്ങള്‍ക്ക് അറിഞ്ഞുകൂട‘. അപ്പോൾ ‍അഹറോണ്‍ അവരോട് പറഞ്ഞു: ‘നിങ്ങളുടെ ഭാര്യമാരുടെയും പുത്രന്മാരുടെയും പുത്രിമാരുടെയും കാതിലെ സ്വര്‍ണവളയങ്ങള്‍ എടുത്ത് എന്റെ അടുത്ത് കൊണ്ടുവരൂ‘. അതനുസരിച്ച് എല്ലാവരും തങ്ങളുടെ കാതുകളില്‍ ഉണ്ടായിരുന്ന സ്വര്‍ണവളയങ്ങളെടുത്ത് അഹറോണിന്റെ അടുത്ത് കൊണ്ടുവന്നു. അയാള്‍ അവ വാങ്ങി. ഒരു കൊത്തുളികൊണ്ട് രൂപം നല്‍കി ഒരു കാളക്കുട്ടിയെ വാര്‍ത്തുണ്ടാക്കി. അവര്‍ പറഞ്ഞു: ‘ഇസ്രായീലെ, ഇതാ നിന്നെ ഈജിപ്തില്‍നിന്ന് കൊണ്ടുവന്ന നിന്റെ ദേവന്മാര്‍!’. ഇതുകണ്ടപ്പോള്‍ അഹറോണ്‍ കാളക്കുട്ടിയുടെ മുമ്പില്‍ ഒരു ബലിപീഠമുണ്ടാക്കി. അയാള്‍ പ്രഖ്യാപിച്ചു: ‘നാളെ കര്‍ത്താവിന് ഒരു ഉത്‌സവമായിരിക്കും‘. ജനങ്ങള്‍ അടുത്ത ദിവസം രാവിലെ എഴുന്നേറ്റ് ഹോമബലി കഴിക്കുകയും സമാധാന ബലി അര്‍പ്പിക്കുകയും ചെയ്തു. ജനങ്ങളിരുന്ന് തീനും കുടിയും കഴിഞ്ഞു കൂത്താടാന്‍ തുടങ്ങി” (പുറ: 32:1-6). പ്രവാചകനായ ഹാറൂന്‍ (عليه السلام) വിഗ്രഹാരാധന നടത്തുകയും അത് പ്രോത്‌സാഹിപ്പിക്കുകയും ചെയ്തുവെന്ന ബൈബിള്‍ പരാമര്‍ശം ഖുര്‍ആന്‍ അംഗീകരിക്കുന്നേയില്ല. സാമിരിയുടെ ദുരുപദേശംമൂലം ജനം വഴിപിഴച്ചുപോകുമ്പോള്‍ അവരെ തടഞ്ഞുനിര്‍ത്തി സത്യമാര്‍ഗത്തിലേക്ക് അവരെ ക്ഷണിക്കുന്നവനായാണ് ഹാറൂനി(عليه السلام)നെ ഖുര്‍ആന്‍ പരിചയപ്പെടുത്തുന്നത്. ”മുമ്പുതന്നെ ഹാറൂന്‍ അവരോട് പറഞ്ഞിട്ടുണ്ടായിരുന്നു: എന്റെ ജനങ്ങളേ, ഇത് (കാളക്കുട്ടി) മൂലം നിങ്ങള്‍ പരീക്ഷിക്കപ്പെടുക മാത്രമാണ് ഉണ്ടായത്. തീര്‍ച്ചയായും നിങ്ങളുടെ രക്ഷിതാവ് പരമകാരുണികനത്രെ. അതുകൊണ്ട് നിങ്ങള്‍ എന്നെ പിന്തുടരുകയും എന്റെ കല്‍പന അനുസരിക്കുകയും ചെയ്യുക. അവര്‍ പറഞ്ഞു: മൂസാ ഞങ്ങളുടെ അടുത്തേക്ക് മടങ്ങിവരുവോളം ഞങ്ങള്‍ ഇതിനുള്ള ആരാധനയില്‍ നിരതരായി തന്നെ ഇരിക്കുന്നതാണ്” (വി.ഖു. 20:90,91). യഥാര്‍ത്ഥത്തില്‍ അഹറോണ്‍ വിഗ്രഹാരാധനയെന്ന മഹാപാപം ചെയ്തിട്ടില്ലെന്നുതന്നെയാണ് പുറപ്പാട് പുസ്തകത്തിന്റെ മുപ്പത്തിരണ്ടാം അധ്യായം ഒന്ന് മനസ്സിരുത്തി വായിച്ചാല്‍ നമുക്ക് മനസ്സിലാവുക. താഴെപറയുന്ന കാര്യങ്ങള്‍ ശ്രദ്ധിക്കുക: i)വിഗ്രഹാരാധനയെന്ന മഹാപാപം ചെയ്തവര്‍ക്ക് മോശ വിധിച്ച ശിക്ഷയെപ്പറ്റി പുറപ്പാട് പുസ്തകം പറയുന്നത് ഇങ്ങനെയാണ്: ”അഹറോണ്‍ അവരെ കെട്ടഴിച്ചുവിടുകയാല്‍ ശത്രുക്കളുടെ മുമ്പില്‍ പരിഹാസ്യരാകുമാറ് ജനം നിയന്ത്രണംവിട്ടുപോയെന്ന് കണ്ട മോശെ പാളയത്തിന്റെ വാതില്‍ക്കല്‍ നിന്നിട്ടു പറഞ്ഞു: ‘കര്‍ത്താവിന്റെ പക്ഷത്തുള്ളവര്‍ എന്റെ അടുത്ത് വരട്ടെ‘. ലേവിയുടെ പുത്രന്മാരെല്ലാം ഉടനടി മോശെയുടെ ചുറ്റും വന്നുകൂടി. അപ്പോള്‍ അയാള്‍ പറഞ്ഞു: ‘ഇസ്രായീലിന്റെ ദൈവമായ കര്‍ത്താവ് ഇങ്ങനെ അരുള്‍ ചെയ്യുന്നു: ഓരോരുത്തനും തന്റെ വാളുമേന്തി പാളയത്തിലെ കൂടാരവാതിലുകള്‍തോറും ചെന്ന് തന്റെ സഹോദരന്മാരെയും സുഹൃത്തുക്കളെയും അയല്‍ക്കാരെയും കൊന്നുകളയുക.’. ലേവിയുടെ പുത്രന്മാര്‍ മോശെ പറഞ്ഞതുപോലെ പ്രവര്‍ത്തിച്ചു. അന്ന് ജനത്തില്‍ മൂവായിരത്തോളം പുരുഷന്മാര്‍ കൊല്ലപ്പെട്ടു” (പുറ 32:25-28). പാപം ചെയ്തവരെ കൊന്നുകളയാനാണ് ഇവിടെ മോശ കല്‍പിക്കുന്നത്. എന്നാല്‍ ബൈബിള്‍ പ്രകാരം ഈ പാപത്തിന് കാരണക്കാരനായ അഹറോണ്‍ കൊല്ലപ്പെടുന്നതായി നാം കാണുന്നില്ല. അദ്ദേഹം ഈ സംഭവത്തിനുശേഷവും കുറെനാള്‍ ജീവിച്ചിരുന്നതായി പഴയ നിയമം വ്യക്തമാക്കുന്നു. അഹറോണായിരുന്നു സ്വര്‍ണവിഗ്രഹം നിര്‍മിക്കുകയും അതിന് പ്രേരിപ്പിക്കുകയും ചെയ്തതെങ്കില്‍ അദ്ദേഹം ഒന്നാമതായിത്തന്നെ കൊല്ലപ്പെടുമായിരുന്നു. പാപത്തിന് കാരണക്കാരനായ സ്വന്തം സഹോദരനെ സംരക്ഷിക്കുകയും സഹോദരന്‍ വഴി പാപികളായവരെ കൊന്നൊടുക്കുകയും ചെയ്തുകൊണ്ട് മോശെ അനീതി ചെയ്തുവെന്ന് കരുതാന്‍ നിവൃത്തിയില്ല. അഹറോണ്‍ വിഗ്രഹാരാധനക്ക് നേതൃത്വം കൊടുത്തിരുന്നുവെങ്കില്‍ അദ്ദേഹം കൊല്ലപ്പെടുമായിരുന്നു; തീര്‍ച്ച. മോശയുടെ കല്‍പനപ്രകാരം നടന്ന കൂട്ടക്കൊലയില്‍ അഹ്‌റോണ്‍ ഉള്‍പ്പെട്ടിരുന്നില്ലെന്ന വസ്തുത അദ്ദേഹമല്ല സ്വര്‍ണവിഗ്രഹം നിര്‍മ്മിച്ചതെന്ന് വ്യക്തമാക്കുന്നുണ്ട്. ii). സ്വര്‍ണവിഗ്രഹമുണ്ടാക്കുകയും അതിനെ ആരാധിക്കുവാന്‍ ജനത്തെ പ്രേരിപ്പിക്കുകയും ചെയ്ത അഹറോണെ രക്ഷിക്കുവാന്‍ മോശ ധൃഷ്ടനായിരുന്നെങ്കില്‍ തന്നെ വിഗ്രഹാരാധനയെന്ന പാപം ചെയ്ത സ്വന്തം സഹോദരങ്ങളെയും അയല്‍ക്കാരെയും കൊന്നൊടുക്കുവാനുള്ള മോശയുടെ കല്‍പന ശിരസാവഹിച്ച ലേവിയര്‍ അദ്ദേഹത്തിന്റെ പക്ഷപാതിത്വത്തെ ചോദ്യം ചെയ്യുമായിരുന്നുവെന്നത് തീര്‍ച്ചയാണ്. തങ്ങളുടെ സഹോദരങ്ങളെയും സ്വന്തക്കാരെയും കൊന്നൊടുക്കുമ്പോള്‍ ഈ പാപത്തിന് യഥാര്‍ത്ഥത്തില്‍ ഉത്തരവാദിയായ മോശയുടെ സഹോദരന്‍ രക്ഷപ്പെടുന്നത് അവര്‍ക്ക് സഹിക്കില്ലെന്നുറപ്പാണ്. അതുകൊണ്ടുതന്നെ അവര്‍ മോശയെ വിമര്‍ശിക്കുമായിരുന്നു. എന്നാല്‍ അത്തരം വിമര്‍ശനങ്ങളോ ചോദ്യംചെയ്യലുകളോ ഒന്നുംതന്നെ ബൈബിള്‍ ഉദ്ധരിക്കുന്നില്ല. അഹറോണല്ല സ്വര്‍ണവിഗ്രഹം നിര്‍മിച്ചതെന്നാണ് ഇതും മനസ്സിലാക്കിത്തരുന്നത്. iii) കാളക്കുട്ടിയുടെ സ്വര്‍ണവിഗ്രഹം നിര്‍മിക്കുകയും അതിനെ ആരാധിക്കുകയും ചെയ്തവരെക്കുറിച്ചുള്ള ദൈവവിധി ഇങ്ങനെയാണ് പഴയ നിയമം രേഖപ്പെടുത്തിയിരിക്കുന്നത് ”എനിക്കെതിരെ പാപം ചെയ്തവന്റെ പേര്‍ എന്റെ പുസ്തകത്തില്‍നിന്ന് തുടച്ചുനീക്കും” (പുറപ്പാട്32:33). അഹറോന്റെ നാമം ദൈവികഗ്രന്ഥത്തില്‍നിന്ന്ത തുടച്ചുനീക്കിയിട്ടില്ലെന്ന് പഴയനിയമ പുസ്തകങ്ങളിലൂടെ ഒരാവര്‍ത്തിവായിക്കുന്ന ആര്‍ക്കും ബോധ്യപ്പെടും. മാത്രവുമല്ല, ഈ സംഭവത്തിനുശേഷം അഹറോണ് കൂടുതല്‍ ദൈവാനുഗ്രഹങ്ങളും സ്ഥാനമാനങ്ങളും ലഭിക്കുന്നതായാണ് ബൈബിള്‍ മനസിലാക്കിത്തരുന്നത്. ലേവിയരുടെ നേതൃത്വവും വിശുദ്ധ പൗരോഹിത്യത്തിന്റെ പ്രതാപവുമെല്ലാം അഹരോണിലും പുത്ര പാരമ്പര്യത്തിലുമാണ് ദൈവം നിക്ഷിപ്തമാക്കിയത്(സംഖ്യ 18:1-20). ഇതില്‍നിന്നെല്ലാം സ്വര്‍ണവിഗ്രഹം നിര്‍മിക്കുകയെന്നമഹാപാപം ചെയ്തത് അഹരോണായിരിക്കാനിടയില്ലെന്ന് സുതരാംവ്യക്തമാകുന്നു. രണ്ട്) യഹൂദഗ്രന്ഥമായ പിര്‍ഗ്വി റബ്ബി ഏലിയെസറില്‍ മോശയുടെസമൂഹം കാളക്കുട്ടിയുടെ വിഗ്രഹമുണ്ടാക്കി ആരാധിച്ച കഥ പറയുന്നുണ്ടെന്നത് നേരാണ്. ഈ കഥാകഥനത്തിനിടക്ക് സമ്മായെല്‍ (Sammael)കാളവിഗ്രഹത്തിനകത്ത് ഒളിച്ചിരിക്കുകയും മുക്രശബ്ദം പുറപ്പെടുവിച്ചുകൊണ്ട് ഇസ്രായേലിനെ വഞ്ചിക്കുകയും ചെയ്തു”വെന്ന ഒരു പരാമര്‍ശമുണ്ട്. യഹൂദ വിശ്വാസപ്രകാരം മരണത്തിന്റെ മാലാഖയാണ് ‘സമ്മായെല്‍’. ഈ പരാമര്‍ശം തെറ്റായി മനസ്സിലാക്കിക്കൊണ്ടാണ് ‘സാമിരി‘യാണ് വിഗ്രഹം നിര്‍മിച്ചതെന്ന് മുഹമ്മദ്(ﷺ) പറഞ്ഞതെന്നാണ് ആരോപണം. മുഹമ്മദ് നബി (ﷺ) ‘സമ്മായെലി‘നെസാമിരിയായി തെറ്റിദ്ധരിച്ചതുകൊണ്ടാണ് ഖുര്‍ആനില്‍ സാമിരിയാണ് വിഗ്രഹമുണ്ടാക്കിയതെന്ന കഥയുണ്ടായതെന്നാണ് വിമര്‍ശകരുടെ വാദമെന്നര്‍ത്ഥം. ഈ വാദത്തിന്റെ നിരര്‍ത്ഥകത മനസ്സിലാക്കാന്‍ പിര്‍ഗ്വി റബ്ബിഏലിയെസരിനെക്കുറിച്ച് യഹൂദ വിജ്ഞാനകോശം എന്താണ് പറയുന്നതെന്ന് പരിശോധിച്ചാല്‍ മാത്രം മതി. ഈ പുസ്തകത്തെക്കുറിച്ച് ദി ജ്യൂയിഷ് എന്‍സൈക്‌ളോപീഡിയ എഴുതുന്നത് കാണുക. ”പതിമൂന്നാം അധ്യായത്തിന്റെ അവസാനത്തില്‍ രചയിതാവ് അറേബ്യയിലെയും സ്‌പെയിനിലെയും റോമിലെയും മുഹമ്മദന്‍ വിജയങ്ങളുടെ മൂന്ന് ഘട്ടങ്ങളെക്കുറിച്ച് സ്പഷ്ടമായി പ്രതിപാദിച്ചതില്‍നിന്നും ഇശ്മയേലിന്റെ പേരിനോടൊപ്പം ഫാത്തിമയുടെയും ആയിഷയുടെയും പേരുകള്‍ നല്‍കിയതില്‍നിന്നും ജോഷാണ് ഏഷ്യാ മൈനറില്‍ ഇസ്‌ലാം പ്രബലമായിരുന്ന കാലത്താണ് ഈ ഗ്രന്ഥം രചിക്കപ്പെട്ടതെന്ന അഭിപ്രായം ആദ്യമായി അവതരിപ്പിച്ചത്. മുപ്പത്തിയാറാം അധ്യായത്തില്‍ മിശിഹയുടെ ആഗമനത്തിന് മുമ്പുള്ള രണ്ട് സഹോദരന്മാരുടെ ഒന്നിച്ചുള്ള ഭരണത്തെക്കുറിച്ച് പരാമര്‍ശിച്ചതില്‍ നിന്ന് ഈ രചന നടന്നത് ഒമ്പതാംനൂറ്റാണ്ടില്‍ ഹാറൂണ്‍ അര്‍റഷീദിന്റെ രണ്ട് പുത്രന്മാര്‍-അല്‍ അമീനും അല്‍മഅ്മൂനും–ഇസ്‌ലാമിക സാമ്രാജ്യം ഭരിക്കുന്ന കാലത്തായിരിക്കാമെന്നും ഊഹിക്കാവുന്നതാണ്” (The Jewish Encyclopaedia 1905, Funk & Wangnalls Company Vol X Page 59) മുഹമ്മദ് നബി (ﷺ)ക്ക് ശേഷം നൂറ്റാണ്ടുകള്‍ കഴിഞ്ഞ് രചിക്കപ്പെട്ട ഒരുപുസ്തകത്തിലെ പരാമര്‍ശം അബദ്ധത്തില്‍ മനസ്സിലാക്കിയാണ് സാമിരിയെന്ന പദം അദ്ദേഹം ഖുര്‍ആനില്‍ പ്രയോഗിച്ചതെന്ന വിമര്‍ശനം എന്തുമാത്രം വലിയ വങ്കത്തമാണ്! കിട്ടുന്ന ആയുധമെല്ലാമെടുത്ത് ഖുര്‍ആനിനെതിരെ പ്രയോഗിക്കാന്‍ ശ്രമിക്കുമ്പോള്‍ അവ എത്രത്തോളം വസ്തുനിഷ്ഠമാണെന്നുപോലും നോക്കാന്‍ വിമര്‍ശകര്‍ സന്നദ്ധരാകാറില്ലെന്നതിനുള്ള പല ഉദാഹരണങ്ങളിലൊന്നാണിത്. മൂന്ന്) ‘ശോമറോനിം‘ എന്ന ഹിബ്രു പദമാണ് ശമരിയക്കാര്‍ എന്ന് മലയാളത്തിലും Samaritans എന്ന് ഇംഗ്ലീഷിലും പരിഭാഷപ്പെടുത്തപ്പെടുന്നത്. ആരാണ് ശമരിയക്കാര്‍? ബൈബിള്‍ നിഘണ്ടു പറയുന്നത് കാണുക: ”ഇവര്‍ ക്രി.മു. 722ല്‍ സര്‍ഗോന്‍ രാജാവ് ശമര്യയെ കീഴടക്കി തന്റെ ദേശത്തിലേക്ക് നാടുകടത്തിയ ഇസ്രായീല്യര്‍ക്ക് പകരം കുടിപാര്‍പ്പിട്ട വിദേശീയരുടെ സന്തതികളാകുന്നു. ഈ അന്യരായ അശ്യൂര്യര്‍ ആദ്യംവന്നപ്പോള്‍ അശൂര്‍ ദേശത്തില്‍ ആരാധിച്ചതുപോലെ അവരുടെ പഴയദേവതകളെതന്നെ ശമര്യയിലും ആരാധിച്ചു. എന്നാല്‍ ഇവര്‍ക്ക് പല കഷ്ടതകള്‍ സംഭവിച്ചപ്പോള്‍ യഹോവയാണ് കാനാന്‍ ദേശത്തിലെ പരദേവതയെന്ന് വിചാരിച്ച് പ്രവാസത്തില്‍നിന്ന് കൊണ്ടുവരപ്പെട്ട ഒരു പുരോഹിതന്റെ ഉപദേശപ്രകാരം യഹോവയെ ആരാധിച്ചുതുടങ്ങി. ഇവര്‍ ഇതിനായി പുരോഹിതന്മാരെ നിയമിച്ചു. അത് നിമിത്തം യഹൂദന്മാര്‍ ഇവരെ വളരെദോഷിച്ചു. 1: രാജാ 17:33. പിന്നീട് ഏകദേശം 80 സംവല്‍സരങ്ങള്‍ക്ക് ശേഷം അശ്യൂര്‍ രാജാവ് വീണ്ടും പല അന്യജാതിക്കാരെ ശമര്യയില്‍ കുടിപാര്‍പ്പിച്ചു. യസ്ര 4:10. ക്രി. മു 536 യഹൂദന്മാര്‍ പ്രവാസത്തില്‍ നിന്ന് മടങ്ങിവന്നതോടുകൂടി അവരും ശമര്യരും തമ്മില്‍ വിരോധമുണ്ടായി. എസ്ര4:7 നെഹ 4:7. ശമര്യയര്‍ അനന്തരകാലത്ത് ഗരീസി മലയില്‍ ഒരു വലിയദേവാലയം പണിതു. അതുമൂലം യഹൂദന്മാര്‍ക്ക് ഇവരോട് വൈര്യംജ്വലിച്ചു. ഈ വൈര്യം പുതിയ നിയമകാലത്ത് വര്‍ദ്ധമാനമായിരുന്നു” (ബൈബിള്‍ നിഘണ്ടു പുറം 586, 587) ശമരിയക്കാരെക്കുറിച്ച് ഡോ. ഡി. ബാബു പോള്‍ തന്റെ ‘വേദശബ്ദരത്‌നാകരത്തില്‍’ അല്‍പം വിശദമായിത്തന്നെ പ്രതിപാദിക്കുന്നുണ്ട്: ”ശമരിയക്കാര്‍: ശമരിയാ പ്രവിശ്യയില്‍ വസിക്കുന്നവര്‍. ശേഖേമില്‍ പാര്‍ത്ത് എബ്രായരുടെ ദൈവത്തെതന്നെ ആരാധിച്ചവരാണ് തങ്ങള്‍ എന്ന അവകാശവാദം യഹൂദന്മാര്‍ അംഗീകരിക്കുന്നില്ല. അവരുടെ അഭിപ്രായത്തില്‍ രണ്ട് രാജാ 17:24ല്‍ പറയുന്ന കുടിയേറ്റക്കാരാണ് ശമരിയക്കാരുടെ മുന്‍ഗാമികള്‍. യരൂശലേമില്‍നിന്ന് നിഷ്‌കാസിതരായ പുരോഹിതന്മാര്‍ കര്‍മ്മിതരായിരുന്ന ഗെരിസിം ദേവാലയം യവനസ്വാധീനത്തിന് വശഗമായിരുന്നു എന്നും യഹൂദര്‍ ആരോപിക്കുന്നു. പുറജാതിക്കാരുമായി സമ്മിശ്രപ്പെട്ടാണ് ശമരിയായിലെ യഹൂദര്‍ നിലകൊണ്ടത് എന്നതില്‍ തര്‍ക്കമില്ല. എന്നാല്‍ പുറജാതിക്കാരുടെ ദേവന്മാരെ അവര്‍ ആരാധിച്ചുവെന്ന് സ്ഥാപിക്കാവതല്ല. പുറത്തുനിന്ന് കൊണ്ടുവന്ന ദേവന്മാര്‍ക്ക് വലിയ ആയുസ് ആ മണ്ണില്‍ കിട്ടിയെന്ന് തോന്നുന്നില്ല.പ്രവാസത്തില്‍ നിന്ന് മടങ്ങിയവര്‍ യരൂശലേം ദേവാലയം പുനരുദ്ധരിക്കാന്‍ ശ്രമിച്ചപ്പോള്‍ ശമരിയക്കാര്‍ക്ക് സഹകരിക്കണമെന്നുണ്ടായിരുന്നു. അതിന് കഴിയാതെ വന്നപ്പോഴാണ് ‘എന്നാല്‍ കാണിച്ചുതരാം‘ എന്ന മട്ടില്‍ ശമരിയക്കാര്‍ പരാതിയുമായി ഇറങ്ങിയത്. യഹൂദരും ശമരിയക്കാരും തമ്മില്‍ ഇണയില്ലാ പിണക്കം തുടങ്ങുന്നത് ഈ ഘട്ടം മുതലാണ്. യഹൂദര്‍വംശീയ വിശുദ്ധി തെളിയിക്കാന്‍ വംശാവലിക്ക് പ്രാധാന്യം നല്‍കിത്തുടങ്ങിയതും ശമരിയക്കാരെ അകറ്റിനിര്‍ത്താന്‍വേണ്ടി കൂടെയായിരുന്നുവെന്ന് കരുതാവുന്നതാണ്. മഖാബിയ വിപ്ലവകാലത്ത് ശമരിയക്കാര്‍ യഹൂദരുടെ കൂടെയല്ല നിലയുറപ്പിച്ചത്. യരൂശലേം ദേവാലയം അശുദ്ധമാക്കാന്‍ ശമരിയക്കാര്‍ ശ്രമിച്ചതിനെക്കുറിച്ച് ജോസിഫസ് എഴുതിയിട്ടുണ്ട്. ക്രി.പി. 35ല്‍ ശമരിയക്കാര്‍ ഒരു ‘മിശിഹാ‘ യെ കണ്ടെത്തി. പൊന്തിയോസ് പിലാത്തോസ് മശിഹയുടെ ‘ഓശാന യാത്ര‘ അലങ്കോലപ്പെടുത്തിയതിനെ തുടര്‍ന്നുണ്ടായ കലാപങ്ങളാണ് പിലാത്തോസിന്റെ സ്ഥാനചലനത്തിലേക്ക് നയിച്ചത് എന്ന് തോന്നുന്നു. യഹൂദരുടെ പഴയ നിയമം ഇന്നത്തെ രൂപം കൈവരിക്കുംമുമ്പെ ശമരിയക്കാര്‍ പിണങ്ങിയിരുന്നു. അതുകൊണ്ടാണല്ലോ അവര്‍ പഞ്ചഗ്രന്ഥി മാത്രം അംഗീകരിക്കുന്നത്. ഒരു മഹാപുരോഹിതനാണ് ശമരിയക്കാരുടെ നേതാവ്: ആത്മീയമായും ഭൗതികമായും ന്യായപ്രമാണത്തിന് വിശദീകരണവും വ്യാഖ്യാനവും കൊടുക്കുന്നതില്‍ അവര്‍ പരീശന്മാരെപോലെയായിരുന്നു. മശിഹയുടെ ആഗമനം അവരും പ്രതീക്ഷിച്ചിരുന്നു. ‘നേരെയാക്കുന്നവന്‍’ എന്ന് അര്‍ത്ഥമുള്ള താഹേബ് എന്ന പദമാണ് അവര്‍ ഉപയോഗിച്ചിരുന്നത്. പെസഹാ ഉള്‍പ്പെടെ എല്ലാ അനുഷ്ഠാനങ്ങളിലും യഹൂദരില്‍നിന്ന് വ്യതിരിക്തമാണ് ശമര്യാ രീതികള്‍. ശമര്യക്കാര്‍ ഇപ്പോഴുമുണ്ട്: ഏകദേശം നാനൂറ് കുടുംബങ്ങള്‍” (വേദശബ്ദരത്‌നാകരം പുറം 634). ശമരിയക്കാരെക്കുറിച്ച യഹൂദരുടെയും ക്രൈസ്തവരുടെയും പരമ്പരാഗത വാദമാണിത്. എന്നാല്‍ ശമരിയക്കാര്‍ ഈ വാദം അംഗീകരിക്കുന്നില്ല. ഹാര്‍പേഴ്‌സ് ബൈബിള്‍ ഡിക്ഷ്ണറി എഴുതുന്നത് കാണുക: ”ഒരു മതവിഭാഗമെന്ന നിലക്ക് ശമരിയക്കാര്‍ വളരെയേറെ നിഷ്ഠയുള്ളവരും തോറ പ്രകാരം ജീവിക്കുന്നവരും അവരുടെ മതപാരമ്പര്യത്തില്‍ അഭിമാനിക്കുന്നവരുമാണ്. യഹൂദന്മാരല്ല, തങ്ങളാണ് മോശ പഠിപ്പിക്കുകയും പുരാതന കാലത്ത് ഗരിസിം മലയില്‍ അനുഷ്ഠിച്ചുവരികയും ചെയ്ത പൗരാണിക ഇസ്രായീലിന്റെ യഥാര്‍ത്ഥ വിശ്വാസമുള്‍ക്കൊള്ളുന്നവരെന്നാണ് അവരുടെ വാദം. അവര്‍ തങ്ങളെ വിളിക്കുന്നത് ഷാമറിം (Shamerim) എന്നാണ്. ”(തോറ). പ്രകാരം ജീവിക്കുന്നവര്‍” എന്നാണ് ഈ പദത്തിനര്‍ത്ഥം. യഹൂദന്മാര്‍ യഹൂദായുടെ പിന്‍മുറക്കാരാണെന്നതുപോലെ പുരാതന ഇസ്രായേലിലെ യോസഫിന്റെ പിന്‍മുറക്കാരായ ജനവിഭാഗമാണ് തങ്ങളെന്നാണ് അവര്‍ മനസ്സിലാക്കുന്നത്. ഷിലോഹില്‍ ഒരു സമാന്തര ദേവാലയമുണ്ടാക്കിയെന്ന് കരുതപ്പെടുന്ന ഏലിയെന്ന പുരോഹിതനാണ് യഥാര്‍ത്ഥ വിശ്വാസത്തില്‍ നിന്ന് പിഴച്ചുകൊണ്ട് യഹൂദ മതമുണ്ടാക്കിയത്. യഹൂദ ബൈബിളിലെ രണ്ടും മൂന്നും ഭാഗങ്ങളില്‍ പറയുന്ന ഇസ്രായേലിന്റെ വിശ്വാസത്തെക്കുറിച്ച ചരിത്രം വിശുദ്ധമല്ലെന്നും മതഭ്രംശം സംഭവിച്ചവയാണെന്നും അവര്‍ കുറ്റപ്പെടുത്തുന്നു. ശമരിയക്കാര്‍ വിശുദ്ധ ഗ്രന്ഥമായി അംഗീകരിക്കുന്നത് അവരുടെ സവിശേഷമായ സംേശാധനയ്ക്ക് വിധേയമാക്കപ്പെട്ട പഞ്ചഗ്രന്ഥിയെ മാത്രമാണ്” (Harpers Bible Dictionary Page 899) എന്‍സൈക്ലോപീഡിയ ബ്രിട്ടാണിക്ക എഴുതുന്നത് ഇങ്ങനെയാണ്: ‘ശമരിയക്കാര്‍ അവരെ സ്വയം വിളിക്കുന്നത് ബനൂ ഇസ്രായീല്യര്‍ (ഇസ്രായേല്‍ സന്തതികള്‍) എന്നും ഷാമെറിം (ആചരിക്കുന്നവന്‍) എന്നുമാണ്. കാരണം അവരുടെ മതാനുഷ്ഠാനങ്ങളുടെയെല്ലാം പൂര്‍ണമായ പ്രമാണം പഞ്ചഗ്രന്ഥി (പഴയ നിയമത്തിലെ ആദ്യത്തെ അഞ്ച് പുസ്തകങ്ങള്‍)യാണ്. മറ്റ് യഹൂദന്മാര്‍ അവരെ ശൊമോറിം (Shomorim) അഥവാ ശമരിയക്കാര്‍ എന്നാണ് വിളിക്കുന്നത്. തല്‍മൂദില്‍ (നിയമത്തിന്റെയും സിദ്ധാന്തത്തിന്റെയും വ്യാഖ്യാനത്തിന്റെയും റബ്ബിമാരുടെ സംഗ്രഹഗ്രന്ഥം) അവരെ കുത്തിം(Kutim) എന്നാണ് വിളിച്ചിരിക്കുന്നത്. അസീറിയന്‍ വിജയത്തിനുശേഷം ശമരിയയില്‍ കുടിയേറിയ മെസപ്പെട്ടോമിയന്‍ കുത്തിയന്മാരുടെ (Cuthaeans)പിന്‍മുറക്കാരാണ് ഇവര്‍ എന്ന സങ്കല്‍പത്തിലാണ് ഈ അഭിസംബോധന” (“Samiritan” Encyclopaedia Brittanica CD 99 Standard Edition) തങ്ങള്‍ യോസഫിന്റെ പിന്‍മുറക്കാരാണെന്നാണ് ശമരിയക്കാരുടെ വാദമെന്നും ഈ വാദത്തിന് ഉപോല്‍ബലകമായ പൂര്‍ണമായും തള്ളിക്കളയാന്‍ പറ്റാത്ത തെളിവുകളുണ്ടെന്നുമുള്ള വസ്തുതകള്‍ എന്‍സൈക്ലോപീഡിയ ജൂദായിക്കയും സമ്മതിക്കുന്നുണ്ട്. ശമരിയക്കാര്‍ എന്നപേരില്‍നിന്ന് വളരെകുറച്ച് കാര്യങ്ങള്‍ മാത്രമെ മനസ്സിലാക്കാന്‍ കഴിയുന്നുള്ളൂ. 2 രാജാക്കന്മാര്‍ 17:29ല്‍ ഒരു തവണ മാത്രമാണ് ബൈബിള്‍ ”ഷൊറോണിം” എന്ന് പ്രയോഗിച്ചിരിക്കുന്നത്. ഇതിന് ഇംഗ്ലീഷില്‍ Samaritansഎന്നതിനേക്കാള്‍ Samarians എന്ന് ഭാഷാന്തരം ചെയ്യുന്നതാണ് ശരി. ശമരിയക്കാര്‍ ഈ പേര് സ്വയം ഉപയോഗിക്കുന്നില്ലെന്നതാണ് ഒന്നാമത്തെകാര്യം. ദീര്‍ഘകാലമായി അവര്‍ സ്വയം വിളിക്കുന്നത് ഷാമെറിന്‍ (Shamerin)എന്നാണ്. ”സത്യം ആചരിക്കുന്നവര്‍” അല്ലെങ്കില്‍ ”സത്യത്തിന്റെ സംരക്ഷകര്‍” എന്നാണ് ഇതിന്നര്‍ത്ഥം…….. ശമരിയയില്‍ ജീവിച്ചിരുന്നവരുടെയും അസ്സീറിയക്കാരുടെ ശമരിയാവിജയ (722/1 B.C.E) ത്തിന്റെ കാലത്തുണ്ടായിരുന്ന മറ്റുള്ളവരുടെയും മിശ്രണത്തില്‍ നിന്നാണ് ശമരിയക്കാര്‍ ഉണ്ടായതെന്ന വിശ്വാസമായിരുന്നു ഇരുപതാം നൂറ്റാണ്ടിന്റെ മധ്യംവരെ പൊതുവായി നിലനിന്നിരുന്നത്. രാജാക്കന്മാരുടെ രണ്ടാം പുസ്തകത്തിലെ 17ാം അധ്യായമായിരുന്നു ശമരിയക്കാരുടെ ഉല്‍പത്തിയെക്കുറിച്ച് നമുക്ക് അറിവ് നല്‍കുന്ന പ്രധാനപ്പെട്ട ഒരു സ്രോതസ്സ്. എന്നാല്‍ ഈ ബൈബിള്‍ ഭാഗം പുനഃപരിശോധനക്ക് വിധേയമാക്കിയപ്പോള്‍ ശമരിയക്കാരുടെ തന്നെ പുരാവൃത്താന്തങ്ങള്‍ക്കും ചരിത്രങ്ങള്‍ക്കും കൂടുതല്‍ പരിഗണന നല്‍കുന്നതിലേക്ക് നാം നയിക്കപ്പെട്ടിരിക്കുകയാണ്. സെഫര്‍ ഹ യാമീം (Sefer ha-Yamim) എന്ന രണ്ടാം ദിനവൃത്താന്ത (Chronicle II) ത്തിന്റെ പ്രസിദ്ധീകരണത്തോടെ ശമരിയക്കാരുടെ ചരിത്രത്തെക്കുറിച്ച അവരുടെതന്നെ വീക്ഷണം പൂര്‍ണമായും വെളിവായിരിക്കുകയാണ്. ദിനവൃത്താന്തങ്ങളും ശമരിയക്കാരുടേതല്ലാത്ത മറ്റ് പല കാര്യങ്ങളുമെല്ലാം ഇതിലുണ്ട്. ഇതുപ്രകാരം യോസേഫിന്റെ ഗോത്രങ്ങളായ എഫ്രയീമിന്റെയും മനാശ്ശെയുടെയും നേരിട്ടുള്ള പിന്‍ഗാമികളാണ് ശമരിയക്കാര്‍. അഹറോണില്‍ നിന്ന് തുടങ്ങി എലിസറിലൂടെയും ഫിനെഹാസിലൂടെയുമുള്ള മഹാപൗരോഹിത്യവും ക്രിസ്താബ്ദം പതിനേഴാം നൂറ്റാണ്ടുവരെ അവര്‍ അവകാശമാക്കിയിരുന്നു. ഫലസ്തീന്റെ കേന്ദ്രഭാഗത്തുള്ള പുരാതന ഭൂപ്രദേശത്ത് മറ്റ് ഇസ്രായീലി ഗോത്രങ്ങളുമായി സമാധാനത്തില്‍ കഴിയുകയായിരുന്നു ശമരിയക്കാരെന്നും ശേഖേമില്‍നിന്ന് ശിലോഹിലേക്ക് മാറ്റിക്കൊണ്ട് ഉത്തര ഉപാസനാരീതികളെ തകിടം മറിക്കുകയും ചില ഉത്തര ഇസ്രായേലികളെ തന്റെ പുതിയ ആരാധനാ സമ്പ്രദായത്തിലേക്ക് മാറ്റുകയും ചെയ്ത ഏലിയുടെ കാലംവരെ ഇത് തുടര്‍ന്നുവെന്നുമാണ് അവര്‍ വാദിക്കുന്നത്. ശമരിയക്കാരെ സംബന്ധിച്ചിടത്തോളം ഇത് ഒരു വലിയ മാര്‍ഗഭ്രംശമാണ്” (“Samaritans” The Encyclopaedia Judaica CD Rom Edition) ശമരിയക്കാര്‍ തങ്ങള്‍ യോസഫിന്റെ പിന്‍മുറക്കാരാണെന്നാണ് അവകാശപ്പെടുന്നതെന്നും ഈ അവകാശവാദം അപ്പടി നിഷേധിക്കുവാന്‍ സാധ്യമല്ലെന്നാണ് പുതിയ ഗവേഷണങ്ങള്‍ വ്യക്തമാക്കുന്ന തെന്നുമുള്ള വസ്തുതകള്‍ യഹൂദ വിജ്ഞാനകോശംപോലും സമ്മതിക്കുന്നുവെന്നര്‍ത്ഥം. ഒരു വിഭാഗത്തിന്റെ ഉല്‍പത്തിയെയും വിശ്വാസങ്ങളെയും കുറിച്ച് അവരുടെ ശത്രുക്കള്‍ നല്‍കുന്ന അറിവിന്റെ അടിസ്ഥാനത്തിലല്ല വിലയിരുത്തേണ്ടതെന്ന് സാമാന്യ മര്യാദയുടെ വെളിച്ചത്തില്‍ നോക്കുമ്പോള്‍ ശമരിയക്കാരുടെ ഉല്‍പത്തിയെക്കുറിച്ച യഹൂദ വീക്ഷണം തള്ളപ്പെടേണ്ടതാണെന്ന് നമുക്ക് ബോധ്യപ്പെടുന്നു. ശമരിയക്കാരില്‍ ഇന്നും അവശേഷിക്കുന്ന നാനൂറോളം കുടുംബങ്ങള്‍ വിശ്വസിക്കുന്നത് തങ്ങള്‍ യോസേഫിന്റെ പിന്‍മുറക്കാരാണെന്നാണ്. ആ വിശ്വാസത്തിന് നൂറ്റാണ്ടുകളുടെ പഴക്കമുണ്ട്. മാത്രവുമല്ല, പ്രസ്തുത വിശ്വാസത്തില്‍ അല്‍പമെല്ലാം കഴമ്പുണ്ടെന്നു തന്നെയാണ് പുതിയ ഗവേഷണങ്ങള്‍ വ്യക്തമാക്കുന്നത്. പ്രസ്തുത ഗവേഷണങ്ങളാകട്ടെ ഇരുപതാം നൂറ്റാണ്ടില്‍നടന്നവയുമാണ്. ഇതില്‍നിന്ന് ഒരുകാര്യം നമുക്ക് സുതരാം വ്യക്തമാവുന്നു. മൂസാ (عليه السلام)യുടെ കാലത്ത് സ്വര്‍ണവിഗ്രഹം നിര്‍മിക്കുകയും അതിനെ ആരാധിക്കുവാന്‍ ഇസ്രായീല്യരെ പ്രചോദിപ്പിക്കുകയും ചെയ്തത് ഒരു ശമരിയക്കാരനാണെന്ന (അസ്‌സാമിരി) ഖുര്‍ആനിക പ്രസ്താവനയില്‍ ചരിത്രവിരുദ്ധമായി യാതൊന്നുമില്ലെന്ന വസ്തുതയാണത്. ശമരിയക്കാര്‍ യോസഫിന്റെ പിന്‍മുറക്കാരാണെങ്കില്‍ മൂസാ (عليه السلام)യുടെ കാലത്ത് അവരുണ്ടായിരിക്കുമെന്ന് തീര്‍ച്ചയാണ്. തങ്ങളാണ് ഇസ്രാഈല്‍ സന്തതികളുടെ യഥാര്‍ത്ഥ വിശ്വാസാനുഷ്ഠാനങ്ങളുടെ വക്താക്കള്‍ എന്ന് ഇന്നും അവകാശപ്പെടുന്ന അവരുടെ മുന്‍ഗാമികളും സ്വാഭാവികമായിമൂസ(عليه السلام)യോടൊപ്പം കടല്‍ കടന്ന് എത്തിയിരിക്കുമല്ലോ. അവരില്‍ പെട്ടഒരാളായിരിക്കണം സ്വര്‍ണ വിഗ്രഹം നിര്‍മ്മിച്ചുകൊണ്ട് ഇസ്രായീല്യരെ വഴിതെറ്റിച്ചത്. ഖുര്‍ആന്‍ പറഞ്ഞത് പൂര്‍ണമായും സത്യസന്ധമാണെന്ന വസ്തുതയാണ് ഇവിടെ അനാവൃതമാവുന്നത്. സ്വര്‍ണവിഗ്രഹം നിര്‍മിച്ചത് അഹറോണാണെന്ന ബൈബിള്‍ പരാമര്‍ശം അടിസ്ഥാനരഹിതമാണെന്ന് പുറപ്പാട് പുസ്തകം തന്നെ വ്യക്തമാക്കുന്നു. ഒരുസാമിരിയാണ് കുറ്റവാളിയെന്ന ഖുര്‍ആനിക പരാമര്‍ശത്തിന് ഉപോല്‍ബലകമായ തെളിവുകളാണ് പുതിയ ഗവേഷണ ഫലങ്ങളിലൂടെ വെളിവായിക്കൊണ്ടിരിക്കുന്നത്. ബൈബിളില്‍ മാനുഷിക കരവിരുതുകള്‍ നടന്നിട്ടുണ്ടെന്നും ഖുര്‍ആന്‍ തെറ്റുപറ്റാത്ത ദൈവിക ഗ്രന്ഥമാണെന്നുമുള്ള വസ്തുതകള്‍ തന്നെയാണ് ഖുര്‍ആനിനെതിരെയുള്ള വിമര്‍ശനങ്ങളോരോന്നും വെളിച്ചത്തുകൊണ്ടുവരുന്നത്.
ബൈബിളിലെ ഉല്‍പത്തി പുസ്തകം മുപ്പത്തിയൊന്‍പതാം അധ്യായത്തില്‍ യാക്കോബിന്റെ പുത്രനായ യോസഫ് ഈജിപ്തിലെത്തിയതും ഫറോവയുടെ ഉദ്യോഗസ്ഥനായ പോത്തിഫറുടെ വീട്ടില്‍ കഴിഞ്ഞതും അവിടെവെച്ച് യജമാനന്റെ ഭാര്യ അദ്ദേഹത്തെ പ്രലോഭിപ്പിക്കാന്‍ ശ്രമിച്ചതും അതില്‍നിന്ന് അദ്ദേഹം രക്ഷപ്പെട്ടതുമായ കാര്യങ്ങള്‍ വിശദമായിത്തന്നെ പ്രതിപാദിക്കുന്നുണ്ട്. അത് ഇങ്ങനെയാണ്. ”ഇതിനിടയില്‍ യിസ്മാഈല്യന്‍ യോസഫിനെ ഈജിപ്തിലേക്ക് കൊണ്ടുപോയി. അവിടെ ഫറോവയുടെ ഒരു ഉദ്യോഗസ്ഥനും അംഗരക്ഷകസേനയുടെ നായകനുമായ ഈജിപ്തുകാരന്‍ പോത്തിഫര്‍ അയാളെ വിലക്കുവാങ്ങി. കര്‍ത്താവ് യോസഫിന്റെ കൂടെയുണ്ടായിരുന്നതിനാല്‍ അയാള്‍ വിജയിയായി. യജമാനനായ ഈജിപ്തുകാരന്റെ വീട്ടില്‍ അയാള്‍ താമസിച്ചു. കര്‍ത്താവ് യോസഫിന്റെ കൂടെയുണ്ട് എന്നും അയാള്‍ ചെയ്യുന്നതെല്ലാം വിജയ പ്രദമാക്കുന്നതിന് സഹായിക്കുന്നുണ്ടെന്നും യജമാനന്‍ കണ്ടു. അതുകൊണ്ട് യജമാനന്‍ യോസഫില്‍ പ്രീതനായി. യോസഫ് അയാളെ സ്വീകരിച്ചു. പോത്തിഫര്‍ യോസഫിനെ തന്റെ ഗൃഹത്തിലെ മേല്‍നോട്ടക്കാരനായി നിയമിച്ചു. തനിക്കുള്ളതെല്ലാം അയാളുടെ ചുമതലയില്‍ ഏല്‍പിച്ചുകൊടുത്തു. അയാള്‍ യോസേഫിനെ തന്റെ ഗൃഹത്തിനും തനിക്കുള്ള എല്ലാറ്റിനും ചുമതലക്കാരനാക്കി. അപ്പോള്‍ മുതല്‍ യോസേഫ് നിമിത്തം കര്‍ത്താവ് ഈജിപ്തുകാരന്‍ പോത്തിഫറുടെ കുടുംബത്തെ അനുഗ്രഹിച്ചു. കര്‍ത്താവിന്റെ അനുഗ്രഹം വീട്ടിലും വയലിലും അയാള്‍ക്കുള്ള സകലതിന്മേലുമുണ്ടായി. അതുകൊണ്ട് പോത്തിഫര്‍ തനിക്കുള്ളതെല്ലാം യോസേഫിന്റെ ചുമതലയില്‍വിട്ടു. യോസേഫ് വീട്ടിലുണ്ടായിരുന്നതിനാല്‍ ഭക്ഷണത്തിനപ്പുറം മറ്റൊന്നിനും പോത്തിഫറിന് ശ്രദ്ധിക്കേണ്ടിവന്നില്ല. സുന്ദരനും സുമുഖനുമായിരുന്നു യോസേഫ്. കുറെകാലം കഴിഞ്ഞപ്പോള്‍ യജമാനനന്റെ ഭാര്യ യോസേഫില്‍ കണ്ണുവെച്ചു. ”എന്നോടൊപ്പം ശയിക്കുക” എന്ന് അവള്‍ പറഞ്ഞു. യോസേഫ് വിസമ്മതിച്ചു. യജമാനന്റെ ഭാര്യയോട് അയാള്‍ പറഞ്ഞു; ‘നോക്കൂ, ഞാന്‍ ഇവിടെയുള്ളതുകൊണ്ട് എന്റെ യജമാനന്‍ വീട്ടിലെ ഒരു കാര്യത്തെപ്പറ്റിയും ക്ലേശിക്കുന്നില്ല. തനിക്കുള്ളതെല്ലാം അദ്ദേഹം എന്നെ ഏല്‍പിച്ചിരിക്കയാണ്. ഈ വീട്ടില്‍ അയാള്‍ എന്നേക്കാള്‍ വലിയവനല്ല. വീട്ടില്‍ എനിക്കൊന്നും അപ്രാപ്യമാക്കിയിട്ടില്ല; നിങ്ങളെയൊഴിച്ച്. കാരണം നിങ്ങള്‍ അദ്ദേഹത്തിന്റെ ഭാര്യയാണല്ലോ. അപ്പോള്‍പിന്നെ ഞാന്‍ എങ്ങനെ ഈ വലിയ അധര്‍മ്മം പ്രവര്‍ത്തിക്കും; ദൈവത്തിന് എതിരായി പാപം ചെയ്യും?”. ദിവസംതോറും യോേസഫിനോട് അവള്‍ ഇക്കാര്യം പറഞ്ഞിരുന്നെങ്കിലും അവളുടെ കൂടെ ശയിക്കാനോ അവളുടെയടുക്കല്‍ ഇരിക്കാനോ അയാള്‍ കൂട്ടാക്കിയില്ല. ഒരുദിവസം ജോലിചെയ്യുന്നതിനായി യോേസഫ് വീട്ടിനുള്ളിലേക്ക് കടന്നു. പുരുഷന്മാര്‍ ആരും വീട്ടിലില്ലായിരുന്നു. അയാളുടെ മേലങ്കിയില്‍ കയറിപ്പിടിച്ച് ‘എന്നോടൊപ്പം ശയിക്കുക‘ എന്ന് അവള്‍ ആവശ്യം ഉന്നയിച്ചു. മേലങ്കി അവളുടെ കയ്യില്‍ ഉപേക്ഷിച്ച് അയാള്‍ ഓടി വീടിന് വെളിയിലേക്ക് പോയി. അയാള്‍ മേലങ്കി ഉപേക്ഷിച്ച് വീടിന് പുറത്തേക്ക് ഓടിപ്പോയി എന്ന് കണ്ടപ്പോള്‍ അവള്‍ വീട്ടിലുള്ളവരെ വിളിച്ചുവരുത്തി ഇപ്രകാരം പറഞ്ഞു: ‘നോക്കൂ, നമ്മെ അപമാനിക്കാന്‍ അദ്ദേഹം ഒരു എബ്രായനെ നമ്മുടെയിടയില്‍ കൊണ്ടുവന്നിരിക്കുന്നു! എന്റെ കൂടെ ശയിക്കുന്നതിന് അയാള്‍ അകത്ത് കയറിവന്നു. ഞാന്‍ ഉച്ചത്തില്‍ നിലവിളിച്ചു. ഞാന്‍ ശബ്ദം ഉയര്‍ത്തി നിലവിളിക്കുന്നതുകേട്ട ഉടന്‍ അയാള്‍ മേലങ്കി ഉപേക്ഷിച്ച് ഓടി വീടിന് വെളിയിലിറങ്ങിപ്പോയി‘. യോസഫിന്റെ യജമാനന്‍ വീട്ടില്‍ വരുന്നതുവരെ അവള്‍ അയാളുടെ മേലങ്കി കൈവശംവെച്ചു. നേരത്തെ പറഞ്ഞ കഥ തന്നെ അവള്‍ അയാളോടും പറഞ്ഞു: ‘അങ്ങ് ഞങ്ങള്‍ക്ക് കൊണ്ടുവന്ന എബ്രായ ദാസന്‍ മാനം കെടുത്താന്‍ എന്റെ അടുക്കല്‍ വന്നു. ഞാന്‍ ശബ്ദംഉയര്‍ത്തി കരഞ്ഞയുടന്‍ അയാള്‍ മേലങ്കി എന്റെയടുത്ത് ഉപേക്ഷിച്ച് വീടിന് വെളിയിലേക്ക് ഓടിപ്പോയി‘. ‘അങ്ങയുടെ ദാസന്‍ ഈ വിധത്തിലാണ് എന്നോട് പെരുമാറിയതെന്ന് ഭാര്യ പറയുന്നത് കേട്ടപ്പോള്‍ അദ്ദേഹം കോപംകൊണ്ട് ജ്വലിച്ചു. യജമാനന്‍ യോസേഫിനെ പിടിച്ച് രാജാവിന്റെ തടവുകാരെ അടച്ചിരുന്ന ജയിലില്‍ അടച്ചു. യോസേഫ് ജയിലിലായി. എന്നാല്‍, കര്‍ത്താവ് അയാളോട് കൂടെയായിരുന്ന് അചഞ്ചലമായ സ്‌നേഹം പ്രകടിപ്പിച്ചു. തന്മൂലം തടവറ സൂക്ഷിപ്പുകാരന് അയാളോട് പ്രീതി തോന്നി. അവിടെയുള്ള എല്ലാ തടവുകാരെയും തടവറ സൂക്ഷിപ്പുകാരന്‍ യോസേഫിന്റെ മേല്‍നോട്ടത്തില്‍ വിട്ടുകൊടുത്തു; അവിടെ നടന്നതിനെല്ലാം അയാളായിരുന്നു നടത്തിപ്പുകാരന്‍. കര്‍ത്താവ് യോസേഫിനോട് കൂടെയുണ്ടായിരുന്നതിനാല്‍ യോസേഫിനെ ഭരമേല്‍പിച്ച ഒരു കാര്യത്തിനും തടവറ സൂക്ഷിപ്പുകാരന്‍ ശ്രദ്ധിക്കേണ്ടിയിരുന്നില്ല. യോസേഫിന്റെ പ്രവര്‍ത്തികള്‍ എല്ലാം ദൈവം വിജയകരമാക്കി” (ഉല്‍പത്തി 39: 1-23). യൂസുഫ് നബി (عليه السلام)യുടെ ചരിത്രം അല്‍പം വിശദമായിത്തന്നെ ഒരു കഥാകഥനത്തിന്റെ രീതിയില്‍ ഖുര്‍ആന്‍ പന്ത്രണ്ടാം അധ്യായമായ സൂറത്തുയൂസുഫില്‍ വിവരിക്കുന്നുണ്ട്. നടേ ഉദ്ധരിച്ച ബൈബിളില്‍ വചനങ്ങളില്‍ സൂചിപ്പിക്കപ്പെട്ട സംഭവങ്ങള്‍ വിവരിക്കുന്ന ഖുര്‍ആന്‍ സൂക്തങ്ങളുടെ സാരംകാണുക: ”ഈജിപ്തില്‍നിന്ന് അവനെ (യൂസുഫിനെ) വിലക്കെടുത്തയാള്‍ തന്റെ ഭാര്യയോട് പറഞ്ഞു: ഇവന് മാന്യമായ താമസസൗകര്യം നല്‍കുക. ഇവന്‍ നമുക്ക് പ്രയോജനപ്പെട്ടേക്കാം. അല്ലെങ്കില്‍ നമുക്ക് അവനെ മകനായി സ്വീകരിക്കാം. അപ്രകാരം യൂസുഫിന് നാം ആ ഭൂപ്രദേശത്ത് സൗകര്യമുണ്ടാക്കികൊടുത്തു. സ്വപ്‌ന വാര്‍ത്തകളുടെ വ്യാഖ്യാനം അദ്ദേഹത്തിന് നാം അറിയിച്ചുകൊടുക്കുവാന്‍ വേണ്ടിയാണ് അത്. അല്ലാഹു തന്റെ കാര്യം ജയിച്ചടക്കുന്നവനത്രെ. പക്ഷെ, മനുഷ്യരില്‍ അധികപേരും അത് മനസ്സിലാക്കുന്നില്ല. അങ്ങനെ അദ്ദേഹം പൂര്‍ണ്ണ വളര്‍ച്ചയെത്തിയപ്പോള്‍ അദ്ദേഹത്തിന് നാം യുക്തിബോധവും അറിവും നല്‍കി. സുകൃതം ചെയ്യുന്നവര്‍ക്ക് അപ്രകാരം നാം പ്രതിഫലം നല്‍കുന്നു. അവന്‍ (യൂസുഫ്) ഏതൊരുവളുടെ വീട്ടിലാണോ അവള്‍ അവനെ വശീകരിക്കുവാന്‍ ശ്രമം നടത്തി. വാതിലുകള്‍ അടച്ചുപൂട്ടിയിട്ട് അവള്‍ പറഞ്ഞു. ഇങ്ങോട്ട് വാ. അവന്‍ പറഞ്ഞു: അല്ലാഹുവില്‍ ശരണം! നിശ്ചയമായും അവനാണ് എന്റെ രക്ഷിതാവ്. അവന്‍ എന്റെ താമസം ക്ഷേമകരമാക്കിയിരിക്കുന്നു. തീര്‍ച്ചയായും അക്രമം പ്രവര്‍ത്തിക്കുന്നവന്‍ വിജയിക്കുകയില്ല. അവള്‍ക്ക് അവനില്‍ ആഗ്രഹം ജനിച്ചു. തന്റെ രക്ഷിതാവിന്റെ പ്രമാണം കണ്ടറിഞ്ഞില്ലായിരുന്നുവെങ്കില്‍ അവന് അവളിലും ആഗ്രഹം ജനിച്ചേനെ. അപ്രകാരം (സംഭവിച്ചത്) തിന്മയും നീചവൃത്തിയും അവനില്‍നിന്ന് നാം തിരിച്ചുവിടുന്നതിന് വേണ്ടിയത്രെ. തീര്‍ച്ചയായും അവന്‍ നമ്മുടെ നിഷ്‌കളങ്കരായ ദാസന്മാരില്‍ പെട്ടവനാകുന്നു. അവര്‍ രണ്ടുപേരും വാതില്‍ക്കലേക്ക് മത്‌സരിച്ചോടി. അവള്‍ പിന്നില്‍നിന്ന് അവന്റെ കുപ്പായം(പിടിച്ചു. അത്) കീറി. അവര്‍ ഇരുവരും വാതില്‍ക്കല്‍വെച്ച് അവളുടെ നാഥനെ (ഭര്‍ത്താവിനെ) കണ്ടുമുട്ടി. അവള്‍ പറഞ്ഞു. താങ്കളുടെ ഭാര്യയുടെ കാര്യത്തില്‍ ദുരുദ്ദേശം പുലര്‍ത്തിയവനുള്ള പ്രതിഫലം, അവന്‍ തടവിലാക്കപ്പെടുക എന്നതോ, വേദനയേറിയ മറ്റേതെങ്കിലും ശിക്ഷയോ തന്നെ ആയിരിക്കണം. യൂസുഫ് പറഞ്ഞു: അവളാണ് എന്നെ വശീകരിക്കുവാന്‍ ശ്രമം നടത്തിയത്. അവളുടെ കുടുംബത്തില്‍പെട്ട ഒരു സാക്ഷി ഇപ്രകാരം സാക്ഷ്യപ്പെടുത്തി. അവന്റെ കുപ്പായം മുന്നില്‍നിന്നാണ് കീറിയിട്ടുള്ളതെങ്കില്‍ അവള്‍ സത്യമാണ് പറഞ്ഞത്. അവനാകട്ടെ കളവ് പറയുന്നവരുടെ കൂട്ടത്തിലാണ്. എന്നാല്‍ അവന്റെ കുപ്പായം പിന്നില്‍നിന്നാണ് കീറിയിട്ടുള്ളതെങ്കില്‍ അവള്‍ കളവാണ് പറഞ്ഞത്. അവനാകട്ടെ സത്യം പറയുന്നവരുടെ കൂട്ടത്തിലാണ്. അങ്ങനെ അവന്റെ(യൂസുഫിന്റെ) കുപ്പായം പിന്നില്‍ നിന്നാണ് കീറിയിട്ടുള്ളത് എന്ന്കണ്ടപ്പോള്‍ അയാള്‍ (ഗൃഹനാഥന്‍ –തന്റെ ഭാര്യയോട്) പറഞ്ഞു. തീര്‍ച്ചയായും ഇത് നിങ്ങളുടെ (സ്ത്രീകളുടെ) തന്ത്രത്തില്‍പെട്ടതാണ്. നിങ്ങളുടെ തന്ത്രം ഭയങ്കരം തന്നെ. യൂസുഫേ, നീ ഇത് അവഗണിച്ചേക്കുക. (പെണ്ണേ) നീ നിന്റെ പാപത്തിന് മാപ്പ് തേടുക. തീര്‍ച്ചയായും നീ പിഴച്ചവരുടെ കൂട്ടത്തിലാകുന്നു. നഗരത്തിലെ ചില സ്ത്രീകള്‍ പറഞ്ഞു. പ്രഭുവിന്റെ ഭാര്യ തന്റെ വേലക്കാരനെ വശീ കരിക്കാന്‍ ശ്രമിക്കുന്നു. അവള്‍ അവനോട് പ്രേമബദ്ധയായി കഴിഞ്ഞിരിക്കുന്നു. തീര്‍ച്ചയായും അവള്‍ വ്യക്തമായ പിഴവില്‍ അകപ്പെട്ടതായി ഞങ്ങള്‍ കാണുന്നു. അങ്ങനെ ആ സ്ത്രീകളുടെ കുസൃതിയെപ്പറ്റി അവള്‍ കേട്ടറിഞ്ഞപ്പോള്‍ അവരുടെ അടുത്തേക്ക് അവള്‍ ആളെ അയക്കുകയും, അവര്‍ക്ക് ചാരിയിരിക്കാവുന്ന ഇരിപ്പിടങ്ങള്‍ ഒരുക്കുകയും ചെയ്തു. അവരില്‍ ഓരോരുത്തര്‍ക്കും(പഴങ്ങള്‍ മുറിക്കാന്‍) അവള്‍ ഓരോ കത്തികൊടുത്തു. (യൂസഫിനോട്) അവള്‍പറഞ്ഞു. നീ അവരുടെ മുമ്പിലേക്ക് പുറപ്പെടുക. അങ്ങനെ അവനെ അവര്‍ കണ്ടപ്പോള്‍ അവര്‍ക്ക് അവനെപ്പറ്റി വിസ്മയം തോന്നുകയും, അവരുടെ സ്വന്തം കൈകള്‍തന്നെ അറുത്ത് പോവുകയും ചെയ്തു. അവര്‍ പറഞ്ഞു. അല്ലാഹു എത്ര പരിശുദ്ധന്‍. ഇത് ഒരു മനുഷ്യനല്ല, ആദരണീയനായ ഒരുമലക്ക് തന്നെയാണ്. അവള്‍ പറഞ്ഞു: എന്നാല്‍ ഏതൊരുവന്റെ കാര്യത്തില്‍ നിങ്ങളെന്നെ ആക്ഷേപിച്ചുവോ അവനാണ് ഇത്. തീര്‍ച്ചയായും അവനെ ഞാന്‍ വശീകരിക്കാന്‍ ശ്രമിച്ചിട്ടുണ്ട്. അപ്പോള്‍ അവന്‍ (സ്വയം കളങ്കപ്പെടുത്താതെ) കാത്തു സൂക്ഷിക്കുകയാണ് ചെയ്ത്. ഞാന്‍ അവനോട് കല്‍പിക്കുന്നപ്രകാരം അവന്‍ ചെയ്തില്ലെങ്കില്‍ തീര്‍ച്ചയായും അവന്‍ തടവിലാക്കപ്പെടുകയും നിന്ദ്യരുടെ കൂട്ടത്തിലായിരിക്കുകയും ചെയ്യും. അവന്‍ (യൂസുഫ്) പറഞ്ഞു. എന്റെ രക്ഷിതാവേ, ഇവര്‍ എന്നെ ഏതൊന്നിലേക്ക് ക്ഷണിക്കുന്നുവോ അതിനേക്കാളും എനിക്ക് കൂടുതല്‍ പ്രിയപ്പെട്ടത് ജയിലാകുന്നു. ഇവരുടെ കുതന്ത്രം എന്നെവിട്ട് നീ തിരിച്ചുകളയാത്ത പക്ഷം ഞാന്‍ അവരിലേക്ക് ചാഞ്ഞുപോയേക്കും. അങ്ങനെ ഞാന്‍ അവിവേകികളുടെ കൂട്ടത്തിലായിപ്പോവുകയും ചെയ്യും. അപ്പോള്‍ അവന്റെ പ്രാര്‍ത്ഥന തന്റെ രക്ഷിതാവ് സ്വീകരിക്കുകയും അവരുടെ കുതന്ത്രം അവനില്‍നിന്ന് അവന്‍ തട്ടിത്തിരിച്ച് കളയുകയും ചെയ്തു. തീര്‍ച്ചയായും അവന്‍ എല്ലാം കേള്‍ക്കുന്നവനും കാണുന്നവനുമത്രെ. പിന്നീട് തെളിവുകള്‍ കണ്ടറിഞ്ഞതിനുശേഷവും അവര്‍ക്ക് തോന്നി; അവനെ ഒരവധിവരെ തടവിലാക്കുക തന്നെവേണമെന്ന്” (വി.ഖു. 12:21-35). ഖുര്‍ആനും ബൈബിളും ഒരേ കഥയാണ് പരാമര്‍ശിക്കുന്നതെങ്കിലും അവയുടെ വിശദാംശങ്ങളില്‍ ഏറെ വ്യത്യാസങ്ങളുണ്ടെന്ന് ഇവ രണ്ടും വായിച്ചുനോക്കുന്ന ആര്‍ക്കും മനസ്സിലാക്കാവുന്നതേയുള്ളൂ. യൂസുഫ് കുറ്റക്കാരനാണെന്ന് യജമാനനും മറ്റും കരുതിയെന്ന രൂപത്തിലാണ്ബൈബിളിന്റെ വിവരണം. ഖുര്‍ആനിലാകട്ടെ യൂസുഫിന്റെ നിരപരാധിത്വംബോധ്യപ്പെട്ടശേഷവും അദ്ദേഹത്തെ തടവിലാക്കുവാന്‍ യജമാനന്‍തീരുമാനിച്ചതായാണ് പറഞ്ഞിരിക്കുന്നത്. മേലങ്കി ഊരി ഓടിയയൂസുഫിനെയാണ് ബൈബിള്‍ പരിചയപ്പെടുത്തുന്നത്. യജമാന ഭാര്യപിന്നില്‍നിന്ന് പിടിച്ചപ്പോള്‍ കുപ്പായം കീറിയതായാണ് ഖുര്‍ആനിലെവിവരണം. ഇങ്ങനെ വിശദാംശങ്ങളില്‍ നിരവധി വ്യത്യാസങ്ങളുണ്ടെന്നതാണ് വാസ്തവം. യജമാനന്റെ പേരുമായി ബന്ധപ്പെട്ട് ഖുര്‍ആന്‍ നടത്തുന്ന പരാമര്‍ശങ്ങളുടെ വെളിച്ചത്തിലാണ് വിമര്‍ശകന്മാര്‍ ഖുര്‍ആനില്‍ അബദ്ധം ആരോപിക്കുന്നത്. ‘ബൈബിളില്‍ വ്യക്തമായിത്തന്നെ യജമാനന്റെ പേര് പറഞ്ഞിരിക്കുന്നു പോത്തിഫര്‍. ഖുര്‍ആനിലാകട്ടെ യൂസുഫി(عليه السلام)ന്റെ യജമാനനെക്കുറിച്ച് പരാമര്‍ശിക്കുമ്പോള്‍ അസീസ് എന്നാണ് പ്രയോഗിച്ചിരിക്കുന്നത്. പോത്തിഫര്‍ എന്ന നാമവുമായി ഒരു നിലക്കും യോജിക്കുന്ന പേരല്ല അസീസ്. മാത്രവുമല്ല, ഒരു ശുദ്ധ അറബി പദമാണ് ‘അസീസ്‘. യോസഫിന്റെ കാലത്തെ ഈജിപ്തില്‍ ജീവിച്ച ഒരു വ്യക്തിക്ക് ശുദ്ധമായ ഒരു അറബിപ്പേരുണ്ടായിരുന്നുവെന്ന് കരുതുന്നതുപോലും വിഡ്ഢിത്തമാണ്. യഹൂദ ക്രിസ്ത്യാനികളില്‍ നിന്ന് യോസഫിന്റെ കഥകേട്ട് നാളുകള്‍ക്കുശേഷം മുഹമ്മദ് ഖുര്‍ആനില്‍ അത് എഴുതിച്ചേര്‍ത്തപ്പോള്‍ ഉണ്ടായ ഒരു അബദ്ധമാണിത്……..’ വിമര്‍ശകരുടെ ന്യായവാദങ്ങള്‍ ഇങ്ങനെപോകുന്നു. തങ്ങളുടെ വാദം സ്ഥാപിക്കുന്നതിനായി വിമര്‍ശകര്‍ ഉദ്ധരിക്കുന്ന ഖുര്‍ആന്‍ സൂക്തങ്ങളുടെ സാരമാണ് താഴെ: ”നഗരത്തിലെ ചില സ്ത്രീകള്‍ പറഞ്ഞു: പ്രഭു(അല്‍-അസീസ്)വിന്റെഭാര്യ തന്റെ വേലക്കാരനെ വശീകരിക്കാന്‍ ശ്രമിക്കുന്നു. അവള്‍ അവനോട് പ്രേമബദ്ധയായിക്കഴിഞ്ഞിരിക്കുന്നു. തീര്‍ച്ചയായും അവള്‍ വ്യക്തമായ പിഴവില്‍ അകപ്പെട്ടതായി ഞങ്ങള്‍ കാണുന്നു” (വി.ഖു: 12:30). ”(ആ സ്ത്രീകളെ) വിളിച്ചുവരുത്തിയിട്ട് അദ്ദേഹം (രാജാവ്) ചോദിച്ചു.യൂസുഫിനെ വശീകരിക്കുവാന്‍ നിങ്ങള്‍ ശ്രമം നടത്തിയപ്പോള്‍ നിങ്ങളുടെ സ്ഥിതി എന്തായിരുന്നു? അവര്‍ പറഞ്ഞു. അല്ലാഹു എത്ര പരിശുദ്ധന്‍. ഞങ്ങള്‍ യൂസുഫിനെപ്പറ്റി ദോഷകരമായ ഒന്നും മനസിലാക്കിയിട്ടില്ല. പ്രഭു(അല്‍-അസീസ്)വിന്റെ ഭാര്യ പറഞ്ഞു: ഇപ്പോള്‍ സത്യം വെളിപ്പെട്ടിരിക്കുന്നു. ഞാന്‍ അദ്ദേഹത്തെ വശീകരിക്കുവാന്‍ ശ്രമിക്കുകയാണുണ്ടായത്. തീര്‍ച്ചയായും അദ്ദേഹം സത്യവാന്മാരുടെ കൂട്ടത്തില്‍തന്നെയാകുന്നു” വി.ഖു: 12:51). ഈ സൂക്തങ്ങളില്‍ പ്രഭുവെന്ന് പരിഭാഷപ്പെടുത്തിയിരിക്കുന്നത് ‘അല്‍അസീസ്‘ എന്ന അറബി പദത്തെയാണ്. ചില ഇംഗ്ലീഷ് പരിഭാഷകളില്‍ ഈ പദം ഭാഷാന്തരം ചെയ്യാതെ അല്‍-അസീസ് എന്നുതന്നെ അതേപോലെ ഒരു നാമമെന്ന രൂപത്തില്‍ കൊടുത്തിട്ടുണ്ട്. അത് എടുത്തുകൊണ്ടാണ് പോത്തിഫറിന് പകരമായി അസീസ് എന്നാണ് ഖുര്‍ആനില്‍ പ്രയോഗിച്ചിരിക്കുന്നതെന്ന് വിമര്‍ശകര്‍ വാദിക്കുന്നത്. അസീസ് എന്ന അറബി നാമത്തിന്റെ അര്‍ത്ഥം ‘പ്രതാപി‘ യെന്നാണ്. അറബികള്‍ ഇത് ഒരു പേരായും പേരിന്റെ ഭാഗമായും ഉപയോഗിക്കാറുണ്ടെന്നത് ശരിയാണ്. അമുസ്‌ലിംകളും ഈ പേര് ഉപയോഗിക്കാറുണ്ട്. ഇറാഖി മന്ത്രിയായ താരീഖ് അസീസിന്റെ പേര് ഉദാഹരണം. അസീസിന്റെ മുമ്പില്‍ ‘അല്‍’ എന്ന അവ്യയം (definite article) ചേര്‍ത്ത് അല്‍-അസീസ് എന്നാണ് നടേ സൂചിപ്പിക്കപ്പെട്ട ഖുര്‍ആന്‍ സൂക്തങ്ങളില്‍ പ്രയോഗിച്ചിരിക്കുന്നത്. ഒരു പേര് എന്ന നിലയ്ക്ക് അറബിയില്‍ അല്‍-അസീസ് എന്നുപയോഗിക്കാറില്ല. ഇംഗ്ലീഷില്‍ the Lord എന്ന് പറയുന്നതിന് സമാനമായ ഒരു പ്രയോഗമാണിത്. അതുകൊണ്ടാണ് മലയാളത്തില്‍ ‘പ്രഭു‘വെന്ന് പരിഭാഷപ്പെടുത്തിയിരിക്കുന്നത്. വ്യക്തിയുടെ സ്ഥാനമാണ് അല്ലാതെ പേരല്ല അല്‍-അസീസ് എന്ന പ്രയോഗം ദ്യോതിപ്പിക്കുന്നത്. ബൈബിളില്‍ പോത്തിഫര്‍ എന്ന് വിളിച്ച വ്യക്തിയുടെ പേര് സൂചിപ്പിച്ചുകൊണ്ടുള്ള ഒരു പ്രയോഗമെന്ന നിലയ്ക്കല്ല, പ്രത്യുത അദ്ദേഹത്തിന്റെ സ്ഥാനം വ്യക്തമാക്കുന്ന ഒരു പദമെന്ന നിലയ്ക്കാണ് ഖുര്‍ആനില്‍ അല്‍-അസീസ് എന്ന് പറഞ്ഞിരിക്കുന്നത് എന്നര്‍ത്ഥം. പോത്തിഫറിനെക്കുറിച്ച് ബൈബിളില്‍ പറഞ്ഞിരിക്കുന്നത്” ഫറോവാന്റെ ഒരു ഉദ്യോഗസ്ഥനും അംഗരക്ഷക സേനയുടെ നായകനു” (ഉല്‍ 39:1)മെന്നാണല്ലോ. ഈ ഉന്നതസ്ഥാനം വ്യക്തമാക്കാനാണ് ഖുര്‍ആന്‍ ”അല്‍-അസീസ്” എന്ന് പ്രയോഗിച്ചതെന്നാണ് മനസ്സിലാവുന്നത്. ബൈബിളില്‍ പ്രയോഗിച്ച പോത്തിഫര്‍ എന്ന നാമം എന്തുകൊണ്ടാണ്ഖുര്‍ആനില്‍ ഉപയോഗിക്കാതിരുന്നത് എന്ന് ഖണ്ഡിതമായിപ്പറയാന്‍ നമുക്കാവില്ല. സര്‍വ്വശക്തന്റെ വചനങ്ങളുടെ സമാഹാരമാണ് ഖുര്‍ആന്‍. അതിലെ ഓരോ പദവും ഉപയോഗിച്ചതിനു പിന്നില്‍ എന്തെന്ത് യുക്തികളാണുള്ളതെന്ന് പറയാന്‍ പരിമിതമായ അറിവ് മാത്രം നല്‍കപ്പെട്ട നമുക്ക് കഴിയില്ല. ഇക്കാര്യത്തില്‍ സ്ഥിരപ്പെട്ട ഹദീസുകളോ പണ്ഡിതാഭിപ്രായങ്ങളോ ഉദ്ധരിക്കപ്പെട്ടതായി കാണാനും കഴിയുന്നില്ല. ‘പോത്തിഫര്‍’ എന്ന് ഖുര്‍ആനില്‍ പറയാതിരുന്നതിന്റെ കാരണം കൃത്യമായി സര്‍വ്വശക്തനേ അറിയൂ. എങ്കിലും, ‘പോത്തിഫര്‍’ എന്ന ഈജിപ്ഷ്യന്‍ നാമത്തിന്റെ അര്‍ത്ഥമെന്താണെന്ന് മനസ്സിലാക്കുമ്പോള്‍ ഖുര്‍ആനില്‍ പദങ്ങള്‍ പ്രയോഗിച്ചതിലുള്ള സൂക്ഷ്മത നമുക്ക് ഒരിക്കല്‍കൂടി ബോധ്യപ്പെടുമെന്നകാര്യം ഇവിടെ ചൂണ്ടിക്കാട്ടാമെന്ന് തോന്നുന്നു. പോത്തിഫര്‍ എന്ന ഈജിപ്ഷ്യന്‍ പദത്തിനര്‍ത്ഥം ‘റേ‘ യാല്‍ നല്‍കപ്പെട്ടവന്‍” (he whom Re has given-Harpers Bible Dictionary P. 809) ഈജിപ്തുകാരുടെ സൂര്യദേവന്റെ പേരാണ്‘റേ‘ സൂര്യദേവന്റെ ദാനം‘ എന്ന് വേണമെങ്കില്‍ ”പോത്തിഫര്‍” എന്ന പേര് പരിഭാഷപ്പെടുത്താം. സൂര്യാരാധനയുമായി ബന്ധപ്പെട്ടതാണ് ഈ പേര്. ബഹുദൈവാരാധനയുമായി ബന്ധപ്പെടുന്ന നാമങ്ങളോ പ്രയോഗങ്ങളോ ഉപയോഗിക്കുവാന്‍ പാടില്ലെന്നാണ് ഇസ്‌ലാമിന്റെ അനുശാസന. ഇതുകൊണ്ടുകൂടിയായിരിക്കാം ഖുര്‍ആന്‍ ‘പോത്തിഫര്‍’ എന്ന പേരുപയോഗിക്കാതെ അദ്ദേഹത്തിന്റെ സ്ഥാനത്തെ ദ്യോതിപ്പിക്കുന്ന ‘അല്‍-അസീസ് എന്നുമാത്രം പ്രയോഗിച്ചത്. ഖുര്‍ആന്‍ ‘പോത്തിഫര്‍’ എന്ന പേര് ഉപയോഗിക്കുകയാണെങ്കില്‍ ‘സൂര്യദേവന്റെ ദാന‘മാണ് അദ്ദേഹമെന്ന് അംഗീകരിക്കുന്നുവെന്നാണല്ലോ വന്നുചേരുക. ദൈവിക വചനങ്ങളുടെ സമാഹാരമാണ് ഖുര്‍ആന്‍ എന്ന വസ്തുത ഇവിടെയും വ്യക്തമായി വെളിപ്പെടുന്നു. ബൈബിളില്‍ നിന്ന് മുഹമ്മദ് നബി(ﷺ) പകര്‍ത്തിയെഴുതിയതായിരുന്നു യോസേഫിന്റെ കഥയെങ്കില്‍ സ്വാഭാവികമായും പോത്തിഫര്‍ എന്ന യജമാനനാമം ഖുര്‍ആനില്‍ സ്ഥാനം പിടിക്കേണ്ടതായിരുന്നു. പോത്തിഫര്‍ എന്നാല്‍ ”സൂര്യദേവന്റെ ദാനം”എന്നാണ് അര്‍ത്ഥമെന്ന് മുഹമ്മദ് നബിക്ക് അറിയാമായിരുന്നുവെന്നും അത് ബഹുദൈവ വിശ്വാസമുള്‍ക്കൊള്ളുന്നതായതിനാല്‍ ഖുര്‍ആനില്‍നിന്ന് ഒഴിവാക്കിയെന്നും കരുതുന്നത് ശുദ്ധ ഭോഷ്‌ക്കാണെന്ന് പറയേണ്ടതില്ല. ഖുര്‍ആനിന്റെ അവതരണ കാലത്ത് ഈജിപ്ഷ്യന്‍ ഭാഷ അറിയാവുന്നവരായി ആരും തന്നെ അറേബ്യയിലുണ്ടായിരുന്നില്ലെന്ന വസ്തുത ഓര്‍ക്കുക. അതേപോലെതന്നെ പോത്തിഫര്‍ എന്ന നാമമൊഴിവാക്കിയത് യാദൃച്ഛികമാണെന്ന് കരുതാനും ന്യായമില്ല. ബൈബിളില്‍നിന്ന് കാര്യങ്ങള്‍ പകര്‍ത്തിയെഴുതുമ്പോള്‍ അതില്‍ പല തവണ പ്രയോഗിക്കപ്പെട്ട ഒരു പേര് യാദൃച്ഛികമായി ഒഴിവായിപ്പോയെന്ന് കരുതുന്നതെങ്ങനെ? ഖുര്‍ആനിന്റെ ദൈവികതയ്ക്കുള്ള തെളിവാണിതെന്ന് പറയുന്നത് അതുകൊണ്ടാണ്. സര്‍വ്വജ്ഞനായ തമ്പുരാന്റെ വചനങ്ങളിലെവിടെയും യാതൊരു സ്ഖലിതവുമുണ്ടാവുകയില്ലെന്ന വസ്തുത ഇവിടെ നമുക്ക് ബോധ്യപ്പെടുന്നു. ഖുര്‍ആനിനെതിരെ ഉന്നയിക്കപ്പെട്ട എല്ലാ ആരോപണങ്ങളും ഖുര്‍ആനിന്റെ ദൈവികത വ്യക്തമാവുന്നിടത്തേക്കാണ് നമ്മെ നയിക്കുന്നതെന്നുള്ളത് എന്തുമാത്രം വിസ്മയകരമായിരിക്കുന്നു!

ഖുര്‍ആനില്‍ ചരിത്രപരമായ അബദ്ധങ്ങളുണ്ടെന്ന് സ്ഥാപിക്കുവാന്‍ ശ്രമിക്കുന്ന ക്രിസ്തുമത പ്രചാരകര്‍ സ്ഥിരമായി ഉന്നയിക്കുന്ന ഒരുവിമര്‍ശനമാണിത്. മോശയുടെ സഹോദരനായിരുന്ന ഹാറൂണി ന്റെസഹോദരിയാണ് മര്‍യമെന്ന് തെറ്റിദ്ധരിച്ച മുഹമ്മദ് നബി(ﷺ) യുടെരചനയിലുണ്ടായ ഒരു അബദ്ധമാണ് ഖുര്‍ആനിലെ സൂറത്തു മര്‍യമിലുള്ളതെ്(19:27-28)ന്നാണ വാദം. മോശയുടെയും അഹറോന്റെയും സഹോദരിയായ മിറിയാമിനെ പഴയ നിയമം പരിചയപ്പെടുത്തുന്നുണ്ട് (പുറപ്പാട് 15:20-21;ആവര്‍ത്തനം 24:9). അഹറോന്റെ സഹോദരിയായ മിറിയാമും യേശുമാതാവായ മറിയാമും തമ്മില്‍ മുഹമ്മദ് നബിക്ക് മാറിപ്പോയതാണെന്നും അതുവഴിയാണ് മറിയമിനെ ഹാറൂണിന്റെ സഹോദരിയായി അവതരിപ്പിച്ചതെന്നുമാണ് മിഷനറിമാരുടെ പ്രധാനപ്പെട്ടവാദം. ഈ വാദം എത്രമാത്രം വസ്തുനിഷ്ഠമാണെന്ന് ഖുര്‍ആനില്‍ ഹാറൂനി(അ)നെയും മര്‍യമിനെയും പറ്റി പരാമര്‍ശിക്കുന്ന സൂക്തങ്ങളിലൂടെഒരാവര്‍ത്തി കടന്നുപോയാല്‍ തന്നെ സുതരാം വ്യക്തമാവും. ഹാറൂന്‍ (അ) മൂസാ നബി(ﷺ )യുടെ സഹോദരനും ഫിര്‍ഔനിന്റെഅടുത്തേക്ക് നിയോഗിക്കപ്പെട്ട പ്രവാചകനുമാണെന്ന് വ്യക്തമാക്കുന്നനിരവധി ഖുര്‍ആന്‍ സൂക്തങ്ങളുണ്ട്. 2:248, 4:163, 6:84, 7:122, 7:142, 7:150-151, 10:75, 10:87, 10:89, 19:28, 19:53, 20:30-35, 20:42, 20:70, 20:92-94, 21:48, 21:23, 21:45, 24-35-36, 26:13, 26:48, 28:34-35, 37:114-122 എന്നീ സൂക്തങ്ങള്‍ ഇവയില്‍ ചിലത്മാത്രം. ഇസ്രായീലിലേക്ക് നിയോഗിക്കപ്പെട്ട പ്രഥമ പ്രവാചകനായി മൂസാ(u)യെ അവതരിപ്പിക്കുന്ന ഖുര്‍ആന്‍ സൂക്തങ്ങളും നിരവധിയുണ്ട്. 2:108, 2:248, 4:153, 5:20-26, 6:84, 14:8, 19:51-53, 22:44, 25:35-36, 28:3, 37:114-122, 18:60-82, 20:9-48, 27:7-12, 28:29-35, 79:15-19, 7:109-126, 10:79-82, 20:56-73, 26:38-51, 33:7, 2:53, 2:87, 2:136, 3:84, 6:91, 6:154, 7:144-145, 10:87, 11:110, 14:5, 17:2, 21:48 തുടങ്ങിനിരവധി സൂക്തങ്ങള്‍! മോശെ പ്രവാചകന് ശേഷം തലമുറകള്‍ കഴിഞ്ഞാണ്യേശു ക്രിസ്തുവിന്റെ നിയോഗമെന്ന് സൂചിപ്പിക്കുന്ന ഖുര്‍ആന്‍സൂക്തങ്ങളും നിരവധിയുണ്ട്. 3:49-51, 5:46, 5:72, 43:59, 61:6, 61:14, 4:171, 5:75, 42:13, 4:172, 19:30, 43:64, 43:63, 3:52-54, 5:11-113, 57:27, 61:14 എന്നീ സൂക്തങ്ങള്‍ഇക്കാര്യം വ്യക്തമാക്കുന്നുണ്ട്. യേശുവിന്റെ മാതാവാണ് മര്‍യമെന്ന്വ്യക്തമാക്കുന്ന സൂക്തങ്ങളും അനവധിയുണ്ട്. 3:35-37, 19:22-26, 21:91, 66:12, 5:75, 21:91, 4:156, 4:171, 5:116, 21:91, 3:42-51, 19:16-21 എന്നിവ ഉദാഹരണം. ഈസൂക്തങ്ങള്‍ വ്യക്തമാക്കുന്ന ഒരു യാഥാര്‍ത്ഥ്യമുണ്ട്. ഖുര്‍ആന്‍അവതരിപ്പിച്ചവന് ഹാറൂനിന്റെ സഹോദരിയല്ല മര്‍യമെന്ന്അറിയാമായിരുന്നുവെന്ന വസ്തുതയാണത്. യേശുവിന് തലമുറകള്‍ക്ക് മുമ്പ്ജീവിച്ച വ്യക്തിയായി നിരവധി സൂക്തങ്ങളില്‍പരിചയപ്പെടുത്തപ്പെട്ടിരിക്കുന്ന ഹാറൂനിന്റെ സഹോദരിയായിരുന്നു യേശുമാതാവായ മര്‍യമെന്ന് തെറ്റിദ്ധരിച്ച് എഴുതിയതല്ല ‘ഹാറൂനിന്റെസഹോദരി‘യെന്ന സംബോധനയെന്ന് ഇതില്‍നിന്ന് വ്യക്തമാണ്.

പിന്നെയെന്തുകൊണ്ടാണ് ഖുര്‍ആനില്‍ മര്‍യത്തെ സൂചിപ്പിച്ചുകൊണ്ട്‘ഹാറൂനിന്റെ സഹോദരി‘യെന്ന അഭിസംബോധനയുണ്ടായത്? മര്‍യത്തെഹാറൂനിന്റെ സഹോദരിയായി അഭിസംബോധനചെയ്യുന്ന സൂക്തം കാണുക: ”അനന്തരം കുഞ്ഞിനെയും വഹിച്ചുകൊണ്ട് അവള്‍ തന്റെ ആളുകള്‍ക്കടുത്ത്ചെന്നു. അവര്‍ പറഞ്ഞു: മര്‍യമേ….. ആക്ഷേപകരമായ ഒരുകാര്യംതന്നെയാണ് നീ ചെയ്തിരിക്കുന്നത്. ഹാറൂനിന്റെ സഹോദരീ……നിന്റെ പിതാവ് ഒരു ചീത്തമനുഷ്യനായിരുന്നില്ല. നിന്റെ മാതാവ് ഒരുദുര്‍ന്നടപടിക്കാരിയുമായിരുന്നില്ല” (ഖുര്‍ആന്‍ 19:27,28).

ഈ സൂക്തങ്ങളില്‍ യേശുവിന്റെ ജനനത്തോടനുബന്ധിച്ചഇസ്രായീല്യരുടെ പ്രതികരണമാണ് രേഖപ്പെടുത്തിയിരിക്കുന്നത്. ‘ഹാറൂനിന്റെ സഹോദരീ……’യെന്നത് ഖുര്‍ആനിന്റെ ഒരു കേവലപ്രസ്താവനയല്ല; പ്രത്യൂത, അക്കാലത്തെ ഇസ്രായീല്യരുടെഅഭിസംബോധനയെ ഖുര്‍ആന്‍ ഉദ്ധരിക്കുകയാണെന്ന കാര്യം പ്രത്യേകംശ്രദ്ധേയമാണ്. അതുകൊണ്ടുതന്നെ യേശുവിന്റെ ജനനകാലത്ത് ഇത്തരം വല്ലഅഭിസംബോധനാ രീതിയും നിലവിലുണ്ടായിരുന്നോ എന്ന അന്വേഷണംഏറെ പ്രസക്തമാണ്. അങ്ങനെയൊരു അന്വേഷണം നടത്തുമ്പോള്‍ഖുര്‍ആനിന്റെ ചരിത്രപരതയെ സംശയിക്കുവാനായി മെനഞ്ഞുണ്ടാക്കിയഈ വാദം അതിന്റെ അമാനുഷികതയ്ക്കും അജയ്യതയ്ക്കുമുള്ളതെളിവാണെന്ന വസ്തുതയാണ് ബോധ്യപ്പെടുക.

മോശ കഴിഞ്ഞാല്‍ പിന്നെ പഞ്ചപുസ്തകങ്ങളില്‍ ഏറ്റവുമധികംനിറഞ്ഞുനില്‍ക്കുന്ന വ്യക്തിയാണ് അഹറോണ്‍. മോശയുടെ സഹോദരനുംസഹായിയുമായ പ്രവാചകന്‍. അദ്ദേഹത്തെയും മക്കളെയുമാണ് യഹോവപൗരോഹിത്യത്തിനുവേണ്ടി തെരഞ്ഞെടുത്തത്. ‘നിന്റെ സഹോദരനായഅഹരോനെയും അയാളുടെ പുത്രന്മാരായ നാദാബ്്, അബിഹൂ,ലെയാസാര്‍, ഈതാമാര്‍ എന്നിവരെയും എനിക്ക് പുരോഹിതരായിസേവനം അനുഷ്ഠിക്കാന്‍ ഇസ്രായേല്‍ മക്കളുടെ ഇടയില്‍നിന്ന് നിന്റെഅടുത്തേക്ക് വിളിക്കുക” (പുറ: 28:1) എന്ന മോശയോടുള്ള യഹോവയുടെകല്‍പനയുടെ അടിത്തറയിലാണ് യഹൂദ പൗരോഹിത്യംരൂപപ്പെട്ടിരിക്കുന്നത്. പ്രധാന പുരോഹിതനായി കരുതപ്പെട്ട അഹരോന്റെപുത്രന്മാരെയും (ലേവ്യ 1:7) ഭവനത്തെയും (സങ്കീ 115:10) സൂചിപ്പിക്കാന്‍അഹറോണ്‍ വംശജര്‍ (Aaronites) എന്ന് പ്രയോഗിച്ചതായി കാണാം.പൗരോഹിത്യം അഹറോണില്‍നിന്ന് ആരംഭിച്ചതാണെന്നാണ് ഈ പ്രയോഗംവ്യക്തമാക്കുന്നതെന്ന് സൂചിപ്പിക്കപ്പെടുന്നു. പഴയ നിയമത്തിന്റെപൗരോഹിത്യ കൂട്ടായ്മയെ 24 ഗണങ്ങളായി വിഭജിച്ചിരുന്നു. ഓരോഗണവും ഓരോ ആഴ്ചയില്‍ ദേവാലയ ശുശ്രൂഷയ്ക്ക് നേതൃത്വം വഹിച്ചിരുന്നു (1 ദിനവൃത്താന്തം 24:1-19) ഈ പുരോഹിത ഗണത്തില്‍പെട്ടഒരാളായിരുന്നു സ്‌നാപകയോഹന്നാന്റെ പിതാവായ സഖറിയാ. (ലൂക്കാ1:5-9). പൗരോഹിത്യഗണത്തില്‍പെട്ട വിശിഷ്ട വ്യക്തികളെ അഹറോനോട്ചേര്‍ത്ത് വിളിക്കുന്ന സമ്പ്രദായം അന്ന് നിലവിലുണ്ടായിരുന്നു.സഖറിയായുടെ ഭാര്യയായ എലിസബത്തിനെക്കുറിച്ച് ലൂക്കോസ് പറയുന്നത്ഇങ്ങനെയാണ്: ”അഹറോന്റെ പുത്രിമാരില്‍ ഒരുവളായിരുന്നു അവന്റെഭാര്യ: അവളുടെ പേര് എലിസബത്ത്” (ലൂക്കാ1:5). അഹറോണുശേഷംനൂറ്റാണ്ടുകള്‍ കഴിഞ്ഞ് ജീവിച്ച വ്യക്തിയാണ് എലിസബത്ത്. എന്നിട്ടുംബൈബിള്‍ അവരെ ക്കുറിച്ച് പറയുന്നത് ”അഹറോന്റെ പുത്രി”യെന്നാണ്.യേശുവിന്റെ ജനനത്തിന് തൊട്ടുമുമ്പാണ് യോഹന്നാന്റെ ജനനം.അക്കാലത്തെ ഒരു പൊതുവായ പ്രയോഗശൈലിയായിരുന്നു വിശിഷ്ടവ്യക്തികളെ അഹറോന്റെ പേരുമായി ബന്ധപ്പെടുത്തിക്കൊണ്ട്പരാമര്‍ശിക്കുന്ന ശൈലിയെന്ന വസ്തുത എലിസബത്തിനെ ‘അഹറോന്റെപുത്രി‘യെന്ന് വിളിച്ചതില്‍നിന്ന് വ്യക്തമായി മനസ്സിലാക്കാനാവും. യേശുവിന്റെ മാതാവായ മര്‍യം എലിസബത്തിന്റെ ബന്ധുവായിരുന്നുവെന്ന് ബൈബിള്‍ പറയുന്നുണ്ട് (ലൂക്കോസ് 1:36).മര്‍യത്തെക്കുറിച്ച് ബൈബിളിലില്ലാത്ത ഒട്ടനവധി വിവരങ്ങള്‍ ഖുര്‍ആന്‍ നല്‍കുന്നുണ്ട്. മര്‍യം ഗര്‍ഭത്തിലായിരുന്നപ്പോള്‍ മാതാവ് അവരെനേര്‍ച്ചയാക്കിയതും പ്രതീക്ഷയ്ക്ക് വിപരീതമായി ജനിച്ചത് പെണ്‍കുഞ്ഞായതും എങ്കിലും അവര്‍ നേര്‍ച്ച പൂര്‍ത്തീകരിക്കുവാന്‍ തീരുമാനിച്ചതും ബന്ധുവായിരുന്ന സകരിയ്യായെ അവരുടെ സംരക്ഷണച്ചുമതല ഏല്‍പിച്ചതുമെല്ലാം ഖുര്‍ആന്‍ വിശദീകരിക്കുന്നുണ്ട്. സകരിയ്യാ(അ)യുടെ സംരക്ഷണത്തില്‍ വളര്‍ന്നവളായിരുന്നു മര്‍യമെന്നര്‍ത്ഥം. അങ്ങനെയുള്ള മര്‍യമിനെക്കുറിച്ച് ഇസ്രായീല്യര്‍ ”ഹാറൂണിന്റെ സഹോദരി”യെന്ന് പറയാനുള്ള സാധ്യത നിഷേധിക്കാനാവില്ലല്ലോ. പുരോഹിത ഗണത്തിലെ പ്രധാനികളെ അഹരോണുമായി ബന്ധപ്പെടുത്തി അഭിസംബോധന ചെയ്യുന്ന രീതി ഇസ്രായീല്‍ സമൂഹത്തില്‍ നിലനിന്നിരുന്നുവെന്ന വസ്തുത നടേ നാം മനസ്സിലാക്കിയിട്ടുണ്ട്. പുരോഹിത പ്രധാനിയും പ്രവാചകനും ദേവാലയത്തിന്റെ സംരക്ഷണ ഉത്തരവാദിത്തം വഹിച്ച വ്യക്തിയുമായിരുന്ന സെഖറിയാവിന്റെ സംരക്ഷണത്തില്‍ കഴിയുന്ന മര്‍യത്തെ അവര്‍ ‘ഹാറൂണിന്റെ സഹോദരി‘യെന്ന് വിളിച്ചിരിക്കാന്‍ തീര്‍ച്ചയായും സാധ്യതയുണ്ട്. അതുകൊണ്ടു തന്നെയാണ് ഖുര്‍ആന്‍ ഇക്കാര്യം ഉദ്ധരിച്ചതെന്ന് വ്യക്തമാണ്. ഈ വസ്തുത നബി (ﷺ) തന്നെ വ്യക്തമാക്കിയിട്ടുള്ളതാണ്്. സഹീഹുമുസ്‌ലിം ഉദ്ധരിച്ചിരിക്കുന്ന ഒരു ഹദീസ് കാണുക: ”മുഗീ റത്തുബ്‌നുശുഅ്ബയില്‍ നിന്ന് നിവേദനം: ഞാന്‍ നജ്‌റാനില്‍ വന്നപ്പോള്‍ അവിടത്തെ ക്രൈസ്തവര്‍ എന്നോട് ചോദിച്ചു: ”നിങ്ങള്‍ ഖുര്‍ആനില്‍ മറിയത്തെക്കുറിച്ച്” ”ഹാറൂണിന്റെ സഹോദരീ”യെന്ന് വായിക്കുന്നു. എന്നാല്‍ മോശ യേശുവിന് ദീര്‍ഘകാലം മുമ്പ് ജനിച്ച വ്യക്തിയാണല്ലോ”. ഞാന്‍ ദൈവദൂതന്റെ (ﷺ)സന്നിധിയില്‍ മടങ്ങിയെത്തിയപ്പോള്‍ ഇക്കാര്യം അദ്ദേഹത്തോട് ചോദിച്ചു.അപ്പോള്‍ പ്രവാചകന്‍ (ﷺ) പറഞ്ഞു: പ്രവാചകന്മാരുടെയുംസജ്ജനങ്ങളുടെയും പേരിനോടൊപ്പം തങ്ങളുടെ പേര് ചേര്‍ത്തുവിളിക്കുന്നപതിവ് അവര്‍ക്കുണ്ടായിരുന്നു” (സഹീഹുമുസ്‌ലിം: കിത്താബുല്‍ അദബ്). മൂസാ(u)യുടെ സഹോദരനും പ്രവാചകനുമായിരുന്ന ഹാറൂനിന്റെപേരിനോട് ചേര്‍ത്ത് മര്‍യത്തെ വിളിച്ചതാണ് ‘ഹാറൂനിന്റെ സഹോദരീ‘യെന്ന വസ്തുത വ്യക്തമാക്കുകയാണ് പ്രവാചകന്‍ (ﷺ) ഇവിടെചെയ്തിരിക്കുന്നത്. പതിനാല് നൂറ്റാണ്ടുകള്‍ക്ക് മുമ്പ് മുഹ മ്മദ് നബി (ﷺ)തന്നെ മറുപടി പറഞ്ഞു കഴിഞ്ഞ വിമര്‍ശനമാണിതെന്നര്‍ത്ഥം. ഖുര്‍ആനില്‍ ചരിത്രാബദ്ധങ്ങള്‍ ആരോപിക്കുവാന്‍ ശ്രമിക്കുന്നവര്‍ കാണാന്‍ കൂട്ടാക്കാത്ത ഒരു വസ്തുതകൂടി ഇവിടെ വ്യക്തമാക്കേണ്ടതുണ്ട്.പുണ്യ പുരുഷന്മാരുടെ പേരിനോടൊപ്പം ചേര്‍ത്ത് പ്രധാന വ്യക്തികളെ വിളിക്കുന്ന സമ്പ്രദായം യേശുവിന്റെ കാലത്ത് വ്യാപകമായിരുന്നുവെന്ന വസ്തുത ബൈബിള്‍ വ്യക്തമാക്കുന്നതായി നാം മനസ്സിലാക്കി. എന്നാല്‍ ഇത്തരമൊരു സമ്പ്രദായം ക്രൈസ്തവര്‍ക്കിടയില്‍ നിലനിന്നിരുന്നില്ലെന്നാണ് നജ്‌റാനിലെ ക്രൈസ്തവരുടെ മുഗീറത്തുബ്‌നു ശുഅ്ബയോടുള്ള ചോദ്യത്തില്‍ നിന്ന് മനസ്സിലാകുന്നത്. പിന്നെയെങ്ങനെയാണ് അത്തരമൊരു സമ്പ്രദായം നിലനിന്നിരുന്നുവെന്ന് മുഹമ്മദ് നബി (ﷺ)ക്ക് മനസ്സിലായത്? യേശുവിന്റെ കാലത്ത് നിലനിന്ന രീതിയില്‍തന്നെ ഹാറൂനിന്റെ പേരിനോട് ചേര്‍ത്താണ് മര്‍യത്തെ ഇസ്രായീല്‍ പ്രമാണിമാര്‍ വിളിച്ചതെന്ന് മുഹമ്മദ് നബി (ﷺ) എങ്ങനെ അറിഞ്ഞു? ഈ ചോദ്യങ്ങള്‍ക്ക് ഒരേയൊരു ഉത്തരം മാത്രമേയുള്ളൂ. വിശുദ്ധ ഖുര്‍ആന്‍ പറഞ്ഞ ഉത്തരം: ”അദ്ദേഹം തന്നിഷ്ടപ്രകാരം സംസാരിക്കുന്നുമില്ല. അത് അദ്ദേഹത്തിന് ദിവ്യസന്ദേശമായിനല്‍കപ്പെടുന്ന ഒരു ഉല്‍ബോധനം മാത്രമാകുന്നു” ”(നബിയേ) നാം നിനക്ക് ബോധനം നല്‍കുന്ന അദൃശ്യ വാര്‍ത്തകളില്‍ പെട്ടതാകുന്നു അവയൊക്കെ. അവരില്‍ ആരാണ് മര്‍യത്തിന്റെ സംരക്ഷണം ഏറ്റെടുക്കേണ്ടതെന്ന് തീരുമാനിക്കുവാനായി അവര്‍ തങ്ങളുടെ അമ്പുകള്‍ ഇട്ടുകൊണ്ട് നറുക്കെടുപ്പ് നടത്തിയിരുന്ന സമയത്ത് നീ അവരുടെ അടുത്തുണ്ടായിരുന്നില്ലല്ലോ.അവര്‍ തര്‍ക്കത്തിലേര്‍പ്പെട്ടുകൊണ്ടിരുന്നപ്പോഴും നീ അവരുടെ അടുത്തുണ്ടായിരുന്നില്ല” (വി.ഖു. 3:44).

കുറെയധികം ചരിത്ര പരാമര്‍ശങ്ങളുണ്ട് ഖുര്‍ആനില്‍. ആദിമനുഷ്യനായ ആദമില്‍നിന്ന് തുടങ്ങി വിവിധ കാലങ്ങളില്‍ വിവിധ ദേശങ്ങളിലേക്ക്  നിയോഗിക്കപ്പെട്ട വ്യത്യസ്ത പ്രവാചകന്മാരുടെയും സമൂഹങ്ങളുടെയും ചരിത്രത്തില്‍നിന്നുള്ള ശകലങ്ങള്‍ ഖുര്‍ആനിലുടനീളം കാണാം. കാലാനുക്രമമായ രീതിയിലുള്ള ചരിത്രാഖ്യാനമോ കഥാകഥനമോ അവതരിപ്പിക്കുകയെന്നതിലുപരിയായി പഠിപ്പിക്കുവാനുദ്ദേശിക്കുന്ന ആശയത്തിന് ഉപോല്‍ബലകമായ ചരിത്രസംഭവങ്ങള്‍ വിവരിക്കുന്ന ഒരുസവിശേഷമായ രീതിയാണ് പൊതുവായി ഖുര്‍ആനില്‍ സ്വീകരിക്കപ്പെട്ടിരിക്കുന്നത്. സത്യാസത്യ വിവേചകമായ വേദഗ്രന്ഥം ധര്‍മവും അധര്‍മവുമെന്തെന്ന് വ്യവഛേദിച്ച് പഠിപ്പിക്കുന്നതിനിടെ ഉദാഹരണങ്ങളെന്ന വണ്ണമാണ് ചരിത്രശകലങ്ങള്‍ കടന്നുവരുന്നതെന്ന്വേണമെങ്കില്‍ പറയാം. വ്യത്യസ്ത ഘട്ടങ്ങളില്‍ വിവിധ സാഹചര്യങ്ങളിലായാണല്ലോ ഖുര്‍ആന്‍ സൂക്തങ്ങള്‍ അവതരിപ്പിക്കപ്പെട്ടത്. ഓരോ സാഹചര്യങ്ങളിലേക്കും ഉപയുക്തമായ രീതിയിലുള്ള ചരിത്രശകലങ്ങള്‍ പ്രസ്തുത സാഹചര്യത്തില്‍ അവതീര്‍ണമായ ഖുര്‍ആന്‍ സൂക്തത്തിലാണുണ്ടാവുക. എങ്കിലും പ്രസ്തുതചരിത്ര പരാമര്‍ശങ്ങളെല്ലാം അബദ്ധരഹിതവും വൈരുധ്യങ്ങളില്‍ നിന്ന് മുക്തവുമായി നിലനില്‍ക്കുന്നുവെന്നത് ഖുര്‍ആനിന്റെ അമാനുഷികതവ്യക്തമാക്കുന്നുണ്ട്.

എന്നോ കഴിഞ്ഞുപോയ സംഭവങ്ങളാണ് ഖുര്‍ആന്‍ സൂക്തങ്ങളില്‍പരാമര്‍ശിക്കപ്പെടുന്നത്. പ്രസ്തുത സംഭവങ്ങള്‍ക്ക് സാക്ഷിയായഒരാളിലൂടെയല്ല അത് ലോകം ശ്രവിക്കുന്നത്. ആ സംഭവങ്ങളെക്കുറിച്ചവ്യക്തവും സൂക്ഷ്മവുമായ അറിവ് മറ്റേതെങ്കിലും ഭൗതിക സ്രോതസ്സില്‍നിന്ന് അദ്ദേഹം സമ്പാദിച്ചിട്ടുമില്ല. ആ വ്യക്തിയിലൂടെ മാനവരാശിക്ക് അവതരിപ്പിക്കപ്പെട്ട സംഭവ വിവരണങ്ങളിലാവട്ടെ അബദ്ധങ്ങളൊന്നും കാണാന്‍ കഴിയുന്നുമില്ല. എങ്കില്‍ പിന്നെ ഈ സംഭവവിവരണങ്ങള്‍ അദ്ദേഹത്തിന്റെ സ്വന്തം വകയാണെന്ന് എങ്ങനെ സങ്കല്‍പിക്കാനാവും?അദ്ദേഹം പറയുന്നതാകട്ടെ സര്‍വ്വജ്ഞനായ തമ്പുരാനാണ് ഈ സംഭവവിവരണങ്ങളെല്ലാം അടങ്ങിയ ഖുര്‍ആന്‍ തനിക്ക് അവതരിപ്പിച്ച് തന്നതെന്നുമാണ്. അന്തിമ പ്രവാചകന്‍ (ﷺ) കളവ് പറഞ്ഞതായി സമകാലികരായ അവിശ്വാസികള്‍ പോലും ആരോപിച്ചിരുന്നുമില്ല. ഖുര്‍ആനിലെ സംഭവവിവരണങ്ങളുടെ സൂക്ഷ്മതയും കൃത്യതയും അതിന്റെ അമാനുഷികതയ്ക്കും ദൈവികതയ്ക്കും തെളിവാണെന്ന വസ്തുതയാണ് നമുക്ക് ഇവിടെ കാണാന്‍ കഴിയുന്നത്. ഇക്കാര്യം ഖുര്‍ആന്‍ തന്നെ സ്വയം വ്യക്തമാക്കുന്നുമുണ്ട്. യൂസുഫ് നബി (അ )യുടെ ചരിത്രംവിവരിച്ചുകൊണ്ട് ഖുര്‍ആന്‍ പറഞ്ഞു: ”(നബിയേ) നിനക്ക് നാം സന്ദേശമായിനല്‍കുന്ന അദൃശ്യവാര്‍ത്തകളില്‍ പെട്ടതത്രെ അത്. (യൂസുഫിനെതിരില്‍)തന്ത്രം പ്രയോഗിച്ചുകൊണ്ട് അവര്‍ തങ്ങളുടെ പദ്ധതി കൂടിത്തീരുമാനിച്ചപ്പോള്‍ നീ അവരുടെ അടുക്കല്‍ ഉണ്ടായിരുന്നില്ലല്ലോ” (വി.ഖു.12:102). പ്രവാചകന്മാരെ സംബന്ധിച്ചും മറ്റുമുള്ള കൃത്യമായസംഭവവിവരണങ്ങള്‍ സര്‍വ്വശക്്തന്‍ ബോധനം നല്‍കിയ അദൃശ്യ വര്‍ത്തമാനങ്ങളാണെന്നും പ്രസ്തുത വര്‍ത്തമാനങ്ങളുള്‍ക്കൊള്ളുന്ന പരിശുദ്ധ ഖുര്‍ആന്‍ ദൈവികമാണെന്നതിന് അവതന്നെ സ്വയം സാക്ഷിനില്‍ക്കുന്നുവെന്നും അര്‍ത്ഥം.

ഭര്‍ത്താവ് മരിച്ച സ്ത്രീ നാലുമാസവും പത്തു ദിവസവും ദുഃഖമാചരിക്കണമെന്ന് ഖുര്‍ആന്‍ അനുശാസിക്കുന്നുണ്ട്.”നിങ്ങളാരെങ്കിലും തങ്ങളുടെ ഭാര്യമാരെ വിട്ടേച്ചുകൊണ്ട് മരണപ്പെടുകയാണെങ്കില്‍ അവര്‍ തങ്ങളുടെ കാര്യത്തില്‍ നാലു മാസവും പത്തുദിവസവും കാത്തിരിക്കേണ്ടതാണ്. എന്നിട്ട് അവരുടെ ആ അവധിയെത്തിയാല്‍ തങ്ങളുടെ കാര്യത്തില്‍ അവര്‍ മര്യാദയനുസരിച്ചു പ്രവര്‍ത്തിക്കുന്നതില്‍ നിങ്ങള്‍ക്ക് കുറ്റമൊന്നുമില്ല” (2:224).എന്തിനാണത്?

വിധവയുടെ ദുഃഖാചരണത്തിന് പിന്നിൽ രണ്ടു ഉദ്ദേശ്യങ്ങളാണുള്ളത്. തന്റെ ജീവിത പങ്കാളിയുടെ വേര്‍പാടില്‍ ദുഃഖാചരണം നടത്തുകയും മറ്റൊരു വിവാഹത്തിലേർപ്പെടുവാനുള്ള മാനസികപക്വത നേടിയെടുക്കുകയുമാണ് ഒന്ന്. അന്തരിച്ച ഭര്‍ത്താവില്‍നിന്ന് താന്‍ ഗര്‍ഭം ധരിച്ചിട്ടുണ്ടോ എന്ന സംശയം ദുരീകരിക്കുകയാണ് മറ്റൊന്ന്.

ദുഃഖാചരണകാലത്ത് അവള്‍ ചെയ്യേണ്ടതെന്താണ്? ദുഃഖാചരണകാലത്ത് അവള്‍ വിവാഹിതയാകാന്‍ പാടില്ല. വിവാഹാലോചനകളും ഇക്കാലത്ത് വിലക്കപ്പെട്ടിരിക്കുന്നു. അഴകും മോടിയും കൂട്ടി പുരുഷന്മാരെ ആകര്‍ഷിക്കുകയോ സ്വമനസ്സില്‍ ലൈംഗികതൃഷ്ണ വളര്‍ത്തുകയോ ചെയ്തുകൂടാ. വര്‍ണശബളമായ ആടയാഭരണങ്ങള്‍ ധരിക്കുകയും ചായവും സുറുമയും ഉപയോഗിക്കുകയും സുഗന്ധദ്രവ്യങ്ങള്‍ പുരട്ടുകയും ചെയ്യുന്നതില്‍നിന്ന് ഇക്കാലത്ത് അവള്‍ വിലക്കപ്പെട്ടിരിക്കുന്നു. അത്യാവശ്യകാര്യങ്ങള്‍ക്കായി പുറത്തുപോകുന്നതിനെയോ മാന്യവും വൃത്തിയുള്ളതുമായ വസ്ത്രം ധരിക്കുന്നതിനെയോ നിരോധിച്ചതായി കാണാന്‍ കഴിയില്ല. ചുരുക്കത്തില്‍, ലൈംഗിക ചിന്തയുണ്ടാക്കുന്ന എല്ലാ കാര്യങ്ങളില്‍നിന്നും അകന്നുനില്‍ക്കാന്‍ ദുഃഖാചരണകാലത്ത് സ്ത്രീ ബാധ്യസ്ഥയാണ്.

ഭര്‍ത്താവ് മരിച്ച് നാലു മാസവും പത്തു ദിവസവും കഴിഞ്ഞാല്‍- ഗര്‍ഭിണിയാണെങ്കില്‍ പ്രസവിച്ചാല്‍- അവള്‍ക്ക് ഇഷ്ടമുള്ളതുപോലെ ചെയ്യാവുന്നതാണ്. ഒന്നുകില്‍ പുനര്‍വിവാഹം ചെയ്യാം. അല്ലെങ്കില്‍ തല്‍ക്കാലം വിവാഹം വേണ്ടെന്നു വെക്കാം. എല്ലാം അവളുടെ ഇഷ്ടത്തിന് വിടേണ്ടതാണ്.

അജ്ഞാന കാലത്ത് അറേബ്യയില്‍ വിധവകള്‍ ഒരു വര്‍ഷം ദുഃഖാചരണം നടത്തുമായിരുന്നു. അങ്ങേയറ്റം മലിനമായി വസ്ത്രം ധരിച്ച്, കുളിക്കുകയോ വൃത്തിയാവുകയോ ചെയ്യാതെയുള്ള ദുഃഖാചരണം. ഇതില്‍നിന്ന് പരിവര്‍ത്തനം ഉണ്ടാക്കുകയാണ് ഇസ്‌ലാം ചെയ്തത്.

ഭര്‍ത്താവ് മരിച്ച ഹൈന്ദവ സ്ത്രീ എന്തു ചെയ്യണം?

മനുസ്മൃതിയുടെ വിധി കാണുക:

കാമം തുക്ഷ പയേ ദ്ദേഹം പുഷ്പ മൂല ഫലൈഃ ശുഭൈഃ

ന തു നാമാഭി ഗൃഹ്ണീ യാത്പത്യൗ പ്രേത പരസ്യതു

ആസീതാ മരണാല്‍ ക്ഷാന്താ നിയതാ ബ്രഹ്മചാരിണീ

യോ ധര്‍മ്മ ഏകപത്‌നീ നാം കാംക്ഷന്തീ തമനുത്തമം (5:157, 158).

(ഭര്‍ത്താവ് മരിച്ച ശേഷം പരിശുദ്ധമായ കിഴങ്ങ്, ഫലം, പുഷ്പം മുതലായ ആഹാരങ്ങള്‍കൊണ്ട് ദേഹത്തിന് ക്ഷതം വരുത്തി കാലം നയിക്കേണ്ടതാണ്. കാമവികാരോദ്ദേശ്യത്തിന്മേല്‍ മറ്റൊരു പുരുഷന്റെ പേരുപറയരുത്. ഭര്‍ത്താവ് മരിച്ച ശേഷം ജീവാവസാനം വരെ സഹനശീലയായി പരിശുദ്ധയായി ബ്രഹ്മധ്യാനമുള്ളവളായും മധുമാംസഭക്ഷണം ചെയ്യാത്തവളായും ഉത്കൃഷ്ടയായ പതിവ്രതയുടെ ധര്‍മത്തെ ആഗ്രഹിക്കുന്നവളായും ഇരിക്കേണ്ടതാകുന്നു).

ഇത് മനുസ്മൃതിയുടെ വിധി. ഇന്ത്യയില്‍ നിലവിലുണ്ടായിരുന്ന അവസ്ഥ ഇതിലും ഭീകരമായിരുന്നു. ഭര്‍ത്താക്കന്മാര്‍ മരിച്ചാല്‍ അവരുടെ ചിതയില്‍ ചാടി മരിക്കണമെന്ന് സ്ത്രീ നിര്‍ദേശിക്കപ്പെട്ടിരുന്നു. ക്രൂരമായ സതി സമ്പ്രദായം! അത് അനുഷ്ഠിക്കുവാന്‍ വിസമ്മതിക്കുന്ന വിധവകള്‍ തലമൊട്ടയടിച്ച് സമൂഹത്തില്‍ ഒറ്റപ്പെട്ടു കഴിയണമായിരുന്നു. ശൈശവ വിവാഹത്തിന് ശേഷം വിധവകളാകുന്ന ആറും ഏഴും വയസ്സ് പ്രായമുള്ള പെണ്‍കിടാങ്ങള്‍പോലും തലമൊട്ടയടിച്ച് ജീവിതകാലം മുഴുവന്‍ ഭിക്ഷുണികളായി കഴിഞ്ഞുകൂടണമെന്നായിരുന്നു നിയമം. ഇവര്‍ക്ക് അനുവദിക്കപ്പെട്ടിരുന്നതോ ഒരു നേരത്തെ ഭക്ഷണം മാത്രം!

വിധവകളെ പുനര്‍വിവാഹത്തില്‍നിന്ന് ഖുര്‍ആന്‍ വിലക്കുന്നില്ല. അവര്‍ നാലു മാസവും പത്തു ദിവസവും കാത്തിരിക്കണമെന്നു മാത്രമാണ് അനുശാസിക്കുന്നത്. ഈ കാത്തിരിപ്പാകട്ടെ തികച്ചും ശാസ്ത്രീയവും സ്ത്രീക്ക് ഗുണം ചെയ്യുന്നതുമാണുതാനും. പുരുഷനിൽ നിന്ന് വ്യത്യസ്തമായി ഇണയായി ഒരാളെ മാത്രമേ സ്ത്രീക്ക് തന്റെ മനസ്സിൽ ഉൾക്കൊള്ളാൻ കഴിയൂ. പുതിയൊരു ദാമ്പത്യം വിജയകരമായാവണമെങ്കിൽ മനസ്സിലുള്ള ഇണയെ താഴെയിറക്കാൻ കഴിയണം. ഏറെക്കാലം കൂടെ ജീവിച്ച ഇണയെ മനസ്സില്നിന്നിറക്കി വെക്കാനും മറ്റൊരു ഇണയെ സ്വീകരിക്കുവാൻ മാനസികമായി ഒരുങ്ങാനും ദുഃഖാചരണം വഴി അവൾക്ക് കഴിയും.

ഭർത്താവിന്റെ മരണം നടന്ന ഉടനെ സ്ത്രീ വിവാഹിതയാവുകയും  ഗര്‍ഭിണിയായി അവര്‍ക്ക് കുഞ്ഞുണ്ടാവുകയുമാണെങ്കില്‍ അതിന്റെ പിതൃത്വത്തെക്കുറിച്ച് സംശയം ഉണ്ടാകുവാന്‍ സാധ്യതയുണ്ട്. ഈ സംശയം തന്റെ കുടുംബഭദ്രതയും മനസ്സമാധാനവും തകര്‍ക്കുന്നതിലേക്ക് നയിച്ചേക്കും. ഖുര്‍ആന്‍ പറഞ്ഞ പ്രകാരം കാത്തിരുന്ന ശേഷം പുനര്‍വിവാഹം ചെയ്യുന്ന സ്ത്രീ ഗര്‍ഭിണിയാകുമ്പോള്‍ ഈ പ്രശ്‌നം ഉദിക്കുന്നില്ല. അത് രണ്ടാം ഭര്‍ത്താവിന്റെ കുഞ്ഞുതന്നെയാണെന്ന് ഉറപ്പിക്കാനാവും. വിധവയുടെ ദുഃഖാചരണം സംബന്ധിച്ച ഖുര്‍ആനിക ഇദ്ദയുടെ നിയമവും സ്ത്രീക്ക് അനുഗുണമാണെന്നും പ്രയാസപ്പെടുത്താതിരിക്കാനുള്ളതാണെന്നുമുള്ള വസ്തുതയാണ് ഇവിടെ വ്യക്തമാകുന്നത്.

മുന്നോട്ടു പോകാൻ കഴിയാത്ത വിധം പരാജയമാണ് ദാമ്പത്യജീവിതമെങ്കിൽ വിവാഹമോചനത്തിന് ആവശ്യപ്പെടാൻ സ്ത്രീക്കും അവകാശമുണ്ട്. സ്ത്രീയുടെ വിവാഹമോചനം രണ്ടു തരമാണ്. ഖുല്‍ഉം ഫസ്ഖും.

തന്റെ ഭര്‍ത്താവിനെ വെറുക്കുകയും അയാളോടൊപ്പം ജീവിക്കുവാന്‍ ഇഷ്ടപ്പെടാതിരിക്കുകയും ചെയ്യുന്ന സ്ത്രീക്ക് അയാളോട് വിവാഹമോചനത്തിന് ആവശ്യപ്പെടാവുന്നതാണ്. ഇതാണ് ‘ഖുല്‍അ്‘.ഭര്‍ത്താവില്‍നിന്ന് ലഭിച്ച വിവാഹമൂല്യം തിരിച്ചുകൊടുക്കണമെന്നുള്ളതാണ് ‘ഖുല്‍ഇ‘നുള്ള നിബന്ധന. വിവാഹം വഴി ഭാര്യക്ക് ലഭിച്ച സമ്പത്ത് തിരിച്ചുകൊടുക്കണമെന്നര്‍ഥം. ഇക്കാര്യം വിവരിക്കുന്ന ഖുര്‍ആന്‍ സൂക്തം നോക്കുക: ”അങ്ങനെ അവര്‍ക്ക് (ദമ്പതികള്‍ക്ക്) അല്ലാഹുവിന്റെ നിയമപരിധികള്‍ പാലിക്കുവാന്‍ കഴിയില്ലെന്ന് നിങ്ങള്‍ക്ക് ഉത്കണ്ഠ തോന്നുകയാണെങ്കില്‍ അവള്‍ വല്ലതും വിട്ടുകൊടുത്ത് സ്വയം മോചനം നേടുന്നതിന് അവര്‍ ഇരുവര്‍ക്കും കുറ്റമില്ല. (2:229).

‘ഖുല്‍ഇ‘നുള്ള നിബന്ധനകള്‍ താഴെ പറയുന്നവയാണ്.

ഒന്ന്: ത്വലാഖിനെപ്പോലെതന്നെ അനിവാര്യമായ സാഹചര്യങ്ങളില്ലാതെ ഖുല്‍അ് ചെയ്യാന്‍ പാടില്ലാത്തതാകുന്നു. പ്രവാചകന്‍ (ﷺ) പറഞ്ഞു: ”പ്രയാസമുണ്ടാവുമ്പോഴല്ലാതെ ഭര്‍ത്താവില്‍നിന്ന് വിവാഹമോചനം ആവശ്യപ്പെടുന്ന സ്ത്രീക്ക് സ്വര്‍ഗത്തിന്റെ സുഗന്ധം പോലും നിഷിദ്ധമാണ്” (അബൂദാവൂദ്, തിര്‍മുദി).

രണ്ട്: സ്ത്രീ ഖുല്‍അ് ആവശ്യപ്പെട്ടാല്‍ അവളെ മോചിപ്പിക്കേണ്ടത് പുരുഷന്റെ ബാധ്യതയാണ്.

മൂന്ന്: താന്‍ നല്‍കിയ വിവാഹമൂല്യം പൂര്‍ണമായോ ഭാഗികമായോ ആവശ്യപ്പെടാന്‍ പുരുഷന് അവകാശമുണ്ട്. വിവാഹമൂല്യത്തില്‍ കവിഞ്ഞ യാതൊന്നും ആവശ്യപ്പെടാവതല്ല.

നാല്: താന്‍ ആവശ്യപ്പെട്ട തുക നല്‍കുന്നതോടുകൂടി ഖുല്‍അ് സാധുവായിത്തീരുന്നു. അഥവാ ആ സ്ത്രീ പുരുഷന്റെ ഭാര്യയല്ലാതായിമാറുന്നു.

ഇത്തരം വിവാഹമോചനങ്ങള്‍ പ്രവാചകന്റെ(ﷺ) കാലത്തു നടന്നതായി കാണാനാവും. താന്‍ ഇഷ്ടപ്പെടാത്ത ഭാര്യയെക്കൊണ്ട് ഖുല്‍അ് ചെയ്യിക്കുന്നതിനുവേണ്ടി അവളെ പ്രയാസപ്പെടുത്തുന്ന സമ്പ്രദായം നിലവിലുണ്ടായിരുന്നു. താന്‍ നല്‍കിയ വിവാഹമൂല്യം തിരിച്ചുവാങ്ങുന്നതിനുവേണ്ടിയായിരുന്നു അത്. ഖുര്‍ആന്‍ ഇൗ സമ്പ്രദായത്തെ ശക്തിയായി വിലക്കുന്നുണ്ട്.

”നിങ്ങള്‍ കൊടുത്തിട്ടുള്ളതില്‍ ഒരു ഭാഗം തട്ടിയെടുക്കാനായി നിങ്ങള്‍ അവരെ മുടക്കിയിടുകയും ചെയ്യരുത്” (4:19).

സ്ത്രീയുടെ രണ്ടാമത്തെ വിവാഹമോചന രീതിയാണ് ‘ഫസ്ഖ്‘.ഭാര്യയുടെ അവകാശങ്ങള്‍ നിഷേധിക്കുകയും അതോടൊപ്പം വിവാഹമോചനം നല്‍കാതിരിക്കുകയും ചെയ്യുന്ന പുരുഷന്മാരില്‍നിന്ന് ന്യായാധിപന്റെ സഹായത്തോടെ നേടുന്ന വിവാഹമോചനമാണിത്. ഭര്‍ത്താവിന് സന്താനോല്‍പാദനശേഷി ഇല്ലെന്ന് തെളിയുക,ലൈംഗികബന്ധത്തിന് സാധിക്കാതിരിക്കുക, അവിഹിത വേഴ്ചകളില്‍ മുഴുകുക, ക്രൂരമായി പെരുമാറുക, തന്നെ അധാര്‍മിക വൃത്തിക്ക് നിര്‍ബന്ധിക്കുക, ജീവിതത്തിന്റെ അടിസ്ഥാനാവശ്യങ്ങള്‍ നിഷേധിക്കുക,തന്റെ സ്വത്തുക്കള്‍ അന്യായമായി ഉപയോഗിക്കുക, ഒന്നിലധികം ഭാര്യമാരുള്ളയാളാണെങ്കില്‍ തന്നോട് നീതിപൂര്‍വം വര്‍ത്തിക്കാതിരിക്കുക, തുടങ്ങിയ അവസരങ്ങളില്‍ ഭാര്യക്ക് ന്യായാധിപന്‍ മുഖേന വിവാഹബന്ധം വേര്‍പെടുത്താവുന്നതാണ്. ഇതാണ് ഫസ്ഖ്. തന്റെ അനുവാദമില്ലാതെ രക്ഷാധികാരികള്‍ വിവാഹം ചെയ്തുകൊടുത്താലും ഭര്‍ത്താവ് എവിടെയാണെന്നറിയാത്ത സ്ഥിതി ഉണ്ടെങ്കിലും ഭാര്യക്ക് ഫസ്ഖ് ചെയ്യാവുന്നതാണ്.

ഫസ്ഖ് ചെയ്യുന്നത് ന്യായാധിപനിലൂടെയായിരിക്കണമെന്നുള്ളതാണ് അതിനുള്ള നിബന്ധന. ഭാര്യ ഉന്നയിക്കുന്ന കാരണങ്ങള്‍ ഫസ്ഖിന് പ്രേരിപ്പിക്കാവുന്ന തരത്തിലുള്ളതാണോ എന്ന് പരിശോധിക്കുന്നത് ന്യായാധിപനാണ്. അങ്ങനെയാണെങ്കില്‍ വിവാഹമൂല്യം തിരിച്ചുനല്‍കാതെതന്നെ അവള്‍ക്ക് അവനുമായുള്ള ബന്ധത്തില്‍നിന്ന് പിരിയാനുള്ള സംവിധാനമുണ്ട്.

അനിവാര്യമായ സാഹചര്യത്തിൽ ഇസ്‌ലാം വിവാഹമോചനം അനുവദിക്കുന്നുണ്ട്. പുരുഷന്‍ തന്റെ അധികാരമുപയോഗിച്ച് വിവാഹബന്ധം വേര്‍പെടുത്തുന്നതിനാണ് സാങ്കേതികമായി ത്വലാഖ് എന്നു പറയുന്നത്. ത്വലാഖിലെത്തിച്ചേരാതെ സൂക്ഷിക്കുവാന്‍ കഴിയുന്നത്ര ശ്രമിക്കണമെന്നാണ് ഖുര്‍ആനിന്റെ താല്‍പര്യം. പുരുഷന്‍ തന്റെ ഇണയെ ഇഷ്ടപ്പെടുന്നില്ലെങ്കില്‍തന്നെ സാധ്യമാകുന്നത്ര അവളോടൊത്തു ജീവിക്കുവാന്‍ പരിശ്രമിക്കണമെന്നാണ് അത് അനുശാസിക്കുന്നത്. ”അവരോട് നിങ്ങള്‍ മര്യാദയോടെ സഹവര്‍ത്തിക്കുകയും ചെയ്യുക. നിങ്ങള്‍ക്ക് അവരോട് വെറുപ്പ് തോന്നിയേക്കാം. എന്നാല്‍, നിങ്ങള്‍ക്ക് വെറുപ്പ് തോന്നുന്ന ഒന്നില്‍ തന്നെ അല്ലാഹു ധാരാളം നന്മ നിശ്ചയിച്ചിരിക്കുകയും ചെയ്യാം” (4:19)

ദമ്പതിമാര്‍ക്കിടയില്‍ ഐക്യം നിലനിര്‍ത്താന്‍ ആവുന്നതൊക്കെ ചെയ്യേണ്ടതുണ്ടെന്നാണ് ഖുര്‍ആനിന്റെ നിലപാട്. എന്നാല്‍, സ്‌നേഹവും ഐക്യവും ഇല്ലാതായിത്തീരുകയും വൈവാഹിക ജീവിതത്തിന്റെ ലക്ഷ്യങ്ങള്‍ സാക്ഷാത്കരിക്കപ്പെടാതിരിക്കുകയും ചെയ്യുന്ന അവസ്ഥ സംജാതമായാല്‍ അവര്‍ തമ്മില്‍ വേര്‍പിരിയുന്നതിന് വിരോധമില്ല. ഈ വേര്‍പിരിയലിന് പുരുഷന്‍ മുന്‍കൈയെടുക്കുമ്പോള്‍ അതിന് ത്വലാഖ് എന്നു പറയുന്നു. സ്ത്രീയാണ് മുന്കയ്യെടുക്കുന്നതെങ്കിൽ അത് ഖുൽഅ എന്നും ന്യായാധിപനിലൂടെ നടക്കുന്നതാണെങ്കിൽ ഫസ്ഖ് എന്നുമാണ് അറിയപ്പെടുക.

ആര്‍ത്തവ സമയത്ത് സ്ത്രീയെ ത്വലാഖ് ചെയ്യുന്നത് ഇസ്‌ലാം വിലക്കിയിട്ടുണ്ട്. ഇക്കാലത്ത് സ്ത്രീയുടെ ശാരീരിക-മാനസിക നിലകളില്‍ സ്പഷ്ടമായ മാറ്റമുണ്ടാവുമെന്ന കാര്യം തെളിയിക്കപ്പെട്ടതാണ്. അവള്‍ക്ക് ശുണ്ഠിയും മറവിയും കൂടുതലായിരിക്കും. അക്കാരണത്താല്‍തന്നെ ആര്‍ത്തവകാലത്ത് തമ്മില്‍ പിണങ്ങാനും സാധ്യത കൂടുതലാണ്. ഈ പിണക്കം വിവാഹമോചനത്തിലേക്ക് നയിച്ചുകൂടാ. ദമ്പതികള്‍ തമ്മില്‍ താല്‍പര്യവും ആഭിമുഖ്യവുമുണ്ടാക്കുവാനുതകുന്ന ലൈംഗികബന്ധം ഇക്കാലത്ത് നിഷിദ്ധവുമാണ്. പിണക്കമെല്ലാം തീരുന്നത് കിടപ്പറയില്‍ വെച്ചാണല്ലോ. ആര്‍ത്തവകാലത്തുണ്ടാകുന്ന പിണക്കം തീരാന്‍ ശുദ്ധിയായതിന് ശേഷമുള്ള ലൈംഗികബന്ധം മതിയാവും. അതുകൊണ്ടുതന്നെ ആര്‍ത്തവകാലത്ത് ഭാര്യയെ മോചിപ്പിക്കുന്നത് ശരിയല്ലെന്നും അങ്ങനെ മോചിപ്പിച്ചവര്‍ അവളെ തിരിച്ചെടുക്കേണ്ടതുണ്ടെന്നും പ്രവാചകന്‍ (ﷺ) പഠിപ്പിച്ചിട്ടുണ്ട്.

ശുദ്ധികാലത്ത് തന്റെ ഭാര്യയെ ത്വലാഖ് ചെയ്യുന്ന പുരുഷന്‍ പക്ഷേ, അവളെ വീട്ടില്‍നിന്ന് പുറത്താക്കാന്‍ പാടില്ല. അവള്‍ പുറത്തുപോകാനും പാടില്ല. മൂന്നു തവണ ആര്‍ത്തവമുണ്ടാകുന്നതുവരെ അവള്‍ ഭര്‍തൃഗൃഹത്തില്‍തന്നെ താമസിക്കേണ്ടതാണ്. ആര്‍ത്തവം നിലച്ചവര്‍ക്ക് മൂന്നു മാസക്കാലവും ഗര്‍ഭിണികള്‍ക്ക് പ്രസവം വരെയുമാണ് ഈ കാലാവധി. ഇദ്ദാ കാലമെന്നാണ് ഈ കാലാവധിക്ക് സാങ്കേതികമായ പേര്. ഈ കാലത്ത് വിവാഹമോചിത ഭര്‍തൃഗൃഹത്തില്‍തന്നെ താമസിക്കണമെന്നാണ് ഖുര്‍ആനിന്റെ വിധി.

”വിവാഹമുക്തകള്‍ തങ്ങളുടെ സ്വന്തം കാര്യത്തില്‍, മൂന്ന് തവണ ആര്‍ത്തവമുണ്ടാവുന്നത് വരെ കാത്തിരിക്കേണ്ടതാണ്. അവര്‍ അല്ലാഹുവിലും അന്ത്യദിനത്തിലും വിശ്വസിക്കുന്നവരാണെങ്കില്‍ തങ്ങളുടെ ഗര്‍ഭാശയങ്ങളില്‍ അല്ലാഹു സൃഷ്ടിച്ചിട്ടുള്ളതിനെ അവര്‍ ഒളിച്ചുവെക്കുവാന്‍ പാടില്ല” (2:228).

”നിങ്ങള്‍ സ്ത്രീകളെ വിവാഹമോചനം ചെയ്യുകയാണെങ്കില്‍ അവരുടെ ഇദ്ദാ കാലത്തിന് (കണക്കാക്കി) വിവാഹമോചനം ചെയ്യുകയും ഇദ്ദാകാലം നിങ്ങള്‍ എണ്ണികണക്കാക്കുകയും ചെയ്യുക. നിങ്ങളുടെ രക്ഷിതാവായ അല്ലാഹുവെ നിങ്ങള്‍ സൂക്ഷിക്കുകയും ചെയ്യുക. അവരുടെ വീടുകളില്‍നിന്ന് അവരെ നിങ്ങള്‍ പുറത്താക്കരുത്. അവര്‍ പുറത്തുപോവുകയും ചെയ്യരുത്. പ്രത്യക്ഷമായ വല്ല നീചവൃത്തിയും അവള്‍ ചെയ്യുകയാണെങ്കിലല്ലാതെ… അങ്ങനെ അവര്‍ അവരുടെ അവധിയില്‍ എത്തുമ്പോള്‍ നിങ്ങള്‍ ന്യായമായ നിലയില്‍ അവരെ പിടിച്ചുനിര്‍ത്തുകയോ ന്യായമായ നിലയില്‍ അവരുമായി വേര്‍പിരിയുകയോ ചെയ്യുക” (65:1,2).

ഇദ്ദയുടെ കാലത്ത് സ്ത്രീയും പുരുഷനും ഭാര്യാഭര്‍ത്താക്കന്മാരല്ല. എന്നാല്‍, അന്യരുമല്ല. പുരുഷന്റെ വീട്ടിലാണ് അവള്‍ കഴിയുന്നത്. വിവാഹമോചനം ചെയ്ത ശേഷവും സ്ത്രീ ഭര്‍ത്താവിന്റെ വീട്ടില്‍തന്നെ താമസിക്കുന്നത് ഇരുവരുടെയും മനസ്സ് മാറ്റുവാന്‍ ഉപകരിക്കും. ഇന്നലെവരെ കൂടെക്കിടന്നവര്‍ ഇന്ന് രണ്ടായി കഴിയുകയാണ്. അവളെയാണെങ്കില്‍ അയാള്‍ കാണുകയും ചെയ്യുന്നു. അയാളുടെ ആസക്തിയെ ഇളക്കിവിടുവാനും കോപം ശമിപ്പിക്കുവാനും ഇതുമൂലം കഴിഞ്ഞേക്കും. ഇദ്ദാകാലത്ത് അവളെ മടക്കിയെടുക്കുവാന്‍ പുരുഷന് അവകാശമുണ്ട്. നിരുപാധികം അയാള്‍ക്ക് അതിന് സാധിക്കും. കുടുംബസ്ഥാപനം തകരാതിരിക്കുന്നതിന് എത്ര ശാസ്ത്രീയമായ മാര്‍ഗങ്ങളാണ് ഖുര്‍ആന്‍ സ്വീകരിക്കുന്നത്; കര്‍ക്കശമായ നിയമങ്ങള്‍ അടിച്ചേല്‍പിക്കാതെതന്നെ.

വിവാഹമോചനം നടത്തി. മൂന്ന് ആര്‍ത്തവകാലം കഴിയുന്നതുവരെ ഭര്‍തൃഗൃഹത്തില്‍ അവള്‍ താമസിക്കുകയും ചെയ്തു. എന്നിട്ടും അവര്‍ തമ്മില്‍ ഇണങ്ങാന്‍ മാര്‍ഗമില്ല. എങ്കില്‍ പിന്നെ മോചനംതന്നെയാണ് പരിഹാരം. ഈ മോചനംപോലും മാന്യമായിരിക്കണമെന്നാണ് ഖുര്‍ആനിന്റെ അനുശാസന. ”ഒന്നുകില്‍ മാന്യമായി അവളെ പിടിച്ചുനിര്‍ത്തുക, അല്ലെങ്കില്‍ മാന്യമായി അവളെ പിരിച്ചയക്കുക” (65:2).

വിവാഹസമയത്ത് വരന്‍ നല്‍കിയ വിവാഹമൂല്യം പൂര്‍ണമായി ഇങ്ങനെ മോചിപ്പിക്കുന്ന സ്ത്രീക്ക് അവകാശപ്പെട്ടതാണ്. കൂടുതലായാലും കുറച്ചായാലും അത് തിരിച്ചുവാങ്ങാന്‍ പാടില്ല. ഖുര്‍ആന്‍ പറയുന്നു:”നിങ്ങള്‍ ഒരു ഭാര്യയുടെ സ്ഥാനത്ത് മറ്റൊരു ഭാര്യയെ പകരം സ്വീകരിക്കുവാന്‍ ഉദ്ദേശിക്കുന്നപക്ഷം അവരില്‍ ഒരുവള്‍ക്ക് നിങ്ങള്‍ ഒരു കൂമ്പാരംതന്നെ കൊടുത്തിട്ടുണ്ടായിരുന്നുവെങ്കിലും അതില്‍നിന്ന് യാതൊന്നുംതന്നെ നിങ്ങള്‍ തിരിച്ചുവാങ്ങരുത്”(4:20).

”എന്നാല്‍, ഭാര്യയെ സ്പര്‍ശിക്കുന്നതിനു മുമ്പാണ് മോചനമെങ്കില്‍ നിശ്ചയിക്കപ്പെട്ട വിവാഹമൂല്യത്തിന്റെ പകുതി അവള്‍ക്ക് നല്‍കിയാല്‍ മതിയാകുന്നതാണ്”(2:237).

വിവാഹമോചന സമയത്ത് സ്ത്രീകള്‍ക്ക് മാന്യമായ പാരിതോഷികം നല്‍കണമെന്നും ഖുര്‍ആന്‍ അനുശാസിക്കുന്നുണ്ട്. ”വിവാഹമോചിതരായ സ്ത്രീകള്‍ക്ക് ന്യായപ്രകാരം എന്തെങ്കിലും ജീവിതവിഭവമായി നല്‍കേണ്ടതുണ്ട്. ഭയഭക്തിയുള്ളവര്‍ക്ക് അതൊരു ബാധ്യതയത്രേ” (2:241).

ഒരാള്‍ ഒരു സ്ത്രീയെ വിവാഹമോചനം നടത്തി. അല്‍പകാലത്തിനുശേഷം തന്റെ പ്രവൃത്തിയില്‍ അയാള്‍ക്ക് പാശ്ചാതാപം തോന്നി. മോചിതയായ സ്ത്രീയാണെങ്കില്‍ പുനര്‍വിവാഹം ചെയ്യപ്പെട്ടിട്ടുമില്ല. അയാള്‍ക്ക് അവളെ തന്റെ ഭാര്യയായി സ്വീകരിക്കണമെന്ന് ആഗ്രഹം ജനിച്ചു. എങ്കില്‍ അയാള്‍ക്ക് അവളെ തിരിച്ചെടുക്കാന്‍ ഖുര്‍ആന്‍ അനുവദിക്കുന്നു. ഇങ്ങനെ തിരിച്ചെടുത്തതിനുശേഷം ഒരിക്കല്‍കൂടി അതേസ്ത്രീയെതന്നെ വിവാഹമോചനം ചെയ്യുന്നുവെന്നു കരുതുക. ഒരു പ്രാവശ്യം കൂടി മാത്രമേ അയാള്‍ക്ക് അവളെ തിരിച്ചെടുക്കാന്‍ അവകാശമുള്ളൂ. മൂന്നാം തവണയും അയാള്‍ അവളെ ത്വലാഖ് ചെയ്യുകയാണെങ്കില്‍ പിന്നെ അയാള്‍ക്ക് അവളെ തിരിച്ചെടുക്കാന്‍ കഴിയില്ല. ഇതാണ് ഖുര്‍ആന്‍ പ്രതിപാദിക്കുന്ന മൂന്നു ത്വലാഖുകള്‍. ഖുര്‍ആന്‍തന്നെ പറയട്ടെ:”(മടക്കിയെടുക്കാന്‍ അനുമതിയുള്ള) വിവാഹമോചനം രണ്ടു പ്രാവശ്യം മാത്രമാകുന്നു. പിന്നെ ഒന്നുകില്‍ മര്യാദയനുസരിച്ച് കൂടെ നിര്‍ത്തുകയോ അല്ലെങ്കില്‍ നല്ല നിലയില്‍ പിരിച്ചയക്കുകയോ ആണ് വേണ്ടത്… ഇനിയും (മൂന്നാമതും) അവന്‍ അവളെ വിവാഹമോചനം ചെയ്യുകയാണെങ്കില്‍ അതിനുശേഷം അവളുമായി ബന്ധപ്പെടല്‍ അവന് അനുവദനീയമാവില്ല” (2:229-230).

ഇതാണ് ഖുര്‍ആനില്‍ പ്രതിപാദിക്കുന്ന മൂന്ന് ത്വലാഖുകള്‍. മൂന്നും മൂന്നു പ്രാവശ്യമായി നടക്കുന്ന വിവാഹമോചനങ്ങളാണവ. ഒരേസമയം മൂന്ന് ത്വലാഖ് ചൊല്ലുന്നത് നിഷിദ്ധമാണെന്ന കാര്യത്തില്‍ പ്രമുഖ മുസ്‌ലിം പണ്ഡിതന്മാര്‍ക്കിടയില്‍ പക്ഷാന്തരമില്ല. മൂന്നു ത്വലാഖും ഒന്നിച്ചു ചൊല്ലിയ ഒരാളെ ഉമര്‍(റ) ചമ്മട്ടികൊണ്ട് അടിക്കുവാന്‍ കല്‍പിക്കുകയുണ്ടായി. ഇതില്‍ നിന്ന് ഇത്തരമൊരു നടപടിയെ ഇസ്‌ലാം എന്തുമാത്രം വെറുക്കുന്നുവെന്ന് മനസ്സിലാക്കാന്‍ കഴിയും.

മൂന്ന് ത്വലാഖുകള്‍ എന്ന പദ്ധതി യഥാര്‍ഥത്തില്‍ സ്ത്രീക്ക് ഗുണകരമാണെന്നതാണ് വാസ്തവം. ഖുര്‍ആന്‍ പറഞ്ഞ രീതിയില്‍ ജീവിക്കുന്ന ഒരാള്‍ക്ക് അയാളുടെ ഹൃദയത്തിനകത്ത് സ്‌നേഹത്തിന്റെ ലാഞ്ഛനയെങ്കിലും ബാക്കിയുണ്ടെങ്കില്‍ മൂന്നാമത് ത്വലാഖ് ചെയ്യാന്‍ കഴിയില്ല. സ്വന്തം ഭാര്യയോടൊപ്പം ഒന്നിച്ചുകഴിയാന്‍ എന്തെങ്കിലും പഴുതുണ്ടോയെന്ന് അന്വേഷിക്കുകയും ഉണ്ടെങ്കില്‍ അതുപയോഗപ്പെടുത്തുകയും ചെയ്യുകയാണ് മൂന്നാമത്തെ ത്വലാഖിന് മുമ്പ് അയാള്‍ ചെയ്യുക. രണ്ടു പ്രാവശ്യം അയാള്‍ സഹിച്ച വിരഹദുഃഖം അയാളെ അലട്ടിക്കൊണ്ടിരിക്കും. ഇനിയൊരിക്കലും ഒന്നിച്ചുകഴിയാന്‍ സാധിക്കില്ലെന്ന് ഉറപ്പായതിന് ശേഷം മാത്രമേ മൂന്നാം പ്രാവശ്യം അയാള്‍ അവളെ വിവാഹമോചനം ചെയ്യുകയുള്ളൂ. ഏറ്റവും ശാസ്ത്രീയവും സ്ത്രീയെ പരിഗണിക്കുന്നതുമാണ് ഇസ്‌ലാമിലെ വിവാഹമോചനം എന്ന് പറയുന്നത് അതുകൊണ്ടാണ്.

വിഷയവുമായി ബന്ധപ്പെട്ട വീഡിയോ

”ഭാര്യമാരെ അടിക്കുന്നവര്‍ മാന്യന്മാരല്ല” എന്ന് പഠിപ്പിച്ച മുഹമ്മദ് നബി (സ) പൂര്ത്തീകരിച്ച ഇസ്‌ലാം നിരുപാധികം പെണ്ണിനെ അടിക്കാൻ പുരുഷൻ യാതൊരു അവകാശവും നൽകിയിട്ടില്ല. എന്നാൽ ഒരു ശിക്ഷണനടപടി എന്ന നിലയിൽ, വിവാഹമോചനം എന്ന ദുരന്തം ഇല്ലാതാക്കുന്നതിനുള്ള ഒരു അവസാന ശ്രമമെന്ന രീതിയിൽ ഇന്നലെ വരെ സ്നേഹിച്ച് തലോടിയ കൈ കൊണ്ടുള്ള ഒരു താഡനം ഇസ്‌ലാം അനുവദിച്ചിട്ടുണ്ട്. അത് എപ്പോൾ എങ്ങനെയെന്നും ഇസ്‌ലാം കൃത്യമായി പഠിപ്പിച്ചിട്ടുണ്ട്.

കുടുംബമെന്ന സ്ഥാപനത്തിലെ രണ്ട് പാതികളാണ് പുരുഷനും സ്ത്രീയും. എന്നാല്‍, സ്ഥാപനത്തിന്റെ നിയന്ത്രണാധികാരം പുരുഷനിലാണ് നിക്ഷിപ്തമായിരിക്കുന്നത്. സ്ഥാപനം തകരാതെ സൂക്ഷിേക്കണ്ടത് അവന്റെ ബാധ്യതയാണ്. ഇതിനുവേണ്ടി പരമാവധി പരിശ്രമിക്കണമെന്ന് ഖുര്‍ആന്‍ പുരുഷനോട് നിഷ്‌കര്‍ഷിക്കുന്നു. അതിനുവേണ്ടിയുള്ള നടപടിക്രമങ്ങളെക്കുറിച്ച് വിശദീകരിക്കുന്ന സൂക്തം ശ്രദ്ധിക്കുക:

”അതിനാല്‍ നല്ലവരായ സ്ത്രീകള്‍ അച്ചടക്കമുള്ളവരും അല്ലാഹു കാത്തത് മറവിലും കാത്തുസൂക്ഷിക്കുന്നവളുമാണ്. അച്ചടക്കരാഹിത്യം നിങ്ങള്‍ ഭയപ്പെടുന്ന സ്ത്രീകളെ നിങ്ങള്‍ ശാസിക്കുക; കിടപ്പറകളില്‍ അവരുമായി അകന്നുനില്‍ക്കുക; അവരെ അടിക്കുകയും ചെയ്യുക. എന്നിട്ട് അവര്‍ നിങ്ങളെ അനുസരിക്കുന്നപക്ഷം പിന്നെ നിങ്ങള്‍ അവര്‍ക്കെതിരില്‍ യാതൊരു മാര്‍ഗവും തേടരുത്” (4:34).

ഈ സൂക്തത്തില്‍ അച്ചടക്കരാഹിത്യത്തെക്കുറിച്ച് പറയുന്നത് നല്ല സ്ത്രീ ആരാണെന്ന് നിര്‍വചിച്ചതിനുശേഷമാണ്. ”അച്ചടക്കമുള്ളവളും അല്ലാഹു കാത്തത് മറവിലും കാത്തുസൂക്ഷിക്കുന്നവളും” ആണ് ഖുര്‍ആനിക വീക്ഷണത്തിലെ നല്ല സ്ത്രീ. കുടുംബത്തിന്റെ ഭദ്രതയ്ക്കും സമൂഹത്തിന്റെ ധാര്‍മികതക്കും സ്ത്രീകളില്‍ ഈ സ്വഭാവങ്ങള്‍ ആവശ്യമാണ്. അവള്‍ അച്ചടക്കമുള്ളവളായിരിക്കണം. അതോടൊപ്പംതന്നെ അല്ലാഹു കാത്തത് മറവിലും സംരക്ഷിക്കുന്നവളുമായിരിക്കണം.

ഭര്‍ത്താവിനോട് കയര്‍ക്കുകയും അയാള്‍ പറയുന്നതിനോടെല്ലാം എതിരു പ്രവര്‍ത്തിക്കുകയും ചെയ്യുന്ന ഭാര്യയുമൊത്തുള്ള ജീവിതം സഹിക്കാന്‍ എത്ര പേര്‍ക്ക് കഴിയും? പരസ്പരം പിണങ്ങിയും ശണ്ഠ കൂടിയും നിലനില്‍ക്കുന്ന കുടുംബാന്തരീക്ഷത്തില്‍ വളരുന്ന കുഞ്ഞുങ്ങളുടെ അവസ്ഥയെന്തായിരിക്കും? അത്തരമൊരു അവസ്ഥയുണ്ടാകുവാന്‍ പാടില്ലെന്ന് ഖുര്‍ആന്‍ നിഷ്‌കര്‍ഷിക്കുന്നു. ഭാര്യയില്‍നിന്ന് ഭര്‍ത്താവിന് മാത്രം അര്‍ഹതപ്പെടുന്ന പലതുമുണ്ട്. അവ അയാളുടെ സാന്നിധ്യത്തില്‍ അയാള്‍ക്ക് നല്‍കുകയും അസാന്നിധ്യത്തില്‍ മറ്റു പലര്‍ക്കും നല്‍കുകയും ചെയ്യുക നല്ല സ്ത്രീയുടെ സ്വഭാവമല്ല. ഭര്‍ത്താവിന് മാത്രം അവകാശപ്പെട്ട ഒരു നോട്ടമോ വാക്കോ പോലും അവളില്‍നിന്ന് അന്യര്‍ക്കായി ഉണ്ടായിക്കൂടാ. അതുണ്ടാവുന്നത് കുടുംബത്തിന്റെ തകര്‍ച്ചക്ക് കാരണമാവും. ഒരു കാരണവശാലും അത്തരമൊരു തകര്‍ച്ചയുണ്ടാവരുത്. ഖുര്‍ആനികമായ മാര്‍ഗനിര്‍ദേശങ്ങള്‍ ഈയൊരു ലക്ഷ്യത്തോടുകൂടിയുള്ളവയാണ്.

കുടുംബത്തിന്റെ തകര്‍ച്ചക്ക് നിമിത്തമായേക്കാവുന്ന അച്ചടക്കരാഹിത്യത്തെ മുളയിലേ നുള്ളിക്കളയണമെന്നാണ് ഖുര്‍ആനിന്റെ അനുശാസന. അത് പരമകാഷ്ഠ പ്രാപിച്ച് ധിക്കാരത്തിന്റെ പാരമ്യത്തിലെത്തുന്നതുവരെ കാത്തിരിക്കുന്നത് കുടുംബമെന്ന സ്ഥാപനം പൊട്ടിപ്പിളരുന്നതിന് കാരണമാവും. ആ തലത്തിലെത്തിയാല്‍ പിന്നെ ചികില്‍സകള്‍ ഫലിക്കുകയില്ല. ശാന്തിയും സമാധാനവും തകര്‍ന്ന് സര്‍വനാശത്തിലേക്ക് പോയിക്കൊണ്ടിരിക്കുന്ന കുടുംബത്തില്‍ ജീവിക്കുന്ന കുട്ടികളുടെ സ്ഥിതി പരിതാപകരമായിരിക്കും. അതിനാല്‍ അച്ചടക്കരാഹിത്യത്തിന്റെ ലക്ഷണങ്ങള്‍ വളരെ വിദൂരത്തുതന്നെ പ്രത്യക്ഷമായിത്തുടങ്ങിയാല്‍ കുടുംബത്തെ നാശത്തില്‍നിന്നു രക്ഷിക്കുവാന്‍ ക്രമപ്രവൃദ്ധമായ ചില നടപടികളാവശ്യമാണ്.അങ്ങനെയുള്ള സന്ദര്‍ഭത്തില്‍ അച്ചടക്കരാഹിത്യം ഇല്ലാതാക്കുന്നതിനുവേണ്ടി ചില നടപടികള്‍ കൈക്കൊള്ളുവാന്‍ പുരുഷനെ ഇസ്‌ലാം അനുവദിച്ചിട്ടുണ്ട്. ഈ നടപടികള്‍ നിന്ദിക്കുന്നതിനോ പ്രതികാരം ചെയ്യുന്നതിനോ വേണ്ടിയുള്ളതല്ല. പ്രത്യുത, സംസ്‌കരണത്തിനും അച്ചടക്കരാഹിത്യം ഇല്ലാതാക്കുന്നതിലൂടെ ഐക്യപ്പെടുത്തുന്നതിനും വേണ്ടിയുള്ളതാകുന്നു.

ഖുര്‍ആന്‍ നിര്‍ദേശിക്കുന്ന പ്രസ്തുത നടപടിക്രമം ഇങ്ങനെയാണ്:”ശാസിക്കുക, കിടപ്പറയില്‍ അവളെ ബഹിഷ്‌കരിക്കുക, പിന്നെ അവളെ അടിക്കുക”.

അച്ചടക്കമില്ലാത്ത സ്ത്രീയെ ആദ്യം ശാസിക്കുകയാണ് വേണ്ടത്. അവളുടെ പ്രവര്‍ത്തനങ്ങള്‍ കൊണ്ട് ഇഹത്തിലും പരത്തിലുമുണ്ടാകുവാന്‍ പോകുന്ന പ്രതിഫലനങ്ങളെക്കുറിച്ച് ബോധ്യപ്പെടുത്തുക. പെണ്ണിന്റെ പ്രത്യേകമായ സ്വഭാവങ്ങളാല്‍ സംഭവിച്ചുപോയ പാകപ്പിഴവുകളാണെങ്കില്‍ തിരുത്തുവാന്‍ ഉപദേശം ഫലം ചെയ്യും.

ശാസനയും ഉപദേശവും ഫലം ചെയ്യാത്ത സ്ഥിതിയുണ്ടാവാം. ഭര്‍ത്താവിന്റെ സ്‌നേഹവായ്‌പോടെയുള്ള ശാസനയും വികാരസാന്ദ്രമായ ഉപദേശവും ഫലം ചെയ്യാതിരിക്കുന്നതിന് കാരണം പലപ്പോഴും അഹങ്കാരമായിരിക്കും. സൗന്ദര്യത്തിന്റെയും ധനത്തിന്റെയും കുടുംബ മാഹാത്മ്യത്തിന്റെയും പേരിലുള്ള അഹന്ത. ഇവിടെയാണ് രണ്ടാമത്തെ നടപടിക്രമം വരുന്നത്. കിടപ്പറയില്‍ അവളെ ബഹിഷ്‌കരിക്കുക. ആകര്‍ഷണത്തിന്റെയും പ്രലോഭനത്തിന്റെയും കേന്ദ്രമാണ് കിടക്ക. അച്ചടക്കമില്ലാത്ത അഹങ്കാരിയായ സ്ത്രീയുടെ അധീശത്വത്തിന്റെ ഉച്ചകോടി അവിടെയാണല്ലോ. അവിടെ അവള്‍ ബഹിഷ്‌കരിക്കപ്പെടുകയെന്നു പറഞ്ഞാല്‍ അവളുടെ അഹന്തയെ പുല്ലുവില പോലും കല്‍പിക്കാതെ പുച്ഛിച്ചുതള്ളുന്നുവെന്നര്‍ഥം. അച്ചടക്കമില്ലാത്ത സ്ത്രീയുടെ ഏറ്റവും മൂര്‍ച്ചയുള്ള ആയുധത്തിനെതിരെയുള്ള ശക്തമായ നടപടി. ഈ നടപടിക്കു മുതിരുന്ന പുരുഷന് അപാ രമായ നിയന്ത്രണവും നിശ്ചയദാര്‍ഢ്യവുമാവശ്യമാണ്. ഏതൊരു അഹങ്കാരിയെയും ചിന്തിപ്പിക്കുന്ന രാത്രികളായിരിക്കും അത്. എന്തിന്റെ പേരിലാണോ താന്‍ അധീശത്വം നടിച്ചിരുന്നത് അത് തന്റെ ഇണയ്ക്ക് ആവശ്യമില്ലെന്ന രീതിയിലുള്ള ബഹിഷ്‌കരണം സ്ത്രീയുടെ മനസ്സ് മാറ്റുകതന്നെ ചെയ്യും.

ബഹിഷ്‌കരണവും പരാജയപ്പെടുന്ന സന്ദര്‍ഭങ്ങളിലും കുടുംബത്തെ തകരാന്‍ അനുവദിക്കരുതെന്നാണ് ഖുര്‍ആനിന്റെ അനുശാസന. ശാസനകള്‍ ഫലിക്കാതിരിക്കുകയും ശയ്യാബഹിഷ്‌കരണം വിജയിക്കാതിരിക്കുകയും ചെയ്യുന്ന സാഹചര്യം വളരെ വിരളമായിരിക്കും. അത്തരം സാഹചര്യങ്ങളുണ്ടാവുകയാണെങ്കില്‍ അവിടെ അച്ചടക്കരാഹിത്യം അതിന്റെ പരമകാഷ്ഠയിലെത്തിയിരിക്കും. ചെറിയ ശിക്ഷകളല്ലാതെ ഇനി മാര്‍ഗങ്ങളൊന്നുമില്ല.

അടുത്ത മാര്‍ഗമെന്ന നിലയ്ക്കാണ് ഖുര്‍ആന്‍ ‘പ്രഹരം‘നിര്‍ദേശിക്കുന്നത്. സമാധാനപരമായ മാര്‍ഗങ്ങളെല്ലാം പരാജയപ്പെടുമ്പോള്‍ ഒരു കരുതല്‍ നടപടിയെന്ന നിലയില്‍ നിര്‍ദേശിക്കപ്പെട്ടതാണ് അവളെ അടിയ്ക്കുകയെന്നത്. സാധാരണ ഗതിയില്‍ സ്ത്രീയെ അടിക്കുന്നതിനെതിരെ ശക്തമായി സംസാരിച്ചിട്ടുള്ള വ്യക്തിയാണ് മുഹമ്മദ്(ﷺ). ”ഭാര്യമാരെ അടിക്കുന്നവര്‍ മാന്യന്മാരല്ല” (അബൂദാവൂദ്, ഇബ്‌നുമാജ) എന്നാണ് അദ്ദേഹം അഭിപ്രായപ്പെട്ടത്. അദ്ദേഹം ചോദിച്ചു: ”നാണമില്ലേ നിങ്ങള്‍ക്ക്? അടിമയെ അടിക്കുന്നതുപോലെ സ്വന്തം ഭാര്യയെ അടിക്കാന്‍; പിന്നെ അവളോടൊത്ത് ശയിക്കാനും” (മുസ്‌ലിം, അഹ്മദ്). ”നിങ്ങളില്‍ ഭാര്യമാരോട് നന്നായി പെരുമാറുന്നവരാണ് ഏറ്റവും നല്ലവന്‍” (ബുഖാരി,തുര്‍മുദി) എന്നു പഠിപ്പിച്ച പ്രവാചകനിലൂടെ അവതീര്‍ണമായ ഖുര്‍ആന്‍ വെറുതെ സ്ത്രീയെ അടിക്കണമെന്ന് കല്‍പിക്കുകയില്ലെന്നുറപ്പാണ്.

വലിയ തിന്മയില്ലാതാക്കുവാനുള്ള ശിക്ഷണമായി, മറ്റു മാര്‍ഗങ്ങള്‍ പരാജയപ്പെടുമ്പോഴുള്ള അവസാന മാര്‍ഗമായിട്ടാണ് ഖുര്‍ആന്‍ അടി നിര്‍ദേശിക്കുന്നത്. അതുതന്നെ അവള്‍ക്ക് അഭിമാനക്ഷതമുണ്ടാകുന്ന രീതിയില്‍ മുഖത്തോ മറ്റോ ആകരുതെന്ന് പ്രവാചകന്‍ (ﷺ) പ്രത്യേകം നിര്‍ദേശിച്ചിട്ടുമുണ്ട്. സ്ത്രീയെ നിന്ദിക്കുവാനോ അപമാനിക്കുവാനോ വേണ്ടിയല്ല, പ്രത്യുത നന്നാക്കുന്നതിനും സംസ്‌കരിക്കുന്നതിനും വേണ്ടിയുള്ള അവസാനത്തെ മാര്‍ഗമെന്ന നിലയ്ക്കാണ് ഖുര്‍ആന്‍ അടി നിര്‍ദേശിക്കുന്നത്. പിതാവ് മക്കളെ അടിക്കുന്നതുപോലെ, അധ്യാപകന്‍ വിദ്യാര്‍ഥികളെ കൈകാര്യം ചെയ്യുന്നതുപോലെ, ഒരു പരിശീലകന്റെ വികാരത്തോടെയുള്ള ശിക്ഷണമാണത്. എപ്പോഴും സ്‌നേഹം നല്‍കുകയും തന്റെ സ്‌നേഹപ്രകടനങ്ങള്‍ക്ക് പാത്രമാവുകയും ചെയ്യുന്ന തന്റെ ഇണയുടെ പ്രഹരം അവളെ വീണ്ടു വിചാരത്തിനും ഖേദപ്രകടനത്തിനും അങ്ങനെ തെറ്റുതിരുത്തലിനും പ്രേരകമാക്കിയേക്കാം. അങ്ങനെ തകര്‍ച്ചയുടെ വക്കിലെത്തിനില്‍ക്കുന്ന കുടുംബം തകരാതെ രക്ഷപ്പെടാനിടയുണ്ട്. ഈ ശിക്ഷണത്തിന്റെ ആത്യന്തിക ലക്ഷ്യം കുടുംബമെന്ന സ്ഥാപനത്തെ തകര്‍ച്ചയില്‍നിന്ന് രക്ഷിക്കുകയാണെന്നര്‍ഥം.

ഇസ്‌ലാമിക വസ്ത്രധാരണം അടിമത്തത്തിന്റെ അടയാളമല്ല. പ്രത്യുത ആഭിജാത്യത്തിന്റെ ചിഹ്‌നമാണ് എന്ന് അല്‍പം ചിന്തിച്ചാല്‍ ബോധ്യമാകും. മുഖവും മുന്‍കൈയും ഒഴികെയുള്ള ശരീരഭാഗങ്ങളെല്ലാം മറക്കണമെന്ന് ഇസ്‌ലാം സ്ത്രീയോട് കല്‍പിക്കുന്നുവെന്നത് ശരിയാണ്. എന്തിനാണ് ഈ കല്‍പന? സ്ത്രീകളെ അടിമത്തത്തിന്റെ കാരാഗൃഹത്തിലടക്കുകയോ സുരക്ഷിതത്വത്തിന്റെ താഴ്‌വരയില്‍ വിഹരിക്കാനനുവദിക്കുകയോ എന്താണ് ഈ കല്‍പന ചെയ്യുന്നത്?ഇസ്‌ലാമിക വസ്ത്രധാരണം നിര്‍ബന്ധമാക്കിക്കൊണ്ടുള്ള ഖുര്‍ആന്‍ സൂക്തങ്ങള്‍ ഈ ചോദ്യങ്ങള്‍ക്ക് വ്യക്തമായ ഉത്തരം നല്‍കുന്നുണ്ട്. അത് ഇങ്ങനെയാണ്:

”നബിയേ, താങ്കളുടെ പത്‌നിമാരോടും പുത്രിമാരോടും സത്യവിശ്വാസികളുടെ സ്ത്രീകളോടും അവര്‍ തങ്ങളുടെ മൂടുപടങ്ങള്‍ തങ്ങളുടെ മേല്‍ താഴ്ത്തിയിടാന്‍ പറയുക. അവര്‍ തിരിച്ചറിയപ്പെടുവാനും അങ്ങനെ അവര്‍ ശല്യം ചെയ്യപ്പെടാതിരിക്കാനും അതാണ് ഏറ്റവും അനുയോജ്യമായത്. അല്ലാഹു ഏറെ പൊറുക്കുന്നവനും കരുണാനിധിയുമാകുന്നു” (33:59).

”സത്യവിശ്വാസിനികളോട് അവരുടെ ദൃഷ്ടികള്‍ താഴ്ത്തുവാനും അവരുടെ ഗുഹ്യാവയവങ്ങള്‍ കാത്തുരക്ഷിക്കാനും അവരുടെ ഭംഗിയില്‍നിന്ന് പ്രത്യക്ഷമായതൊഴിച്ച് മറ്റൊന്നും വെളിപ്പെടുത്താതിരിക്കുവാനും നീ പറയുക. അവരുടെ മക്കനകള്‍ കുപ്പായമാറുകള്‍ക്ക് മീതെ അവര്‍ താഴ്ത്തിയിട്ടുകൊള്ളട്ടെ” (24:31).

”പഴയ അജ്ഞാനകാലത്തെ സൗന്ദര്യപ്രകടനം പോലെയുള്ള സൗന്ദര്യ പ്രകടനം നിങ്ങള്‍ നടത്തരുത്” (33:33).

സ്ത്രീയോട് മാന്യമായ വസ്ത്രധാരണരീതി സ്വീകരിക്കാന്‍ കല്‍പിച്ചതിന് പിന്നിലുള്ള ലക്ഷ്യങ്ങള്‍ ഈ സൂക്തങ്ങളില്‍ല്‍നിന്ന് സുതരാം വ്യക്തമാണ്.

ഒന്ന്, തിരിച്ചറിയപ്പെടുക.

രണ്ട്, ശല്യം ചെയ്യപ്പെടാതിരിക്കുക.

സമൂഹത്തിന്റെ വ്യത്യസ്ത തുറകളില്‍ ജീവിക്കുന്നവര്‍ തിരിച്ചറിയപ്പെടുന്നതിനുവേണ്ടി വ്യത്യസ്ത വസ്ത്രധാരണരീതികള്‍ സ്വീകരിക്കാറുണ്ട്. സ്ത്രീകെള സംബന്ധിച്ചിടത്തോളം അവരുടെ വസ്ത്രധാരണരീതിയില്‍നിന്നുതന്നെ ഒരളവോളം അവരുടെ ജീവിതരീതിയെയും പെരുമാറ്റ രീതിയെയും നമുക്ക് അളക്കുവാന്‍ സാധിക്കും.

ആവശ്യക്കാര്‍ക്ക് തിരിച്ചറിയുവാന്‍ സാധിക്കുന്ന രീതിയിലുള്ള വസ്ത്രധാരണരീതിയാണ് വേശ്യകള്‍ സ്വീകരിക്കുക. ക്ഷേത്രങ്ങളോട് ബന്ധപ്പെട്ട് ജീവിച്ചിരുന്ന ദേവദാസികള്‍ക്ക് അവരുടേതായ വസ്ത്രധാരണ രീതിയുണ്ടായിരുന്നു. ഗ്രീസിലെ ഹെറ്റേയ്‌റേകള്‍ക്കും ചൈനയിലെ ചിന്‍കുവാന്‍ ജെന്നുകള്‍ക്കും ജപ്പാനിലെ ഗായിഷേകള്‍ക്കുമെല്ലാം അവരുടേതായ വസ്ത്രധാരണരീതികളുണ്ടായിരുന്നതായി കാണാന്‍ കഴിയും. ഈ വസ്ത്രധാരണത്തില്‍ നിന്ന് അവരെ മനസ്സിലാക്കാം. ആവശ്യക്കാര്‍ക്ക് ഉപയോഗിക്കുവാന്‍ ക്ഷണിക്കുകയും ചെയ്യാം.

ഇസ്‌ലാം വിഭാവനം ചെയ്യുന്ന സ്ത്രീ, മാന്യയും കുലീനയുമാണ്;ചാരിത്രവതിയും സദ്‌വൃത്തയുമാണ്. അവളുടെയടുത്തേക്ക് ലൈംഗികദാഹം പൂണ്ട ചെന്നായ്ക്കള്‍ ഓടിയടുക്കേണ്ടതില്ല. കാമാഭ്യര്‍ഥനയുമായി അവളെ ആരും സമീപിക്കേണ്ടതില്ല. ഇത് അവളുടെ വസ്ത്രത്തില്‍നിന്നുതന്നെ തിരിച്ചറിയണം. പതിനഞ്ചാം നൂറ്റാണ്ടിലെ വെനീസിലെ നിയമസംഹിതയില്‍ വേശ്യകള്‍ മാറുമറയ്ക്കാതെ ജനാലക്കല്‍ ഇരുന്നുകൊള്ളണമെന്ന കല്‍ പനയുണ്ടായിരുന്നു. മാംസദാഹം തീര്‍ക്കുവാന്‍ വരുന്നവര്‍ക്ക് മാംസഗുണമളക്കുവാന്‍ വേണ്ടിയുള്ള നടപടി! ഇന്നലെകളില്‍ ആവശ്യക്കാരെ ആകര്‍ഷിക്കുന്നതിനുവേണ്ടി അഭിസാരികകള്‍ സ്വീകരിച്ചിരുന്ന വസ്ത്രങ്ങള്‍ക്ക് സമാനമായ ഉടയാടകളാണ് ആധുനിക വനിതകളുടെ വേഷമെന്ന കാര്യം എന്തു മാത്രം വിചിത്രമല്ല! സത്യവിശ്വാസികളെയും മാംസവില്‍പനക്കാരികളെയും തിരിച്ചറിയണമെന്ന് ഖുര്‍ആന്‍ നിര്‍ദേശിക്കുന്നു; അവരുടെ വസ്ത്രധാരണത്തിലൂടെ.

എക്കാലത്തും ശല്യം ചെയ്യപ്പെട്ടുകൊണ്ടിരിക്കുന്ന ഒരു വിഭാഗമാണ് സ്ത്രീകള്‍. അവരുടെ മാംസത്തിനുവേണ്ടി-ചാരിത്ര്യത്തിനുവേണ്ടി-കടിപിടി കൂടുന്നവരാണ് എന്നത്തെയും സാഹിത്യ-സാംസ്‌കാരിക രംഗത്തെ നായകന്മാര്‍. നഗ്‌നനൃത്തങ്ങളും നഗ്‌നതാ വിവരങ്ങളുള്‍ക്കൊള്ളുന്ന കവിതകളും ഉപയോഗിച്ചുകൊണ്ടായിരുന്നു ഇന്നലെ സ്ത്രീയുടെ മാനത്തെ പിച്ചിച്ചീന്തിയിരുന്നതെങ്കില്‍ ഇന്നത്‘വിഡ്ഢിപ്പെട്ടി‘കളിലൂടെയും ഇന്റര്‍നെറ്റിലൂടെയും കുടുംബത്തിന്റെ ഇടനാഴികളിലേക്ക് കടന്നുവന്നുകൊണ്ടിരിക്കുകയാണ്. ആധുനിക ജനതയുടെ മുഴുജീവിതവും ലൈംഗികവത്കരിക്കപ്പെട്ടിരിക്കുകയാണ്. അതിരാവിലെ കുടിക്കേണ്ട കാപ്പിയേതാണെന്ന് തെരഞ്ഞെടുക്കുന്നതിനും രാത്രി ഉറങ്ങുമ്പോള്‍ വെയ്‌ക്കേണ്ട തലയിണ ഏതാണെന്ന് തീരുമാനിക്കുന്നതിനുംപോലും പെണ്ണിന്റെ നിമ്‌നോന്നതികളിലൂടെ കണ്ണ് പായിക്കണമെന്നുള്ള അവസ്ഥയാണിന്നുള്ളത്.

അതുകൊണ്ടുതന്നെ, പെണ്ണിനു നേരെയുള്ള കൈയേറ്റങ്ങളും കൂടി ക്കൊണ്ടിരിക്കുന്നു. സ്വന്തം മകളെ മാനഭംഗം ചെയ്യുന്ന അച്ഛനും പെറ്റമ്മയുമായി ലൈംഗികകേളികളിലേര്‍പ്പെടുന്ന മകനും നമ്മുടെ മസ്തിഷ്‌കങ്ങളില്‍ യാതൊരു ആന്ദോളനവും സൃഷ്ടിക്കാത്ത കഥാപാത്രങ്ങളായിക്കൊണ്ടിരിക്കുന്നു. വിദ്യാര്‍ഥിനികളെ മാനഭംഗപ്പെടുത്തുന്ന അധ്യാപകര്‍! അധ്യാപികമാരുമായി ഊരുചുറ്റുന്ന വിദ്യാര്‍ഥികള്‍! വനിതാ സെക്രട്ടറിയുമായി ബന്ധപ്പെട്ട ലൈംഗിക അപവാദങ്ങള്‍ മൂലം രാജിവെച്ചൊഴിയേണ്ടിവരുന്ന ഉദ്യോഗസ്ഥ പ്രമുഖര്‍! പലരുമായി ലൈംഗികബന്ധമുണ്ടെന്ന് പരസ്യമായി പ്രഖ്യാപിക്കുന്ന രാജകുമാരിമാര്‍! ഇങ്ങനെ പോകുന്നു ദിനപത്രങ്ങളില്‍ ദിനേന നാം വായിക്കുന്ന വര്‍ത്തമാനങ്ങള്‍. സ്ത്രീകള്‍ക്ക് സൈ്വരമായി യാത്ര ചെയ്യാന്‍ കഴിയാത്ത അവസ്ഥ! സ്വൈര്യമായി ജോലി ചെയ്യാനാവാത്ത സ്ഥിതി! എന്തിനധികം, സ്വൈര്യമായി വീട്ടില്‍ അടങ്ങിക്കൂടി നില്‍ക്കുവാന്‍ പോലും കഴിയാത്ത അവസ്ഥയിലേക്കാണ് നമ്മുടെ സമൂഹം പൊയ്‌ക്കൊണ്ടിരിക്കുന്നത്. ഇതിനെന്താണ് കാരണം?പക്വമതികളായ വിദഗ്ധര്‍ പറയുന്ന ഉത്തരം ശ്രദ്ധിക്കുക:

‘കുമാരി‘ വാരികയിലെ ‘പ്രതിവാര ചിന്തകള്‍‘ എന്ന പംക്തിയില്‍ എന്‍. വി. കൃഷ്ണവാരിയര്‍ എഴുതി: ”സ്ത്രീകളുടെ മാദകമായ വസ്ത്രധാരണവും ചേഷ്ടകളും നിമിത്തം മതിമറന്ന് താല്‍ക്കാലികമായ ഒരു ഉന്മാദാവസ്ഥയിലാണ് പുരുഷന്‍ ബലാല്‍സംഗം നടത്തുന്നതെന്ന് പൊതുവെ വിശ്വസിക്കപ്പെടുന്നു. പുരുഷനെ ഉത്തേജിപ്പിക്കുമാറ് വസ്ത്രം ധരിച്ച ഒാരോ സ്ത്രീയും ബലാല്‍സംഗം അര്‍ഹിക്കുന്നുവെന്ന് ഇന്ത്യയില്‍ ഒരു സുപ്രീംകോടതി ജഡ്ജി കുറെമുമ്പ് പരസ്യമായി പ്രസ്താവിക്കുകയുണ്ടായി” (കുമാരി വാരിക 11.3.83).

അപ്പോള്‍ വസ്ത്രധാരണത്തില്‍ മാന്യത പുലര്‍ത്തുന്നതുവഴി സ്ത്രീ സ്വന്തം ശരീരത്തെ സംരക്ഷിക്കുകയാണ് ചെയ്യുന്നത്. പടച്ചതമ്പുരാന്‍ പറഞ്ഞതെത്ര ശരി!

”അവര്‍ തിരിച്ചറിയപ്പെടാനും ശല്യം ചെയ്യപ്പെടാതിരിക്കാനും അതാണ് അനുയോജ്യം” (33:59).

വ്യഭിചാരവും ബലാല്‍സംഗങ്ങളും സ്ത്രീകള്‍ക്ക് നേരെയുള്ള കൈയേറ്റങ്ങളും അവസാനിപ്പിക്കുന്നതിന്റെ ആദ്യപടിയെന്ന നിലക്കാണ് മാന്യമായി വസ്ത്രധാരണം ചെയ്യണമെന്ന് ഖുര്‍ആന്‍ സ്ത്രീകളോട് ഉപദേശിക്കുന്നത്.

മുഖവും മുന്‍കൈയും ഒഴികെയുള്ള ശരീരഭാഗങ്ങളെല്ലാം മറയ്ക്കണമെന്നുതന്നെയായിരുന്നു സത്യവിശ്വാസിനികളായ സ്ത്രീകളോട് മുന്‍ പ്രവാചകന്മാരും പഠിപ്പിച്ചിരുന്നത് എന്നാണ് മനസ്സിലാക്കാന്‍ കഴിയുന്നത്. അന്യപുരുഷന്മാരെ കാണുമ്പോള്‍ മൂടുപടം അണിയുന്ന പതിവ് ഇസ്രായേല്‍ സമൂഹത്തില്‍ ആദ്യം മുതല്‍ക്കുതന്നെ നിലനിന്നിരുന്നുവെന്നാണ് പഴയനിയമ ചരിത്രം നല്‍കുന്ന സൂചന (ഉല്‍പത്തി 24:62-65). ഒരു സ്ത്രീയുടെ മൂടുപടം എടുത്തുകളയുന്നത് അവളെ മാനഭംഗം ചെയ്യുന്നതിന് തുല്യമായിക്കൊണ്ട് വിശേഷിപ്പിക്കപ്പെട്ടതില്‍നിന്ന് (ഉത്തമഗീതം 5:7) അതിനുണ്ടായിരുന്ന പ്രാധാന്യം എത്രത്തോളമായിരുന്നുവെന്ന് ഊഹിക്കുവാന്‍ കഴിയും.

യേശുക്രിസ്തുവിന് ശേഷവും മൂടുപടം ഉപയോഗിക്കുന്ന സമ്പ്രദായം നിലനിന്നിരുന്നതായി കാണാന്‍ കഴിയും. പൗലോസിന്റെ എഴുത്തുകളില്‍നിന്ന് നമുക്ക് ഇക്കാര്യം മനസ്സിലാക്കാനാവും. അദ്ദേഹം എഴുതി: ”സ്വന്തം ശിരസ്സ് മൂടാതെ പ്രാര്‍ഥിക്കയോ പ്രവചിക്കയോ ചെയ്യുന്ന സ്ത്രീ തന്റെ ശിരസ്സിനെ അപമാനിക്കുന്നു. അവളുടെ തല മുണ്ഡനം ചെയ്യുന്നതിന് സമമാണത്. തല മൂടാത്ത സ്ത്രീ തന്റെ മുടി മുറിക്കണം. മുടി മുറിക്കുന്നതും മുണ്ഡനം ചെയ്യുന്നതും അപമാനമാണെന്ന് കരുതുന്നവര്‍ ശിരോവസ്ത്രം ധരിക്കട്ടെ” (1കൊരിന്ത്യര്‍ 11:5-7).

”വ്യഭിചാരത്തെ സമീപിക്കുകപോലും ചെയ്യരുത്” (17:32) എന്ന സത്യവിശ്വാസികളോടുള്ള ഖുര്‍ആനിക കല്‍പനയുടെ പ്രയോഗവത്കരണത്തിന്റെ ഭാഗമായിട്ടാണ് മാന്യമായ വസ്ത്രധാരണം വേണമെന്ന് അത് സ്ത്രീകളോട് അനുശാസിക്കുന്നത്. കാമാര്‍ത്തമായ നോട്ടവും വാക്കും അംഗചലനങ്ങളുമെല്ലാം വ്യഭിചാരത്തിന്റെ അംശങ്ങളുള്‍ക്കൊള്ളുന്നവയാണെന്നാണ് മുഹമ്മദ് നബി(ﷺ)പഠിപ്പിച്ചത്. വ്യഭിചാരത്തിലേക്കും തദ്വാരാ സദാചാര തകര്‍ച്ചയിലേക്കും നയിക്കുന്ന ‘കൊച്ചു വ്യഭിചാരങ്ങ‘ളുടെ വാതിലടയ്ക്കണമെന്ന് ഇസ്‌ലാം നിഷ്‌കര്‍ഷിക്കുന്നു. മാദകമായ വസ്ത്രധാരണവും ലൈംഗികചേഷ്ടയിലെ അംഗചലനങ്ങളുള്‍ക്കൊള്ളുന്ന നൃത്തനര്‍ത്യങ്ങളും ഇസ്‌ലാം നിരോധിക്കുന്നത് അതുകൊണ്ടാണ്.

മാന്യമായാണ് സ്ത്രീ വസ്ത്രം ധരിക്കേണ്ടതെന്ന് കാര്യബോധമുള്ളവരെല്ലാം സമ്മതിക്കും. സ്ത്രീകൾക്ക് നേരെയുള്ള അക്രമങ്ങൾക്കുള്ള കാരണങ്ങളിലൊന്ന് പുരുഷനെ ത്രസിപ്പിക്കുന്ന അവളുടെ വസ്ത്രധാരണയാണെന്നും അവർ പറയും. എങ്ങനെയാണ് ഒരു സ്ത്രീ മാന്യമായി വസ്ത്രം ധരിക്കേണ്ടത്?കാര്‍ക്കൂന്തലുകളും മാറിന്റെ സിംഹഭാഗവും വയറുമെല്ലാം പുറത്തുകാണിച്ചുകൊണ്ടുള്ള പഴയ ദേവദാസികളുടേതിനു തുല്യമായ വസ്ത്രധാരണാ രീതിയോ? കാല്‍മുട്ടുവരെയും കഴുത്തും കാര്‍ക്കൂന്തലുകളും പുറത്ത് കാണിച്ചുകൊണ്ടുള്ള ഗ്രീസിലെ ഹെറ്റയ്‌റേകളുടെ വസ്ത്രധാരണ സമ്പ്രദായമോ? ഇറുകിയ വസ്ത്രങ്ങളിലൂടെ ശരീരത്തിന്റെ നിമ്‌നോന്നതികള്‍ പുരുഷന് മുന്നില്‍ പ്രദര്‍ശിപ്പിക്കുന്ന ചൈനയിലെ ചിന്‍കുവാന്‍ ജെന്നുകളുടെ ഉടയാടകള്‍ക്ക് തുല്യമായ പുടവകളോ? അതല്ല, മുഖവും മുന്‍കൈയും മാത്രം പുറത്തുകാണിക്കുകയും ശരീരഭാഗങ്ങള്‍ വെളിവാകാത്ത രൂപത്തില്‍ അയഞ്ഞ വസ്ത്രം ധരിക്കുകയും ചെയ്യുന്ന ഇസ്‌ലാമിക രീതിയോ?മുന്‍ധാരണയില്ലാത്ത ആര്‍ക്കും അവസാനത്തേതല്ലാത്ത മറ്റൊരു ഉത്തരം തെരഞ്ഞെടുക്കാന്‍ കഴിയില്ല.

ഇസ്‌ലാം സ്ത്രീയോട് മാന്യമായി വസ്ത്രം ധരിക്കാന്‍ പറയുക മാത്രമല്ല, എങ്ങനെയാണ് ആ വസ്ത്രധാരണരീതിയെന്ന് പഠിപ്പിക്കുക കൂടി ചെയ്തുവെന്നുള്ളതാണ് അതിന്റെ ഏറ്റവും വലിയ സവിശേഷത. പുരുഷന്മാരെ വഴിതെറ്റിക്കുന്ന രീതിയിലുള്ള വസ്ത്രധാരണം സ്വീകരിക്കരുതെന്ന് പറയുന്ന മറ്റുള്ളവര്‍ക്ക് പലപ്പോഴും പ്രസ്തുത വസ്ത്രധാരണ രീതിയെക്കുറിച്ച് വ്യക്തമായൊരു ചിത്രം നല്‍കാന്‍ കഴിയാറില്ല. ഇസ്‌ലാം വിജയിക്കുന്നത് ഇവിടെയാണ്. ഇസ്‌ലാമിക വസ്ത്രധാരണാരീതി സ്വീകരിച്ചിരിക്കുന്ന സമൂഹങ്ങളില്‍ സ്ത്രീകള്‍ക്കെതിരെയുള്ള കുറ്റകൃത്യങ്ങള്‍ തുലോം വിരളമാണെന്ന വസ്തുത ‘അവര്‍ ശല്യപ്പെടാതിരിക്കാന്‍ വേണ്ടി” (33:59)എന്ന ഖുര്‍ആനിക നിര്‍ദേശത്തിന്റെ സത്യതയും പ്രായോഗികതയും വ്യക്തമാക്കുന്നതാണ്.

ഇസ്‌ലാമിക വസ്ത്രധാരണം സ്ത്രീയെ അടുക്കളയില്‍ തളച്ചിടുന്നതിനുവേണ്ടി സൃഷ്ടിച്ചെടുത്തതാണെന്ന ആരോപണം അടിസ്ഥാനരഹിതമാണ്. ഈ വസ്ത്രധാരണാരീതി സ്വീകരിച്ചുകൊണ്ടുതന്നെ സമൂഹത്തിന്റെ വിവിധ തുറകളില്‍ പ്രശോഭിച്ച ഒട്ടനവധി മഹിളാരത്‌നങ്ങളെക്കുറിച്ച് ചരിത്രം നമുക്ക് പറഞ്ഞുതരുന്നുണ്ട്. പ്രവാചകനില്‍നിന്ന് കാര്യങ്ങള്‍ പഠിക്കുകയും അദ്ദേഹത്തിന്റെ ജീവിതകാലത്തും ശേഷവും പ്രവാചകാനുചരന്മാരെ പഠിപ്പിക്കുകയും ചെയ്യുന്നതിന് പ്രവാചകപത്‌നി ആഇശ(റ)ക്ക് ഇസ്‌ലാമിക വസ്ത്രധാരണം ഒരു തടസ്സമായി നിന്നിട്ടില്ല. പ്രസ്തുത വസ്ത്രം ധരിച്ചുകൊണ്ടുതന്നെയായിരുന്നു അവര്‍ ജമല്‍ യുദ്ധം നയിച്ചത്. പുരുഷന്മാരില്‍ ഭൂരിപക്ഷവും യുദ്ധരംഗം വിട്ടോടിയ സന്ദര്‍ഭത്തില്‍ -ഉഹ്ദ് യുദ്ധത്തില്‍ -ആയുധമെടുത്ത് അടരാടിയ ഉമ്മു അമ്മാറ(റ)ധരിച്ചത് ഹിജാബ് തന്നെയായിരുന്നു. ഏഴ് യുദ്ധങ്ങളില്‍ പ്രവാചകനോടൊപ്പം പങ്കെടുത്ത് പരിക്കേറ്റവരെ പരിചരിച്ചും ഭക്ഷണം പാകം ചെയ്തും പ്രശസ്തയായ ഉമ്മുഅത്വിയ്യ(റ)ക്ക് തന്റെ ദൗത്യനിര്‍വഹണത്തിനു മുമ്പില്‍ ഇസ്‌ലാമിക വസ്ത്രധാരണം ഒരു വിലങ്ങായിത്തീര്‍ന്നിട്ടില്ല. ഇങ്ങനെ പ്രവാചകാനുചരന്മാരില്‍തന്നെ എത്രയെത്ര മഹിളാരത്‌നങ്ങള്‍! മുഖവും മുന്‍കൈയും മാത്രം പുറത്തുകാണിച്ചുകൊണ്ടുതന്നെ സമൂഹത്തിന്റെ വ്യത്യസ്ത തുറകളില്‍ വ്യക്തിമുദ്ര പതിപ്പിച്ച മഹതികള്‍!

ഇന്നും ഇസ്‌ലാമിക സമൂഹത്തില്‍ ഇത്തരം സഹോദരിമാരുണ്ട്. ഇസ്‌ലാമിക വസ്ത്രധാരണരീതി സ്വീകരിച്ചുകൊണ്ട് സാമൂഹിക മേഖലകളിലേക്ക് സേവന സന്നദ്ധരായി സധൈര്യം കയറിച്ചെല്ലുന്ന സഹോദരികള്‍. ഇസ്‌ലാമിക വസ്ത്രധാരണം സ്ത്രീയെ ചങ്ങലകളില്‍ ബന്ധിക്കുന്നുവെന്ന ആരോപണം അര്‍ഥമില്ലാത്തതാണെന്ന വസ്തുത ഇവിടെ അനാവൃതമാകുന്നു.

സത്യത്തില്‍, മാന്യമായി വസ്ത്രം ധരിക്കണമെന്ന് നിര്‍ദേശിക്കുക വഴി ഖുര്‍ആന്‍ സ്ത്രീകളുടെ ആത്മാഭിമാനം ഉയര്‍ത്തുകയും അവര്‍ ആക്രമിക്കപ്പെടുന്ന അവസ്ഥ ഇല്ലാതാക്കുവാനുള്ള പ്രായോഗിക പദ്ധതിക്ക് രൂപം നല്‍കുകയുമാണ് ചെയ്യുന്നത്.

വിഷയവുമായി ബന്ധപ്പെട്ട വീഡിയോ

ഒരു പുരുഷനു പകരം രണ്ടു സ്ത്രീകള്‍ സാക്ഷികളായി ഉണ്ടാവണമെന്ന ഖുര്‍ആനിന്റെ അനുശാസന സ്ത്രീയെ അവഗണിക്കുകയും അവളോട് അനീതി ചെയ്യുന്നതുമല്ലേയെന്നാണ് വിമർശനം.

കടമിടപാടുകളെ സംബന്ധിച്ച് പ്രതിപാദിക്കുന്നിടത്ത് ഖുര്‍ആന്‍ പറയുന്നു:

”നിങ്ങളില്‍പെട്ട രണ്ടു പുരുഷന്മാരെ നിങ്ങള്‍ സാക്ഷിനിര്‍ത്തുക. ഇനി ഇരുവരും പുരുഷന്മാരായില്ലെങ്കില്‍ നിങ്ങളിഷ്ടപ്പെടുന്ന സാക്ഷികളില്‍നിന്ന് ഒരു പുരുഷനും രണ്ടു സ്ത്രീകളും ആയാലും മതി. അവരില്‍ ഒരുവള്‍ക്ക് തെറ്റുപറ്റിയാല്‍ മറ്റവള്‍ അവളെ ഓര്‍മിപ്പിക്കുവാന്‍ വേണ്ടി” (2:283).

പല മതഗ്രന്ഥങ്ങളും സ്ത്രീ, സാക്ഷ്യത്തിനുതന്നെ അയോഗ്യയാണെന്നാണ് വിധിച്ചിരിക്കുന്നത്. ഉദാഹരണത്തിന്,യാജ്ഞവല്‍ക്യസ്മൃതിയുടെ വിധി കാണുക:

സ്ത്രീ ബാലവൃദ്ധ കിവത മത്തോന്‍മത്താഭിശസ്തകാഃ

രംഗാവതാരി പാഖണ്ഡി കുടകൃദ്വിലേന്ദ്രിയഃ

പതിതാപതാര്‍ത്ഥ സംബന്ധി സഹായരി പുതസ്‌കരാഃ

സാഹസീ ദൃഷ്ട ദോഷശ്ച നിര്‍ദ്ധുതാദ്യാസ്ത്വ സാക്ഷിണഃ (2:70,71).

(സ്ത്രീ, ബാലന്‍, വൃദ്ധന്‍ ചൂതുകളിക്കാരന്‍, മത്തനായവന്‍,ഉന്മാദമുള്ളവന്‍, ബ്രഹ്മഹത്യ തുടങ്ങിയ പാപമുള്ളവന്‍, ചാരണന്‍ (ഗായകന്‍, നടന്‍ തുടങ്ങിയവര്‍), പാഖണ്ഡി (നാസ്തികന്‍), വ്യാജരേഖ ചമക്കുന്നവന്‍, വികലാംഗന്‍, പതിതന്‍, സുഹൃത്ത്, പണം കൊടുക്കുന്നവന്‍, സഹായി, ശത്രു, കള്ളന്‍, സാഹസി (പിടിച്ചുപറിക്കാരന്‍), പ്രത്യക്ഷമായ ദോഷമുള്ളവന്‍, ബന്ധുക്കള്‍ ഉപേക്ഷിച്ചവന്‍ തുടങ്ങിയവര്‍ സാക്ഷികളാവാന്‍ യോഗ്യരല്ല)

എന്തുകൊണ്ടാണ് സ്ത്രീകളെ സാക്ഷ്യത്തിനു പറ്റാത്തത്?മനുസ്മൃതിയുടെ വിശദീകരണം ഇങ്ങനെയാണ്:

ഏകോലുബ്ധസ്തു സാക്ഷീസ്യാല്‍ ബഹ്യശ്ശൂ ച്യോപിന സ്ത്രീയഃ

സ്ത്രീ ബുദ്ധേര സ്ഥിരത്വാത്തു ദോഷൈശ്ചാന്യോപിയേ വൃതാഃ

(8:77)

(നിഷ്‌കാമനായ ഒരുത്തനെ സാക്ഷിയായി സ്വീകരിക്കാം. സ്ത്രീകള്‍ വളരെപ്പേരായാലും അവരുടെ ബുദ്ധിക്കു സ്‌ഥൈര്യമില്ലാത്തതിനാലും അവരെയും മുന്‍പറഞ്ഞ ദോഷികളെയും കടം മുതലായ വിഷയത്തില്‍ സാക്ഷിത്വേന സ്വീകരിക്കരുത്).

സ്ത്രീയെ സാക്ഷ്യത്തിനേ കൊള്ളുകയില്ലെന്ന നിലപാടുമായി ഇസ്‌ലാം വിയോജിക്കുന്നു. അവളെ സാക്ഷിയാക്കാമെന്നുതന്നെയാണ് ഇസ്‌ലാമിന്റെ നിലപാട്. എന്നാല്‍, അവളുടെ സാക്ഷ്യത്തിന് വ്യത്യസ്ത തലങ്ങളുണ്ട്. വിവാഹമോചനത്തെയും മരണസമയത്തെ വസ്വിയത്തിനെയും കുറിച്ച് പ്രതിപാദിക്കുമ്പോള്‍ ഖുര്‍ആന്‍ അവക്ക് രണ്ടു സാക്ഷികള്‍ വേണമെന്ന് നിഷ്‌കര്‍ഷിക്കുന്നുണ്ട് (65:2, 5:106).ഇവിടെയെല്ലാം സ്ത്രീയായിരുന്നാലും പുരുഷനായിരുന്നാലും രണ്ടു സാക്ഷികളാണ് വേണ്ടതെന്ന അഭിപ്രായക്കാരാണ് പ്രമുഖരായ ഇസ്‌ലാമിക പണ്ഡിതന്മാര്‍. അതുപോലെതന്നെ ആര്‍ത്തവം, പ്രസവം തുടങ്ങിയ കാര്യങ്ങളുമായി ബന്ധപ്പെട്ട വിഷയങ്ങളില്‍ സ്ത്രീകളുടെ സാക്ഷ്യം മാത്രമേ സ്വീകാര്യമാകൂ എന്ന കാര്യത്തില്‍ ഇസ്‌ലാമിക പണ്ഡിതന്മാര്‍ക്കിടയില്‍ അഭിപ്രായാന്തരങ്ങളൊന്നുമില്ല. സാദാചാരലംഘനം ആരോപിക്കപ്പെടുന്ന ഘട്ടങ്ങളില്‍ സത്യം ചെയ്യുകയും സ്വയം സാക്ഷ്യം വഹിക്കുകയും ചെയ്യേണ്ടിവരുമ്പോഴും സ്ത്രീ-പുരുഷ വ്യത്യാസങ്ങളൊന്നുമില്ലെന്നതാണ് ഖുര്‍ആനിക നിലപാട്. (ഖുര്‍ആന്‍ 24:6-9). എന്നാല്‍, കടമിടപാടുകളുടെ സ്ഥിതി ഇതില്‍നിന്ന് വ്യത്യസ്തമാണ്. സാക്ഷ്യത്തിനുതന്നെ സ്ത്രീകളെ കൊള്ളുകയില്ലായെന്ന ‘മത‘ വീക്ഷണം പുലര്‍ത്തുന്ന കാലത്താണ് സ്ത്രീയെ സാക്ഷ്യത്തിന് കൊള്ളുമെന്നും കടമിടപാടുകളുടെ കാര്യത്തില്‍ രണ്ടു സ്ത്രീകള്‍ ഒരു പുരുഷനു പകരം സാക്ഷ്യം വഹിച്ചാല്‍ മതിയെന്നുമുള്ള നിയമം ഖുര്‍ആന്‍ പ്രഖ്യാപിക്കുന്നത്. എന്തുകൊണ്ട് ഒരു പുരുഷനുപകരം രണ്ട് സ്ത്രീകള്‍ വേണം? ഉത്തരവും ഖുര്‍ആന്‍ തന്നെ പറയുന്നുണ്ട്: ”അവരില്‍ ഒരുവള്‍ക്ക് തെറ്റു പറ്റിയാല്‍ മറ്റവള്‍ അവളെ ഓര്‍മിപ്പിക്കുവാന്‍ വേണ്ടി”.

സത്യത്തില്‍ ഈ ഖുര്‍ആനിക നിര്‍ദേശം അതിന്റെ ദൈവികത മനസ്സിലാക്കിത്തരികയാണ് ചെയ്യുന്നത്; സ്ത്രീയെയും പുരുഷനെയും വ്യക്തമായി അറിയാവുന്ന സ്രഷ്ടാവിന്റെ നിയമസംഹിതയാണ് ഖുര്‍ആന്‍ എന്ന വസ്തുത. സ്ത്രീയെ തരം താഴ്ത്തുകയല്ല പ്രത്യുത അവളുടെ അബലതകള്‍ മനസ്സലാക്കുകയാണ് ഇവിടെ ഖുര്‍ആന്‍ ചെയ്യുന്നത്. നീതി നിര്‍വഹണത്തിന് ഉപയുക്തമാകുംവിധമായിരിക്കണം സ്ത്രീയുടെയും പുരുഷന്റെയും ഓരോ രംഗത്തെയും പങ്കാളിത്തം നിര്‍ണയിക്കേണ്ടതെന്ന ഖുര്‍ആനിന്റെ പൊതുതത്ത്വംതന്നെയാണ് ഇവിടെയും തെളിഞ്ഞുകാണുന്നത്. താഴെ പറയുന്ന വസ്തുതകള്‍ ശ്രദ്ധിച്ചാല്‍ ഇക്കാര്യം ബോധ്യമാവും.

ഒന്ന്: ഈ സൂക്തത്തില്‍ കടമിടപാടുകളെക്കുറിച്ചാണ് പരാമര്‍ശിച്ചിരിക്കുന്നത്. സാമ്പത്തിക ബാധ്യത പുരുഷന്മാരിലാണ് ഇസ്‌ലാം നിക്ഷിപ്തമാക്കുന്നതെന്നതിനാല്‍തന്നെ സാമ്പത്തിക ക്രയവിക്രയങ്ങളില്‍ പൊതുവേ പുരുഷന്മാരായിരിക്കും പങ്കാളികളായുണ്ടാവുക. ഇസ്‌ലാമിക സമൂഹത്തില്‍ പരസ്ത്രീ-പുരുഷ സംഗമം പ്രോല്‍സാഹിപ്പിക്കപ്പെടാത്തതിനാല്‍ പുരുഷന്മാര്‍ പരസ്പരമുള്ള ഇടപാടുകളിലും അവര്‍ മാത്രം വിഹരിക്കുന്ന രംഗങ്ങളിലും സ്ത്രീകള്‍ സാക്ഷികളായുണ്ടാവുക സ്വാഭാവികമല്ല. ഇടപാടുകള്‍ക്ക് സ്ത്രീകള്‍ സാക്ഷികളാണെങ്കില്‍തന്നെ അവര്‍ ഇസ്‌ലാമികമായ അച്ചടക്കം പാലിക്കാന്‍ ബാധ്യസ്ഥരുമാണ്. അങ്ങനെ അച്ചടക്കം പാലിക്കപ്പെടുന്ന അവസ്ഥയില്‍ ഇടപാടുകളുമായി ബന്ധപ്പെട്ട വ്യക്തികളെ വേണ്ട വിധം തിരിച്ചറിയാന്‍ കഴിഞ്ഞുകൊള്ളണമെന്നില്ല.

രണ്ട്: സ്ത്രീകള്‍ പൊതുവേ വികാരജീവികളാണ്. ചടുലമായ വികാരത്താല്‍ സ്വാധീനിക്കപ്പെടുന്ന സ്ത്രീ സത്യത്തില്‍നിന്നും വ്യതിചലിച്ചേക്കാന്‍ ഇടയുണ്ട്. സാക്ഷ്യം വഹിക്കപ്പെടുന്നത് സാക്ഷിനില്‍ക്കുന്നവളുടെ അസൂയയെ ഇളക്കിവിടാന്‍ മാത്രം സൗന്ദര്യമുള്ളവളുടെ കാര്യത്തിലായിരിക്കാം. അല്ലെങ്കില്‍ അവളിലെ മൃദുല വികാരങ്ങളെ തൊട്ടുണര്‍ത്താന്‍ പോന്ന ഒരു യുവാവിന്റെ കാര്യത്തിലായിരിക്കാം. മാതൃത്വത്തെ തഴുകിയുണര്‍ത്തുന്ന സാഹചര്യങ്ങളുടെ സ്വാധീനമുണ്ടാകാനും സാധ്യതയുണ്ട്. ഈ അവസ്ഥകളിലെ വൈകാരിക സമ്മര്‍ദങ്ങള്‍ അവളുടെ സാക്ഷ്യത്തെ സ്വാധീനിക്കാനിടയുണ്ട്.

മൂന്ന്: സ്ത്രീയുടെ ശാരീരികമായ പ്രത്യേകതകള്‍ അവളില്‍ പല തരത്തിലുള്ള പ്രയാസങ്ങളുമുണ്ടാക്കാറുണ്ട്. ആര്‍ത്തവത്തിന് തൊട്ടുമുമ്പുള്ള ദിവസങ്ങളിലെ മനഃസംഘര്‍ഷം, ഗര്‍ഭധാരണത്തിന്റെ ആദ്യനാളുകളിലെ ശാരീരിക- മാനസിക പ്രശ്‌നങ്ങള്‍, ആര്‍ത്തവ വിരാമത്തോടനുബന്ധിച്ചുള്ള പ്രയാസങ്ങള്‍, പ്രസവകാലത്തെ പ്രശ്‌നങ്ങള്‍,ഗര്‍ഭഛിദ്രമുണ്ടാക്കുന്ന മാനസികാഘാതം ഇങ്ങനെ ഒട്ടനവധി പ്രശ്‌നങ്ങള്‍ സ്ത്രീകള്‍ മാത്രം നേരിടേണ്ടവയാണ്. ഈ സാഹചര്യങ്ങളില്‍ ശാരീരിക പ്രയാസങ്ങള്‍ക്കുപുറമെ ഒട്ടനവധി മാനസിക പ്രശ്‌നങ്ങള്‍ക്കും സ്ത്രീകള്‍ വിധേയരാവുന്നുവെന്നാണ് ശാസ്ത്രീയ ഗവേഷണങ്ങള്‍ വ്യക്തമാക്കുന്നത്. മനോമാന്ദ്യം (slow mindedness), ഏകാഗ്രതയില്ലായ്മ, ഓര്‍മക്കുറവ് തുടങ്ങിയവ ഈ സാഹചര്യങ്ങളിലെ മാനസിക പ്രശ്‌നങ്ങളാണെന്നാണ് കണ്ടെത്തിയിട്ടുള്ളത്. സ്ത്രീകളുടെ സാക്ഷ്യത്തെക്കുറിച്ച് പറയുമ്പോള്‍ ഇത്തരം സാഹചര്യങ്ങളെകൂടി കണക്കിലെടുക്കേണ്ടതുണ്ടല്ലോ. ഒരു പുരുഷനുപകരം രണ്ടു സ്ത്രീകള്‍ സാക്ഷികളാവണമെന്ന് പറഞ്ഞ സൂക്തത്തില്‍ ‘ഒരുവള്‍ക്ക് തെറ്റിയാല്‍ മറ്റെവള്‍ ഓര്‍മിപ്പിക്കാന്‍ വേണ്ടി‘യെന്നാണ് പറഞ്ഞിട്ടുള്ളതെന്ന കാര്യം പ്രത്യേകം ശ്രദ്ധേയമാണ്.

സത്യത്തില്‍, ഈ ഖുര്‍ആനിക നിയമം സ്ത്രീകളുടെ വിലയിടിക്കുകയല്ല, പ്രത്യുത അവളുടെ അബലതകളും പ്രയാസങ്ങളും മനസ്സിലാക്കി അതിനുള്ള പരിഹാരമാര്‍ഗം നിര്‍ദേശിക്കുകയും അവള്‍ക്കുകൂടി പുരുഷനെപ്പോലെ സാക്ഷിയാകുവാനുള്ള അവസരം നല്‍കുകയുമാണ് ചെയ്യുന്നത്. മനോമാന്ദ്യത്തിന്റെയും ഓര്‍മക്കുറവിന്റെയും അവസ്ഥകളില്‍ ഒരുവളെ തിരുത്താന്‍ മറ്റവള്‍ക്ക് സാധിക്കുകയെന്നതാണ് ഇതിന്റെ താല്‍പര്യം. അതേസമയം,സ്ത്രീകളുമായി ബന്ധപ്പെട്ട വിഷയങ്ങളിലും അവളുടേതായ ഇടപാടുകളിലും ഒറ്റ സ്ത്രീയുടെ സാക്ഷ്യംതന്നെ പൂര്‍ണമായി പരിഗണിക്കപ്പെടുകയും ചെയ്യും. പ്രകൃതിമതത്തിന്റെ നിയമ നിര്‍ദേശങ്ങളെല്ലാം പ്രകൃതിയുമായി പൂര്‍ണമായും പൊരുത്തപ്പെടുന്നതാണെന്ന യാഥാര്‍ഥ്യമാണ് ഇവിടെ നമുക്ക് കാണാന്‍ കഴിയുന്നത്.

വിഷയവുമായി ബന്ധപ്പെട്ട വീഡിയോ

അനന്തര സ്വത്തില്‍ പുരുഷന് സ്ത്രീയുടേതിന്റെ ഇരട്ടി അവകാശമുണ്ടെന്നാണല്ലോ ഖുര്ആന്‍ അനുശാസിക്കുന്നത്. ഇത് വ്യക്തമായ വിവേചനമല്ലേ?

ത്യത്തില്‍, സ്ത്രീകള്‍ക്ക് അനന്തരസ്വത്ത് നല്‍കുവാന്‍ ആഹ്വാനം ചെയ്യുന്ന ഏക മതഗ്രന്ഥമാണ് ഖുര്‍ആന്‍. പരിഷ്‌കൃതമെന്നു വിശേഷിപ്പിക്കപ്പെടുന്ന പല രാജ്യങ്ങളും ഇരുപതാം നൂറ്റാണ്ടില്‍ മാത്രമാണ് സ്ത്രീക്ക് അനന്തര സ്വത്തില്‍ അവകാശം നല്‍കിയത്. ഖുര്‍ആനാകട്ടെ ഏഴാം നൂറ്റാണ്ടില്‍തന്നെ അസന്നിഗ്ധമായി പ്രഖ്യാപിച്ചു.”മാതാപിതാക്കളും അടുത്ത ബന്ധുക്കളും വിട്ടേച്ചുപോയ സ്വത്തില്‍ പുരുഷന്മാര്‍ക്ക് ഓഹരിയുണ്ട്. മാതാപിതാക്കളും അടുത്ത ബന്ധുക്കളും വിട്ടേച്ചുപോയ സ്വത്തില്‍ സ്ത്രീകള്‍ക്കും അവകാശമുണ്ട്” (4:7).

ബൈബിള്‍ പഴയനിയമപ്രകാരം പുത്രന്മാരുണ്ടെങ്കില്‍ അവര്‍ക്കു മാത്രമാണ് അനന്തര സ്വത്തില്‍ അവകാശമുള്ളത്. മരിച്ചയാളുടെ സ്വത്തുക്കള്‍ മക്കള്‍ക്കാണ് ഭാഗിച്ചുകൊടുക്കപ്പെടുന്നതെന്ന് സൂചിപ്പിക്കുന്ന വചനങ്ങള്‍ കാണാനാവും (ആവര്‍ത്തനം 21:15-17). പുത്രന്മാരില്ലെങ്കില്‍ പുത്രിമാര്‍ക്ക് അവകാശം നല്‍കണമെന്ന് നിര്‍ദേശമുണ്ട്. ”പുത്രനില്ലാതെ മരിക്കുന്നയാളുടെ പിന്തുടര്‍ച്ചാവകാശം പുത്രിക്കു ലഭിക്കുമാറാകണം” (സംഖ്യ 27:8). വിധവയ്ക്കുപോലും ഭര്‍ത്താവിന്റെ സ്വത്തില്‍ അവകാശമുണ്ടായിരുന്നില്ല (റവ. എ.സി. ക്ലെയ്റ്റണ്‍: ബൈബിള്‍ നിഘണ്ടു, പുറം 113).

ബൈബിള്‍ പുതിയ നിയമത്തിലാകട്ടെ ദായക്രമത്തെക്കുറിച്ച് പുതിയ നിയമങ്ങളൊന്നുംതന്നെ കാണാന്‍ കഴിയുന്നില്ല. ക്രൈസ്തവസഭ പൊതുവെ ഇക്കാര്യത്തില്‍ പഴയ നിയമത്തിലെ കല്‍പനകള്‍ അനുസരിക്കുകയായിരുന്നു ചെയ്തിരുന്നത്. അതുകൊണ്ടുതന്നെ ക്രൈസ്തവ ഭൂരിപക്ഷ പ്രദേശങ്ങളില്‍ ഈ അടുത്ത കാലംവരെ അനന്തരാവകാശം മാത്രമല്ല, സ്വത്തു സമ്പാദിക്കുവാന്‍ വരെ സ്ത്രീകള്‍ക്ക് അവകാശം നല്‍കപ്പെട്ടിരു ന്നില്ല. സ്വന്തം പേരില്‍ സ്വത്ത് സമ്പാദിക്കാന്‍ ന്യൂയോര്‍ക്കിലെ സ്ത്രീകളെ അനുവദിക്കുന്നത് 1848-ലാണ്. 1850-ലാണ് അമേരിക്കയിലെ എല്ലാ സംസ്ഥാനങ്ങളിലും സ്ത്രീകള്‍ക്ക് അനന്തരാവകാശം നല്‍കുന്ന നിയമം പ്രാബല്യത്തിലായത്.

പുരുഷന്റെ സ്വകാര്യ സ്വത്തായി സ്ത്രീയെ കണക്കാക്കുകയും അതുപ്രകാരമുള്ള നിയമങ്ങളാവിഷ്‌കരിക്കുകയും ചെയ്യുന്ന ഹൈന്ദവഗ്രന്ഥങ്ങള്‍ അവളെ അനന്തര സ്വത്തില്‍ പങ്കാളിയാക്കുന്നതിനെക്കുറിച്ച് പരാമര്‍ശിക്കുന്നുപോലുമില്ല. ഭര്‍ത്താവിനു ദാനം ചെയ്യാനും വില്‍ക്കാനും ഉപയോഗിക്കുവാനുമെല്ലാം അവകാശമുള്ള സ്വകാര്യ സ്വത്താണ് ഭാര്യ എന്നാണ് ഇതിഹാസകഥകള്‍ വായിച്ചാല്‍ നമുക്ക് ബോധ്യപ്പെടുക. അതിഥി പൂജക്കുവേണ്ടി സ്വന്തം ഭാര്യയെ നല്‍കുന്ന സുദര്‍ശനനും (മഹാഭാരതം അനുശാസനപര്‍വം) ഭാര്യയെ വസിഷ്ഠന് നല്‍കുന്ന മിത്രസഹനും (ശാന്തിപര്‍വം) നല്‍കുന്ന സൂചനയിതാണ്. പിതാവിന്റെ സ്വത്തില്‍ പെണ്‍മക്കള്‍ക്ക് അവകാശമുള്ളതായി സൂചിപ്പിക്കുന്ന വചനങ്ങളൊന്നും ഹിന്ദുമതഗ്രന്ഥങ്ങളില്‍ കാണാന്‍ കഴിയില്ല. പുത്രന്മാരാണ് അനന്തര സ്വത്തില്‍ അവകാശികളായിട്ടുള്ളവരെന്നാണ് മനുസ്മൃതിയുടെ നിയമം.

ഉൗര്‍ദ്ധ്വം പിതുശ്ച മാതുശ്ച സമേത്യ ഭ്രാതരഃ സ്സമം

ഭജേരന്‍ പൈതൃകം രിക്ഥമനീശാസ്‌തേ ഹി ജീവതൊ (9:104)

(മാതാപിതാക്കള്‍ രണ്ടുപേരും മരിച്ചശേഷം പുത്രന്മാരെല്ലാം ഒന്നുചേര്‍ന്ന് അവരുടെ സമ്പാദ്യം വിഭജിച്ച് എടുക്കണം. എന്തുകൊണ്ടെന്നാല്‍ മാതാപിതാക്കന്മാര്‍ ഇരിക്കുമ്പോള്‍ അവരുടെ ധനം പുത്രന്മാര്‍ക്കു സ്വാധീനമല്ല).

മാതാപിതാക്കളുടെ സ്വത്തില്‍ പുത്രന്മാര്‍ക്കും പുത്രിമാര്‍ക്കുമുള്ള അവകാശം ഖുര്‍ആന്‍ അംഗീകരിക്കുന്നു. പുത്രന്മാര്‍ക്കും പുത്രിമാര്‍ക്കും മാത്രമല്ല, മാതാപിതാക്കള്‍ക്കും ഭാര്യാഭര്‍ത്താക്കന്മാര്‍ക്കും സഹോദരീസഹോദരന്മാര്‍ക്കുമെല്ലാം മരണപ്പെട്ടയാളുടെ സ്വത്തിലുള്ള അവകാശം എത്രയാണെന്നും എങ്ങനെയാണെന്നുമെല്ലാം ഖുര്‍ആന്‍ വ്യക്തമായി പ്രതിപാദിക്കുന്നുണ്ട്. ഇതിന്റെ ഭാഗം മാത്രമാണ് പുത്രന്റെയും പുത്രിയുടെയും അവകാശം. അനന്തരാവകാശത്തെക്കുറിച്ച് വിശദമായി വിവരിക്കുന്ന ഖുര്‍ആന്‍ സൂക്തങ്ങളുടെ (4:11,12) തുടക്കം ഇങ്ങനെയാണ്: ”നിങ്ങളുടെ സന്താനങ്ങളുടെ കാര്യത്തില്‍ അല്ലാഹു നിങ്ങള്‍ക്ക് നിര്‍ദേശം നല്‍കുന്നു. ആണിന് രണ്ട് പെണ്ണിന്‍േറതിന് തുല്യമായ ഓഹരിയാണുള്ളത്” (4:11) മരിച്ചയാളുടെ പുത്രന് പുത്രിക്കു ലഭിക്കുന്നതിന്റെ ഇരട്ടി സ്വത്ത് അനന്തരമായി ലഭിക്കുമെന്ന് സാരം.

ഇത് സ്ത്രീകളോടുള്ള അവഗണനയാണോ? പുരുഷപക്ഷപാതം പ്രകടിപ്പിക്കുന്ന നിയമമാണോ? വിധി പറയുന്നതിനുമുമ്പ് താഴെ പറയുന്ന വസ്തുതകള്‍ മനസ്സിലാക്കുക.

ഒന്ന്: സ്ത്രീക്ക് സ്വത്ത് സമ്പാദിക്കുവാനുള്ള അവകാശം ഖുര്‍ആന്‍ അംഗീകരിക്കുന്നു. എത്ര വേണമെങ്കിലും സമ്പാദിക്കാം. പ്രസ്തുത സമ്പാദ്യത്തില്‍ പുരുഷന് യാതൊരു അവകാശവുമില്ല. അവളുടെ സമ്പാദ്യം അവളുടേതു മാത്രമാണ്.

രണ്ട്: സ്ത്രീയുടെയോ കുട്ടികളുടെയോ മാതാപിതാക്കളുടെയോ സംരക്ഷണത്തിനുള്ള ഉത്തരവാദിത്തം ഒരു പരിതസ്ഥിതിയിലും സ്ത്രീയുടെ ബാധ്യതയായിത്തീരുന്നില്ല. എത്രതന്നെ സമ്പത്തുള്ളവളായിരുന്നാലും തന്റെയും മക്കളുടെയും മാതാപിതാക്കളുടെയും ഭര്‍ത്താവിന്റെയും ചെലവ് വഹിക്കാന്‍ സ്ത്രീക്ക് ബാധ്യതയില്ലെന്നര്‍ഥം.

മൂന്ന്: വിവാഹാവസരത്തില്‍ വരനില്‍നിന് വിവാഹമൂല്യം നേടിയെടുക്കുവാന്‍ സ്ത്രീക്ക് അവകാശമുണ്ട്. പ്രസ്തുത വിവാഹമൂല്യം (മഹ്ര്‍) അവളുടെ സമ്പത്തായാണ് ഗണിക്കപ്പെടുന്നത്.

നാല്: കുടുംബത്തിന്റെ സംരക്ഷണം പുരുഷന്റെ ബാധ്യതയാണ്. ഭാര്യയുടെയും കുട്ടികളുടെയും ചെലവുകള്‍ വഹിക്കാന്‍ പുരുഷന്‍ ബാധ്യസ്ഥനാണ്. മാതാപിതാക്കളെയും അടുത്ത ബന്ധുക്കളെയും സംരക്ഷിക്കേണ്ട ബാധ്യതയും പുരുഷന്റെതുതന്നെ. എല്ലാവിധ സാമ്പത്തിക ബാധ്യതയും പുരുഷനാണുള്ളതെന്നര്‍ഥം.

അഞ്ച്: ഭാര്യ എത്രതന്നെ വലിയ പണക്കാരിയാണെങ്കിലും അവളുടെ സ്വത്തില്‍നിന്ന് അവളുടെ അനുവാദമില്ലാതെ ഒന്നും എടുത്തുപയോഗിക്കുവാന്‍ ഭര്‍ത്താവിന് അവകാശമില്ല.

ഇനി പറയൂ, സ്ത്രീയോട് നീതി പുലര്‍ത്തുകയാണോ അതല്ല വിവേചനം കാണിക്കുകയാണോ എന്താണ് ഖുര്‍ആന്‍ ചെയ്തിട്ടുള്ളത്?

സ്ത്രീക്ക് ലഭിക്കുന്ന അനന്തരസ്വത്ത് അവളുടേത് മാത്രമാണ്. മറ്റാര്‍ക്കും അതില്‍ യാതൊരു പങ്കുമില്ല. പുരുഷന് ലഭിക്കുന്നതോ?അവന്‍ വിവാഹമൂല്യം നല്‍കണം, സ്ത്രീയുടെ സംരക്ഷണം ഏറ്റെടുക്കണം, അവള്‍ക്കും കുട്ടികള്‍ക്കുമുള്ള ചെലവുകള്‍ വഹിക്കണം. എല്ലാം പുരുഷന്റെ ഉത്തരവാദിത്തം. അപ്പോള്‍ സ്ത്രീയെയാണോ പുരുഷനെയാണോ ഖുര്‍ആന്‍ കൂടുതല്‍ പരിഗണിച്ചിരിക്കുന്നത്?

സാമ്പത്തിക ബാധ്യതകള്‍ പുരുഷനില്‍ നിക്ഷിപ്തമാക്കുന്ന മറ്റു മതഗ്രന്ഥങ്ങളെല്ലാം പ്രസ്തുത ബാധ്യതകള്‍ക്കു പകരമായി അനന്തരാവകാശം പുരുഷനില്‍ പരിമിതപ്പെടുത്തുകയാണ് ചെയ്തിരിക്കുന്നത്. ഖുര്‍ആനാകട്ടെ എല്ലാ സാമ്പത്തിക ബാധ്യതകളും പുരുഷനാണെന്ന് പഠിപ്പിക്കുന്നതോടൊപ്പംതന്നെ സ്ത്രീക്ക് അനന്തരാവകാശം നല്‍കുകയും ചെയ്യുന്നു. പുരുഷന്റെ പകുതി അനന്തരസ്വത്ത് നല്‍കിക്കൊണ്ട് അത് അവളെ ബഹുമാനിക്കുകയാണ് ചെയ്തിരിക്കുന്നത്.

ഖുര്‍ആനിന്റെതല്ലാത്ത ഏത് നിര്‍ദേശമാണ് ഈ രംഗത്ത് വിമര്‍ശകരുടെ കൈവശമുള്ളത്? രണ്ട് നിര്‍ദേശങ്ങള്‍ ഉന്നയിക്കപ്പെടാം.

1.സ്ത്രീക്ക് പുരുഷന്റെ ഇരട്ടി സ്വത്ത് നല്‍കുക. സാമ്പത്തിക ബാധ്യതകള്‍ സ്ത്രീയില്‍ നിക്ഷപ്തമാക്കുക.

2.സ്ത്രീക്കും പുരുഷനും സ്വത്തില്‍ തുല്യാവകാശം നല്‍കുക. സാമ്പത്തിക ബാധ്യതകള്‍ തുല്യമായി വീതിച്ചെടുക്കുക.

ഈ രണ്ട് നിര്‍ദേശങ്ങളിലും സാമ്പത്തിക ബാധ്യതകള്‍ സ്ത്രീയില്‍ കെട്ടിയേല്‍പിക്കുകയാണ് ചെയ്യുന്നത്. സ്‌ത്രൈണപ്രകൃതിക്ക് വിരുദ്ധമായ ഒരു ആശയമാണിത്. ഗര്‍ഭകാലത്തും പ്രസവകാലത്തുമെല്ലാം പുരുഷന്റെ പരിരക്ഷയും സഹായവുമാണ് അവള്‍ കാംക്ഷിക്കുന്നത്. സാമ്പത്തിക ബാ ധ്യതകള്‍ ഒരു നിയമമെന്ന നിലയില്‍ സ്ത്രീയുടെ ചുമലില്‍ വെക്കുന്നത് ദൂരവ്യാപകമായ പ്രത്യാഘാതങ്ങള്‍ക്കു കാരണമാകും. അതുകൊണ്ട് സ്ത്രീക്ക് ഏറ്റവും അനുഗുണമായ നിയമംതന്നെയാണ് സ്വത്തവകാശത്തിന്റെ വിഷയത്തില്‍ ഖുര്‍ആന്‍ മുന്നോട്ടുവെച്ചിരിക്കുന്നത്.

‘നിങ്ങളുടെ ഭാര്യമാര്‍ നിങ്ങളുടെ കൃഷിയിടമാകുന്നു. അതിനാല്‍ നിങ്ങള്‍ ഇച്ഛിക്കുംവിധം നിങ്ങള്‍ക്ക് നിങ്ങളുടെ കൃഷിയിടത്തില്‍ ചെല്ലാവുന്നതാണ്‘ (2:223) എന്ന ഖുര്‍ആന്‍ സൂക്തമാണ് ഇവിടെ വിമര്‍ശിക്കപ്പെട്ടിരിക്കുന്നത്. ഭാര്യയെ കൃഷിയിടത്തോടുപമിക്കുന്ന ഖുര്‍ആന്‍ അവളെ വെറുമൊരു ഉല്‍പാദനോപകരണം മാത്രമാക്കിയെന്നാണ് ആക്ഷേപം.

ഖുര്‍ആനില്‍ ഒരുപാട് ഉപമാലങ്കാരങ്ങളുണ്ട്. സ്ത്രീയെ കൃഷിയിടത്തോടും വസ്ത്രത്തോടും ഉപമിക്കുന്നത് അവയില്‍ ചിലതുമാത്രം. ഉപമകള്‍ക്ക് ഒരു പ്രത്യേകതയുണ്ട്. ഓരോരുത്തര്‍ക്കും അവരുടെ മനോഗതംപോലെ അവയെ വ്യാഖ്യാനിക്കാന്‍ കഴിയും. പ്രസ്തുത വ്യാഖ്യാനങ്ങള്‍ വ്യാഖ്യാതാവിന്റെ മനസ്സിന്റെ നിമ്‌നോന്നതികളെയാണ് പ്രതിഫലിപ്പിക്കുക. കൃഷിസ്ഥലത്തോട് ഭാര്യയെ ഉപമിച്ചതിനാല്‍ കൃഷിയിടം ചവിട്ടിമെതിക്കുന്നതുപോലെ അവളെ ചവിട്ടിമെതിക്കാമെന്നും അത് വില്‍ക്കുന്നതുപോലെ സ്ത്രീയെ ഏതു സമയത്തും വില്‍പന നടത്താമെന്നും അതിനെ ഉഴുതുമറിക്കുന്നതുപോലെ അവളെ ഉഴുതുമറിക്കാമെന്നുമാണ് ഖുര്‍ആന്‍ പറയുന്നതെന്ന് ഒരാള്‍ക്ക് വാദിക്കാം. ഭാര്യയെ വസ്ത്രത്തോടുപമിച്ചതില്‍നിന്ന് അവളെ വസ്ത്രം മാറുന്നതുപോലെ മാറ്റുവാനാണ് ഖുര്‍ആന്‍ കല്‍പിക്കുന്നതെന്ന് വ്യാഖ്യാനിക്കാനും സാധിക്കും. പക്ഷേ, ഈ വ്യാഖ്യാനങ്ങളെല്ലാം വ്യാഖ്യാതാക്കളുടെ മനോഗതിയെയും മുന്‍ധാരണകളെയുമല്ലാതെ മറ്റൊന്നിനെയും പ്രതിഫലിപ്പിക്കുന്നില്ല എന്നല്ലോ മനഃശാസ്ത്ര മതം.

ഏതൊരു ഗ്രന്ഥത്തിലെയും ഉപമാലങ്കാരങ്ങളെ വ്യാഖ്യാനിക്കുവാന്‍ ആ ഗ്രന്ഥത്തിന്റെ ഇതിവൃത്തത്തെയും അത് പ്രഖ്യാപിക്കുന്ന ആദര്‍ശത്തെയും അത് മുന്നോട്ടുവെക്കുന്ന സാമൂഹികസംവിധാനത്തെയും കുറിച്ച അടിസ്ഥാന വസ്തുതകള്‍ അറിയേണ്ടതുണ്ട്. ‘സ്ത്രീകള്‍ക്ക് ബാധ്യതയുള്ളപോലെ അവകാശങ്ങളുമുണ്ട്‘(2:228) എന്ന ഖുര്‍ആന്‍ സൂക്തം സ്ത്രീപുരുഷബന്ധത്തെക്കുറിച്ച അതിന്റെ വീക്ഷണത്തെ സംബന്ധിച്ച അടിസ്ഥാനപരമായ അറിവ് നല്‍കുന്നുണ്ട്. ‘ഭൂമിയിലെ വിഭവങ്ങളില്‍ ഉത്തമമാണ് സദ്‌വൃത്തയായ സ്ത്രീ‘(മുസ്‌ലിം) എന്ന പ്രവാചക വചനം ഇതിന് അനുബന്ധമായി സ്ഥിതി ചെയ്യുന്നു. ഈ അടിത്തറയില്‍നിന്നുകൊണ്ട് സ്ത്രീയെക്കുറിച്ച ഉപമകള്‍ മനസ്സിലാക്കിയാലേ പ്രസ്തുത ഉപമകളുടെ സൗന്ദര്യം ആസ്വദിക്കാനാവൂ.

സ്ത്രീയെ വസ്ത്രത്തോടുപമിച്ച ഖുര്‍ആന്‍ എന്താണ് അര്‍ഥമാക്കുന്നത്? ശരീരവുമായി ഒട്ടിച്ചേര്‍ന്നുനില്‍ക്കുന്ന ഭൗതികമായി ഏറ്റവും അടുത്ത വസ്തുവാണ് വസ്ത്രം. അത് അന്യന്‍ കാണാതിരിക്കേണ്ട ശരീരഭാഗങ്ങളെ മറച്ചുവെക്കുന്നു. കാലാവസ്ഥയുടെ അസുഖകരമായ അവസ്ഥകളില്‍നിന്ന് ശരീരത്തെ സംരക്ഷിക്കുന്നത് വസ്ത്രമാണ്. മനുഷ്യന്റെ അന്തസ്സിന്റെ പ്രകടനവും വസ്ത്രത്തില്‍ കുടികൊള്ളുന്നു. സൗന്ദര്യവും ആനന്ദവും വര്‍ധിപ്പിക്കുന്നതിനും വസ്ത്രം ഉപയോഗിക്കുന്നു. സര്‍വോപരി ഒരാളുടെ സംസ്‌കാരത്തിന്റെ പ്രകടനമാണ് വസ്ത്രം. ഖുര്‍ആന്‍ സ്ത്രീയെ പുരുഷന്റെ വസ്ത്രമായി മാത്രമല്ല പറഞ്ഞിട്ടുള്ളതെന്ന കാര്യം പ്രത്യേകം പ്രസ്താവ്യമാണ്. ‘അവര്‍ നിങ്ങള്‍ക്കൊരു വസ്ത്രമാണ്, നിങ്ങള്‍ അവര്‍ക്കും ഒരു വസ്ത്രമാണ്‘ (2:187) എന്നാണ് ഖുര്‍ആന്‍ പറയുന്നത്. ഖുര്‍ആനിന്റെ ഉപമ എത്ര സുന്ദരം! കൃത്യം. പരസ്പരം വസ്ത്രമാകാതിരിക്കുന്നതല്ലേ ഇന്നത്തെ കുടുംബപ്രശ്‌നങ്ങളുടെ പ്രധാനപ്പെട്ട കാരണം?

സ്ത്രീയെ കൃഷിയിടത്തോടും പുരുഷനെ കൃഷിക്കാരനോടും ഉപമിച്ച ഖുര്‍ആന്‍ എന്താണ് അര്‍ഥമാക്കിയിരിക്കുന്നത്? കൃഷിയിടവും കൃഷിക്കാരനും തമ്മിലുള്ള ബന്ധമറിയാന്‍ കൃഷിക്കാരനോടുതന്നെ ചോദിക്കണം. കൃഷിയിടത്തിനുവേണ്ടി മരിക്കാന്‍ സന്നദ്ധനാണവന്‍. മണ്ണെന്ന് കേള്‍ക്കുമ്പോള്‍ അയാള്‍ വികാരതരളിതനാവും. കൃഷിഭൂമിയുടെ നിയമത്തെക്കുറിച്ച് അറിയുന്നവനാണവന്‍. സ്വന്തം കൃഷിയിടത്തില്‍ അന്യനെ വിത്തിടാന്‍ അയാള്‍ അനുവദിക്കുകയില്ല. അപരന്റെ കൃഷി സ്ഥലത്ത് വിത്തിറക്കാന്‍ അയാളൊട്ട് മുതിരുകയുമില്ല. കൃഷിഭൂമി പാഴാക്കരുത്്. തരിശിടരുത്. വളമിടണം. ജലസേചനം ചെയ്യണം. മണ്ണിന്റെ ഗുണം കൂട്ടണം. മണ്ണൊലിപ്പ് തടയണം.‘നിങ്ങളുടെ ഭാര്യമാര്‍ നിങ്ങളുടെ കൃഷിയിടമാണ്‘ (2:223) എന്ന ഖുര്‍ആനികാധ്യാപനം ശ്രവിക്കുന്ന കര്‍ഷകന് പെണ്ണിനെ കേവലം ഒരു ഉല്‍പാദനയന്ത്രമായി കാണാന്‍ കഴിയില്ല. കൃഷിയിടവും കര്‍ഷകനും തമ്മിലുള്ള ബന്ധത്തിന്റെ ആന്തരികമായ ആഴമറിയാത്തവര്‍ക്ക് ഈ ഉപമ ആസ്വദിക്കാന്‍ കഴിയില്ല. എന്നാല്‍ കൃഷിക്കാരന്റെ സ്ഥിതി അതല്ല. അവന്‍ പ്രസ്തുത ഉപമയുടെ അര്‍ഥം മനസ്സിലാക്കുന്നു. സൗന്ദര്യമുള്‍ക്കൊള്ളുന്നു. ഖുര്‍ആന്‍ സംസാരിക്കുന്നത് പച്ചയായ മനുഷ്യരോടാണ്; സാങ്കല്‍പിക ലോകത്ത് ബുദ്ധി വ്യായാമം ചെയ്യുന്ന‘ജീവി‘കളോടല്ലെന്ന കാര്യം പ്രത്യേകം പ്രസ്താവ്യമാണ്.

സ്ത്രീയെ കൃഷിയിടത്തോടുപമിച്ച ഖുര്‍ആനിക സൂക്തത്തിന്റെ അവതരണ പശ്ചാത്തലംകൂടി മനസ്സിലാക്കുന്നത് നന്നായിരിക്കും. സ്ത്രീകളുമായി ലൈംഗികബന്ധം പുലര്‍ത്തുന്നത് ചില പ്രത്യേക രീതികളിലായിരുന്നാല്‍ അത് പാപമാണെന്നും ജനിക്കുന്ന കുഞ്ഞിന്റെ കണ്ണിനു തകരാറുണ്ടാവുമെന്നുമുള്ള അന്ധവിശ്വാസങ്ങള്‍ മദീനയിലെ യഹൂദര്‍ക്കിടയില്‍ പ്രചരിച്ചിരുന്നു. ഇതിനെക്കുറിച്ച് അനുചരന്മാര്‍ പ്രവാചകനോട് (ﷺ) ചോദിച്ചു: അപ്പോഴാണ് ഈ സൂക്തം അവതരിപ്പിക്കപ്പെട്ടതെന്നാണ് പല ഖുര്‍ആന്‍ വ്യാഖ്യാതാക്കളുടെയും അഭിപ്രായം. ‘നിങ്ങളുടെ ഭാര്യമാര്‍ നിങ്ങളുടെ കൃഷിസ്ഥലമാകുന്നു. അതിനാല്‍ നിങ്ങള്‍ ഇച്ഛിക്കുന്ന വിധം നിങ്ങളുടെ കൃഷിസ്ഥലത്തു ചെല്ലുക‘ എന്ന സൂക്തത്തിന്റെ വിവക്ഷ ഈ പശ്ചാത്തലം വെച്ചുകൊണ്ട് മനസ്സിലാക്കുന്നത് തെറ്റിദ്ധാരണ നീങ്ങാന്‍ സഹായകമാവും. കൃഷിയിടത്തിലേക്ക് പല മാര്‍ഗങ്ങളുപയോഗിച്ച് കടന്നുചെല്ലുന്ന കൃഷിക്കാരനെപ്പോലെ ലൈംഗികബന്ധത്തില്‍ വ്യത്യസ്ത മാര്‍ഗങ്ങളുപയോഗിക്കുന്നതുകൊണ്ട് കുഴപ്പമില്ലെന്നാണ് ഖുര്‍ആന്‍ ഇവിടെ പഠിപ്പിക്കുന്നത്. കൃഷി സ്ഥലത്തുതന്നെയാണ് വിത്തുവിതക്കുന്നതെന്ന് ഉറപ്പുവരുത്തണമെന്നുമാത്രം. ഗുദമൈഥുനമൊഴിച്ചുള്ള മാര്ഗങ്ങളിലൂടെയെല്ലാം ലൈംഗീകാസ്വാദനം നേടാമെന്ന് പഠിപ്പിക്കുന്ന ഈ വചനം പെണ് വിരുദ്ധമാണെന്ന് പറയുന്നവർക്ക് ലൈംഗികതയെക്കുറിച്ചും അതിന്റെ മഴവിൽ വർണങ്ങളിലുള്ള ആസ്വാദനരീതികളെക്കുറിച്ചും യാതൊന്നുമറിയില്ലേയെന്ന് ആരെങ്കിലും സംശയിച്ചാൽ അവരെ കുറ്റം പറയാനാവില്ല. .

  'പുരുഷന്മാര്‍ സ്ത്രീകളുടെമേല്‍ നിയന്ത്രണാധികാരമുള്ളവരാകുന്നു‘(4:34).  

‘പുരുഷന്മാര്‍ക്ക് അവരേക്കാളുപരി ഒരു പദവിയുണ്ട്‘ (2:228).

വിശുദ്ധ ഖുര്‍ആനില്‍ പുരുഷമേധാവിത്തമാരോപിക്കുന്നവര്‍ഉദ്ധരിക്കാറുള്ള സൂക്തങ്ങളാണിവ. ഈ സൂക്തങ്ങള്‍ അറബികളുടെ ആണ്‍കോയ്മാവ്യവസ്ഥിതിയുടെ ഉല്‍പന്നമാണ് ഖുര്‍ആന്‍ എന്ന് വ്യക്തമാക്കുന്നതായി വാദിക്കപ്പെടുന്നു. എന്നാല്‍, വസ്തുതയെന്താണ്?

ആദ്യം ഉദ്ധരിക്കപ്പെട്ട സൂക്തത്തില്‍ പുരുഷന്‍ സ്ത്രീയുടെ മേല്‍‘ഖവ്വാം‘ ആണ് എന്നാണ് ഖുര്‍ആന്‍ പറയുന്നത്. ഒരാളുടെയോ സ്ഥാപനത്തിന്റെയോ കാര്യങ്ങള്‍ യഥോചിതം കൊണ്ടുനടക്കുകയും മേല്‍നോട്ടം വഹിക്കുകയും അതിനാവശ്യമായത് സജ്ജീകരിക്കുകയും ചെയ്യുന്ന വ്യക്തിക്കാണ് അറബിയില്‍ ‘ഖവ്വാം‘ എന്നും ‘ഖയ്യിം‘എന്നുമെല്ലാം പറയുന്നത്. അത് ഒരു അവകാശത്തേക്കാളധികം ഉത്തരവാദിത്തത്തെയാണ് ദ്യോതിപ്പിക്കുന്നത്. സ്ത്രീയും കുട്ടികളും അടങ്ങുന്ന കുടുംബമെന്ന സ്ഥാപനത്തിന്റെ നിയന്ത്രണാധികാരം,അതല്ലെങ്കില്‍ നിയന്ത്രണത്തിനുള്ള ഉത്തരവാദിത്തം പുരുഷനിലാണ് നിക്ഷിപ്തമായിരിക്കുന്നത് എന്നാണ് പ്രസ്തുത സൂക്തത്തിന്റെ സാരം.

കുടുംബം ഒരു സ്ഥാപനമാണ്. ഇരുപാതികള്‍ക്കും ശാന്തിയും സമാധാനവും സായൂജ്യവും പ്രദാനം ചെയ്യുന്ന മഹത്തായ സ്ഥാപനം. സാമൂഹിക സ്ഥാപനങ്ങളില്‍ ഏറ്റവും പ്രധാനപ്പെട്ടതാണ് കുടുംബമെന്നുള്ളതാണ് വാസ്തവം. ഏതൊരു സ്ഥാപനത്തിനും അതിന്റെ പ്രശ്‌നങ്ങള്‍ പരിഹരിക്കുന്നതിനും ഉയര്‍ച്ചക്കും വളര്‍ച്ചക്കും വേണ്ടിയത്‌നിക്കുന്നതിനും ഒരു മേലധികാരി ഉണ്ടായിരിക്കണമെന്ന കാര്യം ആരും അംഗീകരിക്കുന്നതാണ്. എത്രതന്നെ ആത്മാര്‍ഥമായ സംരംഭമാണെന്നിരിക്കിലും ഒരു നിയന്ത്രണാധികാരിയുടെ അഭാവത്തില്‍ അത് മുരടിച്ചുപോവുമെന്നത് കാര്യനിര്‍വഹണശാസ്ത്രത്തിന്റെ (administrative science) ബാലപാഠമെങ്കിലും മനസ്സിലാക്കിയിട്ടുള്ളവര്‍ക്ക് അറിയാവുന്നതാണ്്. അപ്പോള്‍ പിന്നെ സമൂഹത്തിന്റെ ധാര്‍മികാടിത്തറയുടെ രൂപീകരണം നടക്കുന്ന കുടുംബത്തിന് ഒരു മേലധികാരി ആവശ്യമില്ലേ? സ്ത്രീയും പുരുഷനും ചേര്‍ന്നുണ്ടാവുന്ന കൂട്ടുസ്ഥാപനമായ കുടുംബത്തിന്റെയും അതില്‍ വളര്‍ന്നുവരുന്ന സന്താനങ്ങളുടെയും അവയോടനുബന്ധിച്ചുണ്ടാവുന്ന ഉത്തരവാദിത്തങ്ങളുടെയും നടത്തിപ്പിന് ഒരു മേല്‍നോട്ടക്കാരന്‍ അത്യാവശ്യമാണ്. അല്ലാത്തപക്ഷം അരാജകത്വവും സര്‍വത്ര വിനാശവുമായിരിക്കും ഫലം.

കുടുംബത്തിന് നായകത്വം വഹിക്കുവാന്‍ ഒരാള്‍ വേണമെന്ന് വ്യക്തം. ആര്‍ക്കാണിതിന് അര്‍ഹതയുള്ളത് എന്നു ചോദിക്കുന്നതിനേക്കാള്‍ ആര്‍ക്കാണതിന് സാധിക്കുകയെന്ന് പരതുന്നതാവും ശരി. ഒന്നുകില്‍ രണ്ടുപേരും കൂടി നായകത്വം വഹിക്കുക. അല്ലെങ്കില്‍ സ്ത്രീ കുടുംബത്തിന്റെ നായകത്വമേറ്റെടുക്കുക. ഇവ രണ്ടും പ്രായോഗികമല്ലെങ്കില്‍ മാത്രം പുരുഷനെ ആ ചുമതല ഏല്‍പിക്കുക എന്ന പൊതുധാരണയുടെ അടിസ്ഥാനത്തില്‍ നാം പ്രശ്‌നത്തെ സമീപിക്കുക; നിഷ്‌കളങ്കതയോടുകൂടി.

ഒരു സ്ഥാപനത്തിന് മേലധികാരിയില്ലാതിരിക്കുന്നതിനേക്കാള്‍ അപകടമാണ് അതിന് രണ്ടു നായകന്മാരുണ്ടാവുകയെന്നത്. സ്ഥാപനങ്ങള്‍ നോക്കിനടത്തിയിരുന്ന അച്ഛന്‍ മരിച്ചാല്‍ ഉടന്‍ അവ വിഭജിച്ചെടുക്കുകയോ അല്ലെങ്കില്‍ മക്കളില്‍ ആരെങ്കിലുമൊരാളെ നിയന്ത്രണാധികാരം ഏല്‍പിക്കുകയോ ചെയ്യാതിരുന്നാലുണ്ടാവാറുള്ള പ്രശ്‌നങ്ങളില്‍ പലപ്പോഴും സ്ഥാപനങ്ങള്‍ തന്നെ തകര്‍ന്ന് നാശമാകാറാണ് പതിവ്. ഒന്നിലധികം നായകന്മാരുള്ള സ്ഥാപനങ്ങളില്‍ നായകത്വത്തിന് വേണ്ടിയുള്ള കിടമല്‍സരങ്ങളും പ്രശ്‌നങ്ങളും കാരണം അതു തകരും. തകരാതെ നിലനില്‍ക്കുന്നുവെങ്കില്‍തന്നെ അതിന്റെ‘ഉല്‍പന്നങ്ങള്‍‘ക്ക് എന്തെങ്കിലും വൈകല്യങ്ങളുണ്ടാവും. കുടുംബത്തിന്റെ അവസ്ഥയും ഇതുതന്നെ. രണ്ടുപേരെയും നായകന്മാരാക്കിയാല്‍ പ്രശ്‌നങ്ങളിലുള്ള സമീപനത്തെക്കുറിച്ച സംഘട്ടനങ്ങളുണ്ടാവും. ഇതു നേതൃത്വത്തിനുവേണ്ടിയുള്ള മല്‍സരത്തില്‍ കലാശിക്കും. അശാന്തമായ കുടുംബാന്തരീക്ഷമായിരിക്കും ഇതിന്റെ ഫലം. അത്തരമൊരു കുടുംബാന്തരീക്ഷത്തില്‍ വളരുന്ന കുഞ്ഞുങ്ങളില്‍ മാനസിക സംഘര്‍ഷങ്ങളും വൈകാരിക താളപ്പിഴകളുമുണ്ടാവും. അത് അടുത്ത തലമുറയില്‍ ധാര്‍മികത്തകര്‍ച്ചക്ക് നിമിത്തമാകും.

കുടുംബത്തിന്റെ നിയന്ത്രണാധികാരം സ്ത്രീക്ക് ഏറ്റെടുക്കുവാന്‍ പറ്റുമോ? അതല്ല പുരുഷനിലാണോ ആ ഉത്തരവാദിത്തം ഏല്‍പിക്കേണ്ടത്? ഈ ചോദ്യത്തിനുള്ള ഉത്തരം ‘കുടുംബത്തിന്റെ നിയന്ത്രണത്തിനാവശ്യം വിചാരമോ അതല്ല വികാരമോ?, എന്ന മറുചോദ്യമാണ്.വിചാരമെന്നാണ് ഉത്തരമെങ്കില്‍ പുരുഷനെയാണ് കുടുംബത്തിന്റെ നിയന്ത്രണമേല്‍പിക്കേണ്ടത്, വികാരമെന്നാണെങ്കില്‍ സ്ത്രീയെയും.

സ്ത്രീയുടെയും പുരുഷന്റെയും ശരീരപ്രകൃതിയും മാനസികാവസ്ഥയും അവരേറ്റെടുക്കേണ്ട ധര്‍മത്തിനനുസൃതമായ രീതിയിലാണ് സംവിധാനിക്കപ്പെട്ടിരിക്കുന്നത്. ശാരീരിക പ്രകൃതിയെന്നു പറയുമ്പോള്‍ കേവലം ബാഹ്യമായ വ്യത്യാസങ്ങള്‍ മാത്രമല്ല വിവക്ഷിക്കുന്നത്. അസ്ഥി വ്യവസ്ഥ മുതല്‍ പേശീവ്യവസ്ഥ വരെയുള്ള ആന്തരിക വ്യവസ്ഥകള്‍ പോലും ഓരോരുത്തര്‍ക്കും പ്രകൃത്യാ നിശ്ചയിക്കപ്പെട്ട ധര്‍മത്തിനനുസൃതമായ രീതിയിലാണുള്ളത്. പ്രസിദ്ധ ലൈംഗികശാസ്ത്രജ്ഞനായ ഹാവ്‌ലോക്ക് എല്ലിസിന്റെ ‘ആണ് തന്റെ കൈവിരല്‍ തുമ്പുവരെ പുരുഷനും സ്ത്രീ തന്റെ കാല്‍വിരല്‍ തുമ്പുവരെ പെണ്ണുമാണ്‘ എന്ന പ്രസിദ്ധമായ അഭിപ്രായം നൂറുശതമാനം ശരിയാണെന്നുള്ളതാണ് വസ്തുത.

പുരുഷന്റെ എല്ലുകള്‍ അധ്വാനത്തിനു പറ്റിയ രീതിയിലുള്ളവയാണെങ്കില്‍ സ്ത്രീയുടേത് ഗര്‍ഭധാരണത്തിന് അനുയോജ്യമായതാണ്. കഠിനാധ്വാനത്തിനാവശ്യമായ പേശികളാണ് പുരുഷനുള്ളതെങ്കില്‍ മാംസളതയും മിനുസവും നല്‍കുന്ന കൊഴുപ്പാണ് സ്ത്രീ ശരീരത്തിലുള്ളത്. അധ്വാനത്തിന് പറ്റിയ രീതിയിലുള്ള ആണിന്റെ കൈകള്‍! ആലിംഗനത്തിന് പറ്റുന്ന പെണ്ണിന്റെ കൈകള്‍…! ഇങ്ങനെ പോകുന്നു ശാരീരിക വ്യത്യാസങ്ങള്‍.

മാതൃത്വത്തിന് പറ്റിയ രീതിയില്‍ സ്ത്രീ ശരീരവും അധ്വാനത്തിന് സാധിക്കുന്ന രൂപത്തില്‍ പുരുഷശരീരവും സംവിധാനിക്കപ്പെട്ടപ്പോള്‍ അവരവരുടെ ധര്‍മത്തിന് അനുഗുണമായ മാനസിക ഗുണങ്ങളും അതിനോടനുബന്ധിച്ച് നല്‍കപ്പെട്ടിരിക്കുമല്ലോ. ദയയും വാല്‍സല്യവും ക്ഷിപ്രവൈകാരികതയുമാണ് സ്ത്രീ മനസ്സിന്റെ പ്രത്യേകതകള്‍. അത് വികാരപ്രധാനമാണ്. ശൈശവത്തിലും ബാല്യത്തിലും പെണ്‍കുട്ടികള്‍ കാണിക്കുന്ന ബൗദ്ധിക കഴിവുകള്‍ പോലും കൗമാരത്തോടെ മന്ദീഭവിക്കുന്നുവെന്നാണ് പഠനങ്ങള്‍ കാണിക്കുന്നത്. മാതൃത്വത്തിന് തയാറാകുമ്പോള്‍ മനസ്സും അതിനൊത്ത് മാറുന്നുവെന്നര്‍ഥം. സ്ത്രീയുടെ മനസ്സിനെക്കുറിച്ച് വ്യവഹരിക്കുമ്പോള്‍ നമ്മുടെ മനസ്സിലോടിയെത്തുന്നതെന്താണ്? അലിവാര്‍ന്ന ഹൃദയം, അതി ലോലമായ മനസ്സ്, പെട്ടെന്ന് പ്രതികരിക്കുന്ന പ്രകൃതം, നിരന്തരം നിര്‍ഗളിക്കുന്ന സ്‌നേഹവായ്പ്, നുരഞ്ഞുപൊങ്ങുന്ന വൈകാരികത…ഇതെല്ലാംതന്നെ സ്ത്രീമനസ്സ് വികാരപ്രധാനമാണെന്ന് വ്യക്തമാക്കുന്നു. എന്നാല്‍,പുരുഷമനസ്സിന്റെ അവസ്ഥയോ? ചിന്തിച്ചുള്ള പ്രതികരണം,പാരുഷ്യത്തോടെയുള്ള പെരുമാറ്റം, അവധാനതയോടുകൂടിയുള്ളപ്രത്യുത്തരം, ആലോചനയോടെയുള്ള പ്രവര്‍ത്തനം. ഇവയാണ് പുരുഷമനസ്സിന്റെ പ്രതിബിംബം. ഇവ വിചാരപ്രധാനമാണ്. അധ്വാനത്തിന് പറ്റിയ രീതിയില്‍ പുരുഷമനസ്സ് സംവിധാനിക്കപ്പെട്ടിരിക്കുന്നവെന്ന് സാരം.

(ഇത് പൊതുവായ വിലയിരുത്തലാണ്. ഭരിക്കാനും നീതിന്യായം നടത്താനും യുദ്ധം നയിക്കാനും ഭാരം ചുമക്കാനും കഠിനാധ്വാനം ചെയ്യാനും കഴിയുന്ന സ്ത്രീകളില്ലേ? പാചകത്തിനും വാല്‍സല്യത്തോടെ ശിശുക്കളെ പോറ്റുവാനും കുടുംബഭരണത്തിനും പറ്റിയ പുരുഷന്മാരില്ലേ? ‘ഉണ്ട്‘ എന്നുതന്നെയാണുത്തരം. ഇത് ചില അപവാദങ്ങള്‍ മാത്രമാണ്. അവര്‍ പലപ്പോഴും ലൈംഗികമായി മാത്രമേ തങ്ങളുടെ ലിംഗത്തിലുള്ളവരില്‍ ഉള്‍പ്പെടുകയുള്ളൂ. പെരുമാറ്റത്തിലും രീതിയിലും ധര്‍മനിര്‍വഹണത്തിലും എതിര്‍ ലിംഗത്തിലുള്ളവരോടായിരിക്കും അവര്‍ക്ക് അടുപ്പം)

കുടുംബത്തിന്റെ രക്ഷാധികാരത്തിന് പുരുഷനെ പ്രാപ്തനാക്കുന്നത് വിചാരത്തോടുകൂടി കാര്യങ്ങള്‍ കൈകാര്യം ചെയ്യാനുള്ള അവന്റെ കഴിവാണ്. അവന്റെ ശാരീരിക ഘടന അവനില്‍ അടിച്ചേല്‍പിച്ച ധര്‍മത്തിന്റെ നിര്‍വഹണമാണത്. അവന്‍ അധ്വാനിക്കണം, കുടുംബത്തെ പോറ്റുവാനുള്ള സമ്പത്തുണ്ടാക്കണം -അവനിലാണ് കുടുംബത്തിന്റെ കൈകാര്യകര്‍തൃത്വം ഏല്‍പിക്കപ്പെട്ടിരിക്കുന്നത്. ആ സ്ഥാപനത്തിന്റെയും അതിലുള്‍പ്പെട്ടവരുടെയും ജീവിതച്ചെലവിനുവേണ്ടിയുള്ള ആസൂത്രണവും ആ മാര്‍ഗത്തിലുള്ള സാമ്പത്തിക മേല്‍നോട്ടവും അവന്റെ ബാധ്യതയാക്കിത്തീര്‍ക്കുകയാണ് ഈ കൈകാര്യകര്‍തൃത്വം ചെയ്യുന്നത്. അതുകൊണ്ടാണ് ‘പുരുഷന്മാര്‍ സ്ത്രീകളുടെമേല്‍ നിയന്ത്രണാധികാരമുള്ളവരാകുന്നു‘വെന്ന് പറഞ്ഞതോടൊപ്പംതന്നെ അതിന്റെ കാരണമായി ‘മനുഷ്യരില്‍ ഒരു വിഭാഗത്തിന് മറുവിഭാഗത്തേക്കാള്‍ അല്ലാഹു കൂടുതല്‍ കഴിവ് നല്‍കിയതിനാലും അവരുടെ ധനം ചെലവഴിച്ചതിനാലുമാണിത്‘(4:34)എന്ന് ഖുര്‍ആന്‍ എടുത്തു പറഞ്ഞത്്. കുടുംബത്തിന്റെ നിയന്ത്രണാധികാരം നല്‍കുക വഴി പുരുഷനുമേല്‍ ഒരു വലിയ ഉത്തരവാദിത്തമേല്‍പിക്കുകയാണ് ഖുര്‍ആന്‍ ചെയ്യുന്നതെന്ന് പറയാന്‍ ഇതാണ് കാരണം.

സ്ത്രീയുടെ മേലും ഗൃഹഭരണത്തിന്‍മേലും പുരുഷന്‍ ഏകാധിപതിയായിരിക്കണമെന്നല്ല അവന്ന് നിയന്ത്രണാധികാരം നല്‍കിയതുകൊണ്ട് വിവക്ഷിക്കുന്നത്. പരസ്പര സഹകരണവും കൂടിയാലോചനയുമുണ്ടാവുമ്പോഴേ നായകത്വം ജീവസ്സുറ്റതാവൂ.‘സ്ത്രീകളുമായി നന്മയില്‍ വര്‍ത്തിക്കണം‘(4:19) എന്ന ഖുര്‍ആനിക നിര്‍ദേശവും, ‘നിങ്ങളുടെ വീട്ടുകാരോട് നന്നായി പെരുമാറുന്നവനാണ് നിങ്ങളില്‍ ഉത്തമന്‍‘ (ബുഖാരി) എന്ന പ്രവാചകന്റെ ഉപദേശവും നായകത്വമേല്‍പിക്കപ്പെട്ട പുരുഷന്‍ സ്വീകരിക്കുമ്പോഴാണ് സംതൃപ്തമായ കുടുംബജീവിതം സംജാതമാവുക.

‘പുരുഷന്മാര്‍ക്ക് സ്ത്രീകളേക്കാള്‍ ഒരു പദവിയുണ്ട്.(2:228) എന്നു ഖുര്‍ആന്‍ പറഞ്ഞതും ഈ അടിസ്ഥാനത്തിലാണ്. കുടുംബത്തിന്റെ സാമ്പത്തിക ഭാരമേറ്റെടുക്കുന്നതിലൂടെ കൈവരുന്ന പദവിയാണിത്. കുടുംബത്തിന്റെ രക്ഷാകര്‍തൃത്വമാണ് ആ പദവി.

ഉയര്‍ന്ന ശമ്പളമുള്ള ഒരു വനിതക്ക് കുടുംബത്തിന്റെ നായകത്വം നല്‍കിയെന്നുവെക്കുക. വൈവാഹിക ജീവിതത്തിന്റെ ആദ്യനാളുകളില്‍ കുടുംബസംരക്ഷണമെന്ന ഉത്തരവാദിത്വം തലയിലെത്തുന്നതിന് മുമ്പ് അത് അവള്‍ക്കൊരു പ്രയാസമായി അനുഭവപ്പെടുകയില്ലായിരിക്കാം. എന്നാല്‍,അവള്‍ ഗര്‍ഭിണിയും അമ്മയുമാവുമ്പോള്‍ നായകത്വത്തിന്റെ ഭാരം ചുമക്കാന്‍ അവള്‍ക്ക് കഴിയില്ല. പുരുഷനില്‍ കുടുംബനായകത്വമേല്‍പിക്കുന്നതിലൂടെ ഖുര്‍ആന്‍ സ്ത്രീക്ക് തണലേകുകയാണ് ചെയ്തിട്ടുള്ളതെന്ന് സാരം. സ്‌ത്രൈണതയെക്കുറിച്ചറിയുന്നവരൊന്നും ഇക്കാര്യത്തില്‍ ഖുര്‍ആനിന് എതിര് നില്‍ക്കുകയില്ല.

വിഷയവുമായി ബന്ധപ്പെട്ട വീഡിയോ

ഇസ്‌ലാമിന് ആണ്കോയ്മയോ പെണ്കോയ്മയോ പരിചയമില്ല. ആണിനോടും പെണ്ണിനോടും തങ്ങളുടെ ബാധ്യത കളെയും അവകാശങ്ങളെയുംകുറിച്ച് സംസാരിക്കുന്ന ആദർശമാണ് ഇസ്‌ലാം. ”സ്ത്രീകള്‍ക്ക് ബാധ്യതകള്‍ ഉള്ളതുപോലെതന്നെ ന്യായമായ അവകാശങ്ങളുമുണ്ട്”(2:228) എന്നാണ് ഖുര്‍ആന്‍ അസന്നിഗ്ധമായി പ്രഖ്യാപിക്കുന്നത്. പെണ്ണവകാശങ്ങളെക്കുറിച്ച് ഈ രൂപത്തിലുള്ള ഒരു പ്രസ്താവന പതിനെട്ടാം നൂറ്റാണ്ടിനു മുൻപ് എഴുതപ്പെട്ട ഏതെങ്കിലും ധര്മശാസ്ത്രഗ്രന്തങ്ങളിൽ കാണാനാവുമോ എന്ന ചോദ്യത്തിന്റെ ഉത്തരം ഇല്ലായെന്നാണ്. ന്യായമായും നൽകേണ്ട പെണ്ണവകാശങ്ങളെക്കുറിച്ച് ഇത്രയും വ്യക്തമായി പ്രഖ്യാപിക്കുന്ന ഖുര്‍ആന്‍ ആണ്‍കോയ്മാ വ്യവസ്ഥിതിയുടെ സൃഷ്ടിയാണെന്ന് പറയുന്നതെങ്ങനെ?സ്ത്രീയുടെ അവകാശങ്ങളെക്കുറിച്ച് ഖുര്‍ആനിനെപ്പോലെ വ്യക്തവും വിശദവുമായി പ്രതിപാദിക്കുന്ന മറ്റൊരു ധര്മശാസ്ത്രഗ്രന്ഥവുമില്ലെന്നതാണ് വാസ്തവം.

സ്ത്രീക്ക് ഇസ്‌ലാം അനുവദിച്ച-അല്ല, നേടിക്കൊടുത്ത-അവകാശങ്ങളുടെ മഹത്വമറിയണമെങ്കില്‍ അതിന്റെ അവതരണകാലത്തുണ്ടായിരുന്ന പെണ്ണിന്റെ പദവിയെന്തായിരുന്നുവെന്ന് മനസ്സിലാക്കണം. യവനന്മാര്‍ പി ശാചിന്റെ പ്രതിരൂപമായിട്ടായിരുന്നു പെണ്ണിനെ കണ്ടിരുന്നത്. പത്‌നിയെ അറുകൊല നടത്താന്‍ പോലും പുരുഷന് സ്വാതന്ത്ര്യം നല്‍കുന്നതായിരുന്നു റോമന്‍ നിയമവ്യവസ്ഥ. ഭര്‍ത്താവിന്റെ ചിതയില്‍ ചാടി മരിക്കണമെന്നതായിരുന്നുവല്ലോ ഭാരതീയ സ്ത്രീയോടുള്ള മതോപദേശം. പാപം കടന്നുവരാന്‍ കാരണക്കാരിയായ (?) പെണ്ണിനു നേരെയുള്ള യഹൂദന്മാരുടെ പെരുമാറ്റം ക്രൂരമായിരുന്നു. യഹൂദമതത്തിന്റെ പിന്തുടര്‍ച്ചയായി വന്ന ക്രിസ്തുമതത്തിലെ സ്ഥിതിയും മെച്ചപ്പെട്ടതായിരുന്നില്ല. സ്ത്രീകള്‍ക്ക് ആത്മാവുണ്ടോ എന്നതായിരുന്നു പതിനാറാം നൂറ്റാണ്ടില്‍ പോലും പാതിരിമാരുടെ ചര്‍ച്ചാവിഷയം. മുഹമ്മദ് നബി(ﷺ)ക്ക് മുമ്പ് അറേബ്യയിലെ പെണ്ണിന്റെ സ്ഥിതിയും ഇതിനേക്കാളെല്ലാം കഷ്ടമായിരുന്നു. അവള്‍ക്ക് ജീവിക്കാനുള്ള അവകാശം പോലും നിഷേധിക്കപ്പെട്ടു. പ്രസവിക്കപ്പെട്ടത് പെണ്‍കുഞ്ഞാണെന്നറിഞ്ഞാല്‍ ജീവനോടെ കുഴിച്ചുമൂടാന്‍ സന്നദ്ധരായിരുന്ന ജനങ്ങളുള്‍ക്കെളളുന്ന സമൂഹം. ഇത്തരമൊരു സാമൂഹിക സാഹചര്യത്തിലാണ് സ്ത്രീകളുടെ അവകാശങ്ങളെക്കുറിച്ച് ഖുര്‍ആന്‍ സംസാരിക്കുവാനാരംഭിച്ചത്.

പെണ്ണിന് ഖുര്‍ആന്‍ നല്‍കിയ അവകാശങ്ങളെ ഇങ്ങനെ സംഗ്രഹിക്കാം:

1.ജീവിക്കാനുള്ള അവകാശം: ഭാര്യ പ്രസവിച്ചത് പെണ്‍കുഞ്ഞാണെന്ന് മനസ്സിലാക്കിയാല്‍ അതിനെ കൊന്നുകളയുന്നതിനെക്കുറിച്ച് ചിന്തിച്ചിരുന്നവരായിരുന്നു അറബികള്‍ (ഖുര്‍ആന്‍16:59). ആധുനിക സാങ്കേതിക വിദ്യകളുപയോഗിച്ച് ഭ്രൂണത്തിന്റെ ലിംഗം നിര്‍ണയിക്കുകയും പ്രസവിക്കാന്‍ പോകുന്നത് പെണ്‍കുഞ്ഞാണെന്നറിഞ്ഞാല്‍ അതിനെ ഭ്രൂണാവസ്ഥയില്‍തന്നെ നശിപ്പിക്കാനൊരുമ്പെടുകയും ചെയ്യുന്ന സമകാലീന സമൂഹത്തിന്റെ ധാര്‍മിക നിലവാരം അജ്ഞാനകാലത്തെ അറബികളില്‍ നിന്ന് അല്‍പം പോലും ഉയര്‍ന്നതല്ല. പെണ്ണിനെ ജീവിക്കുവാന്‍ അനുവദിക്കാത്ത കുടിലതയെ ഖുര്‍ആന്‍ വിമര്‍ശിക്കുന്നു (16:59,81:9) പുരുഷനെപ്പോലെ അവള്‍ക്കും ജനിക്കുവാനും ജീവിക്കാനും അവകാശമുണ്ടെന്ന് അത് പ്രഖ്യാപിക്കുന്നു.

2.സ്വത്തവകാശം: പുരുഷനെപ്പോലെ സമ്പാദിക്കാനുള്ള അവകാശം ഖുര്‍ആന്‍ സ്ത്രീക്ക് നല്‍കുന്നു. സ്വന്തമായി ഉണ്ടാക്കിയതോ അനന്തരമായി ലഭിച്ചതോ ആയ സമ്പാദ്യങ്ങളെല്ലാം അവളുടേത് മാത്രമാണ് എന്നാണ് ഖുര്‍ആനിന്റെ കാഴ്ചപ്പാട്.സ്ത്രീക്ക് സ്വന്തം പേരിൽ സ്വത്ത് സമ്പാദിക്കുവാൻ 1870 നു ശേഷം മാത്രമാണ് യൂറോപ്പിൽ നിയമപരമായ അനുമതി ലഭിച്ചതെന്ന ചരിത്രം ഇതോട് ചേർത്ത് വായിക്കേണ്ടതാണ്. സ്ത്രീയുടെ സമ്പാദ്യത്തില്‍നിന്ന് അവളുടെ സമ്മതമില്ലാതെ യാതൊന്നും എടുക്കുവാന്‍ ഭര്‍ത്താവിന് പോലും അവകാശമില്ലെന്നാണ് ഇസ്‌ലാമിക നിയമം. .”പുരുഷന്മാര്‍ക്ക് അവര്‍ സമ്പാദിച്ചതിന്റെ വിഹിതവും സ്ത്രീകള്‍ക്ക് അവര്‍ സമ്പാദിച്ചതിന്റെ വിഹിതവുമുണ്ട് (ഖുര്‍ആന്‍ 4:32).

3.അനന്തരാവകാശം: മാതാപിതാക്കളുടെ സ്വത്തില്‍ പുത്രിമാര്‍ക്കും ഓഹരിയുണ്ടെന്നാണ് ഖുര്‍ആനിന്റെ അധ്യാപനം. മറ്റൊരു മതഗ്രന്ഥവും സ്ത്രീക്ക് അനന്തരസ്വത്തില്‍ അവകാശമുണ്ടെന്ന് പ്രഖ്യാപിക്കുന്നില്ലെന്നതാണ് വാസ്തവം. പരിഷ്‌കൃതമെന്നറിയപ്പെടുന്ന യൂറോപ്പില്‍ പോലും വനിതകള്‍ക്ക് അനന്തരസ്വത്തില്‍ അവകാശമുണ്ടെന്ന നിയമം കൊണ്ടുവന്നത് ഏതാനും പതിറ്റാണ്ടുകള്‍ക്കു മുമ്പാണ്. ഖുര്‍ആനാകട്ടെ,പതിനാലു നൂറ്റാണ്ടുകള്‍ക്കുമുമ്പുതന്നെ സ്ത്രീകള്‍ക്ക് അനന്തരസ്വത്തില്‍ അവകാശമുണ്ടെന്ന് പ്രഖ്യാപിക്കുകയും അത് നടപ്പാക്കുകയും ചെയ്തിട്ടുണ്ട്. ”മാതാപിതാക്കളും അടുത്ത ബന്ധുക്കളും വിട്ടേച്ചുപോയ സ്വത്തില്‍ പുരുഷന്മാര്‍ക്ക് ഓഹരിയുണ്ട്. മാതാപിതാക്കളും അടുത്ത ബന്ധുക്കളും വിട്ടേച്ചുപോയ സ്വത്തില്‍ സ്ത്രീകള്‍ക്കും ഓഹരിയുണ്ട്” (ഖുര്‍ആന്‍ 4:7).

4.ഇണയെ തീരുമാനിക്കുവാനുള്ള അവകാശം: വിവാഹാലോചനാവേളയില്‍ സ്ത്രീയുടെ ഇഷ്ടാനിഷ്ടങ്ങള്‍ പരിഗണിക്കപ്പെടണമെന്നാണ് ഇസ്‌ലാമിന്റെ ശാസന. ഒരു സ്ത്രീയെ അവള്‍ക്കിഷ്ടമില്ലാത്ത ഒരാള്‍ക്ക് വിവാഹം ചെയ്തുകൊടുക്കുവാന്‍ ആര്‍ക്കും അവകാശമില്ല; സ്വന്തം പിതാവിനുപോലും. മുഹമ്മദ് നബി(ﷺ) പറഞ്ഞു: ”വിധവയോട് അനുവാദം ചോദിക്കാതെ അവളെ വിവാഹം ചെയ്തുകൊടുക്കരുത്. കന്യകയോട് സമ്മതമാവശ്യപ്പെടാതെ അവളെയും കല്യാണം കഴിച്ചുകൊടുക്കാന്‍ പാടില്ല. മൗനമാണ് കന്യകയുടെ സമ്മതം” (ബുഖാരി, മുസ്‌ലിം).

5.പഠിക്കുവാനും ചിന്തിക്കുവാനുമുള്ള അവകാശം: സ്ത്രീകള്‍ക്ക് പഠിക്കുവാനും ചിന്തിക്കുവാനും അവകാശമുണ്ടെന്നാണ് ഇസ്‌ലാമിന്റെ കാഴ്ചപ്പാട്. ഇത് കേവലം ഉപദേശങ്ങളിലൊതുക്കുകയല്ല, പ്രായോഗികമായി കാണിച്ചുതരികയാണ് പ്രവാചകന്‍ (ﷺ)ചെയ്തത്. പ്രവാചകാനുചരകളായ വനിതകള്‍ വിജ്ഞാന സമ്പാദനത്തില്‍ പ്രകടിപ്പിച്ചിരുന്ന ശുഷ്‌കാന്തി സുവിദിതമാണ്. പ്രവാചകന്റെയും പത്‌നിമാരുടെയും അടുക്കല്‍ വിജ്ഞാന സമ്പാദനത്തിനായി വനിതകള്‍ സദാ എത്താറുണ്ടായിരുന്നുവെന്ന് ചരിത്രത്തില്‍ കാണാനാവും. അവരുമായി വിജ്ഞാന വിനിമയം നടത്താനായി പ്രവാചകന്‍ (ﷺ) ഒരു ദിവസം നീക്കിവെച്ചിരുന്നുവെന്ന് ഇമാം ബുഖാരി റിപ്പോര്‍ട്ട് ചെയ്ത ഹദീസില്‍ കാണാം.

6.വിമര്‍ശിക്കുവാനുള്ള അവകാശം: വിമര്‍ശിക്കുവാനും ചോദ്യം ചെയ്യുവാനുമുള്ള അവകാശം ഇസ്‌ലാം സ്ത്രീകള്‍ക്കു നല്‍കുന്നുണ്ട്. ജാഹിലിയ്യാ കാലത്ത് നിലനിന്നിരുന്ന‘ളിഹാർ‘ എന്ന സമ്പ്രദായത്തെക്കുറിച്ച് പ്രവാചകനോട് തര്‍ക്കിച്ച സ്വഹാബിവനിതയുടെ ചോദ്യങ്ങള്‍ക്കുള്ള ഉത്തരമായാണ് മുജാദിലഃ (തര്‍ക്കിക്കുന്നവള്‍) എന്ന അദ്ധ്യായത്തിലെ ആദ്യവചനങ്ങള്‍ അവതരിപ്പിക്കപ്പെട്ടത്. പ്രവാചകന്റെ മുമ്പില്‍ പോലും സ്ത്രീകള്‍ക്ക് അവകാശങ്ങള്‍ക്ക് വേണ്ടി സംവദിക്കാനുളള അവകാശം അനുവദിക്കപ്പെട്ടിരുന്നുവെന്നാണല്ലോ ഇത് വ്യക്തമാക്കുന്നത്. ഈ സൂക്തത്തിലെവിടെയും സ്വഹാബി വനിതയുടെ തര്‍ക്കത്തെ വിമര്‍ശിച്ചിട്ടില്ലെന്ന കാര്യം പ്രത്യേകം ശ്രദ്ധേയമാണ്.

7.സാമൂഹ്യപ്രവര്‍ത്തനങ്ങളില്‍ പങ്കെടുക്കുന്നതിനുള്ള അവകാശം: സാമൂഹ്യപ്രവർത്തനങ്ങളിൽ സംബന്ധിക്കുന്നത് സ്വാഭാവികമായും പുരുഷന്മാരാണെങ്കിലും സ്ത്രീകള്‍ക്കും അത്തരം കാര്യങ്ങളില്‍ പങ്കുവഹിക്കാന്‍ ഇസ്‌ലാം സ്വാതന്ത്ര്യം നല്‍കിയിട്ടുണ്ട്. വിശ്വാസ സ്വാതന്ത്ര്യത്തിനുവേണ്ടിയുള്ള സമരത്തില്‍ നേരിട്ട് സംബന്ധിക്കുന്നതിന് ഇസ്‌ലാം സ്ത്രീകളെ നിര്‍ബന്ധിക്കുന്നില്ല. എന്നാല്‍,യുദ്ധരംഗത്തും മറ്റും പട പൊരുതുന്നവര്‍ക്ക് സഹായികളായി വര്‍ത്തിക്കുവാന്‍ മുസ്‌ലിം വനിതകള്‍ രംഗത്തുണ്ടായിരുന്നു. പുരുഷന്മാരൊടൊപ്പം യുദ്ധത്തിന് പുറപ്പെടുകയും അവര്‍ക്ക് ഭക്ഷണം പാകം ചെയ്യുകയും പാനീയങ്ങള്‍ വിതരണം നടത്തുകയും മുറിവേറ്റവരെ ശുശ്രൂഷിക്കുകയും ചെയ്തിരുന്ന സ്വഹാബി വനിതകളെക്കുറിച്ച് ചരിത്രം നമുക്ക് വിവരിച്ചുതരുന്നുണ്ട്. സന്നിഗ്ധ ഘട്ടങ്ങളിൽ സമരമുഖത്തിറങ്ങാന്‍ വരെ സന്നദ്ധത കാണിച്ചിരുന്ന മഹിളാ രത്‌നങ്ങളുണ്ടായിട്ടുണ്ട്, ഇസ്‌ലാമിക ചരിത്രത്തില്‍. പ്രവാചക പത്‌നിയായിരുന്ന ആയിശ(റ)യായിരുന്നു ഖലീഫ ഉസ്മാന്റെ ഘാതകരെ ശിക്ഷിക്കാതെ അലി(റ)യെ ഖലീഫയായി തെരഞ്ഞെടുക്കരുതെന്ന അഭിപ്രായത്തില്‍നിന്ന് ഉരുണ്ടുകൂടിയ ജമല്‍ യുദ്ധത്തിന് നേതൃത്വം വഹിച്ചത്.

8.വിവാഹമൂല്യത്തിനുള്ള അവകാശം: വിവാഹം ചെയ്യപ്പെടുന്ന സ്ത്രീയുടെ അവകാശമാണ്‘മഹര്‍‘ ലഭിക്കുകയെന്നത്. തനിക്ക് ആവശ്യമുള്ള ‘മഹര്‍‘ തന്റെ കൈകാര്യകര്‍ത്താവ് മുഖേന ആവശ്യപ്പെടുവാന്‍ സ്ത്രീക്ക് അവകാശമുണ്ട്. ഈ വിവാഹമൂല്യം നല്‍കേണ്ടത് പുരുഷന്റെ ബാധ്യതയാണ്. നല്‍കപ്പെടുന്ന വിവാഹമൂല്യം സ്ത്രീയുടെ സമ്പത്തായാണ് പരിഗ ണിക്കപ്പെടുന്നത്. അവളുടെ സമ്മതമില്ലാതെ ആര്‍ക്കും അതില്‍നിന്നും ഒന്നും എടുക്കാനാവില്ല.”സ്ത്രീകള്‍ക്ക് അവരുടെ വിവാഹമൂല്യങ്ങള്‍ മനഃസംതൃപ്തിയോടുകൂടി നിങ്ങള്‍ നല്‍കുക” (4:4) എന്നാണ് ഖുര്‍ആനിന്റെ കല്‍പന.

9.വിവാഹമോചനത്തിനുള്ള അവകാശം: ഭര്‍ത്താവിനോടൊപ്പം ജീവിക്കുവാന്‍ സാധ്യമല്ലാത്ത അവസ്ഥയില്‍ വിവാഹമോചനം നേടാന്‍ സ്ത്രീക്ക് അവകാശമുണ്ട്.‘ഖുല്‍അ്‘, ‘ഫസ്ഖ്‘ എന്നീ രണ്ട് സാങ്കേതിക ശബ്ദങ്ങളിലാണ് സ്ത്രീകളുടെ വിവാഹമോചനം വ്യവഹരിക്കപ്പെടുന്നത്. വിവാഹ മൂല്യം തിരിച്ചുനല്‍കിക്കൊണ്ടുള്ള മോചനമാണ് ഒന്നാമത്തേത്. തിരിച്ചു നല്‍കാതെയുള്ളതാണ് രണ്ടാമത്തേത്. ഏതായിരുന്നാലും താനിഷ്ടപ്പെടാത്ത ഒരു ഭര്‍ത്താവിനോടൊപ്പം പൊറുക്കാന്‍ ഇസ്‌ലാം സ്ത്രീയെ നിര്‍ബന്ധിക്കുന്നില്ല. അവള്‍ക്ക് അനിവാര്യമായ സാഹചര്യത്തില്‍ വിവാഹമോചനം നേടാവുന്നതാണ്.ഇങ്ങനെ എല്ലാ നിലയ്ക്കുമുള്ള പെണ്ണവകാശങ്ങളെ പരിഗണിക്കുകയും അവയെന്തൊക്കെയാണെന്ന് കൃത്യമായി പഠിപ്പിക്കുകയും അവ നൽകേണ്ടത് ബാധ്യതയാണെന്ന് നിഷ്കര്ഷിക്കുകയും ചെയ്യുന്ന ദർശനമാണ് ഇസ്‌ലാം; സ്ത്രീകളുടെ ഒരു അവകാശത്തോടും അത് പുറം തിരിഞ്ഞു നിൽക്കുന്നില്ല.

ഇങ്ങനെ എല്ലാ നിലയ്ക്കുമുള്ള പെണ്ണവകാശങ്ങളെ പരിഗണിക്കുകയും അവയെന്തൊക്കെയാണെന്ന് കൃത്യമായി പഠിപ്പിക്കുകയും അവ നൽകേണ്ടത് ബാധ്യതയാണെന്ന് നിഷ്കര്ഷിക്കുകയും ചെയ്യുന്ന ദർശനമാണ് ഇസ്‌ലാം; സ്ത്രീകളുടെ ഒരു അവകാശത്തോടും അത് പുറം തിരിഞ്ഞു നിൽക്കുന്നില്ല.

വിഷയവുമായി ബന്ധപ്പെട്ട വീഡിയോ
നുഷ്യരെ ഒന്നിപ്പിക്കുകയാണ് വേദഗ്രന്ഥത്തിന്റെ പരമപ്രധാനമായ ധര്‍മമെന്നാണ് ഖുര്‍ആനിക വീക്ഷണം. വിശുദ്ധ ഖുര്‍ആന്‍ സൂചിപ്പിക്കുന്നതു കാണുക: ‘മനുഷ്യര്‍ ഒരൊറ്റ സമുദായമായിരുന്നു. അനന്തരം (അവര്‍ ഭിന്നിച്ചപ്പോള്‍ വിശ്വാസികള്‍ക്ക്) സന്തോഷവാര്‍ത്ത അറിയിക്കുവാനും, (നിഷേധികള്‍ക്ക്) താക്കീത് നല്‍കുന്നതിനും വേണ്ടി അല്ലാഹു പ്രവാചകന്മാരെ നിയോഗിച്ചു. അവര്‍ ഭിന്നിച്ച വിഷയത്തില്‍ ദൈവികമായ തീര്‍പ്പുകല്‍പിക്കുന്നതിനായി അവരുടെ കൂടെ സത്യവേദവും അവന്‍ അയച്ചു കൊടുത്തു‘ (2:213). മനുഷ്യര്‍ ഭിന്നിച്ച വിഷയത്തില്‍ ദൈവികമായ തീര്‍പ്പുകല്‍പിക്കുന്നതിനുവേണ്ടിയാണ് വേദഗ്രന്ഥങ്ങള്‍ അവതരിപ്പിക്കപ്പെട്ടതെന്നാണല്ലോ ഇതില്‍ നിന്ന് മനസ്സിലാവുന്നത്. മനുഷ്യരെ ഭിന്നതയില്‍നിന്ന് കരകയറ്റുവാന്‍ വേണ്ടിയാണ് ഖുര്‍ആനിന്റെയും അവതരണമെന്ന് അത് പ്രഖ്യാപിക്കുന്നുണ്ട്. ‘അവര്‍ ഏതൊരു കാര്യത്തില്‍ ഭിന്നിച്ചുപോയിരിക്കുന്നുവോ, അതവര്‍ക്ക് വ്യക്തമാക്കിക്കൊടുക്കാന്‍ വേണ്ടിയും വിശ്വസിക്കുന്ന ജനങ്ങള്‍ക്ക് മാര്‍ഗദര്‍ശനവും കാരുണ്യവും ആയിക്കൊണ്ടും മാത്രമാണ് ഞാന്‍ നിനക്ക് വേദഗ്രന്ഥം അവതരിപ്പിച്ചുതന്നത്‘ (16:64). വേദഗ്രന്ഥത്തിന്റെ ആളുകളെന്ന് സ്വയം അഭിമാനിച്ചിരുന്നവര്‍ ഭിന്നിച്ചതുപോലെ അഭിപ്രായഭിന്നതകള്‍ രൂപമെടുത്ത് ഛിന്നഭിന്നമാകാതിരിക്കാന്‍ അന്തിമവേദഗ്രന്ഥമായ ഖുര്‍ആനും അതിന്റെ പ്രായോഗിക ജീവിതമാതൃകയായ നബിചര്യയും മുറുകെ പിടിക്കുകയാണ് വേണ്ടതെന്ന് ഖുര്‍ആന്‍ വിശ്വാസികളെ ആഹ്വാനം ചെയ്യുന്നു. ‘നിങ്ങളൊന്നിച്ച് അല്ലാഹുവിന്റെ കയറില്‍ മുറുകെ പിടിക്കുക. നിങ്ങള്‍ ഭിന്നിച്ചുപോകരുത്‘ (3:103). ഇവിടെ അല്ലാഹുവിന്റെ കയറുകൊണ്ടുള്ള വിവക്ഷ വിശുദ്ധ ഖുര്‍ആനും നബിചര്യയുമാണെന്ന് വ്യാഖ്യാതാക്കള്‍ വ്യക്തമാക്കിയിട്ടുണ്ട്. ചുരുക്കത്തില്‍, വേദഗ്രന്ഥത്തിന്റെ പരമപ്രധാനമായ ധര്‍മം ജനങ്ങളെ സത്യത്തിലേക്ക് നയിച്ചുകൊണ്ട് അവര്‍ക്കിടയിലുള്ള ഭിന്നിപ്പും സ്പര്‍ധയും ഇല്ലാതെയാക്കുകയാകുന്നു.

‘വേദം‘ ഒരു സംസ്‌കൃത പദമാണ്. അറിവ്, വിദ്യ എന്നൊക്കെയാണ് ഈ പദത്തിനര്‍ഥം. വേദാന്തദര്‍ശന പ്രകാരം വേദം ശ്രുതിയാണ്. പടച്ചതമ്പുരാനില്‍ നിന്ന് ഋഷിമാര്‍ ശ്രവിച്ച വചനങ്ങളാണ് വേദത്തിന്റെ ഉള്ളടക്കമെന്നാണ് വിശ്വാസം. ‘പരമപുരുഷനില്‍നിന്നാണ് വേദം ഉല്‍പന്നമായത്‘ എന്നാണ് ഋഗ്വേദം (10:90:9) പറയുന്നത്. ഏതായിരുന്നാലും ദൈവികഗ്രന്ഥം എന്ന അര്‍ഥത്തിലാണ് ‘വേദം‘ എന്ന പദം ഇന്ത്യയില്‍ വ്യവഹരിക്കപ്പെട്ടു പോന്നിട്ടുള്ളത്. ഇന്ത്യയില്‍ പ്രചരിപ്പിക്കപ്പെട്ട സെമിറ്റിക് മതങ്ങളുടെ അനുയായികളും കാലാന്തരത്തില്‍ തങ്ങളുടെ മതഗ്രന്ഥങ്ങളെ വേദങ്ങള്‍ എന്നു വിശേഷിപ്പിക്കുകയാണുണ്ടായത്.

വേദഗ്രന്ഥം എന്ന അര്‍ഥത്തില്‍ ഖുര്‍ആന്‍ പ്രയോഗിക്കുന്നത് ‘അല്‍ കിതാബ്‘ എന്ന പദമാണ്. ഗ്രന്ഥം (the scripture) എന്നര്‍ഥം. പ്രവാചകന്മാര്‍ക്ക് പടച്ചതമ്പുരാന്‍ അവതരിപ്പിച്ച ദിവ്യവെളിപാടുകളാണ് വേദഗ്രന്ഥത്തിന്റെ ഉള്ളടക്കമെന്നാണ് ഖുര്‍ആനിക വീക്ഷണം. ദിവ്യ വെളിപാടുകള്‍ക്കാണ് ‘വഹ്‌യ്‘ എന്നു പറയുന്നത്. വേദഗ്രന്ഥത്തില്‍ ‘വഹ്‌യ്‘ മാത്രമേയുണ്ടാവൂ. എന്നാല്‍, ഒരു പ്രവാചകന് ലഭിക്കുന്ന എല്ലാ വഹ്‌യും വേദഗ്രന്ഥത്തില്‍ ഉള്‍ക്കൊള്ളിക്കണമെന്നില്ല. വേദഗ്രന്ഥത്തില്‍ ഉള്‍ക്കൊള്ളിക്കണമെന്ന നിര്‍ദേശത്തോടെ ലഭിക്കുന്ന ‘വഹ്‌യ്‘ ആണ് അതില്‍ ഉള്‍ക്കൊള്ളിക്കുന്നത്.

മൂസാ(അ)ക്ക് നല്‍കപ്പെട്ട വേദഗ്രന്ഥമാണ് തൗറാത്ത്. ഇതേപോലെ ദാവൂദി(അ)നും ഈസാ(അ)ക്കും നല്‍കപ്പെട്ട വേദഗ്രന്ഥങ്ങളാണ് സബൂര്‍,ഇന്‍ജീല്‍ എന്നിവ. പടച്ചതമ്പുരാന്‍ പ്രവാചകന്മാര്‍ക്ക് അവതരിപ്പിച്ച ഗ്രന്ഥങ്ങളായിട്ടാണ് വേദഗ്രന്ഥങ്ങളെ ഖുര്‍ആന്‍ പരിചയപ്പെടുത്തുന്നത്.‘തീര്‍ച്ചയായും നാം തന്നെയാണ് തൗറാത്ത് അവതരിപ്പിച്ചിരിക്കുന്നത്. അതില്‍ മാര്‍ഗദര്‍ശനവും പ്രകാശവുമുണ്ട് (5:44).

‘ദാവൂദിന് നാം ‘സബൂര്‍‘ നല്‍കുകയും ചെയ്തിരിക്കുന്നു‘ (17:55)

‘സന്മാര്‍ഗനിര്‍ദേശവും സത്യപ്രകാശവുമടങ്ങിയ ഇന്‍ജീലും നാം അദ്ദേഹത്തിന് (ഈസാക്ക്) നല്‍കി‘ (5:46).

ഇവയില്‍നിന്ന് സ്രഷ്ടാവ് പ്രവാചകന്മാര്‍ക്ക് അവതരിപ്പിച്ച ഗ്രന്ഥങ്ങളാണിവയെന്ന് സുതരാം വ്യക്തമാണ്.

എന്നാല്‍ ബൈബിള്‍ പുസ്തകങ്ങളുടെ സ്ഥിതി ഇതല്ല. പ്രവാചകന്മാര്‍ക്ക് ശേഷം പതിറ്റാണ്ടുകള്‍ കഴിഞ്ഞ് രചിക്കപ്പെട്ട ഗ്രന്ഥങ്ങളാണവ. ദൈവദൂതന്മാര്‍ക്ക് അവതരിപ്പിക്കപ്പെട്ട ഗ്രന്ഥങ്ങളാണെന്ന് ഖണ്ഡിതമായി പറയാവുന്ന ഒരു പുസ്തകവും ബൈബിളിലില്ല. പഞ്ചപുസ്തകങ്ങള്‍ (തോറാ) മോശെ രചിച്ചുവെന്നാണ് പരമ്പരാഗത യഹൂദ വിശ്വാസം;ദൈവം അവതരിപ്പിച്ച ഗ്രന്ഥമാണെന്നല്ല. മോശെ രചിച്ചതാണ് പഞ്ചപുസ്തകങ്ങളെന്ന പരമ്പരാഗത വിശ്വാസം അടിസ്ഥാന രഹിതമാണെന്നാണ് ആധുനിക ഗവേഷണങ്ങള്‍ വ്യക്തമാക്കുന്നത്. മോശെയുടെ മരണവും മരണാനന്തര സംഭവങ്ങളുമെല്ലാം പഞ്ചപുസ്തകത്തില്‍ വിവരിക്കപ്പെട്ടിട്ടുള്ളതിനാല്‍ (ആവര്‍ത്തനം 34:5-10)അതൊരിക്കലും മോശെ രചിച്ചതായിരിക്കാനിടയില്ലെന്നാണ് പണ്ഡിതാഭിപ്രായം.

സങ്കീര്‍ത്തനങ്ങളുടെ സ്ഥിതിയും തഥൈവ. ദാവീദ് രചിച്ച താണെന്ന് ഖണ്ഡിതമായി പറയാവുന്ന ഒരു സങ്കീര്‍ത്തനം പോലുമില്ലെന്നതാണ് വാസ്തവം.

സുവിശേഷങ്ങളില്‍ യേശു പ്രസംഗിച്ച ദൈവത്തിന്റെ സുവിശേഷത്തെക്കുറിച്ച സൂചനകളുണ്ടെങ്കിലും (മാര്‍ക്കോസ് 1:14,15) പ്രസ്തുത സുവിശേഷത്തെക്കുറിച്ച വ്യക്തമായൊരു ചിത്രം നാലു സുവിശേ ഷങ്ങളും നല്‍കുന്നില്ല. പുതിയ നിയമത്തിലുള്ള സുവിശേഷങ്ങളാകട്ടെ യേശുവിന് അഞ്ചു പതിറ്റാണ്ടുകളെങ്കിലും കഴിഞ്ഞ് രചിക്കപ്പെട്ടതാണ്. യേശുവിന്റെ ജീവിതത്തെയും സന്ദേശത്തെയും കുറിച്ച് വ്യത്യസ്തവും വിരുദ്ധവുമായ ചിത്രങ്ങളാണ് സുവിശേഷങ്ങള്‍ നല്‍കുന്നത്. ഇവയൊന്നുംതന്നെ യേശുവിന് അവതരിപ്പിക്കപ്പെട്ട വേദഗ്രന്ഥമല്ലെന്ന് വ്യക്തമാണല്ലോ.

ചുരുക്കത്തില്‍ തൗറാത്തിലെയും സബൂറിലെയും ഇന്‍ജീലിലെയും പല ആശയങ്ങളും ബൈബിളിലെ വ്യത്യസ്ത പുസ്തകങ്ങളില്‍ ഉദ്ധരിക്കപ്പെട്ടിട്ടുണ്ടെങ്കിലും അവയൊന്നും പൂര്‍ണമായി ബൈബിളില്‍ ഉണ്ടെന്ന് പറയാന്‍ കഴിയില്ല.

ഇംഗ്ലീഷില്‍ ഷെയ്ക്‌സ്പിയറുടെ നാടകങ്ങൾ, ജര്‍മന്‍ ഭാഷയില്‍ ഗോയ്‌ഥേയുടെയും ഷില്ലറുടെയും രചനകൾ, പേര്‍സ്യനില്‍ ഹാഫിളിന്റെയും റൂമിയുടെയും കവിതകൾ, സംസ്‌കൃതത്തില്‍ ഋഗ്വേദം. ഇങ്ങനെ ഓരോ ഭാഷയിലും ഉന്നതമായ സാഹിത്യ സൃഷ്ടികളുണ്ടായിട്ടുണ്ട്. ഇതേ പോലെയുള്ള ഒരു സാഹിത്യകൃതിയായി കണ്ടാൽ പോരെ ക്വുർആനിനെയും. ഖുർആനിന് മാത്രമെന്താണ് സവിശേഷത?

ഇംഗ്ലീഷില്‍ ഷെയ്ക്‌സ്പിയറെ വെല്ലുന്ന ഒരു നാടകകൃത്തില്ല. ജര്‍മന്‍ ഭാഷയിലാണെങ്കില്‍ ഗോയ്‌ഥേയും ഷില്ലറും അവരുടെ നാടകരചനയില്‍ അത്യുന്നതന്മാരാണ്. പേര്‍സ്യനില്‍ ഹാഫിളും റൂമിയും അദ്വിതീയരാണ്. സംസ്‌കൃതത്തില്‍ ഋഗ്വേദം അതുല്യമായ രചനയാണ്. ഓരോ ഭാഷയിലും ഉന്നതമായ സാഹിത്യ സൃഷ്ടികളുണ്ടായിട്ടുണ്ട്. ഇതുപോലെ അറബിയിലും മനോഹരമായ രചനകളുണ്ടായിട്ടുണ്ട്. ഈ രചനകളില്‍നിന്നെല്ലാം തികച്ചും വ്യത്യസ്തമാണ് ഖുര്‍ആനിന്റെ രൂപവും ശൈലിയും ഉള്ളടക്കവുമെല്ലാം. ഷേക്‌സ്പിയറുടെ നാടങ്ങളും ഗോയ്‌ഥേയുടെയും ഹോമറുടെയും കൃതികളുമെല്ലാം കഥനങ്ങളും ആസ്വാദനത്തിനു വേണ്ടിയുള്ളയതുമാണ്. അവ മാനുഷിക വികാരത്തെ മാത്രം സംതൃപ്തമാക്കാനുതകുന്നതാണ്.

ഖുര്‍ആനിക വചനങ്ങള്‍ ആസ്വാദനം നല്‍കുന്നതോടൊപ്പം പരിവര്‍ത്തനങ്ങളുണ്ടാക്കുകയും ചെയ്യുന്നു. സന്തോഷത്തോടൊപ്പം ശാന്തിയും നല്‍കുന്നു. കഥനങ്ങളോടൊപ്പം പാഠങ്ങളും പഠിപ്പിക്കുന്നു. മനുഷ്യരെ ഒന്നും പുറത്തുനിന്ന് അടിച്ചേല്‍പിക്കുകയല്ല ഖുര്‍ആന്‍ ചെയ്യുന്നത്. അവന് അകത്തുനിന്നുതന്നെ കര്‍മങ്ങള്‍ക്കുള്ള പ്രചോദനമുണ്ടാക്കുകയാണ്. ബുദ്ധിക്ക് സംതൃപ്തിയും വികാരങ്ങള്‍ക്ക് പൂര്‍ത്തീകരണവും നല്‍കിക്കൊണ്ട് ആളുകളെ പ്രവര്‍ത്തന നിരതമാക്കുകയാണ് അവ ചെയ്യുന്നത്. മദ്യം നിരോധിച്ചുകൊണ്ടുള്ള സൂക്ത(5:90,91)ങ്ങള്‍ ഉദാഹരണം. പ്രസ്തുത സൂക്തങ്ങളിലെ കല്‍പന സ്വയമേവ നിറവേറ്റുകയാണ് അത് കേട്ടവര്‍ ചെയ്തത്. മദീനാ തെരുവിലൂടെ മദ്യച്ചാലുകള്‍ ഒഴുകിയതിന് കാരണമതായിരുന്നു. മനുഷ്യവിരചിതമായ ഒരു സാഹിത്യ സൃഷ്ടിക്കും സാധിക്കാത്ത ഒരു കാര്യമാണിത്. ഒരാളുടെയല്ല, ഒരായിരം പേരുടെയുമല്ല; ലക്ഷങ്ങളുടെ ഹൃദയങ്ങള്‍ക്കകത്തേക്ക് തുളച്ചുകയറി ഒരേ രൂപത്തിലുള്ള കര്‍മങ്ങള്‍ ചെയ്യുന്നവരായി മാറ്റിയെടുക്കുകയെന്നത് മനുഷ്യകഴിവിന്നതീതമാണ്. മനുഷ്യമനസ്സിന്റെ സ്പന്ദതാളങ്ങളെയും ലയത്തെയും കുറിച്ച് വ്യക്തമായി അറിയാവുന്ന പടച്ചതമ്പുരാനു മാത്രമേ അത്തരമൊരു രചന സാധ്യമാകൂ.

ഏതു ഭാഷയിലെയും സാഹിത്യകൃതികളെടുത്ത് പരിശോധിക്കുക. അവയുടെ സാഹിത്യമൂല്യത്തെക്കുറിച്ച് നാം സംസാരിക്കുന്നത് അത് എഴുതപ്പെട്ട കാലത്തെ ഭാഷയുടെയും അറിവിന്റെയും ഭൂമികയില്‍നിന്നുകൊണ്ടാണ്. അവയിലൊന്നിന്റെയും ഭാഷകള്‍ ഇപ്പോള്‍ ജീവല്‍ ഭാഷകളേയല്ല. ഷേക്‌സ്പിയറുടെ ഇംഗ്ലീഷും ഋഗ്വേദത്തിന്റെ സംസ്‌കൃതവുമൊന്നും ഇന്ന് ജീവിച്ചിരിക്കുന്ന ഭാഷകളല്ല. ഈ ഭാഷകളെല്ലാം ഒട്ടനവധി പരിണാമ പ്രക്രിയകള്‍ക്ക് വിധേയമായി. ഖുര്‍ആനിന്റെ ഭാഷയും സൗന്ദര്യവും ഇവയില്‍നിന്ന് തികച്ചും വ്യത്യസ്തമാണ്. ഖുര്‍ആന്‍ അവതരിപ്പിക്കപ്പെട്ട് പതിനാല് നൂറ്റാണ്ട് കഴിഞ്ഞിട്ടും ഖുര്‍ആനിക അറബിതന്നെയാണ് ഇന്നും അറബികള്‍ക്ക് ആധാരഭാഷ (standard language)യായി നിലനില്‍ക്കുന്നത്. ദൈവിക നിയമങ്ങളെപോലെത്തന്നെ ദൈവിക ഗ്രന്ഥത്തിന്റെ ഭാഷക്കും ഗണ്യമായ മൗലികമാറ്റങ്ങളൊന്നും കൂടാതെ പതിനാലു നൂറ്റാണ്ടുകാലം നിലനില്‍ക്കുവാന്‍ കഴിഞ്ഞുവെന്നതുതന്നെ ഒരു ദൈവിക ദൃഷ്ടാന്തമാണ്. ഭാഷാ പരിണാമത്തെയും കഴിഞ്ഞ പതിനാലു നൂറ്റാണ്ടുകള്‍ക്കിടയില്‍ മറ്റു ഭാഷകള്‍ക്കുണ്ടായ മാറ്റങ്ങളെയും കുറിച്ച് പഠിച്ചവര്‍ക്കേ ഖുര്‍ആനിന്റെ മാത്രമായ ഈ സവിശേഷത വ്യക്തമായി മനസ്സിലാവൂ.

സത്യത്തില്‍, മറ്റു സാഹിത്യ കൃതികള്‍ ഖുര്‍ആനുമായി താരതമ്യം ചെയ്യാനേ അര്‍ഹമല്ലാത്തവയാണ്. അവയെല്ലാം ഓരോ പ്രത്യേക സാഹചര്യങ്ങളുടെ സൃഷ്ടി; ആ സാഹചര്യങ്ങളുമായി പൊരുത്തപ്പെട്ട സംഭവങ്ങളുമായി മാത്രം ബന്ധപ്പെട്ടവ; ജനങ്ങളെ ആസ്വദിപ്പിക്കുകയെന്ന ഉദ്ദേശ്യത്തോടുകൂടി മാത്രം രചിക്കപ്പെട്ടവ. ഖുര്‍ആനാകട്ടെ ജനങ്ങളെ അഭ്യസിപ്പിക്കുവാനുള്ളതാണ്. അത്തരമൊരു ഗ്രന്ഥം ആസ്വാദനം നല്‍കുകയെന്നത് വളരെ വിരളമാണ്. ഖുര്‍ആനിക സൂക്തങ്ങള്‍ ഒരേസമയംതന്നെ അത് അവതരിപ്പിക്കപ്പെട്ടകാലത്തെ സാഹചര്യങ്ങളോടും മറ്റു കാലങ്ങളിലെ തത്തുല്യമായ സാഹചര്യങ്ങളോടും പൊരുത്തപ്പെടുന്നവയാണ്. ബാഹ്യമായി ആസ്വദിപ്പിക്കുക ഖുര്‍ആനിന്റെ ലക്ഷ്യമേയല്ല. എന്നാല്‍ ഖുര്‍ആനിക വചനങ്ങള്‍ മനസ്സിന് സംതൃപ്തിയും കുളിര്‍മയും നല്‍കുകയും അതിന്റെ മനോഹാരിതയില്‍ മനസ്സ് പകച്ചുനിന്നുപോവുകയും ചെയ്യുന്നു.

മറ്റു ഗ്രന്ഥങ്ങളില്‍നിന്നെല്ലാം ഖുര്‍ആനിനെ വ്യതിരിക്തമാക്കുന്ന അതിന്റെ സുപ്രധാനമായ പ്രത്യേകത അത് മുന്നോട്ട് വെക്കുന്ന വെല്ലുവിളിയാണ്. മറ്റുകൃതികളുടെയൊന്നും രചയിതാക്കള്‍ക്ക് തങ്ങളുടെ ഗ്രന്ഥത്തിനു തുല്യമായ ഒരു ഗ്രന്ഥം കൊണ്ടുവരാനായി വെല്ലുവിളിക്കുവാനുള്ള ധൈര്യമുണ്ടായിട്ടില്ല; ധൈര്യമുണ്ടാവുകയുമില്ല. മറ്റൊരാളുടെ കഴിവ് എത്രമാത്രമുണ്ടെന്ന് മനസ്സിലാക്കാന്‍ ആര്‍ക്കാണ് സാധിക്കുക? അതിന് ഒരാള്‍ക്കും സാധിക്കുയില്ലെന്നതുകൊണ്ടുതന്നെ അത്തരമൊരു വെല്ലുവിളി നടത്താന്‍ സര്‍വശക്തനായ സ്രഷ്ടാവിനല്ലാതെ ഒരാള്‍ക്കും കഴിയുകയില്ല. ലോകോത്തര സാഹിത്യകൃതികളൊന്നുംതന്നെ അത്തരമൊരു വെല്ലുവിളി നടത്തുന്നുമില്ല.

ചുരുക്കത്തില്‍, ഖുര്‍ആനുമായി താരതമ്യത്തിനുപോലും മറ്റു സാഹിത്യഗ്രന്ഥങ്ങളൊന്നും അര്‍ഹമല്ലെന്നതാണ് വാസ്തവം.

ക്വുർആൻ ഉന്നതമായ ഒരു സാഹിത്യകൃതിയാണെന്നും അതിനാൽ അത് ദൈവികമാണെന്നും മുസ്ലിംകൾ വാദിക്കാറുണ്ട്. നല്ല സാഹിത്യകൃതിയാണ് എന്നതുകൊണ്ടുമാത്രം ഒരുഗ്രൻഥം ദൈവികമാണെന്ന് പറയാൻ കഴിയുമോ ?

ന്നതമായ സാഹിത്യകൃതിയാണ് എന്നതുകൊണ്ടുമാത്രം ഒരു ഗ്രന്ഥവും ദൈവികമാണെന്ന് പറയുക സാധ്യമല്ല; വടി നിലത്തിട്ട് സര്‍പ്പമാക്കി കാണിക്കുന്നവരെയെല്ലാം ദൈവ പ്രവാചകന്മാരായി അംഗീകരിക്കാന്‍ പറ്റാത്തതുപോലെ. ദൈവിക ദൃഷ്ടാന്തവും മാനുഷിക വിദ്യകളും തമ്മില്‍ അടിസ്ഥാനപരമായ ഒരു അന്തരമുണ്ട്. ദൃഷ്ടാന്തങ്ങള്‍ മനുഷ്യരുടെ കഴിവുകളെ മുഴുവന്‍ വെല്ലുവിളിക്കുന്നതായിരിക്കുമെന്നതാണത്. അതിനു മുകളില്‍ നില്‍ക്കുവാന്‍ മാനുഷിക വിദ്യകള്‍ക്കൊന്നിനും കഴിയില്ല. അവ എത്രസാര്‍ഥമാണെന്നിരിക്കിലും. മോശെയുടെ സര്‍പ്പം മാന്ത്രികന്മാരുടെ സര്‍പ്പങ്ങളെ മുഴുവന്‍ വിഴുങ്ങിയതുപോലെ ദൈവിക ദൃഷ്ടാന്തങ്ങള്‍ മാനുഷിക വിദ്യകളെ മുഴുവന്‍ വെല്ലുവിളിച്ചുകൊണ്ട് നിലനില്‍ക്കും; തീര്‍ച്ച.

ഖുര്‍ആന്‍ ഉന്നതമായ സാഹിത്യനിലവാരം പുലര്‍ത്തുകയും മനുഷ്യമനസ്സുകളെ സ്വാധീനിക്കുകയും ചെയ്യുന്നു. അതോടൊപ്പം അത് മാനവരാശിയോട് ഒരു അത്യുജ്വലമായ വെല്ലുവിളി നടത്തുകയും ചെയ്യുന്നു. അതിനു സമാന്തരമായി ഒരു രചന നിര്‍വഹിക്കുവാനാണ് പ്രസ്തുത വെല്ലുവിളി. ഈ വെല്ലുവിളിക്കുമുമ്പില്‍ മറ്റു സാഹിത്യ കൃതികളെല്ലാം മോശെയുടെ സര്‍പ്പത്തിനു മുന്നിലെ മാന്ത്രികപ്പാമ്പുകളെപ്പോലെ നിസ്സഹായരായി നില്‍ക്കുകയാണ്.

ഖുര്‍ആന്‍ ആദ്യം വെല്ലുവിളിച്ചത് അതുപോലൊരു ഗ്രന്ഥം കൊണ്ടുവരുവാനാണ്. ഖുര്‍ആന്‍ പറഞ്ഞു: ”പറയുക: ഈ ഖുര്‍ആന്‍ പോലൊന്ന് കൊണ്ടുവരുന്നതിനായി മനുഷ്യരും ജിന്നുകളും ഒന്നിച്ചു ചേര്‍ന്നാലും തീര്‍ച്ചയായും അതുപോലൊന്ന് അവര്‍ കൊണ്ടുവരികയില്ല. അവരില്‍ ചിലര്‍ ചിലര്‍ക്ക് പിന്തുണ നല്‍കുന്നവരായാല്‍ പോലും”(17:88).

ഖുര്‍ആനിന് തുല്യമായ ഒരു ഗ്രന്ഥം കൊണ്ടുവരുന്നതിനുവേണ്ടിയുള്ള വെല്ലുവിളിക്കു മുമ്പില്‍ അറബി സാഹിത്യകാരന്മാരെല്ലാം മുട്ടുമടക്കി. എങ്കിലും ഖുര്‍ആന്‍ കെട്ടിച്ചമച്ചതാണെന്നും മാരണമാണെന്നും വാദിക്കുന്നവരോട് അത് വീണ്ടും വെല്ലുവിളിച്ചു: ”അതല്ല, അദ്ദേഹം അത് കെട്ടിച്ചമച്ചുവെന്നാണോ അവര്‍ പറയുന്നത്? എന്നാല്‍ ഇതുപോലുള്ള പത്ത് അധ്യായങ്ങള്‍ ചമച്ചുണ്ടാക്കിയത് നിങ്ങള്‍ കൊണ്ടുവരൂ. അല്ലാഹുവിന് പുറമെ നിങ്ങളെ സഹായിക്കുന്നവരെയെല്ലാം നിങ്ങള്‍ വിളിച്ചുകൊള്ളുകയും ചെയ്യുക. നിങ്ങള്‍ സത്യവാന്മാരാണെങ്കില്‍”(10:13).

ഖുര്‍ആനിലെ പത്ത് അധ്യായങ്ങള്‍ക്ക് തുല്യമായ അധ്യായങ്ങളെങ്കിലും രചിച്ചുകൊണ്ട് അത് മനുഷ്യനിര്‍മിതമാണെന്ന വാദം സ്ഥാപിക്കുവാനുള്ള ഖുര്‍ആനിന്റെ വെല്ലുവിളിക്ക് ഉത്തരം നല്‍കാന്‍ സമകാലികരായ മനുഷ്യര്‍ക്കൊന്നും കഴിഞ്ഞില്ല. എന്നാല്‍,അവിശ്വാസികള്‍ ഖുര്‍ആന്‍ വചനങ്ങള്‍ മുഹമ്മദി(ﷺ)ന്റെ രചനയാണെന്ന പ്രചാരണം നിര്‍ത്തിയതുമില്ല. അപ്പോള്‍ ഖുര്‍ആന്‍ വീണ്ടും പറഞ്ഞു: ”അതല്ല, അദ്ദേഹം അതുകെട്ടിച്ചമച്ചുവെന്നാണോ നിങ്ങള്‍ പറയുന്നത്? പറയുക: എന്നാല്‍, അതിനു തുല്യമായ ഒരു അധ്യായം നിങ്ങള്‍ കൊണ്ടുവരൂ. അല്ലാഹുവിന് പുറമെ നിങ്ങള്‍ക്ക് സാധിക്കുന്നവരെയെല്ലാം വിളിച്ചുകൊള്ളുകയും ചെയ്യുക; നിങ്ങള്‍ സത്യവാന്മാരാണെങ്കില്‍”(10:38).

ഈ വെല്ലുവിളികള്‍ക്കൊന്നിനും മറുപടി നല്‍കുവാന്‍ അന്നു ജീവിച്ചിരുന്ന സാഹിത്യകാരന്മാര്‍ക്കൊന്നും കഴിഞ്ഞില്ല. അവരില്‍ പലരും അതിനു ശ്രമിച്ചുവെങ്കിലും പരാജയപ്പെട്ടു പിന്‍വാങ്ങേണ്ടിവന്നു. ഖുര്‍ആന്‍ അവസാന നാളുവരെയുള്ള മുഴുവന്‍ മനുഷ്യര്‍ക്കുമുള്ള ദൃഷ്ടാന്തമാണല്ലോ. അതുകൊണ്ടുതന്നെ മുഴുവന്‍ മാനവസമൂഹത്തോടുമായി ഈ വെല്ലുവിളി അത് ഒരിക്കല്‍കൂടി ആവര്‍ത്തിച്ചു: ”നമ്മുടെ ദാസന് നാം അവതരിപ്പിച്ചു കൊടുത്തതിനെപ്പറ്റി നിങ്ങള്‍ സംശയാലുക്കളാണെങ്കില്‍ അതിന്‍േറതുപോലുള്ള ഒരു അധ്യായമെങ്കിലും നിങ്ങള്‍ കൊണ്ടുവരിക. അല്ലാഹുവിനു പുറമെ നിങ്ങള്‍ക്കുള്ള സഹായികളെയും വിളിച്ചുകൊള്ളുക; നിങ്ങള്‍ സത്യവാന്മാരാണെങ്കില്‍. നിങ്ങള്‍ക്കത് ചെയ്യാനായില്ലെങ്കില്‍- നിങ്ങള്‍ക്കത് ഒരിക്കലും ചെയ്യാന്‍ കഴിയുകയില്ല -മനുഷ്യരും കല്ലുകളും ഇന്ധനമായി കത്തിക്കപ്പെടുന്ന നരകാഗ്‌നിയെ നിങ്ങള്‍ കാത്തു സൂക്ഷിച്ചുകൊള്ളുക. സത്യനിഷേധികള്‍ക്കു വേണ്ടി ഒരുക്കപ്പെട്ടതാകുന്നു അത്” (2:23,24).

ദൈവമൊഴിച്ചുള്ള മുഴുവന്‍ പേരും ഒരുമിച്ചു കൂടിയാല്‍ പോലും ഖുര്‍ആനിലെ ഏറ്റവും ചെറിയ അധ്യായതിനു തുല്യമായ ഒരു രചനപോലും കൊണ്ടുവരാന്‍ കഴിയില്ലെന്നതാണ് വെല്ലുവിളി. ഈ വെല്ലുവിളിക്ക് ഉത്തരം നല്‍കാന്‍ അറേബ്യന്‍ സാഹിത്യത്തറവാട്ടിലെ കാരണവന്മാര്‍ക്ക് കഴിഞ്ഞില്ല. ഇന്നും ആ വെല്ലുവിളി ലോകത്തിനു മുന്നില്‍ സ്പഷ്ടമായി നിലനില്‍ക്കുന്നു. മാനവരാശിയുടെ കര്‍ണപുടങ്ങളില്‍ ഖുര്‍ആനിന്റെ വെല്ലുവിളി അലച്ചുകൊണ്ടിരിക്കുന്നു. മറ്റു വേദഗ്രന്ഥങ്ങളുടെ ഭാഷകളെപ്പോലെ ഖുര്‍ആനിന്റെ ഭാഷ ഒരു നിര്‍ജീവ ഭാഷയല്ല. അത് സജീവമായൊരു സംസാരഭാഷയാണ്. അറബി സംസാരിക്കുന്നവരായ കുറേ അമുസ്‌ലിംകളുണ്ട്. ഇസ്‌ലാമിന്റെ കഠിന വിരോധികളായ കുറെ അറബി സാഹിത്യകാരന്മാരുമുണ്ട്. അവര്‍ക്കൊന്നുംതന്നെ ഖുര്‍ആനിന്റെ ഈ വെല്ലുവിളിക്കു മറുപടി നല്‍കാന്‍ കഴിഞ്ഞിട്ടില്ല; ഇനിയൊട്ട് കഴിയുകയുമില്ല.

ഖുര്‍ആന്‍ കേവലമായ ഒരു മാനുഷിക രചനയായിരുന്നെങ്കില്‍ ഇത്തരമൊരു വെല്ലുവിളി നടത്താന്‍ അതിന് സാധിക്കുമായിരുന്നില്ല. മനുഷ്യര്‍ മുഴുവന്‍ ഒന്നിച്ചു ചേര്‍ന്നാല്‍ പോലും തന്റെ രചനയിലെ ഒരു അധ്യായത്തിനു തുല്യമായ ഒരെണ്ണം കൊണ്ടുവരാന്‍ കഴിയില്ലെന്ന് പറയാന്‍ ഒരു മനുഷ്യന് ധൈര്യം വരുന്നതെങ്ങനെ? ഖുര്‍ആനിന്റെ അമാനുഷികത പ്രകടമാക്കപ്പെടുന്നത് ഈ വെല്ലുവിളിയിലാണ്. ഈ വെല്ലുവിളിയില്ലായിരുന്നുവെങ്കില്‍, ഖുര്‍ആനിക സാഹിത്യത്തിന് മാത്രമായി ദൈവികതയുണ്ടെന്ന് പറയാന്‍ കഴിയുകയില്ലായിരുന്നുവെന്നര്‍ഥം; മറ്റേത് ഉന്നതമായ സാഹിത്യ കൃതിയെയും പോലെ.

 സമൂഹത്തിൽ മാറ്റമുണ്ടാക്കുന്ന രീതിയില്‍ വ്യക്തിയെ പരിവര്‍ത്തിപ്പിക്കുവാനും അടിസ്ഥാനപരമായ മാറ്റത്തിന് നിമിത്തമാകുവാനും കഴിയുന്നതാകണം സാഹിത്യമെന്നു പറയാറുണ്ട്. ഈ വീക്ഷണത്തിന്റെ അടിസ്ഥാനത്തിൽ ഖുർആനിനെ എങ്ങനെ വിലയിരുത്താനാകും? 

സാമൂഹ്യമാറ്റത്തിന് നിമിത്തമാകുന്ന രീതിയില്‍ വ്യക്തിയെ പരിവര്‍ത്തിപ്പിക്കുവാന്‍ കഴിയുന്നതാകണം സാഹിത്യമെന്ന വീക്ഷണത്തിന്റെ അളവുകോല്‍ ഉപയോഗിച്ച് പരിശോധിച്ചാല്‍ ഖുര്‍ആന്‍ ഒരു കുറ്റമറ്റ സാഹിത്യകൃതിയാണെന്ന് പറയാന്‍ കഴിയും. ശ്രോതാവിന്റെ ബുദ്ധിക്ക് തൃപ്തിയും മനസ്സിന് സമാധാനവും നല്‍കുന്നതോടൊപ്പം അവന്റെ ഹൃദയത്തില്‍ മാറ്റങ്ങള്‍ സൃഷ്ടിക്കുകയും കൂടി ചെയ്യുന്നവയാണ് ഖുര്‍ആന്‍ സൂക്തങ്ങള്‍. മനുഷ്യമനസ്സുകളെ സ്വാധീനിക്കുവാനും അവയില്‍ പരിവര്‍ത്തനത്തിന്റ ആന്ദോളനങ്ങള്‍ സൃഷ്ടിക്കുവാനുമുള്ള ഖുര്‍ആനിന്റെ കഴിവ് അതിനെ അതുല്യമാക്കുന്ന പല സവിശേഷതകളിലൊന്നാണ്.

മുഹമ്മദ് നബി(ﷺ) ഒരിക്കല്‍ കഅ്ബയുടെ സമീപം വെച്ച് ഖുര്‍ആന്‍ പാരായണം ചെയ്യുകയാണ്. ശ്രോതാക്കളില്‍ മുസ്‌ലിംകളും അമുസ്‌ലിംകളുമെല്ലാം ഉണ്ട്. സൂറത്തുന്നജ്മിലെ സാഷ്ടാംഗത്തിന്റെ സൂക്തം ഓതിക്കൊണ്ടിരിക്കെ അല്ലാഹുവിന്റെ കല്‍പന പ്രകാരം നബി(ﷺ) സാഷ്ടാംഗം ചെയ്തു. അവിടെ കൂടിയിരുന്ന മുഴുവന്‍ ആളുകളും, മുസ്‌ലിം – അമുസ്‌ലിം വ്യത്യാസമില്ലാതെ നബിയോടൊപ്പം സാഷ്ടാംഗം ചെയ്തുപോയി. ഉമയ്യത്തുബ്‌നു ഖലഫ് എന്ന അഹങ്കാരിയൊഴികെ. (അബൂദാവൂദ്, തിര്‍മിദി). ഖുര്‍ആനിന്റെ സ്വാധീനശക്തി! അതിന്റെ കഠിന വിരോധികള്‍ പോലും അതിന്റെ ആജ്ഞയനുസരിച്ച് സാഷ്ടാംഗം ചെയ്യുന്ന അവസ്ഥ!!

ലബീദുബ്‌നു റബീഅഃ നബി(ﷺ)യുടെ കാലത്തെ അറേബ്യയിലെ അതിപ്രഗത്ഭനായ സാഹിത്യകാരനായിരുന്നു. അദ്ദേഹത്തിന്റെ അതിസുന്ദരമായ ഒരു കവിത കഅ്ബയുടെ വാതിലിന്മേല്‍ പറ്റിച്ചുവെച്ചിരുന്നു. അങ്ങനെ വെക്കുന്നത് ഒരു വെല്ലുവിളിയാണ്. പ്രസ്തുത കവിതയെ വെല്ലുവാന്‍ ആര്‍ക്കെങ്കിലും കഴിയുമോയെന്നാണ് വെല്ലുവിളി. അവിടെയുണ്ടായിരുന്ന ഒരു കവിക്കും അതിനടുത്ത് മറ്റൊരു കവിതയൊട്ടിച്ച് വെക്കാനുള്ള ധൈര്യം വന്നില്ല. അത്രക്ക് മനോഹരമായിരുന്നു ആ കവിത. എന്നാല്‍ അതിനടുത്തു തന്നെ ഏതാനും ഖുര്‍ആന്‍ സൂക്തങ്ങള്‍ എഴുതിത്തൂക്കുവാന്‍ പ്രവാചകാനുചരന്മാര്‍ തയാറായി. തന്റെ വെല്ലുവിളിക്ക് ഉത്തരം നല്‍കിയവനെ പരിഹസിക്കുവാനുള്ള വെമ്പലോടെ ലബീദ് ഖുര്‍ആന്‍ വചനങ്ങള്‍ വായിച്ചു. ഏതാനും വചനങ്ങള്‍ വായിച്ചതേയുള്ളൂ;അദ്ദേഹം ഖുര്‍ആനിന്റെ വശ്യതയില്‍ ആകൃഷ്ടനായി ഇസ്‌ലാം സ്വീകരിച്ചു. പുച്ഛത്തോടെ നോക്കുന്നവന്റെ മനസ്സില്‍പോലും മാറ്റം സൃഷ്ടിക്കുവാനുള്ള ഖുര്‍ആനിന്റെ കഴിവാണ് ഇവിടെ പ്രകടമായത്.

ഉമറുബ്‌നുല്‍ ഖത്താബിന്റെ ഇസ്‌ലാം ആശ്ലേഷം ചരിത്ര ്രപസിദ്ധമാണ്. മുഹമ്മദ് നബി(ﷺ)യുടെ തലയെടുക്കുവാനായി ഊരിയ വാളും കൊണ്ട് പുറപ്പെട്ട ഉമറി(റ)ന്റെ മനസ്സുമാറ്റിയത് സഹോദരിയില്‍നിന്നും ലഭിച്ച ഫലകത്തിലെ ഖുര്‍ആന്‍ വചനങ്ങളുടെ വശ്യതയും ആശയ ഗാംഭീര്യവുമായിരുന്നു.

ജുബൈർ ഇബ്ൻ മുത്വ്ഇം എന്ന ബഹുദൈവ വിശ്വാസി ഒരിക്കല്‍ വഴിയിലൂടെ നടന്നുപോവുകയാണ്. മുഹമ്മദ് നബി(ﷺ) മഗ്‌രിബ് നമസ്‌കാരത്തില്‍ സൂറത്തു ത്വൂര്‍ ഓതിക്കൊണ്ടിരിക്കുന്നത് അദ്ദേഹം കേട്ടു. അതിലെ ഓരോ പദവും അദ്ദേഹത്തിന്റെ ഹൃദയത്തില്‍ പതിഞ്ഞു. അതിന്റെ മനോഹാരിത അദ്ദേഹത്തെ ആകര്‍ഷിച്ചു. അതിന്റെ സ്വാധീനത്തില്‍ അദ്ദേഹം അതിശയിച്ചു. അവിടെ വെച്ചുതന്നെ അദ്ദേഹം ഇസ്‌ലാം സ്വീകരിച്ചു. ഇങ്ങനെ എത്രയെത്ര സംഭവങ്ങള്‍!

ഖുര്‍ആനിന്റെ മനോഹരവും വശ്യവുമായ ശൈലിയെപ്പറ്റി മക്കാ മുശ്‌രിക്കുകള്‍ ബോധവാന്മാരായിരുന്നു. പ്രസ്തുത മനോഹാരിതയാണ് പാരമ്പര്യമതത്തില്‍നിന്ന് ജനങ്ങള്‍ കൊഴിഞ്ഞുപോകാന്‍ ഇടയാക്കുന്നത് എന്ന് അവര്‍ക്ക് അറിയാമായിരുന്നു. നാടുവിടാനൊരുങ്ങിയ അബൂബക്കറി(റ)നെ തിരിച്ചുകൊണ്ടുവന്ന ഇബ്‌നുദുഗ്‌നയോട് മക്കാനിവാസികള്‍ പറഞ്ഞത് ഇപ്രകാരമായിരുന്നു: ‘അബൂബക്കര്‍ ഖുര്‍ആന്‍ ഉറക്കെ പാരായണം ചെയ്യുകയും ഞങ്ങളുടെ സ്ത്രീകളും കുട്ടികളും അത് കേള്‍ക്കുവാന്‍ ഇടവരികയും ചെയ്യരുത്. എങ്കില്‍ മാത്രമേ ഇവിടെ താമസിക്കുവാന്‍ അബൂബക്കറിനെ ഞങ്ങള്‍ അനുവദിക്കുകയുള്ളൂ.

ഖുര്‍ആനിന്റെ ഈ സ്വാധീനശക്തിയാണല്ലോ ജനങ്ങളെ ഇസ്‌ലാമിലേക്ക് ആകര്‍ഷിച്ചത്. കേവലം 23 വര്‍ഷക്കാലം കൊണ്ട് അന്ധകാരത്തിന്റെ അഗാധ ഗര്‍ത്തങ്ങളില്‍ ജീവിച്ചിരുന്ന ഒരു സമൂഹത്തെ ലോകത്തിന് മുഴുവന്‍ മാതൃകായോഗ്യരായ സമുദായമാക്കിയതിനു പിന്നില്‍ പ്രവര്‍ത്തിച്ചത് ഖുര്‍ആനിന്റെ ഈ ദൈവികതയായിരുന്നുവെന്നതാണ് സത്യം. ആര്‍.വി.സി ബോഡ്‌ലി എഴുതിയത് അതാണല്ലോ.

This book transformed the simple shepherds, the merchants and nomads of Arabia into warriors and empire builders (R.V.C Bodley: The Messenger, The Life of Mohammed- Newyork (1943:page239)

‘അറേബ്യയിലെ ആട്ടിടയന്മാരും കച്ചവടക്കാരും അലഞ്ഞുനടക്കുന്നവരുമായിരുന്ന സാധാരണക്കാരെ പടയാളികളും സാമ്രാജ്യ സ്ഥാപകരുമാക്കിത്തീര്‍ത്തത് ഈ ഗ്രന്ഥമാണ്‘.

ഖുര്‍ആനിന്റെ സ്വാധീനശക്തിയെക്കുറിച്ച് മോര്‍ഗന്‍ എഴുതി:

The Qur’an succeeded so well in captiving the mind of the audience that several of the oppenents thought it the effect of witch craft and enchantment (K.W. Morgan: Islam interpreted by Muslims, London (1958 page:27)

‘ശ്രോതാവിന്റെ മനസ്സിനെ സ്വാധീനിക്കുന്നതിലുള്ള ഖുര്‍ആനിന്റെ അത്യപാരമായ ശേഷിയാല്‍ അത് മാരണമാണെന്നും ആഭിചാരമാണെന്നുമാണ് അതിന്റെ എതിരാളികള്‍ കരുതിയത്‘.

ഖുര്‍ആന്‍ ദൈവവചനമാണ്. അതില്‍ യാതൊരുവിധ വൈരുധ്യവുമില്ല.മനുഷ്യനിര്‍മ്മിതമായ ഒരു വചനമെങ്കിലും ഖുര്‍ആനില്‍ഉണ്ടായിരുന്നുവെങ്കില്‍ അത് ഖുര്‍ആനിന്റെ മറ്റു ഭാഗങ്ങളുമായി സാരമായവൈരുധ്യങ്ങള്‍ ഉള്ളതാകുമായിരുന്നു. എന്നാല്‍ മനുഷ്യരുടെകൈകടത്തലുകളില്‍ നിന്ന് ദൈവം തമ്പുരാന്‍ തന്നെ തന്റെ അന്തിമവേദഗ്രന്ഥത്തെ സംരക്ഷിച്ചിട്ടുണ്ട്; ഇനിയും അന്ത്യനാളുവരെ അത്സംരക്ഷിക്കപ്പെടുകയും ചെയ്യും. ഇത് അല്ലാഹുവിന്റെ വാഗ്ദാനമാണ്.

തീര്‍ച്ചയായും നാമാണ് ആ ഉദ്‌ബോധനം അവതരിപ്പിച്ചത്. നിശ്ചയം നാംഅതിനെ കാത്തുസൂക്ഷിക്കുന്നതുമാണ്. (വി.ഖു.15:9)

മനുഷ്യരുടെ കൈകടത്തലുകളുണ്ടായപ്പോഴാണ് പൂര്‍വ്വവേദങ്ങള്‍വികലമാക്കപ്പെട്ടത്; പ്രസ്തുത വൈകല്യത്തിന്റെ അനിവാര്യതയായിരുന്നുഅവയിലെ വൈരുധ്യങ്ങള്‍. വ്യത്യസ്ത വ്യക്തികള്‍ ഒരേ കാര്യത്തെ കുറിച്ചുതന്നെ പ്രതിപാദിച്ചാലും അവയില്‍ വൈരുധ്യങ്ങളുണ്ടാവുകസ്വാഭാവികമാണ്. ബൈബിളിലും മറ്റു വേദഗ്രന്ഥങ്ങളിലുമെല്ലാംകാണപ്പെടുന്ന വൈരുധ്യങ്ങള്‍ ഇത്തരത്തിലുള്ളവയാണ്. വൈരുധ്യങ്ങളാല്‍നിബിഡമായ വേദഗ്രന്ഥങ്ങളുടെ സ്വന്തക്കാര്‍ എന്ന് അവകാശപ്പെടുന്നവര്‍പ്രസ്തുത ഗ്രന്ഥങ്ങളിലെ വൈരുധ്യങ്ങള്‍ വിശദീകരിക്കുവാന്‍പ്രയാസപ്പെടുകയാണ് ചെയ്യുന്നത്. പ്രസ്തുത വൈരുധ്യങ്ങള്‍മറച്ചുവെക്കാനും അതില്‍ നിന്ന് ശ്രദ്ധ തിരിച്ചുവിടാനും വേണ്ടിയാണ്ഖുര്‍ആനില്‍ വൈരുധ്യങ്ങളുണ്ടെന്ന വാദവുമായി അത്തരക്കാര്‍രംഗത്തുവരുന്നത്.

ഖുര്‍ആനില്‍ വൈരുധ്യങ്ങളൊന്നുമില്ലെന്ന് പറയുമ്പോള്‍ അതില്‍വൈവിധ്യങ്ങളില്ലെന്ന് അര്‍ത്ഥമാക്കിക്കൂടാത്തതാണ്. വൈവിധ്യവുംവൈരുധ്യവും ഒന്നല്ല; അവ തികച്ചും വ്യത്യസ്തങ്ങളാണ്. വൈവിധ്യങ്ങളെവൈരുധ്യങ്ങളായി തെറ്റിദ്ധരിപ്പിച്ചുകൊണ്ടാണ് ഖുര്‍ആനില്‍വൈരുധ്യങ്ങളുണ്ടെന്ന വാദവുമായി വിമര്‍ശകന്‍മാര്‍ രംഗത്തുവരാറുള്ളത്.ഒരു ഉദാഹരണം: ബൈബിള്‍ പുതിയ നിയമത്തിലെ പ്രധാനപ്പെട്ടവൈരുധ്യമാണ് വംശാവലിയിലെ വൈരുധ്യങ്ങള്‍. മത്തായിയും (1:6-16)ലൂക്കോസും (3:23-31) രേഖപ്പെടുത്തിയ യേശുവിന്റെ വംശാവലികള്‍ തമ്മില്‍കുറേയധികം വൈരുധ്യങ്ങളുണ്ട്. അതിനുകാരണം മത്തായി, ദാവീദിന്റെപുത്രനായ സോളമന്റെ പുത്രപരമ്പരയിലും ലൂക്കോസ്, ദാവീദിന്റെമകനായ നാഥാന്റെ പുത്രപാരമ്പര്യത്തിലും യേശുവിനെ പ്രതിഷ്ഠിക്കാന്‍പരിശ്രമിച്ചതാണ്. മത്തായിയുടെ വംശാവലി പ്രകാരം ദാവീദു മുതല്‍യേശുവരെ 28 പേരാണ് ഉള്ളതെങ്കില്‍ ലൂക്കോസ് നല്‍കിയ വംശാവലി പ്രകാരം 43 പേരാണുള്ളത്. യേശുവിന്റെ പിതാവായി അറിയപ്പെട്ടയോസേഫിന്റെ പിതാവ് ആരാണെന്ന പ്രശ്‌നം മുതല്‍ വൈരുധ്യങ്ങള്‍ആരംഭിക്കുന്നു. മത്തായി പറയുന്നത് യാക്കോബാണെന്നും ലൂക്കോസ്പറയുന്നത് ഹേലിയാണെന്നുമാണ്. ഒരാള്‍ക്ക് ഒരൊറ്റപിതാവേയുണ്ടാവൂയെന്നതിനാല്‍ ഇതൊരു വ്യക്തമായ വൈരുധ്യമാണ്.എന്നാല്‍ മത്തായിയും ലൂക്കോസും യോസേഫിന്റെ സഹോദരന്റെപേരായിരുന്നു പറഞ്ഞതെങ്കിലോ? മത്തായി യോസേഫിന്റെ സഹോദരന്‍യാക്കോബ് എന്നും, ലൂക്കോസ് യോസേഫിന്റെ സഹോദരന്‍ ഹേലിയെന്നുംപറഞ്ഞുവെന്നിരിക്കട്ടെ. ഈ പരാമര്‍ശങ്ങള്‍ തമ്മില്‍ വൈരുധ്യംആരോപിക്കുന്നത് ശരിയായിരിക്കുകയില്ല. ഒരാള്‍ക്ക് രണ്ടു സഹോദരന്‍മാര്‍ഉണ്ടാവുക സ്വാഭാവികമാണല്ലോ. മത്തായി, യോസേഫിന്റെ യാക്കോബ്എന്ന സഹോദരനെ കുറിച്ചും ലൂക്കോസ്, ഹേലിയെന്ന സഹോദരനെസംബന്ധിച്ചുമാണ് പറഞ്ഞതെന്ന് വിചാരിക്കാവുന്നതാണ്. ഇത് രണ്ടുപേരുടെപരാമര്‍ശങ്ങളിലുണ്ടാകാവുന്ന വൈവിധ്യത്തിന് ഉദാഹരണമാണ്; ഈവൈവിധ്യം വൈരുധ്യമല്ലെന്ന വസ്തുത മനസ്സിലാക്കേണ്ടതുണ്ട്.

ഖുര്‍ആന്‍ ഒരു ചരിത്രഗ്രന്ഥമല്ല. എന്നാല്‍ ചരിത്ര സംഭവങ്ങളെ കുറിച്ചപ്രതിപാദനങ്ങള്‍ ഖുര്‍ആനിലുണ്ട്. പ്രസ്തുത പ്രതിപാദനങ്ങള്‍ബൈബിളിലേതുപോലെ ഓരോന്നും സംഭവിച്ച ക്രമത്തിലല്ല ഖുര്‍ആനില്‍പ്രത്യക്ഷപ്പെടുന്നത്. അതിന്ന് കാരണമുണ്ട്, ഇസ്‌റാഈല്‍ സമുദായത്തിന്റെചരിത്രമാണ് ബൈബിള്‍ പഴയനിയമത്തിന്റെ പൊതുവായ പരാമര്‍ശം.ഉല്‍പത്തി മുതല്‍ മോശയുടെ മരണം വരെയുള്ള സംഭവങ്ങളാണ്പഞ്ചപുസ്തകത്തിലുള്ളത്. മറ്റു പ്രവാചകന്‍മാരുടെയുംദീര്‍ഘദര്‍ശിമാരുടെയും ചരിത്രങ്ങള്‍ മറ്റു പഴയ നിയമ ഗ്രന്ഥങ്ങളില്‍ കാണാം.പുതിയ നിയമത്തിലെ സുവിശേഷങ്ങളിലാകട്ടെ യേശുവിന്റെ കഥയാണ്നമുക്ക് കാണാന്‍ കഴിയുക. ഇവയെല്ലാം ചരിത്രപ്രതിപാദനഗ്രന്ഥങ്ങളായതിനാല്‍ സംഭവവിവരണത്തിന്റെ രീതിയാണ്സ്വീകരിക്കപ്പെട്ടിരിക്കുന്നത്. ഖുര്‍ആനാകട്ടെ സംഭവവിവരണത്തിന്റെരീതിയിലല്ല ചരിത്രങ്ങള്‍ പ്രതിപാദിച്ചിരിക്കുന്നത്. ഖുര്‍ആന്‍ പ്രദാനംചെയ്യുന്ന ധാര്‍മ്മിക നിര്‍ദേശങ്ങള്‍ക്ക് ഉപോല്‍ബലകമായചരിത്രസംഭവങ്ങള്‍ എടുത്തുദ്ധരിക്കുകയാണ് അത് ചെയ്യുന്നത്. അതുകൊണ്ട്തന്നെ പ്രതിപാദ്യ വിഷയങ്ങളുടെ ആവശ്യകതയ്ക്കനുസരിച്ച് ചരിത്രത്തിലെസംഭവങ്ങള്‍ എടുത്തുദ്ധരിക്കുന്ന രീതിയാണ് ഖുര്‍ആന്‍സ്വീകരിച്ചിരിക്കുന്നത്. ഇങ്ങനെ ഉദ്ധരിക്കുമ്പോള്‍ ചരിത്രത്തിലെ കാലക്രമംഖുര്‍ആന്‍ പരിഗണിക്കുന്നേയില്ല. അത്തരമൊരു പരിഗണനഅനാവശ്യമാണല്ലോ.

ചരിത്ര പ്രതിപാദനത്തിന് ഖുര്‍ആന്‍ സ്വീകരിച്ചിരിക്കുന്ന രീതിയുടെസവിശേഷത മറച്ചുവെച്ചുകൊണ്ടാണ് ചില വൈരുധ്യങ്ങള്‍ അതിന്മേല്‍ആരോപിക്കപ്പെടുന്നത്. മോശയുടെ ചരിത്രം പറഞ്ഞതിനു ശേഷമായിരിക്കുംഖുര്‍ആന്‍ ചിലപ്പോള്‍ അബ്രഹാമിന്റെ ചരിത്രത്തില്‍ നിന്നുള്ള സംഭവങ്ങള്‍ഉദ്ധരിക്കുന്നത്. അബ്രഹാമിന് മുമ്പാണ് മോശ ജീവിച്ചത് എന്ന് ഖുര്‍ആന്‍ഇതുകൊണ്ട് അര്‍ഥമാക്കുന്നില്ല. മോശയുടെ ചരിത്രത്തില്‍നിന്ന്പാഠമുള്‍ക്കൊള്ളേണ്ട കാര്യങ്ങള്‍ പ്രതിപാദിക്കുമ്പോള്‍ അത്ഉദ്ധരിക്കപ്പെടുന്നു; അബ്രഹാമിന്റെ ജീവിത സംഭവങ്ങള്‍പറയേണ്ടിവരുമ്പോള്‍ അതും ഉദ്ധരിക്കുന്നു. അവയെ കാലിക ക്രമത്തില്‍എടുക്കേണ്ടതില്ല. അങ്ങനെ എടുക്കണമെന്ന് ഖുര്‍ആന്‍ ഒരിടത്തുംആവശ്യപ്പെടുന്നില്ല. അതുകൊണ്ട് തന്നെ അത്തരം സംഭവവിവരണങ്ങള്‍വൈരുധ്യങ്ങളുടെ ഗണ ത്തില്‍ ഉള്‍പ്പെടുന്നില്ല.

ഖുര്ആന് ശുദ്ധമായ അറബി ഭാഷയിലാണ് എന്ന് 16:103 ല് പറയുന്നു. എന്നാല് ഖുര്ആനില് ഒട്ടനവധി അനറബി പദങ്ങള് ഉപയോഗിക്കപ്പെട്ടതായി കാണുന്നുണ്ട്. സൂചിപ്പിക്കപ്പെട്ട ഖുര്ആന് വാക്യം തെറ്റാണെന്നല്ലേ ഇതിനര്ഥം?

സൂറത്തുന്നഹ്‌ലിലെ 103 ാം വചനം ഖുര്‍ആനിനെതിരെയുള്ളസത്യനിഷേധികളുടെ ഒരു വാദത്തെ ഖണ്ഡിക്കുകയാണ് ചെയ്യുന്ന ത്. ജാബിര്‍റൂമി എന്ന ഒരു അനറബിയുമായി പ്രവാചകനുണ്ടായിരുന്ന അടുപ്പത്തെഅടിസ്ഥാനമാക്കി ഖുര്‍ആന്‍ വചനങ്ങള്‍ അയാള്‍ പറഞ്ഞുകൊടുക്കുന്നതാണെന്ന ഒരു വിമര്‍ശനം മക്കാമുശ്‌രിക്കുകള്‍ഉന്നയിക്കുകയുണ്ടായി. അറബി സാഹിത്യകാരന്‍മാരെ വെല്ലുവിളിക്കുന്നഒരു മഹല്‍ ഗ്രന്ഥത്തിലെ വചനങ്ങള്‍ ഒരു അനറബിയുടെ സൃഷ്ടിയാണെന്നവാദത്തിന്റെ ബാലിശത വ്യക്തമാക്കുകയാണ് 16:103 ചെയ്യുന്നത്.

ഈ വചനത്തില്‍, ഇതാകട്ടെ സ്പഷ്ടമായ അറബിഭാഷയുമാകുന്നുവെന്നാണ് ഖുര്‍ആനിനെ സൂചിപ്പിച്ചുകൊണ്ട്പറഞ്ഞിരിക്കുന്നത്. അറബിയ്യുന്‍ മുബീന്‍ എന്നാണ് പ്രയോഗം. ഇതിന്ശുദ്ധമായ അറബി ഭാഷ എന്നര്‍ഥമില്ല. സ്പഷ്ടമായ അറബി ഭാഷഎന്നാണര്‍ഥം. ലോകത്തിലെ എല്ലാ ഭാഷകളിലും ഇതര ഭാഷകളില്‍ നിന്നുള്ളപദങ്ങള്‍ കടന്നുകൂടിയിട്ടുണ്ട്. ആധുനിക ഭാഷകളില്‍ മിക്കതിലും അവയുടെശബ്ദ സമ്പത്തില്‍ കാല്‍ഭാഗത്തിലധികവും ഇതര ഭാഷകളില്‍ നിന്നുള്ളപദങ്ങളാണുള്ളത്. അറബിയില്‍- വിശേഷിച്ചും പൗരാണിക അറബിയില്‍-ഇത്തരം പദങ്ങള്‍ തുലോം വിരളമാണ്. എങ്കിലും ഗ്രീക്കിലെഇവാന്‍ഗലിയോണ്‍ എന്ന പദത്തില്‍ നിന്നുണ്ടായ ഇഞ്ചീല്‍ എന്ന അറബിപ്രയോഗത്തെപോലെയുള്ള ചില അറബീകരിക്കപ്പെട്ട പദങ്ങള്‍ഖുര്‍ആനിലുണ്ട്. എന്നാല്‍ ഇവ അന്യഭാഷാ പ്രയോഗങ്ങളാണ് എന്ന്പറയുന്നതില്‍ യാതൊരു അര്‍ത്ഥവുമില്ല.ഇവാന്‍ഗലിയോണ്‍’ഇഞ്ചീലാകുന്നതോടെ ആ പദം അറബിയായിമാറികഴിഞ്ഞുവെന്നുള്ളതാണ് വാസ്തവം. കീസ് എന്ന അറബിപദത്തില്‍നിന്നാണ് മലയാളത്തിലെ കീശയുടെ വ്യുല്‍പത്തി. ഇതിനാല്‍ കീശമലയാള പദമല്ല എന്ന് പറയുന്നത് വിവരക്കേടാണ്. ഇതേ പോലെതന്നെയാണ്എല്ലാ ഭാഷകളുടെയും സ്ഥിതി.

ചുരുക്കത്തില്‍ ഖുര്‍ആന്‍ സ്പഷ്ടമായ അറബിയിലാണെന്നപ്രസ്താവനയുമായി അതിലെ മറ്റു ഭാഷകളിലെ പദങ്ങളില്‍ നിന്ന് കടന്നുവന്നവാക്കുകളുടെ സാന്നിധ്യം യാതൊരു വിധത്തിലും വൈരുധ്യംപുലര്‍ത്തുന്നില്ല.

തന്റെ മരണത്തിനു മുമ്പ് ഫറോവ പശ്ചാത്തപിക്കുകയും അയാളെ ദൈവം രക്ഷപ്പെടുത്തുകയും ചെയ്തതായി 10:90 92 വരെ സൂക്തങ്ങളില് പറയുന്നു. എന്നാല് മരണം ആസന്നമാകുമ്പോഴുള്ള പശ്ചാത്താപം സ്വീകാര്യമല്ലെന്ന് 4:18ലും പറയുന്നു. ഇത് വൈരുധ്യമല്ലേ?

റിവില്ലായ്മ നിമിത്തം തിന്മ ചെയ്യുകയും എന്നിട്ട് താമസിയാതെ ആത്മാര്‍ഥമായിപശ്ചാത്തപിക്കുകയും ചെയ്തവര്‍ക്ക് പൊറുത്തു കൊടുക്കാമെന്ന ദൈവിക വാഗ്ദാനത്തിന് ശേഷം, മരണംആസന്നമായി ജീവന്‍ തൊണ്ടക്കുഴിയിലെത്തുമ്പോള്‍ ഞാന്‍ പശ്ചാത്തപിച്ചിരിക്കുന്നു എന്ന് പറഞ്ഞുകൊണ്ടുള്ള പ്രഹസനമല്ല ഇതുകൊണ്ടു വിവക്ഷിച്ചിരിക്കുന്നതെന്ന് വ്യക്തമാക്കുകയാണ് ഖുര്‍ആന്‍ 4:18ല്‍ചെയ്യുന്നത്. മരണത്തെ മുഖാമുഖം കണ്ടുകൊണ്ടിരിക്കുമ്പോള്‍ ഏത് ക്രൂരനായ അവിശ്വാസിയുംപശ്ചാത്താപവിവശനായി തീരുമെന്ന വസ്തുത ഖുര്‍ആനിലെ വ്യത്യസ്ത സൂക്തങ്ങളില്‍വ്യക്തമാക്കപ്പെട്ടിട്ടുണ്ട്. (ഉദാ:63:10,11) ഏകച്ഛത്രാധിപതിയായിരുന്ന ഫറോവയുടെ അന്ത്യവും ഇക്കാര്യത്തിനുള്ളതെളിവായിട്ടാണ് ഖുര്‍ആന്‍ എടുത്തുകാണിക്കുന്നത്.

‘ഇസ്‌റാഈല്‍ സന്തതികളെ നാം കടല്‍ കടത്തികൊണ്ടുപോയി. അപ്പോള്‍ ഫിര്‍ഔനും അവന്റെസൈന്യങ്ങളും ധിക്കാരവും അതിക്രമവുമായി അവരെ പിന്തുടര്‍ന്നു. ഒടുവില്‍ മുങ്ങിമരിക്കാറായപ്പോള്‍ അവന്‍പറഞ്ഞു, ഇസ്‌റാഈല്‍ സന്തതികള്‍ ഏതൊരു ദൈവത്തില്‍ വിശ്വസിച്ചിരിക്കുന്നുവോ അവനല്ലാതെ യാതൊരുദൈവവുമില്ല എന്ന് ഞാന്‍ വിശ്വസിച്ചിരിക്കുന്നു. ഞാന്‍ മുസ്ലിംകളുടെ കൂട്ടത്തിലാകുന്നു. (അല്ലാഹു അവനോട്പറഞ്ഞു) മുമ്പൊക്കെ ധിക്കരിക്കുകയും കുഴപ്പക്കാരുടെ കൂട്ടത്തിലായിരിക്കുകയും ചെയ്തിട്ട് ഇപ്പോഴാണോ(നീ വിശ്വസിക്കുന്നത്). എന്നാല്‍ നിന്റെ പുറകെ വരുന്നവര്‍ക്ക് നീ ഒരു ദൃഷ്ടാന്തമായിരിക്കേണ്ടതിന് വേണ്ടിഇന്ന് നിന്റെ ശരീരത്തെ നാം രക്ഷപ്പെടുത്തിയെടുക്കുന്നതാണ്. തീര്‍ച്ചയായും മനുഷ്യരില്‍ ധാരാളം പേര്‍നമ്മുടെ ദൃഷ്ടാന്തങ്ങളെപ്പറ്റി അശ്രദ്ധരാകുന്നു.’ (10:90-92)

ഈ സൂക്തങ്ങളിലെവിടെയും മരണ വക്ത്രത്തിലുള്ള ഫറോവയുടെ പശ്ചാത്താപം അല്ലാഹുസ്വീകരിച്ചുവെന്നോ അവന് പൊറുത്തുകൊടുത്തുവെന്നോ പറയുന്നില്ല. ഏതൊരു സ്വേച്ഛാധിപതിയുംമരണത്തെ മുഖാമുഖം കാണുമ്പോള്‍ പശ്ചാത്താപ വിവശനായിത്തീരുമെന്ന വസ്തുതക്കുള്ളതെളിവായികൊണ്ടാണ് ഈ സൂക്തത്തില്‍ ഫറോവയുടെ ചരിത്രം വിവരിച്ചിരിക്കുന്നത്.

എന്നാല്‍ നിന്റെ പുറകെ വരുന്നവര്‍ക്ക് നീ ഒരു ദൃഷ്ടാന്തമായിരിക്കേണ്ടതിന് വേണ്ടി ഇന്ന് നിന്റെശരീരത്തെ നാം രക്ഷപ്പെടുത്തിയെടുക്കുന്നതാണ് എന്ന ദൈവീക വചനം ഫറോവയ്ക്ക് രക്ഷ ലഭിച്ചുവെന്നാണ്വ്യക്തമാക്കുന്നതെന്ന് ഈ സൂക്തങ്ങള്‍ ഒരു തവണ വായിച്ചവരൊന്നും പറയുകയില്ല. മറ്റു ശരീരങ്ങളെ പോലെഫറോവയുടെ ശരീരം ജീര്‍ണിക്കരുതെന്നും അത് മാനവരാശിക്ക് ദൃഷ്ടാന്തമായി തീരണമെന്നുമുള്ള ദൈവീകതീരുമാനത്തിന്റെ പ്രഖ്യാപനമാണിത്. പ്രസ്തുത പ്രഖ്യാപനത്തിന്റെ പുലര്‍ച്ചയെന്നോണം ഇന്നുംഫറോവയുടെ ശരീരം ജീര്‍ണിക്കാതെ കിടക്കുന്നുവെന്നത് ഖുര്‍ആനിന്റെ ദൈവീകതയ്ക്കുള്ള ജീവിക്കുന്നതെളിവുകളിലൊന്നാണ്.

അല്ലാഹുവിന്റെ വചനങ്ങള്ക്ക് മാറ്റം വരുത്താന് ആരുമില്ല എന്ന് 6:115 ല് പറഞ്ഞതിന്ന് വിരുദ്ധമായി ആയത്തുകള് അല്ലാഹു ദുര്ബലപ്പെടുത്തുമെന്ന് 2:106 ലും പറയുന്നതായി കാണാം. ഈ വൈരുധ്യത്തെ എങ്ങനെ വിശദീകരിക്കും?

ത്യ സമ്പൂര്‍ണങ്ങളും നീതി യുക്തങ്ങളുമായ ദൈവീക വചനങ്ങളില്‍യാതൊരു വിധ നീക്കു പോക്കുകളും പാടില്ലെന്നാണ് 6:115 ഉം ഇക്കാര്യംവിശദീകരിക്കപ്പെട്ട മറ്റു സൂക്തങ്ങളും വ്യക്തമാക്കുന്നത്. ദൈവീകവചനങ്ങള്‍ക്ക് പകരം അവയോട് കിടയൊക്കുന്ന തോ അവയേക്കാള്‍പ്രായോഗികമോ യുക്തമോ ആയ വേറെ വചനങ്ങള്‍ ആവിഷ്‌ക്കരിക്കുവാന്‍ആര്‍ക്കും കഴിയില്ല. മാനവരാശിക്ക് ആത്യന്തികമായി ഗുണകരമായത്എന്താണെന്നും ദോഷകരമായതെന്താണെന്നും കൃത്യമായി അറിയാവുന്നപടച്ചതമ്പുരാന്റെ വചനങ്ങള്‍ക്ക് പകരം വെക്കുവാന്‍ പറ്റിയവചനങ്ങളൊന്നും കൊണ്ടു വരാന്‍ മനുഷ്യര്‍ക്ക് കഴിയില്ല. ജനഹിതം മാനിച്ച്ദൈവീക വിധിവിലക്കുകള്‍ക്ക് വിരുദ്ധമായ മാര്‍ഗങ്ങളിലൂടെ സഞ്ചരിച്ചജനസമൂഹങ്ങള്‍ക്ക് തിക്തമായ ഫലങ്ങള്‍ നേരിടേണ്ടി വന്നിട്ടുണ്ട്. അതിനാല്‍ദൈവീക വചനങ്ങളെ മാറ്റി മറിച്ച് പ്രായോഗികവും മനുഷ്യര്‍ക്കാകമാനംആത്യന്തികമായ നന്‍മ വരുത്തുന്നതുമായ നിയമങ്ങള്‍ നിര്‍മ്മിക്കുവാന്‍ ആര്വിചാരിച്ചാലും സാധ്യമല്ല. മുകളില്‍ പരാമര്‍ശിക്കപ്പെട്ട വചനങ്ങളെയോഅവയുള്‍ക്കൊള്ളുന്ന വിധിവിലക്കുകളെയോ മാറ്റി മറിക്കുവാന്‍ ആര്‍ക്കുംഅവകാശമില്ലെന്നും അങ്ങനെ മാറ്റിമറിക്കുവാന്‍ ആരെങ്കിലും ധൃഷ്ടരായാല്‍അതിന്റെ തിക്ത ഫലം അനുഭവിക്കേണ്ടി വരുമെന്നുമാണ് ഇവ നല്‍കുന്നപാഠം.

ദൈവീക വചനങ്ങളെ മാറ്റിമറിക്കുവാന്‍ സൃഷ്ടികള്‍ക്കാര്‍ക്കുംഅവകാശമില്ലെന്ന പരാമര്‍ശം ഏതെങ്കിലും വചനത്തെദുര്‍ബലപ്പെടുത്തുവാന്‍ അല്ലാഹുവിന് അധികാരമുണ്ടായിരിക്കുന്നതിന്വിരുദ്ധമാകുന്നില്ല. സമൂഹത്തിന്റെ പരിണാമത്തിനിടയില്‍ ചില നിയമങ്ങള്‍ദുര്‍ബലപ്പെടുത്തുകയും പുതിയ നിയമങ്ങള്‍ നിര്‍ദേശിക്കുകയുംചെയ്യുവാനുള്ള അവകാശവും അല്ലാഹുവിന് തന്നെയാണ്. പൂര്‍വ്വവേദങ്ങളിലെ വിധികളില്‍ ചിലവ ശേഷം വന്ന വേദങ്ങളില്‍തിരുത്തപ്പെട്ടിട്ടുണ്ട്. തൗറാത്തിലും ഇഞ്ചീലിലുമുള്ള ചില വിധികള്‍ഖുര്‍ആനിലുള്ള പുതിയ വിധികളാല്‍ ദുര്‍ബലമാക്കപ്പെട്ടിട്ടുണ്ട്. ഖുര്‍ആനില്‍തന്നെ ആദ്യകാലത്ത് അവതരിപ്പിക്കപ്പെട്ട ചില വിധികള്‍ പിന്നീ ട്ദുര്‍ബലമാക്കിയിട്ടുണ്ട്. ഇതെല്ലാം ചെയ്തത് മനുഷ്യ സമുഹത്തെ യുംഅതിന്റെ പരിണാമത്തെയും കുറിച്ച് നന്നായി അറിയാവുന്ന അല്ലാഹുതന്നെയാണ്. അതുകൊണ്ട് തന്നെ ഈ മാറ്റങ്ങളൊന്നും സമൂഹത്തില്‍പ്രശ്‌നങ്ങള്‍ സൃഷ്ടിക്കാറില്ല. എന്നാല്‍ ധാര്‍മിക രംഗത്തെ ദൈവികവിധിവിലക്കുകളെ തൃണവല്‍ഗണിക്കുകയും തന്നിഷ്ടം പ്രവര്‍ത്തിക്കുകയുംചെയ്ത സമൂഹങ്ങളെല്ലാം അതിന്റെ തിക്ത ഫലം അനുഭവിക്കേണ്ടിവന്നിട്ടുണ്ട്. ദൈവിക വിധിവിലക്കുകളെ മാറ്റി മറിക്കാന്‍ ആര്‍ക്കുംഅവകാശമില്ലെന്ന് വ്യക്തമാക്കുന്ന സൂക്തങ്ങള്‍ സാമൂഹ്യപരിണാമത്തിനനുസരിച്ച് അല്ലാഹു തന്നെ ചില നിയമങ്ങളില്‍ മാറ്റംവരുത്തുമെന്ന് പഠിപ്പിക്കുന്ന സൂക്തങ്ങളുമായി യാതൊരു വിധത്തിലുംവൈരുധ്യം പുലര്‍ത്തുന്നില്ലെന്നര്‍ഥം.

വ്യഭിചാര കുറ്റത്തിന് നൂറടി നല്കണമെന്ന 24:2 ലെ വിധിക്ക് വിരുദ്ധമായി അവരിലെ സ്ത്രീകളെ വീട്ടുതടങ്കലില് വെക്കണമെന്ന് 4:15 ലും പുരുഷന്മാരെ പീഡിപ്പിക്കണമെന്ന് 4:16 ലും പറയുന്നു. ഇത് വൈരുധ്യമല്ലേ?

സാംസ്‌കാരികമായി വട്ടപ്പൂജ്യത്തിലായിരുന്ന ഒരു ജനതയെ 23 വര്‍ഷംകൊണ്ട് മാതൃകായോഗ്യമായ ഒരു സമൂഹമാക്കി മാറ്റിയ ഒരു ഗ്രന്ഥമാണ്ഖുര്‍ആന്‍. പ്രസ്തുത സമൂഹത്തിന്റെ മാറ്റം നടന്നത് ഒരൊറ്റ നിമിഷംകൊണ്ടായിരുന്നില്ല. വികല വിശ്വാസങ്ങളില്‍ നിന്ന് അവരെവിമലീകരിക്കുകയും സംസ്‌കരിക്കാന്‍ പോന്ന കര്‍മങ്ങളിലൂടെ അവരെവിശുദ്ധീകരിക്കുകയും ചെയ്തതോടൊപ്പം തലമുറകളായി അവര്‍ആമഗ്‌നരായിരുന്ന അധാര്‍മികവൃത്തികളില്‍ നിന്ന് ഘട്ടംഘട്ടമായി അവരെമോചിപ്പിക്കുകയുമാണ് ഖുര്‍ആന്‍ ചെയ്തത്. ആ സമൂഹത്തിന്റെ,പരിണാമത്തിന്റെ വ്യത്യസ്ത ഘട്ടങ്ങളില്‍ അവതരിപ്പിക്കപ്പെട്ട നിയമങ്ങള്‍വ്യത്യസ്തങ്ങളായിരിക്കുക സ്വാഭാവികമാണ്.അങ്ങനെയല്ലായിരുന്നുവെങ്കില്‍ അത്തരമൊരു സമൂഹത്തെ ആമൂലാഗ്രംപരിവര്‍ത്തിപ്പിക്കുക സാധ്യമാവുമായിരുന്നില്ല.

മദ്യപാ നവുംവ്യഭിചാരവുമെല്ലാം നിയമം മൂലം നിരോധിക്കപ്പെട്ടത് ഘട്ടങ്ങളായിട്ടായിരുന്നു.വ്യഭിചാരിക്കും വ്യഭിചാരിണിക്കും ആദ്യം വിധിക്കപ്പെട്ട ശിക്ഷയാണ്4:15,16 സൂക്തങ്ങളില്‍ പരാമര്‍ശിക്കപ്പെട്ടിരിക്കുന്നത്.വ്യഭിചാരത്തിലേര്‍പ്പെടുന്ന സ്ത്രീ പുരുഷന്‍മാരെപീഡിപ്പിക്കണമെന്നായിരുന്നു ആദ്യത്തെ കല്‍പന. അതോടൊപ്പം പ്രസ്തുതദുര്‍വൃത്തിയിലേര്‍പ്പെടുന്ന സ്ത്രീകളെ വീടുകളില്‍ തടഞ്ഞുവെക്കണമെന്നുംഅത് വ്യാപിക്കുവാന്‍ ഇടവരുത്തരുതെന്നും കൂടി കല്‍പ്പിക്കപ്പെട്ടു. എന്നാല്‍ഈ കല്‍പന അല്ലാഹു അവര്‍ക്കൊരു മാര്‍ഗം ഉണ്ടാക്കുന്നത് വരെ(4:15)യാണെന്ന് വ്യക്തമാക്കപ്പെടുകയും ചെയ്തു. വ്യഭിചാരവൃത്തിയില്‍മുങ്ങിക്കുളിച്ചിരുന്ന ഒരു സമൂഹത്തെ പരിവര്‍ത്തിപ്പിക്കുന്നതിന്റെ ആദ്യപടിയായിരുന്നു ഇത്. ശേഷം ഇസ്‌ലാമിക സമൂഹം വളര്‍ന്നപ്പോള്‍പ്രസ്തുത ദുര്‍വൃത്തിക്കുള്ള കൃത്യവും വ്യക്തവുമായ ശിക്ഷാവിധികള്‍നിര്‍ദേശിക്കപ്പെടുകയും ചെയ്തു. വിവാഹിതരല്ലാത്തവ്യഭിചാരികള്‍ക്കുള്ള വ്യക്തമായ ശിക്ഷ 24:2ല്‍ അവതരിപ്പിക്കപ്പെട്ടതോടെ4:15,16 ലെ നിയമം ദുര്‍ബലപ്പെടുത്തപ്പെട്ടുവെന്ന് വ്യാഖ്യാതാക്കള്‍വ്യക്തമാക്കിയിട്ടുണ്ട്. അഥവാ സമൂഹപരിണാമത്തിന്റെ വ്യത്യസ്തഘട്ടങ്ങളിലുണ്ടായ രണ്ട് നിയമങ്ങളാണ് 4:15,16 ലും24:2ലുംവ്യക്തമാക്കപ്പെട്ടിരിക്കുന്നത്. ഇവ തമ്മില്‍ വൈരുധ്യമില്ല. അവസാനംഅവതരിപ്പിക്കപ്പെട്ട നിയമമെന്ന നിലയ്ക്ക് 24:2ലെ നിര്‍ദേശമാണ് അതിന്റെഅവതരണത്തിന് ശേഷം അവസാന നാളുവരെയുള്ള വിശ്വാസികള്‍ക്ക്ബാധകമായിട്ടുള്ളത്.

ഖുര്‍ആനിനുമുമ്പ് അവതരിപ്പിക്കപ്പെട്ട വേദഗ്രന്ഥങ്ങളെയെല്ലാം അത് അംഗീകരിക്കുന്നു. ആകെ എത്ര വേദഗ്രന്ഥങ്ങള്‍ അവതരിപ്പിക്കപ്പെട്ടുവെന്ന് ഖണ്ഡിതമായി ഖുര്‍ആന്‍ പ്രസ്താവിക്കുന്നില്ല. നാല് വേദഗ്രന്ഥങ്ങളുടെ പേര് മാത്രമാണ് ഖുര്‍ആനില്‍ പരാമര്‍ശിക്കപ്പെട്ടിരിക്കുന്നത്. മൂസാ നബി(അ)ക്ക് അവതരിപ്പിക്കപ്പെട്ട തൗറാത്തും ദാവൂദ് നബി(അ)ക്ക് അവതരിപ്പിക്കപ്പെട്ട സബൂറും ഈസാനബി(അ)ക്ക് അവതരിപ്പിക്കപ്പെട്ട ഇന്‍ജീലും മുഹമ്മദ്( ﷺ)ക്ക് അവതരിപ്പിക്കപ്പെട്ട ഖുര്‍ആനുമാണവ. ഈ നാലു വേദഗ്രന്ഥങ്ങള്‍ക്കു പുറമെയും എഴുതപ്പെട്ട രേഖകള്‍ പടച്ചതമ്പുരാനില്‍ നിന്ന് അവതരിപ്പിക്കപ്പെട്ടിട്ടുണ്ടെന്നാണ് ഖുര്‍ആന്‍ നല്‍കുന്ന സൂചന.

‘നിങ്ങള്‍ പറയുക: അല്ലാഹുവിലും അവങ്കല്‍നിന്ന് ഞങ്ങള്‍ക്ക് അവതരിപ്പിച്ചു കിട്ടിയതിലും ഇബ്‌റാഹീമിനും ഇസ്മായിലിനും ഇസ്ഹാഖിനും യഅ്ഖൂബിനും യഅ്ഖൂബ് സന്തതികള്‍ക്കും അവതരിപ്പിച്ചുകൊടുത്തതിലും മൂസാ, ഈസാ എന്നിവര്‍ക്ക് നല്‍കപ്പെട്ടതിലും സര്‍വപ്രവാചകന്മാര്‍ക്കും അവരുടെ രക്ഷിതാവിങ്കല്‍നിന്ന് നല്‍കപ്പെട്ടതിലും ഞങ്ങള്‍ വിശ്വസിച്ചിരിക്കുന്നു‘ (2:136)

‘തീര്‍ച്ചയായും ഇത് ആദ്യത്തെ ഏടുകളില്‍തന്നെയുണ്ട്, അഥവാ ഇബ്റാഹീമിന്റെയും മൂസായുടെയും ഏടുകളില്‍ (87:18,19).

മുമ്പുള്ള വേദങ്ങളെ മുഴുവന്‍ ഖുര്‍ആന്‍ സത്യപ്പെടുത്തുന്നു: ‘അവന്‍ ഈ വേദഗ്രന്ഥത്തെ മുന്‍വേദങ്ങളെ ശരിവെക്കുന്നതായിക്കൊണ്ട് സത്യവുമായി നിനക്ക് അവതരിപ്പിച്ചുതന്നിരിക്കുന്നു. മനുഷ്യര്‍ക്ക് മാര്‍ഗദര്‍ശനത്തിനായി ഇതിനുമുമ്പ് അവന്‍ തൗറാത്തും ഇന്‍ജീലും അവതരിപ്പിച്ചു. ഫുര്‍ഖാനും അവന്‍ അവതരിപ്പിച്ചിരിക്കുന്നു‘ (3:3).

അല്ലാഹുവില്‍നിന്ന് അവതരിപ്പിക്കപ്പെട്ട വേദഗ്രന്ഥങ്ങളിലെല്ലാം വിശ്വസിക്കേണ്ടത് മുസ്‌ലിമിന്റെ നിര്‍ബന്ധബാധ്യതയാണ്. മുമ്പ് അവതരിപ്പിക്കപ്പെട്ട വേദഗ്രന്ഥങ്ങളിലേതെങ്കിലും ദൈവികമല്ലെന്ന് വിശ്വസിക്കുന്നത് വലിയൊരു അപരാധമായിട്ടാണ് ഖുര്‍ആന്‍ കാണുന്നത്.

‘സത്യവിശ്വാസികളേ, അല്ലാഹുവിലും അവന്റെ ദൂതനിലും അവന്റെ ദൂതന് അവന്‍ അവതരിപ്പിച്ച ഗ്രന്ഥത്തിലും അവന്‍ മുമ്പ് അവതരിപ്പിച്ച ഗ്രന്ഥത്തിലും നിങ്ങള്‍ വിശ്വസിക്കുവിന്‍. അല്ലാഹുവിലും അവന്റെ മലക്കുകളിലും അവന്റെ ഗ്രന്ഥങ്ങളിലും അവന്റെ ദൂതന്മാരിലും അന്ത്യദിനത്തിലും വല്ലവനും അവിശ്വസിക്കുന്ന പക്ഷം തീര്‍ച്ചയായും അവന്‍ ബഹുദൂരം പിഴച്ചുപോയിരിക്കുന്നു‘ (4:136).

എല്ലാ ജനസമൂഹങ്ങളിലേക്കും പ്രവാചകന്മാര്‍ നിയോഗിക്കപ്പെട്ടിട്ടുണ്ട്. ‘ഒരു താക്കീതുകാരന്‍ കഴിഞ്ഞുപോകാത്ത ഒരു സമുദായവുമില്ല‘ (35:24) എന്നാണ് ഖുര്‍ആന്‍ അര്‍ഥശങ്കക്കിടയില്ലാത്തവിധം വ്യക്തമാക്കുന്നത്. അപ്പോള്‍ ചിരപുരാതനമായ ഒരു സംസ്‌കാരം നിലനിന്നിരുന്ന പ്രദേശമെന്ന നിലയ്ക്ക് ഇന്ത്യയിലും പ്രവാചകന്മാര്‍ വന്നിട്ടുണ്ടാവണം. ആ പ്രവാചകന്മാരില്‍ ചിലര്‍ക്ക് വേദഗ്രന്ഥങ്ങളും നല്‍കപ്പെട്ടിരിക്കണം. ഈ പ്രവാചകന്മാരെയോ വേദഗ്രന്ഥങ്ങളെയോ ഇകഴ്ത്തുകയോ അവമതിക്കുകയോ ചെയ്യാ ന്‍ മുസ്‌ലിമിന് പാടില്ല. പ്രവാചകന്മാര്‍ക്കിടയില്‍ വിവേചനം കല്‍പിക്കുന്നതിനെതിരെ ഖുര്‍ആന്‍ ശക്തമായി താക്കീത് നല്‍കുന്നുണ്ട് (4:150). അപ്പോള്‍ ഇന്ത്യയിലേക്കു പ്രവാചകന്മാര്‍ നിയോഗിക്കപ്പെട്ടിട്ടുണ്ടെങ്കില്‍ അവരെയും അവര്‍ക്ക് അവതരിപ്പിക്കപ്പെട്ട വേദഗ്രന്ഥങ്ങളെയും ഖുര്‍ആന്‍ ആദരി ക്കുന്നു. അംഗീകരിക്കുന്നു. എന്നാല്‍, ഇന്നുനിലനില്‍ക്കുന്ന ശ്രുതി ഗ്രന്ഥങ്ങളിലേതെങ്കിലും (വേദ സംഹിതകള്‍, ബ്രാഹ്മണങ്ങള്‍, ആരണ്യകങ്ങള്‍, ഉപനിഷത്തുകള്‍) പടച്ച തമ്പുരാന്‍ പ്രവാചകന്മാര്‍ക്ക് അവതരിപ്പിച്ച ഗ്രന്ഥങ്ങളാണെന്ന് പറയാന്‍ കഴിയുമോ? ഇവ ദൈവത്തിങ്കല്‍നിന്ന് ശ്രവിക്കപ്പെട്ടതിനാലാണ് ശ്രുതിയെന്നു വിളിക്കുന്നതെന്നാണ് വിശ്വാസം. ദൈവത്തിങ്കല്‍നിന്ന് മനുഷ്യര്‍ക്ക് പ്രത്യേകമായ സന്ദേശങ്ങള്‍ അവതരിപ്പിക്കപ്പെടുന്നുവെന്ന വിശ്വാസം ഹിന്ദുക്കള്‍ക്കിടയില്‍ നിലനിന്നിരുന്നുവെന്ന് ശ്രുതി സങ്കല്‍പം വ്യക്തമാക്കുന്നു. നടേ പറഞ്ഞ ഗ്രന്ഥങ്ങളെല്ലാം ശ്രുതികളായി വ്യവഹരിക്കപ്പെടുന്നുണ്ടെങ്കിലും അവയില്‍ ഏതെല്ലാം പ്രാമാണികമാണെന്ന കാര്യത്തില്‍ അഭിപ്രായാന്തരങ്ങളുണ്ട്. ആര്യസമാജ സ്ഥാപകനായ സ്വാമി ദയാനന്ദസരസ്വതി നാല് വേദസംഹിതകള്‍ക്കു മാത്രമാണ് അപ്രമാദിത്വമുള്ളതെന്ന് വാദിക്കുമ്പോള്‍ സ്വാമി വിവേകാനന്ദനെ പോലുള്ളവര്‍ ഉപനിഷത്തുകള്‍ക്കാണ് പ്രഥമ പ്രാധാന്യം നല്‍കുന്നത്. അടിസ്ഥാന ശ്രുതിഗ്രന്ഥങ്ങള്‍ക്കുപോലും തെറ്റുകള്‍ പറ്റാമെന്ന് അഭിപ്രായപ്പെട്ട ഹിന്ദുമത പണ്ഡിതന്മാരുണ്ട്.‘വേദങ്ങള്‍ തെറ്റു പറ്റാത്തവയോ സര്‍വതും ഉള്‍ക്കൊള്ളുന്നവയോ അല്ല‘ (Indian Religions page 22) എന്ന ഡോ. രാധാകൃഷ്ണന്റെ വീക്ഷണവും ‘വേദങ്ങളില്‍ യുക്തിയുമായി പൊരുത്തപ്പെടുന്നിടത്തോളം ഭാഗങ്ങള്‍ ഞാന്‍ സ്വീകരിക്കുന്നു. വേദങ്ങളിലെ ചില ഭാഗങ്ങള്‍ പ്രഥമ ദൃഷ്ടിയില്‍ പരസ്പരവിരുദ്ധങ്ങളാണ്‘. (വിവേകാനന്ദ സാഹിത്യ സര്‍വസ്വം വാല്യം-4, പുറം-55) എന്ന സ്വാമി വിവേകാനന്ദന്റെ നിലപാടും വേദങ്ങള്‍ നൂറുശതമാനം ദൈവിക വചനങ്ങളാണുള്‍ക്കൊള്ളുന്നതെന്ന വാദഗതിയുടെ മുനയൊടിക്കാന്‍ പോന്നതാണ്. പൊതുവെ പറഞ്ഞാല്‍, ഇന്ത്യയില്‍ നിലനിന്നിരുന്ന വിശ്വാസങ്ങളെയും ആചാരങ്ങളെയും കുറിച്ച വ്യക്തമായ ചിത്രം നല്‍കുന്ന ഗ്രന്ഥങ്ങളാണ് ശ്രുതികള്‍. ഇന്ത്യയിലേക്ക് നിയുക്തരായ പ്രവാചകന്മാര്‍ പ്രബോധനം ചെയ്ത ആശയങ്ങളുടെ ശകലങ്ങള്‍ ഇവയില്‍ കാണാനാവുമായിരിക്കണം. എന്നാല്‍, ഇവ പൂര്‍ണമായും ദൈവികമാണെന്ന വാദം ഇസ്‌ലാം അംഗീകരിക്കുന്നില്ല. വേദപണ്ഡിതന്മാർക്കും അങ്ങനെയൊരു വാദമില്ലല്ലോ.
മൂസാനബി(عليهالسلام)യുടെ കാലത്ത് ക്രൂശീകരണം ഒരു ശിക്ഷയായി നിലനിന്നിരുന്നുവെന്ന് സൂചിപ്പിക്കുന്ന ഒന്നിലധികം ഖുര്‍ആന്‍ സൂക്തങ്ങളുണ്ട്. ചില സൂക്തങ്ങളുടെ സാരം കാണുക: ”നിങ്ങളുടെ കൈകളും കാലുകളും എതിര്‍വശങ്ങളില്‍നിന്നായി ഞാന്‍ മുറിച്ചുകളയുക തന്നെ ചെയ്യും. പിന്നെ നിങ്ങളെ മുഴുവന്‍ ഞാന്‍ ക്രൂശിക്കുകയും ചെയ്യും, തീര്‍ച്ച”(വി.ഖു.7:124). ”അവന്‍(ഫിര്‍ഔന്‍) പറഞ്ഞു: ഞാന്‍ നിങ്ങള്‍ക്ക് അനുവാദം തരുന്നതിന് മുമ്പായി നിങ്ങള്‍ അവനില്‍ വിശ്വസിച്ചുവെന്നോ. തീര്‍ച്ചയായും ഇവന്‍ നിങ്ങള്‍ക്ക് ജാലവിദ്യ പഠിപ്പിച്ച നിങ്ങളുടെ തലവന്‍ തന്നെയാണ്. വഴിയെ നിങ്ങള്‍ അറിഞ്ഞുകൊള്ളും. തീര്‍ച്ചയായും നിങ്ങളുടെ കൈകളും നിങ്ങളുടെ കാലുകളും എതിര്‍വശങ്ങളില്‍നിന്നായിക്കൊണ്ട് ഞാന്‍ മുറിച്ചുകളയുകയും നിങ്ങളെ മുഴുവന്‍ ഞാന്‍ ക്രൂശിക്കുകയും ചെയ്യുന്നതാണ്” (വി.ഖു. 26:49). ”അവന്‍(ഫിര്‍ഔന്‍) പറഞ്ഞു: ഞാന്‍ നിങ്ങള്‍ക്ക് സമ്മതം തരുന്നതിന് മുമ്പ് നിങ്ങള്‍ അവനെ വിശ്വസിച്ചുകഴിഞ്ഞുവെന്നോ? തീര്‍ച്ചയായും നിങ്ങള്‍ക്ക് ജാലവിദ്യ പഠിപ്പിച്ചുതന്ന നിങ്ങളുടെ നേതാവ് തന്നെയാണ് അവന്‍. ആകയാല്‍ തീര്‍ച്ചയായും ഞാന്‍ നിങ്ങളുടെ കൈകളും കാലുകളും എതിര്‍വശങ്ങളില്‍നിന്നായി മുറിച്ചുകളയുകയും ഈന്തപ്പന തടികളില്‍ നിങ്ങളെ ഞാന്‍ ക്രൂശിക്കുകയും ചെയ്യുന്നതാണ്. ഞങ്ങളില്‍ ആരാണ് ഏറ്റവും കഠിനമായതും നീണ്ടുനില്‍ക്കുന്നതുമായ ശിക്ഷ നല്‍കുന്നവന്‍ എന്ന് തീര്‍ച്ചയായും നിങ്ങള്‍ക്ക് മനസ്സിലാകുകയും ചെയ്യും” (വി.ഖു. 20:71). മൂസാനബി(عليهالسلام)യുടെ മുമ്പ് യൂസുഫ് നബി (عليهالسلام)യുടെകാലത്തുതന്നെ ക്രൂശീകരണം നിലനിന്നിരുന്നതായി സൂചിപ്പിക്കുന്ന ഖുര്‍ആന്‍ വചനങ്ങളുമുണ്ട്. യൂസുഫ് നബി (عليهالسلام)യോടൊപ്പം ജയിലിലടക്കപ്പെട്ടയാളുടെ സ്വപ്‌നത്തിന് അദ്ദേഹം നല്‍കിയ വ്യാഖ്യാനത്തെക്കുറിച്ച് പരാമര്‍ശിക്കവെ ഖുര്‍ആന്‍ ഇങ്ങനെ പറയുന്നത്കാണാം: ”ജയിലിലെ രണ്ട് സുഹൃത്തുക്കളേ എന്നാല്‍ നിങ്ങളിലൊരുവന്‍ തന്റെ യജമാനന് വീഞ്ഞ് കുടിപ്പിച്ചുകൊണ്ടിരിക്കും. എന്നാല്‍ മറ്റേയാള്‍ ക്രൂശിക്കപ്പെടും. എന്നിട്ട് അയാളുടെ തലയില്‍നിന്ന് പറവകള്‍ കൊത്തിത്തിന്നും. ഏതൊരു കാര്യത്തെപ്പറ്റി നിങ്ങളിരുവരും വിധി ആരായുന്നുവോ ആ കാര്യം തീരുമാനിക്കപ്പെട്ടുകഴിഞ്ഞിരിക്കുന്നു” (വി.ഖു. 12:41) മൂസാനബി (عليهالسلام)ക്ക് മുമ്പുതന്നെ ഈജിപ്തില്‍ നിലനിന്നിരുന്ന ഒരു ശിക്ഷാമുറയായാണ് ഖുര്‍ആന്‍ ക്രൂശീകരണത്തെ പരിചയപ്പെടുത്തുന്നത് എന്ന് ഈ സൂക്തങ്ങളില്‍നിന്ന് സുതരാം വ്യക്തമാണ്. പൗരാണിക ഈജിപ്തിനെക്കുറിച്ച പഠനഗ്രന്ഥങ്ങളിലൊന്നുംതന്നെ ഫറോവമാരുടെകാലത്ത് ക്രൂശീകരണം നിലനിന്നതായി വ്യക്തമാക്കുന്ന രേഖകള്‍ അവതരിപ്പിക്കുന്നില്ലെന്നത് ശരിയാണ്. എന്നാല്‍ അതുകൊണ്ടുമാത്രം അന്ന് അങ്ങനെയൊരു ശിക്ഷാസമ്പ്രദായം തന്നെ നിലനിന്നിരുന്നില്ലായെന്ന് പറയാനാവില്ല. പൗരാണിക ഈജിപ്തിനെക്കുറിച്ച പഠനത്തിന് അവിടെനിന്ന് ഉല്‍ഖനനം ചെയ്‌തെടുത്ത ശിലാരേഖകളെയും സീലുകളെയും പ്രതിമകളെയുമെല്ലാമാണ് പ്രധാനമായും ഉപയോഗിക്കുന്നത്. ഇവയിൽ ‍നിന്ന് മാത്രമായി ഈജിപ്തിന്റെ ഒരു സമ്പൂര്‍ണ്ണ ചരിത്രം നിര്‍മിക്കാനാവില്ല. ഇവയില്‍ രേഖപ്പെടുത്തപ്പെടാത്ത നിരവധി കാര്യങ്ങളുണ്ടായിരിക്കാം. രേഖപ്പെടുത്തപ്പെട്ട കാര്യങ്ങളില്‍ തന്നെ വായിക്കാന്‍ കഴിയാത്തവയുണ്ടായിരിക്കാം; വായിക്കാന്‍ കഴിഞ്ഞവയില്‍ തന്നെ സ്ഖലിതങ്ങളുണ്ടാകുവാനുള്ള സാധ്യതയും പൂര്‍ണമായി തള്ളിക്കളയാനാവില്ല. അതുകൊണ്ടുതന്നെ പൗരാണിക ഈജിപ്തിനെക്കുറിച്ച പഠനങ്ങള്‍ അവിടെ ക്രൂശീകരണം നിലനിന്നിട്ടില്ലായെന്ന് വ്യക്തമാക്കുന്ന രേഖകള്‍ വെളിപ്പെടുത്താത്തിടത്തോളം അവിടെ ക്രൂശീകരണമുണ്ടായിരുന്നുവെന്ന് വ്യക്തമാക്കുന്ന ഖുര്‍ആനിക പരാമര്‍ശങ്ങള്‍ അബദ്ധമാണെന്ന് പറയാനാകില്ല. ഖുര്‍ആനിക പരാമര്‍ശങ്ങള്‍ക്ക് ഉപോല്‍ബലകമായ രേഖകള്‍ ലഭിച്ചിട്ടില്ലാത്തതുപോലെതന്നെ അതിനെ നിഷേധിക്കുന്ന രേഖകളുമില്ലാത്ത സ്ഥിതിക്ക് ആ പരാമര്‍ശങ്ങളില്‍ അബദ്ധമാരോപിക്കുന്നതിന് യാതൊരു അടിസ്ഥാനവുമില്ല. എന്താണ് ക്രൂശീകരണം? ‘ഓക്‌സ്‌ഫോര്‍ഡ് കംപാനിയന്‍ ടു ദ ബൈബിള്‍’പറയുന്നത് കാണുക: ”വധശിക്ഷയായോ മൃതശരീരത്തെ പ്രദര്‍ശിപ്പിക്കുന്നതിന്നായോ കുരിശിലോ മരത്തിലോ ഒരാളെ ആണിയടിക്കുകയോ ബന്ധിക്കുകയോ ചെയ്യുക”. (Bruce M Metzger Michael D Coogan (Ed), Oxford Companion to the Bible, 1993, Oxford University Press, Oxford & Newyork page 141) ഇന്ന് ക്രൈസ്തവര്‍ മതചിഹ്‌നമായി ഉപയോഗിക്കുന്ന രീതിയിലുള്ളതായിരുന്നില്ല ആദ്യകാലത്തെ കുരിശ്. ഒരു മരത്തടിയില്‍ കൈകളും കാലുകളും അരക്കെട്ടുമെല്ലാം ആണിയില്‍ തറച്ച് ഇഞ്ചിഞ്ചായികൊല്ലുന്ന സമ്പ്രദായമാണ് ക്രൂശീകരണത്തിന്റെ ആദിമരൂപം. പിന്നീട്എക്‌സ് (X) ആകൃതിയില്‍ രണ്ട് തടികള് ‍വെച്ച് അതില്‍ ക്രൂശീകരിക്കുന്നരീതിയുണ്ടായി. അതിനും ശേഷമാണ് ഒരു തടിമരത്തിന്റെ മുകള്‍ഭാഗത്ത് മറ്റൊരു മരക്കഷണം കൂട്ടിവെച്ച് ടി (T) ആകൃതിയിലും ഇന്ന് ക്രൈസ്തവര്‍ മതചിഹ്‌നമായി ഉപയോഗിക്കുന്ന + ആകൃതിയിലുമെല്ലാമുള്ളകുരിശുകളുണ്ടായത്. അതുകൊണ്ടാണ് ‘മരത്തിലോ കുരിശിലോ ആണിയടിച്ചുകൊല്ലുന്നതാണ് ക്രൂശീകരണം‘ എന്ന് ഓക്‌സ്‌ഫോര്‍ഡ് കംപാനിയന്‍ ടു ദി ബൈബിളില്‍ പറഞ്ഞത്. ആണിയടിച്ച് കൊല്ലുകയെന്നകര്‍മ്മമാണ്, അതിനുപയോഗിക്കുന്ന വസ്തുവിന്റെ ആകൃതിയല്ല ക്രൂശീകരണത്തെ അന്വര്‍ത്ഥമാക്കുന്നത് എന്ന് സാരം. മരത്തില്‍ തറച്ചുകൊല്ലുന്ന ഏര്‍പ്പാട് മോശയുടെ കാലത്തും യോസഫിന്റെ കാലത്തുമെല്ലാം നിലനിന്നിരുന്നുവെന്നതിന് ബൈബിള്‍തന്നെ തെളിവുകള്‍ നല്‍കുന്നുണ്ട്. യോസഫിന്റെ സ്വപ്‌നവ്യാഖ്യാനത്തെക്കുറിച്ച് ഉല്‍പത്തി പുസ്തകം പറയുന്നത് ഇങ്ങനെയാണ്: ”യോസഫ് പറഞ്ഞു: ഇതാണ് സ്വപ്‌നത്തിന്റെ വ്യാഖ്യാനം. മൂന്ന് കുട്ട മൂന്ന് ദിവസമാണ്. മൂന്ന്ദിവസത്തിനകം ഫറോവാന്‍ നിന്റെ തലവെട്ടി നിന്നെ കഴുമരത്തില്‍ കെട്ടിത്തൂക്കും. പക്ഷികള്‍ നിന്റെ മാംസം ഭക്ഷിക്കും” (ഉല്‍പത്തി 40:18-19) ഇക്കാര്യം പരാമര്‍ശിക്കുമ്പോള്‍ ഖുര്‍ആന്‍ ‘ക്രൂശിക്കുക‘യെന്നാണ്പറഞ്ഞിട്ടുള്ളതെന്ന കാര്യം പ്രത്യേകം ശ്രദ്ധേയമാണ്. മോശ എഴുതിയതായി കരുതപ്പെടുന്ന ആവര്‍ത്തന പുസ്തകത്തിലുംമരത്തില്‍ തൂക്കിക്കൊല്ലുന്നതിനെക്കുറിച്ച പരാമര്‍ശങ്ങളുണ്ട്: ”വധശിക്ഷഅര്‍ഹിക്കുന്ന കുറ്റം ചെയ്തവനെ വധിച്ച് മരത്തില്‍ തൂക്കിക്കഴിഞ്ഞാല്‍അയാളുടെ ജഡം രാത്രി മുഴുവന്‍ ആ മരത്തില്‍ കിടക്കരുത്. ആ ദിവസംതന്നെഅയാളെ സംസ്‌ക്കരിക്കണം. തൂക്കിക്കൊല്ലപ്പെടുന്നവന്‍ ദൈവത്താല്‍ശപിക്കപ്പെട്ടവനാണ്. നിന്റെ ദൈവമായ കര്‍ത്താവ് നിനക്ക് അവകാശമായിതരുന്ന ദേശം നീ മലിനമാക്കരുത്” (ആവ 21:22-23) മോശക്ക് ശേഷം വന്ന യോശുവയുടെ കാലത്തും ഈ ശിക്ഷാ രീതിനിലനിന്നിരുന്നുവെന്ന് ബൈബിള്‍ വ്യക്തമാക്കുന്നുണ്ട്: ”ആയിയിലെരാജാവിനെ വൈകുന്നേരംവരെ ഒരു മരത്തില്‍ കെട്ടിത്തൂക്കിയിട്ടു.സൂര്യാസ്തമയം ആയപ്പോള്‍ യോശുവയുടെ കല്‍പനപ്രകാരം ശവംമരത്തില്‍നിന്ന് ഇറക്കി; നഗരവാതില്‍ക്കലിട്ടു. അവര്‍ അതിനുമുകളില്‍ ഒരുകല്‍ക്കൂന ഉണ്ടാക്കി. അത് ഇന്നോളം അവിടെയുണ്ട്” (യോശുവ 8:29) ആവര്‍ത്തന പുസ്തകത്തിലെ ‘മരത്തില്‍ തൂക്കപ്പെട്ടവനെ‘ക്കുറിച്ച പരാമര്‍ശങ്ങളെ യേശുവില്‍ ആരോപിക്കപ്പെട്ട ക്രൂശീകരണവുമായി പൗലോസ് ബന്ധിപ്പിക്കുന്നത് കാണുക: ”നമുക്കുവേണ്ടി ശാപവിധേയനായിത്തീര്‍ന്ന ക്രിസ്തു നിയമത്തിന്റെ ശാപത്തില്‍ നിന്ന് നമ്മെ മോചിപ്പിച്ചിരിക്കുന്നു. ‘മരത്തില്‍ തൂങ്ങി മരിക്കുന്നവരെല്ലാം ശപിക്കപ്പെട്ടവരാണ്‘ എന്ന് എഴുതപ്പെട്ടിരിക്കുന്നുവല്ലോ. അങ്ങനെ അബ്രാഹാമിന് ദൈവം നല്‍കിയ അനുഗ്രഹം ക്രിസ്തുയേശുവിലൂടെ വിജാതീയര്‍ക്കും ലഭിക്കാനും തത്ഫലമായി വാഗ്ദാനം ചെയ്യപ്പെട്ട പരിശുദ്ധാത്മാവ് വിശ്വാസം വഴി നമുക്കും ലഭിക്കാനും യേശുക്രിസ്തുഅരുള്‍ചെയ്തു” (ഗലാത്യര്‍ 3:13) അപ്പോസ്തല പ്രവൃത്തികളിലും യേശുവിനെക്കുറിച്ച് പറയുമ്പോള്‍ ‘അവര്‍ അവനെ മരത്തില്‍ തൂക്കിക്കൊന്നു‘ (അപ്പോ 10:39) വെന്നാണ്പറയുന്നത്. ഇതില്‍നിന്നെല്ലാം യോസഫിന്റെ കാലത്ത് നിലനിന്നതും മോശ ആവര്‍ത്തനപുസ്തകത്തില്‍ പറഞ്ഞതും യോശുവ നടപ്പിലാക്കിയതുമെല്ലാം യേശുവിന്റെ കാലത്ത് നിലനിന്നിരുന്ന ക്രൂശീകരണത്തിന്റെ തന്നെ വ്യത്യസ്ത രൂപങ്ങളായിരുന്നുവെന്ന് സുതരാം വ്യക്തമാണ്. ഈ തെളിവുകളുടെ അടിസ്ഥാനത്തില്‍ ഈജിപ്തുകാര്‍ക്കിടയില്‍ ക്രൂശീകരണമെന്ന ശിക്ഷാ സമ്പ്രദായം നിലനിന്നിരുന്നതായി ബൈബിള്‍പണ്ഡിതന്മാര്‍ തന്നെ വ്യക്തമാക്കിയിട്ടുണ്ട്. സ്മിത്തിന്റെ ബൈബിള്‍ ഡിക്ഷ്ണറി പറയുന്നത് കാണുക: ”ഈജിപ്തുകാരുടെയും (ഉല്‍പത്തി 40:19), കാര്‍ത്തേജിനിയന്മാരുടെയും പേര്‍ഷ്യക്കാരുടെയും (എസ്‌തേര്‍ 7:10) അസീറിയക്കാരുടെയും സ്‌കീത്യരുടെയും ഇന്ത്യക്കാരുടെയും ജര്‍മന്‍കാരുടെയും വളരെ ആദ്യകാലം തൊട്ടുതന്നെ ഗ്രീക്കുകാരുടെയും റോമക്കാരുടെയും ഇടയില്‍ ക്രൂശീകരണം ഉപയോഗിക്കപ്പെട്ടിരുന്നു. ആദിമയഹൂദന്മാര്‍ക്ക് ഈ ശിക്ഷാ സമ്പ്രദായം അറിയാമായിരുന്നോയെന്ന വിഷയത്തില്‍ തര്‍ക്കം നിലനില്‍ക്കുന്നുണ്ട്. യഹൂദന്മാര്‍ റോമക്കാരില്‍നിന്നായിരിക്കണം ഈ സമ്പ്രദായം സ്വീകരിച്ചത്. ഇത് ഏറ്റവുംഭീകരമായ മരണരീതിയായി എല്ലാവരും ഒരുപോലെ അംഗീകരിച്ചിരുന്നു” (“Crucifixion” Smith’s Bible Dictionary Online) പുരാതന ഈജിപ്തില്‍ ക്രൂശീകരണം നിലനിന്നതിന് തെളിവുകളില്ലെന്ന് പറഞ്ഞ് ഖുര്‍ആനില്‍ അബദ്ധം ആരോപിക്കുന്നതിന് മുമ്പ് മിഷനറിമാര്‍ സ്വന്തം വേദഗ്രന്ഥം ഒന്ന് മനസ്സിരുത്തി വായിച്ചുനോക്കേണ്ടതായിരുന്നു. യോസഫിന്റെയും മോശയുടെയും കാലത്ത് ക്രൂശീകരണം നിലനിന്നിരുന്നുവെന്ന് പറയുന്നത് ഖുര്‍ആന്‍ മാത്രമല്ല; ബൈബിളും കൂടിയാണ്. ഇരുവേദഗ്രന്ഥങ്ങളും ഒരുപോലെ പറയുന്ന ഇക്കാര്യത്തിന് വിരുദ്ധമായ തെളിവുകള്‍ പുരാതന ഈജിപ്തിനെക്കുറിച്ച പഠനങ്ങള്‍ നല്‍കാത്തിടത്തോളം ഇതില്‍ അബദ്ധമാരോപിക്കുന്നതില്‍ യാതൊരു കഴമ്പുമില്ല.
മുഹമ്മദ് നബി(ﷺ)യുടെ നിശാപ്രയാണത്തെക്കുറിച്ച് പറയുന്നത് ഖുര്‍ആനില്‍ പതിനേഴാം അധ്യായത്തിന്റെ ഒന്നാം വചനത്തിലാണ്. പ്രസ്തുത വചനത്തിന്റെ സാരം ഇങ്ങനെയാണ്: ”തന്റെ ദാസനെ (നബിയെ) ഒരു രാത്രിയില്‍ മസ്ജിദുല്‍ ഹറാമില്‍നിന്ന് മസ്ജിദുല്‍ അഖ്‌സായിലേക്ക്–അതിന്റെ പരിസരം നാം അനുഗൃഹീതമാക്കിയിരിക്കുന്നു–നിശായാത്ര ചെയ്യിച്ചവന്‍ എത്രയോപരിശുദ്ധന്‍. നമ്മുടെ ദൃഷ്ടാന്തങ്ങളില്‍ ചിലത് അദ്ദേഹത്തിന് നാംകാണിച്ചുകൊടുക്കുവാന്‍ വേണ്ടിയത്രെ അത്. തീര്‍ച്ചയായും അവന്‍ (അല്ലാഹു) എല്ലാം കേള്‍ക്കുന്നവനും കാണുന്നവനുമത്രെ” (വി.ഖു: 17:1) ‘ഒരൊറ്റ രാത്രികൊണ്ട് മുഹമ്മദ് നബിയെ മസ്ജിദുല്‍ ഹറാമില്‍നിന്ന് മസ്ജിദുല്‍ അഖ്‌സായിലേക്ക് നിശായാത്ര ചെയ്യിച്ചുവെന്ന് പറയുമ്പോള്‍ ഇങ്ങനെ രണ്ട് സ്ഥലങ്ങള്‍ നിലനില്‍ക്കണമല്ലോ. എന്നാല്‍ മസ്ജിദുല്‍ അഖ്‌സായെന്ന പേരില്‍ ഇന്ന് ജെറുസലേമില്‍ നിലനില്‍ക്കുന്ന കെട്ടിടം നിര്‍മ്മിക്കപ്പെട്ടത് മുഹമ്മദ് നബിക്ക് ശേഷം വര്‍ഷങ്ങള്‍ കഴിഞ്ഞാണ്. സോളമന്‍ നിര്‍മിച്ച ജെറുസലേം ദേവാലയമാണ് ഇവിടെ വിവക്ഷിക്കപ്പെടുന്നതെങ്കില്‍ അത് മുഹമ്മദ് നബിയുടെ കാലത്ത് നിലനിന്നിരുന്നില്ലെന്നുറപ്പാണ്. ക്രിസ്താബ്ദം 70ല്‍ തന്നെ–ഖുര്‍ആനില്‍ പറഞ്ഞ ഇസ്രാഇന് അഞ്ചര നൂറ്റാണ്ടുകള്‍ക്ക് മുമ്പുതന്നെ അത് തകര്‍ക്കപ്പെട്ടിട്ടുണ്ട്. പിന്നെയെങ്ങനെയാണ് ജെറുസലേം ദേവാലയത്തിലേക്ക് മുഹമ്മദ് നബി നിശായാത്ര നടത്തുക? യഹൂദരില്‍നിന്ന് ജെറുസലേം ദേവാലയത്തെക്കുറിച്ച് കേട്ടറിഞ്ഞ മുഹമ്മദ് നബി അത് ഇപ്പോഴും നിലനില്‍ക്കുന്നുവെന്ന് തെറ്റിദ്ധരിച്ചാണ് മസ്ജിദുല്‍ ഹറാമില്‍നിന്ന് മസ്ജിദുല്‍ അഖ്‌സയിലേക്ക് നിശായാത്ര നടത്തിയെന്ന കഥ മെനഞ്ഞെടുത്തത്‘- വിമര്‍ശകരുടെ വാദം ഇങ്ങനെ പോകുന്നു. ഈ വാദങ്ങളില്‍ എത്രത്തോളം കഴമ്പുണ്ടെന്ന് നോക്കുവാന്‍ മസ്ജിദുല്‍ അഖ്‌സാ, മസ്ജിദുല്‍ ഹറാം തുടങ്ങിയ പദങ്ങളുടെ വിവക്ഷയെന്താണെന്ന് പരിശോധിക്കണം. ‘മസ്ജിദ്‘ എന്ന അറബി പദം ‘സജദ‘യെന്ന ധാതുവില്‍നിന്നുണ്ടായതാണ്. ‘സജദ‘യെന്നാല്‍ സാഷ്ടാംഗം ചെയ്യുക എന്നര്‍ത്ഥം. സുജൂദ് അഥവാ സാഷ്ടാംഗം ചെയ്യപ്പെടുന്ന സ്ഥല(ഇസ്മുമകാന്‍)മാണ് ‘മസ്ജിദ്‘. പൊതുവായി ‘ആരാധനാ സ്ഥലം‘ എന്നുംഅര്‍ത്ഥം പറയാം. സര്‍വ്വശക്തനായ സ്രഷ്ടാവിനെ ആരാധിക്കുവാന്‍ വേണ്ടി നിശ്ചയിക്കപ്പെട്ട സ്ഥലത്തെക്കുറിച്ചുകൊണ്ടാണ് ഖുര്‍ആനില്‍ ‘മസ്ജിദ്‘ എന്ന് പ്രയോഗിച്ചിരിക്കുന്നത്. ജെറുസലേം ദേവാലയത്തിന്റെ തകര്‍ച്ചയെ സൂചിപ്പിക്കുന്ന ഖുര്‍ആനിക വചനത്തിലും ആരാധനാലയത്തെക്കുറിക്കാന്‍ ‘മസ്ജിദ്‘ (17:7) എന്നുതന്നെയാണ് പ്രയോഗിച്ചിരിക്കുന്നത് എന്ന കാര്യം പ്രത്യേകം പ്രസ്താവ്യമാണ്. മസ്ജിദ് എന്ന പദത്തിന് ആരാധനാ സ്ഥലം എന്ന്മാത്രമെ അര്‍ത്ഥമുള്ളൂ; ഒരു കെട്ടിടമുണ്ടെങ്കിലും ഇല്ലെങ്കിലും സര്‍വ്വശക്തന് ആരാധനകളര്‍പ്പിക്കപ്പെടുന്ന സ്ഥലത്തെക്കുറിക്കുവാന്‍ മസ്ജിദ് എന്ന്പറയും. ‘മസ്ജിദ്‘ എന്ന പദം ഒരു കെട്ടിടത്തെക്കുറിക്കുന്നില്ല, പ്രത്യുത ചെയ്യുന്ന പ്രവൃത്തിയെ –ആരാധന–മാത്രമാണ് ദ്യോതിപ്പിക്കുന്നത് എന്നര്‍ത്ഥം. സഹീഹുല്‍ ബുഖാരി റിപ്പോര്‍ട്ട് ചെയ്തിട്ടുള്ള ഒരു ഹദീസ് ഇക്കാര്യം കുറെക്കൂടി വ്യക്തമായി മനസ്സിലാക്കിത്തരുന്നു. ജാബിര്‍ (റ)ല്‍നിന്ന് നിവേദനം: തിരുമേനി (ﷺ) അരുളി: എനിക്ക് മുമ്പുള്ളവര്‍ക്ക് നല്‍കാത്ത അഞ്ച് കാര്യങ്ങള്‍ എനിക്ക് അല്ലാഹു നല്‍കിയിരിക്കുന്നു. ഒരു മാസത്തെ വഴിദൂരത്തെ ഭയംകൊണ്ട് ഞാന്‍ സഹായിക്കപ്പെട്ടു. ഭൂമിയെ (സര്‍വ്വവും) എനിക്ക് സാഷ്ടാംഗം ചെയ്യുവാനുള്ള സ്ഥലമായും ശുചീകരിക്കുവാനുള്ള ഒരു വസ്തുവായും അല്ലാഹു അംഗീകരിച്ചുതന്നു. എന്റെ അനുയായികളില്‍ ഏതെങ്കിലും ഒരാള്‍ക്ക് നമസ്‌കാര സമയം എത്തിയാല്‍ (പള്ളിയും വെള്ളവും ഇല്ലെങ്കിലും) അവിടെവെച്ച് അവന്‍ നമസ്‌കരിക്കട്ടെ. ശത്രുക്കളുമായുള്ള യുദ്ധത്തില്‍ പിടിച്ചെടുക്കുന്ന ധനം ഉപയോഗിക്കുവാന്‍ എനിക്ക് അനുമതി നല്‍കിയിരിക്കുന്നു. എനിക്കുമുമ്പ് ആര്‍ക്കും അത് അനുവദിച്ചുകൊടുത്തിരുന്നില്ല. ശുപാര്‍ശ എനിക്ക് അനുവദിച്ചുതന്നു. നബിമാരെ അവരവരുടെ ജനതയിലേക്ക് മാത്രമാണ് മുമ്പ് നിയോഗിച്ചിരുന്നത്. എന്നെ നിയോഗിച്ചിരിക്കുന്നതാകട്ടെ മനുഷ്യരാശിയിലേക്ക് ആകമാനവും” (സഹീഹുല്‍ ബുഖാരി). മുമ്പുള്ള പ്രവാചകന്മാര്‍ക്കൊന്നും നല്‍കപ്പെടാത്ത, മുഹമ്മദ് നബി(ﷺ)ക്ക് മാത്രമായി നല്‍കപ്പെട്ടിട്ടുള്ള അഞ്ച് കാര്യങ്ങളില്‍ രണ്ടാമതായി പറഞ്ഞ കാര്യം ശ്രദ്ധിക്കുക: ”ഭൂമിയെ (സര്‍വ്വവും) എനിക്ക് സാഷ്ടാംഗം ചെയ്യുവാനുള്ള സ്ഥലമായും ശുചീകരിക്കുവാനുള്ള ഒരു വസ്തുവായും അല്ലാഹു അംഗീകരിച്ചുതന്നു. എന്റെ അനുയായികളില്‍ ഏതെങ്കിലും ഒരാള്‍ക്ക് നമസ്‌കാര സമയം എത്തിയാല്‍ (പള്ളിയും വെള്ളവും ഇല്ലെങ്കിലും)അവിടെവെച്ച് അവന്‍ നമസ്‌കരിക്കട്ടെ”. ഇവിടെ ‘സാഷ്ടാംഗം ചെയ്യുവാനുള്ള സ്ഥലം‘ എന്ന് പരിഭാഷപ്പെടുത്തിയിരിക്കുന്നത് ‘മസ്ജിദ്‘ എന്ന പദത്തെയാണ്. മുമ്പുള്ള പ്രവാചകന്മാര്‍ക്കും അനുയായികള്‍ക്കുമെല്ലാം സാഷ്ടാംഗത്തിനായി ചില പ്രത്യേക സ്ഥലങ്ങള്‍ നിര്‍ദ്ദേശിക്കപ്പെട്ടിരുന്നുവെന്നും അവിടെവെച്ചുള്ള ആരാധനകള്‍ മാത്രമെ സ്വീകാര്യമായി പരിഗണിക്കപ്പെട്ടിരുന്നുള്ളൂവെന്നും മുഹമ്മദ് നബിയോടെ ഈ അവസ്ഥയ്ക്ക് മാറ്റമുണ്ടായിയെന്നും അദ്ദേഹത്തിന്റെ സമുദായത്തിന് ഭൂമിയില്‍ എവിടെവെച്ചും ആരാധനകള്‍ക്ക് അനുമതി നല്‍കിയിട്ടുണ്ട് എന്നുമാണല്ലോ ഈ പരാമര്‍ശം വ്യക്തമാക്കുന്നത്. മുഹമ്മദ് നബി (ﷺ)യുടെ സമുദായത്തിന് ഭൂമി മുഴുവന്‍ മസ്ജിദാണെന്ന് പറയുമ്പോള്‍ ഭൂമിയില്‍ എല്ലായിടത്തും ആരാധനകള്‍ക്കായി മന്ദിരമുണ്ടാക്കിയിട്ടുണ്ട് എന്ന് ആരും മനസ്സിലാക്കുന്നില്ല. ഭൂമിയില്‍ എവിടെവെച്ചും സാഷ്ടാംഗം നമിക്കുവാനും ആരാധനകള്‍ അര്‍പ്പിക്കുവാനുമുള്ള സ്വാതന്ത്ര്യം അവര്‍ക്കുണ്ടെന്നാണല്ലോ ഇതിന്നര്‍ത്ഥം. മസ്ജിദ് എന്ന പദം ഒരു കെട്ടിടത്തെക്കുറിക്കുന്നില്ലെന്ന വസ്തുതയാണ് ഇതില്‍നിന്നെല്ലാം വ്യക്തമാകുന്നത്. വിമര്‍ശിക്കപ്പെട്ട സൂക്തത്തില്‍ മസ്ജിദുല്‍ ഹറാമില്‍നിന്ന് മസ്ജിദുല്‍ അഖ്‌സായിലേക്കുള്ള നിശാപ്രയാണത്തെക്കുറിച്ചാണല്ലോ സൂചിപ്പിച്ചിരിക്കുന്നത്. മസ്ജിദുല്‍ അഖ്‌സായെക്കുറിച്ച് മുഹമ്മദ് നബി (ﷺ)തെറ്റിദ്ധരിച്ചുകൊണ്ട് എഴുതിയതാണെന്ന വിമര്‍ശകരുടെ വാദം അംഗീകരിച്ചാല്‍ തന്റെ കണ്‍മുമ്പിലുള്ള കാര്യങ്ങളെക്കുറിച്ചുപോലും അദ്ദേഹം തീരെ ബോധമില്ലാത്തവനായിരുന്നുവെന്ന് പറയേണ്ടിവരും. മക്കയിലെ കഅ്ബാലയത്തിന് ചുറ്റുമുള്ള വിശുദ്ധ ദേവാലയമാണ്മസ്ജിദുല്‍ ഹറാം എന്ന് അറിയപ്പെടുന്നത്. കഅ്ബാലയത്തിന് ചുറ്റുംഇപ്പോള്‍ നിലനില്‍ക്കുന്ന രീതിയിലുള്ള ഒരു കെട്ടിടം നബി (ﷺ)യുടെ നിശായാത്രയുടെ കാലത്ത് ഉണ്ടായിരുന്നതായി ഒരു ചരിത്രരേഖയുമില്ല. എന്നാല്‍ കഅ്ബാലയത്തിന് ചുറ്റുമുള്ള നിര്‍ണിതമായ പ്രദേശങ്ങള്‍ ഇബ്രാഹീം (عليه السلام) നബിയുടെ കാലംമുതല്‍തന്നെ ഹറം എന്ന് വിളിക്കപ്പെട്ടിരുന്നു. കഅ്ബാലയത്തിന് ചുറ്റും നിര്‍ണിതമായ സ്ഥലത്തെഉദ്ദേശിച്ചുകൊണ്ട് ഖുര്‍ആനില്‍തന്നെ മസ്ജിദുല്‍ ഹറാം എന്ന്പറഞ്ഞിട്ടുമുണ്ട്. ”മസ്ജിദുല്‍ ഹറാമിനടുത്തുവെച്ച് നിങ്ങള്‍ അവരോട് യുദ്ധം ചെയ്യരുത്. അവര്‍ നിങ്ങളോട് അവിടെവെച്ച് യുദ്ധം ചെയ്യുന്നതുവരെ” (വി.ഖു. 2:191). ”ഇനി ആര്‍ക്കെങ്കിലും അത് (ബലിമൃഗം) കിട്ടാത്തപക്ഷം ഹജ്ജിനിടയില്‍ മൂന്നുദിവസവും, നിങ്ങള്‍ (നാട്ടില്‍) തിരിച്ചെത്തിയിട്ട് ഏഴുദിവസം ചേര്‍ത്ത് ആകെ 10 ദിവസം നോമ്പ് അനുഷ്ഠിക്കേണ്ടതാണ്. കുടുംബസമേതം മസ്ജിദുല്‍ ഹറാമില്‍ താമസിക്കുന്നവരല്ലാത്തവര്‍ക്കാകുന്നു ഈ വിധി” (വി.ഖു. 2:196). ഇവയില്‍നിന്ന് ഇബ്രാഹീം നബി (عليه السلام) നിര്‍ണയിക്കുകയും മക്കാവിജയത്തിന്റെ ദിവസം നബി (ﷺ) അനുചരനായ തമീമുബ്‌നു ഖുസാഅ(റ)യെ പറഞ്ഞയച്ച് ഉറപ്പിക്കുകയും ചെയ്ത അതിര്‍ത്തിക്കകത്തെ പ്രദേശങ്ങളെ പൊതുവായി ഉദ്ദേശിച്ചുകൊണ്ടാണ് ഖുര്‍ആന്‍ മസ്ജിദുല്‍ ഹറാം എന്ന് പറഞ്ഞിട്ടുള്ളതെന്ന് വ്യക്തമാണ്. മാത്രവുമല്ല, ഇസ്‌റാഇനെക്കുറിച്ച് വിശദീകരിക്കുന്ന ഹദീസുകളില്‍ നിശായാത്രയാരംഭിച്ചത് മസ്ജിദുല്‍ ഹറാമില്‍ നിന്നാണെന്നും ഉമ്മുഹാനീ(റ)യുടെ വീട്ടില്‍നിന്നാണെന്നും പറഞ്ഞിട്ടുമുണ്ട്. കഅ്ബാലയത്തിനടുത്തായിരുന്നു ഉമ്മു ഹാനീ(റ)യുടെ വീട്. ഉമ്മു ഹാനീ(റ)യുടെ ഗൃഹത്തില്‍നിന്ന് ആരംഭിച്ച നിശായാത്രയെക്കുറിച്ച് ഖുര്‍ആനിലും ഹദീസുകളിലുമെല്ലാം മസ്ജിദുല്‍ ഹറാമില്‍നിന്ന് എന്ന് പറഞ്ഞതില്‍നിന്നുതന്നെ മസ്ജിദുല്‍ ഹറാം എന്നതുകൊണ്ട് ഒരു കെട്ടിടമല്ല വിവക്ഷിക്കപ്പെട്ടിരിക്കുന്നത് എന്ന് വ്യക്തമാണ്. വാദത്തിനുവേണ്ടി വിമര്‍ശകരുടെ വാദം അംഗീകരിച്ചാല്‍പോലും മുഹമ്മദ് നബി തന്നെ ഇസ്രാഅ് ആരംഭിച്ചത് മസ്ജിദുല്‍ ഹറാമില്‍ നിന്നാണെന്നും ഉമ്മു ഹാനീ(റ)യുടെ വീട്ടില്‍ നിന്നാണെന്നും പറയുമ്പോള്‍ ഇവിടെ ഒരു പ്രത്യേക കെട്ടിടമല്ല പ്രത്യുത കഅ്ബാലയത്തിന് ചുറ്റുമുള്ള സാഷ്ടാംഗം ചെയ്യപ്പെടുന്ന സ്ഥലം എന്നാണ് അദ്ദേഹം മസ്ജിദുല്‍ ഹറാംകൊണ്ട് വിവക്ഷിച്ചിട്ടുള്ളതെന്ന് സമ്മതിക്കേണ്ടിവരും. മസ്ജിദുല്‍ ഹറാമിനെക്കുറിച്ച പരാമര്‍ശങ്ങളില്‍ ഒരു പ്രത്യേക കെട്ടിടമല്ല ഉദ്ദേശിക്കപ്പെട്ടിരിക്കുന്നതെന്ന വസ്തുത വിമര്‍ശകര്‍ക്കുപോലും അംഗീകരിക്കാതിരിക്കാനാവില്ല എന്നര്‍ത്ഥം. ഇതേപോലെന്നെയാണ് ‘മസ്ജിദുല്‍ അഖ്‌സാ‘യെക്കുറിച്ച പരാമര്‍ശങ്ങളുടെ സ്ഥിതിയും. ‘അങ്ങേ അറ്റത്തെ സാഷ്ടാംഗസ്ഥാനം‘ എന്നാണ് മസ്ജിദുല്‍ അഖ്‌സായെന്നാല്‍ അര്‍ത്ഥം. സോളമന്‍ നിര്‍മിച്ച ദേവാലയത്തെ ബേത്ത്–ഹ–മിഖ്ദാഷ് (Bet ha-Miqdash) എന്നാണ് യഹൂദന്മാര്‍ വിളിക്കുന്നത്. അറബിയില്‍ ‘ബൈത്തുല്‍ മുഖദ്ദസ്‘ എന്ന് പറയുന്ന ഈ ദേവാലയം നിന്നസ്ഥലമാണ് മസ്ജിദുല്‍ അഖ്‌സാ കൊണ്ട് വിവക്ഷിക്കപ്പെടുന്നത്. സോളമന്‍ നിര്‍മിച്ച ദേവാല യത്തിന്റെ അറ്റകുറ്റപണികള്‍ യോശിയാവ് തീര്‍ത്തതായും ക്രിസ്തുവിന് 586 വര്‍ഷങ്ങള്‍ക്ക് മുമ്പ് നെബുക്കദ് നെസര്‍ അത് തകര്‍ത്തതായും അന്‍പത് വര്‍ഷങ്ങള്‍ക്ക് ശേഷം യഹൂദര്‍ വീണ്ടും അത് പണിതതായും നെഹമിയായുടെ നിര്‍ദ്ദേശപ്രകാരം അതിന്റെ മതിലുകളും കോട്ടകളുമെല്ലാം വീണ്ടും പണിതതായുമെല്ലാം നമുക്ക് ബൈബിള്‍ പഴയനിയമത്തില്‍നിന്ന് മനസിലാകുന്നുണ്ട്. ഈ ദേവാലയം പിന്നീട് പൂര്‍ണമായി നശിപ്പിക്കപ്പെട്ടത് ക്രിസ്താബ്ദം 70ല്‍ റോമാക്കാരുടെ ഭരണകാലത്താണ്. സോളമന്റെ കാലംമുതല്‍ ഏകദൈവാരാധന നിര്‍വ്വഹിക്കപ്പെട്ടിരുന്ന സ്ഥലമായിരുന്നു ബൈത്തുല്‍ മുഖദ്ദസ് എന്ന വസ്തുത ബൈബിളില്‍നിന്നുതന്നെ വ്യക്തമാണെന്നര്‍ത്ഥം. ഇസ്രായീല്യര്‍ വിശുദ്ധ സ്ഥലമായി ഗണിച്ചിരുന്ന ബേത്ത്–ഹ–മിഖ്ദാഷിലായിരുന്നു പ്രാര്‍ത്ഥനകള്‍ക്കും ബലിയര്‍പ്പണത്തിനും മറ്റ് കൂദാശകള്‍ക്കുമെല്ലാം ജനം ഒരുമിച്ച് കൂടിയിരുന്നത്. അവിടെ വെച്ചായിരുന്നു യാഹ്‌വെക്ക് മുമ്പില്‍ യഹൂദ പുരോഹിതന്മാര്‍ സാഷ്ടാംഗം നമിച്ചിരുന്നത്. ജറുസലേം ദേവാലയത്തിലെ ആരാധനയെക്കുറിച്ച് യഹൂദവിജ്ഞാന കോശം പറയുന്നത് കാണുക: ”കല്‍ക്കരിക്കഷ്ണങ്ങള്‍ പെറുക്കിയെടുത്ത പുരോഹിതന്‍ വിശുദ്ധസ്ഥലത്ത് പ്രവേശിച്ച് ജ്വലിച്ചുകൊണ്ടിരിക്കുന്ന അള്‍ത്താരയിലേക്ക് അവ എറിയുകയും സ്വയം സാഷ്ടാംഗം നമിക്കുകയും ചെയ്ത ശേഷം അവിടെ നിന്ന് പിരിയുന്നു. പിന്നെ നറുക്കെടുപ്പിലൂടെ സുഗന്ധം പുകയ്ക്കുവാനായി തെരഞ്ഞെടുക്കപ്പെട്ട പുരോഹിതന്‍ തന്റെ കയ്യില്‍ കുന്തിരിക്കത്തിന്റെ താലവുമായി കടന്നുവരുന്നു. കര്‍മ്മത്തിന് ഉത്തരവാദപ്പെടുത്തപ്പെട്ട പുരോഹിതനും അയാളെ അനുഗമിക്കുന്നു. ഈ പുരോഹിതന്റെ നിര്‍ദ്ദേശമനുസരിച്ചാണ് അനുഷ്ഠാനങ്ങള്‍ നടക്കുന്നത്. അയാള്‍ ‘പുകയ്ക്കുക‘യെന്ന് നിര്‍ദ്ദേശിക്കുമ്പോള്‍ മാത്രമെ മറ്റേയാള്‍ പുകയ്ക്കുകയുള്ളൂ. ഹാളിനും അള്‍ത്താരക്കുമിടയിലുള്ള സ്ഥലത്തുനിന്ന് ജനം നീങ്ങിക്കഴിഞ്ഞാല്‍ ഔദ്യോഗിക കൃത്യനിര്‍വ്വഹണത്തിനായി നിയോഗിക്കപ്പെട്ട പുരോഹിതന്‍ സുഗന്ധം പുകയ്ക്കുകയും സ്വയം സാഷ്ടാംഗം നമിച്ച് പിരിയുകയും ചെയ്യുന്നു. വിശുദ്ധ സ്ഥലത്ത് സുഗന്ധം പുകയ്ക്കല്‍ നടന്നുകൊണ്ടിരിക്കുമ്പോള്‍ ദേവാലയത്തിനകത്തും പുറത്തും കൂടിയിരിക്കുന്ന ജനം പ്രാര്‍ത്ഥനയില്‍ മുഴുകുന്നു. സുഗന്ധം പുകച്ചപുരോഹിതന്‍ പോയതിനുശേഷം പുരോഹിതന്മാരെല്ലാം വരിവരിയായി വിശുദ്ധ സ്ഥലത്ത് പ്രവേശിക്കുകയും സ്വയം സാഷ്ടാംഗം നമിച്ചശേഷം പുറത്തുപോവുകയും ചെയ്യുന്നു”. (“Temple” Encyclopaedia Judaica: CD Rom Edition: 1997, Judaica Multimedia (Israel) Limited) പ്രധാനികളായ പല റബ്ബിമാരും യെരൂശലേം ദേവാലയത്തിനകത്തുവെച്ച് മാത്രമെ സാഷ്ടാംഗം നമിക്കാന്‍ പാടുള്ളൂവെന്ന് വാദിച്ചിരുന്നുവെന്ന യഹൂദ വിജ്ഞാനകോശത്തിന്റെ പ്രസ്താവന ശ്രദ്ധേയമാണ് (Ibid). മുന്‍ പ്രവാചകന്മാര്‍ക്കെല്ലാം ചില പ്രത്യേക സ്ഥലങ്ങള്‍ മസ്ജിദുകളായി (സാഷ്ടാംഗസ്ഥലം) നിശ്ചയിച്ചിരുന്നുവെന്ന നടേസൂചിപ്പിക്കപ്പെട്ട ഹദീസിലെ പ്രസ്താവന ഇതോട് ചേര്‍ത്ത് വായിക്കുക. മസ്ജിദുല്‍ അഖ്‌സയിലേക്കാണ് നിശാ പ്രയാണമുണ്ടായതെന്ന് ഖുര്‍ആന്‍ പറയുമ്പോള്‍ ഒരു പ്രത്യേക കെട്ടിടമല്ല വിവക്ഷിക്കുന്നതെന്നും ‘അങ്ങേയറ്റത്തെ സാഷ്ടാംഗ സ്ഥാനമായ ബൈത്തുല്‍ മുഖദ്ദസ്സ് സ്ഥിതിചെയ്തിരുന്ന സ്ഥലമാണെന്നുമുള്ള വസ്തുതകള്‍ വ്യക്തമാക്കുന്നതിനു വേണ്ടിയാണ് ഇക്കാര്യങ്ങള്‍ സൂചിപ്പിച്ചത്. ഇസ്രായീല്യര്‍ക്ക് സാഷ്ടാംഗത്തിനുവേണ്ടി നിശ്ചയിക്കപ്പെട്ടിരുന്ന സ്ഥലമായിരുന്നു ബൈത്തുല്‍ മുഖദ്ദസ് സ്ഥിതിചെയ്തിരുന്ന മസ്ജിദുല്‍ അഖ്‌സായെന്നും അത് ഒരു പ്രത്യേക കെട്ടിടമെന്ന നിലയ്ക്കല്ല ഖുര്‍ആന്‍ പറഞ്ഞിട്ടുള്ളതെന്നുമുള്ള വിശദീകരണത്തെ ഖണ്ഡിക്കുവാന്‍വേണ്ടി സഹീഹുല്‍ ബുഖാരിയിലുള്ള ഒരു ഹദീസ് വിമര്‍ശകര്‍ ഉദ്ധരിക്കാറുണ്ട്. പ്രസ്തുത ഹദീസ് ഇങ്ങനെയാണ്: ”അബൂദര്‍റ് (റ)നിവേദനം: ഞാന്‍ ചോദിച്ചു. ദൈവദൂതരേ ആദ്യമായി ഭൂമിയില്‍ സ്ഥാപിതമായ പള്ളിയേതാണ്? നബി (ﷺ) അരുളി: മസ്ജിദുല്‍ഹറാം. പിന്നീട് ഏത് പള്ളിയാണെന്ന് ഞാന്‍ ചോദിച്ചു. നബി (ﷺ) അരുളി: മസ്ജിദുല്‍ അഖ്‌സാ. എത്രകൊല്ലം ഇടവിട്ടാണ് ഇവ രണ്ടും സ്ഥാപിതമായത്? നബി (ﷺ) അരുളി: നാല്‍പത് കൊല്ലം ഇടവിട്ട്. ഇനി എവിടെവെച്ചാണോ നമസ്‌ക്കാര സമയമായത് അവിടെവെച്ച് നീ നമസ്‌കരിച്ചുകൊള്ളുക. തീര്‍ച്ചയായും അതിലാണ് പുണ്യമിരിക്കുന്നത്. (സഹീഹുല്‍ ബുഖാരി) ഈ ഹദീസില്‍ പറഞ്ഞിരിക്കുന്നത് മസ്ജിദുല്‍ അഖ്‌സായുടെ നിര്‍മ്മിതിയെക്കുറിച്ചാണെന്നും അതുകൊണ്ടുതന്നെ നിര്‍മ്മിക്കപ്പെട്ട ഒരുകെട്ടിടമാണ് മസ്ജിദുല്‍ അഖ്‌സാകൊണ്ട് വിവക്ഷിക്കപ്പെടുന്ന തെന്നുമാണ് വിമര്‍ശകരുടെ വാദം. അബ്രഹാം നിര്‍മിച്ചതെന്ന് കരു തപ്പെടുന്ന കഅ്ബയുടെയും സോളമന്‍ നിര്‍മിച്ച ദേവാലയത്തിന്റെയും നിര്‍മാണത്തിനിടയ്ക്ക് നാല്‍പതുവര്‍ഷത്തെ ഇടവേള മാത്രമാണുള്ളതെന്ന് പറഞ്ഞ മുഹമ്മദ് നബി (ﷺ)യെ പരിഹസിക്കുവാനും ചില വിമര്‍ശകര്‍ ഒരുമ്പെടുന്നുണ്ട്. ഹദീസിനെക്കുറിച്ച വിമര്‍ശനങ്ങള്‍ക്ക് പ്രഗല്‍ഭരായ ഹദീസ് പണ്ഡിതന്മാര്‍തന്നെ മറുപടി പറഞ്ഞിട്ടുണ്ട്. പ്രസ്തുത മറുപടി ഇങ്ങനെ സംക്ഷേപിക്കാം. ഒന്ന്) ഇബ്രാഹീം നബി (عليه السلام)യല്ല മസ്ജിദുല്‍ ഹറാമിന്റെ സ്ഥാപകന്‍. അദ്ദേഹത്തിന് മുമ്പുതന്നെ അത് നിലനിന്നിരുന്നുവെന്ന് ഖുര്‍ആനില്‍ നിന്നുതന്നെ വ്യക്തമാവുന്നുണ്ട്. ”ഞങ്ങളുടെ രക്ഷിതാവേ, എന്റെ സന്തതികളില്‍നിന്ന് (ചിലരെ)കൃഷിയൊന്നുമില്ലാത്ത ഒരു താഴ്‌വരയില്‍, നിന്റെ പവിത്രമായ ഭവനത്തിന്റെ (കഅ്ബയുടെ) അടുത്ത് ഞാനിതാ താമസിപ്പിച്ചിരിക്കുന്നു” (വി.ഖു. 14:37) രണ്ട്) ഏകദൈവാരാധനയ്ക്കുവേണ്ടി ഭൂമിയില്‍ സ്ഥാപിക്കപ്പെട്ട ആദ്യത്തെ കേന്ദ്രമാണ് കഅ്ബ. ”തീര്‍ച്ചയായും മനുഷ്യര്‍ക്കുവേണ്ടി സ്ഥാപിക്കപ്പെട്ട ഒന്നാമത്തെ ആരാധനാ മന്ദിരം ബക്കയിലുള്ളതത്രെ. (അത്) അനുഗൃഹീതമായും ലോകര്‍ക്ക് മാര്‍ഗദര്‍ശകമായും (നിലകൊള്ളുന്നു)” (വി.ഖു. 3:96) ആദിമനുഷ്യനായ ആദമിന്റെ കാലംമുതല്‍ തന്നെ ഈ ആരാധനാ കേന്ദ്രമുണ്ടായിരുന്നു. ഇബ്രാഹീം നബി പ്രസ്തുത കേന്ദ്രത്തില്‍ ഒരു ആലയം പണിയുകയാണ് ചെയ്തത്. മൂന്ന്) ബൈത്തുല്‍ മുഖദ്ദസ് സ്ഥാപിച്ചത് സുലൈമാനാ(അ)ണെന്ന് ഖുര്‍ആനിലോ ഹദീസുകളിലോ കാണുന്നില്ല. ആദമിന്റെയോ പുത്രന്മാരുടെയോ കാലത്ത്–കഅ്ബാ നിര്‍മാണത്തിന് നാല്‍പത് വര്‍ഷങ്ങള്‍ക്ക് ശേഷമായിരിക്കണം ഈ കേന്ദ്രം സ്ഥാപിക്കപ്പെട്ടത്. നാല്) ഇബ്രാഹീം നബി (عليه السلام)ക്ക് മുമ്പുതന്നെ കഅ്ബാലയമുണ്ടായിരുന്നുവെന്നതുപോലെ സുലൈമാന്‍ നബി (عليه السلام)ക്കുമുമ്പും മസ്ജിദുല്‍ അഖ്‌സയുണ്ടായിരുന്നു. ഇബ്രാഹിം നബിയും സുലൈമാന്‍ നബിയും പ്രസ്തുത സ്ഥലങ്ങള്‍ പുനരുജ്ജീവിപ്പിക്കുകയും അവിടെ ദേവാലയങ്ങള്‍ പണിയുകയുമാണ് ചെയ്തത്. സോളമന് മുമ്പുതന്നെ ജെറുസലേമില്‍ ഒരു വിശുദ്ധ സ്ഥലമുണ്ടായിരുന്നുവെന്നതിലേക്ക് ബൈബിള്‍ പഴയനിയമം സൂചനകള്‍ നല്‍കുന്നുണ്ട്. ജെറുസലേം നഗരത്തിലെ ഒരു പ്രധാന കേന്ദ്രമായ മോരിയാ മലയെക്കുറിച്ച് ബൈബിള്‍ നിഘണ്ടു എഴുതുന്നത് കാണുക: ”മോരിയാ മല–ഓഫാല്‍ കുന്നിന് വടക്കാണ് മോരിയാ മല സ്ഥിതിചെയ്തിരുന്നത്. പുരാതന കാലംമുതല്‍ ഇത് കാനന്യര്‍ക്ക് ഒരു വിശുദ്ധസ്ഥലമായിരുന്നു. അബ്രഹാം തന്റെ മകനായ ഇസ്ഹാഖിനെ ബലികഴിക്കുന്നതിന് പോയതും ഇവിടെതന്നെ. ഉല്‍പത്തി 22: 8 (റവ. എ.സി.ക്ലെയിറ്റണ്‍: ബൈബിള്‍ നിഘണ്ടു. പേജ് 447). ആദാമിന്റെ കാലംമുതല്‍ ആദരിക്കപ്പെട്ട ഒരു ആരാധനാസ്ഥലമായതുകൊണ്ടാകാം ഈ സ്ഥലം പ്രത്യേകമായി പരിഗണിക്കപ്പെട്ടത്. ഇതുകൊണ്ടുതന്നെയാവണം സോളമന്‍ അവിടെത്തന്നെ ഒരു ദേവാലയം സ്ഥാപിച്ചത്. നടേ ഉദ്ധരിക്കപ്പെട്ട ഹദീസില്‍ ഉണ്ടെന്ന് വിമര്‍ശകര്‍ ഉന്നയിച്ച അബദ്ധം യഥാര്‍ത്ഥത്തില്‍ അവരുടെ മൂഢസങ്കല്‍പങ്ങളില്‍നിന്ന് ഉയിര്‍ക്കൊണ്ടതാണെന്ന സത്യമാണ് ഇവിടെ വ്യക്തമാകുന്നത്. ഭൂമിയില്‍ മാനവന്‍ കാല് കുത്തിയതിനുശേഷം ആദ്യമായി നിര്‍ണയിക്കപ്പെട്ട ആരാധനാസ്ഥലമായ മസ്ജിദുല്‍ ഹറാമില്‍നിന്ന് രണ്ടാമതായി ഉണ്ടാക്കിയ മസ്ജിദുല്‍ അഖ്‌സായിലേക്കാണ് മുഹമ്മദ് നബി (ﷺ) നിശാപ്രയാണം നടത്തിയത്. പ്രത്യേകമായി അനുഗ്രഹിക്കപ്പെട്ട ഒരു പ്രദേശത്തുനിന്ന് അതേപോലെ തന്നെയുള്ള മറ്റൊരു സ്ഥലത്തേക്കുള്ള യാത്രയാണിത്; ഒരു കെട്ടിടത്തില്‍നിന്ന് മറ്റൊരു കെട്ടിടത്തിലേക്കുള്ള യാത്രയല്ല. ഖുര്‍ആനില്‍ അബദ്ധങ്ങള്‍ ചികയുന്നവര്‍ക്കാണ്, ഖുര്‍ആനിനല്ല യഥാര്‍ത്ഥത്തില്‍ തെറ്റ്പറ്റുന്നത് എന്ന വസ്തുതയാണ് ഇവിടെയും നാം കാണുന്നത്.
ഖുര്‍ആനിലെ പത്തൊന്‍പതാം അധ്യായമായ സൂറത്തു മറിയം തുടങ്ങുന്നതു തന്നെ സകരിയ്യാ (عليه السلام)യുടെ വൃത്താന്തവുമായിക്കൊണ്ടാണ്. വാര്‍ധക്യകാലത്ത് വന്ധ്യയായ ഭാര്യയോടൊപ്പം ജീവിക്കുന്ന സകരിയ്യാ (عليه السلام)യുടെ ഒരു അനന്തരാവകാശിക്കുവേണ്ടിയുള്ള പ്രാര്‍ത്ഥനയും പ്രസ്തുത പ്രാര്‍ത്ഥനക്കുള്ള ഉത്തരമായി ഒരു ആണ്‍കുഞ്ഞുണ്ടായ കഥയുമെല്ലാം ഖുര്‍ആന്‍ വിവരിക്കുന്നുണ്ട്. സകരിയ്യായുടെ പ്രാര്‍ത്ഥനയ്ക്കുള്ള ഉത്തരമെന്നോണം സര്‍വ്വശക്തന്‍ അദ്ദേഹത്തിന് നല്‍കിയ വാഗ്ദാനം ഏഴാം വചനത്തില്‍ പറയുന്നുണ്ട്. അതിന്റെ സാരം ഇങ്ങനെയാണ്: ”ഹേ സക്കരിയാ, തീര്‍ച്ചയായും നിനക്ക് നാം ഒരു ആണ്‍കുട്ടിയെ പറ്റി സന്തോഷവാര്‍ത്ത അറിയിക്കുന്നു. അവന്റെ പേര്‍ യഹ്‌യാ എന്നാകുന്നു. മുമ്പ് നാം ആരെയും അവന്റെ പേരുള്ളവരാക്കിയിട്ടില്ല.” (വി.ഖു.19:7) ഈ വചനത്തില്‍ ”മുമ്പ് നാം ആരെയും അവന്റെ പേരുള്ളവരാക്കിയിട്ടില്ല”യെന്ന് യഹ്‌യാ (عليه السلام)യെക്കുറിച്ച് പറഞ്ഞത് ചരിത്രപരമായി അബദ്ധമാണെന്നാണ് ആരോപണം. ‘യോഹന്നാന്‍ സ്‌നാപക‘ന് അറബിയില്‍ പറയുന്ന പേരാണ് യഹ്‌യായെന്ന് പല ഖുര്‍ആന്‍ വ്യാഖ്യാതാക്കളും വ്യക്തമാക്കിയിട്ടുണ്ട്. ഇംഗ്ലീഷില്‍ പുറത്തിറങ്ങിയ ചിലഖുര്‍ആന്‍ പരിഭാഷാ ഗ്രന്ഥങ്ങളില്‍ യഹ്‌യായെന്നതിന് പകരമായി ജോണ്‍ (John) എന്നെഴുതുകയും മറ്റുചിലവയില്‍ യഹ്‌യായെന്നെഴുതി ജോണ്‍ എന്ന് ബ്രാക്കറ്റില്‍ ചേര്‍ക്കുകയും ചെയ്തിട്ടുണ്ട്. അപ്പോള്‍ യോഹന്നാന്‍ സ്‌നാപകനുമുമ്പ് യോഹന്നാന്‍ എന്ന പേരുള്ളവരായി ആരുംതന്നെ ജീവിച്ചിരുന്നില്ല എന്നാണ് ഈ സൂക്തത്തില്‍ പറഞ്ഞതെന്ന് വരുന്നു. പഴയനിയമത്തില്‍തന്നെ ഇരുപത്തിയേഴ് പ്രാവശ്യം യോഹന്നാന്‍ എന്ന നാമം പ്രയോഗിക്കപ്പെട്ടിട്ടുണ്ട്. പഴയ നിയമത്തിലെ യോഹന്നാന്‍മാരൊന്നും തന്നെ അത്ര പ്രസിദ്ധരല്ലാത്തതിനാല്‍ മുഹമ്മദി(ﷺ)ന് അവരെക്കുറിച്ച് അറിയുമായിരുന്നില്ല. അതുകൊണ്ട് വന്നുഭവിച്ച അബദ്ധമാണിത്‘: ഖുര്‍ആന്‍വിമര്‍ശകരുടെ വാദം പോകുന്നത് ഇങ്ങനെയാണ്. ഈ വിമര്‍ശനം പ്രധാനമായും ഒരു പദത്തെ ചുറ്റിപ്പറ്റിയുള്ളതാണ്. ‘യഹ്‌യാ‘യെന്നതാണ് ആ പദം. യോഹന്നാന്‍ സ്‌നാപകനെക്കുറിക്കാന്‍ ഖുര്‍ആനില്‍ പ്രയോഗിക്കപ്പെട്ട പദമാണത്. പദോല്‍പത്തിയെക്കുറിച്ച് സൂക്ഷ്മമായി ഗവേഷണം നടത്താത്ത വ്യാഖ്യാതാക്കള്‍ യഹ്‌യായെന്ന പദത്തിന് പകരമായും തത്തുല്യമായും യോഹന്നാന്‍ എന്ന് പ്രയോഗിച്ചിട്ടുണ്ടെന്നത് നേരാണ്. ഖുര്‍ആനില്‍ പറഞ്ഞിരിക്കുന്നത് ”മുമ്പ് നാം ആരെയും അവന്റെ പേര് (യഹ്‌യാ) ഉള്ളവരാക്കിയിട്ടില്ല”യെന്നാണ്. ഖുര്‍ആനില്‍ മുമ്പ് ആര്‍ക്കുമുണ്ടായിരുന്നില്ലെന്ന് പറയുന്നത് ‘യഹ്‌യാ‘യെന്ന നാമമാണ്; യോഹന്നാന്‍ എന്ന പേരല്ല. യഹ്‌യ=യോഹന്നാന്‍ എന്ന് കരുതിയ വ്യാഖ്യാതാക്കളാണ് ഖുര്‍ആനില്‍ പറഞ്ഞത് യോഹന്നാന്‍ എന്നാണെന്ന് വരുത്തിത്തീര്‍ത്തത്. ബൈബിള്‍ പഴയ നിയമത്തില്‍ പലതവണ യോഹന്നാന്‍ എന്ന പേര് ഉപയോഗിക്കപ്പെട്ടിട്ടുണ്ടെന്നത് നേരാണ്. ”യഹ്‌യാ”യെന്നല്ല അവിടെയൊന്നും പ്രയോഗിച്ചിരിക്കുന്നത് എന്ന കാര്യം പ്രത്യേകം ശ്രദ്ധേയമാണ്. അറബി ബൈബിളില്‍ നിന്നുള്ള ഏതാനും ഉദ്ധരണികള്‍കാണുക: ”യഹ്‌യാ”യെന്ന അറബി പദത്തിന് തത്തുല്യമായ ഹിബ്രു പദമാണോ ‘യോഹന്നാന്‍’ എന്നാണ് നാം ആദ്യമായി പരിശോധിക്കേണ്ടത്. ഇവ്വിഷയകമായ പ്രാഥമിക പരിശോധനയ്ക്ക് നാം അറബി ബൈബിള്‍ പരിശോധിച്ചാല്‍ മതിയാവും. 1 രാജാക്കന്മാര്‍ 25:23, 1 ദിനവൃത്താന്തരം 3:15, 1ദിനവൃത്താന്തം 3:24, എസ്രാ 8:12 തുടങ്ങിയ പഴയ നിയമ ഉദ്ധരണികളില്‍ യോഹന്നാനെക്കുറിച്ച് പറയുന്നുണ്ട്. ഇവിടെയെല്ലാം അറബി ബൈബിളില്‍ യൂഹന്നായെന്നാണ് പറഞ്ഞിരിക്കുന്നത് ”യോഹന്നാന്‍ സുവിശേഷം” എന്നതലക്കെട്ട് അറബി ബൈബിളില്‍ ‘ബിശാറത്തു യൂഹന്നാ‘ () യെന്നാണ്. അറബിപുതിയ നിയമത്തില്‍ യോഹന്നാന്‍ സ്‌നാപകനെയും യേശു ശിഷ്യനായ യോഹന്നാനെയുമെല്ലാം ‘യൂഹന്നാ‘യെന്നു തന്നെയാണ് വിളിച്ചിരിക്കുന്നത്; എവിടെയും ‘യഹ്‌യാ‘യെന്ന് കാണുന്നില്ല. ‘യോഹന്നാന്‍’ എന്ന ഹിബ്രു ശബ്ദത്തിന് തത്തുല്യമായ അറബി പദമായിരുന്നു ‘യഹ്‌യാ‘യെങ്കില്‍ അറബി ബൈബിളില്‍ യോഹന്നാന്‍ എന്ന പദത്തിന് പകരമായി യഹ്‌യായെന്ന് പ്രയോഗിക്കുമായിരുന്നുവെന്ന് തീര്‍ച്ചയാണ്. സത്യത്തില്‍, യഹ്‌യാ, യോഹന്നാന്‍ എന്നിവ തികച്ചും വ്യത്യസ്തങ്ങളായ രണ്ട് നാമങ്ങളാണ്. യോഹന്നാന്‍ എന്ന ഹിബ്രുപദത്തിനര്‍ത്ഥം ‘യഹോവ കാരുണ്യം ചെയ്തിരിക്കുന്നു” (Jehovah has graced)എന്നാണ്. രണ്ട് പദങ്ങള്‍ ചേര്‍ന്നുണ്ടായ ഒരു നാമമാണ് യോഹന്നാന്‍. യൂ+ഹന്നാന്‍. യഹോവയുടെ ചുരുക്കമായാണ് ‘യൂ‘യെന്ന് പ്രയോഗിച്ചിരിക്കുന്നത്. ‘ഹന്നാന്‍ എന്ന ഹിബ്രു പദം ‘ഹനാന്‍’ എന്ന അരമായിക് മൂലത്തില്‍ നിന്നുണ്ടായതാണ്. ‘അനുകമ്പ‘യെന്നാണ് അര്‍ത്ഥം. ‘യഹോവ അനുകമ്പയുള്ളവനായിരിക്കുന്നു” എന്നോ ”യഹോവയുടെഅനുകമ്പ” എന്നോ ആണ് യോഹന്നാന്‍ എന്ന പദത്തിന്റെ മൂലാര്‍ത്ഥം. എന്നാല്‍ ‘യഹ്‌യാ‘യെന്ന അറബി പദമുണ്ടായിരിക്കുന്നത് ‘ഹയാ‘ എന്ന മൂലത്തില്‍നിന്നാണ്. ഈ പദത്തിന് രണ്ട് അര്‍ത്ഥമുണ്ട്. ഒന്ന്‘അല്‍ഹയാത്തി‘ല്‍നിന്ന് നിര്‍ധരിക്കപ്പെട്ടത്. ‘ജീവന്‍’ എന്നര്‍ത്ഥം. മറ്റൊന്ന് ‘അല്‍-ഹയാഇ‘ല്‍ നിന്നുള്ളത്. ‘നാണം‘ എന്ന് സാരം ‘യഹ്‌യാ‘യുടെ ഉല്‍പത്തി ഇവ രണ്ടില്‍ ഏതില്‍ നിന്നായിരുന്നാലും യോഹന്നാന്‍ എന്ന ഹിബ്രു പദവുമായി യാതൊരു ബന്ധവും ഇതിനില്ലെന്ന് വ്യക്തമാണ്. രണ്ടിന്റെയും മൂലങ്ങള്‍ വ്യത്യസ്തങ്ങളാണ്; അര്‍ത്ഥങ്ങള്‍ തമ്മില്‍ യാതൊരു വിധസാമ്യവുമില്ലതാനും. യേശുവിന് തൊട്ടുമുമ്പ് വന്ന സകരിയ്യായുടെ പുത്രനെയാണ് ഖുര്‍ആന്‍ ‘യഹ്‌യാ‘യെന്ന് വിളിക്കുന്നത്. സകരിയ്യയുടെയും എലിസബത്തിന്റെയും പുത്രനാണ് ബൈബിളിലെ യോഹന്നാന്‍ സ്‌നാപകന്‍. എന്നാല്‍ ‘യഹ്‌യാ‘യെന്ന പദവും ‘യോഹന്നാന്‍’ എന്ന പദവും തമ്മില്‍ യാതൊരുവിധ ബന്ധവുമില്ല. രണ്ടും രണ്ട് മൂലങ്ങളില്‍ നിന്നുണ്ടായവ; രണ്ട് അര്‍ത്ഥങ്ങളുള്‍ക്കൊള്ളുന്നവ. ഇതെങ്ങനെ സംഭവിച്ചു? ഒരാളുടെ തന്നെ രണ്ട് നാമങ്ങളാണോ യഹ്‌യായും യോഹന്നാനും? അതല്ല വിമര്‍ശകര്‍ ആരോപിക്കുന്നതുപോലെ മുഹമ്മദ് നബി(ﷺ)ക്ക് പറ്റിയ ഒരു കൈപ്പിഴയാണോ ഇത്? ഈ ചോദ്യങ്ങള്‍ക്ക് വസ്തുനിഷ്ഠമായ ഉത്തരം ലഭിക്കണമെങ്കില്‍ ഖുര്‍ആനും ബൈബിളുമല്ലാത്ത മറ്റുവല്ല രേഖകളും സ്‌നാപകയോഹന്നാനെക്കുറിച്ച് പറയുന്നതായി നിലനില്‍ക്കുന്നുണ്ടോയെന്ന് പരിശോധിക്കണം. അങ്ങനെ വല്ല രേഖകളുമുണ്ടെങ്കില്‍ അവ ഇക്കാര്യത്തില്‍നല്‍കുന്ന അറിവ് ഏറെ പ്രധാനപ്പെട്ടതാണ്. യോഹന്നാനോ യഹ്‌യയോ എന്താണ് യഥാര്‍ത്ഥ നാമമെന്ന് കണ്ടുപിടിക്കുന്നതിനുവേണ്ടി മാത്രമല്ല പ്രസ്തുത രേഖകള്‍ പ്രയോജനപ്രദമാവുക. അതുവഴി ഏത് ഗ്രന്ഥമാണ് കൃത്യവും സൂക്ഷ്മവുമായ പരാമര്‍ശങ്ങള്‍ നടത്തുന്നതെന്ന് മനസ്സിലാക്കുവാനും ഏതിനാണ് അപ്രമാദിത്വമുള്ളതെന്ന് വ്യക്തമായി അറിയുവാനും കഴിയും. യോഹന്നാന്‍ സ്‌നാപകനെക്കുറിച്ച്, അദ്ദേഹം ഒരു വിശുദ്ധപുരുഷനായിരുന്നുവെന്ന് വിശ്വസിക്കുന്ന ക്രൈസ്തവരും മുസ്‌ലിംകളുമല്ലാത്ത മറ്റേതെങ്കിലും വിഭാഗങ്ങളുണ്ടോയെന്ന അന്വേഷണം പ്രസക്തമാണ്. അങ്ങനെയൊന്നുണ്ടെങ്കില്‍ ഇക്കാര്യത്തില്‍ ഖുര്‍ആനും ബൈബിളുമല്ലാത്ത ഒരു സ്രോതസ്സായി അവരുടെ ഗ്രന്ഥങ്ങളോ രേഖകളോ സ്വീകരിക്കുവാന്‍ പറ്റുമെന്ന കാര്യത്തില്‍ സംശയമൊന്നുമില്ല. പക്ഷെ,അങ്ങനെയെന്തെങ്കിലുമുണ്ടോ? യോഹന്നാന്‍ സ്‌നാപകനെ പിന്തുടരുന്നവരാണ് തങ്ങളെന്നും അദ്ദേഹം അന്തിമ പ്രവാചകനായിരുന്നുവെന്നും അവകാശപ്പെടുന്ന ഒരു വിഭാഗം ഇറാഖിലും ഇറാനിലും ഇന്നുമുണ്ട്. പോര്‍ച്ചുഗീസ് ക്രിസ്ത്യന്‍ മിഷനറിമാര്‍ ഇവരെ വിളിച്ചത് ‘യോഹന്നാന്‍ സ്‌നാപകന്റെ ക്രിസ്ത്യാനികള്‍’ (Christians of John the Baptist) എന്നായിരുന്നു. ഏകദൈവാരാധകരായ ഇവരുടെ പ്രധാനപ്പെട്ട ഒരു ആചാരമാണ് ജ്ഞാനസ്‌നാനം (Baptism). തങ്ങളുടെ മതത്തെയും വര്‍ഗത്തെയും കുറിക്കുവാന്‍വേണ്ടി മന്‍ഡായി (Mandai) എന്നും മതവിശ്വാസികളെ സൂചിപ്പിക്കുവാന്‍ മാന്‍ഡിയന്മാര്‍ (Mandaens) എന്നുമാണ് അവര്‍ ഉപയോഗിക്കുന്നത്. ഇസ്‌ലാമിനോട് സമാനമായ ഒട്ടനവധി വിശ്വാസാചാരങ്ങള്‍ മാന്‍ഡിയന്‍മാര്‍ക്കുണ്ട്. ജ്ഞാനസ്‌നാനം, പ്രാര്‍ത്ഥനകള്‍, ഉപവാസം, ദാനം തുടങ്ങിയവയാണ് ഇവരുടെ അടിസ്ഥാനാചാരങ്ങള്‍. അരമായ ഭാഷയോട് സാദൃശ്യമുള്ളതും സെമിറ്റിക് മൂലത്തില്‍നിന്ന് നിര്‍ധരിക്കപ്പെട്ടതുമായ മാന്‍ഡിയാക് ഭാഷ (Mandiac language) യിലാണ് ഇവരുടെ മതഗ്രന്ഥങ്ങളെല്ലാം എഴുതപ്പെട്ടിരിക്കുന്നത്. ഗിന്‍സാ റാബ, ദ്രാഷഇദ് യഹ്‌യ, ആദാം ബോഗ്‌റ, ദി കിലെസ്ത, നിയാനി എന്നിവയാണ് ഇവരുടെ മതഗ്രന്ഥങ്ങള്‍. ബാഗ്ദാദിലെ കൗണ്‍സില്‍ ഓഫ് ജനറല്‍ അഫയേഴ്‌സിന് കീഴില്‍പ്രവര്‍ത്തിക്കുന്ന മാന്‍ഡിയന്‍ റിസര്‍ച്ച് സെന്ററില്‍നിന്ന് ഈമതവിഭാഗത്തെക്കുറിച്ച കൂടുതല്‍ അറിവ് ലഭിക്കും. www.mandaean.com-au,www.mandaean.org എന്നീ വെബ്‌സൈറ്റുകളില്‍നിന്ന് ഈ മതവിഭാഗത്തിന്റെ വിശ്വാസാചാരങ്ങളെക്കുറിച്ച് മനസ്സിലാക്കാനാവും. മാന്‍ഡിയന്‍മാര്‍ തങ്ങളുടെ പ്രവാചകനും ഗുരുവുമായി സ്വീകരിച്ചിരിക്കുന്നത് യോഹന്നാന്‍ സ്‌നാപകനെയാണെന്ന് പറഞ്ഞുവല്ലോ. അവര്‍ അദ്ദേഹത്തെ വിളിക്കുന്നത് യഹ്‌യാ യൂഹന്നായെന്നാണ്. സ്‌നാപകയോഹന്നാന്‍േറതായി അവര്‍ വിശ്വസിക്കുന്ന ഉപദേശങ്ങളുടെയും അധ്യാപനങ്ങളുടെയും സമാഹാരമാണ് ‘ദ്രാഷാ ഇദ് യഹ്‌യ‘യെന്ന ഗ്രന്ഥം. ‘യഹ്‌യായുടെ പുസ്തകം‘ എന്നര്‍ത്ഥം. അവരുടെ ഏറ്റവും പ്രധാനപ്പെട്ട ഗ്രന്ഥമായ ഗിന്‍സ റാബയിലെ നാനൂറ്റി പത്താം അധ്യായം തന്നെ ‘യഹ്‌യായുടെ പ്രാര്‍ത്ഥനകള്‍’ എന്ന തലക്കെട്ടോടുകൂടിയതാണ്. ഇവയില്‍ നിന്നെല്ലാംതന്നെ മാന്‍ഡിയന്‍മാര്‍ യോഹന്നാന്‍ സ്‌നാപകനെ വിളിക്കുന്നത് യഹ്‌യാ യൂഹന്നായെന്നായിരുന്നുവെന്ന് വ്യക്തമാകുന്നു. ഖുര്‍ആനില്‍ പറഞ്ഞതുപോലെ യോഹന്നാന്‍ സ്‌നാപകന് യഹ്‌യായെന്ന പേര് കൂടിയുണ്ടായിരുന്നുവെന്ന് ഇതില്‍നിന്ന് കൃത്യമായി മനസ്സിലാകുന്നുണ്ട്. മാന്‍ഡായിക്കുകാരുടെ ഗ്രന്ഥങ്ങളില്‍നിന്നും ലേഖനങ്ങളില്‍ നിന്നുമെല്ലാം യോഹന്നാന്‍ സ്‌നാപകന്റെ നാമം യഹ്‌യാ യൂഹന്നായെന്നായിരുന്നുവെന്ന് മനസ്സിലാകുന്നുണ്ടെന്ന് പറഞ്ഞല്ലോ. എന്നാല്‍ ഖുര്‍ആനിലൊരിടത്തും അദ്ദേഹത്തെ യഹ്‌യാ യൂഹന്നായെന്ന് വിളിച്ചിട്ടില്ല. എന്തുണ്ടൊണിത്? ഈ ചോദ്യത്തിന് ഉത്തരം ലഭിക്കണമെങ്കില്‍ എന്തുകൊണ്ടാണ് ഈ ഇരട്ടനാമം ഉപയോഗിക്കുന്നതെന്ന് കൃത്യമായി മനസ്സിലാക്കണം. മാന്‍ഡിയന്‍മാരെയും അവരുടെ വിശ്വാസസംഹിതകളെയും ആചാരരീതികളെയുംകുറിച്ച് വിശദമായി പഠിച്ചയാളാണ് ഇ.എസ്. ഡ്രോവര്‍. അവരുടെ ‘ദി മാന്‍ഡിയന്‍സ് ഓഫ് ഇറാഖ് ആന്റ് ഇറാന്‍’, ‘ദി കാനോനിക്കല്‍പ്രെയര്‍ ബുക്ക് ഓഫ് ദി മാന്‍ഡിയന്‍സ്‘ എന്നീ പുസ്തകങ്ങള്‍ ഇവ്വിഷയകമായ ആധികാരിക രേഖകളായി പരിഗണിക്കപ്പെടുന്നവയാണ്. അവരും ആര്‍. മാക്കൂച്ചും കൂടിച്ചേര്‍ന്ന് എഴുതിയ ഗ്രന്ഥമാണ് എമാന്‍ഡായിക് ഡിക്ഷ്ണറി. (E.S. Drowoer: & R. Marcuch: A MANDAIC DICTIONARY 1963 OXFORD) മാന്‍ഡിയന്മാരുടെ സാങ്കേതിക ശബ്ദങ്ങളും അവര്‍ അവയുപയോഗിച്ചിരുന്ന രീതിയുമെല്ലാം ഈ ശബ്ദകോശത്തിലുണ്ട്. പ്രസ്തുത ഡിക്ഷ്ണറിയുടെ 185-ാം പുറത്തില്‍ യഹ്‌യാ (iahia)യുടെയും 190-ാംപുറത്തില്‍ യോഹന്നാ (iuhana) യുടെയും സാരം നല്‍കിയിട്ടുണ്ട്. അവനോക്കുക: ഇവയില്‍നിന്നും മറ്റ് മാന്‍ഡിയന്‍ സാഹിത്യങ്ങളില്‍ നിന്നുമായി അവരുടെ പേരുകളെക്കുറിച്ച് നമുക്ക് മനസ്സിലാകുന്ന ഒട്ടേറെ കാര്യങ്ങളുണ്ട്. എല്ലാ മാന്‍ഡിയന്മാര്‍ക്കും പൊതുവെ രണ്ട് പേരുകളുണ്ടായിരിക്കും. ഒന്നാമത്തെ പേര് മല്‍വാഷാ നാമമെന്നും (malwasha name) രണ്ടാമത്തെ പേര് ലഖബ് (lagab) എന്നുമാണ് അറിയപ്പെടുക. എന്തിനാണ് ഈ രണ്ട് പേരുകള്‍? ഇവ എന്താണ് അര്‍ത്ഥമാക്കുന്നത്? ഇ.എസ്. ഡ്രോവര്‍ എഴുതുന്നു: ”രണ്ടാമത്തെ പേര് പൊതുവെ ഒരു മുഹമ്മദന്‍ നാമമായിരിക്കും. ഇതാണ് എല്ലാ സാധാരണ ആവശ്യങ്ങള്‍ക്കും ഉപയോഗിക്കപ്പെടാറുള്ളത്. ആദ്യത്തെ പേര് (malwasha) ആണ് അയാളുടെ യഥാര്‍ത്ഥ ആത്മീയ നാമം. മതപരവും മാന്ത്രികവുമായ സന്ദര്‍ഭങ്ങളിലെല്ലാം ഈ പേരാണ് ഉപയോഗിക്കുക” (E.S. Drower: The Mandaeans of Iraq and Iran (1962-Lieden) Page 81) യോഹന്നാന്‍ സ്‌നാപകന്റെ മാല്‍വാഷാ നാമമാണ് യഹ്‌യ. യോഹന്നാന്‍ എന്നത് അദ്ദേഹത്തിന്റെ ലഖബും. ജനങ്ങള്‍ പൊതുവെ അദ്ദേഹത്തെ വിളിച്ചിരുന്നത് യോഹന്നാന്‍ എന്നായിരിക്കണം. കാരണം മാന്‍ഡിയന്‍മാര്‍ എല്ലാ സാധാരണ ആവശ്യങ്ങള്‍ക്കും പൊതുവായി ലഖബാണ് വിളിച്ചിരുന്നത്. എന്നാല്‍ അദ്ദേഹത്തിന്റെ യഥാര്‍ത്ഥ ആത്മീയ നാമം യഹ്‌യായെന്നായിരുന്നു. മതപരമായ സന്ദര്‍ഭങ്ങളുമായി ബന്ധപ്പെടുത്തി ഈ നാമമാണ് ഏറെ ഉപയോഗിക്കപ്പെട്ടിരിക്കുന്നത്. യഹ്‌യായുടെ പുസ്തകത്തിലെ മിക്ക അധ്യായങ്ങളും ആരംഭിക്കുന്നതുതന്നെ ”യഹ്‌യാ രാത്രികളില്‍ പ്രഖ്യാപിക്കുന്നു; യോഹന്നാ രാത്രിയുടെ സന്ധ്യകളിലും” എന്ന് പറഞ്ഞുകൊണ്ടാണ്. ചുരുക്കത്തില്‍ യോഹന്നാന്‍ സ്‌നാപകന്റെ യഥാര്‍ത്ഥ ആത്മീയ നാമം ‘യഹ്‌യാ‘യെന്നായിരുന്നു; ജനങ്ങള്‍ ആ സമൂഹത്തില്‍ നിലനിന്നിരുന്ന സമ്പ്രദായപ്രകാരം അദ്ദേഹത്തിന്റെ ലഖബ് ആയ‘യോഹന്നാ‘ എന്ന പേരിലാണ് പൊതുവായി അദ്ദേഹത്തെ വിളിച്ചിരുന്നത് എന്ന് മാത്രമേയുള്ളൂ. ഖുര്‍ആനില്‍ ‘യഹ്‌യാ‘യെന്ന് മാത്രമെ പ്രവാചക നാമമായിഉപയോഗിക്കപ്പെട്ടിട്ടുള്ളൂവെന്ന് പറഞ്ഞുവല്ലോ. അതാണ്, അത് മാത്രമാണ്അദ്ദേഹത്തിന്റെ യഥാര്‍ത്ഥ നാമം എന്നുള്ളതുകൊണ്ടാണിത്. ദൈവികകല്‍പന പ്രകാരം മാതാപിതാക്കള്‍ അദ്ദേഹത്തിന് നല്‍കിയ പേരായാണ്ഖുര്‍ആന്‍ ‘യഹ്‌യാ‘യെന്ന നാമത്തെ പരിചയപ്പെടു ത്തുന്നത്. അതായിരുന്നുഅദ്ദേഹത്തിന്റെ യഥാര്‍ത്ഥ ആത്മീയ നാമം എന്ന് മാന്‍ഡിയന്‍സാഹിത്യങ്ങളില്‍നിന്ന് നമുക്ക് മനസ്സിലാകുന്നതോടെ ഖുര്‍ആനിന്റെദൈവികത ഒരിക്കല്‍കൂടി വ്യക്തമാവുകയാണ് ചെയ്യുന്നത്; ഒപ്പം,ബൈബിളില്‍നിന്ന് പകര്‍ത്തിയെഴുതിയതാണ് ഖുര്‍ആന്‍ എന്ന വാദത്തിന്റെമൂലത്തില്‍തന്നെ ഈ വസ്തുതകള്‍ കഠാരകുത്തിക്കയറ്റുന്നു.ബൈബിളിലെവിടെയും പരാമര്‍ശിക്കപ്പെട്ടിട്ടില്ലാത്ത യോഹന്നാന്‍സ്‌നാപകന്റെ യഥാര്‍ത്ഥ നാമമായ ‘യഹ്‌യാ‘ ഖുര്‍ആനില്‍ വന്നത്യാദൃച്ഛികമാകാനിടയില്ലെന്ന് ഏതൊരു സാധാരണക്കാരന്നുംമനസ്സിലാക്കാവുന്നതേയുള്ളൂ. സകരിയ്യാ (عليه السلام)യുടെ വാര്‍ധക്യകാലത്ത്വന്ധ്യയായ ഭാര്യയില്‍ പുത്രനെ പ്രദാനം ചെയ്യുകയും പുത്രന് ‘യഹ്‌യാ‘യെന്ന് പേരിടാന്‍ നിര്‍ദ്ദേശിക്കുകയും ചെയ്ത തമ്പുരാനില്‍നിന്ന്അവതീര്‍ണമായ ഗ്രന്ഥമായതിനാലാണ് ഖുര്‍ആനില്‍ ഇക്കാര്യത്തിലുംകൃത്യവും സൂക്ഷ്മവുമായ പരാമര്‍ശങ്ങളുണ്ടായത് എന്ന് മാത്രമേചിന്തിക്കുവാന്‍ കഴിയുകയുള്ളൂ. ബൈബിളില്‍ സ്‌നാപകനെ കുറിക്കുവാന്‍ യോഹന്നാന്‍ എന്ന് മാത്രമെപ്രയോഗിക്കപ്പെട്ടിട്ടുള്ളൂവെന്നതും സ്വാഭാവികമാണ്. അദ്ദേഹത്തിന് ശേഷംപതിറ്റാണ്ടുകള്‍ ഏറെക്കഴിഞ്ഞ് രചിക്കപ്പെട്ട സുവിശേഷങ്ങളിലാണ്യോഹന്നാന്‍ സ്‌നാപകനെക്കുറിച്ച പരാമര്‍ശങ്ങളുള്ളത്. നടേസൂചിപ്പിച്ചതുപോലെ തന്റെ കാലത്ത് അദ്ദേഹം ജനങ്ങളാല്‍ പൊതുവായിവിളിക്കപ്പെട്ടത് യോഹന്നായെന്നായിരിക്കണം. അതുകൊണ്ടുതന്നെവാമൊഴിയായി പ്രചരിച്ച അദ്ദേഹത്തിന്റെ ജീവിതകഥനങ്ങളിലുംഉപദേശങ്ങളിലും യോഹന്നായെന്ന പേരായിരിക്കണം പ്രധാനമായുംഉപയോഗിക്കപ്പെട്ടത്. പതിറ്റാണ്ടുകള്‍ കഴിഞ്ഞ് രചിക്കപ്പെട്ടസുവിശേഷങ്ങളുടെ കര്‍ത്താക്കള്‍ യോഹന്നായെന്നാണ് അദ്ദേഹത്തിന്റെനാമമെന്ന് കരുതിയത് സ്വാഭാവികം മാത്രം. എന്നാല്‍ ബൈബിള്‍പുസ്തകങ്ങളുടെ കര്‍ത്താക്കള്‍ക്ക് തങ്ങളുടെ ഗ്രന്ഥരചനയില്‍ദൈവനിവേശനമുണ്ടായിരുന്നുവെന്ന ക്രൈസ്തവ വിശ്വാസത്തിന് കോട്ടംതട്ടിക്കുന്ന പല തെളിവുകളിലൊന്നാണ് ഇതുമെന്ന വസ്തുതവിസ്മരിച്ചുകൂടാ. അങ്ങനെയൊരു ദൈവിക ഇടപെടലിന്റെ സ്വാധീനത്താല്‍രചിക്കപ്പെട്ടതായിരുന്നു സുവിശേഷങ്ങളെങ്കില്‍ തീര്‍ച്ചയായും സ്‌നാപകന്റെയഥാര്‍ത്ഥമായ ആത്മീയ നാമമായിരുന്നു അവയില്‍പ്രതിപാദിക്കേണ്ടിയിരുന്നത്. എ ന്നാല്‍ ജനങ്ങള്‍ക്കിടയില്‍പ്രചാരത്തിലിരുന്ന യോഹന്നാന്‍ എന്ന പേര് മാത്രമെ സുവിശേഷങ്ങള്‍ഉപയോഗിക്കുന്നുള്ളൂ. തങ്ങള്‍ക്ക് വാമൊഴിയായിപകര്‍ന്നുകിട്ടിയതിനേക്കാള്‍ അധികമായ യാതൊരു അറിവും സുവിശേഷകര്‍ത്താക്കള്‍ക്ക് ഉണ്ടായിരുന്നില്ലെന്ന വസ്തുതയാണല്ലോ ഇത്വെളിപ്പെടുത്തുന്നത്. യഹ്‌യായുടെ ലഖബ് ആയ ‘യോഹന്നാ‘യെക്കുറിച്ച് എന്തെങ്കിലുംഅറിവ് ഖുര്‍ആന്‍ നല്‍കുന്നുണ്ടോയെന്ന് പരിശോധിക്കു േമ്പാള്‍ അതിന്റെദൈവികത ഒന്നുകൂടി നമുക്ക് ബോധ്യപ്പെടുകയും സര്‍വ്വശക്തനായസ്രഷ്ടാവിന് മുമ്പില്‍ നമ്രശിരസ്‌കരാവുന്നതിലേക്ക് നാം നയിക്കപ്പെടുകയുംചെയ്യുന്നു. ‘യൂ‘, ‘ഹന്നാന്‍’ എന്നീ രണ്ട് വാക്കുകളുടെ സമ്മേളനത്തില്‍നിന്നാണ്യൂഹന്നായെന്ന പദമുണ്ടായിട്ടുള്ളതെന്നും ‘ഹന്നാന്‍’ എന്ന ഹിബ്രു പദം‘ഹനാന്‍’ എന്ന അരമായ മൂലത്തില്‍നിന്നുണ്ടായതാണെന്നും‘അനുകമ്പ‘യെന്നാണ് ഈ പദത്തിന് അര്‍ത്ഥമെന്നും നേരത്തെ സൂചിപ്പിച്ചത്ഓര്‍ക്കുക. ‘ഹനാന്‍’ എന്ന അറബി പദവും ഇതേഅര്‍ത്ഥമുള്‍ക്കൊള്ളുന്നതാണ്. അറബി–ഹിബ്രു–അരാമിക് തുടങ്ങിയഭാഷകളെല്ലാം ഒരേ സെമിറ്റിക് മൂലത്തില്‍നിന്നുണ്ടായവയാണല്ലോ. ഖുര്‍ആനില്‍ ഒരു തവണമാത്രമെ ‘ഹനാന്‍’ എന്ന പദംഉപയോഗിച്ചിട്ടുള്ളൂ; സൂറത്തുമര്‍യമിലെ പതിമൂന്നാം (19:13) സൂക്തത്തില്‍.ആ സൂക്തത്തിന്റെ മലയാളം ലിപ്യന്തരണം ഇങ്ങനെയാണ്: വ ഹനാനന്‍ മിന്‍ ലദുന്നാ വ സകാത്തന്‍ വ കാന തഖിയ്യാ ”ഈവചനത്തിന്റെ മലയാള പരിഭാഷ ”നമ്മുടെ പക്കല്‍നിന്നുള്ള അനുകമ്പയുംപരിശുദ്ധിയും നല്‍കി; അദ്ദേഹം (യഹ്‌യാ) ധര്‍മ്മനിഷ്ഠയുള്ളവനായിരുന്നു” (19:13)വെന്നാണ്. ഈ വചനത്തില്‍ യഹ്‌യായെക്കുറിച്ച് ‘നമ്മുടെ പക്കല്‍നിന്നുള്ളഅനുകമ്പ‘ (ഹനാനന്‍ മിന്‍ ലദുന്ന)യെന്ന് പ്രയോഗിച്ചത് പ്രത്യേകംശ്രദ്ധേയമാണ്. യഹ്‌യാ ‘ദൈവത്തില്‍നിന്നുള്ള അനുകമ്പ‘യാണെന്നര്‍ത്ഥം.യൂഹന്നയെന്ന പദത്തിനര്‍ത്ഥം ‘ദൈവത്തില്‍നിന്നുള്ള അനുകമ്പ‘യെന്നാണെന്ന്മുമ്പ് സൂചിപ്പിച്ചത് ഓര്‍ക്കുക. ‘യോഹന്ന‘യിലെ അതേ ഹനാന്‍ തന്നെയാണ്ഖുര്‍ആന്‍ ഇവിടെ പ്രയോഗിച്ചിരിക്കുന്നത്. യൂഹന്നയിലെ യൂ ഒഴിവാക്കിഅതിന്റെ മൂലരൂപത്തിന് തത്തുല്യമായ ‘ഹനാന്‍’ എന്ന്പ്രയോഗിക്കുകയാണ് ഖുര്‍ആന്‍ ചെയ്തിരിക്കുന്നത്. ഇവിടെ ‘യൂ‘ ഒഴിവാക്കിയിരിക്കുന്നതും പ്രത്യേകം ശ്രദ്ധിക്കേണ്ടതാണ്. ‘യഹോവ‘യുടെ ചുരുക്കമായാണ് ‘യൂ‘യെന്ന് ഉപയോഗിച്ചിരിക്കുന്നതെന്ന്നേരത്തെ സൂചിപ്പിച്ചുവല്ലോ. അറബിയില്‍ ഏകദൈവത്തെക്കുറിക്കുവാന്‍യഹോവയെന്ന് ഉപയോഗിക്കാറില്ല. അതുകൊണ്ടുതന്നെ യഹോവയുടെചുരുക്കപ്പേരായ ‘യൂ‘യെന്ന് ഖുര്‍ആനില്‍ പ്രയോഗിക്കുന്നത് സംഗതമല്ലല്ലോ.യൂഹന്നായെന്നത് യഹ്‌യായുടെ യഥാര്‍ത്ഥ നാമമല്ലെന്നും അദ്ദേഹത്തിന്റെസ്വഭാവ സവിശേഷതകളുടെ അടിസ്ഥാനതില്‍ ജനം വിളിച്ചിരുന്നപേരായിരുന്നുവെന്നും അത് അദ്ദേഹത്തിന്റെ സവിശേഷത മാത്രമാണ്വെളിപ്പെടുത്തുന്നതെന്നും ഓര്‍ക്കുക. അതുകൊണ്ടുതന്നെ‘ദൈവത്തില്‍നിന്നുള്ള അനുകമ്പ‘യെന്ന അര്‍ത്ഥത്തിലുള്ള യൂഹന്നായെന്ന്അതേപോലെ അറബിയില്‍ പ്രയോഗിച്ചിരുന്നുവെങ്കില്‍ ആ പദംഅര്‍ത്ഥരഹിതമാകുമായിരുന്നു. യൂഹന്നായെന്നത് അദ്ദേഹത്തിന്റെപേരല്ലല്ലോ. എന്നാല്‍ ‘യൂ‘ ഒഴിവാക്കിക്കൊണ്ട് ‘ദൈവത്തില്‍നിന്നുള്ള ഹനാന്‍’എന്ന് കൃത്യമായി ഖുര്‍ആന്‍ പ്രയോഗിച്ചത് കാണുമ്പോള്‍ അതിന്റെസൂക്ഷ്മതയും കൃത്യതയും നമുക്ക് വ്യക്തമായി മനസ്സിലാവുകയുംതെറ്റുപറ്റാത്ത സ്രഷ്ടാവില്‍ നിന്നുള്ളതാണ് ഖുര്‍ആനെന്ന് സുതരാംബോധ്യപ്പെടുകയും ചെയ്യുന്നു. ഖുര്‍ആനില്‍ ഒരേയൊരു സ്ഥലത്ത് മാത്രമെഹനാന്‍ എന്ന പദം ഉപയോഗിച്ചിട്ടുള്ളൂവെന്നും അത്യഹ്‌യായെക്കുറിച്ചാണെന്നതുംകൂടി ഇതോടൊപ്പം ചേര്‍ത്തുവായിക്കുമ്പോള്‍ആര്‍ക്കാണ് അതിന്റെ ദൈവികത ബോധ്യപ്പെടാതിരിക്കുക? ഇനി നാം ചോദ്യത്തിലേക്ക് തിരിച്ചുപോവുക. ഖുര്‍ആനിലെസൂറത്തുമര്‍യം ഏഴാം വചനത്തില്‍ (19:7) പറയുന്നതെന്താണ്? ”ഹേ സക്കരിയാ, തീര്‍ച്ചയായും നിനക്ക് നാം ഒരു ആണ്‍കുട്ടിയെപറ്റിസന്തോഷവാര്‍ത്ത അറിയിക്കുന്നു. അവന്റെ പേര്‍ യഹ്‌യാ എന്നാകുന്നു.മുമ്പ് നാം ആരെയും അവന്റെ പേരുള്ളവരാക്കിയിട്ടില്ല” (വി.ഖു.19:7) ഈ വചനം രണ്ടുതരത്തില്‍ വ്യാഖ്യാനിക്കപ്പെട്ടിട്ടുണ്ട്.: ഒന്ന്) ഇവിടെ ”ലം നജ്അല്‍ ലഹു മിന്‍ ഖബ്‌ലു സമിയ്യാ”യെന്ന വചനഭാഗത്തെയാണ് ”മുമ്പ് ആരെയും അവന്റെ പേരുള്ളവരാക്കിയിട്ടില്ല”യെന്ന് പരിഭാഷപ്പെടുത്തിയിരിക്കുന്നത്. ‘സമിയ്യന്‍’എന്ന പദത്തെയാണ് പേരുള്ളവന്‍ എന്ന് ഭാഷാന്തരം ചെയ്തിരിക്കുന്നത്. ഇതിന് ”മിഥ്‌ലന്‍” എന്നും ”ശബീഹന്‍” എന്നുമെല്ലാം അര്‍ത്ഥമുണ്ട്. അദ്ദേഹത്തെ പോലെയുള്ളവന്‍ എന്നര്‍ത്ഥം. അപ്പോള്‍ ഈ വചനഭാഗത്തിന്” മുമ്പ് ആരെയും അദ്ദേഹത്തെപ്പോലെയുള്ളവനാക്കിയിട്ടില്ല” എന്ന അര്‍ത്ഥം വരും. വൃദ്ധനായ പിതാവിന് വന്ധ്യയായ ഭാര്യയിലുണ്ടായകുഞ്ഞാണ് യഹ്‌യ. ഇങ്ങനെയൊരു സംഭവം അദ്ദേഹത്തിന് മുമ്പുണ്ടായിട്ടില്ല. ഈ അര്‍ത്ഥത്തില്‍ യഹ്‌യായെപ്പോലെ ഒരാള്‍ അദ്ദേഹത്തിന്മുമ്പുണ്ടായിട്ടില്ലെന്നാണ് ഈ വചനത്തിന് ചില പണ്ഡിതന്മാര്‍ നല്‍കിയ വ്യാഖ്യാനം. രണ്ട്) ഈ വചനഭാഗത്തിന്റെ നേര്‍ക്കുനേരെയുള്ള അര്‍ത്ഥം പരിഗണിച്ചുകൊണ്ട് ‘യഹ്‌യാ‘യെന്ന പേര് സ്‌നാപകനുമുമ്പ് മറ്റാര്‍ക്കുമുണ്ടായിട്ടില്ലെന്നാണ് മറ്റൊരു വിഭാഗം പണ്ഡിതന്മാരുടെ വ്യാഖ്യാനം. രണ്ട് വ്യാഖ്യാനങ്ങള്‍ പ്രകാരം പരിശോധിച്ചാലും ഈ ഖുര്‍ആന്‍ സൂക്തത്തില്‍ യാതൊരു വിധ അബദ്ധവുമില്ലെന്നതാണ് വാസ്തവം. യഹ്‌യാ(അ)ക്കുമുമ്പ് വൃദ്ധനായ പിതാവിന് വന്ധ്യയായ മാതാവിലുണ്ടായ ഒരു കുഞ്ഞിന്റെ കഥ ബൈബിളോ ഖുര്‍ആനോ പരാമര്‍ശിക്കുന്നില്ല. രണ്ടാമത്തെ വ്യാഖ്യാനത്തില്‍ കടിച്ചുതൂങ്ങി ഖുര്‍ആനില്‍ അബദ്ധം ആരോപിക്കുവാന്‍ വേണ്ടി ശ്രമിക്കുന്നവരുടെ വിമര്‍ശനങ്ങള്‍ ഖുര്‍ആനിന്റെ പ്രോജ്ജ്വല പ്രകാശത്തിന് മുമ്പില്‍ കരിഞ്ഞുവീഴുന്നതാണ് നാം കണ്ടത്. യഹ്‌യായെന്ന ഒരു നാമം സ്‌നാപകനുമുമ്പ് ആര്‍ക്കെങ്കിലും നല്‍കപ്പെട്ടതായി സൂചിപ്പിക്കുന്ന രേഖകളൊന്നും തന്നെയില്ല. യഹ്‌യാ=യോഹന്നാന്‍ എന്ന സമവാക്യം ഖുര്‍ആനിന്‍േറതല്ല. അതുകൊണ്ടുതന്നെ അത് വിമര്‍ശനങ്ങള്‍ക്കുമുമ്പില്‍ തകരും. എന്നാല്‍ ഖുര്‍ആന്‍ മുന്നോട്ടുവെക്കുന്ന ആശയങ്ങളാകട്ടെ ഓരോ വിമര്‍ശനങ്ങളുന്നയിക്കപ്പെടുമ്പോഴും പൂര്‍വ്വാധികം പ്രോജ്ജ്വലമായി വിളങ്ങുക മാത്രേമയുള്ളൂ.